Sintesis U De Chile Tomo I-1.pdf

  • Uploaded by: orlando gonzalez
  • 0
  • 0
  • March 2021
  • PDF

This document was uploaded by user and they confirmed that they have the permission to share it. If you are author or own the copyright of this book, please report to us by using this DMCA report form. Report DMCA


Overview

Download & View Sintesis U De Chile Tomo I-1.pdf as PDF for free.

More details

  • Words: 299,527
  • Pages: 676
Loading documents preview...
Expropiado, reeditado y remasterizado por: VMAM (TOMO I 2019)

SÍNTESIS TOMO I MÓDULO 1: Medicina Interna Cardiología Diabetes y nutrición Endocrinología Enfermedades infecciosas Enfermedades respiratorias Gastroenterología Geriatría Hémato-oncología Nefrología Neurología Reumatología

MÓDULO 1: Medicina Interna Cardiología

PÁGINA 16

1. Angina Crónica Estable ...................................................................................... 16 2. Bloqueos Aurículo-Ventriculares ........................................................................ 18 3. Endocarditis Bacteriana Sub Aguda ................................................................... 20 4. Estenosis Aórtica................................................................................................ 22 5. Estenosis Mitral .................................................................................................. 23 6. Fibrilación Auricular Crónica .............................................................................. 24 7. Fibrilación Auricular Paroxística ......................................................................... 26 8. Hipertensión Arterial Esencial ............................................................................ 28 9. Insuficiencia Aórtica ........................................................................................... 31 10. Insuficiencia Cardiaca ........................................................................................ 33 11. Insuficiencia Mitral .............................................................................................. 35 12. Pericarditis Aguda .............................................................................................. 37 13. Taquicardia Paroxística Supraventricular ........................................................... 39 14. Arritmia Supraventricular .................................................................................... 40 15. Angina Inestable................................................................................................. 55 16. Disección Aórtica................................................................................................ 56 17. Infarto Agudo Al Miocardio ................................................................................. 57 18. Insuficiencia Cardíaca Aguda ............................................................................. 59 19. Paro Cardiorespiratorio ...................................................................................... 64 20. Shock ................................................................................................................. 66 21. Taquiarritmias y Bradiarritmias con Compromiso Hemodinámico ...................... 68 22. Cardiopatía congénita en adulto ........................................................................ 70 23. Corazón pulmonar crónico ................................................................................. 71 24. Dislipidemias ...................................................................................................... 72 25. Embolia cardiogénica ......................................................................................... 75 26. Embolia pulmonar .............................................................................................. 76 27. Enfermedad reumática activa ............................................................................. 78 28. Extrasistolía ventricular benigna ........................................................................ 80 29. Flutter auricular .................................................................................................. 81 30. Hipertensión arterial secundaria ......................................................................... 82 31. Miocardiopatías .................................................................................................. 83 32. Paciente con soplo ............................................................................................. 85 33. Síndrome metabólico ......................................................................................... 87 34. Taponamiento pericárdico .................................................................................. 88

Diabetes y nutrición

PÁGINA 89

1. Desnutrición ....................................................................................................... 89 2. Diabetes Gestacional ......................................................................................... 91 3. Diabetes Pregestacional .................................................................................... 94 4. Diabetes Mellitus Tipo 1 ..................................................................................... 96 5. Diabetes Mellitus Tipo 2 ....................................................................................101 6. Cetoacidosis Diabética ......................................................................................105 7. Hipoglicemia ......................................................................................................107 8. Diabetes por Corticoides ...................................................................................109 9. Dislipidemias .....................................................................................................110 10. Insuficiencia Renal Y Diabetes ..........................................................................112 11. Neuropatía Diabética.........................................................................................113 12. Obesidad ...........................................................................................................114 13. Coma Hiperosmolar ..........................................................................................116 14. Síndrome Hiperglicémico Hiperosmolar No Cetósico ........................................118 15. Obesidad Mórbida .............................................................................................120 16. Pie Diabético y Otras Infecciones en Diabetes .................................................121 17. Retinopatía Diabética ........................................................................................122 18. Síndrome Metabólico ........................................................................................123 19. Vasculopatía Periférica .....................................................................................125 20. Hipertrigliceridemia Grave .................................................................................127 21. Acidosis láctica ..................................................................................................129 22. Déficit agudo de tiamina ....................................................................................130 23. Hipertensión arterial en Diabetes tipo 2 ...........................................................131 24. Nefropatía incipiente .........................................................................................132 25. Sindromes carenciales de vitaminas y minerales ..............................................133 26. Trastornos de conducta alimentaria ..................................................................135

Endocrinología

PÁGINA 137

1. Bocio .................................................................................................................137 2. Hipotiroidismo ...................................................................................................139 3. Hipertiroidismo ..................................................................................................140 4. Síndrome De Cushing .......................................................................................142 5. Insuficiencia Suprarrenal Crónica......................................................................144 6. Hipertensión Arterial de Origen Endocrino ........................................................146 7. Hipopituitarismo.................................................................................................148 8. Diabetes Insípida...............................................................................................149 9. Síndrome de Secreción Inapropiada de ADH ...................................................151 10. Osteoporosis Secundaria ..................................................................................153 11. Tormenta Tiroídea .............................................................................................155 12. Mixedema ..........................................................................................................157 13. Insuficiencia Suprarrenal Aguda .......................................................................158 14. Amenorrea ........................................................................................................160 15. Cáncer de Tiroides ............................................................................................166 16. Hipercalcemia aguda.........................................................................................167 17. Hipercalcemias e hipocalcemias .......................................................................169 18. Hiperprolactinemia ............................................................................................171 19. Hipogonadismo masculino ................................................................................173 20. Hirsutismo .........................................................................................................175 21. Síndrome climatérico.........................................................................................176 22. Tetania ..............................................................................................................179 23. Tiroiditis .............................................................................................................181 24. Tumores hipofisiarios ........................................................................................183

Enfermedades infecciosas

PÁGINA 184

1. Celulitis Bacteriana............................................................................................184 2. Enfermedades De Transmisión Sexual (ETS) ...................................................186 3. Enfermedades Eruptivas No Complicadas (Varicela, Herpes Zóster) ...............188 4. Infección Por VIH ..............................................................................................190 5. Influenza............................................................................................................192 6. Neutropenia Febril .............................................................................................194 7. Sífilis Secundaria, Terciaria y Congénita...........................................................196 8. Síndrome Febril Prolongado .............................................................................198 9. Endocarditis Infecciosa .....................................................................................199 10. Infección Invasiva de Partes: Celulitis, Fasceítis, Miositis Necrotizantes o Septicémicas .....................................................................................................201 11. Meningitis Aguda ...............................................................................................203 12. Septicemia ........................................................................................................205 13. Parasitosis Intestinales......................................................................................206 14. Síndrome Mononucleósico ................................................................................208 15. Tóxico-Infección Alimentaria .............................................................................209 16. Triquinosis .........................................................................................................210 17. Tétanos .............................................................................................................211 18. Micosis Invasora (Aspergillosis, Candidiasis, Mucormicosis) ............................212 19. Candidiasis Oral y Esofágica ............................................................................214 20. Cólera................................................................................................................215 21. Absceso cerebral...............................................................................................217 22. Adenitis y adenoflegmón ...................................................................................218 23. Angina úlcero-necrótica, úlcero-membranosa y pseudomembranosa...............219 24. Ántrax o carbunco .............................................................................................220 25. Brucelosis..........................................................................................................221 26. Dengue..............................................................................................................222 27. Diarrea asociada a antibióticos .........................................................................223 28. Encefalitis aguda ...............................................................................................224 29. Fiebre tifoídea y paratifoídea .............................................................................226 30. Flegmón submaxilar, submandibular y del piso de la boca ...............................227 31. Hidatidosis.........................................................................................................228 32. Infecciones asociadas a catéteres vasculares ..................................................229 33. Leptospirosis .....................................................................................................230 34. Malaria ..............................................................................................................231 35. Osteomielitis......................................................................................................232 36. Síndrome pulmonar por hanta virus ..................................................................233 37. Tuberculosis extrapulmonar ..............................................................................234 38. Varicela complicada: neumonitis, cerebelitis, encefalitis ...................................235

Enfermedades respiratorias

PÁGINA 236

1. Asma Bronquial Grave ......................................................................................236 2. Asma Bronquial Leve y Moderada ....................................................................238 3. Bronquitis Aguda ...............................................................................................240 4. Bronquitis Crónica .............................................................................................241 5. Derrame Pleural Neoplásico .............................................................................242 6. Derrame Pleural Paraneumónico Complicado ..................................................243 7. Derrame Pleural Paraneumónico Simple ..........................................................244 8. Derrame Pleural Por Tuberculosis ....................................................................245 9. Enfermedad Pulmonar Obstructiva Crónica Avanzada .....................................246 10. Enfermedad Pulmonar Obstructiva Crónica Leve y Moderada .........................247 11. Hipertensión Pulmonar ......................................................................................249 12. Neumonías Adquiridas en la Comunidad (Tipo 4 de la Sociedad Chilena de Enfermedades Respiratorias) .......................................................................251 13. Neumonía Adquirida En La Comunidad (Tipo 1, 2 Y 3 De La Sociedad Chilena De Enfermedades Respiratorias) .........................................................253 14. Neumonía en Inmunosuprimidos.......................................................................255 15. Neumonía Nosocomial ......................................................................................256 16. Neumotórax Moderado y Grave ........................................................................257 17. Crisis de Asma Bronquial ..................................................................................258 18. Absceso pulmonar.............................................................................................261 19. Asfixia por Inmersión .........................................................................................262 20. Aspiración de cuerpo extraño con asfixia ..........................................................264 21. Aspiración de cuerpo extraño sin asfixia ...........................................................266 22. Bronquiectasias .................................................................................................268 23. Cáncer bronquial primario .................................................................................269 24. Corazón pulmonar crónico (cor pulmonale) ......................................................271 25. Daño pulmonar secundario a drogas ................................................................272 26. Edema pulmonar no cardiogénico .....................................................................273 27. Embolia pulmonar .............................................................................................274 28. Embolia Pulmonar Masiva.................................................................................276 29. Enfermedad Bronquial Obstructiva Crónica (EPOC) Descompensada .............277 30. Enfermedades del intersticio pulmonar .............................................................279 31. Hemoptisis leve y moderada .............................................................................281 32. Hemoptisis moderada, severa y masiva............................................................283 33. Hidatidosis pulmonar .........................................................................................284 34. Infecciones de las vías aéreas superiores ........................................................286 35. Insuficiencia respiratoria aguda o crónica reagudizada que requiere ventilación mecánica ....................................................................288 36. Insuficiencia respiratoria aguda o crónica reagudizada, leve o moderada (que no requiere ventilación mecánica).........................................290 37. Insuficiencia respiratoria crónica .......................................................................291 38. Intoxicación leve por CO y otros gases .............................................................293 39. Intoxicación por monóxido de carbono ..............................................................295 40. Metástasis Pulmonares .....................................................................................297 41. Neumoconiosis..................................................................................................298 42. Neumotórax hipertensivo (a tensión).................................................................300 43. Neumotórax pequeño ........................................................................................301

44. Paro cardiorrespiratorio .....................................................................................302 45. Síndrome de Apnea del Sueño .........................................................................305 46. Síndromes mediastínicos ..................................................................................306 47. Tos crónica ........................................................................................................307 48. Traumatismo torácico grave ..............................................................................309 49. Traumatismo torácico simple ............................................................................311 50. Tuberculosis Pulmonar......................................................................................313 51. Tuberculosis Pulmonar (Fracaso al tratamiento) ...............................................316

Gastroenterología

PÁGINA 318

1. Ascitis ................................................................................................................318 2. Cáncer de Páncreas..........................................................................................320 3. Cáncer de Vesícula y Vía Biliar .........................................................................321 4. Cáncer Gástrico ................................................................................................323 5. Cirrosis Hepática ...............................................................................................325 6. Colelitiasis .........................................................................................................326 7. Colestasia .........................................................................................................327 8. Diarrea Asociada a Antibióticos ........................................................................328 9. Diarrea en Inmunosuprimidos ...........................................................................330 10. Enfermedad Diverticular No Complicada ..........................................................331 11. Hepatitis Aguda A No Complicada ....................................................................333 12. Hepatitis Aguda B, C, por Otros Virus, por Drogas y Toxinas ...........................335 13. Pancreatitis Crónica ..........................................................................................337 14. Enfermedad por Reflujo Gastroesofágico (ERGE) ............................................339 15. Colangitis ..........................................................................................................342 16. Colecistitis Aguda ..............................................................................................344 17. Hemorragia Digestiva Alta y Baja ......................................................................346 18. Hepatitis Aguda A Complicada ..........................................................................348 19. Insuficiencia Hepática Aguda ............................................................................350 20. Pancreatitis Aguda ............................................................................................351 21. Síndrome Hepatorrenal .....................................................................................353 22. Abdomen Agudo................................................................................................355 23. Afagia aguda .....................................................................................................358 24. Colopatía funcional............................................................................................359 25. Constipación .....................................................................................................361 26. Diarrea aguda ...................................................................................................363 27. Diarrea Crónica .................................................................................................365 28. Disfagia .............................................................................................................367 29. Dispepsia ..........................................................................................................368 30. Encefalopatía hepática (EH) .............................................................................369 31. Enfermedad Celíaca..........................................................................................371 32. Enfermedad diverticular complicada .................................................................373 33. Enfermedad inflamatoria crónica intestinal ........................................................375 34. Hepatitis crónica ................................................................................................377 35. Insuficiencia hepática crónica ...........................................................................378 36. Intolerancia a la lactosa.....................................................................................379 37. Masa hepática ...................................................................................................380 38. Parasitosis Intestinales......................................................................................382 39. Peritonitis bacteriana espontánea del cirrótico ..................................................383 40. Síndrome de malabsorción ...............................................................................385 41. Tumores de colon..............................................................................................387 42. Úlcera péptica ...................................................................................................389

Geriatría

PÁGINA 391

1. Caídas ...............................................................................................................391 2. Constipación y Fecalomas ................................................................................394 3. Demencia ..........................................................................................................396 4. Depresión en el Adulto Mayor ...........................................................................398 5. Diabetes Mellitus II ............................................................................................400 6. Enfermedad Cerebro Vascular ..........................................................................402 7. Hipertensión Arterial ..........................................................................................403 8. Inmovilidad ........................................................................................................405 9. Evento Coronario En Adulto Mayor ...................................................................407 10. Fractura de Cadera ...........................................................................................408 11. Síndrome Confusional Agudo (SCA, Delirium) ..................................................409 12. Escaras o Úlceras por Presión (UPP) ...............................................................411 13. Fragilidad ..........................................................................................................414 14. Malnutrición .......................................................................................................416 15. Osteoporosis .....................................................................................................417 16. Temblor .............................................................................................................419 17. Trastornos de la Marcha ...................................................................................420 18. Abdomen agudo en adulto mayor .....................................................................423 19. Agitación y agresividad .....................................................................................424 20. Hipotensión postural (ortostática) ......................................................................425 21. Hipotermia .........................................................................................................426 22. Incontinencia urinaria ........................................................................................428

Hémato-oncología

PÁGINA 431

1. Anemia de Enfermedad Crónica .......................................................................431 2. Anemia Ferropriva .............................................................................................433 3. Anemia Hemolítica ............................................................................................435 4. Anemia Megaloblástica .....................................................................................437 5. Coagulopatías Adquiridas .................................................................................439 6. Leucemias Agudas ............................................................................................441 7. Púrpuras Vasculares .........................................................................................443 8. Síndrome Paraneoplásico .................................................................................445 9. Trombofilias.......................................................................................................447 10. Coagulación Intravascular Diseminada .............................................................449 11. Lisis Tumoral Aguda..........................................................................................450 12. Neutropenia Febril .............................................................................................451 13. Síndrome De Vena Cava Superior ....................................................................452 14. Leucemias Crónicas ..........................................................................................453 15. Síndromes Mielodisplásicos ..............................................................................455 16. Síndromes Mieloproliferativos Crónicos ............................................................456 17. Hipercalcemia ...................................................................................................458 18. Coagulopatía congénita sangrante....................................................................462 19. Coagulopatías congénitas (hemofilias, von Willebrand) ....................................463 20. Disproteinemias (gammapatías M)....................................................................465 21. Hipofunción medular .........................................................................................466 22. Linfomas ............................................................................................................467 23. Punción lumbar .................................................................................................468 24. Púrpuras trombopénicos ...................................................................................471 25. Quimioterapia intratecal ....................................................................................473 26. Reacción leucemoide ........................................................................................475 27. Síndrome compresión medular .........................................................................476 28. Síndrome leucoeritroblástico .............................................................................478 29. Trombopenia severa .........................................................................................479

Nefrología

PÁGINA 480

1. Alcalosis Metabólica ..........................................................................................480 2. Anemia en Nefrópata ........................................................................................482 3. Hipertensión Arterial Esencial ...........................................................................483 4. Hipertensión Arterial Secundaria .......................................................................486 5. Insuficiencia Renal Crónica ...............................................................................487 6. Nefropatía Diabética..........................................................................................492 7. Pielonefritis Aguda No Complicada ...................................................................493 8. Síndrome Nefrítico ............................................................................................495 9. Síndrome Nefrósico (Nefrótico) .........................................................................497 10. Síndrome Urémico ............................................................................................499 11. Uropatía Obstructiva .........................................................................................501 12. Crisis Hipertensivas, Emergencias Hipertensivas .............................................502 13. Hiperkalemia Grave...........................................................................................504 14. Hipokalemia ......................................................................................................506 15. Hiponatremia Aguda Grave ...............................................................................508 16. Insuficienca Renal Aguda Obstructiva (1) ........................................................510 17. Insuficienca Renal Aguda Prerrenal ..................................................................511 18. Pielonefritis Aguda Complicada ........................................................................512 19. Acidosis Metabólica...........................................................................................513 20. Acidosis e Hiperkalemia en insuficiencia renal crónica .....................................516 21. Bacteriuria Asintomática....................................................................................517 22. Cólico nefrítico, Urolitiasis y Complicaciones (Obstrucción, Sepsis e Insuficiencia Renal) ......................................................518 23. Edema generalizado grave (Anasarca) .............................................................519 24. Edema Generalizado grave (Anasarca) ............................................................520 25. Enfermedad Tubular Aguda ..............................................................................521 26. Glomerulopatía Lúpica ......................................................................................522 27. Hipernatremia, poliuria ......................................................................................523 28. Hiponatremia Crónica Asintomática ..................................................................524 29. Hipovolemia ......................................................................................................525 30. Infección Urinaria Baja (Cistitis) ........................................................................526 31. Instalación de pigtail ..........................................................................................527 32. Insuficienca renal aguda prostrrenal u obstructiva (2) ......................................528 33. Nefritis Tubulointersticial Aguda y Crónica ........................................................529 34. Osteodistrofia en Insuficiencia Renal Crónica ...................................................530 35. Pre-eclampsia ...................................................................................................531 36. Progresión de la Insuficiencia Renal Crónica ....................................................532 37. Proteinuria .........................................................................................................533 38. Riñon Poliquístico..............................................................................................535 39. Urolitiasis...........................................................................................................536 40. Vasculitis o glomerulonefritis rápidamente progresiva ......................................538

Neurología

PÁGINA 540

1. Compromiso De Conciencia ..............................................................................540 2. Epilepsia............................................................................................................542 3. Lumbago Mecánico Y Lumbociática..................................................................545 4. Migraña .............................................................................................................548 5. Parálisis (Hemiplejias, Tetraplejias, Hemiparesias, Tetraparesias) ...................550 6. Síndrome de Hipertensión Endocraneana ........................................................553 7. Cefalea Aguda en Urgencias ............................................................................555 8. Crisis Convulsiva ...............................................................................................557 9. Estado Confusional Agudo ................................................................................559 10. Encefalopatía Tóxica Metabólica ......................................................................561 11. Enfermedad Cerebro Vascular ..........................................................................562 12. Hipertensión Intracraneana ...............................................................................564 13. Síndromes Meníngeos con Meningitis Purulenta o Bacteriana .........................566 14. Status Convulsivo..............................................................................................568 15. Status Migrañoso ..............................................................................................570 16. Esclerosis Múltiple.............................................................................................571 17. Neuralgia Esencial del Trigémino ......................................................................573 18. Neuropatía por Atrapamiento ............................................................................575 19. Síndromes Miasténicos .....................................................................................577 20. Síndrome Miopatico (Distrofias Musculares, Polimiositis) .................................579 21. Síndromes Convulsivos Sintomáticos ...............................................................581 22. Síndromes Sensitivos........................................................................................583 23. Cefalea tensional...............................................................................................586 24. Complicaciones neurológicas de abuso de sustancias .....................................587 25. Cuadros de deterioro orgánico cerebral potencialmente reversibles ................589 26. Cuadros deficitarios de funciones cerebrales superiores ..................................590 27. Demencia ..........................................................................................................591 28. Distonía aguda por neurolépticos ......................................................................593 29. Enfermedad de Parkinson y parkinsonismos ....................................................594 30. Examen de Líquido Cefaloraquídeo ..................................................................595 31. Hemorragia subaracnoídea ...............................................................................598 32. Hérpes zóster ....................................................................................................600 33. Lumbociáticas y cervicobraquialgias .................................................................601 34. Movimientos anormales inducidos por fármacos ..............................................603 35. Papiloscopía ......................................................................................................605 36. Parálisis facial periférica....................................................................................606 37. Paraplejia aguda ...............................................................................................608 38. Polineuropatías, radiculopatías, mononeuropatías ...........................................609 39. Síndrome atáxico ..............................................................................................611 40. Síndrome cuadrapléjico agudo ..........................................................................612 41. Síndrome cuadripléjico fláccido: polirradiculoneuritis aguda o 613 guillain-barré (sgb) ............................................................................................613 42. Síndrome encefalítico........................................................................................614 43. Síndromes convulsivos somáticos ....................................................................615 44. Síndromes vertiginosos centrales .....................................................................617 45. Síndromes vertiginosos periféricos ...................................................................619 46. Temblor esencial ...............................................................................................621

47. Trastornos del sueño.........................................................................................622 48. Traumatismo encéfalo-craneano grave (TEC grave) ........................................624 49. Traumatismo encéfalo-craneano leve (TEC leve) .............................................626 50. Traumatismo raquimedular ...............................................................................628 51. Tumores de la region selar................................................................................630

Reumatología

PÁGINA 632

1. Artritis Reumatoídea..........................................................................................632 2. Fibromialgia .......................................................................................................634 3. Síndrome De Hombro Doloroso ........................................................................635 4. Lumbago Mecánico ...........................................................................................637 5. Lupus Eritematoso Sistémico ............................................................................639 6. Monoartritis .......................................................................................................641 7. Oligoartritis ........................................................................................................642 8. Osteoporosis .....................................................................................................643 9. Síndrome De Túnel Carpiano ............................................................................645 10. Tendinopatías y Bursitis ....................................................................................646 11. Vasculitis Sistémicas .........................................................................................648 12. Artritis Séptica ...................................................................................................651 13. Gota Aguda .......................................................................................................653 14. Poliartritis ..........................................................................................................655 15. Síndrome De Sjögren ........................................................................................657 16. Síndrome Antifosfolípido (SAAF).......................................................................659 17. Lumbago Infeccioso y Tumoral .........................................................................661

18. Artritis psoriásica ...............................................................................................663 19. Artritis reactivas .................................................................................................665 20. Artrosis ..............................................................................................................666 21. Cervicalgia ........................................................................................................667 22. Columna dolorosa .............................................................................................669 23. Condrocalcinosis ...............................................................................................671 24. Epicondilalgia, epitroclealgias ...........................................................................672 25. Esclerosis sistémica progresiva ........................................................................673 26. Osteodistrofia en insuficiencia renal crónica .....................................................674 27. Pelviespondilopatías seronegativas ..................................................................675 28. Polimiositis, dermatomiositis .............................................................................676

MÓDULO 1: Medicina Interna

Compendio Cardiología Angina Crónica Estable

Etiología-Epidemiología-Fisiopatología La ateroesclerosis de las arterias coronarias (enfermedad coronaria) es la causa más frecuente de angina crónica estable. En este caso, la placa ateromatosa constituye una obstrucción fija del flujo sanguíneo coronario, disminuyendo la oferta de O2. Por mecanismos locales de autoregulación, la arteria coronaria afectada se vasodilata a distal, aumentando el flujo sanguíneo, pero disminuyendo su reserva coronaria. De esta manera, un paciente que en reposo es asintomático y ante un aumento de la demanda (por ejemplo esfuerzo físico), será incapaz de aumentar más la oferta de O2 (depleción de reserva coronaria), apareciendo isquemia y con ella, angina. La enfermedad coronaria es la 1° causa de muerte en los países industrializados y también en Chile, y en forma concordante, la angina crónica estable es causa frecuente de ingreso a servicios de cardiología. Se asocia a varios factores de riesgo cardiovascular tales como: tabaquismo, hipertensión arterial, diabetes mellitus, dislipidemia, sedentarismo, síndrome metabólico, entre otros.

Nivel de manejo del médico general: Diagnóstico Específico Tratamiento Inicial Seguimiento Completo

Aspectos Esenciales • • • • •

El diagnóstico es clínico. Confirmación con estudios no invasivos, test de esfuerzo primera elección. Antiagregantes, estatinas, betabloqueadores, aumentan la sobrevida y DEBEN utilizarse. Manejo sintomático: nitratos, betabloqueadores y bloqueadores de canales de calcio. Coronariografía indicada en pacientes de alto riesgo o en estudios no invasivos positivos.

Caso Clínico Tipo

Diagnóstico

Varón de 54 años, DM2, dislipidemia y tabaquismo activo. Presenta desde hace 4 meses dolor precordial leve al subir escaleras o caminar rápido, que cede rápida y completamente con reposo. Refiere “saber cuándo le va a doler el corazón”.

El diagnóstico es clínico, confirmándose con estudios no invasivos. Clínica • Malestar o dolor torácico anterior (retroesternal o precordial). • Puede irradiarse a hombros, brazos, manos y dedos, espalda e incluso epigastrio y mandíbula • Transitorio, de corta duración (no más de 5 minutos), que inicia y culmina progresivamente. • Desencadenado por estrés físico y/o emocional, comidas copiosas, frío. • Desaparece paulatinamente con reposo o nitratos. • Puede acompañarse de disnea o síntomas neurovegetativos (náuseas, vómitos, sudoración, etc.).

Definición Es una de la expresiones clínicas de isquemia miocárdica crónica. Es un dolor torácico retroesternal opresivo cuyas características (factores desencadenantes y calmantes, duración, intensidad y síntomas asociados) son relativamente estables en el tiempo.

Algunos pacientes (ejemplos: diabéticos y/o añosos), pue-

1 Facultad de Medicina, Universidad de Chile

den no presentar angina típica frente a isquemia miocárdica, manifestando por el contrario nauseas o disnea (“equivalente anginoso”). Estudios no invasivos: La radiografía de tórax y el ECG (en reposo) pueden ser normales o mostrar alteraciones inespecíficas. El test de esfuerzo (ergometría) permite objetivar la aparición de síntomas o alteraciones en el ECG desencadenadas por el esfuerzo físico; también permite valorar la capacidad funcional, la respuesta de la presión arterial y la aparición de arritmias. • •



Seguimiento Principalmente el control de los síntomas, con énfasis en la aparición de inestabilidad (cambios en duración, intensidad, magnitud del desencadenante, etc.) así como en la frecuencia de presentación y severidad de los episodios, además de la pesquisa y tratamiento de factores de riesgo cardiovascular.

La ergometría es clínicamente positiva si aparece angina durante la prueba. La prueba es eléctricamente positiva si aparece infradesnivel de ST rectilineo o descendente > 1,5 mm. Si aparece SDST (poco frecuente), debe ser en derivaciones que no tengan onda q patológica en reposo. Los cambios de la onda T no tienen valor diagnóstico.

Autor / Editor Victor Valdés

El ecocardiograma de estrés también reproduce la isquemia miocárdica, y se considera positivo ante la aparición de angina o alteraciones de la motilidad. Su gran limitante es que es operador dependiente. Estudios invasivos: Corresponde a la coronariografía, la cual permite visualizar la anatomía coronaria mediante la inyección de medio de contraste. Está indicada cuando los métodos diagnósticos no invasivos son positivos mostrando alto riesgo (isquemia a baja carga), o son no concluyentes frente a una alta sospecha clínica, o cuando el paciente no responde al tratamiento médico o en pacientes de alto riesgo de enfermedad coronaria.

Tratamiento El tratamiento médico tiene como objetivos: • Aumento de sobrevida • Aumento calidad de vida No farmacológicos: • Cambios en el estilo de vida: 1. Dieta cualitativa basada en la pirámide alimenticia de la OMS, con variaciones según antecedentes del paciente. 2. Ejercicio físico aeróbico, debe ser indicado caso a caso. Farmacológico. • Antiagregantes plaquetarios (AAS o clopidogrel), estatinas, betabloqueadores. Control de síntomas: Nitratos, bloqueadores de canales de calcio, betabloqueadores. En caso de falla del tratamiento médico o en paciente de alto riesgo (miocardio en riesgo) se puede proceder a tratamientos invasivos: • Revascularización percutánea: Principalmente mediante intervención coronaria percutánea (PCI). • Cirugía de revascularización coronaria (By-pass coronario).

2 Facultad de Medicina, Universidad de Chile

Año 2016

Bloqueos Aurículo-Ventriculares

50 lpm. Toma ECG, que muestra PR prolongado hasta que uno no conduce, con su diagnostico la deriva al especialista.

Nivel de manejo del médico general: Diagnóstico Específico Tratamiento Inicial Seguimiento Derivar

Definición Trastornos de la conducción del impulso cardiaco a través del NAV y/o del Haz de His.

Aspectos Esenciales • • •

Etiología-Epidemiología-Fisiopatología

De alta prevalencia en la población. Causa más frecuente de instalación de marcapasos definitivo. El diagnostico es mediante ECG. El bloqueo AV puede tener tres grados, diferenciados mediante ECG, con distintos manejos cada uno.

La causa puede ser irreversible o reversible, aumenta su prevalencia con la edad. Dentro de las irreversibles lo más común es la fibrosis idiopática del sistema éxito-conductor. También es una causa importante el daño estructural en contexto de insuficiencia cardíaca, independientemente de la causa de esta. Dentro de las reversibles están los trastornos electrolíticos (hiperkalemia e hipermagnesemia) y fármacos (Betabloqueo, digitálicos y antiarrítmicos), infecciones (miocarditis aguda, fiebre reumatica.

Caso Clínico Tipo Mujer de 68 años, es traída al servicio de urgencias por familiares quienes la encontraron en el suelo, la paciente no recuerda el episodio, su único hallazgo al examen es FC de

Clasificación

3 Facultad de Medicina, Universidad de Chile

Diagnóstico Se debe sospechar en contexto de un paciente con una bradiarritmia sintomática: Anamnesis: Interrogatorio dirigido a sintomas como: Dolor de tórax, disnea, fatiga, o episodios de síncope o pre síncope a repetición. Examen físico: Enfocado en búsqueda de signos como: Palidez, diaforesis, signos de mala perfusión periférica, compromiso de conciencia, signos de EPA, Hipotensión. Se confirma con un electrocardiograma, donde según lo relatado anteriormente es probable que encontremos una Bloqueo AV asociado a frecuencias cardiacas bajas (<60-50 lpm). Puede ser también un hallazgo en paciente asintomático.

Tratamiento Evaluar causa subyacente y tratarla. De no ser así dependerá del tipo de BAV: 1° grado y 2° grado Mobitz I, pueden ser seguidos y con control periódico, evitando fármacos favorecedores. Los de 2° grado Mobitz II y 3° grado tienen indicación de marcapaso definitivo. Si la bradicardia fuese sintomática o con compromiso hemodinámico, se puede instalar un marcapaso transitorio o drogas intravenosas en infusión continua como Isoproterenol o atropina (en general tienen mala respuesta).

Seguimiento Por parte del especialista, se debe monitorizar el ritmo cardíaco y si es que existe progresión del bloqueo, indicar marcapaso definitivo.

Autor / Editor Karim Gallardo

Año 2016

4 Facultad de Medicina, Universidad de Chile

Endocarditis Bacteriana Sub Aguda Nivel de manejo del médico general: Diagnóstico Sospechar Tratamiento Inicial Seguimiento Derivar

oclusiones hacia distal, provocando infartos. Esta infección suele destruir las estructuras valvulares, desencadenando problemas hemodinámicos y compromiso de otros tejidos cardíacos. En casos en donde la infección se mantenga por semanas o meses; puede observarse una respuesta inmunológica intensa, provocando un daño de tipo autoinmune en otros parénquimas.

Aspectos Esenciales

Diagnóstico

• •

Se sospecha con la anamnesis y el examen físico. Los síntomas de EBSA pueden ir cambiando durante el tiempo tales como fiebre, escalofríos y sudoración nocturna (que orientan a infección sistémica). Malestar general, astenia y adinamia son frecuentes. El paciente también suele referir sentirse en un “estado gripal”, con pérdida de peso, mialgias, artralgias y lumbago.

• •

Agente más frecuente: S. viridans Hemocultivo y ETE confirman el diagnóstico y guían tratamiento. Tratamiento con PNC sódica + gentamicina por 4 semanas. Profilaxis posterior previa a procedimientos dentales con Amoxicilina por via oral.

La afectación intravascular es la que provoca los síntomas de la insuficiencia cardíaca izquierda o derecha, las manifestaciones embólicas en diversos territorios (SNC, abdomen, extremidades, etc).

Caso Clínico Tipo Paciente varón de 50 años con antecedente de enfermedad reumática, presenta cuadro de inicio insidioso de dos semanas, caracterizado por CEG, artralgias, esplenomegalia, pulsos distales disminuidos, fiebre y un soplo de regurgitación en el foco mitral.

Criterios de Duke modificados: Criterios Mayores: • Cultivos de sangre positivos para Endocarditis Bacteriana. • Evidencia de compromiso endocárdico (ecografía positiva para Endocarditis o nueve regurgitación valvular).

Definición Es una enfermedad infecciosa que afecta al miocardio, la cual tiene como forma una presentación clínica que se mantiene por semanas o meses y generalmente es causada por gérmenes de poca virulencia como S. viridans, estafilococos, entrococos y rara vez Gram (-).

Criterios Menores • Lesión cárdica predisponerte, uso de drogas inyectables. • Fiebre: temperatura > 38º C. • Fenómenos vasculares. • Fenómenos inmunológicos. • Evidencia microbiológica (que no cumpla con criterio mayor)

Etiología-Epidemiología-Fisiopatología

Endocarditis bacteriana en presencia de: 2 criterios mayores, 1 criterio mayor y 3 menores o 5 criterios menores.

Hay un aumento en la edad promedio de los pacientes, con un promedio de 50 años y una mayor proporción de hombres (aproximadamente 2/3). Se produce principalmente en válvulas previamente dañadas. Las lesiones predisponentes son principalmente secuelas de endocarditis reumática, válvula aórtica biscúspide congénita y lesiones endocárdicas parietales por impacto del chorro sanguíneo, preferentemente en defectos septales ventriculares e insuficiencia mitral y aortica. Tiene un curso arrastrado, la puerta de entrada casi siempre pasa inadvertida. Produce con frecuencia graves deformaciones de los velos.

Paciente tipo: • Antecedente de enfermedad reumática, cirugía cardiaca, procedimiento dental, valvulopatía o cardiopatía • Paciente joven usuario de drogas IV Paciente FOD con: • Soplo cardiaco • Fenómenos vasculares: Hemorragia conjuntival, manchas de Janeway, émbolos a distancia, aneurismas micóticos • Fenómenos inmunológicos: Nódulos de Osler, Manchas de Roth, glomerulonefritis Destacar que estos últimos fenómenos son mas frecuentes en Endocarditis Aguda. Se confirma con el hemocultivo (3 son suficientes para aislar el germen causal) y la ecocardiografía transesofágica (ETE),

En lo fisiopatológico, esta enfermedad se gesta a partir de una colonización bacteriana de estructuras cardíacas con endotelio dañado, cuerpos extraños intracardíacos, entre otros. La colonización, que se produce principalmente en las válvulas, al desarrollarse, genera la formación de vegetaciones, la cuales pueden fragmentarse y generar émbolos, causando

5 Facultad de Medicina, Universidad de Chile

ambos guían el tratamiento. Exámenes complementarios como hemograma, orina completa, creatininemia, búsqueda de complejos inmunes circulantes son útiles. El electrocardiograma debe registrarse al ingreso y repetirlo en el curso de la enfermedad, la radiografía de tórax puede evidenciar ICC. Permiten descartar los diagnósticos diferenciales que puede adoptar como fiebre de origen desconocido, debe descartarse fiebre reumática, osteomielitis, TBC, meningitis, abscesos cerebrales, AVE, malaria, pericarditis aguda, vasculitis lupus y coagulación intravascular diseminada.

Tratamiento Depende del agente etiológico. En el caso del Streptococo viridans: Penicilina sódica 5 millones cada 6 horas e.v. por 4 semanas solo o en combinación con Gentamicina 1 mg/ kg cada 8 horas durante las dos primeras semanas. Como alternativa se utiliza Ceftriaxona 2 g/día por 4 semanas o Vancomicina 15 mg/kg EV sin exceder 1 g cada 12 horas por 4 semanas.

Seguimiento Es tarea del especialista, debe evaluarse la respuesta al tratamiento antimicrobiano y la posibilidad de resolución quirúrgica. Por otra parte hay que realizar profilaxis en procedimientos dentales, en ciertas condiciones, con 2 g de amoxicilina v.o. previo al procedimiento.

Autor / Editor Marcelo Fres

Año 2016

6 Facultad de Medicina, Universidad de Chile

Estenosis Aórtica

Diagnóstico Triada Clínica: Angina-Síncope-Disnea. Otros síntomas pueden ser mareos, palpitaciones o síntomas de insuficiencia cardiaca. Al examen físico, EA leve puede encontrarse un click de eyección en área aórtica principal y un soplo sistólico de eyección leve o moderado. En EA moderada-severa: Pulso arterial parvus y tardus, Presión diferencial disminuida, onda “a” predominante en el pulso venoso, R2 apagado y retardado, Cuarto ruido, Soplo Sistólico de eyección en área aórtica aspero, romboidal, de baja frecuencia que en el área Mitral se escucha más como de regurgitación, Soplo Diastólico aspirativo y que se irradia a ambas carótidas.

Nivel de manejo del médico general: Diagnóstico Específico Tratamiento Inicial Seguimiento Derivar

Aspectos Esenciales • • •

Diagnóstico clínico y ecocardiográfico. Tratamiento definitivo: Quirúrgico. Pronóstico se relaciona con la aparición de síntomas y con el grado de compromiso de la función del Ventrículo Izquierdo.

En el Electrocardiograma es habitual la presencia de hipertrofia ventricular, la que puede estar oculta por la presencia de Bloqueo de Rama Izquierda.

Caso Clínico Tipo

En la Radiografía de tórax puede haber dilatación de la raíz aórtica y discretos signos de crecimiento del ventrículo izquierdo.

Paciente de 65 años que consulta por sincope durante actividad física en tres oportunidades con antecedentes de hipertensión no controlada, al examen se constata soplo eyectivo aórtico. ECG muestra signos de Hipertrofia ventricular y sobrecarga, sin arritmias.

El Ecocardiograma permite apreciar el grado de hipertrofia ventricular y la presencia de engrosamiento y calcificación de los velos aórticos. Mediante el uso de Doppler se puede calcular el gradiente valvular a partir de la velocidad del flujo. Estudio hemodinámico y angiográfico: En caso de angina.

Definición

Tratamiento

Cambios patológicos, fisiopatológicos y clínicos que se asocian a la disminución del área valvular aórtica a menos de 2 cm2 con obstrucción del flujo de salida del ventrículo izquierdo. Significa un aumento de la gradiente transvalvular, lo que ocaciona como consecuencia una sobrecarga crónica de presión sobre el ventrículo izquierdo con la formación de una hipertrofia cardiaca concéntrica secundaria.

Independiente de su severidad, debe incluir prevención de endocarditis infecciosa y eventualmente de Fiebre Reumática. En los pacientes con una estenosis significativa (gradiente medio > 40 mmHg.) asintomática, debe limitarse los esfuerzos físicos y establecerse un programa de controles cada 6 a 12 meses. En los pacientes con estenosis aórtica significativa sintomática, está indicada la cirugía de reemplazo valvular. En algunos pacientes poco sintomáticos puede plantearse la cirugía cuando el gradiente transvalvular es muy importante (gradiente medio > 64 mmHg ).

Etiología-Epidemiología-Fisiopatología Existe un obstáculo al flujo de salida del ventrículo izquierdo en sístole. En pacientes menores de 70 años las causas en orden de frecuencia son: • Válvula bicúspide calcificada. • Secundaria a enfermedad reumática (postinflamatoria). • Degenerativa. En pacientes mayores de 70 años las causas en orden decreciente son: • Degenerativa • Calcificación de una válvula bicúspide • Postinflamatoria.

Seguimiento Derivar a especialista.

Autor / Editor Karim Gallardo

7 Facultad de Medicina, Universidad de Chile

Año 2016

Estenosis Mitral

cha, manifestaciones embolicas (ACV) o edema pulmonar agudo. Al examen hay un soplo en el foco mitrálico, mesodiastólico “en rodada”, con refuerzo présistólico (si presenta ritmo sinusal) y chasquido de apertura {RUFFF ta ta}.

Nivel de manejo del médico general: Diagnóstico Específico Tratamiento Inicial Seguimiento Derivar

El diagnóstico se confirma con Ecocardiograma transtorácico. Para definir terapia invasiva es absolutamente necesario el ecocardiograma transesofágico. ECG nos muestra crecimiento de AI, a veces sobrecarga del VD o bloqueo de rama derecha. Radiografía de Tórax permite evaluar crecimiento AI, AD y VD, así como puede observarse calcificación intraauricular y líneas B de Kerley. (criterios de severidad).

Aspectos Esenciales • • •

La causa más importante es la enfermedad reumática. El hallazgo que más la sugiere es el soplo meso diastólico en rodada, con chasquido de apertura. En caso de FA o trombo.

Tratamiento Se basa en la prevención de la enfermedad reumática, con el uso de penicilina. Una vez establecida la enfermedad tenemos el manejo conservador (pacientes asintomáticos y/o con área >1,5 cm2), que involucra restricción de sodio, uso de B bloqueo en casos indicados, anticoagulación en caso de FA, trombo intracavitario o embolía. El manejo invasivo (que puede ser quirúrgico o percutáneo), esta indicado en pacientes sintomáticos según el área valvular:

Caso Clínico Tipo Mujer de 35 años, embarazada de 28 semanas, comienza con disnea progresiva y edema de extremidades inferiores, se le añade en el último día esputo sanguinolento, por lo cual consulta. En el examen físico encuentra un soplo diastólico y un chasquido de apertura en el foco mitral.



Definición



Obstrucción del tracto de salida del ventrículo izquierdo debido a una anomalía estructural de la válvula mitral con reducción del área de la válvula menor de 2 cm2.

AVM ≤ 1 cm2 . Cirugía o Balonplastía percutánea (BPP), según anatomía valvular. AVM entre 1.1 y 1.5 cm2. Si anatomía es favorable, se intentará BPP. De lo contrario, dar 6 meses de tratamiento médico y reevaluar CF. Si al cabo de 6 meses está en CF II, III o IV, debe ir a cirugía. En caso de tener presión sistólica de arteria pulmonar en reposo > 50 mmHg, no esperar los 6 meses y va a cirugía.

Etiología-Epidemiología-Fisiopatología Seguimiento

La principal etiología es la enfermedad reumática (95%), y luego la calcificación del anillo mitral en adultos mayores. Preferentemente en mujeres, con una relación de género 2:1. En la ER se produce una reacción autoinmune, mediada por anticuerpos dirigidos contra el S.pyogenes, lo que produce fusión de las comisuras de la válvula mitral, lentamente progresiva y rigidez de los velos de la válvula, todo lo que contribuye a la reducción del área valvular. Esta estenosis lleva a un aumento de la presión y del tamaño de la aurícula izquierda, generando aumento de presión capilar pulmonar, grados variables de hipertensión pulmonar y falla cardiaca derecha en los casos más avanzados. Además, por la remodelación de la AI se puede producir FA. Las alteraciones progresan lentamente.

Una vez efectuado el diagnóstico el seguimiento y la decisión de resolución quirúrgica son por parte de especialista.

Autor / Editor Karim Gallardo

Diagnóstico La sospecha es clínica, en mujeres preferentemente y en paciente con el antecedente de ER en la infancia. Clínica: Cuadro larvado de disnea de esfuerzos progresiva, palpitaciones, hemoptisis, edema bimaleolar por falla dere-

8 Facultad de Medicina, Universidad de Chile

Revisor Formato Diego Ugalde

Año 2016

Fibrilación Auricular Crónica • Nivel de manejo del médico general: Diagnóstico Específico Tratamiento Inicial Seguimiento Completo

patía estructural lleva a remodelación del tejido auricular, lo que facilita y perpetúa FA. Automatismo anormal: Aumento de descarga a una frecuencia rápida por periodo refractario más corto, el tejido auricular no es capaz de conducir en forma homogénea y se produce dispersión de refractariedad y la fibrilación.

Aspectos Esenciales

Diagnóstico

• • • •

La sospecha es fundamentalmente clínica y se confirma con ECG. Cuadro Clínico: Palpitaciones (Regulares o Irregulares), Disnea, sincope recurrente, ACV criptogénicos, descompensación de patología de base, etc. Factores de Riesgo: 1. Precipitantes (OH, Ejercicios, Stress). 2. Comorbilidad (HTA, E. Ao, IC, EAO, ACV,DM, EPOC, Alt. tiroideas). 3. Hábitos (TBQ, OH, Drogas ilícitas). 4. Antecedentes familiares FA. ECG 12 Derivaciones: Evaluar alteraciones estructurales (IAM, HVI, Bloqueo de rama, pre-excitación). Ritmo irregular, ausencia de onda P. Evaluación de Riesgo de Embolia y necesidad de ACO: Utilización de escala CHA2DS2-VASC.

• •

Arritmia sostenida más frecuente. ECG: Ritmo irregularmente irregular sin onda P. Clínica: Palpitaciones. Tratamiento: Lo esencial es la anticoagulación y control de frecuencia. FA Persistente: Anticoagulación, control del ritmo o de la frecuencia. FA permanente: Control de frecuencia y anticoagulación.

Caso Clínico Tipo Paciente sexo femenino, 65 años, desde hace 2 años presenta pulso irregular, con palpitaciones ocasionales que la han llevado a policonsultar al SU. Antecedentes de IC e HTA en tratamiento. Al ECG: Taquicardia irregular con ausencia de onda P.

Tratamiento Definición

Objetivos del tratamiento: Reducir los síntomas y prevenir las complicaciones. Posee tres componentes: Control de Frecuencia, Control de Ritmo, Manejo antitrombótico.

Taquiarritmia irregular, con ausencia de onda P, de origen supraventricular, con ciclo auricular generalmente menor de 200 ms. La FA crónica puede clasificarse en: 1. Persistente: Episodio de > 7 días de duración o que requiere de cardioversión eléctrica o farmacológica. 2. Permanente: FA sin lograr cardioversión, por lo tanto no se utilizan estrategias para el control del ritmo.

1. 1. FA Persistente: Pueden ser abordados con control del ritmo o control de frecuencia, sin diferencias en la mortalidad entre una u otra estrategia. • Control del ritmo: Fármacos anti arrítmicos o cardioversión eléctrica. Si se logra cardiovertir exitosamente y en general en un primer episodio, beta bloqueadores para mantener el ritmo, de no ser así, anti arrítmicos clase Ic, la o amiodarona son útiles en caso de que el paciente no tenga o tenga patología cardiaca estructural respectivamente. • Control de frecuencia: Tratar como FA permanente.

Etiología-Epidemiología-Fisiopatología Etiología: Puede ser idiopática o en contexto de factores de daño endotelial y cardiopatía estructural. Epidemiologia: Corresponde a la arritmia sostenida más frecuente. Prevalencia: 1-2 % población general. Incidencia en aumento, asociada a un aumento de morbilidad y mortalidad cardiovascular. Prevalencia aumenta con edad y sexo masculino.

2. FA Permanente: Controlar frecuencia cardiaca, ya que por definición no se ha logrado la conversión a ritmo sinusal, teniendo como meta 60-80 lpm en reposo y 90115 en ejercicio moderado. Iniciar con monoterapia con beta bloqueador o bloqueador canal calcio. Si no se controla, adicionar digoxina durante las actividades cotidianas. Amiodarona en caso de no controlar frecuencia con 2 fármacos. • Anticoagulación: Indicado en pacientes con riesgo de tromboembolismo (CHA2DS2-VASC: ICC, HTA, edad > 75, DM, ACV, EVP, edad 65-74, sexo femenino; 2 puntos o mas es indicativo de TACO, con un punto TACO o AAS,

Fisiopatología: Los mecanismos electrofisiológicos que generan FA son la microreentrada y el automatismo aumentado. Ambos contribuyen a la remodelación eléctrica y estructural, permitiendo la autoperpetuación. • Microreentrada: Por heterogeneidad de las propiedades electrofisiológicas en la aurícula. La presencia de cardio-

9 Facultad de Medicina, Universidad de Chile



0 puntos Nada o AAS). También debe evaluarse el riesgo de sangrado (HA1-2SBLED1-2: HTA, Alteraciones renales o hepáticas, ACV, Sangrado, INR lábil, edad > 65, uso de drogas o alcohol; 3 o más es alto riesgo). El TACO puede utilizar Inhibidores de la Vitamina K [Acenocumarol, Warfarina] o inhibidores de la trombina [Dabigatran] para llegar a un INR de 2-3 como rango terapéutico. Ablación con radiofrecuencia: Especialmente útil en FA persistente o permanente con focos automáticos alrededor de venas pulmonares, y en pacientes con falla cardiaca cuando la medicación antiarritimica ha fracasado (incluyendo Amiodarona).

Seguimiento Control clínico cada 12 meses como mínimo. Evaluar frecuencia cardíaca, situación funcional, control analítico con pruebas hepáticas, de función renal y de función tiroidea (en uso de amiodarona) y en usuario de TACO, control mensual de INR para mantener 2-3.

Autor / Editor Julián Bravo

Año 2016

10 Facultad de Medicina, Universidad de Chile

Fibrilación Auricular Paroxística

ciencia, mala perfusión periférica, hipotensión, disnea por insuficiencia cardiaca, o angina. La pausa que sigue a la FA paroxística puede provocar síncope.

Nivel de manejo del médico general: Diagnóstico Específico Tratamiento Inicial Seguimiento Derivar

ECG: actividad auricular rápida y desorganizada, con ausencia de onda P, sustituida por ondulaciones de la línea de base, frecuencia auricular de 350-500 lpm. Respuesta ventricular totalmente irregular.

Aspectos Esenciales • • • •

Tratamiento

ECG: taquiarritmia con ausencia de onda P. FA paroxística: autolimitado por 7 días. Sospechar en palpitaciones con pulso irregular. Ejes del tto: control de frecuencia, del ritmo y anticoagulación.

Enfrentamiento general de pacientes con FA (En debut de FA paroxística): 1. Toda FA hemodinámicamente inestable: Cardioversión eléctrica urgente (Corriente directa sincronizada de 200 a 360 J). Otras indicación de CVE es FA con pre-excitación. Criterios AHA de Taquiarritmia inestable: • Compromiso de conciencia • Signos de shock • Disnea • Angina • Signos de insuficiencia cardiaca aguda

Caso Clínico Tipo Paciente sexo masculino, 45 años. Comienza en forma brusca hace 6 horas con palpitaciones de alta frecuencia. Pulso irregular, hemodinámicamente estable. Se solicita ECG: taquicardia con ausencia de onda P, respuesta ventricular rápida (300lpm).

2. Las FA hemodinámicamente estables: 2.1 Control de Frecuencia (Objetivo: 60-80, 90-115 en ejercicio lpm): Betabloqueadores [Atenolol, Propanolol] o Bloqueadores de canal de calcio [Verapamilo, Diltiazem]. Amiodarona se usará excepcionalmente en pacientes con IC descompensada, por riesgo de convertir a RS y embolía posterior. 2.2 Control del Ritmo: En FA que se presenta en menos de 48 hrs: cardioversión (eléctrica o farmacológica según corresponda). Cardioversión farmacológica: con Antiarritmicos clase IC [Propafenona, Flecainida] o Antiarritmicos clase III [Amiodarona]. En la FA que lleva más de 48 hrs (o tiempo de evolución desconocido): TACO por 3 semanas y luego cardioversión eléctrica guiada por ETE, seguido de TACO por 4 semanas o indefinido según riesgo embolígeno. Si hay trombos, diferir por al menos 3 meses cualquier intento de Cardioversión. 2.3 Anticoagulación: Hay 2 puntajes para evaluar esto, el CHADS2y el CHA2DS2-VASC. (C = Insuf Cardiaca o FEVI < 0.4; H= HTA, A= edad, que es 1 en CHADS2 y en CHA2DS2-VASC se divide en 1 pto entre 65 y 74 años y 2 ptos en 75 o más; D = diabetes; S = stroke, 2 puntos; VA= compromiso vascular periférico (ATE carotídea o Aórtica; Enfermedad arterial oclusiva); SC= condición de sexo, Femenino). Entonces, si el sujeto tiene ≥ 2 puntos CHADS2 , tiene indicación de TACO indefinido. Si tiene CHADS2 = 0 o 1, se analiza el otro puntaje CHA2DS2VASC. Así, si con este último el paciente tiene ≥ 2, necesita TACO, y si puntúa 0 no necesita nada o sólo Aspirina; y si tiene 1 puede quedar sólo con AAS. Se pueden utilizar Inhibidores de la Vit.K [Acenocumarol, Warfarina] para llegar a INR de 2-3 como rango terapéutico.

Definición Taquiarritmia supraventricular, irregular, sin onda P, que implica una pérdida de la contracción auricular efectiva. Se denomina FA Paroxística cuando es autolimitado, usualmente dura menos de 48 hrs pudiendo continuar por 7 días máximo.

Etiología-Epidemiología-Fisiopatología Etiología y Fisiopatología: La FA puede ser generada por dos mecanismos: microreentrada y el automatismo aumentado. Suele desencadenarse por estrés emocional, quirúrgico, alteraciones metabólicas o hemodinámicas, hipertiroidismo, etc. FA focal: pacientes jóvenes sin cardiopatía estructural con múltiples episodios de FA paroxística, por focos automáticos en aurícula alrededor de venas pulmonares. Epidemiología: Arritmia extremadamente frecuente, particularmente en su forma paroxística no recurrente. Prevalencia en población general de un 0,4%, que se eleva hasta el 10% en >80 años. El 12% de ellas se instala en corazón sano.

Diagnóstico Clínico y Electrocardiográfico. Clínica: La principal manifestación son las palpitaciones. Si la respuesta ventricular es excesiva: compromiso de con-

11 Facultad de Medicina, Universidad de Chile

Clasificación

Seguimiento

2.4 Terapias Ablativas por catéter se deben considerar en: (1) Pacientes con FA paroxística sintomáticos donde la terapia farmacológica ha fracasado o (2) Pacientes con falla cardíaca cuando la medicación antiarrtimica ha fracasado incluyendo amiodarona.llegar a INR de 2-3 como rango terapéutico.

Control cada 6 meses como mínimo y antes si se presentan síntomas. Control mensual del INR en pacientes en TACO. Derivar a urgencias los casos con síncopes o muy sintomáticos y que necesitan control urgente de la frecuencia cardíaca. Considerar interconsulta a cardiología en: <50 años, dificultades para su clasificación, tomar decisión de control de la frecuencia o del ritmo, sospecha de enfermedad valvular o disfunción sistólica, sospecha síndrome de Wolf-Parkinson-White, la mayoría de los casos de FA paroxística para valoración de tratamiento de control del ritmo.

Objetivo tratamiento: • Reducción de los paroxismos y mantención a largo plazo del ritmo sinusal. •

En los pacientes con pocos o infrecuentes episodios de FA o pocos síntomas, estrategia sin fármacos o esquema “pill-in-the-pocket”, utilizando beta bloqueadores y anti arrítmicos de la clase Ic (flecainide o propafenona), si no hay contraindicaciones.



Tratamiento inicial para pacientes con paroxismos sintomáticos frecuentes (con o sin enfermedad cardiaca estructural) son los beta bloqueadores. Si no son efectivos o tolerados, considerar anti arrítmicos de la clase Ic. Como tercera opción Amiodarona o tratamientos no farmacológicos (ablación).



Siempre al diagnóstico descartar hipertiroidismo, intoxicaciones por fármacos, alcohol u otros.

Autor / Editor Karim Gallardo

12 Facultad de Medicina, Universidad de Chile

Año 2016

Hipertensión Arterial Esencial

Diagnóstico El diagnóstico se consigue con al menos dos mediciones adicionales de presión en cada brazo, separados al menos en 30 segundos, en días distintos y en un lapso no mayor a 15 días. Se clasificará como hipertenso a personas cuyo pro-

Nivel de manejo del médico general: Diagnóstico Específico Tratamiento Completo Seguimiento Completo

medio de mediciones sea mayor a 140/90 mmHg.

Aspectos Esenciales • • • • • •

Tratamiento

Tratamiento no farmacológico • Dieta y ejercicio: Alimentación sana e hipocalórica. Ejercicios aeróbicos durante 30-60 minutos, 3-5 veces a la semana. • Reducción ingesta de sal: 6gr/día. • Reducción del consumo de alcohol. • Reducción del consumo de café: 4 tazas máximo. • Dejar de fumar. • Terapias de relajación.

Principal factor de riesgo para enfermedad cerebrovascular y enfermedad coronaria. Todo paciente con HTA debe ser clasificado según su riesgo cardiovascular (RCV), con tal de graduar la intensidad de las intervenciones. El objetivo del tratamiento es lograr una normotensión y el control de todos los factores de RCV modificables. Todo paciente con PA >160/100 mmHg y RCV alto o muy alto se beneficia de tratamiento farmacológico combinado desde el inicio. Pacientes <55 años con RCV bajo o moderado, iniciar con IECA, ARA II o betabloqueadores. Pacientes > 55 años, iniciar con calcio antagonistas o diuréticos.

Tratamiento farmacológico Se debe iniciar el tratamiento farmacológico en personas con una presiones mayores a 160/100 mmHg y aquellas con RCV alto o muy alto. En pacientes hipertensos con RCV bajo, es posible iniciar cambios en el estilo de vida y esperar 3 meses antes de iniciar el tratamiento farmacológico. Si después de 3 meses la PA no se normaliza, se debe iniciar la terapia farmacológica. Se puede iniciar monoterapia en pacientes hipertensos con RCV bajo o moderado. Se debe utilizar terapia combinada en pacientes con PA mayor a 160/100 mmHg y en aquellos con RCV alto o muy alto.

Caso Clínico Tipo Paciente asintomático, acude a control, en quién se encuentran cifras de presión arterial elevadas.

Definición

Pacientes hipertensos con RCV promedio o bajo y sin comorbilidad: En <55 años los beta bloqueadores, IECA o ARA II serían de mejor elección En > 55 años iniciar con calcioantagonistas o diuréticos tiazídicos cuando hay predominio de hipertensión diastólica y sistólica respectivamente. Si luego de 3 meses no se logran las metas terapéuticas se deberá agregar un 2º fármaco de mecanismo de acción diferente y complementario (ver tabla 1).

Elevación persistente de la presión arterial sobre 140/90 mmHg, en donde el mecanismo inicial del proceso se desconoce.

Etiología-Epidemiología-Fisiopatología La HTA es la principal causa de muerte a nivel mundial, se asocia estrechamente con el tabaquismo y la hipercolesterolemia. Es el principal factor de riesgo para enfermedad cerebro vascular y enfermedad coronaria. La HTA aumenta según edad. Es más frecuente en hombres ( 28,7% vs 25,3%). Los niveles elevados de presión arterial producen cambios estructurales en el sistema arterial, lo que afecta a órganos como cerebro, corazón, riñón, determinando las principales complicaciones: Enfermedad coronaria, insuficiencia cardiaca, insuficiencia renal y ateromatosis periférica.

Tabla 1: Combinaciones lógicas de fármacos antihipertensivos.

13 Facultad de Medicina, Universidad de Chile

Existe escasa evidencia que apoye el agregar un 3º fármaco, sin embargo se promueve la combinación: IECA ( o ARAII)+ calcio antagonista + diurético tiazídico. Si no se logra la normotensión con 3 fármacos, derivar a especialista.



Pacientes hipertensos con RCV alto o muy alto, daño a órgano blanco, condiciones clínicas asociadas o comorbilidad.

Diabetes Mellitus

Meta PA < 130/80 mmHg

Enfermedad Cardiovascular y Cerebrovascular

Meta PA < 130/80 mmHg, siempre que exista buena tolerancia hemodinámica

14 Facultad de Medicina, Universidad de Chile

Nefropatía no diabética

Meta PA < 130/80 mmHg

Seguimiento

Las metas del tratamiento son las mencionadas anteriormente.Todo paciente con HTA esencial confirmada debe continuar su tratamiento de por vida. Se debe lograr ajustar el tratamiento hasta obtener el control de la PA, luego de ello la frecuencia de consultas puede reducirse, adecuando la periodicidad de los controles e intensidad de las intervenciones según el RCV.

Autor / Editor Matilde Pedrero

Año 2016

15 Facultad de Medicina, Universidad de Chile

Insuficiencia Aórtica

Diagnóstico Clínica: Asintomáticos por años, aunque pueden presentarse síntomas asociados a percepción del crecimiento cardíaco: disconfort por choque de punta en pared torácica (que se perciben sobre todo en decúbito ventral y decúbito lateral izquierdo), taquicardia o extrasístoles. Generalmente los síntomas no aparecen hasta que se presenta falla ventricular izquierda: DPN, ortopnea, disnea de esfuerzo, palpitaciones, edema y en ocasiones angina y síncope (sobretodo en esfuerzo). Sin embargo, la presencia de angina suele ser poco común en la IA aislada por ensanchamiento de las arterias coronarias. Más raro aún es la angina abdominal por isquemia esplácnica.

Nivel de manejo del médico general: Diagnóstico Específico Tratamiento Inicial Seguimiento Derivar

Aspectos Esenciales • • • •

Las causas más comunes son Enfermedad Reumática (ER) y Aorta bicúspide. El diagnóstico es clínico. Síntomas aparecen con enfermedad avanzada. El manejo definitivo es quirúrgico, en pacientes sintomáticos y asintomáticos con baja FE.

Al examen físico, signos clásicos por pulso hiperdinámico (debido a presión diferencial elevada): signo Musset (movimiento de cabeza con pulso), de Quincke (pulsación lecho ungueal), pulso celler, diferencia entre PAS entre extremidades superiores e inferiores > 60 mmHg (signo de Hill), pulso bisferiens, danza arterial del cuello, ruido de Traube y a la auscultación encontramos R1 normal o disminuido; 3º Ruido, soplo diastólico precoz, “in decrescendo” (se ausculta mejor inclinado hacia adelante, en espiración, y en foco aórtico y accesorio) y soplo Austin-Flint (mesodiastólico, infrecuente, traduce turbulencia al paso de la sangre de AI a VI por choque del flujo regurgitado con la válvula mitral). Choque de punta hiperdinámico y desplazado a la izquierda y abajo. La intensidad del soplo diastólico no tiene correlación con la severidad, no así la duración de éste: en leves suele ser sólo en el inicio de diástole, mientras que en las severas suele ser holodiastólico.

Caso Clínico Tipo

Mujer De 68 años, con antecedentes de ER. Presenta desde hace 2 años disnea progresiva, hasta hacerse de reposo, asociada a palpitaciones y edema de EEII. Al examen físico presenta pulso celler y se ausculta un soplo diastólico de regurgitación.

Definición Es la incompetencia de la válvula aórtica en diástole, apareciendo reflujo de la sangre desde aorta al ventrículo izquierdo luego de la sístole ventricular.

Imagenología: Ecocardiograma confirma el diagnóstico. Doppler permite evaluar la severidad (Grave cuando la fracción de regurgitación es >50%). Además permite evaluar repercusión estructural (HVI), volumen y dimensión de fin de diástole, fracción de eyección (<50% indica fase descompensada) y evaluación anatómica de la válvula. En caso de que la etiología sea debido a Aorta Bicúspide, puede existir formación de aneurismas o disección como resultado de dilatación de la raíz aórtica y/o aorta descendente. Otros: ECG con HVI; Rx tórax con cardiomegalia.

Etiología-Epidemiología-Fisiopatología Las causas más comunes de IA Crónica son: enfermedad reumática (ER) o dilatación de una válvula bicúspide congénita. Otras causas son AR, degeneración mixomatosa, espondilo artritis anquilosante, HTA, aortitis (por sífilis) y síndrome de Marfán. Causas de IA Aguda son Endocarditis y disección aórtica. El eje fisiopatológico es la sobrecarga de volumen ventricular (llenado desde la aorta y la aurícula izquierda), lo que genera finalmente hipertrofia ventricular excéntrica; además, dado que hay aumento del volumen expulsado, se produce hipertrofia concéntrica. Entonces, el VI mantiene por largo tiempo la relación masa/volumen conservado y aumenta su compliance, por lo cual se mantiene el GC y la presión de fin de diástole hasta etapas avanzadas. Ello no sucede en la IA Aguda, elevándose bruscamente la presión de fin de diástole ventricular y cayendo marcadamente el GC. El volumen de regurgitación (y por tanto el soplo) se incrementa al aumentar la postcarga por aumento de RVP.

Tratamiento Médico: NO existe un tratamiento médico de esta enfermedad cuando es severa y sintomática. Se administrarán diuréticos, IECA, en espera de cirugía. Si los pacientes son asintomáticos, si son hipertensos se tratará la HTA con IECA. ß-bloqueadores están relativamente contraindicados al aumentar el tiempo de diástole y, por tanto, el reflujo. Quirúrgico: Es el tratamiento definitivo (recambio valvular por prótesis). Indicado en pacientes sintomáticos. Cuando la FEVI es 0.3 o menos, los pacientes son INOPERABLES. Los pacientes asintomáticos con FE < del 50% y/o diámetro de

16 Facultad de Medicina, Universidad de Chile

fin de sístole ≥ 25 mm/m2 deben ser operados pues dilatar la cirugía empeora el pronóstico. 1 En IA Aguda: Nitroprusiato, inótropos, diuréticos y cirugía precoz.

Seguimiento Por especialista. Monitorizar la función ventricular con ecocardiograma seriado, además de prevenir endocarditis y enfermedad reumática. En IA leves con FE normal se realizará evaluación anual con ecocardiografía cada 2 a 3 años. Pacientes con IA severa estable y FE>50%, tendrán un seguimiento más estricto según las dimensiones del VI.

Autor / Editor Karim Gallardo

Año 2016

17 Facultad de Medicina, Universidad de Chile

Insuficiencia Cardiaca

ción diastólica). Se distinguen causas subyacentes (isquemia, hipertrofia, valvulopatías), y factores desencadentantes (Cardíacos: arritmias; Extracardíacos: Anemia, infección, embarazo, TEP, transgresión dietética, abandono tratamiento, estrés emocional).

Nivel de manejo del médico general: Diagnóstico Específico Tratamiento Inicial Seguimiento Completo

IC Aguda: Etiología: Síndrome coronario agudo (60%), insuficiencia mitral aguda, secuela de endocarditis infecciosa. Al no existir un tiempo suficiente para que existan cambios estructurales, predominan los mecanismos neuro-humorales de compensación (sistema simpático). Esto lleva a una caída del gasto cardíaco, con hipoperfusión tisular, aumento de la presión diastólica con congestión aguda del territorio pulmonar.

Aspectos Esenciales • • •

Diagnóstico basado en clínica, ECG y ecocardiograma doppler. Tratamiento ICC: Medidas generales y terapia farmacológica (IECA, B bloqueador y Diuréticos). Tratamiento ICA: Oxígeno, Furosemida, Nitroglicerina o Nitroprusiato.

Diagnóstico Fundamentalmente basado en una sospecha clínica inicial y confirmado por ECG y Ecocardiograma doppler. IC Crónica: Criterios de Framingham (2 mayores o 1 mayor y 2 menores).

Caso Clínico Tipo Hombre de 70 años HTA, que presenta disnea de esfuerzos progresiva hasta hacerse de reposo. Al examen se aprecia taquipneico, taquicardico, hipertenso. Yugulares ingurgitadas 45º, crépitos bilaterales, soplo holosistólico en foco mitral y edema de EEII.

CRITERIOS DE FRAMINGHAM

Definición Síndrome clínico que es consecuencia de una alteración cardiaca funcional o estructural, que altera o impide la capacidad ventricular de llenado y/o eyección, produciendo síntomas y signos en reposo o ejercicio. Según la rapidez de instalación del cuadro clínico, se clasifica en Crónica (deterioro progresivo de la función cardiaca), o Aguda (deterioro súbito).

Etiología-Epidemiología-Fisiopatología IC Crónica: Prevalencia de un 2% en Europa y EE.UU, que aumenta con la edad. Etiología: Principalmente por enfermedad coronaria e HTA crónica. Luego le siguen cardiopatías congenitas y valvulopatías. Frente a una noxa, se estimula activación del sistema RAA y simpático, permitiendo inicialmente la adaptación del órgano y la mantención de su función, pero con el tiempo se vuelven deletéreos y provocan la dilatación del ventrículo y su disfunción (proceso de remodelación).

Exámenes: • ECG debe realizarse en todo paciente con sospecha de IC: buscar evidencias de infarto al miocardio previo, signos de hipertrofia ventricular izquierda y dilatación auricular izquierda, bloqueo de rama izquierda (se asocia a daño estructural). • Radiografía de Tórax: Índice cardio torácico > 0.5, congestión pulmonar, líneas B de Kerley (edema septal), permite diagnóstico diferencial con enfermedades pulmonares.

Puede clasificarse en IC Derecha (congestión venosa sistémica con aumento presión venosa yugular, edemas y hepatoesplenomegalia), o Izquierda (Congestión pulmonar), así como también clasificarse en IC con disfunción sistólica o con función sistólica preservada (generalmente, por disfun-

18 Facultad de Medicina, Universidad de Chile



Eco Doppler: Define daño estructural y permite medir la fracción de eyección. Debe evaluarse la enfermedad coronaria y/o isquemia. IC Aguda: Inicio súbito de disnea, ortopnea, fatigabilidad importante y esputo asalmonado espumoso. En el examen físico: Signos de hipoperfusión tisular (extremidades frías, oliguria, compromiso de conciencia) y signos congestivos (crépitos pulmonares, edema de extremidades, ingurgitación yugular, hepato-esplenomegalia). Además se debe buscar elementos que orienten a la etiología del cuadro. Son de utilidad ECG, Rx tórax y eco cardiograma, sin embargo NO son necesarios para iniciar la reanimación.

Medicamentos a usar con precaución: Metformina (mayor riesgo de acidosis láctica), inhibidores de fosfodiesterasa-3 (cilostazol) y fosfodiesterasa- 5 (sildenafil, especialmente en conjunto con uso de Nitratos), metilxantinas. IC Aguda: Tiene dos vertientes. La primera es la reanimación del paciente: Sentarlo con los pies colgando, O2 al 100% (considerar uso de VPPI) y el uso de fármacos que de primera línea tenemos Diuréticos (Furosemida iv), Analgesia (Morfina) y Vasodilatación (Nitroglicerina iv, Nitroprusiato iv). Lo segundo es tratar la causa subyacente; instalar marcapaso transitorio si fuera por arritmia, trombolisis, etc. Debe ser derivado a un centro terciario una vez estabilizado, lo más pronto posible.  

Tratamiento

Seguimiento

IC Crónica: Identificar factores precipitantes o descompensantes y tratarlos. Según etapa: • A (factores de riesgo, sin cardiopatía estructural): Control de factores de riesgo y medidas generales (Restricción de Sal, suspención de tabaco, ejercicio físico regular y baja de peso, limitar consumo OH); • B (Cardiopatía estructural asintomática): IECA (Fármaco de elección), ARA 2 (si no se puede usar IECA) y B bloqueo (iniciar en bajas dosis para evitar descompensación e idealmente una vez estable con IECA/ARA 2). • C (B + síntomas): Añadir diuréticos (furosemida, sin impacto en sobrevida), espironolactona (CFIII-IV; CF II con FE<30%). • D (C con refracteriedad a tto): Otras terapias como resincronización ventricular y en casos seleccionados el trasplante cardiaco.

IC Crónica: Control de peso en cada control, evaluar sintomatología y fomentar adhesión a la terapia. IC Aguda: Derivar a especialista luego de manejo de urgencia.

Autor / Editor Karim Gallardo

En caso de pacientes con HTA, se debe privilegiar el uso de IECA/ARA 2, espironolactona y beta-bloqueo como tratamiento, sin embargo, en pacientes que se mantengan refractarios, se pueden utilizar antihipertensivos como diuréticos de asa, Hidralazina o dinitrato de isosorbide (estos 2 últimos se pueden dar como terapia combinada en pacientes que persisten sintomáticos pese a tratamiento estable de IC, especialmente en aquellos de raza negra, inciando en dosis de 25 y 20 mg c/8 hrs respectivamente). En el caso de Digoxina, se puede utilizar en pacientes sintomáticos (fatiga, disnea y tolerancia al ejercicio) o que requieran control de frecuencia por Fibrilación auricular concomitante, ya que disminuyen hospitalizaciones, pero no mejoran la sobrevida. En el caso de pacientes con disfunción diastólica y FE preservada, el tratamiento se enfoca en reducir factores de estrés hemodinámico, manejando los niveles de presión arterial, controlando la frecuencia en caso de FA y controlar la congestión pulmonar con diuréticos en caso de que exista. Se debe recordar que algunos de estos pacientes con baja compliance de VI son sensibles a la disminución de precarga, por lo que el uso de diuréticos y venodilatadores (nitratos y bloqueadores de calcio dihidropiridínicos) puede llevar a una baja de la FE.

19 Facultad de Medicina, Universidad de Chile

Año 2016

Insuficiencia Mitral

o falla cardíaca. Aguda Etiologías 1. Alteración de músculos papilares en post IAM. 2. Rotura tendinosa valvular en Sd. de prolapso valvular mitral. 3. EBSA. 4. Enfermedad reumática.

Nivel de manejo del médico general: Diagnóstico Específico Tratamiento Inicial Seguimiento Derivar

Aspectos Esenciales • • •

En la IM aguda hay reflujo importante hacia la aurícula izquierda, que no ha tenido tiempo de dilatarse, lo que eleva la presión auricular en forma severa, transmitiéndose en forma retrógrada al territorio veno-capilar pulmonar. La función sistólica VI habitualmente está preservada. La congestión pulmonar predominante gatillará los síntomas de disnea y ortopnea. Puede haber edema de EEII (compromiso de VD). En caso de IM aguda puede debutar como EPA.

IM crónica: Asintomática por años hasta descompensación. IM aguda: Cuadro potencialmente mortal (EPA, shock cardiogénico). La indicación de cirugía depende de la presencia de síntomas y de la fracción de eyección.

Caso Clínico Tipo

Diagnóstico

Paciente 55 años, con diagnóstico de IM leve hace 10 años. Hace 1 años comienza con disnea de esfuerzos progresiva hasta hacerse de reposo, ortopnea y palpitaciones. Al examen físico destaca MP (+) con crépitos bibasales, soplo holosistólico en foco mitral III/VI y edema bimaleolar.

Las manifestaciones clínicas habituales son disnea progresiva de esfuerzo, ortopnea, eventual edema de EEII, palpitaciones., entre otros. Al examen físico: R1 disminuido, desdoblamiento de R2 (por rápido cierre de A2 y en estadíos avanzados el aumento de tiempo de P2). Soplo generalmente holosistólico, con foco en ápex e irradiación a axila. Generalmente poca variación con respiración. El ECG y la Radiografía de Tórax pueden mostrar signos de complicación (ACxFA, HVI, hipertensión pulmonar, “p” ancha en dromedario y “p” negativa en V1). El ecocardiograma permite el diagnóstico de certeza, la valoración de la severidad y el seguimiento.

Definición Reflujo de sangre desde el ventrículo izquierdo a la aurícula izquierda durante la sístole debido a la incompetencia de la válvula mitral. Puede presentarse de forma crónica y aguda.

Etiología-Epidemiología-Fisiopatología Tratamiento

Crónica Etiologías 1. Enfermedad reumática (ER). 2. Endocarditis infecciosa (EI). 3. Mesenquimopatías. 4. Congénita. 5. Prolapso / enfermedad mixomatosa / degenerativa (causa más frecuente en países desarrollados).

Crónica 1. Asintomática: Prevención de enfermedad reumática. Uso de vasodilatadores puede enmascarar desarrollo de síntomas y retardar indicación quirúrgica. 2. Con Disfunción Sistólica: IECA. 3. Con FA: TACO. 4. Resolución quirúrgica en pacientes seleccionados (Considerar en IM crónica que se hace sintomática, asintomáticos con IM severa, FA en asintomático con FE límite y de bajo riesgo quirúrgico).

También existen causas secundarias (funcionales) como IAM, disfunción diastólica del VI, miocardiopatía hipertrófica. Debido a la sobrecarga de volumen hay dilatación de las cavidades izquierdas, efecto compensador que mantiene el gasto cardíaco por un largo período (años). Una vez superado este mecanismo aparece la falla ventricular (HT Pulmonar, compromiso derecho) y disminución del gasto cardíaco. Puede sobreagregarse una ACxFA. La severidad de los síntomas (disnea de esfuerzo, fatiga, palpitaciones) depende de la severidad de la IM, su velocidad de progresión, la presencia de FA y enfermedades cardíacas asociadas, siendo generalmente asintomática en IM leve y moderada e incluso en estadíos severos si no se desarrolla arritmia, HT Pulmonar

Actualmente ya no se considera la necesidad de profilaxis de Endocarditis según las últimas guías de la AHA ante procedimientos dentales. Aguda Hospitalización en UCI, monitorización hemodinámica, tratamiento depletivo intenso con diuréticos EV, drogas inotrópicas y vasodilatadores EV, mientras se diagnostica y trata la enfermedad de base, con el fin de estabilizar al paciente, que

20 Facultad de Medicina, Universidad de Chile

en su mayor parte de las veces, proceden a cirugía sobre la válvula mitral.

Seguimiento

Por especialista.

Autor / Editor Karim Gallardo

Año 2016

21 Facultad de Medicina, Universidad de Chile

Pericarditis Aguda

Pericarditis No Infecciosas: Pericarditis asociada al infarto del miocardio (dos situaciones diferentes: pericarditis durante la fase aguda del infarto, secundaria a la necrosis del miocardio; y pericarditis del Síndrome de Dressler, secundaria a fenómenos autoinmunes, desde la segunda semana post-infarto). Pericarditis urémica (complicación de insuficiencia renal crónica); Pericarditis neoplásica (más común metástasis de cáncer de pulmón, mama y Linfoma de Hodking; mucho menos común mesotelioma primario de pericardio); Pericarditis asociada a enfermedades del colágeno; Secundaria a drogas.

Nivel de manejo del médico general: Diagnóstico Específico Tratamiento Inicial Seguimiento Completo

Aspectos Esenciales • • • • • •

Manifestaciones cardinales: dolor torácico, frote pericárdico, cambios ECG, derrame pericárdico. Diagnóstico diferencial más importante: IAM Clínica menos súbita que IAM. ECG: alteraciones extensas a diferencia de IAM. Hospitalizarse por 24hrs. Complicaciones: taponamiento y arritmias. Una vez superado el evento agudo, si no cursó con derrame pericárdico importante, no requiere seguimiento.

Diagnóstico La evaluaciópn y manejo inicial del paciente que consulta en un servicio de urgencias con sospecha de pericarditis aguda se debe basar en 4 pilares diagnósticos: Clínico-electrocardiográfico- biomarcadores - ecocardiografía transtorácica Las principales manifestaciones clínicas son: • Dolor torácico: intenso, retroesternal y precordial, irradiado a espalda y anillo del trapecio izquierdo. • Comienzo gradual o súbito • Suele ser pleurítico (agudo y agravado por la inspiración, tos). Se alivia sentado e inclinado hacia adelante (posición mahometana) y se agrava en decúbito supino. En las de origen urémico o reumatológico puede ser menos intenso. • Frote pericárdico: signo muy útil cuando está presente, paciente sentado o decúbito prono. Suele ser inconstante e intermitente. Fiebre de bajo grado Recordar que pericarditis puede elevar enzimas cardiacas al asociarse a miocarditis. • Cuando asocia derrame pericardico los ruidos cardiacos se auscultan apagados.

Caso Clínico Tipo Hombre de 35 años, con dolor torácico que empeora con inspiración y tos, y se alivia al inclinarse hacia adelante. El dolor fue precedido de cuadro febril de horas de evolución. FC 94 lpm regular, PA 140/80, ritmo regular, sin soplos con presencia de frote pericárdico a la auscultación.

Definición Inflamación del Pericardio. Puede ser clasificada etológicamente en infecciosa, o no infecciosa. Se considera pericarditis aguda a la presencia de signos y/o síntomas de una a dos semanas de evolución (menos de 6 semanas).

Electrocardiograma (ECG): En ausencia de derrame masivo, se observan cambios clásicamente distribuidos en 4 etapas: Primero elevación difusa y de concavidad superior del segmento ST, depresión segmento PR; segundo ST y PR vuelven a lo normal (primera semana del cuadro); tercero Ondas T negativas difusas (generalmente luego de un ST isoeléctrico a diferencia de IAM); cuarto ondas T normales (aunque pueden quedar T invertidas indefinidas como una “pericarditis crónica”). Cuando hay derrame pericárdico caen los voltajes QRS, pueden haber extrasístoles y fibrilación auricular. Biomarcadores cardiacos: elevados en conterxto de pericarditis secundaria a IAM, o en contexgto de pericarditis asociada a miocarditis.

Etiología-Epidemiología-Fisiopatología Patología que con mayor frecuencia afecta al pericardio. Más frecuente en hombres que en mujeres, en adultos que en jóvenes y en autopsias que en clínica. Pericarditis comprende aproximadamente el 5% de los dolores torácicos de tipo angina Asocia miocarditis en aproximadamente el 15% de los casos: implicancias de tratamiento y pronósticas importantes. Pericarditis Infecciosas: la mayoría son de origen viral, especialmente por virus Coxsakie o Echovirus, por lo que las virales y las idiopáticas se consideran la misma entidad. Pericarditis Tuberculosa: poco frecuente, considerar en lugares de alta prevalencia, de evolución insidiosa y generalmente crónica. Las pericarditis bacterianas son raras, muy graves y generalmente asociadas a infecciones pulmonares, cirugía o traumatismos torácicos; principalmente estafilocócicas y neumocócicas.

Ecocardiograma: Si bien no hace el diagnóstico, es una herramienta de apoyo, mostrando a veces aumento de la ecorefringencia del pericardio, presencia y magnitud del derrame, y la patología de base en el caso de un infarto por ejemplo. Además, la Rx de tórax puede ayudarnos en la sospecha de un derramne pericárdico, esto cuando el corazón se ve “en forma de botella”.

22 Facultad de Medicina, Universidad de Chile

Tratamiento Hospitalización con 3 objetivos primarios: Iniciar tratamiento, Estratificar riesgo, y Observar evolución por eventuales arritmias y porque no es infrecuente la asociación con miocarditis, lo cual tiene otra connotación pronóstica. Factores de riesgo incluyen: Fiebre + leucocitosis; evidencia de taponamiento, efusión importante (>20 mm); inmunosuprimidos; TACO; trauma agudo; sin respuesta a tratamiento con AINES por 1 semana; elevación de enzimas cardíacas. Tratamiento farmacológico: • AINEs a altas dosis (preferentemente ibuprofeno 600 mg c/8 hrs e ir disminuyendo paulatinamente por 1-2 semanas o AAs 500 mg c/8 hrs especialmente si es post IAM). Asociar inhibidores de bomba de protones. • Cortos periodos de Prednisona (0.25-0.5 mg/kg/día por 1-2 semanas), pero preferentemente se reserva para refractarios a AINEs. • En casos de recurrencia, la colchicina puede ser útil (0.50.6 mg c/12 hrs por 3 meses), aunque también puede recomendarse asociada AINEs en un primer episodio. • Drenaje pericárdico o pericardiotomía: Reservado para derrames importantes, alteración hemodinámica, sospecha de etiología bacteriana/neoplásica o evidencia de pericarditis constrictiva.

Seguimiento Si el paciente cursó con derrame, debe ser controlado por especialista. Si no cursó con derrame no requiere seguimiento en particular.

Autor / Editor Matías Jerez

Año 2016

23 Facultad de Medicina, Universidad de Chile

Taquicardia Paroxística Supraventricular

Tratamiento Si no existe compromiso hemodinámico, iniciar tratamiento con maniobras vagales (masaje carotídeo, estimulación faríngea) que el 80% de las veces interrumpe la arritmia; si no revierten el cuadro, usar fármacos que actúan a nivel del NAV: de elección Adenosina 1 ampolla (6mg) vía endovenosa, seguido de bolos de solución fisiológica. Si no revierte con una ampolla de adenosina, readministrar el doble de dosis (12mg). En su defecto, usar Verapamilo (5mg iv), Diltiazem o Amiodarona (hasta 5mg/kg).

Nivel de manejo del médico general: Diagnóstico Específico Tratamiento Inicial Seguimiento Completo

Aspectos Esenciales • • • •

Si existe compromiso hemodinámico: cardioversión eléctrica. Criterios de inestabilidad hemodinámica según la AHA • Signos de mala perfusión periférica • Compromiso de conciencia • Hipotensión • Angina • Disnea

Mecanismo más frecuente es por reentrada en el Nodo AV. En mujeres jóvenes, sin cardiopatía. El manejo se inicia con maniobras vagales. El fármaco de elección, de no responder al manejo inicial, es la Adenosina.

Caso Clínico Tipo

Seguimiento

Paciente sexo femenino de 40 años sin antecedentes mórbidos. Consulta en SU por palpitaciones de inicio brusco y sensación de malestar torácico. Al examen físico presenta FC de 170 x’, PA 120/80, bien perfundida. Al ECG: taquicardia regular de complejo angosto sin onda P.

La gran mayoría revierte con las medidas indicadas, sin embargo un porcentaje presenta recurrencias. Frente a la recurrencia o la mala tolerancia hemodinámica a una TPSV debe plantearse la derivación a especialista y la necesidad de estudio electrofisiológico (EEF) para posible fulguración del haz eliminando el circuito de re entrada. En personal como aviadores, deportistas alto rendimiento se debe plantear la fulguración desde el inicio. Evaluación por especialista indicara B-bloqueo permanente u otros fármacos, junto con determinar necesidad de EEF.

Definición Es una taquiarritimia regular de origen supra ventricular, generalmente se presenta con una frecuencia de 150-180 lpm. Presentación en episodios esporádicos de inicio y terminación bruscos. Etiología-epidemiología-fisiopatología Existen diversas causas, la más frecuente es una reentrada en el nodo AV, por la presencia de fibras con distintas velocidades de conducción. La segunda causa más importante es la presencia de un haz paraespecífico o vía accesoria oculta (en este caso, si se asocia a Síndrome de preexcitación [diagnóstico ECG], se denomina Sd. Wolf Parkinson White, es decir Sd. preexcitación más arritmia). Se presenta por lo general en mujeres jóvenes, sin antecedentes mórbidos ni cardiopatía de base.

Autor / Editor Catalina Luvecce

Diagnóstico Se presenta principalmente como palpitaciones mal toleradas, en general sin compromiso hemodinámico. Otros síntomas posibles son mareos, malestar precordial inespecífico. El diagnóstico es Electrocardiográfico, evidenciándose taquicardia regular de complejo angosto (QRS < 0,12 seg) en ausencia de onda P.

24 Facultad de Medicina, Universidad de Chile

Año 2016

Arritmia Supraventricular

• Flúter auricular 1. Fibrilación auricular c) Arritmias de la Unión Auriculoventricular (AV): • Extrasístole nodal • Latido de escape de la unión AV • Ritmo de escape de la unión AV 1. Taquicardia de la unión AV d) Arritmias Ventriculares: • Contracción prematura ventricular • Taquicardia ventricular (TV) sostenida y no sostenida • Latido de escape ventricular • Ritmo de escape ventricular • Ritmo idioventricular acelerado

Introducción Una arritmia es todo ritmo cardíaco distinto al ritmo sinusal normal o todo trastorno en la formación y/o conducción de los impulsos cardíacos y constituyen un problema frecuente en la asistencia en urgencias. Su forma de presentación varía desde una anormalidad electrocardiográfica aislada, detectada casualmente en un paciente asintomático, hasta una emergencia médica. Su carácter potencialmente letal y su frecuente repercusión hemodinámica hacen que sea necesario establecer un diagnóstico y un tratamiento precoz. Clínicamente, puede ser asintomático o manifestarse por palpitaciones, signos y síntomas de insuficiencia cardíaca, bajo gasto cardíaco (mareos, síncope) y síndrome coronario agudo y el diagnóstico se sospecha por la auscultación cardíaca y la palpación del pulso, y se confirma mediante el electrocardiograma (ECG).

1. Alteraciones en la conducción del impulso: a) Bloqueos • Bloqueos sinoauriculares 1. Bloqueos AV b) Pre - excitación: • Síndrome de Wolff Parkinson – White 2. Arritmias mixtas: • Enfermedad del seno

En el manejo clínico de las arritmias no existen pautas rígidas de tratamiento ni esquemas inflexibles, por lo que es conveniente adaptar los protocolos terapéuticos a cada situación y usarlos sólo como guía de referencia. Por lo tanto, el médico que se enfrenta a una arritmia debe conocer y aplicar correctamente las maniobras de soporte vital básico y avanzado. En Chile, no existen extensos reportes sobre la epidemiología de las arritmias. Según los registros del Departamento de Estadísticas e Información en Salud (DEIS) del MINSAL, la tasa de mortalidad por BAV 3º, aumentó progresivamente en los últimos años y es más elevada en los adultos mayores (no se especifican otras patologías asociadas en este grupo). Así mismo, en el grupo de 75 y más años, los trastornos de la conducción cardíaca y arritmias se ubican dentro de las primeras 20 causas de AVISA (años de vida ajustados por discapacidad), lo que adquiere especial relevancia en el contexto del envejecimiento poblacional, que hace que hoy día el país tenga una creciente proporción de adultos mayores, que dependen en su mayoría del sistema público de salud para la resolución de sus patologías, por lo que se podrían esperar una mayor carga de enfermedad.

Fisiopatología El término arritmia cardiaca implica no sólo una alteración del ritmo cardiaco, sino que también cualquier cambio de lugar en la iniciación o secuencia de la actividad eléctrica del corazón que se aparte de lo normal. El ritmo cardíaco será considerado normal, cuando se origine en el nódulo sinusal y se conduzca a través de todas las estructuras cardíacas por las vías acostumbradas en forma normal. Por lo tanto, y de acuerdo con este concepto, un simple retardo en la conducción de los impulsos o una secuencia de activación anormal (como ocurre en los bloqueos de ramas y en las preexcitaciones) serán considerados también una arritmia cardiaca. Una manera útil de clasificar los mecanismos arritmogénicos, se muestran a continuación: Clasificación de arritimias según mecanismo de generación: 1. Alteración en la generación del impulso: • Automatismo normal • Automatismo Anormal • Potenciales gatillados 2. Alteración en la conducción del impulso: • Conducción lenta y bloqueo • Reentrada 3. Alteración simultánea en la generación y conducción del impulso

Dependiendo de su origen, éstas se pueden clasificar en arritmias supraventriculares y ventriculares. Las primeras, y sobre lo que trata este artículo, son aquellas que se originan antes del Haz de His, es decir, en las aurículas o en el nodo aurículoventricular. En el cuadro 1 se detalla la clasificación de las arritmias más frecuentes: 1. Alteraciones en la formación del impulso: a) Arritmias Sinusales: • Taquicardia sinusal • Bradicardia sinusal • Arritmia sinusal 1. Paro sinusal b) Arritmias Auriculares: • Extrasístole auricular • Marcapasos migratorio • Taquicardia auricular

1. Alteración en la conducción del impulso: La bradicardia se produce por una falla de la iniciación o de la conducción del impulso cardíaco. La falla en la iniciación del impulso puede ser causada por depresión de la automaticidad como resultado del enlentecimiento o falla de la des-

25 Facultad de Medicina, Universidad de Chile

polarización diastólica de la fase 4 del potencial de acción, la cual es provocada por enfermedad o exposición a fármacos. El sistema nervioso autónomo juega un rol principal en la modulación de la despolarización diastólica de fase 4 y así de las frecuencias tanto del marcapaso primario (nódulo sinusal) como de los marcapasos subsidiarios.

sinusal y del nodo aurículoventricular). Las alteraciones del automatismo se pueden clasificar en dos grandes categorías: automatismo exagerado y pospotenciales. El primero, es el mecanismo involucrado en taquicardias sinusales, en muchas taquicardias auriculares y en una forma de taquicardia incesante originada en fascículos del haz de His en niños y adolescentes. La hipokalemia y la estimulación simpática favorecen la aparición de arritmias por exageración del automatismo. Por otro lado, los pospotenciales se producen cuando un potencial de acción es seguido de una oscilación de voltaje. En caso de que esta oscilación alcance el potencial umbral, se producirá un nuevo potencial de acción. Las oscilaciones de voltaje pueden ocurrir antes de que se haya completado el potencial de acción anterior (pospotenciales precoces) o una vez que éste se ha completado (post potenciales tardíos). Los pospotenciales pueden determinar respuestas generativas aisladas o sostenidas. Los pospotenciales precoces constituyen el mecanismo involucrado en la génesis de taquicardias ventriculares polimorfas asociadas a síndrome de QT largo. Además, muchas de las arritmias por intoxicación digitálica son atribuibles a postpotenciales tardíos.

Un trastorno de la conducción puede determinar la aparición de un bloqueo cardiaco y se clasifican en cuanto al grado de severidad en bloqueos de primer grado (cuando un impulso atraviesa con lentitud la zona de bloqueo); bloqueo de segundo grado (cuando no todos los impulsos atraviesan la zona de bloqueo) y bloqueo de tercer grado(cuando ningún impulso atraviesa la zona de bloqueo). También, los bloqueos pueden clasificarse de acuerdo al sitio en que ocurren, como aquellos que se localizan entre el nódulo sinusal y la aurícula (bloqueos sinoauriculares), a nivel del tejido auricular (bloqueos intra e interauriculares) en el nódulo auriculoventricular, en el His, o en las ramas del haz de His. Los trastornos de la conducción pueden determinar la aparición no sólo de bradiarritmias por bloqueos, sino que también originar extrasístoles y taquicardia por un mecanismo denominado reentrada. Normalmente, un impulso eléctrico originado en el nódulo sinusal se propaga activando, por una parte, a las aurículas, y por otra, difunde hacia los ventrículos a través de los haces internodales, nodo auriculoventricular, tronco común del haz de His, ramas de His (derecha y fascículos izquierdos) y red de Purkinje distribuida en el seno del miocardio ventricular. Una vez que los ventrículos se han activado, el impulso eléctrico se extingue ya que no encuentra nuevo tejido en condiciones de depolarizarse. El concepto de reentrada implica que un impulso no se extingue después de haber activado al corazón, sino que vuelve a excitar fibras previamente depolarizadas.

Diagnóstico y manejo de las arritmias supraventriculares •

• • •

Las condiciones necesarias para que se produzca una reentrada son un bloqueo unidireccional de un impulso en algún lugar (habitualmente el impulso corresponde a una extrasístole), lenta propagación del mismo sobre una ruta alterna y reexcitación del tejido proximal al lugar inicial del bloqueo en dirección retrógrada.



• • •

2. Alteración en la generación del impulso En relación al automatismo cardíaco, las células del nódulo sinusal, al igual que las de la unión auriculoventricular y del sistema His – Purkinje, exhiben depolarización diastólica. La depolarización diastólica en las fibras del nódulo sinusal es la más rápida y de ahí, es que sean ellas las que constituyen el marcapaso fisiológico del corazón. Sin embargo, en condiciones patológicas o experimentales cualquier fibra miocárdica puede generar actividad espontánea y esto puede ocurrir como resultado de una enfermedad miocárdica o a través de manipulaciones experimentales como ocurre al elevar artificialmente los niveles de potasio extracelular. También, dicho fenómeno se observa durante isquemia miocárdica. Los potenciales de acción generados como consecuencia de estas depolarizaciones diastólicas patológicas, son del tipo de fibra lenta (semejante a los potenciales de acción del nódulo

• •

Como estrategia diagnóstica común para todas las arritmias, el objetivo fundamental en urgencias es la identificación de aquellas que requieren tratamiento urgente. Entre ellas hay que destacar: Las que ocasionan un deterioro hemodinámico, ya sea directa o indirectamente. Las que supongan per se un peligro para la vida del enfermo. Las que agraven o descompensen una enfermedad preexistente. Para detectar estas arritmias de alto riesgo hay que basarse en la anamnesis, la exploración física y en un reducido número de exploraciones complementarias. Los objetivos terapéuticos son los siguientes: Estabilizar hemodinámicamente al paciente. Controlar los síntomas.Idioma Controlar la respuesta ventricular hasta conseguir una frecuencia cardiaca que mantenga estable al paciente sin exponerlo a los efectos secundarios y complicaciones del tratamiento administrado (farmacológico, eléctrico). Evitar las complicaciones que pudieran generar la arritmia o el tratamiento antiarrítmico. Reversión urgente a ritmo sinusal, si estuviera indicado.

Clasificación de las arritmias Para la clasificación de las arritmias listadas en el cuadro 1, desarrollaremos el diagnóstico y tratamiento específico para cada una de ellas.

26 Facultad de Medicina, Universidad de Chile

1. Arritmias sinusales a) Taquicardia Sinusal

Afecciones no cardíacas: mixedema, hipertensión intracraneal, fármacos (beta-bloqueadores, amiodarona, verapamilo, ditiazem).

Etiología: Puede ser secundaria a fiebre, dolor, insuficiencia cardíaca, hipovolemia, hipertiroidismo, pericarditis, embolia pulmonar, feocromocitoma, estimulación simpática o inhibición vagal. Puede aparecer en sujetos sanos, lactantes y niños, ejercicio físico, ansiedad, tabaco e ingesta de estimulantes como café, té o alcohol.

Electrocardiograma:

Electrocardiograma:

• •

Ritmo sinusal Frecuencia cardiaca inferior a 60 lat/min.

Tratamiento: Esta arritmia no requiere de tratamiento, excepto cuando produce inestabilidad hemodinámica. • •

Ritmo sinusal Frecuencia cardíaca superior a 100 lat/min.

c) Arritmia Sinusal

Tratamiento: • • • •

Clasificación: • Respiratoria: El ritmo sinusal se enlentece con la espiración y aumenta con la inspiración debido a cambios del tono vagal con el ciclo respiratorio. Aparece en individuos sanos, sobre todo en niños y adolescentes. • No respiratoria: Las variaciones del ritmo sinusal no están relacionadas con la respiración. Se desconoce la etiología. Aparece fundamentalmente en individuos sanos, si bien puede asociarse a bradicardia sinusal en la fase precoz del IAM inferior. • Ventriculofásica: Se caracteriza porque los intervalos PP que no contienen un complejo QRS son más largos que los intervalos PP que sí lo contienen. Se asocia a bloqueo aurículoventricular (BAV) completo y, con menos frecuencia, a BAV de segundo grado 2:1, extrasistolía ventricular y ritmos de escape.

Supresión de sustancias estimulantes (alcohol, café o tabaco). Corrección del factor desencadenante o de la causa aguda que la ha generado: anemia, hipoxemia, etc. Ansiolíticos, como lorazepam en dosis de 1 mg por vía sublingual. Si no se observa mejoría, administrar propanolol en dosis inicial de 10 mg/8 horas por vía oral. Si aparece en el contexto de una crisis tirotóxica, se administra metoprolol en dosis inicial de 2 mg por vía intravenosa. Si no aparecen efectos secundarios, se administra a los 5 min una dosis de 3 mg en otros 2 min, y se continúa con dosis de 5 mg cada 5 min hasta un máximo de 15 mg. Transcurridos 15 min de la última dosis intravenosa, puede iniciarse la administración por vía oral en dosis de 100 mg/12 horas. La monitorización de este fármaco por vía intravenosa requiere monitorización electrocardiográfica continua.

Electrocardiograma:

b) Bradicardia Sinusal Etiología: Individuos sanos: hipervagotomía, deportistas, constitución asténica y durante el sueño. Cardiopatías: fase inicial del infarto agudo de miocardio (IAM), sobre todo de cara inferior. Enfermedad del seno.

• •

Presencia de ritmo sinusal. Intervalos PP o RR irregulares, con una variación superior

27 Facultad de Medicina, Universidad de Chile

a 0.12 segundos entre el intervalo PP más corto y el de mayor duración.

mo de 7 gotas/min (21 ml/h), ajustando velocidad de perfusión según la respuesta del paciente hasta una dosis máxima de 400 mg/24 horas. Si no se dispone de marcapasos transcutáneo, hasta la colocación de uno intravenoso se inicia una perfusión intravenosa de isoproterenol en dosis inicial de 1 µg/min en iniciar perfusión a 5 gotas/min (15 ml/h). Esta dosis se incrementa de 5 en 5 gotas (1 µg) cada 10 min, hasta conseguir una frecuencia ventricular que mantenga al paciente hemodinámicamente estable, hasta alcanzar una dosis máxima de 20 µg/min (100 gotas/min; 300ml/h) o hasta la aparición de complicaciones.

Tratamiento: La arritmia sinusal aislada no requiere tratamiento. d) Paro Sinusal Etiología:

Otra opción consiste en la administración, también en perfusión intravenosa continúa, de adrenalina en una dosis inicial de 2 µg/min y se perfunde a 3 gotas/min (9 ml/h). Esta dosis se incrementa de 2 en 2 gotas (1.2 µg) cada 10 min, hasta controlar la frecuencia ventricular o alcanzar la dosis de 10 µg/min o hasta la aparición de complicaciones.

1. Personas normales vagotónicas o con hipersensibilidad del seno carotídeo. Se asocia con frecuencia a arritmia sinusal. 2. Secundario a fármacos: digitálicos, quinidina, parasimpaticomiméticos. 3. Alteraciones electrolíticas: hiperkalemia. 4. Cardiopatías (miocarditis o IAM) que afecten el nódulo sinusal. 5. Iatrogénico: masaje del seno carotídeo en sujetos ancianos o con seno hipersensible.

2.

Electrocardiograma:

• • •

Arritmias auriculares a) Contracciones auriculares prematuras o extrasístoles auriculares

Etiología: En personas sanas pueden aparecer espontáneamente o inducidas por estimulantes, como alcohol, tabaco o café. Patología cardíaca: valvulopatías reumáticas (fundamentalmente insuficiencia mitral y tricúspide) y coronariopatías. Patología no cardíaca: hipertiroidismo, hipoxia, alteraciones hidroelectrolíticas.

Ritmo sinusal. Intervalos PP irregulares por la presencia de pausas que surgen sin una cadencia determinada. El intervalo PP que incluye la pausa no es múltiplo del intervalo PP del ritmo de base, característica que lo diferencia del bloqueo sinoauricular de segundo grado tipo II.

Electrocardiograma:

Tratamiento: Está indicado cuando produce alteraciones hemodinámicas, se constatan pausas de más de 3 segundos o la frecuencia ventricular es inferior a 40 lat/min, de forma mantenida. Tratamiento de la enfermedad causante. Se administra atropina en dosis inicial de 0.5 a 1 mg (dosis <0.5 mg pueden producir un efecto paradójico), en bolo intravenoso que puede repetirse cada 5 min hasta conseguir una frecuencia ventricular que permita mantener al paciente hemodinámicamente estable o administrar dosis máxima de 0.04 mg/kg (3 mg). Si el paciente no se estabiliza hemodinámicamente, existen dos alternativas: Colocación de un marcapasos transcutáneo previa administración de un analgésico, como tramados, en dosis inicial de 200 mg/24 horas por vía intravenosa y se perfunde a un rit-

28 Facultad de Medicina, Universidad de Chile

• • •

• • • •

• • • •

Ondas P de origen sinusal entre las que se intercalan ondas P anticipadas de origen no sinusal (P´) con morfología distinta. Según su precocidad, la onda P´ puede seguirse de un complejo QRS de morfología similar a la del ritmo de base, conducirse con aberrancia ventricular (contracción auricular prematura [CAP] aberrada) o no seguirse de QRS (CAP bloqueda). El intervalo P´R es superior a 0.12 segundos. Los intervalos PP (o RR) son irregulares, con pausa compensadora. Las CAP pueden aparecer aisladas, en bigeminismo (se alternan con los latidos del ritmo de base), trigeminismo (alternancia de dos latidos del ritmo de base y una CAP). Las CAP pueden ser unifocales, multifocales o muy precoces (pudiendo generar una fibrilación auricular).

Ondas P inicialmente sinusales, que luego se hacen auriculares o de la unión aurículoventricular (AV). Cada onda P va seguida de un complejo QRS con un intervalo PR variable, que puede oscilar desde valores normales hasta una duración inferior a 0.12 segundos. Morfología de onda P variable, con cambios sucesivos hasta aplanarse o hacerse negativa; son necesarios tres o más focos supraventriculares para considerarla marcapasos migratorio. Frecuencia cardíaca entre 60 y 100 lat/min, que disminuye a medida que el marcapasos se aproxima a la unión AV. Intervalos PP (o RR) irregulares.

Tratamiento: No precisa tratamiento antiarrítmico. c) Taquicardia Auricular Esta arritmia puede tener dos presentaciones: Taquicardia auricular unifocal o monomórfica y la taquicardia auricular multifocal. 2.c.i. Taquicardia Auricular Unifocal o Monomórfica Etiología: Cardiopatías: reumáticas, isquémicas, hipertensiva, prolapso de la válvula mitral, pericarditis. Enfermedad pulmonar obstructiva crónica (EPOC), cor pulmonar, tirotoxicosis. Intoxicación digitálica (es frecuente la taquicardia auricular con BAV 2:1).

Tratamiento: Se tratan las que sean sintomáticas. Se basa en: Supresión de sustancias estimulantes (café, alcohol, tabaco). Tratamiento de la enfermedad de base. En individuos ansiosos se administran ansiolíticos como lorazepam en dosis de 1 mg/12 horas por vía oral o sublingual. Si no cede, administrar propanolol en dosis inicial de 10 mg/8 horas por vía oral o metoprolol en dosis inicial de 100 mg/12 horas por la misma vía.

Electrocardiograma:

b) Marcapasos Migratorio Etiología: Las causas son similares a las de arritmia sinusal. Electrocardiograma:

29 Facultad de Medicina, Universidad de Chile

Ondas P´ (ectópicas o sinusales) que, en ocasiones son de difícil o imposible identificación si la frecuencia cardiaca es alta. Suelen tener la misma morfología (taquicardia auricular unifocal o monomórfica). •

• •

2. Estabilidad Hemodinámica: a) Control de la frecuencia ventricular: En ausencia de insuficiencia cardíaca, se administra un antagonista del calcio no dihidropiridínico (verapamilo, diltiazem) o un betabloqueador (metoprolol) por vía intravenosa u oral, en función de la intensidad de los síntomas: Verapamilo en dosis inicial de 5 mg por vía intravenosa. La administración de este fármaco por la vía indicada requiere monitorización continúa del ritmo y la frecuencia cardíacos. Si se elige en dosis oral inicial de 80 mg/8 horas, o 120 mg/12 horas del preparado. El verapamilo puede agravar la hipoxemia por vasodilatación de los territorios pulmonares hipoventilados, y es inotropo negativo. Diltiazem en dosis de 25 mg por vía intravenosa en 10 min y puede repetirse, si es necesario, a los 20 min. La administración de este fármaco por la vía indicada requiere de la monitorización continúa del ritmo y la frecuencia cardíacos. Si se elige por vía oral, se administra en dosis inicial de 60 mg/8 horas por vía oral. Este fármaco es menos inotropo negativo que el verapamilo. Metoprolol en dosis inicial de 2 mg por vía intravenosa. Si no aparecen efectos secundarios, se administra a los 5 min una dosis de 3 mg en otros 2 min y se continúa con dosis de 5 mg cada 5 min hasta un máximo de 15 mg (3 ampollas). Transcurridos 15 min de la última dosis intravenosa, puede iniciarse la administración por vía oral en dosis de 100 mg/12 horas. La administración intravenosa de este fármaco requiere monitorización electrocardiográfica continúa. Este fármaco está contraindicado en la EPOC y en la insuficiencia cardíaca grave. En presencia de insuficiencia cardíaca o contraindicación de los fármacos anteriores, se administra digoxina por vía intravenosa en dosis de 0.25 mg cada 2 horas, hasta la respuesta ventricular o alcanzar la dosis máxima de 1.5 mg. Durante su administración, por vía oral o intravenosa, no es necesaria la monitorización electrocardiográfica. Si se opta por la oral, se administra en dosis de 0.25 mg/8 horas, durante 48 horas, para continuar con 0.25 mg/día. Si el paciente está en tratamiento con digoxina, no se administra dosis de carga. Como alternativa a la digoxina, o si la respuesta ventricular no ha sido adecuada, después de administrar .075 mg se utiliza amiodarona en las mismas dosis y vías descritas anteriormente. Si la respuesta ventricular está aumentada, puede confundirse con una taquicardia paroxística nodal. Para confirmar que se trata de una taquicardia auricular, se realizan maniobras vagales o se administra adenosina, produciéndose un BAV transitorio durante el cual se detectan las ondas P´ ectópicas, para volver a la situación inicial al cabo de unos segundos.

Cada onda P´ puede seguirse de un complejo QRS de morfología normal, conducido con aberrancia, o no ir seguida de ningún complejo si existen un BAV funcional 2:1 (típico de la intoxicación digitálica). El intervalo P´R es normal o prolongado. Frecuencia auricular entre 100 y 240 lat/min. La frecuencia ventricular depende de la existencia de BAV y su grado. Intervalos P´P´ regulares en el caso de conducción normal o BAV 2:1, con la línea basal visible entre dos ciclos consecutivos.

Formas de Presentación: • Paroxística: Es la más frecuente. Suele ser sintomática. • No paroxística: Rara vez es sintomática.

Tratamiento: El tratamiento urgente está indicado si presenta inestabilidad hemodinámica, síntomas o una frecuencia ventricular superior a 120 lat/min.

1. Inestabilidad Hemodinámica: Se procede del mismo modo que el descrito en el tratamiento general de las taquiarritmias con inestabilidad hemodinámica, aunque en este proceso no suele haber respuesta a la cardioversión eléctrica, por lo que si fracasa, se administra amiodarona en dosis inicial de 5-7 mg/kg por vía intravenosa y se perfunde en 15 min. Si no se consigue la reversión, puede revertirse la misma dosis, pero perfundida en 60 min y a un ritmo de 40 gotas/min. Si después de administrar 900 mg persiste la inestabilidad hemodinámica, se repite la cardioversión eléctrica. Posteriormente, se procede de la siguiente forma: Si se estabiliza hemodinámicamente, pero sin reversión, se continúa con una perfusión intravenosa continua de amiodarona, en dosis de 1.2 – 1.8 g en 24 horas, descontando la cantidad ya administrada y perfundir a un ritmo inicial de 12 gotas/min (36 ml/h). Si se ha conseguido la reversión, se completa la impregnación de amiodarona por vía oral, cuya dosis depende de la dosis intravenosa administrada, sabiendo que la dosis de impregnación por vía oral es de 200 mg/8 horas durante 7 días, seguidos de 200 mg/12 horas durante otros 7 días, para continuar con 200 mg/ 24 joras, 5 días a la semana, como dosis de mantenimiento. Si el paciente está en tratamiento con amiodarona, se administra la dosis de mantenimiento, y no es necesario completar la dosis de impregnación.

b) Reversipon a ritmi sinusal: En ausencia de cardiopatía estructural y una vez controlada la frecuencia ventricular, se administra flecainida, por vía intravenosa en dosis de 1.5 – 3 mg/kg. Si se elige la vía oral, se administra en dosis inicial de 200 – 300 mg (por esta vía no se precisa monitorización electrocardiográfica). Si se consigue la cardioversión, se prosigue con 100 mg/12 horas por vía oral. La elección inicial de la vía intravenosa u oral depende de la intensidad de los síntomas. En presencia de cardiopatía estructural, se administra amio-

30 Facultad de Medicina, Universidad de Chile

darona por vía intravenosa en dosis mencionada con anterioridad y se optimiza el tratamiento de la insuficiencia cardíaca.

Si se sospecha de hipomagnesemia, se administra sulfato de magnesio por vía intravenosa en dosis inicial de 1.5 g y se perfunde en 10 min. La administración de este fármaco requiere que la concentración plasmática de potasio sea superior a 4 mEq/L.

c) Prevención de recurrencias: Se utiliza el mismo fármaco, por vía oral, que ha servido para la restauración a ritmo sinusal. 2.c.ii) Taquicardia auricular multifocal (Ritmo auricular caótico)

Si es necesario administrar fármacos antiarrítmicos, se utilizan, en función de los antecedentes del paciente, verapamilo, diltiazem o metoprolol por vía intravenosa u oral, en función de la gravedad de los síntomas, como ya se ha descrito.

Etiología: Aparece fundamentalmente en la EPOC y en la insuficiencia cardíaca descompensada. Con menos frecuencia, puede aparecer en ancianos o en el transcurso de hipocalcemia, hipokalemia, hipomagnesemia, embolia pulmonar, hipertensión arterial, hipoxemia, intoxicación por metilxantinas y, raras veces, en la intoxicación digitálica.

Si no puede utilizarse ninguno de los fármacos anteriores, o si estos no son efectivos, se administra amiodarona por vía intravenosa u oral, según la intensidad de los síntomas, en las dosis ya mencionadas, aunque este fármaco tampoco ha demostrado su efectividad. d) Flúter Auricular (FLA)

Electrocardiograma: Etiología: En crisis, raras veces paroxístico: ausencia de cardiopatía estructural y relacionado con otras situaciones, como alcoholismo, tirotoxicosis o pericarditis. FLA permanente o crónico: cardiopatías (reumática, isquémica, miocardiopatía) y dilatación auricular por embolia pulmonar insuficiencia respiratoria crónica o valvulopatía mitral o tricuspídica. Como complicación en el IAM (poco frecuente) o como manifestación de la intoxicación digitálica.



Clasificación: FLA común o tipo 1: Se caracteriza por frecuencia auricular entre 250 y 350 lat/min, mecanismo de reentrada, circuito localizado en la aurícula derecha y ondas F predominantemente negativas en la cara inferior. Otros tipos de FLA (menos frecuentes). Se caracterizan porque no cumplen criterios de reentrada, no tienen la frecuencia auricular del tipo 1. Exploraciones complementarias:

Las alteraciones electrocardiográficas son idénticas a las descritas anteriormente para marcapasos migratorio, a excepción de la frecuencia auricular, que oscila entre 100 y 180 lat/min. La frecuencia ventricular puede ser inferior, si hay BAV.

Electrocardiograma:

Tratamiento: El tratamiento en urgencias está indicado cuando presenta síntomas, teniendo en cuenta que los fármacos recomendados son de limitada eficacia y están reservados para los casos en los que la taquicardia no se resuelve después de estabilizar la patología de base. Se basa en: Si hay inestabilidad hemodinámica, se procede como se ha expuesto en la taquicardia auricular unifocal. Optimizar el tratamiento de la insuficiencia cardíaca y/o respiratoria, procurando mantener la teofilinemia dentro de los límites terapeúticos, corregir las alteraciones hidroelectrolíticas que se produzcan (sobre todo hipokalemia e hipomagnesemia) y disminuir en lo posible la dosis de fármacos betaadrenérgicos.



Las ondas P son sustituidas por ondas F que distorsionan la línea basal (dientes de sierra).

31 Facultad de Medicina, Universidad de Chile

• • • •

La frecuencia ventricular oscila entre 250 y 350 lat/min, la ventricular depende del grado de BAV existente. Los complejos QRS pueden ser: Rítmicos, conducción 2:1, 3:1, 4:1, etc.; con intervalos RR regulares. Arrítmicos: Cuando existen distintos grados de conducción AV sin ningún orden preestablecido (FLA con conducción AV variable). Si el grado de arritmia es mayor, se denomina fibriloflúter auricular, considerándose una Fibrilación Auricular (FA).

toma poco frecuente pero grave, ya que se relaciona con enfermedad del seno, obstrucción hemodinámica, ACV o preexcitación. Miocardiopatía inducida por la taquicardia: aparece en pacientes con taquiarritmias de larga evolución, cuya frecuencia ventricular no se ha controlado Sintomática con inestabilidad hemodinámica: Frecuente en presencia de frecuencias ventriculares extremas y está asociada a cardiopatías graves. Electrocardiograma:

Otras exploraciones Se solicitan las mismas que en la FA (Ver fibrilación auricular). e) Fibrilación auricular (FA) Etiología: Aguda: Surge en determinadas circunstancias y cuadros; por ejemplo, intoxicación etílica, cirugía, Infarto Agudo al Miocardio (IAM), entre otras. Una vez tratada, desaparece la posibilidad de recurrencia. Asociada a enfermedades cardíacas: valvulopatías, cardiopatías isquémicas, pericarditis constrictivas, entre otras. Asociada a otras enfermedades con afección cardíaca secundaria: Hipertensión Arterial, diabetes mellitus, enfermedad broncopulmonar, etc. Neurogénica: Poco frecuente. FA no valvular: Este término está restringido a los casos en que el trastorno del ritmo aparece en ausencia de enfermedad mitral reumática o prótesis valvular cardíaca.

• • •

Clasificación: Primera crisis detectada: Es la primera crisis de FA que se diagnostica a pesar que el paciente pudiera haber padecido otras. Puede evolucionar manteniéndose en FA (permanente) o revirtiendo a ritmo sinusal, tanto espontáneamente (primera crisis FA paroxística) como post tratamiento farmacológico o eléctrico (primera crisis FA persistente). 1. FA permanente: La arritmia es aceptada por el paciente y el médico, no considerándose la adopción de estrategias de control de ritmo. 2. FA paroxística: Es autolimitada (48 horas aprox.). Aunque los paroxismos pueden continuar por 7 días, las 48 horas son clínicamente fundamentales, pues transcurrido este tiempo la probabilidad de cardioversión espontánea es baja y debe considerarse la anticoagulación. 3. FA persistente: El episodio dura más de 7 días, requiriendo cardioversión farmacológica o eléctrica. 4. FA persistente de larga duración: Arritmia con una data de al menos 1 año al momento en que se decide adoptar estrategia de control de ritmo (cardioversión).



Permite identificar la existencia de arritmia y su tipo. La FA se caracteriza por: Ausencias de ondas P, sustituidas por ondas f que distorsionan la línea de base. Frecuencia auricular entre 400 y 600 lat/min. La frecuencia ventricular es variable. Complejos QRS arrítmicos, con intervalos RR irregulares, excepto a que coexista con un bloqueo auriculoventricular de tercer grado.

Otras exploraciones: • Radiografía posteroanterior y lateral de tórax. • Pulsioximetría (en caso de disnea). • Gasometría arterial si la saturación de oxígeno es inferior al 90% o si el paciente está en shock (en este caso se solicitan niveles séricos de ácido láctico si el gasómetro no lo detecta). • Hematimetría con fórmula y recuento leucocitarios. • Bioquímica sanguínea (glucosa, urea, creatinina, sodio y potasio). Si existe dolor torácico de patrón coronario se solicita troponina). • Estudio de coagulación en caso que el paciente esté en shock, padezca algún tipo de coagulopatía o se trate (o estuviese indicado) con anticoagulantes orales. • Estudio toxicológico en caso de sospecha de intoxicación por fármacos. • Otras, en función de la clínica y disponibilidades técnicas del servicio.

Formas clínicas: Asintomática. Sintomática con estabilidad hemodinámica: Generalmente, los síntomas varían con la frecuencia ventricular, la duración de la FA, existencia de cardiopatía asociada y la percepción individual. Las formas clínicas de inicio incluyen: Embolias (sobre todo cerebrovasculares). Exacerbación de una enfermedad cardíaca. Palpitaciones, dolor torácico, disnea, mareo o síncope (sín-

32 Facultad de Medicina, Universidad de Chile

Manejo en urgencias de la Fibrilación Auricular y Flúter Auricular



El tratamiento debe ser individualizado y depende de la frecuencia ventricular, presencia de síntomas, existencia de cardiopatía estructural significativa o insuficiencia cardiaca, presencia de estabilidad hemodinámica, medios disponibles y experiencia del médico, primando la seguridad del paciente sobre la efectividad del tratamiento. Los objetivos terapéuticos secuenciales son: 1. Estabilización hemodinámica. 2. Control de los síntomas y de la frecuencia ventricular (entre 60 y 100 lat/min). No obstante, de urgencia: 3. Si la frecuencia ventricular es lenta (FAL) debe intentarse conseguir la frecuencia ventricular mínima que mantenga al paciente estable, evitando efectos secundarios de fármacos o marcapaso transcutáneo, hasta su tratamiento definitivo (marcapasos permanente o cese de la causa). 4. Si la frecuencia ventricular es rápida (FAR) se debe mantener el control estricto de esta (80 lat/min en reposo y 110 lat/min en ejercicio). Los fármacos que se utilicen dependerán de la existencia o no de insuficiencia cardiaca: 5. Ausencia de insuficiencia cardíaca: utilizar un betabloqueador (metoprolol) o un antagonista del calcio no dihidropiridina (verapamilo, diltiazem). Si estuvieran contraindicados o no se toleran se utilizaría digoxina, y como última opción amiodarona (solo en fase aguda). Se pueden administrar por vía intravenosa u oral, en función de la intensidad de los síntomas. 6. Insuficiencia cardiaca: utilizar digoxina. En caso de no conseguir el control se podría añadir un betabloqueador en dosis bajas por vía oral. Si no fueran efectivos o no estuvieran indicados se utilizaría amiodarona (solo en fase aguda).



Crisis de FA de menos de 48 horas: si el paciente no está anticoagulado, se administra heparina de bajo peso molecular (HBPM) en una única dosis de 100 UI/kg por vía subcutánea. Si revierte, se instaura el tratamiento crónico o mantenimiento. Si el paciente está anticoagulado debería seguir con el mismo tratamiento sin precisar nada más Crisis de FA de más de 48 horas o duración desconocida: si el paciente no está anticoagulado, se anticoagula con AVK o dabigatrán, manteniendo un INR entre 2 a 3, durante 3 semanas antes y 4 después de la cardioversión. Si revierten se instaura tratamiento crónico. Si está anticoagulado se debe seguir con el mismo tratamiento.

Tratamiento crónico o de mantenimiento: Se inicia desde la documentación de la primera crisis de FA, modificándolo de acuerdo a la evolución del paciente. Se basa en la presencia de riesgo tromboembólico y hemorrágico, y la decisión del paciente. Los fármacos utilizados son antiagregantes, plaquetarios (ácido acetilsalicílico, clopidogrel) y anticoagulantes orales como AVK, anti-IIa (dabigatrán) y anti-Xa (apixabán y rivaroxabán. Durante períodos cortos se utiliza HBPM (enoxaparina) en dosis terapéutica. 3. Arritmias de la unión auriculoventricular (av) a) Contracciones nodales prematuras, extrasístoles noda les o de la unión auriculoventricular. Etiología: Se relacionan con cardiopatías (isquemia coronaria, insuficiencia cardíaca congestiva, intoxicación digitálica, etc.) y son infrecuentes en individuos sanos. Se atribuyen a una probable potenciación del automatismo normal. Electrocardiograma:

1. Control del ritmo: Cardioversión: puede ser eléctrica o farmacológica. Para esta última se utilizan antiarrítmicos en función de que exista o no cardiopatía estructural significativa. La cardioversión eléctrica urgente (CEU), está indicada cuando existe inestabilidad hemodinámica y cuando la crisis de FAR es inferior a las 48 horas sin conseguir el objetivo con fármacos. La cardioversión eléctrica electiva (CEE) está indicada en las crisis de FAR mayores a 48 horas o desconocida y en la FA persiste de larga duración, ya que el paciente necesita estar previamente anticoagulado (mínimo 3 semanas). La CEE la decide el cardiólogo. Prevención de recurrencias: una vez controlado el ritmo, se puede proceder al tratamiento farmacológico para prevenir otros episodios.

• •

Tratamiento antitrombótico: 1. F.A valvular: se anticoagula permanentemente con fármacos antivitamina K (AVK), en la dosis necesaria para mantener un I NR entre 2,5 y 3,5. 2. FA no valvular:

• •

Ondas P sinusales entre las que se intercalan con variable frecuencia ondas P ectópicas (P’). Cada onda P’ puede preceder, superponerse o seguir a un complejo QRS de morfología igual o muy similar a la del ritmo de base, con intervalos P’R menores a 0.12 s, o RP’ (cuando la onda P’ es posterior al complejo QRS) inferiores a 0.20 s. Intervalos PP (o RR) intermitentemente irregulares, con pausa compensadora completa, incompleta o incluso ausente. Intervalo de acoplamiento constante para cada foco ectópico.

33 Facultad de Medicina, Universidad de Chile



Pueden aparecer aisladas, presentar bigeminismo o trigeminismo, estar acopladas o ser unifocales o multifocales.

Tratamiento: Suspensión de sustancias estimulantes. Corrección de la causa desencadenante. En individuos ansiosos se administran ansiolíticos, como lorazepam en dosis de 1 mg/12 hr por vía oral o sublingual. Si no cede, se administra propranolol en dosis inicial de 10 mg/8 hr por vía oral o metoprolol en dosis inicial de 100 mg/12 hr por la misma vía. b) Laido de escape de la unión auriculoventricular y ritmo de escape de la unión

Ritmo de escape de la unión: • Sucesión rítmica de seis o más LEU. • La frecuencia ventricular oscila entre 40 y 60 lat/min. • Puede presentarse disociación AV si el marcapasos sinusal tiene una frecuencia similar a la nodal (disociación AV isorítmica), típica de la intoxicación de digoxina.

Etiología: En individuos sanos que tienen bradicardia sinusal. Cardiopatías o intoxicación por fármacos (digital, bloqueadores beta, etc.), que produzcan ritmos lentos (bradicardia sinusal, bloqueo sinoauricular, paro sinusal). El ritmo de escape de la unión (REU) actúa como marcapasos subsidiario en presencia de disminución del automatismo del nódulo sinusal, BSA o BAV de tercer grado.

Tratamiento: Está indicado cuando produce alteraciones hemodinámicas, se constatan pausas de más de 3 s o la frecuencia ventricular es inferior a 40 lat/min. Se basa en: • Tratamiento de la enfermedad causante. • Control de la frecuencia ventricular con atropina, marcapasos, aleudrina o adrenalina.

Electrocardiograma:

c) Taquicardias de la unión auriculoventricular 3.c.i) Taquicardia no paroxística de la unión auriculoventricularuicardia no paroxística de la unión auriculoventricular

• • •



Etiología: Puede ser causada por intoxicación digitálica, IAM inferior o miocarditis.

Latidos retrasados sobre la cadencia del ritmo de base. Ondas P sinusales entre las que se intercalan, con variable frecuencia, ondas P’. Cada onda P’ puede preceder, superponerse o seguir a un complejo QRS de morfología igual o muy similar a la del ritmo de base, con intervalos P’R menores a 0.12 s, o RP’ (cuando la onda P’ es posterior al complejo QRS) inferiores a 0.20 s El intervalo de escape es constante para los latidos de escape de la unión AV (LE) del mismo foco.

Electrocardiograma:

34 Facultad de Medicina, Universidad de Chile







Ondas P’ negativas en D2 que pueden aparecer, precediendo al complejo QRS con un intervalo P’R inferior a 0.12 s, inscribirse a la vez que el QRS (no se detecta en ECG) o seguirle (la onda P’ es posterior al complejo QRS) con un intervalo RP’ inferior a 0.20 s. Los complejos QRS son rítmicos, con una duración normal, salvo coexistencia de trastorno de la conducción intraventricular, a una frecuencia que oscila entre 60 y 130 lat/min. Si la frecuencia del nodo AV es similar a la del nódulo sinusal, puede objetivarse disociación AV isorítmica, típica de la intoxicación digitálica.

una extrasístole de la unión AV.

Tratamiento: Siempre requiere tratamiento urgente, considerando que ante la presencia de inestabilidad hemodinámica se trata como una taquiarritmia con complejo QRS estrecho. Si esto no ocurre, se aplican las mismas medidas generales de la taquiarritmia con estabilidad hemodinámica: Maniobras vagales (masaje del seno carotídeo, maniobras de Valsalva, entre otras). Si no revierte, administrar adenosina en dosis inicial de 3 mg (1 ml) en bolo intravenoso rápido. Si no revierte o si la adenosina está contraindicada, se administra verapamilo en dosis inicial de 5 mg por vía intravenosa.

Tratamiento: Se tratan las que originan los síntomas o cuando la frecuencia ventricular es mayor a 100 lat/min. Se basa en:

En el caso que no revierta con los fármacos anteriores o estuvieran contraindicados, se administra amiodarona por vía intravenosa en dosis de 5-7 mg/kg (si no lo tomaba previamente).

Tratamiento de la causa subyacente. Etiología desconocida: administración de propranolol en dosis inicial de 10 mg/ 8 hr por vía oral o metoprolol en dosis inicial de 100 mg/ 12hr por la misma vía.

Si la taquicardia nodal paroxística aparece en el contexto de una crisis tirotóxica, se administra propanolol por vía intravenosa en dosis de 1 mg. La administración de este fármaco requiere monitorización electrocardiográfica contínua.

3.c.ii)Taquicardia paroxística de la unión auriculoventricular o nodal paroxística. Etiología: Suele aparecer en individuos sanos. Cardiopatía: reumática, pericarditis aguda, prolapso de la válvula mitral, IAM o síndromes de preexcitación, etc.

4. Alteración en la conducción del impulso Los trastornos en la conducción del impulso se pueden clasificar en: a)Bloqueos: Es el enlentecimiento o la detención del impulso a través de una zona del sistema específico de conducción. Aquí se tratarán los más frecuentes, representados por los bloqueos sinoauriculares (BSA) y los bloqueos auriculoventriculares (BAV). a) Preexcitación: Es la transmisión del estímulo auricular a los ventrículos, o viceversa, por vías distintas de las habituales del sistema específico de conducción (vías accesorias). Dentro de este grupo, la más frecuente es el síndrome de Wolff-Parkinson-White (WPW), en el que el haz anómalo (haz de Kent) comunica la aurícula con el ventrículo.

Electrocardiograma:

4.1. Bloqueos Sinoauriculares (BSA) El trastorno de la conducción se produce en la unión entre el nódulo sinusal y la aurícula derecha. • •



Etiología: En sujetos sanos por aumento del tono vagal o hipersensibilidad del seno carotídeo. Cardiopatías: IAM inferior, miocarditis agudas. Intoxicación: digitálicos, quinidinas, betabloqueadores, verapamilo, diltiazem. Hiperkalemia. Enfermedad del seno.

Cumple los criterios del REU o de la taquicardia no paroxística de la unión AV, pero con una frecuencia ventricular de 160-250 lat/min. La duración de los complejos QRS es inferior a 0.12 s, excepto si hay aberrancia ventricular, bloqueo previo de rama o síndrome de preexcitación con conducción antidrómica (complejo QRS superiores a 0.12 s). Se instaura y cesa de forma brusca. Comienza a partir de

35 Facultad de Medicina, Universidad de Chile

Tratamiento:

Clasificación: De primer grado: imposible traducirse en el electrocardiograma (ECG). De segundo grado: • Tipo 1 o Mobitz I (Wenekebach). • Tipo 2 o Mobitz II De tercer grado: imposible de traducirse en el ECG. Por lo tanto, en este artículo solo se expondrá el BSA de segundo grado que se produce cuando uno de los impulsos originado en el nódulo sinusal no se conduce a las aurículas por ser bloqueado en la unión sinoauricular.

Está indicado cuando produce alteraciones hemodinámicas, se constatan pausa de más de 3 segundos o la frecuencia ventricular es inferior a 40 lat/min. Se basa en el control de la frecuencia ventricular con atropina, marcapasos, aleudrina o adrenalina. 4.2. Bloqueos Auriculoventriculares (BAV) El estímulo se enlentece o detiene en la unión auriculoventricular (AV).

Electrocardiograma: Tipo I

Etiología: Puede ser orgánica o funcional: aparece en individuos sanos, por procesos degenerativos en el sistema de conducción (personas mayores de 60 años). Puede ser la primera manifestación de la intoxicación digitálica. Clasificación: 1. Bloqueo auriculoventricular (BAV) de primer grado. 2. BAV de segundo grado. Se divide en tipo I (Mobitz I) y tipo II (Mobitz II). 3. BAV de tercer grado.

Ondas P sinusales de morfología constante en cada derivación. • Cada onda P va seguida de un complejo QRS con un intervalo PR normal, salvo coexistencia de BAV de primer grado. • Frecuencia cardíaca variable, generalmente dentro de los límites normales, más lenta cuanto mayor es el grado de bloqueo. • Intervalos PP (o RR) irregulares, que se acortan progresivamente hasta que se produce una pausa cuya duración es menor que la del doble de cualquiera de los intervalos PP. El intervalo PP más corto es el que precede a la pausa y el más largo, el que la sigue.

4.2.1. BAV de Primer Grado Electrocardiograma:

Tipo II

• • • • • •



Ondas P sinusales de morfología constante en cada derivación. Cada onda P va seguida de un complejo QRS con un intervalo PR normal, salvo coexistencia de BAV de primer grado. Frecuencia cardíaca que se reduce súbitamente a la mitad (BSA 2:1, el más frecuente), la tercera parte (BSA 3:1), etc., por no aparecer una o más ondas P en el momento esperado. Dicha pausa puedes ser el doble o el triple (según el grado del bloqueo), y es múltiplo de un intervalo PP normal. Intervalos PP (o RR) regulares.

Ritmo sinusal. Cada onda P va seguida de un complejo QRS. El intervalo PR es superior a 0.20 segundos.

Tratamiento: No está indicado el tratamiento urgente. Si se debe a efectos adversos o a intoxicación por fármacos, se ingresa el paciente en el área de observación del servicio de urgencias, para vigilar el desarrollo de otras arritmias más graves.

4.2.2. BAV de Segundo Grado Tipo 1 o Mobitz I Etiología:

IAM de cara inferior.

36 Facultad de Medicina, Universidad de Chile

Intoxicación digitálica Infecciones (fiebre reumática, escarlatina).

y porque los intervalos RR que incluyen una onda P bloqueada son múltiplos de los intervalos RR de los ciclos conducidos.

Electrocardiograma:

Tratamiento: Se procede a controlar la frecuencia ventricular y el uso de atropina no está indicada. Hay un tipo especial de BAV denominado bloqueo AV 2:1 en el que, con cadencia fija, una onda P se sigue de un QRS, mientras otra u otras no lo hacen, sin poder demostrar un fenómeno Mobitz II. Las indicaciones de tratamiento y las pautas terapéuticas son similares a las de BAV de segundo grado tipo 2.

4.2.3. BAV de Tercer Grado o Completo Etiología: • • • •

Transitorio: IAM inferior y antiarrítmicos. Permanente: congénito o adquirido por IAM anterior o por un proceso esclerodegenerativo del sistema de conducción.

Ondas P, con morfología constante en cada derivación. Intervalos PP constantes (eta característica lo diferencia de la contracción auricular prematura). Alargamiento progresivo de los intervalos PR, hasta que una onda P no se sigue de complejo QRS (se queda bloqueada). Intervalos RR progresivamente más cortos.

Electrocardiograma:

Tratamiento: De la causa desencadenante. Si se constatan pausas de más de 3 segundos o la frecuencia ventricular media es inferior a 40 lat/min en vigilia, se procede a controlar la frecuencia ventricular. •

Tipo 2 o Mobitz II



Etiología:

IAM anteroseptal Cirugía cardíaca.



Electrocardiograma:

A nivel auricular puede observarse cualquier ritmo (sinusal, fibrilación o flúter auriculares, etc.). Aparece un ritmo de escape generado por un marcapasos situado por debajo del bloqueo (de la unión AV o ventricular). Existe disociación AV, es decir, no hay relación entre el ritmo existente por encima del bloqueo (auricular) y el ritmo que aparece por debajo de este (de la unión AV o ventricular).

Tratamiento: Se procede a controlar la frecuencia ventricular y si el ritmo de escape es ventricular, la atropina no está indicada. 4.3. Síndromes de Preexitación: a) Síndrome de Wolff-Parkinson-White (WPW) •

Etiología: Más frecuente en individuos sanos. Mayor incidencia en personas mayores de 30 años con pre-

Se diferencia del anterior por intervalos PR constantes, que pueden ser normales o superiores a 0.20 segundos

37 Facultad de Medicina, Universidad de Chile

dominio del sexo masculino. Frecuente asociación con taquiarritmias supraventriculares: taquicardia paroxística de la unión AV, fibrilación y flúter auricular.

igual que la taquicardia nodal paroxística. Si el QRS en superior a .012 segundos (QRS ancho) corresponde a una taquicardia antidrómica (rítmico) o a una fibrilación auricular (arrítmico). Se tratan de la siguiente manera: Procainamida en dosis inicial de 50 mg (0.5 mL) por vía intravenosa, que puede repetirse, si es necesario, cada 5 minutos hasta la cardioreversión, la aparición de efectos secundarios (hipotensión, insuficiencia cardíaca, prolongación de la duración del QRS > 50%) o hasta alcanzar la dosis máxima de 1 g. Otra opción consiste en la administración de flecainida en dosis de 1.5-3 mg/kg por vía intravenosa. La administración de estos fármacos por vía intravenosa requiere de monitorización electrocardiográfica continua.

Electrocardiograma:

Si hay contraindicación o existe cardiopatía estructural significativa, se administra amiodarona en las dosis y vías referidas anteriormente. En esta arritmia, están contraindicados los siguientes fármacos: adenosina, betabloqueadores, antagonistas del calcio no dihidropiridínicos (verapamilo, diltiazem) y digoxina.

• •

c) Taquicardia con frecuencia irregular Las taquicardias irregulares, independientemente de la duración del QRS, tienen un pronóstico grave por el alto riesgo de fibrilación ventricular, especialmente cuando el intervalo RR es inferior a 0.20 segundos. En el paciente inestable, si fracasa el tratamiento anterior o si la frecuencia es superior a 250 lat/min, está indicada la cardioversión eléctrica. Además, de los fármacos contraindicados anteriormente, la lidocaína no se recomienda.

Complejo QRS ensanchado (>.012 s), a expensas de un empastamiento inicial (ondas delta). Trastorno secundario de la repolarización, con el segmento ST isoeléctrico o discretamente infradesnivelado, convexo hacia la línea de base, y onda T negativa y asimétrica en las derivaciones que tienen un complejo QRS predominantemente positivo.

Tratamiento:

5. Arritmias mixtas

En períodos asintomáticos, el tratamiento definitivo es la ablación de la vía accesoria. El tratamiento específico de las principales arritmias de este síndrome de WPW se expone a continuación:

5.1. Enfermedad del seno o disfunción del nódulo sinusal (DNS) La disfunción de nódulo sinusal (DNS) incluye trastornos que afectan a la generación del impulso en el nódulo sinusal y/o su transmisión a las aurículas. Puede generar bradiarritmias, taquiarritmias auriculares o alternancia entre ambas (síndrome bradicardia-taquicardia). Las bradiarritmias más frecuentes incluyen la bradicardia sinusal persistente, el paro sinusal, el bloqueo sinoauricular y la incompetencia cronotrópica. Las taquiarritmias más frecuentes incluyen la taquicardia, el flúter y la fibrilación auricular. La incompetencia cronotrópica se define como la incapacidad del corazón de ajustar adecuadamente su frecuencia en respuesta al aumento de la actividad física o al cambio de las demandas metabólicas. Actualmente, se diagnostica cuando el paciente no alcanza el 80% de la frecuencia cardíaca máxima esperada (220 menos la edad) en el ejercicio máximo de una prueba de esfuerzo.

b) Taquicardia Nodal • Taquicardia con QRS estrecho por conducción ortodrómica (el estímulo llega a los ventrículos por la unión AV y vuelve a las aurículas por la vía accesoria). • Taquicardia con QRS ancho, bien por la conducción antidrómica (el estímulo llega a los ventrículos a través de la vía accesoria y vuelve a las aurículas a través de la unión AV), bien por conducción ortodrómica en presencia de aberrancia de conducción o por bloqueo de rama previo.

Tratamiento:

Etiología: Puede ser secundario a fármacos (antiarrítmicos, litio, fenitoína), trastornos hidroelectrolíticos, apnea del sueño, hipoxemia, hipotiroidismo, hipotermia, aumento del tono vagal, fibrosis degenerativa ideopática, colagenosis, cirugía cardía-

Siempre requieren tratamiento urgente. Este se basa en la cardioversión eléctrica urgente si existe inestabilidad hemodinámica o farmacológica. Si el QRS es inferior a 0.12 segundos (QRS estrecho) se trata

38 Facultad de Medicina, Universidad de Chile

ca, cardiopatía isquémica, endocarditis o miocarditis.

Tratamiento: El tratamiento urgente está indicado cuando produce alteraciones hemodinámicas, y dependiendo de la arritmia que origine, es decir, marcapasos en las bradiarritmias y tratamiento específico en las taquiarritmias.

Conclusión Las arritmias cardíacas constituyen un problema frecuente en la asistencia en urgencias. Su forma de presentación varía desde una anormalidad electrocardiográfica aislada hasta una emergencia médica. Su carácter potencialmente letal y su frecuente repercusión hemodinámica hacen que sea necesario establecer un diagnóstico y un tratamiento precoz, por ende, es indispensable tener un claro conocimiento de cómo suceden para dar el manejo clínico adecuado. Siempre hay que tener presente que se trata a un paciente que padece una arritmia y no a una alteración electrocardiográfica. Por ello, es imprescindible, antes de adoptar una actitud terapéutica, realizar una valoración urgente del enfermo con el fin de descartar la existencia de un paro cardiorespiratorio o inestabilidad hemodinámica, y conocer si los síntomas son producidos por la arritmia o esta forma parte de la expresión clínica de otro proceso cardíaco o extracardíaco. En el primer caso, es fundamental tratar la arritmia (especialmente tratando de controlar la respuesta ventricular), mientras que en el segundo, dicha actuación no está indicada o tiene importancia secundaria. Las taquiarritmias incluyen un grupo de arritmias caracterizadas por tener una frecuencia ventricular superior a 100 lat/ min (habitualmente >140 lat/min). Generalmente, aparecen en forma de crisis, presentan una variable repercusión clínica (desde episodios asintomáticos hasta paro cardiorespiratorio) y conllevan, en muchos casos, dificultades diagnósticas. Las bradiarritmias incluyen un grupo de arritmias caracterizadas por tener una frecuencia ventricular inferior a 60 lat/min. La etiología más frecuente en urgencias es la interacción e intoxicación farmacológica, seguida por la cardiopatía isquémica. La repercusión clínica es variable, en función de la forma de presentación (en crisis o no), y varía desde episodios asintomáticos hasta paro cardiorespiratorio. Para el correcto manejo de un trastorno del ritm o, debe tenerse presente la siguiente regla, en la que se resume la actitud terapéutica ante una arritmia cardíaca: no ser ni más ni menos agresivos con la arritmia de lo que esta lo es con el paciente, primando la seguridad del mismo sobre la efectividad del tratamiento.

39 Facultad de Medicina, Universidad de Chile

Angina Inestable

fermedad coronaria. Se presenta como: Dolor de inicio agudo pudiendo aparecer en reposo, sin relación a los esfuerzos, de < 30 min de duración, retroesternal o precordial. Puede evolucionar desde una angina estable. Debe solicitarse ECG, pudiendo mostrar cambios evolutivos segmento ST, infra desnivel del ST o inversión de onda T (SCA SSDST); y Biomarcadores de daño miocárdico negativos (en caso contrario correspondería a IAM SSDST). Si el paciente persiste sintomático se debe realizar ECG y Marcadores De daño Miocárdicos seriados, esto aumenta la sensibilidad diagnóstica (Ojo con IDST en pared septal, pues puede tratarse de imagen especular de SDST en pared posterior estricta; siempre realizar ECG con derivaciones derechas y posteriores).

Nivel de manejo del médico general: Diagnóstico Específico Tratamiento Inicial Seguimiento Derivar

Aspectos Esenciales • • • • •

SCA SSDST con enzimas (-). ECG es el examen de elección. En DM, la presentación pude ser atípica. Tratamiento debe iniciarse precozmente (Corregir hipertensión, taquicardia). Derivar para decidir conducta según riesgo (TIMI).

Tratamiento Medidas Generales: Hospitalizar, Reposo absoluto, O2 para sat> 90%, monitorización. Antiagregante: Al inicio Aspirina 500 mg no recubierto a masticar, Clopidrogrel en dosis de carga de 300 mg. Anti isquémico: Nitroglicerina sublingual seguida por administración EV luego de 3 dosis SL sin respuesta al dolor o en caso de hipertensión, taquicardia o falla cardiaca. Morfina 2-4 mg sc en caso de dolor insoportable por el paciente (No recomendado como primera línea) repitiendo según dolor cada 5-15 min. B-bloqueo: Propranolol 5 mg ev c/5-10 min hasta alcanzar FC de 50-60 x’ y luego traspaso a vía oral con paciente estable 25 mg c/6 h. Importante descartar contraindicaciones como insuficiencia cardiaca o hipotensión. Anticoagulante: HNF 60 U/kg bolo seguido de 16 U/kg/hora, ajustando según TTPA (2 veces el basal) o HBPM; Fragmin(dalteparina) 100 U/kg cada 12 horas o Clexane (enoxoparina) 1 mg/kg cada 12 horas. Debe ser derivado. Estatinas: En dosis altas (atorvastatina 80 mg al día) para estabilización de placa ateroesclerótica.

Caso Clínico Tipo Hombre de 45 años, fumador, presenta hace 20 min dolor de inicio súbito, mientras dormía, opresivo e intensidad progresiva (8/10 EVA) en precordio. El dolor no cedió por lo que decide consultar. Realiza ECG, que evidencia IDST en cara lateral y biomarcadores de daño miocárdico que resultan (-).

Definición Es la aparición reciente de dolor anginoso, en un paciente que no lo presentaba o el cambio en las características del dolor en un paciente con ángor crónico (Progresión clínica: Mayor dolor, menor intensidad del esfuerzo o ángor de reposo). Incluido dentro del diagnóstico de Sd. Coronario Agudo sin Supradesnivel ST (SCASSDST)

Etiología-Epidemiología-Fisiopatología Seguimiento

Se produce por la oclusión aguda incompleta de una arteria coronaria, secundaria a la rotura aguda de una placa ateroesclerótica, con exposición de material subendotelial protrombótico, activación plaquetaria y de la cascada de la coagulación y posterior formación de un trombo plaquetario al interior de la arteria. Esto disminuye el flujo sanguíneo al miocardio, produciendo isquemia y manifestándose principalmente como dolor. Como NO hay necrosis del miocardio, no hay alza de biomarcadores de necrosis miocárdica (CK, CK-MB, Troponina, o puede haber alza de troponinas sin elevación de enzimas.

Será por parte del especialista, quién de acuerdo al riesgo del paciente (TIMI risk-score) decidirá coronariografía precoz (alto riesgo: 5-7 puntos) o estudio no invasivo (bajo riesgo: 0-2 puntos). TIMI score (7 puntos): Edad ≥ 65ª, ≥3 FR cardiovascular, enfermedad coronaria conocida (revascularización previa, aneurisma aórtico, enfermedad arterial oclusiva, enfermedad arterial carotídea), AAS en últimos 7 días, ≥2 episodios angina en < 24hrs, aumento enzimas cardiacas, desviación ST ≥0,5mm.

Diagnóstico El diagnóstico es clínico más electrocardiográfico. En general el paciente presenta factores de riesgo para en-

Autor / Editor Francisco Weber

40 Facultad de Medicina, Universidad de Chile

Año 2016

Disección Aórtica

• •

Nivel de manejo del médico general: Diagnóstico Específico Tratamiento Inicial Seguimiento Completo

Diagnóstico Se sospecha por la clínica, al enfrentarnos a un paciente con dolor de inicio súbito de máxima intensidad en precordio, dorso o región interescapular que lo lleva a consultar a un servicio de urgencia minutos o horas posterior al inicio. Ex Físico: HTA, asimetría de PA y de los pulsos de EESS, se puede auscultar un soplo interescapular o diastólico de regurgitación a lo largo del borde esternal derecho (Por insuficiencia aórtica, cuando compromete la raíz aórtica y los velos aórticos), pueden existir signos de taponamiento cardíaco (en la tipo A, cuando la aorta se rompe al saco pericárdico) como también compromiso neurológico focal por efecto de masa a estructuras circundantes como por ejemplo disminución de perfusión a nivel de la medula o parálisis de cuerdas vocales por compresión de nervio laríngeo recurrente izquierdo.

Aspectos Esenciales • • • • •

Rotura de la Aorta. Oclusión de vasos que salen de la Aorta, como las coronarias y/o arterias renales y el lumen falso puede llegar a ser más grande que el verdadero, y colapsarlo.

HTA y la edad son factores de riesgo. Síntoma cardinal es el dolor de máxima intensidad de instalación súbita. El examen de elección es el AngioTAC de tórax. El manejo inicial es con B-bloqueo. Se debe derivar dada su altísima mortalidad.

Caso Clínico Tipo Mujer de 68 años hipertensa, es traída por el SAPU al SU, por un cuadro de dolor de inicio súbito EVA 10/10 en el precordio, no irradiado. Al examen presenta PA 180/110, FC 100x’, con pulso asimétrico en las EESS. El ECG muestra taquicardia sinusal sin cambios en el ST, solicita una Rx Tórax que muestra un mediastino ancho.

La confirmación es con imagen siendo de elección el AngioTAC de tórax, también presta utilidad el Ecocardiograma TE (más sensible y específico que el TT) y la Rx tórax que muestra un mediastino ensanchado, y pedir ECG para dg diferencial con un SCA. Importante destacar que estos exámenes se deben realizar con el paciente hemodinamicamente estable.

Definición Es el desgarro de la capa íntima de la aorta (puerta de entrada), determinando el llene de un falso lumen, que puede continuar disecando esta arteria. Según la clasificación de Stanford, tenemos dos tipos: • Tipo A, que compromete desde la aorta ascendente, corresponde al 60% del total. • Tipo B, que compromete la aorta descendente.

Tratamiento El manejo inicial incluye el ABC de la reanimación, hospitalizar, monitorizar y gestionar la derivación. Luego el manejo de la presión arterial con LABETALOL, teniendo como meta PAS 90-100 y FC de 60x’, se pueden añadir vasodilatadores, pero siempre con B-bloqueo. En los casos que corresponda a un tipo A el tratamiento es quirúrgico (reemplazo protésico de la aorta ascendente), ya que la mortalidad aumenta 1% por cada hora. Las tipo B se operan sólo si se complican (Manejo endovascular). También son importantes el reposo absoluto y la psedoanalgesia del paciente (En caso de ser necesaria).

Etiología-Epidemiología-Fisiopatología Existen factores de riesgo que son: HTA, tabaquismo, ateromatosis difusa de la aorta, aneurisma aorta o disección previa, vasculitis, Sd. de Marfán, Sd. de Ehlers Danlos, válvula aórtica bicúspide, coartación aórtica, vasculitis. En estas situaciones se puede producir el desgarro, que lleva a una hemorragia de la media, esta hace que progrese el desgarro y se forme un falso lumen. Por este falso lumen ingresa el flujo sanguíneo haciendo que el rasgo se extienda hacia proximal o distal de la lesión.

Seguimiento Es por parte del especialista, se debe de monitorizar la presión arterial y las eventuales complicaciones como insuficiencia aórtica.

Es más frecuente en hombres y el promedio de edad es de 65 años. La mortalidad de la disección aórtica sin tratamiento es altísima; según la mayoría de los autores más de un tercio de los pacientes mueren en las primeras 24 horas, la mitad en las siguientes 48 horas, dos tercios en la primera semana y casi el 90% mueren en el primer mes. Las complicaciones pueden ser:

Autor / Editor Francisco Weber

41 Facultad de Medicina, Universidad de Chile

Año 2017

Infarto Agudo Al Miocardio

con Supradesnivel del ST (SDST) y sin SDST, lo que determina en cierto modo el tratamiento y los plazos que exige el GES. La definición de SDST en pacientes sin hipertrofia ventricular izquierda, ni bloqueo de rama izquierda es:

Nivel de manejo del médico general: Diagnóstico Específico Tratamiento Inicial Seguimiento Completo

Nueva elevación desde el punto J en al menos 2 derivaciones contiguas, de ≥ 2mm (0,2 mV) en hombres o ≥ 1,5mm en mujeres en derivaciones V2-V3; ó ≥ 1mm en cualquier otra derivación. Nuevo bloqueo completo de rama izquierda en pacientes con ECGs anteriores que no muestran dicha alteración. No debe tomarse aisladamente como diagnóstico. Infradesnivel ST ≥ 2mm en precordiales V1-V4, que puede ser espejo de SDST en coordenadas posteriores. Marcadores de injuria miocárdica: Hacen el dg de IAM independiente del patrón ECG. Son menos precoces y por tanto no determinan conducta inmediata. Hay mayor sensibilidad con curva de biomarcadores c/6 horas, y sirven para evaluar reperfusión. Los más importantes son CK, CK-MB (vida media de 36 horas) y Troponina, que es más sensible (dado que la vida media es de 7-14 días, también es útil para evaluar IAM reciente).

Aspectos Esenciales • • • • •

Principal causa: Ateromatosis coronaria. Sospecha es clínica y el dg: ECG + biomarcadores. La principal causa de muerte son las arritmias ventriculares. Nitroglicerina, AAS, ß-bloqueo y Estatina de urgencia. Reperfusión precoz en SDST.

Caso Clínico Tipo Paciente con dolor torácico de 2 hrs de evolución, toma ECG que evidencia SDST V2-V4, por lo que inicia las medidas generales, se encuentra a 3 hrs de un centro con hemodinamia. Inicia Trombolisis sistémica con lo que disminuye el dolor y el segmento ST desciende a la mitad.

Tratamiento Tratamiento Farmacológico: NTG sublingual 0.4 mg c/5 min por 3 dosis. Evaluar uso de NTG intravenosa (5-10 mcg/min titulando cada 10 minutos hasta cese de dolor, sin bajar de PAS < 100. No en IAM pared inferior ni en hipotensión). Morfina ev (2-4 mg en bolo cada 10 minutos. No en IAM pared inferior, ni hipotensión). Aspirina (500mg vía oral primera dosis, luego 160 - 325 mg/ día). Clopidogrel (dosis inicial 300 mg VO una vez o 600 mg si irá a angioplastía, seguido de al menos 75 mg/día). Estatinas. (Atorvastatina 80mg/día VO) Si el paciente está en Killip I o II, puede iniciar ß-bloqueo oral salvo contraindicación absoluta (por ejemplo, Propanolol en dosis de 5 mg vo c/6-8 hrs e ir titulando, o Carvedilol 6,25mg c/12 hrs).Importante en pacientes con infarto al miocardio por cocaìna se encuentra contraindicado el uso agudo de betabloqueadores en forma aguda. (En estos pacientes se encuentra recomendado el uso de benzodiazepinas). El uso de IECA debe posponerse hasta que el paciente se encuentre estable Tratamiento definitivo dependerá del patrón ECG: IDST se debe tratar como SCA sin SDST: Se debe realizar estratificación de riesgo del paciente con TIMI score. TIMI Score: ≥65 años, 3 factores de riesgo CV, estenosis previa ≥50%, desnivel segmento ST > 0,5mm, 2 eventos anginosos en 24 hrs, aspirina en los últimos 7 días y elevación de biomarcadores. (1 punto para cada ítem), que lo clasifica en riesgo alto (5-7 puntos), medio (3-4) y bajo (<3). Tratamiento común incluye Aspirina, NTG sublingual 0.4 mg c/5 min por 3 dosis y evaluar necesidad de NTG e.v.; ß-bloqueo dentro de las primeras 24 hrs si no hay contraindica-

Definición Es la necrosis miocárdica, producto de la isquemia aguda, secundaria a la oclusión de una o más arterias coronarias.

Etiología-Epidemiología-Fisiopatología Causa de muerte del 8% de la población chilena. La principal causa es la enfermedad ateromatosa, otras causas son angina de Prinzmetal, vasculitis, cocaína y disección coronaria. La oclusión coronaria aguda suprime el flujo sanguíneo, causando necrosis del miocardio.

Diagnóstico Clínica: Dolor anginoso típico de más de 20 min de duración, asociado a cortejo neurovegetativo y sensación de muerte inminente. Sin embargo en ancianos o diabéticos se puede manifestar de maneras atípicas (Por ejemplo, con ausencia de dolor). Por lo general el examen físico es normal. Electrocardiograma: Debe ser realizado en menos de 10 minutos desde que el paciente llega al SU. La primera alteración son las onda T hiperagudas, luego alteraciones del segmento ST en 2 derivadas contiguas, ondas Q patológicas (si hay necrosis transmural) y finalmente inversión de onda T. En términos de clasificación, generalmente se agrupan en Infartos

42 Facultad de Medicina, Universidad de Chile

ción (Verapamilo o diltiazem son opciones, no dar en falla cardíaca), un IECA dentro de las primeras 24 hrs a pacientes con congestión pulmonar o FE <40% en ausencia de hipotensión). Se agregará anticoagulante (Heparina no fraccionada o HBPM) + doble antiagregante (Clopidogrel y AAS). Posterior a esto, se decidirá la opción de realizar manejo invasivo precoz en pacientes con angina refractaria, inestabilidad hemodinámica/eléctrica o pacientes estables de alto riesgo. En pacientes en que se decide sólo manejo conservador, debe plantearse angiografía en caso de síntomas recurrentes, falla cardíaca o aparición de arritmias severas. En IAM con SDST se debe realizar reperfusión, para lo cual existen 2 opciones: Trombolisis sistémica (TS) o Angioplastía (AP). Con respecto a TS se recomienda aplicar antes de las 12 hrs de iniciado el cuadro, con estreptoquinasa 1.500.000 UI en solución salina 0,9% 250cc ev en 45 minutos (RAM más frecuente es la hipotensión: se detiene la infusión, se elevan EEII, aporte de volumen y el resto se pasa en 30 min). No administrar en alto riesgo de hemorragia. La AP tiene ventajas en pacientes con contraindicación de TS, en cuadros de más de 3 hrs. de evolución, o con shock cardiogénico. Requisitos para realizarla: el tiempo puerta-balón (primera consulta a intervención) de 90 min, en un centro con alta experiencia. Criterios de reperfusión exitosa: • Inversión onda T < 24 hrs. • Peak enzimático < 12 hrs. • Descenso ST a la mitad en 90 min (el mejor signo). • Disminución del dolor a la mitad. Importante en los IAM derecho (sospecharlo IAM de pared inferior asociado a hipotensión e ingurgitación yugular), se debe reanimar con volumen y no usar nitratos. Previo al alta, realizar un ecocardiograma y evaluar FEVI.

Seguimiento Debe ser en intensivo, monitorizado (principal causa de muerte son arritmias ventriculares malignas). Vigilar aparición de complicaciones: Mecánicas: La más común es la rotura de pared libre (se manifiesta como taponamiento cardiaco); perforación del tabique interventricular; disfunción de musculo papilar (EPA) y la formación de aneurismas o pseudoaneurismas. En general son tardías (> 4 días). Eléctricas: Incluyen bloqueos AV, FA, taquicardia ventricular y fibrilación ventricular.

Autor / Editor Francisco Weber

Año 2016

43 Facultad de Medicina, Universidad de Chile

Insuficiencia Cardíaca Aguda

adherencia al tratamiento o transgresión alimentaria, infecciones y FA.

Nivel de manejo del médico general: Diagnóstico Específico Tratamiento Inicial Seguimiento Derivar

La IC aguda es un problema creciente de salud pública. Actualmente constituye la principal causa de hospitalización en mayores de 65 años y asimismo, la patología que determina el mayor costo de hospitalización. Sólo en EEUU se producen más de 1.1 millones de hospitalizaciones por esta causa al año y la tendencia es al alza. El impacto no solo radica en lo anterior, sino también en la elevada recurrencia de rehospitalización (50% a 6 meses) y mortalidad a mediano plazo (30% a 1 año) de la hospitalización índice.

Aspectos Esenciales • • • •

Es una situación clínica que amenaza la vida. El diagnóstico es clínico. El manejo inicial debe centrarse en el ABC y estabilización del paciente. Se debe estudiar causa desencadenante.

En el registro ICARO de pacientes ingresados por IC en hospitales chilenos, la edad promedio fue 69 años, con predominio del género masculino (59%) y con alta comorbilidad. Las etiologías más importantes de IC fueron, en orden decreciente: hipertensiva, isquémica y valvular La gran mayoría (86%) correspondió a descompensación de una IC crónica. Las formas de presentación más habituales fueron la congestión pulmonar o sistémica, mientras que la hipotensión o shock cardiogénico estuvieron presentes en menor de 10%. La fisiopatología en común es un gasto cardíaco insuficiente y gran predominio de los mecanismos adrenérgicos generándose circulación periférica insuficiente, con hipotensión arterial, hipoperfusión renal y presión de llenado ventricular izquierdo elevada con importante congestión pulmonar e hipoxemia.

Caso Clínico Tipo Paciente 65 años, hospitalizado hace 3 días por IAM. Inicia súbitamente disnea y dolor torácico. Al examen destaca PA 90/60, crepitaciones en ambas bases pulmonares y soplo holosistólico precordial IV/VI. Ecocardiografía revela insuficiencia mitral con ruptura de cuerda tendinosa.

Definición Instalación brusca o deterioro de síntomas y signos de falla cardíaca. Es una condición que amenaza la vida requiriendo evaluación y tratamiento inmediato. El espectro clínico varía desde un empeoramiento generalizado con predominio del edema periférico a un shock cardiogénico/EPA.

Diagnóstico Eminentemente clínico: tos, disnea, taquipnea, ortopnea, yugulares ingurgitadas, crépitos pulmonares, edema periférico que rápidamente se van haciendo más severos. Se describen también sibilancias, lo que se denomina “asma cardíaca”. Se deben realizar tres valoraciones:

Etiología-Epidemiología-Fisiopatología



Causas de ICA que generan un deterioro rápido: • IAM • Arritmia severa (taqui/bradicardia). • TEP • Taponamiento cardíaco • Disección aórtica • Insuficiencias valvulares agudas. Gatillan un deterioro más insidioso: • Infecciones • Anemia • Falta de adherencia a tto de IC. • Hipo/Hipertiroidismo.

• • •

¿Los síntomas son debido a ICA o existe una causa alternativa (EPOC descompensado, anemia)? Si hay ICA ¿Hay algún precipitante y requiere este intervención inmediata (arritmia, IAM)? ¿Hay riesgo vital por hipoperfusión o hipoxemia? El ECG y la Radiografía de Tórax pueden ayudar en el diagnóstico específico.

Fisiopatológicamente la IC aguda es un diagnóstico sindromático que comprende un grupo heterogéneo de presentaciones clínicas, como se describe en las Guías clínicas europeas de IC: descompensación de falla cardiaca crónica, edema pulmonar agudo, falla cardiaca hipertensiva, falla ventricular derecha aislada y el shock cardiogénico.

Recordar también como causas precipitantes las interacciones farmacológicas y efectos secundarios de medicamentos como bloqueadores de canales de calcio o altas dosis de betabloqueadores, así como también la sobrecarga de volumen administrado. De acuerdo con datos del registro ICARO en nuestro país las causas de descompensación más frecuentes son: falta de

El síntoma más común es la disnea, aunque también son frecuentes la fatiga, el edema periférico y el malestar torácico. Algunos pacientes pueden presentar síntomas de bajo débito como intolerancia al ejercicio, anorexia y trastornos cognitivos. Hallazgos clínicos encontrados en el paciente como la

44 Facultad de Medicina, Universidad de Chile

presencia de signos de retención hidrosalina y el estado de la perfusión periférica permiten clasificar al paciente en alguna de las categorías de Forrestier: húmedo vs seco y caliente vs frío.

El tratamiento incluye la evaluación y estabilización inicial: Paciente en posición sentado, toma de signos vitales, monitorización continua, vías venosas, medición de diuresis. Para definir el tratamiento más adecuado, lo primero debemos clasificar al paciente en alguna de las categorías de la Clasificación de Forrester. Su valoración es simple y toma pocos minutos. Esta categorización es de utilidad en la toma de decisiones terapéuticas, pero también tiene implicancias pronósticas.

Tratamiento (ver algoritmo) Debe ser administrado en paralelo con el trabajo diagnóstico etiológico y bajo monitorización estricta.

Clasificación Clínico-hemodinámica de Pacientes con IC Aguda Signos y Síntomas de Congestión

Aquellos pacientes en los que predomina la congestión, pero que mantienen una adecuada perfusión, se beneficiarán del uso de diuréticos y vasodilatadores. Al contrario, en los pacientes catalogados como “fríos” con hipotensión y manifestaciones de hipoperfusión, se considerará el inicio de inótropos.

llos que conllevan un riesgo vital a corto plazo como las arritmias ventriculares y el síndrome coronario agudo. 1. Monitorización hemodinámica invasiva a) Catéter en la arteria pulmonar (Catéter Swan - Ganz): no se recomienda el uso rutinario del catéter de la arteria pulmonar. Sin embargo, puede tener utilidad y ser considerado en las siguientes condicio nes clínicas:

Se debe indagar los posibles factores de descompensación y realizar la corrección oportuna de ellos, en particular aque-

45 Facultad de Medicina, Universidad de Chile





Falla cardiaca aguda con síntomas persistentes, re fractariedad a tratamiento y/o los datos hemodiná micos o presiones llene son inciertos • Hipotensión persistente • Deterioro progresivo de la función renal • Requerimiento de drogas vasoactivas en dosis altas o combinación de dos o más drogas vasoactivas • Cuando se plantea terapias avanzadas como ACM o trasplante cardiaco b) Catéter arterial: la monitorización continua de la presión sanguínea con un catéter arterial puede ser útil en casos con una presión sanguínea marginal y permite la valoración óptima de vasodilatadores in travenosos.

co en pacientes con shock. El uso de opiáceos, con el fin de aliviar el apremio respiratorio se debe evaluar en forma individual y evitar su uso en aquellos pacientes con hipotensión. Se recomiendan dosis bajas, entre 1-3 mg EV.

2. Maximizar la oxigenación es vital Se administrará oxígeno a aquellos pacientes cuya saturación de oxígeno sea menor de 92%. Todos los pacientes con edema cardiogénico pulmonar agudo deben ser colocados en posicicón vertical y recibir oxígeno suplementario. La ventilación a presión positiva no invasina (VPPNI) debe ser considerada en estos pacientes con un aumento en curso del trabajo respiratorio, acidosis respiratoria o hipoxemia persistente. Se ha demostrado que la VPPNI se traduce en una resolución más rápida de los síntomas. Si bien no existe evidencia de una reducción a corto plazo de la mortalidad, ésta puede ser una herramienta a menudo muy valiosa, anticipándose a la intubación. Los pacientes que no responden a la VPPNI deberían ser intubados sin demora. El uso de presión positiva a final de la espiración (PEEP) puede ser efectiva para mejorar la oxigenación, pero los altos niveles de PEEP tienen un coste añadido de provocar una disminución en el retorno venoso y un gasto cardiaco reducido, lo cual puede ser poblemáti-

3. Vasodilatadores En ausencia de hipotensión sintomática, los vasodilatadores por vía intravenosa constituyen el tratamiento de primera línea para el manejo del edema cardiogénico pulmonar. Entre las alternativas para el uso vía endovenosa se cuentan: a) Nitroglicerina: cuyo efecto primordial es venodila tador, reduce la precarga y mejora la congestión pul monar. La cefalea es el efecto colateral más frecuen te, y la utilización está contraindicada en el marco del uso reciente de inhibidores de la fosfodiesterasa 5 (PDE-5). b) Nitroprusiato de sodio: posee un efecto vasodi latador dual, venoso y arterial. Dado el potencial de reducir bruscamente la presión arterial, es recomen dable durante su uso el monitoreo con catéter arte rial y manejo en una unidad de complejidad ma yor. El nitroprusiato es útil en particular en casos do de se requiere una gran y rápida reducción en la postcarga (por ejemplo, shock cardiogénico e insu ficiencia aórtica o insuficiencia mitral aguda grave). Si bien la toxicidad de la cianida y el tiocianato es ex tremadamente rara en tratamientos de corta dura ción debe utilizarse con precaución en pacientes con disfunción renal grave. A largo plazo deben evitarse las infusiones a altas dosis. En pacientes con isque mia miocárdica se prefiere la nitroglicerina o una combinación de nitroglicerina y nitroprusiato para evitar el riesgo teórico de robo coronario.

4. Diuréticos Constituyen el pilar fundamental en el manejo de los pacientes ingresados por descompensación de insuficiencia cardiaca. Además de su capacidad para reducir de forma gradual el volumen intravascular, los diuréticos tienen un efecto vasodilatador inmediato, que puede ser responsable del alivio rápido de los síntomas que producen. Debido a que muchos pacientes con edema cardiogénico pulmonar agudo no tienen un exceso total de sal y agua en su organismo, se recomienda el uso juicioso de los diuréticos. En cambio, en pacientes con evidencias de retención hidrosalina se debe inciar diuréticos intravenosos precozmente. Los pacientes sin una exposición crónica a diuréticos de asa responden en general a 20-40 mg de furosemida por vía intravenosa. Los pacientes sometidos a un tratamiento con furosemida a largo plazo, en general

necesitan una dosis de bolo intravenoso al menos equivalente a su dosis oral. Los resultados del estudio DOSE publicado recientemente demuestran que no hay beneficios con la administración de forma contínua frente a bolo intravenoso de diuréticos, ni perjuicios por altas dosis (una dosis intravenosa de 2,5 veces la dosis crónica oral de furosemida del paciente). Si se opta por una infusión contínua de diuréticos, ésta debe ir precedida por una dosis en bolo, que también debe ser administrada ante cualquier aumento de ritmo de la infusión contínua. Más que un objetivo terapéutico arbitrario de equilibrio de fluidos neto o un peso seco estimado, las evaluaciones clínicas frecuentes del estado de volumen (signos vitales, registro de peso, balance hídrico y evaluación de los síntomas) deberían guiar el tratamiento y definir el punto en el cual debería ocurrir la conversión a un régimen de mante-

46 Facultad de Medicina, Universidad de Chile

nimiento oral. El uso rutinario de sonda urinaria no está indicado, a excepción que se requiera de un control estricto de volumen urinario o se sospeche una obstrucción urinaria. Es necesario el monitoreo de la función renal y electrolitos en plasma, cuando se utiliza diuréticos por vía endovenosa. Suele tolerarse algún grado de empeoramiento de la función renal para lograr una descongestión adecuada. Sin embargo, si ocurre una insuficiencia renal progresiva a pesar de la congestión persistente, debe considerarse la ultrafiltración o la adición de un vasodilatador intravenoso o inótropo. Entre los efectos adversos importantes se encuentran hipotensión, hipopotasemia, hipomagnesemia e hipocalcemia. La resistencia a los diuréticos puede abordarse con dosis escaladas de diuréticos de asa y posteriormente con la adición de un diurético de tiazida.

inotrópicos contribuyan con una mejoría en cuanto a prolongar la sobrevida de los pacientes. Pueden provocar arritmias e inducir isquemia por aumento del consumo miocárdico de oxígeno. Se recomienda el uso de inotrópicos en las siguientes situaciones: signos de bajo débito, deterioro de función de órganos blanco, como riñón, y congestión persistente a pesar de la terapia diurética. Se describen acontinuación características de dos de los agentes inótropos más usados: a) Dobutamina: actúa en receptores Beta 1 y en me nor grado en receptores Beta 2 y Alfa 1 adrenérgi- cos. Tiene una vida media más corta que la milrinona y con frecuencia es el fármaco de elección en un con texto agudo. b) Milrinona: es un inhibidor de la PDE que aumenta el inotropismo cardiaco mediante la inhibición de la degradación del monofosfato de adenosina cíclico. Es un potente vasodilatador pulmonar y sistémico debido a sus efectos sobre la PDE vascular. Debido a que no se dirige a los receptores beta, la milrinona puede ser más efectiva que la dobutamina en el ma co de un tratamiento betabloqueante reciente o en curso.

5. Tratamiento inótropo La indicación de la terapia inotrópica está restringida para los pacientes con hipotensión, con PAS inferior a 85 mmHg. La mayoría de estos pacientes posee disfunción sistólica avanzada y presentan al momento del ingreso signos de mala perfusión tisular y compromiso de la función de órganos blanco. No se dispone de evidencia que los fármacos

Fármacos de acción inotrópica positiva

6. Ultrafiltración Su uso está reservado para pacientes resistentes al tratamiento con diuréticos intravenosos o con diuresis complicada con el empeoramiento de la función renal.

pensada. 8. Soporte circulatorio mecánico temporal Los pacientes con shock cardiogénico resistente al tratamiento y edema cardiogénico pulmonar pueden beneficiarse del uso temporal de un balón de contrapulsación intraaórtico o un medio temporal alternativo de soporte circulatorio mecánico (por ejemplo, circulación extracorpórea venoarterial) que pueden facilitar el puente a la estabilización o a una toma de decisión posterior.

7. Antagonistas de la vasopresina El antagonista oral del receptor de tipo 2 de vasopresina, tolvaptan, y el inhibidor de receptores de vasopresina intravenoso no selectivo, conivaptan, están aprobados para el tratamiento de la hiponatremia hipervolémica o euvolémica que puede acompañar a la insuficiencia cardiaca descom-

47 Facultad de Medicina, Universidad de Chile

9. El diagnóstico y manejo de las taquiarritmias auriculares y ventriculares es crítico para el cuidado de los pacientes con IC aguda descompensada, ya que éstas pueden con frecuencia precipitar exacerbaciones y alterar el curso de la enfermedad.

plementada una vez que se alcanza la estabilidad clínica. En general, los vasodilatadores (IECA, ARA II o hidralazina/dinitrato de isosorbida) son reintroducidos primero conforme a la retirada de los vasodilatadores intravenosos. Si los betabloqueadores se retiraron debido al shock cardiogénico, pueden ser cuidadosamente reintroducidos en los pacientes euvolémicos estables.

10. La Transición al tratamiento farmacológico crónico es im-

Algoritmo Manejo de Pacientes que Consultan a Urgencia por IC

Guía Clínica Insuficiencia Cardiaca. MINSAL 2015 Autor / Editor Año Seguimiento Francisco Weber 2016 Por especialista

48 Facultad de Medicina, Universidad de Chile

Paro Cardiorespiratorio

pulmones durante algunos minutos, tras lo cual sobreviene el paro cardíaco por anoxia miocárdica. Los ritmos del PCR son de dos tipos: desfibrilables (Fibrilación ventricular y Taquicardia ventricular sin pulso) y los no desfibrilables (Actividad eléctrica sin pulso y Asistolía). Lo más frecuente es el PCR de causa cardiológica, producido por taquiarritmias ventriculares secundarias a sindrome coronario agudo; si no reciben tratamiento progresarán a asistolía. Los ritmos no desfibrilables son poco frecuentes como presentación inicial del PCR y se producen generalmente en PCR de causa no cardiológica.

Nivel de manejo del médico general: Diagnóstico Específico Tratamiento Inicial Seguimiento Completo

Aspectos Esenciales • • • •

Es esencial el inicio inmediato de RCP, pues por cada minuto sin RCP la sobrevida del paro cardíaco, presenciado en FV, disminuye 7 – 10%. Cadena de supervivencia: 1) Activación del sistema de emergencia 2) RCP básica 3) Desfibrilación 4) RCP avanzada. 5) Cuidados integrados post paro cardiaco. Secuencia de maniobras de RCP: C-A-B (iniciar y hacer énfasis en las compresiones torácicas) La desfibrilación es la medida más efectiva para reanimar

Diagnóstico Su diagnóstico es clínico, cuyas manifestaciones principales son la inconciencia, ausencia de pulso y la respiración anormal (apnea o gasping). La ausencia de pulso se debe determinar en menos de 10 segundos y en caso de duda se debe asumir que no hay pulso.

un paciente en FV o TVSP.

Tratamiento Caso Clínico Tipo

La cadena de supervivencia considera cuatro eslabones: 1. Reconocimeinto y activación precoz del sistema de emergencias. 2. RCP básica. 3. Desfibrilación precoz. 4. RCP avanzada 5. Cuidados integrados post paro cardiaco.

Paciente con pérdida brusca de conciencia, ausencia de pulsos centrales y la presencia de apnea o gasping.

Definición

Los primeros tres eslabones se engloban en el concepto de Soporte Vital Básico (SVB), mientras que los últimos corresponden a Soporte Vital Avanzado (SVA). Se inician las maniobras de RCP mientras se consigue un desfibrilador. La secuencia recomendada es C-A-B (Chest Compressions, Airway, Breathing), comenzando con 30 compresiones y alternándolas con 2 ventilaciones. Como ayuda memoria el ritmo de las compresiones es el mismo de la canción “Staying alive”. Las compresiones deben ser a más de 100/minuto, de más de 5 cm de profundidad, evitando interrupciones, permitiendo la completa reexpanción del tórax entre compresiones y cambiando al operador cada 2 minutos. Las ventilaciones deben ser de 1 segundo de duración, hasta lograr un levantamiento normal del tórax. Si utiliza un dispositivo avanzado de vía aérea (tubo endotraqueal o dispositivo supraglótico) debe realizar compresiones contínuas y 8-10 ventilaciones por minuto. Utilice oxígeno al 100% y volumen corriente de 500-600 ml; evite la hiperventilación.

Detención súbita de la actividad miocárdica y ventilatoria que determina una brusca caída del transporte de oxígeno a los tejidos, determinando la pérdida de conciencia. Es una emergencia médica potencialmente reversible a través de las maniobras de Reanimación Cardiopulmonar (RCP).

Etiología-Epidemiología-Fisiopatología Causas más frecuentes: 1. cardiológicas: sindrome coronario agudo, arritmias, taponamiento cardiaco, tromboembolismo pulmonar, etc. 2. respiratorias: obstrucción, depresión del centro respiratorio, broncoaspiración, asfixia o neumotórax a tensión. 3. Otras: traumatismos, trastornos electrolíticos y metabólicos, shock, hipotermia y drogas. Si el evento primario es cardíaco al colapso cardiovascular se le agrega el paro respiratorio por hipoxia de los centros bulbares, pudiendo presentarse una respiración anormal e ineficaz conocida como gasping o respiraciones agónicas. Si el evento primario es un paro respiratorio el corazón continúa la oxigenación de los tejidos con el aire contenido en los

Es recomendable utilizar la capnografía (EtCO2) para monitorizar el adecuado emplazamiento del dispositivo de vía aérea y monitorizar la calidad de la RCP (un EtCO2 <10 mmHg obliga a mejorar la calidad de la RCP). Apenas tenga disponible el desfibrilador debe evaluar el ritmo cardiaco, lo cual debe repetirse cada 2 minutos. Si es desfibrilable, debe realizar una desfibrilación (120-200 J bifásico o 360 J monofásico) y continuar inmediatamente la RCP comenzando por las

49 Facultad de Medicina, Universidad de Chile

compresiones. Si no es desfibrilable, debe continuar inmediatamente la RCP comenzando por las compresiones. Los fármacos a utilizar son: • Adrenalina: 1 mg cada 3-5 minutos IV o IO. • Amiodarona: en caso de ritmos desfibrilables. 300 mg IV o IO inciales y 150 mg más en en casos refractarios. Siempre se deben identificar y tratar las causas reversibles (las 5H y las 5T): hipovolemia, hipoxia, acidosis, hipo/hiperkalemia, hipotermia, neumotórax a tensión, taponamiento cardiaco, intoxicaciones, tromboembolismo pulmonar y trombosis coronaria. Tras el restablecimiento de la circulación espontánea, se procura la optimización de la función cardiopulmonar y la perfusión de órganos vitales. Si hay sospecha de sindrome coronario agudo el paciente se debe realizar una coronariografía. Trasladar al paciente a una UCI apropiado que disponga de un sistema completo de tratamiento post-PCR. Considerar el control de la temperatura (optimizar recuperación neurológica), así como prevenir y tratar disfunciones orgánicas, lo que incluye evitar la ventilación excesiva y la hiperoxia.

Seguimiento Por especialista. Se adjuntan algoritmos de resumen de reanimación ACLS.

50 Facultad de Medicina, Universidad de Chile

Shock



Nivel de manejo del médico general: Diagnóstico Específico Tratamiento Inicial Seguimiento Completo

Cardiogénico: • Mecanismo: ‘Disminución del gasto cardíaco por falla en la función sistólica y/o diastólica cardíaca (Falla bomba, disminuye GC). • Causas: Cardiopatía coronaria (IAM extenso, el > frecuente), miocardiopatías, arritmias, valvulopatías, mixomas, etc.

Aspectos Esenciales • • •

Causas: Hemorragia (Mas común trauma penetrante, hemorragia digestiva alta por varices esofágicas o ulceras gástricas y hemorragia digestiva baja de diversas causas), deshidratación, secuestro en tercer espacio, pérdidas digestivas, urinarias, insensibles, etc.

Shock es el síndrome caracterizado por hipotensión y signos de hipoperfusión tisular con la consiguiente disfunción orgánica. Existen distintos tipos de shock. Un alto grado de sospecha y manejo oportuno son fun-

Obstructivo: • Mecanismo: ‘Falla diastólica por compresión extrínseca de cavidades cardíacas. • Causas: Taponamiento cardiaco, neumotórax a tensión, embolia pulmonar masiva, pericarditis constrictiva severa, etc.

damentales para un buen pronóstico.

Caso Clínico Tipo

Distributivo: • Mecanismo: ‘Caída severa de la resistencia vascular periférica por vasodilatación. • Causas: Shock séptico la más frecuente (Pensar siempre en Neumococo, agentes multiresistentes como Klebsiella, Pseudomonas, Enterococos y Candidas), SIRS, alergias (shock anafiláctico), lesiones graves SNC (shock neurogénico), insuficiencia suprarrenal aguda, drogas, toxinas, etc.

Paciente de 57 años, con antecedentes de dolor epigástrico de 1 mes de evolución, después de una deposición con características de melena sufre fuertes mareos, acudiendo a consultorio. El paciente luce pálido y sudoroso, constatándose al examen físico taquicardia y taquipnea. No se logra medir presión arterial.

Con el fin de preservar los órganos vitales, los mecanismos de compensación son puestos en marcha, tales como aumento del tono adrenérgico, de la frecuencia y contractilidad cardíacas, vasoconstricción cutánea, muscular, esplácnica, entre otros, los cuales pueden o no contrarrestar el shock en forma inicial. De acuerdo a la causa del shock y a los mecanismos compensadores existentes, un determinado patrón hemodinámico se pone en evidencia, el cual siendo característico de cada tipo de shock presenta utilidad para el diagnóstico diferencial.

Definición Estado de inadecuada perfusión tisular (O2 y nutrientes) de acuerdo a los requerimientos metabólicos del organismo, llevando a falla orgánica múltiple y muerte. Inicialmente reversible, pero si la hipoperfusión se mantiene se torna irreversible. Shock: Hipotensión Arterial (PAS < 90mmHg, disminución PAS > 40 mmHg, o PAM < 60 mmHg) + signos hipoperfusion

Patrones hemodinámicos según tipo de Shock :

tisular.

Tipo de Shock

Etiología-Epidemiología-Fisiopatología

Gasto Cardiaco

Resistencia Presión Vascular Periférica Venosa Central

Hipovolémico Cardiogénico Obstructivo Distributivo

Una adecuada perfusión tisular requiere un sistema respiratorio funcional (intercambio de O2 y CO2), una adecuada cantidad de O2 y nutrientes disueltos en sangre, un buen funcionamiento cardíaco (bombeo de sangre) y un íntegro sistema de vasos sanguíneos. Alteraciones en cualquiera de estos componentes pueden llevar al shock. De acuerdo a la alteración predominante, se distinguen varios tipos de shock. Tipos de Shock: Hipovolémico: • Mecanismo: Disminución del volumen intravascular (Precarga).

Diagnóstico Clínico, no obstante determinar la etiología puede requerir exámenes complementarios. Cuadro clínico: Hipotensión arterial, taquicardia, palidez, piel fría y moteada, llene capilar lento en lechos ungueales, ta-

51 Facultad de Medicina, Universidad de Chile

quipnea, compromiso de conciencia, oliguria, acidosis metabólica (por acumulación de lactato). Otras manifestaciones clínicas dependerán del tipo de shock y la causa de este.

Tratamiento El tratamiento inicial consiste en la aplicación de medidas generales de soporte vital en función del cuadro clínico, asegurar un adecuado acceso vascular con el objeto de restablecer la perfusión tisular (reposición o expansión de volumen intravascular, uso de drogas vasoactivas, entre otros), corrigiendo concomitante o posteriormente la causa desencadenante específica. El tratamiento inicial es empírico, se realiza ABC de la reanimación y se debe intentar estabilizar para un rápido traslado al servicio de urgencia más cercano. Luego el tratamiento específico dependerá del tipo específico de Shock. Shock Hipovolémico tratar de detener la hemorragia, reanimación con cristaloides o soluciones coloidales si lo amerita. Shock Cardiogénico: Despejar vía aérea, ventilación adecuada, aporte de O2, aporte de volumen con precaución (recordar que es por falla de bomba), ECG, monitorización, tratamiento agudo de IAM en caso de ser su causa, manejo del ritmo, DVA si es necesario. Shock Obstructivo: En casos de neumotórax a tensión con compromiso hemodinámico se debe realizar de urgencia una punción pleural a nivel del segundo espacio intercostal en la línea medio clavicular. Shock Distributivo: Despejar vía aérea, ventilación adecuada, control hemorragias si las hay, aportar cristaloides, cultivos, tratamiento ATB empírico en caso de sepsis; adrenalina 0,5mg IM en caso de anafilaxis.

Seguimiento Derivar a SU lo más pronto posible, monitorizado, con aporte de O2, cristaloides, etc.

Autor / Editor Francisco Weber

Año 2017

52 Facultad de Medicina, Universidad de Chile

Taquiarritmias y Bradiarritmias con Compromiso Hemodinámico

compromiso hemodinámico: Hipotensión, frialdad de piel y extremidades, oligo-anuria, compromiso de conciencia. Se confirma con un electrocardiograma (ECG) de 12 derivaciones, que evidencia bradicardia (< 60 lpm) o taquicardia (> 100 lpm) asociada a alteraciones propias de cada arritmia. También se considera cuando la FC no es la adecuada para la condición clínica.  

Nivel de manejo del médico general: Diagnóstico Específico Tratamiento Inicial Seguimiento Completo

Tratamiento Aspectos Esenciales • •

Evaluar hemodinamia y adecuada perfusión tisular. Taquicardia y compromiso HD: Cardioversión eléctrica o desfibrilación.



En bradicardia: atropina, si no responde: MET.

El manejo inicial en ambas consta de apoyo con oxígeno al 100% y/o ventilatorio (si lo requiere), establecer una vía venosa periférica, monitorización ECG y PA. Bradiarritmia: La presencia de signos adversos (hipotensión significativa, FC < 40 lpm, arritmias ventriculares significativas, insuficiencia cardíaca aguda, deterioro del nivel de conciencia, síncopes de repetición en reposo), o un elevado riesgo de asistolia (bloqueo AV avanzado con presencia de QRS ancho por escape ventricular o trastornos de conducción intraventricular asociados, pausas ventriculares de > 3 seg) obligan a tomar una actitud agresiva y urgente, con el fin de restablecer una frecuencia cardíaca óptima y perfusión adecuada de los órganos principales. Administrar Atropina (0,5 mg, c/3 min, hasta 3 mg) si no responde considerar marcapaso externo transitorio (transcutaneo o transvenoso).

Caso Clínico Tipo Mujer de 65 ingresa con Angor de reciente aparición y palpitaciones. ECG evidencia taquicardia de complejo angosto a 180 lpm, persistiendo con angina. Se decide monitorizar y cardiovertir, tornándose asintomática y la FC baja a 70 lpm, con ritmo sinusal.

Taquiarritimia: Nos podemos plantear 2 escenarios, que el complejo QRS sea ancho (regular o irregular) o angosto. Si presenta: hipotensión, compromiso de conciencia, shock, SCA, o Insuficiencia cardiaca aguda; se debe cardiovertir, a menos que presente ausencia de pulsos o paro cardiorrespiratorio que se debe desfibrilar. En las de complejo angosto (que no cumple lo anterior) se prefiere iniciar maniobras vagales en una primera instancia (Compresas frías en cabeza y cuello, maniobra de Valsalva). No se recomienda el masaje carotídeo, por riesgo de soltar un trombo a este nivel y producir un ACV. Si continua luego de las maniobras vagales, el fco. de elección es la adenosina, pueden utilizarse B bloqueo o bloqueadores de canales de calcio no DHP. En las de complejo ancho se debe dar infusión de antiarrítmico (amiodarona). En TV polimorfas: sulfato de magnesio. Evaluar constantemente aparición de PCR.

Definición Son alteraciones del ritmo cardíaco, con aumento (taquiarritmia) o disminución (bradiarritmia) de la frecuencia normal, que si no son tratadas de manera adecuada y oportuna pueden comprometer la vida del paciente.

Etiología-Epidemiología-Fisiopatología Las arritmias pueden comprometer la vida del paciente, al impedir que la bomba cardíaca pueda mantener eficazmente la perfusión propia y de los principales órganos (compromiso hemodinámico), creando isquemia e insuficiencia funcional de los mismos. Las condiciones que pueden llevar a esta son múltiples: Taquiarritmias: FA, flutter auricular, taquicardia ventricular, fibrilación ventricular, taquicardia ventricular polimorfa (torsión de puntas), etc. Bradiarritmias: Bloqueo AV, bradicardia, bloqueo y paro sinusal, enfermedad del nodo, entre otras.

Seguimiento Es por parte del especialista.

Diagnóstico

Autor / Editor Francisco Weber

La sospecha se debe de tener en todo paciente que presenta compromiso de conciencia (SÍncope) o que presenta un síndrome coronario agudo, edema pulmonar y otros signos de

53 Facultad de Medicina, Universidad de Chile

Año 2017

BIBLIOGRAFÍA •

Torres, J.M., Murillo, L., (2015). Arritmias cardíacas: estrategia diagnóstica y tratamiento general en J.M. Torres, L. Jiménez y M.A. Romero (Eds.), Urgencias Médicas cardiovasculares (pp 146 – 151). Elserver, España.



Torres, J.M., Murillo, L., (2015). Arritmias por alteración en la conducción del impulso. Enfermedad del seno en J.M. Torres, L. Jiménez y H. Degayón (Eds.), Urgencias Médicas cardiovasculares (pp 178 – 181). Elserver, España.



Torres, J.M., Murillo, L., (2015). Arritmias sinusales y auriculares en J.M. Torres, L. Jiménez y H. Degayón (Eds.), Urgencias Médicas cardiovasculares (pp 153 – 159). Elserver, España.



Torres, J.M., Murillo, L., (2015). Fibrilación y Flúter auriculares. Arritmias auriculoventriculares y ventriculares en J.M. Torres, L. Jiménez y H. Degayón (Eds.), Urgencias Médicas cardiovasculares (pp 160 – 176). Elserver, España.



Guía clínica MINSAL (2011) trastornos de generación del impulso cardíaco y conducción en personas de 15 años y más que requieren marcapasos. Santiago de Chile, MINSAL.



Montagud, V., et al. (2015). Arritmias. Fundación española del corazón. Recuperado el 25 de septiembre de 2016, de http://www.fundaciondelcorazon.com/ informacion-para-pacientes/enfermedades-cardiovasculares/arritmias.html



Fajuri, A. (2009). Manual de arritmias. Unidad de cardiología de la Facultad de Medicina de la universidad católica de Chile. Recuperado el 26 de septiembre de 2016, de http://www.cardiouc.cl/CardiologiaUC/Arritmias/Arritmias_4.pdf



https://www.uptodate.com/contents/overview-of-the-acute-management-of-unstable-angina-and-non-st-elevation-myocardial-infarction?source=search_result&search=inestable%20angina&selectedTitle=1~150



https://www.uptodate.com/contents/clinical-features-and-diagnosis-of-acute-aortic-dissection?source=search_result&search=diseccion%20aortica&selectedTitle=1~150



https://www.uptodate.com/contents/overview-of-the-acute-management-of-st-elevation-myocardial-infarction?source=search_result&search=infarto%20agudo%20al%20miocardio&selectedTitle=1~150



https://www.uptodate.com/contents/overview-of-sudden-cardiac-arrest-and-sudden-cardiac-death?source=search_result&search=paro%20cardiorespiratorio&selectedTitle=1~150



https://www.uptodate.com/contents/definition-classification-etiology-and-pathophysiology-of-shock-in-adults?source=search_result&search=shock&selectedTitle=1~150



https://www.uptodate.com/contents/arrhythmia-management-for-the-primary-care-clinician?source=search_result&search=arritmias&selectedTitle=1~150

54 Facultad de Medicina, Universidad de Chile

▪ Cianóticas (Cortocircuito Der a Izq.):

Cardiopatía congénita en adulto

1 Obstrucción corazón derecho: Tetralogía de Fallot (Es la más frecuente de las cianóticas, se caracteriza por episodios de crisis hipoxémicas). 2 Mezcla total: Ventrículo único. 3 Falta de mezcla: Transposición de los grandes vasos.

Nivel de manejo del médico general: Diagnóstico Sospecha Tratamiento Inicial Seguimiento Derivar

Existe asociación entre algunas malformaciones con distintas etiologías, como:

Aspectos esenciales  



Sd. Turner (coartación aórtica) Sd. Down (canal AV) Rubéola materna (estenosis pulmonar, ductus) Sd. Holt-Oram (CIA)

Las 3 más frecuentes son: Comunicación interventricular (CIV), persistencia del ductus arterioso y comunicación interauricular (CIA). Estos cortocircuitos pueden progresar con un síndrome de Eissenmenger (inversión del flujo de VD a VI), caracterizado por cianosis y dedos en palillo de tambor. El examen de elección es el Ecocardiograma.

Por tanto, la pesquisa en forma dirigida en ese grupo de pacientes es fundamental (ecocardiograma en el RN).

Clínica CIV: Si es pequeña son asintomáticas y el soplo es holosistólico intenso; si hay hipertensión pulmonar se deben buscar lesiones asociadas. En pacientes adultos con CIV pequeña no operada muestran mayor incidencia de arritmias, estenosis subaórtica, e intolerancia al ejercicio. En CIV grandes hay signos de cardiomegalia y congestión pulmonar en la radiografía. El estudio de estos pacientes incluye ECG, ecocardiograma, estudio hemodinámico y oximetría.

Caso clínico tipo Mujer de 37 años, cursando su primer embarazo. Antecedente de soplo en la infancia, en la semana 32 comienza a presentar disnea de esfuerzos pequeños y edema de extremidades. RR2T, R2 pulmonar aumentado y ampliamente desdoblado, cursa con BIRD en el ECG.

Ductus: Es más frecuente en mujeres. Si el defecto es grande, puede cursar con síntomas de insuficiencia cardíaca congestiva, al examen físico hay soplo sistodiastólico continuo "en maquinaria" en región infraclavicular izquierda, hay signos de crecimiento auricular y ventricular izquierdo en radiografía. El estudio de estos pacientes incluye ECG y ecocardiograma.

Definición Alteraciones anatómicas de una o varias de las cuatro cámaras cardíacas, de los tabiques o de las válvulas o tractos de salida originadas durante el desarrollo intrauterino, pesquisadas en el paciente adulto.

CIA: Es del tipo no cianótica, es más frecuente en mujeres, hay crecimiento de AD, VD y dilatación de la arteria pulmonar. La mayoría puede ser asintomático en la niñez, se puede pesquisar por un soplo sistólico de eyección, R1 fuerte a veces acompañado de click de eyección pulmonar, y R2 fuerte y desdoblado amplia y fijamente en todas las fases de la respiración, puede haber una rodada tricuspídea (indicador de CIA severa). Se confirma el diagnóstico con radiografía de tórax, ECG, ecocardiograma, y estudio hemodinámico.

Etiología-epidemiología-fisiopatología Actualmente el adecuado control de salud pediátrico ha aumentado el número de adultos respecto a los niños que son portadores de una cardiopatía. Debido a lo amplio del tema se abarcarán las más frecuentes: ▪ Comunicación interventricular (24-40%) ▪ Persistencia del ductus arterioso (8.6-12%) ▪ Comunicación interauricular (6.6-9.2%) ▪ Aorta bicúspide (0.5 a 2% en población general, 24% en familiares de primer grado).

Tratamiento CIV: Observación, médico o quirúrgico según tamaño del defecto, magnitud cortocircuito, localización y antecedente de EBSA. Ductus: Cierre quirúrgico o por catéter. No requiere profilaxis para endocarditis bacteriana, a diferencia de la CIV y la CIA. CIA: Puede ser quirúrgico, evaluado y realizado por especialista.

Diagnóstico El Ecocardiograma es el examen de elección para el estudio y diagnóstico de estas patologías. Una clasificación que permite sistematizar cardiopatías congénitas es la siguiente:

las

▪ Acianóticas:

Seguimiento

1 Cortocircuito Izq. a Der: CIV, CIA, Ductus y Canal AV. 2 Obstrucción corazón izquierdo: Coartación de la aorta. 3 Insuf. Valvulares y otras: Estenosis pulmonar.

Por especialista. La práctica de ejercicios moderados es recomendable. Respecto al embarazo, hay que analizar los riesgos individuales dependiendo de la cardiopatía subyacente. 1

utilidad son: RxTx evidencia crecimiento cavidades derechas.

Corazón pulmonar crónico

Electrocardiograma, en él se puede ver bloqueo de rama derecha (rSR' en V1, S profunda en DI y si es completo, QRS > 120ms de ancho), desviación de eje a derecha, P pulmonar y signos de Hipertrofia y sobrecarga de VD (T negativas en V1 a V4).

Nivel de manejo del médico general: Diagnóstico Específico Tratamiento Inicial Seguimiento Derivar

Tratamiento

Aspectos esenciales    

El control de la causa desencadenante es la medida más importante. Se debe procurar mantener una saturación de O2 adecuada (O2 suplementario si fuese necesario), evitar poliglobulia. Los fármacos utilizados son vasodilatadores, dentro de los que destacan el sildenafilo (no mezclar con nitratos) y los bloqueadores de canales de calcio. Puede utilizarse diuréticos en pacientes con evidencia de VEC expandido y en pacientes con VD dilatado y gran interdependencia ventricular. En pacientes hospitalizados con cor pulmonar agudo o crónico reagudizado pueden utilizarse inótropos como milrinona y dobutamina y ultrafiltración como manejo de la sobrecarga de volumen. El manejo definitivo es el transplante Cardio-pulmonar.

Enfermedad poco frecuente. Se debe sospechar frente a síntomas de IC derecha. El examen de elección es el Ecocardiograma. El manejo inicial: identificar y tratar la causa subyacente.

Caso clínico tipo Hombre 70 años con antecedentes de TVP recurrente. Presenta disnea de 4 meses de evolución, aumento de volumen abdominal y de EEII. Al examen se evidencia además yugulares ingurgitadas y soplo mesosistólico en foco pulmonar.

Seguimiento Es por parte del especialista.

Definición Es la afectación cardiaca (principalmente dilatación y/o hipertrofia de cavidades derechas asociado o no a disfunción sistólica), secundaria a hipertensión pulmonar crónica (HTPC), que se define como presión media de arteria pulmonar (PAPm) >25 mmHg.

Etiología-epidemiología-fisiopatología Las causas son variadas, en Chile: EPOC (80-90% de los casos), Fibrosis pulmonar, TEP crónico, hipoxemia crónica, sarcoidosis, idiopática, etc. Todas estas condiciones producen hipertensión pulmonar (HTP), que lleva a dilatación e hipertrofia del ventrículo derecho (VD), aumento de presión de fin de diástole con dilatación secundaría de aurícula derecha y finalmente hipertensión venosa sistémica. Recordar a la Hipertensión pulmonar primaria en mujeres de edad fértil como causa más importante de contraindicación de embarazo. Es importante de la definición que la HTP y falla cardíaca derecha secundaria a falla cardíaca izquierda o cardiopatías congénitas NO se considera Cor pulmonale.

Diagnóstico La sospecha es clínica. Dentro de los elementos sugerentes tenemos: disnea (con frecuencia corresponde al primer síntoma), hipoxia, ascitis y edema periférico; al examen físico, yugulares ingurgitadas, hepatoesplenomegalia, acropaquia, edema periférico, soplo sistólico eyectivo en foco pulmonar con R2 aumentado en intensidad, palpación del VD en el precordio. Se confirma con Ecocardiograma, que permite objetivar compromiso de VD, además de calcular PAP; otros exámenes de 2

perfil lipídico completo cada 5 años a pacientes de 20 años o más.

Dislipidemias

Coltotal TG

Nivel de manejo del médico general: Diagnóstico: Completo Tratamiento: Específico Seguimiento: Completo

Valores normales Niveles de riesgo <200 mg/dl. Nivel de riesgo >240 mg/dl. <150 mg/dl.

Nivel de riesgo >200 mg/dl.

LDL

<130 mg/dl.

Alto riesgo >500 mg/dl. Nivel de riesgo >130 mg/dl.

HDL

>50 mg/dl mujeres.

Aspecto esenciales     

Alteración en la concentración de lípidos en la sangre. El manejo debe basarse en el riesgo cardiovascular (RCV) individual. El tratamiento consta de medidas no farmacológicas y farmacológicas. Se debe comenzar un tamizaje de la enfermedad a partir desde los 20 años, cada 5 años, con un perfil lipídico completo. Niveles de Colesterol LDL >190mg/dL se considera de alto RCV independientemente de la presencia y/o ausencia de otros FR.

Alto riesgo >160 mg/dl. en Nivel de riesgo <35-40 mg/dl.

>40 mg/dl en hombres. *Col-total: LDL+HDL+VLDL(TG/5). Actualmente los niveles de Colesterol LDL >190mg/dL se considera de alto RCV independientemente de la presencia y/o ausencia de otros FR.

Tratamiento Tratamiento no farmacológico: -Objetivos: reducción de Col-total, LDL y TG, y elevación HDL. -Promoción actividad física (aérobica, 150 min semanales). Dieta baja en carbohidratos simples, colesterol, grasas saturadas, rica en fibra, ácidos grasos insaturados, omega3. Suspensión de tabaco y disminuir ingesta de alcohol. -Los jóvenes de bajo riesgo cardiovascular solo necesitarán de estas medidas para lograr metas.

Caso clínico tipo Paciente 40 años, hipertenso y diabético, con poca adherencia al tratamiento, se encuentra con Coltotal >240 mg/dl en exámenes de control.

Definición Concentraciones anormales de lípidos en la sangre: colesterol total (Col-total) lipoproteínas de alta densidad (HDL), lipoproteínas de baja densidad (LDL) y triglicéridos (TG), las cuales implican un riesgo para la salud, especialmente un riesgo cardiovascular.

Tratamiento farmacológico Se inicia al tener dos perfiles lipídicos concluyentes. El 1º objetivo es alcanzar las metas de LDL según el riesgo cardiovascular (RCV) del paciente. Esta terapia está indicada en individuos con enfermedad ateromatosa o múltiples factores de riesgo, si tiene RCV alto, diabetes o si no se logra cumplir las metas de LDL con terapia no farmacológica.

Etiología- epidemiología- fisiopatología Se puede clasificar a las dislipidemias según su etiopatogenia en: -Dislipidemias primarias (genéticas): Se caracterizan por valores de lípidos muy altos (Col-total >300 mg/dl; TG >400 mg/dl) o niveles de Col-HDL muy bajos (<25 mg/dl), con TG normales. -Dislipidemias secundarias (secundarias a otras patologías o factores ambientales): Debe considerarse la obesidad y el sedentarismo como factores condicionantes ya que su abordaje mejora el pronóstico del paciente.

Las metas terapéuticas de LDL según RCV son las siguientes:   

O según su la clínica: -Hipercolesterolemia aislada. -Hipertrigliceridemia aislada. -Dislipidemias mixtas. -HDL bajo.

Diagnóstico: El diagnóstico se basa en los niveles de Col-total, LDL, HDL y TG. Se solicita un perfil lipídico, el cual debe ser tomando con un ayuno de 12 horas. Se debe realizar un 3

Alto: Col-LDL <70mg/dl o >50% de reducción si no es posible lograr el objetivo anterior. Moderado: Col-LDL <100mg/dl. Bajo: Col-LDL <130mg/dl.

Los fármacos más utilizados son:

Seguimiento:

De estos fármacos, las estatinas son los más eficaces en reducir los niveles de colesterol LDL. Además de su efecto hipolipemiante, se han descrito otros efectos beneficiosos como: antiarrítmico, antiinflamatorio y antioxidante. Su principal efecto adverso son miopatías (debilidad muscular y/o mialgias), con frecuencia de presentación dosis dependiente, que pueden presentarse dentro de los 3 primeros meses de tratamiento. Se recomienda medir niveles basales de CK. Si el paciente refiere síntomas se deben controlar los niveles de CK, si estos son 10 veces el nivel basal, se debe suspender el medicamento (riesgo de rabdomiolisis). Se recomienda también medición de niveles basales de transaminasas hepáticas (ALT o GPT), dentro de los 3 meses de inicio del tratamiento, se debe suspender el medicamento si estos valores son 3 veces el nivel basal.

Pacientes de bajo y moderado riesgo CV se puede controlar con el Col-total, cuyo valor es una buena aproximación al Col-LDL. Para la mayoría de los pacientes un nivel de Col-total de 200-240 mg/dL equivale a un Col-LDL de 130-160 mg/dL; una meta de Col-LDL <160 mg/dL se puede homologar a un Col-total<240 mg/dL, y un nivel de Col-LDL<130 mg/dL equivale a un Col-total <200mg/dL. Una vez se logre la meta referida al Col-total, se debe confirmar con una medición del nivel de Col-LDL. La frecuencia de los controles dependerá del RCV del paciente. Inicialmente se debe realizar control cada 7 a 14 días hasta lograr las metas según RCV. Luego de lograr las metas el control debe ser cada 3 meses en pacientes con RCV alto, cada 6 meses en pacientes con RCV moderado y anual en pacientes con RCV bajo.

Los fibratos (como el gemfibrozilo) se usan fundamentalmente para la reducción de los TG cuando estos se encuentran en concentraciones mayores a 500mg/dL dado el riesgo de desarrollar de pancreatitis aguda. El uso concomitante de estatinas y gemfibrozilo está CONTRAINDICADO.

4

5

Tratamiento

Embolia cardiogénica Nivel de manejo del médico general: Diagnóstico Específico Tratamiento Inicial Seguimiento Completo

Siempre se debe de iniciar anticoagulación con heparina (no fraccionada o de bajo peso). En los casos en que exista compromiso de la extremidad se debe realizar la embolectomía (previo a que exista compromiso motor, ya que después el daño es irreversible). Una vez estabilizado el paciente debe de ser derivado a un centro terciario.

Aspectos esenciales

Seguimiento

   

El lugar más frecuente de embolia son las EEII. Las 6 P son indicadores clínicos de insuficiencia arterial aguda. Hasta el 30% de los AVE isquémico son de origen cardiembólico. El manejo inicial incluye anticoagulación.

Se debe mantener el tratamiento anticoagulante, manejo de las secuelas y la patología subyacente. El seguimiento es por parte del especialista.

Caso clínico tipo Hombre de 75 años, con antecedentes de palpitaciones, consulta al Servicio de urgencia por cuadro de 2 horas de evolución de parestesias en extremidad inferior izquierda asociadas a frialdad. Lo examina y encuentra ausencia de pulsos distales, palidez, sin compromiso motor. Al examinar pulsos, encuentra un pulso irregular con un ECG compatible con FA. Inicia HBPM y deriva un centro terciario.

Definición Es la impactación de un embolo proveniente del corazón en un vaso sanguíneo.

Etiología-epidemiología-fisiopatología Las causas más frecuentes son FA no valvular (45%), IAM reciente, aneurisma ventricular, FA asociada a valvulopatía reumática, válvulas protésicas y otras (EBSA, mixoma auricular entre otros). El compromiso arterial lleva a la lesión de los tejidos, siendo el más susceptible a la injuria, el cerebro.

Diagnóstico Para llegar al diagnóstico lo más importante es mantener una ALTA sospecha, ya que las manifestaciones clínicas variarán de acuerdo al territorio comprometido. Si compromete el cerebro se manifestará como un AVE (en cualquiera de sus formas), en caso de comprometer una extremidad presentará: Palidez, paresia, parestesia, ausencia de pulso, dolor (Pain) y poiquilotermia (6P), además de buscar signos de la patología de base (como el pulso irregular en la FA). Dentro de los exámenes complementarios son de utilidad el TAC de cerebro, electrocardiograma (evidencia FA), ecocardiograma (TT y TE) (permite evidenciar presencia de trombos auriculares, valvulopatías, etc.) y el AngioTAC de extremidades, que evidencia la amputación de la circulación arterial.

6

resistencia del territorio vascular secundaria a tanto a obstrucción por el émbolo como por vasoconstricción pulmonar secundaria a la hipoxemia; este aumento de la resistencia vascular pulmonar lleva a dilatación del ventrículo derecho, y eventualmente a disminución del gasto cardíaco debido a caída de la precarga.

Embolia pulmonar Nivel de manejo del médico general: Diagnóstico: Sospecha Tratamiento: Inicial Seguimiento: Derivar

Diagnóstico La sospecha es clínica, debe tenerse cuando nos encontramos frente a un paciente portador de alguno de los factores de riesgo y que presente súbitamente, disnea, dolor torácico de tipo pleurítico, taquipnea, y/o hemoptisis. Frecuentes, pero menos específicos son la presencia de tos, fiebre y taquicardia. Los TEP masivos pueden presentar síncope, hipotensión o incluso paro cardiorespiratorio. En el examen físico debemos buscar signos de TVP, crépitos a la auscultación pulmonar o signos de compromiso cardíaco: 3T o un soplo de Graham Steel (soplo protodiastólico producido por una insuficiencia pulmonar relativa que se ausculta en el foco pulmonar). Podemos dividir a los pacientes con sospecha de TEP en pacientes con alta probabilidad pre-test y baja probabilidad pre-test usando el score de Wells simplificado.

Aspectos esenciales    

La causa más frecuente es TVP femoro-poplítea. La disnea y el dolor torácico son cardinales. El Gold standard para el diagnóstico es la AngioTAC de tórax. Si la sospecha es alta se debe iniciar el tratamiento anticoagulante sin demora.

Caso clínico tipo Mujer hospitalizada por Fx de cadera no resuelta, presenta de manera súbita disnea y dolor torácico con tope inspiratorio. Al examen está normotensa, taquicárdica y taquipneica. Dada la alta sospecha clínica solicita Angio TAC de tórax que muestra TEP. Inicia manejo con HBPM.

Otros exámenes que son de utilidad (sospecha): ECG (Bloqueo de rama derecha, desviación del eje a derecha y patrón S1Q3T3), Ecocardiograma que evidencia compromiso de ventrículo derecho y mide la PAP (no hace el diagnóstico), y las enzimas cardíacas que pueden estar elevadas (indica compromiso del miocardio).

Definición La tromboembolia pulmonar (TEP) es la obstrucción al flujo arterial pulmonar, secundario a trombo que frecuentemente se origina y desprende del territorio venoso. De acuerdo a su localización se caracterizan como centrales o proximales (de la art. pulmonar principal, lobar o segmentaria) y distales o subsegmentarios. Pueden ser de presentación aguda, manifestando síntomas a las horas de la obstrucción, de presentación sub-aguda, con manifestaciones clínicas a los días o semanas de la obstrucción, o de presentación crónica, desarrollándose a través del curso de años (por ej., la hipertensión pulmonar trombo-embólica crónica). Se define como TEP masivo en sujetos sin enfermedad cardiopulmonar previa una obstrucción > 50% del lecho vascular pulmonar y en aquéllos con enfermedad cardiopulmonar una obstrucción > 23%, que causa hipotensión (PAS < 90 mmHg o una disminución en la PAS ≥ 40 mmHg desde su basal, por un período > 15 minutos).

La probabilidad pre-test nos ayuda a definir la conducta diagnóstica, de acuerdo al siguiente algoritmo:

Tratamiento El manejo inicial debe ir enfocado a la estabilización del paciente: hemodinamia, suplementación de oxígeno en caso de hipoxemia (y valorar la necesidad de ventilación mecánica). Si el paciente tiene una baja probabilidad de TEP, se solicita un dimero-D, el cual tiene un alto valor predictivo negativo, de esta manera, una baja probabilidad de TEP más dimero-D negativo, descarta la posibilidad de TEP. En casos de alta sospecha con hemodinamia estable, se debe iniciar tratamiento con heparina corriente o de bajo peso molecular mientras se prepara al paciente para el angioTAC. Una vez confirmado por este examen, se inicia el tratamiento anticoagulante oral paralelo a la heparina, la que se suspende una vez que el tratamiento oral esté en niveles terapéuticos (INR 2-3). El tratamiento anticoagulante oral se mantiene por 3 a 6 meses, y si el factor de riesgo se mantendrá activo, será de por vida.

Etiología-epidemiología-fisiopatología La principal causa de TEP es la migración de un émbolo arterial proveniente de una TVP (60% de los pacientes con TEP presentan una TVP). Los factores de riesgo principales son los que determinan la denominada Tríada de Virchow (estasia venosa, daño endotelial e hipercoagulabilidad): cirugías mayores (abdominpelvianas, cadera y EEII), cesárea y puerperio, neoplasias, fracturas/traumas de EEII, inmovilización prolongada, tabaquismo, uso de anticonceptivos orales y el antecedente personal de TVP o TEP.

En casos de alta sospecha de TEP con hemodinamia inestable, se inicia tratamiento con heparina no fraccionada o de bajo peso molecular a dosis plena, en espera del resultado del AngioTAC. Una vez confirmado, se realiza trombolisis, siempre y cuando no haya contraindicaciones absolutas. La trombolisis se puede realizar con estreptoquinasa (250.000 U EV los primeros 30 minutos y luego 100.000 U/h por 24 horas, con atención a aparición de hipotensión, anafilaxia, asma o reacciones alérgicas), tPA (100 mg EV en 2 horas).

El émbolo puede producir infartos pulmonares, alteración de la relación V/Q, y compromiso de corazón derecho. La afectación cardíaca está mediada por aumento de la 7

Generalmente se descontinúa la terapia anticoagulante durante la infusión del trombolítico.

Algunos autores plantean que ante un paciente con evidente probabilidad de TEP y hemodinamia inestable, podría iniciarse de entrada trombolisis, idealmente con tPA (tenecteplase 100mg a pasar en 2 horas) acompañado de heparina. Y realizar luego sin premura el AngioTAC para confirmar la sospecha, sin embargo, está en discusión.

Se contraindica la trombolisis en caso de neoplasia intracraneana, trauma o cirugía intracraneana reciente (<2 meses), hemorragia interna activa o reciente en los últimos 6 meses, historia de ACV hemorrágico, coagulopatía, HTA severa e incontrolada, ACV no hemorrágico en los últimos 2 meses, cirugía en los últimos 10 días y trombocitopenia. Si estuviese contraindicada se debe realizar estudio angiográfico y trombectomía o lisis del coágulo.

Seguimiento Debe ser por especialista, decidir por cuánto tiempo mantener ACO (INR 2-3) y evaluar la necesidad de buscar algún factor protrombótico (por ejemplo, neoplasia oculta o trombofilia).

8

Diagnóstico

Enfermedad reumática activa

Se manifiesta principalmente con compromiso articular aproximadamente en un 75-80% de los casos, correspondiente a una artritis migratoria de grandes articulaciones (rodillas, tobillos, muñecas, codos).

Nivel de manejo del médico general: Diagnóstico Específico Tratamiento Completo Seguimiento Completo

Compromiso cardiaco (pancarditis) entre 50-60%, manifestado por daño a nivel del miocardio por degeneración fibrinoidea del colágeno, el granuloma de Aschoff (lesión endocárdica, principalmente a nivel de las válvulas), y lesión pericárdica que se manifiesta como un derrame serofibrinoso de tipo inespecífico; se pesquisará soplos de insuficiencia mitral o aortica, frotes pericardicos, cardiomegalia o signos de congestión pulmonar.

Aspectos esenciales      

Diagnóstico: 2 criterios mayores o 1 criterio mayor y 2 menores siempre que se demuestre infección por SGA. El 75% de los casos son autolimitados y remiten espontáneamente antes de las 6 semanas. Aparición de enfermedad reumática (ER) en promedio es de 3 semanas después del episodio agudo de infección por SGA. Manejo agudo: Reposo, erradicación de SGA, manejo de artritis y carditis. La prevención primaria reduce un 70% la aparición de ER. El compromiso cardíaco permanente estará dado por la aparición de nuevos brotes, el grado de severidad y la duración de estos episodios.

Compromiso neurológico (Corea de Sydenhan), de inicio más tardío, en aproximadamente un 10-20%, se presentará como movimientos descoordinados, involuntarios e inapropiados realizados de manera inconsciente por el paciente. Presencia de nódulos subcutáneos y eritema marginado en un 2-3%.; además puede presentarse sintomatología inespecífica como CEG, artralgias, fiebre, etc. Exámenes: Elevación en los títulos para anticuerpos antiestreptolisina O (ASO), que pueden permanecer alrededor de 6 a 8 semanas (hasta un año), PCR y VHS elevados y ECG con bloqueo AV de 1° grado algunas veces.

Caso clínico tipo

El diagnóstico se realiza mediante los Criterios de Jones, con 2 criterios mayores o 1 criterio mayor más 2 menores junto con la evidencia de infección por SGA (ASLO +). La confirmación de una infección estreptocócica se hace mediante una prueba bacteriológica directa o un cultivo faríngeo. Indirectamente podremos evidenciarla mediante la determinación de títulos de Antiestreptolisina O (ASO), anti-DNasa B, anti-estreptokinasa o anti-hialuronidasa.

Paciente de 12 años consulta por gonalgia bilateral, EVA 6/10, de 3 días de evolución asociado a fiebre, cefalea y anorexia. Relata molestias similares en tobillos y codo izquierdo no asociado a evento traumático. En la historia se encuentra el antecedente de una faringoamigdalitis tratada hace 1 mes. Al examen físico impresiona un leve aumento de volumen, doloroso a la movilización pasiva, en las articulaciones en cuestión y soplo sistólico en foco aórtico 3/6.

Diagnóstico con 2 criterios mayores o 1 criterio mayor y 2 menores siempre que se demuestre infección por SGA. CRITERIOS DE JONES

Definición Enfermedad inflamatoria con compromiso multisistémico, principalmente en corazón, articulaciones, tejido subcutáneo y sistema nervioso central, que se produce como secuela retardada a una infección faríngoamigdaliana o cutánea por Streptococo grupo A (pyogenes) (SGA).

CRITERIOS MAYORES

CRITERIOS MENORES

Carditis

Artralgia

Poliartritis

Fiebre

Corea

Elevación de VHS o PCR

Nódulos cutáneos

Etiología-epidemiología-fisiopatología Correspondería a una reacción inmune cruzada dada por similitud de antígenos en algunos tejidos (válvulas cardíacas, articulaciones) con antígenos del SGA. Existe una mayor susceptibilidad del huésped dada por ciertos antígenos de histocompatibilidad: HLA DRB1, DR6, DQB. La edad de presentación es preferentemente entre los 515 años y se manifiesta en promedio a las 3 semanas (15 semanas) de ocurrida la infección por SGA. Su prevalencia fue en marcado descenso hasta el año 1998, año que dejó de notificarse.

Sub- Intervalo P-R prolongado en el ECG

Eritema marginado

Carditis reumática previa

Tratamiento El cuadro suele ser autolimitado y remitir espontáneamente en un 75% antes de 6 semanas y un 90% antes de 12 semanas. Sin embargo, un 5% se prolonga más de 6 meses, pudiendo llegar a ser mortales en casos de carditis severas. Dentro del tratamiento se debe evaluar el manejo del episodio agudo y la profilaxis para la prevención de recurrencias.

9

Para el episodio agudo: Reposo y erradicación de SGA: Penicilina Benzatina 1.2 millones UI IM, Penicilina oral, Amoxicilina o Eritromicina por 10 días. Manejo de artritis: Aspirina 50-100 fraccionado en 4 dosis, por 2 a 4 semanas.

mg/Kg/día,

Manejo de la carditis: En caso de carditis severa se utiliza Prednisona 2 mg/Kg por 2 a 4 semanas y posterior cambio gradual a aspirina. Profilaxis Primaria (reduce 70% aparición ER): Tratamiento adecuado de todas las faringoamigdalitis estreptocócicas o sospechosas de estreptocócica con Penicilina Benzatina 1.2 millones UI, Penicilina oral, Amoxicilina o Eritromicina por 10 días.

Seguimiento La recurrencia de esta patología estará determinada por la nueva exposición a episodios de infección faríngoamigdaliana por SGA. A más largo plazo, la presencia de daño cardíaco permanente dependerá de la duración y gravedad de la carditis; de la aparición de nuevos brotes y de la magnitud y localización del daño valvular. Profilaxis Segundaria: PNC Benzatina 1.2 millones UI im cada 4 semanas por 5 años o hasta los 18 años de edad. En el caso de haber presentado carditis debe prolongarse por 10 años o hasta cumplir 25 años. Los pacientes con daño valvular residual debieran recibir profilaxis secundaria por períodos aún más prolongados.

10



Extrasistolía ventricular benigna

Tratamiento

Nivel de manejo del médico general: Diagnóstico Específico Tratamiento Inicial Seguimiento Completo

En el caso de EVB no requieren mayores intervenciones. En el caso de EV amenazantes o malignas requieren la derivación a especialista para la valoración de inicio de tratamiento con antiarrítmicos (en insuficiencia coronaria, miocardiopatía hipertrófica, historia familiar de muerte súbita, historia personal de síncope, prolapso mitral sintomático, síndrome QT largo).

Aspectos esenciales    

En pacientes sanos es de curso benigno. Su diagnóstico es por lo general como hallazgo en ECG. En su gran mayoría no requiere de tratamiento farmacológico. Derivar a especialista si se asocia a cardiopatías.

Seguimiento En EVB no se requiere de seguimiento (en general).

Caso clínico tipo Paciente de 65 años, con HTA, asintomático que dentro de exámenes de rutina se realiza ECG. Como único hallazgo se evidencian complejos QRS anchos aislados, no precedidos de onda P y que tras estos existe un aumento del intervalo de tiempo previo al inicio de un nuevo QRS.

Definición Una extrasístole ventricular (EV) es la presencia al ECG de complejos QRS prematuros, anchos y no precedidos de onda P debido a un foco ectópico ventricular. La presencia de 3 o más extrasístoles se denomina taquicardia ventricular. La consecución de 1 complejo sinusal seguido de un EV se denomina bigeminismo.

Etiología-epidemiología-fisiopatología Se produce por focos ectópicos ventriculares capaces de generar una contracción. La prevalencia depende del método de pesquisa utilizado; se describe hasta en un 75% de los holter de arritmia, mientras que se encuentra sólo en un 3% de los ECG de 12 derivaciones de 3060seg. en pacientes sin antecedentes cardiológicos.

Diagnóstico En general, son hallazgos al ECG tanto en pacientes sanos como con cardiopatía subyacente. Una apropiada valoración debe considerar:    

Características ECGs. Asociación de síntomas. Presencia de enfermedad subyacente. Función ventricular [FV].

Sobre esta base se pueden clasificar las EV en:  

Malignas: EV compleja y arritmias ventriculares, cardiopatía sintomática, FV comprometida.

Benignas: Aisladas, asintomáticas, ausencia de cardiopatía, FE preservada. Amenazantes: EV complejas, con o sin síntomas, con cardiopatía.

11

insuficiencia cardíaca aguda, estado mental alterado, dolor torácico isquémico), la cardioversión sincronizada está indicada [50 Joules]. En caso de estabilidad HDN, se debe priorizar el control de la frecuencia cardíaca (Digoxina, Bloq Ca+, B Bloq). Si la arritmia es de >48 hrs de duración se debe instaurar anticoagulación antes de cualquier modo de cardioversión. La cardioversión posterior puede conseguirse farmacológica o eléctricamente. El tratamiento definitivo es la ablación por radiofrecuencia del istmo cavotricuspídeo.

Flutter auricular Nivel de manejo del médico general: Diagnóstico Específico Tratamiento Inicial Seguimiento Derivar

Aspectos esenciales    

Seguimiento Taquicardia con pulso 150 lpm, regular, con QRS angosto al ECG. Su diagnóstico es por ECG. En el manejo es fundamental la evaluación hemodinámica El tratamiento definitivo es la ablación por radiofrecuencia.

Por especialista.

Caso clínico tipo Paciente de 65 años, con HTA, que posterior a cardiocirugía inicia cuadro de palpitaciones, disnea, dolor torácico e hipotensión. Se realiza ABC. Monitor revela taquicardia regular de QRS angosto, 150 lpm compatible con flutter auricular. Se procede a cardioversión sincronizada retornando ritmo sinusal.

Definición Es una taquiarritmia caracterizada por ritmo organizado, típicamente con una frecuencia auricular de 250 a 350 lpm, originado por una macroreentrada a nivel auricular derecho. Generalmente, se acompaña de un bloqueo AV 2:1, generando una frecuencia ventricular de 150 lpm.

Etiología-epidemiología-fisiopatología Se presenta de preferencia en edades avanzadas. El 60% se asocia a un factor desencadenantes (cirugía mayor, IAM, NAC, etc.). Sólo un 1,7% se presenta de manera aislada. La mayoría de los flutter son dependientes del istmo cavotricuspídeo (75% aprox.), nivel donde se establece la macroreentrada paralela al anillo valvular, debido a cambios en el ritmo auricular (p.ej.: extrasístole).

Diagnóstico Es fundamentalmente Electrocardiográfico, reconociendo elementos característicos:    

Taquicardia regular de complejo QRS angosto. Ondas “F” o en serrucho a frecuencia de 300 x min. BAV 2:1 con frecuencia ventricular 150 lpm. Ante bloqueo de nodo AV aumenta el BAV y se visualizan mejor las ondas F.

Tratamiento El manejo agudo implica la evaluación del ABC. Si el paciente presenta inestabilidad HDN (hipotensión, 12

1. Comienzo <30 ó >55 años, especialmente si no hay antecedentes familiares de HTA u otros factores de riesgo como obesidad. 2. PA ≥ 160/100 mmHg, particularmente si se asocia a repercusión orgánica. 3. HTA refractaria (uso de 3 antihipertensivos a dosis plenas, uno de ellos un diurético). 4. Síntomas y signos de enfermedad causante.

Hipertensión arterial secundaria Nivel de manejo del médico general: Diagnóstico: Sospecha Tratamiento: Inicial Seguimiento: Derivar

Durante el seguimiento de pacientes hipertensos, sospechar HTA secundaria si aparecen las siguientes claves en general:

Aspectos esenciales   

1. Hipertensión refractaria a tres drogas en dosis plenas, una de ellas un diurético. 2. Elevación significativa y persistente de la PA sobre valores previamente estables. 3. Hiperaldosteronismo primario: Hipokalemia bajo 3.0 meq/l en pacientes que usan diuréticos en dosis habituales o sin asociarse a medicamentos (pese a que esta última ocurre sólo en el 30% de estos pacientes). 4. Sospechar enfermedad renovascular si durante el tratamiento se pesquisa elevación de creatininemia (al menos 30%) en relación al uso de IECA o ARA II o episodios recurrentes de EPA o ICC inexplicables. También sospecharlo en pacientes con riñón pequeño unilateral o asimetría mayor a 1.5cm sin otra causa que lo explique. Generalmente se da en un contexto de ateroesclerosis difusa, por lo que se debe sospechar si inicia HTA después de los 55 años. Recordar que el examen de orina no debería estar alterado (a diferencia de la HTA secundaria a ERC).

10% de las HTA en Chile. Sospecharla frente a comienzo tardío, PA altas y HTA refractaria. Tratamiento médico y/o quirúrgico según etiología específica.

Caso clínico tipo Paciente de 56 años recientemente diagnosticado de HTA el que se manejó con IECA y bloqueador de canales de calcio. Se le agrega espironolactona observándose normalización de PA a las 8 semanas ¿cuál es el diagnóstico etiológico más probable?

Definición Hipertensión Arterial (HTA) que posee una etiología potencialmente curable.

Etiología-epidemiología-fisiopatología Tratamiento Epidemiología En Chile existiría una población estimada de 1.400.000 hipertensos, de los cuales sólo 140.000 (10%) personas tendrían una HTAS. La fisiopatología dependerá de cada enfermedad.

El tratamiento específico dependerá de la etiología de la HTAS. Las enfermedades hipertensivas que son tratables y eventualmente corregibles por métodos quirúrgicos y/o intervencionales son:  

Etiologías Causas frecuentes según grupo etáreo: 

   

    

<18 años: Enfermedad del parénquima renal (glomerulopatías, malformaciones congénitas y nefropatía por reflujo vésico ureteral), coartación aórtica. Adulto Joven: Estenosis de la arteria renal por displasia fibromuscular, hipo/hipertiroidismo, uso de ACO. Adulto Mediana Edad: Hiperaldosteronismo primario, apnea obstructiva del sueño, feocromocitoma, síndrome de Cushing. Adulto Mayor: Estenosis de la arteria renal por ateroesclerosis, ERC. Las posibles etiologías y los antecedentes de la anamnesis, examen físico y exámenes de laboratorios que nos orientan a un trastorno específico de la hipertensión secundaria se resumen en la tabla 1.

Estenosis de la arteria renal. Nefropatía unilateral secundaria a hidronefrosis o pielonefritis crónica. Adenomas productores de aldosterona. Síndrome de Cushing. Coartación aórtica. Feocromocitoma. Hiperparatiroidismo primario y acromegalia.

una

En cambio, aquellas que tienen un tratamiento médico específico y son potencialmente corregibles son:     

Hiperaldosteronismo por hiperplasia suprarrenal. Hiperaldosteronismo supresible por dexametasona. Hipertiroidismo. Hipotiroidismo. Síndrome de apnea hipopnea obstructiva del sueño (SAHOS).

Seguimiento

Diagnóstico

Derivar a especialista.

Es fundamental mantener un alto grado de sospecha de HTA secundaria si existen las siguientes claves:

13

caracteriza por hipertrofia segmentaria, habitualmente del septum interventricular, que puede causar obstrucción dinámica a nivel del tracto de salida del ventrículo izquierdo, dificultando su vaciamiento. Se produce falla ventricular diastólica, secundaria a un ventrículo grueso y rígido (poco elástico), resultando en una presión de llenado diastólico elevada, incluso a pesar de haber un ventrículo hiperdinámico.

Miocardiopatías Nivel de manejo del médico general: Diagnóstico: Sospecha Tratamiento: Inicial Seguimiento: Derivar

Miocardiopatía restrictiva: Disfunción diastólica secundaria a la rigidez de las paredes ventriculares que impide su llenado, causada por fibrosis e infiltración del ventrículo, sin dilatación y habitualmente sin hipertrofia. Entre las causas infiltrativas, la amiloidosis ocupa un lugar importante por su prevalencia. Una complicación importante es el trombo embolismo pulmonar. Presenta paredes ventriculares simétricas y algo gruesas con volúmenes normales, los índices de función sistólica son normales o poco alterados. Existe dilatación de ambas aurículas.

Aspectos esenciales    

Miocardiopatía hipertrófica (MCH) es la primera causa de muerte súbita en jóvenes deportistas. Ecocardiograma es el examen más costo-efectivo para el diagnóstico de la mayoría de las miocardiopatías (MC). Ante la sospecha de MC suspender ejercicios intensos inmediatamente. El antecedente de muerte súbita o diagnóstico de MC de un familiar hacen aumentar las probabilidades de diagnóstico con MC.

Miocardiopatía arritmogénica de ventrículo derecho (MAVD): Genética, presenta arritmias ventriculares con pared libre de VD reemplazada por tejido fibroadiposo. Presenta akinesia focal o diskinesia.

Caso clínico tipo

Miocardiopatía no clasificada: Enfermedades que no pueden ser incluidas en ninguna categoría. Son ejemplos la disfunción sistólica con mínima dilatación, enfermedad mitocondrial y fibroelastosis.

Hombre deportista de 18 años, el cual consulta porque su padre murió súbitamente a los 35 años, no presenta síntomas cardiovasculares, A la auscultación, soplo sistólico de eyección II/VI, no irradiado a cuello, Resto del examen normal.

Miocardiopatías específicas: Otras enfermedades que afectan al miocardio y que se asocian a una alteración cardiaca específica o que forman parte de una enfermedad sistémica.

Definición Grupo heterogéneo de enfermedades del miocardio asociadas con disfunción mecánica y/o eléctrica, debidas a una variedad de causas, aunque frecuentemente genéticas.

Diagnóstico Las manifestaciones clínicas están relacionadas con la falla cardíaca que produce cada una. En el caso de la MCR, el diagnóstico diferencial más importante es con la pericarditis constrictiva, esto por el carácter potencialmente curable de esta última.

Etiología-epidemiología-fisiopatología Pueden estar confinadas al corazón o ser parte de un desorden sistémico generalizado. Con frecuencia llevan a muerte cardiovascular o discapacidad debida a insuficiencia cardíaca progresiva. Se clasifican en hipertrófica (MCH), dilatada (MCD) y restrictiva (MCR).

Miocardiopatía dilatada: Síntomas y signos de insuficiencia cardiaca global. - Disnea paroxística nocturna - ortopnea - disnea de esfuerzos - edema EEII - tos nocturna.

Miocardiopatía dilatada: Dilatación de uno o ambos ventrículos con paredes de grosor normal o adelgazadas y deterioro de la función sistólica. Evoluciona a insuficiencia cardíaca progresiva, al deterioro de la función contráctil del ventrículo izquierdo, arritmias ventriculares y supraventriculares, trastornos del sistema de conducción eléctrica, tromboembolismo y muerte, sea súbita o relacionada con la insuficiencia cardíaca. Aunque puede afectar a todas las edades es más frecuente en la tercera y cuarta década de la vida. Tanto su incidencia como su gravedad son mayores en la raza negra y en hombres. Corresponde al resultado del daño miocárdico por mecanismos citotóxicos, metabólicos, inmunológicos, familiares e infecciosos, diferenciándola de aquella causada por enfermedad coronaria o valvular. Presenta fracción de eyección disminuida.

Al examen físico: - Ingurgitación yugular - choque de la punta desplazado crepitaciones pulmonares - 3er ruido - edema abdominal y EEII. Sospechar miocardiopatía de etiologías antes comentadas en pacientes jóvenes, que muchas veces no tienen factores de riesgo de daño cardiovascular. Miocardiopatía hipertrófica: Sospechar en pacientes jóvenes con episodios de: - Palpitaciones - disnea de esfuerzo - angina - presíncope y síncope

Miocardiopatía hipertrófica: 50% de los pacientes con trastorno genético demostrable, es la causa más común de muerte súbita en jóvenes, incluso deportistas. Se

Al examen físico:

14

- Pulso carotídeo de ascenso rápido (céler) - presencia de 4to ruido cardíaco - choque de la punta sostenido - soplo sistólico de eyección. Sin embargo, la presentación más habitual suele ser la muerte súbita. Miocardiopatía restrictiva: Clínica similar a miocardiopatía dilatada, llama la atención la mala respuesta a tratamiento, y sobre todo al manejo de volumen con diuréticos, esto debido a que al disminuir erróneamente la precarga del corazón perjudicaremos el gasto cardíaco. Exámenes complementarios: La radiografía de tórax puede ser de utilidad en caso de miocardiopatía dilatada, debido al crecimiento de la silueta cardíaca El ECG en cambio, mostrará su mayor utilidad en la miocardiopatía hipertrófica, esto debido a utilizar los criterios de Sokolow-Lyon para hipertrofia miocárdica (S en V1 + R en V5 o V6 >35mm y/o R en aVL >11mm). El diagnóstico definitivo se realiza con Ecocardiografía.

Tratamiento MCD: tratamiento sintomático de la IC; diuréticos, digitálicos, betabloqueadores e IECAs, más anticoagulación a permanencia. Antiarrítmicos deben evitarse. Otros tratamientos incluyen miomectomía septal o ablación septal con alcohol. MCH: evitar ejercicios extenuantes, deshidratación, digitálicos, nitritos, vasodilatadores, alcohol y diuréticos. Betabloqueadores útiles en alivio sintomático, al igual que Amiodarona. Se recomienda hacer un tamizaje con ecocardiograma doppler transtorácico en los familiares de primer grado de los pacientes conocidos y en atletas jóvenes, esto porque es la primera causa de muerte súbita en ellos. MCR: No hay tratamiento satisfactorio. Se recomienda anticoagular a permanencia (riesgo elevado de TEP). MAVD: Es posible utilizar β-bloqueo, desfibrilador implantable en prevención secundaria, pudiendo usarse en prevención primaria si también compromete VI o si es extensa. Se pueden utilizar antiarrítmicos e incluso ablación con radiofrecuencia.

Seguimiento Derivar a especialista para tratamiento y posible trasplante cardíaco. Considerar hospitalización de aquellos pacientes con síntomas refractarios a tratamiento médico, con síncope o muerte súbita recuperada, así como del que desarrolló fibrilación auricular o compromiso hemodinámico. La educación del paciente es importante para lograr la adherencia terapéutica. Debe tranquilizarse razonablemente a los de bajo riesgo, e indicar a todos que eviten los ejercicios extenuantes.

15

Paciente con soplo

Intensidad Descripción

Nivel de manejo del médico general: Diagnóstico Específico Tratamiento Inicial Seguimiento Derivar

Aspectos esenciales   

I/VI

Soplo Débil, se ausculta con dificultad, no todos pueden auscultarlo

II/VI

Soplo que es audible con técnica adecuada, por todos los examinadores

III/VI IV/VI

Soplo fácilmente audible Soplo Intenso, puede auscultarse un frémito

V/VI

Soplo muy intenso, puede palparse frémito

VI/VI

Soplo audible sin usar fonendoscopio

 

Soplos inocentes: Protomesosistólicos, nunca pansistólicos o diastólicos. Los soplos originados en cavidades derechas aumentan con la inspiración profunda. En la miocardiopatía hipertrófica obstructiva y el prolapso mitral, el soplo aumenta con maniobra de valsalva o bipedestación, al contrario del resto de los soplos.

Describir irradiaciónes. Describir maniobras que alteran la intensidad de los soplos.

El diagnóstico es Clínico. Se clasifican según el tiempo del ciclo cardiaco en que aparecen: a) Sistólicos (Después de R1 y antes de R2): 

Caso clínico tipo 

Paciente hombre de 72 años, refiere cuadro de disnea, ángor y síncope. Al examen físico: Pulso parvus y tardus, frémito sistólico carotídeo y sistólico en foco aórtico principal, R2 apagado y retardado, R4, soplo sistólico de eyección en área aórtica, soplo diastólico aspirativo.

De eyección: Con refuerzo mesosistólico, traducen obstrucción a la salida del ventrículo. Ej: Estenosis aórtica, estenosis pulmonar, coartación aórtica. También lo vemos en contexto de soplos hiperdinámicos: Fiebre, anemia, embarazo, etc De regurgitación: Borran R1 y a veces R2, indican flujo desde una cámara de alta presión a una de baja presión. Ej: Insuficiencia Mitral, Insuficiencia Tricuspídea, CIV.

b) Diastólicos (Después de R2 y antes de R1):

Definición



Corresponden a ruidos generados por un flujo sanguíneo turbulento a través del corazón y/o vasos sanguíneos.



Etiología-epidemiología-fisiopatología

c) Continuos: Por ductus arterioso persistente, intenso, con acentuación sistólica.

Pueden ser patológicos o fisiológicos. Los soplos patológicos surgen producto de alteraciones estructurales, como en valvulopatías o en las cardiopatías congénitas. Los soplos fisiológicos pueden corresponder a soplos inocentes (que acontecen en pacientes sanos con corazones normales, generalmente afectan a niños y adolescentes) o a soplos funcionales, como los que ocurren en situaciones de circulación hiperdinámica (fiebre, anemia, embarazo, hipertiroidismo, etc).

Existen patrones de irradiación característicos que pueden apoyar el diagnóstico; por ejemplo, la irradiación hacia las carótidas y huecos supraclaviculares en la estenosis aórtica o la irradiación hacia la axila en la insuficiencia mitral. Distintas maniobras alteran la intensidad de los soplos, prestando utilidad al diagnóstico. Deben distinguirse adecuadamente los soplos inocentes o funcionales: proto y/o mesosistólicos. Nunca pansistólicos ni diastólicos. No irradian en general. Baja intensidad la mayoría de las veces. Nunca con frémito.

Diagnóstico Cuando nos enfrentemos a un paciente con soplo, además de evaluar según sus características y contexto si se trata de un soplo fisiológico o patológico, se debe abordar del siguiente modo:    

De regurgitación: Protodiastólicos aspirativos, indican insuficiencias valvulares. Ej: Insuficiencia Aórtica, Insuficiencia Pulmonar. Por llene ventricular: Principalmente mesodiastólicos, con refuerzo presistólico si hay contracción de aurícula. Ej: Estenosis mitral, estenosis tricuspídea.

Maniobras que alteran la intensidad de los soplos: 

Describir momento del ciclo en que se escuchó el soplo (sístólico, diastólico, ambas). Describir característica (mesosistólico, pansistólico, rodada diastólica, presencia de clicks, etc). Describir foco (aórtico, pulmonar, mitral o tricuspídeo). Describir intensidad según la escala de Levin:



 

16

Maniobra de Valsalva y bipedestación (↓ precarga): Miocardiopatía hipertrófica obstructiva y prolapso valvular mitral. Mayoría de los soplos. Cuclillas y elevación pasiva de piernas (↑ precarga): Mayoría de los soplos. Miocardiopatía hipertrófica obstructiva y prolapso valvular mitral. Vasodilatadores (↓ postcarga): Estenosis aórtica, miocardiopatía hipertrófica obstructiva y prolapso valvular mitral. Insuficiencias aórtica y mitral. Ejercicio isométrico, vasoconstrictores (↑postcarga): Insuficiencias aórtica y mitral.



Estenosis aórtica, miocardiopatía hipertrófica obstructiva y prolapso valvular mitral. Inspiración profunda (signo de RiveroCarvallo): Soplos originados en cavidades cardíacas derechas.

Tratamiento Según etiología.

Seguimiento Según etiología y por especialista.

17

Diagnóstico

Síndrome metabólico

Es importante tener en consideración que frente a la presencia de un elemento (clínico o laboratorio) que sugiera síndrome metabólico, es necesario buscar dirigidamente los restantes.

Nivel de manejo del médico general: Diagnóstico: Específico. Tratamiento: Completo. Seguimiento: Completo.

Existen distintos criterios diagnósticos; los más utilizados son: - ATP III (diagnóstico con 3/5):

Aspectos esenciales    

 Patología de etiología multifactorial. Muy prevalente en nuestra población (25%). Criterios diagnósticos sin consenso único. Componentes esenciales: Obesidad abdominal, hipertrigliceridemia, HDL bajo, HTA y DM.

 

Caso clínico tipo



Paciente sexo masculino, hipertenso en tratamiento, sedentario y con malos hábitos alimentarios, con obesidad abdominal evidente, que en exámenes de control presenta TG 200 mg/dL, HDL 30 mg/dL, Glicemia 110 mg/dL.



Obesidad abdominal: Circunferencia de cintura > 102 cm en hombres y >88 cm en mujeres. TAG sérico ≥150 mg/dL o tratamiento por TAG elevados. HDL <40 mg/dL en hombres y <50 mg/dL en mujeres o tratamiento farmacológico por HDL bajo. Presión arterial ≥130/85 mmHg o tratamiento farmacológico antihipertensivo. Glicemia en ayunas ≥100 mg/dL o tratamiento farmacológico hipoglicemiante.

- International Diabetes Federation:

Hay variadas definiciones para síndrome metabólico, todas coinciden en una combinación de insulinoresistencia con o sin Diabetes, obesidad abdominal, HTA y dislipidemia, junto a un estado protrombótico y proinflamatorio común en individuos obesos y que incrementa los riesgos mayores de enfermedad cardiovascular. El eje central sería insulinoresistencia y obesidad abdominal.

Debe estar presente la obesidad abdominal según raza / etnia. Más dos de los siguientes criterios:  TAG >150 mg/dL o en tratamiento  HDL <40 mg/dL en hombres o <50 mg/dL en mujeres, o en tratamiento.  PAS >130mmHg, PAD >85mmHg, o tratamiento antihipertensivo.  Glicemia en ayuna >100 mg/dL o diagnóstico previo de DM 2. Se recomienda evaluación con PTGO, pero no es requerido para el diagnóstico.

Etiología-epidemiología-fisiopatología

Tratamiento

Definición

1. Detener tabaquismo, y consumo alcohol moderado (esto último sólo si TAG normales, en caso de estar elevados debe suprimirse el alcohol). 2. Disminución de peso mediante dietoterapia (baja ingesta de grasas saturadas, grasas trans, y colesterol, reducción de azúcares simples, y aumento de frutas, vegetales y granos enteros) y actividad física (ejercicio aeróbico moderado a intenso al menos 30 minutos al día al menos 150 min a la semana, e idealmente, más de una hora al día.) 3. Modificaciones en estilo de vida: Elemento central. 4. Farmacoterapia: solo si no hay respuesta con modificación de estilo de vida. Fármacos aprobados: insulinosensibilizadores, hipolipemiantes, antihipertensivos, orlistat (este último uso restringido por efectos secundarios). 5. Cirugía: frente al fracaso del tratamiento médico o en caso que de asociarse a obesidad mórbida con IMC > 40 u obesidad grado II con IMC > 35 y otra comorbilidad como HTA, DM o dislipidemia.

Se estima que un 25% de la población adulta padece síndrome metabólico; en Chile 22,5% de la población adulta, sin diferencias por sexo. La prevalencia aumenta con la edad. La etiología es compleja, intervienen tanto factores genéticos como ambientales, que influyen sobre el tejido adiposo y la inmunidad innata. El tejido adiposo abdominal actúa como un órgano endocrino que libera el exceso de ácidos grasos libres (AGL), angiotensina II (ATII), y adipokinas. El aumento de AGL plasmático inhibe la captación de glucosa por el tejido muscular. El exceso de AGL y ATII produce un daño en el páncreas, el cual produce más insulina, pero insuficiente para contrarrestar la hiperglicemia. La ATII aumenta la presión arterial por vasoconstricción. Adipokinas provocan reacciones inflamatorias que aumentan la insulinorresistencia y la HTA. La hiperglicemia y el aumento de circulación AGL elevan la síntesis hepática de TAG (triglicéridos), aumentando el transporte de TAG y disminuyendo el de HDL por lipoproteínas. Existen condiciones asociadas al SM. Destacan el hígado graso no alcohólico, el síndrome de ovario poliquístico, el hipogonadismo, la apnea obstructiva del sueño, la lipodistrofia y la enfermedad renal crónica.

Seguimiento La finalidad del tratamiento consiste en disminuir los riesgos de enfermedad cardiovascular sobre todo la enfermedad coronaría cardiaca. Requiere control periódico de cada uno de los componentes del Sd. Metabólico y otros factores de riesgo cardiovascular. 18

Ingurgitación yugular, que aumenta con la inspiración (signo de Kussmaul), pulso paradójico (disminución de la PAS de más 10 mmHg durante la inspiración), hipotensión, taquicardia, oliguria, matidez precordial y ruidos cardíacos apagados. Importante recalcar que la severidad de los síntomas dependerá en gran parte del tiempo de evolución del taponamiento, siendo muy sintomática en pacientes agudos (Trauma, post cirugía, ruptura aortica o como consecuencia de algún procedimiento invasivo).

Taponamiento pericárdico Nivel de manejo del médico general: Diagnóstico Específico Tratamiento Inicial Seguimiento Completo

Aspectos esenciales   

El compromiso hemodinámico dependerá de la severidad del taponamiento y de la reserva fisiológica del paciente, pudiendo presentar disnea (por bajo débito pulmonar), fatigabilidad (por bajo débito muscular) e incluso shock (piel fría y palidez, llene capilar lento, oliguria y compromiso de conciencia).

Son características clásicas del taponamiento la ingurgitación yugular con signo de Kussmaul, el pulso paradójico, la hipotensión y taquicardia. Tríada de Beck: Ingurgitación yugular, Hipotensión y Ruidos cardiacos apagados. La pericardiocentesis de urgencia está indicada en el taponamiento cardíaco con compromiso hemodinámico.

Otros métodos de estudio con menor sensibilidad: Radiografía de tórax, que mostrará cardiomegalia de bordes cardiofrénicos agudos y pedículos vasculares estrechos (imagen de cantimplora). Importante que para ser visible a nivel radiográfico la cuantía del derrame debe superar los 200 ml.

Caso clínico tipo Paciente politraumatizado por accidente automovilístico. Se encuentra agitado, pálido, hipotenso, con frialdad y discreta sudoración fría. Presenta las yugulares ingurgitadas y se decide realizar ecocardiograma por sospecha de taponamiento.

ECG, que puede mostrar disminución difusa del voltaje, o en algunos casos, alternancia eléctrica, con complejos QRS de pequeño voltaje intercalados con QRS normales (muy específico, pero poco sensible).

Definición

Tratamiento

Síndrome producido por la compresión extrínseca de las cámaras cardíacas comprometiendo su llenado, secundario a un aumento excesivo en la presión intrapericárdica.

La pericardiocentesis terapéutica, que debe hacerse permitirá un rápido alivio de los síntomas y permitirá el posterior diagnóstico etiológico del derrame pericárdico mediante el estudio del líquido obtenido en casos en los que la etiología no este clara.

Etiología-epidemiología-fisiopatología

Su realización urgente no está indicada en derrames pericárdicos severos sin taponamiento (es decir, sólo hallazgo ecocardiográfico). Los diuréticos para disminuir la presión venosa están contraindicados, dado que pueden contribuir al shock al disminuir el volumen circulante.

La naturaleza fibroelástica del pericardio le permite adaptarse a los cambios en el volumen cardíaco y a la presencia de derrame pericárdico. No obstante, un derrame de gran magnitud y/o rápida instalación no darán tiempo a la adaptación, aumentando súbitamente la presión intrapericárdica y provocando el taponamiento.

Seguimiento

De la misma manera, patologías que cursen con una disminución de la elasticidad del pericardio (por ejemplo, pericarditis constrictiva) determinarán una mayor probabilidad de sufrir taponamiento ante un derrame de menor cuantía. Cualquier pericarditis puede producir taponamiento, siendo más frecuente en la neoplásica, urémica y tuberculosa.

Por especialista.

El hemopericardio traumático es también una causa importante de taponamiento agudo. En general, las causas médicas son derrames de lento crecimiento y por tanto el paciente se presenta con gran volumen de derrame pericárdico; las causas quirúrgicas son de rápida instalación y suelen tener menos cuantía de derrame (ejemplo: post esternotomía, post toracotomía, post traumática).

Diagnóstico El diagnóstico es clínico, confirmándose la presencia y cuantía del derrame pericárdico mediante ecocardiograma, si la condición del paciente lo permite. Las manifestaciones clínicas características son: 19

MÓDULO 1: Medicina Interna

Diabetes y Nutrición Desnutrición Se clasifica según etiología: Primaria: Insuficiente aporte (pobreza, ignorancia o iatrogenia). Es la más frecuente. Secundaria: Por enfermedades (ej. parasitosis, enf. Celíaca, déficit enzimáticos) Mixta: Ambas causas participan en su aparición.

Nivel de manejo del médico general: Diagnóstico Específico Tratamiento Inicial Seguimiento Completo

Aspectos Esenciales • • •

Clínica: Calórica-protéica: Existe un déficit de energía, por lo que se utilizan proteínas como energía. Es una enf crónica, en su grado más extremo llega a marasmo. Mayor frecuencia entre los 6 y 18 meses. A la anamnesis hay historia de LM insuficiente, reemplazo por fórmulas diluidas, infecciones a repetición. Clínica: baja de peso, pérdida de panículo adiposo, disminución de masa muscular, irritabilidad, somnolencia, retraso de crecimiento. Laboratorio: normal. Protéica: Hay una dieta sin proteínas, pobre en lípidos y calorías, compuesta mayormente de H. de C. En una enfermedad aguda, mayor frecuencia entro los 1-5 años (después del destete), se puede asociar a malaabsorsión intestinal, pcte crítico, ayuno prolongado. Clínica: edema, baja de peso, dism. masa muscular, anorexia, alt. psíquicas, lesiones cutáneas, hepatomegalia, trast. gastrointestinales, alt. inmunológicas y retraso de talla. Al laboratorio: baja albúmina, anemia, hipocalcemia, hipofosfemia, hipomagnesemia, hipokalemia, hipoprotrombinemia, déficit de zinc y hierro, carencias vitamínicas y folato. Si albúmina < 2.5 mg./dl, se produce sd. edematoso agudo, o “Kwashiorkor”.

La ingesta insuficiente de energía es la causa más frecuente. Compromiso de talla implica desnutrición crónica. Intervención nutricional inicial programando Kcal/día según peso ideal para la talla.

Caso Clínico Tipo Niño de 4 años, acude a SU por dolor cólico en epigastrio, flatulencias, distensión abdominal y náuseas. Ex físico: peso y talla baja para la edad, disminución de la consistencia de los cartílagos auriculares, piel seca, atrofia muscular. Dg: desnutrición, anemia y parasitosis por Giardia lamblia.

Definición Estado patológico inespecífico, sistémico y potencialmente reversible, originado en insuficiente aporte, transporte o utilización de nutrientes por las células del organismo, caracterizado por pérdida o falta de incremento del peso y/o la talla.

Mixta: déficit global de nutrientes Duración: Aguda: no compromete talla. Crónica: compromete talla, puede ser descompensada (P/T menor a - 2DS) o compensada (P/T N) Intensidad: según el parámetro antropométrico para la edad Leve (riesgo de desnutrición): entre -1 y -2 DS Moderada: entre -2 y -3 DS Grave: menor a 3 DS

Etiología-Epidemiología-Fisiopatología Generalmente producida por ingesta insuficiente y/o aumento de las pérdidas. La desnutrición leve (o riesgo de desnutrición) es la más frecuente.

1 Facultad de Medicina, Universidad de Chile

Diagnóstico Anamnesis: Existen factores de riesgo socioeconómicos, ambientales y familiares. Evaluar control del embarazo, lactancia materna, enf. intercurrentes, control del crecimiento y desarrollo. Luego se realizará la evaluación antropométrica: 1. Si es < 1 año se utiliza P/E bajo -2 DS, bajo 80% del ideal, o 1 año, P/T bajo -2 DS, bajo 80% del ideal o 6 años se utiliza el IMC
Tratamiento Objetivos: Aportar nutrientes para mejorar deficiencias. Recuperar funciones metabólicas. Proveer los depósitos corporales. Mantener un crecimiento normal. Riesgo de desnutrición y desnutrición moderada: Corregir errores dietéticos, cubrir necesidades energéticas y de nutrientes esenciales, aumentar densidad calórica (más cereales/aceite), evaluación periódica de curvas. Derivar en caso de desnutrición moderada. Desnutrición grave: Se debe hospitalizar para estabilizar.

Seguimiento

Hasta recuperación: normalización de peso/edad y peso/talla.

Autor / Editor Catalina Luvecce

Año 2016

2 Facultad de Medicina, Universidad de Chile

Diabetes Gestacional

HbA1c > 10 al momento de embarazarse presentaron 50% de Malformaciones congénitas, a diferencia de mujeres con HbA1c < 7 al momento de la concepción, donde la tasa es cercana a cero. Otra complicación a considerar es el aumento de la muerte fetal in útero.

Nivel de manejo del médico general: Diagnóstico Específico Tratamiento Inicial Seguimiento Completo

Riesgos de DPG: Fetales: Parto prematuro Hipoglicemia neo-natal Hiperbilirrubinemia Macrosomía Distocia de hombros Malformaciones congénitas: Cardiovasculares 36,8% Neurológicas 20,8% Urogenitales 13,6% Musculo-esqueléticas 12,8% Digestivas 8,8% Orofaciales 1,6% Otras 5,6% Maternas: Cesáreas Infección/dehiscencia herida operatoria Hemorragia post parto TVP Endometritis post parto

Aspectos Esenciales •

Diabetes Gestacional es la intolerancia a la glucosa que se inicia o es reconocida por primera vez durante el embarazo. Es independiente de su severidad y del requerimiento de insulina.

En Chile se hace screening a todas las embarazadas. • Diagnóstico con 2 Glicemias en Ayuna ≥ 105 mg/dl, o 1 PTGO ≥ 140 mg/dl. • Derivar a alto riesgo obstétrico.

Caso Clínico Tipo Paciente de 38 años, multípara de 2 (primer hijo con PN 4.300 grs), con embarazo de 33 semanas. Al examen presenta altura uterina de 34 cm. La prueba de tolerancia a la glucosa a las 27 semanas muestra glicemia basal de 89 mg/dl y de 145 mg/dl a las 2 horas.

Con respecto a la DG, al igual que la DM2, su incidencia está en constante ascenso, algunos estudios indican prevalencias de un 1 a 25% aproximadamente.

Definición

El sobrepeso es el factor de riesgo más importante (y predispone también a DM2), también lo es la Insulinorresistencia y haber presentado hiperglicemia en condiciones de stress (infecciones, Tratamiento con corticoesteroides), la edad avanzada va en relación a que las mujeres mayores tienden a tener un IMC mayor (por tanto, mayor tasa de sobrepeso). Los riesgos de la Diabetes Gestacional son similares a los de DPG, a excepción de no asociarse a malformaciones congénitas ni muerte fetal in útero.

Diabetes Pre-gestacional (DPG): Paciente con patología conocida DM 1 o 2 que se embaraza, o que cumple con los criterios clásicos establecidos según la OMS durante el primer trimestre del embarazo. Diabetes Gestacional: Patología que inicia en el 2do trimestre de embarazo en adelante, caracterizado como cualquier grado de intolerancia a la glucosa manifestada durante este período.

Etiopatogenia Epidemiología

Para comprender este apartado hay ciertos conceptos que debemos tener claros. La insulina materna NO atraviesa la barrera hemato-placentaria, por lo tanto el feto debe regular su glicemia a base de su generación propia de insulina. Los carbohidratos atraviesan la barrera hemato-placentaria por difusión facilitada. Por lo tanto niveles elevados de glicemia materna, conllevarán niveles elevados de glicemia fetal. La insulina es la principal hormona ANABÓLICA del feto. Además hay que saber que los aminoácidos atraviesan esta barrera por transporte activo, y los ácidos grasos por difusión simple.

Según la encuesta nacional de salud del año 2003, en mujeres embarazadas entre 25-44 años la prevalencia de diabetes del embarazo es de 1,2% en Chile, con 68,8% correspondiendo a DG. A nivel mundial, más del 85% actual de Diabetes del embarazo son del tipo Gestacional. La diabetes mellitus tipo 2 es la Diabetes mas frecuente pregestacional. Existe una asociación de HTA y pre-eclampsia con mujeres DM1 y HTA crónica en DM de tipo 2. La DPG se asocia con distintas patologías, donde el grado de HbA1c es fundamental para evaluar riesgo, mujeres con

En el primer trimestre del embarazo, los niveles elevados de

3 Facultad de Medicina, Universidad de Chile

hormonas esteroidales (estrógenos y progesterona) conllevan una hiperplasia de islotes pancreáticos, por lo tanto la glicemia basal de la mujer tiende a disminuir (un 20% aprox) de la que tenía en el período pregestacional.

2 horas post-comidas: <120 mg/dL Noche: 60-99 mg/dL

En el segundo trimestre en adelante, y debido al aumento de los requerimientos nutricionales del feto, la secreción de hormonas placentarias (Lactógeno placentario, cortisol, prolactina) llevan a una insulinorresistencia global que aumenta los niveles basales de glicemia, provocando que mujeres predispuestas (obesas, resistentes a la insulina,etc ) no tengan los mecanismo homeostáticos necesarios para compensar la hiperglicemia, y desarrollar así DG.

Pilares de Tratamiento: Nutricional: Prevenir aumento exagerado de peso y asegurar nutrientes adecuados para la gestación. Consumo calórico según pirámide alimenticia de OMS. En el primer trimestre, el óptimo es calcular 1600- 1800 kCal Diarias. Incrementando en 400- 500 kcal desde el segundo trimestre. H de C: 210 gr/día , proteínas: 1,1 gr/kg/día Incentivar LME. Realizar ejercicio de modo regular, evitando actividad extenuante y con alto impacto. Farmacológico: Siempre complementario a la dieta Se realiza con insulina: Rápida, Lispro y Aspártica. Acción prolongada: NPH

HbA1c: <6,0%

Diagnóstico El diagnóstico de Diabétes Pregestacional se hace en el primer semestre de embarazo, utilizando los criterios clásicos de DM. Glicemia de ayunas alterada en 2 ocasiones > 125 mg/dL Paciente sintomático (polidipsia, poliuria, hiperfagia) con glicemia > 200mg/dL.

Tener en cuenta que: Los requerimientos durante el primer trimestre son similares a los pregestacionales. En las semanas 7 a 15 son los requerimientos mas bajos (ojo con hipoglicemias) Las semanas 28 a 32 son las de mayor demanda de insulina Requerimientos se estabilizan e incluso disminuyen desde la semana 35.

PTGO alterada: > 100 en ayunas y/o > 140 post-carga La detección de Diabetes del embarazo es una práctica universal en mujeres embarazadas. 1er control pre-natal: solicito glicemia de ayunas. Si 100-125 mg/dL, sospecha de DG Si > 125, repetir o tomar PTGO, si alterado: Diabetes Pre Gestacional

DM1: Continuar con esquema antes utilizado, en general NPH SC. 0,4 – 0,5UI/Kg, repartidas 2/3 en la mañana y 1/3 en la noche. En general esto corresponde al 50% de la insulina utilizada al día, siendo la insulina basal, agregando a esto bolos prandiales de insulina rápida, o ultrarápida (Lispro o Aspártica). Realizar ajustes y correciones según el automonitoreo con HGT.

Semanas 24-28: solicito PTGO. Si > 99 en ayunas y/o > o = a 140mg/dL post-carga (2hrs) : Diabetes Gestacional. Semana 30-33 a mujeres con factores de riesgo para DG: Solicito PTGO. Si > 99 en ayunas y/o > o = a 140mg/dL post-carga (2hrs) : Diabetes Gestacional. Factores de riesgo para DG: Polihidroamnios Macrosomía fetal Aumento de peso > 2DS

Recordar que estas pacientes tienden mas a la hipoglicemia nocturna (3:00 am) e hiperglicemia reactiva (7:00 am) Aumentos post prandiales de glicemia son más acentuados DM2: Ideal es iniciar tratamiento con insulina antes de la concepción, incluso ante buenos controles metabólicos con Hipoglicemiantes orales, cambiando esto por insulina. Dosis utilizadas generalmente mayores que en DM1, pero pacientes con mayor estabilidad glicémica, y con necesidad de esquemas menos complejos. Ante pacientes con hiperglicemia sostenida o Hb1Ac > 9, el tratamiento se inicia Hospitalizado: Iniciar con 0,4- 0,6 UI/ kg/día

Tratamiento OBJETIVO: Lograr un control metabólico adecuado, duranto todo el embarazo. Se sugiere realizar un examen de Hemoglobina glicosilada cada trimestre (DPG). En el caso de la glicemia, también se debe realizar mediciones seriadas, basta con HGT para trabajar con metas de glicemia. Antes del desayuno: 60- 90 mg/dL Antes de comidas: 60- 105 mg/dL 1 hora post-comidas: <140 mg/dL

Si la alteración metabólica es intermitente, y Hb1Ac <9, siempre y cuando sea posible garantizar controles frecuentes, se inicará tratamiento de modo ambulatorio con 1 o 2 dosis dependiendo de criterio clínico. Iniciar con 0,2 UI/kg/día. Los resultados en la glicemia de ayunas obligan a aumentar tratamiento de insulina nocturno, y resultados alterados de glicemia antes de almuerzo y cena, requieren aumentos en

4 Facultad de Medicina, Universidad de Chile

insulina matinal. Estos cambios deben corresponder en general al 10% de la dosis antes administrada. Una vez alcanzadas las metas pre prandiales, debemos evaluar las glicemias post prandiales Valores post prandiales 140- 180 mg/dL : iniciar Insulina cristalina 2U; valores >180, iniciar 4U, siempre media hora antes de las comidas. Considerar siempre consultar con especialista. En mujeres con DG: Iniciar tratamiento con insulina si en 2 semanas no se logra un óptimo control metabólico con manejo conservador. El inicio del tratamiento se realiza idealmente ambulatorio, según criterio clínico podría realizarse una hospitalización breve de 24 hrs, con el fin de orientar y educar bien a la paciente. Dosis inicial generalmente baja: 0,1-0,2 UI/kg/día para insulina NPH Algunas recomendaciones: Si sólo alterada glicemia de ayunas (>100mg/dL) inicio NPH nocturna 0,1 UI/kg Alteración post desayuno: insulina rápida pre desayuno (2U si glicemia 140-179mg/dl y 4 U si >180mg/dL) Pre y post desayuno en rangos normales, alteración post almuerzo, onces y pre cena: NPH matinal (0,15/kg/día antes del desayuno) Si la alteración es pre desayuno y post comidas :0,2U/ kg/ día: 2/3 antes desayuno y 1/3 en la noche) Solicitar ayuda a un especialista.

Seguimiento En nivel secundario según la normativa Minsal.

Autor / Editor Matilde Pedrero

Revisor de Formato Año Macarena Moya 2016

5 Facultad de Medicina, Universidad de Chile

Diabetes Pregestacional

experimenta agravación significativa durante la gestación; la cardiopatía coronaria en diabéticas embarazadas, se asocia a una mortalidad materna muy elevada, cercana al 70%; y la hipertensión puede generar preeclampsia y eclampsia.

Nivel de manejo del médico general: Diagnóstico Específico Tratamiento Inicial Seguimiento Completo

Diagnóstico Paciente embarazada con diagnóstico de diabetes previo a gestación, ya sea DM tipo I y II.

Aspectos Esenciales • • •

Es importante el consejo preconcepcional, idealmente tener un embarazo programado. El control metabólico tiene un rol fundamental en complicaciones para el feto y la madre. Tratamiento: Insulina, dieta y control médico; con objetivo de HbA1C < 7%, glicemia postprandial a las 2 horas < 120 mg/dl.

Tratamiento Es fundamental el manejo de la paciente por un equipo multidisciplinario y seguimiento con controles rigurosos. Los Pilares del tratamiento son: • Idealmente la programación del embarazo. • Los controles médicos periódicos. • La dieta y la insulinoterapia. En pacientes diabéticas que quieran embarazarse es importante que lleven un estilo de vida saludable junto con un buen control de su enfermedad. Si tienen sobrepeso u obesidad es necesario que bajen de peso. La programación del embarazo servirá para pesquisar a tiempo las complicaciones de la diabetes, llevar una HbA1c menor a 7% al menos 2 a 3 meses previos al embarazo, prevenir malformaciones congénitas con el uso de ácido fólico (4 mg/día), suspender aquellos medicamentos contraindicados en embarazo (estatinas, IECA, ARAII) y continuar con el uso de hipoglicemiantes que se hayan usado previo al embarazo.

Caso Clínico Tipo Paciente 35 años, obesa, nulípara, con diagnóstico de DM tipo 2 e HTA desde los 33 años, en tratamiento con Metformina y Enalapril. Consulta por amenorrea de 8 semanas. Se diagnostica embarazo con beta HCG plasmática.

Definición Se considera diabetes pregestacional aquella que se diagnostica previa al embarazo. Incluye la DM I y II.

En el control médico del embarazo es importante solicitar exámenes para estudiar la repercusión de los parénquimas, entre ellos están la HbA1c, orina completa, urocultivo, proteinuria de 24 horas, BUN, creatinina y ECG. Este se realizará en el nivel secundario de salud.

Etiología-Epidemiología-Fisiopatología En Chile la prevalencia de Diabetes pregestacional es de un 5-10% en las mujeres embarazadas, en donde la mayoría de ellas corresponden a pacientes con antecedente de DM II. De hecho, se ha visto un aumento en la asociación entre la DM II y el embarazo, principalmente debido al retraso del embarazo en la población femenina.

La insulina es el tratamiento de elección para estas pacientes. Se usa en 2 dosis diarias, dejando ⅔ en la mañana y ⅓ nocturna, la dosis puede variar según los valores de glicemia en ayuna y postprandiales. También se pueden tratar con hipoglicemiantes orales (metformina y glibenclamida).

La existencia de esta patología aumenta las probabilidades de efectos adversos durante el embarazo, aumentando la morbimortalidad tanto de la madre como del feto, aún más si los embarazos no se planifican. Se ha evidenciado que la no adecuación al tratamiento en etapas tempranas de la gestación (embriogénesis), aumenta la incidencia de abortos espontáneos y anomalías congénitas. En etapas más tardías del embarazo, aumenta el riesgo de macrosomía y sus secuelas. En el caso de las mujeres con DM I, la presencia de complicaciones vasculares se asocia con un mayor riesgo de parto prematuro y de mortalidad perinatal.

Seguimiento

Durante tres meses previos al embarazo, en la gestación misma y en el post-parto para asegurar un buen control glicémico, en caso de que se desee un nuevo embarazo. Los objetivos metabólicos durante el control de esta patología son: una glicemia en ayunas < 90 mg/dl, glicemia 2 horas postprandial < 120 mg/dl, HbA1c < 7%, cetonuria y glucosuria negativas y una ganancia de peso límite de un 20% del peso inicial. El manejo de la diabetes pregestacional incluye: Detección, monitoreo e intervención de las complicaciones médicas (retinopatía, nefropatía, hipertensión, enfermedades cardiovasculares, cetoacidosis). Monitoreo e intervención de las complicaciones fetales y

Si la paciente presenta retinopatía previa al embarazo, ésta tiende a agravarse, por lo que se recomienda un control oftalmológico cada dos meses; la nefropatía, en general, no

6 Facultad de Medicina, Universidad de Chile

obstétricas (anomalías congénitas, macrosomía, preeclampsia). Control con diabetólogo cada 1 a 3 semanas aproximadamente para el ajuste de la terapia insulínica. Exámenes de seguimiento: Rutina: los mismo de cualquier embarazo. Cobra más importancia la orina completa y el urocultivo por la mayor prevalencia de bacteriuria asintomática. HbA1c cada 2 meses. Ultrasonido según indicaciones habituales, es útil efectuar un estudio Doppler de la arteria uterina en la semana 16, y eventualmente en el tercer trimestre. Fondo de ojo en cada trimestre, se realiza en el primero sólo si no tiene en los últimos 6 meses. Microalbuminuria en cada trimestre. Otros aspectos importantes: • En caso de amenaza de parto prematuro: manejo con nifedipino, indometacina o tractocile para tocolisis, en lugar de un agonista receptor beta-adrenérgico. • Si se administra betametasona prenatal para maduración pulmonar fetal, monitorizar glicemia frecuentemente. Usar insulina según requerimientos para mantener glicemia dentro de rangos normales. • Si el peso estimado del feto es más de 4500 gramos, se sugiere un parto por cesárea. • Se sugiere la inducción del parto a las 39 hasta las 40 semanas de gestación en mujeres con cuellos uterinos favorables y los fetos a menos de 4500 g. En presencia de factores de riesgo como el control deficiente de la glucosa, el empeoramiento de la nefropatía o retinopatía, la preeclampsia, o restricción del crecimiento intrauterino se recomienda adelantar el parto. La espera del inicio espontáneo del trabajo de parto es razonable si hay un buen control glicémico y no hay complicaciones maternas o del feto, pero no más allá de 40 semanas de gestación.

Autor / Editor Nicole Fritzche

Año 2016

7 Facultad de Medicina, Universidad de Chile

defectos en la secreción o acción de la insulina. La hiperglicemia crónica de la diabetes se asocia a daño en el largo plazo, disfunción y falla en diferentes órganos, particularmente ojos, riñones, nervios, corazón y vasos sanguíneos. En el desarrollo de la enfermedad participan diferentes procesos patológicos, que en el caso de la diabetes tipo 1, corresponde a una deficiencia absoluta de la secreción de insulina.

Diabetes Mellitus Tipo 1 Nivel de manejo del médico general: Diagnóstico Específico Tratamiento Inicial Seguimiento Derivar

Epidemiología En Chile no se tiene información sobre la incidencia de diabetes tipo 1 a nivel nacional, sin embargo, si se extrapolan los resultados del estudio en población menor de 15 años de la Región Metropolitana realizado entre los años 2000 y 2004 al resto del país, sería un país con una incidencia intermedia, 6,58 por 100.000 habitantes/año. En el período analizado se observó un incremento significativo de la tasa de incidencia de 5,44 a 8,33 por 100.000 habitantes/año, respectivamente. No se observan diferencias significatovas por según sexo. El número total de casos diagnósticados fue significativamente mayor durante el período otoño-invierno y en comunas urbanas de mayor nivel económico (19,9/100.000 habitantes) y con una muy baja incidencia en población indígena (1,05/100.000 habitantes). La prevalencia de diabetes tipo 1 se duplicó en las últimas décadas en el mundo, y se proyecta aumentará aún más, lo que permite concluir que la diabetes tipo 1 será una carga para más pacientes y para la mayoría de los sistemas de atención de salud. En Chile no existen estudios de prevalencia de DM tipo 1. El registro de la Superintendencia de Salud informa que a marzo de 2011 había un total de 10.760 casos de diabetes tipo 1 acumulados, 5.929 en FONASA y 4.840 en las ISAPRES. Lo más probable es que este total de diabéticos tipo 1 corresponda a una sobreestimación del número de personas con esta condición, ya que es utilizado para efectuar un seguimiento del cumplimiento del programa GES.

Aspectos Esenciales • • • • •

La diabetes tipo 1 tiene una deficiencia absoluta en la secreción de insulina (es insulinodependiente por definición). El método de elección para pesquisar y diagnosticar la diabetes es la glicemia plasmática, en sangre venosa y determinada en laboratorio. La HbA1c es fundamental en el seguimiento, pero no en el diagnóstico. La prueba de tolerancia a la glucosa oral (PTGO) no está indicada para el diagnóstico de la diabetes mellitus tipo 1. Los marcadores inmunológicos y el péptido C no son necesarios para el diagnóstico a menos que haya duda diagnóstica.

Caso Clínico Tipo Paciente consulta en servicio de urgencia, por cuadro de dolor abdominal, náuseas y vómitos. Refiere baja de peso en las últimas 3 semanas, a pesar de que ha aumentado el consumo de comida. Refiere que ha tenido mucha sed, por lo que bebe abundantes cantidades de líquidos. Se toma una glicemia capilar que resulta “High”, y una cetonas capilares positivas.

Fisiopatología Está caracterizada por la destrucción de las células ß, mediada por mecanismos inmunológicos, este mecanismo a veces se ve, y a veces no, pero se sospecha. Por tanto existe una división en DM tipo Ia, en la cual el mecanismo inmunológico se puede demostrar en el paciente por exámenes; y DM tipo Ib o idiopática, en la que se asume que existe un mecanismo de daño de tipo inmunológico, ya que tiene el mismo comportamiento que una DM tipo I, pero no se puede demostrar. Se describen 3 etapas en el desarrollo de DM tipo I: • Una etapa inicial, donde tenemos la presencia de marcadores génicos. Hay susceptibilidad del individuo que lo predispone a hacer una DM tipo I, sin embargo la presencia del gen NO determina la aparición de la enfermedad, sólo aumenta su probabilidad a lo que se suma algo ambiental que no se conoce bien, existe el antecedente por ejemplo de exposición a algunos virus, toxinas, etc. que son factores que en un individuo con genes susceptibles, determinarían la aparición de la DM tipo I. • Luego se da paso a la segunda etapa inmunológica o de marcadores inmunológicos (ICA: Antocuerpos Anti-islotes), en la cual uno puede demostrar un montaje de una respuesta inmune contra las células ß del páncreas, se produce destrucción de estas células por mecanismo

Definición La diabetes mellitus tipo 1 es una enfermedad metabólica, que se caracteriza por hiperglicemia crónica, que resulta de la destrucción autoinmune de las células Beta del páncreas, determinando un déficit absoluto de insulina y dependencia vital de la insulina exógena.

Etiología-Epidemiología-Fisiopatología La existencia de esta patología aumenta las probabilidades de efectos adversos durante el embarazo, aumentando la morbimortalidalicaciones vasculares se asocia con un mayor riesgo de parto prematuro y de mortalidad perinatal. Etiología La diabetes incluye a un grupo de enfermedades metabólicas que se caracterizan por hiperglicemia como resultado de

8 Facultad de Medicina, Universidad de Chile



de daño tipo IV, dado principalmente por la inmunidad celular, pese a que existen anticuerpos éstos no tienen un rol patogénico. El daño es por destrucción linfocítica citotóxica. Y posteriormente, una tercera etapa, donde se monta una respuesta que va deteriorando el número celular,

apareciendo marcadores metabólicos. Cuando ya hay un 80% de destrucción celular ocurre la insulinopenia y la hiperglucagonemia (por falta de inhibición paracrina). En una primera etapa vamos a tener un paciente no hiperglicemico, posteriormente hará intoleracia a la glucosa, hasta llegar a volverse un hiperglicémico crónico.

Diagnóstico * La PTGO no está indicada en el diagnóstico de DM tipo 1 y la HbA1c no es un examen que esté validado para el diagnóstico de DM tipo 1.

La diabetes tipo 1 se puede presentar a cualquier edad, con síntomas clásicos como polidipsia, poliuria, polifagia, nicturia y pérdida de peso de rápida evolución, o con cetoacidosis diabética. Los síntomas clásicos de DM1 son similares en niños y adultos, pero la presentación al debut en niños menores suele ser más grave, el período prodrómido más breve y la cetoacidosis y el compromiso de conciencia más intenso. A medida que aumenta la edad, la sintomatología es más variable en intensidad desde muy aguda hasta síntomas escasos. En adultos jóvenes la DM1 suele presentarse en forma atípica con escasa sintomatología clínica y lenta evolución a la insulinodependencia (Diabetes Latente Autoinmune del Adulto, LADA). En este caso los pacientes suelen tener normopeso, ausencia de signos clínicos de insulinorresistencia y sin antecedentes familiares de diabetes. Los elementos clínicos que orientan al diagnóstico son edad menor de 20 años, eutrofia, ausencia de signos de insulino-resitencia y tendencia a la rápida descompensación. Confirmación diagnóstica: Los criterios diagnósticos de DM1 se basan en las cifras de glicemia: • Glicemia al azar mayor de 200 mg/dl con síntomas asociados. • Dos glicemias de ayuno mayores de 126 mg/dl. • Prueba de tolerancia a la glucosa mayor de 200 mg/dl a las 2 horas.

Otros exámenes de laboratorio: 1. Marcadores inmunológicos: Si bien existen marcadores de destrucción autoinmune de la célula beta (anticuerpos antiislotes, antidescarboxilasa del ácido glutámico, anti GAD, anticuerpo anti-insulina) que están presentes al diagnóstico y en etapas muy tempranas de la enfermedad, si la clínica es clara, no es necesario solicitarlos. Es recomendable medirlos en casos de duda diagnóstica con diabetes tipo 2, diabetes monogénicas (ej. MODY), diabetes secundaria. 2. Péptido C: esta molécula se produce en una relación 1:1 con la insulina y se utiliza en clínica para cuantificar la secreción de la célula ß ya que este no se metaboliza en el hígado (lo hace principalmente en el riñón) y tiene una vida media mucho mas larga a diferencia de la insulina que es metabolizada en más del 50% en el hígado a penas es secretada a la sangre por el páncreas por lo que sus niveles en la sangre no se corresponden con lo secretado por las células beta. Este examen se solicita sólo frente a la duda diagnóstica. Su valor puede ser normal en el período de luna de miel y está ausente en el período de que la enfermedad está establecida (diabetes

9 Facultad de Medicina, Universidad de Chile

total). Diagnóstico diferencial 1. Diabetes monogénica: • Diabetes neonatal o diagnosticada dentro de los primeros 6 meses de vida. • Diabetes familiar con un padre y abuelo afectados. • Hiperglicemia en ayunas leve (100-153 mg/dL) especialmente si es joven o con antecedentes familiares. • Diabetes asociada con afeccciones extrapancreáticas. 2. Diabetes tipo 2: • Obesos con antecedentes familiares de DM tipo 2 y signos clínicos de insulinorresistencia (acantosis nigricans). • Pacientes que debutan con cetoacidosis y evolucionan en forma muy estable en los meses siguientes, con evidencia de producción de insulina (Diabetes tipo 2 propensa a la cetosis). 3. Síndromes de insulino-resistencia: • Retraso de crecimiento.

• •

Signos de insulino-resistencia en ausencia de obesidad. Lipodistrofias.

Tratamiento

(para más detalles ver el Guía Clínica AUGE, Diabetes Mellitus tipo 1, MINSAL 2013). Objetivo: lograr un control metabólico óptimo para prevenir o retrasar las complicaciones crónicas y mejorar la expectativa de vida. Los pilares del tratamiento incluyen la terapia insulínica, estilo de vida y alimentación saludable (conteo de hidratos de carbono), autocontrol, educación del paciente diabético y su familia y el apoyo psicosocial. 1. Insulinoterapia: • Tipos de insulina: las características de las insulinas disponibles en Chile y sus tiempos de acción se muestran en las siguientes tablas:

Tabla Insulinas registradas en el Instituto de Salud Pública de Chile, según tipo y laboratorio que la comercializa

Tabla Tipos de Insulina y Tiempos de Acción

*Modificado de Eyzaguirre F, Codner E, Rev. Méd. Chile 2006, 134: 239 - 250

10 Facultad de Medicina, Universidad de Chile



Aspectos para el buen uso de la insulina:

*Guía clínica AUGE, Diabetes Mellitus tipo 1. Serie Guías clínicas MINSAL, 2013.





Esquemas a utilizar: deben ser individualizados para cada paciente. Existen los siguientes: a) 1-2 inyecciones diarias de insulina de acción intermedia, habitualmente mezcladas con insulina de acción rápida o análogo ultrarrápido. b) Esquemas intensivos: -Múltiples inyecciones de insulina rápida o análogo ultrarrápido pre-prandiales, junto a insulina de acción intermedia o de acción prolongada. -Infusión subcutánea contínua con infusor (bomba) de insulina. Dosis a utilizar: la dosis depende de la edad, peso, estadio puberal, duración de la diabetes, estado de los sitios de inyección, aportes y distribución de la ingesta alimentaria, patrón de ejercicio y rutina diaria, del control metabólico y de la presencia de enfermedades intercurrentes. La dosis habitual total es de 0,7 a 1 U/Kg/día. En período de luna de miel el requerimiento de insulina baja a menos de 0,5 U/kg/día y durante la pubertad aumenta a 1-2 U/Kg/día. La dosis debe ajustarse a los requerimientos de cada paciente, de tal manera de lograr el mejor

control glicémico con el menor riesgo de hipoglicemia. 2. Control glicémico: se realiza principalmente mediante el autocontrol o automonitoreo de la glicemia por los pacientes y/o sus familiaresla mediante la técnica de autoevaluación capilar de manera de proveer en forma oportuna y confiable los niveles de glucosa para facilitar una adecuada desición en realación a la dieta, ejercicio, insulinoterapia u otras conductas en personas con diabetes (ej. manejar). Se utiliza la hemoglobina glicosilada (HbA1c) como indicador promedio de los valores de las glicemias en los últimos tres meses. La meta para los pacientes adultos con DM tipo 1 es tener valores de HbA1c bajo 7,2%. 3. Alimentación: las recomendaciones nutricionales para las personas con DM tipo 1 no difieren sustancialmente de las que se dan a la población no diabética, pero se requiere una coordinación entre la insulinoterapia y la alimentación; se trata básicamente de una alimentación equilibrada, en que las comidas están cubiertas por insulina prandial.

11 Facultad de Medicina, Universidad de Chile

4. Educación: su objetivo es favoreceer la autonomía de la persona en el cuidado de su diabetes, de manera que tanto la persona como la familia fortalezcan el proceso de toma de decisiones y permitan mantener un óptimo control metabólico. Requiere un equipo multidisciplinario (médico, enfermera, nutricionista, asistente social, psicólogo, etc.) para ser efectivo.

3. Evaluaciones clínicas anuales: • Fondo de ojo por oftalmólogo a contar del tercer año desde el diagnóstico. • Control anual por nutricionista especializada. 4. Exámenes discrecionales: • Anticuerpos antiendomisio y/o anticuerpos antitransglutaminasa en IgA (en pacientes con sospecha.

Seguimiento El seguimiento en el paciente diabético tiene como objetivo que el paciente tenga una buena calidad de vida, el mejor control glicémico, evitar episodios de hipoglicemia y prevenir el desarrollo de complicaciones crónicas. El equipo de salud debe realizar educación permanente, monitorear la adherencia al tratamiento, ajuste de dosis de insulina, autocontrol para definir intervenciones durante la evolución de la enfermedad. Es fundamental la detección precoz de enfermedades asociadas y complicaciones crónicas. • El adecuado control de la diabetes tipo 1 requiere necesariamente la activa participación del paciente o de su familia en su autoontrol: 1. Realizar automonitoreo con glicemias capilares en promedio 4-6 determinaciones al día: - Una glicemia antes de cada comida (4). - Una glicemia 2 horas post comidas alternando horarios (2). - En caso de descompensaciones, en días de enfermedad o ajustes de terapia se aumenta el número de controles glicémicos. 2. Determinación de cuerpos cetónicos (en orina o sangre), con niveles de glicemia persistentemente elevadas, mayores o iguales a 250 mg/dL. • En el control médico del paciente se debe efectuar: - Hemoglobina glicosilada cada 3 meses - Para detectar comorbilidades o complicaciones crónicas realizar con frecuenta anual: • Creatininemia. • Perfil lipídico. • T4-TSH-Anticuerpos antitiroídeos. • Razón albuminuria/creatininuria (microalbuminuria).

Autor / Editor Paula Saavedra

12 Facultad de Medicina, Universidad de Chile

Año 2016

Diabetes Mellitus Tipo 2

esta vez trae una glicemia de 135.

Definición Nivel de manejo del médico general: Diagnóstico Específico Tratamiento Inicial Seguimiento Completo

La diabetes tipo 2 se define por la condición de hiperglicemia crónica. La diabetes se asocia a una reducción en la expectativa de vida, aumento del riesgo de complicaciones y de eventos mórbidos relacionados con las complicaciones crónicas, disminución en la calidad de vida y aumento en los costos.

Aspectos Esenciales •







La diabetes tipo 2 es una enfermedad poco sintomática, por lo que su diagnóstico se efectúa en alrededor del 50% de los casos por exámenes de laboratorio solicitados por otra causa y no por sospecha clínica. El método de elección para pesquisar y diagnosticar la diabetes tipo 2 en adultos es la glicemia plasmática en ayunas (al menos 8 horas) en sangre venosa determinada en el laboratorio. La HbA1c es el examen de elección para el control metabólico de la diabetes, en nuestro país no se debe usar como método diagnóstico, porque el examen no está estandarizado. La glicemia capilar con cintas reactivas no es un examen para el diagnóstico de diabetes, sólo para su control.

Diagnóstico Diabetes: • Síntomas clásicos de diabetes (polidipsia, poliuria, polifagia y baja de peso) y una glicemia en cualquier momento del día ≥ 200 mg/dl. • Glicemia en ayunas (GA) ≥ 126 mg/dl. Debe confirmarse con una segunda GA ≥ 126 mg/dl, en un día diferente. (ayuno se define como un período sin ingesta calórica de por lo menos ocho horas). • Glicemia ≥ 200 mg/dl dos horas después de una carga de 75 g de glucosa durante una PTGO Prediabetes: GAA y/o IGO • Glicemia alterada en ayunas (GAA): GA ≥ 100 y < 126 en 2 días distintos. • Intolerancia a glucosa oral (IGO): GA ≥ 100 y glicemia a las 2 horas post carga entre 140-199.

Caso Clínico Tipo Paciente de 58 años, obeso, HTA, con una glicemia plasmática de 138 en control anterior, acude nuevamente a control,

13 Facultad de Medicina, Universidad de Chile

Algoritmo 1: Diagnóstico de diabetes, glicemia en ayunas alterada (GAA) e intolerancia a la glucosa oral (IGO). *Realizar un examen de laboratorio confirmatório en un día distinto en todos aquellos casos en que no hay síntomas clásicos de diabetes o una descompensación metabólica inequívoca.

Tratamiento El tratamiento tiene como objetivos: • corregir el trastorno metabólico (mantener glicemia lo más cercana al valor de normalidad) • prevenir o retrasar la aparición de complicaciones

• •

tratar los trastornos asociados a la diabetes (como HTA, dislipidemia). mejorar la calidad de vida

Etapa 3

Algoritmo 2: Conductas terapéuticas en personas con diabetes tipo 2 *Terapias menos validadas en sujetos asintomáticos y glicemias <200 mg/dl, se puede intentar tratamiento sólo con cambios en el estilo de vida como etapa 1 y reevaluar.

Los pilares del tratamiento son: 1. Cambios en el estilo de vida (CEV) son la primera línea de tratamiento ya que reducen el riesgo de diabetes en un 58%, y son transversales a todo el tratamiento.

• •

Dieta con abundantes verduras, frutas, cereales y leguminosas, pescados (dos veces a la semana)preferir lácteos descremados, aves sin piel y carnes magras. Evitar el consumo excesivo de grasas y aceites, especial-

14 Facultad de Medicina, Universidad de Chile

mente las grasas saturadas y sodio (evitar vísceras, yema de huevo y frituras, alimentos procesados y aquellos con alto contenido de sodio, reducir cantidad de sal que utiliza en la preparación de los alimentos y elimine el salero de la mesa. Mantenga una ingesta calórica que guarde relación con el gasto energético. En individuos con sobrepeso u obesidad, aumente la actividad física y reduzca el consumo de alimentos de alto contenido calórico. Realice actividad física de intensidad moderada la mayoría de los días de la semana, al menos durante treinta minutos cada vez (ej. caminata rápida). Consumir alcohol, beba con moderación, máximo dos tragos en cada ocasión

2. Tratamiento farmacológico: • Insulina • Hipoglicemiantes orales: un grogupo heterogéneo de drogas que a través de mecanismos pancreáticos y/o extrapancreáticos producen una disminución de los niveles de glicemia, luego de su administración por via oral. El hipoglicemiante ideal es que prevenga complicaciones, debe ser fisiológico, cuidar el páncreas y ser favorable para el organismo. Debe ser seguro, evitando hipoglicemia, eventos cardiovasculares mayores, pancreatitis, cáncer. El efecto del hipoglicemiante en disminuir la HbA1c, va a depender del nivel de HbA1c previo, y menor evolución de la diabetes (a más alta la HbA1c mayor es la baja, y si el diagnóstico es reciente, también va a responder mejor.) La siguiente tabla reúne los hipoglicemiantes.

La metformina es el único fármaco que se ha podido demostrar previene las complicaciones macrovasculares. Se debe partir con dosis pequeñas, ya que el principal efecto secundario es la mala tolerancia digestiva, que suele durar 2-4 semanas, y luego el paciente se va acostumbrando a este. No se puede usar en insuficiencia hepática, renal (VFG <45) ni en estados de hipoperfusión (ej. insuficiencia cardiaca congestiva). La metformina puede producir hipoglicemia en aquellos pacientes que se cuidan mucho de las comida

En Chile, se recomiendan algunas conductas para realizar este seguimiento de las complicaciones crónicas: Nefropatía diabética: • Al momento del diagnóstico, se debe evaluar la concentración de proteínas en la orina y la creatinina plasmática. La evaluación se debe hacer anualmente • Si no hay proteinuria en una muestra matinal de orina comprobar si hay albúmina en orina, usando la razón albúmina/creatinina (RAC) o una prueba semicuantitativa si no se dispone del examen para determinar RAC. • Si se comprueba microalbuminuria, RAC entre 30-300 mg/g, se requiere al menos una segunda prueba confirmatoria durante los siguientes 6 meses • En toda persona con diabetes tipo 2 con nefropatía incipiente o nefropatía diabética clínica, iniciar tratamiento con un IECA o ARA II, independientemente de su nivel de presión arterial.



• •

Seguimiento Para realizar el seguimiento de la diabetes, se debe tener en cuenta cuáles son las complicaciones agudas y crónicas de ésta, como se observa en el siguiente mapa conceptual.

15 Facultad de Medicina, Universidad de Chile



Aquellos sujetos con proteinuria, o una VFGe <30 ml/ min/1.73m2, deben ser referidos a evaluación por especialista. Retinopatía diabética: • Al momento de diagnosticar una diabetes mellitus tipo 2, se debe realizar tamizaje de retinopatía diabética (por medio de un fondo de ojo, realizado por un oftalmólogo) • El tamizaje se debe hacer de forma periódica según la clínica. Si el paciente no tiene alteraciones, se repite cada 2 años, pero si hay alteraciones u otros factores de riesgo, se hace en intervalos más cortos

Autor / Editor Bruno Peña

Neuropatía diabética: • Toda persona con diabetes debe ser examinada al menos una vez al año para pesquisar potenciales problemas en sus pies. • El examen anual de los pies debe concluir con una clasificación del riesgo de ulceración, si el riesgo es alto, se debe derivar a un equipo multidisciplinario de cuidados del pie o a un médico especialista en pie diabético • Es importante educar constantemente sobre los cuidados del pie diabético y realizar curaciones básicas en pie diabético (revisar: Curaciones básicas del pié diabético código: 1.02.5.001).

Año 2016

16 Facultad de Medicina, Universidad de Chile

Cetoacidosis Diabética

degradan a ácidos grasos y glicerol. Los ácidos grasos libres salen a la circulación unidos a lipoproteínas, aumentando la cantidad de VLDL en plasma. Cuando los ácidos grasos llegan al hígado se degradan a AcCoA, y como esta vía está muy aumentada, se acumulan dando paso a la condensación y a la excesiva formación de cuerpos cetónicos. Al igual que el ácido láctico, los cuerpos cetónicos se encuentran disociados en el plasma generando la acidosis. La diuresis osmótica, puede provocar una pérdida de electrolitos plasmáticos: cloro, sodio y potasio, o mantener estos parámetros normales.

Nivel de manejo del médico general: Diagnóstico Específico Tratamiento Inicial Seguimiento Completo

Aspectos Esenciales • • • •

Complicación aguda de DM, generalmente tipo 1. Buscar factor descompensante en paciente diabético que llega con CAD: Infección, trauma. Sospecha clínica y confirmación con laboratorio. Es vital no demorar tratamiento. Lo más importante es hidratar precozmente al paciente. Inicio de insulina después de verificar kalemia.

Normalmente se produce en las etapas iniciales del tratamiento de los pacientes diabéticos, ya sea por una falla en la adecuación de la dosis de insulina, por un mal manejo de ésta, o por desconocimiento de la enfermedad. Es más, muchos de los pacientes con DM I debutan con cetoacidosis.

Diagnóstico

Caso Clínico Tipo

Clínica: Desarrollo rápido. Poliuria, polidipsia, polifagia y pérdida de peso de tiempo variable (2-4 semanas, en lactantes menos de 1 semana), dolor abdominal (tipo abdomen agudo), náuseas y vómitos, deshidratación, compromiso neurológico que puede llegar al coma. Al examen físico: respiración de Kussmaul, taquicardia, hipotensión, piel seca, y compromiso de conciencia.

Paciente de 15 años, sin antecedentes de diabetes, quien refiere polidipsia y poliuria, compromiso del estado general, sudoración y cansancio físico extremo, consulta a urgencia al examen físico le encuentran turgor disminuido, respiración entrecortada y rápida, presión 90/60, pulso=100 por minuto, t° 38°C. Se le realiza un hemoglucotest = 400mgr/dl, Leucocitosis de 15000/mm3 en el hemograma rápido.

Laboratorio: 1. Criterios diagnósticos: • Glicemia >250 mg/dL en adultos • pH <7,3 (el cual puede ser menor a 7 en los casos graves) • Bicarbonato <15 mEq/L • Cuerpos cetónicos positivos en orina y sangre +++ 2. Otros: • Anión GAP >12 • Glucosuria.

Definición La cetoacidosis diabética es una trastorno metabólico grave de la Diabetes, fundamentalmente de la insulinodependiente o tipo I. Se produce cuando los niveles de insulina son insuficientes para manejar los altos niveles de azúcar en la sangre, lo que genera un aumento en la formación de triglicéridos, lipolisis y reacciones de ß-oxidación aumentadas, con la consiguiente formación excesiva de cuerpos cetónicos. Es más frecuente en niños que en adultos.

Clasificación:

Etiología-Epidemiología-Fisiopatología Se asocia principalmente a Diabetes Mellitus (DM) tipo I, pero también se puede desarrollar en pacientes con DM II ante condiciones de estrés severo, como sepsis o trauma, pancreatitis, tratamiento con esteroides y en casos de ayuno prolongado. También se observa ante falta de tratamiento insulínico (suspensión o dosis insuficiente). La falta de insulina en la sangre, y el aumento de las hormonas contrarregulatorias (HCR) (glucagón, adrenalina, cortisol y hormona del crecimiento) generan un aumento del azúcar en la sangre, la hiperglicemia genera una diuresis osmótica que provoca deshidratación, y como no se puede utilizar para la producción de energía, se utiliza en la formación de triglicéridos (TG). Debido al aumento de las HCR, los TG se

17 Facultad de Medicina, Universidad de Chile

Diagnósticos diferenciales: cetoacidosis alcohólica, acidosis láctica, falla renal aguda, intoxicación por polietilenglicol o ácido acetil salicílico en cuadro de diarrea aguda, coma hiperosmolar.

(diferir administración hasta alcanzar un potasio de 3,3 mEq/l). Suspender la infusión de insulina cuando se haya alcanzado un pH > 7.3 (recordar mantener la infusión de insulina al menos 15 minutos después de administrada la primera dosis de insulina SC). 3. Corregir la acidosis severa (pH < 6.9) que persiste después del período de hidratación inicial con administración de bicarbonato. Sobre este pH no está recomendado su uso. 4. Corregir el desbalance electrolítico: • Sodio: Basta para reponer la natremia el aporte que se hace con las soluciones salinas, no requiriéndose un aporte extraordinario de sodio. • Potasio: - K+ > 5.2 mE/l. No requiere suplementar de forma inmediata. Control cada 2 horas. - K+ 3.3 - 5.2 mE/l. Aportar 20-40 meq/hr por litro de solución salina. - K+ < 3.3 meq/l. aportar 20-30 meq en el primer litro de hidratación y diferir el uso de insulina. • Fosfato: no está recomendado su uso en CAD. Utilizar sólo si existe hipofosfemia severa y compromiso de conciencia persistente. 5. Identificar y corregir factores desencadenantes.

Tratamiento El tratamiento debe realizarse de forma precoz, y las dosis empleadas de NaCl, insulina, bicarbonato y potasio, dependerán de la edad del paciente. Según las guías clínicas del MINSAL 2013, debe enfocarse en: 1. Restablecer volumen circulatorio y perfusión tisular dependiendo del grado de deshidratación del paciente, utilizando solución isotónica de NaCl al 0,9%, y suero glucosado cuando la glicemia disminuya a 200 mg/dl para prevenir hipoglicemias. 2. Disminuir la concentración de glucosa plasmática y la cetoacidosis con insulina cristalina endovenosa de preferencia (si no hay vía venosa permeable puede administarse IM o SC), una vez confirmado el diagnóstico de laboratorio, 1 a 2 horas después de iniciada la hidratación y, siempre y cuando, no exista hipokalemia severa

* Criterios diagnósticos: glicemia > 250 mg/dl, pH venoso < 7.3, bicarbonato <15mEq/L, cetonuria moderada o cetonemia. * Manejo del paciente Adulto con Cetoacidosis Diabética. Guía Clínica AUGE, Diabetes Mellitus tipo 1, MINSAL 2013.

Seguimiento Autor / Editor Francisco Weber

Derivación a especialista.

18 Facultad de Medicina, Universidad de Chile

Año 2017

Hipoglicemia

• •

Nivel de manejo del médico general: Diagnóstico Específico Tratamiento Inicial Seguimiento Completo



Aspectos Esenciales • • • •

Más frecuente en pacientes con DM I, aunque también puede verse en pacientes no diabéticos. Frecuente en pacientes diabéticos por ayuno, por dosis errónea de terapia hipoglicemiante, o por tratamiento con Insulina. Paciente conciente: glucosa por vía oral (½ vaso de agua + 1 cucharada sopera de azúcar). Paciente inconciente: 1 ampolla de glucagón o 20 ml glucosa al 20-30%.

respuesta a la ingesta). Hipoglicemia facticia. Tumores productores de IGF-II (fibrosarcoma, mesotelioma, carcinoma adrenal, entre otros). La hipoglicemia constituye una complicación frecuente en diabéticos por retrasar u omitir una comida, beber alcohol en exceso o sin ingesta simultánea de alimentos, hacer ejercicio intenso sin haber ingerido una colación apropiada o por dosis errónea del hipoglicemiante oral o insulina. Hay que recordar que al progresar la falla renal en un diabético con nefropatía, es más frecuente la aparición de hipoglicemia por aumento de la vida media de los hipoglicemiantes y la insulina.

Diagnóstico Normalmente, los síntomas y signos de hipoglicemia se presentan a niveles de glicemia <50 a 60 mg/dl, pero dependerá de cada paciente. Según la Guía clínica de Diabetes Mellitus I (DM I) del MINSAL no existe consenso sobre el nivel de glicemia que define la hipoglicemia. Sin embargo, la ADA ha definido que niveles de glicemia >70 mg/dl disminuyen el riesgo de una Hipoglicemia Severa. Las guías de práctica clínica de la Sociedad Americana de Endocrinología indican que se debe estudiar y manejar la hipoglicemia cuando se presente la tríada de Whipple:

Caso Clínico Tipo Paciente de 50 años con antecedentes de HTA y DM2, usuario de insulina NPH. Consulta en Servicio de urgencia por sudoración, temblor y palpitaciones. Al interrogatorio dirigido refiere no haber consumido el desayuno.

1. Síntomas, signos o ambos sugestivos de hipoglicemia. 2. Una concentración de glucosa plasmática baja. 3. Resolución de los síntomas o signos después de que la concentración plasmática de glucosa aumenta.

Definición Es una de las consecuencias más severas de la descompensación en la Diabetes Mellitus que normalmente requiere hospitalización y manejo inmediato. Se caracteriza por la tríada de Whipple: 1. Síntomas hipoglicémicos o probablemente hipoglicémicos, especialmente si el paciente se encuentra en ayuno o después de ejercicio intenso. 2. Hipoglicemia concurrente con los síntomas. 3. Alivio de los síntomas al administrar glucosa al paciente.

Los signos y síntomas son: • Autonómicos: Palidez, temblor, sudoración fría, taquicardia. • Neuroglucopénicos: Alteración del juicio y conducta, confusión, compromiso de conciencia, visión borrosa, parestesias, alteración del habla, convulsiones, coma y muerte. • Inespecíficos: Irritabilidad, terrores nocturnos, llanto, náuseas, hambre, cefalea, otros. Se clasifica en: 1. Hipoglicemia leve o moderada: paciente conciente con hipoglicemia sintomática o con hipoglicemia asintomática. 2. Hipoglicemia severa: paciente con compromiso de conciencia que requiere de ayuda de un tercero para tratar su hipoglicemia, con convulsiones o coma.

Etiología-Epidemiología-Fisiopatología Existe una gran diversidad de causas: • Ayuno prolongado • Fármacos: dosis incorrecta de insulina, RAM de hipoglicemiantes orales. • Tumores pancreáticos (insulinoma: la mayoría intrapancreático, 10% maligno asociado a MEN 1). • Insuficiencia renal severa (falta de contribución del riñón a la gluconeogénesis, bloqueo de gluconeogénesis hepática por toxinas, disminuye clearence de insulina). • Hepatopatía severa (falla gluconeogénesis hepática y reserva hepática de glucógeno). • Hipoglicemia reactiva verdadera (rara, en mujeres jóvenes, hay liberación aumentada o retardada de insulina en

Tratamiento Depende del setting clínico. 1. Pacientes con hipoglicemia leve o moderada, deben recibir hidratos de carbono en forma de glucosa 20 g en adultos y 0,3 g/kg en niños. Con esto se espera un aumento de 45 a 65 mg/dl en la glicemia.

19 Facultad de Medicina, Universidad de Chile





Con glicemia de 60 – 70 mg/dl: Si falta 1 hora para la comida: adelantarla y controlar nuevamente la glicemia. Según eso, decidir dosis de Insulina a utilizar (pacientes DM IR con manejo avanzado del tratamiento). Si falta más de 1 hora para la comida: consumir una colación de 12 – 15 g de CHO. Con glicemia <60 mg/dl: Consumir colación de 10 – 15 g CHO de fácil digestión, y controlar glicemia a los 15 minutos. Si no se normaliza, repetir la toma. Ejemplos de colación: ½ taza de jugo o de bebida con azúcar 1 yogur o 1 vaso de leche 2 galletas 3 pastillas de glucosa 2 cucharaditas de pasas 1 cucharadita de miel

2. Pacientes con hipoglicemia severa (compromiso de conciencia, convulsiones o vómitos), deben recibir una inyección IM de glucagón 0,5 mg en menores de 12 años, 1 mg en mayores de 12 años, o bien 10 a 30 mcg/ kg. Administración de solución glucosada al 10 a 30%. Se considera que la hipoglicemia ha revertido cuando alcanza valores de 100 mg/dL.

Seguimiento Depende de etiología. Si persiste sintomatología derivar a especialista.

Autor / Editor Francisco Weber

Año 2017

20 Facultad de Medicina, Universidad de Chile

Diabetes por Corticoides

Mecanismo: La terapia con GC aumenta la proteólisis, pero disminuye su utilización a nivel del músculo y aumenta la gluconeogénesis hepática. Por otra parte, se produce disminución de la secreción de insulina (por disminución de GLUT 2), reduciendo la traslocación de GLUT 4, y con ello, la captación de glucosa a nivel muscular y de adipocitos generando resistencia a la insulina, intolerancia a la glucosa y finalmente, desarrollando la DM.

Nivel de manejo del médico general: Diagnóstico Específico Tratamiento Inicial Seguimiento Completo

Aspectos Esenciales • • • • •

Diagnóstico

Hiperglucemia debido al uso de corticoides. Principales FR: Dosis y duración del tto con GC. Efectos dosis dependientes. Se caracteriza por hiperglicemia postprandial. Tratamiento consiste en disminuir hiperglicemias, ajustar alimentación, insulina y/o HGO, y GC. Monitorear constantemente.

Criterios de diabetes con antecedentes de uso prolongado de GC: Glicemia ayunas ≥ 126mg/dl Glicemia al azar ≥ 200 mg/dl o Prueba de tolerancia a la glucosa oral (PTGO) ≥ 200mg/dl. Normalmente, los GC no provocan hiperglicemia en ayunas, pero si lo hacen post comida (postprandiales), por lo que el examen de glicemia en ayunas podría subdiagnosticar los casos, debiendo recurrir entonces a glicemias al azar. La HbA1c es de especial utilidad, sin embargo en Chile no está validada como medio diagnóstico.

Caso Clínico Tipo Paciente mujer de 72 años, HTA, DM2 en tratamiento con metformina y glibenclamida, que hace 4 meses inició tratamiento con GC por Artritis Reumatoidea, que presenta elevación de sus glicemias postprandiales.

Tratamiento Al igual que en la DM, se deben normalizar las glicemias. Primero, se debe estudiar el tiempo, dosis y vía de administración de los GC, y evaluar una posible disminución o supresión en los casos pertinentes. Si los GC no pueden ser suprimidos, se deben trabajar en un equipo multidisciplinario (médico tratante, enfermera y nutricionista) para monitorear las glicemias capilares, ajustar ingesta de carbohidratos, y según esto recurrir al tratamiento con insulina (ajustando las dosis en la medida que sea necesario). También se puede estudiar el uso de hipoglicemiantes orales (HGO), como metformina y monitorear respuesta. Metas del control: lo recomendado para la mayoría de pacientes con DM: glicemia en ayunas <130mg/dl, postprandial <180mg/dl y HbA1c <7%.

Definición Es un trastorno secundario debido al uso de glucocorticoides (GC) que genera hiperglicemias en rangos de DM, sin antecedentes previos de esta enfermedad.

Etiología-Epidemiología-Fisiopatología El desarrollo de DM en el contexto de uso de corticoides es poco común en individuos previamente euglicémicos. Sin embargo la hiperglicemia es una RAM del uso de glucocorticoides (GC) en dosis suprafisiológicas por cualquier vía. Se caracterizan en general por una elevación moderada de la glicemia de ayuna con marcada hiperglicemia post-prandial.

Seguimiento

Epidemiología: Los GC empeoran la diabetes conocida y pueden precipitar una diabetes desconocida. La frecuencia es muy variable (entre el 1 y el 46%) y en pacientes postrasplante, es entre el 10 y el 20%. Puede ocurrir a cualquier edad y se considera un marcador del futuro desarrollo de diabetes.

Efectuado por especialista.

Autor / Editor Paulina Lira

Factores de riesgo: Los principales factores de riesgo son tratamiento en dosis altas y prolongadas. Otros factores de riesgo incluyen la edad, antecedentes familiares de DM, alteraciones previas en el metabolismo de los hidratos de carbono (resistencia a la insulina e intolerancia a la glucosa), IMC y perímetro de cintura elevados. La vía oral y parenteral tienen mayor incidencia de DM asociada a corticoides que los administrados de forma inhalatoria o por vía intraarticular.

21 Facultad de Medicina, Universidad de Chile

Año 2016

Dislipidemias

Etiología-Epidemiología-Fisiopatología Pueden ser primarias: de origen genético o hereditario (hipercolesterolemia familiar, hiperlipidemia familiar combinada, defecto de apoproteína B e hipertrigliceridemia familiar) o secundarias, asociadas a enfermedades (DM, obesidad, hipotiroidismo, síndrome metabólico, etc.). El proceso de ateroesclerosis se inicia en etapas tempranas de la vida, una detección oportuna de dislipidemia mejora su pronóstico y previene complicaciones en la vida adulta. El 40 a 50% de los niños con Hiperlipidemia continúan entre 4 a 15 años después.

Nivel de manejo del médico general: Diagnóstico Específico Tratamiento Completo Seguimiento Completo

Aspectos Esenciales • • • •

Producción excesiva de lipoproteínas séricas. Pueden ser primarias o secundarias. Tratamiento dietético y hábitos saludables son la terapia de primera línea. La promoción de hábitos saludables es fundamental para la prevención.

Caso Clínico Tipo Niño de 13 años es llevado para control de salud, con antecedentes de padre con IAM a los 50 años y madre Diabética. Al examen físico: peso de 51 kg y talla de 1,45 m. Es asmático y no realiza actividad física, su alimentación es variada.

Definición Las dislipidemias son un conjunto de patologías caracteriza-

das por alteraciones en las concentraciones de los lípidos sanguíneos a un nivel que significa un riesgo para la salud. Es un término genérico para denominar cualquier situación donde existan concentraciones anormales de colesterol total, HDL, LDL o triglicéridos.

Diagnóstico Se debe realizar tamizaje a todos los niños desde los 2 años y antes de los 10 años, que pertenezcan a grupos de riesgo (obesidad, HTA, DM, historia familiar de dislipidemia o enfermedad CV precoz). Además de una anamnesis completa con examen físico enfocado a detectar sobrepeso (obesidad) y síndrome metabólico (acantosis nigricans).

Tratamiento El tratamiento dietético y los hábitos saludables siguen siendo el primer nivel terapéutico de dislipidemia infantil. Las principales medidas de prevención primaria de ECV ateroesclerótica están relacionadas con una dieta sana, se busca mantener un peso, perfil de lípidos y presión arterial adecuados, realizar tamizaje de índice de masa corporal, circunferencia de cintura, determinación de colesterol y lipoproteínas, glucosa e insulina séricos, y control de peso y diabetes. El tratamiento con estatinas es el de elección en la hipercolesterolemia familiar y mejora la morbimortalidad. En caso de requerir tratamiento farmacológico se debe derivar al especialista.

22 Facultad de Medicina, Universidad de Chile

Seguimiento En APS se deben tomar medidas urgentes en educación, detección y atención temprana. En caso de dislipidemias hereditarias y/o patologías secundarias derivar a especialista.

Autor / Editor Paulina Lira

Año 2016

23 Facultad de Medicina, Universidad de Chile

Insuficiencia Renal Y Diabetes

Diagnóstico La pesquisa de nefropatía diabética debe comenzar al momento del diagnóstico en los pacientes con diabetes tipo 2 y a los 5 años del diagnóstico en los tipo 1. Realizar albuminuria en 24 hrs o índice urinario en muestra aislada albúmina/creatinina, además de parámetros para determinar VFG (creatinina plasmática). Si el primer resultado es positivo: repetir antes de hacer el diagnóstico (gran variabilidad de la excreción urinaria). Si la segunda muestra resulta negativa: repetir una tercera vez.

Nivel de manejo del médico general: Diagnóstico Específico Tratamiento Inicial Seguimiento Derivar

Aspectos Esenciales • • •

La ERC en diabéticos va desde la nefropatía incipiente hasta IRC terminal. La principal causa de ingreso a programas de hemodiálisis es la nefropatía diabética. El principal objetivo del tratamiento es evitar la progresión del daño renal.

El diagnóstico se realiza con dos resultados positivos en un intervalo de 3 meses. La pesquisa no debe realizarse en presencia de: Infección urinaria, hipertensión arterial no controlada, Insuficiencia cardiaca congestiva, fiebre, ejercicio intenso, hematuria, haber tenido relaciones sexuales el día previo.

Caso Clínico Tipo Tratamiento

Paciente diabético de larga data, con regular control metabólico y mal control de otras patologías de base, como HTA y dislipidemia.

• •

Definición



Deterioro permanente de la función renal en un paciente diabético. La nefropatía diabética se define como la excreción urinaria de albúmina en ausencia de otras enfermedades renales. Se categoriza en: microalbuminuria 30–299 (g/mg creatinina o mg/24 hr) o macroalbuminuria ≥ 300. La última se considera nefropatía establecida. Si consideramos la proteinuria, ésta debe ser sobre 500 mg/24 horas para diagnosticar nefropatía diabética establecida (no es el método estándar utilizado).



Seguimiento Derivar.

Autor / Editor Paulina Lira

Etiología-Epidemiología-Fisiopatología •





Control glicémico estricto HbA1C < 7%. IECA o ARA II son fármacos que están indicados como primera medida frente a cualquier grado de nefropatía diabética, aún en pacientes normotensos, con el objeto de disminuir el grado de proteinuria. Control de Presión arterial. Meta de 130/80 mm/Hg o 125/75 mmHg para pacientes con falla renal establecida. Mantener Colesterol LDL < 100 mg/dl.

La principal causa de ingreso a diálisis es la nefropatía diabética con 34% de todos los ingresos. El mal control glicémico es el principal determinante de la progresión del deterioro en la función renal. En DM1 la incidencia de microalbuminuria es de 2040/1000 pacientes-año, a partir de los 5 años de evolución de la DM. A los 20-30 años de evolución 50% de los pacientes la presenta. Esta fase es reversible hasta un 50% de los casos. Entre 40 y 80% de los pacientes pueden progresar a proteinuria. La microalbuminuria es una fase potencialmente reversible, un 20-40% de los pacientes progresa a nefropatía clínica y de éstos, un 20% llega a IR terminal a los 20 años. Confiere un mayor riesgo cardiovascular y es un marcador de disfunción endotelial.

24 Facultad de Medicina, Universidad de Chile

Año 2016

Neuropatía Diabética



Nivel de manejo del médico general: Diagnóstico Sospecha Tratamiento Inicial Seguimiento Derivar



Aspectos Esenciales • • •

Formas clínicas: motora, sensitiva y autonómica. El pie diabético sigue siendo la neuropatía predominante en la clínica, pero también podemos encontrar la gastroparesia o la vejiga neurogénica.

Factores de crecimiento: Los niveles adecuados de insulina mantienen la supervivencia y el crecimiento del axón, pero cuando se manifiesta la DM I o en etapas avanzadas de DM II cuando los niveles de insulina son muy bajos o nulos, ya no es capaz de mantener estas funciones y se produce apoptosis de las células axonales. Déficit de tiamina: La falta de insulina también incide en una menor absorción de tiamina, provocando la disminución de este micronutriente que frena el ciclo de las pentosas y favorece la vía de la fructosa que aumenta la proteína kinasa C, y que por otra parte disminuye la reducción de NADH disminuyendo la capacidad antioxidante del organismo.

Diagnóstico Caso Clínico Tipo

La clasificación más frecuente de la ND es la siguiente: • Motora: Poco frecuente, afecta principalmente a los nervios craneales, y puede causar desde debilidad muscular hasta parálisis. • Sensitiva: Es la más frecuente, sus síntomas van desde dolor, pinchazos, hormigueos, calambres en las extremidades, sobretodo en las piernas. • Autonómica: Afecta el sistema nervioso autónomo y con ello los principales sistemas: digestivo, genitourinario, cardiovascular. • Examen Físico: • Pies, manos o brazos en busca de micosis, úlceras o pérdida de sensibilidad. • Problemas gastrointestinales a repetición que no tienen una causa determinada (patógenos o virus). • Problemas genitourinarios (incontinencia urinaria, disfunción eréctil, infecciones urinarias a repetición, disfunción eréctil). • Hipotensión ortostática.

Paciente diabético descompensado con HbA1c >7% que consulta por infecciones urinarias a repetición y periodos de diarrea y constipación con pérdida de sensibilidad en sus extremidades.

Definición Es la más común de las complicaciones de la Diabetes Mellitus, tanto la tipo I como la tipo II, y puede ir desde la pérdida de sensibilidad hasta el dolor y debilidad muscular, ya que compromete principalmente al sistema nervioso periférico (SNP).

Etiología-Epidemiología-Fisiopatología La neuropatía diabética afecta al 50% de los diabéticos tipo II al momento del diagnóstico e incluso previo a éste y aumenta la probabilidad de su aparición a medida que evoluciona la enfermedad con los años.

Tratamiento • • • •

El sistema nervioso codifica de manera errónea las señales que envía el organismo y produce la neuropatía que se produce debido a una asociación de factores: • Metabólicos: La hiperglicemia sostenida provoca que la glucosa ingrese al axón y a la célula de Shwann, generando hiperosmolaridad y edema. Por otra parte el aumento de NADP+ evita que se neutralicen las ROS y NOS generando daño oxidativo, y la acumulación de NADH favorecerá la vía inflamatoria y la fibrosis. • Vasculares: Al producirse daño en los vasos que irrigan los nervios, éstos dejan de entregar los nutrientes necesarios para que la célula nerviosa codifique correctamente el mensaje. • Glicosilación de las proteínas: En la diabetes descompensada existe una acumulación de los productos de glicosilación (AGEs) en proteínas de larga vida como en la mielina del sistema nervioso periférico, transformándose en sustancias blanco de los macrófagos (RAGE) que contribuirán al daño.

Óptimo control glicémico Ha1bc <7. Suplementos vitamínicos de tiamina. Cuidados dermatológicos. Reeducación alimentaria y énfasis en correcto tratamiento nutricional y farmacológico.

Seguimiento Se recomienda test de monofilamento y diapasón como screening una vez por año, entregando la clasificación de riesgo de ulceración. Pacientes con alto riesgo, deben ser derivados a especialista.

Autor / Editor Paulina Lira

25 Facultad de Medicina, Universidad de Chile

Año 2016

Obesidad

Diagnóstico: La obesidad se define como la presencia de una cantidad excesiva de grasa corporal que conlleva un riesgo para la salud. El indicador más utilizado para su diagnóstico es el índice de masa corporal (IMC), considerando su buena correlación con el exceso de adiposidad y la morbimortalidad asociada. La obesidad, para fines prácticos, se define con un IMC >30, Calculo de IMC: IMC= peso (kg) / estatura 2 (metros)

Nivel de manejo del médico general: Diagnóstico Específico Tratamiento Completo Seguimiento Completo

Aspectos Esenciales • • • • •

Enfermedad crónica por desbalance energético de etiología multifactorial. Principal factor hiperalimentación y sedentarismo. Factor de Riesgo Cardiovascular. Prevalencia alta en la población general. Aumento de prevalencia en la población infantil

Tratamiento: Dada la etiología multifactorial de la obesidad, la base del tratamiento es una alimentación sana, equilibrada y realizar ejercicio físico, es decir un cambio de estilo de vida. Idealmente en conjunto con equipo multidisciplinario conformado por médicos especialistas (nutriólogos, endocrinológos), nutricionistas, kinesiólogos y profesores de educación física, equipo de salud mental y, eventualmente, cirujanos con experiencia en técnicas de cirugía bariátrica. Hábitos Recomendados: 1. Cambios en hábitos dietarios • 4 comidas diarias, educar respecto a porciones, inclusión de colaciones entre comidas. • Disminución del consumo de grasas, especialmente saturadas (cecinas y embutidos, carnes grasas, mantequilla, margarina, mayonesa, manteca, crema, etc.) • Disminución del consumo de azúcar y alimentos azucarados (refrescos, mermeladas, etc.) • Aumento del consumo de fibra (verduras y frutas crudas, leguminosas y cereales de grano entero). • Preferir carnes blancas (pescado, pavo y pollo o rojas con bajo contenido graso como posta, lomo liso, punta de ganso). • Aumentar el consumo de lácteos descremados • Reducir el consumo de sal 2. Dejar el tabaco 3. No consumir bebidas alcohólicas 4. Crear un plan de actividad física apropiado para cada paciente, para lograr idealmente 150 minutos a la semana de ejercicios de moderada intensidad. • Ventajas de la actividad física durante la reducción de peso - Previene la reducción del gasto energético inducid por dieta hipocalórica - Disminuye la pérdida de masa magra - Mejora la adherencia a un plan multidisciplinario del tratamiento - Da una sensación de bienestar físico y mental - Induce mayor gasto energético y favorece el balance energético negativo

Caso Clínico Tipo Paciente con IMC 31 kg/mt2, cuadro se inicia en la infancia. Se asocia a sedentarismo y otros factores de riesgo cardiovascular: DM e HTA, entre otros. Nivel socioeconómico bajo y jornada laboral de 10 hrs, con una hora de almuerzo en todo el día.

Definición Presencia de una cantidad excesiva de grasa corporal, objetivado con un IMC Igual o superior a 30 Kg/m2.

Etiología-Epidemiología-Fisiopatología La mayoría de los casos de obesidad son de origen multifactorial. Se reconocen factores genéticos, metabólicos, endocrinológicos y ambientales. Sin embargo, la obesidad exógena o por sobrealimentación constituye la principal causa, produciendo un desbalance energético, lo que trae como consecuencia un exceso de grasa en el organismo. Sólo un pequeño porcentaje (2 a 3%) de los obesos tendrán como causa alguna patología de origen endocrinológico. Según la encuesta nacional de salud de 2010, el 25,1 % de la población chilena es obesa y 2,3% es obeso mórbido (lMC > 40). Los adultos obesos tienen tasas más altas de enfermedad cardiovascular y de mortalidad por todas las causas. La prevalencia es más elevada en las mujeres y en las personas de nivel socioeconómico más bajo. El riesgo para la salud de un exceso de grasa corporal depende de la distribución de ella. Así, la distribución de grasa de tipo androide, abdominal o central tiene una mejor correlación con la grasa visceral y un mayor riesgo de presentar morbilidades asociadas a ella (hipertensión arterial, dislipidemias, etc).

En caso de fracaso a tratamiento con cambios de estilo de vida, se hace necesaria una terapia farmacológica. Actualmente los fármacos aprobados por la FDA son: •

A corto plazo (menos de 12 semanas):

26 Facultad de Medicina, Universidad de Chile

- Fentermina: inhibidor del apetito. Libera NA y DA. - Químicamente similar a anfetamina. - Aprobado en EE.UU. para uso a corto plazo por potencial abuso, (retirado de Europa). - Dosis:15 – 30 mg diarios. - Baja de peso promedio 3,6 kg sobre el placebo en monoterapia - Dada tolerancia, se hace necesario aumentar la dosis con el tiempo - Efectos adversos: estimulación del sistema nervioso central (SNC), cefalea, insomnio, palpitaciones, taquicardia, aumento de la presión arterial, rabdomiolisis, hemorragia intracraneal - Dietilpropión - Químicamente similar a bupropión, - Aprobado para uso a corto plazo por potencial abuso - Dosis 75 mg. - Baja peso promedio 10 kg en los primeros 3 meses - Efectos adversos: boca seca, insomnio y estimulación del SNC •

Largo plazo: - Orlistat: interfiere la absorción de lípidos al inhibir la enzima lipasa pancreática - Dosis recomendada: 1 capsula 3 veces al día con las comidas. - Efecto dosis dependiente: 120 mg: disminuye hasta un 30 % grasa ingerida - 60 mg: disminuye hasta un 25%



Otros beneficios de Orlistat: - 37% reducción en la conversión de intolerancia a la glucosa a diabetes - Reducción de colesterol LDL más allá de lo esperado por la baja de peso - Reducción de la lipemia postprandial

dable y actividad física, motivar al paciente a ingresar nuevamente al Programa y reforzar los logros alcanzados por el paciente en el módulo de obesidad en cualquier control de salud posterior. En el caso de pacientes con IMC > 40 la indicación es la cirugía barlátrica, igual que en pacientes con IMC >35 y comorbllidades de riesgo CV (diabetes, HTA, dlslipldemia, esteatosis hepática, etc.)

Autor / Editor Tomás González



Efectos adversos: Incontinencia fecal, diarrea, goteo oleoso, flatulencia, dolor abdominal, tienden a disminuir con el cambio de dieta. Todo cambio de hábitos debe acompañarse de metas a alcanzar en un tiempo determinado. Para que los pacientes sigan las Indicaciones, estas deben ser lo más específicas posibles y adecuadas a su realidad.

Seguimiento: El seguimiento se debe realizar a través de una intervención específica para los individuos obesos del, en APS, como parte del Programa de Salud Cardiovascular que constituye un plan intensivo para el manejo de la obesidad, con una duración de 4 meses donde el paciente será intervenido por un equipo multidisciplinario Integrado por médico, enfermera, nutricionista y profesor de educación física, según disponibilidad local. Al finalizar los 4 meses se debe evaluar al paciente en base a las metas propuestas, si estas no se logran: referirlo al equipo de promoción del consultorio donde se hará énfasis en las recomendaciones de alimentación salu-

27 Facultad de Medicina, Universidad de Chile

Año 2016

Coma Hiperosmolar

sión) abandono de terapia, infecciones (ITU, neumonía), insuficiencia renal, cardiaca, hepática, secuelas neurológicas invalidantes, alcoholismo, infartos, sepsis, hemorragia digestiva, AVE, pancreatitis, cirugía, etc.

Nivel de manejo del médico general: Diagnóstico Específico Tratamiento Inicial Seguimiento No requiere

Diagnóstico El coma hiperosmolar aparece casi siempre en adultos mayores. Los síntomas se desarrollan lentamente y no posee cuadro clínico característico, excepto cuando hay una causa precipitante. Los síntomas más frecuentes son poliuria, polidipsia, puede haber alteraciones del estado de conciencia, crisis convulsivas y coma, y signo de Babinski, que generalmente remiten con el tratamiento. Al examen físico, se deben buscar signos de depleción de volumen extracelular, como taquicardia, hipotensión arterial y deshidratación grave que puede llevar a shock hipovolémico. A nivel abdominal puede haber náuseas, vómitos dolor abdominal más íleo gástrico y trombosis arteriales por hipercoagulabilidad e hiperviscosidad sanguínea. Criterios diagnósticos: • Glucosa plasmática >600 mg/dl • pH arterial < 7,30 • Bicarbonato sérico < 18 • Cetonas urinarias y plasmáticas escasas • Osmolaridad efectiva (2x [Na+] + glucosa/18) • > 320 mOsm/kg, anión GAP (Na+ – [Cl- + HCO3-]) variable • estado de conciencia variable, llegando hasta sopor o coma.

Aspectos Esenciales • • • • •

Emergencia hiperglicémica. Hiperglicemia, deshidratación extrema, hiperosmolaridad, en ausencia de cetoacidosis. Inicio insidioso. Polidipsia, poliuria, compromiso de conciencia, deshidratación respiración normal, sin halitosis. Tratamiento: hidratación, reducción gradual de glicemia y osmolaridad, corrección del desbalance ELP y pH y tratamiento del factor desencadenante. Ocurre en pacientes con DM2.

Caso Clínico Tipo Paciente de 72 años, con antecedente de DM2, HTA presenta hace tres días cuadro de náuseas, vómitos y fiebre. Familiares relatan que ha estado decaído y que hace dos días no se levanta de la cama y permanece somnoliento. El día de hoy lo llevan a urgencias por convulsión. HGT revela glicemia de 700.

Definición

Tratamiento

Emergencia metabólica aguda de DM2, grave por su alta mortalidad, que se caracteriza por: hiperglicemia severa, hiperosmolaridad, deshidratación severa, con variable compromiso de conciencia (puede simular ACV) y ausencia/leve cetonemia/cetonuria.

Siempre hospitalizar, idealmente unidad de paciente crítico para control hemodinámico e iniciar reanimación del shock, que causa la elevada mortalidad. Protocolo ADA 2009: La base del tratamiento es estabilizar la deshidratación, la hiperglicemia, el desequilibrio electrolítico; identificar el factor precipitante y monitorear. • Fluidoterapia: Si hay shock cardiogénico realizar monitoreo hemodinámico y dar drogas vasoactivas, si es deshidratación severa administrar NaCl 0.9% EV a 1 lt/hr, si es deshidratación leve evaluar Na+ corregido (sumar 1,6 mg/dl de Na+ por cada 100 mg/dl de glucosa sobre 100 mg/dl). Si el Na+ está alto o normal administrar 250 – 500 ml/hr de NaCl al 0,45% dependiendo del estado de hidratación. Si está bajo administrar NaCl al 0.9% dependiendo del estado de hidratación. Cuando la glucosa sérica se encuentre entre 250 - 300 mg/dl cambiar a dextrosa al 5% con 0.45% NaCl a 150 - 250 ml/hr. • Bicarbonato: Si pH > 6.9 no se administrará bicarbonato. De lo contrario, dar 100 mmol en 400 ml de H20 + 20 mEq de KCL, en infusión por 2 horas. Repetir cada 2 horas hasta que pH > 7. Monitorear potasio sérico cada 2 horas. • Insulina: administrar 0,1 U/kg de cristalina en bolo EV y posteriormente la misma dosis en BIC o 0.14 como infu-

Etiología-Epidemiología-Fisiopatología El coma hiperosmolar corresponde al 10% de las complicaciones, principalmente en DM2 sin diagnóstico previo (debut), asociado a una causa desencadenante. (30 – 50% sin diagnóstico previo), posee una mortalidad entre 35-40%. El mecanismo fisiopatológico subyacente es una insulinopenia relativa, que a nivel hepático es suficiente para prevenir la lipólisis y cetogénesis pero no para estimular la utilización de glucosa, lo que sumado al aumento de las hormonas de contrarregulación, determina un estado de hiperglicemia masiva, que se asocia a grados variable de hiperosmolaridad y de hipercoagulabilidad, aumentando el riesgo de enfermedades tromboticas. La hiperglicemia más la deshidratación por diuresis osmótica determinan alteraciones neurológicas de grado variable. Factores desencadenantes Medicamentos (diuréticos, glucocorticoides, inmunosupre-

28 Facultad de Medicina, Universidad de Chile



sión EV en BIC. Si la glucosa sérica no cae al menos un 10% la primera hora dar 0.14 U/KG en bolo EV. Cuando glucosa sérica alcance los 300 mg/dl, reducir infusión continua de insulina cristalina a 0.02 – 0.05 U/kg/hr EV. Mantener glucosa sérica entre 200 y 300 mg/dl hasta que el paciente vuelva a estar vigil. Potasio: Establecer adecuada función renal (diuresis 50 ml/hr). Si K+<3.3 mEq/lt dar 20–30 mEq/hr previo a la insulina hasta que K+>3.3 mEq/lt. Si K+ está entre 3.3 y 5.2 mEq/lt dar 20 a 30 mEq/lt de K+ en cada litro de fluido EV para mantener K+ sérico entre 4 y 5 mEq/lt. Si K+ > 5.2 no dar K+ y monitorear potasio sérico cada 2 horas.

Seguimiento Luego de recuperado no requiere seguimiento específico, sí mantener los controles habituales de su patología.

Autor / Editor Francisco Weber

Año 2017

29 Facultad de Medicina, Universidad de Chile

Síndrome Hiperglicémico Hiperosmolar No Cetósico

mado al aumento de las hormonas contrarregulatorias (glucagón, adrenalina, cortisol, hormona del crecimiento), hace que los niveles de azúcar en la sangre se disparen. El riñón intenta eliminar el azúcar a través de la orina, lo que provoca un balance negativo de agua y sodio secundario a diuresis osmótica. Finalmente se provoca un círculo vicioso: la deshidratación, concentra la glucosa en la sangre que continuará enviado señales al riñón para que la elimine, y aumenta la deshidratación, confusión, y generando además un estado dehipercoagulabilidad aumentando el riesgo de ACV.

Nivel de manejo del médico general: Diagnóstico Específico Tratamiento Inicial Seguimiento No requiere

Aspectos Esenciales • • • •

Etiología: las causas más frecuentes corresponden a infecciones intercurrentes y abandono de tratamiento de DM II. Otras causas corresponden a: Enfermedades severas: IAM, ACV, sepsis, pancreatitis. Medicamentos como diuréticos (tiazidas), corticoides, medicamentos simpáticomiméticos.

Complicación aguda de DM y una emergencia hiperglicémica. Hiperglicemia, deshidratación extrema, hiperosmolaridad, en ausencia de acidosis y ausencia o leve presencia de cetosis. Inicio insidioso. Polidipsia +++, poliuria +++, compromiso de conciencia, deshidratación, respiración normal, sin halitosis. Tto: hidratación, reducción gradual de glicemia y osmolaridad, corrección del desbalance ELP y pH, y tratamiento del factor desencadenante.

Diagnóstico Clínica: Cuadro de inicio insidioso (a diferencia de la cetoacidosis), con poliuria +++, polidipsia +++, y eventual baja de peso en los días anteriores al ingreso. Compromiso relativo de conciencia de forma progresiva (pueden debutar también con focalidad neurológica, compromiso de conciencia cualitativo-cuantitativo). Signos de Deshidratación al examen físico (taquicardia, signo del pliegue, mucosa oral seca) y eventualmente hipotensión.

Caso Clínico Tipo Paciente de 72 años, con antecedente de DM II, HTA presenta hace tres días cuadro de náuseas, vómitos y fiebre. Familiares relatan que ha estado decaído y que hace dos días no se levanta de la cama y permanece somnoliento. El día de hoy lo llevan a urgencias por convulsión. HGT revela glicemia de 700 mg/dl.

Laboratorio: glicemia >600 mg/dl (frecuentemente mayor a 1000 mg/dl), osmolaridad >320 mOsm/Kg, sin acidosis (pH ≥7,3), ni cetosis. Cálculo de anión gap normal o levemente amplio (anión gap sérico: Na - (Cl + HCO3)). Hipovolemia e hiponatremia. Para cálculo de Osmolaridad: Osm=2 x (natremia en mEq/L) + (glicemia en mg/dl /18) + (NU en mg/dl /2,8). Considerar que para Glicemias mayores a 300 se debe ajustar la natremia: natremia + (1,6* [ (glicemia-100)100]).

Definición Complicación aguda y emergencia metabólica aguda de DM II, grave por su alta mortalidad, que se caracteriza por: hiperglicemia severa, hiperosmolaridad, deshidratación severa, con variable compromiso de conciencia, sin cetosis ni acidosis.

Tratamiento Hospitalización inmediata (emergencia Médica). Antes de iniciar el tratamiento constatar ELP. Base del tratamiento consiste en:

Etiología-Epidemiología-Fisiopatología

1. Reposición de volumen: se utiliza solución isotónica (suero salino al 0,9% en hiponatremia, al 0,45% en normo o hipernatrémicos). Meta es completar la mitad del déficit de agua calculado en 12-24 horas (1L/hora durante la primera hora y luego 4-14 ml/kg/hora según deshidratación). Posterior al descenso bajo los 200mg/ dL se utiliza Glucosalino al 5% a 100ml/hora x 48 horas. Para el cálculo de volumen a reponer: Déficit de Agua = H2O Ideal - H2O real H2O Ideal= Peso x 0,6 // H2O real = H2O ideal x osmolaridad ideal/osmolaridad real Osmolaridad Ideal = 300

Epidemiología: Es la forma de presentación más frecuente de la DM II, presentándose también en la DM I (en DM I es más frecuente la cetoacidosis por la deficiencia absoluta de insulina). Alta mortalidad, pero esto dependerá del manejo y del estado del paciente al momento del ingreso. Se observa mayormente en pacientes mayores de 65 años. Fisiopatología: Este síndrome se caracteriza por una leve insulinopenia o insulinopenia relativa y un aumento exacerbado del glucagón. La insulinopenia es suficiente para evitar la lipolisis y la producción de cuerpos cetónicos, pero no es suficiente para estimular la utilización de glucosa. Esto su-

30 Facultad de Medicina, Universidad de Chile

2. Reducción Glicemia: Bolo de 0,1U x Kg de Peso e infusión continua de insulina cristalina (rápida) de 0,1 U/Kg/hora siempre con K mayor a 3.3. Lo ideal es obtener descenso de glucosa 10% por hora. Si no se tiene Bomba: Insulina EV e IM horaria con dosis inicial de 0,4-0.6 U/kg, la mitad EV y la mitad IM y luego 0,1 U/kg/h de insulina regular SC o IM. Luego con glicemia bajo 200 mg/dl se inicia la infusión de suero glucosalino al 5% a 100 ml/h. Se disminuye la infusión de insulina a la mitad. 3. Corrección de Eventual Acidosis y ELP: Se utiliza bicarbonato únicamente con pH<7,0.

Seguimiento Seguimiento realizado por especialista para eventual ajuste de terapia y reeducación. Autor / Editor Francisco Weber

Año 2017

31 Facultad de Medicina, Universidad de Chile

Obesidad Mórbida

Equipo multidisciplinario: Médico, nutricionista, enfermera, psicológo, profesor de educación física, kinesiólogo. Dietoterapia: Dieta hipocalórica con suplementación de oligoelementos. Suspensión de tabaco y alcohol. Ejercicio físico: En forma progresiva, al menos 200 min/semana de actividad moderada. Farmacoterapia: Cuando el IMC > 30 Kg/m² y la terapia con dieta y ejercicio ha fracasado, y ante la presencia de enfermedades concomitantes (DM, dislipidemias, e hipertensión). Cirugía bariátrica: Con IMC > 40 ó > 35 con comorbilidades: bypass gástrico y gastrectomía vertical en manga.

Nivel de manejo del médico general: Diagnóstico Específico Tratamiento Inicial Seguimiento Completo

Aspectos Esenciales • • • •

Riesgo muy severo de salud global. Multifactorial. Historia clínica y examen físico completo, considerando patologías asociadas a obesidad. Tratamiento del obeso mórbido incluye dietoterapia, actividad física, farmacoterapia (orlistat y metformina) y cirugía bariátrica.

Seguimiento Derivar a especialista.

Caso Clínico Tipo Autor / Editor Catalina Luvecce

Paciente mujer de 47 años, con antecedentes de HTA, DM2, SAHOS. Antecedente de infarto en padre. Al examen físico se objetiva IMC de 43, PA de 190/165, acantosis nigricans y acrocordones.

Definición La obesidad mórbida u obesidad clase III se presenta con un IMC ≥ 40 kg/m², teniendo un riesgo muy severo de salud global según la OMS.

Etiología-Epidemiología-Fisiopatología Según la última Encuesta Nacional de Salud (ENS 2010) la obesidad mórbida alcanza un 2,3% de la población. En 2003, 148.000 personas tenían esta condición y que aumentó a 300.000 en 2010. La obesidad mórbida, se produce cuando se sobrepasa el IMC de 40 kg/m², aumentando el riesgo de salud global. Debido a esto, se indica cirugía bariátrica, al igual que en pacientes con IMC > 35 Kg/m² y comorbilidades de riesgo CV (diabetes, HTA, dislipidemia, esteatosis hepática, etc.). En la actualidad, las cirugías más utilizadas son gastrectomía vertical en manga y bypass gástrico que mejoran la sensibilidad y la secreción de insulina, aumentan los niveles de incretinas y en el caso de la primera disminuye los niveles de Ghrelina.

Diagnóstico El diagnóstico se realiza en pacientes cuyo IMC > 40 Kg/m². Tratamiento

32 Facultad de Medicina, Universidad de Chile

Año 2016

Pie Diabético y Otras Infecciones en Diabetes

Diagnóstico Diagnóstico es clínico. Anamnesis: Sexo, edad, sintomatología, y antecedentes personales (antecedentes de DM, tiempo de evolución, tratamiento, y si se encuentra controlada), familiares (DM y otras patologías de importancia) y de las complicaciones (complicaciones de la DM). Examen físico: Signos asociados a la DM descompensada (signos infecciosos, oftalmológicos, renales, neurológicos, inmunológicos, entre otros). Examen rutinario minucioso de pies: pulsos, deformidades, heridas, hiperqueratosis. Evaluación de sensibilidad: monofilamentos, diapasón de 128 Hz, propiocepción.

Nivel de manejo del médico general: Diagnóstico Específico Tratamiento Inicial Seguimiento Completo

Aspectos Esenciales • • •

La Diabetes mal controlada aumenta la ocurrencia de complicaciones. Su frecuencia aumenta con el paso del tiempo. Al examen físico hay que buscar signos de otras complicaciones crónicas.

Tratamiento Como profilaxis se debe mantener un buen control metabólico de la glicemia, y para ello hacer énfasis en el tratamiento nutricional y farmacológico adecuado. Es importante recalcar las siguientes medidas de prevención: Inspección regular de heridas en el pie. Hacer énfasis en el uso de calzado adecuado y de higiene diaria. Tratamiento de las patologías no ulcerativas. Educación del paciente y su familia. Y una vez que la úlcera se produzca realizar un manejo integral y adecuado por el equipo multidisciplinario.

Caso Clínico Tipo Se presenta a su consulta una paciente de 65 años, que acude por control de su diabetes. A la anamnesis se aprecia que la paciente ha presentado compromiso oftalmológico progresivo, infecciones a repetición, y mal cumplimiento terapéutico. Al examen físico destaca una herida plantar de su pie izquierdo con signos inflamatorios y secreción de material purulento, que la paciente refiere no haber sentido.

Definición Seguimiento

El pie diabético se define como “la ulceración, infección y/o gangrena del pie, asociados a neuropatía diabética y diferentes grados de enfermedad arterial periférica” (OMS).

Derivar.

Etiología-Epidemiología-Fisiopatología

Autor / Editor Catalina Luvecce

El pie diabético y otras infecciones son complicaciones crónicas de la DM que se pueden prevenir, pero que son frecuentemente consultadas en la clínica por pacientes diabéticos que no controlan de forma adecuada su enfermedad y es la causa más frecuente de amputación no traumática. La DM provoca un estado proinflamatorio que incide en una mayor probabilidad de generar infecciones, ya sea a nivel del sistema genitourinario (ITU a repetición) y en extremidades inferiores (ulceraciones en pies). En el caso del pie diabético, el aumento de la humedad favorece la micosis, y con ello, las ulceraciones en los pies. La pérdida de sensibilidad, la disminución de la cicatrización, y la falta de preocupación de los pacientes por revisar sus pies, genera que esa ulcera aumente, se infecte, llegando a los casos más graves de necrosis y posterior amputación.

33 Facultad de Medicina, Universidad de Chile

Año 2016

Retinopatía Diabética

Describe una forma juvenil, con evolución rápida y fatal y una forma del adulto, cuyo curso es menos acelerado. Diagnóstico Sospecha clínica y hallazgo oftalmoscópico en el contexto de una Diabetes Mellitus: • Microaneurismas: Lesiones típicas y precoces. Son dilataciones saculares de la pared vascular capilar, con exudación con edema y hemorragias. • Exudados duros o lipídicos: Acúmulo de macrófagos cargados de lípidos y material proteico. Traducen la existencia de edema retiniano. • Hemorragias retinianas: Son infrarretinianas, puntiformes o redondeadas. En las formas proliferantes pueden ser subhialoideas o intravítreas. • Edema macular: Engrosamiento de la mácula por edema. Reversible en los primeros estadios, pero aparecen cavitaciones, dando lugar a edema macular quístico, crónicamente. • Exudados algodonosos: Debido a microinfartos retinianos, reflejan la existencia de isquemia. • Neovasos: Típicos de formas proliferativas, secundario a isquemia retiniana. Pueden aparecer en la papila del nervio óptico o en el polo posterior de la retina. Hay proliferación de bandas conectivas, se retraen provocando desprendimiento de retina de tipo avascular.

Nivel de manejo del médico general: Diagnóstico Específico Tratamiento Completo Seguimiento Completo

Aspectos Esenciales • • • • •

La alteración clásica de la retinopatía diabética son los microaneurismas. El edema macular es la causa más importante de pérdida de agudeza visual en el diabético. Se trata mediante láser focal. Otras alteraciones oftálmicas de la diabetes incluyen la catarata diabética y el glaucoma neovascular. Evaluar fondo de ojo y derivar a oftalmólogo en el momento del diagnóstico, para estandarizar el estadio de la retinopatía para tomar una conducta apropiada. Precaución con usar tratamiento insulínico en paciente con retinopatía proliferativa.

Caso Clínico Tipo

Se recomienda en DM1 Fondo de ojo desde el tercer año del Diagnóstico (Guía MINSAL 2013). En el caso de DM2 la revisión oftalmológica debe ser en el momento del diagnóstico.

Paciente de 43 años con diagnóstico hace 6 meses de DM2, consulta por disminución de agudeza visual de ojo derecho. Al explorar el fondo de ojo, vemos dilataciones saculares de la pared vascular capilar con exudación con edema y hemorragias.

Tratamiento Médico: Control óptimo de glicemia con buen control metabólico, de la presión arterial (IECAs, ARAII), y corrección de dislipidemias, si hubiese. Quirúrgico: Disminuir la neoproliferación mediante fotocoagulación. • En caso de hemovitreo, esperar que reabsorba, para fotocoagular. Si no reabsorbe, realizar vitrectomía. • En caso de no visualizar la retina, está indicada la ecografía para descartar desprendimiento retiniano. • En el edema macular se recomienda fotocoagulación.

Definición Microangiopatía diabética a nivel retiniano. Posee 2 formas evolutivas: una inicial, la retinopatía no proliferativa y secundariamente la retinopatía proliferativa. Principal morbilidad de diabéticos y gran problema de salud pública, pues es la principal causa de ceguera bilateral irreversible en menores de 65 años en occidente.

Etiología-Epidemiología-Fisiopatología

Seguimiento

Mecanismo de retinopatía no proliferativa: La hiperglicemia mantenida produce pérdida de los pericitos, los que dan soporte y forman parte de la barrera hematoretinal, llevando a cambios en la hemodinamia regional y en la membrana capilar. Esto favorece la aparición de lesiones clásicas y activación de la cascada de coagulación, llevando a isquemia capilar.

Buen control de la diabetes, presión arterial y dislipidemias retrasa la aparición y enlentece el progreso de la retinopatía diabética. Revisión oftalmológica independiente del tipo de diabetes, cada 6 meses o cada 1 año.

Mecanismo de retinopatía proliferativa: Durante la isquemia se estimulan producción de factores proangiogénicos (VEGF), mecanismo que no solo se asocia a hiperglicemia, sino también a estímulo directo de hormonas contrarreguladoras como la insulina y la hormona del crecimiento. Provoca pérdida de visión brusca e indolora por hemorragia vítrea.

Autor / Editor Paulina Lira

34 Facultad de Medicina, Universidad de Chile

Año 2016

Síndrome Metabólico

grasos libres (AGL). La grasa de origen visceral llega directamente al sistema porta y por lo tanto al hígado, provocando una insulino-resistencia hepática que lleva a un aumento de la liberación de glucosa, la síntesis de citoquinas proinflamatorias (proteína C reactiva, TNF α, resistina, interleuquina 6 y 18) y cambios en el metabolismo lipoproteico que se traducen en un exceso de triglicéridos circulantes.

Nivel de manejo del médico general: Diagnóstico Específico Tratamiento Inicial Seguimiento Completo

Aspectos Esenciales • • • •

Diagnóstico

Patología de etiología multifactorial. Prevalente en nuestra población (25%). Criterios diagnósticos sin consenso único. Componentes esenciales: Obesidad abdominal, hipertrigliceridemia, HDL bajo, HTA y DM.

Es importante tener en consideración que frente a la presencia de un elemento (clínico o laboratorio) que sugiera síndrome metabólico, es necesario buscar dirigidamente los restantes. Existen distintos criterios diagnósticos, los más utilizados son: ATP III (diagnóstico con al menos 3 criterios) 2005: • Obesidad abdominal: Circunferencia de cintura > 102 cm en hombres y > 88 cm en mujeres. • TG sérico ≥ 150 mg/dL o tratamiento por TG elevados. • HDL < 40 mg/dL en hombres y < 50 mg/dL en mujeres o tratamiento farmacológico por HDL bajo. • Presión arterial ≥ 130/85 mmHg o tratamiento farmacológico antihipertensivo. • Glicemia en ayunas ≥ 100 mg/dL o tratamiento farmacológico hipoglicemiante. International Diabetes Federation 2006: Debe estar presente la obesidad abdominal según raza/etnia. Más dos de los siguientes criterios: • TAG > 150 mg/dL o en tratamiento • HDL < 40 mg/dL en hombres o < 50 mg/dL en mujeres, o en tratamiento. • PAS > 130mmHg, PAD > 85mmHg, o tratamiento antihipertensivo. • Glicemia en ayuna > 100 mg/dL o diagnostico previo de DM2. Se recomienda evaluación con PTGO, pero no es requerido para el diagnóstico.

Caso Clínico Tipo Paciente sexo masculino 38 años, hipertenso en tratamiento, sedentario y con malos hábitos alimentarios, con obesidad abdominal evidente, que en exámenes de control presenta TG 200 mg/dL, HDL 30 mg/dL, glicemia 110 mg/dL. Acude a control para evaluación anual de su trabajo.

Definición Corresponde a un conjunto de factores de riesgo asociados entre sí y que sumados en una misma persona promueven un estado protrombótico y proinflamatorio común en individuos obesos y que incrementa los riesgos mayores de enfermedad cardiovascular y diabetes mellitus 2.

Etiología-Epidemiología-Fisiopatología Epidemiología: Se estima que un 25% de la población adulta padece síndrome metabólico; en Chile 22,5% de la población adulta, sin diferencias por sexo. La prevalencia aumenta con la edad.

Tratamiento 1. Modificaciones en estilo de vida es el elemento central: Disminución de peso mediante dietoterapia (baja ingesta de grasas saturadas, grasas trans y colesterol, reducción de azúcares simples, y aumento de frutas, vegetales y granos enteros) y actividad física (ejercicio aeróbico moderado a intenso al menos 150 min a la semana, e idealmente, más de una hora al día). Detener tabaquismo, y consumo alcohol moderado (sólo si TG normales, en caso de estar elevados debe suprimirse el alcohol). 2. Farmacoterapia: Solo si no hay respuesta con modificación de estilo de vida. Fármacos aprobados: Orlistat, insulinosensibilizadores (metformina, tiazolidenionas), hipolipemiantes (estatinas, fibratos), antihipertensivos (IECA). 3. Cirugía: frente al fracaso del tratamiento médico o en caso que de asociarse a obesidad mórbida con IMC > 40

Etiología y Fisiopatología: La etiología es compleja, intervienen tanto factores genéticos como ambientales, que influyen sobre el tejido adiposo y la inmunidad innata. El tejido adiposo abdominal actúa como un órgano endocrino que libera el exceso de ácidos grasos libres (AGL), angiotensina II (ATII), y adipokinas. El aumento de AGL plasmático inhibe la captación de glucosa por el tejido muscular. El exceso de AGL y ATII produce un daño en el páncreas, el cual produce más insulina, pero insuficiente para contrarrestar la hiperglicemia. La ATII aumenta la presión arterial por vasoconstricción. Adipokinas provocan reacciones inflamatorias que aumentan la insulinorresistencia y la HTA. Una gran cantidad de tejido adiposo presente en los obesos resulta habitualmente en un flujo aumentado de ácidos

35 Facultad de Medicina, Universidad de Chile

u obesidad grado II (IMC > 35) y otra comorbilidad como HTA, DM o dislipidemia.

Seguimiento La finalidad del tratamiento consiste en disminuir los riesgos de enfermedad cardiovascular sobre todo la enfermedad coronaría cardíaca. Requiere control periódico de cada uno de los componentes del Sd. Metabólico y otros factores de riesgo cardiovascular.

Autor / Editor Matilde Pedrero

Año 2016

36 Facultad de Medicina, Universidad de Chile

Vasculopatía Periférica

aporte de oxígeno hacia la musculatura y los tejidos. Epidemiología y Etiología: Vasculopatía periférica en contexto de diabetes es el principal factor de riesgo de amputación de extremidades, esta presente en el 20% de la población mayor de 70 años, y aumenta el Riesgo Cardiovascular por 5 veces. Corresponde a una alteración de origen multifactorial, asociado a los Factores de Riesgo Cardiovascular clásicos (Tabaquismo, DM, Dislipidemia e HTA, además de edad, Sexo, Antecedente de IAM personal o Familiar). Implica mayor morbi-mortalidad tanto por los factores desencadenantes (edad, hábito tabáquico, hipertensión arterial, dislipemia o diabetes mellitus) como por sus posibles complicaciones (isquemia y sepsis). Su gravedad se asocia con el riesgo de infartos, evento vascular cerebral isquémico, así como muerte por causas vasculares.

Nivel de manejo del médico general: Diagnóstico Específico Tratamiento Inicial Seguimiento Completo

Aspectos Esenciales • • • •

Relacionado a factores de riesgo cardiovascular clásicos. Clínica clásica dada por dolor en zona posterior de la pierna y claudicación intermitente, que cede con el reposo, asociado a una distancia determinada. Diagnóstico se apoya en Estudio vascular no invasivo Tratamiento consiste en el de los factores de riesgo y fármacos como AAS, clopidogrel y cilostazol.

Diagnóstico Caso Clínico Tipo

Clínica: Dependiendo de la gravedad puede manifestarse desde dolor durante la actividad Física hasta el reposo. El hallazgo clave corresponde a la Claudicación Intermitente: Aparición de dolor o parestesia referida en relación a los músculos (generalmente Gastrocnemios en la zona posterior de la pierna) que se gatilla con el ejercicio y que cede con el reposo, y que se reproduce con una distancia determinada y exacta. Se distingue de la Claudicación Neurogénica en que ésta no tiene una distancia definida, el dolor es migratorio, y no cede generalmente con el reposo o lo hacen en forma más larvada. Se puede evidenciar hacia distal, disminución y/o asimetría de los pulsos Arteriales poplíteos, Tibial posterior y Pedio. En los casos más avanzados evoluciona hasta el dolor en reposo (Isquemia Crítica), generalmente de predominio nocturno, y es característico que los pacientes durante la noche duermen con la extremidad fuera de la cama (“dormir Anclado”) para ayudar con la gravedad al descenso de la sangre y evitar el dolor de isquemia. Se puede observar Rubeosis cutánea extrema (coloración rojo fuerte de la extremidad) con el declive y vasodilatación cutánea. Y en forma más crónica se pueden observar Cambios Cutáneos (pérdida de anexos cutáneos, Lesiones atróficas) y úlceras persistentes dolorosas, excepto en los diabéticos, localizadas en la zona distal de los dedos, en el lecho ungueal o sobre eminencias óseas.

Paciente de sexo masculino de 55 años, con antecedentes de hipertensión arterial, tabaquismo activo, diabetes mellitus y dislipidemia, presenta dolor en la zona posterior de la pierna al caminar 4 cuadras, que cede con el reposo y que después de ello puede seguir avanzando otras 4 cuadras más. Enfermedad arterial oclusiva.

Definición La vasculopatía periférica es una de las manifestaciones más importantes de ateroesclerosis sistémica. Corresponde a un proceso de avance crónico secundario a la inflamación arterial crónica que se presenta en enfermedades de Grandes y pequeño vaso. Existen tres entidades prevalentes a mencionar: Enfermedad arterial oclusiva de extremidades inferiores, enfermedad cerebro vascular y aneurisma de la aorta abdominal. La primera de ellas es la que causa más graves complicaciones.

Etiología-Epidemiología-Fisiopatología Fisiopatología: Los mecanismos fisiopatológicos más relevantes en la enfermedad vascular periférica comprenden la disfunción de la célula endotelial que resultan en un estrés oxidativo mantenido y el shear stress que se produce secundario a la prevalencia de Hipertensión Arterial y que se traducen en la alteración de los mecanismos vasodilatadores y la migración anormal de células al subendotelio con disminución progresiva del lumen vascular. Esto condiciona posteriormente que secundario a un evento de actividad, la demanda sanguínea requerida por la musculatura y los tejidos supera al aporte dado por la disminución del lúmen vascular, lo que condiciona isquemia y necrosis de los tejidos en forma crónica. Existe un mecanismo compensatorio que es la producción vasculatura colateral, que permite aumentar el

Según clínica se puede clasificar en Grados I a IV de Fontaine: Vasculopatía periférica0.png Dependiendo de la evaluación del dolor y de los pulsos se puede localizar inicialmente el sitio de Oclusión Vascular: Vasculopatía periférica1.png Diagnóstico Por medio del índice tibio-braquial podemos darnos cuenta del pronóstico a largo plazo del paciente, ya que un índice tibio-braquial bajo tiene una mortalidad anual del 25%. Se considera un valor normal Indice Tobillo/Brazo mayor a 0,9. Entre 0,4 a 0,9 se evidencia la claudicación intermitente, y bajo 0,4 se considera isquemia Crítica. En pacientes nefrópatas y Diabéticos pierde sensibilidad debido a la

37 Facultad de Medicina, Universidad de Chile

baja capacidad de compresión vascular. --> Otros Métodos de evaluación corresponden a la Pletismografía (evaluación de las ondas de volúmen del pulso a lo largo de las extremidades, útil en caso de calcificaciones arteriales) --> Para confirmación diagnóstica y previo a eventual resolución quirúrgica, se puede realizar Arteriografía para definir la estrategia quirúrgica. También se usan actualmente estudios imagenológicos como el AngioTC y la AngioRM.

Tratamiento Tratamiento Médico: Control de los factores de riesgo (tabaquismo, hiperlipidemia, hipertensión y diabetes mellitus) Aspirina 75-100mg y/o Clopidogrel. Permite disminuir la incidencia de eventos vasculares y la mortalidad. No mejora la sintomatología. Todos los pacientes deben tener de base este tratamiento. Tratamiento Anticoagulante: En caso de embolia diagnosticada o antecedente de eventos embólicos. Pentoxifilina: Disminuye la viscosidad sanguínea y aumenta la flexibilidad de los GR. Mejora la clínica en forma leve, siendo ésta no constante entre los usuarios. Cilostazol: Inhibidor de la Fosfodiasterasa 3 que produce vasodilatación y efecto antiagregante. Mejora la clínica en forma mayor que la pentoxifilina, pero no tiene un buen perfil de seguridad en pacientes con antecedente de Insuficiencia Cardíaca. Tratamiento Quirúrgico: Generalmente para pacientes con Clasificación Fontaine IIb, III y IV Angioplastía Tromboendarterectomía By Pass Arteriales

Seguimiento Derivar para manejo por especialista.

Autor / Editor Julián Bravo

Revisor de Formato Angel Eloy

Año 2016

38 Facultad de Medicina, Universidad de Chile

Hipertrigliceridemia Grave

Diagnóstico Triglicéridos plasmáticos en ayuna mayor a 1.000 mg/dl.

Nivel de manejo del médico general: Diagnóstico Específico Tratamiento Inicial Seguimiento Completo

Tratamiento Tratamiento inicial: Cambios en estilo de vida con alimentación saludable baja en grasas, ejercicio físico, control estricto de peso. Medidas farmacológicas: con fibratos, omega 3, acido nicotínico. En casos en que se requiera descender de forma urgente los niveles de Triglicéridos plasmáticos por riesgo de Pancreatitis aguda puede usarse Insulina regular endovenosa a razón de 0.1-0.3 UI/Kg/hora manteniendo glicemia entre 150 a 200 mg/dL Dado la magnitud del trastorno es poco probable obtener resultados óptimos a corto plazo, motivo por el cual el tratamiento definitivo y a largo plazo está a cargo del especialista.

Aspectos Esenciales • • •

Diagnóstico con valores sobre 1.000 mg/dl. Debido a disminución de actividad de LPL por mecanismo genético. Tratamiento multidisciplinario: dieta, ejercicio y fármacos.

Caso Clínico Tipo Paciente con o sin antecedentes de hipertrigliceridemia familiar, que luego de algún factor desencadenante presenta elevación de los niveles de TG. A veces la primera manifestación puede ser por las complicaciones, como la pancreatitis aguda.

Seguimiento Derivar con control entre 6 a 8 semanas posterior al inicio de tratamiento.

Definición Niveles plasmáticos de triglicéridos superiores a 1.000 mg/ dl. Autor / Editor Francisco Weber

Etiología-Epidemiología-Fisiopatología La hipertrigliceridemia grave aumenta el riesgo de pancreatitis aguda, siendo etiología de la pancreatitis aguda entre un 1,3 y un 11%. El origen de este trastorno metabólico es una disminución genética de la lipoproteinlipasa (LPL), enzima sintetizada en el tejido muscular y en los adipocitos. La LPL es transportada al endotelio capilar, donde se mantiene ligada a través de glicosaminglicanos. La LPL actúa sobre los quilomicrones ricos en triglicéridos, hidrolizándolos a quilomicrones remanentes que son transportados al hígado y ácidos grasos libres que se transportan al músculo esquelético, miocardio y adipocitos. Las mutaciones del gen de LPL dan origen a la hiperquilomicronemia (ausencia completa) y a la hipertrigliceridemia mixta (déficit parcial). Ciertos factores ambientales pueden desencadenar las manifestaciones clínicas: dieta, drogas, trastornos de metabolismo y enfermedades. Entre ellos se pueden nombrar enfermedades metábolicas (por ej. DM), alcohol, embarazo, estrógenos y sus análogos como tamoxifeno y clomifeno, betabloqueadores, tiazidas e inhibidores de proteasa usados en el tratamiento de SIDA. Estos factores provocan una disminución de LPL y de la apoproteína C-II, lo que lleva a que estas HTG primarias latentes se manifiesten.

39 Facultad de Medicina, Universidad de Chile

Año 2017

BIBLIOGRAFÍA •

https://www.uptodate.com/contents/diabetic-ketoacidosis-and-hyperosmolar-hyperglycemic-state-in-adults-clinical-features-evaluation-and-diagnosis?source=see_link



https://www.uptodate.com/contents/diabetic-ketoacidosis-and-hyperosmolar-hyperglycemic-state-in-adults-treatment?source=search_result&search=cetoacidosis%20diabetica&selectedTitle=1~150



https://www.uptodate.com/contents/hypoglycemia-in-adults-without-diabetes-mellitus-diagnostic-approach?source=searc



https://www.uptodate.com/contents/diabetic-ketoacidosis-and-hyperosmolar-hyperglycemic-state-in-adults-clinical-features-evaluation-and-diagnosis?source=see_link



https://www.uptodate.com/contents/diabetic-ketoacidosis-and-hyperosmolar-hyperglycemic-state-in-adults-treatment?source=search_result&search=cetoacidosis%20diabetica&selectedTitle=1~150



https://www.uptodate.com/contents/diabetic-ketoacidosis-and-hyperosmolar-hyperglycemic-state-in-adults-treatment?source=search_result&search=coma%20hiperglicemico%20hiperosmolar&selectedTitle=1~150



https://www.uptodate.com/contents/diabetic-ketoacidosis-and-hyperosmolar-hyperglycemic-state-in-adults-clinical-features-evaluation-and-diagnosis?source=search_result&search=coma%20hiperglicemico%20hiperosmolar&selectedTitle=3~150



https://www.uptodate.com/contents/approach-to-the-patient-with-hypertriglyceridemia?source=search_result&search=hipertrigliceridemia&selectedTitle=1~150

40 Facultad de Medicina, Universidad de Chile

Acidosis láctica

producción hepática de glucosa (neoglucogénesis), generando la acumulación de lactato, y generalmente se produce cuando existen factores predisponentes como los antes mencionados.

Nivel de manejo del médico general: Diagnóstico: Específico. Tratamiento: Inicial. Seguimiento: Completo.

También se ha encontrado asociación entre los inhibidores de la transcriptasa inversa análogos de los nucleósidos (ITIN) y la acidosis láctica, utilizados en la terapia de VIH, siendo más peligrosos en mujeres embarazadas que usen en conjunto el Zerit y Videx. Entre otros medicamentos que pueden producir Acidosis Láctica encontramos el Linezolid, describiéndose casos en los que tanto en tratamientos prolongados como al usar la primera dosis se presentaría Acidosis. También está documentado el uso de Propofol como causante.

Aspectos esenciales 

  

Es un tipo de Acidosis metabólica con anión gap aumentado, aunque también puede darse con anión gap normal en casos de hipoalbuminemia no diagnosticada. Principalmente en contexto intrahospitalario. Principales causas asociadas a hipoxia tisular. Cuidado en el uso de metformina en pacientes con factores predisponentes. Suspender uso en caso de ser necesario.

Importante tener en cuenta pacientes alcohólicos, en los cuáles el mecanismo preponderante es la falla hepática. La mayoría de los casos de acidosis láctica se debe a hipoperfusión tisular marcada en estado de shock (debido a la hipovolemia, insuficiencia cardíaca o sepsis) o durante un paro cardiorrespiratorio. Es importante destacar que en estos casos puede haber acidosis láctica tanto aeróbica como anaeróbica.

Caso clínico tipo Paciente hospitalizado con comorbilidades o factores de riesgo de presentación de acidosis láctica: Shock de cualquier etiología, uso de hipoglicemiantes orales, terapia antiretroviral. Se presenta con anorexia, fatiga, taquicardia, hipotensión arterial, compromiso de conciencia, disnea, oligoanuria, marcada vasoconstricción (Extremidades frías) e hiperventilación.

Diagnóstico Clínica: las manifestaciones más comunes son anorexia, fatiga, taquicardia, hipotensión arterial, compromiso de conciencia, disnea, e hiperventilación. Laboratorio: niveles de lactato superiores a 2 mmol/L, y pH <7,35 en plasma. La tasa de producción de H+, condiciona la gravedad de la acidosis.

Definición Tratamiento

La acidosis metabólica láctica es un trastorno severo del metabolismo intermediario, en donde los niveles de lactato plasmático y anión GAP se encuentran aumentados. Se produce cuando los niveles de lactato en plasma son mayores a 2 mmol/L y el pH sanguíneo es < 7,35.

En primera instancia se debe corregir la causa de origen, es decir, tratar la enfermedad que causó la acidosis, aumentar la perfusión tisular de oxígeno, eliminar toxinas o fármacos. La administración de bicarbonato es controversial, se debe utilizar con precaución, recomendándose su uso en acidosis severas con pH <7,1 y HCO3 < de 6 meq/l. La dosis a usar en un paciente con adecuada ventilación es de 12 meq/Kg en bolo intravenoso, controlando electrolitos plasmáticos y pH a los 30-60 minutos, volviendo a repetir el bolo si el pH se mantiene bajo 7.1.

Etiología-epidemiología-fisiopatología La acidosis láctica es la causa más común de acidosis metabólica en los pacientes hospitalizados. Se asocia con un anión gap elevado al igual que de lactato plasmático. La acidosis láctica se clasifica en dos tipos principales:

Seguimiento

Tipo A o acidosis anaerobia: Se produce por hipoxia que favorece la vía de la glicolisis anaeróbica, y con ello aumenta la formación de lactato. Esta hipoxia tisular se puede producir por menor ingreso de oxígeno a los tejidos (insuficiencia cardiaca, respiratoria o incluso anemia), y por aumento de la demanda o bloqueo de la respiración tisular (isquemia). Tipo B o acidosis aerobia: En este caso, existe una falla en la eliminación del lactato. El lactato se metaboliza en un 50 a 60% en el hígado, y es por eso que cuando existe alguna falla hepática (cirrosis, esteatosis), aumentan las probabilidades que se genere este trastorno. A este tipo, se asocia la utilización de biguanidas específicamente, la metformina. Este fármaco es ampliamente utilizado en el tratamiento de la Diabetes tipo II. La metformina disminuye la

Derivar para continuar manejo.

1

continuo temporal de dos síndromes, la Encefalopatía de Wernicke (EW) y la Demencia de Korsakoff (DK). Esta última corresponde a la condición neurológica crónica que ocurre como consecuencia de una EW, se manifiesta con déficit de memoria y psicosis confabulatoria, además de los síntomas de EW.4 La EW es un síndrome neuropsiquiátrico agudo, caracterizado por trastornos en la marcha, oculomotilidad y del estado mental.

Déficit agudo de tiamina Nivel de manejo del médico general: Diagnóstico: Específico Tratamiento: Inicial Seguimiento: Completo

Etiología y Fisiopatología

Aspectos esenciales   

Con respecto a la etiopatogenia del déficit de tiamina se ha postulado diversos mecanismos a través del cual se produce ya sea disminución de la absorción de tiamina, disminución de la ingesta o aumento de las pérdidas. De las causas de déficit agudo de tiamina destacan: anorexia nerviosa, leucemia, cáncer,3 paciente con nutrición parenteral total,5 niños alimentados con formulas en base de soya o pacientes con abuso crónico de alcohol.4

Cuadro neurosiquiátrico grave, con alta mortalidad si es que no se trata precozmente. Subdiagnóstico frecuente por clínica variada y ausencia de exámenes diagnósticos. Importancia de la prevención y tratamiento precoz.

Caso clínico tipo

Si bien la fisiopatología no se encuentra del todo dilucidada. Se postula que ciertas áreas específicas del cerebro, como la sustancia gris periacueductal, cuerpos mamilares, tálamo medial presentan un elevado metabolismo y requerimientos de tiamina, predisponiéndolas al daño por déficit. Se necesitaría un déficit de 2-3 semanas para depletar las reservas corporales, posterior a lo cual se comenzaría a evidenciar el daño orgánico.

Paciente de 60 años con antecedentes de alcoholismo. Traído al SU por cuadro de 4 días de compromiso de conciencia, vómitos y diarrea. Se precisan alteraciones cognitivas en examen neurológico. TAC de cerebro y exámenes de laboratorio básico normales.

Definición y Epidemiología

Diagnóstico

La tiamina corresponde a la vitamina B1. Es una vitamina hidrosoluble, que forma parte del complejo B. Su absorción se realiza en el intestino delgado (yeyuno e íleon) como tiamina libre y difosfato de tiamina (TDP). Su absorción se ve favorecida por vitamina C y ácido fólico, por otro lado, se disminuye con la presencia de etanol. Su composición química consta de dos estructuras cíclicas orgánicas interconectadas: un anillo pirimidina con un grupo amino y un anillo tiazol azufrado unido a la pirimidina por un puente metileno.

Basado fundamentalmente en la clínica, la cual revierte después de la administración de tiamina. La Triada básica son: trastornos de la marcha (23%), del estado mental (82%) y de la oculomotilidad (29%). También pueden presentar alteraciones conductuales como agitación y confusión. El factor predisponente abuso de OH.

La incidencia a nivel mundial del déficit de vitamina B1 en ancianos de forma crónica oscila entre 3-5% 1, debido mayormente a un bajo aporte en la dieta. Sin embargo, el déficit agudo muchas veces pasa desapercibido y es difícil poder tener una incidencia. Existe una distribución mundial variable según país, mayor asociación a pacientes consumidores crónicos de OH. En base a estudios de autopsias, la prevalencia en adultos variaría entre un 0,8 y 2,8%, elevándose hasta un 12,5% en pacientes OH. Mayor prevalencia en hombres, con distribución 1,7:1, con respecto a mujeres2. Letalidad estimada en un 17%.1,7

más

En cuanto al Laboratorio: diagnósticos de EW.

importante

No

hay

es

el

exámenes

Como apoyo diagnóstico podemos usar las Imágenes: RNM tiene baja sensibilidad (53%) pero altamente específica (93%). Donde ocurre un aumento de señal en T2 en áreas del cerebro mencionadas previamente.

Tratamiento Debe instaurarse de forma urgente, para prevenir progresión a coma o muerte. No hay evidencias sobre la dosis, frecuencia, duración y vía de administración de tiamina en forma profiláctica. De todas maneras, se recomienda su uso en pacientes alcohólicos hospitalizados, en dosis de 500 mg, tres veces al día, en infusión a pasar en 30 minutos, por 2 días, luego 250 mg endovenoso o intramuscular diariamente por 5 días más, en combinación con otras vitaminas B.1 Si no hay respuesta, se puede suspender. Si hay buena respuesta, prolongar por 3-5 días más.3

Es importante tenerla en mente dentro de las causas de un deterioro cognitiva agudo sobretodo en paciente mayores1 como en paciente expuestos de forma brusca a un estrés importante (cirugía, poli traumatizado, sepsis) o pacientes expuestos a nutrición parenteral total. La deficiencia aguda de tiamina puede ocasionar cambios patológicos en el sistema nervioso y cardiovascular3. La mayoría de los pacientes presentan distintos grados de neuritis periférica, parálisis oculares, cambios mentales y alteraciones cognitivas. Se puede desarrollar el Síndrome de Wernicke-Korsakoff. Por otro lado, otros pacientes pueden presentar alteraciones cardiovasculares, desde edema de extremidades hasta insuficiencia cardiaca. El síndrome de Wernicke-Korsakoff corresponde a la manifestación más conocida del déficit de tiamina. Es un

Seguimiento Derivación a especialista. o 2

PA ≥ 140/90 mmHg: cambios estilo de vida y tratamiento farmacológico desde el diagnóstico.

Hipertensión arterial en Diabetes tipo 2

Tratamiento farmacológico: Primera elección: IECA o ARA II por su efecto protector adicional que ejerce al bloquear el sistema reninaangiotensina, que se relaciona con el desarrollo y progresión de daño renal, de manera que retrasan la progresión de microalbuminuria a macroalbuminuria en personas con DM 2 e hipertensión.

Nivel de manejo del médico general: Diagnóstico Específico Tratamiento Inicial Seguimiento Completo

Si no se logra el objetivo de PA con IECA o ARA II, se asocia como segunda droga un diurético tiazídico en dosis baja (12,5-25 mg/día), siempre que la VFG sea ≥ 30 ml/min. En pacientes con microalbuminuria o nefropatía clínica que no logran las metas considerar el uso de antagonistas de los canales de calcio del tipo no dihidropiridínicos (diltiazem, verapamilo).

Aspectos esenciales Los diabéticos deben tener un estricto control de presión arterial. HTA y DM2 son factores de riesgo cardiovasculares, potenciando la posibilidad de desarrollar complicaciones. Generalmente requieren tratamiento asociado.

En pacientes con angina o IAM previo se debe incluir un betabloqueador.

Caso clínico tipo

Los calcioantagonista dihidropiridinicos no están recomendados porque no reducen la progresión de la nefropatía.

Paciente 55 años, sexo masculino, DM tipo 2 diagnosticada hace 5 años. En control de rutina se pesquisa PA 150/100 mm Hg, que se repite dos días después, diagnosticando HTA. Iniciar tto con IECA o ARA-II.

Seguimiento El objetivo de presión arterial es ≤ 130/80 mmHg. Si el paciente presenta proteinuria persistente, la meta es PA < 125/75 mmHg. La mayoría de los pacientes DM requiere la combinación de dos o más antihipertensivos para lograr los valores objetivos de PA. Se debe controlar la PA en cada control médico y evaluar la presencia de hipotensión ortostática por neuropatía autonómica.

Definición Paciente con diagnostico de DM 2 con presión arterial ≥ 130/80 mmHg

Etiología-epidemiología-fisiopatología Al momento del diagnóstico 30% de los pacientes DM2 tiene HTA, cifra que aumenta a 70% cuando se desarrolla nefropatía. En DM2 la hipertensión es uno de los principales factores de riesgo en el desarrollo de complicaciones macrovasculares (enfermedad coronaria y ACV) y microvasculares (retinopatía y nefropatía). El manejo óptimo y agresivo de la PA es efectivo en reducir ambas complicaciones.

Diagnóstico La definición estándar de hipertensión es una presión arterial (PA) ≥ 140/90 mmHg, pero estudios epidemiológicos y ensayos clínicos indican que una presión arterial ≤ 130/80 mm Hg confiere mejor protección macro y microvascular en pacientes con DM. Si se obtiene dicho valor en una medición debe confirmarse con otra medición durante otro día (MAPA).

Tratamiento El tratamiento dependerá del nivel de HTA: PAS 130-139 mm Hg o PAD 80-89 mmHg: terapia no farmacológica (cambios estilo de vida: reducción peso y disminución ingesta sal) por 3 meses. Si los objetivos no se logran, iniciar tratamiento farmacológico.

3

Seguimiento

Nefropatía incipiente

Controles cada 4 meses con médico, enfermera y nutricionista. Nivel de manejo del médico general: Diagnóstico Específico Tratamiento Inicial Seguimiento Completo

DM II: Exámenes al diagnóstico y luego control anual (OC, RAC). DM I: Microalbuminuria anual en pacientes con ≥ 5 años de evolución de la enfermedad. Si el índice proteinuria creatinuria es mayor o igual a 0,3 derivar a nefrología. Si es menor de 30 mg/g, realizar un nuevo control en un año.

Aspectos esenciales    

Asintomático. Screening anual desde el diagnóstico en DM 2 y desde el 5° año de Dg en DM1. RAC: 30-300 mg/g. IECA o ARA II disminuye progresión de nefropatía.

Caso clínico tipo Paciente de 58 años, IMC: 30 kg/m2, fumador de 20 cigarrillos al día, HTA, con dislipidemia mixta. Se le diagnostica diabetes mellitus 2 y en EOC proteinuria negativa, pero con microalbuminuria en RAC.

Definición Daño renal precoz en diabético objetivado con microalbuminuria estimada con relación albuminuria/creatinuria ≥ 30 mg/g y < 300 mg/g.

Etiología-epidemiología-fisiopatología El 20 a 30% de las personas con DM2 tiene daño renal al diagnóstico. La hiperglicemia crónica condiciona, a largo plazo, el desarrollo de nefropatía, retinopatía, neuropatía y cardiopatía, determinando alta morbilidad y mortalidad respecto a la población general.

Diagnóstico Examen de orina completa realizado al diagnóstico, puede evidenciar la presencia de proteinuria. Si la prueba es positiva, se debe realizar una confirmación diagnóstica con la razón proteínas/creatinina (RAC) en muestra aislada de orina matinal. Si el índice es mayor a 0,3 el paciente debe ser evaluado por un nefrólogo. Si es negativo, debe determinarse la presencia de microalbuminuria (30 a 300 mg/día de albúmina urinaria). Si es < 30 mg/d realizar otro control al año siguiente. Si está entre 30 y 300 mg/d confirma la presencia de microalbuminuria y se debe repetir la prueba en un máximo de 6 meses.

Tratamiento Iniciar tratamiento con IECA o ARA II, ya que disminuyen progresión de nefropatía, aún en pacientes normotensos Controlar otros factores de riesgo CV: suspensión de tabaco, control de PA < 120/75, control LDL < 100. 4

o formación estructural de la córnea en el caso de la Vit A.

Síndromes carenciales de vitaminas y minerales Nivel de manejo del médico general: Diagnóstico Específico Tratamiento Inicial Seguimiento Completo

Epidemiología: Los principales factores de riesgo se relacionan con Nivel socioeconómico, asociación a enfermedades orgánicas, cirugías resectivas a nivel de sistema digestivo (Resección intestinal, Gastrectomías), Patologías Autoinmunes (Gastritis Crónica Atrófica o Anemia Perniciosa), Edad, infecciones concomitantes y estados de Estrés Metabólico, Fármacos que interactúan con Vitaminas y Minerales Traza.

Aspectos esenciales

Diagnóstico

  

Déficit de vitaminas liposolubles se dan en relación a síndromes de malabsorción, colestasia u otros factores que alteren digestión y/o absorción. Déficit de vitaminas hidrosolubles tienen distintas causas, pero destaca el alcoholismo como la más importante. En nefrópatas habitualmente se observan déficit de vitaminas hidrosolubles, por baja ingesta, intolerancia digestiva, restricción dietaria y diálisis.

Clínica: Es específica con síndromes característicos para cada déficit de Vitamina o Minerales Traza. Vitaminas Liposolubles: Vitamina A: Alcoholismo, produce desde xeroftalmia a Ulceración Corneal (cuadro que produce desde pérdida de visión nocturna hasta ceguera por lesión corneal), hiperkeratosis folicular y atrofia de glándulas sebáceas y alteraciones reproductivas. Vitamina D: Nefropatía, insuficiencia hepática, ancianos. Causa raquitismo en los niños y osteomalacia y Osteoporosis en adultos.

Caso clínico tipo Paciente de sexo masculino de 53 años con antecedentes de alcoholismo, que presenta un cuadro caracterizado por confusión, nistagmus, ataxia y oftalmoplejia.

Raquitismo: Niños con crecimiento deficiente, Huesos Anchos y curvados, deformación a nivel de huesos costales, engrosamiento de articulaciones de codos, muñecas y talones, frente Amplia. A la radiografía se observan placas epifisiarias extensas. Debilidad Muscular y cuadros de Tetania.

Encefalopatía de Wernicke por déficit de tiamina (B1).

Definición

Vitamina E: Alteraciones hematológicas (Anemia Hemolítica), Polineuropatías y Escotomas Visuales.

Síndromes Clínicos secundarios a la ausencia de cantidades requeridas de Vitaminas y Minerales para la mantención de la homeostasis, sea por ingesta inadecuada, depósitos insuficientes, aumento de los requerimientos corporales o interacciones con drogas.

Vitamina K: Hipoprotrombinemia y disminución de factores dependientes de Vit K. Equimosis, Epistaxis, Hemorragias Gastrointestinales, Sangrado Uterino Anormal, Hematuria.

Etiología-epidemiología-fisiopatología

Vitaminas Hidrosolubles:

Etiología: Ingesta Inadecuada: Dieta desbalanceada o insuficiente, pérdida de vitaminas en los alimentos, enfermedades crónicas que cursan con anorexia.

Tiamina (B1): Alcoholismo. Puede causar: Beri Beri Húmedo (Déficit Crónico): Edema de EEII, cara y escroto, disnea, taquicardia y cardiomegalia con falla cardiocirculatoria.

Depósitos insuficientes: Malabsorción: Esteatorrea (Vit A, D, K); Fístulas (Déficit Biotina), Síndrome Intestino Corto (B9, B12), Gastrectomía (B12), Desnutrición Proteica.

Beri Beri Seco (Déficit Crónico): Parestesias EEII, Marcha Atáxica, Parálisis de EEII, Hipotonía Muscular.

Aumento de los Requerimientos: Fase de recuperación de pacientes con desnutrición severa, estrés secundario a infección o trauma, Embarazo y lactancia.

Encefalopatía Wernicke-Korsakoff (Déficit Agudo): Alteración de conciencia, nistagmus, oftalmoplejia y ataxia. Amnesia, fabulación y pérdida de memoria de inicio reciente.

Interacción con Drogas: Fármacos que reducen la biodisponibilidad. Enfermedades Crónicas.

Riboflavina (B2): Se presenta como dermatitis seborreica, queilosis, estomatitis angular, glositis, Vasculización Corneal.

Fisiopatología: Dependiendo del elemento traza o Vitamina ausente, se producen alteraciones específicas en las cadenas metabólicas de los procesos celulares involucrados en la mantención de los tejidos, ya que la mayor parte de éstos funcionan como catalizadores o son requeridos como enzimas para la síntesis de proteínas específicas. Ejemplo Vit B12 para la formación de Mielina

Niacina (B3): Alcoholismo. Causa pelagra: "las 3D" Dermatitis pigmentada, agrietada simulando quemaduras de sol, Depresión, apatía, irritabilidad, confusión y demencia, y dolor abdominal, vómitos, y Diarrea con alteración de mucosa a nivel oral e intestinal.

5

Piridoxina (B6): Raro, provoca también sintomatología inespecífica: Dermatitis y glositis y estomatitis. Crisis Convulsivas, Anemia Normo o Microcitica y disminución de Linfocitos. Ácido fólico (B9): Produce anemia megalobástica, al igual que el déficit de Cobalamina (B12), pero esta última se acompaña de signos neurológicos. Vitamina C: Escorbuto: Gingivitis, Mala Cicatrización, Síndromes Hemorragíparo, Fracturas Óseas. Elementos Traza: Hierro: Anemia Ferropriva. Calcio: Osteomalacia, Osteoporosis. Fósforo: Alteraciones de Conciencia, Miopatía. Magnesio: Alteraciones Neuromusculares, Hipocalcemia, Hipokalemia. Zinc: Dermatitis, Inmunosupresión.

Alteración

del

Crecimiento,

Yodo: Bocio. Diagnóstico es de carácter clínico complementado con niveles de los elementos a evaluar dependiendo del déficit estudiado. En casos particulares se pueden hacer test más específicos tales como el Test de Schilling para evaluar a qué nivel ocurre el déficit de Vitamina B12.

Tratamiento El de la patología de base y suplemento de los elementos deficitarios, especialmente en alcoholismo, enfermedades hepáticas, ancianos, enfermedades renales, entre otras.

Seguimiento No requiere seguimiento controlado el déficit.

6

Trastornos de conducta alimentaria

Factores Desencadenantes:  

Nivel de manejo del médico general: Diagnóstico: Específico. Tratamiento: Inicial. Seguimiento: Derivar.

  

Pérdidas cercanas (muerte de cercanos, separaciones) Fracaso de relación de pareja o experiencia sexual, abuso sexual Comentarios peyorativos sobre su cuerpo. Enfermedades físicas, consuntivas Dietas Restrictivas

Factores Perpetuantes:

Aspectos esenciales 



   

    

Los trastornos de la conducta alimentaria se definen como una alteración persistente del comer que perjudica la salud o el funcionamiento psicosocial. Los más frecuentes son: Anorexia nerviosa, bulimia nerviosa y trastorno por atracones. Nuevos trastornos se han incluido en el DSM V, pero existe poca o nula información sobre su prevalencia y curso. Afecta principalmente a mujeres adolescentes y adultas-jóvenes. Su incidencia va en aumento en el mundo. Requiere un manejo multidisciplinario. Puede requerir de hospitalización.

Emaciación Alteraciones fisiológicas intestinales Distorsión de Imagen Corporal Estructura de Personalidad Interacción Familiar Patogénica, sociocultural.

Presión

Trastornos Psiquiátricos Asociados:  

Anorexia Nerviosa: 50-75% Distimia o Trastorno de Depresión Mayor, 10-13% TOC, 5-20% Suicidio Bulimia: Trastorno de abuso y dependencia de sustancias 45-50%, Trastorno de Ansiedad 43%, Trastorno del Ánimo Bipolar 12%

Diagnóstico

Caso clínico tipo

El diagnóstico es clínico, en la entrevista se debe evaluar lo conductual, pero también es importante el examen físico del paciente para observar complicaciones médicas asociadas. Se debe realizar un buen examen psiquiátrico para detectar comorbilidades. Suele haber problemáticas familiares complejas, que también hay que explorar.

A su consulta se presenta una madre con su hija de 16 años, ya que la madre está preocupada porque su hija no se alimenta bien, ha bajado de peso, y la ha notado menos comunicativa, lo cual es negado por la paciente. Al examen físico se constata una paciente desnutrida, aletargada, deprimida, y con erosión del esmalte dentario.

Criterios diagnósticos DSM V:

Definición - Anorexia nerviosa. Grupo de trastornos psiquiátricos caracterizados por una conducta alterada ante la ingesta alimentaria, o la aparición de comportamientos de control de peso que llevan como consecuencia problemas físicos o del funcionamiento psicosocial del individuo.

A. Restricción de la ingesta de energía que conduce a un bajo peso corporal, dada la edad del paciente, el sexo, la etapa del desarrollo y la salud física, con un marcado rechazo a mantener el peso corporal normal o sobre el mínimo normal (IMC <18,5). B. Miedo intenso a ganar peso o engordar o comportamiento persistente que impide el aumento de peso, a pesar de tener bajo peso.

Etiología-epidemiología-fisiopatología Epidemiología: Bulimia: 1-3% Población general, Trastorno por Atracones 2% población general, Anorexia Nerviosa 0,5-1% mujeres adolescentes. 90-95% corresponden al sexo femenino, siendo la edad de inicio más frecuente a los 15 años.

C. Percepción distorsionada del peso y la forma corporal, influencia indebida del peso y la forma corporal en la autoestima, o la negación de la gravedad médica del propio peso corporal bajo.

Factores Predisponentes:  

    

Especificar el tipo: restrictivo o compulsivo-purgativo. Especificar si está en remisión parcial o total, especificar gravedad (Leve IMC < o = 17, Moderado: 1616.99, Severa: 15-15.99, Extrema < 15).

Alto nivel intelectual Rasgos de personalidad dependientes (necesidad de aprobación, complaciente) y Obsesivos (Perfeccionismo, rigidez) en el caso de la Anorexia Nerviosa. Estructura de personalidad límite en Bulimia Medio Familiar Patológico y con alta autocrítica Pensamiento Dicotómico Autoestima Pobre, Alexitimia Sobrepeso Pre-mórbido (25% pacientes)

- Bulimia Nerviosa. A. Episodios de atracones (consumo de una cantidad de alimentos en un periodo discreto de tiempo que es mucho más grande que lo que la mayoría de las personas comería en las mismas circunstancias) en que los 7

pacientes sienten que no pueden controlar su ingesta durante el episodio.

Tratamiento 

B. Conductas compensatorias para evitar la ganancia de peso (vómitos autoinducidos, mal uso de laxantes, diuréticos u otros fármacos, ayuno o ejercicio excesivo).



C. A y B al menos una vez a la semana durante tres meses. D. Autoevaluación del paciente es indebidamente influenciada por la forma y el peso corporal.



E. La alteración no aparece exclusivamente durante episodios de anorexia nerviosa.



Especificar: remisión parcial o total; si es leve (1-3 conductas compensatorias por semana), moderado (4-7), grave (8-13), extremo (14 o más).



- Trastorno por atracones.



Dependiendo de la severidad de las complicaciones médicas coexistentes (y en casos de alto índice de suicidio) se puede Hospitalizar para manejo en sala. Se debe realizar evaluación de Examen Físico por Nutricionista y Laboratorio Completo (Hemograma, VHS, Perfil Bioquímico, Creatinina, Perfil Lipídico, Pruebas Tiroídeas y Hepáticas, ELP, Densitometría Ósea y ECG). Abordaje multidisciplinario incluyendo nutricionista, Psiquiatra, Médico Internista, Psicólogo y Terapeuta Familiar. Se pueden utilizar modelos de Psicoterapia de tipo Cognitivo Conductual y Terapia familiar para abordar los desencadenantes y perpetuantes. Reeducación Nutricional y Negociación con la Paciente. Farmacología Complementaria: ISRS, Antipsicóticos y Ansiolíticos. Manejo esencial es la PSICOTERAPIA.

A. Los episodios de atracones (consumo de una cantidad de alimentos en un periodo discreto de tiempo que es mucho más grande que lo que la mayoría de las personas comería en las mismas circunstancias), durante los cuales el paciente siente que no tiene control sobre la alimentación.

Pronóstico: 40-50% buena Evolución, 60% mala evolución con cronificación de anorexia. 25-30% remitirán sin tratamiento en 1-2 años. Considerar en Anorexia Nerviosa tiene alto índice de suicidio.

B. Episodios de atracones compulsivos se caracterizan por al menos tres de los siguientes:

Seguimiento Derivar para tratamiento con Psiquiatra. Seguimiento de caso en COSAM posterior a su alta.

1. Comer más rápido de lo normal. 2. Comer hasta sentirse incómodamente lleno. 3. Comer grandes cantidades de alimentos cuando no se siente físicamente hambriento. 4. Comer solo debido a la vergüenza que genera la cantidad de alimentos que consume. 5. Sentirse disgustado consigo mismo, deprimido o culpable después de comer en exceso. C. Los episodios ocurren al menos una vez por semana durante tres meses. D. Sin uso regular de conductas compensatorias. E. Los atracones de comida no ocurren exclusivamente en el transcurso de bulimia nerviosa o anorexia nerviosa. Especificar gravedad: Leve: 1-3 atracones por semana, Moderado: 4-7, Grave: 8-13, Extremo: 14 o más. Para el resto de los trastornos revisar los criterios diagnósticos del DSM V, ya que son menos frecuentes y de mayor manejo por especialista. Al examen Físico: Evaluar sin ropa, piel seca, amarillenta, presencia de lanugo. Erosión del esmalte dental, Hipertrofia parotídea y signos carenciales específicos de cada déficit nutricional. Complicaciones Médicas asociadas: Esofagitis, RGE, Sd. Mallory Weiss, Megacolon y esteatorrea. Deshidratación, Hipokalemia, alcalosis metabólica, neumonía aspirativa, Crisis convulsivas, déficits carenciales específicas. Leucopenia, Anemia y trombocitopenia. Disfunción tiroidea.

8

MÓDULO 1: Medicina Interna

Endocrinología Bocio Nivel de manejo del médico general: Diagnóstico Específico Tratamiento Inicial Seguimiento Derivar

I Bocio palpable y visible con cuello en hiperextensión II Bocio palpable y visible con el cuello en posición normal III Bocio visible a larga distancia

Aspectos Esenciales

Etiología-Epidemiología-Fisiopatología

• • • •

El bocio es la patología tiroidea de mayor prevalencia en el mundo. Hablamos de bocio endémico cuando su prevalencia en la población de 6 a 12 años es > 10%, y de bocio esporádico cuando es ≤ 10%. Se presenta con mayor frecuencia en el sexo femenino.

Bocio es el aumento del tamaño de la tiroides. Patología tiroidea de mayor prevalencia en el mundo. Niveles de TSH es la prueba de laboratorio inicial. Alternativas de tratamiento son la supresión con tiroxina, cirugía y el radioyodo.

La causa más frecuente a nivel mundial el producido por deficiencia de yodo. Sin embargo, en países con complementación de yodo en la sal y otros productos alimentarios, como Chile, las principales causas son el Bocio multinodular, la tiroiditis crónica autoinmune (Tiroiditis de Hashimoto) y la Enfermedad de Graves, existiendo otras causas menos frecuentes, como otras tiroiditis, enfermedades infiltrativas, o tumores.

Caso Clínico Tipo A una mujer de 25 años, asintomática se le pesquisa un bocio difuso II de la clasificación OMS en un examen físico de rutina.

Definición

En su desarrollo participan factores tanto genéticos como ambientales. La causa más frecuente de bocio endémico es el déficit de yodo dietario, mientras que el bocio esporádico es generalmente idiopático. Su desarrollo es debido fundamentalmente a un aumento en la replicación de las células foliculares tiroideas, cuya etiopatogenia permanece aún sin esclarecer, sin embargo se considera a la TSH como el principal factor etiopatogénico. En la historia natural del bocio existe una tendencia natural al crecimiento, la multinodularidad y al desarrollo de autonomía funcional, pudiendo desarrollarse con el tiempo un hipertiroidismo subclínico o incluso clínico (bocio multinodular tóxico)

Se denomina bocio al aumento de tamaño de la glándula tiroides, cuyo peso normal oscila entre 12 y 20 g. Este aumento de volumen puede asociarse a una función tiroidea normal (bocio no tóxico o simple), o bien, asociarse a una hiper o hipofunción tiroidea, y, en base a sus características morfológicas, se puede clasificar según su extensión en simétrico o asimétrico; según su estructura en difuso, uninodular o multinodular; o según su tamaño, según los siguientes estadios (OMS): Estadío Caracteristica 0A No hay bocio 0B Detectable sólo por palpación, pero no visible con el cuello en hiperextensión

1 Facultad de Medicina, Universidad de Chile

Diagnóstico



La evaluación clínica debe ir dirigida a determinar la causa subyacente, la existencia de síntomas de disfunción tiroídea y signos que sugieran sintomatología compresiva (disnea o estridor debido a compresión traqueal, voz ronca o bitonal por parálisis de cuerdas vocales secundaria compresión de nervio recurrente, disfagia, o signos de compresión de vasos subclavios, yugulares o vena cava superior) o tumoral (rápido crecimiento, consistencia dura en un bocio nodular o adenopatías). La exploración física debe valorar el tamaño de la glándula a través de la palpación y también evaluar la presencia de nódulos y adenopatías. Sin embargo, se debe recordar que la mayoría de los pacientes con bocio son asintomáticos.

sencia o sospecha de malignidad a la PAAF. Radioyodo (131I): Se considera una alternativa terapéutica a la cirugía en caso de Bocio no tóxico, ancianos, pacientes con alto riesgo quirúrgico o rechazo del procedimiento y bocios que recurren tras la tiroidectomía. Con ello se consiguen importantes reducciones en el volumen tiroideo y mejoría de la sintomatología compresiva.

Seguimiento Debido a que la mayoría de los bocios son asintomáticos y crecen lentamente, nunca ocasionan ningún problema, por lo que no precisan tratamiento alguno. Sin embargo, debe realizarse monitorización periódica en busca de disfunción tiroidea, crecimiento continuo o sintomatología obstructiva. En estos pacientes se sugiere evaluar con examen físico y TSH anualmente. El seguimiento completo es llevado a cabo por especialista, por lo que en caso de pesquisar la presencia de bocio se indica su derivación.

En la evaluación funcional, la medición de TSH es la prueba de laboratorio inicial necesaria para la valoración de todo bocio, la que deberá complementarse con T4L o T4L y T3 en el caso de encontrarse elevada o suprimida, respectivamente. La indicación de realizar ecografía tiroidea a todo paciente con bocio es controversial, sin embargo, se sugiere en caso de bocio no tóxico no relacionado a tiroiditis de Hashimoto o deficiencia de yodo, en vías de establecer su tamaño, morfología, el número y características individuales de los nódulos presentes y existencia de adenopatías. El estudio con otras técnicas imagenológicas o Punción por aspiración con aguja fina (PAAF) no está indicado en la evaluación inicial. Del mismo modo, a menos que exista la sospecha fundada de déficit de yodo como etiología, la evaluación de la ioduria no está indicada.

Autor / Editor Paulina Lira

Tratamiento La finalidad de la terapia es corregir la disfunción tiroidea subyacente, si está presente, y disminuir el tamaño del bocio, en el caso de que éste vaya en crecimiento o produzca síntomas obstructivos. Para ello existen diversos métodos de tratamiento, cada uno con ventajas y desventajas, no habiendo consenso sobre el ideal tratamiento en el Bocio no tóxico. Por ello, la indicación de terapia debe ser llevada a cabo por el especialista. Las principales opciones son las siguientes: • Terapia supresora con hormona tiroidea. Actualmente controvertida. Se realiza con el objeto de suprimir la secreción de TSH y así conseguir una reducción del tamaño del bocio o al menos evitar el futuro crecimiento del mismo. Cuando es usada debe ser mantenida por largo tiempo, y cualquier reducción del tamaño del bocio se pierde con su discontinuación. Se debe utilizar la menor dosis de Levotiroxina posible, no estando bien definidos los niveles óptimos de supresión de TSH. Si el bocio disminuye de tamaño o permanece estable, el tratamiento se deberá mantener indefinidamente. • Tiroidectomía parcial o total: Es preferida en pacientes con síntomas compresivos, bocios cuyo crecimiento se mantenga a pesar del tratamiento médico o en quienes signifique un problema estético o este método sea de su preferencia, estando indicada además, en caso de pre-

2 Facultad de Medicina, Universidad de Chile

Año 2016

Hipotiroidismo

determinación del nivel de TSH en sangre obtenida del talón de los recién nacidos. El nivel de TSH, por definición está siempre elevado en el hipotiroidismo primario.

Nivel de manejo del médico general: Diagnóstico Específico Tratamiento Inicial Seguimiento Completo

Diagnóstico Clínico. Anamnesis completa y exhaustiva. Las manifestaciones clínicas dependen de la edad de inicio, causa y magnitud del cuadro. Los síntomas habituales incluyen ánimo bajo, letargo, alteraciones en la memoria y concentración, intolerancia al frío, estreñimiento y aumento de peso leve, sequedad cutanea, calvicie, disfonía, calambres musculares y menorragia.

Aspectos Esenciales • • • • •

Disminución o ausencia en la produccion de Hormonas Tiroideas. Se clasifica en Hipotiroidismo Congénito o Adquirido. Resulta fundamental su detección precoz. La confirmación es con medición de TSH y T4 libre. Programa de Screening nacional con medición de TSH neonatal

Examen Físico completo y exhaustivo: constipación, letargo, intolerancia al frío, piel seca, extremidades frías, bradicardia, hipertensión diastólica leve, prolongación de los reflejos osteotendineos; en etapas avanzadas mixedema.

Caso Clínico Tipo

Se debe sospechar en todo niño con disminución de la velocidad de crecimiento estatural, con ganancia anormal de peso, retardo de edad ósea, con o sin síntomas de hipofunción tiroidea. Si el hipotiroidismo es de larga evolución produce retraso de talla desproporcionado y/o retardo mental. El RN puede ser asintomático o presentarse con ictericia neonatal prolongada, edema de párpados piel y manos, GEG, hipotermia, fontanelas amplias.

Niño de 3 años de edad con detención en el crecimiento, sin antecedentes patológicos, con constipación crónica y piel seca. La madre refiere tener Hipotiroidismo desde hace muchos años, y que a su hijo le hicieron el control de TSH cuando nació y era normal.

Definición Disminución o ausencia en la producción de hormonas tiroideas.

Examen Complementario: se confirma midiendo T4 libre y TSH. Presencia de anticuerpos antimicrosomales o antitiroglobulina, orienta a tiroiditis autoinmune (Hashimoto). Ecotomografía de Tiroides, TAC y RMN.

Etiología-Epidemiología-Fisiopatología

Tratamiento

En áreas de suministro suficiente de yodo, la etiología más frecuente es la de origen autoinmune. La mayor incidencia se da a la edad de 60 año, y la prevalencia aumenta con la edad. Se pueden clasificar en Congénito o Adquirido y a su vez según la localización en primarios, por falla en la glándula tiroides, y secundarios o terciarios por falla a nivel hipofisiario o hipotalámico respectivamente. La causa más frecuente de hipotiroidismo adquirido es la tiroiditis crónica autoinmune, más frecuente en mujeres, en el 60% existen antecedentes familiares de patología tiroidea. Puede asociarse con otras enfermedades autoinmunes (DM-1, vitíligo, etc). El hipotiroidismo subclínico cursa con niveles dentro de rangos adecuados de T4L asociado a leve elevación de TSH; habitualmente es asintomático, si bien en algunos casos puede cursar con síntomas leves. El hipotiroidismo transitorio puede originarse por tiroiditis subaguda o asintomática (como la tiroiditis post parto).

El tratamiento es Levotiroxina en dosis de 100 ug/m2/día, debiendo ajustarse periódicamente de acuerdo a la respuesta clínica y bioquímica de TSH. En los lactantes se evaluará cada 3 ó 4 meses y en el niño mayor cada 6 meses.

Seguimiento Control y seguimiento a nivel secundario por especialista.

Autor / Editor Catalina Luvecce

Es una de las enfermedades endocrinas más frecuentes en pediatría, de gran relevancia por su impacto en el crecimiento y desarrollo. El cribado sistemático neonatal se basa en la

3 Facultad de Medicina, Universidad de Chile

Año 2016

Hipertiroidismo

Definición Estado clínico provocado por el exceso de hormonas tiroideas circulantes, y la exposición de los tejidos a éstas. Puede ser por aumento de producción o de circulación de hormonas tiroideas.

Nivel de manejo del médico general: Diagnóstico Específico Tratamiento Inicial Seguimiento Completo

Etiología-Epidemiología-Fisiopatología

Aspectos Esenciales • • •

Los niveles circulantes elevados de hormonas tiroideas pueden ser debidos a: • incremento en la síntesis y secreción a nivel de la glándula tiroides, • aumento en la liberación por destrucción de la glándula tiroides, • causas iatrogénicas, • alteración de los tejidos diana, • producción de hormonas tiroideas a nivel extratiroideo.

Hiperfunción de glándula tiroides. Enfermedad de Basedow-Graves es causa más frecuente. Valores elevados de T4 libre y suprimidos o bajos de TSH.

Caso Clínico Tipo Paciente mujer de 42 años. Consulta por “nerviosismo”, taquicardia e insomnio progresivo en los últimos meses. Al interrogatorio dirigido, señala aumento de apetito con baja de peso, polidefecación, palpitaciones, intolerancia al calor. Al examen físico, se aprecia piel húmeda y caliente, temblor fino y exoftalmo. Se solicitan hormonas tiroídeas, que muestran T4L elevada y TSH disminuida.

El hipertiroidismo es menos frecuente que el hipotiroidismo, con prevalencia entre 0,5 y 1%, siendo aproximadamente 5 - 10 veces más frecuente en mujeres. Su mayor incidencia se da entre los 20 a 50 años.El hipertiroidismo subclínico puede llegar al 4,6%, especialmente en población geriátrica. La enfermedad de Graves es la causa más frecuente de hipertiroidismo (60-80% de los casos), seguidas del bocio tóxico multinodular y adenoma tóxico.

Causas de Tirotoxicosis

4 Facultad de Medicina, Universidad de Chile

Diagnóstico

Tratamiento

En la clínica, las principales repercusiones son a nivel metabólico y circulatorio, siendo los síntomas más habituales la astenia o cansancio, fatigabilidad y debilidad muscular, el nerviosismo, ansiedad, labilidad emocional, la pérdida de peso con apetito conservado o aumentado, las palpitaciones, temblor y la intolerancia al calor. Los hallazgos más característicos en la exploración física son la taquicardia, la piel caliente y sudorosa, el temblor muscular fino y la retracción palpebral. La presencia de bocio u otra sintomatología específica dependerá de la etiología.

En la actualidad existen tres modalidades de tratamiento para la hiperfunción de la glándula tiroides: fármacos antitiroideos, yodo radioactivo (I131) y cirugía de tiroides. Para el control de los síntomas se utilizan betabloqueadores al momento de diagnosticar, se usa atenolol 25-50 mg/día. Se recomienda el uso de metimazol como antitiroideo por su gran eficacia y duración de acción, además de la baja tasa de efectos adversos. El propiltiouracilo es preferido su uso en el primer trimestre del embarazo por el potencial efecto teratogénico del metimazol. La elección de la modalidad terapéutica es de resorte del especialista y viene condicionada por la causa de la hiperfunción, por los medios disponibles y por las preferencias del paciente.

En el adulto mayor los signos habitualmente descritos pueden no ser evidentes, siendo los principales síntomas la pérdida de peso y el cansancio (“tirotoxicosis apática”). Son signos propios de enfermedad de Graves: oftalmopatía (exoftalmos, quemosis, oftalmoplejia), mixedema pretibial (muy infrecuente) y acropaquia tiroidea. Si bien la oftalmopatía puede presentarse en forma aislada y sin asociación a tirotoxicosis.

Seguimiento Derivar a especialista.

En el caso presentar síntomas de hipertiroidismo durante el embarazo, será necesario solicitar medición de TSH, T4L y eventualmente T3. Es necesario recordar que durante el embarazo, tanto por aumento de la concentración de TBG, aumento de T4 y T4L y el efecto estimulante de la βHCG (debido a su particular semejanza con TSH) es normal encontrar niveles de TSH bajo los valores usuales. Por ello, se han establecido niveles específicos para cada trimestre (1er trimestre 0,1 a 2,5; 2do trimestre 0,2 a 3,0; 3er trimestre 0,3 a 3,0), los que se pueden utilizar en caso de que el laboratorio que realice el test no tenga niveles predeterminados para utilizar en embarazadas. Las dos principales etiologías son la enfermedad de Graves y el hipertiroidismo gestacional, inducido por la acción comentada de la βHCG, requiriendo tratamiento sólo en el primer caso y sólo cuando se presenta en forma moderada o severa.

Autor / Editor Matilde Pedrero

En exámenes de laboratorio, la determinación de los niveles de T4L y TSH suelen ser suficientes para llegar al diagnóstico de tirotoxicosis. Encontraremos valores elevados de T4L junto con niveles suprimidos de TSH. En el caso de hipertiroidismo subclínico se encontrarán niveles de TSH bajo lo normal pero T4L, T3 y T3L normales. En el caso de encontrar TSH y T4L disminuidas, con presencia de síntomas de hipertiroidismo, es necesario profundizar el estudio con la medición de T3, en busca de tirotoxicosis por T3, la que puede deberse a ingestión subrepticia de hormonas, consciente o inconscientemente. La utilización de otros métodos de estudio, como gamagrafía tiroidea o captación de iodo radioactivo pueden ser útiles en la búsqueda de la etiología del hipertiroidismo. En el caso de la enfermedad de Graves, la gammagrafía muestra un bocio difuso y la captación aparece elevada. En caso de encontrarse un nódulo frío durante este primer estudio se debe realizar PAAF de éste, debido al mayor riesgo de malignidad.

5 Facultad de Medicina, Universidad de Chile

Año 2016

Síndrome De Cushing

de la corteza suprarrenal, que causa el exceso de cortisol y de este modo suprime la secreción tanto de CRH como de ACTH.

Nivel de manejo del médico general: Diagnóstico Específico Tratamiento Inicial Seguimiento Completo

En el primer caso, la causa más frecuente (65 a 70% de los pacientes) es la enfermedad de Cushing (por la hiperproducción a nivel hipofisiario de ACTH, frecuentemente por microadenomas hipofisiarios), le siguen en frecuencia el síndrome de secreción ectópica de ACTH (10 a 15%, por carcinoma pulmonar de células pequeñas, tumor carcinoide, tumor de islotes pancreáticos, timoma, etc.) y luego el síndrome de secreción ectópica de CRH (también de origen tumoral <1%). En el segundo caso, la causa más frecuente son tumores de la corteza suprarrenal (15%) y luego la hiperplasia bilateral macro o micronodular (<1% cada una).

Aspectos Esenciales •

• •



Importante conocer el fenómeno de pseudo Cushing en que los pacientes comparten fenotipo similar y algunas alteraciones de laboratorio y que se ve en alcoholismo, obesidad, depresión. Diagnóstico es clínico. Los síntomas que son más sugerentes de la presencia de hipercortisolismo incluyen depósito de grasa supraclavicular, atrofia cutánea, estrías amplias violáceas y debilidad muscular proximal. Para el diagnóstico etiológico se mide ACTH.

Diagnóstico El diagnóstico etiológico es complejo, por lo que tras la sospecha del síndrome se debe realizar la derivación a especialista. Las manifestaciones clínicas son variadas, pudiendo estar todas o sólo algunas presentes, lo que depende del grado y duración del hipercortisolismo y la etiología. Los más comunes son la obesidad (característicamente de distribución centrípeta, afectando la facies, cuello y tronco, produciendo redondeamiento de la cara (“de luna llena”), depósito de grasa a nivel dorso-cervical y en fosas supraclaviculares), trastornos menstruales (oligoamenorrea o amenorrea), signos de hiperandrogenismo cutaneo (hirsutismo, acné, etc.), trastornos neuropsicológicos, hipertensión arterial, debilidad y atrofia muscular proximal, estrías rojo-vinosas, atrofia de la piel, equimosis y hematomas, hiperpigmentación, disminución de la densidad mineral ósea, intolerancia a la glucosa o diabetes mellitus, hipertrigliceridemia, linfopenia, aneosinofilia, hipokalemia, infecciones, eventos tromboembólicos, etc.

Caso Clínico Tipo Mujer de 50 años que presenta cara redonda, obesidad centrípeta, estrías violáceas e hipertensión. Los exámenes de laboratorio muestran: hiperglicemia, hipertrigliceridemia, linfopenia e hipokalemia.

Definición El síndrome de Cushing corresponde al cuadro clínico derivado de la exposición crónica y excesiva a glucocorticoides. Establecer el diagnóstico suele ser difícil porque ninguno de los síntomas o signos son patognomónicos del síndrome. Hay un gran espectro de manifestaciones subclínicas, dependiendo de la duración y la intensidad del exceso de producción de esteroides.

Es importante descartar la ingestión de prednisona u otro corticoesteroide, el cual puede ser la causa del síndrome. Para la confirmación del diagnóstico, se recomienda que a lo menos dos de los siguientes exámenes resulten anormales: Cortisol libre urinario (CLU). La excreción de cortisol urinario de 24 horas es una forma directa y confiable de medición de la secreción de cortisol. Determina la concentración de cortisol no unido a proteínas. El hallazgo de cortisol aumentado 2 o más veces sobre el límite superior del rango normal es categórico de síndrome de Cushing. Es necesario medir creatinina en la misma muestra para asegurarse que la recolección de orina fue completa.

Etiología-Epidemiología-Fisiopatología La causa más frecuente es el síndrome de Cushing exógeno o iatrogénico, por ingestión excesiva de glucocorticoides sintéticos, generalmente por indicación médica. El síndrome de Cushing endógeno es una enfermedad muy rara, con una incidencia aproximada de 2 a 4 casos por millón de habitantes. Clásicamente se ha hecho la separación entre dos categorías:

Test de supresión a dosis bajas de dexametasona. Mediante la prueba de supresión con 1 mg (“test de Nugent”, que consiste en la administración a las 11 a 12 pm y medición de cortisol sérico a las 8 am del día siguiente), el valor de corte es <1,8 μg/dL.

síndrome Cushing ACTH dependiente, que cursa con niveles inapropiadamente altos de ACTH, estimulando a la corteza adrenal a una producción excesiva de cortisol, y, síndrome de Cushing ACTH independiente, en el cual la producción de cortisol se debe a un funcionamiento anormal

Cortisol salival a las 11 pm (al menos dos veces): Método simple y confiable que mide el cortisol libre, permitiendo la

6 Facultad de Medicina, Universidad de Chile

evaluación sin requerir hospitalización. Es especialmente útil en pacientes con sospecha de síndrome de Cushing cíclico. Los valores de referencia deben interpretarse según las indicaciones del laboratorio.

Autor / Editor Diego Cáceres

Excepcionalmente, cuando hay discordancia entre las otras pruebas, puede realizarse la prueba con 2 mg de dexametasona en 2 días o test de Liddle (administración de 0,5 mg cada 6 horas en un total de 8 dosis y medición del cortisol sérico 2 a 6 horas después de la última dosis). Debe tenerse la seguridad que el paciente ingirió todas las dosis indicadas. Para la interpretación de los exámenes es importante considerar: factores de estrés, factores que aumentan la proteína transportadora (CBG), factores que aumentan el metabolismo de la Dexametasona, volumen urinario (>4L o <500 ml) entre otros, que pueden alterar los resultados de las pruebas. Para el diagnóstico etiológico se realiza la medición de ACTH sérica. Posterior a ello, el especialista indicará la realización de otros exámenes orientados a determinar la causa última del cuadro clínico.

Tratamiento El objetivo del tratamiento de todos los pacientes con síndrome de Cushing es lograr la normalización de la función hipotálamo-hipófisis-suprarrenal y posterior reversión de los signos y síntomas cushingoides y comorbilidades. Generalmente quirúrgico. La mayor parte de las veces se trata de microadenomas hiposifiarios, por lo que se realiza resección transesfenoidal. En el caso de tumores suprarrenales benignos, el tratamiento también es quirúrgico. En el caso de carcinomas además puede requerirse terapia coadyuvante con drogas antiglucocorticoideas y quimioterápicos. En los pacientes con enfermedad de Cushing que no fueron curados con cirugía pituitaria, la terapia médica dirigida al tumor corticotropo como cabergolina o pasireotida puede resultar en la normalización del cortisol libre en orina de 24 horas, especialmente si tienen sólo hipercortisolismo leve. La irradiación de la hipófisis es otro tratamiento de segunda línea para la enfermedad persistente o recurrente de Cushing. Inhibidores de la enzima suprarrenales se deben utilizar para controlar el hipercortisolismo hasta que sea efectiva. Los síntomas físicos y signos del síndrome de Cushing se resuelven gradualmente durante un período de 2 a 12 meses después de la curación efectiva de síndrome de Cushing. La hipertensión, la osteoporosis y la intolerancia a la glucosa mejoran pero pueden no desaparecer.

Seguimiento El seguimiento y estudio completo es llevado a cabo por especialista, por lo que en caso de sospecha de síndrome de Cushing está indicada la derivación.

7 Facultad de Medicina, Universidad de Chile

Año 2017

Insuficiencia Suprarrenal Crónica

Diagnóstico La presentación clínica de la insuficiencia suprarrenal depende de si ésta se presenta en forma aguda o crónica. Esta última, cuando su etiología es primaria, se caracteriza por astenia, debilidad o fatiga fácil como principal síntoma, a lo que se suma de forma insidiosa hiperpigmentación cutánea, pérdida de peso, hipotensión, alteraciones de la función gastrointestinal (diarrea, náuseas, vómitos, anorexia, dolor abdominal, etc.), avidez por la sal, hipotensión ortostática. En los casos autoinmunes puede encontrarse vitíligo. A nivel bioquímico, es frecuente encontrar hiponatremia (presente en 85 a 90% de los pacientes), hiperkalemia (aproximadamente en 60 a 65%), aumento del nitrógeno ureico, hipoglicemia. Las manifestaciones clínicas de la Insuficiencia suprarrenal crónica secundaria o terciaria son similares a las de causa primaria, recién expuestas, con algunas excepciones, como ausencia de hiperpigmentación, ausencia de deshidratación e hipotensión menos prominente, ausencia de hipokalemia (debido a la preservación de la función de la aldosterona), menor frecuencia de signos gastrointestinales, mayor frecuencia de hipoglicemia y posibles síntomas secundarios a la existencia de un tumor a nivel hipofisiario o hipotalámico (cefalea, alteraciones visuales, déficit de otras hormonas, etc.).

Nivel de manejo del médico general: Diagnóstico Específico Tratamiento Inicial Seguimiento Completo

Aspectos Esenciales • • •

Las causas fundamentales son la adrenalitis autoinmunitaria y la tuberculosa. Hiperpigmentación melánica es frecuente Tratamiento de sustitución con hidrocortisona.

Caso Clínico Tipo Paciente varón de 68 años. Consulta por cuadro de semanas de evolución de nauseas y vómitos, dolor abdominal y baja de peso. Su acompañante señala que está “más salado” para comer, y que al pararse se marea. Al examen físico, se aprecia hiperpigmentación de piel y mucosas. Se encuentra hipotenso, con test de hipotensión ortostática positivo. En el laboratorio, los electrolitos destacan con hiperkalemia e hiponatremia.

La confirmación diagnóstica debe ser realizada a nivel secundario, por lo que tras la sospecha se indica la derivación a especialista. El proceso diagnóstico consiste en tres etapas:

Definición

(1) demostración de la secreción disminuida de cortisol, (2) determinar si la deficiencia de cortisol es dependiente o independiente de un ACTH, además de evaluar la secreción de mineralocorticoides en pacientes sin déficit de ACTH, y (3) buscar una causa tratable de una patología primaria.

Es el síndrome clínico derivado de la deficiencia en la producción de hormonas a nivel de la corteza suprarrenal.

Etiología-Epidemiología-Fisiopatología

Tratamiento

La etiología de la Insuficiencia suprarrenal puede dividirse en primaria (enfermedad de Addison, debida a la afectación las glándulas suprarrenales), secundaria (debida a la alteración de la secreción de ACTH, a nivel hipofisiario) o terciaria (debida a la alteración de la secreción de CRH, a nivel hipotalámico).

El tratamiento está basado en la sustitución de la deficiencia hormonal, el que debe ser ajustado a las características y bienestar del paciente. No obstante, a grandes rasgos, éste se apoya principalmente en la sustitución del déficit de glucocorticoides.

En relación a la insuficiencia suprarrenal primaria, sus causas principales son: adrenalitis autoinmune (70 a 90% de los casos) y adrenalitis infecciosa (7 a 20%), siendo esta última secundaria a infección diseminada por Mycobacterium tuberculosis, Histoplasma capsulatum, VIH, entre otras. La insuficiencia suprarrenal secundaria puede se producida por cualquier proceso que involucre a la hipófisis e interfiera con la secreción de ACTH (ej: panhipopituitarismo). En el caso de la insuficiencia suprarrenal terciaria, las causas más comunes son el cese abrupto de tratamiento con altas dosis de glucocorticoides y la corrección de hipercortisolismo.

El régimen usual es mediante el uso de un glucocorticoide de acción corta (hidrocortisona), dividido en una dosis a las 7-8 am, y una o dos dosis vespertinas, con una dosis total recomendada de 10-12 mg/m². Hay que recalcar que frente a la sospecha de una insuficiencia suprarrenal, se debe iniciar inmediatamente la sustitución de hidrocortisona mientras se realiza la derivación y confirmación diagnóstica, por el riesgo inminente de crisis suprarrenal. Si se requiere el especialista podria empezar con fludrocortisona con mayor actividad mineralocorticoide

8 Facultad de Medicina, Universidad de Chile

Seguimiento Es llevado a cabo por el especialista, quien realizará ajustes necesarios en la terapia hormonal, intentando siempre utilizar la menor dosis que alivie los síntomas del déficit de corticoides y evite los síntomas del exceso de corticoides.

Autor / Editor Diego Cáceres

Año 2017

9 Facultad de Medicina, Universidad de Chile

Hipertensión Arterial de Origen Endocrino

- Acromegalia: Revisada en otro capítulo.

Diagnóstico

Nivel de manejo del médico general: Diagnóstico Específico Tratamiento Inicial Seguimiento Completo

Hiperaldosteronismo primario: - Clínica: La presentación clásica consiste en hipertensión e hipokalemia (no más del 13% de los casos). Además, puede presentar signos musculares de hipokalemia, hipernatremia leve, hipomagnesemia y alcalosis metabólica. Ocasionalmente se presenta con poliuria y nicturia, secundarias a la excreción de agua inicialmente retenida por la aldosterona. Se indica hacer screening de hiperaldosteronismo primario a: pacientes con HTA e hipokalemia, HTA resistente a tratamiento, HTA y hallazgo de incidentaloma suprarrenal, en toda sospecha de HTA secundaria. - Laboratorio: El diagnóstico inicialmente se realiza con la evaluación de la relación Aldosterona plasmática / Actividad de renina plasmática (AP/ARP): en el hiperaldosteronismo primario hay una producción excesiva y autónoma de aldosterona, por lo tanto la renina estará suprimida, resultando en una relación AP/ARP aumentada. Valores >30 apoyan el diagnóstico, mientras que valores <20 descartan hiperaldosteronismo. Sin embargo, la relación AP/ARP no es diagnóstica de hiperaldosteronismo primario, por lo que posterior a ello deberán realizarse otros estudios para demostrar la secreción inapropiada de aldosterona.

Aspectos Esenciales • • • • •

Forma parte de las HTA secundarias, es decir, tiene causa tratable. Ocurre en jóvenes. Múltiples etiologías. Alta morbimortalidad. Tratamiento generalmente quirúrgico.

Caso Clínico Tipo Paciente varón de 37 años, presenta un episodio de síncope. Refiere antecedentes de cefalea pulsátil y palpitaciones. Al examen físico destaca PA 190/110 mmHg y FC 140 lpm. No consume drogas. Hemograma normal. Crea 0,8; K 3,6; Na 137; Ca 9,2. Se sospecha Feocromocitoma

Feocromocitoma: - Clínica: La triada clásica está compuesta por taquicardia, sudoración y cefalea (sin embargo, la mayoría de los pacientes no tienen los tres síntomas). Se presenta con HTA de difícil tratamiento, a veces hipertensión severa paroxística (50% de los casos aproximadamente), en espacial cuando se presenta en relación a anestesia, exámenes con medio de contraste o trabajo de parto; paroxismos de cefalea, temblor, palpitaciones, palidez, debilidad generalizada y disnea, de comienzo súbito y corta duración. Menos frecuentemente se manifestan con hipotensión ortostática, visión borrosa, baja de peso, hiperglicemia, leucocitosis o trastornos psiquiátricos. Los feocromocitomas pueden localizarse en la pared vesical, en estos casos puede manifestarse con síntomas por exceso de catecolaminas asociados a la micción. Se sugiere sospechar feocromocitoma principalmente en caso de sintomatología acorde, hipertensión resistente, síndromes familiares que predispongan a tumores secretores de catecolaminas, historia familiar de feocromocitoma, incidentaloma adrenal, respuesta hipertensiva durante anestesia, cirugía o angiografía e hipertensión a edades tempranas (<20 años), entre otras. - Laboratorio: Se observa hiperglicemia, hipercalcemia, hipokalemia. El diagnóstico se hace con la determinación de metabolitos de metanefrinas y catecolaminas en orina de 24 horas. La confirmación bioquímica será seguida de una evaluación imagenológica para la localizar el tumor, que se ubica en un 95% en abdomen y pelvis.

Definición Hipertensión arterial secundaria a causas endrocrinológicas, generalmente a nivel suprarrenal.

Etiología-Epidemiología-Fisiopatología - Hiperaldosteronismo primario: Causado por adenoma suprarrenal (Síndrome de Conn) o hiperplasia unilateral (aprox. 65%), hiperplasia suprarrenal bilateral o idiopática (30%) o carcinoma (extremadamente raro). Otras causas poco frecuentes comprenden el hiperaldosteronismo remediable con glucocorticoides y otros trastornos de exceso verdadero o aparente de mineralocorticoides - Feocromocitoma: Tumor de células cromafines, productor de catecolaminas (Epinefrina, Norepinefrina y Dopamina), generalmente en las glándulas suprarrenales (90%) o en tejido paraganglionar fuera de las suprarrenales. Se puede manifestar como hipertensión sostenida o paroxística en jovenes o adultos. Es la causa más infrecuente de HTA secundaria (<0,1% del total de HTA), pero su importancia radica en su alta morbimortalidad. - Síndrome de Cushing: Revisado en otro capítulo.

10 Facultad de Medicina, Universidad de Chile

Tratamiento Dependerá de la etiología causante. En general, el tratamiento de las causas tumorales es quirúrgico, acompañado de tratamiento sintomático. El hiperaldosteronismo por hiperplasia suprarrenal bilateral se trata con bloqueadores del receptor de aldosterona o del ENAC. El feocromocitoma debe recibir tratamiento preoperatorio con bloqueo alfa adrenérgico por 10 a 14 días, y posteriormente agregar bloqueadores beta adrenérgicos. En el caso de pesquisar una neoplasia que tenga asociación descrita con feocromocitoma (NEM 2A o 2B), se debe buscar dirigidamente la presencia de feocromocitoma, cuya existencia condicionará el tratamiento a indicar.

Seguimiento Derivar a especialista.

Autor / Editor Catalina Luvecce

Año 2016

11 Facultad de Medicina, Universidad de Chile

Hipopituitarismo

aracnoidocele, apoplejía, genético, infarto pituitario (síndrome de Sheehan), autoinmune.

Nivel de manejo del médico general: Diagnóstico Específico Tratamiento Inicial Seguimiento Completo

2. Hipotalámico: Craneofaringioma, meningiomas, germinomas, radioterapia, infiltrativas (sarcoidosis, hemocromatosis), genética (Síndrome de Kallman).

Diagnóstico

Aspectos Esenciales • • • •

Al sospecharse clínicamente del déficit de una o más hormonas, se debe realizar exploración hipofisiaria global, para establecer si alguna otra hormona del eje presenta hipo o hiperfunción. Para el déficit de TSH, se solicita T4L; para el déficit de ACTH, se solicita cortisol matinal (en valores intermedios 3-18 ug/ dL se observa reserva de ACTH); para déficit de gonadotropinas, solicitar FSH y estradiol en mujeres con amenorrea, y Testosterona total en hombres; para déficit de GH, se solicita IGF-I. No está indicado el estudio de déficit de prolactina de forma rutinaria.

Si el origen es hipotalámico puede acompañarse de diabetes insípida central y existir hiperprolactinemia. Muchas veces no es necesario esperar el resultado de los exámenes para realizar el tratamiento. El tratamiento debe ser permanente y ajustarse en situaciones de estrés. El Sd. Sheehan es poco frecuente actualmente.

Caso Clínico Tipo Paciente varón de 17 años. Consulta por falta de desarrollo puberal. Al examen: Talla 1,65 (talla familiar 1,75) sobrepeso leve, ausencia de vello facial, axilar y pubiano. Genitales masculinos T2, testes de 3cc. En los exámenes de laboratorio: glicemia 60mg/dl, Na 135meq/lt, Hto 33%.

Tratamiento - Hidrocortisona (déficit de ACTH): debe ser la primera en ser sustituida ante sospecha de compromiso del eje. Se puede usar hidrocortisona 15mg/m2, o Prednisona 5mg AM. El control de tratamiento es clínico.

Definición

- Levotiroxina (déficit de TSH), si los niveles de T4L son >0,9 ng/dL las dosis de tiroxina deben ser mayores a las de sustitución por falla tiroidea. Ej: Levotiroxina 2,1 u/Kg al día.

Es el déficit de una o más hormonas hipofisiarias secundario a alteración hipofisiaria o a secreción hipotalámica deficiente de las hormonas que controlan la adenohipófisis. Los ejes más lábiles son GH y gonadotropinas, pero puede observarse déficit aislado y combinaciones de todo tipo. Se llama déficit secundario si ocurre en la hipófisis, y déficit terciario si se debe al hipotálamo.

- Estradiol y Progesterona (déficit de LH/FSH): si hay déficit total, se usan ambas, como si fuera post-menopáusica. Si la paciente es histerectomizada, se usa sólo estradiol. - Testosterona (déficit de LH/FSH): se puede usar testosterona inyectable.

Etiología-Epidemiología-Fisiopatología

- Hormona de crecimiento (déficit de GH); en el adulto aún es discutible su sustitución. Está indicada en niños, especialmente en con talla baja patológica, (velocidad de crecimiento < - 2 DE, IGF-1, IGFBP 3 bajas, , edad ósea retrasada).

El déficit en las hormonas hipofisiarias puede ser ocasionado por múltiples trastornos: congénitos, genéticos, neoplásicos (adenoma hipofisario, tumoración parasillar, craneofaringioma, metástasis, meningiomas), vasculares (apoplejía hipofisaria, necrosis posparto y drepanocitosis), traumáticos (lesiones craneoencefálicas, radioterapia, procedimientos quirúrgicos), infiltrativos (hemocromatosis, hipofisitis linfocítica, sarcoidosis, histiocitosis X) o infecciosos (tuberculosis, micosis y enfermedades parasitarias). De manera general, según la localización del compromiso, se clasifica en dos grandes grupos:

Seguimiento Derivar a especialista.

1. Hipófisis: Tumoral (adenomas, aproximadamente el 50% de los casos de hipopituitarismo), cirugía, radioterapia (hipopituitarismo puede aparecer hasta 15 años post terapia),

Autor / Editor Catalina Luvecce

12 Facultad de Medicina, Universidad de Chile

Año 2016

Diabetes Insípida Nivel de manejo del médico general: Diagnóstico Específico Tratamiento Inicial Seguimiento Completo

• • •

Aspectos Esenciales • • •



Diagnósticos diferenciales de poliuria: diuresis osmótica por diabetes mellitus y polidipsia sicógena, que es más frecuente en mujeres y no hay nicturia. Diabets insípida central: responde a la administración de AVP exógena y no así la nefrogénica. Causa importante de DI central: post TEC y Cx.

Polidipsia primaria: Cuadro caracterizado por un aumento primario del consumo de agua, más frecuente en adolescentes y adultos ansiosos, con patología psiquiátrica, lesiones hipotalámicas, pudiendo ser de causa iatrogénica, secundaria a recomendación médica.

Caso Clínico Tipo Paciente de 34 años, que post TEC presenta poliuria de inicio brusco que se acompaña de nicturia y polidipsia. Con necesidad imperiosa de beber agua de día y de noche. La orina es hipotónica y su volumen oscila entre 4 a 15 l/d.

Diagnóstico Manifestaciones clínicas: Poliuria (>3 L/día en el adulto y >2 L/día en niños), persistente, sed excesiva, nicturia y polidipsia. La orina presenta densidad baja (<1.010) y osmolaridad baja (<300 mOsm/Kg) o normal, dependiendo del acceso al agua.

Definición Cuadro clínico con diuresis mayor de 3 L/día (>40 mL/Kg al día) y una osmolalidad < 300 mosmol/kg, densidad específica <1.010, causado por la secreción insuficiente de ADH o por defectos en su acción. Se distinguen dos tipos: hipotalámica o central y nefrogénica.

Evaluación inicial: Medir volumen urinario en 24 horas, examen de orina (densidad urinaria y descartar glucosuria), osmolaridad urinaria en orina matinal, ELP, glicemia, uremia y creatinemia.

Etiología-Epidemiología-Fisiopatología

Test de deshidratación con VAP exógena final: Consiste en comparar la osmolaridad urinaria y plasmática después de la deshidratación y la obtenida tras la administración de DDAVP (4 ug SC o 10 ug intranasal). Esta última se administra cuando se alcanza una osmolaridad plasmática superior a 300 mOsm/Kg o si el peso corporal disminuye un 5%. Los sujetos normales en esta prueba alcanzan concentraciones urinarias de 500 a 1.200 mOsm/Kg en 4 a 10 horas de deshidratación y no responde a AVP exógena.

Diabetes insípida (DI) central: Disminución parcial o total de la secreción de ADH. Se requiere una pérdida del 80% para que aparezca el cuadro. Se produce poliuria que lleva a un incremento de la osmolaridad plasmática y disminución leve del volumen plasmático que activa el centro de la sed. Esto hace que el paciente ingiera agua hasta normalizar la osmolaridad, por eso no habrá hipernatremia. En el caso de la DI total, se alcanza una máxima dilución urinaria (< 100 mosmol/kg), y el débito urinario puede alcanzar hasta 10-20 lt. Dentro de su etiología destaca: •

• •

cuerpos neuronales se libera a través del tallo hipofisiario abundante cantidad de la hormona, con el riesgo de producir intoxicación hídrica. Por último, si el daño progresa hay lesión de los somas neuronales y DI permanente. Hereditaria: De inicio brusco en la infancia, incontrolable, por mutaciones de neurofisina y molécula de AVP. Diabetes insípida nefrogénica: hay una falta de la respuesta renal a la hormona ADH. Congénita: se presenta al nacer, por mutación del receptor V2 o del gen de aquaporina. Adquirida: nefropatías que provocan tubulopatías o deforman la arquitectura renal (riñones poliquísticos, amiloidosis, uropatía obstructiva), drogas que alteran la síntesis renal de cAMP (litio, demeclociclina), hipokalemia e hipercalcemia, que producen disminución de las aquaporinas.

DI central: Preferencia por líquidos fríos, permanente (día y noche), si pierde la conciencia se deshidrata, hay hiperuricemia. Durante el test de deshidratación no concentran más la orina y presenta respuesta a desmopresina concentrando más del 50%. Una vez confirmado el diagnóstico, debe realizarse estudio de imágenes con RNM para el diagnóstico etiológico definitivo.

Idiopática (50%). Entre los 20 y 30 años, de inicio brusco, gran avidez por líquidos helados, con libre acceso al agua los síntomas son poliuria y nicturia. Si ésta se restringe aparece irritabilidad, confusión, ataxia, hipertermia, hipotensión y puede llegar al coma. Tumores y neoplasias hematológicas (25%). Posterior a TEC o cirugía (respuesta trifásica, 30%): Primera fase con disminución de ADH por lesión directa de la neurohipófisis, posteriormente al estar indemne los

DI nefrogénica: Se presenta en recién nacidos o hay patología renal o drogas de por medio. Hay niveles séricos elevados de AVP junto a poliuria hipotónica y no presenta respuesta a desmopresina. En el caso de la DI nefrogénica congénita, los

13 Facultad de Medicina, Universidad de Chile

síntomas por lo general aparecen durante la primera semana de vida, presentando vómitos, estreñimiento, trastorno del desarrollo, fiebre y poliuria. Los análisis demuestran presencia de hipernatremia con baja osmolaridad de la orina.

Tratamiento DI central: Desmopresina (análogo sintético de AVP), usualmente vía spray intranasal, con dosis iniciales de 5 ug al acostarse hasta 20 ug 1 a 2 veces cada día. DI nefrogénica: Mantener ingesta de agua adecuada, en algún déficit parcial puede responde a dosis alta de desmopresina. El objetivo del tratamiento es reducir la poliuria sintomática, disminuyendo el volumen de orina lo que se logra con dieta hiposódica y diuréticos tiazidicos. Es aconsejable administrar amiloride. Eliminar factores causantes.

Seguimiento Por especialista.

Autor / Editor Catalina Luvecce

Año 2016

14 Facultad de Medicina, Universidad de Chile

Síndrome de Secreción Inapropiada de ADH

siendo los pacientes euvolémicos. No hay alteraciones en el equilibrio ácido-base ni en la homeostasis del potasio producto del SSIADH. Las causas de SSIADH son diversas. El SSIADH es particularmente frecuente en pacientes hospitalizados y representa el 50% de las hiponatremias euvolémicas. Otras causas de hiponatremia euvolémica que deben buscarse dirigidamente son el hipotiroidismo, insuficiencia suprarrenal, polidipsia psicógena, etc.

Nivel de manejo del médico general: Diagnóstico Específico Tratamiento Inicial Seguimiento Completo

Causas de SSIADH: Neoplasias: Tumor de células pequeñas, tumores de cabeza y cuello, tumores de duodeno y páncreas, timoma, etc. Enfermedades pulmonares no malignas: Neumonías, asma, atelectasias, insuficiencia respiratoria aguda, neumotórax, ventilación mecánica, etc. Alteraciones SNC: Infecciones, ictus, TEC, psicosis, cirugía hipofisiaria, etc. Fármacos: Clorpropamida carbamazepina y derivados, clofibrato, ciclofosfamida, tricíclicos, IMAO, ISRS, oxitocina, etc. Otros: Hipotiroidismo, insuficiencia suprarrenal, cirugía mayor torácica o abdominal, VIH, arteritis de la temporal, SSIADH hereditario, etc.

Aspectos Esenciales • • • • •

Sospechar en todo paciente con hiponatremia e hipoosmolaridad plasmática sin edemas, hipotensión o signos de deshidratación. Frecuente en pacientes hospitalizados. Deben descartarse hipotiroidismo e insuficiencia suprarrenal. Restricción acuosa y corrección de la natremia constituyen el manejo. Tener siempre presente el riesgo de mielinolisis pontina al corregir la natremia.

Caso Clínico Tipo Mujer de 65 años, con antecedentes de epilepsia, EPOC y tabaquismo activo, cursando quinto día postoperatorio tras gastrectomía subtotal, secundaria a cáncer gástrico. Asintomática, presenta hiponatremia e hipoosmolaridad plasmática leves, sin alteraciones en la kalemia ni estado ácido-base.

Diagnóstico Clínico y de laboratorio. Las manifestaciones están dadas por la hiponatremia y dependerán de su severidad y velocidad de instalación. a.Hiponatremia leve (125 a 135 mEq/L): anorexia, náuseas, vómitos, cefalea, sensación de inestabilidad, etc. b.Hiponatremia grave (<125 mEq/L): agitación, irritabilidad, confusión, coma, convulsiones (síntomas producto de edema cerebral). Las alteraciones de laboratorio características son: hiponatremia (<135 mEq/L), hipoosmolaridad plasmática (<275 mOsm/Kg), ausencia de dilución máxima de la orina (osmolaridad urinaria >100 mOsm/Kg en el contexto de hipoosmolaridad plasmática), excreción urinaria de sodio aumentada (>20 mEq/L) y niveles plasmáticos disminuidos de: creatinina, BUN, ácido úrico, albúmina, etc. Además, son característicos y requeridos para el diagnósticio: euvolemia (ausencia de edemas, hipotensión o signos de deshidratación) y exclusión de hipotiroidismo e insuficiencia suprarrenal.

Definición Síndrome caracterizado por la presencia de hiponatremia hipotónica euvolémica, producto de una secreción inapropiada de ADH en relación a la osmolaridad plasmática, parcial o totalmente autónoma respecto al control osmótico.

Etiología-Epidemiología-Fisiopatología La ADH regula la osmolaridad plasmática al permitir la reabsorción de agua libre en el nefrón distal, en respuesta al aumento de la osmolaridad. En el SSIADH, el aumento patológico de la ADH impedirá la excreción de agua libre, conduciendo tras la ingesta de agua, a la disminución de la osmolaridad plasmática con osmolaridad urinaria aumentada. El hematocrito, la hemoglobina y los niveles plasmáticos de otros solutos también pueden verse disminuidos (albúmina, creatinina, BUN, ácido úrico, etc.). Los mecanismos reguladores de la volemia (y por ende de la cantidad de sodio) se encuentran intactos, por lo tanto, la hipervolemia generada en un principio por el aumento en la reabsorción de agua libre es compensada mediante un aumento en la natriuresis,

Tratamiento Consta de un tratamiento etiológico (identificar y tratar la causa subyacente) y un manejo sindromático (restricción de la ingesta de agua y corrección de la natremia): a) Hiponatremias agudas y/o sintomáticas y/o severas (<125 mEq/L) • Restricción de la ingesta de agua (régimen 0, por lo me-

15 Facultad de Medicina, Universidad de Chile





nos hasta corrección). Solución salina hipertónica (3%): 1 a 2 mEq/L/hora en sintomáticos y 0,5 a 1 mEq/L en sintomáticos, con un límite de 9 mEq/L en 24 horas (para evitar mielinolisis pontina, sin embargo, una excepción a esto sería cuando una hiponatremia “hiperaguda” se ha desarrollado en pocas horas). El déficit de sodio se calcula como: déficit de sodio = agua corporal total x (natremia deseada – natremia actual). El agua corporal total se estima con el peso x 0,5 en mujeres y 0,6 en hombres. Así mismo, el incremento de la natremia se calcula como: incremento de natremia = ([Na]infundido – natremia) / (peso corporal + 1). Sin embargo, con el uso de estas fórmulas, el incremento de la natremia es frecuentemente mayor al predicho. Furosemida si osmolaridad urinaria es muy alta.

b)Hiponatremias crónicas y asintomáticas y/o leves (>125 mEq/L) • Restricción de la ingesta de agua (ingesta no mayor a la diuresis.) • Furosemida en dosis bajas y aporte de sal. • Antagonistas tubulares de ADH (litio y demeclociclina) y antagonistas de receptores V2 (tolvaptan y conivaptan) pueden considerarse.

Seguimiento Según etiología.

Autor / Editor Diego Cáceres

Año 2017

16 Facultad de Medicina, Universidad de Chile

Osteoporosis Secundaria

Etiología-Epidemiología-Fisiopatología Epidemiología. En estudios internacionales se estima que en hombres (≥50 años) la prevalencia promedio de osteoporosis es de 6%, en mujeres (≥50 años) estas cifras serían 7.2%. La incidencia de osteoporosis sería de 3.47 x 100.000 personas por año. En Chile no existen estudios de prevalencia de osteoporosis, pero en la ENS 2009-2010 se menciona una prevalencia de fracturas de 17.9% (estudios internacionales mencionan que puede ser un indicador de osteoporosis), esta prevalencia aumenta con la edad, siendo sobre los 65 años mayor en mujeres que en hombres, situación inversa en los pacientes menores de 65 años. En general la osteoporosis es una patología más frecuente en mujeres posmenopáusicas, cuando aparece en mujeres premenopáusicas y hombres se debe descartar causa secundaria, estimándose que cerca del 4050% de la osteoporosis en hombres se debe a causa secundaria.

Nivel de manejo del médico general: Diagnóstico Específico Tratamiento Inicial Seguimiento Completo

Aspectos Esenciales • • • •

Alrededor del 80% de las osteoporosis en mujeres son primarias, a diferencia de los varones, en los que el 40 a 50% tiene al menos una causa secundaria. Es muy importante tener un alto índice de sospecha para buscar y eventualmente corregir las diferentes causas de osteoporosis secundaria. Dentro de las causas más frecuentes destaca el uso de glucocorticoides. En general el tratamiento es por especialista.

Etiología. Entre las causas más frecuentes se encuentra el tratamiento esteroidal, donde la pérdida de hueso trabecular es mayor que la cortical. El riesgo depende de la dosis y duración del tratamiento. En estos casos, el mecanismo es el efecto esteroidal sobre los osteoblastos. En el caso de enfermedades endocrinas tenemos por ejemplo el exceso de PTH, lo cual estimula a los osteoclastos; en el caso de déficit de vitamina D, hay menor absorción de calcio en el intestino, reducción de actividad osteoblástica y aumento de PTH por no inhibición de su síntesis.

Caso Clínico Tipo Mujer de 19 años, portadora de LES en tratamiento hace 5 años con prednisona y cloroquina. Consulta con signos de hipercortisolismo y refiere dolor intenso e incapacitante a nivel de columna dorsolumbar anterior bilateral. El estudio radiológico de columna dorsolumbar mostró fracturas por acuñamiento múltiples y un grueso borde esclerótico en múltiples cuerpos vertebrales.

Dentro de las causas secundarias encontramos: Inducida por medicamentos: Glucocorticoides (≥5 mg al día de prednisona o equivalente por tres o más meses), Aluminio (antiácidos), anticoagulantes (heparina), anticonvulsivantes, inhibidores de la aromatasa, barbitúricos, quimioterapia en cáncer, tacrólimus, ciclosporina, medroxiprogesterona, agonistas GnRH, litio, metotrexato, IBP, ISRS, tomoxifeno, hormonas tiroídeas, nutrición parenteral. Estados hipogonadales: Menopausia prematura, hiperprolactinemia, falla ovárica prematura, panhipopituitarismo, sd. de Turner y Klinerfelter. Desórdenes endocrinos: Insuficiencia adrenal, diabetes mellitus, sd. de cushing, hiperparatioridismo, tirotoxicosis. Desórdenes gastrointestinales: Enfermedad celiaca, bypass gástrico, cirugía gástrica, enfermedad inflamatoria intestinal, malaabsorción, enfermedad pancreática, cirrosis biliar primaria.

Definición Enfermedad esquelética sistémica caracterizada por baja masa o densidad ósea y deterioro de la microarquitectura del tejido óseo, con aumento de la fragilidad ósea y riesgo de fractura. Los sitios más frecuentes de fracturas relacionadas con esta patología son: vértebras, cadera y porción distal del radio. Se clasifica en : Primaria: A su vez se clasifica en tipo I o posmenopáusica y tipo II o senil (tema que se aborda en otro apartado del manual). Secundaria: Cuadro en el que existe un factor causal identificable, excluyendo la menopausia y el envejecimiento, como enfermedades endocrinas, enfermedades hematopoyéticas, enfermedades del tejido conectivo, inducida por drogas, enfermedades renales, inmovilización prolongada, enfermedades gastrointestinales y nutricionales.

Hematológica: Mieloma mútiple, hemofilia, talasemia, leucemia y linfoma. Reumatológicas y autoinmunes: Espondilitis, lupus, artritis reumatoide. Desorden del SNC: Epilepsia, esclerosis múltiple, sd. parkinsoniano, ACV. Enfermedad renal crónica.

17 Facultad de Medicina, Universidad de Chile

Diagnóstico En todo paciente que se sospeche causa secundaria se debe realizar una anamnesis y examen físico exhaustivos (no olvidar medir la talla y la fuerza muscular), además de: Pruebas de laboratorio básicas: hemograma/VHS, perfil bioquímico, creatinina plasmática, PTH, TSH y orina de 24 hrs para creatinina, calcio y sodio (sin suplemento de calcio ni diuréticos en las dos semanas previas). Densitometría ósea (DXA): en general se utiliza el Z-score para definir osteoporosis secundaria. Derivar a especialista: Toda sospecha de causa secundaria, cualquier paciente cuando se pesquise en la DXA: Z-score < -2, DMO más disminuida en cadera que columna, fracaso de tratamiento adecuado.

Tratamiento El tratamiento de la osteoporosis secundaria es de manejo de especialista. Medidas preventivas no farmacológicas para evitar las fracturas: Ingesta adecuada de calcio (1000-1200 mg/día) y vitamina D (800-1000 UI/día). Actividad física regular. Suspender hábito tabáquico. Limitar ingesta alcohol: Máximo 2 bebidas por día. Prevención de caídas. En lo posible, elegir la dosis más baja de glucocorticoides posible, hay evidencia que respalda el uso de risedronato y alendronato en el caso de uso prolongado de glucocorticoides, para disminuir el riesgo de fractura vertebral. Cuando existe exceso de PTH por hiperparatiroidismo primario, el tratamiento es resección quirúrgica de las glándulas paratiroides. Y así dependiendo de la causa se define el manejo específico de la patología causante de osteoporosis.

Seguimiento Toda causa secundaria de osteoporosis requiere manejo y seguimiento por especialista.

Autor / Editor Paulina Lira

Año 2016

18 Facultad de Medicina, Universidad de Chile

Tormenta Tiroídea

Diagnóstico Se realiza por la clínica, no habiendo criterios universalmente aceptados, pero son utilizados los criterios diagnósticos de Burch & Wartofsky, ya que los niveles séricos de hormonas tiroideas no distinguen entre un hipertiroidismo severo y la tormenta tiroidea. Un puntaje de 45 o más es altamente sugestivo de tormenta tiroidea, puntajes entre 25 y 44 apoya el diagnóstico, puntajes bajo 25 hace el diagnóstico improbable. Entre los síntomas más habituales se encuentran hipertermia, disfunción cardiovascular y compromiso de conciencia en contexto de un paciente en que sus pruebas tiroideas están alteradas mostrando T4 y T3 elevadas, con TSH disminuida bajo los valores normales.

Nivel de manejo del médico general: Diagnóstico Específico Tratamiento Inicial Seguimiento Derivar

Aspectos Esenciales • • • • •

Es un hipertiroidismo severo asociado a falla multiorgánica, con alta mortalidad si no hay un tratamiento oportuno. Se presenta usualmente en hipertiroidismos mal controlados o no diagnosticados, ante un factor de stress. El diagnóstico se debe realizar por la clínica, ya que los niveles de hormonas tiroideas no son confiables. Es una emergencia médica, por lo que se debe sospechar y tratar en forma precoz. El tratamiento implica medidas generales, PTU, Yodo, Propanolol y Dexametasona.

Tratamiento La tormenta tiroidea tiene una mortalidad que varía del 10% al 30%, por lo que requiere monitorización en UCI. El tratamiento consiste en tres pilares: 1. Medidas de soporte: • Administración de O2. • Corrección de la deshidratación y administración de suero glucosado al 5% y fisiológico al 0,9% 80-100 ml/h. • Digitálicos y drogas inotrópicas en caso de shock o insulinoterapia si coexiste cetoacidosis diabética. • Control de hipertermia por medidas físicas (compresas frías) y farmacológicas (Paracetamol EV 500 – 1.000 mg c/6 horas). • Contraindicado AAS (desplaza la T4 de la TBG).

Caso Clínico Tipo Paciente hombre de 78 años, con antecedentes de HTA, obesidad, DM2 y Enfermedad de Graves. Consulta por cuadro de fiebre asociado a tos con expectoración mucopurulenta y disnea, además de compromiso de conciencia cualitativo. Rx tórax muestra focos de condensación lóbulo inferior derecho.

2. Tratamiento del hipertiroidismo: • Bloqueo de la síntesis hormonal: de primera elección PTU en dosis inicial de 200-400 mg c/6 horas VO o por SNG. Dosis de mantención 200 mg c/6-8 horas hasta normalizar la función tiroidea; de segunda elección metamizol dosis inicial 20 mg c/6 horas VO y de mantención 10 a 20 mg c/8 horas hasta normalizar la función tiroidea. • Bloqueo de la liberación hormonal: administrar luego de dos horas de la 1era dosis de PTU, yodo VO (solución de Lugol) 10 gotas c/8 horas o yoduro potásico saturado (SSKI) 5 gotas c/8 horas VO. También puede utilizarse algún medio de contraste yodado EV, una dosis/día.

Definición Exacerbación brusca de las manifestaciones clínicas del hipertiroidismo, en general en respuesta a un factor estresante, acompañado de fiebre, compromiso de conciencia variables e insuficiencia de otros sistemas (cardiovascular, hepático, renal, etc.), que presenta alta mortalidad en caso de no ser sospechado y tratado precozmente.

Etiología - Epidemiología - Fisiopatología

3. Tratamiento coadyuvante: • Antagonistas beta adrenérgicos, siendo de primera elección Propanolol 40 a 80 mg c/4-6 horas VO o 0,5 a 1 mg c/5 minutos EV (sólo en caso de monitorización hemodinámica), hasta control de frecuencia, en pacientes con ICC preferir usar beta bloqueadores cardioselectivos. • Corticoides, de primera elección Hidrocortisona 100 mg c/8 horas EV, y segunda elección Dexametasona 2 mg c/6 horas EV. • Tratamiento enérgico del factor desencadenante.

Corresponde a una complicación muy rara, causa de 1 a 2% de las hospitalizaciones por hipertiroidismo. Se produce en pacientes con hipertiroidismo no diagnosticado o con tratamiento insuficiente, en los cuales se agrega un factor de estrés que desencadena la tormenta tiroídea. Algunos factores desencadenantes son: sepsis (más frecuente), cirugías, traumatismos, partos, cetoacidosis diabética, IAM, ACV, TEP, exposición a yodo y fármacos. Dentro de las etiologías, éstas son las mismas causas que las del hipertiroidismo no manifestado como tormenta tiroidea, siendo la más frecuente la Enfermedad de Graves.

19 Facultad de Medicina, Universidad de Chile

Seguimiento

Derivar a endocrinólogo para manejo causal de hipertiroidismo.

Autor / Editor Francisco Weber

Año 2017

20 Facultad de Medicina, Universidad de Chile

Mixedema

do, puede complicarse con la aparición de coma mixedematoso, que se caracteriza por compromiso cuantitativo de conciencia, convulsiones e hipotermia, aunque también se puede presentar hiponatremia, hipotensión, bradicardia e hipoglicemia. Se da principalmente en ancianos (Con mayor incidencia en mujeres), no adecuadamente diagnosticados o tratados. Se desencadena por factores que alteran la respiración, como fármacos, neumonía, insuficiencia cardiaca congestiva, IAM, ACV, hemorragia digestiva, exposición al frío, presencia de sepsis, opioides o post cirugías. La hipoxia e hipercapnia secundarias a la hipoventilación desempeñan un papel importante en la patogenia, la hipoglucemia y la hiponatremia por dilución también pueden contribuir al desarrollo de coma mixedematoso.

Nivel de manejo del médico general: Diagnóstico Específico Tratamiento Inicial Seguimiento Completo

Aspectos Esenciales • • •

Tumefacción y borramiento de pliegues sin signo de la fóvea. Asociado principalmente al hipotiroidismo. Coma mixedematoso, complicación grave en ancianos y pacientes mal tratados: alteración de conciencia + hipotermia + convulsiones.

Tratamiento Para el tratamiento del coma mixedematoso se debe actuar bajo sospecha del cuadro sin esperar resultados de laboratorio (Realizar previo a tratamiento: TSH, T4 y Cortisol en sangre). Se administra una dosis de carga de levotiroxina (dosis de carga) 200-400 ug, seguido de 1.6 ug/kg cada día EV, asociada a una dosis de carga de Triyodotironina (T3) de 5-20 ug, seguida de 2.5 a 10 ug cada 8 horas, con posterior traslape a tratamiento oral con el paciente estable. Importante tener precaución en pacientes cardiópatas por el riesgo de IAM con el uso de Hormonas tiroideas.

Caso Clínico Tipo Paciente mujer de 75 años. Antecedentes de tiroidectomía, dislipidemia y angina de pecho. Tras cirugía de hernia inguinal estrangulada, desarrolla cuadro de compromiso cuantitativo de conciencia, hipotermia, hiponatremia e insuficiencia respiratoria que requiere apoyo mecánico. T3: 20 ng/dL, T4: no detectable, TSH: > 100 µUI/mL.

Para la corrección de hipotermia, se debe usar calentamiento externo sólo si la temperatura es menor a 30°. Se incluye además administración de hidrocortisona EV (100 mg c/8 horas), por la posible insuficiencia suprarrenal asociada, antibioterapia de amplio espectro en sospecha de sepsis y la corrección de cualquier otro factor desencadenante.

Definición Aumento de volumen de la dermis y tejido subcutáneo debido a la acumulación de glucosaminoglicanos (GAGs) en la piel.

Seguimiento

Etiología - Epidemiología - Fisiopatología

Derivar.

El mixedema se puede asociar tanto a hipotiroidismo como a hipertiroidismo. En hipotiroidismo, se cree que la disminución de la función tiroídea, lleva a la deposición de mucopolisacáridos en la piel, lo que se acentuaría con el stress físico (cirugía, infección). Sin embargo, el mecanismo exacto es incierto. En hipertiroidismo, se desconoce la causa.

Autor / Editor Francisco Weber

Diagnóstico Se puede observar aumento de volumen difuso en la piel y tejido subcutáneo, con tumefacción y borramiento de pliegues, pero NO presenta signo de la fóvea (a diferencia del edema clásico). La piel toma un color amarillento, se pone fría, áspera, seca y escamosa, con disminución de secreciones sebáceas y sudoríparas. El paciente con un síndrome hipotiroideo descompensa-

21 Facultad de Medicina, Universidad de Chile

Año 2017

Insuficiencia Suprarrenal Aguda

normal (niños: sepsis por Pseudomona aeruginosa y meningococcemia, síndrome de Waterhouse-Friederichsen. Adultos: paciente crítico, uso de TACO o trastorno de la coagulación).

Nivel de manejo del médico general: Diagnóstico Específico Tratamiento Inicial Seguimiento Completo

d.Infarto suprarrenal bilateral

Diagnóstico Aspectos Esenciales • • •

Clínica: Fiebre elevada, deshidratación severa, hipotensión o shock. Otras manifestaciones inespecíficas como anorexia, náuseas, vómitos, dolor abdominal, fatiga, fiebre (pensar en infección), confusión, sopor y coma. Buscar siempre hiperpigmentacion de pliegues, cicatrices y mucosa bucal. Laboratorio: Hiponatremia, hiperkalemia, hipoglicemia, eosinofilia, hipercalcemia, aumento de BUN y acidosis metabólica orientan a causa primaria, ya que en las crisis por insuficiencia suprarrenal secundaria, no aparecen alteraciones hidroelectrolíticas salvo hiponatremia. Niveles plasmáticos de cortisol <11 μg/dL en un paciente grave, es altamente indicador.

Generalmente se da en pacientes sépticos o bajo condiciones de stress, que tienen una insuficiencia suprarrenal de base. También en pacientes con consumo crónico de corticoides que suspenden bruscamente la terapia. Tratamiento basado en la reposición de fluidos e hidrocortisona. Emergencia endocrinológica, por lo que debe ser urgente.

Caso Clínico Tipo

Tratamiento

Mujer de 45 años, usuaria crónica de corticoides por LES hace 5 años. Traída a urgencia por dolor abdominal progresivo, náuseas y vómitos aislados. Intervenida quirúrgicamente al ingreso, con diagnóstico postoperatorio de peritonitis apendicular generalizada. A las 6 horas postoperatorias, presenta cifras tensionales cercanas a 80/40, con dolor abdominal, náuseas y vómitos. Se indaga con familiares y no había recibido sus medicamentos desde el inicio del cuadro abdominal, por mala tolerancia oral. Tampoco se habían indicado corticoides EV durante hospitalización. Se inicia reposición de fluidos con suero fisiológico y se indica hidrocortisona con buena respuesta a tratamiento.

Urgencia: instaurarse ante sospecha fundada sin pensar en exámenes. Reposición de fluidos con solución de NaCl al 0,9% EV, 2 a 3 litros (primeras 24 horas); glucocorticoides: hidrocortisona 100 mg EV cada 8 horas (en caso de no existir una enfermedad grave asociada, a las 24 horas se puede comenzar el descenso progresivo de la dosis de hidrocortisona); solución glucosada al 5% más electrolitos; hidratación parenteral 3 a 4 L diarios por las 24 a 28 horas siguientes. Reposición de mineralocorticoides no es necesaria en periodo agudo, ya que, a pesar que los principales problemas médicos son la hipotensión y las alteraciones hidroelectrolíticas se corrige mediante reposición de electrolitos y el aporte de volumen a través del SF 0,9%. Siempre se debe buscar la causa desencadenante.

Definición Déficit súbito y global de la función de la corteza suprarrenal. También llamada “crisis suprarrenal”. Es una emergencia endocrinologica.

Seguimiento Derivación a especialista.

Etiología - Epidemiología - Fisiopatología Se puede deber a varios procesos: a.Interrupción brusca de corticoides en pacientes con atrofia suprarrenal secundaria a tratamiento corticoidal prolongado, es la causa más frecuente.

Autor / Editor Francisco Weber

b.Intensificación rápida y fulminante de una insuficiencia suprarrenal crónica (ya diagnosticada), en general desencadenante por cualquier condición de estrés moderado o severo. c.Hemorragia suprarrenal bilateral en paciente previamente

22 Facultad de Medicina, Universidad de Chile

Año 2017

BIBLIOGRAFÍA



ce=search_result&search=adh%20secretion&selectedTitle=2~150 •



N. Hillman Gadea, C. Álvarez Escolá, C. Dassen. Bocio. Bocio difuso y multi-

source=search_result&search=adh%20secretion&selectedTitle=5~150

Universitario La Paz. Universidad Autónoma. Madrid.



Bocio. Hipo e hipertiroidismo. Guía de Actuación Clínica en A. P. José Vicente

ria&selectedTitle=1~150



Muestra de preguntas Endocrinología EUNACOM, N° 351.



Jameson JL, Weetman AP. Trastornos de la glándula tiroides, cap. 341. Harrison.



sis%20secundaria&selectedTitle=3~150

Screening neonatal. Gema Matilde Calderón López, Francisco Jiménez Parrilla,



Antonio Losada Martínez. Servicio de Neonatología. H. Infantil. Virgen del Rocío

Jameson JL, Weetman AP. Trastornos de la glándula tiroides, cap. 341. Harrison.



https://www.uptodate.com/contents/myxedema-coma?source=see_link



https://www.uptodate.com/contents/clinical-manifestations-of-adrenal-insu-

Principios de medicina interna, 18° edición. McGraw Hill.

fficiency-in-adults?source=search_result&search=insuficiencia%20suprare-

https://www.uptodate.com/contents/epidemiology-and-clinical-manifesta-

nal%20aguda&selectedTitle=1~150

tions-of-cushings-syndrome?source=search_result&search=cushing&selected-



Title=2~150 •

https://www.uptodate.com/contents/thyroid-storm?source=search_result&search=tormenta%20tiroidea&selectedTitle=1~150

de Sevilla. AEPED.



https://www.uptodate.com/contents/clinical-manifestations-diagnosis-and-evaluation-of-osteoporosis-in-men?source=search_result&search=osteoporo-

Principios de medicina interna, 18° edición. McGraw Hill.



https://www.uptodate.com/contents/epidemiology-and-etiology-of-premenopausal-osteoporosis?source=search_result&search=osteoporosis%20secunda-

Herrero Ballester. Médico de Familia, CS Alcora



https://www.uptodate.com/contents/treatment-of-hyponatremia-syndrome-of-inappropriate-antidiuretic-hormone-secretion-siadh-and-reset-osmostat?-

nodular normofuncionante. Servicio de Endocrinología y Nutrición. Hospital



https://www.uptodate.com/contents/causes-of-hyponatremia-in-adults?sour-

https://www.uptodate.com/contents/diagnosis-of-adrenal-insufficiency-in-adults?source=see_link

https://www.uptodate.com/contents/overview-of-the-treatment-of-cushings-syndrome?source=search_result&search=cushing&selectedTitle=3~150



https://www.uptodate.com/contents/establishing-the-diagnosis-of-cushings-syndrome?source=search_result&search=cushing&selectedTitle=1~150



https://www.uptodate.com/contents/clinical-manifestations-of-adrenal-insufficiency-in-adults?source=search_result&search=addison&selectedTitle=1~150



https://www.uptodate.com/contents/diagnosis-of-adrenal-insufficiency-in-adults?source=search_result&search=addison&selectedTitle=2~150



https://www.uptodate.com/contents/treatment-of-adrenal-insufficiency-in-adults?source=search_result&search=addison&selectedTitle=3~150



Arlt W. Trastornos de la corteza suprarrenal, cap. 342. Harrison. Principios de medicina interna, 18° edición. McGraw Hill.



Hipertensión, cap. 126. Harrison. Manual de medicina. 18° edición. McGraw Hill.



Trastornos de la hipófisis anterior y el hipotálamo, cap. 179. Harrison. Manual de medicina. 18° edición. McGraw Hill.



Diabetes insípida y síndrome de secreción inapropiada de hormona antidiurética, cap. 180. Harrison. Manual de medicina. 18° edición. McGraw Hill.

23 Facultad de Medicina, Universidad de Chile



Amenorrea

 Nivel de manejo del médico general: Diagnóstico Específico Tratamiento Inicial Seguimiento Completo

  

Aspectos esenciales    



Primaria o secundaria, dependiendo de la ausencia o presencia de menarquía. Primero descartar causas fisiológicas: embarazo es la más frecuente. Síndrome de Turner, principal causa amenorrea primaria. SOP: se diagnostica por hiperandrogenismo (clínico o bioquímico) y oligo/anovulación u ovario poliquístico (signos ecográficos).

Oligomenorrea: disminución de la frecuencia de los ciclo menstruales con duración de éstos de > 35 días o < 10 menstruaciones por año. Polimenorrea: aumento de la frecuencia de los ciclos menstruales, con duración de estos de 21 días o menos. Hipomenorrea: disminución de la cantidad de flujo menstrual o del número de días totales que dura la menstruación. Menorragia: sangrado menstrual abundante o prolongado, pero con ciclos regulares. Metrorragia: sangrado irregular o sangrado entremedio del ciclo. Spotting (“mancha menstrual”): sangrado de escasa cuantía, entremedio del ciclo.

EPIDEMIOLOGÍA Se estima que un 5% de las mujeres tendrán amenorrea no producida por embarazo alguna vez en su vida.(1) Las amenorreas secundarias son mucho más frecuentes que las primarias y dentro de ellas, las más prevalentes son las funcionales (centrales).

Caso clínico tipo Paciente de sexo femenino de 19 años de edad, con antecedentes de menarquia a los 12 años. Consulta porque hace más de 3 meses que no ha presentado menstruación. Tiene pareja estable, test de embarazo negativo. Al examen físico destaca vello terminal en región facial y espalda.

ETIOLOGÍA Y FISIOPATOLOGÍA: 1) Amenorreas primarias(1,2) -Con caracteres sexuales secundarios: implica que la función estrogénica del ovario esta conservada, lo que nos habla de cierta indemnidad del eje hipotálamo – hipófisis – gónada.

DEFINICIÓN

 Criptomenorrea: En realidad si existe flujo menstrual, solo que no sale, porque el canal vaginal está interrumpido. La causa más frecuente es el himen imperforado, pero puede deberse también a un tabique vaginal transverso. A pesar de no haber menstruación, si hay dolor abdominal cíclico, producto de la acumulación de sangre en la vagina (hematocolpos) y en el útero (hematómetra). El diagnóstico se hace al observar el himen abombado y violáceo sin perforaciones y el tratamiento es perforarlo. El tabique requiere cirugía.

Amenorrea: se define como ausencia del flujo menstrual por al menos 3 ciclos menstruales seguidos, en una mujer en edad fértil. La definición operacional dependerá de si la mujer ha tenido sangrados previos (menarquia) o no. Existen 2 tipos de amenorreas (1): Amenorrea primaria: se presenta en mujeres que nunca han menstruado, en cualquiera de los siguientes escenarios: o >14 años sin desarrollo aún de caracteres sexuales secundarios.

 Síndrome de Rokitansky-Kuster-Hauser: Malformación mülleriana congénita, con agenesia vaginal y útero ausente o hipoplásico. Existe función ovárica normal y cariotipo 46XX. Es responsable de un 15% de las amenorreas primarias, con una incidencia de 1/5000. A pesar de no menstruar, pueden tener las otras manifestaciones clínicas de la ovulación. Se debe a una falla parcial de la canalización del conducto vaginal. Se diagnostica mediante RNM y se debe pedir cariotipo. Es importante evaluar la existencia de otras malformaciones asociadas: renales, óseas, auditivas, cardiacas.

o >16 años si comenzó ya su desarrollo puberal (caracteres sexuales secundarios). o Adolescente sin presencia de menarquia, con telarquia de más de 3 años.(2) Amenorrea secundaria: se presenta en mujeres que ya han menstruado antes, en los siguientes casos:(3) o Sin flujo menstrual por 90 días si los ciclos son normales y regulares.

 Testículo feminizante: Se presenta con una incidencia de 1/20.000 a 1/99000.(2) Se debe a una insensibilidad congénita a los andrógenos, por una alteración de los receptores de testosterona. Por lo que se desarrollan con fenotipo femenino pero sin vello púbico (respuesta a andrógenos). Se les debe pedir cariotipo y se deben extraer los testículos por riesgo de malignización.

o Sin flujo menstrual por 9 meses si tenía previamente oligomenorrea. Otras definiciones respecto a Sangrado uterino anormal:

1

Causas de Amenorrea. (Fuente: copiado de MasterHunter T, Heiman DL. Am Fam Physician. 2006 Apr 15;73(8):1374-82.)

2

-Sin caracteres sexuales secundarios: 



Hipoganadismo hipogonadotrófico: FSH y LH bajas, demuestra causa central. Lo más frecuente en este grupo es el retraso constitucional del crecimiento y pubertad, por lo que deben indagarse antecedentes en la familia, en este caso la conducta es expectante.



Otra causa, rara, es el síndrome de Kallman, en el que hay panhipopituitarismo y anosmia por alteración en la migración celular durante el desarrollo embrionario. 

Hipogonadismo Hipergonadotrófico: FSHA y LH elevadas, implica falla ovárica. o Sindrome de Turner: está englobado dentro de las disgenesias gonadales, siendo la primera causa de estas. El síndrome de Turner se debe a la ausencia de uno de los cromosomas X necesarios para hacer a una mujer (X0), que puede ser total o mosaico (algunas células si y otras no).



2. Causa ovárica (es Hipergonadotrófica, el ovario no produce estrógenos con una hipófisis funcional, donde no existe feedback negativo). 

Se ha descrito un fenotipo característico, siendo niñas de talla baja, cuello corto y alado, hipertelorismo, tórax en escudo y cubito valgo. Los genitales son femeninos pero infantiles y se asocia a HTA, coartación de la aorta y aorta bicúspide, y a malformaciones renales. Sin embargo el diagnóstico clínico no siempre es fácil y siempre requiere confirmación con cariotipo. Existen también otras disgenesias gonadales, como el síndrome de Sawyer (disgenesia gonadal pura) donde el cariotipo es 46XY pero hay una deleción completa del brazo corto del cromosoma Y (sonde va el gen SRY). 

Pseudohermafroditismo femenino: genitales ambiguos. Lo más frecuente es hiperplasia suprarrenal congénita.



Las causas son más variadas, pero existe un “orden” en el que se van descartando las patologías. Lo primero que se debe pedir siempre es: Test de Embarazo: b-hCG Prolactinemia
 TSH



El embarazo es la primera causa de amenorrea secundaria, pero no se considera patológica. En general las amenorreas secundarias se dividen dentro de sus causas en centrales y ovárica. 1. Causas centrales (hipogonadismos hipogonadotróficos con FSH y LH bajo los límites normales). 

Falla ovárica precoz: el ovario, al igual que en la menopausia, deja de cumplir sus funciones, solo que lo hace antes de los 40 años de edad (también se le llama “menopausia precoz”). No siempre es irreversible, aproximadamente la mitad de estas pacientes tienen función ovárica intermitente. Casi la mitad de los casos se deben a ooforitis autoinmune y puede asociarse a otros cuadros de esta clase, como hipotiroidismo, enfermedad Celíaca, enfermedad de Addison y Diabetes Mellitus tipo 1 (mantener en control, pueden aparecer después). También puede haber asociación con patología infecciosa (viral). Por el déficit temprano de estrógenos, estas pacientes tienen mayor riesgo cardiovascular y osteoporosis.

3. Amenorrea normogonadotrófica: Los niveles de gonadotrofinas son normales, ya que no es el eje hipotálamo-hipófisis-gónada el alterado.

2) Amenorreas secundarias:(3,6)

  

anticonvulsivantes, entre otros) o a hipotiroidismo (la TSH estimula la liberación de PRL). Por otra parte el hipotiroidismo por sí mismo puede alterar el flujo menstrual, aunque lo más frecuente es menorragia u oligomenorrea. Se puede sospechar por sus otras manifestaciones características. Funcionales: son las causas más frecuentes de amenorrea patológica, aunque no pueden diagnosticarse sólo con la clínica ya que son un diagnóstico de descarte, deben plantearse frente a: estrés psicoemocional, desnutrición extrema (anorexia nerviosa), ejercicio intenso (deportistas de alto rendimiento). Hipopituitarismo: Neoplasias del SNC, lesiones de TBC, hipertensión endocraneana, entre otras.

Hiperprolactinemia: la prolactina inhibe al eje Hipotálamo-Hipófisis-Gónada, provocando la amenorrea pero además produce galactorrea, que es característica. Valores elevados ( >100) y síntomas compresivos como hipertensión endocraneana y alteraciones del campo visual, deben hacer sospechar un prolactinoma. Este se diagnostica mediante RNM y se trata con Bromocriptina o Cabergolina. Cuando no es por un tumor, la hiperprolactinemia puede deberse a fármacos (antidepresivos, antieméticos, 3

Síndrome de Asherman: se producen sinequias y adherencias intrauterinas, habitualmente secundarias a un legrado enérgico u otros procedimientos u procesos que generen inflamación y posteriormente cicatrización. 
 Se diagnostica mediante histerosalpingografía o histeroscopía. La ecografía también puede ser de utilidad.
 Síndrome de ovario poliquístico (SOP): El SOP es un cuadro frecuente, y que forma parte del síndrome metabólico. Este cuadro se presenta en la adolescencia temprana, generalmente poco después de la menarquia, aunque puede afectar a mujeres adultas en edad fértil. Se estima una prevalencia del 7 al 10%.(4) Si bien se desconoce la etiología exacta, se ha postulado que el hiperinsulinismo tendría un rol fundamental, al aumentar la producción de andrógenos en la célula de la teca, que a su vez aumentan la producción de estrógenos por la granulosa, pero que finalmente es sobrepasada. El aumento de andrógenos produce el “hiperandrogenismo” y el aumento de estrógenos la poliquistosis. Además, la relación entre LH y FSH se altera (aumenta LH y disminuye FSH) con LH/FSH > 2,5.(4,8)

negativa existe falla del endometrio o tracto de salida. En cambio si es positiva hay que medir la FSH y LH que si son elevadas, significa que existe falla ovárica (5), en cambio si son normales o bajas, existe falla central.

DIAGNÓSTICO Se realiza por la clínica, que junto a los exámenes orientan la etiología.

-El síndrome de ovario poliquístico (SOP) se caracteriza por la presencia de hiperandrogenismo + oligo o anovulación o ovario poliquiístico. El Hiperandrogenismo es necesario que se encuentre presente, pudiendo ser clínico o bioquímico (4,6):

1.Anamnesis: Preguntar dirigidamente por:          

 

Desarrollo de caracteres sexuales (de no ser así, sugiere falla ovárica, hipofisiaria o anormalidad cromosómica). Antecedentes familiares de retraso o ausencia de pubertad. Signos de hiperandrogenismo (síndrome de ovario poliquístico). Estatura en relación al resto de su familia (estatura baja sugiere síndrome de Turner o enfermedad hipotalámica/hipofisiaria). Enfermedad neonatal o en la niñez (crisis neonatales sugieren hiperplasia suprarrenal congénita). Virilización (tumor ovárico o suprarrenal secretor de andrógenos). Estrés, cambios en el peso, dieta, ejercicio o enfermedad (sugieren amenorrea hipotalámica). Fármacos que causen o se asocien a amenorrea (metoclopramida, antipsicóticos, danazol, dosis altas de progestinas, corticoides).(8) Galactorrea (sugiere hiperprolactinemia). Signos de otra enfermedad hipotalámica/hipofisiaria, como dolor de cabeza, alteraciones de la visión, fatiga, poliuria o polidipsia, actividad sexual y embarazos. Síntomas de déficit de estrógenos (bochornos, depresión, artralgias, etc.). Síntomas de hipotiroidismo.

 

Hiperandrogenismo bioquímico: Testosterona >2ng/mL, Índice de Andrógenos Libres (relación entre testosterona total y SHBG) > 4,5. Hiperandrogenismo clínico: hirsutismo (puntaje en escala de Ferriman > o = 7), pubarquia precoz (antes de los 8 años), acné.

Imagen por ecografía transvaginal de ovario poliquístico. (Fuente: Carvajal Jorge A., Ralph Constanza. Manual Obstetricia y Ginecología 2014). Es diferente a la virilización, esta incluye además, cambios en la voz, alopecia androgénica, aumento de la masa muscular y clitorimegalia.(4)

2. Examen físico: Evaluar: peso, talla, IMC, desarrollo de caracteres sexuales secundarios, tamaño de clítoris, desarrollo de vello púbico, perforación del himen, profundidad vaginal y presencia de cérvix, útero y ovarios (puede ayudarse por ecografía), presencia de acné, hirsutismo, galactorrea, estrías, pigmentación incrementada y vitíligo, signos de síndrome de Turner (implantación baja del cabello, cuello corto, tórax ancho, mamilas separadas, presión arterial en ambos brazos (por coartación aórtica), talla baja, micrognatia, paladar ojival, etc).

Se considera un ovario poliquistico cuando en fase folicular temprana se encuentran mediante ecografía transvaginal (cualquiera): ≥ 12 folículos menores a 1 cm (2 a 9 mm) o volumen ovárico estimado >10mL.

TRATAMIENTO Se debe tratar la causa de la amenorrea y evitar actividades o hábitos que potencialmente pueden causar amenorrea, como son: tabaquismo, consumo de alcohol, estrés, ejercicio competitivo o extremo o bajas bruscas de peso.

3. Laboratorio -Test de embarazo: A toda mujer. Si es (+) en amenorrea primaria sospechar abuso sexual. -En caso de amenorrea primaria: ecotomografía pélvica: para evaluar la anatomía de los genitales internos. En caso de ausencia de útero, la evaluación debe incluir cariotipo y medición de testosterona plasmática.(5) En presencia de útero y sin evidencia de alteraciones anatómicas, una evaluación endocrina debe ser llevada a cabo (FSH, prolactina, etc.).

El el caso del tratamiento del SOP una de las medidas más importantes de tratamiento es la baja de peso, con lo que disminuye la androgénesis y la insulinoresistencia, y puede restaurar incluso la función ovárica. El manejo farmacológico dependerá de los síntomas y molestias de la paciente, además de si desea o no fertilidad: 

-En amenorrea secundaria: los primeros exámenes a solicitar son: b-Hcg, TSH y PRL, para descartar causas como embarazo, hipotiroidismo e hiperprolactinemia.(5) Si son negativas se sigue con la prueba de progesterona: si es positiva es una falla de la foliculogénesis. Por otro lado si es negativa la prueba se debiese hacer prueba de progesterona + estrógeno, si esta prueba es 4

Metformina: para el tratamiento de la insulinorresistencia, pero además, disminuye los niveles de andrógenos y LH, y aumenta la SHBG. Disminuye el riesgo de abortos y de Diabetes gestacional. (4)

Algoritmo diagnóstico de Amenorrea primaria. (Fuente: Carvajal Jorge A., Ralph Constanza. Manual Obstetricia y Ginecología 2014).

5

Algoritmo diagnóstico de Amenorrea secundaria. (Fuente: Carvajal Jorge A., Ralph Constanza. Manual Obstetricia y Ginecología 2014).





Anticonceptivos Orales Combinados: son útiles para la regular regular el ciclo menstrual y disminuir el hiperandrogenismo clínico. Para esto se usan ACO con progestinas antiandrogénicas como el Acetato de Ciproterona, Clormadinona y Drospirenona. En caso de que la paciente desee fertilidad se pueden dar progestinas periódicamente para regular los ciclos. (7) Antiandrógenos: además de la ciproterona, existen otros fármacos con efecto antiandrogenico como la espironolactona, flutamida, finasteride y corticoides (en hiperandrogenismo con gran componente suprarrenal) pero no están indicados de primera línea y son de manejo de especialista.(4,8)

En caso de que el problema principal sea la infertilidad producto de la anovulación se puede ofrecer:  

Induccion de la Ovulación: con Acetato Clomifeno o Gonadotrofinas. Drilling: técnica quirúrgica que consiste destruir parcialmente el ovario, con lo que disminuye la síntesis de andrógenos y estabiliza la relación LH/FSH. En retirada.(5)

de en se se

SEGUIMIENTO Derivar a especialista según sea la causa de la amenorrea.

6

a distancia por vía hematógena a hígado, pulmón y huesos. -Carcinoma indiferenciado o anaplásico. Infrecuente pero muy agresivo, con rápida progresión, infiltración de tejidos venicos (pudiendo producir disnea, disfonía, disfagia, etc.), difícil tratamiento y mortalidad cercana al 100%. -Cáncer medular (5 a 10%). Al ser de células parafoliculares, secreta calcitonina. Tiene una variante esporádica y hereditaria (que suele presentarse antes) dentro de las NEM tipo 2.

Cáncer de Tiroides Nivel de manejo del médico general: Diagnóstico: Específico. Tratamiento: Inicial. Seguimiento: Completo.

Aspectos esenciales   

Diagnóstico El examen más importante para el estudio de los nódulos tiroideos es la punción con aguja fina Ante la sospecha de un cáncer medular hereditario, descartar feocromocitoma Pronóstico depende de la edad, el tamaño y la histología.

El estudio comienza con la anamnesis y examen físico. En caso de encontrar signos que sugieran la posibilidad de cáncer, por ejemplo, nódulos tiroídeos (de ellos, 4 a 6% serán cáncer), o adenopatías, se recomienda solicitar Ecografía tiroidea, para confirmar la presencia de dichos nódulos, sus características ecográficas y establecer la presencia de otros adicionales o de adenopatías. Si se encuentran signos sospechosos a la Ecografía (hipoecogenicidad, microcalcificaciones, vascularización central, márgenes irregulares, halo incompleto, crecimiento documentado de un nódulo, etc., o presencia de adenopatías), se debe proceder a su estudio: PAAF el examen a considerar.

Caso clínico tipo Mujer de 45 años derivada por médico tratante por presentar nódulo de 1 cm palpable en región cervical izquierda. Sin otros síntomas asociados y pruebas tiroideas normales.

Clasificación de Bethesda: (1) benigno (2) lesión folicular o atipia de significancia indeterminada (3) neoplasia folicular (no distingue entre adenoma o carcinoma) (4) sospecha de malignidad (5) malignidad (6) no diagnóstica. Una vez hecho el diagnóstico, se realizará estudio imagenológico para determinar invasión linfática, hematógena o de tejidos vecinos. El sexo masculino, edad menor a 20 o mayor a 45 años, y presencia de nódulos de mayor tamaño conllevan un peor pronóstico.

Definición Neoplasia maligna del tiroides.

Etiología - epidemiología – fisiopatología Las neoplasias tiroideas pueden ser benignas (adenomas) o malignas (carcinomas, linfomas, sarcomas). El cáncer de tiroides es la neoplasia maligna más frecuente del sistema endocrinológico (aproximadamente 90%), con mayor prevalencia en mujeres. Su incidencia es de aprox. 9 cada 100.000 pacientes al año. Los cánceres tiroideos se pueden dividir según su origen:

Tratamiento El manejo debe ser realizado por especialistas. El tratamiento es quirúrgico. Para la mayoría de los carcinomas diferenciados está indicado tratamiento complementario con I131 posterior a la resección, y terapia con Levotiroxina a dosis supresoras, manteniendo MINIMOS niveles de TSH plasmática, evitando así su efecto trófico sobre las células neoplásicas. En el caso del carcinoma anaplásico se puede utilizar radioterapia externa. Es tarea del especialista, y debe ser controlado de por vida. El tratamiento es quirúrgico. Para la mayoría de los carcinomas diferenciados está indicado tratamiento complementario con I131 posterior a la resección, y terapia con Levotiroxina a dosis supresoras, manteniendo MINIMOS niveles de TSH plasmática, evitando así su efecto trófico sobre las células neoplásicas. En el caso del carcinoma anaplásico se puede utilizar radioterapia externa.

(1) derivados de las células foliculares (lo más frecuentes): carcinoma papilar (cáncer tiroideo más frecuente, 70-90%), carcinoma folicular y el carcinoma anaplásico. (2) derivados de las células parafoliculares (células C): carcinoma medular de tiroides, que representa el 5% (esporádico o familiar). (3) Otros: linfomas o sarcomas, son más raros. Factores de riesgo: irradiación externa del cuello, exposición con fines no diagnósticos o terapéuticos a I133, deficiencia de yodo, etc. Tipos de Cancer: -Carcinoma papilar (70 a 90%). 3ra y 4ta década, preferente en mujeres. Frecuentemente multifocal y bilateral diseminación linfática generalmente de buen pronóstico sólo un 10% metastiza a ganglios. -Carcinoma folicular (11%). Malignidad está dada por el compromiso capsular y/o vascular. Habitualmente se presenta como un nódulo único de crecimiento lento. MTT

Seguimiento Por especialista. En el seguimiento de carcinomas diferenciados, tiene utilidad los niveles plasmáticos de tiroglobulinas; en ausencia de glándula tiroides su existencia en el plasma (> 5ng/ml) sugiere recidiva local o MTT. Calcitonina >5ng/mlo para seguimiento de Ca medular de tiroides. 7

Hipercalcemia aguda

El cáncer es la causa más probable en un paciente hospitalizado y el HPTH primario en un paciente ambulatorio. Los tumores más frecuentes son: mama, pulmón y mieloma múltiple.

Nivel de manejo del médico general: Diagnóstico Específico Tratamiento Inicial Seguimiento Derivar

Causas de hipercalcemia aguda: Dependientes de PTH: Hiperparatiroidismo primario. Independiente de PTH: Cáncer (20%, tumores sólidos y hematológicos), exceso de vitamina D (ingesta, enfermedad granulomatosa como TBC).

Aspectos esenciales    

Causa más frecuente: cáncer (mama y pulmón) e hiperparatiroidismo primario. Importante considerar situaciones en que se deba considerar el calcio libre. Sintomatología variada, importate ECG. Tto: rexpandir VEC, aumentar excreción de calcio, disminuir acción osteoclástica y tratar causa.

Diagnóstico En la historia: antecedente familiar de hipercalcemias, investigar uso o abuso en la ingesta de leche o antiácidos (ej. síndrome de álcali-leche), tiazidas, litio, o grandes dosis de vitamina A o D, presencia de dolor óseo (mieloma múltiple, enfermedad metastásica), o dolor abdominal (pancreatitis).

Caso clínico tipo

La sintomatología en agudo depende de la etiología y del grado de la hipercalcemia. Las manifestaciones más frecuentes son: anorexia, náuseas, vómitos, poliuria y polidipsia (diabetes insípida nefrogénica), depresión, cefalea, pérdida de memoria y disfunción cognitiva, fatiga muscular de predominio proximal, hiporreflexia (Ca >15 mg/dL), confusión hasta el coma, bradicardia, BAV de 1er grado.

Hombre de 65 años con alteración del nivel de conciencia y antecedentes de neoplasia pulmonar. Al SU llega con vómitos. Refiere letargia y depresión después de un episodio de dolor abdominal por estreñimiento.

Definición

La crisis hipercalcémica es una emergencia médica caracterizada por calcemia >15 mg/dL, deshidratación severa, insuficiencia renal y compromiso de conciencia progresiva (coma), arritmias del tipo de las bradiarritmias: bloqueos de rama, bloqueo auriculoventricular completo o paro cardiaco.

Emergencia endocrina, definida por aumento de calcio total en el plasma sobre 10,5 mg/dl o calcio ionizado >5 mg/dl.

Etiología-epidemiología-fisiopatología

En el examen físico puede buscarse evidencia de neoplasias primarias (ej. pulmón, mama) o de queratopatía en banda de la córnea. Buscar hipertensión arterial, arritmias cardíacas y consignar el estado neurológico.

La concentración sérica de calcio normal va de 8.5 a 10.5 mg/dl, el 50 a 60% del calcio en la sangre está unido a proteínas del plasma (no filtra libremente) o forma complejos con citrato y fosfato. El calcio ionizado restante (libre) controla las acciones fisiológicas y filtra libremente por el riñón. Los mecanismos que mantienen la homeostasis del calcio son los hormonales (PTH y vitamina D) y los no hormonales (excreción renal y secreción intestina).

Laboratorio: niveles de calcio sérico, albúmina, fósforo, magnesio, electrolitos, BUN, creatinina, ECG buscando signos de hipercalcemia (acortamiento del intervalo QT) y arritmias.

Clasificación de severidad hipercalcemia: Leve entre 10.5 y 12mg/dl, moderada entre 12 y 14 mg/dL y severa > 14mg/dL. Sin embargo, pacientes que cursan además con hipoalbuminemia, pueden tener la calcemia subvalorada. Por lo tanto, se debe usar la siguiente fórmula para corregir la calcemia:

Tratamiento La indicación de tratamiento dependera tanto de los valores de calcio plasmático como también de la clínica del paciente, por ejemplo en pacientes con una hipercalcemia leve con concentraciones menores a 12 mg/dL no requeriran un tratamiento inmediato pero si la suspensión de factores agravantes como el uso de diuréticos tiazídicos o carbonato de litio, sumado a una hidratación importante a lo largo del día para prevenir Nefrolitiasis. Em el otro extremo tenemos pacientes con concentraciones mayores a 14 mg/dL que si requeriran un tratamiento mas agresivo debido a las posibles complicaciones.

Calcio libre= Calcemia total + 0,8 x [4-Albuminemia (g/dl)] Para que se produzca una crisis hipercalcémica debe asociarse a hidratación insuficiente en pacientes no autovalentes o a alteración del mecanismo de la sed, lo que lleva a deshidratación, depleción de volumen lo que agrava la hipercalcemia. La acidosis produce un aumento de la fracción del calcio ionizado. Más del 90% de los casos se deben a hiperparatiroidismo primario o enfermedades malignas. 8

Fundamentos del tratamiento: 1. Aumentar VEC, aumentar la excreción urinaria de calcio 2. Disminuir acción osteoclástica en el hueso 3. Disminuir absorción intestinal de calcio 4. Tratar la causa de hipercalcemia Se indica: Expansión de volumen: Solución de NaCl al 0,9% 200300 ml/hora que luego se ajusta para mantener una diuresis de 100-150 ml/hora (precaución en ancianos y cardiópatas). Suspender fármacos que aumenten la calcemia, como digitálicos, diuréticos tiazídicos, vitamina D u otro. Calcitonina 4 UI/kg, con control de electrolitos posterior para definir nuevo uso de dosis de calcitonina cada 6 o 12 horas. Ácido zoledrónico 4 mg ev a pasar sobre 15 minutos. No se recomienda el uso de diuréticos de asa en ausencia de falla renal o cardíaca. Glucocorticoides (en caso de cánceres hematológicos, sarcoidosis, intoxicación por Vitamina D, no en hiperparatiroidismo primario): Se usa Hidrocortisona 100 mg EV c/8 horas. Hemodiálisis: en casos graves extremos con tratamiento etiológico siempre. Caso de arritmia ventricular, IRA, falla de medidas anteriores, calcemia entre 18 y 20 mg/dL y/o síntomas neurológicos.

Seguimiento Derivar.

9



Hipercalcemias e hipocalcemias

Hipocalcemia. Manifestaciones clínicas: la sintomatología depende en gran parte de si se trata de un cuadro de instalación crónica o aguda. Las más frecuentes de la hipocalcemia crónica son:

Nivel de manejo del médico general: Diagnóstico: sospecha. Tratamiento: inicial. Seguimiento: derivar.

      

Aspectos esenciales     

Calcemia normal: 8,5-10,5 mg/dl. Corregir calcemia según albúmina séricia. Principal causa de hipercalcemia hiperparatiroidismo. Tto. hipercalcemia: según la causa Tto. hipocalcemia: reposición de calcio.

es



Paciente de 75 años diagnosticado con carcinoma de próstata, que acude a la Urgencia por confusión mental, náuseas, vómitos y constipación. Presenta una calcemia de 15 mg/dL ¿Cuál es la primera decisión terapéutica que es preciso tomar? Solución salina y furosemida por vía ev.

Tratamiento En hipercalcemias leves y moderadas no se requiere tratamiento inmediato, educar para disminuir factores que aumentan la glicemia e hidratación adecuada. En hipercalcemias severas requieren terapia más agresiva, solución salino isotónica 200-300 ml/hora que luego se ajusta para mantener una diuresis de 100-150 ml/hora, calcitonina 4 UI/kg, ácido zoledrónico 4 mg ev a pasar sobre 15 minutos. No se recomienda el uso de diuréticos de asa en ausencia de falla renal o cardíaca. En la hipocalcemia crónica el objetivo terapéutico es mantener la calcemia cercana a 8,5 mg/dL y evitar la hipercalciuria (que puede producir nefrolitiasis, nefrocalcinosis y falla renal). El tratamiento se basa en la suplementación de calcio en forma de carbonato o citrato de calcio y vitamina D. La dosis de calcio requerida inicialmente es de 1.500 a 2.500 mg de calcio elemental, con un rango de 1.000 a 3.000 mg cada día, dado como carbonato de calcio (en 3 a 4 dosis) o citrato de calcio, diariamente, más vitamina D en caso de deficiencia (la dosis depende del grado de déficit; dosis usual de vitamina D2 50.000 U/semana VO; dosis usual de vitamina D3 1.000 U/día). En hipocalcemia aguda el objetivo terapéutico es aumentar la calcemia en 2–3 mg. Se utiliza gluconato de calcio al 10%, 1 ampolla (10 mL) diluida en 90 mL de SG 5% EV lento (10 a 30 mL a pasar en 10 min). Se puede repetir a las 4 horas, hasta que la tetania desaparezca. Una ampolla de gluconato de calcio de 10 mL al 10% aporta 90 mg de calcio elemental. Además, se debe corregir la posible magnesemia e iniciar conjuntamente calcio y vitamina D oral.

Definición Clásicamente, se define como hipocalcemia niveles de calcio total <8,5 mg/dL e hipercalcemia con niveles >10,5 mg/dL pero, en la actualidad, las técnicas de laboratorio automatizadas en uso tienen un rango normal que va de 8.2-10.2mg/dl. Se considera hipercalcemia leve con valores <12, moderada entre 12 y 14 y severa >14 mg/dL. El calcio que comúnmente se mide en los laboratorios corresponde al calcio total, es decir, el que se encuentra unido a albúmina y globulinas, por lo tanto, cuando existe hipoalbuminemia se puede obtener una calcemia falsamente normal o disminuida. Para obtener el valor real, se corrige con la fórmula: Calcemia real (mg/dl) = Calcemia medida (mg/dl)+ 0,8 x [4 -Albuminemia (g/ dl)].

Diagnóstico Hipercalcemia. Los síntomas dependen de la etiología, patologías asociadas, tiempo de evolución y niveles de calcemia. La presentación más frecuente es la asintomática. Las manifestaciones clínicas más frecuentes son:

   

Calcificación ectópica (de ganglios basales) Signos extrapiramidales Parkinsonismo Catarata subcapsular Demencia Anormalidades en la dentición Piel seca y aspera; cabello frágil, quebradizo y alopecia; uñas quebradizas, dermatitis exfoliativa. Signos de Chvostek y Trousseau

El diagnóstico de la hipocalcemia se basa en las manifestaciones clínicas, los hallazgos electrocardiográficos y la titulación de los niveles séricos de calcio. Con respecto al laboratorio, se caracteriza por hipocalcemia e hiperfosfatemia, y, dependiendo de la causa, alteración en los niveles de PTH, magnesio, vitamina D, etc.

Caso clínico tipo



Otras: HTA, acortamiento del QT, bradicardia, BAV de primer grado, artralgias, queratopatía en banda, anemia normocítica normocrómica.

Neuropsiquiátricas: fatiga, debilidad, neuropatía periférica, depresión, psicosis, somnolencia, parestesia, coma. Neuromusculares: debilidad muscular proximal, mialgias, atrofia muscular. Gastrointestinales: dolor abdominal, anorexia, náuseas, vómitos, constipación, úlcera gastroduodenal, pancreatitis. Óseas: dolor óseo, osteoporosis, osteítis fibrosa, reabsorción subperiostal. Renales: hipercalciuria, poliuria, polidipsia, nefrolitiasis, nefrocalcinosis.

Seguimiento Derivar a especialista.

10

Etiología

11

Pasos fundamentales en el diagnóstico:

Hiperprolactinemia

1. Interrogar acerca de la posibilidad de embarazo, fármacos, síntomas neurológicos y de otras alteraciones hormonales 2. Determinación de PRL basal matutina en ayunas, el que puede ser repetido si la sospecha es fuerte, debido a la variación natural de concentraciones de PRL. Si se detecta hiperprolactinemia leve se realizará un segundo análisis. Se solicitará también TSH y función renal. 3. Realizar neuroimagen, de elección es la RNM, para explorar la región supra y paraselar.

Nivel de manejo del médico general: Diagnóstico: sospecha. Tratamiento: inicial. Seguimiento: derivar.

Aspectos esenciales 

Cifras de la prolactina entre 100-200 ng/ml nos deben hacer pensar en un tumor.

Tratamiento El objetivo de tratar la hiperprolactinemia es corregir las consecuencias clínicas de su exceso: restauración de la función gonadal y/o fertilidad y prevenir la osteoporosis. Si existe tumor, se deberán corregir las alteraciones visuales, preservar la función hipofisiaria y prevenir la progresión de la enfermedad; por tanto las necesidades de los pacientes depende de distintos factores. En la sospecha está indicada la derivación a especialista para el estudio y tratamiento. Éste último consiste en agonistas dopaminérgicos (Bromocriptina, Cabergolina, etc.) como primera línea, y en caso de resistencia, intolerancia o imposibilidad de mantener el óptimo tratamiento, o de compresión del quiasma óptico, se indica cirugía y eventualmente radioterapia. En la mujer embarazada, el tratamiento de prolactinomas debe comenzar antes de la concepción, advirtiendo a la mujer y su pareja sobre los riesgos del embarazo para ella y el feto, principalmente el crecimiento del adenoma y el riesgo potencial del feto al exponerse a agonistas dopaminérgicos. Si es necesario disminuir la concentración sérica de prolactina para permitir la ovulación, se recomienda el uso de bromocriptina o cabergolina. En caso de macroadenomas, estos deben ser tratados previo al embarazo. Durante el embarazo, las mujeres con micro o macroadenoma debe evaluarse cada 3 meses, en búsqueda de cefalea y cambios en el campo visual, y en caso de pesquisar, se debe recurrir al estudio con RNM, con el consiguiente manejo terapéutico más adecuado.

Caso clínico tipo Paciente mujer de 38 años. Consulta por cuadro de amenorrea, galactorrea y cefalea de días de evolución. En el último día, ha presentado hemianopsia bitemporal. Al laboratorio, se constata Prolactina de 250 mg/dL. Se toma una RM que muestra un macroprolactinoma.

Definición Secreción inadecuadamente elevada de prolactina, por diversas causas.

Etiología Compendio de medicina interna. Rozman. V Edición. Elsevier.

Diagnóstico La sintomatología del cuadro depende del paciente y, al igual que todos los tumores hipofisiarios e hipotalámicos, puede producir hipofunción glandular y asociarse o no a manifestaciones de compresión de estructuras vecinas: en mujeres premenopáusicas se presenta generalmente como oligomenorrea o amenorrea, galactorrea, infertilidad o disminución de libido, asociado a aumento de masa corporal y disminución de la densidad ósea; en mujeres posmenopáusicas los síntomas son menos frecuentes y pueden manifestarse por síntomas derivados del efecto de masa, como cefalea o alteración de la visión; en hombres, suele presentarse como disminución de la libido, impotencia, eyaculación precoz, infertilidad, siendo poco frecuente la galactorrea. Ante la presencia de niveles elevados de prolactina (PRL), lo primero es descartar una posible causa secundaria. Realizado esto, se debe solicitar evaluación imagenológica (RNM) que descarte una posible lesión local responsable de los síntomas, como tumores hipofisiarios o hipotalámicos, sean o no productores de PRL, o bien, produzcan compresión del tallo de la hipófisis por extensión supraselar.

Seguimiento El seguimiento debe ser realizado por el especialista, por lo que se debe derivar cuando se sospeche.

12

Etiología Compendio de medicina interna. Rozman. V Edición. Elsevier.

Causas de Hiperprolactinemia Enfermedades Tumores: craneofaringiomas, hamartomas, germinomas, metástasis hipotalámicas Enfermedades infiltrativas: sarcoidosis, histiocitosis de células de Langerhans, leucemia, tuberculosis Seudotumor cerebral Lesiones: traumatismos, irradiación Sección física o Traumatismo, compresión tumoral, etc. funcional del tallo hipofisiario Enfermedades Prolactinoma hipofisiarias Tumores mamosamatotropos Tumores mixtos productores de GH y PRL Enfermedad de Cushing Tumor secretor de TSH Enfermedades infiltrativas Otros tumores intraselares Fármacos Bloqueantes de H2: neurolépticos, metoclopramida, domperidona, sulpirida Antidepresivos: tricíclicos y no tricíclicos, inhibidores de la MAO o de la recaptación de serotonina Antihipertensivos: verapamilo, metildopa, reserpina Inhibidores de proteasas. Otras causas Hipotiroidismo primario Insuficiencia renal crónica Cirrosis hepática Lesiones irritativas de la pared toráxica Estímulo de la mama Ovario poliquístico Estrés Seudociesis Hiperprolactinemia idiopática GH: Hormona del crecimiento; MAO: monoaminoxidasa; PRL: prolactina; TSH: hormona tirotropa

13



Hipogonadismo masculino

b) Hipofisiario (secundario) FSH, LH: o

Nivel de manejo del médico general: Diagnóstico: Específico. Tratamiento: Inicial. Seguimiento: Completo.

o o o

Aspectos esenciales    

Hipogonadismo hipergonadotropo: 

Síntomas y signos dependientes de falta de hormonas sexuales en hombres. Es importante determinar el nivel para estudiar posible etiología. La terapia se realiza con sustitución de testosterona. Es importante monitorizar parámetros asociados a retención hidrosalina, riesgo de apnea de sueño e hiperplasia prostática.

o o 

o

orquitis

Se realiza principalmente por clínica, se debe averiguar en anamnesis: edad de inicio, pubertad retrasada, frecuencia afeitado disminuida, anosmia, antecedentes de criptorquidia, parotiditis, TBC, traumatismos, cirugía, radiación.

Síndrome clínico que resulta de la falla en algún nivel del eje hipotálamo-hipófisis-gonadal (HHG). Los testículos son insuficientes para producir niveles fisiológicos de testosterona circulante y/o espermatozoides en número y calidad suficientes para lograr fertilidad.

Al examen físico: talla, pubis suelo, pubis cabeza, envergadura, características de la piel, distribución del tejido adiposo, caracteres sexuales secundarios, ginecomastia.

Las anormalidades del eje HHG pueden ser a nivel gonadal (hipogonadismo hipergonadotropo), hipofisiario o hipotalámico (hipogonadismo hipogonadotropo), y pueden presentarse en la etapa pre o postpuberal.

Se diagnostica hipogonadismo primario (hipergonadotrópico) cuando las concentraciones de testosterona se encuentran bajas y las gonadotropinas se encuentran elevadas; hipogonadismo secundario (hipogonadotrópico) cuando las concentraciones de testosterona y de gonadotropina están bajas.

Etiología-epidemiología-fisiopatología En el caso del hipogonadismo primario, el síndrome de Klinefelter (cariotipo 47, XXY) es la causa más frecuente (<1 en 1.000 nacimientos masculinos). Causas genéticas, alteraciones en la biosíntesis o acción de los andrógenos son poco comunes

Las manifestaciones de déficit de testosterona están determinadas por el momento de aparición durante el desarrollo reproductivo: 

El síndrome de Kallmann se debe al mal desarrollo de las neuronas productoras de GnRH y se caracteriza por bajos niveles de LH y FSH asociado a anosmia. Otros deficits de GnRH o de gonadotropinas se presentan sin anosmia.

 

Hipogonadismo hipogonadotropo: a) Hipotalámico (terciario) GnRH:

o

Traumatismos o cirugía. Enfermedades inflamatorias: urliana, TBC. Radiación, quimioterapia.

Diagnóstico

Definición

o o

Disgenesia gonadal: Sd de Klineffelter (47XXY). Deficiencias enzimáticas (17-alfa hidroxilasa). Criptorquidea.

b) Postpuberal o o

Paciente hombre de 26 años, consulta por sospecha de infertilidad. Al examen físico, destaca escaso desarrollo de genitales, con ausencia de vello pubiano y axilar, ginecomastia y distribución adiposa de tipo ginecoide.

o o

a) Prebuberal o

Caso clínico tipo



Panhipopituitarismo: Tumores, cirugía, radioterapia, apoplejía hipofisiaria, hipofisitis. Déficit aislado de gonadotrofinas o asociado a déficit de GH. Otras causas endocrinas: hiperprolactinemia, hipercortisolismo. Abuso de anabólicos.



Lesiones o alteraciones funcionales. Sd genéticos: Prader-Willi, Kahlman (asociado a anosmia). Tumores: Craniofaringioma. Lesiones infiltrativas: sarcoidosis, hemocromatosis Abuso de anabólicos

Primer trimestre intrauterino: Virilización incompleta de los genitales externos, desarrollo incompleto de los conductos de Wolff para formar los genitales internos masculinos. Tercer trimestre: Micropene. Prepuberal: Maduración puberal incompleta, hábito eunucoide, pobre desarrollo genital y muscular, voz infantil y peak de densidad ósea disminuido. Pospuberal: Disminución de la energía, ánimo y libido; disminución de vello sexual, hematocrito, masa muscular y fuerza, y densidad mineral ósea.

En el laboratorio, se debe solciitar testosterona total am (niveles normales: 200 a 800 ng/dL), FSH/LH (en caso de niveles de testosterona bajo lo normal en 2 ocasiones). Habitualmente en los hipogonadismos hipogonadotropos, si corresponde, es conveniente medir prolactina 14

(descartar hiperprolactinemia), ferremia (hemocromatosis). En varones con hipogonadismo hipergonadotropo se agrega un espermiograma y, si corresponde, cariograma para evaluar cromosomas. El cariograma permite evaluar la presencia de cromosompatías, como 47XXY. Se agrega estudio de imágenes en caso de necesidad.

Tratamiento Terapia de sustitución con testosterona.

Seguimiento Si se reemplaza a los pacientes claramente hipogonádicos deben controlarse factores como el hematocrito, perfil lipídico, factores cardiovasculares asociados a retención hidrosalina, monitorizar el riesgo de apnea del sueño y monitorizar los riesgos asociados a la hiperplasia prostática. Se debe derivar a especialista.

15

Para el Síndrome de ovario poliquístico, la etiología más frecuente (75 a 80% de los casos), el o los criterios diagnósticos propuestos son:

Hirsutismo Nivel de manejo del médico general: Diagnóstico: Específico. Tratamiento: Inicial. Seguimiento: Completo.

Aspectos esenciales    

El hirsutismo se refiere al aumento de vello corporal en áreas dependientes de andrógenos. La causa más frecuente es el idiopático, producido por un aumento de la sensibilidad de la unidad pilosebácea a andrógenos circulantes. Existe tratamiento farmacológico y cosmético. Seguimiento se realiza con la escala de Ferriman - Gallway.

Criterios de consenso NIH, 1990 (se requieren todos los criterios)

Criterios de Rotterdam, 2003 (se requieren 2 de 3 criterios)

Irregularidad en menstruaciones , debido a oligo o anovulación

Oligo anovulación

Evidencia de hiperandrogeni smo clínico y/o bioquímico

Caso clínico tipo Exclusión de otras causas: tumores secretores de andrógenos o Hiperplasia suprarrenal congénita no clásica

Paciente mujer, consulta por aparición de vello en el mentón, tórax y línea infraumbilical de 9 meses de evolución, además de aumento progresivo de peso en el último tiempo. Al examen físico: pesa 86Kg, mide 1,59m, IMC 43, percentil IMC >95. Perímetro de cintura 110cm. Perímetro de cadera 115cm. Se aprecia acné en la cara, y acantosis nigricans en el cuello.

Definición

Definición de AES, 2008 (se requieren todos los criterios)

o Hiperandrogeni smo clínico o bioquímico (exceso de andrógenos) Signos clínicos Disfunción o bioquímicos ovárica: oligo o de anovulación y/o hiperandrogeni ovarios smo poliquísticos (por Ecografía) Ovarios Exclusión de poliquísticos otras (por alteraciones Ecografía) ovulatorias o causas de exceso de andrógenos.

Tratamiento

Excesiva presencia de pelo terminal en la mujer, en áreas asociadas a madurez sexual masculina.



Etiología - epidemiología – fisiopatología Las causas se pueden clasificar en 3 grupos:



1) Hiperandrogenismo endógeno: Síndrome de Ovario Poliquístico (SOP, 10% mujeres fértiles), tumores adrenales u ováricos, hiperplasia suprarrenal del adulto. 2) Hiperandrogenismo exógeno: Fármacos como danazol, esteroides anabólicos, terapia de reemplazo hormonal con andrógenos. 3) Aumento de la sensibilidad androgénica de la unidad pilosebácea: Hirsutismo idiopático. Aumento del metabolismo de los andrógenos en la piel y folículos pilosos por mayor actividad de la alfa reductasa.

Tratamiento Farmacológico: La primera línea son los anticonceptivos orales. Efectivos son el Acetato de Ciproterona asociado a Etinilestradiol, y también la Espironolactona, pero son tratamientos manejados por especialista. Tratamiento No Farmacológico: Se usa solo, o asociado a terapia farmacológica. Incluye depilación y decoloración. No existe evidencia que compare su eficacia con el tratamiento farmacológico, ya que más bien es un tratamiento cosmético.

Seguimiento El seguimiento se realiza con la escala de FerrimanGallwey. En caso de ajuste de tratamiento farmacológico o progresión del hirsutismo, se debe derivar a especialista.

Diagnóstico La Escala de Ferriman-Gallwey evalúa la cantidad de pelo existente en mujeres en áreas propiamente masculinas (bigote, barba, tórax superior, línea alba, región suprapúbica, antebrazos, muslos anteriores, tórax posterior parte alta y baja). Utilizando una puntuación de 1 a 4, según la severidad del hirsutismo, las mujeres se puede clasificar en: I. Normal: < 6 puntos II. Hirsutismo leve: 6 – 15 puntos III. Hirsutismo moderado: 16 – 25 puntos IV. Hirsutismo severo: > 25 puntos

16

Etiología - epidemiología – fisiopatología

Síndrome climatérico

El periodo perimenopáusico corresponde al periodo de tiempo comprendido entre la aparición de las primeras alteraciones hormonales y del ciclo menstrual y el transcurso de 12 meses de amenorrea (diagnóstico de menopausia). Inicialmente, este periodo se caracteriza por un aumento variable en la duración del ciclo menstrual, acompañado de un aumento en los niveles plasmáticos de FSH (consecuencia de una disminución de la inhibina ovárica por el bajo número de folículos) con LH y estrógenos normales. Conforme disminuye el número de folículos, se instala una insuficiencia glandular ovárica, disminuyendo importantemente la secreción de estradiol, a la par con un aumento de FSH y LH por la pérdida del mecanismo de retroalimentación negativa. A su vez, las irregularidades del ciclo menstrual se hacen más pronunciadas, alcanzando finalmente la amenorrea. De esta manera, las alteraciones mencionadas se consolidan con la depleción de los folículos primordiales, dando paso al periodo postmenopáusico.

Nivel de manejo del médico general: Diagnóstico: Específico. Tratamiento: Completo. Seguimiento: Completo.

Aspectos esenciales    

El aumento de FSH constituye la alteración hormonal más precoz en el climaterio. El síndrome climatérico es fundamentalmente provocado por la disminución del estradiol. La terapia de reemplazo hormonal tiene importantes riesgos que deben considerarse al momento de iniciar el tratamiento. Nunca se deben dar estrógenos sin asociar progestágenos, excepto en histerectomizadas (riesgo de cáncer de endometrio).

Diagnóstico Caso clínico tipo

El diagnóstico es clínico, pudiendo apoyarse en determinaciones hormonales si fuera necesario. Las manifestaciones clínicas dependen fundamentalmente del descenso en los niveles de estrógeno, encontrándose:

Mujer 51 años, hipertensa, sin otros antecedentes relevantes. Consulta por bochornos y palpitaciones de 4 meses de evolución. Al interrogatorio refiere amenorrea de 7 meses de evolución, irritabilidad e insomnio.



Definición Síndrome provocado por el hipoestrogenismo existente durante el periodo peri y posmenopáusico (Hipogonadismo hipergonadotrófico: FSH > 40 mU/mL y estradiol < 20 pg/mL).

 

En torno al síndrome climatérico hay varias conceptos a destacar: 







 

Climaterio: Fase que comprende la transición de un estado reproductivo a uno no reproductivo. Es un periodo caracterizado por la claudicación de la gónadas femeninas que comienza con las disminución de la fertilidad y termina insensiblemente con la senectud. Síndrome climatérico: Síntomas asociados a la fase de climaterio: vasomotores y neurovegetativos, somáticos, cognitivos y emocionales, y genitourinarios. Menopausia: Cese permanente de la menstruación (última menstruación ocurrió hace más de 12 meses), resultado de una pérdida de la actividad folicular ovárica. Ocurre de manera fisiológica entre los 45 y 56 años. Temprana:40-45 años. Precoz: antes de los 40 años. Tardía: después de los 56 años. Perimenopausia: Periodo desde el comienzo de las manifestaciones clínicas, biológicas y endocrinológicas de la aproximación a la menopausia y 12 meses posterior al último periodo menstrual. Premenopausia: Periodo reproductivo desde la menarquia hasta el último ciclo menstrual. Postmenopausia: Periodo desde el último ciclo menstrual.



Síntomas vasomotores y neurovegetativos: Bochornos (que pueden ocurrir a cualquier hora del día) y sudoración nocturna son los síntomas más frecuentes. Además se mencionan:palpitaciones, parestesias, sicca, vértigo, etc. Somáticos: Dolor osteoarticular, fatigabilidad, astenia, cefalea. Genitourinarios y sexuales: incontinencia urinaria, urgencia miccional, disuria, prurito y sequedad vaginal, infecciones urinarias, infecciones vaginales, disminución de la libido, aversión o evitación del contacto sexual; dificultad, retardo o ausencia en alcanzar orgarsmos; dispareunia, vaginismo, alteraciones dolorosas sexuales no-coitales. Síntomas cognitivos y emocionales: disminución de la memoria reciente y semántica, ánimo depresivo, ansiedad, irritabilidad, cambios de humor, insomnio, letargia y falta de energía.

Además, puede aparecer alteraciones corporales (piel fina y seca, aumento de tejido adiposo, pérdida de masa muscular, rigidez de articulaciones, etc.), osteoporosis y aumento del riesgo cardiovascular (pérdida de la protección estrogénica). En mujeres normales sobre 45 años, el diagnóstico de transición menopáusica o perimenopausia no exige medir niveles de FSH. El diagnóstico de menopausia se hace tras 12 meses de amenorrea, en ausencia de otras causas biológicas o psicológicas. Tampoco es necesario medir niveles de FSH para el diagnóstico. En cuanto al laboratorio, las alteraciones hormonales conforman un hipogonadismo hipergonadotrófico: FSH > 40 mU/mL y Estradiol < 20 pg/mL.

17

Primer control médico:         



Además de:    

Previo a la entrada al box se debe entregar formulario MRS y complementario. Anamnesis próxima y remota. Análisis de MRS y formulario complementario. Aplicación de FRAX u ORAI EMPA Cálculo de riesgo cardiovascular según programa ministerial. Detección de depresión. Examen físico general (no olvidar IMC y circunferencia de cintura), examen ginecológico. Solicitud de exámenes: según orientaciones del programa de salud cardiovascular, mamografía según guía de cáncer de mama, hemograma y VHS si hay sangrado anómalo, perfil lipídico, VDRL o RPR - VIH, TSH, FSH (en casos que se quiera evaluar fertilidad en la perimenopausia), Orina completo, ecotomografía ginecológica de preferencia transvaginal, según protocolo. Educación y consejería: Se debe realizar educación en cambios de hábito hacia una vida saludable, con una dieta adecuada, dejar de fumar y hacer ejercicio.

Tratamiento - Tratamientos no hormonales: Calcio, vitamina D, lubricantes, etc. - Tratamientos hormonales: 

Indicaciones de TRH en posmenopausia: En general, la TRH está indicada sólo en pacientes sintomáticas (se traduce en MRS> o = 15, también es adecuada en MRS > o = 8, MRS somático > o= 8, MRS psicológico > o =6, MRS urogenital > o =3), en la mínima dosis efectiva y por el menor tiempo posible, ya que su uso presenta importantes riesgos, que forman parte de las contraindicaciones, pero, esto se debe evaluar caso a caso. Siempre se debe contar con una mamografía antes de indicar TRH. Se contraindica el tratamiento en caso de pacientes con patología coronaria, cáncer de mama, antecedentes de trombosis venosa profunda o accidente cerebrovascular, enfermedad hepática activa o quienes están en alto riesgo para estas complicaciones. El principal beneficio de la TRH durante el climaterio es la disminución del riesgo de osteoporosis. El uso de estrógenos en forma aislada aumenta la incidencia de cáncer de endometrio, por lo que siempre debe acompañarse de progestágenos (excepto en histerectomizadas). El Raloxifeno es un modulador selectivo de receptores estrogénicos (SERM) que presenta efectos agonistas a nivel de hueso, prestando utilidad en la prevención de la osteoporosis.



Déficit hormonal en la perimenopausia. Insuficiencia lútea o con ciclos monofásicos: la resposición progestínica de la fase lútea se realiza para prevenir alteraciones del sangrado uterino y proliferación endometrial. No evita embarazos. Administración por 10-14 días de manera cíclica (progesterona micronizada, dihidropregesterona, nomegestrol, desogestrel, medroxiporgesterona acetato).



Reposición estrogénica en la perimenopausia: En mujeres sin útero: estradiol transdérmico, estrógeno oral. En mujeres con útero: valerato de estadiol+progestina cíclica, estradiol micornizado+progestina cíclica, estradiol transdérmico+progestina cíclica.

Segundo control médico: Agendado a los tres meses como máximo. Detección de riesgos:    

Revisar, registrar e interpretar resultados de exámenes. Realizar apgar familiar y genograma. Indagar sobre violencia intrafamiliar y facilitar formularios en casos sospechosos. Identificar riesgos personales y clasificar estado de salud: a) Sana. b) Con morbilidad de baja complejidad. c) Con morbilidad de alta complejidad.

Al médico general le compete el manejo de las pacientes sanas y con baja complejidad, que serían las siguientes pacientes:             

Definición de plan de atención personalizado. Coordinar con los programas pertinentes. Acciones educativas y consejería. Siempre promover el cambio a un estilo de vida saludable: dieta adecuada, ejercicio, aporte de calcio y vitamina D, exposición solar (con moderación) y evitar café, alcohol y tabaco.

Hipertensión controlada y sin daño de parénquima. Diabetes compensada y sin daño de parénquima. Varicoflebetis. Antecedente de cirugía vascular venosa superficial. Antecedentes de sólo una trombosis superficial. Mioma uterino asintomático. Riesgo cardiovascular moderado o síndrome metabólico ATP-III< 3 puntos. Obesidad con IMC < 40 sin comorbilidad. Alteración del flujo rojo en perimenopáusica. Osteopenia u osteoporosis sin contraindicación de TRH. Antecedentes de epilepsia sin tratamiento actual. Trastorno de ansiedad. Depresión, consumo perjudicial o dependencia de alcohol y otras drogas.

18

Contraindicaciones para iniciar o mantener TRH. - Absolutas     

Sangrado inexplicado. Cáncer de endometrio activo. Cáncer de mama presente o pasado. Tromboembolismo pulmonar. Hepatopatía aguda.

- Relativas   

Antecedente de tromboembolismo. LES. Porfiria.

Interconsultas cuando corresponda.    

Asistente Social. Salud Mental. Kinesiológico (piso pélvico). Nutrición.

Seguimiento Tercer control médico: Si la paciente está en TRH o presenta factores de riesgo que requieren mayor seguimiento, se debe controlar a los 6 meses desde el último control. Controles posteriores cada un año. Evaluar resultado de intervenciones, repetir MRS, adecuar intervenciones según resultado de intervenciones previas. En el caso de pacientes no usuarias de TRH tercer control al año. Las pacientes que requieren TRH y presentan morbilidad de alta complejidad deben ser derivadas a nivel secundario:          

     

Hipertensión arterial descompensada o con daño de parénquima. Riesgo alto de enfermedad tromboembólica. Diabetes descompensada o con daño de parénquima. Antecedentes de más de una trombosis profunda. Antecedente familiar de trombosis o trombofilia diagnósticada. Menorragia o hipermenorrea asociada que persiste después del tratamiento inicial. Riesgo cardiovascular ATP-III>3. Mioma sintomático o mayor de 8 cm. Epilepsia en tratamiento. Antecedente personal de tumor o cáncer estrógeno/progesterona dependiente (mama, endometrio, melanoma, colon, cáncer de ovario de células claras, cáncer hepático, meningioma). Mesenquimopatías. Prolapso genital sintomático. Incontinencia de orina o urgencia miccional que falla a tratamiento inicial. Patología de mama. Patología cervical. Depresión grave con síntomas psicóticos, alto riesgo suicida o refractariedad.

19

transfusiones de sangre con citrato; en tales pacientes el calcio iónico puede disminuir (hipocalcemia verdadera), pero en otros la hipocalcemia total se debe a hipoalbuminemia. La alcalosis favorece la unión del calcio a proteínas y se debe cuantificar calcio iónico. Calcemias entre 7 y 8 mg/dL pueden no provocar signos típicos de tetania espontáneamente, pero pueden ser provocados por el médico (tetania latente).

Tetania Nivel de manejo del médico general: Diagnóstico Específico Tratamiento Inicial SeguimientoCompleto

Diagnóstico

Aspectos esenciales    

Se realiza por la clínica y con los exámenes y la anamnesis se orienta a la etiología.

Contracción muscular prolongada, puede provocar una posición alterada o pequeños movimientos. Causado por hipocalcemias bajo 7 mg/dL. Son característicos de la tetania los signos de Chvostek y Trousseau. Tto inicial de formas agudas sintomáticas: gluconato de calcio endovenoso.

Clínica: En la etiología por hipocalcemia, en un comienzo hay adormecimiento en labios, parestesias en manos y pies, calambres musculares, paresias a la movilidad fina y disminución de capacidad táctil, seguido de espasmo musculares inducidos o espontáneos: signo de Chvostek (contracción unilateral de músculos faciales tras percusión sobre nervio facial) y signo de Trousseau (espasmo carpopedal tras compresión con manguito apenas por sobre la PAS). Si no es tratado, puede evolucionar a convulsiones generalizadas y laringoespasmo. A nivel cardiovascular se presenta con prolongación del intervalo QT, hipotensión, insuficiencia cardíaca y arritmias. En la etiología por Clostridium tetanii existe el antecedente de herida penetrante o sucia, manifestándose posteriormente (Periodo de incubación entre 2 a 38 días, actuando en promedio entre los 7 a 10 días posterior a la realización de la herida) con fiebre héctica, hiperactividad autonómica (arritmias, hipertensión lábil) seguido de hipertonía muscular. El tétanos tiene presentaciones clínicas locales, generalizados y cefálicas.

Caso clínico tipo Paciente femenino, 53 años, acude a servicio de urgencia con signos evidentes de tetania, Trousseau espontáneo y Chvostek franco a la maniobra semiológica. Al examen físico se observa cicatriz de tiroidectomía.

Definición Se define como una contracción de un músculo o grupo muscular prolongada que puede provocar desde una posición alterada a un pequeño movimiento, causada por irritabilidad neuromuscular o la acción de una toxina o fármaco.

Laboratorio: calcemia total (siempre se debe corregir por albúmina), y en pacientes graves o desnutridos pedir calcio iónico en plasma

Etiología - epidemiología – fisiopatología Etiología: Las causas más comunes de tetania son aquellas que afectan el metabolismo del calcio, produciendo hipocalcemia; hipoparatiroidismo de cualquier etiología, hiperfosfemia, hipomagnesemia, alcalosis respiratoria. Otra causa es la toxina de Clostridium tetanii, patógeno que vive normalmente en el suelo y que por medio de una puerta de entrada (Heridas principalmente), infecta al organismo impidiendo la acción de interneuronas inhibitorias a nivel medular por medio de metaloproteasas.

Tratamiento Hipocalcemia: Formas agudas hipocalcémicas sintomáticas se tratan inicialmente con 10 a 30 mL de solución de Gluconato de calcio al 10%, diluídas en 100 ml de SG 5%, infundidas EV, en 10 minutos. Hipocalcemias persistentes obligan al goteo intravenoso constante de 15 a 20 mg de calcio elemental por Kg de peso en 1.000 mL de SG 5% o SF en 12 a 24 horas (10 ampollas de gluconato de calcio = 900 mg de calcio). En caso de hipomagnesemia, suplementar con Sulfato de magnesio. La hipocalcemia crónica por hipoparatiroidismo se trata con suplementos de calcio oral (1 a 3 g c/8h) más vitamina D (por ejemplo, colecalciferol 50.000 U VO/día o, Ergocalciferol 600000 U cada 1-2 semanas) o Calcitriol (0,25 a 2 ug/día). El objetivo terapéutico es calcemia alrededor de 8.5 mg/dl y calciuria <400 mg/24h, para evitar hipercalciuria y nefrolitiasis.

Epidemiología: En Chile, la hipocalcemia se observa como una de las complicaciones más frecuentes de la cirugía tiroidea, a causa de un hipoparatiroidismo posquirúrgico, transitorio o permanente. Debido a esto es que actualmente se indica aportes de calcio oral. En estudios realizados en Chile, en pacientes sometidos a tiroidectomía total, encontramos hipocalcemia posoperatoria hasta en 50% de los casos, la mayor parte asintomáticos u oligosintomáticos, no se observaron hipocalcemias severas asociadas a tetania.

Tétanos: El objetivo es evitar su aparición, que se puede hacer con una adecuada profilaxis y manejo de heridas. Se adjunta esquema de profilaxis con vacuna antitetánica MINSAL:

Fisiopatología: Hipocalcemia de tipo crónica, suele ser secundaria a deficiencias de forma iatrogénica posterior a cirugía de tiroides (cuando hay daño también en las glándulas paratiroides). Se manifiesta hipocalcemia transitoria en caso de sepsis grave, quemaduras, insuficiencia renal aguda y grandes 20

Seguimiento Derivar a especialista

21

Tiroiditis

-Tiroiditis crónica autoinmune (de Hashimoto): Corresponde a la principal causa de hipotiroidismo en Chile, siendo causada por destrucción celular y humoral del tejido tiroideo, pudiendo derivar en forma de bocio o atrófica. Cursa con anticuerpos anti-tiroglobulina, antitiroperoxidasa y anti-TSHr elevados. Su prevalencia aumenta en las edades medias con un pico etario entre los 30 y los 50 años de edad, afectando principalmente a mujeres. La relación entre mujeres/hombres afectados es de 9:1.

Nivel de manejo del médico general: Diagnóstico Específico Tratamiento Inicial Seguimiento Completo

Aspectos esenciales    

-Tiroiditis posparto: De etiología autoinmune y aparición dentro de 1 año después del parto. Ocurre en 8 a 10% de las embarazadas, aunque no todas son clínicamente significativas. Cursa con anticuerpos anti-tiroideos elevados.

Proceso inflamatorio de la glándula tiroides por diversas causas. Tipo aguda y subaguda son de origen infeccioso. De Hashimoto y post parto son de origen autoinmune. En general afecta mayoritariamente a mujeres.

-Tiroiditis inducida por fármacos: Los pacientes que reciben interferón-alfa, la interleucina-2, amiodarona o litio pueden desarrollar tiroiditis sin dolor. Su aparición en los pacientes que reciben estos fármacos no parece una coincidencia, pero sólo para el interferón-alfa hay datos suficientes para indicar y estar seguros de una relación causal.

Caso clínico tipo Paciente de sexo femenino, 47 años de edad, con antecedentes de LES e insuficiencia suprarrenal consulta por irritabilidad, intolerancia al frío, astenia, adinamia y aumento de peso no explicado por cambios dietario.

Diagnóstico -Tiroiditis aguda: Se manifiesta por inicio brusco de dolor y sensibilidad local, generalmente unilateral, acompañados de inflamación local y síntomas generales de infección (fiebre, calofríos, etc). El diagnóstico se hace por el cuadro clínico, en combinación con el uso de Ecografía tiroidea y PAAF.

Definición Conjunto de patologías caracterizadas por la inflamación de la glándula tiroides, con etiologías y características diversas.

-Tiroiditis subaguda: Los síntomas aparecen después de un pródromo de malestar general, mialgias, febrícula, etc. Posteriormente aparece dolor, generalmente unilateral, sobre el tiroides, referido a los oídos o mandíbula. Característicamente en el examen físico hay sensibilidad importante a la palpación tiroidea y bocio nodular doloroso. En el laboratorio se puede encontrar elevación de parámetros inflamatorios y, dependiendo de la fase, híper o hipotiroidismo.

Etiología - epidemiología – fisiopatología -Tiroiditis aguda (infecciosa aguda o supurada): Poco frecuente, de origen principalmente bacteriano (Streptococcus, Staphylococcus) y producto de la extensión vía hematógena, linfática o por contigüidad. Se distribuye principalmente en niños, ancianos, inmunosuprimidos y pacientes con patología tiroidea previa. Otras infecciones de la tiroides, incluyendo micobacterias, hongos y las infecciones por Pneumocystis, son más crónicas, y estas infecciones ocurren con mayor frecuencia en pacientes inmunocomprometidos.

-Tiroiditis Radiación: El dolor en el cuello es generalmente leve y desaparece espontáneamente en pocos días a una semana. También puede haber exacerbación transitoria del hipertiroidismo.

-Tiroiditis subaguda (de De Quervain, granulomatosa o vírica): Se presume que es causada por una infección viral o proceso inflamatorio postviral, porque muchos pacientes tienen antecedentes de una infección respiratoria superior anterior a la aparición de la tiroiditis. Cursa clásicamente de forma trifásica:

-Tiroiditis crónica autoinmune: Manifestaciones de hipotiroidismo (que amerita confirmación con laboratorio), pudiendo asociarse a otras enfermedades autoinmunes. Para el diagnóstico no es necesario medir en forma rutinaria anticuerpos, sin embargo, serían útiles en casos de hipotiroidismo subclínico, tiroiditis silenciosa o tiroiditis posparto.

tirotóxica (secundaria a destrucción tiroidea, con ↑ de T4 y T3, y ↓ de TSH y captación de I131,

-Tiroiditis posparto: Pueden cursar con una evolución trifásica como la de la tiroiditis subaguda (20 a 30%), hipertiroidismo aislado (20 a 40%) o hipotiroidismo aislado (40 a 50%). En general, los síntomas tanto de híper como hipotiroidismo son leves. Se indica la medición de TSH y T4L y, en caso de TSH ↓, solicitar T3; la medición de niveles de anticuerpos anti-tiroideos tampoco está indicada. Actualmente no está recomendado el screening.

hipotiroidismo (↑ de TSH y ↓ de T4 y T3, y fase de recuperación. Duración entre 1 y 4 meses -Tiroiditis Radiación: Un paciente con hipertiroidismo causado por la enfermedad de Graves, que es tratado con yodo radiactivo, puede desarrollar dolor de la tiroides 5 a 10 días más tarde, debido a la lesión inducida por la radiación y la necrosis de las células foliculares de la tiroides y la inflamación asociados. 22

Tratamiento -Tiroiditis aguda: Drenaje percutáneo y tratamiento antibiótico (de preferencia EV) en relación al resultado de los estudios realizados. Con poca frecuencia, se requiere drenaje quirúrgico o extirpación en pacientes que no responden al drenaje percutáneo y tratamiento antibiótico sistémico. -Tiroiditis subaguda: El tratamiento es sintomático, de preferencia con Ácido acetilsalicílico (500 mg c/5 a 6 horas) o AINE como Ibuprofeno (400 c/8 horas). En caso de ausencia de respuesta en, se pueden cambiar a tratamiento con corticoides orales, debiendo derivarse a especialista. Se utiliza Prednisona 0.5-1 mg c/día, dependiendo de la gravedad e intentando administrar la menor dosis necesaria, por el menor tiempo posible con reducción gradual durante 6 a 8 semanas, en relación a los síntomas. Se puede utilizar beta bloqueadores (Propanolol 40 a 120 mg c/día) en caso de síntomas importantes de hipertiroidismo, así como Levotiroxina en caso de hipotiroidismo muy sintomático. Se sugiere controlar T4L y TSH cada 2 a 4 semanas. -Tiroiditis crónica hipotiroidismo.

autoinmune:

Tratamiento

del

-Tiroiditis posparto: Tratamiento dependiendo de la fase y sintomatología que presente. En caso de hipertiroidismo, se sugiere el uso de Propanolol (en madres que dan lactancia materna); durante el hipotiroidismo, Levotiroixina; en caso de hipotiroidismo asintomático, se sugiere tratamiento con TSH > 10 mU/L.

Seguimiento Depende de la etiología, pero como regla general, evaluar la presencia o persistencia de híper o hipotiroidismo y derivar en caso de ser necesario.

23

Diagnóstico

Tumores hipofisiarios

Dentro de las manifestaciones clínicas se encuentran las producidas por el efecto de masa (cefalea, alteraciones visuales, hipertensión endocraneana, etc.), comunes para todos los tumores hipofisiarios, y las derivadas de la hipo o hiperfunción glandular.

Nivel de manejo del médico general: Diagnóstico Específico Tratamiento Inicial Seguimiento Completo

En el caso los tumores somatotropos, productores de GH y causantes de acromegalia, la clínica manifestada va a depender del tamaño (microadenomas o macroadenomas, menores o mayores a 1 cm, respectivamente) y de las hormonas secretadas (en el caso de la acromegalia, un 60% es producida por adenomas productores de GH en forma exclusiva, siendo menos frecuente la producción de GH y PRL u otras hormonas). En la acromegalia las manifestaciones suelen aparecer de forma insidiosa, siendo las más frecuentes el crecimiento acral (nariz, arco superciliar, pies, manos, lengua, mandíbula, etc.), visceral y de tejidos blandos, hiperhidrosis, hirsutismo, miocardiopatía e hipertrofia del ventrículo izquierdo, HTA, artropatía, poliposis colónica, intolerancia a la glucosa o diabetes mellitus e hiperprolactinemia (ya sea por producción tumoral o por compresión del tallo hipofisiario).

Aspectos esenciales   

El hallazgo de incidentalomas es frecuente. Estos son compatibles con microadenoma hipofisiario no funcionante, son estables en el tiempo. Es importante evaluar el compromiso del campo visual. La apoplejía hipofisiaria es la manifestación más grave de un tumor hipofisiario.

Caso clínico tipo Mujer 48 años, con historia de larga evolución: cefalea frontal, artralgias y parestesias en manos, hiperhidrosis. Al examen físico: sobrepeso, PA 150/90, fascie con protrusión de arcos ciliares, prognatismo leve, macroglosia, acantosis nigricans. Laboratorio: glicemia 107 mg/dl, TG 238. Sospechar acromegalia, buscar tumor.

Se recomienda la resonancia magnética como el único y mejor procedimiento de imagen para las masas de silla turca. Se recomienda la evaluación de la función hormonal hipotálamo-hipófisis siempre que se encuentre una masa selar.

Definición Tratamiento

Dentro de los tumores hipofisiarios se encuentran: prolactinoma, tumores productores de GH, ACTH, TSH, mixtos y tumores no funcionantes. Otros tipos menos frecuentes son: tumor de la bolsa de Rathke, meningioma, tumores metastásicos y linfomas.

Prolactinoma. De elección es médico, con agonistas dopaminérgicos. Se prefiere cabergolina, debe ser prolongado y se suspenden durante el embarazo. La cirugía está indicada en caso de macroadenomas con amenaza visual o ante el fracaso de la terapia médica. Tumores productores de GH. El tratamiento de elección es la adenomectomía por resección transesfenoidal, con una tasa de curación entre el 50 y 90%, dependiendo del tamaño tumoral, siendo complicaciones del procedimiento el panhipopituitarismo, diabetes insípida, fístula de LCR y meningitis. Como segunda línea se utiliza tratamiento farmacológico, principalmente análogos de somatostatina o agonistas dopaminérgicos. La radioterapia se reserva para pacientes que no logran control mediante cirugía y fármacos, tarda años en producir efecto y no está exenta de complicaciones (panhipopituitarismo, déficit neurológicos, etc.). Los criterios de curación son: niveles de GH post sobrecarga de glucosa <1 ng/mL, IGF-I normal y ausencia de síntomas, estableciéndose tres grados de control de la enfermedad: controlada, con GH e IGF-I normales y sin síntomas; control inadecuado con GH e IGF-I anormales y sin síntomas; y mal control, con GH e IGF-I anormales y sintomático. Otros tumores hipofisiarios. En general el tratamiento de primera opción es la cirugía transesfenoidal, reservándose el tratamiento médico y la radioterapia en los casos refractarios o para lograr un mejor manejo de la resección tumoral.

Etiología-epidemiología-fisiopatología Los tumores hipofisiarios más frecuentes son los adenomas. Los adenomas hipofisarios son la causa más común de las masas de la silla turca de la tercera década, que representa hasta un 10% de todos los tumores intracraneales y dentro de estos, el prolactinoma representa aproximadamente el 40% de ellos, seguido de tumores no funcionantes y posteriormente los productores de GH. Los adenomas somatotropos (productores de GH) en su mayoría son productores exclusivos de GH, cuyos niveles se encuentran elevados, estimulando la producción hepática de IGF-I, quien es responsable de la mayor parte de la sintomatología del cuadro conocido como acromegalia (si se produce luego del cierre de fisis) o gigantismo (previo al cierre). Los tumores no funcionantes, que corresponden al 20% de los adenomas hipofisiarios, se pueden dividir en los que secretan LH/FSH, los que secretan subunidades y los no secretores. Los adenomas secretores de TSH y los secretores de ACTH son mucho menos frecuentes. El prolactinoma se revisará en el capítulo de hiperprolactinemia.

Seguimiento Derivar al especialista.

24

MÓDULO 1: Medicina Interna

Infectología Celulitis Bacteriana

tes causales más frecuentes son Gram (+) principalmente Staphylococcus aureus, y Streptococcus pyogenes (Streptococcus beta hemolítico grupo A). Las extremidades inferiores son el lugar más frecuente de infección. Otras formas de celulitis son la celulitis periorbitaria (recordar en sinusitis), la celulitis de la pared abdominal (en obesos mórbidos), la celulitis bucal (por streptococcus pneumoniae y previo a la era de vacunación por haemophilus influenzae tipo b) y la celulitis perianal (por streptococcus beta hemolítico grupo A). Frente a celulitis crepitantes o gangrenosas pensar en clostridium u otros anaerobios.

Nivel de manejo del médico general: Diagnóstico Específico Tratamiento Completo Seguimiento Completo

Aspectos Esenciales • • •

El principal agente de celulitis bacteriana es Staphylococcus aureus y Streptococcus pyogenes. Principal diagnóstico diferencial con Erisipela (tratamiento es similar). En caso de mordedura, nos obliga a pensar en anaerobios.

Factores predisponentes: Puertas de entrada cutánea (trauma, picaduras de insectos, drogas iv, etc), inflamación (eczema o radioterapia), infecciones previas como impétigo, tiña y varicela, edema y obstrucción linfática (característica celulitis en mujeres post linfadenectomía en cáncer de mama). Casos especiales: En pacientes inmunocomprometidos o grandes quemados puede aparecer Pseudomonas (se asocia a lesiones en pies asociada a humedad), Mucor o Aspergillus. En las celulitis secundarias a bacteremia predominan S. pneumoniae y H. influenzae. Cuando es secundaria a una mordedura se deben considerar anaerobios como Fusobacterium, Bacteroides o Eikenella corrodens. Cuando la mordedura es por gatos también se debe pensar en Pasteurella multocida. La celulitis por Erysipelothrix rhusiopathiae es típica de manipuladores de carne y pescados.

Caso Clínico Tipo Una paciente de 50 años, carpintero, consulta por aumento de volumen en antebrazo derecho, eritematoso, con aumento de temperatura local y bordes mal definidos, asociado a sensación febril.

Definición Infección de la piel y tejidos blandos subyacentes superficiales hasta la fascia, con márgenes poco definidos. La erisipela en cambio compromete dermis y linfáticos superficiales solamente.

Patógenos de transmisión por el agua: Exposición al agua dulce (Aeromonas hydróphila) o salada (Vibrio vulnificus).

Diagnóstico

Etiología-Epidemiología-Fisiopatología

La mayor parte de las veces es clínico. El paciente puede referir dolor en la zona afectada acompañado de síntomas sistémicos como fiebre, compromiso del estado general, mialgias, calofrios. Al examen físico lesión eritematosa, de límites mal definidos, con aumento de temperatura y dolor a la pal-

Producida generalmente por inoculación de una bacteria a través de una “puerta de entrada” (herida, picadura de insecto, herida traumática o quirúrgica, sarna, mordedura, etc.), aunque también puede ser por vía hematógena. Los agen-

1 Facultad de Medicina, Universidad de Chile

pación y ocasionalmente adenopatías. Si hay presencia de secreción esta se debe cultivar. En pacientes hospitalizados siempre tomar hemocultivos. Frente a necrosis o sospecha de infección de tejidos profundos siempre tomar imágenes y evaluar por cirugía (descartar fasceitis). Diagnósticos diferenciales: erisipela (infección más superficial generalmente por strepotococcus pyogenes que da una lesión solevantada, más roja, bordes característicamente definidos), dermatitis estásica, dermatitis por contacto, tromboflebitis, paniculitis o eritema migrans.

Tratamiento El principio es cubrir tanto streptococcus pyogenes como Staphylococcus aureus.En general, 5-7 días de tratamiento con antibióticos es suficiente para el tratamiento de la celulitis no purulenta; extensión de hasta 14 días puede estar justificada por enfermedad grave o una respuesta lenta a la terapia. Tratamiento ambulatorio: Cefadroxilo 1 gr c/24 hrs o Amoxi-clavulánico 500/125mg c/8 hrs, por 7-10 días. Hospitalizar: Pacientes con sepsis severas, comorbilidades importantes o descompensadas, necesidad de tratamiento iv (mala tolerancia oral por ejemplo), sospecha de compromiso más profundo (descartar fasceitis) o tromboembolismos venosos subyacentes, múltiples consultas. Tratamiento hospitalizado empírico: Cefazolina 1-2 gr c/8 hrs IV. Alternativa: PNC + cloxacilina. Si alergia a PNC considerar macrólidos o clindamicina. Si sospecha de SAMR (staphylococcus aureus metacilino resistente) considerar vancomicina. Pacientes diabéticos pueden requerir coberturas más amplias (en general se usa Clindamicina y si son más graves Ceftriaxona + clindamicina o un carbapenémico) Siempre cubrir anaerobios en mordeduras.

Seguimiento Se recomienda el marcaje de las lesiones, para monitorizar la regresión v/s progresión o manutención de la lesión, junto a la evolución general del paciente. En pacientes hospitalizados la evolución de los cultivos y parámetros inflamatorios son un buen complemento.

Autor / Editor Gustavo Muñoz R.

Año 2017

2 Facultad de Medicina, Universidad de Chile

Enfermedades De Transmisión Sexual (ETS)

de personas contraen alguna de las cuatro infecciones más prevalentes en el mundo: chlamydia, gonorrea, sífilis o tricomoniasis. Las ITS se propagan predominantemente por contacto sexual, incluidos el sexo vaginal, anal y oral. También se pueden propagar por medios no sexuales, por ejemplo, las transfusiones de sangre o productos sanguíneos.

Nivel de manejo del médico general: Diagnóstico Específico Tratamiento Completo Seguimiento Completo

Diagnóstico Algunos síndromes clínicos y enfermedades que con más frecuencia se asocian a ITS son: • Uretritis: La uretritis en el hombre genera secreción uretral y disuria urente generalmente sin polaquiuria. Se clasifica como Uretritis Gonocócica (N. gonorrhoeae) y No Gonocócica (UNG) (C. trachomatis en su mayoria, Ureaplasmas, VHS-2, T. vaginalis). En una muestra de secreción con tinción de Gram, más de 4 PMN por campo hace el diagnóstico. La presencia de diplococos Gram (-) dentro de cel. Inflamatorias nos habla de Gonorrea (los gonococos de vida libre pueden ser flora saprofita). También se puede hacer tinción de Thayer-Martin. Cuando se diagnostica uretritis por gonococo se trata cubriendo gonococo y también Chlamydia. Cuando es una UNG se trata sólo a la Chlamydia. Puede tener complicaciones como una infección gonocócica diseminada (artritis y lesiones cutáneas), estenosis uretrales, infecciones como Prostatitis u Orquitis, y Artritis reactivas (Sd. de Reiter).

Aspectos Esenciales • • • • • • •

Concepto actual: Infecciones de transmisión sexual (ITS) En la mayoría de los casos, las ITS son asintomáticas o solo van acompañadas de síntomas leves que no necesariamente permiten un diagnostico certero. Primer agente etiológico de UNG: Chlamydia Trachomatis. Diagnóstico de Gonococo en hombre: diplococos dentro de PMN Vaginosis no son ITS. En T. vaginalis el pH vaginal es mayor a 5, en Candida se mantiene normal. En toda úlcera genital: estudio para sífilis y VIH.

Caso Clínico Tipo Paciente masculino 32 años, de hábitos sexuales riesgosos, presenta desde hace 2 días secreción uretral abundante con disuria. No refiere polaquiuria, pujo o tenesmo. Al estudio microbiológico de la secreción uretral: 10 PMN por campo y diplococos gram (-) al interior de los PMN.



Vulvovaginitis: Sus principales causales son Candida albicans y Trichomonas vaginalis (secreción anormal por vaginosis bacteriana no se considera una ITS). La candidiasis vaginal causa prurito, sensación urente, irritación y disuria externa (cuando orina pasa por vulva), generalmente sin leucorrea, aunque puede presentar placas blanquecinas adherentes. El virus herpes simplex puede dar un clínica muy similar, incluso cuesta diferenciar las fisuras de la candidiasis con la lesiones del VHS. El pH vaginal no suele cambiar. El diagnóstico de Candidiasis se basa en la observación de hifas o seudohifas en el estudio microbiológico. La tricomoniasis vaginal presenta aumento de la secreción, abundante y amarilla de mal olor, con irritación vulvar. pH vaginal suele elevarse a >5.0. En el examen al fresco de la secreción vaginal se pueden ver los protozoos móviles. También se puede hacer cultivo o PCR en la orina.



Cervicitis y Enfermedad Inflamatoria Pélvica (EIP): Análogo a la Uretritis del hombre. Los microorganismos y tratamiento son los mismos. Secreción mucopurulenta amarilla proveniente del orificio cervical externo. Existe aumento de los PMN en el frotis del moco endocervical. Si persiste puede dar EIP.



Úlceras genitales: Pueden ser causadas por varias enfermedades que incluyen el Herpes (el más frecuente), Chancro duro, Chancro blando, Linfogranuloma venéreo y Donovanosis/Granuloma inguinal. Existen caracterís-

Definición Constituyen un grupo de infecciones transmisibles que se caracterizan porque su principal modalidad de transmisión es la vía sexual.

Etiología-Epidemiología-Fisiopatología Entre los más de 30 virus, bacterias y parásitos que se sabe se transmiten por contacto sexual, ocho se han vinculado a la máxima incidencia de enfermedades de transmisión sexual. De esas 8 infecciones, 4 son actualmente curables, a saber, la sífilis, la gonorrea, la clamidiasis y la tricomoniasis. Las otras 4 –hepatitis B, virus del herpes simple (HSV o herpes), VIH y virus del papiloma humano (VPH)– son infecciones virales incurables, aunque existen tratamientos capaces de atenuar o modificar los síntomas o la enfermedad. Según estimaciones de la Organización Mundial de la Salud (OMS), más de 1 millón de personas adquieren una ITS cada día, estimándose que anualmente unos 500 millones

3 Facultad de Medicina, Universidad de Chile

ticas clínicas que orientan al diagnóstico etiológico, sin embargo todo paciente con úlceras genitales se debe hacer un estudio serológico y estudio de campo oscuro para T. pallidum, y serología para VIH. En el herpes simple genital existen vesículas dolorosas que pueden ulcerarse. Hasta en dos tercios aparecen recidivas, menos sintomáticas que la primoinfección. El diagnóstico se hace con la observación de células gigantes multinucleadas con inclusiones citoplasmáticas o mediante PCR o serológicas. En la sífilis primaria existen úlceras duras, por lo general lesión única, e indolora con adenopatía inguinal firme e indolora. El chancro blando (o chancroide) es una infección por Haemophilus ducreyi. Se presenta como úlceras dolorosas y purulentas que se acompañan de adenopatías fluctuantes o eritema de la piel que las cubre. Frecuente en países en desarrollo. Se deben aspirar ganglios para muestras para cultivo o PCR. El linfogranuloma venéreo es una linfadenopatía dolorosa muy grande en los ganglios linfáticos femorales e inguinales separados por el ligamiento inguinal. Su causa son Chlamydia trachomatis del serotipo L1-L3. Su diagnóstico se hace con serología. El granuloma inguinales una infección por Calymmatobacterium granulomatis. Su diagnóstico es por biopsia y el tratamiento es con tetraciclinas. •



• • • •

zatinica 2.4 millones de unidades im semanal por 3 semanas consecutivas Neurosífilis: 3 a 4 millones de unidades ev c/4 hrs por 14 días.. Linfogranuloma venéreo: Doxiciclina 100 mg c/ 12 hrs x 21 días Granuloma Inguinal: Tetraciclina 500 mg c/ 6 hrs por 3-5 semanas. Condiloma Acuminado: Podofilotoxina al 0,5 % Aplicación local 2 veces al día por 3 días consecutivos y 4 días sin aplicar (repetir hasta 4 ciclos). Otras opciones: Imiquinod, crioterapia, escisión quirúrgica

Seguimiento A veces se debe tratar con antibióticos también a la pareja o contactos sexuales recientes. Se debe hacer educación sexual para evitar reinfecciones y nuevos contagios. En el caso de sospecha de sífilis que haya dado negativo en primera instancia, se puede hacer un segundo examen posteriormente.

Molusco Contagioso: Producido por un poxvirus homónimo. Clásicamente es una pápula pequeña e indolora, con hoyuelo en el centro, que puede crecer hasta convertirse en un nódulo de color carne y apariencia de perla. El diagnóstico generalmente es una biopsia de piel. En pacientes con sistema inmune normal el trastorno pasa espontáneamente.

Autor / Editor Gustavo Muñoz R.

Lesiones Vegetantes genitales: Se produce por el Virus del Papiloma humano, en especial los serotipos 6 y 11. Los serotipos 16 y 18 están más asociados al cáncer cervicouterino y al anal. La infección por HPV es la ITS más frecuente, pero sólo un 1% da sintomatología. Se presentan como Condiloma Acuminado en el aparato reproductivo externo. El condiloma tiene tratamiento tópico.

Tratamiento • • • • • • •

Uretritis: Una dosis de Ceftriaxona 250 mg im + Azitromicina 1 g vo (para cubrir C. Trachomatis) Candidiasis vaginal: Fluconazol 150mg vo dosis única o clotrimazol por ovulo vaginal de 500 mg por 1 vez Tricomoniasis Vaginal: Metronidazol 2 gr vía oral dosis única (se recomienda combinar con metronidazol en óvulo vaginal 500 mg c/noche x 7 días) Herpes genital, primoinfección: Aciclovir 400 mg vo c/8 hrs x 7 días , o Valaciclovir 1 gr vo c/12 hrs x 7 días Herpes genital, recurrencia: Aciclovir 400 mg vo c/8 hrs x 5 días , o Valaciclovir 500 mg vo c/12 hrs x 5 días Sífilis primaria, Sífilis secundaria, Sífilis latente precoz: Penicilina benzatinica 2.4 millones de unidades im semanal por 2 semanas consecutivas Sífilis latente tardía y sífilis cardiovascular: Penicilina ben-

4 Facultad de Medicina, Universidad de Chile

Año 2017

Enfermedades Eruptivas No Complicadas (Varicela, Herpes Zóster)

El Herpes zoster se manifiesta generalmente en > 60 años o pacientes inmunosuprimidos.

Diagnóstico Nivel de manejo del médico general: Diagnóstico Específico Tratamiento Completo Seguimiento Completo

Varicela: El periodo de incubación es de 10-21 días. Los enfermos son contagiosos 48 horas antes del comienzo del exantema y hasta que todas las vesículas formen costras. Las manifestaciones clínicas incluyen un pródromo de fiebre, malestar general, o faringitis, seguido por el desarrollo de la aparición del exantema. Las lesiones cutáneas son maculopápulas, vesículas y costras en distintos estados evolutivos al principio en cara y tronco, pero luego generalizadas. Herpes Zóster: Se caracteriza por una erupción vesiculosa unilateral circunscrita a un dermatoma. El dolor es intenso, generalmente de carácter lancinante y puede preceder a la aparición de las lesiones. La duración de la enfermedad suele ser 7-10 días. El zóster oftálmico puede terminar en ceguera. El zóster del nervio facial (Sd. de Ramsay Hunt) además de vesículas en el pabellón auditivo externo, presenta ageusia de los tercios anteriores de la lengua. El diagnóstico de estas enfermedades es principalmente clínico, pero puede confirmarse por cultivo de tejidos, seroconversión (IgM), PCR o elevaciones de anticuerpos (IgG).

Aspectos Esenciales • • •

La Varicela y el Herpes Zoster se producen por el mismo virus: VZV. La varicela produce prurito; la mayoría de las erupciones exantemáticas no lo hacen. Tratamiento: Aciclovir vo 800 mg 5veces/d x 7 días.

Caso Clínico Tipo Paciente de 64 años, presenta hace dos días dolor torácico urente en franja unilateral. La noche anterior aparecen vesículas sobre fondo eritematoso en la zona del dolor. Al interrogatorio dirigido dice haber tenido Varicela cuando niño.

Tratamiento Definición

En general depende del estado inmunitario del paciente y factores de riesgo: Para los niños sanos ≤ 12 años, la varicela es generalmente autolimitada y se sugiere no administrar tratamiento antiviral. Sin embargo, se debe considerar el tratamiento en usuarios de corticoides intermitentes, trastornos cutáneos o enfermedad pulmonar crónica (infecciones bacterianas secundarias pueden tener consecuencias graves en estos casos). En huéspedes inmunocompetentes, se detiene la replicación viral por lo general de 72 horas después de la aparición de la erupción (por lo que se debe iniciar su tratamiento antes).

Son una serie de entidades cuya característica común es la presencia de erupciones cutáneas generalizadas. Entre ellas, destacan las infecciones por el Virus Varicela-Zoster (VZV). La escarlatina (Streptococo betahemolítico), eritema infeccioso (parvovirus B19) y exantema súbito (virus Herpes 6) son de interés pediátrico. Otras son rubéola, sarampión (prácticamente sin casos en Chile gracias a la vacunación) y algunas enfermedades por rickettsias (endémicas de otros países).

Etiología-Epidemiología-Fisiopatología



El VZV pertenece a la familia Herpesviridae. Es el agente etiológico de dos entidades clínicas: Varicela (primoinfección) y Herpes Zoster (reactivación). La transmisión se produce a través del contacto con las gotitas de aerosol de las secreciones nasofaríngeas de una persona infectada o por contacto cutáneo directo con líquido de las vesículas de lesiones en la piel. El virus ingresa por la vía respiratoria, se replica en el epitelio e ingresa a los capilares produciendo una viremia primaria; luego se disemina en el sistema reticuloendotelial y por último, establece una viremia secundaria. Los virus colonizan la piel y desde ahí infectan los ganglios de las raíces dorsales de los nervios sensitivos dónde permanecerán latentes. La varicela es una infección altamente contagiosa Generalmente se presenta en menores de 10 años y en invierno y primavera, y tiende a ser más leve en comparación a su presentación en adultos o en inmunocomprometidos.

• • • •



Adultos inmunocompetentes: Aciclovir vo 800 mg 5 veces/día x 5-7 días. En niños menores de 12 años (y con indicación de tratamiento), la dosis de aciclovir es 20 mg/kg (dosis máxima de 800 mg) c/ 6 hrs por 5 días Adultos inmunocomprometidos: Aciclovir 10 mg / kg IV cada 8 horas x 7-10 días Niños >1 año y adolescentes inmunocomprometidos: Aciclovir 1500 mg/m2/día iv dividido en 3 dosis o 30 mg/ kg/día iv dividido en 3 dosis, x 7-10 días Herpes Zóster en inmunocompetente (con menos de 72 hrs después del inicio de aparición de lesiones): Aciclovir 800 mg 5 veces al día vo x 7 días o valaciclovir 1 gr 3 veces al día vo x 7 días. En casos de pacientes inmunocomprometidos con herpes zóster, iniciar mismo esquema aunque hayan transcurrido 72 hrs desde la aparición del lesiones.

5 Facultad de Medicina, Universidad de Chile

Seguimiento Un 50% de los mayores de 50 años refieren neuralgia postherpetica. Posibles complicaciones son sobreinfección bacteriana o enfermedades congénitas.

Autor / Editor Gustavo Muñoz R.

Año 2017

6 Facultad de Medicina, Universidad de Chile

Infección Por VIH Nivel de manejo del médico general: Diagnóstico Específico Tratamiento Inicial Seguimiento Derivar



A finales de 2015 había 36,7 millones de personas infectadas por el VIH en todo el mundo, de las cuales 2,1 millones de personas contrajeron el HIV en 2015.



El África subsahariana (25,6 millones de personas infectadas por el VIH en 2015), es la región más afectada.



En Chile, los primeros casos de SIDA se notificaron en 1984, desde esa fecha hasta el año 2011 las notificaciones alcanzan a 26.740 casos de VIH o SIDA. El principal grupo poblacional afectado, es el de adultos entre 20 y 49 años, destacando una tasa ascendente en el último quinquenio en el grupo de 10 a 19 años. La principal vía de transmisión es la sexual, con el 87,6 % de los casos y sólo el 2,6% de los casos son por transmisión endovenosa.



Se conocen dos subtipos: el VIH-1 y el VIH-2, siendo el primero el más común y de distribución mundial, mientras que el segundo es una variante menos virulenta, más prevalente en África Occidental y Central.

Aspectos Esenciales • • •

Cualquier manisfetación clínica de la etapa C o un conteo de CD4 < 200 células/mm3 independiente de la etapa de la enfermedad es SIDA. Manifestación clínica más frecuente en etapa sintomática: candidiasis de mucosas. Se recomienda iniciar TAR en manifestaciones clínicas de etapa B o C, o en pacientes asintomáticos con CD4 < 350 células/mm3.

Caso Clínico Tipo

Diagnóstico

Varón 30 años, consulta por cuadro de diarrea de 6 semanas de evolución, asociado a sensación febril intermitente. Al examen físico se constatan signos de deshidratación y destaca la presencia de placas blanquecinas en cavidad oral. A la anamnesis dirigida refiere haber tenido sexo sin protección en más de una oportunidad.

Aparecen infecciones oportunistas (ej. Neumonía por PC) y neoplasias inusuales (ej. Kaposi, Linfoma SNC); hay más incidencia de ciertas infecciones comunes en inmunocompetentes (ej. neumonía bacteriana), fenómenos autoinmunes (ej. vasculitis), alérgicos (ej. reacciones cutáneas), y disfunciones de órganos específicos (ej. nefropatía por VIH). 2 a 6 semanas post contagio, 2/3 de los pacientes tiene síntomas atribuibles al síndrome retroviral agudo (fiebre, adenopatías, faringitis, rash, puede ser un sd. mononucleósico); sobreviene luego la infección latente, de progresión lenta y oligosintomática, y finalmente la etapa SIDA. Manifestación clínica más frecuente en la etapa sintomática de la enfermedad es la candidiasis de mucosas.

Definición La infección por VIH tipo 1 o tipo 2, se caracteriza por el deterioro progresivo del sistema inmune. Se asocia en todas sus etapas a una intensa replicación viral, principalmente en linfocitos CD4 y macrófagos. Inicialmente los mecanismos inmunológicos permiten neutralizar los nuevos viriones y regenerar las células inmunes que se destruyen aceleradamente, lográndose un equilibrio entre la cantidad de virus circulante (carga viral) y el sistema inmunológico, lo que se traduce clínicamente por una infección asintomática (etapa A) o poco sintomática durante un periodo variable entre 5-10 años. Finalmente el aumento progresivo de la carga viral provoca un deterioro de la función inmune, certificado con la caida de los Linfocitos CD4, lo que provoca la aparición de diversas infecciones, clásicas y oportunistas, y tumores con lo que se llega a las etapas B y C ( SIDA), y finalmente muerte de no mediar tratamiento.

Debe realizarse un examen físico buscando manifestaciones del VIH-SIDA y de otras ETS, con énfasis en piel, mucosa oral, adenopatías, visceromegalia, examen neurológico, fondo de ojo, y genitales. Además, investigar síntomas (disfagia, alteraciones visuales, cefalea, diarrea, fiebre, baja peso, etc.), prácticas de riesgo, e iniciar educación. En el laboratorio se debe como mínimo solicitar en cada control hemograma, VHS, glicemia,creatininemia, orina completa, pruebas hepáticas, perfil lipídico, recuento de linfocitos CD4, carga viral VIH, VDRL o RPR, HBsAg y de acuerdo a disponibilidad, anticore para virus Hepatitis B (VHB) y serología para virus Hepatitis C (VHC),IgG Toxoplasma gondii, serología para Trypanosoma cruzii (Enfermedad de Chagas), PPD y rx de Tórax, y PAP a todas las mujeres.

Etiología-Epidemiología-Fisiopatología •

El CDC recomienda screening a todo paciente adulto que consulte cualquier servicio de salud. Se realiza con ELISA VIH, que detecta anticuerpos; alta sensibilidad para enfermedad establecida (99%) luego del periodo de ventana (3-6 semanas aprox.). El Western Blot es el usado para confirmación en Chile (ISP).

El VIH sigue siendo un importante problema de salud pública mundial, después de haber cobrado más de 35 millones de vidas hasta ahora. En 2015, 1,1 millones de personas fallecieron a causa del VIH en todo el mundo.

7 Facultad de Medicina, Universidad de Chile

Condiciones definitorias de etapa SIDA (etapa C): TBC pulmonar o extrapulmonar, neumonía por pneumocystis carinii, criptococosis meníngea o extrapulmonar, toxoplasmosis cerebral, enfermedad por micobacterias atípicas, retinitis por CMV, candidiasis esofágica, traqueal o bronquial, encefalopatía VIH, leucoencefalopatía focal progresiva, criptosporidiasis crónica > 1 mes, isosporosis crónica > 1 mes, úlceras mucosas o cutáneas herpéticas > 1 mes, neumonía recurrente, bacteremia recurrente por Salmonella, sarcoma de kaposi, linfoma no hodgkin o linfoma de SNC, cáncer cervicouterino invasor y síndrome consuntivo.

retroviral agudo severo y prolongado, manifestaciones neurológicas significativas, recuento de CD4 persistentemente menores a 350cel/mm3 luego de 3 meses de evolución. En estos casos el TAR debe ser permanente.

Las indicaciones para el inicio de TAR son: • Enfermedad oportunista Tipo C (incluyendo TBC pulmonar), independiente del recuento de CD4. Se difiere el inicio de TAR algunas semanas en pacientes que tienen alguna infección oportunista que requiere ser tratada para evitar un síndrome de reconstitución inmune (ej. Retinitis por CMV, meningitis por criptococo).



En pacientes asintomáticos o con manifestaciones de etapa B que tengan recuento de linfocitos CD4 menores de 350cel/mm3. Con recuentos entre 200-350cel/mm3 se debe contar con 2 mediciones separadas por 1 mes, mientras que con recuentos <200cel/mm3 basta con 1 determinación.

En los pacientes con recuento <100cel/mm3 debe iniciarse el TAR de primera línea dentro de 7 días de la indicación médica (“inicio precoz de TAR”), si los recuentos son <50cel/mm3 se debe iniciar además de la profilaxis de Pneumocystis jiroveci, profilaxis de infección por micobacterias atípicas con azitromicina.



Pacientes con co-infección con virus hepatitis B o C independiente del nivel de CD4.



Pacientes con nefropatía asociada a VIH (HIVAN) independiente de los CD4.



Pacientes con otras condiciones asociadas a VIH como la trombocitopenia asociada a VIH, independiente del nivel de CD4.



En primoinfección: Un 40-90% de los pacientes que adquieren VIH tienen, en un plazo de 2-6 semanas, manifestaciones clínicas diversas que en su conjunto se denominan síndrome retroviral agudo (clínicamente símil a Sd. Mononucléosico). En estos pacientes no se recomienda el inicio de TAR en forma rutinaria. Solamente en casos excepcionales se considera el inicio de TAR: síndrome



Situaciones que requieren profilaxis (accidente cortopunzante de riesgo, agresiones sexuales).

Seguimiento Patología GES, ante sospecha clínica se disponen de 60 días para confirmación diagnóstica.

Los pacientes con CD4<200cel/mm3 deben además iniciar profilaxis de neumonía por Pneumocystis jiroveci con cotrimoxazol y mantenerlo hasta que el recuento supere las 200cel/mm3 y la carga viral sea indetectable.



En embarazadas según protocolo.

Cómo iniciar TAR: Debe ser con asociaciones de 3 drogas antiretrovirales (ARV) que pertenezcan al menos a dos familias distintas de antiretrovirales. Esta es capaz de suprimir la replicación viral (CV indetectable) en más del 70% de los casos, además se recupera cualitativa y cuantitativamente la respuesta inmune y se reduce la morbilidad asociada, la progresión y la mortalidad por SIDA.Habitualmente se inicia con dos inhibidores de la transcriptasa reversa (ej, Zidovudina + Lamivudina, Abacavir + Lamivudina o Tenofovir + Emtricitabina (elección en pac. Co-infectado con VHB)) y como tercera droga, un inhibidor de la transcriptasa reversa no nucleosidico (Ej. Nevirapina, Efavirenz) o un inhibidor de la proteasa reforzado (ej. Lopinavir/ritonavir, Atazanavir/ritonavir).

Tratamiento





Autor / Editor Gustavo Muñoz R.

8 Facultad de Medicina, Universidad de Chile

Año 2017

Influenza

incubación es corto (horas - 4días). El virus alcanza la mucosa respiratoria superior, donde gracias a la neuroaminidasa (rompe enlaces de ácido N-acetil-neuroamínico del mucus), logra liberar viriones.

Nivel de manejo del médico general: Diagnóstico Específico Tratamiento Completo Seguimiento Completo

Diagnóstico Se manifiesta con síntomas de inicio abrupto, inicialmente anorexia y mialgias (espalda, miembros, dolor retro-orbitario al mover los ojos por compromiso de musculatura de la órbita), luego fiebre (38 – 41°C), cefalea intensa posterior a la fiebre, calofríos, odinofagia, rinorrea y tos no productiva. Son comunes las adenopatías cervicales en individuos jóvenes. En menor frecuencia: vómitos, dolor abdominal, diarrea.

Aspectos Esenciales • • • •

Presentación en brotes, en épocas invernales. Síntomas de inicio abrupto, con gran compromiso sistémico. Tratamiento antiviral y profilaxis en grupos de alto riesgo. Vacuna preventiva en grupos de riesgo.

El cuadro no complicado tiene una duración 2 a 7 días, pero la tos puede durar más. Los factores de riesgo para presentar complicaciones son edades extremas (< 2 años > 65 años), embarazo, inmunocomprometidos y patologías crónicas. La complicación más importante es la neumonía, producida por el virus o por sobreinfección bacteriana, donde destaca como agente el Staphylococcus aureus (frente a neumonías siempre buscar y tratar infección bacteriana).

Caso Clínico Tipo Paciente de 30 años con cuadro de 2 días de evolución de fiebre alta, odinofagia, tos, asociado a mialgias y gran CEG. Refiere más familiares con cuadro similar.

Definición

El diagnóstico clínico tiene una sensibilidad de 70% en épocas epidémicas. Existen exámenes confirmatorios para documentar la causa y realizar medidas epidemiológicas. Se pueden detectar antígenos de Influenza A – B mediante test pack (1-2 hr.), panel viral (6-8 hr) tomados con aspirado o hisopado nasofaríngeo con sensibilidad entre 80 – 95%, ambos comunes en la práctica clínica. También se puede realizar PCR (utilidad en diferenciar influenza A estacional de la H1N1), cultivo, y serología.

Infección respiratoria aguda, de carácter estacional. Puede afectar la vía respiratoria alta y/o baja, la mayoría de las veces con síntomas sistémicos. Se presenta en brotes, como epidemias, principalmente en los meses fríos, con una de duración y severidad variable. En general los brotes son entre Abril y Mayo, con una duración de 6 semanas, sin embargo, los últimos años se han desplazado hacia fines de agosto, inicios de septiembre.

Tratamiento

Etiología-Epidemiología-Fisiopatología

El tratamiento es fundamentalmente sintomático, con antipiréticos, analgésicos o antiinflamatorios. Los antivirales están indicados en los grupos de alto riesgo (mayores de 65 años, embarazadas, enfermedades crónicas e inmunocomprometidos). Se debe administrar dentro de 48 hr iniciado el cuadro.

Es producida por el Virus Influenza, de la familia Orthomyxoviridae, compuesto por RNA monoténico, un manto de donde protruyen espículas. Posee 2 estructuras antigénicas; nucleocápside (NP) y matriz (M), que permiten clasificarlos en 3 géneros A, B y C. La Nucleocápside (importante en la infectividad y patogenicidad viral) está constituida por los antígenos de superficie hemaglutinina “H” y neuroaminidasa “N”. El antígeno “H” es el más abundante, y su función es reconocer receptores específicos de la mucosa respiratoria. La neuroaminidasa es una enzima que facilita la liberación viral. Anualmente hay epidemias dado mutaciones puntuales o drift del virus que no provocan cambios en el subtipo. En cambio, las pandemias se producen por mutaciones mayores o shift que si cambia el subtipo. Todos los años se produce una epidemia en cada país. Estas aparecen de forma súbita, alcanzan su peak en 2-3 semanas y duran 2-3 meses.

Los antivirales de uso actual son el oseltamivir (75 mg c/12 hr) y zanamivir (1 inh c/12 hr) por 5 días, que inhiben la neuraminidasa de los géneros A y B. Diversos estudios han demostrado efectividad en reducir el tiempo de los síntomas, disminuir complicaciones graves y tiempo hospitalario, cuando son administrados precozmente (primeras 48 hrs). Frente a un paciente con sospecha clínica, perteneciente a grupos de riesgo, se recomienda iniciar tratamiento previo a toma de examen de detección, en caso de que este esté disponible. No se recomiendan como profilaxis.

La infección se disemina por vía aérea en aerosoles o por contacto con manos u objetos contaminados. El período de

Para la profilaxis se dispone de vacunas inactivadas que se

9 Facultad de Medicina, Universidad de Chile

administran vía subcutánea. La vacuna actual contiene dos cepas de influenza A (H1N1 y H3N2) y una de influenza B. Es necesario vacunar anualmente debido a la corta duración de la inmunidad y la variación antigénica del virus. Indicaciones de vacunación en Chile: • Embarazadas a partir de la 13° semana • Niños entre 6 y 24 meses • Mayores de 65 años • Trabajadores en plantas avícolas y de cerdos. • Paciente con condiciones de riesgo (Diabetes, Enfermedades pulmonares crónicas, ERC etapa IV-V, Cardiopatía, Cáncer en tratamiento con quimioterapia, Trastornos de la deglución, Insuficiencia hepática crónica). • Pacientes con condiciones de riesgo según criterio médico (Cáncer en tratamiento, Inmunodeficiencias, Enfermedades autoinmunes). • Personal de salud.

Seguimiento Realizar tratamiento completo, y seguimiento según respuesta clínica, principalmente buscando precozmente complicaciones en grupos de riesgo.

Autor / Editor Marcelo Fres

Revisor de Formato Verónica Vasquez

Año 2016

10 Facultad de Medicina, Universidad de Chile

Diagnóstico

Neutropenia Febril Nivel de manejo del médico general: Diagnóstico Específico Tratamiento Inicial Seguimiento Derivar

A todo paciente que cursa alguna terapia que pueda generar neutropenia se le indica que consulte a urgencia de inmediato ante la presencia de fiebre. Frecuentemente la única manifestación es la fiebre ya que no se puede montar una buena respuesta inmune.

Aspectos Esenciales

La evaluación diagnóstica debe incluir una anamnesis, la exploración física y las evaluaciones de laboratorio, Esto incluye:

• •

RAN <500 cel/mm3, con una sola temperatura oral >38,3°C o una temperatura >38,0°C sostenida durante> 1 hora. Generalmente post quimioterapia. Principales agentes eran Gram (-) (con especial importancia: Pseudomona aeruginosa), pero ahora se ha visto un equlibrio entre Gram (-) y (+).

Caso Clínico Tipo



Hemocultivos y cultivos obtenidos de otros sitios sospechosos de infección.



Imagen de tórax - En pacientes neutropénicos febriles con signos o síntomas respiratorios, radiografía de tórax en pacientes de bajo riesgo y una tomografía computarizada de tórax en pacientes de alto riesgo.



Imagen abdominal - Los pacientes con neutropenia febril y cualquier signo o síntomas sugestivos de la enterocolitis neutropénica o colitis por Clostridium difficile (por ejemplo, dolor o sensibilidad abdominal, diarrea) deben someterse a un TC abdominal.

Paciente femenino 62 años con antecedente de leucemia mieloide crónica en tratamiento, consulta por presentar fiebre hace dos días (8 días después de quimioterapia). Se objetiva temperatura de 38.5ºC.

Existen distintas tablas y scores para determinar si el paciente es de bajo o alto riesgo (complicaciones y mortalidad), pero principalmente se podría resumir en: • Alto Riesgo: Neoplasia Hematológica, más de 60 años, PMN <100, duración de más de 10 días, inestable, sepsis, con mucositis o con foco.

Definición Presencia de fiebre de 38.3°C en un registro único o de 38°C de más de una hora asociado a un recuento absoluto de neutrófilos <500 o <1000 con declinación a menos de 500 en 24 horas.



Bajo Riesgo: Neoplasia Sólida, menos de 60 años, PMN >100, duración de menos de 7-10 días y sin sepsis, mucositis ni foco.

Etiología-Epidemiología-Fisiopatología Tratamiento

Los neutrófilos son un determinante crítico de infección. Existe reserva de ellos en la medula ósea, sin embargo esto se puede ver afectado, por ejemplo con radioterapia o quimioterapia. Es por esto que la neutropenia febril se encuadra principalmente en las emergencias oncológicas, principalmente relacionadas a neoplasias hematológicas. La mayor caída en el recuento de neutrófilos se da entre los días 9-12 post tratamiento. Por lo anterior, es más grave un cuadro febril antes de los 9 días (porque sabemos que el recuento de neutrófilos va a bajar aún más) y menos grave ya pasando el día 12 (recuento de neutrófilos va a ir aumentando). Cuando existe daño de mucosas, principalmente respiratorias y digestivas, además de neutropenia la posibilidad de infección es muy alta.

El objetivo de la terapia empírica es para cubrir los patógenos más probables y más virulentos que pueden causar rápidamente infección grave o potencialmente mortal en pacientes neutropénicos Pacientes de bajo riesgo: Se recomienda una combinación de una fluoroquinolona ( Ciprofloxacino 750 mg por vía oral dos veces al día o Levofloxacino 750 mg por vía oral una vez al día) y un beta-lactámico como la amoxicilina-ácido clavulánico (500 mg / 125 mg por vía oral tres veces al día). Pacientes de alto riesgo: Emergencia médica, administrar antibióticos tan pronto sea posible. Se recomienda la monoterapia empírica con un agente betalactámico frente a Pseudomonas, tales como cefepima, meropenem, imipenem o piperacilina-tazobactam. No se recomienda el uso de vancomicina (u otro agente que se dirige a cocos gram-positivos) como parte estándar del régimen inicial, a menos que se sospeche infección relacionada con el catéter venoso central, infección de piel o tejidos blandos, neumonía o inestabilidad hemodinámica.

Bacilos Gram-negativos, especialmente Pseudomonas aeruginosa , son los patógenos más comúnmente identificados en pacientes con neutropenia hasta la década de 1980. Posteriormente, las bacterias gram-positivas se han convertido en los patógenos más comunes, encontrándose en un equilibrio desde los años 2000.

11 Facultad de Medicina, Universidad de Chile

Seguimiento Todo paciente con neutropenia febril debe ser hospitalizado y observado para ver su evolución.

Autor / Editor Gustavo Muñoz R.

Año 2017

12 Facultad de Medicina, Universidad de Chile

Sífilis Secundaria, Terciaria y Congénita

tardías no lo son. La evolución de la enfermedad es variable: 1/3 presenta recuperación total sin tratamiento, 1/3 permanece en etapas latentes evidenciada solamente con test serológicos reactivos y el tercio restante, hacia Sífilis Terciaria. La neurosífilis se puede manifestar en cualquiera de las etapas clínicas de la enfermedad, con mayor riesgo en la sífilis tardía. En la sífilis terciaria los test serológicos treponémicos y no treponémicos están reactivos, y en las lesiones la presencia de treponemas es rara siendo destructivas por una reacción de hipersensibilidad.

Nivel de manejo del médico general: Diagnóstico Sospecha Tratamiento Inicial Seguimiento Derivar

Aspectos Esenciales • • • •

La Sífilis congénita es multisistémica, en ella existe paso de anticuerpos IgG maternos (treponémicos y no treponémicos) al feto, que dificulta la interpretación de los resultados serológicos en el recién nacido. La severidad de la infección depende del momento en que la madre la adquirió, el estadio de infección materna al momento del embarazo, la edad gestacional al momento de la infección, la carga de treponemas que infectan al feto y la oportunidad de la respuesta inmunológica de la madre. Antes de la concepción o muy precozmente durante el embarazo: se presentan infecciones más leves o no se produce infección fetal. Después del cuarto mes de embarazo: puede derivar en muerte fetal intrauterina alrededor de la semana 20 de gestación o en enfermedad grave del recién nacido.

Enfermedad transmitida por vía sexual y vertical. Etapas clínicas: precoz (<1 año) y tardía (>1 año). Las lesiones cutáneas de la precoz son altamente contagiosas no así las de la sífilis tardía. La sífilis congénita es una enfermedad multisistémica. El riesgo de contagio depende de entre otros el estado de la infección en la madre y del trimestre del embarazo.

Caso Clínico Tipo Paciente femenino de 24 años quien presenta una dermatosis diseminada a extremidades superiores e inferiores de las que afecta palmas y plantas, bilateral y simétrica constituida por pápulas aisladas y algunas confluentes color rosa pálido con un collarete escamoso. De evolución subaguda y asintomática.

Diagnóstico Sífilis secundaria La lesión primaria puede coexistir con manifestaciones secundarias. Manifestaciones: fiebre, cefalea y decaimiento, acompañado de un rash cutáneo y linfadenopatía generalizada no dolorosa. Lesiones cutáneas:lesiones pápulo escamosas, no pruriginosas, distribuidas simétricamente principalmente en tronco y extremidades. Es frecuente la localización palmo-plantar. Lesiones en mucosas: condilomas planos en región vulvar y perianal, parches mucosos blanquecinos húmedos en mucosa bucal y genital, boqueras y otras. Las lesiones son altamente infectantes por contener gran cantidad de treponemas en su superficie. Alopecía en parches y alopecía de la cola de las cejas. Sin tratamiento estas manifestaciones cutáneas y mucosas desaparecen espontáneamente. Se presentan en episodios de tres a cuatro semanas de duración y en forma recurrente.

Definición •





Sífilis secundaria: Etapa de diseminación hematógena del Treponema pallidum. Se manifiesta dentro de los 6 primeros meses después de la infección, habitualmente 6 a 8 semanas. Sífilis Terciaria: Etapa destructiva de la enfermedad, que se desarrolla años después de la infección primaria en pacientes no tratados o tratados inadecuadamente. En pacientes VIH positivos, inmunodeprimidos, puede ocurrir precozmente. Sífilis congénita: Adquirida por el feto por vía transplacentaria durante el período de gestación, desde una madre con Sífilis no tratada o inadecuadamente tratada.

Sífilis Terciaria

Etiología-Epidemiología-Fisiopatología

Sífilis cardiovascular por compromiso de grandes vasos y válvulas cardíacas, con aortitis que puede complicarse con estenosis coronaria, aneurisma aórtico acompañado de compromiso de las coronarias e insuficiencia valvular aórtica. Gomas sifilíticos cutáneos, mucosos (boca, paladar, faringe, laringe y tabique nasal, se ulceran quedando lesiones con aspecto de sacabocado), y óseos (con mayor frecuencia en los huesos largos, borde anterior de la tibia). Rx: engrosamiento del periostio. Formación de callos óseos irregulares y sensibles

Tiene una incidencia de 23 por 100.000 hab, más frecuente entre 20-30 años en regiones de Tarapacá y Atacama. Sifilis congénita 0.64 por 100.000 RN vivos. Se divide en etapa precoz (primaria, secundaria y latente precoz) y tardía (latente tardía y terciaria), cuyo límite se sitúa en un año. Las lesiones cutáneo mucosas de las formas precoces son contagiosas y las manifestaciones de las formas

13 Facultad de Medicina, Universidad de Chile

que, en ocasiones, pueden palparse. Las lesiones osteolíticas pueden producir perforación del paladar duro o del tabique nasal; las lesiones en el cráneo se describen radiológicamente como “comido por gusanos”. Como método diagnóstico: Ultramicroscpía de campo oscuro, serología test serológicos no treponémicos (VDRL-RPR) reactivos a títulos elevados (> 1:8), test treponémicos en caso de duda con test no treponémicos. Nuevas métodos: PCR, ELISA. IFD.

por kilo de peso por dosis EV cada 12 hrs de 0-7 dias de vida,cada 8 hrs 8-28 dias y cada 6 hrs en mayores de 28 dias, por 14 dias.

Seguimiento Serológico con examen de serología no treponémica cuantitativa (VDRL) en los meses 1º - 3º - 6º y 12º post tratamiento. Reacción de jarisch-herxheimer • Liberacion masiva de ags a las pocas horas de la penicilina. • 90% en sifilis secundaria,50% en primaria y 25% en latente precoz. • Se caracteriza por fiebre alta, rubor facial y cefalea. • No confundir con alergia a penicilina(dura pocas hrs). • No se repite en proximas dosis.

Sífilis congénita Un 60% de los recién nacidos infectados puede ser asintomático al momento del parto. Precoz, se manifiesta desde la concepción hasta los dos primeros años y tardía, después de los dos años. Algunas manifestaciones precoces:abortos, mortinatos, cuadro multisistémico fulminante, lesiones ampollares palmo-plantares, exantema maculopapular simétrico, parches mucosos en lengua, borde lingual y garganta que pueden producir estridor laríngeo, rágades y condilomas planos. La tardía evoluciona a una etapa de latencia, siendo sus manifestaciones similares a la de la enfermedad adquirida, incluyendo la formación de gomas y el desarrollo de neurosífilis. Algunos pueden presentar estigmas como: dientes de Hutchinson, molares de mora, perforación del paladar duro, nariz en silla de montar, tibias en “sable”, opacidades corneales, atrofia óptica, sordera por compromiso del octavo par, hidrartrosis (articulación de Clutton).

Autor / Editor Marcelo Fres

Se confirma con antecedentes de sífilis materna confirmada durante el embarazo no tratada o inadecuadamente tratada, serología del RN 4 títulos superior al de la madre, y criterios clínicos, de laboratorio y radiológicos (radiografía de huesos largos al nacer y control al mes) en el recién nacido. Los exámenes treponémicos (FTAAbs/MHA-TP) no son útiles para el diagnóstico precoz de sífilis congénita, sin embargo, en niños los test treponémicos reactivos después de los 12 meses confirman el diagnóstico de forma retrospectiva. En la misma situación un test treponémico negativo no descarta este diagnóstico.

Tratamiento Sífilis Primaria, Sífilis Secundaria, Sífilis Latente Precoz: • PNC benzatina 2.400.000 UI/semana IM, por dos semanas consecutivas. • En pacientes alérgicos a la PNC (excepto embarazadas): doxiciclina, tetraciclina, ceftriaxona. Sífilis Latente Tardía: • PNC benzatina 2.400.000 UI/ semana IM, por tres semanas consecutivas. • En pacientes alérgicos a la PNC (excepto embarazadas): doxiciclina, tetraciclina. Sífilis congénita • Penicilina sódica es el tratamiento de elección 50.000 UI

14 Facultad de Medicina, Universidad de Chile

Revisor de Formato Verónica Vasquez

Año 2016

Síndrome Febril Prolongado

• • •

Nivel de manejo del médico general: Diagnóstico Específico Tratamiento Inicial Seguimiento Completo

Diagnóstico Se realiza por etapas: Primera etapa - Estudio inicial: Anamnesis: Características de la fiebre, higiene básica, mascotas, viajes, comidas, medicamentos,etc. Examen físico completo: Exantema, lesiones cutáneas, adenopatías, otoscopía, fondo de ojo, ex. neurológico,etc. Exámenes básicos: Hemograma, VHS, PCR, examen de orina. Hemocultivos (2) con búsqueda de fastidiosos, PL en menores de 2 años con síntomas neurológicos. Estudio serológicos para Mycoplasma, Bartonella, VEB, CMV y Parvovirus. Radiografía de tórax en mayores de 2 años, incluso sin síntomas respiratorios. Coprocultivo y EPSD si la anamnesis lo amerita.

Aspectos Esenciales • • • •

Misceláneas: 11% (Drogas/alergias, EII, Enf. de Kawasaki, fiebre facticia, fiebre mediterránea familiar). Mesenquimopatías: 10% (Un 50% ARJ). Neoplásicas: 6-15% (Linfoma, leucemia, tumores sólidos, neuroblastoma, tumor de Wilms).

Fiebre de etiología desconocida que dura más de 7 días, con exámenes básicos normales. Tiene múltiples causas, en general enfermedades comunes de presentación atípica. Etiología principalmente infecciosa. Su estudio es por etapas, desde exámenes básicos hasta los más complejos.

Caso Clínico Tipo

Segunda etapa - Estudios Específicos: Si no se encuentra nada, continuar con: Ecocardiograma, ecografía abdominal, cintigrafía renal (si OC +, UC – ) y ósea (en focalidad osteomuscula), TAC de tórax y abdomen (linfonodos o abscesos), observar HC por más de 7 días (agentes de crecimiento lento), biopsias de sitios sospechosos (linfonodos o lesiones cutáneas), mielograma y mielocultivo, ferritina (elevación en reumatismos o enfermedades malignas crónicas), estudio reumatológico (ANA, FR, ANCA, IgI, IgM, IgA, electroforesis de proteínas) y hemato-oncológico.

Lactante, sin antecedentes mórbidos, consulta por fiebre, sin otros síntomas asociados, examen físico normal. Se realiza hemograma, PCR, examen de orina con urocultivo, una radiografía de tórax, todos sin alteraciones. Actualmente está en su octavo día de fiebre, esperando nuevos estudios.

Definición Múltiples definiciones: • Fiebre que persiste por más de 7 días y cuya causa no se logra identificar después de un estudio de laboratorio inicial para descartar las etiologías más frecuentes (Hemograma, PCR, Ex. orina y Rx. Tórax). Generalmente debido a una enfermedad común de presentación atípica. • Fiebre repetida en paciente ambulatorio > 3 semanas. • Fiebre con temperatura > 38° C rectal o > 37,5° C axilar en paciente hospitalizado con una evaluación completa por > 1 semana sin historia, examen físico o exámenes de laboratorio que orienten a una causa específica. • Aquel síndrome febril cuya duración excede el habitual para el diagnóstico clínico (Ej: IRA Alta > 10 días).

Tercera etapa - Considerar Seguimiento: Reexaminar en busca de nuevos signos, evetualmente repetir exámenes. Dado la amplia cantidad de etiologías posibles, no es recomendable usar antibióticos empíricos en un primer momento a menos que haya una fuerte sospecha de que se trata de una infección bacteriana sin tratamiento.

Tratamiento Según etiología.

Etiología-Epidemiología-Fisiopatología

Seguimiento

Las etiologías son múltiples, y con muy variado pronóstico, sin embargo si después de un estudio acabado no se logra llegar a un diagnóstico, el pronóstico en general es bueno. Las causas más frecuentes son:

Según etiología. Derivación a especialista con estudio básico realizado.

• •

Infección: 50%, destacando las de tipo viral (VEB, CMV, enterovirus) y luego las bacterianas (ITU, bartonellosis, neumonia, abscesos y meningitis subaguda). Sin diagnóstico: 23%.

15 Facultad de Medicina, Universidad de Chile

Endocarditis Infecciosa

medio de edad de los pacientes es 50 años. Mayor proporción hombres/mujeres. Ha aumentado el número de casos de EI en portadores de válvulas protésicas, marcapasos y desfibriladores implantables, y en adultos mayores, en diabéticos y sujetos sometidos a hemodiálisis.

Nivel de manejo del médico general: Diagnóstico Sospecha Tratamiento Inicial Seguimiento Derivar

Fisiopatología: Patologías como la valvulitis reumática, cirugía valvular o roce de un catéter generan lesiones en el endocardio, permitiendo el depósito de plaquetas y fibrina, produciéndose una vegetación o acúmulo fibrino-plaquetario estéril, lesiones que son propias de la endocarditis trombótica no bacteriana. Las bacterias llegan a este sitio y deben adherirse a esta vegetación y producir la colonización. El origen de estas bacteremias puede encontrarse en mucosas traumatizadas y previamente colonizadas, como es el caso de la mucosa oral o en procedimientos urogenitales, gastrointestinales y respiratorios.

Aspectos Esenciales • • • •

Enfermedad de origen generalmente bacteriano. Sin tratamiento tiene desenlace fatal. Factores de riesgo: cardiocirugía, drogadictos endovenosos, marcapasos, daño o prótesis valvular, diabéticos, hemodiálisis. Tratamiento se basa en la antibioterapia, pero puede incluir cirugía.

Diagnóstico Caso Clínico Tipo

Manifestaciones y sospecha clínica: fatigabilidad, disnea, anorexia, sudoración, fiebre remitente con valvulopatía y fenómenos embólicos, lesiones cutáneas y alteraciones cardiacas progresivas. Criterios de Duke para Endocarditis Infecciosa Criterios Mayores: a) Hemocultivos positivos • Microrganismos típicos compatibles con EI con al menos 2 hemocultivos separados. • Microrganismos compatibles con EI en hemocultivos persistentemente positivos definidos como: • 2 muestras de hemocultivos positivos tomados en forma separada por >12 horas. • Todos de 3 o la mayoría de 4 hemocultivos positivos separados (con la primera y la última muestra separados por 1 hora). b) Evidencia de compromiso endocárdico por: • Ecocardiograma positivo con vegetaciones en válvulas o estructuras adyacentes, en dirección del jet de regurgitación, o en material implantado en ausencia de una explicación anatómica alternativa, o abscesos, o nueva dehiscencia parcial de válvula protésica. • Nueva regurgitación valvular (el empeoramiento o cambio de un soplo preexistente es insuficiente).

Paciente de sexo masculino de 49 años de edad consulta por compromiso de estado general, disnea y fiebre de 39°C axilar. Al ingreso se toma una radiografía de tórax sin particularidades y un ecocardiograma que muestra: vegetación de 6 x 12 mm, adherida sobre el lado ventricular de la valva anterior de la tricúspide.

Definición Enfermedad causada por microrganismos, que afecta el endocardio y estructuras cardiacas y/o prótesis endovasculares. Su lesión característica es la vegetación, la que habitualmente se produce en una válvula cardiaca. La mortalidad es de 10-30% con terapia adecuada y del 100% cuando no es tratada.

Etiología-Epidemiología-Fisiopatología Bacterias causantes: Lesión en válvula nativa: • E.I. subaguda: 80 a 95%. S. Viridans, Streptococcus spp., Enterococcus spp., Staphylococcus spp. • E.I. aguda: Staphylococcus aureus.

Criterios Menores: • Cardiopatía predisponente o uso de drogas endovenosas. • Fiebre (temperatura >38,0° C). • Fenómenos vasculares (embolia arterial mayor, infartos pulmonares sépticos, aneurisma micótico, hemorragia intracraneal, hemorragia conjuntival y lesiones de Janeway). • Fenómenos inmunológicos (glomerulonefritis, nódulos de Osler, manchas de Roth, y factor reumatoide). • Evidencia microbiológica (hemocultivos positivos pero no como criterio mayor o evidencia serológica de infección activa con organismos compatibles con EI).

Lesión en válvula protésica: • Aguda (antes de un año) 50%, Staphylococcus spp., Streptococcus spp., hongos (Aspergilluss spp., Candida spp., etc). • Tardía: Streptococcus spp., Enterococcus spp., epidermidis, S. aureus, bacilos gram negativos. Usuarios de drogas intravenosas (raro en Chile): S. Aureus. La incidencia de la EI en la comunidad es de alrededor de 2 nuevos casos por 100.000 personas al año. El pro-

16 Facultad de Medicina, Universidad de Chile



Hallazgos ecocardiográficos (compatible con EI pero no encontrado como criterio mayor).

Criterios clínicos para endocarditis infecciosa requiere: dos criterios mayores, o uno mayor y tres criterios menores, o cinco criterios menores. El hemocultivo es la prueba más importante para el diagnostico etiológico de la enfermedad. Además, el ECG debe registrarse al ingreso. La presencia de un trastorno de conducción (bloqueo AV o de rama) debe orientar a infección miocárdica. Un PR prolongado, puede implicar un remplazo valvular y un peor pronóstico. Infartos silentes por embolización de vegetaciones a arterias coronarias, pueden ser identificados en este examen.

Tratamiento Para Streptococcus grupo viridans: • PNC sódica 5 millones cada 6 hrs ev por 4 semanas + gentamicina 1 mg/kg cada 8 hrs por 2 semanas. • En caso de encontrar resistencia a PNC: vancomicina 15 mg/kg c/12 hrs. • Para S. aureus: • Sensible a PNC:cloxacilina 12 g/día ev (3 g c/6 hrs). • Para S. aureus resistente: vancomicina 15 mg/kg c/12 hrs ev por 4-6 semanas + rifampicina 600 mg/día (opcional) por 4-6 semanas. • Indicaciones de Cirugía para tratamiento en EI: • Falla cardíaca congestiva moderada a severa causada por disfunción valvular. • Prótesis inestable, o con orificio protésico obstruido. • Ausencia de terapia antimicrobiana efectiva: endocarditis causada por hongos. • Endocarditis en válvula protésica por S. aureus con complicación intracardíaca. • Recaída de endocarditis en válvula protésica a pesar de tratamiento antimicrobiano óptimo.

Seguimiento En pacientes con complicaciones intra o extracardíacas, que se encuentren en lugares alejados de centros médicos terciarios que incluyan cardiocirugía, se recomiendan los criterios de derivación: • Insuficiencia cardíaca secundaria a insuficiencia valvular aguda o disfunción protésica. • Sepsis persistente (>7-10) a pesar de terapia antibiótica adecuada. • Etiología no habitual (hongos). • Embolismo a repetición.

Autor / Editor Marco Ayala.

Año 2017

17 Facultad de Medicina, Universidad de Chile

Infección Invasiva de Partes: Celulitis, Fasceítis, Miositis Necrotizantes o Septicémicas

yección, quemadura o vesícula de varicela. Presentación inicial idéntica a celulitis, con progresión a velocidad variable hacia destrucción de tejidos con compromiso sistémico, sepsis y shock. Sospechar en ausencia de respuesta o deterioro con terapia antibiótica adecuada, o en celulitis con necrosis cutánea o bulas. • Dos formas según bacteriología: monomicrobiana casi siempre por S. pyogenes con una mortalidad del 50% a 70%, y polimicrobiana por flora intestinal (coliformes y anaerobios). • Sospechar en el contexto de cirugía/trauma abdominal, úlceras de decúbito, abscesos perianales, sitio de inyección en drogadicción, absceso genital femenino. • Estudio de imágenes: TAC o RNM permiten delimitar extensión del proceso. • Estudio microbiológico: se pueden obtener hemocultivos y/o cultivos de exudado o de muestras de tejido al momento de la disección y debridación. Celulitis necrotizante sinergística (de Meleney) • Muy similar a fasceítis necrotizante, pero compromete músculos profundos además de piel y fascia. Presenta necrosis cutánea importante. • Más frecuente con abscesos perirrectales o isquiorectales. •

Nivel de manejo del médico general: Diagnóstico Sospecha Tratamiento Inicial Seguimiento Derivar

Aspectos Esenciales • • •

Es una urgencia. Sospechar ante celulitis que pese a terapia adecuada no responde o evoluaciona con deterioro clínico. Lo más importante del tratamiento es la debridación de los tejidos necróticos.

Caso Clínico Tipo Paciente de 12 años, con antecedente de dermatitis atópica, cursando actualmente 3er día de varicela, consulta por aparición de eritema asociado a las lesiones y dolor intenso a la palpación. Sospecha principal: fasceitis necrotizante, iniciar tratamiento hospitalizado con ATB ev.

Miositis - Piomiositis: Infección de un músculo aislado. Generalmente por S. aureus. Se caracteriza por dolor en músculo afectado y claudicación en movimiento correspondiente. Puede no palparse absceso. Estudio con ECO, TAC o RNM demuestra el foco. - Miositis estreptocócica anaerobia: Corresponde a Infección de tejidos profundos por estreptococos anaerobios. Es similar a fasceítis necrotizante pero más larvado y generalmente asociado a trauma o cirugía. - Mionecrosis Clostridial o gangrena gaseosa: Infección fulminante de tejidos profundos por especies de Clostridium, la mayoría de las veces C. perfringens. • Forma traumática secundaria a penetración de la piel: Dolor muy intenso, in crescendo, en zona de penetración, iniciado 24 horas post-herida. Piel inicialmente muy pálida, se va tornando rojo-violácea, aparecen bulas y se pueden palpar crépitos. Compromiso sistémico con progresión rápida a sepsis y shock. Gas visible en imágenes (TAC). • Forma espontánea: diseminación hematógena de especies de Clostridium más aerotolerantes, desde lesión intestinal (cáncer).

Definición Infecciones mono o polimicrobianas de compartimientos profundos, con gran destrucción de tejido.

Etiología-Epidemiología-Fisiopatología La celulitis es un trastorno inflamatorio agudo de la piel que puede estar causada por flora autóctona que coloniza la piel y los fanéreos (p. ej., S. aureus y S. pyogenes) o por diversas bacterias exógenas. La fasceítis necrosante puede asociarse a una infección por estreptococos del grupo A, a una infección mixta por bacterias anaerobias o puede formar parte de la gangrena gaseosa causada por Clostridium perfringens. S. pyogenes puede causar una miositis primaria denominada miositis necrosante.

Tratamiento Evaluación inicial: • Observar las estructuras profundas, • Eliminar el tejido necrótico, • Reducir la presión compartimental, • Obtener material apropiado para la tinción de Gram y para cultivos para aerobios y anaerobios. Se requiere manejo agresivo médico-quirúrgico: hospitalizar

Diagnóstico Fasceítis necrotizante • Infección de la fascia subcutánea (no muscular ni aponeurosis). • En el 80% se extiende desde lesión cutánea menor como abrasiones, heridas, mordeduras de insectos, sitio de in-

18 Facultad de Medicina, Universidad de Chile

en centro adecuado. Intervención quirúrgica precoz en ausencia de respuesta a ATB o en deterioro clínico, con debridación de tejido necrótico, conservando tejido viable. Se requieren reevaluaciones en pabellón, hasta retirar todo el tejido necrótico. Es esta la medida más importante. Esquema empírico en F. necrotizante monomicrobiana y gangrena gaseosa: • Penicilina sódica 24 millones UI ev, divididas en 4 a 6 dosis c/día + Clindamicina 600-900 mg ev c/8 horas. Esquemas empíricos en F. necrotizante polimicrobiana: • Cefotaxima 2 gr ev c/6 horas + Metronidazol 500 mg ev c/6 horas ó Clindamicina (dosis idem). • Ampicilina-Sulbactam 2 gr ev c/6-8 horas + Ciprofloxacino 400 mg ev c/12 horas + Clindamicina (dosis idem). • Imipenem 1 gr ev c/6-8 horas ó Meropenem 1 gr ev c/8 horas.

Seguimiento Se recomienda profilaxis a contactos cercanos en pacientes con fasceítis necrotizante por estreptococo tipo A en pacientes inmunosuprimidos o con cirugía reciente. Está indicado en este caso penicilina oral 250 mg cada 4 horas por 24 a 48 horas. Derivar.

Autor / Editor Marco Ayala.

Año 2017

19 Facultad de Medicina, Universidad de Chile

Meningitis Aguda

compatible con una meningitis. Se trata de una emergencia médica por el aumento considerable de la presión intracraneana (PIC) que puede llevar a enclavamiento, dejando severas secuelas neurológicas en el paciente. Las posibles causas para una meningitis pueden ser: • Bacterianas (meningitis piógena): S. pneumoniae, N. meningitidis, Streptococcus del grupo B, H. influenzae y Listeria monocytogenes (más común en niños, adultos mayores, embarazadas y puérperas). Staphylococcus (cuando existe el antecedente de intervenciones quirúrgicas o traumatismos). M. tuberculosis (es común que sea de evolución subaguda o crónica).

Nivel de manejo del médico general: Diagnóstico Específico Tratamiento Inicial Seguimiento Derivar

Aspectos Esenciales • • • • •

Los agentes más frecuentes son S. pneumoniae y N. meningitidis. Signos meníngeos más característicos son: rigidez de nuca, Brudzinski y Kernig. La espondilopatía en ancianos puede ser motivo de un “falso positivo” para rigidez de nuca. La triada clásica es cefalea, fiebre y rigidez de nuca. Cuando se tiene sospecha de meningitis piógena el tratamiento empírico es con ampicilina y una cefalosporina.

• •

Viral: Herpesvirus, enterovirus, arbovirus, virus de la parotiditis, VIH. Micóticas: Cryptococcus (se presenta de forma subaguda en pacientes VIH).



Parasitarias



Recordar que hay muchas otras causas de irritación de las meninges como hemorragias o neoplasias, temas que serán abordados en otros capítulos.

Caso Clínico Tipo Diagnóstico

Paciente masculino de 62 años consulta por cefalea y sensación febril. Refiere haber tenido una neumonía hace 2 días. Al examen físico se encuentra fiebre de 38°C y rigidez de nunca.

Presenta fiebre y síndrome meníngeo, es decir, síndrome de hipertensión endocraneana en grados variables (principalmente cefalea), alteraciones del LCR y manifestaciones dependientes de la irritación (signos como rigidez de nuca, signos de Kernig y de Brudzinski). Pueden existir alteraciones de conciencia o convulsiones en algunos casos. Cabe destacar el caso de la meningitis meningococica la que puede presentar púrpura y equimosis. El uso de TAC sólo se justifica en casos de reciente traumatismo, inmunodeficiencia, neoplasias diagnosticadas del SNC, signos focales, alteración de conciencia o papiledema.

Definición Cuadro clínico agudo debido a un trastorno infeccioso de las meninges.

Etiología-Epidemiología-Fisiopatología

Una vez descartada una lesión focal, se debe hacer una punción lumbar (inclusive puede ser después del tratamiento). El diagnóstico diferencial respecto al LCR se ve en el siguiente cuadro: Antes del tratamiento se deben realizar hemocultivos. Luego

Habitualmente es precedida de un cuadro infeccioso respiratorio o gastrointestinal, aunque también puede no haber antecedente previo. La llegada al SNC es por vía hematógena, generalmente en horas o días el paciente presenta el cuadro

20 Facultad de Medicina, Universidad de Chile

de tomada la muestra de LCR se debe buscar el microorganismo mediante cultivos generales y específicos o técnicas como PCR.

Tratamiento El tratamiento empírico en el adulto para meningitis bacteriana puede ser: ampicilina (2 gr c/4 hrs), + Cefotaxima (2 gr c/4 hrs), o Ceftriaxona (2 gr c/12 hrs). Luego cambiar tratamiento a específico cuando se obtengan los resultados de las muestras. Antes del tratamiento antibiótico se pueden utilizar corticoides (dexametasona 10 a 16 mg c/6 a 12 hr por 2 a 4 días), ya que existe evidencia sugerente de disminución de la mortalidad. En el caso de la meningitis vírica el tratamiento es sintomático con analgésicos, antipiréticos y antieméticos. En el caso de la enfermedad grave por un herpesvirus se recomienda el uso de aciclovir IV (15-30 mg/kg/día en tres dosis), y luego continuar oral por 14 días.

Seguimiento Debe ser derivado a especialista, algunos de estos agentes como N. meningitidis o algunos virus son de notificación obligatoria.

Autor / Editor Marco Ayala.

Año 2017

21 Facultad de Medicina, Universidad de Chile

Septicemia

La sepsis puede avanzar a algo más severo, de manera progresiva: Sepsis grave: Sepsis asociada a disfunción orgánica o hipoperfusión tisular (hiperlactacidemia, oliguria y compromiso de conciencia). Shock séptico: Sepsis asociada a hipotensión e hipoperfusión tisular, en la que no se logra respuesta adecuada a volumen. Disfunción multiorgánica: Alteraciones agudas, aditivas y en cadena, de diferentes órganos o sistemas, de forma que la homeostasis no puede ser mantenida sin intervención. Potencialmente reversible. Falla orgánica múltiple: Disfunción multiorgánica en su fase más grave la que es irreversible.

Nivel de manejo del médico general: Diagnóstico Sospecha Tratamiento Inicial Seguimiento Derivar

Aspectos Esenciales • • • •

Los agentes más frecuentes son bacterias Gram (+). Sospechar cuando se presenten 2 o más criterios de SIRS. Para tratarlo se deben controlar las constantes, tomar medidas generales de estabilización y soporte. La terapia antimicrobiana debe ser de de escalación.

Tratamiento Caso Clínico Tipo

• • • •

Hombre de 70 años acude al servicio de urgencia por cuadro de 3 días de evolución caracterizado por tos, disnea y cefalea. Al ingreso presenta fiebre cuantificada en 38°C, FC 120, FR 35 y en el examen de sangre presenta 3500 leucocitos.



Definición Se trata de un síndrome de respuesta inflamatoria sistémica (SIRS) causada por una infección sospechada o demostrada.

• •

Etiología-Epidemiología-Fisiopatología



Microorganismos causales más frecuentes: bacterias Gram positivas (30-50%), bacterias Gram negativas (25-35%), hongos (3-5%), virus (2-4%) y parásitos (1-3%). • 1era fase: Activación de las células inflamatorias y liberación de sus mediadores. • 2da fase: Activación de los sistemas endocrino, autocrino y paracrino que conduce a SIRS. • 3era fase: aparece la disfunción y falla orgánica múltiple.





Las citoquinas más importantes involucradas son: TNFα, IL-1, IL-2, IL-6, IL-8 e INF γ.

Control de constantes. Medidas generales de estabilización y soporte. Mantenimiento adecuado de perfusión tisular: Fluidoterapia: Suero salino al 0,9% o Ringer lactato para mantener PAM ≥65 mmHg, FC ≥110 lpm y PVC 8-12 cm H20. Si se mantiene la hipotensión a pesar de la administración de 2-3 litros (entre 1-3 horas de iniciado el tratamiento): Utilizar agentes vasopresores: dopamina: 5 μg/kg/min IV incrementandose 5 μg cada 5-10 min hasta 25 μg/kg/ min y noradrenalina: 0,05-0,1 μg/Kg/min IV pudiendose aumentar hasta 4 μg/kg/min. Medidas de soporte de los órganos insuficientes. Manejo quirúrgico en caso necesario (abcesos, colecciones, peritonitis, etc) Terapia antimicrobiana de de-escalación. Prevenir infecciones nosocomiales especialmente si está en UCI. Foco desconocido de “origen extrahospitalario”: imipenem 1 g c/6 h o meropenem 1-2 g c/8 hrs IV + Amikacina 15-20 mg/kg/dia IV +/- Vancomicina 1 g c/8-12 h o linezolid 600 mg c/12 h IV. Foco desconocido de “origen intrahospitalario” o dados de alta recientemente: misma indicación anterior, se prefiere meropenem y se debe agregar vancomicina o linezolid.

Seguimiento Este paciente debe ser ingresado a una unidad para pacientes críticos.

Diagnóstico Para hablar de síndrome de respuesta inflamatoria sistémica se debe contar con la presencia de 2 o más de los siguientes factores: • Temperatura >38°C o <36°C. • Frecuencia cardiaca >90 latidos/min. • Frecuencia respiratoria >20/min o PaCO2 <32 mmHg. • Leucocitos >12000/mm3 o <4000/mm3 o >10% inmaduros.

Autor / Editor Marco Ayala.

22 Facultad de Medicina, Universidad de Chile

Año 2017

Parasitosis Intestinales

Caso Clínico Tipo Un asistente sanitario de 36 años, sufre desde hace dos meses cuadro abdominal intermitente de náuseas, heces pastosas, flatulencia, meteorismo y baja de peso de 3 kg.

Nivel de manejo del médico general: Diagnóstico Específico Tratamiento Completo Seguimiento Completo

Definición Las parasitosis digestivas son originadas por protozoos y helmintos que comprometen fundamentalmente el intestino (delgado y grueso) y, excepcionalmente, otras partes del tubo digestivo.

Aspectos Esenciales • • • •

Fecalismo más frecuente en niños, y carnivorismo en adultos. Eosinofilia presente en helmintos, pero no en protozoos (excepto Isospora). Diagnóstico con EPSD. Protozoos se tratan con Metronidazol, cestodos con Praziquantel y nematodos con Mebendazol o Albendazol.

Etiología-Epidemiología-Fisiopatología Los agentes etiológicos responsables de parasitosis intestinales de mayor importancia en nuestro medio son:

Todos los protozoos intestinales tienen como mecanismo de transmisión el fecalismo humano y destacan especialmente Giardia y Cryptosporidium como agentes de gastroenteritis por aguas contaminadas.

Giardia, Isospora o Cryptosporidium, o disentérica en amebiasis (recordar que E. hystolitica puede dar infecciones extraintestinales como es el absceso hepático amebiano). Las Giardosis pueden presentar un síndrome de malabsorción en el contexto de una diarrea crónica.

Entre los nemátodos, la mayoría es por geofagia excepto Anisakis que es por el consumo de peces no cocidos. En cuanto a los céstodos, el carnivorismo es el principal mecanismo de transmisión, sea de vacuno en T. saginata, de cerdo en T. solium y de peces de agua dulce en Diphyllobotrium. La excepción a esto último lo hace Hymenolepis que es por fecalismo humano.

Los cuadros debidos a cestodos principalmente se manifiestan por dolor abdominal, baja de peso y en algunas ocasiones porque el paciente observa las proglótidas en las heces. Hymenolepis también puede dar vómitos y diarrea con Sd. de malabsorción. En cuanto a los nemátodos, Tricocefalosis puede dar diarrea disentérica, en cambio las Ascariasis no dan mucha sintomatología intestinal, excepto cuando son masivas; generalmente dan más manifestaciones respiratorias cuando realizan el ciclo de Loos (Sd. de Loeffler). Anisakis da más sintomatología digestiva superior al comienzo (disfagia, dolor epigástrico, vómito; incluso se puede expulsar el gusano por la boca) y luego de varios días puede presentar síntomas más intestinales.

La mayoría de las parasitosis intestinales que son por fecalismo afectan más a niños (excepto Isospora y E. histolytica) y las que son por carnivorismo más a los adultos. Los geohelmintos (Ascaris y Trichuris) y los parásitos por carnivorismo han disminuido su frecuencia en zonas urbanas, pero aún se mantienen vigentes en zonas rurales.

Diagnóstico

Una característica particular de las parasitosis es la eosinofilia. Los helmintos tisulares dan las eosinofilias más notorias (en este tema destacaría Ascaris en su paso por el pulmón). Se describe clásicamente que los helmintos intestinales también las presentan, aunque menos notorias. Los protozoos

Cuadro Clínico: Las parasitosis intestinales por protozoo tienen como síntoma principal la diarrea, la que puede ser acuosa en casos de

23 Facultad de Medicina, Universidad de Chile

intestinales no dan eosinofilia, con excepción de Isospora, la que incluso puede presentar Cristales de Charcot-Leiden en las deposiciones. El diagnostico se hace con EPSD (estudio parasitológico seriado de deposiciones) en casi todos estos casos. Cuando se sospecha de Isospora o Cryptosporidium el EPSD debe hacerse con tinción de Ziehl-Nielsen. En el caso de Anisakis, el diagnostico es endoscópico.

Tratamiento Se debe hacer el tratamiento general de una deshidratación o por Sd de malabsorción si lo presenta. El tratamiento es ambulatorio, excepto en T. solium el cuál es hospitalario con aislamiento entérico (para prevenir Cisticercosis). Cuando el agente es un protozoo el tratamiento es con Metronidazol ( Giardasis: Adultos 30 mg/kg -niños 15 mg/kg/dvo c/8 hrs x 5 días / Amebiasis: Adultos 30 mg/kg/día -niños 30-50 mg/kg/día dividido en 3 dosis- en una dosis x 10 días), excepto en el caso de Cryptosporidium (Nitazoxanida 500 mg vo c/ 12 hrs por tres días) e Isospora (Trimetoprima-Sulfametoxazol 160/800 mg c/ 6 hrs por 10 días). Cuando el agente es un cestodo se trata con Praziquantel en dosis única (10 mg/kg/día) Cuando el agente es un nemátodo se trata con un azol, como Mebendazol (500 mg dosis única o 100 mg c/ 12 hrs por 1-3 días, dependiendo el parásito) o Albendazol (400 mg dosis única).

Seguimiento Además de evaluar la efectividad del tratamiento en una consulta posterior, se debe hacer una asesoría educacional sanitaria sobre los mecanismos de infección para evitar otras reinfecciones por la misma causa dentro de la comunidad.

Autor / Editor Gustavo Muñoz R.

Año 2017

24 Facultad de Medicina, Universidad de Chile

Síndrome Mononucleósico

tensa odinofagia, hepatoesplenomegalia, exantema cutáneo máculopapular, compromiso del estado general (astenia, adinamia, anorexia). En el hemograma: leucocitosis mononuclear con linfocitosis atípica >10%.

Nivel de manejo del médico general: Diagnóstico Específico Tratamiento Completo Seguimiento Completo

Para el diagnóstico etiológico la presencia de anticuerpos heterófilos (+) permite diferenciar entre la MI y otras etiologías. En la MI, la confirmación diagnóstica descansa en la serología para VEB, con positividad para IgM anti-VCA (antígeno de la cápside viral) y más tardíamente, anti-EBNA (antígeno nuclear), a las 3-6 semanas.

Aspectos Esenciales • • •

Confirmación diagnóstica mediante serología: IgM anti-VCA y anti-EBNA. Tratamiento sintomático con AINEs. La primoinfección por VIH siempre debe considerarse y descartarse.

Tratamiento En el caso de la MI, el tratamiento es sintomático con paracetamol y AINES. En los SM de otra etiología, el tratamiento también es sintomático y específico para el agente causante si corresponde.

Caso Clínico Tipo

Seguimiento

Hombre de 19 años, sin antecedentes de relevancia, consulta por cuadro de 2 semanas de evolución caracterizado por sensación febril, intensa odinofagia, astenia, anorexia y más recientemente exantema. Al examen físico destacan fiebre de 38,5°C, múltiples adenopatías cervicales y exantema máculopapular.

La MI es autolimitada, persistiendo IgG anti-VCA de por vida. No obstante, puede asociarse a múltiples complicaciones, como: anemia hemolítica, trombocitopenia, rotura esplénica, síndrome de Guillain-Barré, miopericarditis y falla hepática grave.

Definición Síndrome inicialmente descrito para la mononucleosis infecciosa, que se caracteriza principalmente por presencia de adenopatías generalizadas, fiebre y compromiso del estado general.

Autor / Editor Julián Bravo

Etiología-Epidemiología-Fisiopatología La causa más frecuente es la mononucleosis infecciosa (MI), infección de linfocitos B dada por el virus Epstein-Barr (VEB) y caracterizada por la presencia de anticuerpos heterófilos positivos. Transmisión por saliva (“enfermedad del beso”), alcanzando la enfermedad mayor frecuencia en jóvenes entre 15 y 25 años. Causas de síndrome mononucleósico (SM) con anticuerpos heterófilos negativos son la infección por citomegalovirus, toxoplasmosis, hepatitis virales, rubéola, leucemias, linfomas y la primoinfección por VIH.

Diagnóstico La presentación asintomática u oligosintomática es la más frecuente. El cuadro clínico es caracterizado por fiebre, adenopatías (de predominio cervical), faringoamigdalitis con in-

25 Facultad de Medicina, Universidad de Chile

Revisor de Formato Rafaella Reginato

Año 2016

Tóxico-Infección Alimentaria

rior del organismo como es Vibrio cholerae (infrecuente en Chile), Escherichia colienterohemorrágica, Shigella o Vibrio parahaemolyticus, que en general provocan una diarrea inflamatoria, se considera que son una infección propiamente tal, por lo que no corresponde a este capítulo.

Nivel de manejo del médico general: Diagnóstico Específico Tratamiento Completo Seguimiento Completo

Otro caso es el de la llamada Marea roja, término que se usa para las floraciones algales nocivas que producen toxinas marinas con distintas presentaciones clínicas, entre ellas el veneno amnésico, veneno paralizante y veneno diarreico del molusco. También existen sustancias de origenquímico como en Escombrotoxismo, por falla en cadena de frío en peces del género escombroides (atún y palometa), que forman una amina vasoactiva.

Aspectos Esenciales • • •

Los principales organismos preformadores de toxinas son S. aureus, B. cereus, y C. perfringens. Diarrea no inflamatoria, náuseas y vómitos No existe tratamiento específico. Se basa en evitar complicaciones y la rehidratación del paciente.

Diagnóstico Se debe hacer diagnóstico diferencial con el resto de causas de diarrea aguda. Existe una diarrea aguda no inflamatoria, es decir, las deposiciones no presentan moco ni sangre en deposiciones. La fiebre es muy baja o inexistente. Además puede existir nauseas, vómitos y dolor cólico.

Caso Clínico Tipo Mujer de 40 años consulta en la noche porque dos de sus hijos presentan diarrea aguda sin signos inflamatorios. No presentan fiebre y están euvolemicos al examen físico. Al interrogatorio dirigido ella también refiere síntomas, y que durante la tarde comieron papas con mayonesa.

En exámenes de laboratorio no existe o son muy bajos los leucocitos fecales o lactoferrina fecal. En el caso de toxinas por S. aureus el comienzo de síntomas se encuentra generalmente dentro de las primeras 6 horas. C. perfringens entre las primeras 8-16 horas (tiene más dolor cólico y menos vomito). Bacillus cereus puede tener ambas presentaciones. Presentaciones más allá de 16 horas hacen pensar que no son toxinas preformadas, sino una infección intestinal. Se debe buscar signos de deshidratación e hipovolemia. Si la clínica es categórica y no se presentan dudas de otras entidades, NO es necesario mayor estudio.

Definición Causa no inflamatoria de diarrea aguda, habitualmente secretora, cuya etiología son toxinas preformadas por gérmenes en los alimentos antes de su consumo y causas no infecciosas como sustancias vasoactivas mediados por péptidos gastrointestinales, tales como péptido intestinal vasoactivo y gastrina.

Tratamiento Etiología-Epidemiología-Fisiopatología

No existe tratamiento específico ya que la mayoría cura espontáneamente. El tratamiento se centra en reposo y rehidratar al paciente (la OMS recomienda 2.5 g de cloruro de sodio, 2.5 g de bicarbonato sódico, 1.5 g de cloruro de potasio y 20 g de glucosa en un litro de agua). El uso de antiespasmódicos, antipiréticos, antidiarreicos o antisecretores debe ser criterioso y no para todos los casos.

Debido a que la toxina se encuentra en alimentos de consumo general, esto constituye un problema de salud pública. Se puede dar en brotes epidémicos y es más frecuente en niños. Se puede clasificar dependiendo de la clínica que predomina. Cuando el síntoma predominante son los vómitos se debe pensar enStaphylococcus aureus (jamón, pollo, papas, mayonesa) y el Bacillus cereus (arroz frito, carnes, verduras, cereales), ambas forman toxinas preformadas. También se ha asociado a virus tales como Norovirus.

Seguimiento Se debe preguntar por cuadros similares en otros miembros de la familia y en lo posible ver si existe un brote epidémico para investigar su origen.

Cuando la diarrea acuosa predomina: Clostridium perfringens (carne, pollo, legumbres y salsas), con toxinas preformadas. Además se asocia aEscherichia coli enterotoxigénica y otros virus entéricos, como el rotavirus. Además existen bacterias que pueden formar toxinas al inte-

Autor / Editor Marcelo Fres

26 Facultad de Medicina, Universidad de Chile

Año 2016

Triquinosis

movimiento. También puede presentar síntomas oculopalpebrales como edema, inyección conjuntival y dolor al movimiento ocular. También se describe hemorragia subungueal, conjuntival y retinal. Puede presentar otros signos como rash cutáneo. En la enfermedad severa se asocia enfermedad cardiaca, por miocarditis (por inflamación a distancia), lo que produce arritmias severas, la principal causa de muerte en la triquinosis. También se asocia a compromiso pulmonar, cerebral y renal.

Nivel de manejo del médico general: Diagnóstico Específico Tratamiento Completo Seguimiento Completo

El diagnóstico de certeza se realiza por serología. No es recomendable la biopsia muscular ya que es invasiva y además no se sabe con certeza la ubicación de los quistes larvales. Al laboratorio, la eosinofilia es característica (aparece en la segunda semana).

Aspectos Esenciales • • •

Su principal agente causal es Trichinella spiralis. Se adquiere por la ingestión de carne de cerdo insuficientemente cocida con larvas enquistadas en su interior. No se recomienda la biopsia muscular. El diagnóstico confirmatorio es por serología.

Tratamiento Albendazol 400mg cada 12 horas por 10-14 días + corticoides (para disminuir la inflamación muscular).

Caso Clínico Tipo Paciente masculino de 38 años consulta por mialgias generalizadas. Al examen físico presenta 39°C. Dirigidamente refiere haber estado comiendo carne de cerdo de un lugar no autorizado durante un viaje al sur con su familia hace dos semanas. En el hemograma se observa eosinofilia.

Seguimiento Es de notificación obligatoria. Se debe investigar familiares con síntomas similares que hayan estado expuestos.

Definición Zoonosis parasitaria causada por nemátodos del género Trichinella.

Etiología-Epidemiología-Fisiopatología Es una infección reportada en todo el mundo, de gran importancia en América del Sur. Se da generalmente en brotes epidémicos esporádicos asociados a grupos familiares. Su frecuencia es mayor en zonas rurales debido a la matanza clandestina de cerdos. Se adquiere por la ingestión de carne de cerdo insuficientemente cocida con larvas enquistadas en su interior, que son liberadas en jugos gástricos. En el intestino, estas larvas crecen y copulan originando más larvas, las que posteriormente pasan al torrente sanguíneo y se enquistan en el músculo esquelético. La invasión del músculo comienza alrededor del 7° día de la infección y perdura mientras existan hembras en el intestino.

Diagnóstico El cuadro clínico se caracteriza por una fase intestinal la primera semana, que puede ser asintomática o presentarse con dolor abdominal, vómitos, náuseas y diarrea, dependiendo de la carga infectiva. En la fase muscular, después de una semana, se asocia a fiebre y mialgias que empeoran con el

27 Facultad de Medicina, Universidad de Chile

Tétanos

ni debido a la contaminación de una herida con esporas, la cual bloquea la transmisión neuronal. Ésta enfermedad, que se presenta en forma de casos esporádicos, tiene una alta letalidad, con alrededor de un 40% de muertes sobre el total de casos.

Nivel de manejo del médico general: Diagnóstico Inicial Tratamiento Inicial Seguimiento Por Especialista

Diagnóstico Generalmente se presenta con debilidad muscular y rigidez, sumamente dolorosa y espasmos frecuentemente precipitados por estímulos sensoriales, seguidos de disfunción autonómica. Los síntomas iniciales aparecen en los músculos de la cara y del cuello, luego los del tronco como espasmos en el masétero (trismus). Normalmente no se observa confusión ni fiebre alta. El diagnóstico es clínico, en ocasiones se puede aislar la bacteria en la herida. Es importante el antecedente de una puerta de entrada como heridas cutáneas, úlceras varicosas, escaras por decúbito, intervenciones quirúrgicas, inyecciones intramusculares, mordeduras de animales, proyectiles u otro tipo de herida por más insignificante que parezca. Toda herida puede ser tetanígena.

Aspectos Esenciales • • • •

El diagnóstico es clínico. Mejor forma de prevención: vacuna. En heridas de alto riesgo: 250U de inmunoglobulina tetánica humana i.m. Complicaciones: aspirado de secreciones con neumonía secundaria, desnutrición, neumonitis, neumotórax, laceraciones en boca o lengua, entre otras.

Caso Clínico Tipo Paciente de 47 años, sexo masculino quien llega al SU con signos de deshidratación, debilidad y contractura muscular generalizada, trismo y severos espasmos generalizados, opistótono y rigidez abdominal. Un familiar refiere que el paciente sufrió una herida punzante hace una semana.

Tratamiento Es a varios niveles, el desbridamiento quirúrgico de la herida es esencial. Deben administrarse de 3000 a 6000 unidades de inmunoglobulina antitetánica humana i.m. El paciente debe ingresar a UCI, sin estímulos visuales ni auditivos, garantizando una adecuada ventilación e hidratación y tratando precozmente las complicaciones que vayan surgiendo, principalmente infecciosas. Es controversial el uso de antibióticos, normalmente 2 g/día de metronidazol i.v. o v.o. Pueden usarse benzodiacepinas o fármacos paralizantes para controlar los espasmos.

Definición Es una enfermedad aguda, transmisible, no contagiosa, que ataca al ser humano y a los animales y es de distribución mundial. Con frecuencia es mortal, sobre todo en las edades extremas de la vida, siendo prevenible por la inmunización adecuada. Se caracteriza por la presencia de espasmos musculares intensos e intermitentes y rigidez generalizada, secundarios a la acción de una potente neurotoxina, conocida como tetanospasmina, elaborada por Clostridium tetani. La puerta de entrada está dada por roturas de la piel, mucosas y otras veces directamente del músculo.

Seguimiento Una vez instalado el cuadro clínico y hecho el diagnóstico, es imprescindible un adecuado seguimiento del paciente, requiriéndose de su internación en la Unidad de Cuidados Intensivos. Puesto que el tratamiento no puede curar a menudo esta enfermedad, el mejor acercamiento es su prevención. Se adjunta el esquema recomendado para la prevención de tétanos según tipo de herida y antecedentes de vacunación:

Etiología-Epidemiología-Fisiopatología Producido por intoxicación con la toxina de Clostridium teta-

Autor / Editor Marco Ayala

28 Facultad de Medicina, Universidad de Chile

Año 2017

Micosis Invasora (Aspergillosis, Candidiasis, Mucormicosis)



Uso prologando de antibióticos en pacientes inmunosuprimidos.

Nivel de manejo del médico general: Diagnóstico Inicial Tratamiento Inicial Seguimiento Por Especialista

Cuadro clínico: Aspergillosis pulmonar invasora: Cuadro más grave. Fiebre, dolor torácico, tos y hemoptisis. Inespecífico. Sospechar por factores de riesgo. Radiológicamente se manifiesta como nódulos únicos o múltiples, con o sin cavitación, consolidación en parches o segmentaria, o infiltrados peribronquiales con o sin patró de árbol en brote.

Aspectos Esenciales

Otros: traqueobronquitis, rinosinusitis, absceso cerebral, endoftalmitis, endocarditis, compromiso de piel y tracto gastrointestinal.

• • • • •

Las micosis invasoras causan alta morbilidad en pacientes inmunodeprimidos. La mortalidad cruda de las micosis invasoras es muy elevada. Candidiasis es la infección fúngica más común. La aspergilosis pulmonar invasora es el cuadro más grave. La sospecha clínica es fundamental en el pronóstico.

Diagnóstico • Cultivo + evidencia histopatológica de angioinvasión. • Cultivo en lugar habitualmente esteril. • Detección de antigeno Galactomanano en suero o LBA. Tratamiento • Voriconazol: tratamiento de elección (si el cuadro clínico es grave, podría asociarse voriconazol + una equinocandina) • Anfotericina B: En caso de no poder utilizar voriconazol o si existe sospecha de mucormicosis.

Caso Clínico Tipo Mujer de 65 años, con antecedentes de asma crónico y crisis frecuentes que precisan tratamiento con glucocorticoides sistémicos hace 15 días. Una semana antes de ingresar en el hospital comienza con tos, expectoración amarillenta, en ocasiones con sangre, seguido de fiebre y aparición de disnea, que no mejora a pesar del tratamiento antibiótico. En la placa de tórax, se observan múltiples nódulos pulmonares, mal definidos, algunos de ellos cavitados.

Candidiasis Invasora Micosis invasora más frecuente. Patología de alta mortalidad (35-75%) y costos en pacientes críticos. Causado por candida sp, presente habitualmente en el tracto gastrointestinal y genitourinario. Reservorio son humanos y animales. Se define como el aislamiento de alguna especie de Candida, en hemocultivos (candidemia) y/o en algún otro sitio estéril infectado por este patógeno. Factores de riesgo: • Estadía en UCI. • Uso de CVC. • Insuficiencia renal. • Gran cirugía (digestiva). • Neutropenia. • Uso de ATB de amplio espectro. • Hemodialisis. • Nutrición parenteral.

Definición Patología de alta morbimortalidad principalmente en pacientes con factores de riesgo. Debe sospecharse en pacientes con patología pulmonar crónica inexplicada, meningitis crónica, lesiones óseas líticas, lesiones cutáneas crónicas, fiebre de origen indeterminado o citopenias. En pacientes inmunodeprimidos, nuevos signos y síntomas pulmonares, cutáneos, del fondo de ojo o de cabeza y cuello, o fiebre indeterminada persistente, deben llevar a la consideración de estos patógenos.

Cuadro clínico: Alto índice de sospecha en pacientes con factores de riesgo. Cuadro inespecífico. • Fiebre, taquicardia e hipotensión. • Nuevo cuadro de sepsis grave en paciente crítico. • Manifestaciones superficiales: <10%. • Pústulas dolorosas de base eritematosa • Invasión tisular: < 3%. • Genitourinario: Diferenciar de colonización. • Endoftalmitis: Siempre solicitar fondo de ojo ante sospecha de candidiasis invasora. • Osteoarticular, meningitis crónica, endocarditis, peritonitis e infección intraabdmonal, neumonía (raro), empiema, etc.

Aspergilosis Invasora Infección grave causada por Aspergillus sp., principalmente a) fumigatus (organismo ubicuo). Vive en el medio ambiente (reservorio, tierra de plantas ornamentales, basurales, aire acondicionado). Se caracteriza por invasión vascular, trombosis e infarto de tejidos afectados a la histología. Factores de riesgo: • Neutropenia severa y prolongada. • Drogas o condiciones que llevan a inmunodeficiencia celular prolongada: altas dosis de corticoides, QMT, VIH, etc.

Otros:

29 Facultad de Medicina, Universidad de Chile

• • •

Candidiasis esofágica: Infección localizada. Se presenta habitualmente como odinofagia. Se diagnostica por endoscopía digestiva alta.

Autor / Editor Gustavo Muñoz R.

Diagnóstico • Hemocultivos: Gold standard. Baja sensibilidad. • Biopsia y cultivo en lesiones focales. • Fondo de ojo. Tratamiento • Anfotericina B. • Fluconazol: En pacientes no neutropénicos, estables, sin exposición previa a azoles, en centros de baja prevalencia de C. glabrata o C. krusei. • Equinocandinas: caspofungina, micafungina o anidulafungina. En pacientes no neutropénicos o neutropénicos clínicamente inestables, terapia reciente con azoles, en centros de alta prevalencia de C. glabrata o C. krusei. Mucormicosis Hongo filamentoso que vive en el ambiente (suelo y vegetación descompuesta). Afecta principalmente a pacientes inmunosuprimidos y diabéticos mal controlados. Se caracteriza por invasión vascular produciendo infarto y necrosis de tejidos afectados. Factores de riesgo: • Diabetes mal controlada. • Acidosis metabólica. • Uso de deferoxamina (quelante de hierro). • Inmunosupresión celular: Uso crónico de esteroides, trasplante de órganos, leucemia, linfoma, y SIDA. Cuadro clínico: • Rino-oculo-cerebral: Presentación más común. Fiebre, congestión nasal, descarga nasal purulenta, cefalea y dolor sinusal. Progresa rápidamente desde CPN a órbita y cerebro. • Pulmonar: Fiebre y hemoptisis, neumonía con infarto y necrosis, que se disemina a estructuras adyacentes. Diagnóstico • Identificación histopatológica y confirmación por cultivo. Cultivo de baja sensibilidad. • Estudio de imágenes: scaner y RNM. Tratamiento • Tratar factores predisponentes. • Debridamiento quirúrgico. • Antifúngicos: Anfotericina B es el agente de elección. Una vez que se logra pasar a terapia oral, continuar con posaconazol.

Seguimiento Por especialista.

30 Facultad de Medicina, Universidad de Chile

Año 2017

Candidiasis Oral y Esofágica

Puede presentarse secundario a una candidiasis oral, o en forma aislada, donde la sospecha clínica es fundamental. Generalmente el diagnóstico es solamente clínico, ya sea por la visualización de la mucosa oral como el hallazgo endoscópico, aunque se puede confirmar con una preparación con KOH del exudado u otras tinciones. En casos resistentes al tratamiento son útiles los cultivos para excluir la presencia de otras especies distintas a Candida albicans. El diagnóstico diferencial de la candidiasis esofágica es la infección por otros virus (ej, Citomegalovirus, herpes simplex), esofagitis inducida por drogas y otras causas de inflamación crónica como la esofagitis eosinofílica.

Nivel de manejo del médico general: Diagnóstico Específico Tratamiento Completo Seguimiento Derivar

Aspectos Esenciales • •



El principal agente causal es Candida albicans. Sospechar en grupos de riesgo: Bebés, Adultos mayores usuarios de prótesis dentales, uso de ATB o corticoides prolongado, antecedente de QT o RT de cara/cuello y otras condiciones de inmunosupresión como SIDA (sobretodo en candidiasis esofágica). El sello distintivo de la candidiasis esofágica es la odinofagia.

Tratamiento

Caso Clínico Tipo Paciente masculino de 30 años, portador de VIH etapa C3, consulta por disfagia. Al realizar endoscopía se evidencia la presencia de placas blanquecinas adherentes de aspecto algodonoso.

Definición



Candidiasis orofaríngea: Nistatina 200.000 a 400.000 U 5 veces al día por 7-14 días, Clotrimazol 10mg 5 veces/día, o fluconazol 100-200 mg vo por 7-14 días ( En general tratamiento tópico para pacientes sin serología VIH +). También se utiliza colutorios de miconazol 4 veces al día.



Candidiasis esofágica: Se recomienda fluconazol vo (dosis de carga de 400 mg seguida de 200 a 400 mg al día durante 14 a 21 días). Si no responde a tratamiento después de una semana, se recomienda el voriconazol o posaconzaol para pacientes ambulatorios.En pacientes en los que es necesaria la terapia intravenosa, se sugiere el uso de una equinocandina, en lugar de anfotericina B.

Seguimiento

Infección de la mucosa oral y esofágica por hongos del género Candida.

La aparición de candidiasis oral y/o esofágica en jóvenes aparentemente sanos es indicación de investigar VIH u otra inmunodepresión. Se debe hacer un estudio en busca de la causa predisponente.

Etiología-Epidemiología-Fisiopatología Candida es un organismo comensal en seres humanos. El patógeno habitual es la Candida albicans, pero también puede deberse a C. glabrata, C. krusei y C. tropicalis . La enfermedad se asocia a alteración de la flora normal por uso de antibióticos, casos de inmunodepresión, quimioterapia, radioterapia de cabeza y cuello, diabetes, uso de corticoides orales o inhalatorios e inhibidores de la secreción ácida gástrica. Especial importancia en pacientes VIH, siendo la candidiasis esofágica una de las patologías marcadoras de etapa SIDA. La candidiasis oral es habitual en bebés.

Autor / Editor Gustavo Muñoz R.

Diagnóstico La forma más común se caracteriza por placas blanquecinas, adherentes e indoloras en la superficie oral y/o esofágica. Sin embargo, puede haber sólo eritema palatino. Otros sintomas son sensación de sustancia algodonosa en la boca, ageusia, y en algunos casos dolor al masticar. La Candidiasis esofágica se presenta con odinofagia, disfagia y dolor retroesternal.

31 Facultad de Medicina, Universidad de Chile

Año 2017

Cólera

acuosa indolora y vómitos capaces de causar deshidratación severa. Generalmente la diarrea es indolora, sin tenesmo. Las heces son semejantes al agua de arroz. No causa fiebre (o ésta es moderada) debido a que el cuadro se produce por la enterotoxina y no por el germen. Se acompaña de sintomas de deshidratación: xerostomía, xeroftalmia, disminución del turgor de la piel, taquicardia, hipotensión, pulso periférico disminuido, compromiso de conciencia, respiración de Kussmaul.

Nivel de manejo del médico general: Diagnóstico Sospecha Tratamiento Inicial Seguimiento Derivar

Aspectos Esenciales



• • •

Caso Confirmado: Cuadro clínicamente compatible y cuyo coprocultivo (agar TCBS) sea ratificado por el Instituto de Salud Pública.



“Cólera sicca”: Forma infrecuente de cólera donde el fluido se acumula en el lumen intestinal, ocurriendo colapso circulatorio sin diarrea.



Diarrea secretora tipo agua de arroz de inicio súbito. Diagnóstico se confirma con coprocultivo El pilar de tratamiento es la hidratación y antibioterapia es secundaria Notificación obligatoria e inmediata.

Caso Clínico Tipo

Tratamiento

Paciente que viajó al sudeste asiático y consulta por diarrea abundante, acuosa, de inicio súbito y vómitos con el antecedente de haber consumido comida en puesto ambulante un día antes de viajar.

Hidratación es la primera prioridad en el tratamiento del cólera. • Fase de rehidratación: 50-100mL/Kg/h (2-4h). Se prefiere Ringer lactato. • Fase de mantención: Vía oral es de elección. Usar sales de rehidratación oral a 500-1000mL/h, hasta el cese de la diarrea. Antibioterapia (solo en caso de deshidratación severa): • Doxiciclina 7mg/kg, máximo 300 mg, por una vez. • Ciprofloxacino 30mg/kg, máximo 1gr vía oral por una vez. Antibioticoterapia no es un componente esencial en el tratamiento, lo vital es la rehidratación. Sin embargo, se ha descrito que reduce el volumen de la diarrea y la duración del cuadro en un 50%.

Definición El cólera es una infección diarreica aguda causada por los serotipos O1 y O139 de la bacteria Vibrio cholerae, que en su manifestación más grave produce un cuadro que lleva rápidamente a la deshidratación. Es de notificación inmediata.

Etiología-Epidemiología-Fisiopatología Vibrio cholerae es un bacilo Gram negativo aeróbico o anaeróbico facultativo, que a través de la toxina colérica estimula el canal de cloro en las criptas intestinales, lo que incrementa la secreción de agua y electrólitos e inhibe el cotransporte de sodio y cloro en las células de las vellosidades intestinales originando una diarrea secretora. Los reservorios del cólera son los mariscos y el plancton. Se transmite por ingestión de agua o alimentos contaminados con heces y vómitos de pacientes, por la ingestión de alimentos no refrigerados contaminados y por la ingestión de mariscos crudos o mal cocidos provenientes de aguas contaminadas. Raramente el cólera es transmitido por contacto persona a persona.El período de incubación es de 2 a 3 días, con un rango de 0 a 5 días como máximo. Chile permanece libre de cólera desde 1998.

Seguimiento Vigilar contactos por 5 días. Quimioprofilaxis no ha demostrado ser útil.

Autor / Editor Gustavo Muñoz R.

Diagnóstico •

Caso Sospechoso: Toda persona que presente inicio brusco (incubación 24-48hrs) de un cuadro de diarrea

32 Facultad de Medicina, Universidad de Chile

Año 2017

BIBLIOGRAFÍA • • • •

• • • • • • • • • • • • • • •

https://www.uptodate.com/contents/cellulitis-and-skin-abscess-treatment?source=search_result&search=celulitis&selectedTitle=1~150 Infecciones de partes blandas, Peña A. Publicaciones medicina UC : http://publicacionesmedicina.uc.cl/pediatriaHosp/InfeccionesPiel.html Infecciones de transmisión sexual. OMS. 2016: http://www.who.int/mediacentre/factsheets/fs110/es/ Norma de Profilaxis, Diagnóstico y Tratamiento de las Infecciones de Transmisión Sexual (ITS) 2016. MINSAL: http://diprece.minsal.cl/wrdprss_minsal/wp-content/uploads/2014/11/ NORMA-GRAL.-TECNICA-N%C2%B0-187-DE-PROFILAXIS-DIAGNOSTICO-Y-TRATAMIENTO-DE-LAS-ITS.pdf https://www.uptodate.com/contents/treatment-of-varicella-chickenpox-infection?source=search_result&search=tratamiento%20varicela&selectedTitle=1~150#H167447725 https://www.uptodate.com/contents/treatment-of-herpes-zoster-in-the-immunocompetent-host?source=search_result&search=tratameinto%20varicela%20 zoster&selectedTitle=1~150 https://www.uptodate.com/contents/clinical-features-of-varicella-zoster-virus-infection-chickenpox?source=search_result&search=varicela%20y%20 herpes%20zoster&selectedTitle=2~150 Guía clínica AUGE: Síndrome de inmunodeficiencia adquirida VIH/SIDA. MINSAL 2013 VIH/SIDA. OMS. 2016 http://www.who.int/mediacentre/factsheets/fs360/es/ VIH: INFECCION AGUDA, PESQUISA Y MANEJO - Dr. Esteban Cortés S. REV. MED. CLIN. CONDES - 2014; 25(3) 419-424 https://www.uptodate.com/contents/diagnostic-approach-to-the-adult-presenting-with-neutropenic-fever?source=search_result&search=neutropenia%20 febril&selectedTitle=3~150 https://www.uptodate.com/contents/treatment-and-prevention-of-neutropenic-fever-syndromes-in-adult-cancer-patients-at-low-risk-for-complications?source=see_link https://www.uptodate.com/contents/treatment-of-neutropenic-fever-syndromes-in-adults-with-hematologic-malignancies-and-hematopoietic-cell-transplant-recipients-high-risk-patients?source=see_link INFECCIONES POR PARÁSITOS MÁS FRECUENTES Y SU MANEJO - Dr. Werner Apt B. REV. MED. CLIN. CONDES - 2014; 25(3) 485-528 file:///C:/Users/GREEN%20 X%20RESORT/Desktop/12-Dr.Apt.pdf https://www.uptodate.com/contents/epidemiology-and-clinical-manifestations-of-invasive-aspergillosis?source=search_result&search=micosis%20 invasoras&selectedTitle=1~150 https://www.uptodate.com/contents/treatment-and-prevention-of-invasive-aspergillosis?source=search_result&search=micosis%20invasoras&selectedTitle=3~150 https://www.uptodate.com/contents/clinical-manifestations-of-oropharyngeal-and-esophageal-candidiasis?source=search_result&search=candidiasis%20oral%20y%20esofagica&selectedTitle=2~150 https://www.uptodate.com/contents/treatment-of-oropharyngeal-and-esophageal-candidiasis?source=search_result&search=candidiasis%20oral%20y%20 esofagica&selectedTitle=1~150 Cólera. 2015. http://web.minsal.cl/colera/

33 Facultad de Medicina, Universidad de Chile

pacientes VIH (+) con linfocitos CD4 menor de 200, el diagnóstico más probable a considerar es la toxoplasmosis cerebral, con patrones radiológicos sugerentes en RNM y TAC con contraste, presentándose la mayoría de las veces como lesiones múltiples.

Absceso cerebral Nivel de manejo del médico general: Diagnóstico Sospecha Tratamiento Inicial Seguimiento Derivar

En diagnóstico diferencial de masas cerebrales, considerar neurocisticercosis y neoplasias, en general el informe de un radiólogo entrenado aproximará mejor el diagnostico, sobre todo en aquellos casos en que la clínica sea inespecífica.

Aspectos esenciales 



Tratamiento

La infección más frecuentemente es por propagación directa desde un foco contiguo. Otras entradas son el traumatismo o neurocirugía y la diseminación hematógena. El diagnóstico se realiza con neuroimágenes.

Combinación de antibióticos parenterales según etiología, que cubra posibles agentes etiológicos y drenaje neuroquirúrgico. Se puede usar además tratamiento anticonvulsivo profiláctico si es que tuvo una crisis anteriormente.

Caso clínico tipo Seguimiento

Paciente femenino de 7 años con antecedentes de otitis media crónica, presenta desde hace 2 semanas cefalea creciente. Consulta porque en las últimas horas presentó una crisis convulsiva. Al examen físico presenta 37°C.

Se debe hospitalizar y referir a neurocirugía para evaluar e iniciar el tratamiento antibiótico y quirúrgico.

Definición Es una colección supurada y focal dentro del parénquima encefálico rodeado típicamente de una cápsula vascularizada.

Etiología-epidemiología-fisiopatología Se forma por propagación directa (más frecuente) desde un foco de infección craneal contiguo (sinusitis paranasal, otitis media, mastoiditis o infección dental), por traumatismo, intervención neuroquirúrgica, o también puede ser por vía hematógena (esta forma de diseminación usualmente genera múltiples abscesos). Los microorganismos causales dependen del foco de la infección y de la etiología más frecuente de este. Su formación completa demora aproximadamente 14 días, pasando por una cerebritis alrededor del foco necrótico y posteriormente por la formación de la cápsula con gliosis pericapsular. Es una patología infrecuente y que gracias a los nuevos avances no tiene gran mortalidad, sin embargo, puede dejar secuelas neurológicas.

Diagnóstico Se ha descrito la triada clásica de fiebre, cefalea y signos focales, pero esto se ve en menos de la mitad de los pacientes. El cuadro inicial es de una lesión intracraneal en expansión y no el de un evento infeccioso. Se puede observar clínica de síndrome de hipertensión endocraneana y menos del 25% de los pacientes presentan crisis convulsivas focales o generalizadas. El diagnóstico se realiza con neuroimágenes; la resonancia nuclear magnética es de elección, sin embargo, la TAC con contraste tiene muy buena sensibilidad (característicamente captan contraste en anillo). La punción lumbar no juega un rol importante, y está contraindicada si el paciente presenta signos focales o edema de papila, por riesgo de enclavamiento. En 1

Bacteriana: más grande, muy sensible y con signos inflamatorios. Sospechar etiología específica según clínica: por ejemplo arañazo de gato.

Adenitis y adenoflegmón

Adenoflegmón: Diagnóstico Clínico: Es de aparición muy aguda, un ganglio cervical se hace muy doloroso y comienza aumentar de tamaño considerablemente en poco tiempo, aparecen manifestaciones generales como fiebre. Al examen físico se palpa una tumoración dolorosa de bordes mal definidos, evolutivamente y en su parte más prominente aparece una zona de fluctuación.

Nivel de manejo del médico general: Diagnóstico Específico Tratamiento Completo Seguimiento Completo.

Aspectos esenciales Adenitis:  Descartar malignidad.  Sospechar etiología especifica según clínica.  Origen según sitio drenaje.  Bacteriana: beta-lactámicos. Adenoflegmón:  Bacteriana.  Calor húmedo + antibiótico (penicilina)

Tratamiento

Caso clínico tipo

- Adenoflegmón:

- Adenitis: Tratamiento Médico: Antibióticos: Cubrir S. aureus y S. pyogenes (considerar anaerobios si origen dental). Se emplea Cloxacilina 50 - 75 mg/kg/día en 3 o 4 dosis o Amoxicilina-clavulánico 40 mg/kg/día en 3 dosis por 10 a 14 días . El resto según etiología específica.

Medidas locales: calor húmedo local.

Niño 10 años, sin antecedentes mórbidos, consulta por aumento de volumen en región cervical alta. Al examen adenopatía submandibular derecha 1,5cm eritematosa, sensible a la palpación. Sin organomegalias u otras adenopatías.

Tratamiento médico: Antibióticos:  

Definición



Adenitis: Aumento significativo del tamaño (> 1 cm) de un linfonodo, asociado a signos inflamatorios locales.

Compromiso leve: Amoxicilina-clavulánico 40 mg/kg/día en 3 dosis por 10 a 14 días. Compromiso severo: Penicilina 4 millones EV c/6h + gentamicina 240mg EV c/24h Alérgicos a la penicilina: Clindamicina 600mg vía endovenosa c/8h

Tratamiento quirúrgico: drenaje en caso de que sea posible.

Adenoflegmón: Compromiso infeccioso inflamatorio difuso del tejido ganglionar tributario de procesos infecciosos de diverso origen; corresponde a la abscedación de una adenitis aguda.

Seguimiento Adenitis: Se espera regresión del aumento de volumen y de la sintomatología inflamatoria en 48 a 72 hrs.

Etiología-epidemiología-fisiopatología

Si no hay respuesta a las dos semanas: Hemograma VHS, Radiografía de tórax, Eco partes blandas, Derivado Proteico Purificado (PPD/test de Mantoux), serología de acuerdo a sospecha clínica (Virus EbsteinBarr, Citomegalovirus, VIH, Bartonella, Toxoplasmosis, Brucelosis e Histoplasmosis) y punción aspirativa.

Adenitis: Múltiples etiologías virales, bacterianas, parasitarias, tumorales, etc. Adenopatías son frecuentes en atención primaria, > 60 % son de causa inespecífica, 1% de causa maligna. La incidencia real de la adenitis cervical no se conoce pues la mayoría de los casos aparecen en contexto de infecciones respiratorias altas y son autolimitados.

Sospecha etiología maligna: Tamaño mayor de 3 cm, fijación a planos profundos, localización supraclavicular o Radriografía de tórax anormal, se debe derivar de inmediato a un centro especializado para estudio histológico y tratamiento oportuno.

Adenoflegmón: Más prevalente en la población infantil. Es siempre bacteriana, producido por el paso de bacterias virulentas al interior del ganglio, al cual llegan por la red de vasos linfáticos aferentes, procedentes de un foco séptico agudo regional.

Biopsia ganglionar en: Diagnóstico

Crecimiento ganglionar después de 3 semanas de estudio sin diagnóstico etiológico. Falta de disminución de tamaño después de seguimiento de 4 a 6 semanas. Falta de regresión a tamaño considerado normal para la edad del paciente y la localización, en 10 a 12 semanas. Radiografía de tórax anormal (presencia de adenopatías mediastínicas). Localización cervical baja o supraclavicular.

Adenitis: Enfrentamiento de adenopatía debe ir orientado a descartar malignidad: Anamnesis: edad, tiempo de evolución, síntomas asociados. Examen físico: local o generalizada, ubicación, características clínicas (tamaño, consistencia, sensibilidad, movilidad), hepatoesplenomegalia. Según su etiología: Virales: habitualmente blanda, pequeña, en más de un grupo ganglionar y sin cambio de coloración en la piel. 2

la úlcera, pues su consistencia será blanda y elástica, a diferencia de las ulceraciones de origen maligno) y descartar complicaciones como la tromboflebitis séptica de la vena yugular (Sd. de Lemierre). Pedir hemograma, en el caso de AUN cuando es bilateral para descartar agranulocitosis o leucemias, y en el caso de la ASM, para la evidencia de mononucleosis. Para confirmar el diagnóstico en ambos casos, debemos realizar un examen directo del frotis faríngeo. Chile no tiene casos desde 1996.

Angina úlcero-necrótica, úlceromembranosa y pseudomembranosa Nivel de manejo del médico general: Diagnóstico Sospecha. Tratamiento Inicial. Seguimiento Derivar

Tratamiento 

Aspectos esenciales  

La etiología clásica AUN es la Angina de Vincent (F. necrophorum y T. vincetii). Su tratamiento es con Penicilina G. Siempre en angina pseudomembranosa descartar difteria, debido a la gravedad del cuadro.



Caso clínico tipo Paciente masculino de 20 años, consulta por odinofagia, halitosis y CEG. Al examen físico presenta T° de 37.8°C, placas blanquecinas adherentes amigdalianas y una adenopatía cervical derecha en grupo II.

Angina de Vincent : penicilina G o V, 4 millones de U c/ 4 -6 hrs, que se puede dar sola o en asociación con metronidazol. Terapia alternativa:Clindamicina 600 mg c/ 8 hrs. Angina Pseudomembranosa por Mononucleosis: ambulat orio con antipiréticos. Ante complicaciones hospitalizar. En sospecha de Difteria (signos extraamigdalianos como voz áfona, tos ronca, rinolalia o trastornos del ritmo cardíaco) o confirmación diagnóstica, requiere hospitalización urgente en cuidados intensivos para terapia específica y aislamiento.

Seguimiento AUN es ambulatorio, al igual que la ASM por mononucleosis no complicada. La ASM por difteria su manejo es hospitalizado, con notificación obligatoria.

Definición Pertenecen al grupo de las faringoamigdalitis agudas. En el caso de la Angina Úlcero-Necrótica (AUN) presenta un exudado blanquecino-grisáceo, friable y con ulceraciones uni o bilaterales de amígdalas y faringe. Suele estar asociada a una gingivoestomatitis ulceromembranosa. En la Angina pseudomembranosa (ASM) las amígdalas están recubiertas por un exudado blanquecino espeso de coloración grisácea homogéneo, pero muy adherente que puede cubrir las estructuras circundantes. Se puede acompañar de rinorrea mucopurulenta unilateral.

Etiología-epidemiología-fisiopatología La etiología clásica de la AUN corresponde a la Angina de Vincent, infección unilateral, con agentes etiológicos como Fusobacterium necrophorum (Sd. Lemierre: flegmón periamigdaliano en adultos jóvenes) y Treponema vincetti, ambos saprófitos del sarro dentario. Factores predisponentes son mala higiene bucal y pacientes sometidos a estrés, desnutrición o inmunodepresión. La etiología de la Angina Pseudomembranosa principalmente es difteria (Corynebacterium Difteriae) y otras como neumocócica, por chlamydia o mononucleosis. Muy inhabitual pero grave (5%-10% mortalidad).

Diagnóstico El motivo de consulta es odinofagia intensa, febrícula y una marcada astenia. A la exploración física aparecen una o más adenopatías cervicales satélite. En el caso de la ASM descartar difteria y en el caso de las AUN hacer diagnóstico diferencial con cáncer amigdaliano (es importante la palpación de la amígdala en donde asienta 3

tasa de letalidad es de 5% a 20% de los infectados.

Ántrax o carbunco 

Ántrax pulmonar: Los síntomas asemejan un cuadro tipo influenza. Periodo de incubación de 2 -43 días. En 2 a 4 días evolución a síndrome de distress respiratorio agudo. Con hemorragia pulmonar, mediatinitis y linfadenitis edematonecróticas. Letalidad de un 100% para los casos infectados.



Ántrax intestinal: Síntomas inespecíficos. Periodo de incubación de 1 a 6 días. Malestar abdominal y fiebre. Letalidad de un 25-60%

Nivel de manejo del médico general: Diagnóstico: Sospecha. Tratamiento: Inicial. Seguimiento: Derivar.

Aspectos esenciales   

Antecedente de contacto con animales herbívoros Lesión cutánea inicialmente papular que al cabo de una semana se escara Distress respiratorio con fiebre, mialgias, tos, expectoración hemoptoica y ensanchamiento mediastínico

El diagnóstico de ántrax se realiza con un cuadro clínico compatible y cultivo (sangre, lesiones o secreciones) que demuestre B. anthracis en el paciente, o 2 métodos distintos del cultivo (PCR, inmunohistoquímica o detección por ELISA de IgG anti antígeno protector)

Caso clínico tipo

Tratamiento

Hombre de 64 años, proveniente de Haití hace 10 días, sin antecedentes mórbidos, presenta hace 8 días decaimiento, fatiga fácil, sensación febril intermitente y cefalea con aumento de volumen de la cara, en especial en la región peri orbitaria derecha, con secreción amarillenta que se intensificó hace dos días que le imposibilita abrir los parpados y dificultad con la visión del ojo derecho.

En compromiso localizado usar ciprofloxacino vo o doxiciclina vo. Frente a la presencia de compromiso sistémico uso de ciprofloxacino ev o doxiciclina ev y si existe susceptibilidad antimicrobiana usar penicilina G 812 millones de unidades.

Seguimiento Por especialista. Recordar que es ENO (enfermedad de notificación obligatoria).

Definición Infección aguda producida por Bacillus Anthracis. Existen tres formas de presentación de ántrax: cutáneo, pulmonar e intestinal.

Etiología-epidemiología-fisiopatología   



Es una enfermedad infrecuente en Chile Bacillus anthracis, gram positivo, encapsulado, formador de esporas. Reservorio: herbívoros que expulsan los bacilos en hemorragias. Al exponerse al aire, las formas vegetativas esporulan permaneciendo viables en suelos contaminados por muchos años, ya que estas esporas resisten la desinfección y situaciones ambientales adversas. Transmisión: contacto con tejidos de animales que han muerto por la enfermedad (cuero, lana, pelo) exposición laboral de ganaderos. No se transmite de persona a persona.

Diagnóstico 

Ántrax cutáneo: Forma más frecuente, 95% de los casos. El periodo de incubación es de 1 a 12 días. Prurito en la piel expuesta, seguido por una lesión papular que se vuelve vesicular y en dos a seis días termina evolucionando a una escara necrótica rodeada de edema con un halo violáceo. Los sitios más frecuentemente comprometidos son la cabeza, antebrazos y manos. En algunos casos se encuentran linfoadenopatías y fiebre. El curso natural de la enfermedad es a la remisión con cicatrización de la lesión. Sin tratamiento, la 4

presentarse como una enfermedad febril similar a la fiebre tifoidea pero menos grave. Los hallazgos más frecuentes son el dolor musculo esquelético y la afección del esqueleto axial y periférico (osteomielitis vertebral o artritis séptica típicamente de cadera o rodilla). Al examen físico los signos más habituales son la presencia de adenopatías, hepatoesplenomegalia y absceso focal.

Brucelosis Nivel de manejo del médico general: Diagnóstico: Sospecha. Tratamiento: Inicial. Seguimiento: Derivar

El diagnostico de laboratorio se basa cultivos que son exitosos en 50-70% de los casos pero tardan hasta 6 semanas (3 semanas en sistemas especiales de cultivos BACTEC). En etapas tempranas de la infección puede ser útil la serología (IgM) por técnicas de aglutinación estándar (prueba rosa de bengala). Títulos elevados de IgM indican exposición reciente, títulos elevados de IgG sugieren infección activa, mientras que títulos bajos de IgG se pueden ver en sujetos con exposición previa o infección tratada.

Aspectos esenciales     

Infeccion zoonótica más común del mundo. Antecedente de consumo de leche no pasteurizada Fiebre ondulante y CEG Sintomatología polimorfa, puede comenzar con síntomas seudogripales Caso probable: Prueba de Rosa de Bengala positiva (VPP 99%)

Es importante el diagnóstico diferencial con tuberculosis para evitar monoterapias inadvertidas en tuberculosis. La brucelosis tiende a producir menos destrucción ósea y articular

Caso clínico tipo Tratamiento

Paciente de 40 años, carnicero, consulta por cuadro de 5 dias de evolución caracterizado por fiebre muy alta de predominio nocturno que se acompaña de intensa cefalea y diaforesis. Refiere además artralgias y mialgias y manifestaciones clínicas concordantes con una neumonía.

El objetivo del tratamiento es el control de síntomas tan rápido como sea posible, para prevenir complicaciones y recaídas. El cuidado inicial es de soporte. Tratamiento antibiótico debe ser asociado, por la alta frecuencia de recaídas con el uso de monoterapia.

Definición

Para brucelosis aguda en adultos y niños mayores de 8 años, la OMS recomienda el siguente esquema:

Es una zoonosis producida por cocobacilos gramnegativos aeróbicos, del género Brucella. Tambien conocida como fiebre mediterránea, fiebre de Malta.

 

Etiología-epidemiología-fisiopatología

Doxiciclina 100mg VO cada 12h + rifampicina 600900mg VO cada 24h por 6 semanas. Doxiciclina 100mg VO cada 12h por 6 semanas + estreptomicina 1g cada 24h IM por 2-3 semanas.

Para brucelosis aguda en niños menores de 8 años: Brucella se comporta como parásito intracelular facultativo. Es una enfermedad sistémica, que puede comprometer a casi cada órgano. La bacteria puede infectar al ganado vacuno, las cabras, los camellos, los perros y los cerdos. Las vías de transmisión se asocian, en el medio urbano, a la ingesta de productos lácteos contaminados (leche o queso sin pasteurizar) y carne cruda, y en el medio rural, al contacto con animales enfermos (secreciones, inhalación). El germen penetra en el organismo a través de la superficie mucosa, luego es transportado a los ganglios linfáticos regionales, donde se produce una réplica inicial de las Brucellas. Posteriormente por diseminación hematógena se origina una infección localizada crónica en cualquier sitio del cuerpo (como osteoarticular, genitourinaria, hepática, etc). Tiende a afectar más frecuentemente al varón adulto.



Rifampicina + cotrimoxazol forte por 6 semanas es la terapia de elección

El uso de corticoides se reserva para los casos de brucelosis con meningitis sintomática.

Seguimiento Recordar que es ENO. Hay recidiva en el 30% de los casos en general por mal cumplimiento terapéutico. Deben vigilarse los pacientes por al menos 2 años.

Diagnóstico Incubación: usualmente de 2 a 4 semanas. La clínica se caracteriza por fiebre prolongada y ondulante, asociada a sudoración nocturna, compromiso del estado general, apatía y anorexia crecientes, asociado a síntomas inespecíficos como cefalea, mialgias, lumbalgia, constipación, disfagia y tos seca. Puede 5

Dengue

manguito del esfingomanómetro) y leucopenia. Signos de alarma: dolor abdominal, vómitos persistentes, rápido cambio de fiebre a hipotermia, letargo, hepatomegalia, frialdad de extremidades.

Nivel de manejo del médico general: Diagnóstico: Sospecha. Tratamiento: Inicial. Seguimiento: Derivar

- Caso sospechoso de dengue hemorrágico: Todo caso sospechoso o confirmado de Fiebre del Dengue (dengue clásico) con una o más de las siguientes manifestaciones: petequias, equimosis o púrpura, hemorragia de las mucosas, sitios de inyección u otros sitios, hematemesis o melena, trombocitopenia (100.000 células o menos por mm3), indicios de pérdida de plasma debida al aumento de la permeabilidad vascular, con una o más de las siguientes manifestaciones:

Aspectos esenciales  

 

Infección vírica transmitida por mosquito de hábitos nocturnos La infección causas síntomas gripales y en ocasiones evoluciona hasta convertirse en un cuadro potencialmente mortal llamado dengue grave Confirmación con ELISA (IgM) No existe tratamiento específico, el manejo es sintomático

  

Aumento del hematocrito en un 20% o más del valor normal. Disminución del 20% o más del hematocrito después del tratamiento de reposición de pérdidas en comparación con el nivel de base Derrame pleural, hipoproteinemia y ascitis.

- Caso confirmado: ELISA IgM confirmado por ISP.

Caso clínico tipo

Tratamiento

Mujer de 28 años consulta por cefalea retroocular de 48 horas de evolución, asociado a febrículas, CEG, mialgias. Previo a consultar nota aparición de exantema maculopapular pruriginoso. Refiere haber realizado viaje a Brasil hace 2 semanas.

El dengue sólo tiene tratamiento sintomático, con analgesia y reposición de volumen. Está contraindicado el ácido acetilsalicílico y los AINES por el riesgo de hemorragias. Pacientes con sangrado significativo deben manejarse con transfusiones. Durante el período febril, se debe mantener a los pacientes aislados mediante mosquiteros, para evitar la transmisión a los vectores (mosquitos).

Definición Enfermedad viral producido por flavivirus que incluye 4 serotipos (DEN1, DEN2, DEN3 y DEN4) transmitida por el vector Aedes aegypti, de hábitos nocturnos, y cuyo único reservorio es el hombre. La infección por uno de estos serotipos crea inmunidad de por vida solamente contra ese serotipo. El dengue se presenta en los climas tropicales y subtropicales de todo el planeta, sobre todo en las zonas urbanas y semiurbanas. La única forma de prevenir la enfermedad es mediante el control del mosquito y sus criaderos y la detección rápida y temprana de los casos.

Entre finales de 2015 y principios de 2016 se aprobó en varios países el uso de la primera vacuna contra el dengue —Dengvaxia (CYD-TDV), de Sanofi Pasteur— en personas de 9 a 45 años residentes en zonas endémicas. La OMS recomienda que los países consideren la posibilidad de introducir la vacuna CYD-TDV contra el dengue solo en entornos geográficos (nacionales o subnacionales) en los que los datos epidemiológicos indiquen que hay una gran carga de enfermedad.

Seguimiento Etiología-epidemiología-fisiopatología

Enfermedad de notificación obligatoria.

El mosquito Aedes aegypti, se ha adaptado al hábitat humano estableciéndose cerca de viviendas donde existen condiciones deficientes de saneamiento y elementos que permiten contener agua (como maceteros, neumáticos en desuso, etc), donde construyen sus criaderos. El 2011 se confirmó sólo un caso autóctono en Isla de Pascua. Casos en Chile continental son importados.

Diagnóstico - Caso sospechoso de fiebre del dengue: Paciente con enfermedad febril aguda (hasta 40°C) con duración máxima de 7 días, que resida o haya estado los últimos 15 días en zona con circulación de virus del dengue, con dos o más de las siguientes manifestaciones: cefalea, dolor retro orbitario, mialgias, artralgias, erupción cutánea, manifestaciones hemorrágicas (epistaxis, gingivorragia, prueba del torniquete positiva: equimosis al inflar el 6

megacolon tóxico. El diagnóstico se hace con una combinación de criterios clínicos: Cuadro clínico (≥3 deposiciones líquidas en ≤24 h), puede haber leucocitosis, Toxina C.difficile A o B (+) en deposiciones por ELISA (alto rendimiento). PCR ultra rápida en deposiciones tiene mayor sensibilidad y especificidad. No sirve el coprocultivo (bajo rendimiento, alta tasa de portación asintomática, no diferencia de cepas toxigénicas (patógenas) de no patógenas. Colonoscopia compatible con colitis pseudomembranosa. (Evaluar antes de realizar colonoscopía riesgo de perforación).

Diarrea asociada a antibióticos Nivel de manejo del médico general: Diagnóstico: Específico. Tratamiento: Completo. Seguimiento: Completo

Aspectos esenciales     

Tratamiento

Principal factor de riesgo es el uso previo antibiótico. Clostridium difficile principal agente. Hasta 12 semanas post uso ATB. Cualquier antibiótico produce el cuadro. Tratamiento con Metronidazol o Vancomicina vo.

Hidratación, corrección electrolítica, aislamiento de contacto. Discontinuar ATB precipitante (si clínica lo permite). Tratamiento específico: - Infección leve a moderada (o primera recurrencia): Metronidazol 500 mg c/8 hrs vo x 10 días - Infección severa: Vancomicina 125-500 mg 4 veces al día VO por 14 días o Fidaxomicina 200 mg 2 veces al día VO por 10 días

Caso clínico tipo Paciente de 60 años, que ingresa en la UCI por una neumonía severa, siendo tratado con cefalosporinas. Desarrolla a los pocos días una diarrea muy copiosa, con gran deterioro del estado general.

- Infección severa y complicada ( o segunda recurrencia): Vancomicina 125-500 mg 4 veces al día VO o por sonda nasoenteral o 500 mg en 500 ml de suero fisiológico 4 veces al día vía rectal como enema de retención en caso de íleo o distensión abdominal significativa junto con Metronidazol 500 mg 3 veces al día EV por 14 días

Definición Diarrea post administración de antibióticos, sin otra causa aparente que comienza hasta 6-12 semanas post administración. Lo más frecuente y epidemiológicamente importante es la diarrea por Clostridium difficile.

- En 3 o más recurrencias, considerar trasplante de microbiota fecal

Etiología-epidemiología-fisiopatología Seguimiento Los antibióticos pueden dar lugar a una diarrea por dos mecanismos: el primero es su propio efecto tóxico directo en el intestino, y el segundo, una alteración de la flora intestinal, reduciéndose ciertas poblaciones de bacterias intestinales con funciones particulares sobre el metabolismo de los azúcares o permitiendo que puedan multiplicarse bacterias resistentes que producen la diarrea (como el Clostridium difficile).

Si diarrea persiste y resultado de toxina es negativo, repetir. Recurrencia 5-40%.

Clostridium difficile es un bacilo grampositivo, anaerobio estricto, capaz de generar esporas, y el principal agente responsable de las diarreas infecciosas asociada a la atención de salud en adultos y al uso de antimicrobianos (20 a 30% de los casos). La incidencia de infecciones asociadas a C. difficile, en el concierto mundial, incluido Chile, varía entre 0,6 y 2,1% del total de pacientes hospitalizados. Diversos factores de virulencia se asocian al desarrollo de la enfermedad. Los más conocidos son las toxinas A y B, que provocan inflamación intestinal. Otros factores de riesgo, además del uso previo de cualquier antibiótico son: edad avanzada, sexo femenino, uso de inhibidores de la bomba de protones, la existencia de enfermedades crónicas y hospitalizaciones prolongadas.

Diagnóstico El cuadro que produce C. difficile es muy variado, desde diarrea acuosa hasta colitis pseudomembranosa y/o 7

Encefalitis aguda

Diagnóstico Cuadro clínico: fiebre, cefalea, desorientación, alteración del habla o conducta, alteración de la conciencia (cualicuantitativo), signos meníngeos (poco relevante), convulsiones focales, focalización neurológica (paresia), leucocitosis, pleocitosis, enlentecimiento difuso, alteraciones focales en el EEG, anormalidades focales en RNM.

Nivel de manejo del médico general: Diagnóstico: Específico. Tratamiento: Inicial. Seguimiento: Derivar.

Aspectos esenciales    

El diagnóstico se basa en el cuadro clínico, sumado al estudio microbiológico del LCR, más imágenes y estudio extra-meníngeo. Se puede encontrar pleocitosis variable con predominio linfocítico en LCR. El examen central para el diagnóstico etiológico es la PCR en LCR. La RNM y EEG permiten identificar lesiones o alteraciones en el lóbulo temporal sugerente de encefalitis herpética (no patognómonico). Recordar: estudio citoquímico es inespecífico, EEG no reconoce etiología.

Sospechar en pacientes con fiebre, cefalea y alteraciones de conciencia. El examen central es la PCR en LCR. Entre RNM, TAC y EEG, la RNM es el examen más sensible y específico. Ante la sospecha de encefalitis herpética tratar en forma precoz con aciclovir.

Diagnósticos diferenciales:

Caso clínico tipo Tratamiento

Paciente de 35 años, es traído por la familia al servicio de urgencia con cefalea de inicio súbito, sensación febril no cuantificada, compromiso de conciencia progresivo, convulsiones y afasia.

En caso de encefalitis herpética el tratamiento debe ser precoz ante la sospecha: aciclovir 10-15 mg/kg c/8 hrs IV, Controlar función renal y hepática, duración: 14 a 21 días, mejor pronóstico en estados iniciales.

Definición

Muchas causas de encefalitis, especialmente aquellas ligadas a picaduras de mosquitos en zonas tropicales, no tienen tratamiento específico,excepto encefalitis herpética, el manejo es de soporte y sintomático: antibióticos y tratamiento de infecciones bacterianas secundarias, anticonvulsivantes, adecuado manejo de líquidos y electrolitos. Para convulsiones: lorazepam, fenitoína, carbamazepina. Si hay edema cerebral importante añadir dexametasona comenzando con un bolo inicial de 8-12 mg IV y posteriormente 4-6 mg c/6-8 hrs.

Inflamación del parénquima cerebral que se presenta con disfunción neuropsicológica difusa o focal.

Etiología-epidemiología-fisiopatología Causas Frecuentes Herpes simplex tipo 1, Varicela zoster, agentes transmitidos por artrópodos (arbovirus, B. burdorgferi), Virus parotiditis, M. pneumoniae, C. Pneumoniae.

Seguimiento

Causas Raras

Derivar a especialista para tratamiento urgente.

Herpes simplex tipo 2, enterovirus, influenza, dengue, CMV, VEB, VIH, virus coriomeningitis, herpesvirus simio B, rabia, sarampión, rubeola. Frecuentemente son virales, y en Chile son poco frecuentes los transmitidos por artrópodos. En la mayoría de las encefalitis no se conoce el agente patógeno y en las que se llega a identificar un 80% es producido por enterovirus (estacional verano-otoño) y en menor proporción se encuentran virus herpes (esporádico; el de mayor gravedad) y virus paratiroideo. La transmisión se hace persona a persona o por vectores (mosquitos y artrópodos). Los virus entran por las mucosas del tracto gastrointestinal o respiratorio, tiene un periodo de incubación de 4 a 6 días. Se multiplican usualmente en el sitio de infección (primo infección) y se diseminan por vía hematógena, linfática o nerviosa a múltiples órganos. La diseminación vía neuronal inicia por la mucosa nasal, donde se ubica el nervio olfatorio que es una prolongación directa del encéfalo. Mayor riesgo de morbi-mortalidad en <1 año y > 55 años.

8

Diagnósticos diferenciales:

9

Diagnóstico

Fiebre tifoídea y paratifoídea

Importante recopilar antecedentes de viajes a regiones endémicas como el sudeste asiático, África y en menor medida Sudamérica, vacunas para fiebre tifoidea y paratifoidea, uso reciente de antibióticos y de ingesta de alimentos fuera de casa.

Nivel de manejo del médico general: Diagnóstico: Específico. Tratamiento: Completo. Seguimiento: Completo

El cuadro clínico se caracteriza por un síndrome febril prolongado acompañado de cefalea, mialgias, fatigabilidad, anorexia, nauseas, alteraciones del tránsito intestinal (constipación en adultos, diarrea en niños), dolor abdominal, meteorismo y tos seca. Al examen físico es posible encontrar hepatoesplenomegalia, lengua saburral, bradicardia relativa con posterior taquicardia y roseólas tíficas en la 2ª semana (lesiones eritematosas, maculopapulares en tronco y extremidades), a veces roncus y crepitaciones aislado. Al hemograma leucopenia con desviación a izquierda, linfopenia y luego linfocitosis relativa, aneosinofilia, VHS alta, y en casos graves pancitopenia.

Aspectos esenciales     

Bacteremias cíclicas. Transmisión fecal-oral. La alteración gastrointestinal en adultos es la constipación. Examen de mayor sensibilidad hemocultivos 1°-2° semana. Son enfermedades de notificación obligatoria.

Todos los casos sospechosos deben ser confirmados con hemocultivos. Estos son positivosen un 80%, la mayor sensibilidad es entre la 1° y 2° semana. Se deben remitir las cepas al ISP para estudio de sensibilidad y serotipificación. También puede aparecer en coprocultivos (3° semana y portadores crónicos) y mielocultivos. Test de Widal (serológico) no se recomienda por alta tasa de falsos positivos y negativos.

Caso clínico tipo Paciente 29 años regresó esta semana desde un viaje a Bolivia con historia de fatiga, anorexia, cefalea occipital, fiebre. Al examen físico febril, dolor abdominal difuso, hepatomegalia, eritema maculopapular en tronco.

Definición La fiebre tifoidea y paratifoidea (también conocida como fiebre entérica) son enfermedades sistémicas graves causadas por Salmonella typhi y Salmonella paratyphi , respectivamente, y se caracterizan por fiebre sostenida y síntomas abdominales.

Tratamiento El tratamiento antibiótico permite una resolución rápida del cuadro clínico, previene las posibles complicaciones y disminuye las recaídas y la portación crónica. Pero esto se ha complicado por el desarrollo y la rápida difusión de los organismos resistentes a la ampicilina, trimetoprim-sulfametoxazol y cloranfenicol. El esquema actual de primera línea es Ciprofloxacino 500 mg vo cada 12 hrs por 7-10 días o 400 mg ev c/12 hr x 710 días. Alternativa: Ceftriaxona 2gr c/12-24 hrs x 10-14 días (para pacientes con enfermedad más grave).

Etiología-epidemiología-fisiopatología Los agentes implicados son la Salmonella typhi (90%) y la Salmonella paratyphi (10%) A, B y C. Su distribución es mundial, habiendo disminuido su incidencia en los países desarrollados por la mejora de las condiciones higiénicosanitarias y campaña contra el cólera. El reservorio exclusivo es el hombre enfermo o portador, y en la paratifoidea por lo general el hombre, y, en raras ocasiones animales domésticos. La vía de transmisión es por consumo de agua o alimentos contaminados con orina o heces de portadores o enfermos.

Seguimiento El éxito del tratamiento en los casos no complicados por lo general se traduce en una mejoría clínica dentro de tres a cinco días. Los pacientes deben ser monitorizados o instruidos para reportar los síntomas recurrentes, lo que podría reflejar una recaída.

S. Typhi invade la mucosa intestinal a nivel del íleon terminal y alcanza el torrente sanguíneo vía sistema linfático, produciendo la primera bacteremia. Se distribuye en el SRE (hígado, bazo, médula ósea), donde se replica y luego de un período de incubación de 8-14 días se produce una nueva bacteremia. La fase clínica sin tratamiento dura 3-4 semanas. El período de transmisibilidad se mantiene mientras persistan los bacilos en las heces, normalmente desde la primera semana de enfermedad hasta el final de la convalecencia; este período es de 1 a 2 semanas en la fiebre paratifoidea. Un 10% de pacientes no tratados de fiebre tifoidea dispersarán bacilos durante tres meses después del inicio de los síntomas y del 2-5% se tornarán portadores permanentes.

10

Estudio de imágenes:

Flegmón submaxilar, submandibular y del piso de la boca

Radiografía lateral de partes blandas del cuello permite ver edema de la zona y evaluar compromiso de vía aérea. TAC es el examen de elección, permite evaluar foco de origen, extensión de la infección, compromiso óseo, y es indispensable para planear eventual cirugía.

Nivel de manejo del médico general: Diagnóstico Sospecha Tratamiento Inicial Seguimiento Derivar

Tratamiento Hospitalización, estabilización de vía aérea, eventual intubación. Esquema antibiótico: ampicilina-sulbactam 2 g ev cada 4 hrs o penicilina sódica 2-4 millones U ev c/4-6 hrs + Metronidazol 500 mg ev c/6 hrs.

Aspectos esenciales    

Urgencia médica. Antecedente de infección dental. Edema del piso de la boca y elevación de la lengua. Tratamiento médico quirúrgico agresivo con antibioterapia, drenaje y eventual extracción dentaria.

Drenaje en caso de colecciones fluctuantes, o deterioro clínico pese antibióticos adecuados. Extracción de pieza dentaria afectada, si corresponde.

Seguimiento Ancianos, inmunodeprimidos o diabéticos que evolucionan hacia abscesos requieren de un seguimiento cuidadoso y un tratamiento quirúrgico temprano.

Caso clínico tipo Paciente de 23 años de edad consulta por aumento de partes blandas en la región submaxilar y submandibular del lado derecho. Refiere dolor a la palpación cervical, fiebre de 38°C y odinofagia. Refiere además infección dental hace una semana tratada con antibióticos.

Definición Celulitis agresiva, rápidamente progresiva, sin formación de abscesos, con compromiso bilateral de espacios del piso de la boca producida por anaerobios.

Etiología-epidemiología-fisiopatología Extensión por contigüidad desde infección dentaria facilitada por anatomía de la boca. Un porcentaje menor se deben a focos óticos y sinusales. Infección polimicrobiana.

Diagnóstico Paciente febril con dolor bucal, sialorrea, halitosis, rigidez de cuello, y disfagia. Frecuentemente hay antecedente de infección del 2º o 3º molar. Examen físico muestra aumento de volumen indurado, sensible y simétrico de área submandibular con aumento de volumen y elevación de la lengua. Infección progresa rápidamente por contigüidad:  

Compromiso de epiglotis: estridor y cianosis indican pronóstico ominoso. Extensión hasta espacio parafaríngeo y retrofaríngeo puede provocar formación de abscesos, con clínica correspondiente (trismus). Sospechar compromiso parafaríngeo si aparece asimetría. Puede extenderse a mediastino y vasos del cuello.

11

localización pulmonar: dolor torácico, fatiga, cansancio y tos. Los quistes pulmonares suelen presentar vómica asociada a hemoptisis.

Hidatidosis

Laboratorio: Técnicas de inmunodiagnóstico como hemaglutinación indirecta (alta sensibilidad) e inmunoensayo enzimático (ELISA). Al hemograma destaca leve eosinofilia.

Nivel de manejo del médico general: Diagnóstico: Sospecha. Tratamiento: Inicial. Seguimiento: Derivar



Caso Sospechoso: Presencia de quistes (único o múltiples) en distintos órganos y tejidos, siendo los más frecuentes el hígado y pulmones, los que se pueden apreciar mediante ecografía, radiografía, tomografía axial computarizada y/o resonancia nuclear magnética.



Caso Confirmado: Caso sospechoso confirmado por anatomía patológica o visualización directa por microscopía de protoescólices del céstodo.

Aspectos esenciales 



El ser humano se infecta por la ingestión de huevos de parásitos presentes en alimentos, agua o suelo contaminados, o por contacto directo con animales huéspedes. Las dos formas más importantes de la enfermedad en el ser humano son la equinococosis quística (hidatidosis) y la equinococosis alveolar.

Tratamiento Caso clínico tipo Evaluar caso a caso: Paciente de origen rural con cuadro de 5 días de evolución, caracterizado por malestar general, decaimiento, anorexia y fiebre de 38 a 40°C. Posteriormente presenta tos con expectoración de aspecto similar a hollejo de uva.

- Portadores asintomáticos: la conducta a seguir luego de la confirmación del caso, se decidirá teniendo en cuenta el tipo de quiste y su tamaño. - Tratamiento farmacológico: Albendazol 10 mg/kg de peso/día, en dos tomas diarias luego del almuerzo y cena (rico en grasas), por 3 ciclos de 30 días cada uno sin interrupciones. Puede asociarse a inhibidores H1 u omeprazol mientras dure el tratamiento

Definición Zoonosis parasitaria de distribución mundial causada por las formas larvarias de parásitos del género Echinococcus spp., cuyo hospedero definitivo de mayor importancia epidemiológica, en el cono sur de América, es el perro. El ser humano y los animales de producción son hospederos intermediarios

- Pacientes sintomáticos o con quistes hidatídicos complicados (absceso, ruptura a cavidad abdominal, apertura a la vía biliar, tránsito tóraco-abdominal): el tratamiento de elección es la cirugía, ya sea convencional o laparoscópica según el caso en particular y la experiencia del equipo quirúrgico ( siempre que sea posible se efectuará quimioprofilaxis preoperatoria con Albendazol 10 mg/kg/día durante al menos 15 días. Además, se recomienda el uso de Albendazol en todos los casos durante 3 ciclos en el post-operatorio).

Etiología-epidemiología-fisiopatología En Chile, la hidatidosis constituye una enfermedad endémica, cuyo agente etiológico descrito corresponde a la especie Equinococcus granulosus. Tiende a concentrarse en áreas rurales que presentan condiciones de pobreza y vulnerabilidad social, asociado a la práctica de agricultura de subsistencia, particularmente la crianza de ganado ovino, caprino y en menor proporción bovino. El ser humano puede ser huésped intermediario del Equinococcus granulosus, cuyo huésped definitivo es el perro. Los parásitos adultos viven en el intestino de este último y sus huevos son eliminados al ambiente con sus deposiciones y contaminan los pastos que ingiere el ganado. La infección de los humanos se produce por contacto directo o indirecto con las deposiciones del perro. Los huevos ingeridos se transforman en larvas en el intestino, que migran a través de la circulación portal y se alojan principalmente en el hígado y pulmón, otras localizaciones corresponden a riñón, bazo, sistema nervioso central, huesos y músculo.

Seguimiento Estudio con imágenes seriadas para evaluar creciemiento de quistes. Enfermedad de notificación obligatoria inmediata.

Diagnóstico El quiste hepático incluye dolor en el hipocondrio derecho, masa palpable, ictericia y fiebre. Las complicaciones más frecuentes de los quistes hepáticos suelen ser las roturas e infección, transformándose en un absceso. En la 12

hemocultivos (HC). Si se decide dejar el catéter se solicitan HC por tiempo diferencial (HC automatizado 10 cc periféricos más 10 cc del lumen de CVC). Es positivo si existe una diferencia en detección de la positividad de 2 horas en favor de la muestra tomada a través del CVC. Otra alternativa son los HC cuantitativos donde se requiere muestra en jeringa de 2 cc del lumen del catéter más jeringa de punción periférica más 2 HC periféricos. Se considera positivo si existe una relación 3/1 en favor del HC tomado a través de un lumen del CVC v/s los HC periféricos. En caso de retiro del catéter se envía la punta a cultivo más 2 HC periféricos. Es positivo si la punta del catéter tiene el mismo agente de los HC periféricos.

Infecciones asociadas a catéteres vasculares Nivel de manejo del médico general: Diagnóstico: Sospecha. Tratamiento: Inicial. Seguimiento: Derivar

Aspectos esenciales  

Tratamiento

La complicación más frecuente de la cateterización es la infección. Causa más frecuente de bacteriemia nosocomial primaria es infección de CVC.

- Retiro del catéter: Tras el diagnóstico de la infección relacionada con el catéter, su retirada es adecuada en el contexto de una sepsis severa, inestabilidad hemodinámica, tromboflebitis supurativa, endocarditis, bacteriemia persistente después de 72 horas de tratamiento antibiótico al cual el organismo es susceptible.

Caso clínico tipo Paciente masculino, de 69 años, con ERC en HD por CVC transitorio instalado hace 3 meses. Refiere síndrome febril, calofríos, hipotensión e importante CEG durante la diálisis.

- Antibiótico: Se inicia terapia empírica para cocáceas positivas con vancomicina, luego ajustar según cultivos y antibiograma. En general, la terapia antibiótica sistémica no es necesaria en las siguientes circunstancias:

Definición

● Cultivo de la punta del catéter positivo en ausencia de signos clínicos de infección

Infección del tracto sanguíneo (ITS) por catéter venoso central (CVC) se define ante la presencia de bacteriemia o fungemia cuyo origen es el CVC.

● Hemocultivos positivos obtenidos a través de un catéter con cultivos negativos a través de una vena periférica

Etiología-epidemiología-fisiopatología

● Flebitis en ausencia de infección

Alrededor de la mitad de las infecciones nosocomiales torrente sanguíneo se producen en las unidades de cuidados intensivos, y la mayoría están asociados con la presencia de un dispositivo intravascular.

Seguimiento Bacteriemia o fiebre persistente implica el estudio de complicaciones (trombosis séptica, endocarditis bacteriana, osteomielitis).

Fuentes de origen de infección del catéter: la colonización de la piel, la contaminación intraluminal, la siembra secundaria de una infección del torrente sanguíneo y la contaminación del líquido de infusión (raro). La flora de la piel es probablemente la fuente más importante de infección del catéter intravascular. Por lo tanto, los estafilococos coagulasa-negativos y Staphylococcus aureus, son las cepas más comunes de las infecciones nosocomiales torrente sanguíneo

Diagnóstico Se sospecha en pacientes con catéter que presentan fiebre y calofríos. También pueden presentar hipotensión, confusión, hiperventilación, nauseas y vómitos. Algunos pacientes presentan inflamación o pus en sitio de catéter. Disfunción del catéter: Sospecharlo en pacientes que presentan estos signos abruptamente luego de infusión de medicamentos por el catéter infectado o intradiálisis. Importante es evaluar otros focos infecciosos que puedan explicar el cuadro o estar de manera concomitante. Junto al cuadro clínico se debe documentar la infección con 13

y potencialmente mortal, acompañado multiorgánica. Los síntomas habitualmente 10 días después del contacto.

Leptospirosis

La enfermedad se presenta en dos fases, una fase leptospirémica (germen en sangre y LCR) que inicia bruscamente con fiebre, cefalea, mialgias, calofríos, compromiso de estado general y manifestaciones hemorrágicas en distintos órganos. El signo clínico característico es la hemorragia conjuntival. Cuando existe un fallo multiorgánico manifestado por ictericia grave, uremia y neumonitis hemorrágica se denomina Leptospirosis ictérica o Enfermedad de Weil.

Nivel de manejo del médico general: Diagnóstico: Sospecha. Tratamiento: Inicial. Seguimiento: Derivar

Aspectos esenciales  



de falla aparecen

Zoonosis. Su agente causal es la espiroqueta Leptospira interrogans. El diagnóstico debe ser considerado en cualquier paciente que presente fiebre súbita, escalofríos, inyección conjuntival, dolor de cabeza, mialgia e ictericia. Historia de exposición ocupacional o recreacional a animales infectados o a un ambiente posiblemente contaminado con orina de animales

Tras 4-9 días mejora la enfermedad y aparece la segunda fase o fase inmune (anticuerpos), similar a la anterior, con la aparición de meningitis aséptica, anemia hemolítica, ictericia, leucocitosis y trombocitopenia. Se requiere un alto índice de sospecha para hacer el diagnóstico basado en la exposición epidemiológica y las manifestaciones clínicas, ya que los hallazgos clínicos y de laboratorio son inespecíficos. El diagnóstico se realiza con mayor frecuencia por pruebas serológicas.

Caso clínico tipo Tratamiento

Paciente masculino de 28 años ingresa por fiebre de inicio brusco, calofríos y malestar general. Posteriormente desarrolla inyección conjuntival y cefalea con signos meníngeos. Al interrogatorio dirigido describe haber estado hace 1 o 2 semanas haciendo turismo aventura en bosques húmedos.

- Enfermedad leve: tratamiento ambulatorio con doxiciclina 100 mg vo c/12 hrs o Azitromicina 500 mg/día x 3 días. - Enfermedad grave: hospitalizar, tratamiento con penicilina ev 1,5 millones de UI c/6 hrs o doxiciclina ev 100 mg c/12 hrs o ceftriaxona ev 1 gr c/24 hrs. La duración del tratamiento en enfermedad grave es generalmente 7 días.

Definición Es una zoonosis febril aguda con presentaciones clínicas variables causada por espiroquetas del genero Leptospira (en gran mayoría Leptospira interrogans).

Seguimiento Es una enfermedad de notificación obligatoria, hospitalizar según clínica y en caso de optar por manejo ambulatorio recontrolar y educar sobre síntomas de alarma.

Etiología-epidemiología-fisiopatología La leptospirosis es una enfermedad difusa, presente en climas templados como tropicales (más frecuente) y subdiagnosticada. Leptospira es una espiroqueta aeróbica, se encuentra en mamíferos, reptiles y anfibios salvajes y domésticos. El microorganismo llega al hombre al contactar directamente con animales infectados o indirectamente por la orina en agua o suelos contaminados. El microorganismo penetra por piel erosionada o distintas mucosas llegando a la sangre. La Organización Mundial de la Salud (OMS) informa una incidencia anual de leptospirosis humana de 0,1 casos por 100.000 habitantes para climas templados, de 10 a 100 por 100.000 habitantes en climas tropicales y 100 por 100.000 habitantes en brotes y grupos de alto riesgo. Brasil, China y los países del sudeste asiático reportan la mayoría de los casos.

Diagnóstico El curso clínico es variable, puede manifestarse como una enfermedad subclínica seguido de seroconversión, una infección sistémica autolimitada, o una enfermedad grave 14

Malaria

Diagnóstico

Nivel de manejo del médico general: Diagnóstico: Sospecha. Tratamiento: Inicial. Seguimiento: Derivar

El inicio de la enfermedad se produce semanas o hasta meses luego de la infección. Presenta fiebre, cefalea, mialgias y fatiga. Se caracteriza por paroxismos trifásicos periódicos por la ruptura de los glóbulos rojos. Estos empiezan con fiebre alta, cefalea, tos y náuseas que culminan con sudoración abundante. Debe sospecharse y excluirse el diagnóstico de malaria en todas aquellas personas con fiebre que han estado en un área endémica durante un año previo.

Aspectos esenciales   

P. falciparum da cuadros más graves con hiperparasitemias, anemia hemolítica, malaria cerebral (compromiso de conciencia, delirium, convulsiones), hepatoesplenomegalia, hipoglicemia, acidosis láctica, insuficiencia renal, síndrome de distrés respiratorio agudo o coagulopatía. En embarazo los cuadros por lo general son más graves.

En Chile no existen casos autóctonos. Todos son importados. Los agentes causales son principalmente P. vivax y P. falciparum. Clínicamente se caracteriza por paroxismos periódicos de fiebre.

Caso clínico tipo

El diagnóstico se realiza por la visualización de parásitos en el frotis sanguíneo con tinción de Giemsa (los frotis tienen mejor rendimiento antes de los episodios de fiebre).

Paciente femenino de 35 años presenta fiebre, malestar general y sudoraciones abundantes hace 5 días. Cada dos días presenta agravamiento de sus síntomas. Al interrogatorio dirigido relata que hace 6 meses estuvo en la selva peruana

Tratamiento En casos de sospecha alta de la infección, sin posibilidad de diagnóstico precoz, se justifica el uso tratamiento empírico.

Definición

El tratamiento se evalúa caso a caso, ya que depende de la especie de Plasmodium, zona geográfica y forma clínica (preventivo, aguda, crónica, embarazada).

La Malaria o paludismo, es una enfermedad potencialmente mortal causada por parásitos que se transmiten al ser humano por la picadura de mosquitos pertenecientes al género Anopheles infectados, por especies del género Plasmodium.

Algunos antimaláricos son Artesunato (el que tiene mejor perfil de seguridad y eficacia), Atovacuona, Cloroquina, Mefloquina, Primaquina..

Etiología-epidemiología-fisiopatología

Seguimiento

Es endémica de la zona tropical y subtropical del planeta. En Chile ya no se detectan casos autóctonos, por lo que la incidencia existente es por casos importados. El África subsahariana soporta una parte desproporcionada de la carga mundial de paludismo. En 2015, el 90% de los casos y el 92% de los fallecimientos por la enfermedad se produjeron en esta región.

El paciente debe ser hospitalizado y monitorizado. Es de notificación obligatoria.

Las especies de Plasmodium que afectan al hombre son 4: P. vivax(causante del paludismo dominante en la mayoría de los países fuera del África subsahariana), P. falciparum (causante del paludismo más prevalente en el continente africano. Es responsable de la mayoría de las muertes provocadas por el paludismo en todo el mundo) , P. malarie y P. ovale, siendo los dos primeros los frecuentes en Latinoamérica. La infección se produce por la picadura del mosquito Anopheles. En el hombre, existen dos fases: Fase Hepática: los parásitos vía sanguínea llegan a los hepatocitos y se multiplican en ellos hasta que se rompen dejando al parásito libre; y Fase Eritrocítica: se generan merozoítos en los glóbulos rojos que generan rupturas sincrónicas de los eritrocitos cada 48 horas en el caso de P. vivax (terciana benigna) y P. falciparum (terciana maligna) y cada 72 horas en P. malariae (cuartana).

15

diagnostico microbiologico y tratamiento dirigido al patógeno específico. Este procedimiento puede no ser necesario de realizar si existen hemocultivos positivos y hallazgos radiologicos consistentes con el cuadro.

Osteomielitis Nivel de manejo del médico general: Diagnóstico: Inicial. Tratamiento: Inicial. Seguimiento: Por especialista

Tratamiento El tratamiento de la osteomielitis consiste en el desbridamiento y la terapia antimicrobiana. La duración óptima del tratamiento antibiótico no es cierta; se sugiere continuar el tratamiento antimicrobiano parenteral al menos seis semanas desde el última desbridamiento.

Aspectos esenciales    

El 90% de los casos está provocado por el S. aureus. Cuando dispositivos ortopédicos son la causa del cuadro, se requiere su retirada. La infección puede establecerse por tres vías: hematógena, diseminación contigua y directa. Puede presentarse de forma aguda y crónica.

Si no se identifica el organismo causal, hay que seleccionar un tratamiento empírico que cubra S. aureus (Cloxacilina o vancomicina) y otro posible patógeno según la epidemiología local y datos estadísticos de cada centro.

Seguimiento Caso clínico tipo

Por especialista

Paciente varón, 15 años, sufre traumatismo directo en pierna. Se evidencia fractura expuesta, la cual no es manejada con aseo quirúrgico adecuado. Evoluciona posteriormente con fiebre en agujas, taquicardia, cefalea, deshidratación, progresivo compromiso de estado general, dolor y aumento de temperatura local.

Definición Es la inflamacion del hueso causada por un organismo infeccioso, el cual alcanza el tejido óseo por diseminación hematógena (bacteremia), diseminación contigua de tejidos adyacentes o por inoculación directa (trauma, cirugías, prótesis, cuerpos extraños).

Etiología-epidemiología-fisiopatología Causas más frecuentes son postraumática (47%), insuficiencia vascular (más frecuente en diabéticos, 34%) y diseminación hematógena (20%). Aproximadamente el 90% de los casos está provocado por el S. aureus, no obstante, cualquier germen puede ser causal de infección del hueso. Es necesario agregar que los Gram negativos han ido aumentando en frecuencia como causa de infección ósea.

Diagnóstico El cuadro generalmente consiste en dolor asociado al hueso involucrado, puede encontrar enrrojecimiento, inflamación, puede existir fiebre. También puede presentarse como una artritis séptica. El diagnóstico se realiza detectando el hueso expuesto a través de una úlcera en la piel (prueba del estilete) o mediante diagnóstico por la imagen con radiografías simples (erosión cortical, reacción periostal, esclerosis), gammagrafía ósea o resonancia magnética. La realización de biopsia (abierta o con aguja) y cultivos del hueso afectado, antes de iniciar tratamiento con antimicrobianos si es posible, es fundamenal para el 16

Diagnóstico

Síndrome pulmonar por hanta virus

Periodo de incubación: 5-45 días. Fase prodrómica: 1-6 días. Síntomas: Fiebre, vómitos, cefalea, mialgias.

Nivel de manejo del médico general: Diagnóstico Sospecha Tratamiento Inicial Seguimiento Derivar

Fase cardiopulmonar: 6-9 días. Síntomas: Tos, taquipnea, broncorrea serosa, inestabilidad hemodinámica, sangramientos externos, shock cardiogénico. Infiltrado difuso bilateral en la radiografía.

Aspectos esenciales

Fase de recuperación (poliúrica): 10-15 días. Estabilización hemodinámica mejoría de la falla respiratoria, normalización de la radiografía de tórax.

  

Virus Andes es el agente que provoca SPHV en Chile. Tétrada de trombocitopenia, leucocitosis, hemoconcentración y presencia de inmunoblastos sugiere fuertemente SPHV. Rivavirina y corticoides no son útiles para el manejo del cuadro.

Convalecencia: 2-3 meses. Hemograma: Aparece precozmente trombocitopenia, y se observan linfocitos atípicos (inmunoblastos), mas de 10% en frotis. Generalmente leucocitosis con desviación izquierda, pero sin granulación toxica. La hemoconcentración aparece más tardíamente. Tétrada de trombocitopenia, leucocitosis, hemoconcentración y presencia de inmunoblastos sugiere fuertemente SPHV. Suele haber alza de transaminasas y creatinfosfoquinasa, las que son atribuidas a rabdomiolisis. LDH se eleva como respuesta inespecífica a injuria pulmonar. VHS es generalmente normal o levemente elevada.

Caso clínico tipo Hombre de 35 años, consulta al servicio de urgencia por vómitos, cefalea, mialgias y tos. Como antecedente epidemiológico refiere haber viajado al sur del país (Temuco) y en el hemograma presenta leucocitosis.

Diagnóstico diferencial en etapa prodrómica: Influenza y otras virosis sistémicas, adenovirus y otras virosis respiratorias. Gastroenteritis aguda y otras afecciones abdominales agudas. Fiebre tifoidea. Pielonefritis aguda.

Definición Cuadro clínico agudo producido por virus de la familia hantavirus, genero Bunyaviridae, con Argentina compartimos el virus ANDES.

Antecedente epidemiológico de exposición a roedores o situaciones de riesgo (residencia o visita sectores rurales, desmalezar, limpiar casa deshabitada) las últimas 6 semanas, la ausencia de coriza y exudado faríngeo y la presencia de alteraciones hematológicas descritas deben orientar al diagnóstico de SPH.

Etiología-epidemiología-fisiopatología En Chile es una enfermedad endémica, con presentación estacional primavera-verano. Edad promedio: 32,3 años.

Confirmación diagnóstica es por serología: detección de anticuerpos IgM e IgG específicos contra hantavirus presentes en sangre o suero mediante ELISA.

La enfermedad se presenta principalmente en varones (71%) en edad productiva (25-49 años). La gran mayoría son trabajadores agrícolas o forestales. Hay ratones infectados desde la IV hasta la XI región, pero afecta más entre RM y XI, común en gente que trabaja en el bosque y turistas.

Tratamiento Traslado precoz a centro que tenga UCI. Manejo sintomático, el uso de rivavirina es útil en virus europeo, no en Chile. Debe notificarse el caso. Usar soporte mecánico y ventilatorio.

Modo de transmisión al humano es por inhalación de aerosoles provenientes de la saliva, orina y heces de roedores portadores (Oligoryzomys longicaudatus). El virus en el humano se replica en el endotelio pulmonar y cardiaco, sin ocasionar destrucción celular. La producción de anticuerpos neutralizantes de forma precoz posee mejor pronóstico. Los antígenos virales activan los mecanismos de inmunidad celular en el bazo, mediados por inmunoblastos y linfocitos T. El TNF α y la IL-8 aumentan en casos graves. Como consecuencia de esta respuesta ocurre el EPA no cardiogénico y la depresión miocárdica.

Seguimiento Por especialista.

17



Tuberculosis extrapulmonar

 Nivel de manejo del médico general: Diagnóstico Sospecha Tratamiento Inicial Seguimiento Derivar

Tratamiento En todas las formas de tuberculosis extrapulmonar es importante la terapia antituberculosa. Algunas localizaciones pueden requerir asociación de otros fármacos, como corticoides en afectación de meninges, sistema nervioso central, peritoneo, gastrointestinal y pericardio. La adenitis tuberculosa en ocasiones precisa resección quirúrgica. Importante mencionar que la tuberculosis osteoarticular no requiere drenaje, pues responde bien a la medicación.

Aspectos esenciales   

Localizaciones más frecuentes: pleura, seguido por ganglios linfáticos. Habitualmente en pacientes inmunocomprometidos. En meningitis, serositis y compromiso SNC se utilizan corticoides junto a la terapia antituberculosa.

Seguimiento Se debe realizar seguimiento de la respuesta de la infección a la terapia.

Caso clínico tipo Paciente de sexo masculino, 32 años de edad, con antecedentes de VIH sin tratamiento, presenta adenopatía dolorosa de 3 cm de diámetro en región supraclavicular, con características inflamatorias, de consistencia gomosa. Se realiza biopsia excisional y el resultado histopatológico confirma adenitis tuberculosa.

Definición Tuberculosis extrapulmonar es una infección por el Mycobacterium tuberculosis que afecta algún órgano o tejido fuera de la localización pulmonar. Puede comprometer pleura, ganglios linfáticos, sistema urogenital, sistema osteoarticular, gastrointestinal, peritoneal y pericardio, así como también otras localizaciones menos frecuentes.

Etiología-epidemiología-fisiopatología Prácticamente en todos los casos existe un foco primario de tuberculosis pulmonar, desde este foco se puede producir una diseminación por contigüidad, por vía linfática o hematógena, siendo esta última la causante de la mayoría de las TBC extrapulmonares (a excepción de la pleural y la linfática). Habitualmente esta forma se presenta en inmunocomprometidos, pero también puede presentarse en pacientes inmunocompetentes.

Diagnóstico     

Peritoneal o gastrointestinal: biopsia laparoscópica. Pericárdica: signos indirectos en radiografía tórax, ECG y examen físico.

El estudio diagnóstico dependerá de la localización del proceso infeccioso: Linfática: biopsia excisional y cultivo. Urogenital: urocultivo, cultivo de masas y raspado uterino. Osteoarticular: biopsia por aguja para lesiones vertebrales, biopsia sinovial y cultivo de líquido sinovial para articulaciones. Meningitis: punción lumbar (linfocitosis, aumento de proteínas y glucorraquia baja).

18

grupo A invasivo. Aquí destacan celulitis, miositis, fasceitis necrotizantes y síndrome de shock tóxico.

Varicela complicada: neumonitis, cerebelitis, encefalitis

- Complicaciones neurológicas: Difíciles de ver, las más graves son:

Nivel de manejo del médico general: Diagnóstico Sospecha Tratamiento Inicial Seguimiento Derivar

Aspectos esenciales 

 

Ocasionado por el VZV, la primoinfección resulta en un rash vesicular difuso, mientras la reactivación resulta en una infección localizada de la piel. Las complicaciones incluyen infecciones de tejidos blandos y piel, complicaciones neurológicas y neumonía. La administración de Aciclovir intravenoso ha aumentado la sobrevida en pacientes identificados de alto riesgo para varicela grave.



Encefalitis: Ataxia cerebelosa aguda/encefalitis difusa. Estos trastornos típicamente se desarrollan hacia el final de la 1era semana de exantemapero hay casos en que el compromiso del SNC precede a la erupción. Encefalitis difusa: delirio, convulsiones y focalidad neurológica.



Síndrome de Reye: Náusea, vómitos, cefalea, excitabilidad, delirio, con frecuente progresión a coma.



Otras menos comunes: déficit focal transitorio, meningitis aséptica, mielitis transversa, vasculitis y hemiplejía.

- Neumonía: Raro en niños inmunocompetentes, ocurre en la mayoría de los adultos con varicela. Factores de riesgo: tabaco, embarazo, inmunodepresión y sexo masculino. Típicamente se presenta 1 a 6 días después de la aparición del rash con taquipnea progresiva, disnea, y tos seca, la hemoptisis también ha sido reportada. La radriografía de tórax típicamente revela infiltrados bilaterales.

Caso clínico tipo Hombre de 27 años, consulta en SU por disnea, tos seca y hemoptisis. Al examen físico presenta rash vesicular difuso y refiere que su hijo de 5 años presentó varicela hace 5 días.

- Otras complicaciones: Hepatitis, diarrea, faringitis y otitis media.

Tratamiento Definición

Se sugiere que haber recibido un tratamiento agresivo y precoz con Aciclovir intravenoso en aquellos pacientes identificados de alto riesgo para varicela grave (bebés, embarazadas, inmunocompromiso), impactó favorablemente en la sobrevida. La vacunación se utiliza como prevención.

Cuadro ocasionado por el VZV. La primoinfección resulta en un rash vesicular difuso, mientras la reactivación endógena resulta en una infección localizada de la piel conocida como herpes zoster.

Etiología-epidemiología-fisiopatología

Seguimiento

La varicela es altamente contagiosa (tasas infección 2aria >90%). Transmisión ocurre a través del contacto con gotitas de aerosol de secreciones nasofaríngeas de individuos infectados o por contacto directo con fluido de vesículas (piel). Período de incubación es de 14 a 16 días (intervalo 10-21 días). El período de infectividad es considerado desde las 48 hrs anteriores a la aparición de la erupción hasta que las lesiones cutáneas presentan una costra.

Derivar.

Antes de la introducción de la vacuna, los niños y adolescentes sanos representaban el 80% de las hospitalizaciones anuales relacionadas con la varicela en Estados Unidos. Después de la implementación de la vacuna, el número de complicaciones ha disminuido. Las complicaciones incluyen infecciones de piel y tejidos blandos (42%), deshidratación (11%) y complicaciones neurológicas (9%).

Diagnóstico - Infecciones de tejidos blandos/piel: Infección 1aria de varicela en niños ha sido asociada con aumento de la incidencia de infección de tejido blando por estreptococo 19

MÓDULO 1: Medicina Interna

Enf. Respiratorias Asma Bronquial Grave gir con una terapia más agresiva está el riesgo de remodelación de la vía aérea, donde la obstrucción al flujo aéreo se vuelve permanente.

Nivel de manejo del médico general: Diagnóstico Específico Tratamiento Inicial Seguimiento Derivar

Diagnóstico

Aspectos Esenciales • • •

Todo paciente (en tratamiento o no) con clínica de asma que presente episodios frecuentes y síntomas intercrisis: sibilancias a esfuerzos mínimos, síntomas nocturnos más de 2 veces por semana, limitación de cualquier tipo en su actividad diaria, necesidad de beta-agonistas 3 o más veces por semana. En pruebas de función pulmonar, se aprecia un VEF1 (intercrisis) <70% y variabilidad del PEF >30%. En caso de presentarse asma de difícil manejo, debe siempre pensarse en diagnósticos diferenciales: EPOC, insuficiencia cardíaca congestiva (“asma cardíaca”), embolía pulmonar, sarcoidosis, bronquitis, bronquiectasias, RGE, rinitis/rinosinusitis, bronquiolitis, entre otras, y descartar la presencia de aspergilosis broncopulmonar alérgica como causa de descompensación y resistencia a terapia.

Síntomas continuos y nocturnos frecuentes. Requiere terapias más agresivas para evitar remodelación, iniciar con 3° escalón: BAAC a demanda + corticoides inhalados + BAAL. Si requiere del 4to paso de tratamiento derivar a especialista.

Caso Clínico Tipo Paciente conocido como asmático en tratamiento con corticoides inhalados en dosis bajas y salbutamol de rescate. Refiere aumento de síntomas luego de un episodio de influenza hace un mes con limitación de su actividad diaria y necesidad de usar salbutamol como rescate 3 veces la última semana.

Tratamiento El tratamiento busca un control rápido de los síntomas y evitar la remodelación. Para ello se debe partir con el paso 3 (corticoides inhalados en dosis bajas, como fluticasona o budesonida, con β2 agonistas de acción larga, como salmeterol o formoterol, y salbutamol de rescate) y frente a mala respuesta al tratamiento o no lograr los objetivos de control (luego de 3 meses) derivar oportunamente al especialista para una caracterización más específica de su asma y manejo con el 4to o 5to escalón de tratamiento que corresponden a: • Corticoides inhalados en dosis medias o altas más β2 agonistas de acción larga con o sin terapia antileucotrienos. • Corticoides orales como prednisona o prednisolona (menor dosis) con o sin anti-IgE (omalizumab).

Definición El asma bronquial grave es un cuadro clínico que se caracteriza por presentar síntomas diurnos continuos, despertar habitual por asma, VEF1<60% y variabilidad del PEF>30%.

Etiología-Epidemiología-Fisiopatología El asma grave persistente afecta al 10% de los pacientes asmáticos, pero en más del 50% de los casos no se consigue el control total a pesar de un correcto tratamiento. De no corre-

1 Facultad de Medicina, Universidad de Chile

Seguimiento Se espera que luego de comenzado el tratamiento se pierdan los síntomas nocturnos y los intercrisis, quedando sólo con broncodilatación de rescate menor a 2 por semana. Para lo cual se da un período de 3 meses. - Criterios de derivación a especialista: • Todos los pacientes que requieran el paso 4 o superior en la escala de manejo. • Episodio de exacerbación que amenace la vida. • No responde a terapia o no logra objetivos de tratamiento en 3 meses. • Signos y síntomas atípicos que dificultan diagnóstico diferencial.

Autor / Editor Marcelo Fres

Año 2016

2 Facultad de Medicina, Universidad de Chile

importante es la disnea paroxística, acompañada de sibilancias audibles a distancia, sensación de pecho apretado y tos. Los síntomas se exacerban durante la noche y al despertar. Puede haber antecedentes familiares de asma. El examen físico en una intercrisis será normal, mientras que en crisis se verán signos de obstrucción bronquial típicos, uso musculatura accesoria, cianosis y signos de fatiga. La radiografía de tórax suele ser normal. En la espirometría basal y post-broncodilatador, se observa un patrón obstructivo y reversibilidad (VEF1 aumenta 12% y 200 ml al menos post BD), aunque puede ser normal en períodos asintomáticos. La confirmación de la hiperreactividad bronquial se hace con test de metacolina (caída 20% VEF1 con dosis ≤ 8 mg/ml). Sin embargo, la hiperreactividad no es lo mismo que asma, y el diagnóstico debe realizarse principalmente por la clínica, asociado a los exámenes señalados.

Asma Bronquial Leve y Moderada Nivel de manejo del médico general: Diagnóstico Específico Tratamiento Completo Seguimiento Completo

Aspectos Esenciales • • • • •

Hipersensibilidad bronquial secundaria a inflamación crónica de vías aéreas. Factores genéticos y ambientales están involucrados. Examen físico en intercrisis puede ser normal. El diagnóstico se realiza con clínica + test de metacolina positivo. El tratamiento es sintomático, asociado a terapia antiinflamatoria.

Tratamiento Los objetivos del tratamiento del asma bronquial es lograr el control de las manifestaciones clínicas y mantenerlo en el tiempo. El control del asma se puede dividir en dos componentes: • Control actual, que se refiere a los síntomas, nivel o limitación de actividad, terapia de rescate y función pulmonar. • Componente de riesgo futuro, se refiere a la probabilidad de deterioro, exacerbaciones y en forma muy importante la declinación de la función pulmonar.

Caso Clínico Tipo Paciente varón de 21 años. Consulta por episodios de “silbidos” en el pecho y tos de 3 meses de evolución, principalmente nocturnos, y que se han hecho más frecuentes. A la auscultación pulmonar: discreta disminución del murmullo pulmonar, sibilancias leves difusas. Radiografía de tórax: normal.

Definición Inflamación crónica de vías aérea que involucra obstrucción variable al flujo aéreo reversible e hiperreactividad bronquial. Esta patología puede evolucionar a ser irreversible, secundario a la remodelación de las vías aéreas.

Etiología-Epidemiología-Fisiopatología Es la enfermedad respiratoria crónica más prevalente en adultos. En adolescentes es cercana al 10%. Principal causa de ausentismo laboral y escolar en jóvenes. Entre sus causas, destaca la inflamación bronquial crónica (factores genéticos y ambientales), hiperrreactividad de la vía aérea por estímulos inespecíficos (aire frío y ejercicio, irritantes como humo de tabaco, productos de combustión, infecciones respiratorias especialmente micoplasma y clamidia, drogas como agonistas colinérgicos, morfina, codeína) y específicos (alérgenos, AINEs, colorantes, RGE, factores psicológicos).

El tratamiento busca prevenir exacerbaciones y remodelación de vías aéreas. Cada paciente se asigna a uno de los 5 pasos de tratamiento dependiendo de su nivel actual de control del asma, y el tratamiento se ajusta constantemente según los cambios en el control del asma que experimenta el paciente. Es decir, se debe ir escalando por los distintos pasos del control del asma. En la mayoría de los pacientes con asma persistente el tratamiento debe iniciarse en el paso

Diagnóstico Generalmente se inicia en edad temprana. El síntoma más

3 Facultad de Medicina, Universidad de Chile

2. Si los síntomas sugieren un asma mal controlado, debe iniciarse en el paso 3. En todos los pasos de tratamiento se

debe indicar un medicamento de rescate (inicio rápido) para el tratamiento de los síntomas.

Seguimiento Evaluación de respuesta al tratamiento debe ser objetivada, idealmente con PEF domiciliario 2 veces al día. En casos leves y moderados, debido a falta de disponibilidad en nuestro medio, pueden usarse otros índices clínicos (síntomas, consumo de BD, PEF en consulta).

Autor / Editor Marcelo Fres

Año 2016

4 Facultad de Medicina, Universidad de Chile

mente marcado en la influenza), síntomas de compromiso rinofaríngeo, expectoración mucopurulenta y dolor torácico leve al toser. La presencia de paroxismos de tos severos hace necesario descartar infección por Bordetella pertussis.

Bronquitis Aguda Nivel de manejo del médico general: Diagnóstico Específico Tratamiento Completo Seguimiento Completo

Se indica solicitar radiografía de tórax en pacientes con signos vitales anormales (fiebre, taquipnea o taquicardia), signos de consolidación en el examen pulmonar, hipoxemia, confusión mental o signos de enfermedad sistémica. Característicamente no habrá alteraciones de los signos vitales y la radiografía de tórax será normal, pudiendo mostrar cierto grado de engrosamiento peribronquial. Se solicita si se desea descartar neumonía o en situaciones de inmunodepresión y pacientes añosos. El examen físico pulmonar suele ser normal, pudiendo mostrar signos de obstrucción bronquial (roncus, sibilancias, espiración prolongada u otros). Estudios microbiológicos son útiles ante casos graves, persistencia inusual de síntomas, sospecha de etiológica bacteriana o con fines epidemiológicos.

Aspectos Esenciales • • •

Patología frecuente, 90% origen viral. Diagnóstico clínico que no necesita pruebas adicionales. Su principal síntoma es la tos. El manejo es sintomático evitando antibióticos para no generar resistencia.

Caso Clínico Tipo Paciente consulta por cuadro catarral con tos seca de una semana de evolución, que a los 3 días comienza a ser productiva, asociada a dolor urente retroesternal. Se encuentra normotenso y normocárdico, con una frecuencia respiratoria de 16 x’. El examen físico pulmonar no muestra alteraciones.

Tratamiento Básicamente sintomático, tomando medidas generales como hidratación, antipiréticos, reposo y evitar irritantes ambientales. Los antitusígenos están indicados en casos de tos crónica no productiva, emetizante, que impida el sueño o bien que sea muy molesta. Si se auscultan sibilancias, sospechar hiperreactividad u obstrucción bronquial e indicar β2-agonistas inhalados (salbutamol 2 puf c/4-6 hrs) durante 1-2 semanas. No usar antibióticos a menos que cuadro se prolongue mayor a 7–10 días, preferir macrólidos: Eritromicina 500 mg c/6 hrs x 7 días o Claritromicina 500 mg c/12 hrs x 7 días.

Definición Inflamación aguda y difusa de la mucosa bronquial sin extenderse apreciablemente al parénquima pulmonar circundante.

Etiología-Epidemiología-Fisiopatología

Seguimiento

Cerca del 90% de los casos es causado por virus influenza, parainfluenza, VRS, enterovirus y adenovirus. El resto de los casos pueden ser causados por M. pneumoniae, C. pneumoniae y B. pertussis. Afecta a un 5% de la población adulta al año, siendo más frecuente en niños y en fumadores. Puede aparecer en cualquier época del año pero tiene mayor frecuencia en invierno. Este cuadro compromete preferentemente el epitelio respiratorio que se exfolia, con extensas zonas de denudamiento y exposición de receptores tusígenos fácilmente irritables. Causa frecuente de uso de antibióticos.

La persistencia de tos durante >3 semanas, que empeora por la noche o que se desencadena tras la exposición a frío o ejercicio debe hacer sospechar asma. En estas situaciones pueden ser necesarias las técnicas espirométricas correspondientes o realizar estudios de tos crónica.  

Diagnóstico El diagnóstico es principalmente clínico, focalizado en descartar neumonía. Su síntoma cardinal es la tos, que dura cerca de 3 semanas y en algunos casos puede extenderse por más de un mes (B. pertussis o síndrome de hiperrespuesta bronquial post-viral). Puede presentarse sola o acompañarse de fiebre baja, compromiso del estado general (particular-

5 Facultad de Medicina, Universidad de Chile

Tratamiento

Bronquitis Crónica

La principal medida es el cese del hábito tabáquico, lo cual logra disminuir los síntomas. Si hay presencia concomitante de infección respiratoria, se recomienda tratamiento sintomático. Se pueden administrar antibióticos (amoxicilina o macrólidos) sólo si presenta dos de las siguientes características: aumento de la cantidad de expectoración, si la expectoración se torna purulenta, o hay fiebre, asociado a hemograma sugerente.

Nivel de manejo del médico general: Diagnóstico Específico Tratamiento Completo Seguimiento Completo

Aspectos Esenciales • • •

Es un indicador de excesiva exposición tabáquica, conllevando gran riesgo de EPOC y cáncer. Su mayor molestia es la tos productiva crónica. Mayor susceptibilidad de desarrollar infecciones respiratorias bacterianas, con resolución dificultosa.

Seguimiento Los enfermos con bronquitis crónica desarrollan infecciones respiratorias bacterianas agudas con mayor frecuencia. Por otra parte, la velocidad de curación de estas infecciones está retardada, también debido a las fallas de los mecanismos defensivos que colaboran en la curación y reparación de las lesiones. Es importante tener presente que la existencia de bronquitis crónica significa que ha existido una exposición tabáquica importante con el consiguiente obligación de investigar los demás riesgos del tabaco.

Caso Clínico Tipo Paciente de 46 años, fumador crónico, IPA 19. Consulta por cuadro de tos con expectoración mucosa (200 ml diarios) de 5 meses de evolución. Refiere haber tenido un cuadro similar el invierno pasado. A la auscultación pulmonar, se escuchan sibilancias.

Autor / Editor Paulina Lira

Definición Presencia de tos con expectoración en la mayoría de los días por >3 meses al año por 2 o más años consecutivos, excluyendo patologías basales que la provoquen.

Etiología-Epidemiología-Fisiopatología La causa principal es el tabaquismo. Se encuentra presente en el 15% de los fumadores sin enfisema, y en el 80% de los fumadores con enfisema. Otra causa es la exposición a partículas irritantes (de ambiente laboral o contaminación intradomiciliaria). También hay casos sin una clara causa que lo provoque.

Diagnóstico Es eminentemente clínico. El cuadro se caracteriza por tos con expectoración persistente, de distintos grados (leve y matinal, o constante muy productiva). El examen físico puede ser normal, o pueden auscultarse algunos roncus diseminados o crepitaciones basales. La radiografía de tórax debe ser normal, ya que cualquier alteración obliga a descartar que ésta se deba a otra patología. La espirometría debe ser normal. Si existen indicadores de obstrucción bronquial difusa, debe pensarse concomitancia con EPOC, por lo que se debe pedir espirometría buscándola.

6 Facultad de Medicina, Universidad de Chile

Año 2016

Imagenología: radiografía de tórax PA y lateral, TAC con contraste en derrame mínimo. Toracocentesis y análisis del líquido pleural: Exudado seroso o hemático (que en ausencia de traumatismo es muy sugerente), predominio linfocitario >50%, glucosa pleural menor a 60 mg/dl, niveles elevados de ADA y lactato. El estudio citológico puede revelar la presencia de células neoplásicas (diagnóstico 60-80% casos), en caso de ser negativa y de haber alta sospecha, realizar biopsia pleural bajo visión de VTC o toracotomía.

Derrame Pleural Neoplásico Nivel de manejo del médico general: Diagnóstico Específico Tratamiento Inicial Seguimiento Derivar

Aspectos Esenciales • • • •

Causas: 1° cáncer pulmonar, 2° metástasis de cáncer mamario. Confirmación con estudio del líquido pleural con estudio citológico o biopsia. Control de disnea con toracocentesis evacuadora. Tratamiento paliativo con pleurodesis.

Tratamiento El tratamiento del derrame es paliativo para controlar la disnea. Para esto se realiza pleurodesis con talco u otras sustancias o en casos muy avanzados sólo se realizan toracocentesis repetidas. Puede también instalarse un drenaje pleural que el paciente puede incluso utilizar en su domicilio con vaciamientos frecuentes. Tratar la enfermedad de base, en tumores quimiosensibles debe realizarse quimioterapia (linfoma, mama, ovario, tumor de células pequeñas de pulmón).

Caso Clínico Tipo Paciente mujer de 78 años. Consulta por dolor lumbar y disnea asociada a tos seca de 2 meses de evolución. Al examen físico, se detecta retracción de pezón derecho y masas pétreas. A la radiografía de tórax destaca derrame bilateral.

Seguimiento Derivar a especialista. El pronóstico de estos pacientes en general malo sobre todo en los de origen metastásico. La supervivencia en paciente es menor en aquellos con líquido pleural que tiene un pH menor a 7,3 y glucosa menor a 60 mg/dl.

Definición El derrame pleural neoplásico es una entidad común en los pacientes portadores de enfermedades malignas, definiéndose como aquel derrame en el cual se pesquisan células neoplásicas por citología o biopsia.

Etiología-Epidemiología-Fisiopatología

Autor / Editor Marcelo Fres

La causa más frecuente en ambos sexos es el cáncer pulmonar. En mujeres, también puede deberse a metástasis de cáncer mamario. La neoplasia puede dar lugar a derrame pleural por disminución del drenaje linfático pleural (más importante), incremento de permeabilidad en la microcirculación pleural y el aumento de la presión negativa intrapleural. Es la causa más frecuente de derrame sanguinolento, y la segunda causa de exudado.

Diagnóstico Clínica: Disnea, dolor pleurítico o tos seca. Al examen físico puede no haber anormalidades o, dependiendo de la magnitud del derrame, evidenciarse disminución de la expansión torácica, disminución o abolición de las vibraciones vocales, matidez, disminución o abolición del murmullo pulmonar, egofonía y soplo pleurítico.

7 Facultad de Medicina, Universidad de Chile

Año 2016

Derrame Pleural Paraneumónico Complicado

cientes requieren de cirugía.

Diagnóstico

Nivel de manejo del médico general: Diagnóstico Específico Tratamiento Inicial Seguimiento Derivar

Signos clínicos y se confirma con una radiografía simple de tórax. Siempre se hará punción diagnóstica que incluya idealmente: pH, lactato deshidrogenasa (LDH), glucosa, colesterol, tinción Gram, cultivo, adenosindeaminasa (ADA, se eleva en derrame pleural tuberculoso y en caso de empiema) y recuento celular diferencial (predominio PMN). El resultado aislado más relevante es el pH ya que bajo 7.2 indica drenaje. También se evalúa la existencia de tabicación, según signos radiológicos o con ecotomagrafia pleural que además permite orientar la toracocentesis en casos más complejos. El TAC de tórax permite evaluar anatomía para definir si requiere cirugía.

Aspectos Esenciales • • • • •

Complicado: por la necesidad de un drenaje pleural para su vaciamiento. Empiema: pus en la cavidad pleura. Sospecha de empiema por persistencia de mal estado general a pesar del tratamiento ATB. Diagnóstico: radiografía de tórax y toracocentesis. Tratamiento: ATB y drenaje.

Tratamiento •

Caso Clínico Tipo Mujer de 80 años, que presenta dolor en el hemitórax derecho, con tos seca, fiebre y náuseas. Al examen físico: disminución de la expansibilidad torácica en el hemitórax derecho, con abolición de las vibraciones vocales y el murmullo pulmonar en la base. Al tercer día de ATB persiste con fiebre y CEG y en la radiografía de tórax aumenta el derrame.



Definición Colección pleural asociada a neumonías, abscesos pleurales o bronquiectasias infectadas que requiere la colocación de un tubo de drenaje pleural.

DPPN complicado: antibióticos y el drenaje de la cavidad pleural mediante la instalación de un tubo de drenaje pleural. También se indica drenaje en la presencia de cultivos o Gram positivos, pus, mayores a 10 mm, loculados o derrames con engrosamiento de la pleura parietal. Empiema: antibióticos con cobertura para gérmenes aerobios y anaerobios y el drenaje de la cavidad pleural. A veces será suficiente la instalación de un tubo de drenaje, pero en otras situaciones deberá efectuarse una cirugía mediante videotoracoscopía o toracotomía con decorticación.

Seguimiento Se realiza según los resultados del análisis del líquido, pH, LDH y glucosa.

Etiología-Epidemiología-Fisiopatología El derrame paraneumónico (DPPN) se presenta en aproximadamente el 40% de las neumonías adquiridas en la comunidad, Staphylococcus aureus y Streptococcus pneumoniae se presentan en el 70% de los cultivos de gérmenes Gram positivos aeróbicos. Se denomina “complicado” al que requiere la colocación de un tubo pleural. La clasificación varía según el texto usado. El DPPN se puede clasificar en: derrame pleural paraneumónico no complicado (pH >7.2, LDH <1000, glucosa >40 mg/ dl), derrame pleural paraneumónico complicado (pH <7.2, LDH >1000 y glucosa <40 mg/dl) y empiema. Éste último es la acumulación de pus en la cavidad pleural. La clínica es la misma de la neumonía pero a veces se sospecha por la persistencia de la fiebre o deterioro clínico pese a tratamiento. También puede presentarse como un cuadro arrastrado con baja de peso, fiebre, leucocitosis y anemia. Su pronóstico es variables, con 15% de mortalidad y hasta un 40% de los pa-

Autor / Editor Marcelo Fres

8 Facultad de Medicina, Universidad de Chile

Año 2016

Derrame Pleural Paraneumónico Simple

terior y lateral y que es más alta en la zona axilar (curva de Damoisseau), además de silencio respiratorio. Estos signos se desplazan con los cambios de posición (no si existen adherencias pleurales). También puede auscultarse egofonía y un soplo espiratorio en el borde superior del derrame. Se confirma con radiografía de tórax (aparece con >200 ml) en que se ve ocupación de los ángulos costo-frénicos. A medida que el derrame aumenta por sobre los 300 ml, la opacidad adopta un límite superior cóncavo. Esta disposición puede alterarse por tabicamiento. La ecografía es más sensible para la pesquisa de derrames pequeños y sirve de guía para su punción. Un espesor de 10 mm en la radiografía en decúbito lateral es suficiente como para permitir una punción diagnóstica o toracocentesis. La TAC entrega información sobre el parénquima, la pleura y el espacio pleural, y está indicada en casos complejos. En la punción se debe hacer estudio citoquímico, recuento de GB y microbiológico con cultivo y gram.

Nivel de manejo del médico general: Diagnóstico Específico Tratamiento Completo Seguimiento Completo

Aspectos Esenciales • • • •

Causa más común de exudado. Presente en 40% neumonías bacterianas. Si es >10 mm en decúbito lateral, se realiza toracocentesis. El tratamiento es el de la neumonía.

El derrame paraneumónico tiene predominio de PMN. Para distinguir exudado y transudado se utilizan los criterios de Light: • Proteínas pleura / Proteínas sangre > 0,5 • LDH pleura / LDH sangre > 0,6 • LDH pleura > 2/3 del límite alto de normalidad en sangre

Caso Clínico Tipo Paciente de 50 años, presenta NAC lobar inferior derecha, en tratamiento con antibiótico empírico. Evoluciona de manera tórpida, con fiebre persistente, disnea y dolor pleural en hemitórax derecho. La radiografía de tórax muestra velamiento de mitad inferior del mismo hemitórax. La toracocentesis diagnóstica muestra líquido seroso, con características de exudado y pH 7,3.

Tratamiento La terapia es la misma que la usada en la neumonía de base (antibióticos).

Definición Seguimiento

Es la acumulación de líquido en el espacio pleural concomitante a una infección bacteriana pulmonar no tuberculosa (neumonía, bronquiectasias o absceso pulmonar).

Si hay glucosa en líquido pleural <40 mg/dl, LDH >1000 o pH <7,2, se sospecha empiema; si hay bacterias en el gram, cultivo (+) o pH <7, se diagnostica empiema y se debe colocar un tubo de drenaje. Si se sospecha tabicamiento de empiema, se realiza ecografía torácica o TAC para confirmar, con posterior instilación de fibrinolíticos por el tubo pleural, o VTC o toracotomía para debridar la cavidad pleural.

Etiología-Epidemiología-Fisiopatología El 40% de los pacientes con neumonía bacteriana tienen derrame, asociándose a mayor morbimortalidad. Es la causa más común de exudado pleural. Se produce por mayor permeabilidad capilar con salida de proteínas plasmáticas (>0.5 [plasma]) y existe elevación de la LDH (>2/3 límite superior normal o >0,6 [plasma]) por permeabilidad, y liberación local. Cuando el pH <7, tiene mucho riesgo de evolucionar a empiema.

Autor / Editor Marcelo Fres

Diagnóstico Se define como derrame paraneumónico no complicado al exudado con glucosa y pH normal, y cultivos negativos. Los síntomas más típicos son dolor torácico pleurítico, tos y disnea. Al examen físico se percibe si es >300 ml, percutiéndose matidez en las bases pulmonares, ángulo costofrénico pos-

9 Facultad de Medicina, Universidad de Chile

Año 2016

Derrame Pleural Por Tuberculosis

lóbulo inferior (sugerente de infección primaria). Otros estudios de imágenes (ecografía, TAC) pueden ser complementarios.

Nivel de manejo del médico general: Diagnóstico Específico Tratamiento Completo Seguimiento Completo

El líquido pleural es un exudado de color amarillo citrino con predominio mononuclear, clásicamente se describen células mesoteliales escasas (<5%). Presenta glucosa normal o baja, pH ácido (7,2-7,3). También se observa concentración de proteínas > 3 g/dL y LDH generalmente sobre 500 IU/L. Las concentraciones bajas de glucosa y pH son más características de un empiema tuberculoso crónico más que un derrame pleural reciente. ADA ayuda a diferenciarlo principalmente del derrame neoplásico, y tiene como puntos de corte tradicionales: >80 U/L (muy sugerente de TBC) y <30 U/L (muy improbable), para valores intermedios “correlacionar con la clínica”. El diagnóstico definitivo requiere aislar al bacilo con un cultivo de Koch del líquido (10-35% Sensibilidad), de la expectoración (40% sensibilidad) o de una muestra de tejido pleural (39-65% sensibilidad). La baciloscopía del líquido no se recomienda dado su mal rendimiento en el diagnóstico, pero mejora en pacientes VIH. El estudio histológico de la biopsia (con aguja de Cope) es sensible (hasta 78%) y específico (95%), se buscan granulomas caseificados.

Aspectos Esenciales • • • • •

Forma extrapulmonar más frecuente de TBC. Dolor pleurítico, tos seca, fiebre, polipnea y CEG. Derrame pleural unilateral en radiografía de tórax. Diagnóstico definitivo con cultivo de Koch y/o histología. Tratamiento TBC pleural por reactivación: pirazinamida + rifampicina + isoniazida. Pronóstico benigno.

Caso Clínico Tipo Paciente de 34 años, con antecedentes de TBC tratada hace 8 años, consulta por cuadro de 2 semanas de evolución, caracterizado por compromiso del estado general, fiebre hasta 38°C, y dolor pleurítico con tope inspiratorio. A la radiografía de tórax: velamiento base derecha, con curva de Damoiseau.

Tratamiento El tratamiento del derrame pleural por TBC es similar al tratamiento de la tuberculosis pulmonar. En pacientes sin tratamiento TBC previo, conbaciloscopía o cultivo positivos, deberá iniciarse tratamiento con esquema primario. Con baciloscopía y cultivo negativos deberá tratarse como TBC “de baja población bacilar”, con el esquema primario simplificado. En pacientes con VIH el tratamiento inicial es el mismo; debe derivarse a infectólogo, quien decidirá la duración del tratamiento y el seguimiento posterior.

Definición Ocupación de espacio pleural secundario a infección tuberculosa. Generalmente en período post-primario.

Etiología-Epidemiología-Fisiopatología Es la forma extrapulmonar de TBC más frecuente (10%). Si bien el curso es benigno y se resuelve espontáneamente en 3-4 meses, si no se da tratamiento entre el 40- 60% de los pacientes desarrollarán algún tipo de TBC en los 5 años siguientes.

Seguimiento La fiebre remite en las dos primeras semanas; el líquido se reabsorbe gradualmente, y en general no se realiza toracocentesis evacuadora, a menos que haya síntomas muy importantes. El uso de corticoides puede disminuir los síntomas pero no las secuelas fibróticas, y no se recomienda en pacientes VIH (+).

Diagnóstico El derrame pleural secundario a tuberculosis puede ocurrir tanto en la infección primaria como en una reactivación de la enfermedad. Puede tener una presentación aguda o sub-aguda, con dolor pleurítico, tos seca, fiebre, polipnea y CEG. Al examen físico presenta signos de derrame pleural unilateral. En la radiografía de tórax aparece derrame unilateral (90% del total) con curva de Damoiseau, de moderada cuantía, más frecuente a derecha. Ocasionalmente se puede observar patología parenquimatosa asociada al derrame, generalmente ubicada en el lóbulo superior (sugerente de una reactivación de la tuberculosis), y en algunos casos en el

Autor / Editor Marcelo Fres

10 Facultad de Medicina, Universidad de Chile

Año 2016

Enfermedad Pulmonar Obstructiva Crónica Avanzada

Diagnóstico Se sospecha en pacientes fumadores o con exposición a humo de biomasa o químicos, más historia de disnea progresiva persistente, que empeora con el ejercicio, tos y expectoración crónica con esputo (aumentan frente a esfuerzos e infecciones respiratorias), baja de peso y anorexia. Examen físico: taquipnea, taquicardia, hiperinsuflación torácica, uso de musculatura accesoria, respiración paradójica, espiración prolongada, murmullo pulmonar globalmente disminuido.

Nivel de manejo del médico general: Diagnóstico Específico Tratamiento Inicial Seguimiento Derivar

Aspectos Esenciales • • • •

Confirmación espirométrica: VEF1/CVF <70% post broncodilatador (confirma la LCFA).

Asociada fuertemente al hábito tabáquico. Historia de disnea y/o tos y/o expectoración crónica. Confirmación diagnóstica con espirometría. Tratamiento inhalatorio combinado broncodilatador acción corta + corticoides inhalados + broncodilatador de acción prolongada.

Caso Clínico Tipo Paciente varón de 62 años, fumador crónico IPA 25. Consulta por cuadro de disnea de bajo esfuerzo y tos crónica. Se realiza espirometría encontrándose: patrón obstructivo, VEF1/ CVF <70% y VEF1 40% pre y 45% postbroncodilator.

Diagnóstico de EPOC

(VEF1/CVF) post broncodilatador <70% + historia clínica.

EPOC avanzada (estadio III)

VEF1/CVF <70% y VEF1 entre 31% y 49% del teórico con o sin síntomas.

EPOC muy avanzada (estadio IV)

VEF1/CVF <70% y VEF1 ≤30%.

Tratamiento Suspensión del hábito tabáquico es lo más importante. Tratamiento farmacológico inhalatorio destinado a mejorar la sintomatología: broncodilatador de acción corta a demanda desde inicio + broncodilatador acción larga pautado desde estadio II. En estadio III añadir glucocorticoide inhalado sólo si presenta exacerbaciones frecuentes (>3-4/año). Se indica oxigenoterapia si presenta PaO2 ≤55 mmHg o SatO2 ≤88%. También se indica en la presencia de cor pulmonale, hipertensión pulmonar <55 o 55-60, hematocrito >55% o evidencia clínica de falla cardiaca derecha; la meta es mantener SaO2>90% en reposo, sueño y ejercicio. KNT para mejorar la mecánica respiratoria, y vacunación para influenza anual, y neumococo cada 5 años. Sólo suspensión de tabaco y O2 >15 hrs diarias mejora la sobrevida en EPOC.

Definición EPOC se define como una enfermedad prevenible y tratable, con limitación progresiva al flujo aéreo, no completamente reversible, asociada a una respuesta inflamatoria pulmonar anormal a partículas o gases nocivos.

Etiología-Epidemiología-Fisiopatología El principal factor de riesgo es la exposición al humo de tabaco (factor más importante, en directa relación al índice paquete/año considerándose significativo ≥10). La etiopatogenia se basa en la producción de reacción inflamatoria en los pulmones, derivada del humo del tabaco, sumado a otras partículas y gases inhalados, desarrollando destrucción tisular y remodelación de la vía aérea. En las etapas avanzadas destaca la aparición de hipoxemia, con posterior hipercapnia y el desarrollo de hipertensión pulmonar que evoluciona hacia cor pulmonale. Anatomopatológicamente, predominio de bronquitis crónica o enfisema pulmonar.

Seguimiento Derivar a especialista. Sospechar exacerbaciones frente a un acentuamiento de la disnea, tos y/o expectoración.

Es más frecuente en hombres, pero más letal en mujeres. El 15-20% de los fumadores desarrolla EPOC clínico. Las causas de muerte son: insuficiencia respiratoria, cáncer pulmonar (riesgo aumentado), cardiopatía congénita, exacerbaciones.

Autor / Editor Marcelo Fres

11 Facultad de Medicina, Universidad de Chile

Año 2016

Enfermedad Pulmonar Obstructiva Crónica Leve y Moderada

mo (90%), otras exposiciones también aumentan riesgo (ej., laborales). Diagnóstico diferencial: asma mal manejado, secuelas de TBC o bronquiectasias múltiples. Síntomas: tos productiva de predominio matinal, disnea de esfuerzo progresiva, fatiga. Examen físico: tórax en tonel, uso de musculatura accesoria, espiración con labios entrecerrados, posiciones que fijan cintura escapular, hipersonoridad, disminución de MP, espiración prolongada, roncus, sibilancias. En enfermedad avanzada caquexia y signos de cor pulmonale. Estudio básico: radiografía de tórax PA y lateral (atenuación del dibujo pulmonar y signos de hiperinsuflación) y espirometría diagnosticalimitación ventilatoria de tipo obstructivo (VEF/ CVF <0,7) que no revierte con broncodilatador (no mejora 12% VEF1 y 200 mL CVF).Enfermedad avanzada: poliglobulia, hipoxemia crónica.

Nivel de manejo del médico general: Diagnóstico Específico Tratamiento Completo Seguimiento Completo

Aspectos Esenciales • • •

Diagnóstico: clínica (disnea) + espirometría (VEF1/CVF <70%, que no mejora con BD). Tabaquismo principal factor de riesgo. Aumento sobrevida: oxigenoterapia (PO2<55 o 55-60 mmHg con poliglobulia, cor pulmonar o HTTP) y suspender tabaquismo.

La guía GOLD recomienda realizar espirometría en pacientes con clínica y/o FR o antecedentes familiares en pacientes mayores de 40 años. Se debe determinar la severidad, el impacto en la calidad de vida y los riesgos de exacerbaciones, hospitalización o muerte. Se evalúa: • Valoración de los síntomas: escala CAT o MMRC (para evaluar disnea). • Severidad de LCFA: con VEF1 post broncodilatador. • Evaluar riesgo de exacerbaciones, se definen recurrentes cuando son 2 o más en un año. El riesgo de exacerbación aumenta con antecedentes de episodios previos de exacerbaciones y con el empeoramiento de la LCFA. • Comorbilidad: cardiovascular, osteosporosis, síndrome metabólico, depresión y trastornos ansiosos, disfunción muscular y cáncer de pulmón. Todas las comorbilidades influyen en la mortalidad y riesgo de hospitalización.

Caso Clínico Tipo Paciente varón de 50 años, antecedente de tabaquismo crónico activo IPA 18. Consulta por historia de disnea progresiva, que empeora con ejercicio, tos crónica y expectoración serosa matutina. Al examen físico: MP disminuido, SRA. En la radiografía de tórax: aumento de espacios intercostales, diafragma aplanado, sin imágenes condensantes. Espirometría: limitación ventilatoria obstructiva.

Definición Enfermedad prevenible y tratable, con afectación sistémica extrapulmonar y cuyo componente pulmonar se caracteriza por una limitación al flujo de aire que no es completamente reversible, generalmente progresiva y se asocia a una respuesta inflamatoria pulmonar anómala a partículas o gases nocivos (GOLD).

Tratamiento El tratamiento se basa en 4 pilares: evaluación y supervisión de la enfermedad, reducción de los factores de riesgo, tratamiento de la EPOC estable y tratamiento de las exacerbaciones. 1. Cese del tabaquismo, mejora sobrevida. Evitar exposiciones a contaminantes laborales y/o ambientales. 2. Programas de ejercicio físico y/o rehabilitación pulmonar. 3. Vacunación anual contra influenza para todos, vacunación con “neumo23” en pacientes >65 años o con enfermedad avanzada. 4. Fármacos no detienen progresión de la enfermedad sólo alivio sintomático y reducción de las exacerbaciones, mejorando con ello la calidad de vida. Se usan BD de acción corta según necesidad (salbutamol, ipratropio) y BD de acción larga (tiotropio, salmeterol, formoterol), de mantención. Se propone en las guías el manejo escalonado. Puede usarse Teofilina, en formulaciones de liberación retardada, pero hay riesgo de toxicidad. Los anticolinérgicos son los BD de elección cuando los síntomas son continuos. Los corticosteroides inhalados constituyen el eje de terapia antiinflamatoria en los pacientes

Etiología-Epidemiología-Fisiopatología Se debe a una reacción inflamatoria de la vía aérea, secundaria a inhalación de humo de tabaco (altera motilidad ciliar, inhibe función de macrófagos, produce hiperplasia e hipertrofia glandular y contracción musculatura lisa). Agrupa 3 trastornos: bronquitis crónica, enfisema pulmonar y enfermedad de vía aérea pequeña. Prevalencia 9%, más frecuente en hombres, mortalidad mayor en mujeres. Existen casos asociados a exposición a leña en ambientes cerrados o laborales.

Diagnóstico Debe sospecharse en pacientes con antecedente tabaquis-

12 Facultad de Medicina, Universidad de Chile

con exacerbaciones recurrentes, se indican en pacientes con VEF1 menor del 60% del predicho, no debe usarse en monoterapia. Recordar que aumentan la sobrevida el cese del tabaco y el oxígeno en casos indicados y usado más de 15 hrs diarias.

Seguimiento Evolución (hospitalizaciones pasadas, comorbilidades, antecedentes familiares, medicamentos actuales, limitación de AVS), examen físico y evaluación de red de apoyo. En Etapas II, III y IV se debe realizar exámenes complementarios (radiografía de tórax, GSA, prueba de reversibilidad con BD y screening de α-1 antitripsina). Se puede intervenir en la rapidez de la progresión adhiriendo a terapia óptima y disminuyendo las exacerbaciones.

Autor / Editor Marcelo Fres

Año 2016

13 Facultad de Medicina, Universidad de Chile

Hipertensión Pulmonar

12. Alt. metabólicas: tr. tiroideos. 13. Otros: obstrucción tumoral, mediastinitis fibrosante. 14. En los casos de HAP 2ria a cardiopatía izquierda, cursan con HAP postcapilar(presión de enclavamiento elevada, >15 mmHg).Los otros grupos etiológicos cursan con HAP precapilar (presión de enclavamiento: normal (4-12 mmHg).

Nivel de manejo del médico general: Diagnóstico Específico Tratamiento Inicial Seguimiento Derivar

Fisiopatología: Alteraciones en la pared endotelial de los vasos pulmonares (remodelación vascular): • Fibrosis de la íntima. • Hipertrofia de la media y estrechez luminal. • Trombosis intraluminal. La vasoconstricción, el remodelado de la pared vascular y trombosis in situ producen un aumento de la resistencia vascular pulmonar --> sobrecarga de presión del ventrículo derecho. La PºAP aumenta con un GC que inicialmente se mantiene, pero con el tiempo disminuye (falla diastólica del ventrículo Dº). Lo anterior lleva finalmente a cor pulmonale.

Aspectos Esenciales • • • •

Enfermedad infrecuente. Diagnóstico definitivo con cateterismo cardiaco. Todos se benefician con anticoagulantes. El tratamiento definitivo es transplante pulmonar.

Caso Clínico Tipo Paciente 38 años con asma severo, que relata historia de dos años de evolución caracterizada por hemoptisis, disnea progresiva, síncopes y edema de EEII, se realiza radiografía de tórax que muestra aumento de diámetro en trama vascular arterial pulmonar y cor pulmonale.

Diagnóstico a) Clínica • Disnea progresiva (lo más frecuente): su gravedad no se relaciona con la elevación de la presión de AP. • Fatiga y debilidad: por deterioro del gasto cardíaco. • Dolor torácico subesternal: por Insuficiencia coronaria ante el aumento de las necesidades del ventrículo derecho + hipoxemia. • Síncope: por disminución del gasto cardíaco. • Signos de insuficiencia cardíaca derecha (edema, ingurgitación yugular, hepatomegalia, etc). • Soplo de insuficiencia tricuspidea, pulmonar, segundo ruido reforzado.

Definición La hipertensión arterial pulmonar (HAP) se define como la elevación de la presión de la arteria pulmonar media ≥25 mmHg en reposo o ≥30 mmHg en ejercicio.

Etiología-Epidemiología-Fisiopatología Es una enfermedad poco frecuente, puede aparecer en cualquier edad.

b) Exámenes: • Radiografía de tórax: protrusión de AP principal y aumento del ancho de la rama descendente de la AP derecha, oligohemia periférica y cardiomegalia. • ECG: en fases avanzadas hipertrofia auricular y ventricular derecha, trastornos de conducción de rama derecha. • Ecocardiograma útil para estimar la hipertensión pulmonar, descartar causas 2rias de HTP y evaluar la anatomía y funcionamiento cardíaco. • Estudio funcional: espirometría presenta patrón restrictivo leve. La DLCO2 suele disminuir PaO2 disminuida y PaCO2 puede estar disminuida. Test de Marcha 6 minutos: existe correlación entre la distancia recorrida y la gravedad de la enfermedad, es útil para monitorizar la respuesta al tratamiento. • Estudio hemodinámico con cateterismo cardíaco: diagnóstico definitivo. Sirve para descartar causas secundarias y para demostrar aumento de la Pº de AP y aurícula derecha. • Estudio etiológico: dirigido según la sospecha: estudio inmunohistoquímico, pruebas tiroideas, etc.

Etiología: 1. HAP idiopática. 2. HAP hereditaria. 3. HAP secundaria a fármacos y toxinas. 4. HAP asociada a enfermedades del tejido conectivo, infección por VIH, hipertensión portal, enfermedad cardíaca congénita, esquistosomiasis, anemia hemolítica crónica. 5. HAP persistente en el recién nacido 6. HAP 2ria a cardiopatía izquierda: Disfunción sistólica, disfunción diastólica, enfermedad valvular. 7. HAP 2ria a enfermedades pulmonares e hipoxemia: EPOC, enfermedad pulmonar insterticial, hipoventilación alveolar crónica, SAHOS. 8. HAP 2ria a enfermedad tromboembólica 9. HAP 2ria a mecanismos poco claros o multifactoriales: 10. Tr. hematológicos (esplenectomía, sd. mieloproliferativos). 11. Enfermedades sistémicas: sarcoidosis, vasculitis.

14 Facultad de Medicina, Universidad de Chile



En el caso de HAP tromboembólica crónica se sospecha con angioTAC o cintigrafía pulmonar y diagnóstico definitivo con arteriografía pulmonar.

c) Diagnóstico el diagnóstico definitivo se realiza con el estudio hemodinámico (cateterismo cardíaco). Test de vasorreactividad: • Se realiza con una sustancia vasodilatadora: epoprostenol, adenosina, etc. • Tiene implicancias pronósticas y terapéuticas. • Positivo (20%): disminución de la PºAP media de, al menos, 10 mmHg con una presión final <40 mmHg, sin disminución del GC. Sobrevida a 5 años: 95%. • Negativo (80%)

Tratamiento •

Es una enfermedad progresiva, por lo que no existe tratamiento curativo. • Si el test de vasorreactividad resulta positivo la sobrevida a 5 años es de 95%,. • La muerte suele ser por falla cardíaca derecha y, a veces, muerte súbita. • Sus pilares son tratar la enfermedad de base, reducir síntomas, evitar aparición de coágulos y oxigenoterapia para reducir la carga de trabajo cardiaca. Tratamiento no farmacológico: Actividad física y rehabilitación supervisada, evitar embarazo. Tratamiento farmacológico: • Vasodilatadores, si la prueba de vasorreactividad es positiva son útiles lo calcioantagonistas. Si la prueba es negativa son útiles otros fármacos como: análogos de prostaciclina (epoprostenol iv), antagonistas de receptores de endotelina (bosetán), inhibidores de la fosfodiesterasa 5 (sildenafil). • Anticoagulantes orales en todos los pacientes. • Transplante pulmonar en fracaso del tratamiento farmacológico en pacientes seleccionados. Es el tratamiento definitivo. Otras medidas: • Oxígeno para hipoxémicos. • Diuréticos en caso de sobrecarga hídrica. • En HAP tromboembólica crónica el tratamiento es la tromboendarterectomía. • Evitar: gran esfuerzo físico, en mujeres en edad fértil evitar embarazo y el uso de ACO.

Seguimiento Derivar a especialista.

Autor / Editor Marcelo Fres

Año 2016

15 Facultad de Medicina, Universidad de Chile

Neumonías Adquiridas en la Comunidad

tabla a continuación).

(Tipo 4 de la Sociedad Chilena de Enfermedades Respiratorias)

CRITERIOS ATS

Criterios mayores (presencia de 1 criterio ingreso directo a UCI): • Necesidad de ventilación mecánica • Presencia de shock séptico

Nivel de manejo del médico general: Diagnóstico Específico Tratamiento Inicial Seguimiento Derivar

Aspectos Esenciales • • • •

CRITERIOS SOCIEDAD BRITÁNICA DEL TÓRAX (CURB 65) (presencia de ≥ 3 criterios)

Criterios menores (presencia de ≥ 3 criterios hospitalizar directo en UCI): • FR sobre 30. • PaO2 / FiO2 (PaFi) <250 mmHg. • Infiltrados multilobares a la radiografía de tórax. • Confusión/desorientación. • BUN mayor o igual a 20 mg/dL. • Leucopenia bajo 4000 leuc/cc. • Trombocitopenia menor a 100.000. • Hipotermia menor a 36°C. • Hipotensión que requiere fluidos.

Neumonía grave, requiere tratamiento en UCI. Diagnóstico al cumplir criterios CURB- 65 o ATS. Agente más frecuente: S. pneumoniae. Tratamiento antibiótico: EV combinado.

Caso Clínico Tipo Paciente 83 años, ingresa a SU con compromiso de conciencia polipneico (35 resp/min) e hipotenso (96/40 mmHg). Familiares relatan historia previa de resfrío complicado, con fiebre. La radiografía de tórax revela pequeño derrame.

Definición Paciente con neumonía adquirida en la comunidad (NAC) grave que requiere apoyo ventilatorio mecánico y/o hemodinámico en UCI.

Etiología-Epidemiología-Fisiopatología

• • • •



Confusión mental de reciente aparición Nitrógeno Ureico > 20 mg/dl Frecuencia Respiratoria >30 ciclos/min Presión diastólica < 60 mmHg o sistólica <90 mmHg. (Bloody pressure) Edad ≥ 65 años

Tratamiento

1. Epidemiología: 10-30% de los pacientes hospitalizados por NAC. Mortalidad 20-50%. 2. Etiología: Los patógenos aislados con mayor frecuencia son: Streptococco pneumoniae (más frecuente), bacilos gran negativos entéricos,Legionella sp, S. aureus, H. influenzae, anaerobios estrictos, C. pneumoniae, M. pneumoniae y virus respiratorios.

El tratamiento ATB e.v. debe iniciarse dentro de las primeras 4 horas, pues su retraso aumento la morbimortalidad. En el caso de NAC graves debe además realizarse estudio microbiológico para conocer el agente causal (hemocultivo, secreciones respiratorias, líquido pleural, estudio serológico, antígenos urinarios) y determinar resistencias. Las bronquiectasias, desnutrición y uso de esteroides son FR para Pseudomona Aeruginosa, debiendo asegurarse su cobertura en estos casos durante el tratamiento empírico; eventualmente además asegurar cobertura de virus influenza en caso de existir condiciones epidemiológicas para su sospecha.

Diagnóstico La decisión de hospitalizar o no a un paciente se basa en el juicio clínico y en scores como el CURB-65 de la Soc. británica del tórax, o el PSI/PORT score (algoritmo de EEUU que usa aprox. 20 factores, con resultados similares a CURB-65). Una vez que se decide hospitalizar debe decidirse si hacerlo en sala común o en UCI. En general pacientes con 2 o 3 criterios de CURB-65 pueden ir a sala común, 4 o más en UCI; o bien pueden usarse los criterios de la ATS (American Thoracic Society, 2007) para decidir el lugar de hospitalización (ver

Esquemas ATB empíricos de prímera línea (duración: 10–14 días): 1. Ceftriaxona 2 g EV c/día, o 2. Cefotaxima 1 – 2 g EV c/8 h, MÁS Eritromicina 500 mg EV c/8 h, o 3. Levofloxacino 500 – 1.000 mg EV c/día, o Moxifloxacino

16 Facultad de Medicina, Universidad de Chile

400 mg EV c/día. Tratamiento alternativo (segunda línea) 10-14 días: • Amoxicilina + ácido clávulánico 1.000 – 1.200 mg EV c/8 h, • Amoxicilina + Sulbactam 1.000/500 mg EV c/8 h, o • Ampicilina + Sulbactam 1.000/500 mg EV c/8 h, más Eritromicina 500 mg EV c/8 h.

Seguimiento Por su complejidad es necesario derivar a especialista.

Autor / Editor Paulina Lira

Año 2016

17 Facultad de Medicina, Universidad de Chile

Neumonía Adquirida En La Comunidad

Fisiopatología: Los principales mecanismos causantes son: • A. Microaspiración de contenido orofaríngeo: Cierto grado es normal en individuos sanos, pero se favorece por fármacos que deprimen la tos como el alcohol, benzodiacepinas y neurolépticos; además aumenta con el envejecimiento. Se produce una infección por microorganismos que colonizan la cavidad orofaríngea, entre ellos el Streptococcus pneumoniae, Haemophyllus influenza y bacilos Gram negativos entéricos. La presencia de comorbilidades, el tabaquismo y la desnutrición favorecen la colonización por estos agentes. • B. Inhalación: Ingresan microorganismos intracelulares como virus y bacterias atípicas. El tamaño del inóculo es pequeño y su progresión depende de la indemnidad de la inmunidad celular, sobretodo de los macrófagos alveolares; de ahí que los pacientes con tratamiento corticoesteroidal sean más susceptibles. Dado su propagación aérea, estos gérmenes pueden desencadenar brotes epidémicos.

(Tipo 1, 2 Y 3 De La Sociedad Chilena De Enfermedades Respiratorias) Nivel de manejo del médico general: Diagnóstico Específico Tratamiento Completo Seguimiento Completo

Aspectos Esenciales • • • •

Síntomas y signos tienen sensibilidad y especificidad moderada (50-70%). La radiografía de tórax es el examen de elección. Principal agente causal es Neumococo, en cualquier tipo de paciente. Tratamiento según clasificación. NAC que no responde a tratamiento a las 72 hrs: tomar radiografía de tórax en busca de complicación.

Etiología: la principal causa de NAC es el Streptococco pneumoniae, con un 20 a 30%, seguido por los gérmenes atípicos (Chlamidia pneumoniae, Mycoplasma pneumoniae) y virus. En los pacientes EPOC aumenta la frecuencia de Hemophyllus influenza. El Staphylococcus aureus es un agente poco frecuente de neumonía que aumenta tras las pandemias de influenza.

Caso Clínico Tipo Paciente masculino de 72 años, antecedentes de tabaquismo crónico (IPA=40). Consulta en SU por cuadro de dos días de evolución caracterizado por fiebre de 39°C axilar, calofríos, tos con desgarro mucopurulento escaso y dolor pleurítico en hemitórax derecho. Al examen destaca: T° 38.5°C, 110 lpm, PA: 130/70 mmHg, FR 28 rpm, pulsioximetría de 91% sin O2 suplementario. Paciente vigil, orientado, respiración superficial por tope inspiratorio, murmullo pulmonar disminuido con roncus y sibilancias bibasales, dudosas crepitaciones en sector axilar derecho. Radiografía de tórax: condensación en lóbulo superior derecho.

Diagnóstico El diagnóstico es clínico-radiológico, a saber, cuadro clínico sugerente con imagen compatible. Sin embargo NO debe retrasarse el tratamiento ATB en espera de imágenes si el cuadro clínico es sugerente, ya que un retraso de más de 4 horas en el inicio del tratamiento antibiótico puede repercutir en una mayor mortalidad. 1. Clínica: Curso agudo, con CEG, fiebre y calofríos, tos, expectoración, disnea y dolor torácico tipo puntada. 2. Al examen físico: Fiebre, taquicardia, taquipnea y signos focales de condensación pulmonar. 3. Radiografía de tórax: es el examen de elección, donde se muestran alteraciones recientes, entre ella la presencia de condensación pulmonar, sombra no homogénea de bordes poco nítidos con broncograma aéreo, su tamaño y densidad dependerá del lóbulo afectado. La radiografía permite determinar la extensión y detectar complicaciones, como excavación y derrame pleural. 4. Estudio microbiológico: no se logra en más del 50% de las NAC y se justifica sólo en pacientes hospitalizados (graves) y ante fracaso del tratamiento ATB, las técnicas disponibles son: Gram y cultivo de esputo, hemocultivos, cultivo de líquido pleural (todo derrame mayor de 1 cm debe puncionarse y cultivar, junto con estudio citoquímico y pH), serología (anticuerpos IgM o IgG contra virus o bacterias atípicas), antigenuria (para antígeno de Pneumococco o legionella serotipo 1). El inicio del tratamiento no debe estar supeditado al estudio etiológico.

Definición La NAC es un proceso inflamatorio pulmonar de origen infeccioso adquirido en el medio comunitario, se incluyen a pacientes de casas de reposo y similares que se presentan con cuadro clínico compatible dentro de las primeras dos semanas de internación (MINSAL, 2011).

Etiología-Epidemiología-Fisiopatología Epidemiología: Constituye la tercera causa de mortalidad en nuestro medio y es particularmente grave en pacientes mayores de 60 años. Se puede presentar a cualquiera edad, es más frecuente en fumadores y ancianos. Aumenta la gravedad con la comorbilidad asociada y según la presentación clínica.

18 Facultad de Medicina, Universidad de Chile

Tratamiento Aproximadamente el 20% de los pacientes con NAC deben ser hospitalizados. Criterios para para definir la necesidad de hospitalización: 1. Recomendación elaborada por la sociedad Británica de Tórax (CURB-65): • Compromiso de conciencia • Urea sanguínea (BUN) > 20 mg/dl • FR > 30/min • Blood pressure PAS < 90 mmHg o PAD < 60 mmHg • Edad ≥ 65 años

2. La Sociedad chilena de enfermedades respiratorias define criterios de gravedad similares (consenso 2005), agregando además la presencia de comorbilidad, FC> 120 lpm, una FR de 20/min (y no de 30/min como la soc. británica), radiografía de tórax con NAC multilobar, derrame pleural o cavitación, SaO2<90% con FiO2 ambiental, agregando además factores sociales y problemas de adherencia a tratamiento. Si el paciente no cumple ninguno de los criterios puede ser manejado ambulatoriamente; si cumple 2 o más se maneja hospitalizado; si sólo cumple uno cae dentro del juicio del clínico.

En presencia de 2 o más criterios pensar en hospitalizar; 3 hospitalización urgente, 4 pensar en UCI.

Clasificación de las NAC y tratamiento (Sociedad chilena de enfermedades respiratorias, 2005)

NAC tipo I

NAC tipo II

NAC tipo III

Definición

< 65 años, s/comorbilidad ni criterios de gravedad.

>65 años y/o c/comorbilidad, s/ criterios de gravedad.

Cualquier edad, c/criterios de gravedad.

Manejo

Ambulatorio

Ambulatorio

Hospitalario

ATB 7-10 días

Amoxicilina 1 g oral c/8 hrs.

Amoxi-clavulánico 500/125 mg c/8 hrs ó 875/125 mg c/12 hrs o cefuroxima 500 mg c/12 hrs, todas por VO.

Ceftriaxona 1-2 g IV/día o Cefotaxima 1 g IV c/8 hrs. Ante fracaso de β lactámicos o sospecha o confirmación de M. pneumoniae o Legionella spp, agregar eritromicina, claritromicina, azitromicina.

Alternativa

Eritromicina, claritromicina, Eritromicina, claritromicina, azitromicina. azitromicina.

Seguimiento

Autor / Editor Nevelline Salgado

La fiebre suele caer entre las 48 y 72 horas, junto a normalización progresiva de los parámetros inflamatorios y signos vitales. En los pacientes hospitalizados con buena respuesta es posible cambiar a tratamiento oral entre el 3º y 5º día de hospitalización (amoxi-clavulánico, cefuroxima, claritromicina o levofloxacina). Los infiltrados pulmonares en la radiografía de tórax desaparecen en 50% de los casos a las 2 semanas y 73% a las 6 semanas. La resolución es más lenta en los enfermos más graves, en general. Es importante recordar realizar una radiografía de tórax de control al mes para asegurarse que no exista proceso neoplásico u otras lesiones que se hallaban escondidas por la neumonía. Falta de respuesta Se define por ausencia mejoría a las 72 horas de tratamiento. Puede deberse a enfermedades asociadas, uso de ATB inapropiados, presencia de patógenos resistentes o no cubiertos por el esquema inicial, complicaciones de la neumonía (empiema, absceso, SDRA, infección extrapulmonar). Se debe consultar a especialistas o derivar casos muy graves, con derrame pleural, excavación y falta de respuesta a ATB.

19 Facultad de Medicina, Universidad de Chile

Amoxi-clavulánico, amoxi-sulbactam, ampicilina-sulbactam.

Año 2016

Neumonía en Inmunosuprimidos

1. Escenario de VIH-inmunosuprimido (Global): A exámenes de NAC según gravedad, agregar búsqueda de P. jiroveci según disponibilidad del centro y muestra obtenida. 2. Escenario de Neutropenia febril: Agregar búsqueda de hongos, galactomanano en sangre.El cuadro clínico es poco orientador, es por esto que parece ser más justificado el uso de técnicas diagnósticas invasivas. Hay factores que pueden orientar la etiología, tales como el tipo de inmunosupresión y lugar geográfico, las características radiológicas. Se debe realizar una radiografía de tórax y/o un TAC idealmente apenas sea posible. También son de utilidad la fibrobroncoscopía y el lavado broquio-alveolar, que tiene buenos resultados en el diagnóstico de P. jirovecii en pacientes con SIDA, evaluar CMV con lo disponible en el centro. Solicitar VIH ante la sospecha. 3. Escenario de VIH: Es necesario considerar el recuento de CD4+ para una aproximación a los posibles agentes etiológicos involucrados. En términos generales, debe buscarse dirigidamente TBC (independiente del recuento de CD4+) con baciloscopía, hemocultivos, PPD y otras técnicas como T-SPOT o quiantiFERON.

Nivel de manejo del médico general: Diagnóstico Sospecha Tratamiento Inicial Seguimiento Derivar

Aspectos Esenciales • • • •

Los inmunosuprimidos son más susceptibles a patógenos atípicos, hongos y a las bacterias frecuentes. El tipo de inmunosupresión puede orientar a la causa etiológica. Se debe indicar profilaxis con Cotrimoxazol forte en paciente con recuento de CD4 < 200 o en usuario de prednisona > 20 mg/d. La mayoría de las veces se deben emplear exámenes invasivos de diagnóstico.

En caso de CD4+< 200 se debe descartar la presencia de Pneumocystis jirovecii con tinción de Gomoris-Grocket, IFI o PCR en esputo; LDH en sangre (teniendo > 1.000 un VPP alto), y estudio radiográfico. En caso de CD4+ < 50 se debe descartar agentes del complejo micobacterium avium y CMV. En cuanto a hongos, y basándose en la epidemiología o viajes del paciente, estudiar la presencia de criptococosis o histoplasmosis. 4. Escenario trasplantado: Se debe descargar pneumocistis, CMV y hongos, principalmente mucormicosis (90%), aspergilosis y fusarium, especialmente en caso de patología hemato-oncológica.

Caso Clínico Tipo Paciente trasplantado renal el 2011 que consulta por cuadro de 1 mes de evolución caracterizado por astenia, tos no productiva y disnea agregada hace 2 semanas. La Rx Tx no es categórica y el TAC muestra patrón de vidrio esmerilado en la base del pulmón derecho.

Definición Infección del parénquima pulmonar ocurrida en un paciente inmunosuprimido (en esquemas de quimioterapia, trasplantado, con enfermedades autoinmunes o SIDA).

Tratamiento

Debiese ser etiológico, inicialmente esquema según severidad de NAC habitual al que se agrega cotrimoxazol forte cada 8h iv/vo según gravedad. • En neumonía por P. jiroveci con falla respiratoria agregar corticoides. • En neutropenia febril prolongada, de alto riesgo o con galactomanano positivo o imágenes sospechosas de aspergilosis pulmonar agregar antifúngicos. • Es importante dar profilaxis a los pacientes en riesgo, con uso de Cotrimoxazol forte 1tab/día a pacientes tratados con Prednisona > 20mg/día o con CD4+ < 200 cs/mm3.

Etiología-Epidemiología-Fisiopatología Etiología: Puede ser por patógenos primarios que causan enfermedad en individuos inmunocompetentes (S. aureus, S. pneumoniae, M tuberculosis) o por patógenos oportunistas (pseudomonas aeruginosa, pneumocysistis, micobacterias atipicas). Bacterias: S. pneumoniae, P. aeruginosa, S. aureus, M. tuberculosis y no TBC Hongos: P. jiroveci, Aspergillus Virus: CMV, Influenza Según tiempo de evolución, podemos sospechar diferentes etiologías: Aguda (< 5 días), subaguda (5 - 15 días) y crónica (> 15 días).

Seguimiento Realizado por especialista.

Diagnóstico Autor / Editor Nevelline Salgado

Microorganismos variados según condición:

20 Facultad de Medicina, Universidad de Chile

Año 2016

Neumonía Nosocomial

Etiología: Agentes más frecuentes en neumonías nosocomiales son: Bacilos gram (-) 65%: Klebsiella pneumoniae (gran frecuencia), así como algunos gérmenes resistentes a la mayoría de los antibióticos, tales como Pseudomonas aeruginosa y Acinetobacter calcoaceticus. Cualquier otra bacteria gramnegativa puede causar NAH. Cocáceas gram (+) constituyen el 25% siendo principalmente estafilococo aureus o estreptococo pneumonie. Los hongos constituyen el 5% siendo candida sp y aspergilus sp los más relevantes.

Nivel de manejo del médico general: Diagnóstico Inicial Tratamiento Inicial Seguimiento Por especialista

Aspectos Esenciales • • • • • • • •

Diagnóstico

Neumonía que debuta después de 48 horas de hospitalizado. La NIH es la segunda causa de infección nosocomial y la más frecuente en UCI. No se incluyen en este grupo las NIH en el paciente inmunosuprimido. Es la 1° causa de muerte por infección nosocomial. Mortalidad entre 5,5 a 35%. Prolonga la hospitalización en 7 a 9 días en promedio. El diagnóstico es clínico, radiológico y microbiológico. El tratamiento debe ser precoz y empírico.

Se debe tener la sospecha frente a un Infiltrado radiológico progresivo o persistente en presencia de: • Secreciones purulentas • Fiebre/hipotermia • Leucocitosis/leucopenia • Obtener cultivos de vía aérea, hemocultivos, punción de derrame pleural y Rx de tórax PA y Lateral.

Tratamiento Entre los antibióticos para la Neumonía adquirida en el hospital se pueden incluir Vancomicina, tercera y cuarta generación de cefalosporinas, carbapenémicos, fluoroquinolonas y aminoglucósidos. Estos antibióticos se suelen administrar por vía intravenosa. La elección de antibióticos varía de un hospital a otro, debido a las diferencias regionales en los microorganismos más probables, y debido a las diferencias en la capacidad de los microorganismos a resistir a diversos tratamientos antibióticos.

Caso Clínico Tipo Paciente hospitalizado en UCI y asistido con ventilación mecánica que presenta fiebre, calosfríos y leucocitosis, asociada a presencia de infiltrados pulmonares en la radiografía de tórax.

Definición Neumonía que se presenta 48 horas posteriores al ingreso hospitalario y hasta 7 días posteriores al alta del paciente. Pueden ser subdivididas en: a) Precoces: ocurren en los primeros 4 días de hospitalizado, son causada por gérmenes comunitarios y en general de buen pronóstico. b) Tardías: ocurren posterior al 4to día de hospitalizado, tienen mayor asociación con gérmenes nosocomiales y son de mayor mortalidad.

Seguimiento Por especialista.

Autor / Editor Hernán Rubilar

Etiología-Epidemiología-Fisiopatología Epidemiología: En Chile constituye la segunda infección intrahospitalaria más frecuente, correspondiendo a una importante causa de muerte. Fisiopatología: Las neumonías se producen por la deficiencia de los mecanismos defensivos para eliminar los microorganismos que llegan al territorio alveolar, generalmente por aspiración de secreciones de las vías aéreas altas colonizadas por agentes patógenos.

21 Facultad de Medicina, Universidad de Chile

Año 2016

Neumotórax Moderado y Grave

Diagnóstico La clínica es de inicio súbito, caracterizada por dolor pleurítico, disnea y taquicardia. Puede haber asimetría de tórax, hipersonoridad, disminución o ausencia de vibraciones vocales y del murmullo pulmonar. En caso de neumotórax secundario los síntomas suelen ser más graves y pueden presentar hipoxemia e hipotensión, los que se suman a los síntomas y signos de la patología de base. Estudio: La radiografía de tórax muestra aire en el espacio subpleural y permite estimar su cuantía. Aproximadamente en un 15% corresponde a una separación de 2 a 3 cm. entre la pleura visceral (visible) y la pared torácica. La TAC de tórax muestra mejor el colapso pulmonar, permite diagnosticar otras bulas y enfermedades subyacentes.

Nivel de manejo del médico general: Diagnóstico Específico Tratamiento Inicial Seguimiento Derivar

Aspectos Esenciales • • •

El tabaco es un gran factor de riesgo. Aquellos con neumotórax secundario suelen tener más complicaciones respiratorias. El tratamiento se basa en el drenaje pleural. En ciertos casos de riesgo y recidivas se debe optar por la pleurodesis y cirugía.

Tratamiento En el manejo agudo se debe evaluar la presencia de compromiso hemodinámico: • Si no hay compromiso hemodinámico: El neumotórax es pequeño (< 2cm) puede mantenerse con observación y oxigeno suplementario. Si el paciente se encuentra estable y posee un neumotórax de > 3cm debe efectuarse una punción aspiración con aguja fina. En caso de que exista concomitantemente un hemotorax, se debe instalar tubo de drenaje pleural. • Si existe inestabilidad hemodinámica: Corrección de parámetros hemodinámicos y debe instalarse un tubo de drenaje pleural hasta que el pulmón se expanda. • La pleurodesis se indica en aquellos pacientes con un segundo episodio de neumotórax, en quienes realizan actividades de riesgo como buzos y pilotos; y en los neumotórax espontáneos secundarios (mayor mortalidad y recidiva), además de evaluar la cirugía con resección del parénquima dañado.

Caso Clínico Tipo Paciente que llega al Servicio de Urgencias por un cuadro súbito de disnea, taquicardia, dolor en hemitórax izquierdo y apremio respiratorio. La familia refiere que tiene diagnóstico de EPOC desde hace 4 años. Se toma una radiografía de tórax, la que muestra aire subpleural con clara visualización de la pleura visceral.

Definición Presencia de aire en el espacio pleural con un consiguiente colapso pulmonar. Se considera Moderado si compromete del 15% al 35% del parénquima, y Grande si es mayor al 35%. Se puede clasificar en espontáneo (primario si ocurre en un pulmón sano, secundario si ocurre en un pulmón con patologías previas), traumático (abierto o cerrado) o iatrogénico.

Seguimiento

Etiología-Epidemiología-Fisiopatología •



• •

Por especialista.

El neumotórax espontáneo primario típicamente ocurre en hombres jóvenes, altos y delgados, siendo el tabaquismo el mayor factor de riesgo, dado que puede causar bulas subpleurales y cambios enfisematosos en los lóbulos superiores. El neumotórax espontáneo secundario ocurre generalmente en hombres sobre los 60 años con antecedentes de EPOC, Pneumocistis jirovecii o infección por VIH, los que poseen áreas necrosadas de tejido pulmonar. El neumotórax traumático es el que se debe a un trauma directo o indirecto, penetrante o no del tórax. El neumotórax iatrogénico es el que se produce como resultado no intencional de una maniobra diagnóstica o terapéutica.

Autor / Editor Hernán Rubilar

22 Facultad de Medicina, Universidad de Chile

Año 2016

Crisis de Asma Bronquial

(PEF o FEV1).

Etiología-Epidemiología-Fisiopatología

Nivel de manejo del médico general: Diagnóstico Específico Tratamiento Completo Seguimiento Completo

Las exacerbaciones de instauración progresiva, es decir, días (más del 80% de las que acuden a Urgencias) se deben a infecciones respiratorias altas o mal control de la enfermedad por mala adhesión terapéutica (El mecanismo de éstas principalmente es por inflamación). En cambio las de instauración rápida (horas), se deben a tabaquismo, alérgenos inhalados, fármacos (AINES o fármacos b-bloqueantes), alimentos, o estrés emocional (El mecanismo en este caso es más de broncoconstricción).

Aspectos Esenciales • • • •

Los BAAC deben usarse en intervalos regulares durante la crisis en todos los pacientes (Evidencia A). Siempre evaluar técnica inhalatoria. La evaluación de la crisis mediante el PEF y la oximetría de pulso son esenciales para definir gravedad. Corticoides sistemicos estan indicados en crisis moderadas-graves

Diagnóstico

Episodios agudos o subagudos de un aumento progresivo de uno o más de los síntomas clásicos del asma: disnea, tos, sibilancias, opresión torácica, o alguna combinación de estos. Se caracterizan por disminución en el flujo espiratorio

El diagnóstico es esencialmente clínico y basado en los antecedentes del paciente. Los síntomas son los mejores indicadores de cuando se inició la crisis. En las crisis primero aparecen los síntomas y después decae la función pulmonar. Luego de la apreciación general del paciente se debe clasificar según gravedad, para eso se debe evaluar según los siguientes parámetros: • Frecuencia respiratoria. • Frecuencia cardiaca. • Uso de musculatura accesoria. • PEF o VEF1*. • Oximetría de pulso*. • Signología respiratoria obstructiva. • En casos de pacientes graves será necesario evaluar gases arteriales y estado ácido-base. • *La medición del PEF y la saturación de oxígeno son más confiables que la sintomatología para evaluar gravedad Basado en las normas del MINSAL 2013, se determina la gravedad de la crisis según la siguiente tabla:

Tratamiento



Caso Clínico Tipo Paciente de 28 años, tabáquico, asmático, con tratamiento irregular. Consulta al servicio de urgencia por disnea progresiva los últimos días, sibilancias y con aumento de requerimientos de salbutamol. Al examen físico se encuentra taquipneico, con disnea, sibilancias inspiratorias y espiratorias y con uso de musculatura accesoria, saturando 92% y PEF 70% de su basal.

Definición

Objetivos del tratamiento de urgencia: • Aliviar la obstrucción del flujo de aire y la hipoxemia lo antes posible • Definir hospitalización o manejo ambulatorio. • Tratamiento farmacológico de la crisis asmática: • Agonistas de acción corta (BAAC). • Su uso en dosis repetitivas está recomendado para todos los pacientes con crisis asmática (Evidencia A).

• • • •

Todos los pacientes deben ser revaluados y según esto mantener tratamiento con BAAC o ir escalando en la terapia antiinflamatoria si tiene mala respuesta inicial. El uso de BAAC se debe realizar con espaciador, en su defecto con nebulizaciones. Ambos son igual de efectivos. Corticoides orales. La efectividad entre uso oral y parenteral es la misma. En caso que el paciente presente vómitos o esté inestable se usarán por vía parenteral.

23 Facultad de Medicina, Universidad de Chile

• • • • • •

Tienen indicación en los pacientes con crisis asmática, especialmente en los siguientes: Pacientes que han requerido corticoides en crisis previas. Pacientes en quienes pese al uso de BAAC persiste la obstrucción al flujo aéreo. Pacientes en los que desarrollan una crisis asmática pese a estar en tratamiento con corticoides. Oxígeno Se debe suplementar oxígeno para mantener una saturación > 90%.

• • • •

Otras recomendaciones • El uso de BAAC en conjunto con Bromuro de Ipratropio (BI) produciría mejor broncodilatación que cada uno por separado (Evidencia B). • La combinación de BAAC + BI está asociada con menos tasas de hospitalización (Evidencia A) y con mejores valores de PEF al alta (Evidencia B). • No hay ventajas terapéuticas con el uso de Aminofilinas ni con el uso de antibióticos (Evidencia A) • Las recientes guías MINSAL recomiendan el uso de Salbutamol en dosis de 4 a 8 inhalaciones cada 20 minutos, por tres veces (una hora), asociado a Prednisona en dosis de 0,5 mg/Kg (1 dosis). Se refiere también que, en caso de no presentar mejoría de la oximetría de pulso aun habiendo suplementado con oxígeno por máscara o naricera, es necesario derivar.

tamol 2 inhalaciones 4 veces al día. B2 agonistas de acción larga (Salmeterol o Formoterol). Control en salas ERA precoz (dentro de 24 horas) para control evolutivo y reforzar técnica inhalatoria. Control médico dentro de 48 horas. Control con especialista si la crisis fue severa o de riesgo vital, en 48 horas.

Autor / Editor Nevelline Salgado

Manejo: 1. Iniciar Salbutamol 4-8 inhalaciones cada 20 minutos por 3 veces + Prednisona 0,5mg/kg (1 dosis) +Oxigenoterapia. 2. Reevaluar a los 60 min. 3. Si no mejora derivar Respuesta a tratamiento El tratamiento debe ser evaluado idealmente midiendo la función pulmonar (PEF o VEF), lo ideal es que logre volver a su basal.

Resultado y seguimiento: 1. ¿Quiénes se hospitalizan? • Usualmente aquellos pacientes que al ingreso al servicio de urgencia presentan PEF < 25% de su estimado. • Aquellos con PEF < 40% posterior a tratamiento. • Aquellos con PEF 40-60% de su estimado, posterior al tratamiento, podrían ser tratados ambulatoriamente. Requerirán un seguimiento más cercano. Control precoz en Sala ERA. • Aquellos con PEF > 60% de su estimado, posterior al tratamiento, pueden ser manejados ambulatoriamente. 2. Seguimiento en los pacientes en los que se decide tratamiento ambulatorio: • Pulso de corticoides inhalado en dosis media al menos por 7 días. • Mantener terapia con B-agonistas de acción corta. Salbu-

24 Facultad de Medicina, Universidad de Chile

Año 2016

BIBLIOGRAFÍA •

Manual CTO de medicina y cirugía, Neumologia y cirugia toracica 1°edicion chile. Asma

25 Facultad de Medicina, Universidad de Chile

anaerobios (que se encuentran en los surcos gingivales), aunque hay presencia de S. aureus, S. millieri y Klebsiella; además, puede haber hongos y microbacterias, por lo que es importante descartar su presencia, especialmente en inmunocomprometidos.

Absceso pulmonar Nivel de manejo por médico general: Diagnóstico: Específico. Tratamiento: Inicial. Seguimiento: Derivar.

Diagnóstico: La clínica es de comienzo insidioso y se caracteriza por tos productiva con expectoración fétida y ocasionalmente hemoptoica, fiebre no muy elevada, sudoración nocturna, baja de peso y compromiso del estado general. En ocasiones puede presentar broncorrea aguda. Al examen físico es posible encontrar roncus y crepitaciones, evidenciando una condensación, polipnea, taquicardia y halitosis. A la radiografía de tórax: cavidad única, unilateral, de bordes netos y con nivel hidroáereo. Puede realizarse un TAC para descartar un cáncer broncogénico, un empiema concomitante (presente en el 30% de los casos) y para encontrar abscesos pequeños.

Aspectos esenciales:    

Principal factor de riesgo es la enfermedad periodontal. La aspiración es el mecanismo fisiopatológico más importante. En la radiografía de tórax: cavidad de bordes lisos con nivel hidroaéreo. El tratamiento consiste antibióticos por al menos 6 semanas y drenaje.

Caso clínico tipo: Paciente varón de 76 años con antecedentes de DM2 y un ACV hace 2 años. Consulta por cuadro de 3 semanas de evolución caracterizado por tos productiva mucopurulenta abundante, CEG e intensa sensación febril. Destaca evidente mala higiene oral, con múltiples caries e intensa halitosis, además de polipnea, taquicardia y signos de deshidratación.

En exámenes de laboratorio se aprecia elevación de parámetros inflamatorios y de VHS.

Tratamiento: Antibioterapia por 6-8 semanas, con cobertura principalmente para anaerobios, Streptococo y bacilos gram negativos. La conducta más apropiada es determinar el germen causante y establecer el tratamiento correcto.

Definición: Necrosis del parénquima pulmonar secundaria a infección bacteriana, con formación de una cavidad mayor a 2 cm. La formación de múltiples abscesos menores de 2cm es ocasionalmente referido como neumonía necrotizante o gangrena pulmonar, sin embargo, son un continuo del mismo proceso.

El tratamiento de elección es Clindamicina 600 mg EV c/8 hrs con cambio a tratamiento ambulatorio a Clindamicina 150 a 300 mg VO c/6 hrs o Amoxicilina + Ácido Clavulánico. Se sugiere no utilizar Metronidazol por una falla del tratamiento cercana al 50%, sin embargo, si se indica es necesario que sea en combinación con Penicilina. Si se quiere utilizar un esquema alternativo, puede emplearse Ampicilina + Sulbactam 3 g EV c/6 hrs, o Carbapenémicos, aunque algunas fuentes sugieren que este sea el tratamiento a elección. La oxigenoterapia está indicada solo si es necesario.

Etiología – Epidemiología Fisiopatología: Su incidencia es mayor en hombres y aumenta con la edad, debido a una mayor prevalencia de trastornos de la deglución o masticación y su relación con la enfermedad periodontal. Algunos mecanismos que deben considerarse son: la disminución del reflejo de la tos (drogas depresoras, alcoholismo, los accidentes cerebrovasculares y la anestesia), el mal drenaje de secreciones, las cavidades pulmonares preexistentes y los trastornos en la inmunidad (diabetes mellitus, cirrosis, discinesia ciliar, VIH, etc.).

Se espera una mejoría de la clínica (principalmente disminución de la fiebre) dentro de 3 a 4 días después del comienzo del tratamiento antibiótico. En caso de que esto no ocurriera se debería realizar estudios diagnósticos más complejos para definir alteraciones anatómicas que compliquen la infección o un posible agente causante no cubierto por los antibióticos.

Se clasifica como primario o secundario en base a la presencia de comorbilidades que determinen mayor susceptibilidad a la formación del absceso: obstrucción (cuerpo extraño, cáncer, adenopatías), embolia séptica (CVC, endocarditis infecciosa tricuspídea, abscesos pelvianos o abdominales), necrosis avascular (cáncer, infarto), entre otros. Además, pueden ser agudos o crónicos si es que los síntomas han estado presentes por un mes o más. Un review de más de 1000 casos reportados reportó que aproximadamente 80% eran considerados primarios, 40% no-específicos y 40% crónicos.

Se sugiere realizar seguimiento con radiografía de tórax para determinar la desaparición de la lesión o la presencia de masa residual. Debe existir un correcto drenaje de secreciones y en caso de cuerpo extraño, extraer con fibrobroncoscopía. El tratamiento quirúrgico es raro y debe hacerse cuando la respuesta clínica es un fracaso (lobectomía, neumonectomía), cuando hay sospecha de cáncer o complicaciones como hemoptisis masiva, rotura a pleura o infección micótica de la cavitación.

Su principal mecanismo fisiopatológico es la aspiración, siendo muchos una complicación de una neumonía aspirativa. Suelen ser únicos (13% son múltiples). Los gérmenes más frecuentes son los

Seguimiento: Por el especialista, debido a las complicaciones (hemorragias masivas, fístulas, bronquiectasias y cavidades residuales que pueden sobreinfectarse). 1

En adultaos es mas frecuente entre 15 y 24 años por exposicion de conductas de riesgo asociado a consumo de alcohol y drogas.

Asfixia por Inmersión

Patologías con mayor riesgo son la epilepsia y el QT prolongado. El mayor grupo de riesgo lo constituyen lactantes, menores de 4 años y adolescentes. En los menores de 1 año lo más frecuente es que se produzca en la bañera.

Nivel de manejo del médico general: Diagnóstico Específico Tratamiento Completo Seguimiento Completo.

El cuadro se inicia con una reacción de pánico, que lleva a la pérdida del patrón respiratorio normal, una lucha secundaria, aspiración de pequeña cantidad de agua, que desencadena una hiperventilación seguida de apnea involuntaria, la llegada de agua al tracto respiratorio bajo ocasiona un laringoespasmo reflejo, si es sostenido lleva a una asfixia seca , si se produce relajación glótica lleva a una asfixia húmeda, ambas finalmente llevan a anoxia y paro cardiorrespiratorio, comprometiendo pulmones, cerebro, corazón y otros sistemas por la hipoperfusión e hipoxia.

Aspectos esenciales    

3°causa muerte en Chile en edad pediatrica. Objetivo: Evitar lesión anoxo-isquémica. Manejo adecuado en la escena (RCP modificado) y en el hospital (Establecer medidas tempranas esenciales). Énfasis en la prevención.

Caso clínico tipo Diagnóstico

Niño de 3 años, juega al borde de una piscina sin rejas en el patio del hogar sin supervisión, se sumerge en el agua y se mantiene durante 5 min sin que los cuidadores se percaten. Al darse cuenta notan al niño con compromiso de conciencia y taquicardia.

1. Síntomas y signos 

Definición 

Insuficiencia respiratoria producida por sumersión o inmersión en un medio acuoso lo suficientemente serio para requerir soporte vital en el lugar del suceso y transporte a un servicio de emergencia médico para evaluación y tratamiento.

2. Estudios básicos de laboratorio: Gasometría arterial con niveles de lactato sanguíneo, BHC, urea, creatinina, glucosa, pruebas toxicológicas (barbitúricos, BDZ), determinación de niveles de alcohol. RX de tórax, Rx de columna cervical AP-Lateral, cultivo de secreciones bronquiales, ECG, EEG. (Decidir hospitalización, si es así, en sala o en UCI).

Ahogamiento se refiere a la persona que fallece por asfixia por inmersión en un medio líquido, puede llevar a la muerte durante el accidente o durante las 24 horas siguientes. Semiahogamiento se refiere a la persona que sobrevive, aunque sea temporalmente, después de una accidente por inmersión, generalmente pueden presentar lesiones en cerebro, el corazón y riñones. El semiahogamiento a su vez se divide en: 



Leve sintomatología y signos vitales alterados: Siempre sospechar y descartar lesión cervical, Disnea, Déficit neurológico leve, Ansiedad, Cianosis (por Hipoxia), Hipotermia (dependiente de la temperatura del agua), tos intensa, tiraje. Sintomatología grave y SV alterados: Disnea, que puede llegar a apnea, Pérdida del estado de alerta, aumento de secreciones pulmonares, paro cardiorrespiratorio, bradicardia, taquicardia y fibrilación ventricular, edema cerebral.

3. Hospitalizar si: Historia de apnea o cianosis, pérdida de conciencia, riesgo de edema pulmonar, sumersión mayor a 1 min, si requirió RCP.

Húmedo si hubo aspiración de agua, tras la fase de broncoespasmo, se produce hipoxia e hipercapnia, la verdadera asfixia por inmersión (ahogamiento azul). Seco si se presento sólo broncoespasmo reflejo, se produce hipoxia por la apnea, cerca del 10% de los que llegan a los hospitales (ahogamiento blanco).

Tratamiento Objetivo: Evitar lesión anoxo-isquémica cerebral. 

Etiología-epidemiología-fisiopatología Es la tercera causa más común de muerte accidental en EEUU, siendo la 2da causa de muerte no intencional entre 1-4 años. En Chile es la tercera causa de muerte en edad pediatrica. La mayoría se da en agua fresca:



Infantes 78% muerte en bañeras 1-4 años 58% en piscinas, el resto en agua dulce > 4 años 63% en agua fresca

2

En la escena: Realizar ABC modificado para trauma (manejo de la columna cervical y otros), Controlar la hemodinamia, manejar la temperatura (retirar ropa mojada y secar), trasladar a hospital y reanimar hasta normotermia. No se recomienda realizar maniobras para eliminar agua del estomago durante el traslado y periodo prehospitalario, por mayor riesgo de aspiracion. Manejo hospitalario: Medidas tempranas esenciales: Intubación endotraqueal, ventilacion mecanica, monitorización eléctrica cardiaca, medir temperatura central, colocar línea venosa central, manejar la temperatura (en paciente inconsciente: métodos externos hasta bypass extracorpóreo), oxigenoterapia (desde aporte de FiO2 a VM), manejo del broncoespasmo, edema pulmonar y

SDRA, FBC por cuerpo extraño. Manejo de arritmias y función cardíaca. PPC y manejo de edema cerebral, evaluar anticonvulsivantes, antibioticoterapia ante sospecha de aspiración, corrección H/E y función renal.

Seguimiento Presenta entre un 30-50% de mortalidad global. Un10% sobrevive con daño neurológico severo (desde cuadriplejia a estado vegetativo persistente). Los esfuerzos deben ir principalmente a la prevención primaria: 1. En RN- 4 años: Supervisión por adulto, eliminar recipientes con agua, protecciones adecuadas en piscinas, instrucción en RCP a los cuidadores, las clases de natación no sirven. 2. En > 4 años: Clases de natación, supervisión por adulto, normas de seguridad en piscinas, educar sobre riesgos dentro del agua. Supervisar ingestión de alcohol y drogas de abuso durante la práctica de deportes acuáticos, entrenamiento en RCP.

3

Diagnóstico

Aspiración de cuerpo extraño con asfixia

Su forma de presentación habitual es una crisis asfíctica durante la alimentación o el juego con objetos en la boca. Síntomas diversos, desde asintomático a cursar con tos violenta, estridor inspiratorio, disnea, cianosis, apnea, angustia, disfonía o afonía, sibilancias, hasta provocar un PCR y muerte. La sintomatología puede variar debido al tamaño, localización y composición del CE, además del grado de obstrucción y del tiempo de permanencia en el árbol bronquial.

Nivel de manejo del médico general: Diagnóstico: Específico. Tratamiento: Completo. Seguimiento: Completo.

Cuando se ubica en el lumen de la laringe se produce el "Sd. De penetración”, que se caracteriza por una crisis de asfixia por espasmo de la glotis, que desencadena insuficiencia respiratoria con: angustia, tiraje y cornaje. En ese momento se desencadena tos expulsiva que puede eliminar el CE. En otros casos el CE sobrepasa la glotis, localizándose en sitios más inferiores de la vía aérea, dando sintomatología menos evidente. El estudio radiológico es fundamental.

Aspectos esenciales     

Edad principal < 4años. 60% de los cuerpos extraños (CE) son de origen alimentario. Obstrucción completa de vía aérea “Sd. De Penetración”. Manejo con maniobra de Heimlich en adultos y niños > 1 año. Mayor mortalidad en menores de 1 año y adultos mayores

Ubicació n CE

Característic as CE

Larínge o

Disfonía, estridor, tos Grandes (no quintosa, desciende) cianosis ocasional.

Traquea l

“Papirotazo”, ruido Puede quedar producido Rx, atascado o por choque Laringoscop moverse de CE con ía libremente glotis, durante ciclo respiratorio.

Caso clínico tipo Niño de 4 años, mientras jugaba solo, inicia bruscamente quejido e incapacidad para hablar, además de palidez momentánea, sumándose posteriormente cianosis progresiva, agitación psicomotora y angustia. Al poco tiempo sufre pérdida de conciencia.

Definición Obstrucción de vía aérea por aspiración de cuerpo extraño, con riesgo vital por el compromiso agudo de la función respiratoria, que de no ser manejado adecuadamente puede desembocar en la muerte o en severas secuelas.

Etiología-epidemiología-fisiopatología Los cuerpos extraños en la vía aérea constituyen la principal causa de muerte por patología del área otorrinolaringológica en nuestro país. En EEUU, se producen alrededor de 500 muertes anuales por esta causa. El 60% de los cuerpos extraños aspirados son de origen alimenticio y el 40% restante es variable.

Bronqui al

Edad principal: Niños a partir de los 6 meses, que es la edad en que comienza a tomar objetos pequeños por sus propios medios, para llevárselos a la boca, hasta alrededor de los 5 años.

Clínica

Obstrucción parcial: disnea sibilante, dificultad respiratoria. Completa: atelectasia o Sd. Condensació n. Por anatomia mas frecuente bronquio derecho

Exámenes Rx frontal y lateral de cuello (se ven sólo cuerpo radiopacos)

Rx Tórax AP y lateral, Laringoscop ía

Al momento del diagnóstico, el 80% de los cuerpos extraños están situados en el bronquio, en un 60% de los casos en el derecho.

Cuando un niño en edad escolar o adolescente ha aspirado un CE, se debe sospechar el inicio de una enfermedad neuromuscular o que en el momento del accidente haya estado bajo efectos de alcohol o drogas. 60% de los CE aspirados son de origen alimenticio (el mas frecuente es el mani) y 40% restante es variable. Las partículas orgánicas producen una rápida reacción inflamatoria local, mientras que las inorgánicas tienen escasa reacción por lo que se manifiestan con menos síntomas.

Ex fisico pulmonar: buscar disminucion de murmullo pulmonar, taquipnea o sibilancias.

Tratamiento El manejo médico queda reservado para aquel paciente que está en una crisis aguda y súbita de Insuficiencia 4

respiratoria. Clásicamente se describen: golpes en la espalda, golpe-compresión torácica y la maniobra de Heimlich.La elección de una de ellas depende de la edad del paciente y si la obstrucción es completa o no: a) Si la obstrucción es completa y el niño tiene menos de un año se pone al lactante decúbito prono, con la cabeza más abajo que el tronco y apoyándole con los dedos la mandíbula, tratando de mantener la cabeza ligeramente extendida; se golpea moderadamente y con rapidez con el talón de la mano en zona interescapular 5 veces. Luego se pone en la misma ubicación pero decúbito dorsal, sosteniendo la cabeza con la mano y siempre más abajo que el tronco y se harán 5 compresiones rápidas y moderadamente fuerte. Se revisará la boca y se extraerá el cuerpo extraño, si no se ve, se repite la maniobra. b) En la obstrucción completa de un niño que tiene más de un año, se recomienda la maniobra de Heimlich, el fracaso de ella debe generar una cascada de reacción Igual a la anterior. c) Si la obstrucción es Incompleta, el periodo crítico ha pasado y se debe estabilizar el paciente para la extracción definitiva. A pesar de lo descrito anteriormente el manejo es frecuentemente quirúrgico.Broncoscopia (rigida)

Seguimiento Si el CE está bajo la glotis, el tratamiento es la extracción en pabellón bajo anestesia general, por un cirujano entrenado, con broncoscopía rígida. Si existe la sospecha de que el CE por su tamaño no pasará fácilmente por la glotis, se realiza una traqueostomía al inicio del procedimiento.

5

2. Obstrucción valvular por obturación: La luz bronquial queda totalmente obstruida, sin que el aire pueda entrar ni salir. El aire atrapado distalmente se reabsorbe y se producen atelectasias. Es el segundo tipo en frecuencia.

Aspiración de cuerpo extraño sin asfixia Nivel de manejo del médico general: Diagnóstico: Específico. Tratamiento: Inicial. Seguimiento: Derivar.

Diagnóstico 1. Clínica: Presenta una tríada de presentación: tos, sibilancias y disminución de entrada de aire. Además puede presentar síntomas específicos según ubicación del cuerpo extraño:

Aspectos esenciales    

 Accidente frecuente en infancia, ´rincipalmente menores de 4 años. Principal cuidado con inhalación de sustancias vegetales. Triada clásica: tos, sibilancias y disminución de entrada de aire. Contraindicado introducir a ciegas los dedos en la laringe.



Supraglótica: tos, disnea, dificultad para la deglución, sialorrea y cambios de voz. Laringe: si se obstruye completamente, produce asfixia aguda y pueden conducir a la muerte si no se eliminan rápidamente o dejar como secuela encefalopatía hipóxica en los supervivientes.

En Obstrucción parcial: síntomas de estridor, afonía, tos croupal, odinofagia, hemoptisis, sibilancias y disnea.

Caso clínico tipo



Niño de 3 años jugando en el jardín, presenta de forma súbita tos y sibilancias. La madre lo lleva al servicio de urgencias donde se constatan sibilancias y asimetría en el murmullo pulmonar en hemitórax derecho. La localización es: Bronquial.





Definición Ingreso en la vía aérea de un cuerpo sólido, provocando una obstrucción.

Traquea extratorácica: Cuadro similar al anterior que también puede terminar en asfixia. Tráquea intratorácica: Se manifiesta por sibilancias espiratorias y se ha descrito la tos en palmada o bofetada que se produce sobre todo cuando se tose con la boca abierta. Bronquial: predomina la tos y no suele asociarse con dificultad respiratoria. En ocasiones se asocia a sibilancias, rara vez audibles sin fonendo. Asimetría en la auscultación, en forma de hipoventilación localizada o como sibilancias asimétricas.

*Tabla diagnostico diferencial en Capitulo Aspiracion de cuerpo extraño con asfixia.

Etiología - epidemiología – fisiopatología

de

Generalmente debuta con un paroxismo inicial caracterizado por una crisis de tos o sofocación con o sin cianosis, disnea, estridor y dolor. Superada esta etapa hay un periodo relativamente asintomático, de días incluso meses, que puede producir falsa sensación de normalidad. La mucosa bronquial sufre las consecuencias de la impactación extraña: neumonía, atelectasia, bronquiectasia, abscesos o zonas de hiperinsuflación.

Es uno de los accidentes más frecuentes de la infancia (4%). Representa el 0,07%-0,35%, siendo responsable de un elevado número de muertes (100/año). Afecta más a los niños que a las niñas (2:1), entre los 12 meses y los 3 años de vida, menores de 1 año presentan mayor mortalidad. Puede presentar variadas etiologías, se debe tener cuidado principalmente con la inhalación de sustancias vegetales (maní), pues por su poder hidrofílico van a hincharse aumentando su tamaño, además tienen acción irritante sobre la mucosa, lo que contribuye a aumentar la oclusión.

2. Imágenes: Practicar Rx lateral de cuello si se sospecha cuerpo extraño laríngeo o traqueal y/o Rx de torax en inspiración y espiración forzada. Si pese a todo no se confirma la sospecha clínica se debe practicar una fibrobroncoscopia exploradora que además podría ser terapéutica.

Productos pequeños pueden fragmentarse emigrando hacia tractos bronquiales distales que hacen difícil su extracción. La mayor parte de los objetos aspirados son capaces de cambiar de posición con la consiguiente alteración de los signos físicos que desencadenan.

Tratamiento En obstrucción incompleta de la vía aérea: Si el paciente está estable y tiene respiración efectiva, administrar oxígeno y mantener al paciente en posición cómoda, animándole a seguir tosiendo. La broncoscopía es el tratamiento de elección, a realizarse en quirófano con anestesia general.

Fisiopatológicamente encontramos 2 tipos: 1. Obstrucción valvular unidireccional: Durante la inspiración el bronquio se dilata y el aire puede entrar. Durante la espiración el bronquio se contrae retrayéndose sobre el cuerpo extraño e impidiendo la salida del aire. Esto provoca atrapamiento aéreo unilateral o local. Es el tipo más frecuente.

- Está contraindicado introducir a ciegas los dedos en la orofaringepues se podría desplazar el cuerpo extraño a la laringe convirtiendo una obstrucción parcial en completa.

6

- La Maniobra de Heimlich: NO indicada en obstrucción INCOMPLETA de la vía aérea. Complicaciones tras la extraccion: – Obstrucción de la vía aérea – Neumotórax-Neumomediastino – Hemoptisis – Edema laríngeo – Laceración laríngea – Estenosis bronquial – Fístula traqueoesofágica – Bronquiectasias distales

Seguimiento 



Sin diagnóstico precoz, pueden originarse complicaciones a mediano y largo plazo: Estenosis bronquiales, broncoespasmo recidivante, atelectasias persistentes, neumonías recidivantes. Evaluar complicaciones post broncoscopía: Evolución a obstrucción total de la vía aérea, atelectasia, neumonía, estridor, etc.

7

inflamatorio, facilitan las infecciones, por lo que se genera un círculo vicioso. En caso de bronquiectasias localizadas en un segmento pulmonar, debe descartarse la obstrucción bronquial ya sea intra o extraluminal.

Bronquiectasias Nivel de manejo por médico general: Diagnóstico: Sospecha. Tratamiento: Inicial. Seguimiento: Completo.

Diagnóstico: El cuadro clínico clásico se caracteriza por tos y expectoración crónica (broncorrea purulenta) con frecuentes exacerbaciones (aumento de síntomas, fiebre, disnea), pudiendo presentar además hemoptisis leve. Si se deja en evolución espontánea puede desarrollar disnea progresiva, desnutrición, insuficiencia respiratoria e incluso cor pulmonale. La radiografía de tórax puede mostrar imágenes sugerentes, pero el diagnóstico definitivo es clínicoradiológico con tomografía axial computada de alta resolución (TACAR). Se recomienda estudio etiológico con baciloscopía, cultivo de Koch, Gram y cultivo corriente de expectoración. Las pruebas funcionales suelen mostrar patrón obstructivo. Diagnóstico diferencial: EPOC, asma, bronquitis crónica u otras causas de limitación crónica del flujo aéreo.

Aspectos esenciales:    

Diagnóstico clínico-radiológico: broncorrea + TAC alta resolución. Principal causa: secuelar post-infecciosa. Plantear estudio etiológico. Manejo agresivo mejora calidad de vida y sobrevida.

Caso clínico tipo: Paciente varón de 58 años, con antecedentes de TBC pulmonar tratada hace 30 años, y fumador con IPA de 20. Presenta "resfríos recurrentes" desde hace años, caracterizados por tos y broncorrea purulenta que debe tratar con antibióticos. ¿Qué diagnósticos plantea? ¿Cómo inicia el estudio y tratamiento?

Tratamiento: 

Definición: Dilataciones permanentes y anormales de los bronquios, mayores a 2 mm, debido a destrucción de las capas elástica y muscular de la pared bronquial.



Etiología – epidemiología - fisiopatología: La prevalencia de la bronquiectasia es desconocida, probablemente siendo muy variable entre países. Se estima que en EEUU 110000 individuos tienen bronquiectasia, siendo de mayor prevalencia en adultos sobre los 60 años y más comúnmente en mujeres. Estos pacientes utilizan abundantes recursos de salud debido a frecuentes visitas ambulatorias, uso de antibióticos, hospitalizaciones y exámenes complementarios.

 

La principal causa actual es secuelar a infecciones respiratorias, principalmente Adenovirus y TBC. La fisiopatología involucra una noxa infecciosa sobre un sustrato de obstrucción de vía aérea, drenaje alterado o inmunidad deficiente. Otras infecciones causantes son virus influenza y neumonías bacterianas (P. Aeruginosa, K. Pneumoniae, S. Aureus). Sin embargo, la mayoría de las veces no se logra el diagnóstico etiológico.

 

Medidas generales: suprimir tabaquismo, prevenir infecciones respiratorias y tratamiento de enfermedad de base, además de control y apoyo del estado nutricional. Control de infección: se sugiere antibioterapia tanto para fase estable (en forma continua o rotatoria) como en exacerbaciones, siendo en ésta última por 10 a 14 días con: aminopenicilinas con inhibidores de β-lactamasas, quinolonas respiratorias, macrólidos o aminoglicósidos en caso de colonización o infección por Pseudomona. En el tratamiento de Pseudomona aeruginosa el régimen empírico debe estar basado en los patrones locales de resistencia, sin embargo, se sugieren antibióticos antipseudomónicos como β-lactámico + quinolona, β-lactámico + aminoglicósido, o quinolona + aminoglicósido. Manejo de secreciones: kinesioterapia y broncodilatadores. Manejo de la obstrucción: si responde a broncodilatadores. Tratamiento de la causa. Tratamiento quirúrgico: segmentectomía o lobectomía ante bronquiectasias localizadas sin resultados con más de 1 año de tratamiento médico. En hemoptisis masiva: taponamiento broncoscópico o embolización arterial.

Seguimiento:

Otras causas son condiciones genéticas (fibrosis quística, síndrome de Kartagener, déficit de α-1 antitripsina), alteraciones del sistema inmune (déficit de inmunoglobulinas o del complemento, enfermedades autoinmunes), obstrucción bronquial prolongada (tapón mucoso, cuerpo extraño, neoplasias) y aspergilosis.

Derivar a especialista si no responde a tratamiento médico o ante necesidad de resolución quirúrgica.

El proceso empieza con inflamación de la pared que, debido a alteraciones estructurales por el daño 8

el único tratamiento definitivo, pero sólo el 16-17% de los pacientes se presentan al diagnóstico en etapas quirúrgicas con posibilidad de curación. El 52% de los pacientes tiene metástasis al diagnóstico (etapa IV), lo que se traduce en un 3.6% de sobrevida a 5 años. Las metástasis más frecuentes son el pulmón, mediastino, suprarrenales, hueso, cerebro e hígado.

Cáncer bronquial primario Nivel de manejo por médico general: Diagnóstico: Sospecha. Tratamiento: Inicial. Seguimiento: Derivar.

La frecuencia de los distintos tipos de cáncer es: adenocarcinoma 38% (incluido el carcinoma bronquiolo-alveolar), células escamosas 20%, células grandes 5%, células pequeñas 13%, otros no células pequeñas no clasificables 18%, y otros 6%. Los cánceres escamosos y de células pequeñas suelen comprometer más frecuentemente las vías aéreas centrales.

Aspectos esenciales:    

Cáncer más mortal a nivel mundial. En Chile, 3° en hombres y 4° en mujeres. El 90% son secundarios al consumo de tabaco. Cáncer muy agresivo que da metástasis tempranas. La medida más costo-efectiva es la prevención/suspensión del tabaco.

Para la etapificación se utiliza el sistema TNM en su 8va edición (2017), que realiza varios cambios en relación a la 7ma edición (2010).

Diagnóstico:

Caso clínico tipo:

La clínica se caracteriza por tos (aparición o cambio de patrón), hemoptisis (leve a moderada) y disnea. Puede acompañarse de derrame pleural paraneoplásico. Un tumor mayor puede ocasionar obstrucción, y con ello, causar infecciones (neumonías) a repetición. Si se ubica en las cúpulas pulmonares, puede generar síndrome de Claude Bernard Horner. En casos raros y avanzados, pueden manifestarse como síndrome de vena cava superior. Los síndromes paraneoplásicos son más frecuentes en carcinomas de células pequeñas.

Paciente varón de 67 años con antecedentes de tabaquismo activo IPA 53, tosedor crónico. Consulta por un cuadro de 3 meses de evolución caracterizado por disnea progresiva y aumento de su tos habitual, a la que se ha asociado un dolor pleurítico del hemitórax derecho y algunos episodios de hemoptisis. Refiere además baja de peso en el último tiempo.

Definición: Neoplasia maligna originada en las células del epitelio bronquial. Se distinguen dos grupos: carcinoma de células pequeñas (13%) y carcinoma de no células pequeñas (87%), este último clasificado en carcinoma escamoso, adenocarcinoma, carcinoma bronquioloalveolar y carcinoma de células grandes.

El carcinoma de células no pequeñas no manifiesta síntomas hasta que está avanzado, donde aparecen tos, hemoptisis y dolor pleurítico. También puede manifestarse como metástasis extratorácicas (hígado, hueso, cerebro), o como algún síndrome paraneoplásico (hipercalcemia, síndrome de Cushing, alteraciones de la coagulación). Lo más frecuente es diagnóstico incidental en una radiografía de tórax o en estudio extrapulmonar.

Etiología – epidemiología - fisiopatología:

El examen físico puede ser normal o dar manifestaciones de un cáncer avanzado, como baja de peso e hipocratismo digital. Si hay sospecha clínica corresponde realizar un TAC de tórax. La radiografía puede mostrar falsos negativos en lesiones pequeñas, pero sirve para evidenciar grandes masas o compromiso de la pleura o del mediastino. Si existen lesiones sugerentes, se debe derivar a un especialista.

Alrededor del 90% de los cánceres pulmonares son causados por el tabaco, ya sea activa o pasivamente. A nivel mundial, el cáncer pulmonar es la primera causa de muerte por cáncer, aunque es el segundo en incidencia en hombres y el cuarto en mujeres, y en Chile es la tercero más frecuente en hombres y cuarto en mujeres. Se presenta con mayor frecuencia en mayores de 45 años y el promedio de edad al diagnóstico es 71 años, sin embargo, cada día hay pacientes más jóvenes con cánceres más agresivos. Se considera una epidemia.

La detección precoz está recomendada pero aún no es parte de guías, en fumadores de IPA mayor a 30 entre 55 y 74 años con TAC de tórax sin contraste, según lo demostró el estudio NLST (NEJM 2011), y disminuye mortalidad al comparar con la radiografía.

El riesgo para un ex fumador disminuye al 50% en 5 años de abstención, y al 25% en 10 años, persistiendo siempre levemente más elevado que la población no fumadora. Por lo anterior, el método más efectivo para disminuir el número de nuevos casos es evitar que los no fumadores adquieran la adicción, y procurar que los fumadores abandonen el mal hábito lo más pronto posible.

Tratamiento: La evaluación inicial requiere diferenciar si el cáncer es o no de células pequeñas, establecer el estadio de enfermedad según TNM (el factor pronóstico más importante), y ver el estado general del paciente.

Los parientes en primer grado de un enfermo con cáncer pulmonar tienen un riesgo 2.4 veces mayor de desarrollar neoplasias. La sobrevida global en países desarrollados no supera el 15% a 5 años. La Cirugía es

Para pacientes con cáncer de no células pequeñas, el tratamiento depende del estadio de la enfermedad. En 9

etapas iniciales el tratamiento de preferencia es cirugía, reservando la quimioterapia para tumores de mayor extensión intratorácica. En etapas avanzadas, el tratamiento generalmente es paliativo. En el cáncer de células pequeñas, el principal tratamiento es la quimioterapia, ya que en la mayoría de los casos está diseminado al momento del diagnóstico. Cuando se detecta en etapa temprana, se usa radioterapia combinada a quimioterapia, pues esta adición ha mostrado un aumento de la sobrevida. Hoy en el mundo es de vital importancia la presencia de mutaciones susceptibles de terapias individualizadas que pueden mejorar la calidad de vida y sobrevida de algunos pacientes, por ejemplo, mutación EGFR en adenocarcinomas, ALK-EML4, presencia de Kras, ERCC1, etc. Es considerado una conducta estándar en el mundo buscar EGFR en los adenocarcinomas (principalmente en no fumadores, mujeres con ascendencia asiática). Aún en etapas avanzadas, los pacientes con cáncer de pulmón debieran recibir tratamiento, pues mejora su calidad de vida y en algunos casos puede prolongar la sobrevida.

Seguimiento: Por parte del especialista.

10

Diagnóstico:

Corazón pulmonar crónico (cor pulmonale)

Se presenta inicialmente con disnea de esfuerzo, palpitaciones y mareos, generando posteriormente la aparición de fatigabilidad, CEG, baja de peso, anorexia, etc., todo esto tras la disminución del gasto cardíaco.

Nivel de manejo por médico general: Diagnóstico: Específico. Tratamiento: Inicial. Seguimiento: Derivar.

El examen cardíaco muestra signos de hipertensión pulmonar, como aumento del componente pulmonar del segundo ruido y latido vivo del VD en la zona paraesternal derecha o en epigastrio, por hipertrofia del VD. La aparición de un tercer ruido derecho indica insuficiencia ventricular y la insuficiencia tricuspídea por dilatación del VD se traduce por un soplo sistólico de regurgitación y aumento de la onda y pulso venoso. Además, hay ingurgitación yugular, hepatomegalia, reflujo hepatoyugular y edema. Se puede asociar también a síncope y dolor torácico.

Aspectos esenciales:    

Insuficiencia cardíaca derecha ocasionada por hipertensión pulmonar. Principal causa: EPOC. Diagnóstico incluye principalmente clínica, radiografía de tórax y ECG. Manejo por especialista.

En la radiografía de tórax se observan escasas manifestaciones, ya que frecuentemente la silueta cardíaca no aumenta notoriamente de tamaño, aunque podría apreciarse crecimiento de VD en una proyección lateral. El ECG característico puede mostrar: R/S >1 en V5 y V6, S > R en V1, bloqueos de rama derecha, P > 2 mm en D2 (P pulmonar) y desviación a la derecha del eje eléctrico, aunque estos signos pueden estar ausentes incluso en casos avanzados.

Caso clínico tipo: Paciente varón de 70 años con antecedentes de EPOC. Consulta por cuadro caracterizado por CEG, baja de peso, disnea de esfuerzo, palpitaciones, y edema de extremidades. Al examen cardíaco se nota aumento del componente pulmonar del segundo ruido latido vivo paraesternal, tercer ruido, soplo de insuficiencia tricuspídea, yugulares ingurgitadas, reflujo hepatoyugular y edema.

Se puede incluir entre los exámenes de acuerdo a la sospecha clínica: gases arteriales, pruebas de función pulmonar, ecocardiograma, cintigrama pulmonar, anticuerpos sanguíneos, péptido natriurético cerebral (BNP), TAC de tórax, cateterismo cardiaco.

Definición: Tratamiento:

Es una alteración de la estructura (hipertrofia o dilatación) y/o insuficiencia de función del ventrículo derecho, asociada a enfermedad pulmonar crónica y/o hipoxemia. Si bien algunos expertos están en desacuerdo con la inclusión de la disfunción del VD debido a enfermedad vascular pulmonar, la mayoría está de acuerdo en que la disfunción debido a enfermedad cardiaca izquierda o congénita no son considerados cor pulmonale.

El tratamiento persigue tres objetivos: reducir la sobrecarga ventricular derecha, disminuir la presión ventricular derecha y mejorar la contractibilidad ventricular derecha. Siendo el EPOC la causa más frecuente de hipertensión pulmonar crónica, el manejo generalmente incluye la corrección de la hipoxemia, hipercapnia y acidosis.

Etiología – epidemiología - fisiopatología:

Sintomático: Los diuréticos son útiles al disminuir la precarga del VD (ojo con riesgo de disminuir la volemia, acentuando la disminución del gasto cardíaco).

Un estudio retrospectivo de 156 pacientes con enfermedad pulmonar crónica avanzada reportó que 2/3 de los pacientes presentaban disfunción del ventrículo derecho.

Seguimiento:

El desarrollo de hipertensión pulmonar determina una sobrecarga del ventrículo derecho (VD) y conduce a su dilatación e hipertrofia. Como consecuencia el VD se hace insuficiente, aumenta su presión diastólica, se sobrecarga la aurícula derecha y se produce hipertensión venosa sistémica. Casi cualquier enfermedad pulmonar crónica que produzca disminución prolongada del oxígeno sanguíneo puede llevar a cor pulmonale. Entre ellas encontramos: EPOC (causa más frecuente), fibrosis quística, enfermedad pulmonar intersticial, cifoescoliosis, apnea obstructiva del sueño, TEP, etc.

Se debe derivar a especialista tras el diagnóstico.

11

Diagnóstico:

Daño pulmonar secundario a drogas

El cuadro clínico es inespecífico, pues se puede presentar como disnea de diversa intensidad o tos no productiva. Se debe tener el antecedente de tratamiento con medicamentos que se asocien a daño pulmonar (se pueden comprobar los medicamentos en www.pneumotox.com).

Nivel de manejo por médico general: Diagnóstico: Sospecha. Tratamiento: Inicial. Seguimiento: Derivar.

La radiología usualmente describe un patrón de infiltrado intersticial difuso, pero en general no permite diferenciar el daño por drogas del producido por otras enfermedades. La concordancia entre esto y los datos de la anamnesis permite dar más fuerza a la sospecha diagnóstica.

Aspectos esenciales:    

La forma más común es la fibrosis pulmonar intersticial. Factores de riesgo: >60 años, hipoalbuminemia, medicamentos previos y compromiso pleuropulmonar previo. Amiodarona y antineoplásicos asocian frecuentemente EPID. Tratamiento es la suspensión del fármaco.

Dentro de las pruebas de función pulmonar, en la espirometría puede esperarse un patrón restrictivo.

Tratamiento: Suspender la droga/fármaco sospechoso, o reemplazar por alguna alternativa.

Caso clínico tipo: Seguimiento: Paciente varón de 65 años con antecedentes de HTA y arritmia completa por FA, en tratamiento con Amiodarona y TACO. Consulta por cuadro de tos seca y disnea de esfuerzo. La radiografía de tórax muestra un infiltrado pulmonar difuso en ambos campos pulmonares.

Derivación a especialista si la situación es grave o la evolución desfavorable.

Definición: Compromiso a nivel alveolar e intersticial inducido por drogas administradas con fines terapéuticos. Cualquier droga o medicamento puede causar daño o compromiso pulmonar.

Etiología – epidemiología - fisiopatología: La fibrosis pulmonar intersticial es la forma más común y representa el 3% de las enfermedades pulmonares intersticiales difusas (EPID). Los factores de riesgo más importantes son: la edad mayor de 60 años, la afección pleuro-pulmonar por artritis reumatoide, uso previo de medicamentos modificadores de la enfermedad e hipoalbuminemia. La EPID se produce por agentes fibrogénicos que favorecen el engrosamiento difuso de la pared alveolar y de matriz extracelular, manifestándose como un defecto pulmonar restrictivo. El daño pulmonar ocurre como una reacción idiosincrática, sin embargo se han descrito mediadores de la fibrosis, como TNF-α y TGFβ. Algunos fármacos destacan por su frecuencia: Amiodarona, Metotrexato y antineoplásicos.

12

SDRA característicamente oxigenoterapia.

Edema pulmonar no cardiogénico

 

a

A la radiografía de tórax se verá relleno intersticial alveolar en los campos pulmonares, tamaño de silueta cardíaca normal, hilios pulmonares no prominentes. Pueden verse líneas B de Kerley como resultado del edema intersticial (marcas cortas y rectas que se observan en las regiones inferiores de los pulmones).

Aspectos esenciales:



responde

Se debe contextualizar al paciente, por lo que es necesario preguntar por fármacos o procedimientos realizados previamente, exposición ambiental, entre otros.

Nivel de manejo por médico general: Diagnóstico: Sospecha. Tratamiento: Inicial. Seguimiento: Derivar.



no

Múltiples etiologías que alteran la barrera alveolocapilar. El diagnóstico es clínico, se presenta como insuficiencia respiratoria aguda. Manejo de la falla respiratoria y continuar según etiología de base. Diagnóstico diferencial: edema pulmonar cardiogénico.

También se deben evaluar parámetros inflamatorios, ECG, enzimas cardiacas, función renal, electrolitos plasmáticos.

Tratamiento: Lo primero es el manejo de falla respiratoria al asegurar vía aérea, administrar oxígeno. Luego se debe encontrar patología de base para un tratamiento específico. Si el edema pulmonar se asocia con una reacción alérgica, se emplea el oxígeno por mascarilla con corticoesteroides por vía intravenosa. El edema pulmonar agudo debido a una intoxicación por heroína se trata con oxígeno y naloxone; el de las grandes alturas se trata con reposo en cama y oxígeno al 100% y el traslado rápido del paciente a una altitud inferior.

Caso clínico tipo: Joven de 27 años sometido a cirugía ortopédica programada, sin dificultades a la intubación OT. A los 3 min post-extubación desarrolla tiraje subcostal, disnea grave, estridor, estertores por ambos campos pulmonares, cianosis y desaturación arterial que disminuye rápidamente hasta 50%. Ingresa a UCI y se conecta a VM. La radiografía de tórax evidencia edema pulmonar con infiltrados alveolo-intersticiales preferentemente en el hemitórax derecho.

El tratamiento es diferente al del edema pulmonar agudo cardiogénico, pues no se utiliza digital ni diuréticos. Además, no debe usarse morfina si el paciente tiene asma bronquial o enfermedad pulmonar crónica.

Definición: Acumulación de líquido intravascular en los alvéolos, sin evidencia hemodinámica que sugiera origen carcinogénico (PCP <18mmHg) ni disfunción cardíaca.

Seguimiento: Derivar a especialista.

Etiología – epidemiología - fisiopatología: La causa más frecuente es SDRA, que puede ser originado por sepsis o neumonía, otras: ex vacuo, postobstrucción, drogas (salicilatos, opiáceos, hidroclorotiazida, medios de contraste entre otros), gases, altura, post-transfusión, neurogénico. El mecanismo fisiopatológico se debe a una alteración de la permeabilidad de la barrera alveolo-capilar y no por aumento de la presión capilar pulmonar (edema pulmonar cardiogénico), permitiendo paso de proteínas y agua desde el medio intravascular. Otros mecanismos son por aumento de la presión negativa intersticial y alteración en la circulación linfática.

Diagnóstico: El paciente se presenta en falla respiratoria con una sintomatología respiratoria indistinguible del edema pulmonar cardiogénico a excepción de la disnea de evolución rápida, pues igualmente hay taquipnea, taquicardia, angustia, cianosis, tos con expectoración hemoptoica e hipoxia. Lo que genera la sospecha es que no se observará clínica de IC, y en el caso de 13

Etiología – epidemiología - fisiopatología:

Embolia pulmonar

El 70% de las veces el émbolo se origina en una trombosis venosa profunda de las extremidades inferiores y cerca de un 25% en la vena cava inferior. Es la causa más frecuente de muerte en las cirugías electivas y explica hasta un 15% de las muertes postoperatorias.

Nivel de manejo por médico general: Diagnóstico: Sospecha. Tratamiento: Inicial. Seguimiento: Derivar.

El proceso de agregación plaquetaria y coagulación alrededor de las válvulas venosas se ve favorecido por la lentitud de la circulación, daño de la pared del vaso y aumento de la coagulabilidad de la sangre (triada de Virchow).

Aspectos esenciales:      

Más del 70% de las veces se origina en el sistema venoso profundo de las extremidades inferiores. La disnea súbita es su síntoma de presentación más frecuente. La heparina es el tratamiento de elección, se debe iniciar ante sospecha, y confirmar con angioTAC. Si hay compromiso hemodinámico se debe efectuar la trombolisis. El filtro de vena cava es una alterativa si se contraindica la anticoagulación. Sospechar TEP secundario a trombosis hereditarias en: TEP a repetición, TEP sin factores de riesgo identificables, trombosis en sitios no habituales.

Factores de riesgo importantes para presentar TEP son: cirugías mayores (abdominopelvianas, cadera y EEII), cesárea y puerperio, neoplasias, fracturas/traumas de EEII, inmovilización prolongada, tabaquismo, uso de anticonceptivos orales y el antecedente personal de TVP o TEP. El efecto hemodinámico dependerá de magnitud de la obstrucción, el estado previo del corazón y del parénquima pulmonar. Si el corazón no es capaz de soportar el aumento agudo de la presión de la arteria pulmonar puede producirse una insuficiencia cardiaca derecha (>40 mmHg). Se produce un aumento mayor de la presión si el paciente tiene una enfermedad pulmonar previa o una embolia pulmonar crónica.

Caso clínico tipo: Paciente mujer de 38 años, lleva dos semanas de reposo por un esguince de tobillo derecho y está en tratamiento con anticonceptivos orales por acné. Acude a urgencias por disnea de inicio brusco con dolor torácico asociado. En el examen físico se observa una PA de 110/65, frecuencia cardíaca 104, taquipnea leve y auscultación cardiopulmonar normal. El ECG muestra taquicardia sinusal, y la radiografía de tórax es informada como normal.

Diagnóstico: Los síntomas más frecuentes son disnea, dolor pleurítico, tos y hemoptisis; todos de aparición súbita. Al examen físico se puede encontrar taquipnea, crepitaciones, taquicardia y acentuación del componente del 2do tono cardiaco, además de signos de TVP (empastamiento). Se usa el puntaje de Wells para calcular la probabilidad de TEP.

Definición: Obstrucción del árbol vascular pulmonar producida habitualmente por trombos originados en sitios distantes a él. También puede tratarse de émbolos de grasa, gas o de una embolia séptica.

Hallazgos clínicos

Puntaje

Signos de trombosis venosa profunda

3

Frecuencia cardiaca >100/min

1,5

Inmovilización en cama >3 días o cirugía en el 1,5 mes previo Antecedente de embolia pulmonar o trombosis 1,5 venosa Expectoración hemoptóica 1 Cáncer en tratamiento durante los 6 meses 1 previos Diagnóstico de TEP más probable que otros 3 diagnósticos

Puede clasificarse como aguda o crónica. En la primera, los signos y síntomas aparecen inmediatamente después de la obstrucción de los vasos pulmonares; en la segunda, el paciente presenta disnea progresiva durante años, secundario a la hipertensión pulmonar. Las embolias pulmonares agudas pueden ser clasificadas como masivas o submasivas. Se define como masiva el cuadro que causa hipotensión (PAS <90 mmHg o una disminución en la PAS ≥40 mmHg desde su basal, por un período >15 minutos). Todas las embolias que no cumplen el criterio anterior son consideradas submasivas.

Si el puntaje es <2, la probabilidad es baja. Si está ente 2 y 6, es intermedia, mientras que si es >6 la probabilidad es alta. Dentro del estudio inicial también es de utilidad pedir: 

14

Radiografía de tórax: en muchos casos podemos ver una trama vascular congestiva con edema "en alas de mariposa", derrame pulmonar, y elevación de hemidiafragma.



Seguimiento:

ECG: sirve para descartar IAM, y es orientador según varios cambios inespecíficos (70% alterado). o Patrón QR en derivación V1. o Signos de sobrecarga VD: inversión ondas T en V1-V4. o Bloqueo completo o incompleto de rama derecha.

Por especialista.

Para establecer el diagnóstico debemos determinar su probabilidad y contrastarla idealmente con un angioTAC. Si tenemos una probabilidad alta o intermedia y un angioTAC positivo, el diagnóstico está hecho. Si la probabilidad es baja y el angioTAC es negativo, se descarta el diagnóstico. En los casos restantes se deben hacer más pruebas, como el ecodoppler venoso. En pacientes ambulatorios se puede hacer un dímero D como primera aproximación o descarte de TEP (alto valor predictivo negativo).

Tratamiento: El manejo inicial debe ir enfocado a la estabilización del paciente: hemodinamia, suplementación de oxígeno en caso de hipoxemia (y valorar la necesidad de ventilación mecánica). Si el paciente tiene una baja probabilidad de TEP se debe solicitar un dímero-D, el cual tiene un alto valor predictivo negativo. De esta manera, una baja probablidad de TEP más dímero-D negativo descarta definitivamente la posibilidad de TEP. En casos de alta sospecha con hemodinamia estable, se debe iniciar tratamiento con heparina corriente o de bajo peso molecular mientras se prepara al paciente para el angioTAC. Una vez confirmado por este examen, se inicia el tratamiento anticoagulante oral paralelo a la heparina, la que se suspende una vez que el tratamiento oral esté en niveles terapéuticos (INR 23). El tratamiento oral se mantiene por 3 a 6 meses, y si el factor de riesgo se mantendrá activo, será de por vida. En casos de TEP con hemodinamia inestable, se inicia tratamiento con heparina no fraccionada o de bajo peso molecular a dosis plena, en espera del resultado del angioTAC. Una vez confirmado, se realiza trombolisis, siempre y cuando no haya contraindicaciones absolutas. La trombolisis se puede realizar con estreptoquinasa (250.000 U EV los primeros 30 minutos y luego 100.000 U/hr por 24 horas, con atención a aparición de hipotensión, anafilaxia, asma o reacciones alérgicas), tPA (100 mg EV en 2 hrs). Generalmente se descontinúa la terapia anticoagulante durante la infusión del trombolítico. Se contraindica la trombolisis en caso de neoplasia intracraneana, trauma o cirugía intracraneana reciente (<2 meses), hemorragia interna activa o reciente en los últimos 6 meses, historia de ACV hemorrágico, coagulopatía, HTA severa e incontrolada, ACV no hemorrágico en los últimos 2 meses, cirugía en los últimos 10 días y trombocitopenia. Algunos autores plantean que ante un paciente con evidente probabilidad de TEP y hemodinamia inestable, podría iniciarse de entrada trombolisis, idealmente con tPA (tenecteplase 100 mg a pasar en 2 hrs) acompañado de heparina, y realizar luego sin premura el angioTAC para confirmar la sospecha, sin embargo, está en discusión. 15

Diagnóstico

Embolia Pulmonar Masiva

Se necesita un alto grado de sospecha. El TEP masivo se presenta como cuadro de shock circulatorio: Hipotensión arterial, signos de mala perfusión, y alteración de conciencia, que se acompaña de hipoxemia, hipocapnia. Un TEP masivo puede ir acompañado de insuficiencia ventricular derecha aguda, que se manifiesta por un aumento de la presión venosa yugular, tercer ruido, cianosis y shock obstructivo.

Nivel de manejo del médico general: Diagnóstico: Sospecha. Tratamiento: Inicial. Seguimiento: Derivar.

Aspectos esenciales    

Exámenes:

El TEP masivo es un grupo particular dentro de la enf. Tromboembólica, con una altísima mortalidad asociada. La evaluación diagnóstica y terapéutica es diferente a la usada en TEP. El riesgo de la arteriografía pulmonar es bajo y válida como alternativa de diagnóstico eficaz. La terapia endoluminal juega un papel importante en cambiar el curso de la enfermedad.

Ante la sospecha de TEP siempre solicitar: gasometría arterial , péptido natriurético cerebral (BNP) y la troponina y dímero-D, así como la electrocardiografía (ECG). Angiografía pulmonar selectiva y la Tomografía helicoidal pulmonar su mayor rendimiento se encontraría en la detección de émbolos a nivel central (tronco de arteria pulmonar, ramas lobares y segmentarias).

Tratamiento Caso clínico tipo La terapia inicial debe estar dirigida a una rápida restitución de la circulación pulmonar, las medidas terapéuticas no invasivas como heparinización y trombólisis sistémica no han cambiado significativamente el curso de la enfermedad. Por otra parte la embolectomía quirúrgica es un procedimiento complejo con una mortalidad cercana al 30% cuando es usada como último procedimiento terapéutico. Técnicas menos invasivas como trombólisis farmacológica (infusiones de 80.000 100.000 unidades/hora de urokinasa (1300 - 1500 U/Kg/hora) pueden ser suficientes) y fragmentación mecánica/trombectomía mediante catéter han mostrado mejores resultados que trombólisis aislada.

Hombre de 40 años, es traído al SU luego de desplomarse en la calle con disnea de inicio súbito, mala perfusión y compromiso de conciencia.

Definición La tromboembolia pulmonar (TEP) es la obstrucción de la arteria pulmonar por un trombo que frecuentemente se origina y desprende del territorio venoso. Se define como TEP masivo en sujetos sin enfermedad cardiopulmonar previa con obstrucción > 50% del lecho vascular pulmonar y en aquéllos con enfermedad cardiopulmonar y obstrucción > 23%, que causa hipotensión (PAS < 90 mmHg o una disminución en la PAS ≥ 40 mmHg desde su basal, por un período > 15 minutos).

Para los pacientes conTEP confirmado que persisten hipotensos y que no tienen un mayor riesgo de hemorragia, se sugiere el tratamiento trombolítico sistémico seguido de la anticoagulación, en lugar de la anticoagulación sola. Una vez que se decide que la terapia trombolítica se justifica, se sugiere que el agente trombolítico administrado por un catéter venoso periférico, en lugar de un catéter arterial pulmonar.Se aconseja un régimen trombolítico con un tiempo de infusión corta (es decir, ≤ 2 horas

Etiología - epidemiología - fisiopatología. Etiología: La causa más frecuente son los trombos hemáticos embolizados desde el sistema venoso profundo de los miembros inferiores (90%), aunque puede deberse a una trombosis in situ.

Para los pacientes que han fracasado con la trombólisis sistémica, o pacientes con alto riesgo de hemorragia, se sugiere la eliminación dirigida por catéter trombo con o sin trombolisis en lugar de no intervención siempre que se disponga. Aunque las directrices sugieren terapias dirigida por catéter en pacientes con riesgo de muerte antes de la terapia sistémica puede manifestarse eficacia (por ejemplo, en cuestión de horas).

Epidemiología: Incidencia anual estimada de TEP: 6070/100.000, causa más frecuente de muerte en cirugías electivas, 15% de las muertes post operatorias. Sin tratamiento la mortalidad es de 20-30%. El 3-4% del total corresponden a TEP masivo, involucrado en el 70% de las muertes asociadas a TEP, con una mortalidad global que bordea el 60-70%. Fisiopatología: El principal efecto que ocurre es el aumento súbito de la resistencia vascular pulmonar, determinado por la obstrucción al flujo por el émbolo. Se produce un brusco aumento de la presión de la arteria pulmonar y un trastorno circulatorio que hace caer rápidamente al paciente en inestabilidad hemodinámica y a claudicación del ventrículo derecho.

Seguimiento Derivar.

16

Diagnóstico

Enfermedad Bronquial Obstructiva Crónica (EPOC) Descompensada

Anamnesis: Evaluar presencia de uno o más de los síntomas cardinales:  

Nivel de manejo del médico general: Diagnóstico: Específico. Tratamiento: Completo. Seguimiento: Completo.

 



de

Al examen físico: Identificar signos respiratorios principales (respiración rápida y superficial, movimientos paradojales de la pared torácica, sibilancias, atenuación o ausencia del MP, hipersonoro a la percusión y respiración bucinada de los labios), signos cardiovasculares (taquicardia u otras arritmias, insuficiencia cardíaca derecha, edema periférico e inestabilidad hemodinámica) y signos generales (compromiso de conciencia y cianosis central).

Aspectos esenciales 

Aumento de disnea y/o de la tos. Aumento del volumen y/o purulencia expectoración.

Cambios agudos en la expectoración, tos o disnea. Identificar causas infecciosas. VM indicada en exacerbaciones que cursan con acidosis respiratoria leve-moderada y aumento del trabajo respiratorio, siempre que el nivel de conciencia sea aceptable. Contraindicación de O2 en EPOC descompensado: PCO2 > 60, PCO2 > 45 + encefalopatía.

Se debe objetivar la severidad de la crisis con gases en sangre arterial, RxTx, exámenes sanguíneos de rutina, ECG y tinción de Gram y cultivo de expectoración. Diagnóstico diferencial: Neumonía, neumotórax, derrame pleural, cáncer pulmonar, obstrucción de la vía aérea alta, fracturas costales, embolía pulmonar, insuficiencia cardíaca izquierda/derecha y sobredosis de drogas sedantes o narcóticos.

Caso clínico tipo

Se debe considerar este cuadro como forma de debutar de pacientes EPOC no previamente conocidos que no están en controles.

Paciente EPOC de 57 años de evolución, con claro aumento de su disnea basal, asociado a aumento de la expectoración, de 2 días de evolución, sin sensación febril.

Tratamiento Definición

Manejo Ambulatorio

Episodio agudo de rápido empeoramiento de los síntomas mas allá de las variaciones diarias normales y que conduce a un cambio en el tratamiento.

1.-Evaluar gravedad 2.-Identificar comorbilidades descompensadas 3.-Oxigenoterapia a volumenes bajos (0,5-1L/min) 4.-Broncodilatadores, Salbutamol 400ug c/ 20 min por 3 veces 5.-Corticoides sistemicos via oral, Prednisona 40mg c/24 hrs por 7 dias 6.-ATB: Amoxicilina por 7 dias.

Etiología - epidemiología – fisiopatología - Etiología: Generalmente de origen infeccioso. Muchas veces el responsable es un virus aunque puede haber presencia de bacterias como H. influenzae, S. pneumoniae y M, catarrhalis en crisis leves o bacilos entéricos Gram negativos y P. aeruginosa en las graves.

Observar por 2 horas minimo. Hospitalización ante:

- Fisiopatología: El curso progresivo de la EPOC se complica con exacerbaciones cada vez más frecuentes. Durante éstas, el flujo espiratorio se reduce, pero el cambio fisiopatológico fundamental es un empeoramiento del intercambio gaseoso, producido primariamente por un aumento de la inequidad de la relación ventilación / perfusión (V/Q). Así, se desarrolla un aumento progresivo del trabajo de los músculos respiratorios, mayor consumo de oxígeno y amplificación de la falla respiratoria. La alteración V/Q en las exacerbaciones es multifactorial: inflamación y edema de la vía aérea, hipersecreción de mucus y broncoconstricción, vasoconstricción hipóxica refleja de las arteriolas pulmonares.

 

    

Necesidad de apoyo ventilatorio. Comorbilidad importante (neumonía, arritmia, insuficiencia cardíaca, diabetes mellitus, enfermedad renal crónica, insufienciencia hepática). Aumento marcado de la disnea. Incapacidad de dormir o alimentarse a causa de los síntomas. Compromiso de conciencia. Falla de respuesta a tratamiento inicial. Mal acceso a cuidados en domicilio, etc.

Expectorantes, mucolíticos y metilxantinas no han demostrado tener beneficio en las exacerbaciones de EPOC.

- Epidemiología: En Santiago la prevalencia de EPOC etapa A es de 11%, B de 1,9%, C de 1,5% y D de 0,2 %. La mitad de los pacientes con EPOC GOLD C morirá en cuatro a cinco años desde el diagnóstico y no más de 50% de los que se hospitalizan por una exacerbación están vivos después de tres años.

Evaluar necesidad de apoyo ventilatorio no invasivo (uso musc. accesoria importante, FR > 25, acidosis y/o hipercarpnia) o de intubación orotraqueal y VM (VMNI no 17

tolerable, inestabilidad hemodinámica, respiración paradójica, PCR, acidosis grave, FR > 35, etc.). Estos pacientes crónicos exacerbados son los que mejor se benefician de la ventilación no invasiva “BiPAP”.

Seguimiento Criterio para alta: Situación próxima a la basal del paciente, el paciente puede deambular por la habitación (si previamente estaba capacitado), puede comer sin disnea, el sueño no es interrumpido por despertares debidos a la misma, estabilidad clínica y gasométrica durante 12 – 24 horas misma, no precisa broncodilatadores inhalados a intervalos menores de 4-6 horas, el paciente y los cuidadores han comprendido el esquema terapéutico, el domicilio está preparado para la llegada del paciente (oxigenoterapia si lo requiere, por ejemplo) y el Centro de Salud tiene conocimiento del alta.

18

Enfermedades del intersticio pulmonar

Comprende hasta un 15% de las patologías neumológicas. En general se puede decir que hay una injuria que genera un daño alveolar difuso, lo que desencadena una reacción inflamatoria y activación fibroblástica, lo cual puede terminar en una reparación o una secuela (fibrosis).

Nivel de manejo del médico general: Diagnóstico Sospecha Tratamiento Inicial Seguimiento Derivar

Hay 5 grupos etiológicos fundamentales: asociadas a enfermedades sistémicas, gatilladas por drogas o factores ambientales, granulomatosas, neumonías intersticiales idiopáticas y otras enfermedades difusas.

Aspectos esenciales

Diagnóstico

  

Se presentan con tos y disnea progresiva, crépitos bibasales y acropaquia. Funcionalmente presentan un patrón restrictivo, disminución de la DLCO e hipoxemia al ejercicio. Radiológicamente se ve un patrón reticular, vidrio esmerilado y panal de abejas.

El reconocimiento inicial de los pacientes generalmente sigue al inicio de la falta de aire progresiva con esfuerzo (disnea), una tos persistente no productiva o síntomas pulmonares asociados con otra enfermedad, como una enfermedad del tejido conectivo, muchas veces puede además ser asintomático. Si es sintomático, suele presentar tos y disnea progresiva debido a la rigidez del intersticio y disminución del volumen corriente. Se debe poner especial atención a la edad de presentación, factores ocupacionales (neumoconiosis y alveolitis alérgica extrínseca), uso de drogas (ciclofosfamida, amiodarona o bleomicina), antecedente tabáquico (bronquiolitis, neumonía intersticial descamativa, fibrosis pulmonar), exposición a aves (alveolitis alérgica extrínseca) o fiebre y hemoptisis (vasculitis, colagenopatías, COP, hemorragia alveolar).

Caso clínico tipo Paciente de 68 años, tabaquismo suspendido hace 5 años IPA 20. Consulta por tos seca sin fiebre hace 8 meses, a la que se agrega disnea progresiva hace 2 meses. No ha bajado de peso, tiene crepitaciones bibasales y saturación 92%. La radiografía de tórax muestra un patrón de “panal de abejas” bibasal.

El historial médico pasado es importante en la evaluación inicial porque la causa de la enfermedad se reconoce a menudo a partir de la historia clínica del paciente. También se presta atención a la historia del tabaquismo, el uso de medicamentos, las exposiciones ocupacionales y ambientales y la aparición familiar de éste tipo de enfermedades.

Definición Las enfermedades pulmonares intersticiales constituyen un grupo muy heterogéneo de entidades caracterizadas por afectar a los espacios alveolo-intersticiales y a la vasculatura pulmonar, que se clasifican conjuntamente debido a manifestaciones clínicas, radiográficas, fisiológicas o patológicas similares. Comprometen universalmente al pulmón (predominantemente al intersticio) y se manifiestan radiológicamente por una extensa alteración del dibujo radiológico normal del pulmón.

Al examen físico pueden encontrarse crepitaciones basales bilaterales, hipocratismo digital y se debe estudiar la piel y buscar fenómeno de Raynaud, pensando en enfermedades sistémicas como la esclerodermia. La espirometría muestra una limitación restrictiva y la gasometría arterial puede mostrar hipoxemia y la gradiente alveolo arterial aumentada, lo que se exacerba en el ejercicio. Además la DLCO se ve reducida. La radiografía de tórax es fundamental, permite apreciar la disminución del volumen pulmonar, la distribución lesional (silicosis y tuberculosis se presentan en los vértices, FPI las bases) y ver algún patrón radiológico, como el vidrio esmerilado, en “panal de abejas” o patrón reticular. Si no es suficiente puede realizarse una TACAR, un lavado broncoalveolar, biopsia transbronquial o quirúrgica.

Etiología-epidemiología-fisiopatología Entre las EPID, es la fibrosis pulmonar idiopática (FPI) la entidad más frecuentemente diagnosticada, habiéndose descrito una prevalencia estimada de 13-20 casos/100.000 h. La tendencia de los últimos años ha sido un incremento en los datos de incidencia y prevalencia de esta entidad que pueden ser atribuidos a numerosos factores como: mejora de las técnicas diagnósticas utilizadas, mayor sensibilidad de los profesionales en el diagnóstico de esta entidad, el mayor envejecimiento poblacional, etc.

Cuadros clínicos típicos: 

Existen factores ambientales o exógenos (exposición a sustancias orgánicas, maderas, metales, agentes infecciosos como los virus, fármacos, etc.) que están involucrados en la patogénesis de la enfermedad, sin olvidar también factores endógenos como el reflujo gastroesofágico, la autoinmunidad, etc. Sin embargo, en la mayoría de estas enfermedades desconocemos su etiología.

19

Fibrosis intersticial idiopática: adulto 50-60 años con historia de disnea que comienza con pequeños esfuerzos, acropaquias, crepitaciones bibasales en “velcro”, patrón restrictivo a la espirometría, acordarse de que pueden dar antecedente de tabaquismo, sin que guarde relación etiológica con el caso. Afecta habitualmente a las bases pulmonares con patrón vidrio esmerilado.

 



 





• Sarcoidosis: nódulos, habitualmente de 5 a 10 mm de localización peribroncovascular; adenopatías hiliares y mediastínicas; predominio en lóbulos superiores

Histiocitosis X: hombre joven, fumador, quistes aéreos en radiografía de tórax, DLCO disminuido, afecta a campos superiores, patrón restrictivo. Linfangioleiomatosis: mujer en edad reproductiva, con neumotórax recurrentes, quistes aéreos, aumento de los volúmenes pulmonares, derrames recurrentes. Afecta principalmente a las bases pulmonares. Exposición a asbesto: calcificaciones pleurales bilaterales en su parte inferior. Si se asocia además, a alteración del parénquima pulmonar, se puede hablar de asbestosis. Guarda relación con exposición a ciertos metales. Su asociación mas característica es con el mesotelioma, pero lo más frecuente como neoplasia asociada es el carcinoma escamoso pulmonar. Silicosis: gente que trabaja con cuarzo, adenopatías en cáscara de huevo, afecta los segmentos superiores pulmonares. Neumonitis por hipersensibilidad: también llamado pulmón del granjero, presentan granulomas, en el LBA aumento de los LT CD8, imágenes en vidrio esmerilado, y obvio exposición a animales de la granja o si es veterinario. Aspergilosis: pensar en ella cuando el paciente tiene antecedente de asma, opacidades en dedo de guante en la radiografía de tórax, bandas atelectasicas e infiltrados alveolares. Sarcoidosis: típico las adenopatías hiliares bilaterales, granulomas no caseificantes, eritema nodoso pretibial, parálisis facial, artralgias, parotidomegalia, hipergamaglobulinemia y aumento del volumen testicular. Es más frecuente en afroamericanos, y levemente en mujeres.

• Histiocitosis X: miconódulos de distribución peribroncovascular que con frecuencia se cavitan formando quistes aéreos, habitualmente de menos de 10 mm, de predominio en los campos medios y superiores; los ángulos costo-frénicos suelen estar respetados • Linfangioleiomiomatosis: espacios quísticos de distinto tamaño distribuidos difusamente, con un parénquima circundante relativamente normal Un electrocardiograma se obtiene típicamente para evaluar evidencias de hipertensión pulmonar o enfermedad cardíaca concurrente. Si se sospecha insuficiencia cardiaca, se mide el nivel sérico de péptido natriurético cerebral (BNP). Un ecocardiograma también se obtiene cuando hay sospecha de insuficiencia cardíaca o hipertensión pulmonar. Se realizan pruebas completas de función pulmonar (espirometría, volúmenes pulmonares, capacidad de difusión) y oximetría de pulso en ejercicio en todos los pacientes con sospecha. A menudo se obtienen gases de la sangre arterial en el aire de reposo para corroborar los hallazgos de la oximetría de pulso. En prácticamente todos los pacientes que presentan hemoptisis y enfermedad intersticial pulmonar el LBA (lavado bronquio alveolar) se realiza rápidamente para confirmar una fuente alveolar de hemorragia e identificar las etiologías infecciosas, si están presentes. En los pacientes con una presentación más crónica, el LBA es menos útil, ya que los hallazgos son típicamente inespecíficos. Sin embargo, cuando se sospecha que la sarcoidosis, la neumonitis por hipersensibilidad, la histiocitosis de Langerhans pulmonar o la infección se basan en el patrón radiográfico, la historia de exposición o los hallazgos clínicos concomitantes, el LBA puede ayudar a reducir el diagnóstico diferencial.

Estudio La evaluación de laboratorio de rutina es a menudo inespecífica, pero debe incluir pruebas bioquímicas para evaluar la función hepática y renal, pruebas hematológicas con recuento sanguíneo diferencial para detectar evidencia de anemia, policitemia, leucocitosis o eosinofilia, análisis de orina y creatina quinasa para miositis

Cuando no es posible hacer un diagnóstico confiable o escoger la enfermedad después de una evaluación inicial no invasiva, puede ser necesaria una biopsia pulmonar con un examen cuidadoso del tejido pulmonar. Esta decisión se toma caso por caso, ponderando la morbilidad del procedimiento, los diagnósticos probables, la toxicidad de la terapia y los valores y preferencias del paciente.

La TACAR se obtiene en casi todos los pacientes con enfermedad parenquimatosa pulmonar difusa. Usualmente se obtienen imágenes tanto en distintas posiciones para evitar confundir atelectasias dependientes con opacidades intersticiales. Las vistas expiratorias son útiles cuando se sospecha una condición asociada con el atrapamiento de aire. Patrones más característicos de la TACAR en la enfermedad pulmonar intersticial difusa. • Fibrosis pulmonar idiopática: infiltrados reticulares de predominio bibasal, periférico y subpleural. Son frecuentes la panalización, la desectructuración del parénquima y las bronquiectasias por tracción

Tratamiento Por especialista, orientado a la etiología. De acuerdo a las teorías fisiopatológicas clásicas que consideran la inflamación pulmonar crónica como un paso previo al desarrollo de éste tipo de enfermedades, el tratamiento generalmente se basa en fármacos antiinflamatorios (glucocorticoides), asociados o no a inmunosupresonres, y en el último tiempo, a inmunomoduladores.

• Neumonía intersticial no específica: densidad en vidrio esmerilado de predominio bibasal y subpleural; engrosamiento de septos interlobulillares e intralobulillares; desestructuración del parénquima; bronquiectasias por tracción; la panalización es menos frecuente que en la fibrosis pulmonar idiopática

Seguimiento Por especialista.

20



Hemoptisis leve y moderada Nivel de manejo del médico general: Diagnóstico Sospecha Tratamiento Inicial Seguimiento Derivar



Aspectos esenciales



   

Síntoma poco frecuente, pero altamente llamativo para paciente y médico. Diagnóstico diferencial con hematemesis. Buscar causa subyacente. Manejo según etiología.

Iatrogenia: Complicaciones de la broncoscopia, intubación traqueal, traqueotomía, complicación de la punción-aspiración con aguja fina, rotura de la arteria pulmonar por catéter de Swan-Ganz, fármacos anticoagulantes, amiodarona, vinblastina, radioterapia y braquiterapia endobronquial, traumatismos torácicos y aspiración de cuerpos extraños. Coagulopatías: Hemofilia, trombocitopenia y disfunciones plaquetarias, leucemia, CID (coagulación intravascular diseminada). Otras causas: Consumo de cocaína, crack, inhalación de humos, amiloidosis, sarcoidosis, neumoconiosis, secuestro pulmonar, broncolitiasis, formas idiopáticas.

Diagnóstico El diagnóstico es fundamentalmente clínico.

Caso clínico tipo

La evaluación de la hemoptisis debe comenzar con la historia inicial, el examen físico y la evaluación de la oxigenación. Las características importantes de la historia incluyen la edad, el historial de tabaquismo, la duración y la cantidad de la hemoptisis y la asociación con síntomas de bronquitis aguda o una exacerbación aguda de bronquitis crónica o bronquiectasia. Se deben detallar bien las características de la expectoración, para así diferenciarla de la hematemesis. Además, para orientar la etiología de la hemoptisis, preguntar por otros síntomas y signos asociados (tos, rinorrea, fiebre, baja de peso), y antecedentes mórbidos del paciente.

Paciente varón de 58 años, fumador activo. Presenta desde hace 30 años tos con expectoración mucosa permanente, teniendo 3 a 4 exacerbaciones al año, donde la expectoración aumenta y se torna purulenta, en ocasiones se acompañan de fiebre (38ºC) y dolor torácico. En los últimos dos episodios, la expectoración ha sido hemoptoica de 30 ml/día.

Definición Emisión por la boca de sangre procedente del aparato respiratorio, que no compromete la vida del paciente dado su intensidad. Según volumen se clasifica en leve (<30 ml/día) y moderada (30-200 ml/día).

En el examen físico se evalúa la condición general del paciente y se buscan signos orientadores de la causa. El estudio inicial más importante, más allá de la historia y el examen físico, para todos los pacientes con hemoptisis es una radiografía de tórax. Los hallazgos anormales pueden sugerir una variedad de causas específicas de hemoptisis, desde neoplasia pulmonar primaria, infección focal (tuberculosis, aspergiloma) hasta estenosis mitral con edema pulmonar.

Etiología 

    





Origen infeccioso: Tuberculosis, neumonías bacterianas y víricas y abscesos pulmonares, infecciones fúngicas: micetoma y aspergiloma, infecciones víricas del parénquima pulmonar y traqueobronquitis, infecciones parasitarias: quistes hidatídicos y otras parasitosis Asma Bronquitis crónica Bronquiectasias Origen tumoral: Carcinoma broncogénico, tumor carcinoide bronquial, adenoma bronquial, metástasis pulmonares Origen vascular: tromboembolismo pulmonar (TEP) e infarto pulmonar, fístulas de la vía aérea con una arteria sistémica, fístulas arteriovenosas, aneurisma de aorta, endocarditis, telangiectasias pulmonares, enfermedad de Rendu-Osler, hipertensión pulmonar, síndrome de vena cava superior. Colagenosis-vasculitis: Hemosiderosis pulmonar idiopática, síndrome de Goodpasture, granulomatosis de Wegener, enfermedad de Churg-Strauss, púrpura de Schölein-Henoch, enfermedad de Behçet, enfermedad mixta del tejido conectivo, lupus eritematoso sistémico. Cardiopatías: Estenosis mitral, edema agudo de pulmón, insuficiencia cardíaca izquierda, síndrome de Eisenmenger.

Los estudios de laboratorio que pueden ser útiles dependiendo de la situación clínica particular incluyen hemoglobina / hematocrito, recuento de plaquetas, un perfil de coagulación, análisis de orina, nitrógeno ureico en sangre, concentración plasmática de creatinina y recolección de esputo para estudios microbiológicos. Un péptido natriurético cerebral de plasma (BNP) puede ser útil si la insuficiencia cardíaca es un contribuyente sospechoso, mientras que una prueba de D-dímero ayudaría a detectar la embolia pulmonar. Si el cuadro clínico no es sugerente de carcinoma (radiografía de tórax negativa, edad inferior a 40 años, antecedentes de no fumar y hemoptisis inferior a una semana de duración), no se indicará tratamiento inmediato posterior, sobre todo si presenta bronquitis aguda (sangrado sobre esputo purulento). Se recomienda una evaluación adicional si el paciente tiene alguno de los factores de riesgo de carcinoma antes mencionados o si la hemoptisis es recurrente o está asociada con anomalías radiográficas. La tomografía computarizada de tórax y la broncoscopia flexible son procedimientos complementarios, aunque la TAC suele realizarse primero, ya que es menos invasiva, tiene una

21

tasa de diagnóstico más alta y puede ayudar a seleccionar procedimientos de muestreo y ubicación. La TACAR es el siguiente procedimiento preferido en pacientes con menor riesgo de tumor o bronquitis crónica pero con antecedentes o radiografías sugerentes de bronquiectasia o malformación arteriovenosa. Para los pacientes con hemoptisis recurrente y una TACAR normal, la broncoscopia flexible puede llevar a un diagnóstico adecuado. Si la broncoscopia es negativa, la pseudohemoptisis ha sido excluida, y el paciente no está en terapia anticoagulante, se considera que el paciente tiene hemoptisis criptogénica.

Tratamiento Hemoptisis leves: Son las únicas que se pueden estudiar de forma ambulatoria, siempre y cuando la radiografía de tórax sea normal y el paciente se encuentre con buen estado general. Pacientes con radiografía de tórax normal: Adoptar medidas generales: reposo relativo, abstención tabáquica. Si la anamnesis es sugerente de bronquitis: examen citológico del esputo, administrar un antitusígeno suave y un antibiótico de amplio espectro, control clínico en una o dos semanas. Sin una sospecha etiológica clara, no prescribir antibióticos. Pacientes con radiografía de tórax anormal: derivar a neumología. Derivar también si es que la hemoptisis no se encuentra dentro del contexto de una bronquitis aguda. Mientras tanto, recomendar medidas generales citadas anteriormente y antitusígenos. Si tras el proceso diagnóstico se descubre la causa inicial de la hemoptisis, se administrará el tratamiento etiológico. Hemoptisis moderadas: En cualquier caso, hospitalización. Se recomienda: reposo en cama en posición semisentada y descanso nocturno en decúbito lateral del lado sangrante, supresión de la tos con antitusígenos (evitando un exceso de sedación), control de las constantes vitales, oxigenación adecuada (puede ser necesaria la gasometría arterial). Monitorizar, ideal intermedio y fibrobroncoscopía antes de las 48 horas como estudio principalmente.

Seguimiento Derivar, por lo menos a un servicio de urgencia de hospital con disponibilidad de laboratorio e imágenes de ser necesarias.

22

La mayoría de las veces la hemoptisis es leve, y sólo en un 5% de los casos es masiva. En este último caso, la mortalidad llega a ser del 80%.

Hemoptisis moderada, severa y masiva

Diagnóstico El diagnóstico es clínico, hay que diferenciarla de la hematemesis (hemorragia digestiva alta) y de la epistaxis posterior (nariz). Un volumen mayor a 600 ml/día o a 150 ml/hora se considera amenazante para la vida. La causa más frecuente de muerte es la asfixia (no el shock hipovolémico).

Nivel de Manejo del médico general: Diagnóstico Sospecha Tratamiento Inicial Seguimiento Derivar.

Aspectos esenciales      

Estudio: funcion renal, analisisi de orina, pruebas sanguineas adicionales ANCA, Anti GBM y ANA si se sospecha hemorragia alveolar difusa.

Causa mas frecuente : Bronquitis Si es fumador de > 45 años: Pensar en neoplasia. Si hay baja de peso, sudoración nocturna y expectoración: TBC. Si hay anemia e infiltrados pulmonares: Hemosiderosis pulmonar. Si hay vasculitis e infiltrados pulmonares: Hemorragia alveolar. Si hay dolor torácico, disnea, hipoxemia: Embolia pulmonar.

En caso de una hemoptisis leve o moderada, se realiza una radiografía de tórax anteroposterior y latera, y baciloscopías de expectoración (2). Según sospecha, se realiza además: TAC o AngioTAC de torax, ECG, hemocultivo. La Fibrobroncoscopía, realizada en las primeras 48 horas, localiza el sangramiento, permite la toma de muestras y maniobras terapéuticas.

Caso clínico tipo

Tratamiento

Hombre de 65 años, fumador activo (40 paquetes año), llega angustiado al servicio de urgencias porque estos últimos meses tose mucho, y hoy “tosió sangre”. Al interrogatorio relata también una pérdida de peso, y un cansancio correspondiente más o menos al principio de la tos.

Hemoptisis de 30-200 ml dia requiere hospitalizacion. Si es 200-600 requiere tratamiento en urgencias. Cuando un paciente se presenta con hemoptisis severa, el manejo inicial es: - Corregir la posición del paciente (decubito lateral ipsilateral a lesion).Si se desconoce posicion de trendelemburg - Establecer una adecuada vía aérea. - Asegurar una buena ventilación y función cardiovascular. - Controlar el sangrado. - Siempre hospitalizar en caso de hemoptisis moderada o masiva, o en caso de hemoptisis leve pero mantenida. - Oxígeno, IOT si necesario, via venosa permeable o catéter central. - Sedantes de la tos (codeína cada 6-8 horas, si no hay contraindicación), corticoides por vía parenteral si hay broncoespasmo asociado.

Definición Expectoracion de sangre por la boca y desde vias respiratorias. Puede ser: Leve: < 30 ml /día. Moderada: 30 – 200 ml/día. Severa: 200 – 600 ml/día. Masiva: > 600 ml /día , > 150 – 200 ml/hr (5%, mortalidad 80%). o cualquier hemoptisis que conlleve a riesgo vital.

Etiología - epidemiología – fisiopatología

Tratamiento etiológico: ATB si infección (o si EPOC o bronquiectasias), anti TBC, corregir eventuales trastornos de la coagulación según corresponda.

El origen puede ser: Pulmonar o Bronquial (mayor magnitud por la presión sistémica). Las causas pueden ser:

Si se localiza el origen de hemorragia, se puede ailsar la hemorragia con un bloqueante endobronquial o sonda endobronquial de doble lumen.En hemoptisis masiva suele ser necesario la broncoscopia rigida.

- Infecciosas (TBC, absceso pulmonar, bronquiectasias infectadas, neumonía necrotizante, bronquitis aguda/crónica). - Neoplásicas (Neo broncogénico, Adenoma bronquial, Carcinoide). - Cardiovasculares (malformaciones, TEP, estenosis mitral, edema pulmonar agudo). - Traumáticas (Heridas torácicas penetrantes, Tórax volante). - Vasculitis (Poliangeitis microscópica, Enfermedad de Wegener, Lupus Eritematoso Sistémico). - Síndrome pulmón-riñón (Síndrome de Goodpasture). - Cocaína. - Coagulopatías. - Biopsias bronquiales.

En la Fibrobroncoscopía, se inyecta suero fisiológico helado y solución de adrenalina en la zona sangrante. Si ésta resulta ineficaz, se realiza una Angiografía con embolización arterial (riesgo de mielitis transversa por embolización de la arteria espinal anterior). La cirugía se utiliza en último recurso,.

Seguimiento Por especialista.

23

 

Hidatidosis pulmonar

Cerebro Otros: riñón, tejido óseo, tejido muscular.

Nivel de manejo del médico general: Diagnóstico Sospecha Tratamiento Inicial Seguimiento Derivar

Aproximadamente el 60% de la enfermedad de la hidatídica pulmonar afecta al pulmón derecho, y del 50 al 60 % de los casos involucran los lóbulos inferiores. Los quistes múltiples son comunes. Aproximadamente el 20% de los pacientes con quistes pulmonares también tienen quistes hepáticos. La relación entre el pulmón y el hígado es mayor en los niños que en los adultos.

Aspectos esenciales

Diagnóstico

    

Zoonosis parasitaria caracterizada por la presencia de quistes hidatídicos. Producido por fecalismo directo o indirecto. Principal ubicación hígado y pulmón. Principal complicación es la rotura hacia los bronquios manifestada por vómica. La cirugía es el tratamiento de elección.

- Etapa subclínica: el quiste hidatídico crece lentamente, a razón de 1 cm por año. El diagnóstico suele ser accidental al realizarse una radiografía de tórax que evidencia un nódulo pulmonar solitario o múltiple en algunas ocasiones. - Etapa clínica: el quiste hidatídico alcanza un diámetro de 5-6 cm. - No complicada: Dolor, tos, expectoración y disnea, disminución de las vibraciones vocales, matidez, disminución del murmullo pulmonar y soplo. Podría evidenciarse insuficiencia respiratoria.

Caso clínico tipo Paciente sexo masculino, 44 años. Refiere hace un día haber expulsado masivamente por la boca un líquido salado con aspecto de hollejos de uva y arenilla. Habita en zona rural.

- Complicada: En orden de frecuencia tenemos las siguientes complicaciones. 

Definición Zoonosis parasitaria caracterizada por la presencia de quistes hidatídicos (larvas enquistadas de Echinoccocus) en el parénquima pulmonar, los que suelen ubicarse en los segmentos posteriores del lóbulo inferior derecho. Quistes pulmonares pueden ser únicos o múltiples, por lo general no se calcifican, rara vez conduce a la formación de quistes hijos y puede contener aire si el quiste se ha roto.

 

Los síntomas más comunes del quiste hidatídico pulmonar descrita en la literatura incluyen tos (53 a 62%), dolor torácico (49 a 91%), disnea (10 a 70%) y hemoptisis (12 a 21%). Los síntomas menos frecuentes incluyen malestar, náuseas y vómitos, y deformaciones torácicas. La mayoría de los niños y adolescentes con lesiones pulmonares son asintomáticos a pesar de tener lesiones de tamaño impresionante, supuestamente debido a una respuesta inmunitaria más débil y la relativamente mayor elasticidad del parénquima pulmonar en relación con los pacientes mayores.



Tratamiento En general, el enfoque para el tratamiento de los quistes hidatídicos pulmonares consiste en la cirugía. Los tejidos infectados deben eliminarse lo más completamente posible, lo que requiere la extirpación completa del tejido parásito y también puede justificar la resección radical del tejido huésped. En general, es aconsejable al menos 2 años de tratamiento con fármacos adyuvantes junto con al menos 10 años de seguimiento de la recurrencia.

Etiología - Epidemiología – Fisiopatología El perro posee en su intestino la forma adulta del Echinococcus. Este parásito genera huevos eliminados a través de las fecas, los que son ingeridos por humanos en forma directa o indirecta (frutas, verduras o aguas contaminadas). En el intestino delgado del hombre eclosionan liberando un embrión hexacanto que pasa a través de las vellosidades intestinales a la circulación venosa para alcanzar los tejidos donde finalmente se ubica. Los tejidos que suelen alojar al parásito son en orden de frecuencia:  

Rotura hacia los bronquios (hidatidoptisis): vómica (líquido salado, restos de membranas, vesículas y arenilla hidatídica) descrito como hollejos de uva que puede acompañarse de hemoptisis. Rara vez mortal. Rotura hacia las cavidades serosas: genera hidatidosis secundaria. Infección del quiste hidatídico (absceso): fiebre y leucocitosis con desviación hacia la izquierda. Cuando la infección se debe a anaerobios hablamos de pioneumoquiste debido a la presencia de aire dentro del quiste. Reacción anafiláctica: aumento de IgE y eosinofilia puede ir desde la urticaria hasta el shock anafiláctico, el cual es infrecuente (1-7%).

- Cirugía: Es el tratamiento de elección, tanto en quistes no complicados como complicados.   

Hígado (65%) Pulmón (10%) 24

Conservador: esterilización y evacuación de todo el contenido. Radical: quistectomía total con o sin resección de parénquima pulmonar. Complicaciones (25-40%): La más frecuente es la ruptura del quiste hidatídico con diseminación a la cavidad pleural y eventual shock anafiláctico. Para prevenir esto se inyecta formalina o yodo en su

interior con lo que se produce muerte de las escólices. Mortalidad: 0-20%. Recidivas: 10%. - Farmacológico: Pacientes inoperables, quistes menores de 7 cm de diámetro, complemento del tratamiento quirúrgico, quistes múltiples (ej., siembra peritoneal). Administración de albendazol 7,5 mg/kg c/12 hrs. Se toma una vez al día luego del desayuno por 3 meses o más (430 días pre cirugía y hasta 4 semanas postquirúrgico). Desaparición: 30%. Degeneración/reducción de su tamaño: 40-50%. Sin cambios morfológicos: 20-30%.

Seguimiento Derivación a especialista.

cirugía

de

tórax.

Seguimiento

por

25

Infecciones de las vías aéreas superiores

El diagnóstico del resfriado común es clínico, basado en los síntomas reportados y / o en los signos observados. Las radiografías sinusales o la tomografía computarizada (TAC) no están indicadas. Alguna forma de anormalidad de la mucosa puede observarse en la tomografía computarizada hasta en el 42% de los individuos asintomáticos. Los cultivos de secreciones nasales no son prácticos ni rentables para identificar pacientes con infección bacteriana. Las secreciones nasales coloreadas no deben considerarse como evidencia de infección bacteriana secundaria.

Nivel manejo del médico general: Diagnóstico Específico Tratamiento Completo Seguimiento Completo

Aspectos esenciales    

- Rinofaringitis (resfrío común): Rinorrea, malestar general y epífora. Puede complicarse con sinusitis bacteriana, otitis media o crisis obstructivas (asmáticos).

Alta prevalencia. Baja morbilidad y autolimitadas. Abuso de antimicrobianos, su mal uso causa más daño que beneficio. Educar al paciente.

- Faringoamigdalitis: Presenta odinofagia, tos irritativa y fiebre, si se agrega exudado amigdalino (fibrinoso o purulento) y adenopatías es altamente sugerente de infección bacteriana (S. pyogenes). - Sinusitis: Rinorrea mucosa o purulenta, dolor local a la presión externa. Sugiere infección bacteriana (S. pneumonie, H. influenzae) cuando existe empeoramiento de los síntomas después de 5 días de evolución, en caso de que los síntomas sean desproporcionadamente severos a pesar de llevar pocos días, o en caso de persistencia de síntomas por más de 10 días.

Caso clínico tipo Paciente de 13 años, es traído por su madre por presentar tos con expectoración mucopurulenta de 5 días de evolución sin haber presentado fiebre ni alteración de otros signos vitales, examen pulmonar normal. La madre pide una receta de antibióticos para su hijo.

- Laringitis: Disfonía y tos irritativa con escasa expectoración. Laringe enrojecida tanto a nivel glótico como supraglótico.

Definición Infecciones que anatómicamente comprometen estructuras respiratorias altas (desde nariz a bronquios).

- Bronquitis: Tos con o sin expectoración purulenta, sin alteraciones de signos vitales y sin alteración del examen pulmonar (descartar neumonía), cuadro menor a 3 semanas de evolución no es necesario solicitar radiografía.

Etiología-epidemiología-fisiopatología

- Influenza: Fiebre de inicio súbito, escalofríos, cefalea, malestar general y mialgias, a lo que se agregan posteriormente síntomas respiratorios.

Epidemiología: El resfriado común es la enfermedad aguda más frecuente en el mundo industrializado, con dos o tres episodios de enfermedad que ocurren anualmente en adultos. Principal causa de ausentismo escolar y laboral, y principal motivo de consulta ambulatoria. En general de escasa gravedad y autolimitadas. Sólo el 2% se complican. Más frecuentes en invierno. Contagio es por contacto directo, fomites o gotitas.

Tratamiento Los pacientes con síntomas leves no requieren a menudo ninguna terapia sintomática. Se debe aconsejar a los pacientes que regresen para su revisión si su condición empeora o excede el tiempo esperado para la recuperación.

Etiología: Más de 200 subtipos de virus se han asociado con el resfriado común. Los rinovirus causan entre el 30 y el 50% de los resfriados. No es posible identificar el patógeno viral probable sobre la base de los síntomas clínicos.

Medidas higiénicas como el lavado de manos pueden prevenir la propagación de virus respiratorios, especialmente en niños más pequeños.

90% virales (rinovirus, virus influenza A y B, virus parainfluenza, virus sincicial respiratorio y adenovirus) y 10% bacterianas u otros (S. pneumonie, H. Influenzae, M. pneumonie, Chlamydia pneumonie, M. catharralis, Streptococcus spp).

Tratamiento sintomático: aseo nasal, abundante ingesta de líquidos, AINEs, paracetamol, para aliviar los síntomas asociados (por ejemplo, dolor de cabeza, dolor de oído, dolores musculares y articulares, malestar y estornudos).

Fisiopatología: La mayoría de las infecciones respiratorias superiores se transmiten por contacto manual. Los síntomas del resfriado común se deben en gran medida a la respuesta inmune a la infección. Los resfriados suelen persistir durante 3 a 10 días en el huésped normal, aunque la enfermedad clínica puede durar hasta dos semanas en un 25% de los pacientes.

Tratamientos específicos: 



Diagnóstico 26

Faringoamigdalitis estreptocócica: penicilina BZT 1.200.000 UI i.m. por 1 vez. Alternativa: amoxicilina por 10 días, 1 g c/12 hrs o 750 mg c/8 hrs. Sinusitis bacteriana: amoxicilina por 14 días, 1 g c/12 hrs (primera línea); amoxicilina con ácido

 



clavulánico; quinolonas; macrólidos (los últimos en alérgicos a betalactámicos). Laringitis: reposo vocal. Influenza en grupos de riesgo (cuadro grave, ancianos, patología crónica, inmunosuprimidos, embarazadas, edades extremas, casas de reposo y trabajadores de éstas): oseltamivir 75 mg VO c/12 hrs x 5 días, sólo dar en caso de que el cuadro haya iniciado antes de las 48 hrs o en cuadros graves. Los mucolíticos no tienen efecto favorable y no están recomendados para la bronquitis aguda (grado de recomendación I).

Seguimiento No requieren controles posteriores salvo evidencia de complicación.

27

que resulta en hipercapnia y, secundariamente, en hipoxemia (insuficiencia respiratoria global).

Insuficiencia respiratoria aguda o crónica reagudizada que requiere ventilación mecánica

Causas pulmonares y extrapulmonares: intoxicación por opiáceos o barbitúricos, síndrome de Guillain-Barré, miastenia gravis, cifoescoliosis, tórax volante, etc. Con frecuencia, estos mecanismos se combinan en un mismo sujeto.

Nivel de manejo del médico general: Diagnóstico: Específico. Tratamiento: Completo. Seguimiento: Completo.

- Mecanismo no pulmonar: Llegada de sangre con muy bajo contendido de O2 (PO venosa mezclada con PpO2 bajo), en que leves alteraciones del aparato respiratorio hacen imposible alcanzar niveles adecuados de PO2, este fenómeno acontece en la Insuficiencia Cardíaca, en síndrome de bajo débito en que existe un tiempo mayor de extracción de O2. Disminucion de aire inspirado, por ejemplo en individuos que asciende a gran altitud.

Aspectos esenciales    

La instalación aguda no permite generar mecanismos de adaptación. Mecanismos pulmonares: Falla del intercambiador, falla de la bomba. El análisis del intercambio de gases permite hacer una orientación etiológica. En el tratamiento debe corregirse la hipoxemia a niveles normales.

Diagnóstico Las manifestaciones clínicas, derivan de hipoxemia e hipercapnia, pero son inespecíficas y poco sensibles. Signos y síntomas: Disnea, taquipnea, cianosis de tipo central, alteración de la PA (hipertensión, y en estadios más avanzados hipotensión), taquicardia, arritmias, somnolencia, fatiga, cefalea, náuseas y vómitos, sensación de euforia, confusión y alteración de conciencia, hasta llegar al coma, sudoración, temblores y mioclonías. Si hay respiración paradójica ésta denota fatiga muscular, y no es signo patonogmónimo de insuficiencia respiratoria.

Caso clínico tipo Paciente 69 años, tabáquico y con EPOC en tratamiento. Ingresa con historia de varios días de tos con escasa expectoración, disnea progresiva sin fiebre. Al examen físico: disneico, taquicárdico, taquipneico, normotenso, cianosis de lechos ungueales, yugulares planas, espiración prolongada, sibilancias difusas, disminución del MP, sin crépitos.

Exámenes: GSA: PpO2, PCO2, pH (identificar acidosis o alcalosis respiratoria), alcalosis o acidosis metabólica y cálculo de la gradiente. (PAlveolarO2= FiO2 (P. atmosférica x PH2O) x PCO2 /Cuociente respiratorio). VN del CR: 0,8. Luego, a la PA02 calculada se le resta a la Parterial O2. El gradiente normal resulta de la multiplicación de edad por 0.3 que es la fórmula más usada.

Definición Falla del aparato respiratorio en su función de intercambio de gases, con PO2 < 60 mmHg (IR hipoxémica pura), acompañado o no de una PCO2 ≥ 50 mmHg (IR global), respirando aire ambiental y a nivel del mar. Se instala bruscamente.

Cálculo de la Pa/FIO2, nos permite comparar al sujeto consigo mismo, no permite monitorizar la falla respiratoria. No es confiable en: EPOC por alteración V/Q predominante, en fluctuaciones en la extracción periférica de O2 (CaO2 – CvO2), especialmente en situaciones de baja extracción, y en zonas de altura.

Etiología - epidemiología – fisiopatología Puede deberse a: - Falla del intercambiador: por alteración de la superficie alveolar (neumonía, edema pulmonar, atelectasia, enfisema pulmonar, fibrosis pulmonar difusa) o a déficit de la ventilación por obstrucción de las vías aéreas.

En pacientes adultos mayores, la causa de la Insuficiencia muchas veces está retardada o mal diagnosticada, por lo que se recomiendan además de la clínica, gases y el criterio médico: medición de Péptidos natriurético, ecocardiografía BD más doppler, Rx y AngioTAC según sospecha.

Mecanismos fisiopatológicos: alteración V/Q y alteración de difusión (corrigen con O2), y Shunt (no corrige con O2). Se caracteriza por aumento de la diferencia alvéolo-arterial de oxígeno que conduce a hipoxemia. Salvo en los casos más avanzados, la PaCO2 es generalmente normal o baja, porque la hipoxemia, junto a la estimulación de receptores del parénquima pulmonar, aumenta compensatoriamente la ventilación alveolar.

Tratamiento Depende del tipo e instalación de IR. En el caso de la aguda se debe llegar a niveles de oxígeno normales. 2 ejes a recordar: Soporte con oxígeno-ventilación mecánica para evitar la muerte del paciente y Etiológico apuntando a causa.

- Falla de la bomba: (Tórax y músculos respiratorios, centros respiratorios y los nervios): por hipoventilación

28









Soporte: Según gravedad oxígeno suplementario. Titulado por saturación, evitar hiperoxia con depresión en pacientes crónicos. Si no se titula, aumenta la mortalidad en EPOC. Ventilacion mecanica no invasiva:Existen 3 tipos: o 1.-Ventilacion ciclada a volumen: se establece el volumen que se quiere aportar al paciente: eleccion en pacientes con alteraciones restrictivas. o 2.-Ventilacion ciclada a presion: Presion constante durante toda la inspiracion. De eleccion en hospitales y en insuficiencia respiratoria aguda o 3.-Ventilacion con presion positiva continua (CPAP) : se fija los niveles de presion, con ventilacion espontanea. Ventilacion mecanica invasiva: Requiere intubacion o Si requiere, por peor oxigenación, uso de soporte ventilatorio: VMNI, según intercambio o mecánica, en EPOC con falla respiratoria aguda global, pH 7,25 a 7,35; en inmunosuprimido con falla respiratoria, en EPA, mala en asmáticos. Si paciente tiene mayor deterioro, pH menor 7,2, no protege vía aérea, intubar y VMI. En distrés también instalar VMI. En casos extremos de falla respiratoria catastrófica con PaFi menor a 100: ventilación en prono, VAFO. Falla respiratoria: ECMO. Etiológico: Manejo broncodilatador, corticoides en obstructivos, antimicrobianos en neumonía, soporte en causas secundarias de distrés, manejo de EPA habitual.

Entorno de manejo: Suele ser al menos intermedio o UCI. Siempre recordar: Tratamiento guiado por mecánica, gases y causa. Usar los gases arteriales siempre que hay sospecha.

Seguimiento Por especialista, según gravedad.

29

a) Una disminución en la presión de oxígeno en el aire inspirado, b) Una hipoventilación alveolar, c) Una alteración en la capacidad de difusión alvéolocapilar, d) Un desequilibrio en la relación entre la ventilación y la perfusión pulmonares (el mecanismo más frecuente) y e) La existencia de un cortocircuito de derecha a izquierda.

Insuficiencia respiratoria aguda o crónica reagudizada, leve o moderada (que no requiere ventilación mecánica)

Pudiendo corresponder a una falla del pulmón como órgano intercambiador de gases o a una deficiencia de la bomba ventilatoria.

Nivel de manejo del médico general: Diagnóstico Específico Tratamiento Inicial Seguimiento Derivar.

Diagnóstico

Aspectos esenciales   

Sospecha clínica confirmada por: Gasometría Arterial.

Diagnóstico con Gases Arteriales (PaO2 < 60mmHg y/o PaCO2 ≥ 50 mmHg). Tratamiento: Oxigenoterapia + optimizar transporte de O2 y ventilación. Prioridad del tratamiento: Lograr Saturación ≥ 90%

a) Clínica: Disnea, Taquipnea de instalación brusca y mal tolerada (por cuadro agudo o reagudizado), Taquicardia, Cianosis (aparece con PO2 < 50 mmHg), Arritmias, Alteraciones de conciencia, coma. Manifestaciones de: Hipoxia: taquicardia, hipohipertensión, ángor, cianosis, ansiedad, convulsiones. Hipercapnia: hipotensión, vasodilatación, edema, cefalea, edema papilar, asterixis, etc.

Caso clínico tipo Paciente de 69 años, EPOC en tratamiento. Ingresa por varios días de tos con escasa expectoración, disnea progresiva sin fiebre. Disneico, FC 110 lpm, FR 32 rpm, normotenso, cianosis, espiración prolongada, sibilancias difusas, disminución difusa del MP, sin crépitos. Hemograma: 13500 blancos, Gases: PO2 50 , PCO2 52, PH 7.412. BE: 6.4 PaO2/FiO2 = 238.

b) Gases arteriales: PCO2 ≥ 50 mmHg (I.R Hipercápnica); PO2 < 60 mmHg (I.R Hipoxémica), cuando ambas coexisten I.R Global.

Tratamiento

Definición

La clave del tratamiento de la I.R es la resolución de la enfermedad subyacente que la desencadenó.

Incapacidad del aparato respiratorio para mantener los niveles arteriales de O2 y CO2 adecuados para las demandas del metabolismo celular. Por convención con PO2 < 60 mmHg y/o PCO2 ≥ 50 mmHg en GSA. Según su velocidad de instalación y condición basal del aparato respiratorio se clasifican en:

Medidas: a) Corrección de la hipoxemia con oxigenoterapia con PO2 < 60 mmHg o PaFi < 300, incrementar la saturación cercana al 90% (para pacientes crónicos retenedores de CO2) o > a 91% (pacientes previamente sanos) es la mayor prioridad en el manejo.

- Aguda (rápida sobre pulmón sano, no permite instalación de mecanismos adaptativos),

b) Reducción de los requerimientos de oxígeno: Tratar la fiebre, agitación, sepsis, respiración dificultosa conlleva marcada disminución del consumo de oxígeno.

- Crónicas (paulatina, bien tolerada) y - Crónica reagudizada (trastorno agudo que altera la condición crónica del paciente).

c) Optimizar transporte de Oxígeno normalizando el gasto cardíaco y glóbulos rojos.

Si cursa sólo con hipoxemia se denomina parcial, si cursa con hipercapnia asociada, se denomina global.

Seguimiento

Etiología - epidemiología – fisiopatología

Por especialista.

Etiología: La IRA puede deberse a una gran variedad de procesos, tanto de localización intrapulmonar como extrapulmonar. Entre las causas más representativas se encuentran la neumonía, el edema agudo de pulmón y el síndrome del distrés respiratorio agudo. La IRC es la consecuencia final de una gran variedad de enfermedades y trastornos respiratorios y no respiratorios (cardíacos, nerviosos, infecciosos, etc.). Sin embargo, la causa de insuficiencia respiratoria crónica más frecuente es la enfermedad pulmonar obstructiva crónica. Fisiopatología: La insuficiencia respiratoria (I.R) puede deberse a:

30

Insuficiencia respiratoria crónica

alveolar. La hipoventilación resultante se traduce en hipercapnia y, secundariamente, en hipoxemia. Existen dos razones básicas por las cuales se produce una falla de la bomba:

Nivel de manejo del médico general: Diagnóstico Específico Tratamiento Inicial Seguimiento Derivar

Una es la alteración primaria del sistema motor, que puede derivar de una incapacidad del centro respiratorio para generar impulsos a los músculos respiratorios (intoxicación por opiáceos o barbitúricos), de un trastorno en la conducción y transmisión de estos impulsos (síndrome de Guillain-Barré, miastenia gravis), o de un defecto mecánico de los componentes de la caja torácica (cifoescoliosis, tórax volante).

Aspectos esenciales 



Incapacidad del aparato respiratorio para mantener los niveles arteriales de O2 y CO2 adecuados para las demandas del metabolismo celular. Convencionalmente se acepta 60 mmHg como límite inferior para la PaO2 y 49 mmHg como límite superior para la para la PaCO2.

La otra es la falla secundaria de la bomba, debida a fatiga de la musculatura inspiratoria, como consecuencia de un desbalance entre la energía y fuerzas disponibles para generar la ventilación y la existencia de demandas o resistencias aumentadas. Con frecuencia, estos mecanismos se combinan en un mismo sujeto.

Caso clínico tipo Causas:

Paciente varón de 76 años, con EPOC secundario de consumo de tabaco, presenta una capacidad vital forzada [FVC] 1,21 litros (37%), volumen máximo espirado en el primer segundo de una espiración forzada [FEV1] 0,5 litros (23%, FEV1/FVC 42%). Se realizan gases arteriales que muestran una PaO2 en reposo de 52 mmHg.

 

Definición Incapacidad del aparato respiratorio para mantener los niveles arteriales de O2 y CO2 adecuados para las demandas del metabolismo celular. Se produce por una pérdida paulatina de la función respiratoria, por lo que el organismo tiene tiempo para poner en marcha mecanismos de adaptación. No obstante, estos pacientes tienen sus reservas funcionales disminuidas o agotadas, lo que les dificulta soportar exigencias o enfermedades agregadas.

   

Etiología-epidemiología-fisiopatología 

La insuficiencia respiratoria puede deberse a una falla del pulmón como órgano intercambiador de gases o a una deficiencia de la bomba ventilatoria, esta bomba está constituida por el tórax, los músculos que cambian el volumen pulmonar y los centros respiratorios que controlan estos músculos y los nervios que los interconectan.

Enfermedades que cursan con obstrucción bronquial: EPOC, bronquiectasias, fibrosis quística, asma bronquial (raras veces) Enfermedades pulmonares intersticiales crónicas, entre otras fibrosis pulmonar idiopática, sarcoidosis, neumoconiosis, fibrosis postinflamatorias (después de tuberculosis pulmonar u otras neumonías de etiología no tuberculosa) Neoplasias del sistema respiratorio primarias y metastásicas Deformidades del tórax (más frecuentemente cifoscoliosis grave) Obesidad mórbida Enfermedades del sistema nervioso y muscular: esclerosis lateral amiotrófica, esclerosis múltiple, enfermedad de Parkinson, polineuropatías crónicas, daño permanente postraumático de los nervios frénicos o de la médula espinal cervical o torácica, miopatías crónicas (distrofias musculares) Enfermedades del sistema cardiovascular: tromboembolismo pulmonar crónico, cardiopatías cianógenas, insuficiencia cardíaca crónica.

Diagnóstico Encontramos a un paciente con síntomas como: disnea de esfuerzo o de reposo, disminución de la tolerancia al esfuerzo, somnolencia y cefalea, se debe indagar otros síntomas que nos orienten a la enfermedad de base.

- Falla del intercambiador: se puede deber a condiciones que afectan a la superficie de intercambio (neumonía, edema pulmonar, atelectasia, enfisema pulmonar, fibrosis pulmonar difusa) o a déficit de la ventilación alveolar por obstrucción de las vías aéreas. Se manifiesta básicamente por un aumento de la diferencia alvéoloarterial de oxígeno que conduce a hipoxemia. Salvo en los casos más avanzados, la PaCO2 es generalmente normal o baja, porque la hipoxemia, junto a la estimulación de receptores del parénquima pulmonar, aumenta compensatoriamente la ventilación alveolar.

Podemos encontrar en nuestro examen físico: taquipnea, taquicardia, cianosis, dedos en palillo de tambor, enrojecimiento de las conjuntivas y de la piel, hipertrofia de músculos accesorios, tórax en posición inspiratoria. Dado que los mecanismos de compensación y adaptación son muy eficientes, existe gran tolerancia del organismo a la hipoxemia e hipercapnia si éstas se desarrollan lentamente, de modo que el diagnóstico de la insuficiencia respiratoria en estas condiciones sólo puede hacerse mediante el análisis de los gases arteriales.

- Falla de la bomba: se caracteriza por una incapacidad del sistema motor respiratorio para generar la fuerza necesaria para mantener una adecuada ventilación 31

Convencionalmente se acepta 60 mmHg como límite inferior para la PaO2 y 49 mmHg como límite superior para la para la PaCO2.

Estudio Radiografía de tórax, espirometría, gasometría en sangre arterial, y otras pruebas auxiliares dependiendo de la enfermedad sospechada. Además para buscar las complicaciones inherentes a la enfermedad se pueden realizar: estudio morfológico en sangre periférica (policitemia), ECG y opcionalmente ecocardiografía (HAP e Insuficiencia cardiaca derecha).

Tratamiento Tratamiento del cuadro que provoca la descompensación. En la insuficiencia respiratoria crónica la oxigenoterapia debe ser considerada cuando el tratamiento específico de la enfermedad no ha logrado modificarla. Estudios han demostrado que la corrección de la hipoxemia mediante oxigenoterapia continua, prolonga la duración y calidad de vida en pacientes con EPOC y, por analogía, estos hallazgos se han considerado aplicables a otras causas de insuficiencia respiratoria crónica. De acuerdo a estos estudios, su empleo está indicado cuando a pesar de un tratamiento médico óptimo, la PaO2 se mantiene bajo 55 mmHg después de un plazo de observación de al menos un mes, o cuando al estar entre 55 y 60 mmHg se asocia a hipertensión pulmonar, corpulmonale o policitemia (hematocrito >55%). En tales situaciones, la oxigenoterapia debe ser idealmente continua o cubrir al menos 18 horas diarias. Complicaciones oxigenoterapia: atelectasia por reabsorción del O2 en el alveolo (normalmente el nitrógeno es el que mantiene abierto el alveolo). Toxicidad por O2 (producción de radicales libres). Si se corrige la hipoxemia puede disminuir el estímulo respiratorio y aumentar la retención de CO2, por otra parte el efecto Haldane ocurre al aumentar la oferta de O2, este baja la afinidad de la Hb CO2, aumentando la cantidad de CO2 disuelto en la sangre. Se debe realizar fisioterapia respiratoria (rehabilitación física, entrenamiento físico) para obtener una mejoría general y educacar al enfermo y sus familiares, que juegan un papel fundamental en el tratamiento de la insuficiencia respiratoria crónica. Otras consideraciones: Ajustar dieta para prevenir desnutrición, con reducción de la cantidad de carbohidratos, para disminuir la producción del CO2. Ventilación mecánica crónica no invasiva en domicilio, sólo en casos seleccionados.

Seguimiento Por especialista

32

Fisiopatogenia: La cantidad de gas absorbida depende de la ventilación minuto, duración de la exposición y la concentración relativa de CO y O2 ambiental. Las moléculas del gas e une a proteínas (Hb, mioglobina y citocromos) y <1% circula disuelto.

Intoxicación leve por CO y otros gases

La afinidad de Hb por CO es 200-250 veces mayor que por O2, lo que se traduce en una desviación a izquierda de la curva de disociación de la Hb. Esto disminuye la entrega de O2 a los tejidos y conduce a hipoxia celular. El cerebro y el corazón pueden verse gravemente afectados después de una exposición a niveles superiores al 20% de CO-Hb. Donde en el primero predominan lesiones en corteza cerebral, sustancia blanca y ganglios basales; mientras que en el segundo pueden exacerbar patología cardíaca.

Nivel de manejo médico general: Diagnóstico: Específico. Tratamiento: Completo. Seguimiento: Completo.

Aspectos esenciales    

Alta sospecha clínica. Llamado “cefalea de invierno”. Administrar O2 100% por al menos 6 horas. Benzodiazepinas EV ante convulsiones.

Diagnóstico Los hallazgos clínicos del envenenamiento por CO son altamente variables y en gran medida inespecíficos. Los pacientes con intoxicación leve a moderada a menudo presentan síntomas constitucionales, incluyendo cefalea (más común), malestar general, náuseas y mareos, y pueden diagnosticarse erróneamente con síndromes virales agudos. En ausencia de traumas o quemaduras concurrentes, los hallazgos físicos en el envenenamiento con CO suelen limitarse a alteraciones en el estado mental, que van desde una confusión leve hasta convulsiones y coma. Un examen neurológico cuidadoso es crucial.

Caso clínico tipo Paciente de 13 años, mientras cocinaba con un calefactor a querosén presentó cefaleas, náuseas y pérdida del conocimiento de duración desconocida. Fue encontrada por hermano, quién la trasladó al SU. Al examen: lúcida, eupnéica, Glasgow 15, cardíaco: RR2T 80 lpm.

Definición Cuadro clínico secundario a inhalación de gases que pueden ser inertes (nitrógeno y metano) los cuales desplazan al oxígeno en su unión con hemoglobina, tóxicos que interfieren con el metabolismo celular (CO) o irritantes (amoníaco, cloro) que lesionan la vía aérea.

A. Clínica: Síntomas iniciales: cefalea y polipnea leve, se agregan náuseas, irritabilidad, aumento del nivel de dolor de la cefalea, polipnea intensa, dolor torácico, confusión y cianosis.

La intoxicación por monóxido de carbono (CO) es la más común, potencialmente fatal y probablemente subdiagnosticada debido a su presentación clínica inespecífica, que, de no tratarse oportunamente, puede dejar considerables secuelas neurológicas o incluso provocar la muerte del paciente.

La exposición prolongada y en concentraciones elevadas puede generar: compromiso de conciencia, insuficiencia respiratoria y muerte si la exposición dura >1 hora. Los signos clásicos de labios color cereza, cianosis y hemorragia retinal sólo en 2 a 3% de los casos. B. Laboratorio:

Etiología-epidemiología-fisiopatología

 Etiología: El CO es producto de la combustión incompleta de los hidrocarburos. Las concentraciones de carboxihemoglobina normalmente son entre 1 a 3% y en los fumadores va entre el 10% a 15%.



Epidemiología: Intoxicación por CO es una importante causa de morbimortalidad a nivel mundial, secundaria a toxicidad cardíaca y neurológica, depresión respiratoria y acidosis metabólica.



El envenenamiento por CO es más común durante el invierno en climas fríos, pero puede ocurrir en todas las estaciones y ambientes. La inhalación de humo es responsable de la mayoría de los casos no intencionales. Otras fuentes potenciales de CO incluyen sistemas de calefacción con malfuncionamiento, dispositivos de combustión con ventilación inadecuada (por ejemplo, calentadores de queroseno, parrillas de carbón, estufas de camping, generadores eléctricos a gasolina) y vehículos motorizados que operan en áreas mal ventiladas.

 



33

Glicemia: Descartar hipoglicemia como causa de compromiso de conciencia. Concentración de CO-Hb en sangre arterial o venosa: No siempre hay correlación con la severidad del cuadro. Valores sobre 25% se consideran tóxicos. GSA (preferir en sangre arterial): Para cuantificar saturación de Hb y el grado de acidosis metabólica. La saturación periférica (oxímetro de pulso) da un valor falsamente normal, pues CO-Hb tiene un espectro de luz muy similar al de O2Hb. Lactato: marcador de daño tisular. ECG: Puede ocurrir isquemia cardíaca. La evaluación de biomarcadores cardíacos se justifica en pacientes con evidencia de ECG de isquemia, síntomas sugerentes de isquemia, edad superior a 65 años, antecedentes de enfermedad cardiaca o factores de riesgo cardíaco. Realización de pruebas diagnósticas de imagen (TAC craneal), ECG y seriación de enzimas cardíacos en urgencias es conveniente.

Tratamiento ABC de la reanimación. 1. Descontaminación: Remover a la víctima de la zona de exposición. 2. O2 al 100% vía máscara facial (sin bolsa de recirculación) debe iniciarse apenas se sospecha el diagnóstico, en pacientes sin indicación de O2 hiperbárico. Mantener por un mínimo de 6 horas. Si está inconsciente: intubar y ventilar con O2 al 100%. En caso de persistencia de sintomatología, considerar la derivación a oxigenoterapia hiperbárica. 3. Cámara hiperbárica: en pacientes con compromiso de conciencia, signos neurológicos, disfunción cardiovascular, evidencias de isquemia (ECG, dolor torácico, alteración de estado mental), exposición mayor de 24 horas, nivel de CO-Hb mayor a 25%, nivel de CO-Hb mayor a 20% en embarazadas, acidosis metabólica severa (pH <7,1) 4. Tto. convulsiones: Benzodiazepinas ev., diazepam (adulto: 10-20 mg repetir cada 10-15 min si es necesario, niños: 0,2-0,5 mg/kg cada 5 min si es necesario), lorazepam (adulto: 2-4 mg, niños: 0,05-0,1 mg/Kg). Considerar fenobarbital o propofol si no resultan estas medidas. Monitorizar: hipotensión, arritmias, depresión respiratoria (intubación). 5. Medidas generales: Mantener T° corporal normal. Mantener PA. Control de niveles de CO-Hb cada 2-4 hrs. Tratamiento del edema cerebral.

Seguimiento Mantener en reposo 2-4 semanas para minimizar las complicaciones neurológicas. Muchos pacientes con síntomas leves de un envenenamiento no intencional pueden ser manejados en urgencias y ser dados de alta con seguridad. Los pacientes cuyos síntomas no se resuelven, que demuestren ECG o evidencia de laboratorio de intoxicación grave, o que tengan otra causa médica o social de preocupación deben ser hospitalizados. La evaluación psiquiátrica debe ser obtenida para todos los pacientes con envenenamiento autoinfligido de CO.

34

proteínas (Hb, mioglobina y citocromos) y < 1% circula disuelto. La afinidad de Hb por CO es 200-250 veces mayor que por O2, lo que se traduce en una desviación a izquierda de la curva de disociación de la Hb. Esto disminuye la entrega de O2 a los tejidos y conduce a hipoxia celular.Produce un decenso en el transporte de oxigeno con PaO2 y SatO2 normales y alteracion en la respiracion celular.

Intoxicación por monóxido de carbono Nivel de manejo médico: Diagnóstico Específico Tratamiento Completo Seguimiento Completo.

Diagnóstico Lo más importante es la alta especialmente durante el invierno.

Aspectos esenciales    

Alta sospecha clínica. Llamado “Cefalea de invierno”. Administrar O2 100% por al menos 6 hr. Benzodiazepinas EV ante convulsiones.

sospecha

clínica

Síntomas inespecíficos. Según el grado de intoxicación pueden ir de leves a severos. Síntomas: cefalea y polipnea leve, se agregan náuseas, irritabilidad, aumento de la cefalea, polipnea intensa, dolor torácico, confusión(30-40%), somnoliencia (10-30%) debilidad muscular, color rojo cereza de piel y mucosas, incluso edema pulmonar y coma.

Caso clínico tipo

Laboratorio:

Paciente de 13 años, mientras cocinaba con un calefactor a querosén presentó cefaleas, náuseas y pérdida del conocimiento de duración desconocida. Fue encontrada por hermano, quién la trasladó al SU. Al examen físico: Lúcida, eupnéica, Glasgow 15, Cardíaco: RR2T 80 lpm.

Glicemia: Descartar hipoglicemia como causa de compromiso de conciencia. Concentración de CO-Hb en sangre arterial o venosa: No siempre hay correlacion con la severidad. Valores sobre 25% se consideran tóxicos.

Definición

GSA (preferir en sangre arterial): Para cuantificar saturación de Hb y el grado de acidosis metabólica. La saturación periférica (oxímetro de pulso) da un valor falsamente normal, pues COHb tiene un espectro de luz muy similar al de O2Hb.

Cuadro clínico secundario a inhalación de gases, que pueden ser: a) Inertes: Asfixia por desplazamiento de O2 del aire inspirado: Nitrógeno, Metano.

Lactato: marcador de daño tisular.

b) Tóxicos: Interfieren con el metabolismo celular, siendo el pulmón sólo la vía de entrada: CO2, CO [más frecuente] gas tóxico no irritante, inodoro e incoloro que se absorbe fácilmente por los pulmones, producto de la combustión incompleta de los hidrocarburos (tubos de escape de autos, sistemas de calefacción en mal estado, humo).

ECG: Fundamentalmente en busca de alteraciones isquémicas. En pacientes inconscientes, con dolor torácico o con síntomas significativos o COHb > 20%. Solicitar CK y CK-MB en pacientes mayores de 65 años, con factores de riesgo cardiovascular significativos o pacientes jóvenes con dolor torácico o síntomas sugerentes de isquemia.

c) Irritantes: Los más solubles (amoníaco, cloro, etc.) atacan rápidamente las vías superiores (apnea, tos, estornudos). Si la inhalación es masiva pueden llegar a alvéolos (neumonitis graves). Los menos solubles (fosgeno, óxidos de nitrógeno, anhídrido sulfuroso) llegan a los alvéolos, donde forman lentamente los productos nocivos causando neumonitis químicas horas post exposición, por lo que pueden confundirse con neumonías infecciosas.

Considerar TC de cerebro en pacientes con alteraciones del estado mental, para descartar otras etiologías.

Tratamiento ABC de la reanimación.

Etiología - epidemiología – fisiopatología

1) Descontaminación: Remover a la víctima de la zona de exposición.

Epidemiología: La intoxicación por CO es una importante causa de morbimortalidad a nivel mundial. En USA se estiman 600 muertes al año y en Inglaterra alrededor de 50 muertes al año. La mayoría de las muertes ocurren durante la época de invierno. La muerte ocurre principalmente por toxicidad cardíaca y neurológica, depresión respiratoria y acidosis metabólica.

2) O2 al 100% vía máscara facial (sin bolsa de recirculación) debe iniciarse apenas se sospecha el diagnóstico, en pacientes sin indicación de O2 hiperbárico. Mantener por un mínimo de 6 horas. Si está inconsciente: intubar y ventilar con O2 al 100%. En caso de persistencia de sintomatología, considerar la derivación a oxigenoterapia hiperbárica.

Fisiopatología: Cantidad de gas absorbida depende de la ventilación minuto, duración de la exposición y la concentración relativa de CO y O2 ambiental. Se une a

3) Cámara hiperbárica: A todo paciente que presente convulsiones, coma, compromiso grave de conciencia o 35

isquemia miocárdica, aunque no se haya cuantificado la COHb. Considerar en mujeres embarazadas y niños. Indicación relativa: COHb sanguínea 30 a 35 % (importantes las condiciones clínicas y patologías agregadas) e indicación absoluta: sobre 40%. Se indica también con niveles de CO > 25% o 20% si está embarazada y Acidosis metabólica severa (pH < 7,1). 4) Tto. Convulsiones: Benzodiazepinas ev. DIAZEPAM (ADULTO: 10-20mg repetir cada 10-15 min si es necesario, NIÑOS: 0,2-0,5mg /Kg cada 5 min si es necesario) LORAZEPAM (ADULTO: 2-4mg, NIÑOS: 0,05 - 0,1/Kg). Considerar fenobarbital o Propofol si no resultan estas medidas. Monitorizar: hipotensión, arritmias, depresión respiratoria (intubación). 5) Medidas generales: Mantener T° corporal normal. Mantener PA. Control de niveles de COHb cada 2-4 hrs. Tratamiento del edema cerebral.

Seguimiento Mantener en reposo 2-4 semanas para minimizar las complicaciones neurológicas.

36

Metástasis Pulmonares

La TACAR suele ser la primera prueba diagnóstica realizada después de radiografía de tórax convencional, con el fin de caracterizar múltiples nódulos pulmonares. El aspecto radiográfico de las metástasis pulmonares suele ser el de nódulos densos, bien definidos, localizados fundamentalmente en la periferia y sin tractos lineales asociados

Nivel de manejo del médico general: Diagnóstico: Inicial. Tratamiento: Inicial. Seguimiento: Derivar.

Una vez que los nódulos se han caracterizado por TAC, la información radiográfica debe ser considerada en el contexto de la historia y el examen físico. Esto suele ser suficiente para reducir el diagnóstico diferencial y determinar si los nódulos pueden ser seguidos sólo con imágenes.

Aspectos esenciales    

Neoplasia pulmonar más frecuente. Tumores que suelen metastizar a pulmón: sarcomas, mama, colon, genitourinarios, melanoma. La sospecha es radiológica. Resección sólo si el primario está controlado, no hay extensión extrapulmonar y la capacidad funcional del paciente tolerará la resección.

Es de alta utilidad recurrir a estudios previos buscando cambios. La resonancia nuclear magnética del tórax y la tomografía con emisión de positrones pueden ser también utilizadas. El diagnóstico de certeza es histológico (biopsia).

Caso clínico tipo Paciente de sexo femenino de 43 años de edad, con diagnóstico de sarcoma uterino. Se realiza estudio de diseminación y en la TAC de tórax se observan dos nódulos pulmonares en lóbulo superior izquierdo. Se resecan los nódulos pulmonares y el estudio histopatológico confirma metástasis pulmonares de leiosarcoma uterino.

El muestreo de tejido está justificado si los resultados del examen radiológico, anamnesis y examen físico son sugerentes una etiología maligna o persiste la incertidumbre con respecto a la causa de los múltiples nódulos pulmonares. Los métodos de muestreo de tejido incluyen una biopsia de aguja transtorácica guiada por TAC, pinza transbronquial o biopsia con aguja y biopsia pulmonar quirúrgica mediante cirugía toracoscópica asistida por vídeo o toracotomía abierta. La elección de estas técnicas depende de la ubicación y el tamaño de la lesión más accesible, así como de la experiencia disponible en la respectiva institución médica.

Definición Son tumores malignos cuyo (tumor) primario se desarrolla en otro sitio y se encuentra diseminado hacia el pulmón. En los pacientes con tumores malignos primarios conocidos, los nódulos pulmonares múltiples que son ≥10 mm de diámetro o detectados por radiografía de tórax convencional son más comúnmente metátasis de un tumor primario de órganos sólidos malignos, mientras que los nódulos pulmonares múltiples <5 mm de diámetro y yuxtapuestos a la pleura visceral o a una fisura interlobular son más probablemente lesiones benignas, tales como granulomatosis, cicatrices o ganglios linfáticos intrapulmonares.

Tratamiento El único tratamiento potencialmente curativo es la resección. La irradiación pulmonar total es un tratamiento conocido desde hace más de treinta años. Se ha usado a dosis bajas tanto de forma profiláctica en tumores de alto riesgo de metastatizar al pulmón como en lesiones pulmonares conocidas previo al tratamiento quirúrgico definitivo. Las metástasis pueden ser resecadas si cumple con los siguientes criterios: el tumor primario debe estar controlado o ser controlable, no debe existir evidencia de metástasis extrapulmonares y el paciente debe tener una adecuada reserva pulmonar (medida espirométricamente y con cintigrama ventilación-perfusión) que permita realizar una resección completa de todas las metástasis. Sobrevida a cinco años varía entre un 20 y 40% y los factores pronósticos más importantes que han sido identificados son: el intervalo libre de enfermedad (tiempo entre el tratamiento del primario y la aparición de las metástasis), el número de metástasis y la realización de una resección completa. En caso de no cumplir con los criterios mencionados, el tratamiento indicado es paliativo.

Etiología-epidemiología-fisiopatología Epidemiología: Las metástasis pulmonares constituyen el segundo lugar más frecuente de enfermedad metastásica y corresponde a la neoplasia pulmonar más frecuente. Se encuentran en cerca de 30% de los pacientes fallecidos por cáncer. Suelen asentarse en la periferia del pulmón. Etiología: Entre los tumores que frecuentemente metastizan a pulmón se encuentran sarcomas óseos y de partes blandas, mama, colorrectal, melanoma y tumores genitourinario. Fisiopatología: Anatómicamente, los pulmones son un lecho vascular rico además del primer lecho capilar que las células tumorales circulantes encuentran después de abandonar el tumor primario y caer al sistema de drenaje venoso. De este modo los pulmones actuarían como un filtro inicial y las células tumorales serían atrapadas de forma mecánica al llegar a él, creciendo después para formar la metástasis.

La quimioterapia es el tratamiento de elección en casos de afectación pulmonar metastásica múltiple. Especialmente en los tumores de alto índice de proliferación como los tumores germinales.

Se producen por diseminación hematógena siendo poco frecuente la linfática.

Seguimiento

Diagnóstico

Seguimiento clínico y radiológico debe ser estricto evaluando velocidad de crecimiento, progresión local y aparición de nuevas metástasis.

Generalmente son asintomáticos. Pueden presentarse con tos crónica, hemoptisis, disnea o dolor torácico. 37

El agente causal es el polvo fibrogénico, y el resultado de su acción es una cicatriz colágena en el pulmón.

Neumoconiosis

1. Silicosis: Las partículas de sílice (cuarzo) se encuentran en minería, canteras de granito, tunelizaciones, cortadores de piedra, industrias abrasivas, fundiciones e industria de cerámica. Determina un proceso fibrótico pulmonar. En cuanto a la fisiopatología, el macrófago alveolar tiene un gran rol en la reacción tisular. Entre sus complicaciones destaca una predisposición a presentar tuberculosis. Hay poca información epidemiológica sobre silicosis en Chile. 2. Asbestosis: El asbesto se utiliza en materiales de construcción, materiales de fricción (embragues, frenos, etc), materiales termo-resistentes, etc. Su fisiopatología se explica por el depósito de fibras en el parénquima pulmonar y el desarrollo de fibrosis intersticial y alveolar difusa. Se le asocia a la asbestosis el desarrollo de cáncer pulmonar, mesotelioma pleural o peritoneal. 3. Neumoconiosis de los mineros de carbón: Las partículas de carbón se distribuyen ampliamente en el pulmón, con aparición de máculas de carbón alrededor de los bronquiolos. Éstos posteriormente se dilatan, determinando un enfisema focal por polvo. Ésta neumoconiosis aumenta el riesgo de bronquitis crónica y enfisema.

Nivel de manejo del médico general: Diagnóstico: Sospecha. Tratamiento: Inicial. Seguimiento: Derivar.

Aspectos esenciales 

    

Las enfermedades pulmonares profesionales ocurren como resultado directo de la exposición en el lugar de trabajo a metales, polvo, humo o agentes biológicos. Neumoconiosis son importantes como enfermedades ocupacionales. La silicosis aumenta el riesgo de TBC. Asbestosis se asocia con desarrollo de cáncer pulmonar y mesoteliomas. Neumoconiosis de mineros del carbón se asocia a bronquitis crónica. Neumoconiosis no tienen tratamiento.

Caso clínico tipo Paciente varón de 45 años, con antecedentes de haber trabajado 20 años en una fundición de hierro, presenta disnea de esfuerzo progresiva, asociada a tos seca. La radiografía de tórax muestra múltiples opacidades redondeadas, de tamaño pequeño (menor de 1 cm) en ambos campos pulmonares.

Diagnóstico Las neumoconiosis suelen diagnosticarse con una exposición positiva y una presentación radiográfica sugerente. Dada la apariencia radiográfica típica de las neumoconiosis más comunes, generalmente no se requiere biopsia de tejido para establecer el diagnóstico cuando la historia de exposición y el patrón radiográfico son característicos.

Silicosis crónica.

Definición

Silicosis: Clínicamente, se manifiestan por tos y disnea progresiva. Hay tres formas de presentación:

Las neumoconiosis resultan de la inhalación y deposición de partículas inorgánicas y polvo mineral con la subsiguiente reacción del pulmón. La neumoconiosis se puede subdividir en fibrogénica (por ejemplo, sílice, carbón, talco, amianto), benigna o inerte (por ejemplo, hierro, estaño, bario), granulomatosa (por ejemplo, berilio que induce una reacción de tipo sarcoide) y neumonía de células gigantes en inhalación de cobalto.

1. Crónica: suele aparecer tras décadas de exposición. Radiológicamente, presenta opacidades redondas <10 mm, adenopatías hiliares calcificadas en cáscara de huevo, nódulos silicóticos (en linfonodos, pleura u otros órganos); 2. Acelerada: exposición 5-10 años, más intensa. Similar clínica y fisiopatológicamente a la forma crónica; 3. Aguda: exposición de pocas semanas a 5 años, a cantidades importantes, se asocia a disnea importante y baja de peso.

El término médico de neumoconiosis se aplica a aquellas entidades que cumplen 3 requisitos: a) Son enfermedades que asientan en el pulmón (alveolos e intersticio) y pequeñas vías.

La radiografía muestra patrón alveolar difuso. Las formas crónica y acelerada se pueden complicar con fibrosis masiva progresiva, cuando coalescen los nódulos en conglomerados >10 mm. Diagnóstico se realiza con el antecedente de exposición y evidencias radiológicas de la alteración. En cualquiera de las formas, la espirometría puede ser normal o presentar cualquier patrón.

b) Se producen por polvo inorgánico o mineral. c) Originan una cicatriz (cúmulo de tejido colágeno). Actualmente las tres de mayor relevancia son: silicosis (por polvo de sílice cristalina), asbestosis (por exposición a asbesto) y neumoconiosis de los mineros de carbón (por el polvo del carbón).

Asbestosis: La mayor parte de los pacientes quienes desarrollan asbestosis permanecen asintomáticos por 20 a 30 años luego de la exposición inicial. Este período de latencia entre la exposición y los síntomas es inversamente proporcional a la intensidad de la exposición al asbesto. Sin embargo, la patología pleural se produce usualmente en forma temprana.

Etiología-epidemiología-fisiopatología

38

El síntoma más frecuente es disnea. Radiológicamente se observa patrón reticular que predomina en campos inferiores, asociado a placas pleurales. La espirometría habitualmente muestra una alteración ventilatoria restrictiva. El diagnóstico requiere el antecedente de exposición y alguna evidencia de alteración clínica, radiológica o funcional de la alteración. Neumoconiosis de los mineros de carbón: La presentación sintomática más frecuente es tos crónica con expectoración. Radiológicamente, en antracosis simple (generalmente asintomática) se observan opacidades menores de 10 mm bilaterales. En la forma complicada se observan opacidades mayores de 10 mm (al menos una) y habitualmente es sintomático. Diagnóstico requiere antecedente de exposición y evidencias radiológicas.

Estudio Luego de una adecuada historia clínica ocupacional, acompañada de síntomas respiratorios (disnea, tos, expectoración), se realiza un examen físico buscando signología pulmonar clásica (estertores, sibilancias, crépitos finos, disminución de movimientos respiratorios, hipocratismo digital, etc). Se realizan pruebas funcionales (espirometría), y un adecuado examen radiológico (Rx tórax, TAC, RNM). Otras pruebas que se realizan para el diagnóstico son: biopsia de pulmón directa, biopsia transbronquial, lavado bronquio alveolar, microscopía de luz polarizada, entre otras.

Tratamiento Los procesos desencadenados en las neumoconiosis son irreversibles, por lo cual el tratamiento se orienta a la prevención de las complicaciones y de la progresión de la enfermedad. Se realiza el aislamiento de la fuente de trabajo, y medidas generales (humedecer el suelo, protección y ventilación adecuada). Como terapia farmacológica se ocupan corticoesteroides en beriliosis.

Seguimiento Evaluación y pesquisa de complicaciones.

39

Diagnóstico

Neumotórax hipertensivo (a tensión)

El diagnóstico se basa exclusivamente en la clínica y el examen físico. Clínica: Disnea intensa y progresiva, taquipnea, taquicardia, cianosis, hipotensión, inquietud, diaforesis.

Nivel de manejo del médico general: Diagnóstico Específico Tratamiento Completo Seguimiento Completo.

Examen físico: Ingurgitación yugular, Abombamiento del hemitórax, Disminución de los movimientos respiratorios, Timpanismo a la percusión y abolición de los ruidos respiratorios del lado afectado. Desviación de los ruidos cardíacos hacia el lado contralateral.

Aspectos esenciales     

Patología de riesgo vital. Diagnóstico clínico. Tiene indicacion de drenaje Urgente Tratamiento primario: Toracostomía cerrada. Tratamiento definitivo: Tubo de drenaje torácico (Pleurotomía).

Tratamiento El tratamiento del neumotórax a tensión debe ser inmediato a la sospecha clínica, sin esperar exámenes imagenológicos, y consiste en la Toracostomía cerrada: inserción de un catéter IV (bránula) en 2° espacio intercostal línea axilar media del lado afectado para facilitar la salida de aire y restituir las presiones parciales de manera temporal, y despues colocar tubo de drenaje toracico.

Caso clínico tipo Paciente que tras someterse a cirugía, ventilación mecánica o reanimación presenta cuadro agudo caracterizado por ingurgitación yugular, disnea, taquicardia, hipotensión, al examen físico torácico destaca timpanismo a la percusión y la abolición de los ruidos respiratorios del lado afectado.

Seguimiento Solucionada la Urgencia se procede a realizar una pleurotomía con un tubo de drenaje torácico con trampa de agua, que se puede retirar al demostrarse mediante radiografía la expansión pulmonar completa y la no existencia de fuga aérea.

Definición Urgencia médica caracterizada por colapso pulmonar agudo producto de acumulación progresiva de aire en la inspiración, debido a un valvular que impide salida de aire, que conlleva a colapso pulmonar, insuficiencia Respiratoria aguda e inestabilidad hemodinámica.

Etiología-epidemiología-fisiopatología Etiología: Multifactorial, las causas principales son: a) Iatrogenia: Ventilación mecánica con presión positiva al final de la espiración. b) Mecanismos traumáticos (ej: fractura costal, traumas de la pared torácica). c) Otros: Como complicación de un neumotórax espontáneo. Fisiopatología: El neumotórax a tensión se produce cuando el aire entra al espacio pleural durante la inspiración pero no puede ser expulsado por la misma vía, con un funcionamiento semejante a una válvula unidireccional. El resultado de esto es la acumulación de aire intrapleural con una presión sustancialmente superior a la atmosférica, lo que provoca colapso pulmonar y aumento de la presión intratorácica. La acumulación progresiva de aire en el espacio pleural durante cada inspiración, aumenta la presión intratorácica causando colapso del pulmón ipsilateral, desviación contralateral del mediastino, comprometiendo al corazón, los grandes vasos y al otro pulmón. Esto da como resultado la disminución del retorno venoso, compromiso ventilatorio y un gasto cardíaco disminuido. 40

ser más grave, con disnea desproporcionada al tamaño del neumotórax, hipoxemia e hipotensión.

Neumotórax pequeño

2. Imágenes: Nivel de manejo del médico general Diagnóstico: Específico. Tratamiento: Completo. Seguimiento: Derivar.

- Radiografía de tórax: presencia de aire en el espacio subpleural. Para estimar la cuantía se mide la distancia entre pleura parietal y visceral a nivel del hilio pulmonar; encontrándose las siguientes categorías:   

Aspectos esenciales    

El neumotórax puede darse de manera espontánea en sujetos sanos y con patología pulmonar. En casos estables y pequeños: observar y seguir con radiografía. En pacientes más complicados se debe usar drenaje pleural. En pacientes de riesgo y en casos de recidiva: realizar pleurodesis.

Muy pequeño: < 1 cm (< 15%) Pequeño: 1-2 cm (16-50%) Grande: >2 cm (> 50%).

- TAC de tórax: muestra mejor el colapso pulmonar, permite diagnosticar otras bulas y enfermedades subyacentes.

Tratamiento A. Neumotórax espontáneo primario: 1. Primer episodio:  Asintomático y pequeño: se puede observar y dar terapia de sostén (Oxigenoterapia).  Grande o sintomático: Pleurostomía. 1. Segundo episodio (o persistencia de pérdida de aire): Pleurodesis química o toracoscopía.

Caso clínico tipo Paciente varón de 38 años. Consulta por dolor pleurítico de instalación brusca y sensación de agitación. No refiere patologías crónicas y tiene un IPA de 7. Al examen físico, destaca frecuencia cardíaca de 106x'.

B. Neumotórax espontáneo secundario:

Definición



Presencia de aire en el espacio pleural con colapso pulmonar menor al 15% del hemitórax. Se puede clasificar en: 1. Espontáneo (primario si ocurre en un pulmón sano y secundario si ocurre en pulmón con patologías previas). 2. Traumático (abierto o cerrado). 3. Iatrogénico.



Según el tamaño del compromiso podremos clasificar el neumotórax en:

Etiología-epidemiología-fisiopatología

a. Pequeño (1-2 cms):

El neumotórax espontáneo primario típicamente ocurre:  



Hombres jóvenes, altos y delgados El tabaquismo el mayor factor de riesgo dado que puede causar bulas subpleurales y cambios enfisematosos en los lóbulos superiores.



Asintomático: Se puede observar con Rx Tórax cada 24-48 horas (estando hospitalizado). Sintomático: Se puede colocar pleurostomía tipo pigtail.

b. Grande (>2cm):

El neumotórax espontáneo secundario ocurre:  

SIEMPRE Hospitalizar, pues patología pulmonar de base aumenta el riesgo de un resultado adverso. Vigilar signos vitales para evaluar compromiso hemodinámico y respiratorio. Oxígenoterapia: Darlo en forma cuidadosa, en especial en pacientes EPOC por mayor riesgo de hipercapnia. El oxígeno facilita absorción de aire en la pleura parietal (se reabsorberá 1,25% al día).

Instalar pleurostomía formal. Vigilar si hay pérdida de aire persistente, frecuente en esta patología. En caso de haberla se maneja con Videotoracoscopía o pleurodesis química.

Hombres > 60 años Pacientes con antecedentes de: EPOC, infección por VIH o por Pneumocistis jirovecii, los que poseen áreas necrosadas de tejido pulmonar.

Seguimiento Diagnóstico

Derivar a especialista.

1. Clínica y examen físico a) Asintomático. b) Sintomático: Dolor pleurítico, disnea súbita y taquicardia. En neumotórax espontáneo secundario la reserva funcional es menor y, por ende, el cuadro clínico puede 41

3. Otras: Traumatismos, trastornos electrolíticos y metabólicos, shock, hipotermia y drogas.

Paro cardiorrespiratorio

Causas: Nivel de manejo del médico general: Diagnóstico: Específico. Tratamiento: Completo. Seguimiento: Derivar.







5Ts

Hipoxia

Taponamiento Cardíaco

Hipo/Hiperkalemia

Trombosis coronaria

Hidrogeniones (Acidosis) TEP

Aspectos esenciales 

5Hs

Es esencial el inicio inmediato de Resucitación CardioPulmonar, pues por cada minuto sin Resucitación CardioPulmonar la sobrevida del paro cardíaco, presenciado en Fibrilación Ventricular, disminuye 7 – 10%. Cadena de supervivencia: 1) Activación del sistema de emergencia 2) Resucitación CardioPulmonar básica 3) Desfibrilación 4) Resucitación CardioPulmonar avanzada. 5) Cuidados integrados post paro cardiaco. Secuencia de maniobras de Resucitación CardioPulmonar: C-A-B (Chest Compressions, Airway, Breathing) (Iniciar y hacer énfasis en las compresiones torácicas). La desfibrilación es la medida más efectiva para reanimar un paciente en Fibrilación Ventricular o Taquicardia Ventricular Sin Pulso.

Hipotermia

Neumotórax a tensión

Hipovolemia

Tóxicos

Fisiopatología: El colapso circulatorio produce reducción dramática en el transporte de oxígeno de la célula. En especial en cerebro y miocardio, produce un cambio de metabolismo aeróbico a anaeróbico. En primeros 5 min, reservas de ATP celular se han agotado. Pequeña cantidad se obtiene por vía de Adenil kinasa. El AMP a su vez es convertido en adenosina, deprimiendo la conducción a través del nodo AV y ademas dilatación arteriolar. Finalmente se activa la respuesta inflamatoria celular.

Diagnóstico Diagnóstico es clínico, debe ser rápido y seguro. Fundamental es inspección y palpación. Manifestaciones principales son la inconciencia, ausencia de pulso y la respiración anormal (apnea o gasping). La ausencia de pulso se debe determinar en menos de 10 segundos y en caso de duda se debe asumir que no hay pulso.

Caso clínico tipo Paciente de 65 años, llega al servicio de urgencia con compromiso de conciencia, familiares refieren que hace 20 min sintió un dolor opresivo que se irradiaba a mandíbula y brazo izquierdo, EVA 8/10, y luego se “desplomó”. Al examen físico: Paciente no responde, no respira y no hay pulso detectable.

  

Inconsciencia, Apnea o respiración agónica, Ausencia de pulso.

No existen exámenes útiles. Ante la duda, actuar inmediatamente.

Definición Detención súbita de la actividad miocárdica y ventilatoria que determina una brusca caída del transporte de oxígeno a los tejidos, determinando la pérdida de conciencia. Es una emergencia médica potencialmente reversible a través de las maniobras de Reanimación Cardiopulmonar (Resucitación CardioPulmonar).

Etiología - epidemiología – fisiopatología Causas de muerte son muchas. Causas de muerte súbita son principalmente cardíacas en el adulto: 90% de las veces es la enfermedad coronaria a través de un evento isquémico que deriva en una fibrilación ventricular. Causas más frecuentes: 1. Cardiológicas: Síndrome coronario agudo, arritmias, taponamiento cardiaco, tromboembolismo pulmonar, etc. 2. Respiratorias: Obstrucción, depresión del centro respiratorio, broncoaspiración, asfixia o neumotórax a tensión.

42

ALGORITMO MANEJO PCR:

43

Siempre se deben identificar y tratar las causas reversibles (las 5H y las 5T):

Tratamiento y Seguimiento      

La cadena de supervivencia considera cuatro eslabones: 1) Reconocimiento y activación precoz del sistema de emergencias. 2) Resucitación CardioPulmonar básica. 3) Desfibrilación precoz. 4) Resucitación CardioPulmonar avanzada Luego se prosigue con los cuidados integrados post paro cardiaco.

Hipovolemia

Neumotórax a tensión

Hipoxia

Taponamiento cardiaco

Acidosis

Intoxicaciones

Hipo/hiperkalemia Tromboembolismo pulmonar Hipotermia

 Los primeros tres eslabones se engloban en el concepto de Soporte Vital Básico (SVB), mientras que los últimos corresponden a Soporte Vital Avanzado (SVA).

Trombosis coronaria

Tras el restablecimiento de la circulación espontánea se procura la optimización de la función cardiopulmonar y la perfusión de órganos vitales. Si hay sospecha de síndrome coronario agudo el paciente se debe realizar una coronariografía. Trasladar al paciente a una Unidad Cuidados Intensivos apropiado que disponga de un sistema completo de tratamiento post-Paro CardioRespiratorio. Considerar el control de la temperatura (optimizar recuperación neurológica), así como prevenir y tratar disfunciones orgánicas, lo que incluye evitar la ventilación excesiva y la hiperoxia.

Se inician las maniobras de Resucitación CardioPulmonar mientras se consigue un desfibrilador. La secuencia recomendada es C-A-B (Chest Compressions, Airway, Breathing), comenzando con 30 compresiones y alternándolas con 2 ventilaciones. Como ayuda memoria el ritmo de las compresiones es el mismo de la canción "Stayin' alive" de "Bee Gees". Las compresiones efectivas se definen como:     

Seguimiento

Compresiones a 100-120 /minuto De más de 5 cm de profundidad. Evitando interrupciones y pausas. Debe permitir la completa reexpanción del tórax entre compresiones. Cambiar de operador cada 2 minutos.

Por especialista.Manejo del paciente post-paro cardíaco es complejo e implica el desempeño simultáneo de las dos intervenciones diagnósticas y terapéuticas, incluyendo las siguientes: 1)Determinar y tratar la causa de un paro cardíaco

Las ventilaciones deben ser de 1 segundo de duración, hasta lograr un levantamiento normal del tórax.

2)Reducción al mínimo de la lesión cerebral

Si utiliza un dispositivo avanzado de vía aérea (tubo endotraqueal o dispositivo supraglótico) debe realizar compresiones contínuas y 8-10 ventilaciones por minuto. Utilice oxígeno al 100% y volumen corriente de 500-600 ml; evite la hiperventilación. Es recomendable utilizar la capnografía (EtCO2) para monitorizar el adecuado emplazamiento del dispositivo de vía aérea y monitorizar la calidad de la Resucitación CardioPulmonar (un EtCO2 <10 mmHg obliga a mejorar la calidad de la Resucitación CardioPulmonar).

3)Gestión de disfunción cardiovascular 4)La gestión de los problemas que puedan surgir de la isquemia global y reperfusión

Apenas tenga disponible el desfibrilador debe evaluar el ritmo cardiaco, lo cual debe repetirse cada 2 minutos. 



Si es desfibrilable debe realizar una desfibrilación (120-200 J bifásico o 360 J monofásico) y continuar inmediatamente la Resucitación CardioPulmonar comenzando por las compresiones. Si no es desfibrilable debe continuar inmediatamente la Resucitación CardioPulmonar comenzando por las compresiones.

Los fármacos a utilizar son:  

Adrenalina: 1 mg cada 3-5 minutos IntraVenoso Amiodarona: en caso de ritmos desfibrilables. 300 mg IntraVenoso

44

La patología suele asociarse a un deterioro funcional durante el día debido a la interrupción constante de las fases del sueño debido a los despertares, con mayor probabilidad de accidentes de tránsito, laborales, trastornos del ánimo, trastornos conductuales y síndromes cardiovasculares.

Síndrome de Apnea del Sueño Nivel de manejo del médico general: Diagnóstico: Sospecha. Tratamiento: Inicial. Seguimiento: Derivar.

Diagnóstico 1. Anamnesis:  Historial de ronquidos durante el sueño (referido por terceros).  Presencia de factores de riesgo.  Síntomas de cansancio durante el día.

Aspectos esenciales    

Patología asociada a cierto grado de deterioro funcional. Sospecha clínica por anamnesis y presencia de factores de riesgo. Es importante el manejo de factores que contribuyan a la obstrucción, como fármacos, postura, hábitos, entre otros. El tratamiento de patología específica de vía aérea y el seguimiento es por especialista.

2. Examen Físico:  Examen de cabeza, cuello, cavidad nasal, cavidad oral, naso y orofaringe, buscando posibles puntos de obstrucción que refuercen la sospecha diagnóstica.  En caso de disponer de nasofibroscopía, realizar examen de laringe y cuerdas vocales, con el mismo objetivo.

Caso clínico tipo

3. Polisomnografía: Otorga el diagnóstico de certeza una vez hecha la sospecha diagnóstica con la historia clínica. Incluye un registro EEG, electromiográfico, electrooculográfico, ECG, saturación de oxígeno, y medición de flujo nasal y bucal.

Paciente masculino 55 años, obeso y fumador. Refiere hipersomnolencia diurna, irritabilidad y baja del rendimiento en el trabajo. Evaluado por psicólogo descarta trastorno del ánimo. Al interrogatorio dirigido destaca roncopatía de larga data.

Según el número de apneas e hipoapneas o “eventos respiratorios” por hora (lo normal es hasta 5), se clasifican en:

Definición



Episodios recurrentes de colapsos parciales (hipoapneas) o totales de la vía aérea superior durante el sueño, en que ocurre un cese del flujo de aire por 10 segundos o más, pese a persistir los esfuerzos ventilatorios (esto diferencia una apnea obstructiva de una central). La caída de la saturación arterial de oxígeno conduce a un breve despertar característico del cuadro.

 

Apneas leves: más de 5 y hasta 20 eventos por hora. Apneas moderadas: más de 20 y hasta 50 eventos por hora. Apneas severas: sobre 50 eventos.

Según el grado de desaturación de oxígeno que se alcance:  Desaturación leve: hasta 85%  Desaturación moderada: hasta 75%  Desaturación severa: bajo 75%

Etiología-epidemiología-fisiopatología El colapso de la vía puede producirse por reducción del tono muscular orofaríngeo y/o palatino, o por estrechez del pasaje de la vía aérea desde la rinofaringe a la base de la lengua. Los microdespertares permiten normalizar el tono muscular o corregir la postura para desobstruir la vía aérea.

Número de microdespertares por hora de sueño (lo normal es hasta 10). Lo relevante es establecer una relación temporal con los periodos de apnea.

Tratamiento

Dentro de los factores de riesgo encontramos:  Anatómicos: Hipertrofia amigdalina (también adenoidea en niños), macroglosia, hipertrofia cornetes nasales, pólipos o tumores nasales, desviación del septum nasal, obesidad cervical, elongación del velo del paladar y de la úvula, micrognatia y retrognatia  Posicional: más frecuente en decúbito dorsal.  Edad: Riesgo aumenta con la edad.  Obesidad.  Condiciones médicas: Hay condiciones médicas que aumentan el riesgo como el embarazo, falla cardíaca congestiva, enfermedad pulmonar crónica e hipotiroidismo.  Otros: ingesta alimentaria abundante previo a dormir, alcohol, drogas tranquilizantes (hipnóticos, relajantes musculares).

Apneas Leves:  Medidas Generales: alimentación, hábitos (OH, drogas), controlar fármacos, posición al dormir y bajar de peso.  Manejo de patología sinusal: Derivar Especialista. Apneas moderadas a severas:  Uso de CPAP y de BIPAP (Introducen aire (ambiental) a presión en la vía aérea del paciente a través de una mascarilla).  Manejo Quirúrgico: en casos particulares, con un profundo estudio en cada caso; dirigido al punto de colapso y/o obstrucción de la vía aérea.

Seguimiento Por especialista. 45

c) Síndrome de compresión nerviosa: Frénico (dolor irradiado al hombro, respiración paradójica), recurrente laríngeo (voz bitonal, disfonía), ganglios y nervios simpáticos (síndrome de Claude Bernard-Horner). d) Síndrome esofágico: Disfagia por infiltración o compresión extrínseca. e) Síndrome de compresión arterial: Pulmonar (frémitos, soplo sistólico pulmonar) o aórtico (soplo sistólico aórtico, frémito irradiado a cuello, alteración del pulso) f) Síndrome paraneoplasico: Hipercalcemia, miastenia gravis, ginecomastia, HTA.

Síndromes mediastínicos Nivel de manejo del médico general: Diagnóstico: Sospecha. Tratamiento: Inicial. Seguimiento: Derivar.

Aspectos esenciales 

   

Límites mediastino: Cefálico: Plano entre borde superior del esternón con el proceso espinoso de la séptima vértebra cervical; Caudal: Diafragma; Dorsal: Columna dorsal; Ventral: Cara dorsal del esternón; Laterales: Las pleuras parietales mediastínicas. Tumores de mediastino anterior (Las 4 “Ts”: timo, tiroides, teratoma y (el terrible) linfoma). En presencia de síntomas B sospechar linfoma; realizar etapificación y biopsia previa a cirugía. TAC es el método de estudio de elección. Se deben buscar adenopatías.

El estudio debe iniciarse con radiografía de tórax, ecografía (mediastino superior, tiroides y paratiroides), TAC (elección), RNM (estudio vascular sin contraste), mediastinoscopía (permite biopsias). La mediastinitis aguda es un cuadro infrecuente, sospechar si hay fiebre, gran compromiso del estado general, dolor retroesternal irradiado al cuello, taquipnea y signos de sepsis. Si la causa es perforación esofágica, suele haber neumomediastino, que se puede manifestar por enfisema subcutáneo y que se confirma con el examen físico y la radiografía de tórax.

Caso clínico tipo

Tratamiento

Paciente de 55 años, consulta por aumento progresivo de volumen en cara y cuello de 3 meses de evolución. Además refiere sudoración nocturna hace 1 mes. En sospecha de Síndrome de Vena Cava Superior, se realiza TC de Tórax que muestra masa mediastínica de 7 cm de diámetro comprimiendo la vena cava superior. En sospecha de Linfoma se hospitaliza para estudio y manejo.





El tratamiento de la mediastinitis aguda consiste en drenaje quirúrgico de urgencia, soporte hemodinámico y antibióticos que cubran anaerobios, grampositivos y gramnegativos. El tratamiento de tumores y quistes es la extracción quirúrgica asociada a quimioterapia si se sospecha o confirma malignidad (excepto en linfomas y timomas que se hace generalmente sólo quimioterapia).

Definición Seguimiento

Patologías presentes en el mediastino: tumores, quiste, inflamación/infección (mediastinitis).

Por especialista. 

Etiología-epidemiología-fisiopatología Frecuencia de compromiso: 59% mediastino anterior, 29% mediastino medio, y 16% mediastino posterior. La neoplasia más frecuente que compromete al mediastino es el linfoma.  



De las lesiones asintomáticas, 90% son benignas. De las lesiones sintomáticas, 50% son malignas.

Presentación Clínica y Diagnóstico Pueden ir desde presentación asintomática, hasta manifestaciones compresivas o infiltrativas. Varía según el síndrome de presentación: a) Síndrome de vena cava superior: Ingurgitación yugular, cianosis periférica de cabeza, cuello y tórax alto, circulación colateral, edema en esclavina. b) Síndrome de comprensión de vía aérea: Tos seca, disnea inspiratoria con cornaje, tiraje supraesternal, intercostal y epigástrico, obstrucción bronquial. 46

La mediastinitis aguda requiere control en unidad de paciente crítico y evaluación quirúrgica diaria, con régimen cero, alimentación enteral y antibioterapia adecuada. Además de aseo quirúrgico si requiere. En tumores control por oncólogo y cirujano por posible recidiva, y continuar esquemas de radioterapia y quimioterapia según etiología.

Tabla 1. Causas de tos crónica

Tos crónica

Frecuentes  Sindrome de descarga posterior  Asma Bronquial  Reflujo gastroesofágico  Bronquitis crónica

Nivel de manejo del médico general: Diagnóstico: Específico. Tratamiento: Completo. Seguimiento: Completo.

Aspectos esenciales    

Tos que dura más de 8 semanas. La causa más común es el Síndrome de descarga posterior. Le siguen el asma bronquial y el reflujo gastroesofágico. En ausencia de claves clínicas: Rx Tx, se trata como tal y se sigue. Ante mala respuesta: ampliar estudio.

Menos frecuentes  Bronquiectasias  Insuficiencia cardíaca  Cáncer bronquial  Lesiones vía aérea central  Fibrosis pulmonar  Uso de fármacos

De todas formas, hasta en un 10% de los casos no se va a identificar la etiología de la tos, pese a un estudio exhaustivo.

Diagnóstico Caso clínico tipo



Paciente de 35 años que consulta por tos persistente desde hace 9 semanas, asociado a síntomas de congestión nasal, odinofagia y carraspera. Tiene antecedentes de alergia estacional y el uso de jarabes para la tos no ha mejorado el cuadro significativamente. 

Definición Tos que se extiende por más de 8 semanas, sin estar relacionada a un proceso agudo ni tener una causa aparente. Si la duración es entre 3 y 8 semanas hablamos de tos subaguda, y menor a ese período se considera tos aguda.



Etiología-epidemiología-fisiopatología La tos es uno de los síntomas más frecuentes por lo que consultan los pacientes, estando presente en diversas enfermedades, principalmente de las vías respiratorias. En Chile se estima que un 23% de la población adulta presenta el cuadro.

Anamnesis: Consignar el hábito tabáquico (muy importante en nuestro medio), sintomatología digestiva, sensación de obstrucción nasal, carraspeo, rinorrea, sinusitis a repetición o estacionales y el consumo de fármacos como Enalapril o Captopril. Evaluar si hay desencadenantes, episodios de sibilancias o disnea. Examen físico: Búsqueda de signos de enfermedad pulmonar crónica (como hipocratismo digital, crepitaciones, sibilancias), disfonía, alteraciones en la olfación o cualquier signo que pudiese dar cuenta de una enfermedad de base. Exámenes de laboratorio: Se recomienda estudio con radiografía de tórax si no hay causa sospechosa. Según sospecha estudio para otras etiologías (espirometría, test de provocación con metacolina o phmetría esofágica). Se sugiere que pacientes que presenten tos y expectoración por más de 15 días se realice baciloscopía, para determinar la presencia de tuberculosis pulmonar.

Tratamiento Depende de la etiología de la tos (figura 1), pero en líneas generales:

La tos se produce gracias a un complejo arco reflejo. Se inicia por la estimulación de receptores de tos que se encuentran no solo en la vía respiratoria, sino además, en el pericardio, esófago, diafragma y estómago. Estos receptores responden a sustancias irritantes (ácidos, álcalis, capsaicina, entre otros), frío y calor, o estímulos mecánicos. La vía aferente del estímulo es mediante el nervio vago, hasta el “centro de la tos” en la médula. La vía aferente del reflejo viaja por el mismo nervio vago, nervios frénicos y espinales motores, hasta la musculatura espiratoria, produciendo la tos.

  

Las causas de tos crónica son muy variadas (ver tabla 1), pero las afecciones rinosinusales, el asma bronquial, el reflujo gastroesofágico y la bronquitis crónica por irritantes (tabaco principalmente) representan el 95% de todos los casos.

 

47

En rinosinusitis de origen alérgico se deben usar antihistamínicos y descongestionantes, y en caso de infección bacteriana, antibióticos. Para asmáticos, el manejo se basa en corticoides inhalatorios y broncodilatadores en caso necesario. En caso de RGE se recomienda el uso de inhibidores de la bomba de protones. Tos en pacientes no fumadores y sin hiperreactividad bronquial (bronquitis eosinofílica) el manejo es con corticoides. Si se trata de un efecto adverso a IECAs se cambian a ARA2. Además se pueden adicionan antitusígenos al tratamiento etiológico. Cuando la causa no es evidente, el tratamiento va a la vía aérea superior, y se debe seguir el caso.



Siempre se debe suspender el hábito tabáquico y manejar la exposición a irritantes ambientales.

Figura 1. Algoritmo de manejo de la tos crónica

Seguimiento Se debe esperar al menos 8 a 12 semanas desde iniciado el tratamiento para evaluar los resultados. En caso de persistir la tos, ampliar estudio: espirometría inicialmente, y según hallazgos decidir otros exámenes. Se debe considerar derivar a especialista si la tos persiste y no se ha precisado una etiología, después del tratamiento empírico.

48

2.- Hemotórax: Sangre en la cavidad pleural.

Traumatismo torácico grave

a) Masivo: Por herida cardíaca o lesiones de vasos arteriales. Nivel de manejo del médico general: Diagnóstico Específico Tratamiento Inicial Seguimiento Derivar.

b) Progresivo: por sangrado de arterias intercostales o mamaria interna. c) Hemotórax estabilizado. 3.- Tórax Volante: Infrecuente, asociado a fracturas costales múltiples en 2 o más partes de cada costilla, que generan un segmento de parrilla costal aislado del resto.

Aspectos esenciales.   

Generalmente presente en politraumatizados. Diversidad de lesiones según estructura comprometida. Manejo dado por compromiso hemodinámico, vía aérea y ventilación principalmente.

4.- Herida Cardíaca.

Diagnóstico. La tarea más importante en la gestión de los pacientes que han sufrido un traumatismo de la pared torácica se requiere evaluar compromiso intratorácico, intraabdominal, o cualquier otra lesión que amenaza la vida. Los signos vitales anormales, lesión penetrante, o alteración del estado mental debe impulsar de forma automática un mayor nivel de preocupación y evaluación en el servicio de urgencias (SU).

Caso clínico tipo. Paciente que llega a urgencias luego de un accidente de tránsito, herida con arma blanca o de fuego. Viene hemodinámicamente inestable, o con pronta evolución a inestabilidad.

lesiones en la pared del pecho que están asociados con compromiso intratorácico significativo o una lesión intraabdominal incluyen:

Definición. Lesión o herida del tórax que afecta pared ósea, la pleura, los pulmones, el diafragma o el contenido del mediastino.

• fracturas costales múltiples o desplazados • fractura de omóplato • tórax inestable • fractura esternal • luxación esternoclavicular posterior

Etiología - epidemiología - fisiopatología. El trauma es la tercera causa de muerte. En menores de 40 años, se encuentra en el primer lugar. El 50% de la mortalidad por trauma se debe a traumatismos torácicos (75% se debe a trauma torácico como causa primaria o como elemento contribuyente). En Chile, se aprecia una tendencia al alza en frecuencia y gravedad de casos. Las consecuencias de las lesiones torácicas frecuentemente son hipoxia, hipercapnia y acidosis.

● Los hallazgos físicos de trauma de la pared torácica que son consistentes con lesion intratorácica o lesión intraabdominal incluyen: • La hipoxia o signos de dificultad respiratoria (por ejemplo, el uso de los músculos accesorios) • movimiento paradójico de un segmento en el pecho durante la respiración (lo que sugiere tórax inestable) • La inestabilidad hemodinámica • señal del cinturón de seguridad a través del pecho o la pared abdominal • La sensibilidad en múltiples segmentos de nervio (lo que sugiere fracturas costales múltiples) • deformidad palpable del esternón o varios segmentos de costilla • sensibilidad abdominal

Existe una diversidad de diagnóstico que se incluyen dentro del trauma torácico grave: 1.- Neumotórax traumático: Aire en la cavidad pleural. - Abierto: Consecuencia de solución de continuidad en pared torácica de diámetro > 2/3 de la tráquea, con paso de aire a cavidad pleural.

1.- Neumotórax:

- A tensión: Consecuencia de lesiones bronquiales y desgarros pulmonares, que determinan mecanismo de válvula unidireccional que permite entrada de aire, pero no salida. Generando compresión de mediastino y del pulmón contralateral.

- Abierto: Herida evidente en tórax asociado a ventilación alterada con hipoxia más hipercapnia. - A tensión: Antecedente de trauma torácico con cuadro muy agudo de disnea, dolor torácico, taquipnea, hipotensión con examen físico que muestra murmullo pulmonar abolido en hemitórax comprometido, yugulares ingurgitadas, signos de shock. Diagnóstico clínico, no tomar radiografía. Frente a sospecha realizar tratamiento indiciado.

- Simple: a) Incompletos: colapso menor a 25% (en la práctica, no hay separación completa del pulmón y la pleura. b) Completos: colapso entre 25 – 50% (en la práctica separación completa del pulmón y la pleura. c) Totales: Colapso mayor a 50% (pulmón completamente colapsado).

- Simple: Generalmente asociado a herida torácica, aunque también puede producirse en trauma cerrado. Al examen se evidencia disminución o abolición del 49

murmullo pulmonar, timpanismo y disminución de la movilidad del hemitórax comprometido.

Progresivo: Estabilizar hemodinámicamente, toracotomía o videotoracoscopia por cirujano.

2.- Hemotórax: Tiene un espectro de presentación clínica, siendo de gravedad la presencia de hipovolemia, anemia aguda, shock y paro cardiorrespiratorio, dentro de los escenarios más graves.

Estable: Toracocentesis o pleurostomía. 3.- Tórax Volante: Los objetivos son: aporte de O2, analgesia y limpieza del árbol bronquial. Tratar la contusión pulmonar y evitar el distrés respiratorio. Optimizar al máximo el manejo del dolor. “Estabilizar” la pared (ventilación mecánica si la gasometría lo indica apoyar incluso con ventilación mecánica).

A) Masivo: Sospecha diagnóstico ante evidencia de herida de tórax, signos de shock y los elementos semiológicos de ocupación pleural líquida.

4.- Herida Cardiaca:

B) Progresivo: Ingresan estables o hipotensos, pero se recuperan con el aporte de volumen. Evolucionan al poco tiempo con compromiso hemodinámico, llegando a la anemia y el shock.

- Fallecido pero no muerto (moribundo): Toracotomía inmediata en el box de reanimación, ventilación asistida, reposición de volumen.

C) Estable: Inestabilidad muy transitoria o bien siempre estabilidad hemodinámica.

- Agónico: La toracotomía debe practicarse también en el box de reanimación.

3.- Tórax Volante: Caracterizado por fracturas costales múltiples, respiración paradojal, ocupación pleural, contusión pulmonar.

- Shock y taponamiento: A pabellón de urgencia para toracotomía. En caso de taponamiento realizar pericardiocéntesis y traslado a pabellón.

4.- Herida Cardiaca: Sospecha frente a herida precordial o con herida de trayecto sugerente. Cuadro clínico de gran espectro:

- Hemodinámicamente estable: determinar necesidad de cirugía.

a) Muerto

Seguimiento.

b) Clínicamente Fallecido, pero no muerto.

Derivar.

c) Agónico. d) Con diferentes grados de shock. e) Hemotórax. f) Taponamiento Cardiaco: triada de Beck (PAM ↓+ ↑ presión venosa = yugulares ingurgitadas + Ruidos cardiacos apagados). g) Hemodinámicamente estables. Examenes: Radiografia de Torax.

Tratamiento. Según algoritmos ATLS: C-A-B. Según cuadro clínico específico sospechado proceder: 1.-Neumotórax: Abierto: Manejo inmediato es cierre del defecto, colocando sobre herida parche de tres puntas (sellado con cinta en 3 bordes). Manejo posterior pleurostomía y reparación quirúrgica. A tensión: Manejo inmediato es descomprimir con aguja gruesa (bránula Nº 14) en el 2º espacio del lado comprometido en LMC. En un segundo tiempo sonda de pleurostomía. Simple: Según estado general paciente. Tratamiento definitivo con pleurostomía. 2.- Hemotórax: Masivo: Toracotomía inmediata, manejo por cirujano.

50

Eco

fast

para

• lesión aórtica • El neumotórax a tensión • El hemotórax con hemorragia activa, grave • taponamiento pericárdico de una lesión miocárdica • obstruccion bronquial

Traumatismo torácico simple Nivel de manejo del médico general: Diagnóstico: Específico. Tratamiento: Completo. Seguimiento: Completo.

La valoración inicial del paciente politraumatizado se centra en el CAB.

Aspectos esenciales.

Anamnesis: Deberán conocerse las circunstancias del traumatismo y el mecanismo lesivo, la edad, antecedentes de la víctima y el tiempo transcurrido hasta la asistencia. Se valorará la presencia del dolor y su irradiación.

   

No posee daño a órgano blanco. El cuadro principal es compromiso de conciencia con antecedentes de evento de gran transferencia de energía. En 80% de los casos, independiente de la etiología solo se requerirá el uso apropiado de drenaje torácico. Solo el 10 a 15% de los pacientes con heridas penetrantes o contusas necesitaran toracotomía.

Examen físico: Completo para descartar complicaciones. Estudio: ECG, Oximetría de pulso, y Rx tórax: en todo paciente con trauma torácico. Tórax volante: Fractura de más de una costilla en más de un Contusión segmento. Respiración pulmonar paradojal. Alteración en la permeabilidad pulmonar.

Caso clínico tipo. Pacientes que presenten fractura de las 3 primeras costillas y/o Ruptura de aorta y ensanchamiento del mediastino grandes vasos superior apreciable en la Rx de tórax, se debe hacer arteriografía o TAC.

Paciente de 30 años llega al servicio de urgencias luego de un accidente de tránsito. Se aprecia con sopor leve y respiración paradojal. La radiografía de tórax muestra 6 fracturas en parrilla costal derecha.

Definición. Daño producido a nivel torácico por exposición brusca a una fuente de energía que supera su margen de tolerancia. Son aquellos casos que representan un problema diagnóstico, pero sin la urgencia de las causas de compromiso vital postraumático. No existe clinica de compromiso pulmonar o cardiaco

Etiología - epidemiología - fisiopatología. Mecanismo del trauma: Trauma (Aceleración/Desaceleración, Compresión), Penetrante o Ambos.

Cerrado Trauma

Principales causas de traumatismo torácico van asociadas a: Accidentes de tránsito (43%), suicidios (29%), homicidios (22%).

Ruptura traqueobronquial

Predomina enfisema subcutáneo. Signos Rxs: Enfisema mediastínico, enfisema subcutáneo, en ocasiones descenso hilio pulmonar del lado comprometido. Fibrobroncoscopía.

Ruptura laríngea

Restricción de vía aérea, alteración de la voz, hemoptisis, enfisema intersticial.

Hernia diafragmática

Laparotomía.

Contusión miocárdica

Puede verse asociado o no a fracturas costales y ser sospechado en todo paciente con fractura de esternón. Hospitalizar en UCI y solicitar enzimas cardiacas, ecocardiograma y ECG seriados como parte del monitoreo continuo.

Hasta un 25% de las muertes de politraumatizados son por complicaciones derivadas del traumatismo torácico per se.

Diagnóstico.

Tratamiento.

El objetivo prioritario es la estabilización del paciente mediante la identificación y resolución de las alteraciones que ponen en peligro su vida: Al determinar la probabilidad de lesiones graves de un traumatismo torácico cerrado, la alteracion de los signos vitales implica mayor riesgo que el mecanismo de lesión. Lesiones potencialmente mortales inmediatas de traumatismo torácico cerrado incluyen:

Eje del enfrentamiento inicial: CAB. Se debe asegurar la permeabilidad de la vía aérea con una adecuada ventilación y oxigenación, procediendo a la intubación y ventilación mecánica si es necesario manteniendo de manera constante un adecuado control cervical. En pacientes hemodinámicamente inestables se comenzará la infusión de líquidos intravenosos 51

cuidadosamente según pulso radial. Optimizar analgesia con AINES u Opioides. Una vez descartadas las complicaciones con riesgo vital (ver capítulo Trauma Torácico Grave), valorar lesiones sin riesgo vital inminente, como son: - Neumotórax Simple (O2 con FiO2 superior al 85% y vigilancia cuidadosa; toracocentesis y colocación de drenaje pleural si hay signos de IR o si el colapso pulmonar es superior al 10%) o - Fracturas Costales (Movimiento de los fragmentos, resalte palpable en la pared torácica, disnea, taquipnea, equimosis y crepitación. Tratamiento con analgesia, fisioterapia respiratoria precoz y si es necesario, apoyo ventilatorio; Nunca se deben practicar vendajes circulares).

Condición

Manejo específico

Contusión pulmonar

1.- Aumento FiO2 inspirada: Mascarilla al 40% / 2.Optimización del aporte de volumen / 3.- Analgesia.

Derivar urgente a centro Ruptura de aorta especializado para corrección y grandes vasos quirúrgica. Ruptura Derivar urgente para Cirugía. traqueobronquial Traqueostomía de urgencia, Ruptura laríngea derivar urgente para reparación definitiva.

Hernia diafragmática

Derivar para resolución quirúrgica: Liberación de la estructura herniada de las adherencias intratorácicas (toracotomía).

Contusión miocárdica

Derivar urgente para Corrección quirúrgica.

Obstrucción aérea baja

Derivar para Fibrobroncoscopia rígida o flexible que permite la vía extracción del cuerpo extraño. Si se complica requerirá cirugía con resección pulmonar.

Seguimiento. Por especialista.

52

La infección y enfermedad por tuberculosis (TBC) está ligada a una serie de factores de riesgo: pobreza, hacinamiento, desnutrición e inmunosupresión, concentrándose los casos en hombres de edad avanzada y pacientes VIH/SIDA.

Tuberculosis Pulmonar Nivel de manejo del médico general Diagnóstico: Específico. Tratamiento: Completo. Seguimiento: Completo.

El bacilo se transmite por vía aérea como areosol con más de 50m de alcance desde los pacientes bacilíferos (pacientes con tuberculosis activa, baciloscopía positiva) principalmente al toser. En la primoinfección, el bacilo alcanza la vía aérea distal originando un cuadro inflamatorio inespecífico. Los bacilos, fagocitados por macrófagos alveolares, la mayoría de las veces son eliminados por el sistema inmune, pero si esto no ocurre se reproducen sin ser destruidos, invadiendo células cercanas (células epiteliales y endoteliales). Pueden ser transportados vía linfática a linfonodos pulmonares, produciéndose una linfoadenitis. Al desencadenarse la respuesta inmune a la infección, las células inmunes forman un infiltrado, granuloma, luego forman una capa fibrosa calcificada formando el complejo primario o de Ghon, normalmente autolimitado, siendo la multiplicación bacilar controlada por el sistema inmune y los granulomas resueltos, dejando cicatrices. En caso de no mediar un control de la multiplicación, el complejo progresará con compromiso extenso del parénquima, dando lugar a tuberculosis pulmonar o primaria. En caso de compromiso de vasos sanguíneos pulmonares, puede ocurrir bacteriemia, responsable de la diseminación hematógena del bacilo (granulia o milia) y la aparición de cuadros extrapulmonares. Si el sistema inmune del huésped logran controlar el crecimiento del bacilo pero no eliminarlo completamente, no se desarrollara una enfermedad clínica, pero hay un riesgo de 5 a 10% de que esto si ocurra, desarrollándose tuberculosis post primaria.

Aspectos esenciales     



Enfermedad ligada a pobreza, hacinamiento e inmunosupresión (VIH). En todo paciente sospechoso deben realizarse 2 baciloscopías + cultivo. El diagnóstico es bacteriológico, con baciloscopía y/o cultivo positivos. Pronóstico sombrío de no mediar tratamiento. El tratamiento siempre debe ser asociado, prolongado, controlado y normado. Se realizaron cambios importantes en los distintos esquemas de tratamiento, los cuales son válidos desde 2014 en Chile. Se debe estar atento a sus formas extrapulmonares, como la pleural y ganglionar.

Caso clínico tipo Paciente de 41 años, obrero, no fumador, sin antecedentes mórbidos de relevancia. Consulta por marcado CEG, fiebre objetivada en 38,5° y tos productiva mucopurulenta de 3 semanas de evolución. Se solicita radiografía de tórax, que muestra infiltrado de unos 5 cm de diámetro, de límites imprecisos y borrosos localizado en vértice pulmonar derecho.

Historia natural de la infección:  

Definición 

Enfermedad infecto-contagiosa producida por el Mycobacterium tuberculosis (Bacilo de Koch). Normalmente afecta primariamente a los pulmones, pero puede extenderse a otros órganos.



En el mundo esta dentro de las enfermedades infectocontagiosas que causas mayor morbilidad y mortalidad, si bien contamos con una terapia eficaz para curar cada caso, estamos lejos de lograr la erradicación de la enfermedad.

Eliminación inmediata desde el organismo Enfermedad primaria: inicio inmediato de la enfermedad activa. Nueva infección por tuberculosis o enfermedad activa en paciente previamente virgen. Infección/Tuberculosis latente: pacientes con respuesta inmune al bacilo, pero sin desarrollo de la enfermedad, persona PPD +, no vacunada. Reactivación de la enfermedad: inicio de la enfermedad activa varios años luego de un periodo de infección latente.

Diagnóstico El diagnóstico es clínico + laboratorio.

Etiología-epidemiología-fisiopatología

Manifestaciones clínicas:

En Chile la incidencia de TBC según informe del MINSAL fue de 13,4 por 100.000 habitantes, estando dentro de la fase epidemiológica de umbral de eliminación (< 20 por 100.000 habitantes). En las décadas previas Chile presentaba buenos resultados en relación a la caída de la mortalidad y morbilidad por TBC, en los últimos años se ha producido un estancamiento en la incidencia que no ha permitido lograr la meta de eliminación avanzada (incidencia menos de 10 por 100.000 habitantes). Según la proporción de casos de TBC los grupos de riesgo más importantes serian coinfección con VIH, extranjeros, alcoholismo y drogadicción, entre otros.

Tuberculosis Primaria: clínica lentamente progresiva, fiebre es el síntomas mas frecuentes, 1/3 de los pacientes pueden presentar síntomas y signos respiratorios, 25% desarrollan dolor pleurítico o retroesternal. Otros síntomas incluyen fatiga, tos, artralgias y faringitis. La radiografía usualmente normal, se pueden observar adenopatías perihiliares, derrame pleural, infiltrado pulmonar perihiliar y hemitórax derecho (lo más común), la ubicación en los vértices pulmonares es muy sugerente. Reactivación de tuberculosis: frecuentemente compromete los segmentos apicales posteriores de los lóbulos pulmonares superiores o el segmento superior de 53

los lóbulos pulmonares bajos. Puede permanecer sin diagnóstico y potencialmente infecciosa por 2 a 3 años o más, con un desarrollo tardío de los síntomas en el curso de la enfermedad. Presentación insidiosa de la sintomatología, muchos pacientes tienen síntomas vagos e inespecíficos, la mitad a 2/3 de los pacientes desarrollan tos (a medida que progresa la enfermedad se va haciendo más productiva, color amarillo o amarillo verdoso y ocasionalmente con estrías de sangre), pérdida de peso y fatiga. Fiebre y sudoración nocturna se presenta en aproximadamente 50% de los pacientes. 1/3 presenta dolor torácico y/o disnea, ¼ hemoptisis. Sin tratamiento pueden presentar ulceras dolorosas en la boca, lengua, laringe o tracto gastrointestinal mientras desarrolla expectoración crónica y por la deglución de secreciones altamente infecciosas. El examen físico suele ser inespecífico, se puede encontrar opacidad y diminución de murmullo pulmonar (derrame pleural), crépitos inspiratorios o asociados a la tos, signos de consolidación, entre otros. La radiografia en la mayoría de los pacientes se encuentra alterada, con infiltrados intersticiales en los segmentos mencionados anteriormente. En algunos casos, menos frecuente, pueden presentar patrones radiográficos atípicos, adenopatías hiliares (en algunos casos asociado a colapso del lóbulos medio derecho), infiltrado o cavitaciones en zonas medias o bajas del pulmón, derrame pleural y nódulos solitarios.

diagnóstico, y si tiene bacteriología positiva se considera caso secundario de TBC y debe ser tratado.

Tratamiento En el año 2014, se introdujeron cambios importantes al tratamiento de TBC en Chile, basado en las últimas recomendaciones internacionales. Los principales cambios están enfocados en:     

 

Complicaciones de la Tuberculosis: Hemoptisis, neumotórax, bronquiectasias, bronquiolitiasis (presencia de material calcificado u osificado al interior del lumen del árbol bronquial), destrucción extensa del pulmón, shock séptico, malignidad (TBC pulmonar aumentaría el riesgo de cáncer pulmonar), entre otros.



Practicar estudio de susceptibilidad a todos los enfermos, nuevos o antes tratados. Aumentar frecuencia de administración en la segunda fase de tratamiento, de dos a tres veces por semana. Disminución de las dosis de etambutol (para disminuir posibilidad de toxicidad ocular). Mantención de etambutol en la fase intermitente de tratamiento en los casos que sigan con BK positivas en el segundo mes de terapia. Prolongación del tratamiento a 9 meses en casos muy avanzados, en aquellos que persisten con BK positivas al segundo mes de tratamiento y en los co infectados por VIH. Eliminación del esquema secundario de "antes tratados" --> Ahora reciben esquema primario. Eliminación del esquema simplificado en los enfermos con bacteriología no demostrada ---> Reciben esquema primario. Cambios en los Esquemas de Retratamiento en los pacientes portadores de TBC Multirresistentes (TB-MDR).

La administración del tratamiento es de preferencia ambulatorio y siempre estrictamente supervisado, los fármacos se administran todos juntos. El esquema de tratamiento consiste en:

Laboratorio: En la TBC pulmonar las principales técnicas de laboratorio diagnósticas corresponden a la baciloscopía y el cultivo. La baciloscopía (tinción de Ziehl-Nielsen) permite la identificación de bacilos ácido-alcohol resistentes y que en nuestro medio, dada la infrecuencia de mycobacterias no tuberculosas en aparato respiratorio, es prácticamente diagnóstica. El cultivo de Koch, es el gold standard ya que es más sensible y específico, pero dado el tiempo que demora (30-60 días) se hace necesaria la baciloscopía.

ESQUEMA PRIMARIO (Vírgenes a Tratamiento/ Antes tratados (recaídas y abandonos reingresando, excluyendo fracasos de tto) / Con o sin confirmación bacteriológica)

En todo paciente sospechoso de TBC pulmonar, deben realizarse 2 baciloscopías y el cultivo de 1de ellas, en caso de pertenecer a grupo de riesgo ambas muestran deberán ir a cultivo. La prueba de tuberculina o PPD también puede ser usada, principalmente indicada en población infantil. Tambien existen en la actualidad otros métodos más modernos capaz de medir la respuesta inmune en forma más específica midiendo la liberación de interferón gama por linfocitos al exponerlos a antigenos del bacilo de Koch, los denominados IGRAS (Interferon G Release Assays) como el TPSOT-TB, ELISPOT test y Quantiferon Gold in tuve test (QFT-GIT). Tanto el PPD como los IGRAS no discriminan entre infección o enfermedad.

Drogas

Isoniazid Rifampicin Pirazinamid Etambut a a a ol

Fase diaria: 50 dosis / 2 300 meses (mg) Fase trisemana l: 48 dosis 600 / 4 meses (mg)

Estudio de contactos: contacto es todo aquel que ha estado expuesta al contagio con un enfermo de TBC con bacteriología positiva, intradomiciliarios (quinees viven con el caso índice) y extradomiciliarios habituales (relaciones frecuentes con caso índice, más de 6 horas diarias). Se realizará estudio con Rx Tx, Baciloscopía y cultivo. Si ambos resultan normales el sujeto se considera sano, si la Rx resulta con lesiones sospechosas pero bacteriología negativa se realizara seguimiento

600

1500

800

600

-

1400(*)

Las dosis deben ajustarse al peso cuando sea < 45 kg o > 70 Kg. (*) Uso solo en caso de resistencia inicial a la isoniazida, o cuando este dato se desconozca y la BK permanezca positiva al segundo mes de tratamiento, deberá mantenerse la administración de etambutol en la segunda

54

fase, hasta BK negativa o se demuestre sensibilidad a la isoniazida. En la mayoría de los casos el tratamiento se mantiene con una duración total de 6 meses, sin embargo, en casos muy avanzados, o aquellos que persisten con BK positivas al segundo mes de tratamiento y en los co infectados con VIH, el tratamiento se puede prolongar por 9 meses. La quimioterapia tuberculosa presenta múltiples RAM, como hepatotoxicidad, reacciones alérgicas, alteraciones hematológicas, artralgias, etc y se deben repasar las más clásicas. Tiene muchas interacciones en especial el efecto inductor de CYP de rifampicina.

Seguimiento Se debe controlar al paciente al inicio del tratamiento, mensualmente durante el tratamiento, al alta y seis meses posteriores para evaluar mejora clínica, evolución bacteriológica, adherencia a tratamiento, ajuste de dosis si cambio en el peso y RAM a fármacos. Se debe realizar baciloscopía y cultivo mensualmente durante el tratamiento (identificando precozmente recaídas, fracaso o abandono del tto) y RxTx al inicio y termino del tratamiento para evaluar regresión de las lesiones. 

Recaídas: 2 BK (+) posteriores al término del tratamiento.



Sospechosos de fracaso: mantienen cultivo positivo de la baciloscopia al tercer mes, cultivo debe ser enviado a estudio de susceptibilidad.



Fracaso: Persistencia decultivo (+) de la baciloscopía luego de 4 meses de tratamiento o al termino del tratamiento, después de un período de negativización transitoria de dos meses reaparece baciloscopía positiva por dos meses consecutivos. Cultivo (+) deben ser enviados a estudio de susceptibilidad y paciente derivado a nivel secundario.



Abandono: Interrupción de 4 o más semanas en cualquier fase del tratamiento.

También es importante pesquisar la ocurrencia de RAM, que son relativamente frecuentes (2-3%): hepatotoxicidad, reacciones alérgicas, alteraciones hematológicas, artralgias, etc. En caso de ocurrir debe referirse a especialista, suspendiendo el fármaco involucrado o el esquema completo si éste no puede identificarse.

55

Tuberculosis Pulmonar (Fracaso al tratamiento)

fracasen con el esquema primario. El año 2014 en Chile se presentaron 15 casos de tuberculosis multidrogorresistente (TB-MDR). La mayoría de los TBMDR sería secundario a un uso inadecuado de los fármacos.

Nivel de manejo del médico general: Diagnóstico Específico Tratamiento Inicial Seguimiento Derivar.

Diagnóstico Todo paciente que: 1. Mantiene cultivos de baciloscopía positivas hasta el 4to mes de tratamiento. 2. Después de un período de negativización transitoria de dos meses reaparece baciloscopía positiva por dos meses consecutivos.

Aspectos esenciales  

 



Representa el 1% de los enfermos que inician tratamiento. Se considera fracaso al tratamiento primario a todo paciente que: Mantiene baciloscopías positivas hasta el 4to mes de tratamiento, o que después de un período de negativización transitoria de dos meses reaparece baciloscopía positiva por dos meses consecutivos. El fracaso debe confirmarse por cultivo + estudio de susceptibilidad. Las drogas utilizadas en el esquema inicial son: Kanamicina, Moxifloxacino, Etionamida, Pirazinamida y Etambutol (KMEtZ). Por al menos 4 meses. El esquema de continuación utiliza las mismas drogas, excepto Kanamicina, a las mismas dosis por al menos 12 meses.

Clínicamente tienen una presentación comparable a aquellos pacientes que desarrollan una TBC sensible a tratamiento. Existen predictores que nos pueden hacer sospechar aquellos pacientes que podrían desarrollar resistencia a las drogas antiTBC y fracaso al esquema habitual, entre los que destacan un episodio previo de TBC tratado, clínica o signos radiográficos persistentes o progresivos durante la terapia, residir o viajar a un país con alta prevalencia de resistencia a fármacos anti TBC, exposición a un individuo que tuvo o se sospecha una TBMDR, etc. El fracaso siempre debe confirmarse por cultivo.

Tratamiento Caso clínico tipo

Para aquellos pacientes en los que el Estudio de Susceptibilidad confirme que el paciente presenta una Tuberculosis Multidrogorresistente deberán ser tratados con medicamentos de segunda línea los cuales son menos eficaces, con menor poder esterilizante, requiriendo un tratamiento más prolongado en base al siguiente esquema:

Paciente que pese al tratamiento no recupera peso, ni hay mejora en la radiografía de tórax, y se asocia con persistencia de baciloscopías positivas o positivización de estas luego de un periodo negativo.

Toda prescripción de esquema de retratamiento deberá ser comunicada de inmediato por correo electrónico al Nivel Central del Programa de Control y Eliminación de la Tuberculosis (PROCET), a través de la ficha de notificación de pacientes con tuberculosis multirresistentes (TB-MDR), para la solicitud de fármacos de 2da línea, a la cual se debe anexar el estudio de susceptibilidad a todas las drogas empleadas.

Definición Paciente que pese a tratamiento adecuado contra tuberculosis, persiste con exámenes positivos, o estos se vuelven a hacer positivos, luego de un periodo de negativización. Tuberculosis Multidrogorresistente: M. Tuberculosis que presenta resistencia simultánea a Isoniacida y Rifampicina, los medicamentos antituberculosos más efectivos, y posiblemente a otros agentes adicionales.

Esquema Normado de Retratamiento enfermos de 45 a 70 Kg de peso. Fase Fase Inicial (>o Drogas continuación (> o = = a 4 meses) a 12 meses)

Etiología-epidemiología-fisiopatología Del 100 % de los enfermos que iniciaron tratamiento antituberculoso, el 1% entró dentro de la definición de fracaso, según las estadísticas del país entre 2003 – 2006, el año 2013 solo un 0,4% correspondió a fracaso de tratamiento, Se asocia a multiresistencia de los agentes empleados en el esquema de tratamiento, desde el año 2014 comienza a regir una nueva Norma Técnica del Programa de Tuberculosis la cual indica la realización de vigilancia universal de la resistencia a los fármacos antituberculosos, a todos los casos de TBC confirmados mediante cultivo se les realizará un estudio inicial de susceptibilidad a los medicamentos, además de aquellos pacientes que 56

Kanamicina

750(*)

Etionamida

750

750

Moxifloxacino 400

400

Pirazinamida 1.500

1.500

Etambutol

800

800

para

(Ajustar en pacientes con peso inferior a 45 kg, o mayor a 70 kg) (*) 500 mg. en pacientes mayores a 50 años. Fase inicial: 1. Esquema: KMEtZe     

Kanamicina Moxifloxacino Etionamida Pirazinamida Etambutol

1. Duración: Al menos 4 meses. 2. Tratamiento  

Pacientes ambulatorios: Lunes a viernes. Pacientes hospitalizados: Lunes a domingo.

Fase de continuación: 1. Esquema: MEtZE, en las mismas dosis que en fase inicial. 2. Duración: Por lo menos 6 meses después denegativizado el cultivo en dos muestras mensuales consecutivas, con una duración total del tratamiento que no podrá ser menor de 1 año. 3. Tratamiento  Pacientes ambulatorios: Lunes a viernes.  Pacientes hospitalizados: Lunes a domingo. Cuando el estudio inicial de susceptibilidad demuestre resistencia a pirazinamida o etambutol, se debe consultar con el Nivel Central del PROCET la posibilidad de cambiar el esquema de retratamiento. Cirugía de resección pulmonar: Para pacientes con TBMDR que cumplan los siguientes requisitos: 1. Lesiones localizadas 2. Buena reserva cardiopulmonar 3. Falta de un número suficiente de fármacos efectivos que aseguren que se dispone de un esquema curativo.

Seguimiento Todos los pacientes con diagnóstico de TBMDR deben ser derivados a Nivel Secundario. El control del retratamiento se debe realizar (baciloscopía + cultivo) mensualmente hasta el término del tratamiento. Persistencia de baciloscopías positivas al 6º mes o su reaparición con resultado positivo en cualquier control posterior constituye sospecha de fracaso de retratamiento, el cual debe ser confirmado por cultivo y enviar a nuevo estudio de sesceptibilidad.

57

MÓDULO 1: Medicina Interna

Gastroenterología • 2. • • 3. • •

Ascitis Nivel de manejo del médico general: Diagnóstico Específico Tratamiento Inicial Seguimiento Derivar

No relacionada con hipertensión portal GASA < 1,1 • Peritonitis: TB, rotura visceral (aumento de amilasa). • Carcinomatosis Peritoneal. • Pancreatitis. • Vasculitis. • Estado hipoalbuminémicos (sd. nefrótico, enteropatía con pérdida de proteínas). • Síndrome de Meigs (Tumor ovárico). • Obstrucción/Infarto intestinal. • Extravasación linfática postoperatoria.

Aspectos Esenciales • • • •

Neoplasia maligna extensa (CHC o metástasis). Postsinusoidal. ICC derecha. Síndrome de Budd-Chiari. Presinusoidal (asociada a várices > ascitis). Trombosis vena portal o esplénica. Esquistosomiasis.

El diagnóstico diferencial de ascitis se realiza por medio de paracentesis. Es fundamental conocer la técnica del procedimiento. Se debe realizar paracentesis en todo paciente con ascitis de reciente comienzo. Siempre descartar infección de líquido ascítico ya sea PBE o PBS.

Fisiopatología: • Hipertensión Portal: Vasodilatación sistémica (por liberación de NO), disminución de volumen arterial efectivo y retención renal de Na. • Disminución de la presión oncótica sérica: Relacionado con hipoalbuminemia y aumento de producción hepática de linfa.

Caso Clínico Tipo Paciente con antecedente de daño hepático crónico de larga data consulta por disnea asociado a aumento de diámetro abdominal de 3 días de evolución.

Diagnóstico

Definición

Clínica: Aumento de perímetro abdominal, aumento de peso, hernia abdominal de nuevo desarrollo, dolor abdominal, disnea, naúseas, saciedad precoz. Examen Físico: Matidez en flancos (VPN aprox 90%, se necesitan >1500 ml), matidez cambiante (Se. 83%). Imagenología: Ecografia detecta > 100 ml; RM/TC contribuyen al diagnostico diferencial. Paracentesis: Indicada en todo paciente con ascitis de nuevo comienzo y considerarla en todos los cirróticos hospitalizados con asci-

Se entiende por ascitis a la acumulación de líquido en el espacio que existe entre el revestimiento abdominal y los órganos abdominales.

Etiología-Epidemiología-Fisiopatología Etiología: Relacionada con hipertensión portal GASA > 1 1. Sinusoidal: • Cirrosis (81%), incluída PBE. • Hepatitis Aguda.

1 Facultad de Medicina, Universidad de Chile

tis. Complicaciones en <1% (hemorragia). La ecografía aumenta la tasa de éxito pero no disminuye las complicaciones. Gradiente de albúmina entre suero y ascitis (GASA): Precisión 95% para la hipertensión portal. • > 1 g/dL relacionada con HPP. • < 1,1 g/dL no relaciada con HPP. Si existe HPP más otra causa (registrada en aprox 5%) el GASA se mantiene > 1. Si hay cirrosis conocida y GASA < 1,1 pero sin ninguna otra causa fácilmente identificable, es probable que sólo haya cirrosis. Proteínas totales en líquido ascítico (PTLA): Útil cuando GASA > 1 para diferencias entre cirrosis (PTLA < 2,5 g/dl) de la ascitis de origen cardíaco (PTLA > 2,5 g/dl). Descartar infección: El recuento celular con diferencia más tinción Gram/cultivo define la peritonitis bacteriana. Solicitar cultivo fúngico en caso de hospitalización prolongada, uso de antibióticos; cultivo BAAR y ADA en sospecha de TBC.

Tratamiento Si la ascitis es secundaria a hipertensión portal: Disminución de ingresta de NA y diuréticos (espironolactona y furosemida); en casos resistentes realizar paracentesis o DPSIT. Si no se relaciona con hipertensión portal tratar según la causa subyacente.

Seguimiento Por especialista.

Autor / Editor Hernán Rubilar

Año 2016

2 Facultad de Medicina, Universidad de Chile

cópica completa la mediana de sobrevida es de 1 a 2 años, con sobrevida a 5 años de entre 5 a 15%.

Cáncer de Páncreas Nivel de manejo del médico general: Diagnóstico Sospecha Tratamiento Inicial Seguimiento Derivar

Diagnóstico •

Aspectos Esenciales • • • • •

Patología de baja incidencia, pero en aumento y con una mortalidad elevada. El más común es el adenocarcinoma ductal y la ubicación más frecuente es la cabeza pancreática. Su principal manifestación es la Ictericia obstructiva indolora. El examen de elección es la TAC de abdomen. Único tratamiento curativo es la cirugía, sin embargo al momento del diagnóstico sólo entre un 10-20% presentan enfermedad limitada y resecable.



• •

Caso Clínico Tipo



Paciente de 55 años, sexo femenino, consulta por un cuadro de ictericia moderada, CEG y pérdida de peso de 10 kilos durante 3 años de evolución. Al examen físico se palpa una masa abdominal indolora ubicada en el cuadrante superior derecho del abdomen.



Screening: Actualmente no se recomienda screening para el cáncer de páncreas para la población general. La Soc. Chilena de Gastroenterología (2013) recomienda estudios para pacientes con antecedentes de más de un familiar afectado con cáncer de páncreas, y realizar al menos un estudio de imagen abdominal en pacientes diabéticos sin otros factores de riesgo, de preferencia TAC de abdomen. Clínica: Ictericia obstructiva Dolorosa (75-90%). Síntomas constitucionales: Fatiga, pérdida de peso y anorexia. Dolor es su principal síntoma, de carácter continuo (diurno y nocturno), exacerba con las comidas, epigástrico, irradiación a hipocondrios y región dorsolumbar. Al examen físico: Signo Courvoisier-Terrier positivo (masa palpable no dolorosa). Otras Presentaciones: Sd Malabsorción y Esteatorrea, Diabetes Mellitus en el año previo (15%), Ascitis o Sd de retención gástrica. Estudio: El examen de elección para el diagnóstico y etapificación es la TAC abdominal. Tiene un rendimiento cercano al 100% para determinar irresecabilidad. Criterios TAC de potencialmente irresecable son: compromiso extrapancreático; invasión de la vena porta y mesentérica superior; o invasión de la arteria mesentérica superior, celíaca y hepáticas.

En caso de no verse al TAC: ColangioRM o Endosonografía. • Ecografía: sólo en ictericia. Puede ser útil en demostrar dilatación de la vía biliar y presencia de cálculos.

Definición Patología neoplásica maligna del páncreas.

Tratamiento Etiología-Epidemiología-Fisiopatología

Resección quirúrgica curativa (Operación de Whipple o Pancreatoduodenectomía). La sobrevida posterior a la resección del Adenocarcinoma ductal es de 18 meses en promedio. En caso de tumor irresecable se debe realizar tratamiento paliativo basado en 3 pilares: • Control del dolor: Mediante AINES/Opioides o alcoholización del plexo celíaco. • Obstrucción duodenal: Por Gastroyeyuno anastomosis o instalación de una prótesis endoscópica. • Alivio de la ictericia: Mediante tratamiento quirúrgico, endoscópico o percutáneo.

Representa el 3% de todos los cánceres, pero es la 4° causa de muerte por cáncer. Es más frecuente sobre los 65 años de edad. El más frecuente es el cáncer periampular (80%), siendo además el de peor pronóstico (<3% a 5 años). El tipo más común es el Adenocarcinoma ductal (75%) y la ubicación más frecuente es en la cabeza y el proceso uncinado de páncreas (70%). Factores de riesgo: Tabaquismo, dieta rica en grasas y pobre en fibra, alcohol, diabetes mellitus, antecedentes familiares de cáncer de páncreas especialmente antes de los 50 años y pancreatitis crónica (eleva el riesgo en 20 veces independiente de la causa). Entre 80% y 90% son irresecables al diagnóstico, ya sea por ser localmente avanzados (30%) o metastásicos (50%). Sobrevida general es de 6 a 12 meses en el cáncer localmente avanzado sin metástasis y de 3 a 6 meses en presencia de metástasis. En pacientes en que se logra resección macros-

Seguimiento Por especialista. Autor / Editor Hernán Rubilar

3 Facultad de Medicina, Universidad de Chile

Año 2016

nifestación clínica suele ser más precoz, aparece con mayor frecuencia en pacientes con Colangitis esclerosante primaria (asociada a Colitis Ulcerosa). El colangiocarcinoma distal se considera un tumor periampular. Las mayores incidencias en Chile se presentan en las regiones VIII, IX y X, la letalidad más alta del mundo está en la araucania norte.

Cáncer de Vesícula y Vía Biliar Nivel de manejo del médico general: Diagnóstico Sospecha Tratamiento Inicial Seguimiento Derivar

Otros factores de riesgo: Infección crónica con Salmonella Typhi, fármacos (metildopa, ACO, isoniazida), obesidad.

Aspectos Esenciales • • • • • •

Primera causa de muerte oncológica en la mujer. Ante la sospecha clínica el método diagnóstico de primera elección es la ecografía abdominal. Prevención: colecistectomía preventiva. El diagnóstico es tardío. Tiene baja resecabilidad. El pronóstico de los colangiocarcinomas es peor cuanto más proximales son en la vía biliar.

Diagnóstico El diagnóstico es en general tardío. Carcinoma vesicular: Clínica similar a una colelitiasis y colecistitis, con dolor continuo en hipocondrio derecho, náuseas y vómitos. Es frecuente la pérdida de peso, ictericia y masa palpable, Signo de Courvoisier Terrier (hace referencia a la distención de la vía biliar, con ictericia mecánica, y acolia; cuando hay obstrucción de ampolla de vater por neoplasia de éste o Ca de Cabeza de Páncreas, o cálculo en conducto hepático común: hipertermia, escalofríos y dolor). El estudio se realiza con ecografía abdominal, y, para la extensión con TAC.

Caso Clínico Tipo Paciente mujer de 60 años que consulta por CEG, baja de peso y dolor abdominal asociado a ictericia y acolia. Al examen físico presenta masa palpable en hipocondrio derecho.

Laboratorio: Perfil hepático con patrón colestásico. Biopsia de pieza operatoria: Las lesiones tempranas son detectadas en su mayoría luego del examen de pieza operatoria de colecistectomía. Sólo el 10 % del total de neoplasias de vesícula son detectados de esta forma.

Definición Patología neoplásica maligna de vesícula (Cáncer vesicular) y vía biliar (Colangiocarcinoma). El Colangiocarcinoma es la neoplasia maligna de células epiteliales de los ductos biliares intra o extra hepáticos. Son esclerosantes (patrón infiltrativo) bien diferenciados.

Colangiocarcinoma: Las manifestaciones más frecuentes son síntomas constitucionales e ictericia. La colangiografía y TAC son las mejores técnicas diagnósticas. Se estadifica con la clasificación TNM.

Etiología-Epidemiología-Fisiopatología

Tratamiento

El Cáncer de Vesícula representa el 95% de los tumores del sistema biliar. En Chile constituye la primera causa de muerte por causa oncológica en la mujer. La principal asociación es con la colelitiasis (más del 95% de los casos) y vesícula en procelana. Es más frecuente en mujeres mayores de 65 años y el 80% son adenocarcinomas.

Carcinoma Vesicular: Colecistectomía con segmentectomía IV B y V, en caso de estar localizado. En general, tiene mal pronóstico ya que el 90% de los casos diagnosticados está en etapa avanzada. Son contraindicaciones para la resección: Invasión vascular, ascitis, compromiso hepático difuso, invasión peritoneal, compromiso a distancia y pobre estado general. Paliación: Terapia endoscópica o tratamiento percutáneo. QMT. Colangiocarcinoma: En el caso del intrahepático está indicada la resección hepática. Pronóstico es muy malo porque el 75% de los casos al diagnóstico ha metastizado. En el colangiocarcinoma extrahepático se indica la resección del tumor con reconstitución bilioentérica (hepaticoyeyunostomía en Y de Roux) con RDT postoperatoria. El pronóstico de los colangiocarcinomas es peor cuanto más proximales son en la vía biliar. En el caso de paciente sin indicación quirúrgica, tanto el intra como el extra hepático se

Colangiocarcinoma: Tumor poco frecuente, que afecta mayormente a varones de edad avanzada. Más común la ubicación intrahepática. Presenta diseminación linfática y por contigüidad. Se asocian a condiciones de colestasis crónica: Anomalías congénitas ductales (Enfermedad de Caroli y quistes de colédoco), atresia biliar, colangitis esclerosante primaria y colitis ulcerosa. Tumor de Klatskin o colangiocarcinoma hiliar: Aparece en el hepático común cerca de la bifurcación, por lo que la ma-

4 Facultad de Medicina, Universidad de Chile

pueden tratar con endoprótesis biliares paliativas.

Seguimiento Prevención: Colecistectomía en paciente litiásicos independiente de si es sintomática o no. Patología GES: colecistectomía electiva en pacientes entre 35 y 49 años. Seguimiento es por especialista.

Autor / Editor Nevelline Salgado

Año 2016

5 Facultad de Medicina, Universidad de Chile

Cáncer Gástrico

Factores de riesgo: Ambientales: Dieta rica en nitratos, sal, alimentos ahumados y conservados en vinagre, bajo consumo de alimentos ricos en antioxidantes, tabaquismo, nivel socioeconómico bajo, H. pylori. En cuanto a los factores genéticos, los familiares de primer grado de un caso de cáncer gástrico tienen un mayor riesgo de hacer una neoplasia gástrica.

Nivel de manejo del médico general: Diagnóstico Sospecha Tratamiento Inicial Seguimiento Derivar

También se conocen condiciones mórbidas predisponentes de esta patología, entre ellas está el esófago de Barret, gastritis atrófica, anemia perniciosa, metaplasia intestinal, displasia gástrica, adenomas gástricos y gastrectomía subtotal.

Aspectos Esenciales • • •

El cáncer gástrico corresponde generalmente a un adenocarcinoma. El pronóstico está en directa relación con el estadio al momento de la confirmación diagnóstica. La erradicación de Helicobacter pylori influye en la no progresión y en la regresión de las lesiones preneoplásicas.

Caso Clínico Tipo Paciente hombre de 64 años con epigastralgia de más de 20 días de duración, sensación de pesadez y saciedad precoz, y baja de peso de 8 kilos en 6 meses.

Definición Neoplasia maligna que se origina en el estómago.

Origen

Gastritis, gastritis crónica activa, gastritis atrófica, metaplasia intestinal, displasia, adenocarcinoma intestinal.

Edad

> 65 años.

Predominio

Hombres.

Factores asociados

Dietéticos y ambientales, H. pylori, tabaco y OH.

Localización más frecuente

Antro.

Metástasis

Vía vascular.

Frecuencia

Zonas de alto riesgo (epidémico).

Pronóstico

Mejor pronóstico.

Etiología-Epidemiología-Fisiopatología

Diagnóstico

Epidemiología: Es más frecuente en el sexo femenino. En Chile, la mortalidad es de 28 a 20 por 100.000 habitantes, manteniéndose estable desde la década de los 80. La mayor incidencia se concentra en las regiones del Maule y Ñuble.

Suele ser asintomático en estadios tempranos. Dentro de las manifestaciones clínicas están: Pérdida de peso, dolor abdominal “sordo”, náuseas y vómitos, anorexia, disfagia y mal vaciamiento gástrico, hematemesis y melena, saciedad precoz. El screening de rutina no está recomendado para la detección de cáncer gástrico. La endoscopía digestiva alta es indispensable en personas sintomáticas y con factores de riesgo familiar. Las metástasis más frecuentes son a hígado, peritoneo y pulmón. El diagnóstico es endoscópico (con biopsia).

Etiopatogenia: Se piensa que la baja incidencia se relaciona con cambios dietéticos, como el aumento de consumo de frutas y verduras, menor consumo de sal y mejores condiciones sanitarias, lo cual disminuiría la infección por H. pylori. Se cree que el desarrollo del cáncer es un proceso de varios pasos, con una secuencia de epitelio normal a adenoma y carcinoma, cada paso asociado a una mutación genética (como el cáncer de colon). El hallazgo común suele ser una inflamación, la cual asociada a factores como H. pylori aumenta esta condición, llevando al desarrollo de gastritis atrófica, seguida de metaplasia intestinal, luego displasia y finalmente cáncer. El cáncer gástrico se puede dividir en 2 tipos: el intestinal (formación de estructuras tipo glándulas tubulares) y la difusa (más indiferenciada y de peor pronóstico).

Tratamiento El tratamiento de elección es la cirugía. En los casos avanzados la sobrevida promedio es de 6 meses. La sobrevida depende del grado de profundidad y del compromiso ganglionar, y existe a su vez una correlación directa entre ambos, a mayor profundidad, mayor compromiso ganglionar en distancia. La quimioterapia adyuvante ha logrado resultados promisorios en casos seleccionados.

6 Facultad de Medicina, Universidad de Chile

Seguimiento Enviar a la brevedad al especialista, se recomienda seguimiento con endoscopía.

Autor / Editor Victor Valdés

Año 2016

7 Facultad de Medicina, Universidad de Chile

factor mecánico debido a la fibrosis del hígado sino también juegan un rol esencial la contracción activa de las células estrelladas perisinusoidales las cuales se activan en la cirrosis y desarrollan esta capacidad contráctil logrando funcionar como pericitos sinusoidales. Cuando la presión portal aumenta se desarrolla una red de colaterales que derivan el flujo esplácnico a la circulación general sin pasar por el hígado.

Cirrosis Hepática Nivel de manejo del médico general: Diagnóstico Específico Tratamiento Inicial Seguimiento Derivar

Diagnóstico

Aspectos Esenciales • • • •

Para confirmar la cirrosis hepática es necesaria una biopsia hepática (gold standard), pero no siempre es necesario hacerla si se cuenta con una alta sospecha y no va a cambiar el manejo del paciente. También se puede contar con apoyo imagenológico, la ecografía abdominal es útil para evaluar el parénquima hepático y detectar manifestaciones extrahepáticas de la cirrosis. Las manifestaciones clínicas pueden variar desde muy inespecíficas, como pérdida de peso, anorexia, fatiga, hasta síntomas y signos característicos de descompensación hepática, donde se incluyen la ictericia, prurito, hemorragia digestiva alta, distensión abdominal por la ascitis, asterixis, ginecomastia, telangectasias, peritonitis bacteriana espontánea, hipertrofia parotídea, distribución femenina de vello, hepato y/o esplenomegalia. En cuanto a los exámenes de laboratorio podemos encontrar alteración en las aminotransferasas, elevación de bilirrubinemia, FA y GGT altas, INR aumentado, hiponatremia y trombocitopenia.

Sospecharla en pacientes con estigmas de daño hepático crónico. Gold standard para el diagnóstico: Biopsia hepática. Etiología variada: Enf. hepática alcohólica y no alcohólica, VHB y C, hemocromatosis, fármacos etc. Manejo dado por especialista.

Caso Clínico Tipo Paciente masculino de 55 años, con antecedente de ingesta OH crónica, acude a control en APS por sugerencia de familiares debido a distensión abdominal progresiva. Al examen físico destaca ginecomastia, disminución de vello genital y axilas, telangectasias y sensibilidad abdominal a la palpación.

Definición

Son factores predictivos de cirrosis hepática: • Presencia de ascitis. • Plaquetas <160.000/mm³ • Telangectasias. • Razón ALT/AST (GPT/GOT) baja. • INR aumentado.

Es una forma de manifestación de la enfermedad hepática crónica, definida por sus características anatomopatológicas: Distorsión de la arquitectura hepática con fibrosis difusa del hígado que forma tabiques delimitando nódulos de regeneración del parénquima remanente, en respuesta a un daño permanente, alterando la circulación intrahepática. Es el estado final de diversas enfermedad que afectan este órgano. Irreversible en etapas avanzadas.

Tratamiento El tratamiento de esta patología se centra en las complicaciones que puede presentar, ya que la cirrosis se considera irreversible la mayoría de las veces. Es importante centrarse en enlentecer la progresión de la enfermedad, de ser posible, previniendo nuevas noxas hepáticas, ajustando medicamentos con metabolismo hepático, manejar los síntomas y las alteraciones de labotario, prevenir y tratar oportunamente las complicaciones (ascitis, encefalopatía hepática, HDA, PBE, síndrome hepatorrenal y hepatopulmonar, hipertensión portal, etc) y gracias a un seguimiento oportuno dado por el especialista identificar el momento idóneo para un trasplante, si cumple los criterios.

Etiología-Epidemiología-Fisiopatología Etiología: Hay múltiples causas de enfermedad hepática que pueden manifestarse como una cirrosis. Entre las causas más comunes se encuentran: Enfermedad hepática alcohólica y no alcohólica, hepatitis crónica por virus VHB y VHC, hemocromatosis. Causas menos comunes incluyen la hepatitis autoinmune, cirrosis biliar primaria y secundaria, colangitis esclerosante primaria, fármacos, deficiencia de alfa-1 antitripsina, infecciones, enfermedad de Wilson, etc. Epidemiología: Según la encuesta nacional de salud del año 2010 existe un autorreporte de diagnóstico médico de daño hepático alcanza 2,8% a nivel nacional. Y una elevación de las transaminasas de 16% a nivel nacional en ambos sexos. Sin datos exclusivos de cirrosis hepática. Fisiopatología: La cirrosis produce un incremento de la resistencia vascular hepática, el cual no sólo se produce por el

Seguimiento Derivar a especialista.

8 Facultad de Medicina, Universidad de Chile

Autor / Editor Hernán Rubilar

Año 2016

to de triglicéridos.

Colelitiasis

Diagnóstico

Nivel de manejo del médico general: Diagnóstico Específico Tratamiento Inicial Seguimiento Derivar

La litiasis biliar se diagnostica con la ecografía abdominal, la cual se pesquisa en pacientes asintomáticos muchas veces, por la alta prevalencia en la población, o suele ser el estudio elegido ante la sospecha clínica. La colelitiasis se puede manifestar clínicamente de diversas formas o puede ser asintomática. La primera manifestación suele ser el cólico biliar que es un dolor opresivo, continuo y progresivo en hipocondrio derecho y epigastrio, que se presenta 2 a 3 horas posterior a una comida, acompañado de náuseas o vómitos, el episodio dura entre 15 minutos y 2 horas y cede gradual y espontáneamente.

Aspectos Esenciales • • •

Patología muy frecuente en Chile en comparación con el resto del mundo. Factores de riesgo: ser mujer, en edad fértil, multípara, obesidad, con ancestros mapuches. Producido por un desequilibrio de los componentes de la bilis.

Tratamiento Caso Clínico Tipo

La colelitiasis asintomática suele ser controversial en cuanto a su tratamiento, pero sería recomendable la cirugía por su baja morbimortalidad. También se cuenta con tratamiento médico con ácido ursodeoxicólico, dosis de 10 mg/kd/día. El cólico biliar es tratado con AINES y antieméticos para el alivio de los síntomas, con indicación de colecistectomía.

Paciente de 45 años, múltipara de 3, con antecedentes de Sd. metabólico e HTA. Consulta en SAPU por dolor intenso en hipocondrio derecho de 1 hora de evolución, tipo cólico, EVA 5/10, asociado a un episodio de vómito. Ha presentado un par de cuadros similares en los últimos meses.

Seguimiento

Definición

Efectuado por especialista.

La colelitiasis es la presencia de cálculos en la vesícula biliar. Se pueden clasificar según su composición en: colesterol, mixtos (colesterol y carbonato cálcico) y pigmentarios. En Chile, el 90% de los cálculos son de colesterol.

Etiología-Epidemiología-Fisiopatología Epidemiología: En Chile existe una alta prevalencia en comparación al resto del mundo tanto en autopsias como en población viva. Afecta mayormente a mujeres en edad fértil, con aparición precoz durante la juventud, cuya incidencia aumenta a medida que la población envejece, también aumenta en multíparas comparadas con nulíparas, en pacientes con obesidad y descendientes de población mapuche. Etiología y fisiopatología: El desarrollo de litiasis biliar se asocia al desequilibrio de componentes de la bilis, ocurre un defecto en la secreción de lípidos biliares lo que lleva a una sobresaturación biliar de colesterol, lo que forma una solución fisicoquímicamente inestable haciendo que precipiten los cristales de colesterol, primero a nivel microscópico y luego macroscópico. La secreción biliar de colesterol aumenta con la edad. El desarrollo de la colelitiasis no se correlaciona con el colesterol total sérico, pero sí con la disminución de HDL y aumen-

9 Facultad de Medicina, Universidad de Chile

Diagnóstico

Colestasia

La clínica es variable, desde asintomático con alteraciones de laboratorio hasta cuadros clínicos característicos como la ictericia y el dolor abdominal. En la anamnesis no olvidar averiguar si existe dolor abdominal, baja de peso, coluria intermitente, fiebre, prurito, consumo de fármacos (incluso suplementos alimenticios y hierbas) y antecedentes familiares. Signos clínicos característicos de la colestasia son el prurito, ictericia, aumento de pigmentación de la piel, presencia de xantelasmas (en CBP). Cuando se sospecha la colestasia hay que confirmarla bioquímicamente, con valores de FA y GGT. Los valores de bilirrubina pueden ser normales. Elevación de aminotransferasas no suele ser significativa.

Nivel de manejo del médico general: Diagnóstico Específico Tratamiento Inicial Seguimiento Derivar

Aspectos Esenciales • • • • •

Exploración complementaria de mayor utilidad: ecografía abdominal. FA y GGT suelen estar siempre elevados. Ecografía abdominal ayuda a determinar si es intra o extrahepática. En colestasias intrahepáticas sospechar cirrosis biliar primarias. No olvidar anamnesis dirigida a fármacos, suplementos alimenticios y hierbas.

Con la ecografía abdominal podremos clasificar la colestasia de intra o extrahepática, si la vía biliar se encuentra o no dilatada. El TAC obtiene resultados similares a la ecografía para la clasificación de la colestasia, pero la supera en busca de la etiología. La colangioRM las supera en creces estudiando la morfología del árbol biliar, pero no es de fácil acceso. En colestasia intrahepática está indicada la biopsia hepática percutánea por punción para llegar a un diagnóstico específico. Es de gran utilidad pedir anticuerpos mitocondriales buscando CBP.

Caso Clínico Tipo Paciente de 40 años, sexo femenino. Consulta por cuadro de 1 mes de evolución de prurito y dolor abdominal intermitente. Se ha automedicado antihistamínicos, sin lograr buena respuesta. Consulta porque averigua antecedentes familiares de cirugía biliar y le preocupa que tenga relación con su clínica.

Tratamiento Depende de su etiología. Generalmente el tratamiento es sintomático y dirigido a manejar las complicaciones del síndrome. Para el prurito, el tratamiento de primera línea son resinas de intercambio aniónico: colestiramina y coleveselam; como segunda línea antihistamínicos, rifampicina y sertralina. El uso de ácido ursodesoxicólico no es eficaz para el prurito, salvo en la colestasia del embarazo. En colestasia prolongadas, más de 3 meses, suplementar con calcio y vitamina D ya que disminuye su absorción, así como la de las vitaminas liposolubles. La terapia específica de cada patología es de manejo del especialista.

Definición Bloqueo del flujo biliar (total o parcial) que impide la llegada de la bilis desde el hepatocito al duodeno. Hay elevación de la bilirrubina, ácidos biliares, gamma-glutamiltranspeptidasa (GGT) y fosfatasas alcalinas (FA). La obstrucción puede ser a nivel intra o extrahepático. La retención de compuestos potencialmente tóxicos para el hepatocito puede llevar a necrosis y apoptosis de estas células, momento en que se elevan las transaminasas. Si la colestasia es prolongada puede producir fibrosis y terminar en cirrosis hepática.

Seguimiento

Etiología-Epidemiología-Fisiopatología

Efectuado por especialista.

Cualquier patología que logre la obstrucción parcial o total al flujo biliar. Entre las causas de colestasia intrahepática encontramos: hepatitis viral, tóxica, por fármacos, colestasia benigna postoperatoria, asociada a infecciones bacterianas, cirrosis biliar primaria (CBP), hepatitis autoinmune, colangitis esclerosante, cambios hormonales durante el embarazo, etc. Etiología extrahepática incluye: coledocolitiasis, neoplasias de cabeza de páncreas o vías biliares, colangiocarcinoma, pancreatitis, lesiones inflamatorias de los conductos biliares (colangitis esclerosante primaria), etc.

Autor / Editor Hernán Rubilar

10 Facultad de Medicina, Universidad de Chile

Año 2016

de llevar a la aparición de DACD.

Diarrea Asociada a Antibióticos

Diagnóstico

Nivel de manejo del médico general: Diagnóstico Específico Tratamiento Completo Seguimiento Completo

Clínica: La DACD presenta un amplio espectro de manifestaciones clínicas, que va desde una diarrea leve de curso benigno hasta una colitis intensa con desarrollo de megacolon tóxico y complicaciones intraabdominales y sistémicas que pueden llevar a la muerte del paciente.

Aspectos Esenciales • •

Usualmente la diarrea aparece dentro de la primera o segunda semana desde el unicio de la antibioticoterapia y continúa despuées de su suspensión. En un tercio de los casos, la diarrea se inicia después de concluído el tratamiento antibiótico y, a veces, alejado de ella hasta por un par de meses. Las deposiciones son abundantes, acuosas o mucosas, pueden contener sangre y se acompañan de dolor abdominal, fiebre y compromiso del estado general. En casos más prolongados e intensos puede apararecer deshidratación, depleción de electrolitos e hipoproteinemia (por colonopatía perdedora de proteínas). Unos pocos casos evolucionan en forma fulminante, con pancolitis, hemorragia o megacólon tóxico, con todas sus consecuencias. En algunos pacientes, la colitis está confinada al colon derecho y se pueden presentar con abdomen agudo, dolor de rebote en la zona y sin diarrea. Según la gravedad, la DACD se clasifica en: 1. Cuadro leve a moderado, en el que no existe compromiso sistémico o es leve. 2. Cuadro grave, definido por leucocitosis mayor o igual a 15.000 /mm3 o aumento de la creatinina a un nivel de 1,5 veces su nivel premórbido 3. Cuadro grave complicado, si existe hipotensión, shock, hiperlactacidemia, íleo o megacólon. Debe sospecharse en todo paciente hospitalizado que inicie un cuadro diarreico. El diagnóstico se basa en el cuadro clínico, los exámenes de laboratorio y, a veces, en la endoscopía del colon.

La aparición de diarrea asociada a la administración de antibióticos se presenta entre el 5% - 25% de los pacientes. Existen dos entidades: Asociada a Clostridium Difficile y la diarrea simple.

Caso Clínico Tipo Paciente hospitalizado por Neumonía Adquirida en la Comunidad, ATS III al tercer día de tratamiento antibiótico presenta compromiso del estado general, fiebre y deposiciones líquidas.

Definición Es la diarrea producida por la toxina del Clostridium Difficile (DACD), que es un bacilo gram positivo anaerobio estricto, formador de esporas. La mayor parte de las veces está asociada al uso previo de antibióticos.

Etiología-Epidemiología-Fisiopatología La DACD es fundamentalmente una infección nosocomial y constituye la primera causa de diarrea que se presenta en el paciente hospitalizado. Su incidencia varía entre 0,6% y 2,1% del total de pacientes hospitalizados; con una letalidad de 1% a 5%. Es inhabitual en pacientes ambulatorios, aunque su incidencia en este escenario ha ido en aumento.

Laboratorio: En los exámenes generales se puede encontrar anemia, leucocitosis, aumento de la creatininemia, elevación de la velocidad de sedimentación y proteína C reactiva. Los leucocitos fecales pueden o no estar presentes. Los exámenes específicos para el diagnóstico se efectúan en muestras de deposiciones y existe una amplia variedad de ellos: 1. El cultivo de C. difficile es difícil y con poco valor predictivo, ya que existen portadores asintomáticos y también, cepas no toxigénica. 2. Tradicionalmente el “gold standard” ha sido el ensayo de citotoxicidad de filtrado de deposiciones en cultivos celulares, con evaluación de la neutralización de toxina, el que posee alta especificidad. Sin embargo, su costo es elevado, es un método lento y su sensibilidad no es óptima. 3. Una de las técnicas más utilizadas es la detección de toxinas mediante técnica de inmunoensayo enzimático, lo cual es más barato y rápido.

Se ha detectado la presencia de C. Difficile en más del 25% de los pacientes adultos internados en un hospital general. Solo un tercio desarrolla diarrea y los otros son portadores asintomáticos. Todos ellos forman parte del reservorio del agente infeccioso. Esta bacteria sobrevive en el ambiente hospitalario mediante esporas, las que pueden ser transmitidas fácilmente mediante el contacto del paciente infectado con otros pacientes. Se ha demostrado la presencia de esporas viables durante meses en baños, pisos, teléfonos, estetoscopios, etc. El uso de antibióticos es el principal factor de riesgo para desarrollar DACD. Los fármacos más frecuentemente asociados son amoxicilina, ampicilina, cefalosporinas, clindamicina y fluorquinolonas. Sin embargo, cualquier antibiótico es capaz

Existen ensayos que detectan sólo la toxina A y otros que son

11 Facultad de Medicina, Universidad de Chile

capaces de detectar ambas toxinas, siendo mejor esta segunda opción. Su especificidad es muy alta, pero la sensibilidad es un problema, ya que se reporta cifras entre 31 y 99%. 4. Los ensayos moleculares que detectan los genes de C. difficile que codifican las toxinas, entre ellos la reacción de polimerasa en cadena (PCR) y el Lamp (“Loop-mediated isothermal amplification”), han comenzado a imponerse como una excelente alternativa, ya que su costo no es tan elevado, son rápidos y su sensibilidad es superior al 95%; sin embargo, pueden existir falsos positivos si hay contaminación. 5. El inmunoensayo para detectar Glutamato deshidrogenasa, presente en C. difficile, podría ser útil dentro de un algoritmo diagnóstico para disminuir costos, pues si bien su sensibilidad es buena, requiere de un test confirmatorio de toxinas.

b) En el caso de una segunda recurrencia, se recomienda usar vancomicina por un período prolongado y en dosis decrecientes. Un esquema usado es 125 mg cada 6 horas por 14 días, luego 125 mg cada 12 horas por 7 días, seguido de 125 mg al día por 7 días y luego 125 mg cada 2 a 3 días por 2 a 8 semanas.

Seguimiento Se deben evaluar atentamente síntomas de complicación o progresión de la enfermedad.

Autor / Editor Hernán Rubilar

Tratamiento Debe incluir la suspensión o cambio del antibiótico asociado, dependiendo de la situación clínica del paciente. Si no es recomendable suspender la terapia antibiótica, en lo posible se debe optar por alguno menos asociado al desarrollo de DACD. Además, se emplean medidas dietéticas, de apoyo general e hidratación, en concordancia con la gravedad del cuadro clínico. En los casos más leves se ha descrito que puede no ser necesario el tratamiento antibiótico específico. Un punto importante a considerar es que la recurrencia, es decir, la reaparición de diarrea después de un tratamiento exitoso y con demostración de que la causa es el C. difficile, es un problema clínico, pues se presenta en alrededor de 20% de los casos después de un primer episodio de DACD. Además, esta probabilidad aumenta si se producen nuevas recurrencias, por ejemplo es 60% si han existido dos o más recurrencias. 1. Tratamiento de un primer episodio a) En el caso de DACD leve a moderada, el tratamiento específico es metronidazol 500 mg cada 8 horas por 10 a 14 días, por vía oral. b) En cuadros graves no complicados, se recomienda utilizar vancomicina, por vía oral, en dosis de 125 mg cada 6 horas por 10 a 14 días. c) En los casos graves complicados se utiliza vancomicina 500 mg cada 6 horas, vía oral o por sonda nasogástrica, más metronidazol 500 mg cada 8 horas por vía intravenosa. Si existe íleo se debe considerar el uso de vancomicina intracolónica, 500 mg cada 6 horas, diluida en 100 cc de solución salina. Además, se requiere la evaluación conjunta con el equipo de cirugía, pues se puede requerir una colectomía de urgencia si no hay respuesta favorable o aparecen complicaciones abdominales. 2. Tratamiento de la recurrencia a) Ante una primera recurrencia, el tratamiento específico es el mismo indicado para tratar un primer episodio.

12 Facultad de Medicina, Universidad de Chile

Año 2016

Diarrea en Inmunosuprimidos Nivel de manejo del médico general: Diagnóstico Sospecha Tratamiento Inicial Seguimiento Derivar



Aspectos Esenciales • • •

La diarrea crónica es la principal manifestación del compromiso del tracto gastrointestinal en el inmunosuprimido. Debe ser oportuno, con soporte nutricional y derivación precoz a especialista. Recordar que puede tener otros agravantes al mismo cuadro (uso de TARV, intolerancia a la lactosa, enf. inflamatoria intestinal).

Diagnóstico Importante es una buena historia clínica (inicio, descripción de las deposiciones, síntomas asociados, asociación con alimentación, etc.). Es necesario además evaluar el estadío de la inmunodeficiencia (a través de carga viral, recuento de CD4, etc) y complementar con laboratorio general y exámenes coproparasitológicos y microbiológicos. Los estudios radiológicos o endoscópicos aislados, tienen baja sensibilidad diagnóstica, sin embargo, si se suman exámenes (estudio coproparasitológico seriado, la endoscopía digestiva baja y las biopsias, alcanzan una sensibilidad diagnóstica mayor del 90%. Las biopsias múltiples y los estudios histológicos mejoran la detección de agentes oportunistas, como el citomegalovirus, Mycobacterium avium, Cyclospora cayetanensis, etc. La inmunohistoquímica o las coloraciones especiales pueden detectar las esporas del Microsporidium en heces (15,16). La diarrea crónica en estos pacientes también puede ser inducida por medicamentos (antibióticos: toxina B del Clostridium difficile, antiretrovirales), y ser acentuadas por intolerancia a la lactosa, colon irritable, enfermedad inflamatoria intestinal, etc.

Caso Clínico Tipo Paciente de 35 años, con historia de infección por VIH hace 4 años, sin uso de TAR ni controles médicos, acude al servicio de urgencia por 20 días de deposiciones líquidas abundantes que no cedieron con dieta, loperamida ni carbón. Al examen físico, con deshidratación moderada, enflaquecido y candidiasis oral.

Definición Es el cambio en el hábito defecatorio, ya sea por aumento de frecuencia (mayor o igual a 2 deposiciones al día), menor consistencia o mayor peso (> de 200 g/día), por 30 días o más, en una persona con disminución de la respuesta inmunitaria normal del organismo.

Etiología-Epidemiología-Fisiopatología

Tratamiento

El tracto gastrointestinal se relaciona continuamente con numerosos agentes del ambiente externo, por lo que necesita de diferentes formas de protección a través de barreras (epitelio, respuesta inflamatoria local, barrera inmunológica con linfocitos e inmunoglobulinas, placas de Peyer, folículos linfoides, apéndice cecal, entre otros.) Sin embargo, en pacientes que han sufrido inmunosupresión, ya sea por quimioterapia, trasplantados, infección por VIH, colagenopatías, terapia corticoidal prolongada,etc, estas barreras están disminuidas, ocasionando cuadros gastrointestinales severos, persistentes o refractarios al tratamiento. Existen patógenos oportunistas y no oportuistas asociados a diarrea crónica en el inmunosuprimido: •

- Salmonella. - Shigella. - Campylobacter sp. - Mycobacterium tuberculosis. - Giardia lamblia. Patógenos oportunistas. - Mycobacterium avium intracelular. - Citomegalovirus. - Protozoarios coccidiales: cryptosporidium e isospora belli. - Microsporidium. - Cyclospora cayetanensis. - Protozoarios “no patógenos”: Blastocystis hominis, etc. - Cándida albicans. - Criptococcus neoformans.

Debe ser oportuno, con soporte nutricional y derivación precoz a especialista. La indicación de dieta libre de residuos (régimen blando, liviano, sin residuos) no está respaldada por evidencia científica. El uso de pre y probióticos en diarreas de origen infeccioso mejora tiempo y sintomatología. Uso de antimicrobianos según contexto clínico de la diarrea.

Seguimiento Controles rutinarios con especialista.

Enteropatógenos clásicos no oportunistas

13 Facultad de Medicina, Universidad de Chile

Autor / Editor Nevelline Salgado

Año 2016

entre factores dietéticos, genéticos y de motilidad intestinal, que llevan a un aumento de la presión intraluminal y a cambios en la resistencia en algunas zonas de la pared colónica. La baja ingesta de fibras se plantea como un agente etiológico posible para el desarrollo de ED. La teoría que plantea a la fibra como un agente protector contra los divertículos y posteriormente contra la diverticulitis sostiene que: la fibra insoluble provoca la formación de heces más voluminosas, disminuyendo así la efectividad en la segmentación colónica. El resultado general es que la presión intracolónica se mantiene próxima al rango normal durante la peristalsis colónica. La segmentación colónica ocurre como resultado de un aumento de la presión intracolónica en ciertas áreas del colon. Dicha segmentación representa fuertes contracciones musculares de la pared colónica que sirven para hacer avanzar el contenido luminal o detener el pasaje del material.

Enfermedad Diverticular No Complicada Nivel de manejo del médico general: Diagnóstico Sospecha Tratamiento Inicial Seguimiento Completo

Aspectos Esenciales • • • •

Al aumentar la edad, aumenta la prevalencia. La mayoría no presenta síntomas. El diagnóstico puede ser incidental. No presenta síntomas específicos.

En la diverticulosis, la segmentación está exagerada, provocando oclusión de ambos extremos de las cámaras, produciendo altas presiones dentro de las mismas. Lo que favorece la herniación de la misma. El espectro clínico de esta enfermedad se divide en 2 grandes grupos:

Caso Clínico Tipo Paciente femenina de 70 años quien refiere cuadro de dolor abdominal de 1 semana de evolución en fosa ilíaca izquierda asociado a diarrea y meteorismo.

No complicada (75-90%): • Asintomática (mayoría). • Síntomas inespecíficos. Complicada (10-25%): • Diverticulitis. • Sangramiento diverticular.

Definición La enfermedad diverticular (ED) no complicada del colon corresponde a la presencia de divertículos dentro del colon (diverticulosis) que no producen hemorragia severa, obstrucción, perforación o fístula. Los Divertículos son protrusiones saculares de la mucosa a través de la pared muscular del colon que contienen sólo mucosa y submucosa cubiertas de serosa. Estos ocurren en las áreas débiles de la pared intestinal donde pueden penetrar los vasos sanguíneos.

Diagnóstico En la ED asintomática el diagnóstico se realiza en forma incidental al realizar un enema baritado o colonoscopía por otros motivos. La ED sintomática simple presenta síntomas tales como constipación y/o diarrea, meteorismo y dolor en la fosa ilíaca izquierda. Sin embargo son síntomas inespecíficos y son a veces erróneamente atribuidos a una ED, en circunstacias que a menudo corresponden a un trastorno funcional coexistente del colon. De allí que no es posible reconocer síntomas propios y característicos de la ED, con excepción de aquellos que conforman el cuadro de una diverticulitis aguda.

Etiología-Epidemiología-Fisiopatología La prevalencia exacta de la diverticulosis de colon es desconocida ya que muchos pacientes se mantienen asintomáticos. Sin embargo, aumenta con la edad, estimándose en menos de 5% en sujetos menores de 40 años, con un incremento de 30% a los 60 años y alcanzando hasta 65% de la población a los 80 años. La distribución entre hombres y mujeres es similar, con una pequeña preponderancia del sexo femenino. Esta distribución genérica varía con la edad, siendo más frecuente en hombres bajo 50 años, con una leve preponderancia femenina entre los 50 a 70 años y una marcada preponderancia femenina sobre los 70 años de edad. La enfermedad divertcular en países occidentales es principalmente de colon izquierdo, presentándose en el lado derecho en el 1,5% de los casos. En África y Asia en cambio es predominantemente del lado derecho. La ED de colon es el resultado de una compleja interacción

Tratamiento En la ED asintomática o diveticulosis el paciente tiene que estar informado de su condición. El aumento de fibra natural en la dieta podría disminuir las posibilidades de complicación, esto vale también para la ED sintomática simple. Por otro lado, la diverticulitis debe recibir tratamiento médico encaminado a tratar el problema local (reposo digestivo, antibióticos), sin embargo, cuando a pesar del tratamiento médico no es posible controlar el problema local debe surgir el tratamiento quirúrgico.

14 Facultad de Medicina, Universidad de Chile

Seguimiento Ante la presencia de signos y síntomas severos se debe hospitalizar y al aparecer complicaciones como hemorragia, perforación, obstrucción, fístula o diverticulitis recidivantes se debe considerar la cirugía.

Autor / Editor Nevelline Salgado

Año 2016

15 Facultad de Medicina, Universidad de Chile

de suelo), las principales fuentes de contagio. No olvidar la transmisión persona-persona en guarderías, casas de reposo y orfanatos (contacto con pañales, mala higiene).

Hepatitis Aguda A No Complicada Nivel de manejo del médico general: Diagnóstico Específico Tratamiento Completo Seguimiento Completo

Diagnóstico Es clínico + laboratorio. Clínica: lo más frecuente es que sea asintomática y anictérica (más frecuente en <6 años). Si se presenta con síntomas, se encuentran: compromiso del estado general, fiebre, náuseas, vómitos, dolor abdominal, ictericia, coluria, prurito, esteatorrea. La severidad del cuadro de hepatitis A aumenta con la edad, siendo los grupos más susceptibles los escolares, adultos jóvenes y personas de nivel socioeconómico alto debido a su escasa exposición al virus en la niñez. La hepatitis sintomática ocurre en menos del 10% de los menores de 6 años, en 40-50% entre los 6 a 14 años y 70-80% de los mayores de 14 años. Período de incubación: 28 a 30 días en promedio (rango de 15 a 50 días). Período de infectividad máxima: durante la segunda mitad del período de incubación y continua algunos días después del inicio de la ictericia (1° semana). Pueden existir casos de lactantes y niños que excreten el virus hasta por seis meses, pero no en forma crónica. Examen físico: ictericia, sensibilidad abdominal en hipocondrio derecho, hepatomegalia moderada, esplenomegalia (1/5 de casos).

Aspectos Esenciales • • • •

Hepatitis aguda más frecuente en Chile. Vía de transmisión fecal-oral. No existe un tratamiento específico, sí son importantes las medidas de soporte. Luego de la infección por VHA, el individuo queda con inmunidad (anti-VHA IgG) para toda la vida.

Caso Clínico Tipo Paciente de 18 años que 2 semanas luego de regresar mochileando de Arica, comienza con cuadro de ictericia leve, fiebre, dolor abdominal y compromiso del estado general. Sin otras alteraciones al examen físico. Al laboratorio, se observa elevación de transaminasas > 50 veces, hiperbilirrubinemia de predominio indirecto y se detecta anti-VHA IgM (+).

Definición

Laboratorio: pruebas hepáticas (elevación de enzimas hepáticas, especialmente GPT> 10-100 veces, hiperbilirrubinemia de predominio indirecto), detección de anticuerpo anti-VHA IgM, que aparece precozmente coincidiendo con los primeros síntomas, manteniéndose elevado por 3 meses. El anticuerpo anti-VHA IgG, se eleva gradualmente con peaks el 1°-2° mes, persistiendo durante toda la vida, confiriendo inmunidad.

Enfermedad infecto contagiosa aguda de distribución mundial, que se presenta en forma esporádica o epidémica con ciclo estacional, con tres patrones epidemiológicos: endemicidad elevada, intermedia (Chile) y baja o muy baja, con afinidad hepatotropa.

Etiología-Epidemiología-Fisiopatología Epidemiología: El VHA es el agente más frecuente de la hepatitis aguda en Chile. La mayor parte de las infecciones ocurren en niños mayores de 5 años y adolescentes. En Chile los brotes epidémicos son cada 4 o 5 años, preferentemente institucionales.

Tratamiento No existe un tratamiento específico, sino que las medidas son de soporte. Entre ellas: • Durante la fase ictérica: reposo en cama. Evitar los ejercicios físicos por uno a dos meses. • La alimentación debe tender a ser normal, a tolerancia. En la fase de convalecencia: régimen rico en proteínas. • Evitar el alcohol y medicamentos que puedan ser tóxicos a nivel hepático. • Medidas de control de transmisión fecal-oral por una semana después de la aparición de la ictericia. Estas incluyen: lavarse las manos después de estar en contacto con un paciente o con objetos que puedan estar contaminados y antes de atender a otro paciente, desechar adecuadamente los artículos contaminados con material infectante y, dentro de lo posible, uso de guantes para

Etiología: Virus RNA de la familia Picornavirus, siendo muy resistente a agentes físicos y químicos, por lo que se transmite fácilmente por el agua y los alimentos. Fisiopatología: los virus son absorbidos por el intestino, llegando al hígado donde encuentran un receptor en la célula hepática. Es allí donde sintetizan nuevas proteínas virales que se liberan al tracto digestivo y son eliminadas por las deposiciones. La vía de transmisión es fecal-oral, siendo la ingestión de mariscos crudos, la contaminación de aguas y regadío de alimentos por aguas servidas (verduras a ras

16 Facultad de Medicina, Universidad de Chile

• • • •

• • • •

manipular material contaminado. Buen lavado de manos. Cocer alimentos en riesgo (moluscos cudos o mal cocidos) a > 85°C por al menos 4 minutos, pues esta temperatura es capaz de destruir al VHA Desinfectar las superficies contaminadas con hipoclorito de sodio a una dilución de 1% ( una cucharada de cloro por litro de agua). A los contactos (toda persona <20años que duerme bajo el mismo techo o comparte alimentos con un caso confirmado y sin antecedentes de una infección por hepatits A) de casos aislados se les debe aplicar gamma globulina estándar intramuscular, en dosis de 0,02 ml/Kg de peso, tan pronto como sea posible después de la exposición, pero en un plazo máximo de dos semanas desde el inicio de la exposición. No necesitan inmunoglobulina las personas que han recibido una dosis de vacuna contra la Hepatitis A, al menos 30 días antes de la exposición. Disponer de agua potable para consumo humano Eliminar las aguas servidas, a través de un sistema de alcantarillado público o por sistemas particulares, tales como las fosas sépticas o letrinas sanitarias. Lavado minucioso de las manos después de cada cambio de pañales, antes de comer o dar de comer a los niños y ancianos. Vacunación: Vacuna extra MINSAL (1 dosis a los 18 meses: parte del programa PNI para regiones de Arica y Parinacota, Tarapacá y algunas regiones del Bio-Bío), que se aplica en 2 dosis separadas por 6 meses.

Seguimiento Frecuencia de envío de la notificación: diaria a la SEREMI de Salud y semanal al MINSAL.

Autor / Editor Nevelline Salgado

Año 2016

17 Facultad de Medicina, Universidad de Chile

sión es por vía parenteral a través de transfusiones, hemodiálisis, drogadictos endovenosos, contacto sexual, transmisión vertical. La infección por VHC se cronifica en el 80 % de los casos y un 20-35% desarrollarán cirrosis.

Hepatitis Aguda B, C, por Otros Virus, por Drogas y Toxinas

Hepatitis aguda por VHE: Produce una infección aguda, que no va a cronicidad. El mecanismo de transmisión es fecal-oral. La infección por VHE ocurre sobre todo en jóvenes-adultos. El riesgo de hepatitis fulminante es de 1-2% de los casos en general, pero que puede subir hasta el 20 % en el caso de infección en mujeres embarazadas.

Nivel de manejo del médico general: Diagnóstico Específico Tratamiento Inicial Seguimiento Derivar

Aspectos Esenciales • • • •

Hepatitis aguda por fármacos: Los mecanismos de toxicidad hepática por fármacos pueden ser dosis dependiente y dosis independiente (reacción idiosincrática). Por ejemplo en el caso del Paracetamol el daño hepático es dosis dependiente, y aparece en general cuando se consumen más de 10 gramos. Importante consignar que el alcohol aumenta la toxicidad, algunos pacientes pueden desarrollar una falla hepática fulminante con dosis bajas de paracetamol cuando se asocia a alcohol. Otros cuadros de hepatitis aguda por fármacos son producidos por la isoniazida, metildopa, ketoconazol, etc.

Corresponde al proceso inflamatorio del hígado producido por diversas etiologías. El curso clínico varía desde leve hasta falla hepática fulminante Es frecuente encontrar cuadros de hepatitis aguda que son asintomáticos La toxicidad farmacológica puede aparecer a dosis terapéuticas en pacientes con daño hepático de base como por ejemplo alcohólicos.

Caso Clínico Tipo

Diagnóstico

Paciente de sexo masculino de 25 años, con antecedentes de consumo de drogas por vía parenteral, que consulta por cuadro de marcado compromiso del estado general, náuseas, vómitos. Al examen físico destaca ictericia generalizada de piel y mucosas, sensibilidad a la palpación y percusión en hipocondrio derecho. Perfil hepático con elevación de trasaminasas 20 veces superior a los valores normales e hiperbilirrubinemia.

Es frecuente encontrar cuadros de hepatitis aguda que son asintomáticos. En aquellos pacientes sintomáticos, las manifestaciones clínicas iniciales son comunes e independientes de la etiología de la hepatitis. Es importante realizar una detallada anamnesis, destacando aspectos importantes como factores de riesgo y establecer bien la cronología entre la aparición de la hepatopatía y la administración de algún fármaco por ejemplo. Se debe hacer una historia de todos los fármacos que ha tomado en los tres meses previos. El diagnóstico final requiere pruebas de laboratorio y se requiere marcadores específicos.

Definición

Cuadro clínico: • Pródromo: Compromiso del estado general, fiebre variable, anorexia, náuseas, dolor abdominal epigástrico y en hipocondrio derecho, diarrea, artralgias, erupciones cutáneas. • Fase ictérica: Ictericia, coluria, hipocolia, hepatomegalia sensible, esplenomegalia, prurito.

Corresponde al proceso inflamatorio del hígado por diversas etiologías.

Etiología-Epidemiología-Fisiopatología Hepatitis aguda por VHB: VHB es un virus DNA. Los mecanismos de transmisión son: vía parenteral a través de transfusiones de sangre, derivados sanguíneos o contacto con material contaminado, contacto sexual y transmisión perinatal. El VHD puede aparecer simultáneamente con el VHB (coinfección B y D) o generar una infección aguda en un paciente con infección crónica por VHB (este cuadro recibe el nombre de sobreinfección D). Como dato epidemiológico importante aproximadamente el 1% de los pacientes con infección aguda por VHB desarrollará una falla hepática fulminante, el 99% va a recuperación normal.

Alteraciones de laboratorio: • Elevación de aminotransferasas alcanzando 10 a 100 veces los valores normales. Esta prueba es diagnóstica y también se hace como seguimiento • Hiperbilirrubinemia: Es variable • Tiempo de protrombina: Prueba importante ya que su vida media de 24 horas constituye un buen indicador de la función hepática. Marcadores específicos: • VHB: En la infección por el VHB , el primer marcador que aparece es el AgHBs, que lo hace antes de que se eleven

Hepatitis aguda por VHC: VHC es un virus RNA. La transmi-

18 Facultad de Medicina, Universidad de Chile



las transaminasas y permanece elevado durante toda la fase de sintomatología clínica. Otro marcador imprescindible para el diagnóstico de la infección aguda por VHB es IgM anti-HBc. Una vez que se negativiza el AgHBs, aparecen los anticuerpos antiHBs, que permanecen de forma indefinida y que son los anticuerpos protectores que evitan que el paciente se pueda reinfectar de nuevo. El AgHBe se detecta desde el comienzo de la enfermedad, apareciendo poco después del AgHBs, marcando el estado de replicación viral. VHC: Se solicita Anticuerpo anti VHC y ARN de VHC.

Tratamiento Por especialista. En general la infección por aguda por VHB se trata en pacientes con cuadros graves o que evolucionen hacia formas fulminantes. En el caso de la VHC en algunos casos de infección aguda sintomática, el interferón ha sido eficaz para eliminar la infección y evitar su evolución a la cronicidad. Para el caso de las hepatitis agudas por fármacos el tratamiento de elección es retirar el fármaco, vigilar y seguir cuidadosamente la evolución clínica del paciente.

Seguimiento Por especialista.

Autor / Editor Nevelline Salgado

Año 2016

19 Facultad de Medicina, Universidad de Chile

la sensibilidad al alcohol son la dieta rica en grasas y proteínas. También existen mutaciones ligadas al desarrollo de la enfermedad como en el caso de PC hereditaria. Considerar etiología autoinmune.

Pancreatitis Crónica Nivel de manejo del médico general: Diagnóstico Sospecha Tratamiento Inicial Seguimiento Derivar

Fisiopatología: Cada etiología tiene su propia fisiopatología, pero lo común es una destrucción progresiva de la glándula con afectación de la función endocrina y exocrina.

Aspectos Esenciales

Diagnóstico

• • • •

El dolor abdominal es el síntoma cardinal, localizado en hemiabdomen superior con irradiación a dorso. Casi permanente, aumenta su intensidad posterior a la ingesta de alcohol y después de comer. En la evolución de la enfermedad, durante la progresión de la destrucción del páncreas, el dolor tiende a la disminución hasta desaparecer. Puede manifestarse en brotes episódicos.

Factor más importante: consumo crónico de alcohol. La reserva funcional del páncreas es alta. La abstinencia total de alcohol es lo principal. Tratamiento es sintomático.

Caso Clínico Tipo

En las etapas iniciales, el cuadro clínico es a menudo inespecífico: dolor abdominal y meteorismo. Las formas avanzadas de PC se caracterizan por insuficiencia exocrina (esteatorrea) y endocrina (diabetes mellitus) y baja de peso. Durante los años de evolución, la PC puede complicarse con múltiples episodios de Pancreatitis aguda, y complicaciones como pseudoquistes u otras alteraciones locales. La función exocrina y en especial la endocrina, permanecen en rango normal durante largo tiempo, dada la gran reserva funcional del páncreas. Diagnóstico inicial es difícil, ya que, las lesiones son frecuentemente focales, indetectables o de valor diagnóstico incierto y los exámenes complementarios tienen baja sensibilidad para detectar daño leve. Tríada clásica es tardía: Calcificaciones pancreáticas, esteatorrea y diabetes mellitus.

Hombre de 48 años consulta en el SU por náuseas, vómitos y dolor abdominal intermitente por dos días. El dolor se irradia desde el epigastrio y el hipocondrio izquierdo hacia el dorso. Nada alivia el dolor y además aumenta al comer, además refiere deposiciones de muy mal olor y disgregadas. Sin fiebre. Antecedentes: DM2IR, alcoholismo crónico, muchos años de dolor abdominal crónico y diarrea crónica.

Definición Es un síndrome, producto de un proceso inflamatorio crónico del páncreas, que progresa con destrucción del parénquima, reemplazado por tejido fibroso en forma focal o difusa, lo que conduce a un deterioro de la función endocrina y exocrina. Puede ser asintomática por largos períodos, con enzimas pancreáticas en rangos normales. Puede ser dividida en base a la morfología subyacente en las siguientes: tipo gran conducto y tipo pequeño conducto con o sin calcificación. Se pueden producir complicaciones, incluyendo pseudoquistes, obstrucción biliar o duodenal, ascitis pancreática, trombosis de la vena esplénica y pseudoaneurismas.

Tratamiento Los objetivos del tratamiento de la Pancreatitis crónica incluyen el tratamiento del dolor, la corrección de la insuficiencia pancreática, y manejo de complicaciones. El manejo inicial incluye abstinencia del alcohol y el tabaco, y comer comidas pequeñas con bajo contenido en grasa. La abstinencia total de alcohol no previene progresión pero disminuye complicaciones y el pronóstico. • Dolor: Se recomienda uso de suplementos de enzimas pancreáticas en pacientes con dolor persistente. Aunque discutido su uso, pueden aliviar el dolor en algunos pacientes y son generalmente seguros. Analgesia con antiinflamatorios no esteroideos y/o espasmolíticos convencionales pueden considerarse si la terapia de enzimas pancreáticas no logra controlar el dolor. La terapia adyuvante con pregabalina se puede considerar en los pacientes cuyo dolor no se controla adecuadamente con agentes antiinflamatorios no esteroides y/u opiáceos. La cirugía se ha considerado para los pacientes que no responden al tratamiento médico. Se han descrito tres abordajes quirúrgicos: las operaciones de descompresión/

Etiología-Epidemiología-Fisiopatología Epidemiología: En Chile se ha catalogado como infrecuente, aunque los casos diagnosticados irían en aumento, debido a la mayor accesibilidad y mejores métodos diagnósticos. En Europa y EE.UU. la prevalencia es de 10-30 casos/100.000 habitantes. Etiología: El factor etiológico más importante es el consumo crónico de alcohol. Tiene una frecuencia 2 a 4 veces mayor en hombres que en mujeres y la edad de inicio es entre los 40 a 50 años. Su inicio se produce después de más de 10 años de consumo crónico de alcohol. Se cree que el etanol alteraría la activación del zimógeno. Factores que aumentan

20 Facultad de Medicina, Universidad de Chile







drenaje, resecciones pancreáticas, y procedimientos de denervación. Insuficiencia exocrina: enzimas pancreáticas con alto contenido de lipasa, tomadas juntas con todas las ingestas orales de alimentos. Se asocian frecuentemente con antagonistas de receptores H2 o con inhibidores de la bomba de protones para reducir la inactivación por el ácido gástrico. Diabetes mellitus: una vez que se manifiesta, debe administrarse Insulina en bajas dosis por la tendencia a hipoglicemia de estos pacientes (por disminución en la producción de glucagón). Pancreatitis crónica autoinmune: se inicia corticoterapia (Prednisona 30-40mg/día y se reduce en forma paulatina durante 6-10 semanas).

Seguimiento Por especialista.

Autor / Editor Nevelline Salgado

Año 2016

21 Facultad de Medicina, Universidad de Chile

Factores de Riesgo

Enfermedad por Reflujo Gastroesofágico (ERGE)

Tabaco: Relación estrecha entre aparición de síntomas e inicio del habito, asi como también disminución de prevalencia luego del cese de este. Alcohol: Existen resultados contradictorios respecto en pacientes con alto consumo de alto, sin embargo se cree que una alta cantidad de alcohol ingerida tenga relación con la ERGE.

Introducción El paso del contenido gástrico hacia el esófago (reflujo gastroesofagico) es una condición fisiológica normal. La mayoría de los episodios son breves y no causan síntomas, injuria esofágica u otras complicaciones.

Definición

Fármacos: Muchos medicamentos se han asociado a síntomas de ERGE dentro de los que destacan los anticolinergicos, nitratos y corticoides. Por otro lado otros medicamentos con los anticonceptivos orales y la terapia sustitución hormonal disminuirían estos síntomas. Café: Se le ha encontrado asociación pero no categórica.

El reflujo gastroesofagico llega a ser patológico cuando causa daño macroscópico al esófago o síntomas que reducen la calidad de vida.

Clasificación La principal clasificación de la ERGE está basada en la apariencia de la mucosa esofágica en la Endoscopia digestiva alta en los siguientes tipos: • Esofagitis erosiva (EE): Pacientes con visualización endoscópica de lesiones en mucosa esofágica distal con o sin síntomas de reflujo. • ERGE no erosiva: Pacientes con ERGE, sin lesiones en mucosa esofágica a la endoscopia.

Fisiología y Fisiopatología La existencia de mecanismos antirreflujo naturales constituyen una barrera altamente eficiente entre esófago y estómago. El mecanismo valvular está conformado por el esfínter gastroesofágico (EGE), diafragma, ángulo de His, válvula de Gubaroff y la membrana frenoesofágica; y por el clearence esofágico facilitado por la peristalsis y la producción de saliva.

Síntomas

Peristalsis: Entre 40-50% de los pacientes con ERGE tienen peristalsis normal y sólo en 20% la dismotilidad es severa. Esto último condiciona que el material refluido esté por tiempo prolongado en contacto con la mucosa y sea capaz de alcanzar más a menudo el esófago superior y la faringe.

Los síntomas más comunes de enfermedad por reflujo gastroesofágico son pirosis, regurgitación y disfagia. Una variedad de potenciales manifestaciones extraesofagicas han sido también descritas incluyendo broncoespasmo, laringitis y tos crónica. Otros síntomas de RGE incluyen dolor al pecho, hipersalivación, sensación de bulto en la garganta, odinofagia y nauseas.

Esfinter gastroesofagico: La presencia de un EGE efectivo, entendido como aquel que tiene una presión de reposo adecuada y un largo total e intraabdominal normal, no es suficiente para evitar el RGE. Fisiológicamente se explica por la existencia de períodos de relajaciones transitorias del EGE en sujetos normales, en el que la distensión gástrica probablemente juega un rol. Cuando estas relajaciones son más frecuentes y prolongadas, pueden contribuir a la enfermedad por reflujo, fenómeno que explicaría el 40% de los pacientes con ERGE y EGE con presión normal.

Diagnóstico El diagnóstico de ERGE puede ser realizado sólo con la historia clínica. En pacientes que se presenten con alguna de las manifestaciones clínicas descritas, puede realizarse un diagnostico presuntivo. La respuesta a terapia antisecretora no es un criterio diagnostico, sin embargo se considera altamente orientador. No es ni necesario ni práctico iniciar una evaluación diagnóstica invasiva en cada paciente con acidez.

Diafragma: La crura diafragmática constituye una importante protección contra el reflujo inducido por un súbito aumento de la presión intraabdominal, mecanismo que está alterado ante la presencia y tamaño de una hernia hiatal. Actualmente, está bien definido que el RGE y la hernia hiatal pueden existir en forma independiente, sin embargo, se reconoce que esta última por sí sola, puede disrupcionar los mecanismos antirreflujo naturales y, por lo tanto, es considerada un factor independiente de ERGE.

Estudio Endoscopia digestiva alta (EDA): En general se reserva para evaluar los pacientes que tienen

22 Facultad de Medicina, Universidad de Chile

síntomas de alarma, sospecha de complicaciones de ERGE y para seguimiento del esófago de Barrett. La clasificación endoscópica de la esofagitis mas utilizada es la de Los Angeles.

antes tratados se recomienda iniciar con dieta y cambios en el estilo de vida, y antagonistas de receptor 2 de histamina (H2RA) en bajas dosis según sea necesario; se pueden usar antiacidos concomitantes según sea necesario si los síntomas ocurren menos de una vez a la semana. De persistir los sintomas se recomienda aumentar la dosis de los H2RA a la dosis estandar por un minimo de dos semanas. De persistir los sintomas se recomienda el uso de IBP. Dieta y cambios en estilo de vida. Se recomienda: • Baja de peso en pacientes con ERGE y sobrepeso o que recientemente hayan ganado peso • Elevacion de la cabecera de la cama en pacientes con sintomas laringeos (aumentar 6 -8 pulgadas). Abstenerse de asumir una posición supina después de las comidas y evitación de las comidas de dos a tres horas antes de acostarse. • La modificación dietética no debe recomendarse rutinariamente en todos los pacientes con ERGE. Sin embargo, se sugiere la eliminación selectiva de los factores desencadenantes de la dieta (alimentos grasos, cafeína, chocolate, alimentos picantes, alimentos con alto contenido de grasa, bebidas carbonatadas y menta) en pacientes que observan correlación con síntomas de ERGE y una mejoría en los síntomas con eliminación.

Clasificación de Los Angeles: • Grado A: Una o más lesiones longitudinales que no confluyen y no superan los 5 mm • Grado B: Por lo menos una lesión longitudinal mayor de 5 mm, no confluye • Grado C: Lesiones longitudinales que no confluyen, pero comprometen menos del 75% de la circunferencia. • Grado D: Lesiones que confluyen y comprometen por lo menos el 75% de la circunferencia. Indicaciones de endoscopia digestiva alta: Las indicaciones de EDA en ERGE son controversiales, y no es requerida frente a presencia de síntomas típicos de ERGE. Lo que se recomienda es realizar EDA si el diagnostico de ERGE es poco claro y en los siguientes casos: • Pacientes con acidez y síntomas de alarma, con motivo de descartar complicaciones de ERGE y otros diagnósticos. Síntomas de alarma incluyen disfagia, odinofagia, hemorragia digestiva, anemia, baja de peso y vómitos recurrentes. • Pacientes con esofagitis erosiva severa (Grados C y D de Los Angeles) en endoscopia inicial que deben ser sometidos a seguimiento endoscópico luego de 2 meses de tratamiento con inhibidores de bomba de protones (IBP) para evaluar curación y descartar esófago de Barrett. • Tamizaje de esófago de Barrett en pacientes con multiples factores de riesgo para adenocarcinoma de esófago, dentro de los cuales se incluyen ERGE crónico, hernia hiatal, edad > 50 años, sexo masculino, raza blanca, IMC elevado, distribución grasa corporal abdominal. • Pacientes con síntomas típicos de ERGE que persiste después prueba terapéutica de 4-8 semanas de tratamiento IBP dos veces al día.

Antiacidos: No previenen la ERGE, su función se limita al uso intermitente para el alivio de los síntomas leves de la ERGE que ocurren menos de una vez a la semana. Su mecanismo de accion es neutralizacion del pH acido gastrico, disminuyendo la exposicion de la mucosa esofagica a este. Antagonistas de receptor 2 de histamina (H2RA): Su mecanismo de accion es disminuir la secrecion de acido gastrico mediante la la inhibicion del receptor de histamina en la celula parietal. Sin embargo el desarrollo de taquifilaxis dentro de 2 a 6 semanas limita su uso a mantencion. Ademas los H2RA tienen eficacia limitada en pacientes con EE y son inefectivos en Esofagitis severa.

Monitorizacion del pH esofágico: Gold estándar para definir ERGE. Los episodios de reflujo ácido son definidos por una caída del pH bajo 4. El límite máximo normal del tiempo total con pH bajo 4 es de 4-5,5%. Las indicaciones clínicas del test incluyen: • Antes de la fundoplicatura en presencia de reflujo patológico en pacientes con endoscopia normal. • Después de cirugía anti-reflujo, si persiste la pirosis. • Pacientes con síntomas de reflujo, endoscopia normal que no responden a tratamiento con IBP. • Pacientes en los que se sospeche manifestaciones extra esofágicas de la ERGE.

Síntomas severos o frecuentes o Esofagitis Erosiva: En estos pacientes se recomienda iniciar tratamiento con IBP en dosis estardar al dia, por 8 semanas asociado a dieta y cambios en estilo de vida. Posteriormente, disminuir la supresión de ácido con IBP en dosis bajas y luego con H2RA si los pacientes tienen síntomas leves o intermitentes. Descontinuamos la supresión de ácido en todos los pacientes asintomáticos, con la excepción de los pacientes con esofagitis erosiva severa o esófago de Barrett, en quienes sugerimos el mantenimiento de la terapia con IBP. Inhibidores de Bomba de Protones (IBP): Son los más potentes inhibidores de la secreción de ácido gástrico por unión de la bomba hidrógeno-potasio (H-K) ATPasa. Deben utilizarse en pacientes que fallan al tratamiento con H2RA dos veces al día y en pacientes con esofagitis erosiva y / o síntomas frecuentes (dos o más episodios por semana) o severos de ERGE que deterioran la calidad de vida. Los IBP son más efectivos cuando se toman 30 minutos antes de la primera comida del día porque la cantidad de H-K-ATPasa presente

Tratamiento Médico En general los pacientes con ERGE deben ser manejados de acuerdo a tratamiento escalonado, según el control de los síntomas, minimizando de este modo el uso de IBP, asi como sus costos y efectos adversos asociados. Síntomas leves e intermitentes: En general pacientes nunca

23 Facultad de Medicina, Universidad de Chile

en la célula parietal es mayor después de un ayuno prolongado. En comparación con los H2RA, los IBP proporcionan un alivio más rápido de los síntomas y son más eficaces para aliviar los síntomas de la ERGE. Las limitaciones del uso incluyen un costo más alto en comparación con los H2RA y los efectos secundarios potenciales.

incluyen el aclaramiento de garganta, tos persistente, sensación de globo (sensación de un bulto o cuerpo extraño en la garganta), laringoespasmo o sensación de asfixia. Estenosis Laringea y/o traqueal: El RLF puede resultar en estenosis laríngea y traqueal. La presentación de síntomas en pacientes con obstrucción de la vía aérea central es inespecífica y puede ser subaguda o aguda. Las manifestaciones clínicas dependen típicamente del grado de obstrucción luminal, así como de la localización y duración de la obstrucción. Los síntomas incluyen disnea, tos, hemoptisis y sibilancias.

Erradicacion Helicobacter pylori: La Erradicación de H. pylori se ha asociado con una mejora de los síntomas en pacientes con gastritis antral predominante.

Tratamiento Quirurgico La cirugía antirreflujo es claramente indicada en: • Pacientes con esofagitis erosiva que no curan con IBP a dosis plena. • Pacientes con volúmenes importantes de regurgitación, particularmente si ocurren • durante la noche o si tienen evidencia de aspiración. • Pacientes que requieran tratamiento antirreflujo por largo tiempo y que han tenido • eventos adversos graves relacionados a la terapia con IBP, tal como infecciónrefractaria por Clostridium difficile. • También debe ser considerada en pacientes que por razones personales deseen Evitar el uso de fármacos por largo plazo.

Complicaciones Esofago de Barrett: Es una condición en la cual el epitelio columnar metaplásico reemplaza al epitelio escamoso estratificado que normalmente reviste el esófago distal. El epitelio metaplásico se adquiere como consecuencia de la enfermedad de reflujogastroesofágico crónico (ERGE) y predispone al desarrollo de cáncer de esófago. La metaplasia columnar intestinal especializada típica del esófago de Barrett no causa síntomas. Estenosis esofágica: Las estenosis pépticas son el resultado del proceso de cicatrización de la esofagitis ulcerosa. El colágeno se deposita durante esta fase y, con el tiempo, las fibras de colágeno se contraen, estrechando el lumen esofágico. Estas estenosis suelen ser de longitud corta y contiguas a la unión gastroesofágica. Asma: ERGE es común en pacientes con asma y ha sido identificado como un posible desencadenante de asma. Se han propuesto tres mecanismos potenciales por los cuales el ácido esofágico puede producir broncoconstricción y por lo tanto exacerbar la obstrucción del flujo aéreo en asmáticos: aumento del tono vagal, aumento de la reactividad bronquial y microaspiración del contenido gástrico en la vía aérea superior. Laringitis crónica: Los pacientes con laringitis presentan un cambio en la calidad de la voz o ronquera debido a reflujolaringofaringeo (RLF). Otros síntomas asociados con RLF

24 Facultad de Medicina, Universidad de Chile

Nivel de manejo del médico general: Diagnóstico Específico Tratamiento Inicial Seguimiento Derivar

Fisiopatología: La obstrucción de la vía biliar y la colonización bacteriana subsiguiente condicionan la aparición de la infección. Los microorganismos de la flora entérica, aerobios y, en menor grado, anaerobios, son los agentes causales en la mayoría de casos. Microorganismos más habituales: bacterias G(-) E.coli, Klebsiella, Enterobacter; bacterias G(+): > frecuente Enterococcus; también pueden haber anaerobios (Bacteroides, Clostridium).

Aspectos Esenciales

Diagnóstico

• •

De Sospecha: Clínica: La tríada de Charcot (fiebre acompañada por calofríos, ictericia y dolor en hipocondrio derecho) constituye la sintomatología más característica, presente en un 75% de los pacientes. En pacientes más comprometidos: Péntada de Reynolds (se agrega shock séptico y alteración de conciencia) característica de la Colangitis supurativa, estos pacientes presentan 50% de mortalidad.

Colangitis

• •

Triada de Charcot: Ictericia, fiebre y dolor. Pentada de Reynolds: se agrega a la triada de Charcot confusión mental y compromiso hemodinámico. Principal causa: Coledocolitiasis. La base del tratamiento es el drenaje biliar.

Caso Clínico Tipo

Laboratorio: Al hemograma: leucocitosis, aumento de PCR, u otros cambios sugerentes de inflamación. Perfil hepático anormal (fosfatasa alcalina y gamma-glutamil transpeptidasa elevadas [patrón colestásico]; alanina aminotransferasa y aspartato aminotransferasa elevadas). Hiperbilirrubinemia directa. Puede asociarse elevación de enzimas pancreáticas. Elevación de GOT o GPT > 1000 sugiere formación microabcesos hepáticos.

Paciente de 56 años con antecedentes de cólico biliar llega a urgencias con dolor en hipocondrio derecho. Al examen físico destacan escleróticas ictéricas, fiebre.

Definición Síndrome clínico caracterizado por fiebre, ictericia, y dolor abdominal que se desarrolla como resultado de estasis e infección aguda en el tracto biliar por bacterias ascendentes desde el duodeno, pudiendo constituir un cuadro séptico grave. También se conoce como colangitis ascendente. Constituye una urgencia médico-quirúrgica.

Imágenes: En pacientes con pruebas hepáticas anormales y Tríada de Charcot, se procede directamente a CPRE para confirmar el diagnóstico y proporcionar un drenaje biliar inmediato. En pacientes con signos y síntomas sugestivos de Colangitis aguda, pero sin la tríada de Charcot, se recomienda Ecografía transabdominal para buscar dilatación del conducto biliar común o cálculos. Si la Ecografía muestra dilatación ductal o cálculos, debe ser seguido rápidamente (en 24 horas) por CPRE para proporcionar un drenaje biliar. Si la etiología de la obstrucción biliar no está claro después de la CPRE, se puede emplear TAC o Colangiopancreatografía por resonancia magnética (CPRM).

Etiología-Epidemiología-Fisiopatología Epidemiología: Sólo el 50 a 75% de los pacientes se presentan con la tríada de Charcot (fiebre, ictericia y dolor abdominal). La mortalidad global por esta condición es de 5%, pudiendo ser de hasta 40% en pacientes con patologías asociadas.

Definitivo: Si, además de cumplir con los criterios para un diagnóstico de sospecha, el paciente también tiene: Dilatación biliar en imágenes. Evidencia de una etiología en las imágenes (por ejemplo, estenosis, cálculo, o stent).

Etiología: Secundario a obstruccion de la via biliar y la presencia de bacterias. La exctasia biliar produce que las bacterias aciendan por los conductos linfaticose infecten las vias biliares. Bacteria mas frecuente aislada es E.Coli y Anaerobios (Bacteroides fragilis).

Tratamiento

Factor predisponente más importante para la Colangitis aguda es la obstrucción biliar y estasis. Las causas más comunes de la obstrucción biliar en pacientes con Colangitis aguda sin stents en ductos biliares son: cálculos biliares (28 a 70%), estenosis benigna (5 a 28%), y los tumores malignos (10 a 57%). Además, la Colangitis aguda es una complicación común de la colocación de un stent en la obstrucción biliar maligna (18%).

Monitoreo para tratamiento de la sepsis, cobertura antibiótica adecuada, establecimiento de un drenaje biliar y analgesia. • Los pacientes con Colangitis aguda deben recibir tratamiento empírico con antibióticos que cubran bacterias colónicas. Una vez que los resultados de hemocultivos están disponibles, el tratamiento debe ser ajustado, ad-

25 Facultad de Medicina, Universidad de Chile





ministrar cristaloides. ATB:Cefalosporina de 3era generacion + Metronidazol. Se recomienda Esfinterotomía endoscópica con la extracción de cálculo y/o inserción de un stent para colocación de drenaje biliar en la colangitis aguda en lugar de sólo tratamiento con antibióticos. Comúnmente, los cálculos de la vía biliar pueden ser eliminado con éxito en el 90 a 95% de pacientes después de la Esfinterotomía. Si la CPRE no es técnicamente factible o no puede establecer el drenaje biliar, éste a menudo se puede lograr mediante la Colangiografía transhepática percutánea o Descompresión quirúrgica abierta. Aproximadamente 70 a 80% de los pacientes con Colangitis aguda responderá al tratamiento conservador con terapia antibiótica. En estos pacientes, el drenaje biliar entonces puede realizarse de forma electiva (dentro de 24 a 48 horas). Si el paciente no mejora en las primeras 24 horas con tratamiento conservador, se requiere Descompresión biliar urgente. Ésta también indicada en pacientes con signos de Colangitis supurativa aguda, tales como: Dolor abdominal persistente, Hipotensión a pesar de resucitación adecuada, Fiebre superior a 39°C, Confusión mental (predictor de mal pronóstico).

Seguimiento Derivar para drenaje biliar precoz. En causa biliar: Corregir colelitiasis lo antes posible.Si cuadro es secundario a coledocolitiasis estos deben ser colecistectomizados.

Autor / Editor Nevelline Salgado

Año 2016

26 Facultad de Medicina, Universidad de Chile

Nivel de manejo del médico general: Diagnóstico Específico Tratamiento Inicial Seguimiento Derivar

cado por obstrucción del conducto cístico solo. Los estudios sugieren que se requiere un irritante adicional para desarrollar la inflamación de la vesícula biliar. Una vez que la inflamación de la vesícula biliar se inicia, los mediadores inflamatorios se liberan, propagándose aún más la inflamación de la vesícula biliar. En muchos pacientes, la infección del sistema biliar también está involucrada en el desarrollo de colecistitis aguda.

Aspectos Esenciales

Diagnóstico



Clínica: Dolor abdominal en hipocondrio derecho o epigastrio, que con frecuencia se irradia hacia hombro o dorso ipsilateral. El dolor es característicamente constante e intenso dura de 12 -24 horas (a diferencia del colico biliar simple que dura menos de 6 horas), cediendo parcialmente a la analgesia. Se asocia con frecuencia a náuseas, vómitos, escalofríos y fiebre. La fiebre puede estar ausente en pacientes mayores. Examen físico: Defensa muscular involuntaria o una masa dolorosa en región vesicular con tope inspiratorio (signo de Murphy). Puede objetivarse febril y taquicárdico. No es habitual la ictericia, solo cuando se produce una Colecistopancreatitis o Síndrome de Mirizzi. Los pacientes con complicaciones pueden tener signos de sepsis (gangrena), peritonitis generalizada (perforación), crepitación abdominal (colecistitis enfisematosa), u obstrucción intestinal (íleo biliar).

Colecistitis Aguda

• •

Ocurre predominantemente como una complicación de la Litiasis biliar. La Ecotomografía abdominal es la técnica de elección. La Colecistectomía Laparoscópica precoz (dentro de las 48 a 96 horas) es la terapia de elección.

Caso Clínico Tipo Paciente varón de 56 años, IMC 32, HTA y DM II, consulta al SU por dolor EVA 10 en hipocondrio derecho, asociado a náuseas y vómitos, en contexto de fiestas patrias. Al examen, Murphy (+), taquicárdico, subfebril, blumberg (-). Se solicitan exámenes de sangre y ecografía abdominal.

Laboratorio: Leucocitosis y desviación izquierda; algunos signos de deshidratación (BUN elevado); puede existir elevación moderada de amilasa sérica (< 1.000 U/L). La elevación en las concentraciones de bilirrubina y fosfatasa alcalina en suero orientarian a coledocolitiasis concomitante. Imágenes: La Ecotomografía abdominal es la técnica de elección. Colescintigrafía nuclear puede ser útil en casos en que el diagnóstico sigue siendo incierto después de la ecotomografía. engrosamiento de pared de mas de 4mm orienta a colecistitis.

Definición Síndrome clínico caracterizado por la inflamación de la pared vesicular en asociación con obstruccion persistente de la salida del conducto cistico. Es la complicacion mas frecuente de la colelitiasis.

Etiología-Epidemiología-Fisiopatología Epidemiología: Por lo general se desarrolla en pacientes con antecedentes de cálculos biliares sintomáticos. En una revisión sistemática, se vio en el 6 a 11% de los pacientes con litiasis biliar sintomática, durante un seguimiento de 7 a 11 años. Ocurre principalmente en pacientes de 40-80 años

Tratamiento General: Hidratación parenteral, corrección de alteraciones hidroelectrolíticas, analgesia con AINES u opiáceos y se recomienda el uso de antibióticos, aunque no exista evidencia de infección, pues reducen las complicaciones. En caso de vómitos se puede instalar sonda nasogástrica. Tratamiento definitivo: Es quirúrgico, dado que existe un alto riesgo de recurrencia. Es recomendable la Colecistectomía Laparoscópica precoz (dentro de las 48 horas) como terapia de elección, dependiendo del grado de competencia del equipo quirúrgico. La Colecistectomía Clásica continúa siendo una conducta frecuente y válida en nuestro medio.

Etiología: Se produce en el contexto de la obstrucción del conducto cístico. Suele desencadenarse tras una ingesta grasa abundante. Son factores de riesgo para litiasis vesicular: No modificables: Edad, sexo femenino, ancestro amerindio, factores genéticos. Potencialmente modificables: Obesidad, resistencia insulínica, DM, baja brusca de peso en obesos mórbidos, vida sedentaria, embarazos frecuentes, colesterol HDL bajo, hipertrigliceridemia, drogas hipolipemiantes (clofibrato).

Seguimiento.

Fisiopatología: El desarrollo de colecistitis aguda no está totalmente expli-

Derivar.

27 Facultad de Medicina, Universidad de Chile

* Complicaciones de la colecistitis aguda: En caso de Empiema, perforacion o pacientes diabeticos requieren resolución quirúrgica urgente. Colecistitis xantogranulomatosa: Engrosamiento de la pared de aspecto pseudotumoral. Vesícula en porcelana: Inflamacion crónica de la pared. Síndrome de Mirizzi: Fistula colecistocoledociana.

28 Facultad de Medicina, Universidad de Chile

várices esófago-gástricas (VEG) y 8% por Síndrome de Mallory-Weiss. Poco frecuente etiología tumoral, hemobilia y fístulas aorto-entéricas. HDB: 9% se origina en intestino delgado y 6% es de origen indeterminado. Causas más frecuentes son enfermedad diverticular (15-55%) y angiodisplasia en mayores de 65 años. Se divide la etiología según grupo etario: adolescentes-adultos: divertículo de Meckel, enfermedades inflamatorias intestinales (EII), pólipos. Adultos-ancianos: enfermedad diverticular (causa más frecuente de HDB luego de patología orificial), neoplasias colorrectales, angiodisplasia, colitis isquémica. Patología ano-orificial está presente en todas las edades.

Hemorragia Digestiva Alta y Baja Nivel de manejo del médico general: Diagnóstico Sospecha Tratamiento Inicial Seguimiento Derivar

Aspectos Esenciales • • •

La HDA y la HDB se distinguen por localización, es decir, sobre o bajo al ángulo de Treitz (unión duodeno-yeyunal). Sólo la hematemesis permite asegurar que se trata de una HDA. La causa más frecuente de HDA es UGD, en HDB es por divertículos.

Fisiopatología: HDA: La UGD está asociada a la presencia de H. pylori, uso de AINES y stress. Las úlceras subcardiales y de cara posterior del bulbo, sangran más profuso por su proximidad al tronco celiaco. El síndrome Mallory-Weiss se asocia a náuseas y vómitos. Se produce por desgarro de la mucosa y/o submucosa del esófago distal. Las VEG suelen presentarse como hemorragia violenta, con rápido compromiso del estado general y muerte. HDB: Pacientes con divertículos, usuarios de AINE tienen mayor riesgo de sangrado. Las etiologías incluyen angiodisplasia, EII y procesos neoplásicos.

Caso Clínico Tipo Paciente de 60 años, con antecedente de múltiples cirugías, consulta por dolor en hemiabdomen izquierdo EVA 8/10, asociado a fiebre, melena, anemia e inestabilidad hemodinámica.

Diagnóstico HDA: Hematemesis, melena y/o hematoquezia. Sólo hematemesis permite asegurar que la hemorragia digestiva es alta, la aspiración de material hemático por SNG tiene valor similar. Lo principal es establecer el grado de compromiso hemodinámico. Buscar antecedentes de DHC, consumo de AINEs, episodios de vómitos, síndrome ulceroso, etc., pueden ayudar al diagnóstico etiológico. HDB: Principalmente hematoquezia y rectorragia. Puede haber melena ante tránsito lento, dolor abdominal en angiodisplasia, EII y colitis isquémica. Las hemorragias crónicas se pueden presentar como sangre oculta en deposiciones, episodios ocasionales de melena, o incluso rectorragia de pequeña cuantía. Hematocrito de inicio menor a 35%, signos vitales anormales luego de una hora de iniciado el manejo médico y la presencia de abundante sangre al examen rectal, son indicadores de HDB severa y de pronóstico adverso. Antecedentes de dolor anal, prurito y sangre roja fresca en el papel, frecuentemente son expresión de una fisura anal o de patología hemorroidal. Se debe realizar examen rectal. El estudio endoscópico (EDA-colonoscopía) permite diagnóstico etiológico y manejo de la hemorragia digestiva alta y baja respectivamente. Exámenes alternativos: angioTAC, angiografía, estudio con glóbulos rojos marcados con tecnecio, etc.

Definición Es toda pérdida de sangre que proceda del sistema digestivo. Según su localización se define en alta (entre esófago y duodeno, hasta el ángulo de Treitz) y baja (distal al ángulo de Treitz).

Etiología-Epidemiología-Fisiopatología Epidemiología: HDA: La Hemorragia digestiva alta es más frecuente en hombres que en mujeres. Incidencia y gravedad aumentan con la edad y es una causa frecuente de consulta en servicios de urgencia. La HDA no varicosa tiene una mortalidad de 1-14%, en tanto la HDA varicosa tiene una mortalidad a un año de 30%. HDB: La hemorragia digestiva baja corresponde a un tercio de las hemorragias gastrointestinales, es más frecuente en hombres y pacientes de edad avanzada y tiene una mortalidad de 3-6%. El 80% se detienen en forma espontánea y hasta un 25% recurre. Etiología: HDA: Origen péptico es lo más frecuente con úlceras gástricas o duodenales (UGD), gastropatía erosiva o ulceraciones gástricas, duodenitis erosiva, esofagitis erosiva y úlceras esofágicas. Por otro lado, 12 a 15% son por ruptura de

Tratamiento HDA: • Hospitalizar a todo paciente y evaluar compromiso HDN.

29 Facultad de Medicina, Universidad de Chile



• • • • •



Instalar dos bránulas en vías venosas periféricas lo más gruesas posibles, administrar suero fisiológico 0,9% e idealmente alguna solución cristaloide. No administrar suero glucosado, evaluar signos vitales y si se logra estabilización con volumen utilizado. Asegurar vía aérea. Estabilizar coagulación y retirar medicamentos como AINEs. Todos los pacientes deben quedar bajo altas dosis de inhibidores de la bomba de protones. Pacientes cirróticos y aquellos que pudieran haber aspirado deben recibir antibióticos de amplio espectro. El tratamiento definitivo es de los agentes causantes y es endoscópico. Si es “no varicosa” se hace inyección de solución de adrenalina asociada a esclerosante o aplicación de calor local. En la HDA varicosa se usa terlipresina vía EV y ligadura elástica de las várices esofágicas dentro de las primeras 24 hrs. En caso que no se controle el sangrado con técnica endoscópica o mediante uso de fármacos se debe recurrir a procedimientos de segunda línea: TIPS y cirugía. El uso de balón de Sengstaken tiene principal utilidad ante sangrado persistente, como medida puente ante el traslado a un centro especializado (para endoscopía o cirugía).

HDB: • Se hospitalizan las HDB con compromiso HDN o persistentes. Ante patología orificial o HDB muy leve, puede manejarse ambulatoriamente y diferir el estudio. • Se debe evaluar la magnitud y repercusión hemodinámica, e iniciar reposición de volumen. • La hemorragia persistente o clínicamente significativa indica que se debe realizar un estudio durante la hemorragia: angioTAC, angiografía de urgencia o cintigrafía con glóbulos rojos marcados. • Puede ser tratada con radiología intervencional (embolización) o cirugía. Idealmente realizar la colonoscopía dentro de las primeras 12-24 horas del ingreso hospitalario, una vez conseguido la estabilidad hemodinámica del paciente. • Descartar causas de HDA (UGD, SMW, ectasia gástrica vascular antral, esofagitis y gastritis).

Seguimiento Derivar.

Autor / Editor Hernán Rubilar

Año 2016

30 Facultad de Medicina, Universidad de Chile

servidas. Se describe de igual forma en situaciones de mala higiene y hacinamiento; en viajes a países con endemia de VHA, antecedente de uso de drogas y contacto sexual entre hombres.

Hepatitis Aguda A Complicada Nivel de manejo del médico general: Diagnóstico Específico Tratamiento Inicial Seguimiento Derivar

Fisiopatología: El Virus hepatitis A, es un virus RNA, que establece infección aguda con rango de incubación entre 15 a 50 días. Las complicaciones más frecuentes son hepatitis colestásica 38%, encefalopatía 15%, artritis e insuficiencia renal aguda 8%. El proceso inflamatorio se presenta con escasa afectación lobular y predomina la necrosis periportal. Manifestaciones extrahepáticas se originan a partir de depósitos de complejos inmunes en los tejidos.

Aspectos Esenciales • • • •

VHA es del tipo RNA, cuya infección es aguda. La complicación más frecuente en niños es hepatitis colestásica. TP <40% que no corrige con vit. K, es hepatitis aguda grave; si hay encefalopatía, es fulminante. IgM anti VHA denota infección aguda.

Diagnóstico Clínica: Hepatitis colestásica: prurito, ictericia, coluria y acolia de semanas o meses. Elevación de transaminasas, fosfatasa alcalina y del colesterol. Hepatitis fulminante: disfunción hepática severa asociada a encefalopatía, ocurre en 0,1% de los casos. Sospechar con TP prolongado. Hepatitis recurrente: ausencia de síntomas y mejoría de exámenes que posteriormente presentan nueva alza de transaminasas, en general sin ictericia ni síntomas relevantes. Hepatitis autoinmune: Originado por mimetismo molecular con autoantígenos. Presencia de anticuerpos ANA, ASMA, ALKM-1, ANCA. Manifestaciones extrahepáticas: Rash y Artralgias es lo más frecuente.

Caso Clínico Tipo Paciente hombre 15 años, consulta por dolor abdominal, ictericia, coluria y oliguria, refiere haber comido mariscos hace un mes. Dentro de los exámenes destaca: BUN 31 mg/dl, creatininemia 4 mg/dl, GOT 2.111 U/l, GPT 3.696 U/l; FA 428 U/l; GGT 758 U/l. Serología: IgM anti VHA (+).

Definición Enfermedad infecciosa del hígado causada por virus hepatitis A (VHA), caracterizada por inflamación del parénquima hepático. La hepatitis A se ha considerado más bien inocua; sin embargo, existen reportes de casos de hepatitis colestásica, recurrente, fulminante y autoinmune.

SNC

Meningoencefalitis, Síndrome de Guillian-Barré

Renales

Insuficiencia Renal Aguda Oligúrica

Etiología-Epidemiología-Fisiopatología

Hematológicas

PTI, leucopenia, hemólisis masiva aguda

Epidemiología: En Chile se ha presentado como endemia intermedia. Ha tenido epidemias que han disminuido en su frecuencia e intensidad desde 1990, dada por mejor situación sanitaria y socioeconómica del país. Presenta un riesgo menor a 6/100.000 habitantes desde el 2006, y la infección se ha desplazado a edades mayores. El 70% de los casos en adultos son sintomáticos, aumentando la morbilidad. La mayoría de los casos de hepatitis aguda por VHA presentan evolución benigna y autolimitada; existen formas clínicas prolongadas con recaídas y 0,1% a 0,2% de los casos evolucionan a falla hepática aguda. En América Latina es la causa más frecuente de trasplante hepático en niños.

Cardiovasculares

Hipotensión, Bradicardia, vasculitis cutánea

Tracto Gastrointestinal

Pancreatitis Aguda, colecistitis gangrenosa aguda.

Cutáneas

Necrólisis epidérmica tóxica, urticaria, vasculitis cutánea

Otros

Efusión pleural eosinofílica, infección viral mixta

Tratamiento Hepatitis aguda colestásica: Benigna con normalización progresiva de perfil hepático. Puede realizarse ecografía abdominal para descartar obstrucción biliar. Si prurito es muy molesto, puede usarse Colestiramina (4-16g/día).

Etiología: VHA se transmite por alimentos y agua contaminada. Epidemias y casos esporádicos se asocian al consumo de bivalvos, algunas verduras, frutas y alimentos contaminados con aguas

31 Facultad de Medicina, Universidad de Chile

Hepatitis recurrente: Se debe aumentar higiene y descartar infección por VHC o VHB. Hepatitis fulminante: Hospitalización en UCI y soporte en espera de trasplante. Manifestaciones extrahepáticas: Tratamiento antiviral o conservador es controversial.

Seguimiento Derivar a especialista.

Autor / Editor Hernán Rubilar

Año 2016

32 Facultad de Medicina, Universidad de Chile

• Misceláneas: Infiltración maligna, Hepatitis autoinmune. • Indeterminadas. Fisiopatología: Es compleja; estrecha relación con necrosis y apoptosis de los hepatocitos, inducidas por distintas vías moleculares. El mecanismo de la necrosis está dado por depleción de ATP, edema cerebral, despolarización mitocondrial y ruptura de membrana celular; por otro lado en la apoptosis se mantiene el ATP, con activación de caspasas, condensación de cromatina, degradación de ADN y reabsorción de los componentes de la célula. La apoptosis y necrosis de los hepatocitos, conlleva una activación de las células de Kupffer y de la respuesta inmune innata, desencadenándose una respuesta inflamatoria sistémica intensa, que a través de distintos mediadores humorales y celulares, inducirá falla orgánica múltiple.

Insuficiencia Hepática Aguda Nivel de manejo del médico general: Diagnóstico Sospecha Tratamiento Inicial Seguimiento Derivar

Aspectos Esenciales • • •

La identificación de la etiología es importante para guiar el tratamiento y definir el pronóstico. Etiologías más frecuentes: VHA, drogas (paracetamol). Examen mental debe hacerse cada 4-6 horas.

Diagnóstico

Caso Clínico Tipo

Clínica: fundamental la sospecha en todo paciente ictérico que evolucione con náuseas y vómitos, alteración de conciencia o alteraciones de la coagulación (hematoma-equimosis). Para el diagnóstico se requiere la presencia de encefalopatía hepática en cualquiera de sus grados. A diferencia de la encefalopatía de la insuficiencia hepática crónica, ésta desarrolla edema cerebral (principal causa de muerte en encefalopatía grado IV). Es frecuente además la presencia de edema, dolor abdominal y ascitis (peor pronóstico).

Paciente femenino de 4 años con cuadro de 15 días de evolución de ictericia, coluria, acolia y fiebre no cuantificada, posteriormente presenta elevación de enzimas hepáticas, anasarca, distensión abdominal y dificultad respiratoria. 5 días antes de su ingreso presentó deterioro de su estado de conciencia con bradicardia, hipertensión arterial e hipoglicemia.

Laboratorio: destaca alteración del TP (<40% o INR >1.5), que es parte de la definición. Elevación de transaminasas, hiperbilirrubinemia, hipoglicemia, trombocitopenia, y evidencia de falla orgánica múltiple a medida que progresa el cuadro. Se debe dirigir la anamnesis, examen físico y laboratorio, a la búsqueda de etiologías que orienten un tratamiento específico.

Definición Trastorno de la función del hígado, generalmente de inicio súbito, que se caracteriza por la aparición de encefalopatía hepática y alteración del tiempo de protrombina/INR (< 40% o INR > 1.5), dentro de las 8 semanas de inicio de un cuadro ictérico, en ausencia de daño hepático previo (o desconocido). Se incluyen pacientes con hepatopatías crónicas, con diagnóstico menor de 26 semanas (enfermedad de Wilson, reactivación hepatitis B, superinfección por agente Delta, etc.).

Tratamiento •

Etiología-Epidemiología-Fisiopatología



Epidemiología: En general se presenta en pacientes jóvenes. Tiene baja prevalencia, pero con elevada morbimortalidad. Sin trasplante, mortalidad es de 85% aproximadamente. La supervivencia a un año, en pacientes con y sin trasplante es cercana al 65%. Etiología: • Infecciones Virales: VHA, VHB, VHE; CMV, EBV, HSV en pacientes con comorbilidades. • Drogas y toxinas: Paracetamol, Amanita phalloides, éxtasis, idiosincráticas. • Cardiovasculares: Insuficiencia Cardíaca, Hígado de shock, Sd. Budd – Chiari. • Metabólicas: Hígado graso del embarazo, Enfermedad de Wilson.

• •

Derivación precoz a centro donde se disponga de posibilidad de trasplante hepático y hospitalización en UCI para manejo de soporte. Tratamiento específico según etiología: N-acetilcisteína en intoxicación por paracetamol, Aciclovir EV en hepatitis fulminante por Herpes; Interrupción del embarazo en hígado graso agudo de embarazo; Corticoides en hepatitis autoinmune. Considerar trasplante hepático. Manejo de las complicaciones: neurológicas, hemodinámicas, respiratorias, infecciosas, coagulopatías, renales, nutricionales, hipoglicemias.

Seguimiento Derivar a especialista. Autor / Editor Julián Bravo

33 Facultad de Medicina, Universidad de Chile

Revisor de Formato Año Angel Eloy 2016

mia; obstrucción anatómica por tumor o lesión periampular; infecciosas (viral, bacteriana o parasitaria); traumática; post colangiopancreatografía retrógrada endoscópica (CPRE); post quirúrgica; genéticas; autoinmune, por fármacos e idiopática.

Pancreatitis Aguda Nivel de manejo del médico general: Diagnóstico Sospecha Tratamiento Inicial Seguimiento Derivar

Fisiopatología: Cualquiera sea la etiología inicial, se desencadenan mecanismos que llevan a una respuesta local y sistémica. A nivel local, la activación intracelular lleva a la autoactivación enzimática que va a iniciar el proceso autodigestivo y las alteraciones locales. Por otro lado la activación de diversos mediadores, explica las alteraciones a nivel sistémico.

Aspectos Esenciales • • • • • •

Autodigestión pancreática. Causa más frecuente: coledocolitiasis. Epigastralgia, irradiado al dorso en faja. Amilasa y lipasa plasmáticas mayores a 3 veces el normal + clínica hacen el diagnóstico. La infección de la necrosis constituye la complicación más grave, con implicancias en pronóstico y tratamiento. Tratamiento médico: régimen 0, hidratación y analgesia parenteral.

Diagnóstico Debe sospecharse ante el inicio agudo de dolor abdominal epigástrico, permanente con sensibilidad a la palpación al examen físico. Pueden estar presentes distensión abdominal, ictericia y colapso cardiocirculatorio en casos más graves. Al examen físico abdominal destaca dolor a la palpación, pero con escasa o nula resistencia. Blumberg (-). Se puede encontrar taquicardia, taquipnea y fiebre. Signos clásicos como el signo de Mayo-Robson (dolor a la palpación del ángulo costo lumbar posterior izquierdo) y el signo de Cullen (equimosis periumbilical) son poco frecuentes. El diagnóstico se rige según la presencia de 2 de 3 criterios, a saber: 1. Cuadro clínico: aparición de dolor abdominal agudo, epigástrico, intenso, persistente, que puede presentarse con irradiación al dorso en faja o cinturón, con frecuencia asociado a náuseas y vómitos. 2. Laboratorio: elevación de amilasa y/o lipasa 3 veces sobre sus niveles normales. La lipasa es más específica. Sus niveles no se correlacionan con gravedad ni pronóstico. 3. Imágenes: la ecotomografía abdominal es útil para identificar patología biliar, y el TAC de abdomen es el examen de elección para detectar las complicaciones locales, pero su rendimiento es mejor tras 72 hrs de iniciado el cuadro. Otros: En el Hemograma puede haber Leucocitosis y aumento del Hematocrito por hemoconcentración. Es importante estratificar el cuadro según gravedad (por ejemplo score APACHE II), para guiar la búsqueda de complicaciones, tratamiento y pronóstico.

Caso Clínico Tipo Mujer de 56 años, con antecedentes de DM2, dislipidemia y colelitiasis. Consulta por un cuadro de 7 horas de evolución de dolor abdominal en epigastrio e hipocondrio izquierdo permanente, que se irradia al dorso, asociado a náuseas y vómitos persistentes.

Definición Proceso inflamatorio agudo del páncreas que presenta una respuesta inflamatoria tanto local como sistémica, y que se caracteriza por dolor abdominal asociado a elevación de los valores plasmáticos de las enzimas pancreáticas.

Etiología-Epidemiología-Fisiopatología Epidemiología: En un 80% los pacientes cursan con pancreatitis aguda leve y en los que se presentan con pancreatitis aguda grave, la mortalidad puede llegar al 30%. La mortalidad global es de aproximadamente un 5%. En EE.UU la etiología biliar y alcohólica da cuenta del 60 a 75% de los casos.

Tratamiento

Etiología: Tiene múltiples causas, dentro de las cuales la más frecuente es la etiología biliar (35-40% de los casos), por obstrucción mecánica ampular, produciendo aumento de la presión intraductal en forma retrógrada y rotura de células acinares pancreáticas.

• • • • •

Otras causas: drogas como alcohol, tabaco y otras; metabólicas como hipertrigliceridemia (> 1.000 mg/dL) e hipercalce-



Hospitalización en UPC (unidad de intermedio o de cuidados intensivos, según gravedad), Régimen 0 (reposo pancreático), Hidratación parenteral y Corrección de alteraciones hidroelectrolíticas, Analgesia parenteral. Control de inflamación, náuseas y dolor (sobre el uso de morfina, no existen evidencias que agrave el cuadro). Monitorización de signos vitales y débito urinario.

34 Facultad de Medicina, Universidad de Chile

• •



Asistencia nutricional precoz en pacientes con pancreatitis aguda severa o ayuno mayor de 7 días, idealmente nutrición enteral frente a nutrición parenteral. Frente a la sospecha de infección de necrosis pancreática, debe confirmarse tomando hemocultivos y cultivo de la necrosis por punción. Si se confirma, iniciar tratamiento antibiótico (Infección por translocación bacteriana) + desfocalización (necrosectomía quirúrgica). Indicación quirúrgica: CPRE de urgencia cuando se asocia colangitis. Ante pancreatitis biliar severa con compromiso progresivo, realizar Papilotomía Endoscópica. Programar Colecistectomía diferida, tras resolución del cuadro agudo (idealmente antes de 4 semanas).

Seguimiento Derivar. Pancreatitis leve se recuperan sin secuelas. La mortalidad de Pancreatitis grave va de un 7-10% y en el grupo con complicaciones puede llegar al 30%.

Autor / Editor Hernán Rubilar

Año 2016

35 Facultad de Medicina, Universidad de Chile

Fisiopatología: Se piensa que ocurre por un desequilibrio en la hemodinamia renal debido a vasoconstrictores y vasodilatadores renales. Sería mediado por una vasoconstricción renal inapropiada, donde participarían SRAA y diversos mediadores. Los riñones presentan integridad estructural que puede ser comprobada por examen de orina, de imágenes o biopsia. Se comporta como un cuadro prerrenal funcional. Se da en contexto de cirrosis avanzada como parte de su progresión o puede ser desencadenado por hemorragia digestiva grave, sepsis, peritonitis bacteriana espontánea (PBE) o paracentesis evacuadoras.

Síndrome Hepatorrenal Nivel de manejo del médico general: Diagnóstico Sospecha Tratamiento Inicial Seguimiento Derivar

Aspectos Esenciales • • • •

Complicación de pacientes con cirrosis y ascitis. Desencadenantes: hemorragia variceal, PBE, sepsis y paracentesis evacuadora. Manejo inicial: telipresina + albúmina. Muy mal pronóstico.

Diagnóstico Lo característico es la azotemia progresiva, hiponatremia, oliguria, hipotensión y ascitis refractaria. Se debe hacer el diagnóstico diferencial con IRA prerrenal o necrosis tubular aguda. Criterios diagnósticos: • Cirrosis + ascitis • Creatinina sérica >1.5 mg/dL (que no mejora al expandir volumen o con la supresión de los diuréticos) • Ausencia de shock • Ausencia de nefropatía parenquimatosa junto a sedimento urinario sin datos de interés. Existe el Síndrome hepatorrenal tipo 1 y tipo 2 según las características de instalación de la falla renal.

Caso Clínico Tipo Paciente cirrótico de 56 años consulta por ascitis a tensión, refractaria a diuréticos, se pesquisa creatinina de 2,1 (previa de 1,1).

Definición Es una de las muchas causas potenciales de lesión renal aguda en pacientes con enfermedad hepática aguda o crónica. Por tanto, es una complicación grave de un paciente con cirrosis y ascitis, y se caracteriza por hiperazotemia progresiva con retención de sodio y agua, y disminución de la excreción de agua libre, manifestado por oliguria, sin una causa identificable específica de disfunción renal.

Tratamiento El tratamiento de elección es el trasplante hepático. Las medidas de manejo de primera línea es el uso de Telipresina asociada con Albúmina, hay respuesta en ± 65% de los SHR tipo 1. Si no responden de forma adecuada el paso siguiente es TIPS. La principal medida es la prevención del SHR, previniendo las otras complicaciones de la cirrosis hepática como la hemorragia variceal, la PBE y realizando paracentesis de forma segura. PBE: Albúmina (1,5 gr/kg al inicio y 1 gr/kg al día 3) + tratamiento antibiótico. Paracentesis Evacuadora (superior a 5L): Administrar albúmina 8 gr/L extraído.

Etiología-Epidemiología-Fisiopatología Epidemiología: Incidencia se evaluó en un estudio prospectivo de 229 pacientes sin azotemia, con cirrosis y ascitis: el síndrome hepatorrenal (SHR) se desarrolló en 18 y 39% al año y a los cinco años, respectivamente. Los pacientes con hiponatremia y un aumento en actividad de la renina plasmática estaban en mayor riesgo. El SHR también se presenta con frecuencia en pacientes con enfermedad hepática aguda. En un estudio de pacientes con hepatitis alcohólica, SHR se produjo en 28 de 101 pacientes (27%).

Seguimiento

Etiología: En general los pacientes con este síndrome, tienen hipertensión portal debido a cirrosis hepática, hepatitis alcohólica grave, o en menor medida por tumores metastásicos, pero también pueden tener insuficiencia hepática fulminante por otras causas. El SHR representa la etapa final de una secuencia de reducciones en la perfusión renal, inducida por la lesión hepática cada vez más grave.

Derivar. Los pacientes con SHR deben ser manejados por especialista en los hospitales de referencia. Los pacientes con SHR tipo I y rápida progresión deben ser derivados de forma urgente a hospitales base de mayor complejidad con capacidad de manejo intermedio/intensivo y especialistas adecuados. Autor / Editor Hernán Rubilar

36 Facultad de Medicina, Universidad de Chile

Año 2016

BIBLIOGRAFÍA •

https://www.uptodate.com/contents/clinical-manifestations-and-diagnosis-of-gastroesophageal-reflux-in-adults?source=search_result&search=reflujo%20gastroesof%C3%A1gico&selectedTitle=4~150#H2



https://www.uptodate.com/contents/medical-management-of-gastroesophageal-reflux-disease-in-adults?source=search_result&search=reflujo%20gastroesof%C3%A1gico&selectedTitle=2~150#H1



http://sociedadgastro.cl/wp-content/uploads/2016/06/Diagnostico-y-tratamiento-ED.pdf



https://www.uptodate.com/contents/complications-of-gastroesophageal-reflux-in-adults?source=see_link§ionName=Esophageal%20stricture&anchor=H2#H2



Manual de Patologia Quirurgica, F.Crovari,Patologia biliar benigna

37 Facultad de Medicina, Universidad de Chile

Etiología:

Abdomen Agudo

Las principales causas de dolor abdominal agudo:  <50 años: dolor abdominal inespecífico, apendicitis y patología del tracto biliar.  >50 años: patología del tracto biliar, dolor abdominal inespecífico, apendicitis y obstrucción intestinal.  Mujeres: Descartar embarazo/complicación de embarazo (embarazo ectópico, amenaza de aborto, aborto en evolución, aborto completo o incompleto) y considerar la posibilidad de que se trate de una patología ginecológica.

Nivel de manejo del médico general: Diagnóstico Específico Tratamiento Inicial Seguimiento Derivar.

Aspectos esenciales 

   

Corresponde a cualquier dolor abdominal de inicio repentino (de minutos hasta 12 semanas de duración) que presente signos de irritación peritoneal. En la primera evaluación del cuadro se debe establecer la situación hemodinámica y descartar gravedad. Posee una larga lista de diagnósticos diferenciales.Tanto quirurgicas como Medicas No todas las causas de abdomen agudo requieren de cirugía. El tratamiento quirúrgico es necesario, aunque no se conozca el diagnóstico, cuando el paciente presenta mala situación clínica.

Hay que considerar que el diagnóstico diferencial es amplio. A. Etiologías intra-abdominales: En su mayoría requieren de intervención quirúrgica. Hemorragia: Rotura de aneurisma aórtico/aneurisma visceral, rotura de tumor, rotura de órgano sólido (hígado, vaso), sangrado post operatorio, embarazo ectópico roto, rotura de ovario quístico con hemorragia. Inflamación/Infección: Peritonitis: localizada; difusa; química: por ulcera péptica perforada, fuga biliar, ruptura de quiste de ovario. Peritonitis bacteriana secundaria o primaria con cuerpo extraño (catéter para diálisis peritoneal), gastroenteritis, Apendicitis, hepatitis, colecistitis, pancreatitis, diverticulitis, ileitis aguda, colitis, Diverticulitis de Meckel, Absceso intraabdominal, Linfadenitis mesentérica, Enfermedad inflamatoria pélvica, Absceso tubo ovárico. Perforación: Tracto gastrointestinal: Esófago, estómago, duodeno, intestino delgado, colon. Vejiga. Isquemia: Intestinal: émbolo arterial, trombosis venosa mesentérica. Obstrucción de asa ciega. Infarto: hepático, esplénico, de epiplón. Torsión: ovárica, de fibroma uterino. Obstrucción: Gastrointestinal: por adherencias, hernias, tumor, vólvulo, intususcepción, impactación fecal. Biliar: Debido a cálculos, tumor, hematobilia.

Caso clínico tipo Hombre de 24 años consulta al servicio de urgencia por dolor de 6 hrs de evolución; en un principio localizado en región periumbilical y se ha ido irradiando hacia fosa ilíaca derecha alcanzando su intensidad máxima al momento de la consulta. Al examen presenta irritación peritoneal y Blumberg positivo.

Definición Corresponde a cualquier dolor abdominal de inicio repentino que presente signos de irritación peritoneal. Puede ir desde minutos a semanas de evolución. El corte establecido son < de 12 semanas.

B. Etiologías extra-abdominales: pueden no requerir acto quirúrgico. Tórax: Pulmonar: Neumonía, Empiema. Cardíaco: Infarto de miocardio. Metabólico: Cetoacidosis diabética, Crisis Adissoniana, Uremia, Porfiria. Inducido por toxinas: Mordedura de insectos, arañas, anfibios, Tétanos, Drogas Genitourinario: Pielonefritis, Urolitiasis, Torsión testicular, Orqui-Epididimitis. Hematológico: Leucemia, Crisis de Sickle Cell. Neurológico: Tumor del cordón espinal, Herpes Zostér Pared Abdominal: Hematoma parietal Psiquiátrico: Psicógeno

También se incluyen:   

Agudizaciones de procesos crónicos (Pancreatitis crónica, insuficiencia vascular, etc) Constituye una emergencia médico-quirúrgica, pues es necesario descartar que la causa subyacente necesite una resolución quirúrgica. NO es sinónimo de intervención quirúrgica. Los pacientes que requieren cirugía solo representan un subgrupo de enfermos con abdomen agudo.

Fisiopatología:

Etiología-epidemiología-fisiopatología.

Dolor abdominal se divide en tres categorías:

Epidemiología:

Visceral: Es difuso. Ocasionado por la distensión de víscera hueca y es trasmitido a la línea media debido al origen embriológico del órgano comprometido. Parietal o Somático: Es localizado. Ocasionado por irritación mecánica o química del peritoneo parietal. Dolor Referido: Se percibe en una localización distante del lugar de máximo estímulo, lo cual resulta de la confluencia, en la médula espinal, de fibras aferentes

Es una causa muy frecuente de consulta en servicios de urgencia. Es la principal causa de consulta en los servicios de urgencia en EE.UU. Los ingresos varían desde 18 al 42%, con cifras de 63% en pacientes > 65 años.

1

provenientes de diversas áreas del cuerpo. Por ej: dolor epigástrico en un paciente que cursa un IAM.

hipoperfusión), Sepsis de origen abdominal, Isquemia, Perforación y/o peritonitis. Clínica: Obtener una adecuada historia clínica: Antecedentes médicos, quirúrgicos, fármacos, alergias. Antecedentes familiares, Caracterizar el dolor: Tiempo de evolución, localización, irradiación, intensidad, factores agravantes/atenuantes, síntomas asociados. Examen físico: Usar signos al examen físico como rebote o resistencia muscular.Buscar signo de Blumberg, Rovsing, Ademas de descartar masas (Hernias) o visceromegalias. Paciente más estable permite usar analgesia y observación como elemento diagnóstico para decidir conducta. Imágenes: Según hipótesis diagnóstica y disponibilidad, siempre que la estabilidad hemodinámica lo permite. TAC con contraste:Examen de eleccion.Idealmente en causas abdominales (peritonitis, perforaciones)

Diagnóstico. Diagnostico es clinico, Lo escencial es una anamnesis y examen fisico completo. Primera evaluación del paciente: Establecer la gravedad del cuadro y la estabilidad hemodinámica (FC, PA, perfusión tisular). Los cuadros con riesgo vital que primero se consideran son: Síndrome coronario agudo, Abdomen agudo quirúrgico con sepsis de foco abdominal, Isquemia mesentérica, Rotura de aneurisma aórtico. También recordar perforación de víscera hueca. Valorar Causa cardiológica (isquemia: ECG, CPK), Valorar Causas de dolor abdominal agudo: Hemorragia (signos de

2

AngioTAC: Para isquemia mesentérica Transvaginal: en causa gineco-obstétrica.

Ecografía

Exámenes de laboratorio: Hemograma con leucocitosis >10.000/mm3 + cuadro clínico característico orienta a apendicitis. Amilasa y lipasa elevadas en contexto de epigastralgia o dolor en faja irradiado a dorso orientan a pancreatitis. Pruebas hepáticas alteradas en dolor de hipocondrio derecho. Examen de Orina completa en pacientes con hematuria, disuria o dolor en flanco, dolor lumbar+fiebre. Siempre pedir B-HCG en mujeres de edad fértil para descartar causas Ginecologicas. Diagnostico diferencial es extenso, se puede dividir en causas medicas, quirurgicas y extrabdominales. (esquema)

Tratamiento. 1. Valorar y estabilizar hemodinamia mediante examen físico y reanimación agresiva. 2. Manejar dolor: Analgesia 3. Definir si requiere cirugía y si ésta es de emergencia o urgencia. Urgente: Apendicitis, Peritonitis localizada o difusa, Absceso no drenable vía percutánea, Obstrucción intestinal, Isquemia intestinal, Aneurisma aórtico complicado, Neumoperitoneo, Masa inflamatoria o sepsis, Extravasación de contraste en radiografía (perforación). Procesos que en función de su evolución pueden requerir tratamiento quirúrgico: Colecistitis aguda, Pancreatitis aguda, Diverticulitis aguda, Megacolon tóxico. 4. En causa infecciosa (Dolor abdominal+Fiebre>38º) manejar como sepsis: Reanimación y antibióticos para control de foco.

Seguimiento. Derivar a especialista; evaluación junto a equipo quirúrgico según causas expuestas.

3

Los pacientes con Disfunción esofágica tienen dificultad para tragar después de iniciar la deglución y refieren sensación de comida estancada a nivel del hueco supraesternal o retrosternal. Cuando la disfagia es “retrosternal” generalmente coincide con la localización de la lesión; mientras que la disfagia “supraesternal” se refiere desde abajo.

Afagia aguda Nivel de manejo del médico general: Diagnóstico Específico Tratamiento Inicial Seguimiento Derivar.

Ademas hay que indagar sobre los alimentos que desencadenan la afagia o disfagia.La disfagia para sólidos y líquidos desde la aparición de los síntomas es probablemente debido a un trastorno de la motilidad del esófago. En contraste, la disfagia para los sólidos que luego progresa a involucrar a los líquidos es más probable que refleje la obstrucción mecánica. Exámenes complementarios a saber: Radiología convencional con contraste con Bario (Esofagograma con contraste baritado), Endoscopía superior, Test de Motilidad.

Aspectos esenciales.     

Imposibilidad brusca de deglutir. Causas orofaríngeas o esofágicas. Principal causa es impactación de alimento o cuerpo extraño. Tratamiento: extracción endoscópica. Evaluar posible tumor esofágico predisponente.

Tratamiento.

Caso clínico tipo.

-En caso de de presencia de cuerpo extraño, realizar maniobra de hemlich.Si no es posible la extraccion derivar urgentemente para extraccion con esofagoscopía rígida (ER) la que tiene un éxito de entre el 95% a 100%.

Paciente 67 años, sexo masculino, fumador de larga data. Mientras se encuentra en almuerzo familiar, refiere imposibilidad de tragar de inicio brusco. Al examen físico, paciente angustiado, con sialorrea. Realizar EDA, extraer cuerpo extraño, evaluar estenosis esofágica.

- Prevenir la aspiración, indispensable, mediante la colocación de una sonda y la extracción de los líquidos no deglutidos.

Definición.

- El uso de Endoscopía facilita la extracción del objeto en el momento de la observación y torna innecesaria la intervención quirúrgica en la mayoría de los pacientes. Al momento de la extracción endoscópica se recomienda envolver el cuerpo extraño, por ejemplo, en un dedo de guante para evitar el daño y posible perforación iatrogénica del esófago.

Obstrucción esofágica completa que se caracteriza por la imposibilidad para deglutir y se asocia a sialorrea.

Etiología-epidemiología-fisiopatología.

- La administración de Glucagón EV para relajar la musculatura lisa esofágica, se ha propuesto como método para tratar la impactación de alimentos.

La Afagia como Disfagia, pueden ser de causa orofaríngea o esofágica: - Orofaríngea: Se caracteriza por la dificultad para iniciar la deglución debido a la incapacidad del paso del bolo alimenticio desde la faringe, por el esfínter esofágico superior (EES), hacia el cuerpo esofágico. Más frecuente en pacientes de edad avanzada, secundaria a ACV. - Esofágica: Dificultad en la deglución una vez que el bolo ha atravesado la faringe y EES. Su causa se debe a condiciones obstructivas o alteraciones en la motilidad del esófago. La causa más frecuente suele ser el impacto de un bolo alimenticio o cuerpo extraño.

Seguimiento. Luego de solucionada la urgencia, derivar para descartar posibles estrechamientos debido a un tumor u otra enfermedad predisponente que haya facilitado la obstrucción.

Diagnóstico. Se debe basar en primer lugar en la Historia clínica: Distinguir si es orofaríngea o esofágica. Pacientes con Disfunción orofaríngea tienen dificultad para transferir los alimentos desde la boca hasta la faringe y reportan sensación de una obstrucción en el cuello, cuando se les pregunta por el sitio de sus síntomas. Otras molestias pueden incluir tos, ahogo, babeo, y regurgitación al tragar líquidos o alimentos sólidos. (La Disfunción oral puede conducir a regurgitación de alimentos y sialorrea. Disfunción faríngea puede generar tos o asfixia durante el consumo de alimentos, y disfonía).

4

i) Motilidad intestinal anormal: Detectable en el 50% de los pacientes. No existe un patrón de motilidad característico de SII. Puede haber o un aumento de la frecuencia e irregularidad de las contracciones luminales, o un tiempo de tránsito anormal en el (variante constipación), o una respuesta motora exagerada a la colecistoquinina e ingestión de comida (variante diarréica). La estimulación farmacológica de la motilidad intestinal reduce la retención de gases y mejora los síntomas en algunos pacientes.

Colopatía funcional Nivel de manejo del médico general: Diagnóstico: específico. Tratamiento: completo. Seguimiento: completo.

Aspectos escenciales  

  

ii) Hipersensibilidad visceral: Hallazgo frecuente (60%). Hay hiperalgesia y alodinia por hipersensibilización selectiva de nervios aferentes viscerales frente a distensión e hinchazón. No está claro si juega un rol la mediación del sistema nervioso entérico, médula espinal o modulación central. Otros mecanismos posibles incluyen alteraciones de la microbiota fecal, sobrecrecimiento bacteriano intestinal, sensibilidad alimentaria, inflamación, post infeccioso, factores genéticos y factores psicosociales.

Se debe buscar signos y síntomas de alarma que orienten a etiología estructual Se recomienda realizar pruebas serológicas para detectar enfermedad celíaca (por su alta prevalencia en este grupo de pacientes) solo si existen singos de alarma que lo sugieran. Las características principales son dolor abdominal y alteraciones en el hábito intestinal de al menos 3 meses de duración. La alteración del hábito intestinal puede ser con predominio de diarrea o constipación, ambos con manejos distintos. El manejo de las situaciones estresoras desencadenantes es importante.

Diagnóstico Existe un cuadro clínico clásico que se presenta con dolor abdominal crónico tipo cólico de intensidad variable y exacerbaciones periódicas, asociado a diarrea sin elementos patológicos, constipación que puede ser intermitente con diarrea o función normal. Otros síntomas: distención abdominal, meteorismo, dispepsia, síntomas extra-intestinales (dismenorrea, dispareunia, urgencia miccional). El diagnóstico, según los criterios de Roma IV, es: dolor abdominal recurrente al menos 1 día por mes en los últimos 3 meses, asociado a 2 o más de los siguientes:

Caso clínico tipo Consulta paciente de sexo femenino de 18 años, dolor abdominal asociado a distensión, meteorismo y diarrea. En dos semanas más dará la PSU.

Definición

 

La colopatía funcional (o síndrome de intestino irritable SII) es un trastorno digestivo funcional (no orgánico), caracterizado por dolor o malestar abdominal crónico, episódico y recurrente, que asocia a distensión abdominal, cambio de frecuencia y/o cambio de consistencia de las deposiciones



Relacionados con defecación Asociado a cambios de frecuencia de deposiciones Asociado a un cambio en forma de deposiciones

La diferencia ente los criterios de Roma III y IV es la disminución de dolor al menos 3 días al mes a 1 vez al mes, y el criterio de ‘’mejoría’’ con la defecación se cambió por ‘’relacionado’’ con la defecación.

Epidemiología

El examen físico es generalmente normal, pudiendo presentar distensión abdominal.

Es uno de los cuadros clínicos más frecuentes en el mundo, siendo la causa más frecuente de consultas gastroenterológicas ambulatorias. A nivel mundial, se estima una prevalencia alrededor de 11% en población general, con mayor prevalencia en mujeres (2:1). En Santiago, se estimó una prevalencia de 28,6% al año 2013. Según el Consenso Latinoamericano, la prevalencia en la región varía entre 10-20%. Se han reportado que esta población presenta peor calidad de vida que la población general, un alto ausentismo laboral y baja productividad. El diagnóstico generalmente se hace en edades medias de la vida (25-50 años).

Síntomas de alarma que orientan a patología orgánica:     

Pérdida de peso no intencionada (mayor al 10% en tres meses). Presencia de sangre en deposiciones. Síntomas despiertan al paciente en la noche. Historia familiar de cáncer de colon o recto, enfermedad inflamatoria intestinal o enfermedad celíaca. Inicio de síntomas después de los 50 años.

La presencia de alguno de estos signos debe orientar la solicitud de exámenes complementarios (ej: hemograma, VHS, perfil bioquímico, hemorragia oculta en deposiciones, colonoscopía, ecografía abdominal, etc). En paciente que inicia con síntomas luego de los 50 años se debe comenzar el estudio endoscópico.

Etiopatogenia y Fisiopatología Se han descrito varios mecanismos fisiopatológicos en los pacientes con SII. Sin embargo, varios están presentes en la población general asintomática, y ninguno por sí solo explica el síndrome. Los dos mecanismos más aceptados son:

5

que presenten síntomas de alarma para complementación diagnóstico terapéutica.

Tratamiento No existe una terapia específica ni fármaco que pueda englobar toda la fisiopatología. Hay que tener presente que los signos de alarma cambian el manejo. Lo fundamental es la relación médico-paciente y la educación en torno a que es una enfermedad funcional, de carácter benigno, pero que es crónica y recurrente, y se debe tranquilizar, ya muchos pacientes con colopatía funcional tienen temor a presentar un cáncer y buscan organicidad (policonsultan).

Se recomienda derivar a psiquiatría a aquellos pacientes que presenten síntomas persistentes de ansiedad o depresión y no respondan al manejo establecido con antidepresivos.

Tratamiento es sintomático y según gravedad de síntomas. i) Síntomas leves. Manejo no farmacológico: educación, actividad física (2060 minutos de actividad vigorosa por 3-5 días a la semana ha mostrado reducción significativa de la sintomatología), modificación dietaria (dieta FODMAP, baja en carbohidratos fermentables). ii) Síntomas moderados-graves. 





Dolor abdominal: primera línea son antiespasmódicos. Segunda línea se pueden utilizar antidepresivos en dosis pequeñas. Probióticos no se recomiendan rutinariamente. Diarrea: antidiarréicos ej: loperamida 2 mg 45 minutos antes de comidas en dosis regulares (4-8 mg/día). Segunda línea se pueden utilizar secuestrantes de ácidos biliares. No se sugiere utilizar antibióticos rutinariamente. Constipación: dieta rica en fibras. Se pueden hacer ensayos de psyllium o PEG (17-34 gr/día).

Ante síntomas refractarios: reevaluar caso, buscar signos de alarma, adherencia a la terapia y tipo de síntomas.

Seguimiento Se recomienda utilizar el Cuestionario de Gravedad de Síndrome de Intestino Irritable (IBS-SSS), útil para clasificar a los pacientes con SII en función de su gravedad y para valorar su respuesta al tratamiento y determinar la continuidad o modificación del mismo. En una revisión sistemática de 14 estudios, en pacientes con SII, se reportó que:    



Sólo 2-5% fueron diagnosticados con patología gastrointestinal orgánica como causa de los síntomas (seguimiento: 6 meses-6 años). Todos los estudios reportaron persistencia de síntomas al final del seguimiento (seguimiento: 6 meses-15 años). En 5 estudios se reportó ausencia de síntomas solo en 12-38% (seguimiento: 2 años). En 4 estudios se reportó que permanecen sin cambios de síntomas un 30 a 50% de los pacientes y hay empeoramiento de los mismos en 2-18%. (seguimiento 12 meses-7 años). Ningún estudio concluyó con detalle acerca de la frecuencia y duración de las recaídas.

Se deberá derivar a aquellos pacientes que no obtengan respuesta favorable al tratamiento inicial (antiespasmódico y sintomático) asociado al tratamiento antidepresivo posterior a 3 meses de manejo y a aquellos 6

Según la etiopatogenia se puede clasificar como:

Constipación

a) Primaria (Funcional o idiopática): Se plantea una vez que se han descartado las posibles causas de constipación secundaria. Nivel de manejo del médico general: Diagnóstico: Específico. Tratamiento: Completo. Seguimiento: Completo.

b) Secundaria: Cuando nos enfrentamos a un paciente constipado, debemos considerar las diversas causas y mecanismos que pueden conducir a esta patología. Se debe sospechar ante la presencia de signos de alarma tales como: edad de inicio > 50 años, presencia de sangre en deposiciones, anemia, inicio reciente con intensidad creciente, dolor abdominal intenso o nocturno, baja de peso, masa abdominal palpable e historia personal o familiar de cáncer de colon o EII.

Aspectos esenciales   

La constipación es un síntoma el cual abarca numerosas patologías. Existen causas primarias y secundarias. Realizar el diagnóstico diferencial y el tratamiento adecuado es labor tanto del médico general como el especialista.

Clasificación etiopatogénica de la constipación crónica Dieta pobre en residuos, ingesta de alimentos que favorecen heces duras (queso, arroz, chocolate, Falla en la dieta y etc.), vida sedentaria, postración hábitos prolongada, abuso de laxantes, inestabilidad del reflejo rectal.

Caso clínico tipo Paciente de 60 años consulta por dificultad para defecar de aproximandamente 2 meses de evolución, además relata sensación de vaciamiento incompleto y dolor abdominal recurrente.

Anales: fisura, estenosis.

Enfermedades anorrectocolónicas específicas

Definición La constipación es un síntoma, no una enfermedad ni un signo y la no comprensión de ello puede llevar a una serie de problemas para el paciente y su médico (estudio y enfoque terapéutico). Como síntoma, puede ser indicativo de numerosas enfermedades y el diagnóstico diferencial abarca un amplio espectro de patologías. Tiene diferentes significados para los pacientes, según lo que consideren como un patrón defecatorio "normal". Puede implicar que las deposiciones son demasiado pequeñas, duras, difíciles de expulsar, infrecuentes o que existe una sensación de evacuación incompleta después de la defecación.

hemorroides,

Obstructivas: cáncer, vólvulos, hernia, intususcepción, endometriosis, Inflamatorias (diverticulitis, colitis isquémica, TBC, infecciones de transmisión sexual). Rectocele y prolapso rectal. Síndrome del periné descendido.

Tránsito lento idiopático, Alteraciones de la enfermedad diverticular, miopatías motilidad viscerales primarias y secundarias, megacolon y megarrecto idiopático.

Etiología-epidemiología-fisiopatología Estudios epidemiológicos revelan que entre 12 y 30% de la población presenta o ha presentado episodios de constipación, debiendo consultar a un médico. Una revisión sistemática y metaanálisis reciente, reporta una prevalencia en Sudamérica de 18%. La constipación aumenta después de los 65 años, observándose con mayor frecuencia en mujeres, raza negra, individuos sedentarios, obesos y una dieta pobre en fibra e ingesta de líquidos. Algunos estudios también señalan mayor prevalencia en poblaciones con bajo nivel socioeconómico, residencia rural y climas fríos. La diferencia por género entre hombres y mujeres (1,01 a 3,77) se equipara en los mayores de 70 años, sin aclararse el mecanismo de esta situación, sugiriendo un rol de las hormonas sexuales femeninas. Diversos estudios han encontrado que la constipación es un problema importante para el paciente respecto a su calidad de vida. Dado lo anterior, es conveniente que los médicos generales y especialistas, manejen adecuadamente esta patología.

Alteraciones psiquiátricas

Depresión, nerviosa.

psicosis,

anorexia

Farmacológicas

Opiáceos y derivados, antidepresivos, compuestos de fierro, anticolinérgicos, antiácidos, diuréticos, antiparkinsonianos, anticonvulsivantes, quimioterápicos

Neurológicas

Aganglionosis (Hirschsprung, Chagas), lesiones espinales (trauma, esclerosis múltiple, paraplejias, tumores), cerebrales (tumores, enfermedad de Parkinson)

Endocrino metabólicas

Hipotiroidismo, embarazo, diabetes mellitus, síndrome urémico, feocromocitoma, hiperparatiroidismo y otros estados hipercalcémicos

Diagnóstico Con el fin de unificar criterios, se han realizado reuniones de consenso (Roma III, 2006) en que se definió Constipación Crónica como: Aquel paciente que no utiliza laxantes y refiere 2 o más de los siguientes síntomas que hayan aparecido al menos 6 meses antes (no necesariamente consecutivos) y haber 7

estado presentes en los últimos 3 meses + Criterios insuficientes para el diagnóstico de síndrome de intestino irritable.

- Se sugiere que el paciente intente defecar en las mañanas al despertar y luego de las comidas, de modo de aprovechar el aumento normal de la motilidad colónica en dichas situaciones.

1. Evacuaciones con gran esfuerzo, presentes en más de 25% de los movimientos intestinales. 2. Deposiciones duras o caprinas en más de 25% de los movimientos intestinales. 3. Sensación de evacuación incompleta en más de 25% de los movimientos intestinales. 4. Sensación de obstrucción/bloqueo anorrectal más de 25% de los movimientos intestinales. 5. Uso de maniobras manuales para facilitar la evacuación en más de 25% de los movimientos intestinales (evacuación digital o apoyo del piso pélvico) 6. Frecuencia de defecación menor de 3 veces por semana.

Secundaria En este caso, además de las medidas generales ya mencionadas, la terapia consistirá en la corrección si es posible de la alteración específica correspondiente. Pacientes con signos de alarma, síntomas que sugieran una patología orgánica o mayores de 50 años debieran ser estudiados mediante colonoscopía.

Seguimiento Según sea la causa.

Tratamiento Medidas generales: Corregir hábitos higiénicos-dietéticos (educación): - Es importante recuperar el deseo fisiológico de la evacuación evitando inhibirlo. Sin embargo, las exigencias sociolaborales no brindan condiciones adecuadas en términos de espacio físico y psicológico para atender este tipo de necesidades. Se le debe enseñar al paciente que las deposiciones diarias no son la norma y por lo tanto se debe evitar el abuso de laxantes. - Ingesta adecuada de líquidos: Se recomienda por lo menos 2 litros diarios. El agua hidrata la deposición, facilitando su paso a través del intestino, además de ser necesaria para la excreción y actividad enzimática del intestino delgado. A pesar de lo descrito, hay pacientes que incluso ingiriendo gran cantidad de líquido diario, no logran un tránsito intestinal adecuado y requieren medidas complementarias. - Consumo de fibra: La cantidad de fibra ideal para la formación de un peso fecal de 200 g diarios con un tiempo de tránsito de 40 a 48 h, varía de un individuo a otro, pero la mayoría de la población logra este objetivo con el consumo de 20 a 35 g de fibra por día como mínimo (o 10 a 13 g por 1.000 calorías aprox.), ya sean aportadas por la dieta o medidas adicionales. Se puede combinar fibra insoluble (Ej: cereales) y solubles (Ej: frutas y verduras). En ocasiones se justifica la evaluación y educación por parte de una nutricionista, pues el paciente en muchas ocasiones desconoce la cantidad y calidad de fibra que consume diariamente. Existen evidencias que el uso de fibra soluble (Psyllium) sería más beneficioso que el uso de fibra insoluble (salvado de trigo). En los últimos años, algunos trabajos han cuestionado la real utilidad de la fibra y la ingesta de líquido, lo cual está en etapa de reestudio. - En pacientes que no responden o no toleran el uso de fibra de plantea el uso de agentes osmóticos, dentro de los cuales están la lactulosa y el polietilenglicol (PEG). - Aumentar la actividad física en el paciente sedentario (controversial). - Disminuir de peso en el paciente obeso. - Discontinuar fármacos que favorecen la constipación y evitar el abuso de laxantes en especial en el anciano.

8

3. Fármacos (colchicina, quimiotérapicos, etc).

Diarrea aguda

- Fisiopatología: La diarrea puede ser interpretada como un aumento de la cantidad de agua y electrolitos llevando a la producción de deposiciones no formadas. La pérdida del balance entre absorción y secreción en la pared intestinal lleva a un aumento de la cantidad de agua en las deposiciones.

Nivel de manejo del médigo general: Diagnóstico: Específico. Tratamiento: Completo. Seguimiento: Completo.

Hay microorganismos no invasivos, como E.coli enterotoxigénica y Vibrio cholerae, que causan diarrea acuosa sin fiebre a través de enterotoxinas que inducen secreción de fluidos, y otros que son invasivos, como Shigella, Campylobacter y Salmonella, que penetran el epitelio intestinal produciendo una reacción inflamatoria, dando origen a cuadros febriles y con mayor compromiso del estado general. Dentro de las causas no infecciosas, tenemos aquellas en que la toxina está presente en alimentos como salsas, mayonesas, o arroz (intoxicación alimentaria), que generan cuadros masivos y sin fiebre.

Aspectos esenciales.    

 

antibióticos,

Comienzo brusco y por < 2 semanas. Causa más frecuente es infecciosa Hay microrganismos invasivos, que pueden dar cuadros febriles y mayor CEG. Intoxicación alimentaria se produce horas después del consumo del alimento, se asocia a vómitos, preguntar por más personas intoxicadas. Ejemplo: mariscos bivalvos con marea roja. Mayoría es autolimitada. Tratamiento: régimen sin residuos e hidratación.

Diagnóstico. Anamnesis: Indagar sobre viajes recientes (diarrea del viajero, frecuentemente causada por E. coli enterotoxigénica), uso de antibióticos orales dentro de las últimas 4 a 8 semanas, antecedentes de consumo de alimentos de riesgo (huevos y pescados crudos, mariscos, mayonesas etc.), cuadros similares en miembros de la familia o entorno laboral. Siempre diferenciar Disenteria. Clinica: Nauseas, dolor abdominal, vomitos y fiebre, ademas de diarrea.

Caso clínico tipo. Paciente femenina de 24 años que 2 horas después de consumir un pastel con crema, comienza con vómitos, dolor abdominal difuso tipo cólico y diarrea.

Definición. Consiste en el aumento del contenido líquido de las deposiciones (o disminución en la consistencia de éstas) asociado o no a un aumento en la frecuencia defecatoria (más de 3 veces/día), por un período de tiempo menor o igual a 14 días.

Examen físico: Signos vitales, estado hemodinámico, peso (comparar con peso habitual). Obtener evidencia de depleción de volumen extracelular, por ejemplo disminución de la turgencia cutánea, hipotensión ortostática. La fiebre y signos peritoneales pueden ser indicios de infección con un patógeno intestinal invasivo.

Etiología - epidemiología – fisiopatología.

Laboratorio: De utilidad el hemograma, PCR, función renal, electrolitos plasmáticos, leucocitos fecales, lactoferrina fecal; en algunos casos el estudio de agentes patógenos o sangre en las deposiciones. Requieren estudio diagnostico en caso de disenteria, inmunodeprimidos, viajes al extranjero: Coprocultivo corriente EPSD, toxina A y B de C. difficile, antígeno en deposición para Rotavirus.

- Epidemiología: Las enfermedades diarreicas representan una de las cinco causas principales de muerte en todo el mundo. Según la Organización Mundial de la Salud (OMS) y UNICEF, hay alrededor de 2 mil millones de casos de enfermedad diarreica a nivel mundial cada año, y 1.9 millones de niños menores de 5 años de edad fallecen a causa de diarrea anualmente, fundamentalmente en los países en desarrollo. - Etiología: Las causas se pueden dividir en Infecciosas y No infecciosas.

Tratamiento. La mayoría de los casos son episodios breves y autolimitados.

Infecciosas: 1. Virales: Rotavirus, Norovirus, Adenovirus, Sapovirus. 2. Bacterianas: E.coli enterotóxigénica, Shigella, Campylobacter jejuni, Salmonella sp, Aeromonas, Vibrio parahemolyticus, C. difficile, etc. 3. Parasitarias: Giardia Lamblia, E. histolytica, Cryptosporidium, Isospora belli.

Casos leves: medidas generales, reposo domiciliario, rehidratación oral con soluciones formuladas, régimen sin residuos, sin lactosa (leche y derivados), cafeína ni alcohol. Uso racional de antiespasmódicos en caso de dolor abdominal, antipiréticos en caso de fiebre. Agentes antidiarreicos: Absorbentes (hidróxido de Aluminio). Agentes antisecretores: Racecadotrilo (inhibidor de encefalinasas que disminuye la secreción intestinal). Loperamida: uso con precaución y solo en situaciones especiales. Aumenta la absorción de agua y electrolitos, y disminuye la motilidad y secreción. Contraindicada en diarreas por gérmenes enteroinvasores.

No infecciosas: 1. Enterotoxinas de S. aureus, Bacillus cereus, C.perfringens, marea roja. 2. Escombrotoxismo (producido por peces del género escombroides, como atún y palometa)

9

Casos severos: que requieren hospitalización como deshidratación, disentería, intolerancia gástrica por vómitos, fiebre alta: agregar hidratación parenteral con soluciones salinas isotónicas, uso eventual de antieméticos (Ondasentrón o similares). Indicaciones de ATB por sospecha de infección bacteriana invasiva en: disentería, inmunocompromiso, fiebre alta más de 3 días, diarrea del viajero, sepsis.(Ciprofloxacino 500mg c/12 h por 3 a 5 días).En paciente pediatrico es preferible esperar coprocultivo : ATB en caso de Shigella (cefalosporina),

Seguimiento. Evaluación del cese de diarrea y síntomas y signos asociados; en los casos más graves además, evaluar recuperación hemodinámica, orgánica y estado nutricional.

10



Diarrea Crónica



Nivel de manejo del médico general: Diagnóstico: Sospecha. Tratamiento: Inicial. Seguimiento: Derivar.

 



Aspectos esenciales. 

   

El primer paso es evaluar si el paciente efectivamente sufre de diarrea crónica. Puede ser una patología única, o como parte de una enfermedad sistémica. El tratamiento va a depender de la enfermedad de base, y en algunos casos, con medicamentos sintomáticos. Se debe derivar a especialista a todo paciente con diarrea crónica.

     

Infecciones bacterianas crónicas/infecciones por hongos/infecciones por parásitos. Infecciones oportunistas en inmunodeprimidos (Cryptosporidium, Isospora belli, Mycobacterium avium). Enteritis o colitis por radiación. Cirugía previa (gastrectomía, gastroyeyunostomía, vagotomía, colecistectomía, resección intestinal). Insuficiencia pancreática (pancreatitis crónica, pospancreatitis aguda grave, neoplasia de páncreas, fibrosis quística). Causas endocrinas (hipertiroidismo, hipotiroidismo, hipoparatiroidismo, insuficiencia suprarenal, diabetes mellitus). Tumores neuroendocrinos (gastrinoma, VIPoma, carcinoma medular de tiroides, ganglioneuroma). Mastocitosis sistémica. Trastornos infiltrativos intestinales (amiloidosis, esclerodermia). Impactación fecal, incontinencia anal. Alergia alimentaria (proteínas de la leche, de la soja). Diarrea crónica idiopática.

Caso clínico tipo. Paciente de 17 años consulta por diarrea de 2 meses de evolución aproximadamente 10 episodios todos los días, aumento de volumen de extremidades inferiores y baja de peso (15 kg en los últimos 2 meses).

Categorización Acuosa: Cuando nos encontremos con una: Diarrea osmótica: Cede con el ayuno, alimento sospechoso (hidratos de carbono), volumen <1 Lt, sin productos patológicos, hipernatremia, pH fecal < 6, con brecha osmótica > 125

Definición. Se entiende por diarrea crónica al cambio en el hábito defecatorio, ya sea por aumento de frecuencia (3 deposiciones o más al día), menor consistencia o mayor peso (> de 200 g/día), debido al aumento en el contenido de agua. Cuadro que debe prolongarse por 30 días o más.

Diarrea secretora: Persiste con ayuno, volumen > 1 Lt, sin productos patológicos, hipokalemia, alcalosis metabólica por pérdida de K y bicarbonato. pH > 6 (por pérdida de bicarbonato fecal), brecha osmótica < 50.

Plantear el diagnóstico diferencial con la incontinencia fecal, la seudoobstrucción o impactación fecal con diarrea por rebalse.

Grasa: Cuando la clínica y el laboratorio orientan a: Síndrome malabsortivo: Deposiciones amarillentas, oleosas, olor rancio y flotan. Grasa fecal (+), laboratorio carencial, CEG.

Etiología-epidemiología-fisiopatología. Maldigestión: Insuficiencia pancreática o biliar Etiología diarrea Crónica.  Diarrea funcional/síndrome de intestino irritable.  Enfermedad inflamatoria intestinal (colitis ulcerosa, enfermedad de Crohn).  Colitis microscópica (colitis colágena, colitis linfocítica).  Enfermedad celíaca.  Enfermedad de Whipple.  Isquemia intestinal.  Síndromes de malabsorción de hidratos de carbono por déficit de lactasa, hidratos de carbono poco absorbibles de la dieta (sorbitol, fructosa, fibra, fécula, lactulosa).  Malabsorción de ácidos biliares (idiopática, obstrucción vía biliar, alteración íleon terminal).  Diverticulosis intestino delgado.  Neoplasias (adenoma velloso, adenocarcinoma de colon, linfoma intestinal).  Fármacos (antibióticos, hipotensores, antiarrítmicos, antineoplásicos, antiácidos, antidiabéticos, etc.).  Abuso crónico de laxantes.

Inflamatoria: Enfermedad inflamatoria intestinal, enfermedad infecciosa, colitis isquémica, colitis actínica, neoplasia. Alteraciones de la motilidad: Son diagnóstico de exclusión, se asocian a enfermedades sistémicas (diabetes, hipo e hipertiroidismo), esclerodermia, postcirugía, sobrecrecimiento bacteriano.

Diagnóstico. Dentro del proceso diagnóstico resulta fundamental realizar una anamnesis detallada e investigar síntomas de alarma. Según las características de la diarrea se pueden clasificar como: • Intestino delgado: son voluminosas (> 1 litro/día), con dolor periumbilical o en fosa ilíaca derecha, siendo poco frecuente la existencia de pus, sangre, pujo y tenesmo (síndrome rectal).

11

• Colon: son de poco volumen (< de 1 litro), se asocian a pujo, tenesmo, urgencia rectal, pus, sangre, dolor en hipogastrio o fosa ilíaca izquierda.

y electrolitos, sin afectar la secreción basal. Existe mayor experiencia en diarrea aguda y debe evaluarse en diarrea crónica acuosa. 11. Probióticos y prebióticos: mayor experiencia en diarrea aguda. La evidencia apunta hacia el manejo de la diarrea crónica de origen infeccioso, EII, diarrea asociada a antibiótico e intestino irritable.

Signos de alarma que orientan a causa orgánica: Dolor abdominal nocturno, baja de peso, fiebre, sangre y pus en deposiciones, aparición de masas abdominales, síndrome rectal. Datos que sugieren origen funcional: Dolor abdominal diurno, alternancia de diarrea-constipación, buen estado general, sin baja de peso.

Seguimiento Se sugiere derivar al gastroenterólogo en caso de:      

Tratamiento Podemos plantear tratamiento: Específico: Según etiología.     

Cirugía en tumores de colon, ID o páncreas. Antibióticos en la enfermedad de Whipple, SIBO o SBI. Dieta sin gluten en celíacos. 5-ASA (salicilatos) y esteroides en EII. Antiparasitarios.

Sintomático: Indicado: a. Cuando no hemos logrado llegar a una etiología a pesar de las exploraciones realizadas. b. Cuando no existe tratamiento específico de la causa. c. Pobre respuesta a pesar de tratamiento específico. Objetivo: a. Aumentar consistencia de deposiciones. b. Disminuir frecuencia de deposiciones. c. Aliviar síntomas asociados. Opciones terapéuticas 1. Sustancias absorbentes: salvado de trigo, mucílago, plántago, metilcelulosa, coloides hidrófilos que solidifican las heces. 2. Derivados opiáceos: loperamida, codeína, difenoxilato. Ej.: loperamida 2 mg c/4-8 h oral (no sobrepasar 16 mg/día). 3. Análogos de somatostatina: octeotride 50 umg c/12 h subcutánea y aumentar hasta control de síntomas (Existe octeotride liberación lenta sc). Carcinoide, vipoma, glucagonomas, gastrónomas, enteropatía por SIDA. Intestino corto. 4. Colestiramina: 4-12 g/día/oral. Síndrome postvagotomía, postcolecistectomía, resección íleon terminal. 5. Clonidina: 0,1-0,6 mg c/12 h/oral en diarrea del diabético. 6. Inhibidores bomba de protones en diarrea por Zollinger Ellison. 7. Indometacina: 25 mg c/12 h/oral: inhibidor de prostaglandinas. Útil en diarrea secundaria al carcinoma medular tiroideo y adenomas vellosos. 8. Antagonista H1-H2: mastocitosis sistémica: ranitidina 600 mg/día. 9. Enzimas pancreáticos de reemplazo (10.000-25.000 U/oral). 10. Racecadotrilo: inhibidor selectivo, reversible de la encefalinasa, reduce la hipersecreción intestinal de agua 12

Diarrea con sangre y cultivos negativos Diarrea persistente con el ayuno (secretora) Presencia de esteatorrea Sospecha de enfermedad inflamatoria intestinal Falla de respuesta al tratamiento antidiarreico Pacientes inmunosuprimidos con DC

cáncer de esófago (asociado a alcohol y tabaco); estenosis pépticas y las producidas por sustancias alcalinas. Puede también estar causada por una compresión extrínseca (abscesos faríngeos, adenopatías cervicales, neoplasias mediastínicas, mediastinitis, aneurisma aórtico, etc). Disfagias motoras: Debido a incoordinación, debilidad de las contracciones peristálticas o alteración de la relajación del esfínter esofágico inferior. Se subclasifican en: alteraciones de músculo estríado (parálisis faringea, la acalasia cricofaríngea y el globus) y alteraciones de músculo liso (esclerosis sistémica progresiva, espasmo esofágico, acalasia (Rx: ausencia de la burbuja aérea gástrica y una estenosis concéntrica en la unión gastroesofágica, con una dilatación y elongación variable del esófago proximal)).

Disfagia Nivel de manejo del médico general: Diagnóstico Sospecha Tratamiento Inicial Seguimiento Derivar.

Aspectos esenciales.   

Síntoma inespecífico que puede corresponder a un conjunto diverso de patologías. Se clasifican en mecánicas y motoras. Derivar oportunamente y atención a signos de alarma.

Diagnóstico. 

Realizar una buena historia clínica y examen físico de cavidad oral y cuello. A través de la anamnesis es posible distinguir la ubicación de la disfagia en 80% de los casos.  Rx esófago-estómago-duodeno, Pruebas de motilidad (manometría esofágica, estudio dinámico de deglución): Disfagia funcional.  Endoscopía digestiva alta: Generalmente es el mejor examen de costo beneficio para determinar la causa de la disfagia baja. Adicionalmente, permite tomar biopsias o proceder a maniobras terapéuticas inmediatas (dilatacion o instalación de SNG) para reestablecer la alimentación enteral, si fuese necesario. Importante descartar signos de alarma (cáncer esofágico):  Disfagia progresiva, primero a sólidos y luego a alimentos de consistencia paulatinamente más blanda, que progresa a lo largo de algunas semanas o meses  Pérdida de peso  Dolor retroesternal  Hemorragia y la aspiración  Adenopatías  Hepatomegalia.

Caso Clínico Tipo. Paciente de 60 años, fumador, bebedor de alcohol crónico, con historia de 4 meses de dificultad para tragar sólidos, que ha ido progresando en el tiempo hasta tener molestias con líquidos actualmente. Al examen, enflaquecido, con dificultad para tragar saliva, sin otras alteraciones. Se solicita Rx tórax, EDA y derivación inmediata a especialista.

Definición. Dificultad o malestar asociado al paso de los alimentos desde la boca al estómago, ya sea por un compromiso funcional de alguna de las fases de la deglución o por una lesión estructural de alguno de los órganos involucrados en la deglución. Puede ser clasificada según su curso y evolución; la disfagia lógica es aquella que dificulta inicialmente el paso de los sólidos y evoluciona posteriormente comprometiendo el paso de los líquidos, y la disfagia ilógica no respeta este orden en su curso natural (y suele ser intermitente)

*En los tumores benignos el diagnóstico es endoscópico, mostrando estructuras redondeadas, con una mucosa que las reviste habitualmente intacta.

Epidemiología Se desconoce la prevalencia en Chile. A partir de estudios poblacionales se ha estimado que su prevalencia en población occidental es un 6-11%. Ésta aumenta con la edad, se estima que afecta al 13% de la población mayor de 65 años de edad y afectaría hasta el 60% de pacientes institucionalizados. Por otra parte se ha descrito que la presencia de disfagia orofaríngea asociada a aspiración se asocia a una mortalidad de un 45% a un año.

Tratamiento. La indicación quirúrgica de los tumores benignos es principalmente para aquellos con dolor intenso, disfagia, hemorragia, compresión de alguna estructura vital o aumento progresivo de tamaño. Para la Acalasia: bloqueadores de los canales del calcio (nifedipino, diltiazem), los nitratos de efecto prolongado y los agonistas de los receptores alfa adrenérgicos. Dilatación con balón, cirugía, inyección de toxina botulínica. Derivar en caso de signos de alarma.

Etiología – Fisiopatología Se pueden clasificar según su etiopatogenia en 2 grupos: Disfagias mecánicas: Debido a una estrechez absoluta (extrínseca o parietal) o relativa del lúmen, presentándose una obstrucción mecánica al paso del bolo alimenticio en su camino hacia el estómago. Se encuentran las estomatitis, faringitis y esofagitis virales o por hongos; divertículo de Zenker (> 60 años, se manifiesta además con compromiso del estado general); tumores benignos;

Seguimiento. Derivar a especialista en caso de persistencia de patología o signos de alarma.

13

Dispepsia

   

Nivel de manejo del médico general: Diagnóstico Específico Tratamiento Completo Seguimiento Completo.

  

Aspectos Esenciales.    

Conjunto de síntomas gastrointestinales y extraintestinales inespecíficos que pueden estar relacionados a diferentes patologías. Localización del malestar en zona epigástrica asociado o no a diferentes síntomas. Diagnóstico operacional: No permite precisar etiología, pero sí un plan de estudio. Las consideraciones de estudio y manejo van asociadas al riesgo de cáncer gástrico en la población.

Trastornos motores del tubo digestivo Sd. de intestino irritable. Enfermedad Celíaca. Insuficiencias orgánicas (renal, respiratoria, cardíaca). Drogas -> AINEs. Somatización. Idiopática (Dispepsia Funcional) -> diagnóstico de exclusión: "Persistencia de malestar epigástrico al menos 12 semanas en los últimos 12 meses, sin evidencias de condiciones estructurales que expliquen los síntomas, incluyendo una EDA normal y test de ureasa negativo, sin alteración del tránsito ni alivio con la defecación". La Dispepsia Funcional tiene una prevalencia de 25% y su fisiopatología está dada por hipersensibilidad intestinal y trastornos gástricos de vaciamiento y acomodación.

Es importante descartar signos de alarma (cáncer gástrico): 1. Inicio dispepsia > 40-55 años. 2. Evidencia de sangrado digestivo 3. Anemia. 4. Saciedad precoz. 5. Pérdida de peso inexplicable. 6. Disfagia u odinofagia persistente. 7. Vómitos persistentes. 8. Historia familiar de cáncer digestivo. 9. Úlcera péptica documentada previamente. 10. Masa abdominal o linfoadenopatías. 11. Dolor nocturno 12. Uso de AINEs

Caso Clínico Tipo. Paciente de 50 años asiste a consultorio por historia de larga data de malestar epigástrico.

Definición. Es un síndrome, caracterizado por dolor o malestar crónico o recurrente en zona epigástrica que puede o no tener síntomas asociados (ardor, flatulencias, náuseas, saciedad precoz, etc.) que frecuentemente empeora en período postprandial. Es un síndrome frecuente que comprende múltiples etiologías.

Tratamiento. Los pacientes con síntomas predominantes de reflujo, deben ser tratados como tales. Evaluar uso crónico de AINEs en pacientes usuarios de ellos. En general, se usa inhibidor de bomba de protones para los malestares dispépticos del tipo úlcera o reflujo; así como proquinéticos, en el caso de síntomas de dismotilidad, luego de descartar patología orgánica.

Etiología-epidemiología-fisiopatología. Dentro de las causas de dispepsia, se encuentran patologías relacionadas al sistema gastrointestinal como extraintestinales. Es de especial relevancia el diagnóstico de Dispepsia funcional, cuadro debido a diferentes causas (hipomotilidad antral, vaciamiento gástrico retardado, relajación fúndica alterada, estrés psicológico, sensibilidad al ácido, disritmias gástricas, gastritis por H. pylori, entre otras).

Tratamiento de erradicación del Helicobacter pylori (Amoxicilina, Claritromicina e Inhibidor de Bomba de Protones) por 14 días, por encontrarse mayor prevalencia de la bacteria en poblaciones con mayor mortalidad por cáncer gástrico.

Diagnóstico. Clínico y Exámenes complementarios: Es de importancia una adecuada anamnesis y un dirigido examen físico para orientar el diagnóstico. Una Endoscopía digestiva alta es útil para descartar causa orgánica en casos de signos de alarma, en especial a mayores de 40 años. El test de H. pylori es útil para ver existencia de la bacteria que podría estar ocasionando síntomas dispépticos.

Se debe aconsejar sobre un estilo de vida saludable (cese de hábito tabáquico, disminuir la ingesta de alcohol, evitar el sobrepeso), que pueden ayudar a disminuir la sintomatología.

El examen físico es por lo general normal o con sensibilidad epigástrica como hallazgo único. El hallazgo de masa abdominal palpable debiese orientar al clínico en cáncer gástrico u otro tipo de diagnósticos.

En el caso de la DF se recomienda tratar por 3-4 semanas con IBP, independiente de la presentación clínica. En caso que la sintomatología no mejore, se debe continuar con IBP + proquinéticos (con esto se espera que la sintomatología del 80% del total de pacientes mejore). Habiendo cumplido estos ciclos, en caso de no mejorar la sintomatología, los pacientes deben ser derivados a reevaluación por un Gastroenterólogo.

Diagnósticos a descartar:

Seguimiento.

   

Cáncer gástrico. Enfermedad por Reflujo gastroesofágico (ERGE) Úlcera péptica. Pancreatitis crónica.

Control luego del tratamiento para erradicación de bacteria y seguimiento en pacientes con úlceras gástricas y en persistencia de síntomas.

14

La acción sinérgica del amoníaco con otras toxinas, explicarían muchas anormalidades que se presentan en la insuficiencia hepática, tales como los cambios en el transporte de sangre de los precursores de neurotransmisores al cerebro, el metabolismo de neurotransmisores de aminoácidos, y la oxidación de la glucosa cerebral. Estos cambios pueden conducir a la activación de inhibidores (GABA, serotonina) y deterioro de la excitación (glutamato, catecolaminas), lo que resulta en la inhibición neural. Sepsis, neuroinflamación, y alteraciones en la flora intestinal parecen ser factores adicionales en el desarrollo de la disfunción cerebral en la enfermedad hepática avanzada.

Encefalopatía hepática (EH) Nivel de manejo del médico general: Diagnóstico: Específico. Tratamiento: Inicial. Seguimiento: Derivar.

Aspectos esenciales.    

La EH es básicamente un trastorno funcional del SNC, por lo tanto no existen alteraciones morfológicas.

Trastorno funcional y reversible, excepto la EH crónica. El diagnostico es clínico. El pilar central del tratamiento es controlar el factor precipitante. Tratamiento consiste en reducción de la producción de amoníaco y la absorción con medicamentos como la lactulosa , lactitol, o rifaximina

Diagnóstico: El diagnóstico es básicamente Clínico: - EH aguda o subaguda: aparece en forma brusca o en el curso de una semana. - EH aguda o subaguda recurrente: episodios ocasionales, normalidad entre intervalos.

Caso clínico tipo.

- EH recidivante: episodios sucesivos en meses o años, pueden ser reversibles.

Paciente hombre de 56 años, OH crónico, en situación de calle. Es traído a urgencias por presentar convulsiones en vía pública. A examen físico esta consciente, bradipsíquico, presenta además asterixis y rigidez muscular en rueda dentada. ¿Qué indicaciones terapéuticas dejaría?

- EH crónica: evolución oscilante, es permanente, progresiva e irreversible. - EH mínima: sólo detectable mediante exploraciones complementarias.

Definición.

Signos: Asterixis, disfunción neuromuscular, hiperreflexia, convulsiones, posturas de descerebración, hiperventilación, fetor hepático, paraparesia espástica. Pruebas complementarias: ayudan pero no hacen el diagnóstico: EEG; pruebas de psicometría; determinación de amonio y glutamina en LCR y sangre.

Corresponde a un síndrome neuropsiquiátrico complejo y potencialmente reversible, que se presenta en pacientes con disfunción hepática aguda y crónica.

Etiología - Epidemiología - Fisiopatología. Tratamiento.

- Epidemiología: Manifestación clínica más aparente de la insuficiencia hepática aguda grave (IHAG) y también se presenta en pacientes con cirrosis hepática avanzada. Mortalidad de 50% a los 6 meses en IHAG y en encefalopatía aguda del cirrótico. La encefalopatía hepática crónica es la de mejor pronóstico.

El tratamiento de soporte general para los pacientes con encefalopatía hepática incluye evitar la deshidratación y alteraciones electrolíticas, la prestación de apoyo nutricional, y proporcionar un ambiente seguro. Los pacientes no deben restringir su ingesta de proteínas. Los pacientes deben ser instruidos para comer comidas pequeñas durante el día con un aperitivo por la noche de hidratos de carbono complejos. Precauciones para evitar las caídas se deben instituir para los pacientes que están desorientados

- Etiología: Factores predisponentes: insuficiencia hepatocelular y presencia de colaterales portosistémicas. Factores determinantes: probablemente amoníaco; en duda el desequilibrio por descenso de aminoácidos ramificados e incremento de aminoácidos aromáticos.

Identificación del factor precipitante (hemorragia digestiva, infección, PBE, hipoglicemia, constipación, insuficiencia renal, hipokalemia, transgresión dietética, sedantes, etc).

Fisiopatología: En general se produce como consecuencia de la incapacidad del hígado para realizar su acción detoxificadora de sustancias del organismo con actividad cerebral. Probable origen de sustancias sería intestinal, y su paso sería a través de la alteración de la función hepatocelular y la presencia de colaterales portosistémicas, lo que permite que la sangre llegue a nivel cerebral, sin haber sido detoxificada en el hígado. Probablemente el amoníaco sea la sustancia más importante en la patogenia de este síndrome.

Lavado intestinal y administración de fármacos que disminuyan la flora colónica:  

15

1ra línea: Lactulosa: Dosis de 30 cc/6 horas; el objetivo es lograr 2-3 deposiciones pastosas al día, si no es posible vía oral, dar enemas de 1 litro. 2da línea: Rifaximina: 400 mg cada 8 horas, más caro pero mejor tolerado que neomicina o metronidazol. Siempre asociar con Lactulosa.

La neomicina se ha utilizado como un tratamiento de segunda línea en pacientes que no han respondido a los disacáridos, pero no se ha demostrado ser eficaz en ensayos aleatorios y está asociado con nefrotoxicidad y ototoxicidad. Nos reservamos neomicina para pacientes que no pueden tomar rifaximina . La ornitina-aspartato estimula el metabolismo de amoniaco, es una alternativa para el tratamiento de la encefalopatía hepática en países en los que está disponible.

Seguimiento. Derivar a especialista.

Grados de Encefalopatía Hepática:

16

Enfermedad Celíaca

15%), diabetes tipo 1 (3-16%), tiroiditis de Hashimoto (5%), síndrome de Turner (3%), síndrome de Down (5%) y otras enfermedades autoinmunes como hepatitis autoinmune, Sjögren y nefropatía por IgA.

Nivel de manejo del médico general: Diagnóstico Específico. Tratamiento Completo. Seguimiento Completo.

Patogenia: La EC resulta de la interacción entre el gluten y factores genéticos del individuo (fundamentalmente del área inmunológica) y ambientales. El gluten está compuesto por dos proteínas: gliadina y glutenina, que contienen altas concentraciones de prolinas y glutaminas, lo que las hace resistentes a la degradación del ácido gástrico, jugo pancreático y enzimas del borde en cepillo intestinal. El transporte de estos péptidos a través del epitelio del intestino delgado se efectuaría de acuerdo con las siguientes teorías:

Aspectos esenciales.   

La enfermedad celíaca es un trastorno sistémico cuyos síntomas pueden ser gastrointestinales o extraintestinales. El diagnóstico se realiza en base a sospecha clínica y se confirma con exámenes serológicos e histológicos. La contextura corporal del paciente NO constituye un factor importante para la sospecha de Enfermedad Celíaca, la enfermedad se puede presentar en pacientes obesos cómo delgados.

a) Cambios inducidos en las “tight junction”. b) Transporte de los péptidos por transcitosis epitelial. c) Unión de los péptidos a células dendríticas de la lámina propia a través de las células epiteliales. Factores Genéticos: La genética de la EC es compleja, ya que están implicados en la predisposición a la enfermedad al menos 39 genes no HLA, cuyo rol está en estudio, además de los genes de histocompatibilidad mayor HLA-DQ2 y DQ8, que están presentes en sobre el 98% de los casos. Clínicamente la influencia genética en la EC está reflejada por su mayor incidencia en familiares de 1° y 2° grado que en la población general.

Caso clínico tipo. Paciente de 35 años consulta por dolor abdominal recurrente y distensión abdominal de 1 años de evolución. Destaca aparición de síntomas al comer alimentos que contienen gluten como trigo, cebada y centeno.

Factores Ambientales: El factor ambiental más importante es el gluten, ampliamente distribuido en un gran número de alimentos y productos alimenticios. Se ha observado que la introducción precoz de gluten, antes de los 4 meses de edad, se asocia a un mayor riesgo de desarrollo de la EC en los niños susceptibles versus los que lo reciben después de los 7 meses. En niños celíacos se ha detectado mayor frecuencia de infección por Rotavirus que en los controles, su real importancia está por aclararse.

Definición. La enfermedad celíaca (EC), también denominada sprue celíaco o enteropatía por gluten, es un trastorno sistémico mediado por el sistema inmune y gatillado por la ingestión de proteínas del gluten (trigo, cebada y centeno) en individuos genéticamente susceptibles. El compromiso del intestino es variable, desde mínimas lesiones hasta una atrofia total de vellosidades, las que revierten con una dieta libre de gluten.

Diagnóstico.

La atrofia de la mucosa intestinal causa una malabsorción intestinal de micro y macronutrientes, la cual lleva a manifestaciones clínicas que incluyen diarrea crónica, esteatorrea y déficit nutricionales y vitamínicos. Esta condición mejora cuando se suprimen de la dieta los cereales que contienen gluten, y recurre cuando son reintroducidos nuevamente.

El diagnóstico de la EC se basa en la sospecha clínica y la certeza de los estudios serológico e histológico de la mucosa duodenal. Es fundamental que el paciente debe estar ingiriendo una dieta con gluten, en el momento del estudio serológico e histológico, para evitar así los falsos negativos. Clínica.

Etiología-epidemiología-fisiopatología.

La EC en la población infantil tiene variadas formas de presentación, dependiendo de la edad de inicio y duración de la enfermedad. Los niños menores de 2 años, presentan con mayor frecuencia diarrea, distensión abdominal y diversos grados de compromiso de la ganancia de peso y talla. En los niños mayores y adolescentes se puede observar talla baja, anemia, pubertad retardada, aftas recurrentes, hipoplasia del esmalte dental; síntomas gastrointestinales como dolor abdominal recurrente, vómitos y constipación. Además, pueden presentar manifestaciones extraintestinales como artritis o síntomas neurológicos. Los estudios demuestran una disminución de la prevalencia de diarrea como forma de presentación predominante, observándose un aumento de las manifestaciones llamadas atípicas de la enfermedad. Por otra parte, existen condiciones

Etiología y epidemiología: La EC afecta entre 0,6 y 1% de la población mundial. Existe una dispersión muy amplia de prevalencias publicadas y ello se debe a las distintas metodologías empleadas, el estudio de laboratorio practicado y las características de la población estudiada. En América, la frecuencia reportada en EEUU es de 0,75%; México 0,7 a 2,7%; Brasil 0,15 a 0,35% y Argentina de 0,6%. En nuestro país la prevalencia es de 0,3% según la Encuesta Nacional de Salud 2009-2010 (Ministerio de Salud y U. Católica). Se ha reportado una mayor frecuencia en mujeres (2:1). Los estudios serológicos han demostrado que la prevalencia es mayor en personas con factores de riesgo tales como tener un familiar de primer grado celíaco (1017

asociadas como la diabetes tipo I, la cual generalmente precede la EC, además los niños y adolescentes con enfermedades hepáticas autoinmunes tienen una mayor prevalencia de EC.

por los grupos de pacientes y por un equipo médico comprometido. Junto con la dieta sin gluten, en algunos casos se requiere supresión temporal de lactosa; además al paciente se le deben indicar los suplementos que correspondan según sea el caso: hierro, calcio, vitamina D, multivitamínicos.

El espectro clínico es muy amplio, desde sólo alteraciones en el laboratorio en personas asintomáticas (anemia crónica, osteoporosis en mujeres jóvenes, etc.) hasta un cuadro de diarrea crónica severa con baja importante de peso y desnutrición. Esquemáticmente se pueden clasificar los signos y síntomas según sean digestivos o extradigestivos. Dentro de los digestivos el cuadro más florido, antes mencionado (diarrea crónica, desnutrición, signos carenciales), era antiguamente el más frecuente y actualmente es muy raro de observar. Los pacientes en la actualidad, presentan uno más de los siguientes síntomas: pirosis, náuseas, meteorismo, dolor abdominal recurrente, diarrea intermitente de larga data, diarrea crónica (de larga data o de pocos meses de evolución), constipación (5% de nuestros casos) y baja de peso.

Seguimiento. Una vez diagnosticado el paciente debe controlarse con serología a los seis meses de llevar dieta sin gluten; para verificar su cumplimiento, no es necesario practicar endoscopia con biopsias, pues la mejoría de la mucosa puede demorar hasta un año, en cambio los anticuerpos son más sensibles. Una vez el paciente está “estabilizado” (con serología negativa), se recomienda que se controle con su médico tratante al menos una vez al año.

Los síntomas extradigestivos pueden o no acompañar a los anteriores.

Se debe solicitar el laboratorio básico: hemograma, perfil bioquímico, hepático, tiroides, etc, y anticuerpos antiendomisio y antitransglutaminasa. Si el estudio serológico es negativo debe controlarse al año siguiente, si es positivo significa que existe trasgresión alimentaria voluntaria o involuntaria, la que debe estudiarse y ser corregida.

Estudio serológico En la actualidad, las tres pruebas más utilizadas determinan anticuerpos IgA antiendomisio (AAE), IgA antitransglutaminasa (AAT) e IgA antigliadina deamidada (AGD). Los AAE tienen sensibilidad mayor de 90% y especificidad de 98% Los AAT sensibilidad aproximadamente.

y

especificidad

de

95%

Los AGD tienen sensibilidad entre 91 y 98% y especificidad menor de 95%. Se sugiere solicitar dos de estos anticuerpos más IgA sérica, pues en casos de déficit, éstos no marcarán. La sensibilidad es cercana al 100% al sumar dos técnicas serológicas. Estos 3 métodos se deben solicitar midiendo IgG en caso de pacientes con déficit de IgA. A pesar de todas estas pruebas, existe cerca del 5% de celíacos activos en que la serología será negativa y en ellos debe practicarse endoscopia y biopsias si la clínica lo amerita. Estudio histológico El paciente sospechoso debe ser sometido a endoscopia y toma de 4 o más biopsias de la segunda porción de duodeno y del bulbo y en forma escalonada. La confirmación del diagnóstico se basa en el hallazgo de atrofia de la mucosa duodenal. Ampliamente difundida es la clasificación de Marsh-Oberhuber para las biopsias duodenales.

Tratamiento. El tratamiento consiste en consumir una dieta libre de gluten de por vida. De esta manera se deben abandonar los productos derivados del trigo, cebada, centeno, además de alimentos que contengan preservantes o aditamentos que puedan contener trazas de estos antígenos. Existen estudios que han demostrado que el mejor paciente celíaco es el apoyado fuertemente por su familia, 18

Enfermedad diverticular complicada

- Fístula: Más frecuentes en hombres (2:1), pacientes con cirugía abdominal previa y en inmunocomprometidos. La más común es la Colovesical (65-75%). Esta se presenta con neumaturia, fecaluria, hematuria, infecciones urinarias a repetición, disuria y urgencia miccional. La segunda en frecuencia es la fístula colovaginal (25%).

Nivel de manejo del médico general: Diagnóstico Sospecha Tratamiento Inicial Seguimiento Derivar.

- Obstrucción: Cuadro de constipación, distensión abdominal y deposiciones acintadas. Es una complicación tardía. - Perforación: Genera peritonitis purulenta o fecaloídea. Se caracteriza por un cuadro de irritación peritoneal con resistencia abdominal, dolor de inicio súbito e hipotensión. Poco frecuente, se presenta con mayor frecuencia en inmunocomprometidos. Requiere intervención quirúrgica de urgencia.

Aspectos Esenciales.   

La principal complicación es la diverticulítis y sus complicaciones asociadas. El diagnóstico es clínico y se complementa con imágenes, siendo el TAC el método de elección. El tratamiento médico puede ser ambulatorio u hospitalario y dependerá de las condiciones del paciente y tipo de complicación.

Hemorragia Diverticular: El sangrado suele ser abrupto, indoloro y de gran volumen, siendo masivo y requiriendo transfusión de emergencia en un 33%. Se detiene espontáneamente 70-80% de las veces. El examen físico suele no presentar alteraciones. Es de mayor riesgo en ancianos, HTA, enfermedad cardiovascular y uso de AINEs. Existe una mayor incidencia de hemorragia en el lado derecho (50-90%).

Definición. La enfermedad diverticular complicada se presenta en 1525% de los pacientes con presencia de divertículos colónicos. Esta entidad se divide en diverticulitis (simple 75% - o complicada - 25% -) y hemorragia diverticular.

IMÁGENES: 1. TC Abdomen y Pelvis: Es el método de elección para el diagnóstico de diverticulitis y sus complicaciones. Permite ver presencia de divertículos y aumento en la densidad de tejidos blandos adyacentes. Permite la clasificación de la enfermedad.

La diverticulitis es una inflamación de uno o más divertículos, con manifestaciones clínicas variables, que van desde la inflamación subclínica a peritonitis generalizada. Fisiopatológicamente corresponde la perforación micro o macroscópica de un divertículo.

Clasificación de Hinchey

Existen diversas teorías de su etiología:

I: Absceso pericólico o mesentérico pequeño localizado

a. Erosión de pared diverticular por aumento de presión intraluminal o partículas espesadas de alimentos, generándose inflamación y necrosis focal. b. Obstrucción luminal por fecalito en divertículo, generando proliferación de flora bacteriana, disminución de flujo sanguíneo y produciendo necrosis y finalmente perforación. Este es un concepto antiguo.

II: Absceso de mauor tamaño, a menudo en la pelvis

La hemorragia diverticular se presenta en el 30% de los pacientes con diverticulosis, siendo la causa más frecuente de hemorragia digestiva de origen colónico. Es de origen arterial en la vasa recta del cuello del divertículo, por lo que compromete rápidamente la hemodinamia del paciente. No coexiste con episodios de diverticulitis.

2. Enema Baritado: Permite observar el lumen y presencia de fístulas. No debería usarse en la fase aguda de la enfermedad por riesgo de reactivar o complicar la diverticulitis.

III: Diverticulitis perforada, causando peritonitis purulenta generalizada IV: Ruptura de divertículos en la cavidad peritoneal con contaminación fecal, causando peritonitis estercorácea.

3. Ecografía: Buena alternativa si TC no disponible. Muestra divertículos, absceso mural o peridiverticular. En centros de experiencia se ha reportado S 85-98% y E 8098%

Diagnóstico. Diverticulitis: Sospecha clínica y confirmación de laboratorio e imagenológica. Presencia de dolor en el cuadrante inferior izquierdo asociado a fiebre, leucocitosis, náuseas y vómitos, distensión abdominal, cambios de hábito intestinal y síntomas urinarios por irritación vesical. Importante es que adultos mayores y pacientes que reciben corticoides pueden tener pocos hallazgos al examen físico (alto índice de sospecha). Imágen de eleccción es el TAC (S 97%, E 100%).

El uso de colonoscopía esta CONTRAINDICADO en episodio agudo, por aumentar posibilidades de perforación. Después de la resolución de la diverticulitis aguda, se puede realizar de forma electiva para evaluar todo el colon (3 meses después).

Tratamiento. Médico: En presencia de un primer episodio de diverticulitis no complicada. Puede ser ambulatorio u hospitalario.

Diverticulitis complicada: - Absceso: Sospechar cuando persiste con fiebre alta, dolor y CEG a pesar del tratamiento antibiótico. 19

1. Ambulatorio: Paciente con dolor leve, sin síntomas sistémicos, con buena ingesta y sin comorbilidades.

Seguimiento. Por especialista.

Indicaciones:  

Régimen líquido, luego dieta baja en residuos según tolerancia. ATB: se debe cubrir Gram (-) y anaerobios. Alternativas son Metronidazol 500 mg c/8 hrs + Quinolona (Ciprofloxacino 500mg c/12 hrs) ó Amoxi-Clavulánico. Por 7-10 días. Se debería observar mejoría en 48-72 hrs después del inicio del tratamiento.

2. Hospitalización: Se indica cuando existe evidencia de: diverticulitis complicada, mala tolerancia oral, dolor severo que requiera analgesia EV, falla del tratamiento ambulatorio, o pacientes de edad muy avanzada, inmunocomprometidos o con comorbilidades.   

  

Reposo intestinal, hidratación IV Analgesia: Opioides (AINEs y corticoides se han asociado con mayor riesgo de perforación de colon) ATB EV: se recomienda Metronidazol asociado a Cefalosporina 3º (ceftriaxona 1-2g al día) ó fluorquinolona (ciprofloxacino 400 mg EV c/12 o Levo 500 mg c/12h). Si mejoría en 48h, iniciar dieta baja en residuos y paso a ATB VO si 24-48h sin fiebre y disminución de leucocitos. Mantener ATB VO por 10-14 días. Se espera respuesta a los 3 días, en caso contrario sospechar complicación (flegmón o colección) u otro diagnóstico. Al alta se recomienda mantener dieta alta en fibra (no existe evidencia de beneficio de suspensión de alimentos específicos) y realizar colonoscopía para descartar neoplasia colónica, isquemia y enfermedad inflamatoria intestinal.

* Se mantiene en estudio el rol terapéutico de Mesalazina y Rifaximina. Drenaje percutáneo: Indicado en abscesos ≥ 4 cm de diámetro (además del tratamiento antibiótico). Permite una reducción del dolor, fiebre y leucocitosis, y la realización de una cirugía electiva en lugar de una cirugía de urgencia en 60-80% de los pacientes. Sin embargo, si el absceso es cavitado, contiene material fecal franco o si se asocia a perforación, requiere de cirugía precoz. Quirúrgico: indicado complicaciones:

en

caso

de

aparición

de

1. Perforación con peritonitis fecal o purulenta (Hinchey III-IV) 2. Sepsis no controlada 3. Obstrucción intestinal. 4. Abceso que no es posible drenarlo por via percutánea. 5. Fístulas. 6. Deterioro clínico o ausencia de mejoría con manejo conservador. La decisión quirúrgica debe considerar cada caso particular, la edad, comorbilidades, frecuencia y severidad de episodios.

20



Enfermedad inflamatoria crónica intestinal

 Nivel de manejo del médico general: Diagnóstico: Sospecha. Tratamiento: Inicial. Seguimiento: Derivar.



Aspectos esenciales  





Cuadro clínico

La enfermedad inflamatoria intestinal comprende 2 grandes enfermedades: Colitis Ulcerosa y Enfermedad de Crohn. Existe una inflamación y usualmente destrucción de la pared intestinal de etiología desconocida, de curso, crónico, que alterna periodos de remisión y actividad de duración variable. No existe un marcador diagnóstico patognomónico, por lo que el diagnóstico es en base a elementos clínicos, de laboratorio, endoscópicos e histológicos. Los síntomas son la mejor guía para evaluar el grado de la enfermedad y su mejoría es el principal objetivo del tratamiento.

Es muy variado. La presencia de las siguientes molestias gastrointestinales orgánicas, que tienden a permanecer por más de 2-4 semanas o son recurrentes en el tiempo, deben hacer plantear la existencia de una EII. - Dolor abdominal: Frecuentemente cólico intermitente y de predominio postprandial. En ocasiones puede despertar al paciente. - Diarrea prolongada, crónica o recurrente. - Síntomas rectales (pujo, urgencia, tenesmo) y/o dolor anal - Síntomas sistémicos cuando el cuadro es muy intenso o los síntomas son muy prolongados; compromiso del estado general, con baja de peso y anorexia, fiebre o anemia. - Síntomas extraintestinales: articulares, mucocutáneos, hepatobiliares, nefrourológicas y oculares, principalmente.

Caso Clínico Mujer de 19 años, consulta por cuadro de dos semanas de evolución de diarrea con sangre abundante, aproximadamente 6 episodios, cólicos abdominales inferiores, urgencia y tenesmo rectal. Sin astenia, artromialgias ni pérdida de peso. A la exploración física: palidez cutáneo-mucosa y abdomen distendido con ausencia de ruidos intestinales.

Importante buscar en el examen abdominal signos de distensión abdominal, que permite sospechar complicaciones como obstrucción intestinal, megacolon tóxico o peritonitis, en especial si hay signos peritoneales o alteraciones hemodinámicas. Ambos trastornos son de curso crónico, donde se alternan fases de actividad con fases de latencia.

Definición

Diagnóstico

Las enfermedades inflamatorias intestinales (EII) son la colitis ulcerosa (CU) y la enfermedad de Crohn (EC) y corresponden a una serie de patologías inflamatorias de etiología multifactorial que afectan principalmente el tracto intestinal. La CU está limitada al colon y el proceso inflamatorio habitualmente se localiza en la mucosa y la submucosa colónica, más frecuente y más intensa de distal a proximal, siempre existiendo compromiso rectal. La EC puede comprometer cualquier segmento del aparato digestivo, es una inflamación crónica transmural que afecta todas las capas del intestino, con localización predominante en íleon terminal, colon y región perianal. Los tramos entre zonas afectadas histológicamente son normales.

Este grupo de patologías no tienen un marcador diagnóstico patognomónico, por lo que el diagnóstico es el resultado de la integración de los antecedentes clínicos, de laboratorio, endoscópicos e histológicos. Es importante descartar causas infecciosas y no infecciosas que pueden confundirse con EII antes de realizar el diagnóstico. - Hemograma (anemia, leucocitosis con desviación a izquierda, alza VHS) y PCR como indicadores de inflamación; evaluación de actividad y presencia de complicaciones. - Niveles de Calprotectina fecal elevados: identificar pacientes con más posibilidades de presentar EII frente a diarrea líquida no disentérica y justifica la urgencia de una endoscopía. - Albúmina, función renal, ELP y pruebas hepáticas para evaluar el compromiso sistémico de la enfermedad. Perfil de hierro y niveles vitamina B12 en caso de anemia y sospecha de malabsorción. - La serología ayuda en pocos casos por su baja prevalencia. La presencia del anticuerpo ANCA se ha asociado a CU y la del ASCA a EC. - Colonoscopía: Examen de elección para el diagnóstico. Permite establecer la naturaleza y extensión, detectar complicaciones, tomar biopsias y eventualmente realizar procedimiento terapéuticos. En la CU el compromiso

Etiología-Epidemiología-Fisiopatología 



En Chile no existen datos estadísticos de su incidencia, aunque la experiencia clínica demuestra un aumento significativo de casos en los últimos años. Afectan a ambos sexos y se pueden presentar a cualquier edad, con un predominio entre los 15 y 40 años. La fisiopatología de la EII está en investigación. La inflamación de la mucosa intestinal origina ulceración, edema, sangrado y desequilibrio hidroelectrolítico.

Si bien la etiología de las EII aún no se comprende en su totalidad, se han descrito factores genéticos, ambientales e inmunológicos, que contribuyen en su patogénesis. La principal hipóstesis se basa en una pérdida de tolerancia a la flora del lumen intestinal, lo que se traduce en una activación permanente o recurrente del sistema inmune, que lleva a un estado de inflamación patológica. 21

endoscópico es continuo desde el recto, en la EC tiende a ser segmentario, no-continuo. Si las condiciones clínicas no permiten el estudio extenso puede ser suficiente una rectoscopía para visualización y toma de biopsias. - Estudio por imágenes: De utilidad para plantear diagnósticos diferenciales, diagnosticar complicaciones o compromiso de zonas inaccesibles al estudio endoscópico, permiten documentar la longitud y localización de estenosis en áreas que no son accesibles con la colonoscopía, y encontrar perforaciones y fístulas. Por ejemplo: enema baritado, tránsito de intestino delgado con bario, enteroclisis por TC o RNM. La radiografía de abdomen simple se utiliza en sospecha de perforación, megacolon tóxico y obstrucción intestinal. No siempre es posible ni obligatorio precisar si se trata de EC o CU (5-15% de diagnóstico indeterminado), ya que la evaluación y el manejo inicial es bastante similar en ambas enfermedades. Sin embargo, frente a la necesidad de cirugía se debe hacer el mayor esfuerzo en determinar el tipo de EII.

coexistentes, curso de la enfermedad, frecuencia de los brotes, y la respuesta a terapias previas y efectos adversos. Al inicio se pueden usar esteroides y/o moduladores de la inflamación, como mesalazina o ácido 5-amlno-salicílico. En casos más severos se pueden usar inmunosupresores y eventualmente terapia biológica.

Seguimiento La terapia farmacológica es suficiente cuando no aparecen complicaciones, dentro de las cuales las más frecuentes son obstrucción intestinal y fístulas en EC y megacolon tóxico y colitis grave o fulminante en la CU. Todas estas deben resolverse de forma quirúrgica.

Tabla 1.Comparación entre CU y EC de la Sociedad Chilena de Gastroenterología, 2013.

Complicaciones

Pronóstico

Derivan principalmente de la propia enfermedad, pero también pueden contribuir los tratamientos médicos aplicados.

La historia natural de las EII es de enfermedades crónicas con actividad intermitente y progresiva. En el caso de EC 67-73% tienen un curso crónico intermitente y sólo un 10-13% se mantienen en remisión por varios años. Luego de 20 años desde el diagnóstico, >50% van a requerir cirugía.

- Durante las crisis: Infecciones por C. difficile o CMV, anemia, desnutrición, trombosis venosa incluido TEP, obstrucción o perforación intestinal. Importante la detección del megacolon tóxico.

En la CU a los 3-7 años de diagnóstico 25% están en remisión, 18% tienen actividad cada año y 57% tienen un curso intermitente. Luego de 10 años desde el diagnóstico, un 24% requiere colectomía, siendo este riesgo mayor durante el primer año de enfermedad.

- A largo plazo: Infecciones sistémicas, osteoporosis, enfermedades cardiovasculares y neoplasias (específicamente, tienen mayor riesgo de cáncer de colon).

Tratamiento Es importante educar al paciente sobre su patología. Se requiere un enfoque multidisciplinario. El tratamiento depende del grado de actividad y del fenotipo de la enfermedad (extensión y patrón), asi como del potencial de crecimiento en niños y adolescentes, enfermedades 22

Daño hepático crónico: Con síntomas y signos de insuficiencia hepática crónica a hipertensión portal.

Hepatitis crónica

Asintomático: pacientes con hallazgos de laboratorio o signos físicos de DHC. Buscar antecedentes personales (factores de riesgo) y familiares.

Nivel de manejo del médico general: Diagnóstico Específico Tratamiento Inicial Seguimiento Derivar

Diagnóstico Las hepatitis crónicas comparen elementos clínicos y de laboratorio, pero a su vez pueden tener características propias y exámenes específicos. Los elementos del diagnóstico sindromático de la hepatitis crónica son:

Aspectos esenciales.    

Realizar detallada anamnesis personal y familiar. Siempre buscar diagnóstico etiológico. HC-Al tiene mejor respuesta a tratamiento. Derivar para un buen manejo.

- Historia clínica: Síntomas generales y específicos, antecedentes personales y familiares relevantes - Laboratorio: Elevación variable de transaminasas y bilirrubina, con diversos grados de hipoprotrombinemia, según la forma clínica de presentación.

Caso clínico tipo. Paciente de 48 años que consulta por ictericia. Al examen se observan telangectasias aracniformes, eritema palmar, hiperplasia parotídea e hipogonadismo. Tiene el antecedente de una transfusión de sangre hace más de 20 años. El laboratorio muestra PCR+ para VHC.

Definición.

- Imágenes: Puede existir hepatomegalia en estados iniciales y luego disminución del tamaño hepático y diversos grados de esplenomegalia. En etapas más avanzadas se puede detectar ascitis. Ecografía abdominal con el objetivo de buscar DHC o sus complicaciones (HTP y hepatocarcinoma).

Conjunto de enfermedades hepáticas caracterizadas por evidencias bioquímicas de inflamación persistente y aspectos histológicos característicos. Éstas corresponden a inflamación, necrosis hepatocelular y desarrollo progresivo de fibrosis.

- Biopsia hepática: Confirma alteraciones de la arquitectura lobulillar con distintos grados de infiltración inflamatoria portal, que erosioan la placa limitante. Puede haber diversos grados de necrosis, fibrosis y/o cirrosis. Hace el diagnóstico definitivo.

Etiología-epidemiología-fisiopatología.

Para el diagnóstico diferencial ver hemograma (VHS), marcadores virales (Hep B: HBsAg, Ig antiHBc; Hep C: anti-VHC y RNA viral VHC por PCR), autoanticuerpos (hep. autoinmune): AAN (antinucleares), AML (antimúsculo liso), anti-LKM1 y cuantificación de IgG.

La etiología puede ser autoinmune, viral (virus hepatitis B, C o B y D), medicamentosa (nitrofurantoína, metildopa), por enfermedades congénitas (Wilson, hemocromatosis, déficit de alfa 1 antitripsina) o criptogénica.

La hepatitis medicamentosa es indistinguible de las demás y lo principal es la sospecha. La hemocromatosis se asocia a diabetes e hiperpigmentación cutánea. La enfermedad de Wilson presenta depósitos en hígado, cerebro y córneas. El déficit de alfa1 antitripsina se sospecha en lactantes.

En los pacientes que contraen virus hepatitis B alrededor del 5-10% se mantiene como portador crónico y el 1% desarrollará una hepatitis crónica. En los con virus hepatitis C, sobre el 80% desarrollará una hepatitis crónica, siendo excepcional su presentación aguda. El proceso de inflamación no depende de la capacidad citopática del virus, sino de la inmunidad celular del paciente, cuando esta es insuficiente no hay una adecuada respuesta inmune, el organismo no logra eliminar al virus y se perpetúa la respuesta inflamatoria.

Tratamiento. Hepatitis crónica autoinmune: La terapia inmunosupresora es altamente efectiva en el control de los síntomas, mejoría de las alteraciones de laboratorio y en modificar la historia natural de la enfermedad. Corticoides solos o idealmente asociados con azatioprina (en su mayoría de forma indefinida por alta probabilidad de recaídas, muy buena respuesta).

En el caso de la hepatitis autoinmune hay un defecto en las células T, produciendo autoanticuerpos contra antígenos hepatocitarios. Afecta preferentemente a mujeres, especialmente en la adolescencia y climaterio. Generalmente es de comienzo insidioso, pero puede tener un comienzo agudo y excepcionalmente una presentación "fulminante".

Formas de presentación

Hepatitis crónica por virus B y C: Los corticoides están contraindicados, terapia de elección en Hep B es entecavir o tenofovir (excepcionalmente PEG interferón), y en Hep C es PEG interferón + ribavirina.

Se pueden ver 4 presentaciones:

Seguimiento.

Comienzo insidioso: Síntomas generales e inespecíficos (semanas o meses) seguidos de signos y síntomas de DHC e insuficiencia hepática.

Los pacientes con hepatitis crónicas virales o enfermedades congénitas deben ser referidos al especialista para manejo y tratamiento. Todas las hepatitis crónicas no tratadas evolucionarán a cirrosis, insuficiencia hepática grave en etapas terminales y la probabilidad de desarrollar hepatocarcinoma.

Comiendo agudo: Similar a otras causas de hepatitis aguda.

23

enfermedades autoinmunes o hepáticas, historia sexual, resistencia insulina, diabetes mellitus, obesidad.

Insuficiencia hepática crónica

El Score de Child-Turcotte-Pugh (CTP) la clasifica en: Nivel de manejo del médico general: Diagnóstico Específico Tratamiento Inicial Seguimiento Derivar.

Aspectos esenciales.   

Categoría A de Child: puntuación CTP de 5 a 6; categoría B de Child: puntuación CTP de 7 a 9; categoría C de Child: puntuación de 10 a 15. Las tasas de supervivencia a 1 año de los pacienes con clase CTP A, B, C y cirrosis son 100%, 80% y 45% respectivamente.

A diferencia de la IH aguda, la IHC no presenta edema cerebral. Child A: enfermedad compensada; Child B: alteración funcional importante; Child C: enfermedad avanzada y descompensada. La primera causa es el daño hepático por alcohol seguido por el virus de la hepatitis C.

MELD Score (Model End Stage Disease)

Caso clínico tipo.

Debido a la escasez de donación de órganos, el score MELD fue desarrollado para proporcionar una predicción más precisa de la mortalidad a corto plazo y así priorizar los trasplantes hepáticos. El score de MELD precide mejor la supervivencia a 3 meses en los pacientes cirróticos, independientemente de la causa, al compararlo con el score CPT.

Paciente con antecedentes de alcoholismo que consulta por episodio de vómito con sangre. Al examen físico presenta aumento de volumen abdominal compatible con ascitis.

Definición. Enfermedad progresiva e irreversible, que cursa con remisiones, reagudizaciones y descompensaciones que ponen en riesgo la vida del paciente. Las alteraciones funcionales deben permanecer por un período mayor a 6 meses.

MELD Score = (9,57 x ln(Crea sérica) + 3,78 x ln(Bilirrubina sérica) + 11,20 x ln(INR) + 6,43) *Si el paciente se encuentra en hemodiálisis, el valor de creatinina debe ser 4.0

Etiología-epidemiología-fisiopatología. Tratamiento.

Causas más frecuentes:    



Daño hepático por OH Esteatohepatitis no alcohólica (asociada al S. metabólico) Hepatitis crónica viral VHC y VHB Hepatopatías autoinmunes.

  

Manifestaciones comprenden ictericia, hipertensión portal, formación de várices esofágicas y gástricas, hemorragia por várices, ascitis, peritonitis bacteriana espontánea, síndrome hepatorrenal, encefalopatía hepática, síndrome hepatopulmonar, infecciones bacterianas diversas y aparición de carcinoma hepatocelular.

  

Diagnóstico.

Es el tratamiento de la causa subyacente de la hepatopatía. Sin embargo, resulta fundamental: Detener o revertir la progresión de la enfermedad hepática Prevenir injurias superpuestas al hígado Identificar los medicamentos que requieren ajustes de dosis o que se deben evitar por completo Manejo de los síntomas y alteraciones de laboratorio Prevención, identificación y tratamiento de las complicaciones para mejorar la calidad de vida Evaluar la indicación de trasplante según el score de MELD (bilirrubina, INR y creatinina) y clasificación de Child.

Seguimiento.

Fundamentalmente clínico y los hallazgos de laboratorio sugerentes de insuficiencia hepática crónica dependen del daño hepático: anemia, trombocitopenia, hiperbilirrubinemia, aumento enzimas (FA, GPT, GOT), hipoalbuminemia, prolongación del tiempo de protrombina, hiperglobulinemia. *En la anamnesis, se debe preguntar por factores de riesgo: Cantidad y duración del consumo de alcohol, transfusiones, uso de drogas por vía intravenosa, nacimiento en zonas endémicas VHB-VHC, contaminación accidental con sangre o fluidos corporales, piercing o tatuajes, antecedentes familiares de

Derivar.

24

a 2 horas después de ingerido el alimento con lactosa. Son variables en cada individuo.

Intolerancia a la lactosa

Exámenes complementarios:  Test de hidrógeno en aire espirado con carga de lactosa. Una elevación de 20 ppm de hidrógeno por sobre el nivel basal, antes de las 3 horas, se considera como umbral diagnóstico de malabsorción de lactosa. Sin embargo, no permite distinguir entre malabsorción primaria y secundaria.  Test de tolerancia a la lactosa: Si hay intolerancia, no aumenta la glicemia, no se usa de rutina  Test de acidez en deposiciones: Si hay intolerancia serán más ácidas de lo habitual, se usa en pediatría.  Determinación directa de la actividad enzimática de lactasa en una biopsia. Se considera el gold standard, sin embargo, por su carácter invasivo y alto costo es poco utilizado de rutina.

Nivel de manejo del médico general: Diagnóstico Específico Tratamiento Completo Seguimiento Completo.

Aspectos esenciales.    

Tiende a presentarse en la adultez. Diagnóstico: test de hidrógeno espirado. Respuesta a eliminación de lactosa de dieta. Puede ser causada por enfermedades (enfermedad celíaca, EII, infección).

Caso clínico tipo. Tratamiento.

Paciente femenino de 24 años refiere sentir hace unos 5 meses hinchazón, acompañada en ocasiones de diarrea. Estas molestias aparecen unas 2 horas después del desayuno, en el cual refiere tomar vaso de leche y comer un pan con queso.

1. El objetivo del tratamiento es disminuir o eliminar los síntomas experimentados por el paciente ante el consumo de productos lácteos. 2. Restricción de lácteos: Es la estrategia central. Estudios han reportado que el consumo de pequeñas cantidad de lactosa (<12gr equivalentes a 1 taza de leche) no son suficientes para producir síntomas, lo que apoya la estrategia de limitar el consumo de lácteos por sobre su total suspensión. Reemplazar los alimentos con lactosa por homólogos sin este carbohidrato. 3. Probióticos: No existe evidencia suficiente. 4. Suplementación de lactasa: Consumir en conjunto con productos lácteos para hidrolizar lactosa contenida en éstos. 5. Manejo nutricional: Importante mantener una ingesta adecuada de calcio.

Definición. Corresponde al conjunto de manifestaciones clínicas producidas por el consumo de productos lácteos, secundarias a la malabsorción de lactosa (incapacidad de digerir o absorber lactosa, la cual puede ser primaria o secundaria).

Etiología-epidemiología-fisiopatología. Etiología: La causa más frecuente es genética (intolerancia primaria o hereditaria) aunque también algunas enfermedades que producen daño en el intestino delgado, como enfermedad celíaca, enfermedades inflamatorias o infecciosas del intestino, pueden generarla (intolerancia secundaria).

Seguimiento. Buscar la presencia de enfermedad celíaca en toda persona con una intolerancia a la lactosa demostrada. Si no responde a tratamiento: derivar para estudio.

Epidemiologia: En el norte de Europa de reportan prevalencias de hasta un 95%, en cambio, en regiones de Asia la prevalencia puede ser menor al 10% de la población. En Chile la prevalencia en poblacion mestiza se ha estimado en 43%, en contraste con el 12% estimado en población amerindia. Fisiopatología: Falta de expresión de la enzima lactasa, que normalmente es producida en las células del intestino delgado. La lactosa que no es digerida en el colon es metabolizada por la flora bacteriana, produciendo acidificación y aumento de la osmolaridad del lumen intestinal y producción de gas, lo que provoca las molestias intestinales.

Diagnóstico. Clínica: Las molestias físicas pueden incluir náuseas y vómitos, ruidos intestinales audibles, meteorismo, distensión abdominal, flatulencias, dolor abdominal, diarrea. Los síntomas se inician generalmente 30 minutos

25

Habitualmente, el principal problema clínico es asegurar la naturaleza benigna de una lesión ocupante de espacio intrahepático. Ayudan a llegar a esta conclusión varios elementos clínicos como:

Masa hepática



Nivel de manejo del médico general: Diagnóstico Sospecha Tratamiento Inicial Seguimiento Derivar.

 

Aspectos esenciales:   



 Los síntomas en la Hiperplasia nodular focal (HNF) suelen estar presentes en mujeres que usan ACO. La complicación más importante del adenoma hepatocelular es la hemorragia en tumores grandes, durante el periodo menstrual. La causa más común de CHC es la hepatitis crónica viral por virus C que lleva a cirrosis, pero la hemocromatosis y los casos de daño por hepatitis B y C con el consumo exagerado de alcohol predisponen fuertemente a esta condición. Por su asiento frecuente en pacientes cirróticos, el HCH debuta con ascitis, encefalopatía, hemorragia digestiva alta o ictericia.



Ser descubiertas de manera fortuita en personas asintomáticas, con examen físico normal y peso estable. Sin enfermedad maligna concomitante o preexistente. Sin cirrosis ni marcadores (+) de virus hepatitis B ni C. Sin alteración de las pruebas bioquímicas hepáticas, anemia, ni signos de respuesta inflamatoria. Que sean lesiones pequeñas, (menores de 4 cm), hiperecogénicas, sin halo periférico y homogéneas.

Epidemiología En el caso del HMC, el 60-80% de los casos se diagnostican en pacientes entre 30-50 años. En adultos existe una relación mujer : hombre de 3 : 1; las estimaciones de prevalencia de los hemangiomas hepáticos oscilan entre 0,4 y 20%.

Caso clínico tipo.

La HNF ocurre comúnmente en mujeres entre 30-40 años. No se ha establecido aún una relación clara con el uso de ACO.

Paciente varón de 66 años con DM2 y cirrosis hepática por VHC, presenta CEG, fiebre y dolor abdominal difuso. El laboratorio destaca pruebas hepáticas normales e hipercalcemia. Se solicita RM que muestra nódulos de 2,5cm de diámetro.

El adenoma hepático ocurre en hígado no cirrótico, y se observa en mujeres, con promedio de edad de 35 años, especialmente en usuarias de ACO (aumenta 29 veces su incidencia). El CHC es el tumor maligno hepático más frecuente y representa el 90% de éstos; existe diferencia por sexo, siendo la relación hombre : mujer de 2.1-5.7 : 1. EM en occidente, son lesiones malignas frecuentes.

Definición. Corresponde a todo crecimiento hepático, ya sea de origen benigno (como hemangioma, hiperplasia nodular focal o adenoma hepatocelular) y/o maligno como es el carcinoma hepatocelular o metástasis.

Etiología

- Hemangioma cavernoso (HMC): Lesión sólida benigna más frecuente del hígado, con mayor incidencia y prevalencia en mujeres (pueden crecer durante el embarazo o con el uso de anticonceptivos orales).

HMC: Su causa no se conoce bien. Se considera como malformación vascular o hamartoma de origen congénito que se agrandan por ectasia más que por hiperplasia o hipertrofia. Habitualmente es una lesión única, pero hasta en un quinto de los casos puede ser múltiple.

- Hiperplasia nodular focal (HNF): Segundo tumor hepático benigno más frecuente, con mayor prevalencia en mujeres entre 30-50 años.

HNF: Corresponde a una hiperplasia (regenerativa), que se produce en respuesta a hiperperfusión por las arterias anómalas características que se encuentran en el centro de estos nódulos. Suele ser de localización subcapsular, no encapsulada, bien circunscrita, generalmente única, múltiple en el 20% de los casos.

- Adenoma hepatocelular (AH): Tumor benigno muy infrecuente. La incidencia aumenta entre las usuarias de anticonceptivos orales (ACO). Es potencialmente maligno.

AH: Fuertemente asociación con el uso de anticonceptivos orales, andrógenos anabólicos, y la enfermedad de almacenamiento de glucógeno. Menos común la asociación con el embarazo y la diabetes mellitus.

- Carcinoma hepatocelular (CHC): Tumor maligno primitivo del hígado más frecuente del adulto, en su mayoría en cirróticos. En Chile, es más prevalente en varones >65 años y tiene baja incidencia, pero aumenta pacientes con daño por virus C, hemocromatosis, obesos, diabéticos y pacientes con esteatohepatitis no alcohólica.

CHC: Factores de riesgo importantes para el desarrollo de CHC son el estado de portación del virus hepatitis B, la infección crónica por el virus de hepatitis C, la hemocromatosis hereditaria, y cirrosis de casi cualquier causa.

- Enfermedad metastásica (EM): La presencia de una neoplasia extrahepática debe buscarse en los pacientes con lesiones hepáticas característicos en los estudios de imagen.

26

persistencia de duda diagnóstica preferir la biopsia quirúrgica.

Fisiopatología HMC: Pueden variar en tamaño desde pocos milímetros hasta varios centímetros. La lesión más grande puede ser pediculada y en el examen macroscópico aparecen como lesiones quísticas con un color oscuro. Microscópicamente, se compone de espacios vasculares cavernosos de diferentes tamaños revestidos por una sola capa de endotelio plana y llena de sangre. Los compartimentos vasculares están separados por septos fibrosos delgados y pueden contener trombos. Pueden desarrollar cicatriz de colágeno o nódulo fibroso. En ocasiones, puede haber calcificación y la osificación focal estromal.

AH: TAC helicoidal de 3 fases o RNM. CHC: Uso asociado de ECO más Alfa fetoproteína (AFP) > 400ng/mL. En nódulos > 1cm deben aplicarse técnicas confirmatorias y además etapificarse. En nódulos de 1-2 cm, la coincidencia de imagen típica de CHC en 2 exámenes, es diagnóstico sin la necesidad de biopsia, salvo si presenta un patrón atípico. Si no confirma diagnóstico, repetir la biopsia o seguir con imágenes cada 2 ó 3 meses.

HNF: Tiene fuerte margen sin cápsula y puede ser pedunculada. El hallazgo característico es la presencia de una cicatriz estrellada central que contiene una arteria grande anormal con múltiples ramas que irradian a través de los septos fibrosos a la periferia. Criterios microscópicos mínimos para el diagnóstico de FNH clásica son arquitectura nodular, vasos anormales, y la proliferación de conductillos biliares.

En nódulos > 2cm en pacientes cirróticos, concordancia de 2 exámenes TAC o RNM sin necesidad de biopsia. Sin concordancia se realiza biopsia o una tercera imagen (ultrasonido contrastado o arteriografía). En este caso una AFP > 200ng/mL confirma el diagnóstico.

AH: Típicamente lesiones solitarias, varían en tamaño desde unos milímetros hasta varios centímetros. Lesiones a menudo se ubican en el lóbulo derecho del hígado. Apróximadamente 10% son pedunculados. Grandes vasos sanguíneos se aprecian en la superficie y dentro del tumor, con áreas de hemorragia y necrosis. Usualmente no hay cápsula fibrosa; como resultado, la hemorragia de un adenoma puede extenderse libremente en el hígado.

Habitualmente innecesario. Se indica resección quirúrgica en complicaciones como ruptura, asociación con coagulopatía, anemia hemolítica o en cambios de tamaño.

Tratamiento.

HNF: Cuando los síntomas son severos, indicar resección quirúrgica. AH: Suspender ACO. Indicada la resección quirúrgica por riesgo de malignización.

CHC: Ocurre con mayor frecuencia en hígado con enfermedad crónica. La cirrosis hepática es el mecanismo, por el cual los factores de riesgo ocasionan aparición de CHC.

CHC: En el diagnóstico precoz se considera la resección quirúrgica, trasplante hepático o ablación. Diagnóstico en estadios avanzados consideran paliación. La embolización transarterial con o sin quimioterapia es la técnica más utilizada por su acción antitumoral. Sorafenid vía oral, demostró mejoría en sobrevida en pacientes con CHC avanzado.

Diagnóstico. En la clínica:

Seguimiento.

HMC: Habitualmente son pequeños, en su mayoría son asintomáticos, lesiones >10cm o periféricas pueden cursan con dolor sordo en hipocondrio derecho.

CHC: Control y estudio a pacientes cirróticos cada 6 meses, con función hepática aceptable, Child A o B, en condiciones de recibir terapia efectiva. El objetivo es diagnosticar lesiones < 3cm. Casi la mitad de los nódulos < 1 cm en cirróticos, no corresponde a CHC; se recomienda repetir imágenes cada 2-3 meses. Si se mantiene estable a los 2 años, debe realizarse control semestral. Si hay crecimiento, debe usarse TAC o RNM. La ausencia de crecimiento no descarta CHC, pues puede ser estable por largos períodos. En pacientes cirróticos con AFP >20ng/mL, sin nódulos se recomienda el seguimiento con TAC o RNM.

HNF: Mayoría son asintomáticos, hallazgo en estudio de imágenes. Se puede presentar como masa abdominal palpable en lesiones gigantes. Las complicaciones son extremadamente infrecuentes, incluyendo la hemorragia, a pesar de su carácter vascularizado. AH: 25% son asintomáticos. 30-40% presenta dolor en hipocondrio derecho con masa palpable y alteración de las pruebas hepáticas. La ruptura ocurre en el periodo menstrual y en usuarias de ACO. CHC: El cuadro clínico incluye dolor en cuadrante superior derecho, CEG, baja de peso y descompensación de función hepática. En general si se detecta en etapa sintomática tiene mal pronóstico, de hecho actualmente el 70% se detecta en fase incurable. Laboratorio e imágenes: HMC: ECO. y si hay duda diagnostica se puede realizar RNM de preferencia, TAC. HNF: ECO, TAC y/o RNM. Evidencia en estudios contrastados, cicatriz central en el tumor. Ante la 27

Parasitosis Intestinales

capacidad de defensa debido a la alteración en el estado nutricional; alteración en la integridad de tejidos, modificación de la microflora intestinal, entre otras alteraciones.

Nivel de manejo del médico general: Diagnóstico Específico Tratamiento Inicial Seguimiento Derivar.

Diagnóstico. Importante una anamnesis detallada: viajes, contacto con agua contaminada, asistencia a guardería; síntomas como fiebre, sangre en las heces, vómitos, dolor abdominal, prurito anal, etc. Esta información y la sospecha etiológica concreta deben transmitirse al laboratorio para que se puedan realizar las pruebas microbiológicas oportunas.

Aspectos esenciales.     

Principalmente protozoarias. Aumento de pacientes inmunodeprimidos. Clínica leve en su mayoría. Enseñar al paciente la recolección de su muestra Educar para evitar reinfecciones.

El diagnóstico es etiológico, lo principal es obtener una buena muestra y elegir el examen pertinente. El diagnóstico de Helmintos puede ser por observación directa del ejemplar adulto; y tanto protozoos y helmintos por examen parasitológico seriado de deposiciones (EPSD) el cual tiene 2 métodos: Teleman (para huevos y quistes, cuadros intermitentes o con poca clínica); Burrows (trofozoitos, pacientes con clínica activa o aguda).

Caso clínico tipo. Niño de 3 años con diarrea acuosa, dolor abdominal cólico de hace 6 días, febrícula, náuseas y vómitos. Ha ingresado a guardería hace poco. Se pide cultivo de deposiciones siendo negativo para enterobacterias y se descarta adenovirus y rotavirus. Han pasado 5 días y el niño ha bajado de peso. Se le debe pedir EPSD.

Tratamiento.

Definición.

El tratamiento es específico para cada parásito, de ahí deriva la importancia del diagnóstico correcto. No obstante, no todas las parasitosis deben tratarse siempre.

Corresponden a las infecciones gastrointestinales por parásitos.

Protozoosis: Amebiasis: Metronidazol / Tinidazol. Prohibir el consumo de alcohol Balantidiasis: Tetraciclina / Metronidazol Blastocystosis: Metronidazol / TrimetoprimSulfametoxasol / Nitazoxanida (no en Chile) Cryptosporidiosis: En inmunocompetentes se han reportado resultados favorables con Nitazoxanida. Sin embargo en inmunocomprometidos no es efectivo. Cyclosporosis: Trimetoprim / Ciprofloxacino Cystoisosporosis: Trimetoprim-Sulfametoxasol Giardiasis: Metronidazol / Tinidazol / Furazolidona / Nitazoxanida Sarcocystosis: Sólo se pueden aplicar medidas sintomáticas.

Etiología-epidemiología-fisiopatología. Epidemiología: Los parásitos intestinales afectan con más frecuencia a los niños que a los adultos. Existe una estrecha relación entre las condiciones socioeconómicas, saneamiento ambiental e infección parasitaria; en la medida que han mejorado dichos parámetros en Chile, estas infecciones han disminuido en forma significativa. Sin embargo, cambios en la moda culinaria y mayor acceso a alimentos crudos o insuficientemente cocidos; viajes e inmigración, se ha traducido en aumento de agentes que antes no tenían importancia clínica. Se deben reconocer a los agentes que afectan a pacientes con inmunodeficiencia, cuyo tratamiento y prevención son fundamentales.

Helmintiasis Anisakiasis: Extracción con pinza de biopsia en la anisakiasis gástrica y exploración quirúrgica en la ubicación intestinal. Ascariasis: Albendazol / Mebendazol Difilobrotriasis: Praziquantel Hymenolepiasis: Praziquantel Oxiurasis: Albendazol / Mebendazol. El tratamiento se debe extender a todo el grupo familiar, recomendando el aseo cuidadoso del hogar. Teniasis: Praziquantel. Se requiere aislamiento entérico para prevenir Cisticercosis. Tricocefalosis: Albendazol / Mebendazol

Etiología: Esquemáticamente se dividen de protozoosis y helmintiasis. Las más frecuentes (89%) son las protozoosis (Amebiasis, Balantidiasis, Blastocystis homini, Giardia intestinalis, Entamoeba histolytica y por Cryptosporidiosis entre otros). Helmintiasis (10%): anisakiasis (ingesta de pescado crudo), ascariasis, hymenolepiasis, oxiuriasis, teniasis, difilobotriasis, tricocefalosis. Fisiopatología: Pueden producirse por la ingestión de quistes de protozoos, huevos o larvas de gusanos o por la penetración de larvas por vía transcutánea desde el suelo. Cada uno de ellos va a realizar un recorrido específico en el huésped y afectará a uno o varios órganos. Se genera una competencia por nutrientes, se puede producir malabsorción intestinal, pérdida crónica de sangre, utilización excesiva de nutrientes, reducción de la

Seguimiento. Como las parasitosis intestinales tienen mayor prevalencia en los sectores socioculturales bajos por las condiciones de hacinamiento e higiene, lo principal es la educación sanitaria para evitar los distintos mecanismos de infección y su diseminación.

28

Factores de riesgo: Proteínas totales de líquido ascítico menor de 1gr/dL, antecedentes de PBE previa, Hemorragia digestiva en curso.

Peritonitis bacteriana espontánea del cirrótico

Fisiopatología Nivel de manejo del médico general: Diagnóstico: Específico. Tratamiento: Completo. Seguimiento: Derivar.

Ocurriría por traslocación bacteriana desde el lumen intestinal, a raíz de alteraciones en la motilidad y permeabilidad de la mucosa del colon, asociadas a un sobrecrecimiento bacteriano, en el contexto de hipertensión portal.

Aspectos esenciales.    

Diagnóstico.

Aerobios Gram negativos son los más frecuentes, especialmente E. coli. Debe realizarse paracentesis diagnóstica en todo cirrótico con ascitis. Se considera criterio diagnóstico tener un recuento de PMN ≥ 250/uL, debiendo hacer el adecuado diagnóstico diferencial con PBS. Debe iniciarse tratamiento ATB empírico inmediato.

Clínica: Puede manifestarse de varias maneras: paciente asintomático, signos y síntomas de peritonitis, signos y síntomas de infección sistémica o descompensación de cirrosis hepática (encefalopatía hepática, hemorragia digestiva alta, deterioro de función renal). Pueden desarrollar síntomas y signos tales como dolor/distensión abdominal, vómitos, diarrea, fiebre, calofríos, alteraciones del status mental, íleo paralítico, entre otros. La PBE es una condición grave y que potencialmente puede evolucionar a sepsis, falla multiorgánica y muerte, por lo que se vuelve muy relevante el diagnóstico precoz y el inicio temprano del tratamiento.

Caso clínico tipo. Hombre de 65 años, con antecedentes de cirrosis hepática alcóholica. Consulta por cuadro de 1 semana de evolución de síndrome confusional agudo, con ascitis moderada, compromiso del estado general y sensación febril.

Exámenes: Debe realizarse paracentesis diagnóstica en todo paciente cirrótico con ascitis que ingresen al hospital, sea sintomático o no. El diagnóstico es de laboratorio, mediante el estudio del líquido ascítico (LA) y 2 cultivos del mismo (aerobio y anaerobio), sin esperar a estos últimos (ya que tiene un rendimiento cercano a un 40%). Se considera diagnóstico un recuento de PMN ≥ 250/uL.

Definición. Se define como la infección del líquido ascítico (LA), sin una fuente infecciosa intraabdominal quirúrgicamente tratable ni evidente. Clínicamente se observa un amplio espectro de pacientes que van desde la infección asintomática (aprox. un 13%) hasta pacientes que presentan síntomas y signos clásicos de infección.

La presencia de plaquetopenia o INR prolongados no constituyen contraindicación para realizar el estudio diagnóstico, salvo en casos de sindromes hemorragíparos clínicamente evidentes.

La prevalencia de PBE en cirróticos hospitalizados oscila entre el 10 y 30%, con una mortalidad del 20% y una recurrencia anual del 75%. Ya que representa una elevada mortalidad a mediano plazo, y siendo además ésta un factor independiente de mal pronóstico, la PBE contribuye a la indicación de trasplante hepático.

Respecto al diagnóstico diferencial, es importante distinguir la PBE de la peritonitis bacteriana secundaria (PBS), la cual no suele remitir sin intervención quirúrgica. Debe sospecharse PBS cuando coexisten en el estudio del LA o cultivo: Glucosa < 50 mg/dL, Proteínas > 10g/L, LDH > LDH plasmática y Ascitis polimicrobiana (en cultivo o tinción de gram). La falta de respuesta al tratamiento antibiótico es otro parámetro clínico que debe hacer sospechar esta entidad. Al tener alta sospecha de PBS, se debe solicitar rápidamente un estudio imagenológico y evaluación por equipo de cirugía.

Etiología

Tratamiento.

Los agentes etiológicos de PBE más frecuentes corresponden a aerobios Gram Negativos (70%), especialmente Escherichia Coli, Klebsiella; 30% corresponden a Cocáceas Gram positivas incluyendo neumoco y enterococo.

Comprende: control hemodinámico, prevención complicaciones y tratamiento antibiótico empírico.

Epidemiología

de

1. Debe iniciarse tratamiento antibiótico empírico en forma inmediata, por un mínimo de 5 días. Cefotaxima es el antibiótico de elección, en dosis de 2g/12 horas ev. o Ceftriaxona 2g/día ev . Se considera la PBE resuelta cuando existe regresión de la sintomatología, recuento de PMN en LA < 250/uL, negativización de cultivos y normalización de hemograma. Ante un fracaso del tratamiento antibiótico debe sospecharse la presencia de una PBS y debe ser modificado el esquema antibiótico para ampliar la cobertura.

Es importante destacar que algunos casos son de etiologías bacterianas polimicrobianas, sin embargo, no es una condición frecuente y debe hacer sospechar la presencia de peritonitis bacteriana secundaria.

29

2. Prevención de insuficiencia renal a través de administración de Albúmina ev. 1,5g/kg el primer día y 1g/kg el tercer día. Este efecto es más significativo el subgrupo de pacientes con: BUN > 30mg/dL y/o creatinina pl. > 1mg/dL o bilirrubina pl. > 4mg/dL, sin embargo, dado que el SHR es un complicación grave, potencialmente mortal, se sigue recomendando la administración de albúmina a todos los pacientes que cursan con una PBE. Evitar nefrotóxicos, diuréticos, paracentesis evacuadora.

Seguimiento. Por especialista. La profilaxis de la PBE ha demostrado retrasar la indicación de trasplante hepático y mejorar la sobrevida de los pacientes con DHC. La profilaxis secundaria está indicada en todos los pacientes que hayan presentado PBE, y su indicación se prolongará mientras exista ascitis. Se indica el uso de quinolonas (ciprofloxacino 500mg/día o norfloxacino 400mg/día)

30

Etiología

Síndrome de malabsorción Nivel de manejo del médico general Diagnóstico Específico Tratamiento Inicial Seguimiento Derivar.

Las principales etiologías de mala digestión son enfermedades pancreáticas, cirugías gástricas y las ictericias colestásicas. Las de malabsorción parietal son la enfermedad celíaca, resecciones intestinales (Síndrome de intestino corto), infecciones y parasitosis, enfermedades de Crohn y Whipple, intolerancia a lactosa y fármacos.

Aspectos esenciales.

Fisiopatología:

   

Alteración en la absorción de uno o más nutrientes contenidos en la dieta. Síntomas gastrointestinales, baja de peso y avitaminosis. Realizar estudio inicial. Derivar con sospecha etiológica.

Una apropiada absorción de nutrientes consta de 3 fases: 1. Fase luminal: Digestión de los nutrientes en el lumen intestinal. 2. Fase absortiva o de la mucosa: Absorción a través de la mucosa intestinal.

Caso clínico tipo.

3. Fase post-absortiva o de remoción: Transporte a la circulación sanguínea o linfática.

José de 68 años con antecedentes de alcoholismo crónico y dolor abdominal recurrente, ingresa por aumento del dolor con la ingesta, disminución del apetito, pérdida de peso y diarrea. Refiere heces más abundantes, pastosas, e hipocoloreadas.

Alteraciones en cualquiera de estas etapas puede causar malabsorción. Los mecanismos varían según el nutriente comprometido.

Diagnóstico. Definición. Sospecha: El espectro de presentación clínica es amplio y puede ser:

Se refiere a la alteración de la absorción de nutrientes. Puede ser global o parcial. La malabsorción global es el resultado del compromiso difuso de la mucosa intestinal, una superficie de absorción reducida o alteraciones pancreáticas, mientras que la parcial es causada por enfermedades que afectan la absorción de un nutriente específico.

- Asintomático con alteraciones sólo de laboratorio (sospechar frente a: anemia ferropénica, hipocalcemia, hipoalbuminemia, linfopenia, entre otros). - Oligosintomático con síntomas GI leves como: deposiciones blandas, malestar abdominal, meteorismo, distensión abdominal, anorexia. Forma de presentación más frecuente.

Puede ser consecuencia de defectos congénitos en los sistemas de transporte de membrana del epitelio del intestino delgado o de defectos adquiridos en la superficie de absorción epitelial. Otro factor que puede interferir con la absorción de nutrientes es mala digestión, que es debido a la alteración de la digestión de los nutrientes en el lumen intestinal o en el borde en cepillo de las células epiteliales de la mucosa.

- Malabsorción global evidente con esteatorrea, lienteríe o baja de peso con apetito conservado. Es infrecuente. *El dolor abdominal es poco común en malabsorción, excepto en casos de pancreatitis crónica, enfermedad celíaca o enfermedad de Crohn. Posibles hallazgos al examen físico: baja de peso, signos de anemia, equimosis o petequias, hiperqueratosis, glositis, signo de Chvostek y Trousseau, edema, distensión abdominal y disminución de masas musculares. Los hallazgos son variables según nutriente afectado.

Epidemiología Enfermedad celíaca: más frecuente en la raza blanca, en climas tropicales, raro en raza negra porque consumen más maíz que trigo.

Enfermedad de Whipple: principalmente varones entre la cuarta y la séptima décadas de la vida.

Antecedentes: Preguntar por cirugías del tracto GI (gastrectomía, resección de intestino delgado o de páncreas), pancreatitis crónica (consumo de alcohol), colestasia crónica o radioterapia. Antecedentes familiares de enfermedad Celíaca, enfermedad de Crohn, fibrosis quística y déficit de lactasa.

Síndrome de intestino corto: antecedente de resección intestinal de más del 50% de su longitud.

El estudio inicial no requiere en su inicio necesariamente al especialista y puede realizarse de forma ambulatoria.

Esprue tropical: propio de individuos que residen o que han visitado zonas tropicales, en especial la India, Extremo Oriente y el Caribe.

Confirmación: Los exámenes más usados son los que evalúan esteatorrea como tinción Sudán III y esteatocrito >6%. En los estudios etiológicos iniciales están los anticuerpos antiendomisio y antitransglutaminasa, el EPSD para parásitos entre los que se pueden solicitar en

Deficiencia de lactasa: la presencia en adultos de raza blanca es alrededor del 5%; en americanos de raza negra, bantúes y razas orientales es del 60 al 90%.

31

atención primaria. La Endoscopía con Biopsia, test espiratorios, test de absorción de D-Xilosa, entre otros, suelen ser solicitados en nivel secundario.

Tratamiento. General: Comidas pequeñas y fraccionadas, restricción de lactosa, limitación de grasas, reposición electrolitos, minerales y vitaminas. Etiológico: En celíacos dieta sin gluten; antiparasitarios si la causa fuese parasitosis, reposición de enzimas pancréaticas en pancreatitis crónica, etc.

Seguimiento. Confirmar, solicitar estudio inicial y derivar al especialista.

32

Tumores de colon

Factores familiares o genéticos   

Nivel de manejo del médico general: Diagnóstico Inicial Tratamiento Inicial Seguimiento Derivar.

Antecedentes familiares de cáncer colorrectal esporádico Poliposis adenomatosa familiar (PAF) Cáncer colorrectal no asociado a poliposis (HNPCC)

Factores ambientales

Aspectos esenciales.    



Pólipos: usualmente asintomático, puede ulcerarse, sangrar y doler. Si es muy grande, puede obstruir. El 50% de pólipos mayores de 2 cms. desarrollan cáncer. El cáncer de colon es la primera neoplasia maligna del aparato gastrointestinal. Es fundamental el diagnóstico temprano, el cual es accesible al examen del endoscopista.

   

Entre los factores protectores para el CCR destaca: consumo de calcio, folatos, vitamina B6, alta ingesta de frutas y verduras, actividad física, terapia de reemplazo hormonal con estrógenos y progestinas combinados y el uso de Aspirina y AINEs.

Caso clínico tipo. Mujer de 58 años, consulta por dolor abdominal, distensión y vómitos. Con antecedente de cáncer colónico en padre. Refiere que hace 2 días está con dolor en FII, tipo cólico, no irradiado, no se alivia ni se exacerba con ninguna conducta. También refiere cambios en el hábito intestinal y presenta constipación de varios días. La paciente se encuentra distendida, en el momento del comienzo del dolor, desde hace 5 ó 6 semanas.

Fisiopatología Tumores y lesiones pseudotumorales son frecuentes en el intestino grueso. Las lesiones benignas de colon son menos frecuentes y corresponden a un muy bajo porcentaje del total de tumores de colon. El adenoma es la neoplasia más frecuente del intestino grueso y se acepta que éstos son lesiones precancerosas. Se presenta en forma de pólipo sésil o pediculado (adenoma poliposo o pólipo adenomatoso). Pueden ser únicos o múltiples.

Definición. Se denomina cáncer de colon a la presencia de un tumor maligno ubicado en la pared del colon, que corresponden a la última parte del tubo digestivo.

El adenocarcinoma es el tumor maligno más frecuente de colon. La mayoría de los adenocarcinomas se originan de un pólipo, constituyendo la secuencia adenomacarcinoma, la cual demora aproximadamente 10 años en desarrollarse.

Epidemiología

El 90% de los CCR se originan de un adenoma, aunque sólo el 5% de los adenomas progresarán a adenocarcinoma. Dentro de los adenomas con mayor riesgo, se encuentran los adenomas vellosos, mayores de 1 cm, con displasia de alto grado y adenomas múltiples.

En Chile ha tenido un crecimiento progresivo, situándose entre los 3 a 5 tumores que causan más muertes. Sobre 1.000 muertes al año son debidas a cáncer de colon en Chile. En países desarrollados este cáncer se sitúa entre las primeras causas de muerte por cáncer. La mortalidad ha aumentado de 1990 al 2010, de 3,7 a 8 casos/100.000 habitantes. En Chile, en hombre ocupa el cuarto lugar como causa de muerte, por tumores malignos; y el quinto lugar en mujeres.

Diagnóstico. Los pacientes con tumores de colon se pueden presentar de tres formas:

La edad promedio al momento del diagnóstico es la sexta decada de la vida, sin diferencias por sexo.

a) con síntomas y/o signos sospechosos, b) individuos asintomáticos descubiertos por detección de rutina

Etiología Principales factores que aumentan el riesgo de cáncer de colon incluyen:

c) admisión en servicio de urgencia con obstrucción intestinal, peritonitis aguda por perforación de colon, o hemorragia digestiva baja (marcadores de mal pronóstico por presentarse en etapas avanzadas de la enfermedad).

Factores personales    

Dieta y nutrición: Consumo de carnes rojas y procesadas, carnes cocinadas con humo, el beber moderado (2-3 tragos al día) y consumo de leche no fermentada. Obesidad Falta de actividad física Tabaquismo Radiación pélvica

Edad Sexo masculino Antecedentes de cáncer colorrectal Enfermedad inflamatoria intestinal

La mayoría de los tumores de colon se diagnostican después de la aparición de los síntomas. De acuerdo con la localización tumoral, las neoplasia de colon derecho se 33

manifiestan principalmente como anemia ferropriva (por sangrado oculto), diarrea crónica, compromiso de estado general (astenia, adinamia, anorexia, edema), masa palpable en fosa ilíaca derecha, dolor localizado y permanente por infiltración neoplásica. Los tumores de colon izquierdo presentan con mayor frecuencia alteración del hábito intestinal hacia la constipación o diarrea con o sin alternancia, alteración en la forma de las deposiciones (acintadas), hematoquezia o rectorragia intermitente. Los tumores rectales presentan tenesmo, pujo, deposiciones acintadas, dolor al defecar, rectorragia y sensación de recto ocupado.

El seguimiento posterior a la cirugía consiste en la medición de CEA cada 3 meses durante los primeros 2 años, luego cada 6 meses por 5 años. Junto con lo anterior, se realiza una colonoscopía total anual durante los primeros 3 años y luego cada 5 años y estudio con TC TAP anual por 5 años en total.

Pronóstico Respecto al pronóstico, se ha visto que el factor más importante previo a la cirugía es el estadio de la enfermedad al momento del diagnóstico. Se ha visto también que el hecho de tener obstrucción o perforación intestinal confiere peor pronóstico, independiente de la etapa.

Si la enfermedad es metastásica, presenta síntomas variados de acuerdo a la ubicación de la metástasis (linfonodos, hígado, pulmón, peritoneo), como saciedad precoz, ictericia, ascitis, etc.

La sobrevida condicional de los pacientes con cáncer de colon tratados, después de 5 años del diagnóstico, puede llegar a ser mayor al 80% en toda las etapas menos en la etapa IV, que es de un 48% aproximadamente.

Examen de elección: Colonoscopía, permite toma de biopsias para estudio histológico (Biopsia para confirmación diagnóstica y/o buscar presencia de displasia de la mucosa), ademñas de potencialidades terapéuticas (resección de pólipos). Otros exámenes: Radiología con medio de contraste. Enema baritado. Colonografía por TC.

Tratamiento. Los adenomas pueden ser extirpados por colonoscopía, especialmente los que son pediculados. Los pólipos sésiles también pueden ser removidos (En algunos casos en varias sesiones), pero con mayor riesgo de perforación. Cuando el pólipo es grande y no puede ser extirpado por endoscopía (incluyendo la polipectomía con asa), debe ser tratado quirúrgicamente con colectomía segmentaria. Para el tratamiento del cáncer de colon, el tratamiento de elección es la cirugía, la cual consiste en resecar el segmento afectado, dejando márgenes limpios de enfermedad (>5 cms aproximadamente) y llevando consigo el segmento linfovascular correspondiente (linfadenectomía adecuada es >12 ganglios resecados). En casos seleccionados se complementarias, tales como quimioterapia.

emplean terapia radioterapia o

En el caso de metástasis hepáticas o pulmonares aisladas, pueden resecarse y mantener la intención curativa. En enfermedad metastásica avanzada es importante el manejo paliativo. En ocasiones, se realiza tratamiento quirúrgico paliativo en casos de obstrucción (con endoprótesis, ostomías de descarga, colectomías paliativas), perforación o sangrado, constituyendo urgencias quirúrgicas.

Seguimiento. Por especialista. Detección temprana y vigilancia a sujetos de alto riesgo: familiares con adenomas, según grado de parentesco, número de parientes afectados, edad de inicio temprano. La vigilancia debe comenzar a los 40 años o si el caso índice era de menor edad al momento del diagnóstico, 10 años antes de la edad de diagnóstico.

34

En la úlcera gástrica se identifica infección por H. pylori en un 60-80% y en úlcera duodenal en un 90-95%.

Úlcera péptica

Fisiopatología

Nivel de manejo del médico general: Diagnóstico Específico Tratamiento Completo Seguimiento Completo.

Desbalance entre factores agresivos y protectores de la mucosa. En la úlcera gástrica, la disminución de factores protectores de la mucosa (mucus, HCO3, flujo sanguíneo, PGs y uniones intercelulares,) tiene un rol fundamental en la patogenia. En la úlcera duodenal, el aumento de factores agresivos, (H. pylori, AINES, alcohol, ácido gástrico+pepsina o bilis) daña la mucosa del duodeno.

Aspectos esenciales.     

El dolor es epigástrico. Úlceras gástricas pueden ser malignas; extremadamente raro en el caso de las duodenales. Causas más frecuentes son infección por H. pylori y uso de AINES. Tratamiento erradicar H. pylori + IBP o Anti-H2. No olvidar biopsia en úlcera gástrica.

Diagnóstico. Clínica: Desde no presentar síntomas, hasta desarrollar complicaciones con riesgo vital. Síntomas: dolor epigástrico, urente o punzante, se exacerba con ayuno y se alivia con antiácidos, frecuentemente nocturno. Relación variable con la ingesta de alimentos. En las complicaciones: dolor abdominal agudo (perforación), vómitos por obstrucción al vaciamiento gástrico, hematemesis/melena o shock (HDA).

Caso clínico tipo.

Cabe mencionar que por su efecto analgésico los AINEs puede suprimir las manifestaciones clínicas de la UP y, por su efecto antiagregante plaquetario, alteran la coagulación. Esta combinación hace que la HDA en pacientes previamente asintomáticos sea frecuentemente la primera manifestación de una UP en usuarios de AINEs.

Paciente sexo femenino, 50 años, refiere sentir desde hace algunos meses dolor epigástrico de carácter quemante. El dolor aumenta con el ayuno y se alivia al comer o con antiácidos.

Definición.

Examen físico: En úlcera péptica no complicada el examen tiende a la normalidad. Se deben detectar complicaciones: Taquicardia y ortostatismo sugieren HDA (hematemesis/melena), abdomen tenso sugiere perforación. Chapoteo a la sucusión, por retención de líquido en el estómago, sugiere obstrucción.

Solución de continuidad de la mucosa de más de 3- 5 mm de diámetro mayor, que alcanza hasta la submucosa, pudiendo extenderse a todo el espesor del órgano; que cicatriza por reparación de las túnicas subyacentes de la mucosa y por regeneración atípica de ésta, con retracción (cicatriz).

Exámenes complementarios: EDA (de elección) o estudio radiográfico con bario. Métodos para detectar H. pylori, que pueden ser invasivos (Test ureasa rápida, estudio histológico y cultivo) o no invasivos (serología, detección de urea en el aliento y antígeno en las heces). En caso de úlcera gástrica en la EDA siempre debe tomarse biopsia para descartar lesiones malignas y controlar su cicatrización con otra endoscopía.

Etiología La principal causa es el H. pylori, seguido por el uso de AINES (sobretodo en edad avanzada). El 50% de la población esta colonizada por H. pylori pero solo el 5-10% desarrolla úlcera péptica. Esto depende probablemente de la interacción de factores dependientes de la bacteria (cepas "ulcerogénicas") y del huésped (edad de infección, tipo de respuesta inmune local y efecto de la infección sobre la secreción de ácido)

Tratamiento.

Otras causas: fármacos (corticoides, anticoagulantes, bifosfonatos); estados de hipersecreción ácida (ZollingerEllison, mastocitosis sistémica, etc.); neoplasias (adenocarcinoma o linfoma); tabaquismo, úlceras por estrés fisiológico intenso (paciente crítico, trauma, sepsis, gran quemado, cirugía mayor); radio y quimioterapia, dieta (incluido el exceso de alcohol).

Erradicación de H. pylori si hay úlcera activa o antecedentes de enfermedad ulcerosa, mediante terapia durante 7 a 14 días, seguido de IBP o Anti-H2 no más allá de 2 semanas. Con respecto a la evolución natural de la UP se describe; cicatrización espontánea 20-60%, agregando terapia antisecretoria cicatrización 90-95% a las 4 semanas, y recurrencia (sin tratamiento) de 90% a 1 año. La erradicación de H. pylori es la única medida terapéutica que modifica la historia natural de la enfermedad, con tasas de recurrencia de 2-5% al año luego de un tratamiento antibiótico exitoso.

Epidemiología Se estima que 10% de los individuos en países occidentales van a desarrollar una úlcera péptica en algún momento de su vida. En un 70% la edad de presentación es entre los 25 – 64 años. Se estiman 15.000 muertes por año a consecuencia de UP complicada.

35

Opciones de terapia: - Terapia triple: IBP (dosis estándar c/12 hrs) + Claritromicina (500mg c/12 hrs) + Amoxicilina 1 gr c/12 hrs por 7-14 días. - Terapia cuádruple: IBP + Bismuto (240mg c/6 hrs) + Metronidazol (500mg c/8 hrs) + Tetraciclina (500mg c/6 hrs) por 10-14 días. - Terapia secuencial: IBP (dosis estándar c/12 hrs) + amoxicilina 1gr c/12 hrs días 1 a 5 e IBP(dosis estándar c/12 hrs) + claritromicina 500mg c/12 hrs + tinidazol/metronidazol 500mg c/12 hrs días 5 a 10. - Terapia concomitante: IBP (dosis estándar c/12 hrs) + amoxicilina 1gr c/12 hrs + claritromicina 500mg c/12 hrs + tinidazol/metronidazol 500mg c/12 hrs por 10-14 días. En caso de úlcera por AINE, suspender el fármaco, y mantener IBP por 8 semanas. Suprimir el consumo de cigarros. Cirugía: Indicada frente a úlceras complicadas que no se pueden manejar en forma médica o endoscópica. Por ejemplo en HDA masiva, requiere resolución quirúgica clásica; en perforación, aseo y reparación mediante cirugía laparoscópica o clásica. Frente a síndrome pilórico con obstrucción al vaciamiento gástrico, a veces se requiere antrectomía o gastrectomía subtotal, con cirugía clásica o laparoscópica.

Seguimiento. Si la úlcera es gástrica, repetir la EDA 8 a 12 semanas después para comprobar cicatrización y la erradicación de la bacteria y hacer nueva biopsia, en caso de que la úlcera esté todavía presente. El control endoscópico no es necesario en la úlcera duodenal. En estos casos la confirmación de la erradicación es recomendable en los pacientes que han tenido una complicación y optativa en el resto (puede realizarse mediante métodos no invasivos). Si los síntomas persisten o reaparecen, derivar a Gastroenterología.

36

MÓDULO 1: Medicina Interna

Geriatría Caídas da de conocimiento, ni alteración del tono postural. Suele ser caracterizado con frases como “me tropecé y no sé cómo”, “se me aflojaron las piernas”, “de pronto me vi en el suelo”, etc.

Nivel de manejo del médico general: Diagnóstico Específico Tratamiento Completo Seguimiento Derivar

Epidemiología

Aspectos Esenciales • • • •





El riesgo de sufrir caídas aumenta con la edad. Sufren caída un tercio de los pacientes a los 65 años, y la mitad a los 75 años. Y el riesgo se triplica si se trata de pacientes institucionalizados, o si han sufrido caídas previas. El principal problema de las caídas no es sólo la incidencia elevada, sino sus consecuencias: 40% desarrolla deterioro funcional (por el miedo a caer “Síndrome Post Caída”, aislamiento social, depresión), 10% complicaciones graves con alta mortalidad (fracturas, TEC), 40% institucionalizaciones (por la caída o sus consecuencias), mortalidad (son la quinta causa de muerte en adultos mayores).

Las caídas no se deben confundir con un proceso normal de envejecimiento. Se consideran un factor de fragilidad en el anciano. A medida que aumenta la edad, aumenta el riesgo de caída y la gravedad de esta El mayor factor de riesgo de caída es la caída previa. También son factores de riesgo importantes la edad, polifarmacia, los trastornos de la marcha y del equi librio Pueden tener consecuencias físicas producto del golpe (contusión, fracturas) y producto de tiempo que permanece el adulto mayor en el suelo (hipo termia, deshidratación, rabdomiólisis, úlceras por presión, trombosis venosa profunda). Tienen conse cuencias psicológicas (síndrome post caída, pérdida de la autonomía) y también socioeconómicas . Lo más importante es la prevención.

Etiología En la tabla de la página siguiente, se encuentran las causas y cómo hacer la distinción entre ellas:

Caso Clínico Tipo Paciente de 82 años en tratamiento farmacológico por HTA, DM2, dislipidemia, arritmia. Acude a su consultorio por problemas para dormir y se le receta clonazepam. Dos días después se despierta en la noche para ir al baño y sufre caída. Refiere haber tropezado con algo, pero no recuerda qué.

Definición La caída es un evento no intencional, caracterizado por la pérdida de la estabilidad y un desplazamiento del centro de gravedad hacia un nivel inferior, sin el antecedente de pérdi-

1 Facultad de Medicina, Universidad de chile

Diagnóstico Ante el antecedente caída, se debe evaluar caídas previas, las circunstancias en que ocurrió la caída. Revisar los medicamentos que consume, déficits cognitivos, depresión, En el exámen físico, se debe evaluar la presencia de consecuencias físicas por el golpe. Se debe pesquisar alteraciones neurológicas, hipotensión ortostática (Test de Schellong), déficits

visuales, trastornos de la marcha y equilibrio. En el Examen Anual de Medicina Preventiva del Adulto Mayor (EMPAM) se evalúa el equilibrio estático (Estación Unipodal, normal >5 seg) y dinámico (Timed Up and Go, normal <19 seg). Si hay alteraciones, es visto por médico y se evalúa la asistencia a Taller de Prevención de caídas.

2 Facultad de Medicina, Universidad de chile

Tratamiento Los pilares del tratamiento son la valoración médica y funcional, ejercicio físico para mejorar fuerza y equilibrio, ajuste de medicamentos y modificación del entorno del paciente. Uso de vitamina D en pacientes con déficit y en institucionalizados. Otras intervenciones, incluyen manejo de la hipotensión ortostática (hidratación, ajuste medicamentos antihipertensivos, medias compresivas), corrección déficit visual, tratamiento de osteoporosis, manejo de problemas de los pies y uso calzado apropiado (sin taco, base amplia).

Seguimiento Derivar. Autor / Editor Nevelline Salgado

Año 2016

3 Facultad de Medicina, Universidad de chile

Constipación y Fecalomas

pación aumenta después de los 65 años observándose con mayor frecuencia en mujeres, equiparándose la prevalencia sobre los 70 años. La prevalencia en el adulto mayor no ha sido bien definida, los estudios reportan que varía entre un 24 y 50%. Los laxantes se utilizan a diario en un 10 y 18% de los adultos mayores que viven en la comunidad y en el 74% de los institucionalizados. La complicación más importante es el fecaloma. Éste suele aparecer con mayor frecuencia en los ancianos incapacitados o institucionalizados y en los enfermos terminales de cáncer. Sus localizaciones más frecuentes son la ampolla rectal (70%), el colon sigmoides (20%) y el colon proximal (10% de los casos)

Nivel de manejo del médico general: Diagnóstico Específico Tratamiento Completo Seguimiento Completo

Aspectos Esenciales • •

• •

Etiopatogenia

Se debe a disfunción colorrectal 1ª u obedece a causa 2ª. Revisar síntomas de alarma: inicio tardío de constipación, cambio en el perfil de su constipación habitual, sangre en las heces, pérdida de peso, anorexia y antecedentes familiares de cáncer de colon. Por alta asociación a cáncer colorrectal. Si se descarta causa 2ª, realizar manejo no farmacológico, si fracasa utilizar agentes osmóticos. Fecaloma principal complicación.

Factores de Riesgo para constipación. • Edad • Sexo femenino. • Inactividad física. • Bajo nivel socioeconómico y educacional. • Uso de medicamentos constipantes. • Depresión. • Consumo de menos calorías y comidas en los adultos mayores.

Caso Clínico Tipo

En los adultos mayores la constipación se debe a disfunción colorrectal primaria o es secundaria a varios factores etiológicos como:

Paciente de 70 años, DM2, consulta porque hace 3 meses presenta deposiciones una vez a la semana con sensación de evacuación incompleta y además nota presencia de sangre en las deposiciones. Al interrogatorio dirigido relata que presenta baja de aprox. 10 kilos en los últimos 3 meses sin motivo aparente.

• • •

Definición

• •

De acuerdo con los criterios Roma III, constipación se define como aquel paciente que no utiliza laxantes y refiere dos o más de los siguientes síntomas, que hayan aparecido al menos 6 meses antes y estar actualmente activos durante 3 meses: • Evacuaciones con gran esfuerzo, presente en más del 25% de los movimientos intestinales. • Deposiciones duras o caprinas en más del 25% de los movimientos intestinales. • Sensación de evacuación incompleta en más del 25% de los movimientos intestinales. • Necesidad de manipulación digital para facilitar la evacuación en más del 25% de los movimientos intestinales. • Presencia de defecación menor de 3 veces por semana. • La impactación fecal o fecaloma, consiste en una acumulación de heces endurecidas en el recto y/o en el colon que el paciente no puede expulsar es pontáneamente.

• •

Orgánico: cáncer colorrectal, masa extraintestinal, postinflamatorio, isquémico o estenosis quirúrgica. Endocrino-metabólico: DM, hipotiroidismo, hiperkalcemia, porfiria, ERC, panhipopituitarismo. Neurológico: Lesión de médula espinal, enfermedad de parkinson, esclerosis múltiple, neuropatía autonómica. Miogénico: dermatomiositis, esclerodermia, amiloidosis. Anorrectal: fisura anal, estenosis anal, enfermedad inflamatoria intestinal. Drogas: opiáceos, antihipertensivos, antidepresivos tricíclicos, preparados de hierro, antiepilépticos, antiparkinsonianos. Dieta o estilo de vida: Dieta baja en fibras, deshidratación, estilo de vida inactivo.

También se puede presentar como constipación idiopática crónica. En la mayoría de los casos presenta una etiología multifactorial. La patogenia de la impactación fecal no está completamente aclarada. En la literatura se informa que en gran medida se debería a la incapacidad de la persona para detectar y responder a la presencia de heces en el recto, a la disminución de la movilidad y a la disminución de la percepción sensorial.

Diagnóstico Epidemiología

El diagnóstico de constipación se realiza con la historia clínica, revisando los criterios Roma III mencionados en la definición, una vez definido que el adulto mayor presenta constipación, lo primero que se debe hacer es descartar causa secundaria, para ello existen ciertos síntomas y signos de

Estudios epidemiológicos revelan que entre el 12 y el 30% de la población presenta o ha presentado constipación, reportando una prevalencia de 18% en Sudamérica. La consti-

4 Facultad de Medicina, Universidad de chile

alarma: constipación de inicio tardío de o cambio en su perfil de constipación habitual, sangre en las heces, pérdida de peso, anorexia y antecedentes de cáncer de colon y enfermedad inflamatoria intestinal en familiares.



Anamnesis. Interrogar sobre el síntoma predominante o la presencia de síntomas asociados como dolor o distensión que alivian con la defecación. La disfunción del piso pelviano se sospecha si hay esfuerzo excesivo y prolongado antes de defecar, uso de supositorios o enemas, necesidad de compresión vaginal, perineal o digitación rectal para el paso de las heces. También preguntar con qué frecuencia siente el deseo de evacuar y si se atiende tal necesidad. Interrogar al paciente sobre el uso de laxantes: tipo, frecuencia, dosis y efecto y consignar todos los medicamentos que consume. Es necesario realizar una buena historia farmacológica, enfocándose en el uso de fármacos constipantes, como opiáceos, hidróxido de aluminio, calcio u otros.

puede aumentar hasta 2 veces al día. El tratamiento del fecaloma, puede ser conservador (fleet enemas (sin fosfato de sodio), proctoclisis, maniobras manuales bajo anestesia) o quirúrgico (evacuación manual, resección o colostomía en caso de perforación e isquemia intestinal).

Seguimiento Descartada una causa mecánica o secundaria de constipación, se debe ofrecer un período razonable (3 meses) de seguimiento para educación y modificación de hábitos que puedan favorecer un tránsito intestinal “normal”. Menos del 1% de los pacientes que consultan por esta causa, no responde a las medidas previas señaladas y ameritan mayor estudio (ej. estudio funcional de colon y del piso pélvico).

Examen Físico general, rectal y perineal. En búsqueda de cicatrices, fisuras, fístulas, etc. Incluye siempre el tacto rectal en busca de masas o impactación fecal.

Autor / Editor Julián Bravo

Exámenes adicionales. Función tiroidea, calcemia, glicemia, electrolitos plasmáticos, creatininemia, hemograma, VHS y test de sangre oculta en deposiciones. Otros exámenes de resorte de especialista: Tiempo de tránsito colónico con marcadores, Manometría ano-rectal, Test de expulsión del balón, Defecografía. Ante sospecha de cáncer de colon, siempre realizar colonoscopía. El cuadro clínico del fecaloma es muy variable y, a veces, muy escaso en síntomas. Se suele manifestar en el contexto de un cuadro de estreñimiento crónico y puede producir dolor cólico en hipogastrio, anorexia y deterioro del estado general acompañado de diarrea por rebosamiento con incontinencia fecal (paradójica). El diagnóstico del fecaloma se basa en la anamnesis y la exploración física: masa palpable, fija, no dolorosa y tacto rectal con tumoración pelviana. Rx simple de abdomen muestra signos de obstrucción intestinal baja y moteado característico.

Tratamiento Tratamiento de constipación. No farmacológico: • Dieta rica en fibra, de preferencia insoluble (20-25 gr/ día). • Abundantes líquidos (1-2 litros/día). • Ejercicio físico regular. Potenciar prensa abdominal y piso pélvico. • Educación: Aprovechar reflejo gastrocólico (tras comer), en general se recomienda realizar la rutina de defecar 2 veces al día, 30 minutos después de las comidas haciendo esfuerzo por no más de cinco minutos. También se recomienda una posición adecuada (pies bien apoyados). Para los pacientes con disfunción del piso pélvico una opción podría ser el biofeedback. En caso de fallar estas medidas, se recurre al tratamiento farmacológico: • Laxantes, inicialmente osmóticos: La terapia inicial con fibra en la dieta, asociada eventualmente a laxantes osmóticos suaves (Lactulosa/PEG) o lubricantes. Dosis: PEG 17 gr al día en 240 ml de líqudos. Lactulosa 10-20 gr (15-30 ml) cada dos días,

5 Facultad de Medicina, Universidad de chile

Año 2016

Demencia

metabólicas, demencia por cuerpos de Lewy.

Diagnóstico

Nivel de manejo del médico general: Diagnóstico Sospecha Tratamiento Inicial Seguimiento Derivar

El diagnóstico es clínico, se debe indagar información de cuidador o familiar. En general los pacientes con demencia son traídos por familiares, porque no se dan cuenta de sus dificultades o tienen a minimizarlas. Los pacientes que se quejan de memoria, pueden ser depresivos o ansiosos. Algunos síntomas en diferentes dominios puede ser sugerentes de demencia:

Aspectos Esenciales • • • • •

Es un síndrome clínico que se caracteriza por el deterioro cognitivo adquirido, presente al menos durante 6 meses, y que interfiere en la calidad de vida. Se necesita corroborar la información que entrega el paciente con un cuidador o familiar. El principal diagnóstico diferencial de demencia, es el delirium, que se caracteriza por el déficit atencional y por tener curso agudo, fluctuante y reversible. Las causas de demencia pueden ser reversibles (10%) o irreversibles Además de déficit cognitivo, se asocia a alteraciones conductuales (depresión, insomnio, vagabundeo, delusiones y psicosis).

• • • • • •

Caso Clínico Tipo Paciente 70 años, llevado por familiares que refieren olvidos frecuentes, asociado a conductas sociales inadecuadas, afectando actividades que previamente realizaba.

Memoria. Tienden a repetir las cosas, les cuesta recordar conversaciones recientes, eventos, cosas que tienen que hacer, ponen objetos en lugares incorrectos. Ejecutiva. Les cuesta realizar actividades complejas (de muchos pasos) tal como llevar las cuentas de la casa, cocinar, etc. Razonamiento. Frente a problemas, no son capaces de responder con un plan para enfrentarlo. No les importan las reglas de conducta social. Orientación en el espacio. Les cuesta manejar, se pierden en la calle y en lugares frecuentes. Lenguaje. Tienen dificultad para encontrar la palabra adecuada en una conversación. Alteraciones del comportamiento, aparece pasividad, irritabilidad, puede malinterpretar estímulos visuales o auditivos. Puede cambiar el hábito de vestirse, volverse desaseado, desordenado, llegar tarde a la hora de la consulta.

Se debe indagar caídas (hematoma subdural), incontinencia urinaria, alteraciones motoras, comorbilidades (HTA, depresión, OH crónico), fármacos (pueden producir deterioro cognitivo los anticolinérgicos, analgésicos, psicotrópicos y sedantes).

Definición El deterioro cognitivo significativo, adquirido, de al menos 6 meses, que interfiere en las actividades de la vida diaria (AVD). Afecta diferentes áreas cognitivas, como el aprendizaje, memoria, lenguaje, función ejecutiva, atención compleja, visopercepción, visoconstrucción, cognición social.

Para realizar el examen mental se recomienda el Mini-Mental de Folstein que se puede ajustar según nivel educacional y edad. Si la atención está alterada, no se recomienda realizar otras pruebas cognitivas puesto que resultan alteradas En el examen físico se debe buscandor reflejos arcaicos (prehensión palmar, succión), y focalidades que puedan ser sugerentes de demencia vascular (AVE).

Epidemiología Es una patología que se ve principalmente en los adultos mayores. Afecta aproximadamente a un 5-8% de los individuos mayores de 65 años. La prevalencia se va duplicando cada 5 años, alcanzando un 15-20% en los mayores de 75 años y un 25-50% en los mayores de 85 años.

La evaluación de laboratorio inicial incluye, hemograma y VHS, perfil bioquímico (calcio, fósforo, pruebas hepáticas, glicemia, función renal), ELP, TSH, niveles de Vitamina B12. También serología VIH, tamizaje de neurosífilis. Otros estudios, van orientados según la sospecha clínica, comola medición de metales pesados, estudio LCR, EEG, etc.

Etiología

Tratamiento

Causas reversibles: hipotiroidismo, déficit de vitamina B12, vasculitis cerebral, neurosífilis, VIH. Causas irreversibles: Enfermedad de Alzheimer, demencia vascular, degeneración del lóbulo frontal, pseudodemencia depresiva, hidrocefalia normotensiva, hematoma subdural o tumores cerebrales, encefalopatía por VIH, meningitis crónica, enfermedad de Creutzfeldt-Jakob, enfefalopatías tóxico

La implementación de un manejo multidisciplinario, tanto farmacológico como no farmacológico es importante ya que ha demostrado contribuir significativamente a disminuir el impacto de las demencias en los pacientes y su entorno. Manejo no farmacológico destaca: • Vivienda segura para la movilización

6 Facultad de Medicina, Universidad de chile

• • • • • • • •

Mantener una adecuada alimentación e hidratación. Evitar la polifarmacia. Eliminar factores que puedan agitarlos y precipitar conductas agresivas. Estimulación cognitiva, de orientación y coordinación, para fomentar autonomía en AVD. Reeducación de esfínteres Actividad física. Prevenir infecciones (ya que la causa más frecuente de muerte son las neumonías aspirativas asociadas a alteraciones deglución) Es importante el apoyo a la familia y al cuidador (Sd. del cuidador o “burnout syndrome”).

Manejo farmacológico: • Orientado al manejo del déficit cognitivo con fármacos que aumentan la neurotrasmisión colinérgica (tacrina, donepezil, rivastigmina) • Orientado al manejo de las alteraciones conductuales - Depresión (ISRS) - Insomnio (higiene del sueño, hipnóticos de vida media corta) - Vagabundeo (identificación con dirección, teléfono, etc) - Psicosis (neurolépticos a dosis bajas)

Seguimiento Derivar.

Autor / Editor Hernán Rubilar

Año 2016

7 Facultad de Medicina, Universidad de chile

Depresión en el Adulto Mayor

alina y dopamina en el sistema nervioso central. Además existen alteraciones neuroendocrinas (aumento en la secreción de cortisol, con alteración del ciclo circadiano), modificaciones neuroanatómicas y desarreglos de interacción entre el sistema inmune y neuroendocrino (citoquinas activan el eje hipotálamo-hipósis-suprarrenal y alteran la neurotransmisión).

Nivel de manejo del médico general: Diagnóstico Sospecha Tratamiento Inicial Seguimiento Derivar

Son factores asociados especialmente en adultos mayores: Ser divorciado o separado, nivel socioeconómico bajo, escaso apoyo social, presencia de acontecimientos vitales recientes adversos e inesperados y problemas de salud que dan lugar a incapacidad (patologías neuronales, endocrinas, respiratoria, cardiaca, tumoral, etc.). Un tercio de las demencias coexisten con depresión y hasta un 50% de los pacientes tiene al menos, un síntoma depresivo.

Aspectos Esenciales • • • • •

Definición y criterios son iguales en todo grupo etario según DSM-V. Es el trastorno afectivo más frecuente en el anciano. Importante asociación con demencia. Tratamiento de elección son los ISRS. Tratamiento debe durar 18 meses post-desaparición de los síntomas y toda la vida si es una recaída.

Diagnóstico Los criterios diagnósticos son idénticos para todos los grupos etarios, y fueron mencionados en la definición, sin embargo, podemos encontrar singularidades en la presentación en el adulto mayor: • Acentuación patológica de los rasgos premórbidos. • Escasa expresividad de la tristeza. • Tendencia al retraimiento y aislamiento. • Dificultad para reconocer los síntomas depresivos. • Deterioro cognitivo frecuentemente asociado. • Expresión en forma de quejas somáticas. • Mayor frecuencia de síntomas psicóticos. • Posible trastorno de la conducta. Irritabilidad. • Presencia de comorbilidad. Pluripatología. • Enfermedad cerebrovascular frecuente. • Presencia de Polifarmacia.

Caso Clínico Tipo Paciente de sexo femenino de 83 años, con antecedentes de HTA, diabetes mellitus 2 y amputación diabética de los cinco ortejos del pie derecho, con limitación en la movilidad y mala adherencia a tratamiento, asiste a control de rutina acompañado por su cuidadora. Refiere ánimo depresivo todo el último año, anhedonia, insomnio, baja de peso y sentimientos de inutilidad.

Definición

Cuando la depresión se asocia a demencia hay que considerar algunos signos que pueden ser orientadores, como:

Una de las definiciones más utilizadas es la del DSM-V: cinco o más de los siguientes síntomas por al menos dos semanas: Ánimo deprimido y/o anhedonia (al menos uno de los dos), cambios de peso o cambios en apetito, alteraciones del sueño, agitación psicomotora o enlentecimiento, fatiga, sentimientos de inutilidad o culpa excesivos, alteraciones en la concentración y pensamientos recurrentes de muerte. Estos síntomas provocan malestar clínicamente significativo o deterioro en áreas sociales, ocupacionales u otras importantes del funcionamiento. El episodio no es atribuible a los efectos fisiológicos de una sustancia o enfermedad médica.

• • • • • • •

Agitación diurna. Lentitud del lenguaje. Retraso psicomotor. Despertar temprano. Ansiedad afectiva. Autoagresiones. Demanda excesiva de atención.

La historia clínica acerca de síntomas depresivos o el uso de escalas pueden ser de utilidad para el diagnóstico de depresión en el anciano. La Escala de Depresión Geriátrica de Yesavage es un instrumento frecuentemente utilizado para tamizaje de depresión en el adulto mayor, ha sido traducida y validada en diversos idiomas incluyendo el español. Consta de 30 preguntas de formato si-no. Existe una versión abreviada de 15 preguntas, la cual conserva la efectividad de la escala original, mejorando la facilidad de la administración.

Epidemiología Es el trastorno afectivo más frecuente en el anciano. Su prevalencia es falsamente baja (cercana al 6%), debido a la presentación inespecífica y atípica asociada a una reticencia a reconocer los síntomas. La prevalencia es mayor en institucionalizados y hospitalizados.

La importancia del diagnóstico radica en que es una condición que aumenta la morbimortalidad y causa sufrimiento al paciente. Cabe recordar que es un cuadro de difícil diagnóstico, dado su variabilidad de presentación, por lo que la sospecha clínica cumple un rol importante para el diagnóstico oportuno.

Fisiopatología En el desarrollo de la depresión existen diversos factores involucrados entre ellos los biológicos que se caracterizan disminución de neurotransmisores como serotonina, noradren-

8 Facultad de Medicina, Universidad de chile

Tratamiento El tratamiento es farmacológico. La elección de un fármaco debe considerar la severidad de los síntomas, la causa de la depresión y contraindicaciones. Los Inhibidores Selectivos de Recaptación de la Serotonina (fluoxetina, sertralina, paroxetina, citalopram y escitalopram) son de primera elección, administrándose en una única dosis diaria. También es posible usar Mirtazapina (única dosis diaria nocturna) o Venlafaxina (especialmente útil en pacientes con patologías álgicas de base y con demencia). El tratamiento debería mantenerse 18 meses después de desaparecer la sintomatología en un primer episodio y mantenerlo de por vida si hay recaída.

Seguimiento Derivar a especialista (Psiquiatra).

Autor / Editor Julián Bravo

Año 2016

9 Facultad de Medicina, Universidad de chile

Diabetes Mellitus II Nivel de manejo del médico general: Diagnóstico Específico Tratamiento Completo Seguimiento Completo

familiar. El primer evento que conduce a esta patología es la resistencia insulínica generando un hiperinsulinismo compensatorio, lo que permite mantener glicemias normales por años, hasta que la célula beta pierde su capacidad para mantener la hiperinsulinemia generando un déficit relativo de insulina. Finalmente aparece hiperglicemia. Factores que predisponen a la resistencia a la insulina son: sedentarismo, dietas ricas en carbohidratos y grasas y pobre en fibras y obesidad (factores ambientales).

Aspectos Esenciales

Diagnóstico

• • •

Síntomas clásicos (polidipsia, poliuria y baja de peso) + glicemia en cualquier momento ≥ 200 mg/dl. Sin síntomas clásicos como parte de un screening cada 3 años en mayores de 45 años, o a edades más tempranas en pacientes con factores de riesgo: Glicemia en ayunas ≥ 126 mg/dl en al menos 2 oportunidades. Prueba de tolerancia a la glucosa oral con resultado 2 hrs. postcarga ≥ 200 mg/dl.

Diagnóstico basado en síntomas y valores de glicemia y PTGO; HbA1c no validada aún en Chile como método diagnóstico. Tratamiento con cambio del estilo de vida y fármacos, de primera línea metformina. De segunda línea glipizida o insulina. Pacientes mucho más vulnerables a todo tipo de complicaciones, propias de la enfermedad, del tratamiento utilizado (hipoglicemia) y de las comorbilidades asociadas.

Tratamiento Caso Clínico Tipo

No existen diferencias respecto al manejo de pacientes jóvenes según las guías del MINSAL. En las guías internacionales se hace énfasis en un manejo no farmacológico y farmacológico, siendo especialmente cuidadosos con el riesgo de hipoglicemia, el manejo nutricional (obesidad v/s desnutrición) y las interacciones entre medicamentos dada la polifarmacia de estos pacientes.

Paciente adulto mayor, en general con diabetes de larga data, lo cual lleva a una mayor frecuencia de complicaciones tanto micro como macrovasculares, asociado a contexto sindromático (deterioro cognitivo, depresión, polifarmacia, caídas, incontinencia) que se ve afectado tanto por las consecuencias de la hiperglicemia como de la hipoglicemia.

No farmacológico: Modificación de estilo de vida: dieta, baja de peso (5% del peso corporal), ejercicios.

Definición

Farmacológico: El tratamiento farmacológico debe ser individualizado, basado en las habilidades de los pacientes y las comorbilidades. Se recomienda iniciar con las dosis más bajas y escalar lentamente. Es preferible el uso de metformina asociado a modificación del estilo de vida, si no se logran las metas se debe revisar la adherencia y las RAM y se puede agregar sulfonilureas de acción corta, glipizida (también se utiliza cuando la metfornina esta contraindicada o existe intolerancia gastrointestinal). Las sulfonilureas de acción prolongada (clorpropamida y glibenclamida) se asocian con un mayor riesgo de hipoglicemia, por esta razón, debiesen estar contraindicadas en adultos mayores. La dosis inicial de metformina es de 500 mg cada 12 hrs o 850 mg al día, titular con aumento de 500 mg cada semana o 850 mg cada dos semanas, la dosis máxima es de 2550 mg al día, con VFG< 45ml/ min no iniciar metformina. La dosis inicial de glipizida es de 2.5 mg al día y titular 2.5 a 5 mg al día en un intervalo de 1 a 2 semanas, si VFG< 50 ml/min disminuir dosis a la mitad.

Enfermedad metabólica caracterizada por insulinoresistencia y deficiencia no absoluta de insulina generando hiperglicemia crónica que trae como consecuencia daño a nivel micro y macrovascular.

Epidemiología En la encuesta nacional de salud 2009-2010 se informa que en la proyección INE para el 2010 el 12% de la población corresponde a mayores de 65 años. Dentro de este grupo etáreo la prevalencia de Diabetes fue de un 25.8%. Los adultos mayores con diabetes corren el riesgo de desarrollar un espectro similar de complicaciones macrovasculares y microvasculares, como sus contrapartes más jóvenes. Sin embargo, el riesgo absoluto de enfermedad cardiovascular es mucho mayor que los adultos más jóvenes. En Chile el 2007 hubo 1503 defunciones por diabetes tipo 2, 89.3% de éstas ocurrieron en personas mayores de 60 años.

La insulina también es usada en caso de contraindicaciones. Específicamente, las indicaciones para manejar una Diabetes Mellitus 2 con Insulina son: HbA1c>8.5%, hiperglicemia sintomática, fracaso de la terapia oral, enfermedad aguda con necesidad de cirugía, cirugía cardiovascular o IAM, e ingreso a UCI.

Etiopatogenia Presenta una naturaleza genética, demostrada por la alta concordancia entre gemelos (mayor al 90%) y por su transmisión

Metas: En pacientes ancianos con expectativas de vida may-

10 Facultad de Medicina, Universidad de chile

or de 10 años, la meta es obtener una HbA1c< 7%, y con expectativas de vida menor a 10 años o muy frágiles con comorbilidad médica y funcional, HbA1c < 8%. Manejo de comorbilidades y complicaciones: Al diagnóstico, se debe hacer screening de patología microvascular (retinopatía, nefropatía y pie diabético) y manejo del riesgo cardiovascular, con cese del tabaco, baja de peso, manejo de HTA/dislipidemia y uso de aspirina.

Seguimiento Completo

Autor / Editor Paulina Lira

Revisor de Formato Verónica Vasquez

Año 2016

11 Facultad de Medicina, Universidad de chile

Enfermedad Cerebro Vascular Nivel de manejo del médico general: Diagnóstico Sospecha Tratamiento Inicial Seguimiento Derivar •

Aspectos Esenciales • • • • • • •

diovascular y antecedentes de AIT. - Embólico: favorecidos por fibrilación auricular y valvulopatías. El cuadro es brusco, y puede haber transformación hemorrágica. - Lacunar: secundario a HTA o diabetes (microateromas, lipohialinosis), ocurre en regiones cerebrales profundas. Pueden ser causa de demencia vascular. - Trombosis venosa por deshidratación, traumatismos craneales, trombofilia, o paraneoplásico. HEMORRÁGICOS: son el 20 a 30% de los ictus cerebrales. Puede ser intraparenquimatosa (HTA, angiopatía amiloide), o subaracnoidea (rotura de aneurisma, no traumática).

Diagnóstico

Primera causa de discapacidad. La mayoría son isquémicos. Inicio ictal. Sospecha en todo paciente con déficit neurológico o compromiso de conciencia de inicio brusco. TAC de cerebro sin contraste precoz es el examen más importante. Tratamiento AAS + trombolisis en isquémicos, control de la PA en hemorrágicos. Importancia del seguimiento y de la prevención secundaria.

Sospechar en todo paciente con déficit neurológico o compromiso de conciencia de inicio brusco. La clínica depende de la localización, de la extensión y de la etiología. Exámenes: TAC de cerebro sin contraste para diferenciar entre hemorrágico o isquémico, ECG (descartar IAM o FA), glicemia, VHS, ELP, crea, uremia, pruebas de coagulación, y perfil hematológico. Si sospecha HSA con TAC no compatible: Punción lumbar (xantocromía) y angiografía de 4 vasos. En caso de duda diagnóstica: RM.

Caso Clínico Tipo

Tratamiento

Hombre de 85 años con antecedentes de diabetes e hipertensión arterial ingresa por un cuadro agudo de hemiparesia derecha predominante en extremidad superior, asociada a una afasia. El TAC cerebral muestra un área de hipodensidad parietal izquierda.

Epidemiología

Es una urgencia médica: (Tiempo es cerebro), por lo que se debe dar aviso precoz y traslado a un centro de complejidad adecuada para evaluación por neurólogo. En el intertanto, se debe establecer el tiempo de inicio, mantener al paciente con solución fisiológica 60-100 ml/hr (evitar suero glucosado), saturación >93%, tratar fiebre, evitar glicemia bajo 140, prevenir aspiraciones, PAD 105 PAS 140 -180 (uso de Captopril o labetalol si PAS >230 o PAD >140, IAM, disección aórtica, EPA, trombolisis), no usar heparina. En accidentes isquémicos: AAS 250 mg vo dia, trombolisis: rt-PA e.v menos de 4,5 horas tras el inicio de los síntomas o i.a. en menos de 6 horas, pero existe un riesgo de transformación hemorrágica. En accidentes hemorrágicos: control de la HTA. Idealmente hospitalización en una unidad de tratamiento del accidente vascular (UTAC). Inicio de rehabilitación en forma precoz.

La ECV es la 2° a 3ª causa de muerte en el mundo, pero la 1° en mayores de 80 años, y la 1° causa de discapacidad. La mortalidad es de 10 a 25%.

Seguimiento

Definición Conjunto de trastornos de la vascularización cerebral que producen una disminución brusca, transitoria o permanente, del flujo sanguíneo cerebral que se traduce en un déficit neurológico focal o global.



Etiología •

• •

ISQUÉMICOS: Corresponden al 70 a 80% de los itus cerebrales: global por falla hemodinámica o focal. Estos incluyen el accidente isquémico transitorio (AIT): déficit focal brusco, reversible en menos de 24 horas (Ej. Amaurosis fugax); y el infarto cerebral (>24 horas) también llamado ACV. Causas: - Trombóticos: pacientes con factores de riesgo car-



Derivar. Prevención Secundaria: Manejo de los factores de riesgo cardiovascular modificables. Si ictus isquémico de origen aterotrombótico: antiagregantes plaquetarios. En caso de ictus de origen cardioembólico: anticoagulantes orales. Endarterectomía carotídea en pacientes con AIT y estenosis carotídea ipsilateral mayor a 70%. Mantener terapias de rehabilitación.

Autor / Editor Julián Bravo

12 Facultad de Medicina, Universidad de chile

Revisor de Formato Angel Eloy

Año 2016

Hipertensión Arterial

un riesgo de muerte cardiovascular tres veces mayor al adulto mayor normotenso

Nivel de manejo del médico general: Diagnóstico Específico Tratamiento Completo Seguimiento Completo

Fisiopatología Con el envejecimiento, aumenta la rigidez en las arterias y la resistencia periférica, esto produce una disfunción endotelial que es el elemento crucial en la patogenia de la HTA. Además, con el envejecimiento se produce un incremento en la actividad del sistema nervioso simpático secundario a una disminución de los barorreceptores y a una disminución en la actividad de la renina plasmática, en la función renal y en la capacidad de la excreción de sodio.

Aspectos Esenciales • • • •



La HTA es frecuente en el adulto mayor y se asocia a mayor riesgo cardiovascular La HTA del adulto mayor se caracteriza por ser una HTA sistólica aislada con un aumento de la presión diferencial. Una HTA diastólica aislada en el adulto mayor debe hacer sospechar causas de hipertensión arterial secundaria. Se debe evaluar el inicio del tratamiento antihipertensivo y las metas en cada paciente. Ya que pacientes frágiles pueden no verse necesariamente beneficiados del tratamiento. Para realizar el diagnóstico se debe descartar la presencia de hipotensión postural, pseudohipertensión arterial, causas secundarias.

El aumento de la fuerza ejercida sobre la pared arterial, produce una hipertrofia de las células musculares lisas, aumento del colágeno en la pared arterial y esto lleva a un mayor deterioro de la función endotelial. Se acelera la inflamación vascular, la migración de células inflamatorias, la fibrosis de la pared, el estrés oxidativo y la apoptosis, todo lo cual favorece la aterosclerosis.

Diagnóstico Para el diagnóstico de hipertensión en el adulto mayor se deben tener en cuenta algunas particularidades de este. • Por la variabilidad de las cifras tensionales, para hacer el diagnóstico se necesitan 6 determinaciones (realizadas en dos visitas separadas) • Se debe descartar hipotensión postural (ver tema correspondiente 1.07.1.010) que se vería agravada con el tratamiento antihipertensivo • Descartar pseudohipertensión (que sería una falsa elevación de la PAS debida a la esclerosis de la pared arterial que impide su colapso por el manguito, se sospecha en pacientes que no responden al tratamiento antihipertensivo o que presentan síntomas de hipotensión, y que no presentan signos de repercusión de HTA) • Descartar hipertensión del delantal blanco (presente en un 15-25% de los adultos mayores) • Se deben indagar causas de hipertensión arterial secundaria (10% de los pacientes hipertensos). Esta se sospecha en HTA diastólica aislada que es infrecuente en el adulto mayor. Puede estar debida al uso de corticoides, AINEs, alteraciones renales (empeora la función renal con el uso de IECA), etc.

Caso Clínico Tipo •

Paciente varón de 82 años con cifras tensionales en torno a 160/70 mmHg, y se descarta hipotensión postural.

Definición Aumento de la presión arterial sistólica (PAS), disminución de la presión arterial diastólica (PAD), con aumento respectivo de la presión diferencial o presión de pulso (PP) en el adulto mayor.

Epidemiología • • • •



La prevalencia de HTA aumenta con la edad (un 70% de los mayores de 70 años son hipertensos) La HTA es ligeramente mayor en las mujeres que en los hombres La particularidad de la HTA en el adulto mayor es que tiene un predominio sistólico (un 90% de los hipertensos mayores de 70 años tienen HTA sistólica aislada). La presión arterial sistólica (PAS) aumenta de forma lineal hasta los 80 años y la presión arterial diastólica (PAD) comienza a disminuir hacia los 50-60 años. A causa de estas dos situaciones, la presión diferencial (o presión de pulso) que se calcula PP= PAS-PAD, pulso aumenta a partir de los 50 años La presión diferencial sería el mayor predictor de enfermedad coronaria e insuficiencia cardíaca congestiva en que adulto mayor. El adulto mayor hipertenso tiene

Tratamiento En el tratamiento se deben considerar los beneficios del tratamiento antihipertensivo por sobre la fragilidad del adulto mayor. Cuando tiene deterioro funcional importante, deterioro cognitivo avanzado, poca expectativa de vida, no se beneficiarán necesariamente del tratamiento estricto, todo esto se debe considerar al plantear el tratamiento (situación es semejante a el control de glicemia en el adulto mayor). Modificaciones en el estilo de vida (MEV): •

Reducción de peso en aquellos que superan un 10% su

13 Facultad de Medicina, Universidad de chile

• • • •

peso ideal. Ejercicio aeróbico. Reducción en la dieta de contenido de sal, grasas saturadas y colesterol. Suspensión del tabaquismo. Consumo moderado de OH.

Tratamiento farmacológico: •



Los diuréticos tiazídicos en dosis bajas han demostrado un perfil de seguridad en el adulto mayor, además de actuar de forma sinérgica con otros fármacos antihipertensivos. Los IECA y ARA II pueden provocar hipotensión al inicio del tratamiento en pacientes en tratamiento con diuréticos y se debe tener cuidado con la hiperkalemia que producen en pacientes con insuficiencia renal o consumo de AINEs.

Seguimiento Completo. Se debe derivar sospecha de hipertensión arterial secundaria. Autor / Editor Matilde Pedrero

14 Facultad de Medicina, Universidad de chile

Año 2016

Inmovilidad

Etiología Se produce disminución de la actividad metabólica y depósito de colágeno e infiltración fibrograsa de las articulaciones, que conduce a deformidades articulares, alteración de ligamentos y acortamiento de fibras, causando disminución de fuerza y resistencia muscular, contracturas y disminución de la movilidad.

Nivel de manejo del médico general: Diagnóstico Específico Tratamiento Completo Seguimiento Completo

Las causas obedecen a: Factores intrínsecos: • Enfermedades musculoesqueléticas (artritis, artrosis, osteoporosis, fractura de cadera) • Déficits sensoriales • Síndrome post caída Factores extrínsecos: • Hospitalizaciones • Ambiente que dificulta la movilización • Reposo • Fármacos • Calzado inadecuado

Aspectos Esenciales • • • • • •

El síndrome de inmovilidad es frecuente en los adultos mayores. Se produce por el envejecimiento, asociado a factores de predisponentes y desencadenantes. Las consecuencias del síndrome de inmovilidad pueden ser más severas que las causas que lo originaron, por eso es importante la prevención. Se debe evitar la mala prescripción del “reposo”, las actitudes sobreprotectoras con los adultos mayores, fármacos sedantes. Forman parte del tratamiento la terapia física, las ayudas técnicas, la prevención y el tratamiento de las complicaciones. No todos los pacientes inmovilizados se benefician de terapia física, por lo tanto el tratamiento debe ser individualizado.

Diagnóstico Considerar los pacientes con factores de riesgo de inmovilidad intrínsecos con mayor riesgo de inmovilidad, y factores extrínsecos que predisponen a ella. Se debe evaluar equilibrio, la marcha, capacidad para levantarse de una silla sin apoyarse. Se pueden usar escalas, como “Test up and go” o la Escala de Tinetti. Así como también se debe evaluar los sistemas comprometidos por la inmovilidad, y detectar complicaciones de esta. Los sistemas que se complican con la inmovilidad: • Cardiovasculares: hipotensión ortostática, TVP y TEP • Respiratorio: hipoventilación, desaturación, atelectasias y neumonías. • Gastrointestinales: anorexia, REG, constipación. • Musculoesquelético: pérdida de fuerza muscular, masa ósea, contracturas musculares, rigideces. • Endocrino: aumenta la resistencia a la insulina produciendo hiperglicemia. • Genitourinario: incontinencia urinaria, litiasis renal e ITU • Piel: úlceras por presión. • Nervioso: inestabilidad, depresión sensorial. • Psicológicos: depresión, dependencia y, aislamiento social.

Caso Clínico Tipo Varón 84 años, secuelado de ACV hace 6 meses con hemiparesia braquio crural derecha presenta disminución de ánimo y de apetito, constipación, y dolor muscular. Este problema se ha exacerbado durante el invierno, permaneciendo la mayor parte del tiempo acostado.

Definición Es un síndrome que se produce producto de cambios fisiopatológicos en múltiples sistemas, condicionado por el envejecimiento y desuso, y que se manifiesta como un deterioro funcional con limitación de la movilidad. Puede ser agudo o crónico, relativo o absoluto y esto va a depender de las causas que lo producen, la severidad, tiempo de duración, estado previo del paciente y las comorbilidades asociadas.

Tratamiento Lo más importante es la prevención de la inmovilidad. Se debe detectar los factores intrínsecos que conllevan a la inmovilidad, y evitar los factores externos. En Chile, el GES incluyen las órtesis o ayudas técnicas en mayores de 65 años. Una vez instaurado el síndrome de inmovilidad, el tratamiento va principalmente enfocado a un plan rehabilitación física que permita recuperar, compensar o adaptarse a pérdida de funcionalidad. También a prevenir y tratar las complicaciones. Además de la terapia física que incluye trabajar con el paciente encamado, y favorecer la sedestación, transferencias, bipedestación y deambulación, se puede recurrir a ayudas técnicas como bastones con uno o más puntos de

Epidemiología Un 60% de los pacientes adultos mayores que ingresan a hospitalización inician alguna dependencia en las actividades de la vida diaria. La inmovilidad aguda aumenta la mortalidad (30% fallece a los 3 meses, 50% fallece al año) Un 7% de los ancianos se encuentran postrados.

15 Facultad de Medicina, Universidad de chile

apoyo, muletas, andadores fijos o con ruedas, sillas de rueda, colchones antiescaras, etc. Es importante considerar que en pacientes con debilidad muscular extrema, dolor no controlado, falta de motivación en enfermos terminales, miedo a la movilización, etc, no se les debe forzar, ya que la terapia podría tener más efectos adversos (cansancio extremo, HTA, IAM, muerte súbita).

Seguimiento Completo

Autor / Editor Paulina Lira

Año 2016

16 Facultad de Medicina, Universidad de chile

Evento Coronario En Adulto Mayor



Nivel de manejo del médico general: Diagnóstico Específico Tratamiento Completo Seguimiento Completo



Seguimiento Evaluar riesgo y función ventricular: ergonometría, ecocardiografía, coronariografía, ecocardiografía de estrés farmacológico, cintigrafía miocárdica-SPECT, Holter de ritmo. Prevención secundaria: Ejercicios, dieta, control de lípidos y suspender tabaquismo. Terapia antiagregante plaquetaria, beta bloqueadores, IECA o ARA II (en FE < 40%), estatinas, anticoagulación (HNF, ajustar dosis según peso o HBPM, ajustar dosis según Creatinina).

Aspectos Esenciales • •

Fármacos: AAS (500 mg a masticar), Clopidogrel (dosis de carga 300 mg en < 75 años), beta-bloqueadores, Nitroglicerina, Oxígeno por naricera (saturar > 90%), Morfina (excepto en pacientes hipotensos y en Infarto derecho). Terapia de reperfusión (trombolisis vs angioplastía).

La mortalidad en pacientes mayores de 65 años aumenta en comparación con pacientes menores. El cuadro se debe sospechar y confirmar con laboratorio y ECG dada la posibilidad de ausencia de síntomas clásicos de SCA.

Caso Clínico Tipo Paciente 68 años, con antecedentes de HTA hace 7 años, IPA 40 y sobrepeso. Consulta por dolor opresivo de pecho de intensidad leve de 2 horas de evolución, sin irradiación acompañado de náuseas por lo que consulta en servicio de urgencia.

Autor / Editor Marco Ayala

Definición Podemos definir Síndrome Coronario Agudo (SCA) como un episodio agudo de isquemia miocárdica.

Fisiopatología El SCA es causado por la oclusión parcial o total de una arteria coronaria, generalmente por la ruptura del endotelio en relación a una placa ateroesclerótica. A mayor edad del paciente la mortalidad por SCA aumenta. Esto está dado por la presentación atípica del cuadro en pacientes mayores, la presencia de comorbilidades, disminución de la reserva miocárdica, disminución de la circulación colateral, disfunción endotelial y una compliace ventricular disminuida.

Diagnóstico Anamnesis y examen físico: Uno de los síntomas clásicos de IAM, el dolor opresivo, puede estar ausente cuando se trata de un paciente mayor. EL SCA en pacientes de la tercera edad puede ser completamente asintomático (> 20%) o manifestarse sólo por disnea o fatiga. Es importante considerar que la manifestación del SCA puede estar dada solo por la descompensación de su patología de base. Otros síntomas que puede presentar un paciente de estas características son molestia abdominal, sudoración, palpitaciones y síncope. • • • •

Preguntar siempre por factores de riesgo cardiovascular. ECG: Inicialmente puede haber supra o infradesnivel ST, posteriormente aparece onda T invertida y Q patológica. Laboratorio: CK, CK-MB, troponina T, troponina I. Monitorizar y situar al paciente cerca de un desfibrilador.

17 Facultad de Medicina, Universidad de chile

Año 2017

Fractura de Cadera

Diagnóstico Anamnesis. A menudo se da en pacientes de edad avanzada, generalmente de sexo femenino, con grado variable de demencia y antecedente traumático. Presentan dolor de intensidad variable en la cadera afectada (inguino-crural irradiado a cara interna del muslo) e impotencia funcional (carga y movimiento). Examen físico. Extremidad afectada acortada y en rotación externa, con rango de movilidad limitado para rotación y flexión tanto pasiva como activa. Aumento de volumen, equimosis y hematomas, son signos tardíos. Exámenes. Radiografía de pelvis AP y axial. Pueden ser clasificadas mediante la radiografía en intracapsulares (medial) y extracapsulares (lateral), y de acuerdo al nivel del hueso. La curación de las fracturas intracapsulares se complica por la circulación terminal de la cabeza femoral, que si se comprometen los vasos puede llevar a necrosis avascular. Este problema no ocurre en las extracapsulares.

Nivel de manejo del médico general: Diagnóstico Sospecha Tratamiento Inicial Seguimiento Derivar

Aspectos Esenciales • • •

El 90% de estas fracturas se producen en mayores de 50 años. Al examen físico se aprecia la extremidad afectada acortada y en rotación externa. El tratamiento de elección es quirúrgico.

Caso Clínico Tipo

Tratamiento

Mujer de 75 años, con antecedentes de HTA, tabaquismo y DM2, consulta con extremidad inferior izquierda acortada y cadera en rotación externa luego de sufrir una caída a nivel en su domicilio.

Tratamiento general: Prevención. • Actividad física continua. • Prevención de caídas en el hogar. • Tratamiento de comorbilidades. • Resolución de déficit sensoriales. • Alimentación adecuada. • Exposición solar 15 minutos por día. • Promover independencia en ABVD. • Red de apoyo activa. • Una vez que la fractura ya ocurrió: Indicaciones médicas iniciales: Hospitalización, ABC del trauma, exámenes básicos y ECG. Reposo absoluto. KNT motora y respiratoria. • Régimen: según exámenes, plazo operatorio y comorbilidad. • Hidratación: vía venosa permeable con cristaloides según balance hídrico y condición hemodinámica. • Protección gástrica: evita gastritis medicamentosa y HDA. • Analgesia: idealmente endovenosa, suficiente para manejar el dolor. • Prevención de TVP: HBPM o HNF, idealmente post cirugía en contexto de cirugía precoz. • Profilaxis antibiótica: preoperatoria con cefazolina 1 gr cada 8 hrs por 3 veces. • Determinar el nivel de funcionalidad previo y comorbilidades para definir metas de tratamiento. Tratamiento específico. (manejo de especialista): El objetivo es devolver su funcionalidad previa. Salvo algunos casos, el tratamiento de elección es quirúrgico acompañado de una movilización precoz de la extremidad.

Definición Son aquellas que ocurren entre la epífisis proximal del fémur y un plano ubicado por debajo del trocánter menor.

Epidemiología La incidencia se incrementa con la edad, ocurriendo el 90% de ellas en mayores de 50 años. La edad media de presentación es de 80 años y cerca del 80% de los afectados son mujeres (2-3 veces). Pacientes institucionalizados tienen una incidencia 3 veces mayor de fracturas de cadera que aquellos que viven en la comunidad. La mayoría de las fracturas son resultado de caídas o tropiezos (86%), sin embargo cerca del 5% no tiene el antecedente de traumatismo. La tasa de mortalidad a un año luego de haber sufrido una fractura de cadera alcanza entre 15-20%, las fracturas más comunes son las de cuello femoral y las pretrocantereas que representan sobre el 90% del total. Factores de riesgo para sufrir fractura de cadera. • Historia de fractura de cadera materna. • Exceso de consumo de OH. • Exceso de consumo de cafeína. • Inactividad física. • Bajo peso corporal. • Estatura alta. • Fractura de cadera previa. • Uso de psicotrópicos. • Institucionalización. • Discapacidad visual. • Demencia.

Seguimiento Derivar

Autor / Editor Marco Ayala

18 Facultad de Medicina, Universidad de chile

Año 2017

Síndrome Confusional Agudo (SCA, Delirium)

Epidemiología • •

Nivel de manejo del médico general: Diagnóstico Específico Tratamiento Completo Seguimiento Derivar

• • • •

Aspectos Esenciales • • • • •

Es una alteración del nivel de conciencia y atención, de inicio agudo, curso fluctuante y de carácter transitorio. Puede ser de tipo hiperactivo, hipoactivo o mixto. El de tipo hipoactivo es infradiagnosticado y tiene mal pronóstico. Se produce por una noxa en un cerebro vulnerable, y la intensidad de la noxa es indirectamente proporcional al grado de vulnerabilidad. Aumenta la morbimortalidad y además las institucionalizaciones y costos en salud. Es prevenible en un 40% de los casos y las medidas son simples.

El delirium aumenta la mortalidad (un paciente que presenta delirium durante su hospitalización tiene una mortalidad de 10% a 40%) Un adulto mayor que presenta un episodio de delirium, tiene mayor riesgo de morbimortalidad En un 80% de los casos se puede encontrar la causa Un 40% de los casos se puede prevenir El delirium es más frecuente en las UPC (Unidades de Pacientes Críticos) La incidencia varía según el servicio en que se en cuentra hospitalizado el paciente (medicina, cirugía, neurolgía, etc) y también según la causa de hospitalización

Fisiopatología Se produce por una noxa en un cerebro vulnerable, por lo tanto puede ser producto de una noxa grande en un cerebro con un pequeño grado de vulnerabilidad, o una noxa pequeña en un cerebro con gran vulnerabilidad. En la siguiente tabla se encuentran los principales factores involucrados, cuanto más factores presentes haya, es mayor el riesgo de presentar delirium, por lo tanto no sólo se deben tener en cuenta, sino que evitar y tratar.

Caso Clínico Tipo Paciente de 84 años, hipoacúsico usuario de audífonos, usuario de lentes, en tratamiento para HTA, DM2, hipotiroidismo, es hospitalizado por una neumonía ATS III. Ingresa desorientado, se indica régimen cero por boca, medición de diuresis estricta con Foley y se inicia tratamiento con antibióticos endovenosos. Durante la noche evoluciona presenta gritos despertando a los otros pacientes de la sala, intentando levantarse, buscar a su mamá.

Hay muchos fármacos implicados, los siguientes son de uso frecuente en hospitalizados: • Ranitidina • Clorfenamina • Viadil • Furosemida • Metronidazol • Quinolonas (sobre todo el Moxifloxacino)

Definición El delirium o síndrome confusional agudo (SCA), es una alteración del nivel de conciencia y atención, que se caracteriza por tener un inicio agudo, un carácter fluctuante y ser transitorio (dura horas o días). Se debe preguntar en la anamnesis de los adultos mayores (así como se pregunta por antecedentes de IAM, ACV, HTA, DM) porque se asocia a mayor morbimortalidad incluso hasta años después del episodio, prolonga la hospitalización, genera mayor institucionalización, deterioro funcional, déficit cognitivo, y refleja mayor riesgo de presentar nuevos episodios.

Diagnóstico El diagnóstico de delirium es clínico, y basta con pesquisar la alteración del nivel de conciencia y atención. Para evaluar la atención, basta con pedirle que diga una serie y luego la invierta (días de la semana, no acepta errores; meses de año, acepta un error). Se debe preguntar a la enfermera cómo estuvo el paciente durante la noche. El delirium se puede presentar de tres formas: • Delirium hiperactivo (es el más frecuente, muy evidente, fácil de diagnosticar) • Delirium hipoactivo (es poco evidente, por lo que es infradiagnosticado, teniendo peor pronostico) • Mixto

Por lo tanto se debe considerar como una enfermedad, una urgencia médica, que tiene mayor riesgo de caídas, úlceras por presión, infecciones, sedación excesiva. Además, puede ser una forma de presentación de otras enfermedades como un IAM, estatus epiléptico no convulsivo.

19 Facultad de Medicina, Universidad de chile

Con el diagnóstico de delirum, se debe indagar la causa de este. Es importante tener en cuenta que las causas pueden ser múltiples. Siempre preguntar: • • •

¿Ha tenido deposiciones? (el fecaloma es una causa muy frecuente e infradiagnosticado, cuyo diagnóstico está al alcance del dedo) ¿Está orinando adecuadamente? (cuando ocupan muchos pañales por orinar frecuentemente distingir si es orina por rebalse en paciente con globo vesical) ¿Toma algo para dormir? (la suspención brusca de benzodiacepinas de uso crónico puede producir delirium).



Tratamiento Lo más importante es la prevención. Las medidas deben ir encaminadas a corregir o atenuar los factores implicados. • Revisar fármacos que usa el paciente. Evitar fármacos que desencadenen delirium. • Si usa benzodiacepinas de forma crónica no sus pender en la hospitalización. • Evitar la restricción de movilidad (la contención encruz produce mas delirium), si es necesario, preferir la contensión en faja. • Evitar el uso de sonda Foley y uso de pañales (a menos que sea extrictamente necesario). • Mejorar el tránsito intestinal. • Usar tapones de oído en la noche para evitar los ruidos excesivos (ruido de otros pacientes, ruido del personal, de los monitores sobre todo en UPC). • Corregir déficit visuales (traerle sus lentes, audífonos) • Orientar al paciente constamente, que haya un reloj, calendario. • No interrumpir el sueño, favorecer la oscuridad en la noche • Favorecer la presencia de familiares, traer foto de los hijos, la almohada que usa en la casa, etc. La contención física debe evaluarse siempre. NO DEJAR CONTENCIÓN SOS. Se usan fármacos sólo cuando hay agitación severa o alto riesgo de autoagresión. Se usan antipsicóticos (siempre buscando si por el uso se agregan síntomas de síndrome neuroléptico). • Haloperidol, iniciar con 1mg vo ó 2,5mg im. Máximo 5 mg/día. Al día siguiente se usa la mitad de la dosis, dividido en 2 tomas. • Risperdiona 0,25-0,5 mg • Quetiapina 12,5-25mg (OjO produce hipotensión).

Seguimiento Completo

Autor / Editor Marco Ayala

Año 2017

20 Facultad de Medicina, Universidad de chile

Escaras o Úlceras por Presión (UPP) Nivel de manejo del médico general: Diagnóstico Específico Tratamiento Inicial Seguimiento Completo

superficie externa por un período prolongado de tiempo, generando desde un eritema blanqueable en piel indemne hasta úlceras profundas que llegan hasta el hueso.

Epidemiología

Aspectos Esenciales • • • • • • •

La mayoría de los pacientes con UPP son mayores de 65 años. Las UPP son frecuentes en los pacientes hospitalizados (prevalencia 3-17%) y en institucionalizados (prevalencia 3-12%). La literatura informa que en más del 90% de los pacientes hospitalizados con úlceras por presión, la causa de hospitalización fue otra. Los sitios comprometidos en más del 80% de los casos son: sacro (el más frecuente), trocánteres, nalgas, maléolos externos y talones.

Son un indicador de calidad de atención hospitalaria. Ningún paciente hospitalizado debería desarrollar UPP durante su hospitalización. Las úlceras más frecuentes son en sacro, trocánteres, nalgas, maléolos externos y talones. La patogénesis está dada por la fuerza de compresión e isquemia. Lo más importante es identificar los factores de riesgo y la prevención. La medida preventiva más efectiva es el cambio de posición cada 2 horas. La clasificación más utilizada es la de la National Presures Ulcera Advisory Panel (grado I, II, III y IV) Las UPP grado I y II son de tratamiento médico, en cambio las UPP grado III y IV son de tratamiento quirúrgico.

Fisiopatología El factor causal primordial en la génesis de las úlceras es la fuerza de compresión (presión) ya sea de alta intensidad por un corto período o de baja intensidad por largos períodos, asociado a fricción y fuerzas de cizallamiento. Son más sensibles músculos y tejidos subcutáneos, que la piel. El efecto es mayor en los pacientes sentados. Los efectos negativos de la presión por compresión son exacerbados en presencia de fricción, humedad (orina y deposiciones) porque facilita la maceración de la piel, y fuerzas cortantes, (generadas por los tejidos sacrocoxígeos profundos). El segundo componente esencial en la patogenia es la isquemia debido a la compresión de capilares, que es mayor en las prominencias óseas. Son suficientes 2 horas de compresión para producir isquemia.

Caso Clínico Tipo Paciente de 86 años, hospitalizado por NAC ATS IV, lleva 2 meses hospitalizado, comienza con escara en región sacra. Pérdida cutánea limitada a dermis, de 3 cm de diámetro, de aspecto limpio.

Definición

Diagnóstico

Lesiones causadas por un trastorno de irrigación sanguínea y nutrición tisular como resultado de la compresión del tejido blando entre prominencias óseas o cartilaginosas y una

El diagnóstico es clínico. Se clasifican según National Pressures Ulcera Advisory Panel, en:

21 Facultad de Medicina, Universidad de chile

Tratamiento La prevención constituye la medida más importante. Los principios del tratamiento son los siguientes (tabla). Las úlceras grado I y II son de manejo médico. Las úlceras III y IV son de resorte quirúrgico.

Compendio Medicina Interna. C. Rozman. V edición. Elsevier

22 Facultad de Medicina, Universidad de chile

Los factores de riesgo son: • Fisiopatológicos -Lesiones cutáneas patológicas o por envejecimiento -Trastornos del aporte de oxígeno por insuficiencia vascular periférica, estasia venosa, trastornos cardiopulmonares. -Malnutrición por déficit o exceso -Trastornos inmunológicos por infecciones o cáncer -Alteraciones del estado de conciencia por confusión, coma, fármacos -Déficit motores por fracturas, secuelas de ACV -Pérdida de la sensibilidad termoalgésica •

Derivados del tratamiento -Reposo prolongado -Tratamiento inmunosupresor con corticoides, radioterapia o quimioterapia -Sondajes



Situacionales -Falta de higiene por uso de pañales -Arrugas en la ropa -Dolor que produce inmovilidad

De todas las medidas, el cambio de posición cada 2 horas, es lo más efectivo en la prevención puesto que evita el fenómeno fisiopatológico. Para disminuir la presión y fricción se puede usar dispositivos como los colchones “antiescaras” (se encuentran en el GES de Órtesis o ayudas técnicas en mayores de 65 años), almohadas entre rodillas y tobillos, etc Previo al tratamiento quirúrgico, se debe optimizar el aporte de calórico (25-35 Kcal/Kg/día) y proteico (1,5-3gr/Kg/día), lograr una albúmina plasmática >3g/dl. El aporte de vitamina C 1 gr/día es útil sólo cuando hay déficit, y el aporte de vitamina A 25.000 UI/día es beneficiosos, haya o no déficit. Además, la úlcera debe tener tejido de granulación, no estar infectada.

Seguimiento Completo.

Autor / Editor Julián Bravo

Revisor de Formato Angel Eloy

Año 2017

23 Facultad de Medicina, Universidad de chile

Fragilidad

mayor presenta 3 o más criterios alterados, con alta probabilidad estamos en presencia de fragilidad. • Baja actividad física. • Cansancio. • Pérdida de peso (> o igual a 5% del peso previo último año). • Pérdida de fuerza (por autopercepción o evaluado con dinamómetro). • Lentitud (por autopercepción, o >7 segundos para caminar 5 metros).

Nivel de manejo del médico general: Diagnóstico Específico Tratamiento Completo Seguimiento Completo

Aspectos Esenciales •

También se puede hacer screening de fragilidad en el EMPAM, aplicando EFAM. Adulto mayor con un puntaje igual o menor a 42 puntos, debe ser tratado como frágil.

La fragilidad -Es un síndrome asociado al envejecimiento. -Habla de vulnerabilidad ante diferentes estresores. -Implica mayor morbimortalidad, riesgo de dependencia, institucionalización, caídas, hospitalizaciones. -Se puede sospechar con el uso de los criterios de Fried o Puede ser prevenida, y tratada, y constituyen parte del tratamiento la actividad física, suplementos calórico-protéicos, vitamina D y la revisión de polifarmacia.

Además de hacer el diagnóstico de fragilidad, se debe hacer una anamnesis, examen físico y de laboratorio orientados a encontrar las causas de fragilidad en el adulto mayor. En el examen físico se debe dar énfasis fuerza (medición de fuerza del cuádriceps), equilibrio estático (prueba de Estación Unipodal), equilibrio dinámico (prueba de Timed up and go”), evaluación nutricional (peso, talla, IMC, circunferencia braquial, de cintura, pantorrilla y pliegues cutáneos con fin de estimar la masa muscular). Se deben solicitar exámenes de laboratorio. Hemograma, PCR, glicemia, TSH, perfil bioquímico, perfil lipídico, albúmina plasmática, creatinina, electrolitos plasmáticos, ECG y radiografía de tórax, también se pueden solicitar curva de insulina, gases arteriales, 25 OH vitamina D.

Caso Clínico Tipo Mujer de 78 años sin hábitos tóxicos ni alergias medicamentosas conocidas, con los siguientes antecedentes patológicos: prótesis de cadera derecha hace seis meses por fractura, tras sufrir una caída en su hogar. Desde la cirugía presenta tristeza, anhedonia, astenia, sensación de debilidad, anorexia con baja de peso de 9 kgs. Refiere miedo a caerse y a fracturarse de nuevo, motivo por el que no desea salir de su domicilio.

Esta evaluación está orientada a encontrar las causas más frecuentes de fragilidad, entre ellas hay causas fisiopatológicas, funcionales (sedentarismo, inmovilismo, desuso de extremidades), nutricionales (por exceso o déficit), sindromáticas (trastornos del ánimo, baja de peso, astenia, etc). De las causas fisiopatológícas, se encuentran EPOC, anemia, diabetes mellitus, hiperinsulinemia, insulinorresistencia, disminución de testosterona, disminución de vitamina-D, dolor, sarcopenia, alteraciones del equilibrio. Varias de estas tienen en común el aumento de las citoquinas, que conlleva a disminución de la masa muscular, alteración del sistema inmunológico, disminución de albúmina, anemia, anorexia, aumento de lipólisis.

Definición La fragilidad es un síndrome geriátrico producto de la disminución de reserva funcional (por el desgaste de múltiples sistemas fisiológicos) y la disminución de la resistencia frente a factores estresantes, estos últimos pueden ser físicos (infecciones, deshidratación, constipación, quirúrgicos), psicológicos (soledad, maltrato), o funcionales (dificultad para realizar actividades cotidianas).

Tratamiento

Epidemiología

• •





• • •

Al comparar mujeres y hombres de la misma edad, las mujeres tienen mayor índice de fragilidad. La fragilidad es mayor cuando hay bajo nivel educacional y bajos ingresos. La fragilidad es menor en aquellos adultos mayores con buena red social, buena alimentación La prevalencia aumenta con la edad (<10% entre 65-75 años, >30% en mayores 90)

Diagnóstico El diagnóstico de fragilidad es clínico. Una de las formas de diagnosticar, es usando los criterios de Fried. Si un adulto

Lo más importante es la prevención. Se debe identificar adultos mayores frágiles e iniciar medidas para revertir el proceso. Los pilares del tratamiento, incluyen: -Control de comorbilidades. -Actividad física, 45 minutos, 2 o 3 veces por semana, con calentamiento, ejercicios de resistencia y elongación. Aumenta la movilidad y funcionalidad en cuanto a las AVD, mejora la marcha, reduce el riesgo de caídas y mejora la densidad ósea. -Recomendaciones dietéticas, suplementos nutricionales -Vitamina D en pacientes con niveles bajos de 25 OH vitamina-D -Revisión de polifarmacia -Corrección de déficits sensoriales -Detectar y tratar deterioros cognitivos -Manejo del dolor

24 Facultad de Medicina, Universidad de chile

-Aumentar participación social del adulto mayor -Aumentar apoyo de cuidadores y familiares.

Seguimiento Completo.

Autor / Editor Nevelline Salgado

Año 2016

25 Facultad de Medicina, Universidad de chile

Malnutrición

de los problemas más frecuentes es la deshidratación.

Diagnóstico

Nivel de manejo del médico general: Diagnóstico Específico Tratamiento Completo Seguimiento Derivar

A través de parámetros antropométricos y exámenes de laboratorio es posible evaluar el estado nutricional objetivamente. Medir IMC; Antropometría: pliegue tricipital, provee un índice de la grasa corporal, y, circunferencia media del brazo, que provee una medida de la masa muscular. Evaluación Global Subjetiva (EGS); método clínico que reúne parámetros de antecedentes clínicos, síntomas, y examen físico. Hallazgos de la historia y el examen físico permiten clasificar a los pacientes en bien nutridos, levemente DN o severamente DN. Además se utilizan parámetros bioquímicos como albúmina (también prealbúmina), linfocitos transferrina y BUN. Solicitados en determinados casos (gastrectomizados): niveles de vitamina B12 y ác. Fólico.

Aspectos Esenciales • • • • •

Por déficit o exceso de alimentación. Ambas situaciones aumentan la morbimortalidad. Se debe diagnosticar con anamnesis y examen físico. Es importante el cambio del estilo de vida y los factores biopsicosociales asociados. Uno de los problemas más frecuentes en el Adulto Mayor es la deshidratación.

Tratamiento

Caso Clínico Tipo

La recuperación del desnutrido crónico requiere de un enfrentamiento multiprofesional. Debe corregirse los trastornos hidroelectrolíticos y ácido base. Es recomendable en un paciente muy depletado iniciar aportes cercanos a los requerimientos basales, y aumentarlos progresivamente de acuerdo a la tolerancia del paciente en base a fórmulas de fácil digestión, cuidando de suplementar simultáneamente vitaminas y minerales.

Paciente de 84 años, sexo masculino, con IC capacidad funcional III, HTA, DM y DHC. Institucionalizado. Es hospitalizado por fractura de cadera. Presenta IMC 17,5.

Definición Desbalance entre aportes y requerimientos de uno o varios nutrientes. Un déficit de nutrientes de corta duración sólo compromete las reservas del organismo, sin alteraciones funcionales importantes; en cambio, desnutrición de larga duración puede llegar a comprometer funciones vitales. Por el contrario, un exceso de alimentación aumenta el riesgo y morbimortalidad Cardiovascular. La desnutrición se determina por IMC < 18,5, y la obesidad por IMC ≥ 30.

Debe evitarse el “síndrome de realimentación” en casos de desnutrición severa, esta condición es provocada principalmente por descenso brusco de los niveles plasmáticos de potasio y fosfato, al ingresar éstos al compartimento intracelular. La vía oral siempre es de elección, pero debe recurrirse a la alimentación enteral continua cuando la tolerancia oral o la digestiva son malas. La alimentación parenteral está indicada si no es posible utilizar la vía digestiva, o los aportes por esta vía no son suficientes.

Fisiopatología

Es importante suplementar la nutrición con multivitamínicos, folato, hierro y zinc durante la recuperación nutricional, ya que los requerimientos de estos nutrientes aumentan, y hay deficiencias preexistentes de ellos. Puede requerirse también suplementos de cobre, carnitina, calcio, fosfato y magnesio. En los casos de obesidad se debe cambiar el estilo de vida, mejorar la alimentación e implementar ejercicio físico. En caso de obesidad mórbida o IMC ≥35 y morbilidad asociada, se indica cirugía bariátrica (si no presenta una enfermedad de riesgo vital en corto plazo o enfermedad psiquiátrica importante.

El balance energético de un individuo depende de su ingesta de energía y gasto energético: un desbalance entre estos factores, producirá un aumento o pérdida de masa corporal, principalmente masa grasa. La malnutrición puede aumentar el riesgo de padecer enfermedades crónicas no transmisibles (obesidad, cáncer, diabetes, hipertensión arterial, etc.), o producir deficiencias. La desnutrición es frecuente en los pacientes hospitalizados, alrededor de 30%, (frecuencia en geriatría y/u oncología de 50%), el principal problema es que en muchas ocasiones no es detectado. La desnutrición en pacientes hospitalizados se asocia con una disminución de la función respiratoria, inmune, muscular, alteraciones en la cicatrización y peor calidad de vida. Las consecuencias son una mayor morbi-mortalidad, estadía hospitalaria y costos económicos. Según el déficit que predomine, existen 3 tipos de desnutrición; proteica, calórica o mixta. La obesidad es más prevalente en mujeres y personas de bajo nivel socioeconómico. La distribución de grasa de tipo androide, abdominal o central tiene mejor correlación con grasa visceral y mayor riesgo de presentar morbilidades asociadas a ella (hipertensión arterial, dislipidemias, etc). Dentro de los adultos mayores, uno

Seguimiento Por especialista.

Autor / Editor Paulina Lira

26 Facultad de Medicina, Universidad de chile

Año 2016

Osteoporosis

Epidemiología

Nivel de manejo del médico general: Diagnóstico Específico Tratamiento Completo Seguimiento Completo

El peak de masa ósea se logra a fines de los 20 o a comienzo de los 30 años de edad. El peak es más alto en hombres que en mujeres. Luego va disminuyendo lentamente. La incidencia de osteoporosis aumenta con la edad (50-59 años 15% tiene osteoporosis, sobre los 80 años, un 70%). y la causa más frecuente de osteoporosis es la primaria.

Aspectos Esenciales

Diagnóstico

En el adulto mayor la causa más frecuente de osteoporosis es la primaria • Ante fracturas por baja energía en radio distal, vértebras, cadera, se debe sospechar fracturas osteoporóticas • Fármacos de uso habitual como omeprazol, alteran la absorción de calcio • Principales factores de riesgo no modificables son el sexo y la edad. • La densitometría ósea es la base del diagnóstico. • Los hábitos de vida saludable, la suplementación de Calcio y Vit D y los bifosfonatos son la base del tratamiento.

La sospecha puede surgir en pacientes que presentan factores de riesgo ya sea de osteoporosis primaria o secundaria. Hacen sospechar osteoporosis, las fracturas de radio distal (fractura de Colles), vértebras (que van produciendo pérdida de estatura o cifosis dorsal), cadera y húmero proximal (aunque son más raras estas últimas). Las fracturas de vértebras pueden dar dolor en la línea media, pero hasta un 66% de las fracturas son asintomáticas y comprometen región dorsal y lumbar. La presencia de una fractura de vértebra cervical debe hacer sospechar una causa secundaria. Si bien en una radiografía se puede observar una pérdida de la densidad mineral (adoptando una forma cóncava), esta se observa cuando ya ha ocurrido una pérdida mayor 30% de la masa ósea por lo tanto realiza un diagnóstico tardío.

Caso Clínico Tipo

El diagnóstico es por la cuantificación de la masa ósea, lo ideal es la densitometría ósea. En esta, el diagnóstico está dado por la ubicación del punto dentro de la figura que va adjunta al exámen, ya que considera el T-score (la diferencia de la densidad ósea entre el paciente y personas jóvenes del mismo sexo) y el Z-score (diferencia de la densidad ósea entre el paciente y personas de la misma edad y del mismo sexo).

Mujer de 70 años, con antecedentes de artritis reumatoide, sufre caída a nivel desde el borde de su cama, es llevada al servicio de urgencia por familiares. Se le toma radiografía de antebrazo que muestra densidad ósea disminuida y fractura de radio distal.

Definición Pérdida de masa ósea, con alteración de la microarquitectura, conectividad ósea trabecular y adelgazamiento del hueso cortical, que determina un aumento del riesgo de fractura. Se clasifican • Osteoporosis primaria -Tipo I (post-menopáusica, por reducción en los niveles de estrógenos en el período perimenopáusico se asocia a un incremento de la actividad osteoclástica, pero con alto recambio óseo -Tipo II (senil, en ambos sexos, debido a la disminución en la síntesis ósea, probablemente debido a la disminución de la producción de Vitamina D, con bajo recambio óseo) •

Densitometría ósea

Osteoporosis secundaria en adulto mayor -Baja ingesta de calcio, déficit de vitamina D y vitamina C (escorbuto), dieta rica en fósforo (quesos, chocolates), consumo elevado de proteínas, tabaco, OH, exceso de cafeína (que produce hipercalciuria) -Inmovilización, sedentarismo -Tratamientos farmacológicos (corticoides, inhibidores de la bomba de protones, heparina, antiestrógenos) -Patologías que afectan el metabolismo óseo (mieloma, osteomalacia, hiperparatiroidismo, metástasis óseas, enfermedad de Paget)

El laboratorio va orientado a distinguir causas secundarias de osteoporosis. Son útiles niveles séricos de calcio, fósforo y fosfatasas alcalinas además de los exámenes generales. La calciuria de 24 horas es útil para determinar si es por aporte bajo (calciuria baja) o por aumento de resorción (calciuria alta).

27 Facultad de Medicina, Universidad de chile

Tratamiento Lo más importante es prevenir la osteoporosis con una dieta nutritiva, rica en calcio y vitamina D, hacer ejercicio. El objetivo principal del tratamiento es la prevención de las fracturas. Se basa en medidas no farmacológicas y farmacológicas. Tratamiento no farmacológicas: • Aporte de calcio y vitamina D • Evitar OH, tabaco • Evitar la inmovilización • Ambiente seguro para prevenir caídas Tratamiento farmacológico: • Suplementar calcio y vitamina D -Dosis diaria recomendada 1000 UI calcio y 800 UI vitamina D -Mala absorción en hipoclorhidria (no dar con inhibidores de la bomba de protones) -Produce RAM gastrointestinal (flatulencia) -Aumenta el riesgo de litiasis -Para mejorar su absorción es mejor dar la dosis diaria en 2 tomas

Seguimiento Completo

Autor / Editor Julián Bravo

Revisor de Formato Angel Eloy

Año 2016

28 Facultad de Medicina, Universidad de chile

Temblor

medades neurológicas (enfermedad de Parkinson, lesiones mesencefálicas, cerebelosas, etc.), sobreactividad adrenérgica (tirotoxicosis, feocromocitoma, hipoglicemia, etc.) o idiopático.

Nivel de manejo del médico general: Diagnóstico Inicial Tratamiento Inicial Seguimiento No requiere

Diagnóstico Para el diagnóstico etiológico del temblor es necesario evaluar las características de los tipos más importantes: • Parkinsoniano: Característicamente de reposo, distal, afecta preferentemente manos (“pildoreo”), con fre cuencia de 4-6 Hz. Inicialmente asimétrico. En el 60% de los pacientes con enfermedad de Parkinson también se presenta un temblor de tipo postural. • Temblor fisiológico exacerbado: Postural. Ausente en reposo, frecuencia 8-12 Hz. Se debe a una sobre actividad beta-adrenérgica. • Temblor esencial: Fundamentalmente postural, puede ser también de tipo cinético y asociarse a determinadas tareas. Tiene carácter hereditario, puede comenzar a cualquier edad, se exacerba con estrés y mejora con alcohol. • Temblor cerebeloso: Cinético, habitualmente intencional. Puede asociarse a signos de compromiso cerebeloso (ataxia, dismetría, disartria, etc.). • Temblor farmacológico: Puede ser de cualquier tipo, pero lo más frecuente es que produzca temblor postural.

Aspectos Esenciales • • • •

• •

El temblor es frecuente en el adulto mayor Es importante identificar si es consecuencia de procesos fisiológicos o patológicos Se debe indagar fármacos que produzcan temblor Clasificación según la situación en que presenta el temblor -temblor de reposo -temblor postural -temblor de movimiento El temblor esencial es el más frecuente y disminuye con alcohol. El tratamiento del temblor, no suele ser necesario a menos que haya compromiso funcional.

Caso Clínico Tipo Paciente de sexo masculino de 65 años de edad, presenta temblor de pocos meses de evolución que se presenta de manera asimétrica, preferentemente en extremidad superior derecha al mantener una postura y sostener objetos en el aire. Relata que se exacerba en situaciones de estrés y que disminuye con el alcohol. (el paciente presenta temblor esencial).

Tratamiento El tratamiento va a depender de la etiología, y también del compromiso funcional del paciente • Temblor parkinsoniano: Anticolinérgicos. • Temblor fisiológico exacerbado: Beta-bloqueadores (propanolol). • Temblor esencial: Beta-bloqueadores (propanolol) o primidona (anticonvulsivante). Casos refractarios pueden tratarse con toxina botulínica. • Temblor cerebeloso: Tratamiento sintomático no es útil, debe tratarse la patología de base.

Definición Se define como presencia de oscilaciones rítmicas de una parte del cuerpo, secundaria a contracciones alternantes o sincrónicas de grupos musculares opuestos. La clasificación clínica es según la situación en que se presenta en temblor de reposo y de acción (postural o movimiento). El temblor de movimiento, puede ser inicial, transicional (durante el movimiento) o terminal (llamado también intencional).

Seguimiento No requiere.

Epidemiología Los ancianos presentan una mayor prevalencia de temblor que los otros grupos etarios, siendo más frecuente en ellos el temblor esencial.

Autor / Editor Paulina Lira

Etiología Fármacos (beta-adrenérgicos, neurolépticos, anticonvulsivantes, colinérgicos, litio, hormonas tiroídeas, etc.), enfer-

29 Facultad de Medicina, Universidad de chile

Año 2016

Trastornos de la Marcha



Nivel de manejo del médico general: Diagnóstico Específico Tratamiento Completo Seguimiento Derivar

Caso Clínico Tipo Paciente de 75 años, marcha vacilante y caídas recurrentes hace 6 meses. Hace 3 meses se agrega incontinencia urinaria y deterioro cognitivo acelerado. Equilibrio estático conservado y alteración de la marcha.

Aspectos Esenciales • • •



miento. El tratamiento es multidisciplinario, y se basa en un programa de ejercicios, entrenar el equilibrio, entrenar en las actividades de la vida diaria y manejo del espacio donde se va a movilizar el adulto mayor.

Patología de etiología multifactorial. Muchas patologías en el adulto mayor se pueden presentar como un trastorno de la marcha. Se debe preguntar siempre por la evolución de trastorno de la marcha, síntomas concomitantes y síndromes geriátricos (deterioro cognitivo, incontinencia) y realizar un adecuado examen físico con evaluación de la marcha. Importante dado su asociación con caídas y dado que han ido aumentando como consecuencia del envejeci-

Definición La marcha senil se caracteriza por una postura del cuerpo con discreta proyección anterior de la cabeza, flexión del tronco, caderas y rodillas, hay menor braceo, se acorta el paso, aumenta levemente el tamaño de la base de sustentación).

30 Facultad de Medicina, Universidad de chile

Epidemiología • • • • •

(caderas, rodillas, tobillos), todas estas pueden modificar el patrón normal de la marcha, constituyendo la marcha senil. Además, por el envejecimiento se produce disminución en la velocidad de la marcha (un adulto promedio camina a 1,1 m/seg, pero a partir de los 60 años la velocidad de marcha disminuye 1% por año, llegando a perder la capacidad de caminar de forma funcional fuera de sus casas si llega a 0,8 m/seg).

Mayores de 60 años tienen una prevalencia de alrededor de un 32% Mayores de 80 años, la cifra aumenta al 62% En los hogares de ancianos la cifra se estima en un 50% Los trastornos de la marcha se asocian a caídas Son en general de origen multifactorial

Fisiopatología

Etiología

El envejecimiento produce disminución de fuerza muscular, sarcopenia, osteoporosis, pérdida de rangos articulares

La etiología es multifactorial la mayoría de las veces.

Diagnóstico El diagnóstico va encaminado objetivar el trastorno de la marcha, para esto se pueden hacer pruebas especiales. Además se debe intentar identificar el tipo de marcha que tiene el paciente, ya que va a orientar al tipo de problema subyacente. Según la sospecha etiológica, va a realizarse el estudio (el tipo de estudio según sospecha se encuentra en la tabla de más abajo) Pruebas especiales: •

Test de alcance funcionalTest de Romberg progresivo.

31 Facultad de Medicina, Universidad de chile

Tratamiento Determinar las causas que están influyendo en los trastornos de la marcha. Estas intervenciones deben ser multidisciplinarias (fisiatra, kinesiólogo, geriatra, terapeuta ocupacional) El tratamiento disminuye los riesgos de caída y además mejora la funcionalidad del adulto mayor, y consiste en implementar equipos de bajo costo, como ayudas técnicas, marchas con arnés de sujeción, marcha en paralelas, marcha con andador y escalera, así como marcha con pistas visuales (marcas en el suelo o los escalones).

Seguimiento Derivar.

Autor / Editor Nicole Fritzche

• • • • • • • • •

Apoyo monopodal Test de “get up and go” Test de la tarea doble Test de velocidad de la marcha Test de la marcha de 6 minutos Escala de equilibrio de Berg Test de Tinetti Posturografía Laboratorio de marcha

32 Facultad de Medicina, Universidad de chile

Año 2016

estado general. Sin embargo, se presenta de forma atípica con mayor frecuencia en los ancianos: Cuadro confusional agudo, deterioro del estado general y/o de la capacidad funcional de manera aguda, por lo que pueden demorar en consultar hasta 1 semana. Muy importante es la historia clínica (a veces muy difícil por la dificultad en la comunicación: alteración del lenguaje, déficit cognitivo, delirium, deprivación sensorial, etc.).

Abdomen agudo en adulto mayor Nivel de manejo del médico general: Diagnóstico específico Tratamiento inicial Seguimiento derivar

Aspectos esenciales   

Indagar en antecedentes de etiología abdominal, comorbilidad asociada, consumo de determinados fármacos (AINES pueden disminuir percepción del dolor, entre otros) y síntomas asociados. En este grupo etario los síntomas clásicos pueden ser más tardíos o presentarse sólo con síntomas inespecíficos. Exploración física: Puede ser inespecífica. Revisar signos vitales, dentro de ellos destaca hipotensión, taquicardia, hipotermia es 4 veces más frecuente en ancianos acompañando a un proceso intraabdominal. Buscar signos de irritación peritoneal. Pruebas complementarias: Hemograma, función renal, electrolitos, glicemia, dentro de esto destaca que la leucocitosis tiende a ser menor o incluso no aparecer con el mismo grado de inflamación. La ecografía abdominal es una prueba de imagen muy resolutiva, especialmente útil en patología biliar, hepática, aórtica y renal. El TAC debe ser realizado en todo paciente con alta sospecha. Aspectos psicosociales: Considerar posible minimización de síntomas. Nota: Este grupo etario presenta una alta tasa de complicaciones por la patología que causa el abdomen agudo, la cirugía y por sus comorbilidades, entre ellas destacan: infección extra-abdominal (urinaria y respiratoria), sepsis e insuficiencia renal postoperatoria, cardiovasculares (arritmias, síndromes coronarios agudos), tombosis venosa y embolía pulmonar. También presentan una alta tasa de mortalidad.

Frecuente presentación atípica. Primera causa es patología biliar y obstrucción intestinal. Valoración por cirujano ante la menor sospecha diagnóstica de patología quirúrgica.

Caso clínico tipo Una mujer de 62 años consulta por dolor en fosa iliaca izquierda. Refiere 2 episodios previos similares. Al examen hay sensibilidad en la zona y signos de irritación peritoneal. El diagnóstico más probable es: Respuesta: Diverticulitis

Definición Situación crítica que cursa con síntomas abdominales graves y que requiere un tratamiento médico o quirúrgico urgente. Manifestaciones más frecuentes: Dolor abdominal agudo, alteraciones gastrointestinales y repercusión sobre el estado general. En los adultos mayores se presenta la mayoría de las veces con síntomas atípicos (menor intensidad del dolor, afebril, confusión, etc.)

Epidemiología Tratamiento

Motivo frecuente de consulta en la práctica médica habitual, los grupos etiológicos son los mismos que en el paciente adulto, sin embargo, existen diferencias en la frecuencia de aparición. Mientras en el paciente joven la causa más frecuente de dolor abdominal es el dolor abdominal inespecífico y la apendicitis, en el anciano es la patología biliar y la obstrucción intestinal.

Evaluación inicial: Establecer la gravedad del cuadro y detectar la existencia de shock o de cualquier complicación. Iniciar en aquellos pacientes que lo precisen estabilización hemodinámica simultáneamente con la valoración diagnóstica. (tener cuidado con la analgesia ya que puede ocultar síntomas). Obtener historia clínica detallada, exámenes e imágenes, diagnóstico diferencial entre abdomen agudo de origen quirúrgico y de origen médico. Manejar abdomen agudo de origen médico si corresponde. Antibioterapia en determinados casos, que cubra gérmenes anaerobios y gram negativos. Contactar al cirujano. Laparotomía o laparoscopia exploratoria, en caso necesario.

Etiología Causas más frecuentes:  Enfermedades biliares: 25% de los casos de pacientes ancianos que requieren hospitalización.  Obstrucción intestinal y hernia incarcerada.  Úlcera péptica, apendicitis, malignidad, diverticulitis.  También en el anciano son más frecuentes las afecciones vasculares, y el origen extraabdominal del cuadro.

Tratamiento definitivo: La mayoría de las veces es quirúrgico. El manejo del abdomen agudo de origen médico depende de la etiología (CAD, Sd. Urémico, anemia hemolítica, ITU, IAM, NAC, pancreatitis aguda, gastroenteritis aguda).

Causas menos frecuentes:  Abdomen agudo de origen médico: CAD, Sindrome Urémico, anemia hemolítica, ITU, IAM, NAC, pancreatitis aguda, gastroenteirtis aguda, etc.

Seguimiento Derivar a cirujano seguimiento.

Diagnóstico Cuadro clínico: Forma clásica: Aparición aguda, dolor, náuseas, vómitos, alteraciones del ritmo intestinal y repercusión del 1

para

tratamiento

quirúrgico

y



Agitación y agresividad

 Nivel de manejo del médico general: Diagnóstico sospecha Tratamiento inicial Seguimiento derivar.

  

esquizofrenia,

Diagnóstico En general constituyen síntomas de algún síndrome más amplio, su presentación suele ser transitoria, es raro que aparezcan y evolucionen en forma crónica. Va desde leve agitación hasta la auto o heteroagresión. Debido a que responden en general a causas concretas, lo más importante es identificarlas. Principal causa es infectometabólica por lo tanto siempre estudiar focos infecciosos (pulmón, orina y partes blandas), ELP, crea, BUN, hemograma y glicemia. También identificar estresores ambientales como cambio de domicilio, alteración de ciclos sueño-vigilia y déficit sensoriales (pila de audífono, lentes, etc.).

Aspectos esenciales 

Psiquiátrico:psicosis, delirium, trastorno de personalidad. Conducta antisocial

No es un cuadro específico, responde a otro cuadro de base. Amplio espectro de severidad de síntomas. Tratamiento inicial es el manejo de la urgencia con contención verbal, farmacológica y mecánica si es necesario. Siempre identificar causa desencadenante y facilitar su modificación.

Tratamiento Caso clínico tipo

Lo primero es salvaguardar la integridad del paciente y el personal (mantenga distancia y solicite ayuda). Para contener al paciente agitado se debe partir por la contención verbal. En caso que ésta no sea suficiente, se debe aplicar la contención farmacológica y finalmente, la mecánica. Mientras se logra la contención se debe buscar la etiología siempre.

Adulto mayor que debido a patologías de base, cuadro agudo o alteraciones del ambiente presenta cuadro que incluye agitación leve hasta agresividad contra si mismo, otras personas o su entorno.

Definición

Contención Verbal:  Respetar el espacio personal.  No ser provocativo.  Establecer contacto verbal.  Usar lenguaje conciso y simple.  Identificar sentimientos y deseos.  Escuchar atentamente lo que el paciente dice.  Establecer límites claros.  Ofrecer opciones.

Estado de marcada excitación mental acompañado de un aumento inadecuado de la actividad motora, en grado variable, desde una mínima inquietud hasta movimientos descoordinados sin un fin determinado, que en ocasiones se asocia a conductas violentas hacia sí mismo o hacia las personas o material que le rodea.

Contención Farmacológica: Existe una gama de medicamentos indicados según el cuadro específico causante de agitación, o dependiendo de la severidad de los síntomas. Los neurolépticos como haloperidol, en dosis de 1.25-5 mg IM o EV con intervalo de dosificación de 30 minutos, son ampliamente utilizados. Otros fármacos utilizados son las benzodiazepinas como lorazepam en dosis de 0.5-2 mg IM o EV con intervalo de dosificación de 30 minutos si existe agitación severa (de elección en intoxicación por drogas o alcohol, síndrome de abstinencia y agitación de causa desconocida). Nota: precaución con antipsicóticos de primera generación en pacientes que tengan riesgo aumentado de desarrollar síndrome de QT prolongado, controlar con ECG.

Epidemiología El grupo etario de los adultos mayores tienen una elevada prevalencia de trastornos psiquiátricos con cifras que oscilan entre el 15-20%. La agitación representa un problema para el propio paciente (autoagresión), para los familiares, personal de salud y el entorno en general, tanto personas (heteroagresión) como elementos materiales. En general estos estados se asocian a patologías psiquiátricas de base, pero en muchas ocasiones son signo de una condición médica (orgánica) que está afectando al adulto mayor, por lo que en estas situaciones siempre se debe buscar una etiología.

Contención Física: Indicada en:

Etiología

  

Posibles causas:  Toxicológica: intoxicación con alcohol o drogas y síndrome de abstinencia. Reacciones a medicamentos como benzodiazepinas.  Metabólica: hipoglicemia, hipoxia.  Neurológica: ACV, infección al sistema nervioso, demencia.  Otras condiciones médicas: hipertoridismo, hipotermia, hipertermia, cualquier infección sistémica, trastornos hidroelectrolíticos.

Daño inminente al personal o a sí mismo Interrupción significativa del tratamiento. Nota: La contención física puede ser utilizada previamente para lograr la contención farmacológica.

Seguimiento Derivar.

2

Hipotensión postural (ortostática)

Etiología Causas neurogénas  Lesiones que afecten el sistema nervioso central o Hipotensión ortostática pura (fallo autonómico puro) o Trastornos de la médula espinal, hipotálamo o Enfermedad de Parkinson, enfermedad de Huntington o Encefalopatía de Wernicke  Sin afectación del sistema nervioso central o Enfermedades autoinmunes o Enfermedades metabolicas (déficit Vit. B12) o Enfermedades sistémicas) Causas no neurógenas  Cardiovasculares  Disminución del volumen intravascular  Alteraciones endocrinas y metabólicas  Anemia  Encamamiento  Fármacos

Nivel de manejo del médico general: Diagnóstico específico Tratamiento completo Seguimiento Completo.

Aspectos esenciales.   

  



Caída de la PAS ≥ 20 mmHg ó de la PAD ≥ 10 mmHg al pasar del decúbito a bipedestación. Se evidencia a los 3 minutos aprox. de bipedestar. Una cifra en bipedestación puede ser considerada “normal”, pero corresponder a una hipotensión ortostática (no es la cifra, sino la magnitud de la disminución de esta) Puede ser asintomática, o presentar mareos, síncope, incluso angina de pecho o un derrame cerebral. Tiene importancia porque está asociada a caídas y sus complicaciones. Antes de prescribir un tratamiento antihipertensivo a un paciente adulto mayor, descartar hipotensión ortostática, ya que el inicio de tratamiento antihipertensivo la puede acentuar Para diferenciar la etiología de la hipotensión postural (si es causa neurógena o no) es útil el test de Schellong (que se explicará más adelante)

Diagnóstico Lo primero es hacer el diagnóstico de hipotensión postural con esfigmomanómetro. Con el diagnóstico hecho, se puede buscar la etiología. Para orientar la etiología se usa es test de Schellong, que consiste determinar si la caída de la presión se acompaña o no de un alza de la frecuencia cardiáca. Se espera que la caída de la presión arterial esté asociada a un alza de la frecuencia cardíaca (15 latidos por minuto) en causas no neurógenas. Sin embargo, cuando la causa es neurógena, no se produce dicho aumento.

Caso clínico tipo. Paciente de 74 años, sexo masculino, en tratamiento por Enfermedad de Parkinson (EP) con levodopa/carbidopa (250/25) ¾ partes de comprimido cada 3 horas, 5 veces al día. Durante los últimos tres meses ha presentado 2 episodios de caídas, precedidos por sensación de desvenecimiento, debilidad y sudoración. El último despertandose para ir al baño en la noche. Su presión arterial en decúbito es de 140/90 mmHg (FC 68x’), y al ponerlo de pie de 120/80mmHg (FC 66x’) sin síntomas acompañantes.

Tratamiento La hipotensión ortostática se trata si es sintomática. Si bien se deben se deben tratar aquellas causas conocidas con tratamiento específicos (como anemia, reposición de volumen, déficit de vitamina B12), puede ser producto de diferentes causas, o muchas veces no encontrar el diagnóstico etiológico. Por lo tanto, lo más importante para todos los casos es la prevención

Definición Es caída de la presión arterialsistólica (PAS) ≥ 20 mmHg o de la presión arterial diastólica (PAD) ≥ 10 mmHg al pasar de la posición supina a la posición erguida (suele evidenciarse 3 minutos después).

 

Epidemiología    



La hipotensión ortostática es más común en personas de edad avanzada (20% de >65 años). La mayoría son asintomáticas (se estima que un 2% tendría síntomas). Aumenta la frecuencia en adultos mayores hospitalizados (un 60%>65 años). Se ha encontrado una modesta asociación con hipertensión sistólica supina, estenosis carotídea superior al 50% y con el uso de hipoglicemiantes orales. Otros medicamentos asociados, especialmente en adultos mayores, son: antihipertensivos, vasodilatadores, nitratos y bloqueadores de canales de calcio, antidepresivos tricíclicos y opiáceos.Hasta el 40% de los pacientes con hipotensión ortostática no tiene causa conocida

   

Evitar cambios bruscos posturaes (primero sentarse, realizar dorsiflexión del pie para aumentar el retorno venoso) Evitar situaciones que disminuyen el retorno venoso (estar mucho tiempo de pie sin moverse, lugares calurosos, OH, etc) Corregir factores que disminuyen en volumen intravascular (deshidratación, hipokalemia, hiponatremia) Uso de medias elásticas compresivas para favorecer el retorno venoso. Evitar fármacos que produzcan hipotensión Ejercicio físico regular (sobre todo cuando hay componente cardiovascular) Es importante educar al paciente en que al inicio de los síntomas busque donde sentarse y/o apoyarse para evitar las complicaciones de las caídas

Seguimiento Completo. 3

vasoconstricción), por los pulmones por diversos mecanismos. La hipotermia produce alteración de la función transportadora de la membrana celular, disfunción enzimática, y desequilibrios electrolíticos (incluyendo hiperkalemia). Resultando en muerte celular por daño de la membrana celular, disfunción de proteínas, o cristalización del agua intra y extracelular.

Hipotermia Nivel de manejo del médico general: Diagnóstico específico Tratamiento completo Seguimiento completo

Etiología Causas exógenas principales en el adulto mayor (ambientales)

Aspectos esenciales    



El adulto mayor tiene cambios fisiológicos asociados al envejecimiento que predisponen a la hipotermia. La hipotermia no sólo se produce en contexto de temperaturas bajas, sino que también por causas endógenas Frente a temperaturas menores de 35°C, se debe usar termómetros de registro bajo Hipotermias moderadas o severas requieren hospitalización

Exposición prolongada al frío (ya que tienen alterada la percepción térmica)  Inmovilidad  Caída con permanencia prolongada en el suelo  Fármacos (vasodilatadores, hipotensores, antidepresivos tricíclicos, benzodiacepinas, bloqueadores neuromusculares) Causas endógenas 

Caso clínico típico Adulto mayor de 82 años, es hospitalizado en julio por neumonía ATS III. A los 3 días de hospitalización cursando con mejoría clínica, presenta a las 6 de la mañana en la evaluación rutinaria hipertensión, taquicardia, se encuentra algo confuso, al tocarlo está con las extremidades frías y pálidas.

 

Definición



Es una condición provocada cuando las pérdidas de calor superan la producción de calor. Produce un descenso en la temperatura central (axilar o inguinal). En los 35°C ya se habla de hipotermia, y según el grado de descenso, se clasifican en leve (entre 35° a 32°C), moderada (entre 32° y 28°C)y severa bajo los 28°C. El grado de instauración puede ser aguda, subaguda o gradual (días o semanas). Es importante ante temperaturas bajas usar termómetros de registro bajo (es decir, que registre temperaturas bajo 34°C).

Afectación de la termorregulación por el envejecimiento (debido a la alteración de la percepción térmica, alteración de la vasoconstricción periférica, disfunción autonómica, disminución de la movilidad, alteraciones del circadianas de la regulación de la temperatura. Enfermedades neurológicas (ACV, lesiones hipotalámicas, demencia, etc) Metabólicas (hipoglicemia, hipotiroidismo, insuficiencia suprarrenal, cetoacidosis, acidosis láctica, uremia) Úlceras por presión extensas

Diagnóstico Diagnóstico se sospecha con cuadro clínico y se confirma con temperatura corporal menor a 35°C. Las manifestaciones clínicas, depende del grado de hipotermia y se resumen en el siguiente cuadro:

Epidemiología Es frecuente en los adultos mayores, dado que los mecanismos que regulan el calor son menos eficaces, por lo tanto tienen mayor riesgo de desarrollar hipotermia y sus complicaciones. Los factores de riesgo asociados a muerte por hipotermia accidental incluyen el consumo de alcohol, la falta de vivienda, enfermedades psiquiátricas, y la edad avanzada. En hipotermia severa la mortalidad es de un 50% (aumenta con la edad).

Fisiopatología La temperatura corporal es reflejo del equilibrio entre la producción y la pérdida de calor. En centro que regula la temperatura es el hipotálamo. El calor es generado por el metabolismo celular y mecanismos neuroendocrinos (activación noradrenérgica que genera contracción muscular para producir más calor, hormonas tiroídeas) y se regula por la piel (regulado por la vasodilatación y 4

Los exámenes de laboratorio van a ir a buscar etiología y complicaciones. Pensando en la etiología, se debe buscar hipoglicemia, hipotirodismo, signos de laboratorio de sepsis, lactato, urea, según la sospecha clínica Pensando en las complicaciones se debe solicitar hemograma (el hematocrito aumenta 2% por cada grado de descenso y ocurre por depleción de volumen; trombocitopenia por secuestro), electrolitos plasmáticos (buscando hiperkalemia) y gases arteriales (inicialmente se produce taquipnea y luego aumenta el consumo de oxígeno, se producen alteraciones ácido base), pruebas de coagulación (diátesis hemorrágica), pruebas hepáticas, lipasa y amilasa (la pancreatitis es una de las complicaciones), función renal (se produce poliuria en fases iniciales y necrosis tubular aguda con oligoanuria si se agrava), glicemia (si bien la hipoglicemia es causa de hipotermia, la hiperglicemia es complicación de la hipotermia y se produce por disminución de la utilización de glucosa, inhibición de la liberación de insulina y el aumento de las catecolaminas).

Tratamiento Los pilares son: detectar adultos mayores susceptibles (por medio de valoración geriátrica integral), prevenir hipotermia con medidas ambientales, detectar manifestaciones precoces, tratamiento recalentamiento y tratar causas etiológicas. En los pacientes que no responden al tratamiento de recalentamiento intensivo se sugiere el tratamiento con antibióticos de amplio espectro empíricos y una sola dosis de glucocorticoides. En caso de paro cardiorrespiratorio reanimar indefinidamente hasta que la temperatura sea de 30 a 32° C, excepto si las compresiones torácicas son imposibles por la congelación del cuerpo, o si la nariz y la boca están bloqueados por el hielo. Recalentamiento: 



Si la T° ≥ 32°C se utilizarán técnicas de recalentamiento externas pasivas (retirar de ambiente frío y colocarlo en uno tibio, contacto con objetos tibios). Si T° < 32°C se debe monitorizar en unidad de cuidados intensivos, utilizando técnicas de recalentamiento activo externo e interno, lo cual de por si es riesgoso. (inmersión en agua tibia, cobertores eléctricos, inhalaciones tibias, balón intragástrico, irrigación colónica, irrigación mediastínica, hemo o peritoneodiálisis).

Seguimiento Completo.

5



Incontinencia urinaria Nivel de manejo de médico general: Diagnóstico específico Tratamiento inicial Seguimiento derivar

Fisiopatología 

El adulto mayor es más vulnerable a la incontinencia urinaria, ya que con el envejecimiento la vejiga sufre alteraciones fisiológicas (disminuye la capacidad de esta y la acomodación, presenta contracciones involuntarias y queda residuo postmiccional). Sin embargo, esto no significa que el envejecimiento fisiológico lleve a la incontinencia urinaria, siempre es patológica.



A esta vulnerabilidad, se asocian otros factores involucrados, entre los cuales destacan demencia, inmovilidad, ciertos fármacos y la polifarmacia, que por distintos mecanismos van a llevar a la pérdida de la continencia urinaria o Diuréticos (producen poliuria, polaquiuria y urgencia miccional) o Hipnóticos y antipsicóticos (producen sedación, inmovilidad y delirio) o Antidepresivos o Anticolinérgicos o Calcio antagonistas (producen retención urinaria) o Opiáceos



Hay distintos mecanismos por los que se produce la incontinencia, pudiendo clasificarse según si afectan la fase de continencia (incontinencias) o la fase miccional (retenciones y rebosamiento por residuo postmiccional >100cc) y si el origen esta a nivel de detrusor o complejo esfinteriano: o Fase de continencia – Detrusor:  Disminución acomodación (alteración en la inhibición del tono o de la visco elasticidad)  Aumento contractilidad (hiperactividad idiopática o neurogénica). Más frecuente en adulto mayor, urgencia miccional. o Fase de continencia – Complejo esfinteriano:  Incontinencia de esfuerzo por hipermovilidad uretral (alteración musculatura piso pélvico)  Incompetencia enfinteriana intrínsica (Nerógena, daño anatómico del esfínter, falla del sello)  Incontinencia extra uretral (fístulas)  Fase miccional – Detrusor:  Disminución o ausencia de contractilidad (Neurógena o miógena) o Fase miccional – Tracto de salida (Uropatía obstructiva)  Activas (Disinergia detrusoresfinter, externo o interno)  Pasivas: Compresivas (adenoma prostático), Restrictivas (estrechez de uretra), otros.  Inhibición psicógena.

Aspectos esenciales  





El diagnóstico depende casi en su totalidad de que el médico pregunte su existencia. La incontinencia urinaria aguda (o transitoria) refleja alteraciones funcionales, por lo tanto, se recupera cuando cesa el factor que la produjo, suele durar menos de un mes. La incontinencia urinaria crónica (o establecida) refleja alteraciones estructurales (ya sea en el tracto urinario o fuera de él), suelen durar más de un mes Se considera como elevado un residuo postmiccional de más de 100cc

Caso clínico tipo Paciente mujer de 78 años, con antecedentes de HTA y DM tratadas en su consultorio, cuatro partos vía vaginal y una cesárea, no tiene alergias. Refiere que cuando se junta con sus amigas a jugar carioca, cada vez que se ríe se le escapa una pequeña cantidad de orina y moja la ropa interior, no refiere otros síntomas urinarios.

Definición “Pérdida involuntaria de orina que condiciona un problema higiénico y/o social que se puede demostrar objetivamente”. La incontinencia urinaria correspondería a un síntoma, tras el cual existen variados mecanismos fisiopatológicos a partir de distintas enfermedades. Esta patología significa un importante impacto negativo en la calidad de vida de quienes la padecen, en particular en la población mayor llevando secundariamente al aislamiento social, sentimiento de deterioro personal, disminución de la vida en pareja y de la actividad físico, lo cual, junto a otros cambios, obstaculizan un envejecimiento saludable. Además puede significar consecuencias importantes como aumento en la frecuencia de fracturas de cadera por caídas en contexto de urgeincontinencia, alteraciones del sueño por nocturia, generando somnolencia durante el día, entre otros.

Epidemiología 

La prevalencia en el adulto mayor es variable y depende principalmente si están o no institucionalizados. Si el adulto mayor vive en la comunidad es de un 15%, en cambio si está institucionalizado es de un 60%

La incontinencia urinaria es frecuente en la población, fuertemente ligado con la edad. En menores de 50 años se presenta en alrededor de un 25% de las mujeres, con una relación mujer:hombre de 6:1. En los mayores de 60 años es especialmente frecuente en ambos sexos, se observa un aumento en la prevalencia de este cuadro en las mujeres hasta aproximadamente un 33%, y un aumento aún mayor en la población masculina, estrechando la relación a 3:1. 6

Respecto al examen físico además de evaluar elementos generales se debe apreciar el nivel de trofismo vulvovaginal, presencia de fistulas, uretro o cistocele, hipermovilidad uretral (Test de Marshall), estado de musculatura pélvica, tacto rectal especialmente en hombres para descartar hiperplasia prostática.

Diagnóstico El diagnóstico es fundamentalmente clínico y se puede apoyar con estudios complementarios. A nivel de atención primaria se deben descartar las causas más frecuentes de incontinencia urinaria aguda, para derivar al urólogo para un diagnóstico y tratamiento definitivo o en caso de incontinencia urinaria crónica.

Estudios complementarios se deben realizar para descartar patología asociada relevante y determinar el tipo de incontinencia de manera precisa para un mejor plan terapéutico. Se pueden solicitar exámenes básicos iniciales como orina completa y urocultivo, y otros exámenes más específicos según sospecha diagnóstica como uretrocistografía, pielografía de eliminación, ecografía abdominal y pelviana, TAC y PieloTAC, cistoscopia, y estudio urodinámico (uroflujometría, estudio de flujo de presión, etc),

Ante un adulto mayor con incontinencia urinaria se debe establecer el tiempo de evolución de esta. Cuando dura menos de un mes, estamos frente a una incontinencia urinaria aguda (o transitoria), en cambio cuando persiste más de un mes podemos plantear una incontinencia urinaria crónica (o establecida). La importancia de esta diferenciación está en que la incontinencia aguda hay alteraciones funcionales que se recuperan cuando se corrige el factor que las produjo, en cambio la incontinencia crónica refleja que hay alteraciones estructurales. Hay que recordar que la incontinencia puede ser mixta.

Tratamiento El tratamiento va a depender del mecanismo fisiopatológico involucrado, una vez descartadas otras patologías asociadas se puede plantear un manejo inicial según el tipo de incontinencia o derivar a especialista en caso de falla de tratamiento o patología de resolución quirúrgica.

Las causas de incontinencia urinaria aguda se resumen en la mnemotécnica DRIP o DIAPPERS D Delirium R Restricción de movilidad. Retención urinaria I Infección urinaria. Inflamación. Impactación fecal P Poliuria. Polifarmacia

Existen medidas generales como: 

D Delirium I Infección urinaria A Atrofia (vaginitis) P Polifarmacia P Psicológicas E Endocrinopatías R Restricción de movilidad S Stool (impactación fecal)

    

Terapia con técnicas conductuales: importante en población mayor, aplicable para urgeincontinencia e incontinencia de esfuerzo. Terapias de reentrenamiento vesical (educación, refuerzo positivo), entrenamiento de hábitos, micción inmediata según deseo miccional, modificaciones en la dieta e ingesta de líquidos, restricción de alimentos como café y alcohol.

Al enfrentarse a un paciente con incontinencia nuestra atención debe ir enfocada hacia facilitar la consulta por incontinencia generando un ambiente cercano y de confianza con el paciente, objetivar y cuantificar la cantidad de orina perdida, evaluar como impacta en su calidad de vida, determinar posibles hipótesis diagnósticas del tipo de incontinencia y su etiología, descartar patología urológica y condiciones generales agravantes o desencadenantes asociadas. Se debe realizar una anamnesis orientada a las características y causas de la incontinencia.   





Usar ropa que se pueda sacar fácilmente para ir al baño No ingerir líquidos en la noche Ambiente sin barreras físicas para ir al baño Bajar de peso en caso de obesidad Evitar constipación Cese hábito tabáquico

Fisioterapia, biofeedback y rehabilitación del piso pélvico (ejercicios de Kegel): terapia conservadora de primera línea en pacientes con incontinencia urinaria, especialmente de esfuerzo, con el objetivo de reforzar la musculatura pélvica y en particular, el complejo pubococcigeo y elevador del ano. En algunos casos se puede asistir con dispositivos como pesarios.

Factores desencadenantes (tos y esfuerzo en incontinencia de esfuerzo) Presencia de urgencia y urge-incontinencia (hiperactividad vesical) Indagar deterioro cognitivo, inmovilidad, estado confusional, depresión, infección urinaria, uretritis, vaginitis atrófica, exceso ingesta de líquidos o diuresis aumentada (diureticos, hiperglicemia), alteración del tránsito intestinal (fecaloma puede producir incontinencia), fármacos, entre otros. Antecedentes de cirugías pélvicas (próstata), o cualquier intervención pélvica como radioterapia, partos vaginales, traumas, que orientaría a incontinencia de esfuerzo. Una herramienta complementaria que puede resultar práctica es la cartilla miccional (registro calendarizado diario de episodios de escapes de orina, volumen, tipo y magnitud) y cuestionarios previamente validados.

Terapia farmacológica: indicada en incontinencia de esfuerzo y de urgencia, vejiga hiperactiva y algunos trastornos de acomodación y esfinterianos. Entre los fármacos más usados se encuentran los anticolinérgicos, oxibutinina, tolterodina, algunos antidepresivos (Imipramina) y bloqueadores de canales de calcio (diltiazem). Efecto adverso importante y que dificultan su adherencia es la sequedad bucal, especialmente con los anticolinérgicos, lo cual se le debe advertir a los pacientes al iniciar la terapia. Otras terapias: terapia inyectable periuretral, inyección de toxina botulínica tipo A, estimulación percutánea nervio tibial posterior. Cirugía: indicada en algunos casos de incontinencia de esfuerzo (mala TVT o TOT), incontinencia por 7

rebosamiento de causa obstructiva (hiperplasia prostática), alteraciones de la acomodación del detrusor y/o hiperactividad del detrusor que no responden a tratamientos conservador El uso del cateterismo intermitente se usa en situaciones transitorias. El cateterismo permanente debe ser indicado por un urólogo y se considera en casos que no sea posible realizarlo de forma intermitente

Seguimiento El médico general debe buscar las causas más frecuentes de incontinencia urinaria aguda e iniciar el tratamiento. Luego debe derivar al paciente al urólogo para realizar más estudios y/o tratamientos médicos y/o quirúrgicos.

8

MÓDULO 1: Medicina Interna

Hemato-Oncología Anemia de Enfermedad Crónica Etiología-Epidemiología-Fisiopatología

Nivel de manejo del médico general: Diagnóstico Sospecha Tratamiento Inicial Seguimiento Completo

Es la segunda causa de anemia después de la ferropénica a nivel ambulatorio, y la más frecuente en pacientes hospitalizados. La etiopatogenia es multifactorial e incluye 4 mecanismos fundamentales. 1. Anomalías en la utilización del hierro: Dentro de las citoquinas producidas en procesos inflamatorios la IL-6 estimula la producción en el hígado de una proteína de fase aguda conocida como heptacidina. La heptacidina es capaz de disminuir la absorción de hierro a nivel gastrointestinal y además produce una disminución de la utilización del hierro de los macrófagos de depósito al disminuir las proteínas de ingreso al plasma (ferroportina), de esta forma el hierro no pasa al plasma ni a los precursores de la serie roja, ocasionándose como consecuencia una disminución del hierro plasmático (hiposideremia) y un aumento de los depósitos.

Aspectos Esenciales • • • • •

Segunda causa de anemia. Causada por mala utilización del hierro. Generalmente normocítica normocrómica. Ferritina ALTA. (Diferente a la anemia ferropénica) Tratamiento etiológico.

Caso Clínico Tipo Paciente sexo femenino, 68 años, con antecedentes de Artritis Reumatoidea de 15 años evolución, en tratamiento con Metotrexato. Acude a su médico por presentar desde hace 1 mes fatigabilidad, hipersomnia y palpitaciones.

2. Disminución de la vida media de los GR: Es una disminución moderada de 62 a 90 días comparado con los 120 días normales. Las citoquinas podrían disminuir la sobrevida eritrocitaria y estimular la eritrofagocitosis por parte de los macrófagos. Se ha demostrado que el TNF alfa aumenta la fagocitosis de los GR.

Definición La anemia por enfermedad crónica, también denominada “por mecanismos múltiples” inicialmente se asoció a enfermedades infecciosas, inflamatorias y neoplásicas, sin embargo, se puede presentar además en una serie de condiciones tales como la diabetes mellitus, trauma severo, insuficiencia renal crónica, hepatopatías y mesenquimopatías. Por lo general es normocítica, normocrómica e hipoproliferativa, pero puede ser microcítica e hipocrómica hasta un 20 – 30%, por lo que el diagnóstico diferencial con la anemia ferropénica es fundamental.

3. Inhibición de la proliferación y diferenciación: de progenitores eritroides secundaria a citoquinas como interferones y TNF producidos por monocitos y macrófagos en el microambiente medular, los cuales inhiben la eritropoyesis. 4. Deficiencia relativa de Eritropoyetina (EPO).

Diagnóstico El diagnostico se sospecha de manera clínica y se confirma con el hemograma y frotis.

1 Facultad de Medicina, Universidad de Chile





Seguimiento

Cuadro Clínico: Síndrome Anémico: debilidad, palidez mucocutánea, taquicardia, palpitaciones, soplo funcional, disnea, menor apetito, irritabilidad, cefalea, depresión. Signología de la enfermedad crónica causante. Dg Diferencial: - Pruebas Laboratorio:

Hemograma periódico a pacientes con patología inflamatoria crónica.

Autor / Editor Año Catalina San Martín 2016



Hemograma: Hemoglobina baja según corresponda, VCM y CHCM Normocítica y Normocrómica. Aunque puede presentarse con VCM y CHCM bajas. Índice de Reticulocitos Bajo.

Cinética de Fierro: Lo más importante para descartar ferropenia (diagnóstico diferencial más importante). Sideremia BAJA, Ferritina ALTA, Transferrina NORMAL o BAJA, Saturación de Transferrina NORMAL o BAJA. Si es menor 15% hay ferropenia asociada. En cuanto al diagnóstico diferencial, se debe sospechar siempre que haya anemia profunda. No olvidar las anemias carenciales, en especial la falta de folatos (IRC en HD, hepatopatías), déficit vitamina B12 (gastrectomizados, enf. de Crohn), ferropenia (hipotiroidismo, pérdidas gastrointestinales, etc.), mieloptisis (en especial en presencia de cáncer y linfoma con dacriocitos en el frotis) y mielodisplasia en toda anemia que no mejora a pesar de terapias.

Tratamiento Objetivo del tratamiento: Objetivar causa y tratarla. Tratamiento Farmacológico: NO debe administrarse hierro, a menos que se confirme anemia ferropénica asociada. Las transfusiones están indicadas en pacientes sintomáticos, en casos de cirugía mayor, complicaciones hemorrágicas, etc. Los agentes eritropoyéticos son ampliamente usados en la insuficiencia renal crónica. La EPO constituye una opción terapéutica en la Anemia de Enfermedad Crónica asociada a tumores, infecciones crónicas y enfermedades autoinmunes. La administración de EPO disminuye el requerimiento transfusional y mejora la calidad de vida de estos pacientes. Las dosis usadas oscilan entre 150 U/kg tres veces por semana a 40.000 U por semana, Las tasas de respuesta son a veces bajas por lo cual se recomienda una evaluación pre-tratamiento para detectar a los pacientes que más se beneficiarán con esta terapia.

2 Facultad de Medicina, Universidad de Chile

Anemia Ferropriva

Ferropenia y metabolismo del hierro: Es la causa más frecuente de anemia. Corresponde al déficit de hierro, sea por falta de ingesta, absorción o por aumento de las pérdidas. No todos los pacientes con ferropenia desarrollan anemia.

Nivel de manejo del médico general: Diagnóstico Específico Tratamiento Completo Seguimiento Completo

Etiología-Epidemiología-Fisiopatología Fisiología: Metabolismo del hierro. Contenido total de hierro es de 50-55 mg/kg en hombres y 35-40 mg/kg en mujeres. La pérdida de hierro normal diaria corresponde a 1 mg como consecuencia de descamación celular (se incrementa en lactancia, menstruación, embarazo). La ingesta diaria en alimentos es de 10-30 mg de los que se absorben 1 mg en duodeno y yeyuno (proximal y medio). La absorción se ve aumentada en la forma hémica (ferrosa o reducida) y por la acción del ácido gástrico, ácido ascórbico y citrato, y disminuye con los fitatos y cereales de la dieta. Una vez absorbido el hierro se transporta en la sangre principalmente por transferrina y muy poco por ferritina (aunque esta última tiene muy buena correlación con los depósitos de hierro). El hierro que no se utiliza para hematopoyesis queda en depósitos en macrófagos en forma de ferritina y hemosiderina. Fisiopatología: Las causas más frecuentes de ferropenia son: • Disminución del aporte: Dieta inadecuada. • Disminución de la absorción: Cirugía gástrica, aclorhidria, Sd. Malabsortivos (enfermedad celíaca). • Aumento de pérdidas: Sangrado gastro-intestinal (lo más frecuente en varones), pérdidas ginecológicas (lo más frecuente en mujeres), otros sangrados. La anemia ferropriva temprana presenta disminución de la concentración de ferritina (<15 ug/L), aumento de la capacidad total de captación de hierro (TIBC) (>380 ug/100 ml), hierro sérico <50 ug/100 ml y saturación de hierro entre un 10 y 30%. La morfología eritrocitaria suele ser normal hasta que el déficit de hierro se vuelve significativo.

Aspectos Esenciales • • • • • •

Causa más frecuente de anemia. Valor de corte anemia [hemoglobina]: Hombres <13; Mujeres <12; Embarazadas <11. Pruebas de laboratorio: Hemoglobina baja, VCM <80 y CHCH <32: Microcítica e Hipocrómica; Índice de Reticulocitos <2: Arregenerativa. Mejor parámetro para detectar ferropenia: descenso de ferritina. Causa más frecuente: Hombres: Sangrado digestivo, Mujer: Sangrado Menstrual. Tratamiento por 6 meses.

Caso Clínico Tipo Mujer de 18 años, remitida desde consulta de atención primaria para descartar pérdidas digestivas tras detectar en analítica una anemia ferropénica moderada (hemoglobina 10 g/100 ml, VCM < 70 fl y HCM < 25 pg, con hiposideremia) de larga evolución. La paciente aporta estudio ginecológico en el que no se aprecian alteraciones significativas. No hay antecedentes personales, familiares ni tratamientos crónicos de interés. No refiere sintomatología y la exploración abdominal es normal. Presenta palidez mucocutánea. En analítica se encuentra anemia ferropénica moderada, sangre oculta en heces negativa y anticuerpos antigliadina, antiendomisio y antitransglutaminasa positivos. Se realiza endoscopia duodenal cuyo informe destaca la presencia de un aplanamiento de pliegues con mucosa en mosaico. Las biopsias confirman la presencia de atrofia vellositaria subtotal, linfocitos intraepiteliales e hiperplasia de glándulas crípticas. Con el diagnóstico de enfermedad celiaca con manifestación monosintomática y anemia ferropénica, se excluye el gluten de la dieta. A partir de este momento la anemia mejora de forma progresiva.

Diagnóstico El diagnóstico se sospecha de manera clínica y se confirma con el hemograma y frotis. Cuadro Clínico: Síndrome Anémico: debilidad, palidez mucocutánea, taquicardia, palpitaciones, soplo funcional, disnea, menor apetito, irritabilidad, cefalea, depresión. Ferropenia: Estomatitis angular, glositis (lengua lisa y depapilada, dolorosa, inflamada), coiloniquia, uñas quebradizas, pica, pelo frágil, disfagia orgánica por estenosis esofágica en anillo (Sd. Plummer-Vinson), etc. Pruebas Laboratorio: • Hemograma: - Hemoglobina baja según corresponda. - VCM (<80) y CHCH (< 32) baja: Microcítica e Hipocrómica. - Índice de Reticulocitos: <2 : Arregenerativa. - Eventualmente se puede observar: Leucopenia y trombocitosis. • Cinética de Fierro: Lo más importante para certificar ferropenia.

Definición Anemia: Disminución de la masa eritrocitaria, que se refleja como un descenso del número de hematíes, el hematocrito y más especificamente, de la concentración de hemoglobina. Los rangos utilizados para definir anemia varían según sexo, edad (particularmente en población pediátrica) y estado fisiológico (destacando el embarazo). Como guía son útiles los siguientes [concentración de hemoglobina en g/dL]: Hombres <13; Mujeres <12; Embarazadas <11.

3 Facultad de Medicina, Universidad de Chile



- Sideremia BAJA. Autor / Editor - Ferritina BAJA (< 15 ng/ml). Julián Bravo - Transferrina ALTA y Capacidad de fijación de la transferrina ALTA. - Saturación de Transferrina BAJA. Estudio etiológico: Según la causa sospechada en la anamnesis puede necesitarse: - Estudios gastrointestinales: endoscopía digestiva alta con o sin biopsias, colonoscopía, test de hemorragias ocultas en deposiciones, estudio de intestino delgado (enteroscopía de doble balón o con cápsula), anticuerpos antigliadina y antitransglutaminasa (enfermedad celíaca). - Estudios ginecológicos: ecografía transvaginal. - Otros estudios.

Tratamiento Objetivos del tratamiento: • Objetivar CAUSA de ferropenia y buscar su resolución. • Normalizar depósitos de hierro: Tratamiento Farmacológico: Aportar diariamente 100 a 200 mg de Fe elemental al día hasta normalizar depósitos de hierro (normalización de ferritina). Existen varios preparados, los más habituales con sulfato ferroso 200 mg (aporta 40 mg de Fe elemental al día, se requiere dosis de 200 mg 3 veces al día) y fumarato ferroso (109 mg de Fe elemental, requiere dosis de 1 comprimido al día).

Seguimiento En general el tratamiento demora 3 a 6 meses después de corregir la anemia (en relación a la Hb). El primer signo de respuesta al hierro suele ser un incremento en el porcentaje de reticulocitos, aproximadamente a los 10 días. La hemoglobina se normaliza a las 4-10 semanas. Indicaciones de terapia de hierro parenteral: Intolerancia al hierro oral, ausencia de absorción oral, pérdida del hierro a velocidad superior de la reposición por vía oral, enfermedades con mala absorción. Tratamiento etiológico: Buscar siempre la causa de la anemia por déficit de hierro: • Mujeres en edad fértil, pérdida de sangre en área ginecológica. • Hombres y mujeres postmenopáusicas: pérdida de sangre en tubo digestivo, SOSPECHAR CÁNCER GÁSTRICO. • Lactantes y embarazadas: déficit de ingesta por aumento de requerimientos. • Mujer en edad fértil sin pérdida ginecológica evidente u hombre joven con síntomas digestivos, estudio de tubo digestivo, sobre todo si presenta antecedentes familiares. • Tener en cuenta siempre enfermedad celiaca en mujeres.

4 Facultad de Medicina, Universidad de Chile

Año 2016

Anemia Hemolítica

hemoglobina por el glomérulo apareciendo en la orina. Algunas causas de Anemia Hemolitica Intravascular son: CID, prótesis valvulares, transfusión incompatible y Loxocelismo. La hemólisis Extravascular por otra parte ocurre predominantemente en el bazo, y es la causa más común de Anemia Hemolítica. Las causas se pueden dividir en Intracorpusculares y Extracorpusculares. Las Intracorpusculares ocurren por distintos defectos del GR: alteraciones de la membrana (esferocitosis congénita), enzimopatías (déficit G6PDH) y hemoglobinopatías (Talasemias, anemia falciforme), son de carácter hereditario y congénitas a excepción de la hemoglobinuria paroxística nocturna. Las Extracorpusculares, son adquiridas: por hiperesplenismo, inmune (causa más común, destrucción por autoanticuerpos), o por efecto tóxico (infecciones, químicos). El test de Coombs es de utilidad en las de causa inmune.

Nivel de manejo del médico general: Diagnóstico Sospecha Tratamiento Inicial Seguimiento Derivar

Aspectos Esenciales • • • • • • •

Generalmente de etiología autoinmune. En adultos mayores hay que sospechar neoplasias. Anemia regenerativa. Suelen ser normocíticas con aumento de LDH y bilirrubina indirecta. Hay hemosiderosis y hemoglobinuria en hemólisis intravascular grave. Esquistocitosis en hemólisis microangiopática. Transfusiones en casos sintomáticos.

Diagnóstico Cuadro Clínico: Depende de la severidad y forma de presentación. En casos agudos se pude presentar como un Síndrome anémico: debilidad, palidez mucocutánea, taquicardia, disnea, cefalea, astenia, adinamia. En casos crónicos el cuadro clínico puede ser menos marcado. Signos de hemólisis: Ictericia acolúrica, Esplenomegalia. Hemosideuria y/o hemoglobinuria en caso de hemólisis intravascular. Laboratorio: Bilirrubina indirecta aumentada, LDH aumentado, hiperplasia eritroblástica marcada en todas sus etapas de diferenciación. Al hemograma: Anemia normocítica (o microcítica) normocrómica regenerativa (Índice reticulocitario >3). Al frotis: macrocitosis (por reticulocitosis), policromasia, poiquilocitosis. Esquistocitos en anemia hemolítica microangiopática. Al Mielograma: hiperplasia eritroblástica marcada en todas sus etapas de diferenciación. Test de Coombs directo e indirecto: Permite detectar presencia de anticuerpos adheridos al glóbulo rojo (directo) o circulantes en el suero (indirecto). Pruebas complementarias, según clínica: Screening reumatológico: ANA, ENA, C3, C4. Serología viral: VIH, VHB, VHC, Parvovirus, etc. Estudio de imágenes: radiografía de tórax, ecografía abdominal. Pruebas específicas: Test de HAM, Resistencia osmótica, Determinación enzimática (G6PD , PK) y Citometría de Flujo : CD55, CD59, para estudio de hemogobinuria paroxítica nocturna.

Caso Clínico Tipo Mujer de 30 años, con antecedente de LES, al examen físico se encuentra palidez mucocutánea, debilidad, disnea e ictericia. En los exámenes de laboratorio se encuentra LDH aumentada y aumento de la bilirrubina indirecta.

Definición La hemólisis corresponde a la disminución de los glóbulos rojos a causa de su destrucción temprana antes de los 120 días. Esta destrucción compensa con una regeneración medular, pero si la destrucción supera la regeneración medular, entonces, sobreviene la anemia.

Etiología-Epidemiología-Fisiopatología Puede presentarse a cualquier edad, sin diferencias en sexo. La causa puede ser idiopática en aproximadamente 50%, siendo en el resto expresión de una enfermedad subyacente (LES, LLC, Sd. linfoproliferativo, neoplasia, drogas, infecciones etc). En pacientes jóvenes, generalmente hay una causa genética, a mediana edad un factor inmune, a edades mayores puede estar asociado a neoplasias. Según el lugar de destrucción del hematíe, la anemia se puede dividir en: Intravascular y Extravascular. En la Intravascular, la destrucción ocurre dentro de los vasos y puede haber hemosiderinuria y hemoglobinuria esto último debido a que si la hemólisis intravascular es grave, se produce saturación de la haptoglobina, proteina plasmática que liga la hemoglobina liberada, comenzando a filtrar

Tratamiento Sintomático: Transfusiones de sangre en casos sintomáticos, con hematíes lavados para no aumentar la hemólisis. En la hemólisis autoinmune: corticoides, plasmaféresis, esplenectomía e inmunosupresores como aziatropina y ciclo-

5 Facultad de Medicina, Universidad de Chile

fosfamida, el rituximab puede ser una opción en algunos de estos pacientes. En la medicamentosa se debe suspender el fármaco. Esplenectomía en esferocitosis familiar. En hemoglobinuria paroxística nocturna se debe realizar trasplante de progenitores hematopoyéticos. En talasemias transfusión para mantener Hb mayor a 9 g/dL, ácido fólico, prevención de sobrecarga de hierro mediante quelación con desferroxamina (parenteral) o deferasirox (oral); considerar la esplenectomía y el alotrasplante de médula ósea. Y tratamiento de enfermedad subyacente como en LES, leucemia linfática crónica, linfomas.

Seguimiento • •

Derivar a hematología para realizar estudios y seguimiento. Derivar a la especialidad que corresponda en caso de enfermedad subyacente.

Autor / Editor Diego Vidal

Año 2016

6 Facultad de Medicina, Universidad de Chile

Etiología-Epidemiología-Fisiopatología

Anemia Megaloblástica

Como mencionamos anteriormente este tipo de anemia se debe a un déficit de folatos y/o B12 (cobalamina), el que puede ser causado tanto por disminución de la ingesta como es el caso de los vegetarianos extrictos; como por la disminución de la abscorción como en el caso de pacientes gastrectomizados, enfermedad celiaca, resección intestinal, etc. La vitamina B12 o cobalamina ingerida debe formar un complejo con el factor intrínseco (secretado por células parietales gástricas), para ser luego absorbida en el íleon terminal. En el hígado es almacenada por 3 a 5 años, en grandes cantidades, por lo que es difícil su déficit por consumo. Los principales alimentos que aportan dicho nutriente son las carnes y los lácteos. La cobalamina participa en la síntesis de ADN transformando la metionina y metilmalonil Coa a Succinil Coa. La etiología más importante de déficit de B12 es la anemia perniciosa en la cual el paciente presenta anticuerpos anticélulas parietales y anti FI, aunque también puede ocurrir por falla en cualquier punto de su proceso digestivo.

Nivel de manejo del médico general: Diagnóstico Sospecha Tratamiento Inicial Seguimiento Completo

Aspectos Esenciales • • • • • • •

Anemia debido a déficit de Folato y/o B12 (Ac. Fólico + frecuente). Se caracteriza por macrocitosis (VCM > 100); en algunos casos cursa con pancitopenia. Tipo de anemia hiporregenetiva. Principales etiologías: Anemia perniciosa (B12), disminución de ingesta (acido fólico). Aunque la principal causa de macrocitosis SIN anemia es el alcoholismo. Se caracteriza por síntomas generales, GI y neurólogicos. El tratamiento consiste en suplir déficit nutricional, en algunos casos de por vida.

Los folatos se obtienen de verduras verdes y frutas, y las reservas se agotan en 4 o 5 meses. Se absorbe en el tubo digestivo a nivel del duodeno y primera porción del yeyuno. Se deposita en hígado, riñón y eritrocitos, con pocas reservas y es fácil su depleción por consumo. Su función a nivel molecular es la conversión de ácido forminoglutámico en ácido glutámico, metilación de homocisteína en metionina y síntesis de nucleótidos de pirimidinas en la síntesis del DNA. Las principales causas de de su déficit son la ingesta deficiente, que es la más frecuente (alcoholismo), malabsorción (Enf. Celiaca, Sd de Asa ciega.), aumento de requerimientos (embarazo, lactancia, anemia hemolítica, hipertiroidismo), aumento pérdidas (hemodiálisis) y fármacos antifólicos como anticonceptivos orales, antiepilépticos y sulfas.

Caso Clínico Tipo Paciente 65 años, vegetariano hace 7 años, con cuadro de CEG, fatiga, sensación de frío, taquicardia, soplo sistólico, dispepsia, constipación, epigastralgia, calambres. Al hemograma se observa anemia macrocítica; con leucopenia.

Definición Este término identifica a los pacientes con anemia y macrocitosis es decir, glóbulos rojos con volumen corpuscular medio mayor que 100 fL. Sin embargo lo que caracteriza a esta anemia es la megaloblastosis consistente en un gran tamaño de los precursores de las células sanguíneas en la médula osea. Se origina por el deficit de folato y vitamina B12, que participan en las reacciones de síntesis de ADN. El déficit de estos nutrientes produce una disminución en la velocidad de síntesis de ADN con un desarrollo citoplásmatico normal, lo que causa una alteración en la morfología y maduración de los precursores de la serie roja, alcanzando un tamaño mayor al normal. Muchos de estos precursores serán destruidos en la médula osea en un proceso conocido como eritropoyesis ineficaz. Esta alteración puede afectar también otras series hematológicas, por lo que es frecuente que curse con pancitopenia. Es importante recordar que otros trastornos como la aplasia medular, el hipotiroidismo y los síndromes mielodisplásicos también pueden cursar con anemia macrocítica.

Diagnóstico La sospecha es clínica, frente a un paciente con cuadro de sindrome anémico: fatigabilidad fácil, astenia, adinamia, sensación de frío, taquicardia y disnea; síntomas gastrointestinales: pirosis, dispepsia, constipación, glositis atrófica de Hunter y epigastralgia; síntomas neurológicos (sólo en déficit Vit B12 por alteración en la mielinización, ya que la vitamina B12 participa en la formación de adenosilmetionina), parestesia simétrica, en dedos de pies y manos, la alteración más característica es la degeneración combinada subaguda medular, en donde se producen alteraciones en los cordones laterales y posteriores de la médula espinal que se manifiesta como alteración de sensibilidad vibratoria y propioceptiva, en casos más severos ataxia, paraplejia y pérdida de control de esfínteres y en fases avanzadas puede ocasionar demencia. En ancianos, los síntomas pueden ser más sutiles, aunque en fase avanzada puede causar demencia. Al examen físico: palidez de piel y mucosas, glositis o lengua depapilada, lisa, queilitis angular, edema de extremidades inferiores, visceromegalia, petequias, equimosis. En lo neurológico: hiporeflexia, Romberg y Babinsky +. En casos

7 Facultad de Medicina, Universidad de Chile

agudos puede haber pérdida de sensibilidad en las manos, espasticidad e hiperreflexia.

B12 se produce una respuesta reticulocitaria rápida al cuarto o quinto día con normalización de los parametros en 1 a 1,5 meses. Se sugiere que los pacientes sean tratados inicialmente con B12 parenteral intramuscular. Especialmente en pacientes con síntomas neurológicos, donde el objetivo es maximizar la probabilidad de recuperación neurológica completa. Se inicia 1 mg cada día durante una semana, seguido de 1 mg cada semana durante cuatro semanas y luego, si el trastorno subyacente persiste, 1 mg cada mes durante el resto de la vida. Una vez que el estado del paciente se ha normalizado, la terapia de mantenimiento se puede realizar con B12 parenteral u oral.

Frente a estos síntomas se debe solicitar exámenes complementarios: • Hemograma: anemia en grado variable, macrocítica (VCM>100), a veces por compromiso de otras series (pancitopenia). • En el frotis: anisocitosis +, macrocitosis +++, con presencia de megalocitos (glóbulos rojos grandes, ovalados, hipercromos, poiquilocitosis, con dacriocitos). Pueden haber eritroblastos, e inclusiones en los eritrocitos como cuerpos de Howell Jolly, punteado basófilo o anillos de Cabot. En la línea de los leucocitos habrán neutrófilos poli o hipersegmentados (más de 5 lóbulos nucleares). A nivel plaquetario habrá trombocitopenia.

Déficit de folato: Los pacientes con deficiencia de folato deben ser tratados con ácido fólico oral. La dosificación estándar de ácido fólico por vía oral es de 1 a 5 mg/ día por vía oral durante uno a cuatro meses, o hasta que se produce la recuperación hematológica completa. La vía oral es suficiente incluso en aquellos con mala absorción.

A nivel secundario el estudio se complementa con: • Niveles Plasmáticos de Vitamina B12 y Acido fólico: Valores bajos son diagnóstico (VN: Vit B12 150-400 pg/ml, Folato eritrocitario 6-20 ng/ml, Folato sérico 150-600 ng/ cm3) • LDH y bilirrubina indirecta: aumentadas por eritropoyesis ineficaz en la médula ósea. • Test de Schilling: Poco utilizado en la actualidad. Se administra B12 oral marcada, seguido de B12 parenteral no marcada. Si la excreción urinaria a las 24 hrs de B12 es menor de 6 %, los resultados son anormales. En dicho caso se administra B12 marcada unida a FI, seguido de parenteral no marcada. Si la excreción urinaria es anormal (>6%), se hace el diagnóstico de Anemia Perniciosa. • Biopsia médula ósea: Hay asincronía nucleo citoplasma (megaloblastosis), hiperplasia eritroide. Útil ante la sospecha de mielodisplasia, linfoma. • Otros: AC anti FI, AC anti células parietales, Medición de ácido metil malónico y homocisteína plasmática, Endoscopia digestiva alta. A considerar en déficit de B12: hiperhomocisteinemia + aumento de ac. Metilmalónico. En cambio déficit de B6: hiperhomocisteínemia con ac. Metilmalonico en niveles normales.

Seguimiento En anemia perniciosa seguir con endoscopía digestiva alta, ya que 4% de los pacientes desarrolla carcinoma gástrico. Aportar adicionalmente folato frente a aumento de requerimientos o consumo: embarazo, lactancia, prematuros, hemodiálisis, hemólisis, resección íleon terminal, gastrectomizados.

Autor / Editor Nicole Fritzche

Tratamiento Déficit de B12: La deficiencia de vitamina B12 se debe descartar y tratar, de estar presente, antes de dar el ácido fólico a un paciente con anemia megaloblástica, ya que la administración de ácido fólico puede empeorar las complicaciones neurológicas de la deficiencia de vitamina B12 no tratada. El tratamiento se puede administrar por vía parenteral (intramuscular) u oral. Para los pacientes con disminución de la capacidad para absorber la vitamina B12 de la dieta de forma permanente (por ejemplo, anemia perniciosa, gastrectomía total o extirpación quirúrgica del íleon terminal) el tratamiento es de por vida. Si la causa de la deficiencia de B12 puede ser eliminado (por ejemplo, dieta, drogas, síndrome de mala absorción reversible), el tratamiento se puede detener cuando la deficiencia de B12 ha sido completamente invertida y la causa eliminada. Luego de la administración de vitamina

8 Facultad de Medicina, Universidad de Chile

Año 2016

Coagulopatías Adquiridas

mo de estos (CID, hemorragia) y la presencia de anticuerpos inhibidores de factores (factor VIII es el más frecuente). También dentro de las alteraciones de la hemostasia están los desordenes de la fibrinólisis y los desórdenes trombo-hemorrágicos complejos.

Nivel de manejo del médico general: Diagnóstico Sospecha Tratamiento Inicial Seguimiento Derivar

Las causas más frecuentes de coagulopatias adquiridas en la clínica son: enfermedad hepática crónica, coagulación intravascular diseminada (CID) y el déficit de vitamina K. Enfermedad Hepática Crónica: El hígado desempeña un papel importante en el equilibrio hemostático, ya que participa activamente en la síntesis de proteínas procoagulantes (fibrinógeno, factores II, V, VII) así como de otras con función inhibidora o reguladora de la coagulación (antitrombina, proteínas C y S). La buena función hepática no solamente influye en la cantidad de esas proteínas circulantes, sino que puede afectar su estructura o composición bioquímica. Por otra parte, también es conocido que en la enfermedad hepática puede existir un estado de activación exacerbado del sistema fibrinolítico, secundario a una anomalía del aclaramiento plasmático del activador tisular del fibrinógeno. Además la trombocitopenia variabe asociada con el hiperesplenismo, acompaña frecuentemente a la cirrosis hepática, lo que también condiciona una mayor tendencia hemorrágica, que a su vez se intensifica si coexisten alteraciones funcionales de las plaquetas.

Aspectos Esenciales • • • •

Los defectos adquiridos de la coagulación pueden afectar distintos niveles de la hemostasia. Se asocian a enfermedades sistémicas, las más frecuentes: CID, hepatopatías crónicas y déficit de vitamina K. Siempre hay que indagar antecedentes familiares y complicaciones hemorrágicas. Realizar los exámenes iniciales de coagulación para tener una aproximación diagnóstica.

Caso Clínico Tipo Paciente de 50 años, alcohólico crónico y con el antecedente de várices esofágicas. Al examen físico se encuentran equimosis en ambos brazos y petequias en los pies. Refiere además que presentó complicaciones (hemorragia) luego de una cirugía.

Coagulación Intravascular Diseminada (CID): se define como una activación intravascular generalizada de la coagulación asociada a diversas causas como resultado de la formación de fibrina, y en la oclusión trombótica de vasos de mediano y pequeño calibre. Las principales consecuencias de la CID son el consumo de las proteínas hemostáticas y de las plaquetas, lo cual lleva a originar fenómenos hemorrágicos y la obstrucción trombótica de los vasos, lo cual compromete la función de diversos órganos al verse reducido su aporte sanguíneo. El cuadro llevado a sus últimas consecuencias da lugar a un fallo multiorgánico.

Definición Las coagulopatías adquiridas son procesos muy frecuentes en la práctica clínica. En general, son secundarios a una serie de enfermedades sistémicas que alteran algún nivel de la hemostasia (primaria, secundaria o fibrinólisis) y la mayoria de las veces son resultado del consumo de factores de coagulación, como en la coagulación intravascular diseminada (CID), defecto de síntesis de factores, como en las hepatopatías y déficits de vitamina K o como resultado de la presencia de anticoagulantes circulantes o de fármacos que alteran la coagulación (antivitaminas K, acenocumarol o warfarina). La intensidad de la hemorragia es muy variable, existiendo pobre correlación con las alteraciones hemostáticas detectadas en los estudios rutinarios de coagulación.

Déficit de Vitamina K: Es la principal causa de déficit en la síntesis de factores plasmáticos de la coagulación, concretamente de los llamados vitamina K dependientes, factor II, factor VII, factor IX y factor X, afectándose también la síntesis de anticoagulantes naturales, como son las proteínas C y S. La vitamina K es necesaria para la carboxilación de los residuos glutámicos de los citados factores de la coagulación, gracias a la gammaglutamil carboxilasa. Tras la carboxilación estas proteínas ganan afinidad por los fosfolípidos cargados negativamente en la superficie celular, especialmente plaquetaria, iniciándose la coagulación. Además, la célula posee reductasas que van a regenerar la vitamina K utilizada, transformándola de nuevo a su forma activa. La vitamina K aportada por la dieta se denomina VK1, y sus necesidades son en torno a 1 mg/kg al día. Además, las bacterias intestinales producen vitamina K2. El déficit puede deberse a diversos motivos: aporte deficitario, alteración en la absorción y antagonistas de la vitamina K tales como los anticoagulantes orales acenocumarol y warfarina.

Etiología-Epidemiología-Fisiopatología Las alteraciones en la hemostasia primaria son a causa de disfunción plaquetaria (fármacos antiagregantes, uremia, paraproteinemias) o más frecuentemente por estados trombocitopénicos: Inmune (PTI, drogas), disminución de producción (citotóxicos), secuestro (hiperesplenismo), o consumo (CID, PTT, hemorragia masiva) . Las alteraciones en la hemostasia secundaria son debidas a síntesis disminuida de factores (drogas, déficit Vit K, hepatopatía, hipotiroidismo), el consu-

9 Facultad de Medicina, Universidad de Chile

Diagnóstico

consumo de los factores de la coagulación, trombocitopenia secundaria al consumo plaquetario, el fibrinógeno se encuentra elevado al inicio, ya que actúa como reactante de fase aguda para finalmente, si el cuadro se prolonga en el tiempo, se ven reducidos sus niveles, por último aumento de dímero D como consecuencia de la generación de fibrina y descenso de AT-III, proteína C y proteína S. En el Déficit de Vitamina K: El cuadro es poco sintomático. En situaciones más graves pueden aparecer equimosis y hematomas subcutáneos y musculares, junto con hemorragias de mucosas, siendo más frecuente en los tractos gastrointestinal y genitourinario. Diagnóstico al laboratorio se caracteriza por el alargamiento del TP y TTPa.

La anamnesis, debe incluir, al menos: Historia sugerente de patología congénita, tal como antecedentes familiares de hemorragias y hemorragias en la infancia. Importante para identificar una diátesis hemorrágica es el antecedente de cirugías o extracción dentaria con complicaciones hemorrágicas, tipo e intensidad de hemorragias, características de las menstruaciones. En general las hemorragias digestivas, hematuria y menometrorragia de inicio reciente se asocian a problemas locales, más que a coagulopatías. Importante para identificar una causa de coagulopatía es la historia de ingesta de medicamentos, hábitos (OH), cirugías (cardíaca se asocia a Ac inhibidores del factor V).

Tratamiento

El examen físico puede orientar al diagnóstico, equimosis, petequias y sangramiento de mucosas orienta a trastorno primario, y los sangrados profundos como la hemartrosis, sugiere trastorno de hemostasia secundaria. Buscar signos de daño hepático crónico, aliento urémico, signos de autoinmunidad (lupus y enfermedad reumática los más importantes). Dentro de los exámenes iniciales están, para estudio de hemostasia primaria: Recuento de plaquetas y el frotis (esquistocitos, pseudotrombocitopenia por acumulos de plaquetas por EDTA) y tiempo de sangría (que evalúa contacto de plaquetas con vasos sanguíneos), cada vez menos usado por variabilidad de la técnica y por lo tanto de los resultados; Estudio de hemostasia secundaria: tiempo de protrombina (TP) que evalúa via extrínseca (F VII), y vía común (F X, V, II, trombina y fibrinógeno), tiempo de trombina, que evaúa la producción de fibrinógeno y el tiempo de tromboplastina parcial activada (TTPa), que evalúa la vía intrínseca (F VIII, IX, XI y XII) y la vía común. También se usa el estudio de mezcla de plasma (plasma con bajo factor más plasma normal) para diferenciar déficit de factores (TTPa o TP se normaliza) o uso de inhibidores (TTPa o TP se mantiene alterado), y el nivel de fibrinógeno. En Hepatopatías Crónicas: Los pacientes con hepatopatías estables no suelen sangrar, a excepción de algunos hematomas cutáneos y equimosis. El cuadro hemorrágico se precipita por complicaciones de la propia enfermedad como rotura de varices esofágicas o el establecimiento de una CID por necrosis hepática aguda o cuadro séptico. Al laboratorio se observa descenso de los tiempos de coagulación (TP y TTPa) debido al déficit de factores de la coagulación, disminución de la concentración plasmática de proteína C, S y antitrombina III (AT-III), también Trombocitopenia y alteración en las pruebas de función plaquetaria y disminución de 2-antiplasmina y aumento de dímero D. En la CID: Se manifiesta con hemorragia, que va desde un sangrado por puntos de punción hasta equimosis generalizadas y sangrado mucoso que puede llevar a hemorragia gastrointestinal grave. Por otra parte, el depósito de fibrina a nivel de la microcirculación puede originar fenómenos trombóticos que van desde lesiones necróticas a nivel distal, hasta afectar a órganos vitales dando lugar a un fallo multiorgánico, como por ejemplo el síndrome de Watherhousen-Friderichsen que aparece en la sepsis meningocócica. Al laboratorio se ve un alargamiento de los tiempos de coagulación (TP y TTPa) por

El tratamiento dependerá de la patología que causa el desorden de la hemostasia y debe ser individualizado puede requerir el uso apropiado de hemoderivados, plasma, plaquetas y crioprecipitado, en combinación con agentes farmacológicos, tales como el concentrado de factores del complejo protrombínico y antifibrinolíticos. En falla hepática crónica, se trata la hemorragia con ligaduras, TIPS, terlipresina; y transfusión de plasma fresco congelado cada 6-12 horas. En el caso de CID, la principal maniobra terapéutica consiste en corregir la causa que está originando la CID. En segundo lugar, es necesario hacer un tratamiento de soporte para paliar los dos principales síntomas, la hemorragia y la trombosis. Principalmente se realiza un tratamiento sustitutivo con plasma fresco congelado para reponer los factores de la coagulación y los concentrados de plaquetas Por último, el tratamiento del deficit de vitamina K se basa en aportar vitamina, ya sea por vía oral cuando no exista un problema de malabsorción, o por vía parenteral (tanto intramuscular como endovenosa). Generalmente, la administración de 10 mg diarios durante 3 días suele ser necesaria para paliar el déficit. En caso de hemorragias intensas que comprometen la vida del paciente puede ser necesaria la administración de plasma fresco o concentrado de complejo protrombínico activado.

Seguimiento Derivar a hematología para estudio de la patología.

Autor / Editor Marco Ayala

10 Facultad de Medicina, Universidad de Chile

Año 2016

Leucemias Agudas

ciones genéticas que alteran la diferenciación, velocidad de crecimiento y sobrevida celular. Las células neoplásicas se acumulan en la medula ósea, desplazando la hematopoyesis normal, infiltran a la sangre y otros órganos. El mecanismo subyacente más frecuente es la traslocación cromosómica con activación de protoncogenes asociados tales como la mutacion t(9:22) [BCR/ABL], en la leucemia linfática, y las leucemias mieloides se asocian a distintas mutaciones que se asocian a subtipos, tal como la mutación t(15:17), en la LMA M3 promielocítica.

Nivel de manejo del médico general: Diagnóstico Sospecha Tratamiento Inicial Seguimiento Derivar

Aspectos Esenciales • • •

Relacionado con la exposición a radiaciones. Clínica: Anemia, fiebre, hematomas y sangrado. Las medidas buscan disminuir la probabilidad de infecciones, y mantener niveles sanguíneos normales.

Diagnóstico De instalación rápidamente progresiva en uno o dos meses. La clínica viene dada por las citopenias , casi todos los pacientes padecen síndrome anémico, fiebre, infecciones y aparición de hematomas y sangrado a causa de la infiltración de las células malignas a la medula. Por infiltración de los blastos a otros tejidos puede haber: hepato-esplenomegalia (principalmente en linfoblástica), adenopatías, compromiso del SNC, del timo, de piel y encías, del hueso (dolor óseo) y partes blandas, y testículos. Algunas formas presentan CID, tal como la LMA M3 o promielocitica.

Caso Clínico Tipo Hombre de 40 que desde hace un mes refiere debilidad, disnea, fiebre por varios días y múltiples hematomas en el cuerpo. Al examen físico, hepatomegalia leve, palidez mucocutánea, petequias a nivel de boca. Al laboratorio, Hto 30%, plaquetas 100.000, leucocitos 19.000.

Al laboratorio el hemograma muestra anemia y trombocitopenia en 80%. Los leucocitos pueden ser normales, disminuidos (10% de leucemias aleucemincas) o aumentados, pero habitualmente se observa neutropenia. Al frotis se aprecia aumento de las formas inmaduras de la serie blanca (blastos), sin células en estadios madurativos intermedios (hiatus leucémico). LDH aumentada apoya la sospecha. En el mielograma, con una presencia superior de 20% de blastos confirma el diagnostico.

Definición Las Leucemias Agudas son neoplasias cuyo origen es la proliferación clonal de células hematopoyéticas de la médula osea (MO) incapaces de madurar que pierden la capacidad de diferenciación y apoptosis. Por ello, se definen por una proporción de blastos en MO > 20%. Según la línea celular alterada se dividen en Mieloides (LAM), generalmente de peor pronóstico, y Linfoide (LLA), típica en niños y jovenes.

Tratamiento Se debe hospitalizar al paciente y realizarle exámenes generales. Las medidas generales tienen como objetivo: Reducir carga bacteriana, evitar contaminación con gérmenes intrahospitalarios, aislamiento, régimen cocido y lavado de manos, protección renal con hiperhidratación y Alopurinol, para prevenir el síndrome de lisis tumoral, manejo de cuadros febriles con antibióticos de amplio espectro endovenosos y transfusiones para mantener el hematocrito > 20% y plaquetas > 20.000. Notificar GES y derivar a especialista con 24 horas de plazo. De ser necesario, derivación urgente a centro de mayor complejidad.

Etiología-Epidemiología-Fisiopatología Su etiología es frecuentemente idiopática y se puede asociar a mutaciones genéticas. Las leucemias, en general, son mas frecuentes en hombres (56%) que en mujeres. La incidencia varía de acuerdo a la edad y tipo de leucemia. Las leucemias representan menos del 3% del total de canceres con 3 a 4 casos nuevos por 100.000 hab/año y con un predominio mayor en el hombre en una razón 1,5:1. La tasa de mortalidad en Chile para ambos sexos fue 3,7/100.000, 54% correspondieron a hombres y 46% a mujeres, según DEIS 2007. Durante ese mismo año, en la población de 15 años o más, entre un 60% a 70% de los casos nuevos de leucemia correspondieron a leucemias agudas. Las Leucemias Agudas Mieloblásticas se presentan en un 80% en adultos, 20% en niños, y las Linfoblásticas en un 20% en adultos y 80% en niños.

Importante es disminuir la cantidad de blastos, mientras se realiza el estudio específico (citometría de flujo, cariograma, estudio molecular de mutaciones específicas, mielograma), con dexametasona en caso de Leucemia Linfática y de hidroxicarbamida en caso de Leucemias Mieloides. En caso de hiperviscosidad asociada a hiperleucocitosis (> 150.000 leucocitos), se puede hacer leucoferesis, manejo habitualmente realizado en centros especializados.

Son factores de riesgo las radiaciones ionizantes, radioterapia, algunos virus (VEB, HIV, HTLV-1). Se producen muta-

11 Facultad de Medicina, Universidad de Chile

En formas de mal pronóstico el tratamiento es trasplante alogénico, en el resto se realiza quimioterapia. La leucemia promielocitica M3 se asocia con CID y su tratamiento es con acido transretinoico.

Seguimiento Derivar al especialista apenas se sospeche.

Autor / Editor Nicole Fritzche

Año 2016

12 Facultad de Medicina, Universidad de Chile

Púrpuras Vasculares

pequeños mediada por IgA1 (con alteraciones de la glicosilación) y con activación de la vía alternativa del complemento asociado a depósito de inmunocomplejos IgA. Se trata de un cuadro autolimitado que afecta principalmente a niños de 3 a 15 años y es la vasculitis más frecuente en la infancia. Se caracteriza, tanto en estos como en adultos, por la tétrada clásica de púrpura palpable (en ausencia de trombocitopenia o coagulopatía), artritis o artralgias, dolor abdominal y compromiso renal. El antígeno responsable se desconoce, pero aproximadamente en la mitad de los casos existe el antecedente de una infección respiratoria alta (estreptocócica, vírica o de otro tipo).

Nivel de manejo del médico general: Diagnóstico Sospecha Tratamiento Inicial Seguimiento Derivar

Aspectos Esenciales • • • •

Los cuadros se caracterizan por pequeñas hemorragias cutáneas palpables. La etiología más frecuente es inflamatoria. El diagnóstico es principalmente clínico. El manejo inicial consiste en el manejo de los síntomas, el balance hídrico y la anemia.

El Síndrome de Telangiectasia Hereditaria Hemorrágica (de Rendu-Osler-Weber) es un desorden vascular autosómico dominante con una prevalencia de 1: 5000 a 1: 8000. Se caracteriza por epistaxis, sangrado gastrointestinal, anemia ferropénica, telangiectasias en piel y mucosas y presencia de malformaciones arterio-venosas (pulmonares, hepáticas y cerebrales). Aunque esta enfermedad es hereditaria, las manifestaciones clínicas no están generalmente presentes en el nacimiento, sino que se desarrollan con la edad. La epistaxis es generalmente el primer signo de la enfermedad, que se producen a menudo en la infancia, las malformaciones arteriovenosas pulmonares (PAVMs) generalmente se vuelven sintomáticas después de la pubertad.

Caso Clínico Tipo Paciente de 13 años, con antecedente de infección de vía aérea alta reciente, consulta por 10 días de rash petequial palpable y edema en extremidades y dolor abdominal. En el examen de orina se observa proteinuria leve. El diagnóstico más probable es: a. Síndrome de Rendu-Osler-Weber b. Púrpura trombocitopénico trombótico c. Ingestión de AAS d. Púrpura de Schönlein-Henoch e. Púrpura trombocitopénico inmune

Diagnóstico El diagnóstico es principalmente clínico y se sospecha por la presencia de manifestaciones típicas de vasculitis: petequias, púrpura palpable, nódulos subcutáneos, úlceras cutáneas superficiales y compromiso sistémico, en ausencia de trombocitopenia y/u otros trastornos de la coagulación. La presencia de la tétrada clásica de púrpura palpable, artralgias o artritis, compromiso gastrointestinal y renal en un paciente joven, es altamente sugerente de Púrpura de Schönlein-Henoch. En pacientes con una presentación incompleta o inusual, la biopsia del órgano afectado (por ejemplo, piel o riñón) demostrando predominantemente el depósito de inmunoglobulina A (IgA) apoya el diagnóstico. La ecografía abdominal está indicada en pacientes con dolor abdominal severo. Es capaz de detectar un aumento de grosor de la pared intestinal, hematomas, líquido peritoneal, y la invaginación intestinal. El diagnóstico definitivo de Telangiectasia hemorrágica hereditaria se realiza al tener los 4 criterios siguientes: epistaxis espontánea y recurrente, telangiectasias mucocutáneas múltiples, compromiso visceral (gastrointestinal, respiratorio, cerebral o hepático) y existencia de algún familiar de primer grado con la enfermedad. El diagnóstico puede confirmarse mediante pruebas genéticas de mutaciones que implican la endoglina, ALK-1, o SMAD4. El diagnóstico diferencial es con otros cuadros purpúricos, vasculitis, Lupus eritematoso sistémico, enfermedad renal primaria, entre otros.

Definición Cuadros caracterizados por hemorragias cutáneas, en forma de petequias y equimosis, generalmente solevantadas y palpables, por extravasación o salida anormal de glóbulos rojos fuera de los capilares y otros vasos pequeños acumulándose en la dermis y que se producen debido a múltiples desordenes que afectan la estructura de estos vasos sanguíneos. Dentro de los cuadros catalogados como púrpuras vasculares se encuentran el Púrpura de Schönlein-Henoch, el Síndrome de Telangiectasia Hemorrágica Hereditaria (Rendu-Osler-Weber), las vasculitis sistémicas, enfermedades infiltrativas (Amiloidosis), púrpura por autosensibilización eritrocitaria y enfermedades con atrofia del tejido conectivo (Púrpura senil, Síndrome de Ehlers-Danlos), entre otros.

Etiología-Epidemiología-Fisiopatología En general, el origen más frecuente de los púrpuras vasculares es de tipo inflamatorio y/o inmunológico (a su vez asociado a enfermedades con este último componente), con depósito de complejos antigénicos. El púrpura de Schönlein-Henoch es una vasculitis de vasos

13 Facultad de Medicina, Universidad de Chile

Tratamiento

Autor / Editor Marcelo Fres

La mayoría de los pacientes con púrpura de Henoch-Schönlein (HSP), pueden ser atendidos en el medio ambulatorio con la terapia dirigida hacia una adecuada hidratación oral, reposo en cama, y el alivio sintomático del dolor articular y abdominal. La hospitalización está indicada en pacientes que no mantienen la hidratación oral y que requieren la administración de líquidos por vía intravenosa. El manejo hospitalario también puede ser necesario para tratar a los pacientes que tienen sangrado gastrointestinal significativo, dolor abdominal intenso, cambios en el estado mental, grave afectación articular que limita la deambulación y/o cuidado de sí mismo, o evidencia de enfermedad renal significativa (creatinina elevada, hipertensión o proteinuria). En pacientes con dolor en las articulaciones y/o abdominal, se sugiere el uso de un agente antiinflamatorio no esteroideo como naproxeno (10 a 20 mg/kg divididos en dos dosis por día). En pacientes con dolor abdominal intenso que interfiere con la ingesta oral y que no responden a un agente antiinflamatorio no esteroídeo, se sugiere el uso de los glucocorticoides sistémicos como prednisona oral (1 a 2 mg/kg por día) para niños o adultos, con una dosis máxima de 60 a 80 mg por día, o dosis equivalentes de metilprednisolona parenteral (0,8 a 1,6 mg/kg por día, dosis máxima de 64 mg por día). Los glucocorticoides deben disminuirse gradualmente, en no más de 25 por ciento por semana. Los principales objetivos en el manejo de pacientes con síndrome de telangiectasia hemorrágica hereditaria incluyen la prevención de las complicaciones conocidas y tratamiento oportuno de las complicaciones de las malformaciones arteriovenosas (MAVs) sintomáticas. Los pacientes con MAVs pulmonares están en riesgo de secuelas neurológicas por embolia paradójica, lo que indica la necesidad de un diagnóstico precoz y la intervención (por ejemplo: la embolización de MAVs, profilaxis antibiótica para procedimientos intervencionistas, especialmente dental). El tratamiento de las complicaciones es generalmente el mismo que el tratamiento de la misma condición en ausencia de este síndrome. La mayoría de las pacientes con síndrome de telangiectasia hemorrágica hereditaria puede tener un embarazo exitoso, sin embargo, deben ser tratados como de alto riesgo, en vista de la tasa de mortalidad materna de 1%.

Seguimiento Si bien el seguimiento de estos cuadros es de resorte del especialista, se recomienda reevaluar a estos pacientes con mediciones seriadas de presión arterial, de la función renal y análisis de orina.

14 Facultad de Medicina, Universidad de Chile

Año 2016

Síndrome Paraneoplásico

• •

Nivel de manejo del médico general: Diagnóstico Inicial Tratamiento Inicial Seguimiento Derivar

• •

Aspectos Esenciales





• •

• • •

Cualquier manifestación clínica atípica en un paciente canceroso obliga a descartarlos. Muchas veces son hallazgos en exámenes de laboratorio, previos al diagnóstico de la enfermedad neoplásica. Suelen ser asintomáticos Requiere manejo por especialista

Sd SIADH: más común en cáncer microcítico de pulmón, carcinoides y en lesiones del SNC. Generalmente hiponatremia asintomática. Síndrome de Cushing por secreción ectópica de ACTH: la causa más común es el cáncer de células pequeñas de pulmón. Suele ser asintomático. Eritrocitosis: por secreción ectópica de EPO. Suele ser asintomática. Granulocitosis: casi siempre asintomática y no hay desviación izquierda. Trombocitosis: 35% de las trombocitosis poseen una enfermedad neoplásica subyacente. Eosinofilia Tromboflebitis: TVP y TEP son las formas de presentación más frecuentes. Un 15% de los pacientes con TVP o TEP tienen un diagnóstico de cáncer. Más común en cáncer de pulmón, páncreas y tubo digestivo.

b) Síndromes Neurológicos: por distintos mecanismos. Son complejos y variados, principalmente relacionados con mecanismos inmunológicos; causados no sólo por neoplasias del SNC. Su mecanismo de producción no está relacionado con metástasis del cáncer, apoplejía, trastornos metabólicos, infecciosos, o por efectos secundarios de la terapia del cáncer. En el 60% de los casos, las manifestaciones neurológicas preceden el diagnóstico de cáncer.

Caso Clínico Tipo Paciente de 65 años, tabáquico, con diagnóstico de cáncer microcítico de pulmón hace 4 meses. Hace 1 mes presenta letargia, náuseas y confusión. El perfil bioquímico impresiona con una hiponatremia moderada. Al interrogar dirigidamente al paciente, refiere no tener sed durante el día, y haber tenido un episodio de convulsiones.

c) Síndromes Gastrointestinales: • Enteropatía pierde-proteínas • Anorexia y caquexia

Definición

d) Síndromes Renales

Los síndromes paraneoplásicos (SP) son un grupo de síndromes endocrinológicos, dermatológicos, hematológicos y/o neurológicos causados por los efectos a distancia de un tumor, y pueden afectar cualquier órgano blanco. Dicha sintomatología, no se explica por el compromiso de la infiltración tumoral en dicho órgano, sino por ser causada por diferentes factores, hormonas, citoquinas, interleuquinas o respuestas inmunes contra antígenos tumorales que tienen una reacción cruzada con antígenos, y su identificación nos permite sospechar la existencia de un tumor en alguna parte del organismo. La mayoría de estos procesos preceden el diagnóstico del cáncer, por lo que la identificación de ellos permite descubrir el cáncer en fases tempranas, cuando es más posible su curación o tratamiento. La importancia de reconocerlos radica en que pueden ser la primera manifestación temprana de malignidad, como así también pueden complicar el tratamiento y empeorar el pronóstico del paciente con cáncer.

e) Síndromes Cutáneos • Enfermedad de Paget • Enfermedad de Bowen • Acantosis Nigricans • Dermatomiositis • Ictiosis adquiridas • Eritema necrolítico migratorio • Eritema gyratum repens • Otros

Diagnóstico La historia clínica y el exámen físico son esenciales para el diagnóstico sindromático y etiológico. Un 60% de los pacientes con Síndrome Paraneoplásico (SP) presentan signos antes de ser diagnosticados como portadores de una malignidad. Antes de afirmar el origen paraneoplásico de cierto cuadro, es necesario descartar cualquier otra causa, y luego pensar en la posibilidad de asociación maligna, por eso el diagnóstico de un síndrome paraneoplásico es por exclusión. Tras la sospecha de un SP se indica comenzar la búsqueda del cáncer, para esto se deben realizar estudios de imagen altamente sensibles y detección en suero de marcadores tumorales de manera periódica. El diagnóstico se confirma tras el hallazgo del cáncer asociado. En aquellos pacientes ya diag-

Etiología-Epidemiología-Fisiopatología a) Síndromes Endocrino-Hematológicos: • Hipercalcemia por producción ectópica de PTHrP: Más frecuente en cáncer de pulmón, mamas, cabeza y cuello. Suele ser asintomática.

15 Facultad de Medicina, Universidad de Chile

nosticados con cáncer y que presentan signos a distancia, es indispensable antes de hacer el diagnóstico de un SP descartar otras causas mucho más frecuentes como son metástasis, efectos medicamentosos, etc.

Tratamiento El tratamiento está sujeto a especialistas dada la complejidad y amplia gama de síndromes paraneoplásicos. Dependiendo del síndrome, el tratamiento de la causa subyacente suele ser suficiente. En otros casos, el síndrome debe ser tratado de forma primaria también.

Seguimiento Debe ser un seguimiento multidisciplinario por especialistas.

Autor / Editor Julián Bravo

Revisor de Formato Angel Eloy

Año 2016

16 Facultad de Medicina, Universidad de Chile

Diagnóstico

Trombofilias

Cuadro clínico: Aparición de trombosis venosa y/o episodio de embolia pulmonar (en menor proporción, asociado a hiperhomocisteinemia y sd. antifosfolípido). La incidencia de defectos trombofílicos heredados es muy variable y éstos interactúan con los factores de riesgo adquiridos en forma sinérgica. Los elementos clínicos que apoyan una trombofilia hereditaria son: Historia familiar de trombosis (1er. grado), trombosis en paciente joven (<50 años), trombosis sin causa etiológica conocida, trombosis recurrente, trombosis en el embarazo, trombosis en sitios inusuales (portal hepático, mesentérico, axilar, cerebral), trombosis arterial y venosa, y necrosis cutánea inducida por cumarínicos.

Nivel de manejo del médico general: Diagnóstico Sospecha Tratamiento Inicial Seguimiento Derivar

Aspectos Esenciales • • • • • •

Estado de hipercoagulabilidad. Causa hereditaria más frecuente: factor V Leiden. Al factor hereditario se le suma otro gatillante. Se presenta con trombosis venosa o embolia pulmonar. Laboratorio necesario para precisar etiología. Tratamiento en la fase aguda es con HBPM.

Importante investigar los factores que precipitaron este episodio y descartar cáncer (páncreas, gástrico, vejiga, útero, renal, pulmón y mieloma múltiple). Antecedentes de aborto o mortinatos hacen sospechar síndrome antifosfolípido, al examen físico se podría observar livedo reticularis. Laboratorio: hemograma completo con reticulocitos, recuento de plaquetas y observación del frote sanguíneo, estudio basal de coagulación, perfil bioquímico, hepático, función renal y orina completa.

Caso Clínico Tipo Paciente sexo femenino, 37 años, con antecedentes de aborto a las 10 semanas y de TVP distal hace 3 años, un mes post cirugía de rodilla (meniscectomía endoscópica). Dos de sus hermanas tienen infertilidad primaria sin antecedentes de TEV.

Ante sospecha de trombofilia hereditaria y síndrome antifosfolípido deben realizarse un mínimo de exámenes: • PCR Factor V (Factor Leiden). • Mutación G 20210 A Gen de la Protrombina. • Proteína C. • Proteína S Libre. • Anticoagulante Lúpico. • Antitrombina. • Resistencia a la Proteína C Activada.

Definición Alteración hereditaria, adquirida o mixta del balance entre factores procoagulantes y anticoagulantes, resultando en un estado de hipercoagulabilidad, aumentando el riesgo trombosis venosa, arterial o ambas.

Tratamiento

Etiología-Epidemiología-Fisiopatología

El tratamiento del evento trombótico agudo es más o menos similar en todas las trombofilias y no difiere del tratamiento inicial de una trombosis, sin embargo el manejo a largo plazo, la profilaxis de la enfermedad tromboembólica y la morbilidad asociada, varía según el tipo de trombofilia, el número de eventos trombóticos, los sitios comprometidos y la presencia de otros factores de riesgo. Para el manejo inicial: la anticoagulación debe iniciarse de inmediato ya que un retraso aumenta el riesgo de embolización y la muerte. Se debe evaluar el riesgo de hemorragia antes de la anticoagulación. La mayoría de los médicos están de acuerdo en que la anticoagulación debe ser administrado a pacientes con bajo riesgo de hemorragia y evitar en los de alto riesgo.

La etiología del síndrome de hipercoagulabilidad es multifactorial (factores hereditarios, adquiridos o mixtos). Existe un gran número de personas con trombofilia que nunca presentan una trombosis y en otros será recurrente desde su juventud. Por lo tanto, la existencia de una trombofilia no puede considerarse una enfermedad per se. Estados de hipercoagulabilidad hereditarios: Factor V Leiden, Alteración protrombina G20210A, Deficiencia Antitrombina III, Deficiencia Proteína S, Deficiencia Proteína C. La trombofilia hereditaria más frecuente es por mutación del gen del factor V de la coagulación (factor V Leiden), que le da resistencia a la acción proteolítica de la proteína C activada, con una prevalencia del 3-7% en la población general y 20% en pacientes con trombosis. La sigue en frecuencia la mutación G20210A del gen de protrombina, que aumenta la cantidad de protrombina circulante con una prevalencia del 1-3% en la población general y 6% en pacientes con trombosis.

Las opciones de tratamiento para la anticoagulación inicial son los siguientes: heparina de bajo peso molecular (HBPM), fondaparinux, heparina no fraccionada (HNF), el factor Xa oral y los inhibidores directos de la trombina. En general para la mayoría de los pacientes, se sugiere he-

17 Facultad de Medicina, Universidad de Chile

parina de BPM. El fondaparinux es una alternativa aceptable para las pacientes no embarazadas. Para los pacientes con insuficiencia renal grave (por ejemplo, aclaramiento de creatinina <30 ml/min), o para pacientes en los que hay una alta probabilidad de tener que interrumpir de forma aguda la anticoagulación se sugiere HNF. La anticoagulación completa debe garantizarse durante la transición de la terapia inicial a la terapia de mantenimiento. Actualmente se recomienda el inicio conjunto de HBPM y anticoagulante oral (como la warfarina) desde el primer día, y la heparina se continúa durante un mínimo de cuatro a cinco días hasta que la Ratio Normalizada Internacional (INR) esté dentro del intervalo terapéutico (2 a 3; apuntar 2,5) durante un mínimo de 24 a 48 horas. En la suspensión del TACO debe considerarse la trombosis residual por eco-Doppler venoso y el Dímero D. Existen condiciones en las cuales se recomienda la anticoagulación indefinida: • Dos o más trombosis espontáneas. • Una trombosis espontánea en: pacientes con deficiencia de antitrombina o 
síndrome antifosfolípido, presencia de riesgo vital, en un sitio inusual (vena cerebral), en presencia de más de un defecto genético tromboembólico.

Seguimiento Derivar a especialista.

Autor / Editor Diego Vidal

Año 2016

18 Facultad de Medicina, Universidad de Chile

Coagulación Intravascular Diseminada

elevación de productos de degradación como PFD y dímero D. Finalmente todos estos fenómenos llevan al daño de diferentes órganos tanto por isquemia como hemorragia. La CID aguda generalmente se observa en pacientes con una historia de sepsis, cáncer (leucemia promielocítica aguda), o transfusión de sangre ABO incompatible. La CID crónica generalmente se observa en pacientes con una historia de malignidad, especialmente de origen pancreático, gástrico, de ovario o tumores cerebrales, tromboembolismo venoso o arterial.

Nivel de manejo del médico general: Diagnóstico Sospecha Tratamiento Inicial Seguimiento No requiere

Aspectos Esenciales • • • •

Diagnóstico

Proceso sistémico con el potencial para causar trombosis y hemorragia. La CID aguda es secundaria a cuadros de inflamación sistémica la mayoría de las veces. La crónica es secundaria a neoplasias. El diagnóstico es clínico y de laboratorio, con base en los hallazgos de la coagulopatía y/o fibrinólisis. El tratamiento es el de la enfermedad de base.

El primer paso es evaluar al paciente y buscar enfermedades que predisponen a ella. Luego se realizan los exámenes de laboratorio. No hay una sola prueba de laboratorio que pueda confirmar o eliminar el diagnóstico con precisión. Clínica: además del trastorno basal que predispone a la aparición de la CID, se presenta con manifestaciones hemorrágicas y/o trombóticas, disfunción hepática, lesión renal aguda y síndrome de dificultad respiratoria aguda. En los exámenes de laboratorio se puede apreciar trombocitopenia, dimero-D elevado, prolongación del TP/TTPa y disminución del fibrinógeno, elevación de productos de degradación del fibrinógeno. Diagnostico diferencial: déficit de vitamina K, dilución de factores y plaquetas, púrpura trombótica trombocitopénica, falla hepática aguda y hiperfibrinólisis primaria.

Caso Clínico Tipo Paciente que presenta un cuadro de sepsis con hemorragia digestiva alta. En los exámenes de laboratorio se aprecia: plaquetas bajo 100.000, TP prolongado, fibrinógeno bajo 100 mg/dL y dimero-D elevado.

Definición

Tratamiento

La coagulación intravascular diseminada (CID, también llamada coagulopatía de consumo) es un proceso sistémico con el potencial para causar trombosis y hemorragia. Se puede presentar como una emergencia aguda que amenaza la vida o un proceso crónico, subclínico, dependiendo del grado y el ritmo del proceso y la contribución de morbilidades. La identificación de CID y la condición subyacente responsable son fundamentales para un manejo adecuado.

La principal medida en estos pacientes es tratar la enfermedad de base, de lo contrario cualquier medida para tratar la CID no tendrá efecto. Dentro del tratamiento de la CID, se puede usar para la hemorragia: transfusión plaquetaria y de factores coagulación (plasma fresco congelado, crioprecipitados). Para la trombosis: anticoagulación con heparina no fraccionada. También puede usarse Proteína C activada, ácido tranexámico y factor VIIa recombinante. Para los pacientes que no están sangrando, no se utiliza rutinariamente transfusión de plaquetas y factores de coagulación, siempre y cuando el recuento de plaquetas sea ≥10.000/microlitro. Sin embargo, el tratamiento se justifica en pacientes con hemorragia grave, con alto riesgo de sangrado, o que requieren procedimientos invasivos. En contraste, la administración de agentes antifibrinolíticos, como el ácido tranexámico (TXA), ácido épsilon aminocaproico, o aprotinina, están generalmente contraindicados.

Etiología-Epidemiología-Fisiopatología La CID aguda y crónica representan dos extremos de un equilibrio entre factores de coagulación, el consumo de plaquetas y su producción. La CID se inicia con la activación de la cascada de coagulación. Se produce exposición de factor tisular desde endotelio dañado o macrófagos activados, lo que lleva a la activación del factor VII. Esto genera una cascada enzimática que concluye con la generación de trombina lo que permite la formación de fibrina con trombos intravasculares con consumo agudo de factores de coagulación y agregación plaquetaria (lo que produce trombocitopenia). A su vez se produce activación de plasmina que degrada en forma masiva a la fibrina generada, lo que aumenta el riesgo de sangrado, y produce

Seguimiento No requiere.

19 Facultad de Medicina, Universidad de Chile

Autor / Editor Marco Ayala

Año 2016

Lisis Tumoral Aguda

el riñón) e hipocalcemia, ocasionando tetania, arritmias cardiacas y convulsiones.

Nivel de manejo del médico general: Diagnóstico Sospecha Tratamiento Inicial Seguimiento Derivar

Diagnóstico Dos o más de las siguientes alteraciones de laboratorio en un período que abarca 3 días previos hasta 7 días posteriores al inicio de la terapia oncológica: hiperuricemia, hiperkalemia, hiperfosfemia, hipocalcemia. A esto se puede sumar signos y síntomas derivados de dichas alteraciones.

Aspectos Esenciales • • • •

Emergencia oncológica común. Generalmente en Leucemias Agudas y Linfomas no Hodgkin. Cursa con hiperuricemia, hiperkalemia, hiperfosfemia e hipocalcemia. El tratamiento es el de las alteraciones de laboratorio y su clínica.

Tratamiento Lo fundamental es la prevención mediante la correcta estratificación de riesgo de cada paciente y monitorización intensa en aquellos con alto riesgo. Una vez establecido el síndrome, el tratamiento corresponderá al de cada alteración metabólica e hidroelectrolítica y sus complicaciones. La prevención se realiza mediante hiperhidratación endovenosa, administración de alopurinol o agentes urolíticos, monitorización de electrolitos y monitorización cardíaca en intervalos adecuados y adaptados a cada paciente.

Caso Clínico Tipo Escolar de 10 años de edad tratada con quimioterapia a causa de una leucemia linfoblástica aguda, que a pesar de las medidas preventivas presenta hiperuricemia, hiperfosfemia, hipocalemia y disminución de la diuresis. Se inicia tratamiento conservador y luego, por falta de respuesta, se realiza peritoneo-diálisis, eligiéndose esta modalidad por inestabilidad hemodinámica y trastornos de la coagulación, lo que contraindicaba la hemodiálisis. Al tercer día mejoran los valores plasmáticos del ácido úrico, del calcio, del fósforo, del potasio y comienza a aumentar francamente la diuresis, siendo dada de alta en buenas condiciones.

Seguimiento Básicamente el seguimiento se basa en el tratamiento del cáncer subyacente, estratificación del riesgo del paciente para desarrollar síndrome de lisis tumoral, monitorización y controles ajustados a cada paciente.

Definición Emergencia oncológica consecuencia de la liberación de contenido intracelular hacia el torrente sanguíneo producto de la lisis celular espontánea o como respuesta a la terapia oncológica. Puede ser asintomática o cursar con insuficiencia renal aguda, arritmias, convulsiones y muerte por falla multiorgánica.

Autor / Editor Marco Ayala

Etiología-Epidemiología-Fisiopatología La causa subyacente es la patología cancerosa. Las neoplasias más relacionadas a este síndrome son los Linfomas no Hodgkin y las Leucemias Agudas. No obstante, tumores sólidos también pueden causarlo, dependiendo de su tamaño, características del paciente y de la terapia. Datos sobre la frecuencia de esta complicación son escasos y dependen del tipo de cáncer. Cursa con hiperuricemia, pudiendo ocasionar insuficiencia renal aguda por depósito de cristales, hiperkalemia, causante de arritmias cardíacas, hiperfosfemia, provocando hipocalcemia secundaria con precipitación de cristales de fosfato de calcio en distintos órganos (entre ellos

20 Facultad de Medicina, Universidad de Chile

Año 2017

Diagnóstico

Neutropenia Febril

A todo paciente que cursa alguna terapia que pueda generar neutropenia se le indica que consulte a urgencia de inmediato ante la presencia de fiebre. Frecuentemente la única manifestación es la fiebre ya que no se puede montar una buena respuesta inmune. Es fundamental realizar una anamnesis y examen físico orientado a signos y síntomas que nos orienten al foco.

Nivel de manejo del médico general: Diagnóstico Específico Tratamiento Inicial Seguimiento Derivar

Aspectos Esenciales • • •

Se debe confirmar y cuantificar la neutropenia con hemograma y hacer hemocultivos. También se deben hacer otros cultivos o exámenes orientados a la pesquisa del foco infeccioso como una radiografía de tórax, urocultivo, etc. Existen distintas tablas y scores para determinar si el paciente es de bajo o alto riesgo (complicaciones y mortalidad), pero principalmente se podría resumir en: • Alto Riesgo: Neoplasia Hematológica, más de 60 años, PMN <100, duración de más de 10 días, inestable, sepsis, con mucositis o con foco. • Bajo Riesgo: Neoplasia Sólida, menos de 60 años, PMN >100, duración de menos de 7-10 días y sin sepsis, mucositis ni foco.

Fiebre (>38.3ºC) y RAN <500. Generalmente post quimioterapia. Principales agentes son Gram (-), con especial importancia: Pseudomona aeruginosa. Se pueden clasificar en bajo y alto riesgo lo que determina el tratamiento.

Caso Clínico Tipo Paciente femenino 62 años con antecedente de leucemia mieloide crónica en tratamiento, consulta por presentar fiebre hace dos días. 8 días después de quimioterapia. Se objetiva temperatura de 38.5ºC.

Tratamiento

Definición

Si el paciente es de bajo riesgo se puede usar Ciprofloxacino + Amoxicilina/Clavulánico. Si el paciente es de alto riesgo, se debe orientar por epidemiologia y para cubrir Pseudomonas. Si es monoterapia puede ser cefepime, ceftazidima, o algún carbapenémico. Si es biterapia algún aminoglucósido (como Amikacina) + Betalactámico antipseudomónico A las 48 hrs evaluar la posibilidad de Staphylococcus y si es intra o extrahospitalario para pensar en agregar Vancomicina al esquema. De todos modos, lo más probable es que a las 48 horas ya se disponga de los resultados de los cultivos. Si la fiebre es más de 5 días pensar también en hongos.

La sociedad de enfermedades infecciones de America lo define como la presencia de fiebre de 38.3°C en un registro único o de 38°C sostenido por más de una hora asociado a un recuento absoluto de neutrófilos <500 /cc o <1000 con declinación a menos de 500 en 24 horas.

Etiología-Epidemiología-Fisiopatología Los neutrófilos son un determinante crítico de infección. Existe reserva de ellos en la medula ósea, sin embargo, esto puede verse afectado, por ejemplo, con radioterapia o quimioterapia. Es por esto que la neutropenia febril se encuadra principalmente en las emergencias oncológicas, principalmente relacionadas a neoplasias hematológicas. La mayor caída en el recuento de neutrófilos se da entre los días 9-12 post tratamiento. Por lo anterior, es más grave un cuadro febril antes de los 9 días (porque sabemos que el recuento de neutrófilos va a bajar aún más) y menos grave ya pasando el día 12 (va a ir aumentando). Cuando existe daño de mucosas, principalmente respiratorias y digestivas, además de neutropenia la posibilidad de infección es muy alta.

Seguimiento Todo paciente con neutropenia febril debe ser hospitalizado y observado cada 4-24 horas para ver su evolución.

Autor / Editor Diego Cáceres

Los agentes etiológicos son principalmente Gram (-) como E. coli, K. pneumoniae y, de especial relevancia, Pseudomona aeruginosa.

21 Facultad de Medicina, Universidad de Chile

Año 2017

Síndrome De Vena Cava Superior

Tratamiento Generalmente se espera a tener el diagnóstico histológico antes de instaurar el tratamiento específico. Tratamiento de soporte, oxigenoterapia, corticoides endovenosos. Tratamiento específico del tumor con quimioterapia, o radioterapia en tumores radiosensibles. En algunos casos instalación de endoprótesis vasculares, en casos de emergencia. Anticoagulación si es debida a trombosis.

Nivel de manejo del médico general: Diagnóstico Específico Tratamiento Inicial Seguimiento Derivar

Aspectos Esenciales •

• •

Es una complicación sub-aguda que raramente constituye una urgencia vital, salvo que se acompañe de un colapso circulatorio y signos neurológicos de hipertensión intracraneana. Es consecuencia de neoplasias, frecuentemente pulmonares y linfomas Terapia de base: diuréticos, corticoides y anticoagulación si se sospecha trombosis

Seguimiento Derivar

Autor / Editor Diego Cáceres

Caso Clínico Tipo Paciente de 65 años que cursa con cefalea, edema facial, en el cuello y en las extremidades superiores, disnea. Al examen físico se constata circulación colateral torácica superficial, aumento de la presión venosa yugular. Presenta además convulsiones, síncope y papiledema.

Definición Conjunto de síntomas producidos como consecuencia de la obstrucción parcial a completa del flujo sanguíneo a través de la vena cava superior hacia la aurícula derecha.

Etiología-Epidemiología-Fisiopatología Este síndrome se produce en 80-95% de los casos a consecuencia de neoplasias, siendo las más frecuentes: cáncer pulmonar 80% (2/3 de los casos debido a tumores de células pequeñas), Linfoma no Hodgkin y enfermedad de Hodgkin 15%. Las causas no tumorales suponen el 5% de los casos y son debidas a trombosis, fibrosis, etc.

Diagnóstico Radiografía de tórax: ensanchamiento del mediastino superior (60%), derrame pleural (25%), masa hiliar (16%). TAC de tórax con contraste Estudio angiográfico de la vena cava superior, en especial cuando se decide instalar prótesis vascular. Broncoscopía, mediastinoscopía. Biopsia.

22 Facultad de Medicina, Universidad de Chile

Año 2017

Leucemias Crónicas

Incidencia de 0,9-2,4/100.000 hab. Predomina en personas mayores y varones. La causa se desconoce y no tiene relación con radiaciones ionizantes. Es importante mencionar que a pesar de que los linfocitos B están aumentados, estos pacientes tienen inmunodeficiencia humoral ya que disminuye la concentración de inmunoglobulinas (Ig) de superficie y casi nunca secretan Ig a la sangre.

Nivel de manejo del médico general: Diagnóstico Sospecha Tratamiento Inicial Seguimiento Derivar

Aspectos Esenciales

Diagnóstico

• •

LMC: Clínica: Cursa al inicio con una Fase Crónica (95% al Dg), una Fase de Aceleración (Sintomática, pero no siempre presente), y Fase Blástica o de Leucemia Aguda (Fase Terminal). La Fase Crónica se presenta con síntomas de hipermetabolismo: astenia, anorexia, baja de peso, sudoración nocturna. Esplenomegalia por infiltración de granulocitos y síntomas asociados (dolor abdominal, saciedad precoz, dolor en hipocondrio izquierdo). Asintomáticos en 20% de los casos cuando solo se encuentra leucocitosis o masa abdominal. En la Fase de Aceleración se agravan los síntomas, crece más el bazo con mayor leucocitosis, y elevación del número de blastos (10-30%). En la Fase Blástica, los blastos aumentan sobre los 30%, muy sintomáticos. En esta fase la sobrevida es solo de 2-4 meses.

• • •

Hay un porcentaje de pacientes que son asintomáticos. LMC: Esplenomegalia por infiltración, hipermetabolismo (astenia, baja de peso, sudoración nocturna) y Sd. anémico progresivo. LMC fuerte asociación a cromosoma philadelphia t(9;22). LLC: Generalmente asintomática, hallazgo casual de linfocitosis. Esplenomegalia por infiltración. LLC: NUNCA hay blastos en sangre.

Caso Clínico Tipo Paciente varón de 45 años que consulta por saciedad precoz de 3 meses de evolución. A la anamnesis refiere sudoración nocturna y cuadros de dolor abdominal irradiado a hombro izquierdo. Al examen físico se encuentra una masa en el hipocondrio izquierdo.

Laboratorio: El hemograma presenta Reacción Leucemiode (aumento de glóbulos blancos en sus formas inmaduras: mielocitos, juveniles, basófilos), leucocitosis con neutrofilia, anemia normocítica normocrómica y plaquetas con conteo variable. En examen bioquímico aumento de ácido úrico y LDH y disminución de FA de neutrófilos. VHS baja (en ausencia de infección).

Definición Neoplasias del sistema inmune, originadas en células maduras de serie blanca. Pueden ser leucemia mieloide crónica (LMC) en la que predomina la proliferación de la celula mieloide, o linfática crónica (LLC) que corresponde a una neoplasia monoclonal de linfocitos, habitualmente B, funcionalmente incompetentes, con vida media alargada que se van acumulando progresivamente.

Diagnóstico definitivo: Médula ósea hipercelular con relación aumentada mieloide/eritroide y citogenética de MO con Cromosoma Philadelphia (presente en 90-95%) detectable por FISH y PCR. LLC: Clínica: En más de la mitad el diagnostico se hace en pacientes asintomáticos por hemograma alterado, por lo que ante al hallazgo causal de linfocitosis sanguínea hay que pensar en LLC. Posteriormente pueden aparecer síntomas como: astenia, adenopatías, síntomas B (poco habituales), infecciones a repetición, hepato-esplenomegalia (pero no tan masiva como en LMC) y trombocitopenia por ocupación medular. La desregulación inmunitaria conlleva inmunodeficiencia humoral y fenómenos autoinmunitarios (Anemia hemolítica por ac. calientes).

Etiología-Epidemiología-Fisiopatología •



LMC: Síndrome mieloproliferativo crónico más frecuente. Representa el 15-20% del total de leucemias. Incidencia de 1,5/100.000 hab. Predominio en los varones y entre los 40-50 años. Fue la primera patología neoplásica a la que se le asocio una alteración citogenética: el Cromosoma Philadelphia presente en el 95% de LMC y que se debe a la translocación entre el cromosoma 9 y 22 a nivel citogenético o BCR-ABL a nivel molecular. LLC: Proliferación monoclonal y acumulación de Linfocitos, en un 95% estirpe B y 2-3% estirpe T, por lo que al hablar de LLC se refiere principalmente a una neoplasia de células B. Representa el 30% del total de leucemias (la más frecuente en ancianos), de baja agresividad. Invasión progresiva de sangre periférica y MO por linfocitos.

Laboratorio: El diagnóstico de LLC, requiere de linfocitosis persistente superior a 15x103/dL (>4 semanas), sin causa aparente, Manchas de Gumprecht e infiltración de medula ósea > 30% linfocitos. Además se encuentra anemia, trombocitopenia, e hipogammaglobulinemia por inmunodeficiencia humoral.

23 Facultad de Medicina, Universidad de Chile

Tratamiento LMC: El GES en Chile recomienda Mesilato de Imatinib como primera línea de terapia en caso de LMC con cromosoma Philadelphia (+). La dosis recomendada es de 400 mg/día por vía oral. Los pacientes con falla de respuesta o respuesta subóptima, debe aumentarse. El transplante de médula ósea tiene que ser evaluado caso a caso. El tratamiento en fase blastica es habitualmente insatisfactorio. LLC: Antes de iniciar el tratamiento es necesario mantener al paciente en observación, realizar biopsia medular y recuento sanguíneo seriado para conocer el estadio de la enfermedad. Pacientes asintomáticos se deben controlar periódicamente, ya que el tratamiento no ha mostrado mejorar la sobrevida. En caso de pacientes sintomáticos, con síntomas B, hepato-esplenomegalia, progresión de enfermedad (Duplicación de linfocitos en menos de 6 meses o disminución de Hb, o disminución grave de plaquetas), se realiza quimioterapia. La primera línea en pacientes <65 años son análogos de las purinas (ej fludarabina), mientras que en mayores de 65 años o con comorbilidad se prefiere el clorambucil, por ser menos mielotóxico.

Seguimiento Derivar a Hematología.

Autor / Editor Año Valeska Lavanderos 2016

24 Facultad de Medicina, Universidad de Chile

Síndromes Mielodisplásicos

liza un aspirado de médula ósea y una biopsia de hueso con tinción de hierro. La médula ósea es en general hipercelular (o hipo o normocelular), con displasias.

Nivel de manejo del médico general: Diagnóstico Sospecha Tratamiento Inicial Seguimiento Derivar

Clasificación de la FAB: • Anemia refractaria simple • Anemia refractaria con sideroblastos en anillo • Anemia refractaria con exceso de blastos • Anemia refractaria con exceso de blastos en transformación • Leucemia mielomonocítica crónica

Aspectos Esenciales • • • • •

Clasificación de la OMS: • Citopenias refractarias con displasia unilinaje • Anemia refractaria con sideroblastos anillados • Citopenias refractarias con displasia multilinaje • Anemia refractaria con exceso de blastos-1 • Anemia refractaria con exceso de blastos-2 • Síndrome mielodisplásico inclasificable. • Síndrome mielodisplásico asociado a deleción(5q) aislada.

Se sospecha en ancianos con citopenias inexplicables. La mayoría son idiopáticos, el resto se relaciona con quimio o radioterapia. El diagnóstico incluye evaluación de la sangre periférica, aspirado de médula ósea y biopsia de hueso. Hay 2 clasificaciones (FAB y OMS). El tratamiento definitivo es el trasplante alogénico de precursores hematopoyéticos.

Caso Clínico Tipo

Tratamiento

Mujer de 75 años, sin antecedentes de importancia, consulta por fatiga generalizada desde hace unos meses. El hemograma muestra una anemia (Hb: 8,5g/dl) macrocítica (VGM: 108 fL). La paciente no presenta ni deficiencia de vitamina B12 ni de ácido fólico.

El único tratamiento definitivo es el trasplante alogénico de precursores hematopoyéticos, y solo para pacientes menores de 60 años. Otras alternativas son: transfusiones asociadas a deserroxamina, quimioterapia y radioterapia. Más raro son las anemias sensibles a la vitamina B6, a andrógenos, esteroides y ciclosporinas.

Definición Seguimiento

Grupo de enfermedades clonales de la célula madre hematopoyética, caracterizadas por citopenia(s), displasia en una o más líneas celulares, hematopoyesis ineficaz y un riesgo elevado de leucemia mieloide aguda.

Por especialista

Etiología-Epidemiología-Fisiopatología Autor / Editor Diego Vidal

Más frecuentes en varones y en ancianos. Con respecto a la etiología, en el 90% pueden ser idiopáticos, que corresponden a los síndromes mielodisplasicos primarios. El 10% restante son los síndromes mielodisplasicos secundarios, que pueden ser adquiridos (alquilantes, antraciclinas, radiaciones) o hereditarios (anemia de Fanconi). Tienen una elevada frecuencia de transformación hacia Leucemia Aguda.

Diagnóstico Se debe sospechar SMD en citopenias inexplicadas que no responden al tratamiento, o en anemias, con VCM normal o aumentada, exceso de hierro en sangre y aumento de la saturación de transferrina, que no responden a otros tratamientos. Clínicamente, puede haber visceromegalias o síntomas relacionados con las citopenias o el exceso de hierro. Se rea-

25 Facultad de Medicina, Universidad de Chile

Año 2016

Síndromes Mieloproliferativos Crónicos

emigración de las células hematopoyéticas a hígado y bazo.

Nivel de manejo del médico general: Diagnóstico Sospecha Tratamiento Inicial Seguimiento Derivar

Diagnóstico •

Policitemia vera: aumento del número de eritrocitos, con valores normales o bajos de eritropoyetina (lo que la diferencia de otras poliglobulias). Este aumento de GR provoca hiperviscosidad de la sangre, lo que provoca los síntomas: rubicundez, cefaleas, vértigo, parestesias, claudicación de EEII, trastornos neurológicos, baja de peso y sudoración nocturna. Frecuentes son también el prurito y la eritromelalgia (dolor urente en dedos, manos y pies). Al examen físico se puede objetivar HTA, flebitis, esplenomegalia, Hb>18.5 g/dl (en hombres) o 16.5 g/dl (en mujeres). El 95% de ellas presenta una mutación Jak2 (+).



Trombocitosis esencial: es un diagnóstico de exclusión, se deben descartar el resto de causas de trombocitosis, como las infecciones. Asintomático en la mitad de los casos. Síndromes trombóticos y/o hemorrágicos. Eritromelalgia por isquemia de extremidades. Recuento plaquetario mantenido ≥ 450.000/dl.



Leucemia mieloide crónica: 20-40% asintomáticos. Suele aparecer entre las 3ª-6ª décadas, asociada a cromosoma Filadelfia. Como síntomas se presenta: hipermetabolismo, gran esplenomegalia (incluso provocando saciedad precoz y dolor por infarto esplénico), síndrome anémico progresivo. Incremento de los glóbulos blancos en todas sus etapas madurativas. Tras un período variable de 3-4 años de clínica insidiosa ocurre la fase blástica, cuando evoluciona como leucemia aguda.



Mielofibrosis agnogénica: debido a la hematopoyesis extra medular se encuentra gran esplenomegalia e incluso hepatomegalia. Suele presentar anemia e hipermetabolismo. En la médula ósea se encuentra fibrosis medular y mieloptisis. Característico son los dacriocitos o eritrocitos en forma de lágrima.

Aspectos Esenciales • • • •

Trastornos de las células germinales hematopoyéticas. LMC es el más frecuente. Dependiendo de la serie serán las manifestaciones del paciente. Tratamiento depende de la serie celular afectada.

Caso Clínico Tipo Paciente que a la anamnesis refiere episodios recurrentes de cefaleas y baja de peso. Al examen físico presenta rubicundez de piel y mucosas.

Definición Panmielopatías clonales en las que una mutación de la célula germinal pluripotencial causa la proliferación excesiva, dando un incremento de las series hematopoyéticas, habitualmente con predominio de una. En función de la célula predominante, existe la policitemia vera (eritrocitos), leucemia mieloide crónica (leucocitos), trombocitosis esencial (plaquetas) y mielofibrosis agnogénica (fibrosis medular con mieloptisis).

Etiología-Epidemiología-Fisiopatología • •



• •

Alteraciones génicas recurrentes: mutación de tirosina quinasa JAK2, gen de fusión BCR-ABL1 y receptor de trombopoyetina. En Policitemia vera predomina la proliferación de serie roja y cursa con un proceso bifásico constituido por una primera fase de mieloproliferación a expensas de la serie roja y luego una segunda fase con formación de tejido fibroso en la médula (mielofibrosis con metaplasia mieloide). En la LMC (la más frecuente) predomina la serie blanca y ocurre principalmente en hombres de 50-60 años, en el 95% se encuentra presente la alteración citogenetica t(9,22), conocida como cromosoma Philadelfia. En la Trombocitosis Escencial predomina la serie megacariocítica. La Mielofibrosis con metaplasia mieloide cursa con fibrosis de la médula, provocando un patrón mieloptísico con

Tratamiento •

Policitemia vera: el tratamiento depende de la sintomatología. Con poca sintomatología basta con flebotomías, para alcanzar hematocritos <45% en hombres y <42% en mujeres. En caso de gran sintomatología, flebotomía y ASS asociado con tratamiento mielosupresor: hidroxiurea. Interferón en <50 años y mujeres en edad fértil.



Trombocitosis esencial: No precisa tratamiento en pacientes de bajo riesgo asintomáticos y <60 años. En alto riesgo y sintomáticos, el tratamiento es mielosupresor con hidroxiurea, plaquetoféresis o Interferón alfa o anagrelida en jóvenes.

26 Facultad de Medicina, Universidad de Chile



Leucemia mieloide: tratamiento inicial con hidroxiurea, imatinib o busulfán. Trasplante alógeno de precursores hematopoyéticos.



Mielofibrosis agnogénica: trasplante alógeno de precursores hematopoyéticos en jóvenes. En casos en que no se puede realizar trasplante se pueden utilizar transfusiones o eritropoyetina.

Seguimiento Derivar a especialista.

Autor / Editor Diego Vidal

Año 2016

27 Facultad de Medicina, Universidad de Chile

Hipercalcemia

sulta cuando la entrada de calcio en la circulación excede la excreción de calcio en la orina o deposición en los huesos y por lo tanto la concentración sérica de calcio supera los 10,5 mg/dl (VN: 8,5- 10,5 mg/dl). Se considera leve si es < 12 mg/ dl, moderada entre 12 y 14 mg/dl y severa > 14 mg/dl. Si se mide calcio iónico se considera hipercalcemia si > 1,32 mmol/L (VN: 1,16-1,32 mmol/L).

Nivel de manejo del médico general: Diagnóstico Sospecha Tratamiento Inicial Seguimiento Derivar

Aspectos Esenciales

Etiología-Epidemiología-Fisiopatología

• • • •



• • • •

Emergencia oncológica Trastorno metabólico más frecuente El 80% de los casos no presentan metástasis óseas Tumores que más frecuentemente la provocan son cánceres de mama, pulmón y el mieloma múltiple. Síntomas poco específicos, que pueden coincidir o traslaparse con el tratamiento de base Se debe corregir el calcio sérico con la albúmina Tratamiento: Hidratación más bifosfonatos Factor de mal pronóstico

Etiología: entre los tumores que más frecuentemente la provocan encontramos los cánceres de mama, pulmón y el mieloma múltiple. • Epidemiología: la hipercalcemia está presente en hasta un 30% de los enfermos con cáncer. La presencia de hipercalcemia tiene un significado pronóstico, particularmente cuando hay elevación de del PTHrP (proteína relacionada con PTH). Entre un 10 a 30% de los enfermos con hipercalcemia están vivos un año después. Fisiopatología: existen tres mecanismos de producción de hipercalcemia (Tabla 2): 1. Hipercalcemia osteolítica local: el más evidente es la destrucción local de hueso con aumento de la actividad de osteoclastos y liberación de citoquinas. Este mecanismo da cuenta de un 20% de las hipercalcemias. Es visible cuando existe osteolisis importante, como en los casos de mieloma múltiple o en tumores sólidos como mama o pulmón. Las citoquinas más identificadas con el problema son el TNF, la interleuquina 1 y 6 y son las que activan los osteoclastos. 2. Hipercalcemia humoral de malignidad: un segundo mecanismo es el “humoral”, con presencia de PTH-rP, que es una sustancia secretada por los tumores y que es análoga a la PTH (tiene concordancia en los 13 primeros aminoácidos), con una acción similar, produciendo aumento de la reabsorción ósea de calcio y aumento de la absorción a nivel de túbulo distal. Este mecanismo es responsable de cerca de un 80% de los casos de hipercalcemia.

Caso Clínico Tipo Paciente de 75 años de edad con antecedentes mórbidos de tabaquismo suspendido y cáncer de pulmón con último TAC que muestra metástasis hepáticas, actualmente en quimioterapia y utiliza morfina para manejo del dolor. Presenta cuadro de tres semanas de evolución de fatiga, pérdida del apetito, náuseas, estreñimiento, poliuria y desorientación ocasional. Al examen físico se encuentra con signos vitales normales, deshidratada, adelgazada y leve deterioro cognitivo. Se decide tomar exámenes de sangre donde destacan glicemia de 90 mg/dL, nitrógeno ureico de 20 mg/dL, creatinina 1 mg/ dL, sodio de 120 mEq/L, potasio 3 mEq/L, calcio 10 mg/dL y albúmina de 1,5 g/gl.

Definición Corresponde a una emergencia oncológica metabólica. Re-

28 Facultad de Medicina, Universidad de Chile

produce hipercalcemia, este mecanismo es diagnosticado en presencia de hipercalcemia con persistencia de niveles elevados de vitamina D. Esto es más frecuente en los linfomas no Hodgking, observándose la formación de 1,25 hidroxi vitamina D por los linfocitos o macrófagos neoplásicos. Este mecanismo también está presente en enfermedades granulomatosas como la sarcoidosis.

Mecanismo de hipercalcemia humoral de malignidad: La acción de la PTH-rP, secretada a la sangre o en forma paracrina, promueve la expresión del ligando del receptor activador del factor nuclear kappa B (RANKL) en el hueso. el RANKL contribuye a la hipercalcemia uniéndose a RANK en la superficie de los precursores de octeoclastos, llevando a la activación de estos.

3. Linfomas secretores de calcitriol: por último, algunos tumores secretan calcitriol (1,25 hidroxi vitamina D), que

PTH: Hormona paratiroidea; PTHrP: Péptido relacionado con la PTH; 1,25(OH)2D: 1,25 dihidroxi-vitamina D: Linfoma asociado a HTLV: Virus Linfotrófico de células T humanas tipo I.

Diagnóstico

Como aproximación es útil la fórmula:

Los síntomas de hipercalcemia son inespecíficos, dentro de ellos están los síntomas neurológicos, letargia y confusión, síntomas digestivos náuseas, anorexia y constipación. Hay síntomas generales como la fatiga. Son remarcables la presencia de poliuria, polidipsia y aparición de hipovolemia, que puede ser profunda, contribuyendo a un deterioro agudo de la función renal.

En las imágenes, serán visibles las lesiones osteolíticas cuando las hay; la presencia de PTH baja en el contexto de hipercalcemia y ausencia de lesiones líticas, hace sospechar la existencia del mecanismo humoral, explicándose el fenómeno por la supresión de la PTH por su símil. En la práctica clínica es posible la medición y monitoreo de la PTH-rP. Diagnóstico diferencial: • Hiperparatiroidismo primario: habitualmente se presenta con hipercalcemia asintomática, crónica. Calcio sérico y PTH sérica aumentada.

Tratamiento El primer punto a decidir es si vamos a hacer una terapia activa (reducción de los niveles de calcemia) o no, pues el tratamiento activo puede ser adecuado si existe un pronóstico favorable y si tenemos más terapias para ofrecer, sin embargo, no será necesario en un paciente que está cerca del fin de su vida.

El diagnóstico se hace con la medición de calcio iónico o el calcio total en sangre, en este último caso de debe aplicar una fórmula para corregir la calcemia. El calcio iónico es el catión bioquímicamente activo. En la sangre el calcio va unido en 40% de albúmina, en un 50% va ionizado y en un 10% va unido a iones complejos. Cambios en los niveles de albúmina, llevan cambios en la concentración de calcio iónico.

El tratamiento comprende de : 1. Hidratación vigorosa con solución salina: el primer objetivo es la reposición de volumen, tratando de llevar al paciente a euvolemia. La forma más rápida de iniciar

29 Facultad de Medicina, Universidad de Chile

la reducción de la calcemia es el aporte de fluido, aún cuando su utilidad está limitada por la necesidad de aportar varios litros de soluciones intravenosas. Una clara limitante de esta estrategia la vemos en pacientes con insuficiencia cardiaca o renal, que no pueden tolerar sobrecargas de volumen. 2. Diuréticos de asa (Furosemida): promueve la calciuria y acentúa el efecto de la hidratación. Deben utilizarse con precaución, una vez corregida la hipovolemia y con atención a los desbalances hidroelectrolíticos potenciales. 3. Bifosfonatos (análogos inorgánicos de pirofosfato): corresponde al estándar actual, estos actúan impidiendo la acción reabsortiva de los osteoclastos. Su acción es relativamente rápida con un peak cerca del tercer o cuarto día. Los bifosfonatos disponibles son Pamidronato (en dosis única de 60 a 90 mg EV, en 4 horas), o Zolendronato (en dosis única de 4 mg EV, en 15 minutos) o Ibandronato (en dosis única de 4-6 mg EV, en 2 horas). Utilizar con precaución en la insuficiencia renal. Existe riesgo de osteonecrosis mandibular. 4. Calcitonina (SC o EV): es eficaz, aunque de utilidad muy limitada (unas 48 horas) por taquifilaxis. 5. Corticoides: tienen efecto en la reducción de secreción de la vitamina D3 por linfocitos o macrófagos. Este efecto tiene lugar entre 2 a 5 días del inicio. Son particularmente útiles en las neoplasias hematológicas. Las dosis de 40 a 60 mg de prednisona al día son eficaces en estos casos.

Seguimiento Derivar.

Autor / Editor Marco Ayala

Año 2017

30 Facultad de Medicina, Universidad de Chile

BIBLIOGRAFÍA • • • • • • • • • • • • •

• • • • • • •

Marta E. Zerga. Anemia de los trastornos crónicos. HEMATOLOGIA, Vol 8 N° 2: 45-55. Mayo-Agosto, 2004. Baños Madrid, R. Anemia crónica en mujer joven. Casos Clínicos. Sociedad Española de Médicos Generales y de Familia. 2006. Trastornos de los eritrocitos, cap. 68. Harrison. Manual de medicina. 18° edición. McGraw Hill. Dr. Denis Suárez. Hematología. Anemia megaloblástica y anemia hemolítica. Bases de la Medicina Clínica. Universidad de Chile. Manual de Hematología. 8° edición. Grupo CTO. CTO Editorial. Guía Clínica - Leucemia en personas de 15 años y más. Ministerio de Salud. Santiago: Minsal, 2010. Puig Sanz L. Púrpuras. Protocolos de Dermatología. AEP, 2007. David Barreto. Tratamiento de la Púrpura de Henoch Shölein. 2014. Alfonso Yubero Esteban. Boletín 16: Síndromes paraneoplásicos (parte 1°). Boletín Oncológico Teruel. Dalmau J, Rosenfeld M. Síndromes neurológicos paraneoplásicos, cap. 101. Harrison. Principios de medicina interna. 18° edición. McGraw Hill. Gustavo Kiekebusch H., Ernesto Perucca P. Trombofilias hereditarias. REV CHIL OBSTET GINECOL 2003; 68(5): 424-429. Vergara I. Estudio de trombofilias - Análisis molecular de algunos factores relacionados con la alteración en la hemostasia. REV. MED. CLIN. CONDES - 2007; 18(4) 394 - 398. https://www.uptodate.com/contents/treatment-of-neutropenic-fever-syndromes-in-adults-with-hematologic-malignancies-and-hematopoietic-cell-transplant-recipients-high-risk-patients?source=search_result&search=neutropenia%20febril&selectedTitle=2~150 https://www.uptodate.com/contents/overview-of-neutropenic-fever-syndromes?source=search_result&search=neutropenia%20febril&selectedTitle=1~150 https://www.uptodate.com/contents/malignancy-related-superior-vena-cava-syndrome?source=search_result&search=sindrome%20vena%20cava&selectedTitle=1~150 Manual de Hematología. 2° edición. ENARM - México. Guía Clínica - Leucemia en personas de 15 años y más. Ministerio de Salud. Santiago: Minsal, 2010. Manual de Hematología. 8° edición. Grupo CTO. CTO Editorial. Jerry L. Spivak and Richard T. Silver. The revised World Health Organization diagnostic criteria for polycythemia vera, essential thrombocytosis, and primary myelofibrosis: an alternative proposal. Blood 2008 112:231-239. Manual de Hematología. 8° edición. Grupo CTO. CTO Editorial.

31 Facultad de Medicina, Universidad de Chile

precaución y según la etiología. Si no se conoce la etiología de la coagulopatia, lo mejor es evitar transfusiones, recordar que en los pacientes con hemofilia, un mayor número de transfusiones, implica disminución de efectividad de estas a largo plazo.

Coagulopatía congénita sangrante Nivel de manejo del médico general: Diagnóstico Sospecha Tratamiento Inicial Seguimiento Derivar

Seguimiento Derivación a nivel secundario.

Aspectos esenciales   

Situación de urgencia. Al momento de iniciar el tratamiento la estabilización hemodinámica es lo cardinal. Una vez estabilizado se debe trasladar a un centro terciario.

Caso clínico tipo Hombre de 25 años con antecedentes de Hemofilia A. Acude al SU por un cuadro de 1 día de evolución de aumento de volumen rodilla asociado a impotencia funcional, se habría desencadenado luego de un traumatismo. Al examen físico se encuentra pálido con FC de 120 lpm, destaca aumento de volumen de rodilla derecha asociado a rigidez articular.

Definición Cualquier tipo de sangrado en paciente con coagulopatia.

Etiología-epidemiología-fisiopatología Su frecuencia es variable y dependerá de la frecuencia de las coagulopatias en la población donde nos desempeñemos, asi como el control que lleven los pacientes de esta. Según el tipo de coagulopatía congénita es donde se presentarán los sangrados con mayor frecuencia. En la enfermedad de Von Willebrand (cualquiera de sus tipos) predominará el sangrado de mucosas (gingivorragia, menometrorragia, etc). Mientras que en las hemofilias predominarán los sangrados de tipo articular, desencadenados principalmente después de un traumatismo.

Diagnóstico La sospecha es clínica dado el antecedente y el sitio de sangrado (ya mencionado anteriormente). El examen físico debe evaluar el estado hemodinámico, la presencia de petequias y hematomas en cualquier localización. Los exámenes de laboratorio que presentan utilidad son el recuento de plaquetas y la pruebas de coagulación, principalmente para determinar el trastorno subyacente. Como toda hemorragia aguda, el Hto y la Hb, tienen una utilidad clínica menor, frente al examen físico.

Tratamiento Se debe realizar el manejo en urgencias comenzando con el ABC habitual. Las indicaciones de trasfusión de hematíes, plaquetas y plasma se deben realizar con 1

Coagulopatías congénitas (hemofilias, von Willebrand)

nivel de factor VIII contribuya a la expresión y gravedad del cuadro hemorrágico.

Nivel de manejo del médico general: Diagnóstico: Sospecha. Tratamiento: Inicial. Seguimiento: Derivar.

-Tipo 1: Es la forma más frecuente (70-75%). Consiste en un descenso del FvW en plasma (aunque a veces se afecta también la fracción plaquetaria). Se transmite de forma autosómica dominante. -Tipo 2: Deficiencia cualitativa de FvW. El subtipo 2A corresponde al 10-15% de los casos de enfermedad de von Willebrand (EvW). Su anomalía corresponde a un déficit de multímeros de alto peso molecular en plasma y plaquetas. Ello propicia una unión anómala a su receptor plaquetario, la GPIb. El tipo de herencia es autosómica dominante. -Tipo 3: Ausencia completa de FvW. La herencia es autosómica recesiva. Afecta de 1 a 5 pacientes por millón de habitantes.

Existen distintos tipos de enfermedad dependiendo de la calidad/cantidad de FvW presente en plasma y/o plaquetas:

Aspectos esenciales      

Defectos congénitos que afectan a las proteínas plasmáticas de la coagulación. Se sospecha frente a antecedentes de hemorragias. La más frecuente es la Enfermedad de Von Willenbrand seguida de las hemofilias. Se presentan como Sd. hemorragíparo. Hemofilia A (deficit factor VIII), Hemofilia B (déficit factor IX). Recesivas ligadas al X. El diagnóstico se confirma con pruebas funcionales, inmunológicas y test genéticos.

Hemofilias: Las hemofilias son enfermedades recesivas, ligadas al cromosoma X de tal forma que el 100% de las hijas de hemofílicos son portadoras y el 50% de los hijos de mujeres portadoras son hemofílicos. Hay tres clases de hemofilia: A, B y C. La hemofilia A se presenta en 1/10000 nacidos vivo y se debe al déficit del factor VIII, donde el 60% de los casos corresponde a enfermedad severa a causa de la inversión en la secuencia de ADN que se encuentra en el intrón 22 del gen del factor VIII. La Hemofilia B se presenta en 1/30000 nacidos vivos y se debe a déficit del factor IX, el 50 % corresponde a enfermedad severa el principal defecto molecular se debe a deleciones parciales o completas del gen. Por último, la Hemofilia C se debe a déficit del factor XI y en Chile solo se han reportado dos casos.

Caso clínico tipo Paciente de 10 años que es traído por su madre por epistaxis espontáneos a repetición. En la anamnesis refiere también hacer hematomas con golpes leves.

Definición Las coagulopatías congénitas son alteraciones de la hemostasia debido a mutaciones genéticas que se caracterizan por trastornos hemorrágicos que aparecen en la infancia y presentan una historia familiar conocida. Las más frecuentes son la enfermedad de Von Willebrand, caracterizada por la aparición de hemorragias cutaneomucosas en sujetos de ambos sexos y las hemofilias, siendo más frecuente la tipo A por déficit de FVIII que se presenta con hemorragias intrarticulares y musculares en varones.

Diagnóstico Las coagulopatías congénitas se sospechan cuando hay historia de hemorragias precoces y antecedentes familiares y el diagnóstico se confirma con pruebas funcionales, inmunológicas y test genéticos. Enfermedad de Von Willebrand: Se presenta con un Sd. hemorragíparo leve a moderado donde predomina el sangrado en mucosas. Es frecuente que la menorragia sea la primera manifestación en las formas leves. También son frecuentes la epistaxis y gingivorragias espontáneas. Pueden aparecer otros síntomas como hematuria, hematemesis y melenas. Hasta en un 10% del sangrado gastrointestinal se asocia a angiodisplasia. En muchos casos no hay sangrado espontáneo y el diagnóstico se realiza después de una maniobra invasiva o quirúrgica, o tras extracción dental. Son típicos los sangrados postparto (a las 24-48 horas, tras el descenso de FvW, que aumenta durante el embarazo). Son raros los hematomas musculares y de partes blandas que normalmente se dan en el tipo 2N. Los pacientes portadores del tipo 3 tienen hemorragias graves en edades tempranas.

Etiología-epidemiología-fisiopatología Enfermedad de Von Willebrand: Es un trastorno congénito transmitido autosómicamente, caracterizado por el déficit cualitativo y/o cuantitativo del factor de von Willebrand (FvW). Es la coagulopatía congénita más frecuente, habiéndose calculado que en determinadas áreas puede llegar a afectar al 1% de la población, aunque los pacientes con problemas hemorrágicos graves son pocos. El FvW se sintetiza en las células endoteliales y en los megacariocitos, existiendo también una parte que se sintetiza a nivel plaquetario. La función del FvW es facilitar la adhesión de las plaquetas al subendotelio mediante la unión a las glucoproteínas de la membrana plaquetaria GPIb y GPIIb/IIIa, siendo además la molécula transportadora del factor VIII, lo protege de la degradación. Como el FvW es la proteína trasportadora del factor VIII de la coagulación, un descenso de FvW se acompaña de un descenso proporcional de factor VIII, de ahí que el bajo

En el laboratorio hay un aumento del tiempo de sangría y disminución de los niveles plasmáticos del FvW antigénico (FvW:Ag). El factor VIII mostrará un descenso similar, hecho que no sucede en la hemofilia A, donde el FvW es normal. Se ve también un defecto de la aglutinación plaquetaria con ristocetina (RIPA [aglutinación inducida por ristocetina]) y en el subtipo 2N, se prolonga el TTPA (vía intrínseca). 2

ineficaz y en el tipo 2 la respuesta es variable. Se recomienda hacer una prueba previa a su uso terapéutico, ya que hasta un 25% de los pacientes no responde. La dosis recomendada es 0,3 g/kg en 100 cc de suero fisiológico a infundir durante 20- 30 minutos. Se puede repetir la dosis cada 12-24 horas. Si se administra repetidamente a intervalos cortos hay peligro de taquifilaxia. El DDAVP está contraindicado en el tipo 2B al producir trombocitopenia, y en niños menores de 3 años con antecedentes de convulsiones, patología cardiovascular y polidipsia.

Hemofilias: La expresión clínica es la hemorragia fundamentalmente articular, muscular, del sistema nervioso central y de partes blandas. La severidad de la clínica suele estar en relación con la cantidad de factor existente: 1. Forma severa. Cuando la actividad funcional del factor de la coagulación es indetectable, es decir inferior al 1%. Habitualmente encontraremos pacientes con sangrado espontáneo antes de los 6 meses de edad o hemorragia intracraneal en el parto. 2. Forma moderada. Los niveles de factor se encuentran entre el 1 y el 5%. El sangrado aparece generalmente antes de los dos años de edad tras producirse traumas mínimos o postmaniobras invasivas. 3. Forma leve. Los niveles de factor VIII o IX son superiores al 5% e inferiores al 40%. El sangrado es raro y puede aparecer ante traumatismos importantes o postcirugía.

Agentes antifibrinolíticos: para las hemorragias mucocutáneas se ha mostrado especialmente útil la administración de ácido aminocaproico y ácido tranexámico. Su indicación es por vía oral, tópica o endovenosa. Está contraindicado en las hematurias. Los estrógenos de síntesis, con o sin progestágenos, son útiles en las menorragias. Tratamiento sustitutivo con hemoderivados: en los casos de tipo III o en situaciones donde es necesario mantener niveles elevados de forma continuada de FvW se debe recurrir a los concentrados plasmáticos del factor VIII/FvW. La utilización de preparados exclusivos del factor VIII purificado no deben utilizarse. Para la cirugía mayor se recomienda mantener concentraciones plasmáticas de FvW en torno al 50%, y para la cirugía menor alrededor del 30%. Los pacientes tipo 3 tratados con concentrados plasmáticos del factor VIII/FvW pueden llegar a desarrollar una seria complicación como es la aparición de inhibidores al FvW. El tratamiento de elección para las hemofilias son los liofilizados, crioprecipitados y plasma fresco. Esta terapia puede usarse como profilaxis y como manejo de hemorragias agudas.

Manifestaciones hemorrágicas Hemartrosis. Es el sangrado más frecuente (65-90%). Las articulaciones que se afectan con mayor frecuencia son la rodilla, el codo, el tobillo y los hombros. Produce dolor, tumefacción e impotencia funcional. Los sangrados repetidos producen artropatía hemofílica, caracterizada por hipertrofia sinovial, desaparición del cartílago y atrofia muscular. Hematomas musculares. Corresponde al 30% de las complicaciones hemorrágicas. Pueden complicarse con síndromes compartimentales e incluso shock hemorrágico. A la larga producen atrofia muscular. Hemorragia intracraneal. Es la complicación más grave, constituye entre el 2 y 13% de las complicaciones hemorrágicas. Si no hay un tratamiento rápido puede causar la muerte.

Seguimiento Derivar a hematología.

Otras complicaciones. Los hemofílicos también pueden presentar complicaciones hemorrágicas en diferentes localizaciones, destacando la hematuria y la hemorragia gastrointestinal y orofaríngea. Al laboratorio el estudio de coagulación muestra un alargamiento del tiempo de tromboplastina parcial activado, junto con un tiempo de protrombina normal (factor VIII y IX son de vía intrínseca). La confirmación viene por la determinación del factor VIII o IX deficitario. En los hemofílicos, a diferencia de la EvW, el FvW:Ag es normal, así como la aglutinación plaquetaria a la ristocetina. Se han descrito un número importante de alteraciones moleculares responsables de la hemofilia A. Las técnicas moleculares se pueden aplicar en el diagnóstico de portadoras y en el diagnóstico prenatal.

Tratamiento El manejo terapéutico del von Willebrand tiene como objetivo aumentar la cantidad circulante del FvW o mejorar su función favoreciendo la adhesión plaquetaria al subendotelio. 1-deamino-8-D-arginina-vasopresina (DDAVP) es un análogo sintético de la vasopresina capaz de liberar del endotelio FvW vascular al torrente circulatorio, con aumentos de 4-5 veces su valor basal y durante 8-10 horas. Su limitación es que sólo sirve para sujetos que sinteticen FvW normal, por tanto los tipo 1. En el tipo 3 es 3

enfermedades asociadas a paraproteinemias, tales como Mieloma Multiple, la macroglobulinemia o amiloidosis. El Mieloma múltiple es un proceso inmunoproliferativo maligno de células plasmáticas, estas células habitualmente se encuentran en la médula osea pero pueden estar formando un plasmocitoma o incluso pueden estar presentes en el torrente sanguíneo dando origen a una leucemia de células plasmáticas. La incidencia del Mieloma aumenta con la edad siendo la mediana de edad en el momento del diagnóstico 68 años. Afecta más a varones que a mujeres y su incidencia es el doble en la raza negra que en la blanca. Representa el 1% de todos los cánceres y un 13% de las neoplasias hematológicas. Su etiología es desconocida.

Disproteinemias (gammapatías M) Nivel de manejo del médico general: Diagnóstico: Sospecha. Tratamiento: Inicial. Seguimiento: Derivar.

Aspectos esenciales   

Se sospecha con un pico monoclonal en sangre u orina. Lo más frecuente es la gamapatia monoclonal de significado incierto (GMSI) y el mieloma múltiple (MM). El MM se presenta en paciente adulto con anemia, VHS elevada y dolor de espalda. GMSI no se trata.

Diagnóstico El hallazgo principal es un pico monoclonal en la sangre u orina mediante electroforesis en gel de agarosa con un pico alto y delgado en la región beta o en la gamma. Para continuar se hace inmunofijación. Otras enfermedades que pueden dar pico monoclonal son: enfermedad de von Willebrand adquirida, anemia perniciosa, artritis reumatoide, lupus eritematoso, esclerodermia, espondilitis anquilosante y la infección por VIH.

Caso clínico tipo Paciente de 73 años que en los exámenes rutinarios muestran un pico monoclonal. El paciente no refiere síntomas patológicos.

La GMSI no se acompaña de otros síntomas ni alteraciones de laboratorio y generalmente es un hallazgo. En el caso del MM, se acompaña en un 70% de los casos de dolor óseo, principalmente espalda y costal (síntoma más frecuente), secundario a las lesiones osteolíticas producidas por la enfermedad. Además se presenta con hipercalcemia, insuficiencia renal (principalmente por proteinuria de Bence-Jones), Hemograma con anemia NN (mieloptisis) y gran elevación de la VHS .

Definición Las gammapatías monoclonales (paraproteinemias o disproteinemias) son un grupo de trastornos caracterizados por la proliferación de un solo clon de células plasmáticas que sintetizan las inmunoglobulinas, que produce una proteína inmunológicamente homogénea comúnmente conocida como paraproteína o proteína monoclonal.

Lo más relevante es diferenciar mieloma de GMSI: serie ósea, mielograma, creatinina, calcio y hemograma deben ser normales. MM se etapifica con Beta 2 microglobulina. Se deriva, idealmente solicitando estudios básicos para avanzar.

Etiología-epidemiología-fisiopatología En condiciones normales, la maduración de las células plasmáticas secretoras de anticuerpos es estimulada por la exposición al antígeno para el que la inmunoglobulina de superficie posee especificidad. En los trastornos de las células plasmáticas se pierde el control de este proceso. Dentro de las gammapatías más frecuentes está la gammapatía monoclonal de significado incierto (GMSI) y el mieloma múltiple (MM). Dentro de este grupo también están: amiloidosis, enfermedad linfoproliferativa, plasmocitoma solitario y la macroglobulinemia de Waldeström.

Tratamiento La GMSI y el mieloma asintomático no requiere tratamiento, solo seguimiento. El mieloma múltiple se maneja con quimioterápicos: melfelan, bortezomib, corticoides o trasplante de progenitores hematopoyéticos según diversos protocolos.

Seguimiento

Las Gammapatías Monoclonales de Significado Incierto se caracterizan por una proliferación clonal de células plasmáticas que producen una proteina homogénea de carácter monoclonal. Cada molécula de inmunoglobulina consta de cuatro cadena polipeptídicas: dos cadenas pesadas y dos cadenas ligeras, unidas entre sí por puentes disulfuros, la cadena ligera puede ser kappa o lámbda y la cadena pesada (IgG, IgA o IgM) es la que da el nombre al componente monoclonal. Afecta al 1% de la población mayor de 50 años y al 3% de los mayores de 70 años. Tiene buen pronóstico pero hay que recordar que hasta un 25% de los casos desarrollan otras

Derivar a especialista para tipificar enfermedad y definir necesidad de tratar y esquema más adecuado.

4

Las causas se clasifican de acuerdo con los rasgos morfológicos que predominan en la medula ósea en:

Hipofunción medular

Pancitopenia Nivel de manejo del médico general: Diagnóstico: Sospecha. Tratamiento: Inicial. Seguimiento: Derivar.

Con Hipocelularidad de la Sin Hipocelularidad de la Médula Ósea Médula Ósea

Aspectos esenciales

Anemia Aplásica Hereditaria Hemoglobinuria Paroxística [Anemia de Fanconi]. Nocturna.

   

Anemia Aplásica Adquirida.

Las causas más frecuentes son la anemia aplásica adquirida, la mieloptisis y los síndromes mielodisplasicos. Se manifiestan como anemia, púrpura trombocitopénica y neutropenia febril. La mieloptisis se caracteriza por pancitopenia, reacción leucoeritroblástica y dolores óseos. Hay un tratamiento general de soporte y uno específico de acuerdo a cada causa.

Algunos Síndromes Mielofibrosis. Mielodisplásicos. Raras Leucemias Algunas Aleucémicas [LMA]. Aleucémicas.

Algunos Linfomas Médula Ósea.

Paciente de 28 años, sin antecedentes de haber recibido quimioterapia ni radioterapia previamente, presenta leucopenia de 1.300/ml, trombopenia de 25.000/ml y anemia de 7g/dL de Hb, sin blastos en sangre periférica y con aspirado de médula ósea muy hipocelular. Se sospecha cuadro de aplasia medular y se decide realizar una biopsia de médula ósea para confirmar el diagnóstico.

Linfoma de la Médula Ósea.

Diagnóstico Se sospecha por clínica y se confirma con hemograma y biopsia de MO. Las manifestaciones clínicas dependen de: -La etiología: en el caso del daño medular por proliferación anormal de células propias o la invasión de células ajenas hay destrucción de tejido, con debilitamiento óseo y la aparición eventual de dolores óseos y fracturas en hueso patológico. -Líneas celulares afectadas: característicamente hay un Síndrome anémico asociado en diferentes grados a manifestaciones derivadas de trombocitopenia (púrpura) y/o granulocitopenia (fiebre). Exámenes complementarios: Confirman y estiman la gravedad de enfermedad. -Hemograma: Anemia, trombocitopenia y leucopenia, además de reacción leucoeritroblastica (aumento de las formas inmaduras de las 3 series) en la mieloptisis -Frotis sanguíneo: Hay dacriocitos en las mieloptisis -Biopsia de medula ósea: Informa acerca de la celularidad de la médula ósea.

Disminución de las funciones hematopoyéticas de la médula ósea.

Etiología-epidemiología-fisiopatología En términos generales son enfermedades de baja frecuencia en la población. Particularmente se producen en Chile 2-4 casos nuevos al año de anemia aplásica con una distribución bifásica con un peak entre los 10-25 años y otro peak en mayores de 60 años. Se produce por 3 mecanismos fundamentalmente:



de la

Tricoleucemia.

Definición



Leucemias

Algunas Leucemias Mieloptisis. Linfoides Agudas.

Caso clínico tipo



Síndromes Mielodisplásicos.

Atrofia (aplasia): disminución total o parcial de las células hematopoyéticas mieloides. Invasión y destrucción: por proliferación anormal de células propias o ajenas (mieloptisis) Alteración primaria de la maduración: síndromes mielodisplasicos.

Tratamiento Se dividen en dos:  General: Es común para todos los pacientes con hipofunción medular y consiste en transfusión de glóbulos rojos y/o plaquetas cuando sea necesario, además del manejo de la neutropenia febril que muchas veces es la causa de muerte.  Específico: Es diferente para cada causa y habitualmente en pacientes menores corresponde al trasplante alogénico de progenitores hematopoyéticos y en los mayores a corticoides, inmunosupresores o antineoplásicos.

La hipofunción medular puede deberse a la infiltración de la médula ósea por células tumorales o por fibrosis que rebasan a los precursores eritroides normales. Las células que infiltran la médula pueden ser de origen medular (leucemia, mielofibrosis) o extramedular (mieloptisis, cáncer metastásico). Otras causas de hipofunción medular son la ausencia de precursores eritroides (anemia aplásica) a consecuencia de la exposición a fármacos, radiación, sustancias químicas, virus (hepatitis), mecanismos autoinmunitarios o factores genéticos, hereditarios (anemia de Fanconi) o adquiridos (hemoglobinuria paroxística nocturna). La mayoría de los casos de aplasia es idiopática.

Seguimiento Derivar. 5

Linfomas

Barr, HTLV-1 y VHC, y la infección por Helicobacter pylori en el linfoma MALT.

Nivel de manejo del médico general: Diagnóstico: Sospecha. Tratamiento: Inicial. Seguimiento: Derivar.

Diagnóstico Cuadro clínico: se caracteriza por adenopatías indoloras, de crecimiento progresivo en cualquier región, especialmente cervicales, axilares y supraclaviculares. Mayores de 1 cm, de más de 1 mes de evolución y que no regresan con antiinflamatorios y antibióticos (recordar que la mayor parte de las linfoadenopatías ocurren por procesos infecciosos). 30% de los casos de linfoma no Hodking pueden tener presentación extraganglionar, cuya ubicación mas frecuente es digestiva; debutando con sintomatología GI.

Aspectos esenciales    

Ganglios indoloros, de 1 cm, gomosos y que no resuelven con antiinflamatorios. Diagnóstico definitivo con biopsia excisional. Descartar adenopatías por otras causas. Derivar a cirujano o hematólogo.

Al examen físico los ganglios son gomosos, móviles, indoloros, no adheridos a planos profundos, de tamaño mayor a 1 cm.

Caso clínico tipo

El diagnóstico definitivo se realiza mediante biopsia excisional de un ganglio o tejido comprometido. Diagnóstico diferencial con adenopatías de causa infecciosa o metástasis. Importante son los síntomas B: Fiebre sin foco, sudoración excesiva, baja de peso (10% del peso corporal).

Hombre de 57 años que presenta varias adenopatías de las zonas axilar y supraclavicular, de entre 3-5 cm, indoloras y de varios meses de evolución.

Definición Los linfomas son neoplasias del sistema linfático que se caracterizan por aumento de tamaño de los ganglios linfáticos. Constituye una Patología AUGE y se clasifican en dos grandes grupos: linfoma Hodgkin y no Hodgkin. Hasta mediados del siglo XX eran enfermedades fatales, actualmente es posible curar aproximadamente el 80% de los linfomas de Hodgkin y el 50-60% de los linfomas no Hodgkin.

Tratamiento Se basa en la quimioterapia y/o radioterapia. El número de ciclos dependerá del estadio clínico. Estadios I y II: 3 ciclos de quimioterapia con doxorrubicina, bleomicina, vinblastina y dacarbazina y radioterapia de los campos comprometidos. Estadios III y IV: 6-8 ciclos de quimioterapia más radioterapia. Recaídas: Quimioterapia en altas dosis y trasplante autólogo de progenitores hematopoyéticos.

Etiología-epidemiología-fisiopatología Se estima que en Chile se diagnostican 100 linfomas Hodgkin y 600 no Hodgkin al año. Linfoma Hodgkin: Consiste en un tumor de células de Reed-Sternberg. Constituye un 1% de todas las neoplasias y es más frecuente en varones. Su presentación etaria sigue una curva bimodal con un primer pico a los 20-30 años y el segundo a los 60 años. Alta tasa de curación, sobre 80%. La mayor parte provienen de linfocitos B en los que los genes para Ig sufrieron un reordenamiento, pero no se expresan. La mayor parte de las células en los linfonodos afectados corresponden a células plasmáticas, linfoides, eosinófilos y monocitos normales. La etiología es desconocida, pero se ha observado una concordancia en monocigotos 99 veces más alta que la esperada, sugieriendo susceptibilidad genética. Se ha relacionado con el virus Epstein-Barr en la forma clásica del linfoma Hodgkin ya que la tinción inmunohistoquímica ha demostrado infección por este virus en la mitad de los casos aproximadamente. La distribución de los subtipos histológicos es: 75% esclerosis nodular; 20% celularidad mixta, y 5% los subtipos predominio linfocítico y depleción linfocitaria. Linfoma no Hodgkin: Constituye del 2 al 3% de todas las neoplasias, siendo cuatro veces más frecuente que la enfermedad de Hodgkin. Predominan en varones de edad media. Alrededor de 40% cura con el tratamiento actual. Su etiología se relaciona con factores de riesgo como la disfunción inmunológica previa (VIH, trasplantes, inmunodeficiencia común variable), radioterapia o quimioterapias previas, algunos virus como el Epstein-

Seguimiento Ante la sospecha, se deriva a un cirujano para realizar la biopsia excisional o a un hematólogo para confirmar o descartar la posibilidad de linfoma.

6

 

Punción lumbar

- Indicaciones de TAC antes de realizar punción lumbar  Mayores de 60 años.  Inmunodeprimidos.  Lesiones del SNC conocidas.  Convulsión dentro de 1 semana.  Nivel anormal de conciencia.  Signos focales en el examen neurológico.  Edema de papila visto en el examen físico, con sospecha clínica de una elevación de la PIC.  Sospecha de HSA, con el fin de diagnosticar la hemorragia intracraneal o cualquier efecto de masa intracraneal significativa que pudiera estar presente en los pacientes con HSA despierto y alerta con examen neurológico normal.

Nivel de manejo por el médico general: Realizar

Introducción La punción lumbar es un procedimiento que se realiza a menudo en el departamento de emergencia para obtener información sobre el líquido cerebro espinal. A pesar de que generalmente se usa para fines diagnósticos para descartar enfermedades potencialmente mortales (por ejemplo, meningitis bacteriana o hemorragia subaracnoidea), también se utiliza a veces para fines terapéuticos (por ejemplo, tratamiento de pseudotumor cerebri). La punción lumbar debe realizarse sólo después de un examen neurológico minucioso, pero nunca debe retrasar intervenciones que salvan vidas, tales como la administración de antibióticos y esteroides a pacientes con sospecha de meningitis bacteriana.

Materiales e Insumos Una bandeja de punción espinal o lumbar debe incluir los siguientes artículos: - Kit básico:  Apósito estéril.  Mascarilla + Gorra.  Guantes estériles.  Campo estéril.  Solución antiséptica + tórulas.  Lidocaína al 1%.  Jeringa 3 ml.  Agujas calibre 20 y 25.  Agujas espinales, calibre 20 y 22.  Cuatro tubos de ensayo de plástico, numerados 14.  Bandeja desechable.

Indicaciones Se realiza en el caso de sospecha de:    

Meningitis. Hemorragia Subaracnoidea. Sospecha de enfermedades del sistema nervioso central, tales como Sindrome de Guillain- Barre y Meningitis Carcinomatosa. Alivio terapéutico de pseudotumor cerebral.

El procedimiento y la obtención de muestras permiten efectuar múltiples mediciones y estudios, así como administración de fármacos u otras medidas terapéuticas:      

 

Coagulopatías. Absceso cerebral.

- Según situación:  Tres vías llave de paso.  Manómetro.

Medición de presión de LCR Estudios diagnósticos para determinar la presencia de hemorragia, procesos inflamatorios o infecciosos. Estudio citológico y citoquímico. Estudio inmunológico: inmunidad celular y humoral (IgG y bandas oligoclonales). Inyección de sustancias diagnósticas (marcadores isotópicos, contrastes radiológicos, colorantes, etc.) o de fármacos (antineoplásicos y otros). Drenaje de LCR en casos de hipertensión intracraneal idiopática (pseudotumor cerebri) o refractaria a otras medidas (drenaje lumbar continuo). Monitorización y seguimiento de evolución clínica o terapéutica. Intercambio de LCR (“licuorféresis”): remoción de sustancias nocivas mediante filtrado.

Preparación 1-. Consentimiento informado. 2-. Certificar que estén todos los implementos necesarios. 3-. Posicionamiento del paciente. - Posición sentada. Para abrir los espacios interlaminares, el paciente debe inclinarse hacia adelante y ser apoyado (Figura 1). - Decúbito lateral. Caderas, rodillas y la barbilla hacia el pecho con el fin de abrir los espacios interlaminares (Figura 2).

Contraindicaciones - Absolutas  Infección sitio de punción.  Presiones desiguales entre el compartimiento Supra e Infratentorial. - Relativas  Aumento de la presión intracraneal (PIC). 7

Limpieza de la piel con solución antiséptica (Figura 3)

Figura 1

Posicionar paño estéril (Figura 4) 3-. Anestesia local con lidocaína 1%. Inocular a nivel subcutáneo hasta formar una pequeña pápula en el sitio de punción. Profundizar luego la jeringa, aspirado para confirmar que no se está en un vaso sanguíneo, e inocular. Realizar mismo procedimiento encima, por debajo y a los lados muy ligeramente (usando el mismo sitio de la punción). 4-. Punción lumbar. Sujetar trocar de punción con ambos pulgares, disponiendo la aguja con el bisel hacia arriba. Sobre la pápula de anestesia ingresar a la piel con la aguja dirigida ligeramente hacia cefálico. Avanzar hasta sentir cambio de resistencia de duramadre o hasta haber alcanzando 4-5 cm (en este último caso se retira estilete y se observa si la salida de líquido es efectiva). Una vez objetivada la salida de LCE se retira el estilete del trocar y se dispone a su recolección (Figura 5).

Figura 2.

Procedimiento 1-. Verificar lugar de punción. Localizar el espacio interespinoso L3-L4 o L4 - L5 palpando por posterior las crestas ilíacas izquierda y derecha; se traza una línea imaginaria que coincide con espacios a nivel lumbar. 2-. Técnica estéril. El médico debe tener mascarilla y gorra, realizar lavado clínico completo. Colocación de guantes estériles. Limpiar la piel con solución antiséptica y tórulas, extendiendo de forma circular hasta un espacio por sobre y bajo el lugar de punción (Figura 3). Posteriormente se coloca un paño estéril bajo el paciente y un paño estéril fenestrado en el lugar donde se realizará la punción (Figura 4).

Figura 5. 5-. Ontención de muestras (4 tubos). Retirar LCE en 4 tubos rotulados, idealmente 1ml por cada uno (20 gotas).

8

6-. Retiro de trócar. Debe reintroducrise estilete nuevamente, y posteriormente, en conjunto, se retira con el trocar. 7-. Aseptizar nuevamente y colocar apósito estéril. 8-. Reposicionamiento del paciente en decúbito supino posterior a punción.

Complicaciones      

Cefalea (20 – 70%) 24 a 48 hrs posterior al procedimiento, autolimitada. Dolor en zona de punción (35%). Parestesia (transitoria: 13%, persistente: 0,2%). Infección (del sitio de punción o sistema nervioso central). En caso de sepsis, 2% de meningitis. Complicaciones hemorrágicas (raro en pacientes sin coagulopatías, hasta 7% en coagulopatías). Herniación cerebral.

9

2. La PTI crónico o enfermedad de Werlhof (> 6 meses), tiene mayor incidencia en adultos jóvenes de 2040 años y una relación entre mujeres y hombres de 5:1; sin recuperación espontánea y con recidivas. Se debe a la reacción de anticuerpos IgG que van dirigidos a antígenos de membrana de la plaqueta (glicoproteínas Ib y IIb/IIIa), macrófagos esplénicos reaccionan con la fracción constante de IgG destruyendo la plaqueta.

Púrpuras trombopénicos Nivel de manejo del médico general: Diagnóstico: Sospecha. Tratamiento: Inicial. Seguimiento: Derivar.

La PTT suele tener un comienzo brusco y parecerse al CID. Es más frecuente en mujeres de edad media. Tiene una mortalidad de 80-90%. La etiología es desconocida pero hay casos con antecedentes de infección respiratoria alta, fármacos, embarazo (Sd. HELLP), LES. Respecto a la patogenia se ha descrito una deficiencia de una metaloproteasa (ADAMTS 13), responsable de la degradación del factor von Willebrand.

Aspectos esenciales     

Disminución cuantitativa de plaquetas por destrucción. PTI es asintomática o presenta Sd. purpúrico. Descartar causas secundarias. Tto inicial PTI: Prednisona 1 mg/kg. Casos graves hemorrágicos: Ig EV. PTT: emergencia, tratamiento es la plasmaféresis.

Diagnóstico La PTI se presenta en forma asintomática y es descubierta como trombocitopenia aislada al hemograma o como un síndrome purpúrico. Al examen físico pueden haber petequias o equimosis. La presencia de esplenomegalia obliga a buscar otras causas. El hemograma también puede haber anemia en casos de sangrado significativo. La realización de anticuerpos antiplaquetarios no se recomienda por bajo rendimiento. Muy importante descartar otras causas, en especial VIH (ver causas inmunes). La PTT se presenta con una pentada clínica dada por: Anemia hemolítica microangiopática (esquizocitos y Test de Coombs negativo), Trombocitopenia, Fiebre, Compromiso renal y compromiso neurológico (desde trastornos conductuales a convulsiones y coma). Para diagnóstico definitivo se usan biopsias de médula ósea, piel, encías o músculo. Existe también una forma localizada de la enfermedad más frecuente en niños, llamado síndrome hemolítico urémico, este síndrome cursa sin alteración neurológica con predominio renal e hipertensión arterial y suele desencadenarse por infección gastrointestinal asociada a E. coli enterohemorrágica y shigella.

Caso clínico tipo Paciente femenino de 26 años, hace 3 años presenta máculas puntiformes y placas púrpuras en las EEII correspondiente a petequias y equimosis, el último mes refiere epistaxis espontánea a repetición motivo por el que consulta. En el hemograma: recuento plaquetario de 50.000/μL .

Definición Alteración plaquetaria cuantitativa debido a un aumento en la destrucción plaquetaria de causa inmune o no inmune, que cursa con trombocitopenia periférica que corresponde a la disminución del número de plaquetas por debajo de 100.000/μL aproximadamente, disminuciones inferiores a 50.000/μL pueden ocasionar sangrado postrauma y menores a 20.000/μL sangrado espontáneo.

Etiología - Epidemiología – Fisiopatología

Tratamiento No todos los pacientes con PTI requieren tratamiento, esto depende de su sintomatología hemorrágica, el recuento plaquetario y las condiciones del paciente. La terapia inicial son corticoides (Prednisona 1mg/kg), respondiendo en un 70-80% pero sólo un 10-20% se mantienen en remisiones prolongadas. Si no responde o requiere tratamiento prolongado se puede realizar una esplenectomía (previa vacunación antineumocócica), donde el 80% mejora. Ya de tercera línea se utilizan pulsos de dexametasona, inmunosupresores (vincristina, ciclofosfamida, azatioprina, ciclosporina A). La inmunoglobulina EV se indica para casos agudos con riesgo vital, hemorragia craneana o preoperatorio. Recientemente se ha utilizado anticuerpo monoclonal Anti CD 20 (Rituximab) con buenos resultados. La PTT es una emergencia hematológica y casi siempre es mortal si el tratamiento apropiado no se inicia de inmediato. En consecuencia, todos los pacientes con sospecha de PTT deben ingresar inmediatamente a un centro médico capaz de realizar plasmaféresis con recambio plasmático para remover multímeros del factor von Willebrand y anticuerpos contra ADAMTS 13, e infusión PFC o criosupernatante que aporta metaloproteasa. Prednisona 1 mg/kg o Metilprednisolona

La vida media plaquetaria normal es de alrededor de 10 días. En el púrpura trombocitopénico la disminución de la supervivencia plaquetaria debido a la destrucción incrementada de estas obedece a distintas etiologías, puede clasificarse en: 1. Causa inmune: Púrpura trombocitopénico idiopático (PTI), Estados autoinmunes (LES, leucemia linfocítica crónica, linfoma), Inducido por drogas (Causa más frecuente en adultos, heparina, ácido valproico, penicilina, sulfonamidas, cimetidina, tiacidas), Infeccioso (VIH, malaria), Púrpura postransfusional, Púrpura neonatal. 2. Causa no inmune: Coagulación intravascular diseminada (CID), Púrpura trombocitopénico trombótico o Síndrome de Moschcowitz (PTT), Hemangioma cavernoso, Bypass cardiopulmonar, Hiperesplenismo. Según su duración: 1. PTI agudo (< 6 meses), enfermedad infantil que ocurre por igual en ambos sexos, 80% tras infecciones respiratorias virales. En general, se recuperan espontáneamente y tiene escasa recidiva y mortalidad. Suele asociar eosinofilia y linfocitosis. 10

125 mg/12hr. El intercambio de plasma se realiza diariamente hasta que el recuento de plaquetas se ha normalizado y hemólisis cesado en gran medida, como se evidencia por un retorno de la deshidrogenasa láctica sérica (LDH) a la concentración normal o casi normal. El tratamiento aumenta la sobrevida de 10% a un 90%.

Seguimiento Derivar a especialista.

11

Quimioterapia intratecal

Materiales e insumos 1. Material para asepsia o Gorro o Mascarilla o Bata estéril o Guantes estériles y no estériles o Paños estériles (fenestrados y no fenestrados) o Gasas y apósitos estériles o Antiséptico tópico tipo clorhexidina o Mosquisto 2. Material para anestesia local o Jeringa 10 ml o Aguja subcutánea o intramuscular o Solución anestésica 3. Material de punción y extracción LCR o Tubos estériles trasnparentes para analizar LCR o Trócar para punción lumbar (N° 20 o N°22)

Nivel de manejo del médico general: Derivar a especialista

Introducción Se trata de un procedimiento que consiste en una punción con una aguja en la zona de la columna lumbar para introducir uno o varios fármacos para la profilaxis o el tratamiento de enfermedades, normalmente neoplasias hematológicas. El beneficio de la quimioterapia intratecal es que supera la dificultad que tienen la mayoría de los fármacos de uso habitual para atravesar la barrera hematoencefálica.

Indicaciones    

Preparación y procedimiento

Linfomas de alto grado. Leucemia Linfoide Aguda. Carcinomatosis meníngea. Metástasis de tumores sólidos.

1. Posicionamiento del paciente 1. Posición sentada, inclinado hacia adelante 2. Decúbito lateral. Caderas, rodillas y la barbilla hacia el pecho con el fin de abrir los espacios interlaminares 2. Verificar lugar de punción 1. Localizar el espacio interespinoso L3-L4 o L4 - L5 palpando a posterior las crestas ilíacas izquierda y derecha moviendo los dedos en sentido medial hacia la columna. Palpar el lugar más amplio para definir zona de punción interlaminar. 3. Técnica aséptica todo el procedimiento 1. El médico debe tener mascarilla, gorra, delantal y guantes estériles. 2. Limpieza de piel con solución antiséptica y tórulas, extendiéndose de forma circular de adentro hacia afuera (idealmente cubrir una zona amplia para evitar contaminación 3. Colocación de paño estéril bajo el paciente y un paño estéril fenestrado en el lugar donde realizaremos la punción. 4. Anestesiar zona a puncionar 1. Utilizar jeringa de para administrar lidocaína 1%. Levantar una pápula en la piel con la aguja. Insertar la aguja todo el camino hasta el centro, aspirado para confirmar que la aguja no está en un vaso sanguíneo y, a continuación, inyectar una pequeña cantidad retirando la aguja unos pocos centímetros, continúe este proceso encima, por debajo y a los lados muy ligeramente (usando el mismo sitio de la punción). 5. Punción lumbar 1. Teniendo el trocar o aguja espinal dentro de nuestro equipo inicial, se procede a la PL propiamente tal. Se estabiliza trocar con pulgares, y se dispone la aguja con bisel hacia arriba. Sobre la pápula de anestesia se ingresa a la piel de forma perpendicular ligeramente cefálica, se avanza hasta sentir que sobrepasamos la duramadre (“pop” característico) o hasta cuando se haya avanzado 4-5 cm (en este

Contraindicaciones     

Infección del sitio de punción Escoliosis severa Diátesis hemorrágica Inestabilidad hemodinámica Tumores avanzados

Complicaciones         

Cefalea por disminución de la presión secundaria a la extracción de líquido o por irritación meníngea transitoria. Punción de fibras nerviosas. Hematomas en el sitio de la punción. Dolor local. Infección. Herniación transtentorial en pacientes con procesos intracraneales previos. Aracnoiditis. Mielopatía progresiva. Leucoencefalopatía.

Con respecto a complicaciones atribuibles a los fármacos administrados, se han reportado casos de neurotoxicidad posterior a la quimioterapia intratecal, incluyendo lesiones de la médula espinal, encefalopatías y convulsiones. No se han identificado factores predisponentes; sin embargo, la neurotoxicidad parecen ocurrir predominantemente en pacientes que recibieron combinaciones de metotrexato y Ara-C intratecal. Manifestaciones como tetraplejia, paraplejia y síndrome de cauda equina constituyen las lesiones más frecuentemente descritas por neurotoxicidad; mientras que la aparición de convulsiones y encefalopatía han sido reportadas comparativamente en muchos menos casos.

12

último caso se retira estilete y se observa la salida de líquido es efectiva) se retira el estilete del trocar y se dispone a retirar el LCE. 2. Se retira LCE en 4 tubos con cantidad de 1ml por cada tubo (20 gotas por tubo), recordar que tubos deben ser bien identificados y caracterizar el examen a pedir. 6. Administración del fármaco a través del trócar. Una vez finalizado, se coloca estilete nuevamente, se retira trocar. Limpieza y colocación de gasa estéril 7. Reposicionar al paciente 1. Paciente se coloca en posición supina, posterior a punción (al menos por una hora).

13

(sarampión, varicela, parotiditis, mononucleosis y hepatitis) y posterior a la administración de vacunas. Las reacciones leucemoides monocíticas son raras y se observan en las parasitosis.

Reacción leucemoide Nivel de manejo del médico general: Diagnóstico: Sospecha. Tratamiento: Inicial. Seguimiento: Derivar.

Diagnóstico

Paciente internado por un cuadro infeccioso grave que en un hemograma aparece un recuento de leucocitos de 60.000 células/µL, sin presencia de blastos en sangre periférica.

El diagnóstico es de laboratorio, se realiza con un hemograma que muestre un recuento periférico de leucocitos mayor a 50.000 células/µL. Por lo general no hay esplenomegalia, es rara la trombocitosis, no hay cromosoma Filadelfia y la fosfatasa alcalina de los neutrófilos esta aumentada. Se debe diferenciar esta reacción de una leucemia aguda o crónica, para esto recordar que la leucemia mieloide aguda cursa con un porcentaje alto de blastos en sangre periferica y en médula ósea muestra un exceso de blastos que desplazan al resto de la series hematopoyéticas, dando origen a trombocitopenia, anemia y leucopenia. La leucemia mieloide crónica por su parte cursa con esplenomegalia y el nivel de fosfatasa alcalina leucocitaria es baja. Las técnicas citogenéticas o de hibridación fluorescente in situ, revelan la presencia de cromosoma Filadelfia en más del 90% de los casos. Ante cualquier duda debe derivarse el paciente al hematólogo.

Definición

Tratamiento

Es un desorden hematológico en el cual el recuento de leucocitos sanguíneos es mayor a 50.000 células/µL causado por otra patología que induce dicha reacción en la médula. El aumento del recuento de leucocitos es en base principalmente a formas maduras y el recuento diferencial muestra una desviación a izquierda, pueden corresponder a neutrofílicas, linfocíticas o monocíticas, según el predominio celular. Sin embargo, en las reacciones leucemoides graves se pueden observar precursores de neutrófilos menos maduros como los promielocitos y mieloblastos. Se diferencia de la leucemia en que se observa proliferación de todos los elementos mieloides normales en la médula ósea, en contraste con leucemia aguda, en la que los elementos más inmaduros (por ejemplo, promielocitos y mieloblastos) predominan. Otra diferencia es que la reacción leucemoide nunca tiene el cromosoma Filadelfia.

Al ser la reacción leucemoide una respuesta del organismo a otra patología que no afecta a la médula ósea, generalmente se resuelve al tratar la patología que la originó.

Aspectos esenciales    

Recuento de leucocitos mayor a 50.000 células/µL. Por lo general sin presencia de blastos en sangre periférica. Generalmente secundaria a una infección grave. Siempre descartar leucemia.

Caso clínico tipo

Seguimiento Por especialista.

Etiología-epidemiología-fisiopatología Las reacciones leucemoides mieloides neutrofílicas son las más frecuentes y consisten en un aumento de neutrófilos y bandas. Se presenta a cualquier edad, de preferencia en jóvenes y adultos jóvenes y puede ocurrir como respuesta a infecciones bacterianas severas (neumonía, septicemias, endocarditis bacteriana, meningitis e infección por Clostridium difficile y Shigella) también puede ser secundaria a hemorragias, hemólisis severa y eclampsia o como reacción a químicos (intoxicación por mercurio, gas mostaza, benceno), ingesta de sulfas, en recuperación de agranulocitosis o depresión medular, en recuperación de anemia megaloblástica, cetoacidosis diabética, quemaduras graves, neoplasias y en tuberculosis aguda diseminada. Las reacciones leucemoides linfoides presentan una linfocitosis absoluta donde se pueden apreciar linfocitos atípicos. Se observan en las infecciones virales

14

y finalmente isquemia. Esta isquemia del tejido nervioso origina degeneración neural.

Síndrome compresión medular

Diagnóstico

Nivel de manejo del médico general: Diagnóstico Sospecha Tratamiento Inicial Seguimiento Derivar

El diagnóstico de la compresión medular se basa en el diagnóstico clínico, exploración física y la posterior confirmación por pruebasde imagen. El síntoma más temprano y frecuente es el dolor de espalda. Puede afectar a cualquier parte de la columna vertebral y suele orientar hacia el nivel de la compresión. Característicamente empeora con los movimientos, el decúbito, la tos, con la maniobra de Valsalva, la flexión del cuello o las extremidades inferiores y en ocasiones no cede con los analgésicos habituales. Puede asociarse a un dolor radicular cuando se afectan las raíces nerviosas y que se irradian por el dermatoma correspondiente (de forma generalmente bilateral a nivel dorsal y unilateral en los niveles lumbares). La percusión de las apófisis espinosas de las vértebras afectadas ayuda a localizar la lesión. Es de suma importancia determinar dónde se inició el dolor y recordar que si la compresión es extramedular la progresión de los síntomas va a ir desde las extremidades inferiores hasta el nivel de la compresión. El siguiente síntoma en frecuencia es la debilidad en las extremidades (85 %). Esta se acompaña con alteraciones en la marcha y el equilibrio y dos tercios de los enfermos no pueden caminar al momento del diagnóstico. Suele ser bilateral y simétrico, de inicio proximal extendiéndose en sentido distal y se debe a la afección de los haces córticoespinales. La clínica sensitiva es mucho menos frecuente como presentación inicial. Se manifiesta por parestesias y pérdida de sensibilidad. También se presentan alteraciones del sistema nervioso autónomo como la pérdida del control de los esfínteres y la impotencia.

Aspectos esenciales 





El síndrome de compresión medular es una urgencia oncológica que debe ser diagnosticada dentro de las primeras 12 a 24 horas, pues de ello, depende el pronóstico funcional del paciente. Se debe sospechar en cualquier enfermo con patología oncológica, con síntomas como dolor en la columna, debilidad en miembros inferiores y dificultad en la marcha. La prueba diagnóstica de elección es la resonancia magnética nuclear por lo que el paciente debe ser remitido a un centro hospitalario, para realizar de forma multidisciplinaria su estudio y tratamiento.

Caso clínico Hombre 51 años, tabáquico, con antecedentes de cáncer de piso de boca en tratamiento. Refiere 1 semana previo al ingreso disminución de la fuerza e impotencia funcional de miembros inferiores.

Definición

A la exploración se encuentra una primera fase de espasticidad con hiperreflexia y signo de Babinsky bilateral, en fase más evolucionada o en un síndrome de compresión medular de instauración rápida el paciente muestra flacidez e hiporreflexia.

El síndrome de compresión medular es una urgencia oncológica y neurológica de mal pronóstico que se presenta de manera similar en ambos sexos y cualquier enfermedad neoplásica diseminada puede llegar a provocarlo.

Para la confirmación diagnóstica las pruebas de mayor utilidad son:

Etiología-Epidemiología-Fisiopatología

-Radiografía simple: revela en el 80-85 % alteraciones como erosión o pérdida de pedículos, lesiones líticas o blásticas, colapso vertebral, masas paraespinales, mientras que el 15-25 % de los enfermos no presentan alteraciones. - Gammagrafía ósea: es más sensible que la radiografía simple para la detección de metástasis, pero menos específica. - Mielografía: era considerada la técnica de elección, supone la realización de una técnica invasiva, se asocia a incomodidad y a complicaciones potenciales por la punción. - Resonancia magnética (RM): actualmente es la técnica estándar en las situaciones de urgencia. -Tomografía axial computarizada (TAC): es superior al resto de las técnicas de imagen a la hora de evaluar detalladamente la matriz ósea. - Tomografía por emisión de positrones en el diagnóstico precoz de metástasis vertebrales antes de que aparezca el déficit neurológico. - Confirmación histológica por medios invasivos en aquellos pacientes sin antecedentes oncológicos o sin asociación temporal evidente.

Etiología: Este síndrome se presenta de manera similar en ambos sexos y cualquier enfermedad neoplásica diseminada puede llegar a provocarlo. No obstante, los tumores que la producen con mayor frecuencia son: pulmón (12-32 %), próstata (4-28 %), mama en la mujer (12-36 %). Es un poco más raro en los linfomas, carcinoma renal, melanoma y tumores gastrointestinales. Un tercio de los casos que presentan este síndrome son la primera manifestación del tumor, especialmente en el cáncer de pulmón. Es más común a nivel de las vertebras toracicas (60%), seguida de la lumbares (30%) y luego de las cervicales(10%). Fisiopatología: Existe una gran correlación entre los tumores que metastizan con mayor frecuencia a nivel vertebral y el desarrollo de este síndrome. La vía de diseminación tumoral puede ser hematógena y por contigüidad. La mayor parte de las veces la invasión del canal medular se produce desde el cuerpo vertebral. La compresión tumoral provoca estasis venoso, lo que condiciona hipoxia. Como consecuencia aparece edema, lo cual genera más compresión, reducción del flujo capilar

15

Tratamiento Debe instaurarse lo más rápidamente posible (incluso ante la sospecha clínica). Las opciones disponibles son cuatro (bien por separado o bien combinadas): corticoterapia, radioterapia, quimioterapia y descompresión quirúrgica. Este es individualizado y su elección dependerá de la esperanza de vida, la localización, el número de lesiones, el mecanismo de producción de la compresión, la histología tumoral, la velocidad de progresión, la clínica neurológica y tratamiento previo con radioterapia. 1. Dieta baja en sal, inmovilización del paciente y oxígeno suplementario cuando esté indicado: desde el inicio también será necesario utilizar analgésicos y que, aunque a veces los antiinflamatorios pueden controlarlo, en muchas ocasiones el dolor requiere de la utilización de opiáceos y se debe tener en cuenta que estos pueden empeorar algunos síntomas como el estreñimiento y la disfunción vesical. 2. Corticoesteroides, como dexametasona o metilprednisolona: deben iniciarse cuanto antes, en presencia de sospecha clínica de esta complicación, aun cuando no esté confirmada por estudio de imagen. Es la primera medida terapéutica por su acción oncolítica, antiedematosa y antiinflamatoria. La droga de elección es la dexametasona. 3. Radioterapia: Este tratamiento tiene como objetivo aliviar el dolor, reducir las necesidades de analgésicos, prevenir el desarrollo de fracturas patológicas, mejorar la movilidad y actividad del paciente y si es posible prolongar la supervivencia, pues detiene el crecimiento tumoral y reduce la presión de los tejidos periféricos. 4. Quimioterapia: Se emplea en casos selectivos, como aquellos que recién debutan con alguna neoplasia con un buen estado neurológico, y con tumores quimiosensibles (tumor de células germinales y mieloma múltiple). 5. Cirugía: Las indicaciones para la cirugía son: • Inestabilidad de columna. • Compresión ósea de la médula espinal. • Deterioro neurológico durante la radioterapia. • Compresión en un área medular previamente tratada con radioterapia. • Tumores radiorresistentes. • Compresión medular en un paciente sin historia previa de cáncer o con episodio muy lejano que precise diagnóstico histológico.

Seguimiento Derivar.

16

Diagnóstico

Síndrome leucoeritroblástico

El diagnóstico se hace mediante un hemograma y su respectivo análisis de frotis sanguíneo. La presencia de formas nucleadas eritroides y formas mieloides inmaduras hacen el diagnóstico de reacción leucoeritroblástica. Además, es característica la presencia de Dacriocitos (eritrocitos en gota). El diagnóstico etiológico se realiza mediante distintos exámenes dirigidos por el resultado del hemograma y de la anamnesis y examen físico.

Nivel de manejo del médico general: Diagnóstico: Inicial. Tratamiento: inicial. Seguimiento: Derivar.

Aspectos esenciales   

Tratamiento

Presencia de formas eritroides nucleadas y mieloides inmaduras en el frotis sanguíneo. Refleja, generalmente, ocupación de la médula ósea de distinta etiología. Refleja, en otros casos, acortamiento del tiempo de maduración intramedular de las líneas por un aumento de las demandas.

El tratamiento es el de la causa subyacente a la reacción leucoeritroblástica.

Seguimiento Sujeto a especialista dependiendo de subyacente a la reacción leucoeritroblástica.

Caso clínico tipo Paciente de 65 años, diagnosticado de cáncer de próstata avanzado hace 7 meses que no recibe tratamiento. Hace dos semanas inicia cuadro de CEG, ictericia. Al hemograma destaca una anemia normocíticanormocrómica, trombocitopenia, leucopenia con desviación izquierda y formas inmaduras (sin blastos). En sangre periférica se pesquisan eritroblastos.

Definición Presencia de precursores eritroides nucleados, esquistocitos y formas mieloides inmaduras en el frotis de sangre periférica.

Etiología - Epidemiología – Fisiopatología El síndrome o reacción leucoeritroblástica es consecuencia de múltiples patologías. Frecuentemente, refleja la invasión, ocupación o reemplazo de la médula ósea por distintas etiologías. Las principales causas de este síndrome son:  Mieloptisis: generalmente por invasión medular de neoplasias no hematológicas, procesos granulomatosos o fibrosis. Eritrocitos en forma de lágrima.  Mielofibrosis: Fisiopatología similar a la anterior, pero este corresponde a un trastorno primario de la medula ósea.  Carcinoma metastásico a la médula  Leucemias  Mieloma múltiple. También es consecuencia de un acortamiento en el tiempo de maduración dentro de la médula ósea de la línea eritroide y mieloide por estímulos que aceleran su liberación a la circulación periférica. Otros fenómenos también lo pueden manifestar, aunque de forma excepcional: sepsis, hemólisis, hipoxia severa, insuficiencia renal, hemorragias, embolismo pulmonar e insuficiencia cardíaca.

17

la

causa

Caso clínico tipo

petequias espontáneas, hemorragias nasal, gingival, menorragia e inclusive sangrado urinario o gastrointestinal. También debe buscarse linfadenopatía, hepatoesplenomegalia y trombosis. PTI se presenta con síndromes purpúricos o una trombocitopenia aislada en los exámenes de sangre. El SHU y la PTT presentan: anemia hemolítica microangiopática, trombocitopenia, fiebre, compromiso renal y compromiso neurológico. Se debe buscar infecciones por virus o evaluar el consumo de heparina. Si hay sepsis asociado, pensar en CID. No hay pruebas de laboratorio adicionales además del recuento plaquetario y frotis de sangre periférica que sean absolutamente necesaria en un paciente con trombocitopenia aislada. Los adultos con nueva trombocitopenia deben tener pruebas de VIH y VHC. Las pruebas adicionales de laboratorio puede estar justificada en pacientes con otros hallazgos. La urgencia depende del grado de trombocitopenia y otras anomalías, y la estabilidad de los resultados. La evaluación de la médula ósea no es necesaria en todos los pacientes con trombocitopenia. Sin embargo puede ser útil en algunos paciente si se sospecha de una enfermedad hematológica primaria.

Paciente que consulta por hematuria y hematomas en su cuerpo sin relación con traumatismos. El recuento de plaquetas es de 30.000/mm3.

Tratamiento

Trombopenia severa Nivel de manejo del médico general: Diagnóstico Sospecha Tratamiento Inicial Seguimiento Derivar

Aspectos esenciales    

Se habla de grave cuando hay un recuento bajo 20.000 plaquetas /mm3, pues hay sangrado espontáneo Se presenta con equimosis, petequias y hemorragias. Los mecanismos son por déficit de producción o aumento de la destrucción. El tratamiento dependerá de la alteración basal.

El manejo de pacientes con trombocitopenia depende del diagnóstico subyacente. Principios generales que se aplican a todos los pacientes incluyen evitar los medicamentos que interfieren con la función plaquetaria, coordinación con los anestesiólogos y cirujanos antes de procedimientos invasivos, y corrección de alteraciones de la coagulación. A menudo no se necesitan restricciones de actividades. La terapia del PTI se inicia con prednisona 1mg/Kg o metilprednisolona según gravedad de Trombopenia. Si fracasa se puede utilizar Ig-ev o esplenectomía. El SHU requiere plasmaféresis, infusión PFC y soporte renal.

Definición La trombocitopenia se define como un recuento de plaquetas por debajo del límite inferior de la normalidad (es decir, <150.000/microL para los adultos). Los grados de la trombocitopenia pueden subdividirse en leve (recuento de plaquetas de 100.000 a 150.000/microlitro), moderada (50.000 a 99.000/microlitro) y severa ( <50.000/microlitro). La trombocitopenia grave confiere un mayor riesgo de sangrado, pero la correlación entre el recuento de plaquetas y el riesgo de sangrado varía de acuerdo a la condición subyacente y puede ser impredecible.

El tratamiento del PTT es plasmaféresis, PFC y uso de esteroides. En caso de trombopenia causado por heparina, la suspensión de esta es crítica.

Etiología-epidemiología-fisiopatología

En los casos de CID, el tratamiento inicial es tratar la patología de base.

Las causas de trombopenia se pueden dividir en:  Disminución de la producción de plaquetas, dentro de las cuales están: anemia aplástica, leucemia, etc.  Destrucción o consumo de plaquetas: púrpura trombocitopénica idiopática (PTI) (causa autoinmune y afectación de producción), VIH, coagulación intravascular diseminada (CID), púrpura trombocitopénica trombótica (PTT) y síndrome hemolítico urémico (SHU).

Transfusión de Plaquetas: Indicada en forma - Terapéutica: hemorragia activa atribuible a trombocitopenia. - Profiláctica (sin hemorragia activa): en patología médica con recuento <10.000/mm3; y en pacientes quirúrgicos y obstétricos con recuento <50.000/mm3. - Contraindicado en PTI (a menos que tenga riesgo vital), PTT, PT Postransfusional

Diagnóstico

Seguimiento

La historia debe centrarse en los recuentos plaquetarios previos, antecedentes familiares, sangrado, medicamentos, el exceso de medicamentos de venta libre, exposiciones infecciosas, las prácticas dietéticas, y otras condiciones médicas (por ejemplo, trastornos hematológicos, condiciones reumatológicas, cirugía, transfusión). El examen físico debe evaluar el sangrado, ya que se presenta con sangrado cutáneo en forma de equimosis y

Derivar a especialista para estudio etiológico.

18

MÓDULO 1: Medicina Interna

Nefrología Alcalosis Metabólica nistración de citrato, etc. • En los vómitos la pérdida de H+ impide la secreción de bicarbonato pancreático, esto sumado a la pérdida de volumen misma que aumenta el umbral renal de bicarbonato por medio de la acción de la aldosterona, aumentan la concentración de HCO3- en plasma. • El uso de diuréticos de asa y tiazidas aumenta la oferta distal de Na+ a los túbulos colectores que en conjunto con la acción de aldosterona inducida por la pérdida de volumen, favorece la retención de H+ en el lumen tubular, explicando la pérdida de ácido a nivel renal. • La Hipokalemia causada por pérdidas de K+ primarias o en los vómitos permite un intercambio de K+ intracelular por H+, lo que a nivel de túbulo colector aumenta la secreción de H+ y la reabsorción de HCO3-. • En pacientes edematosos, la pérdida de volumen libre de bicarbonato produce aumento en la [HCO3-] plasmática (alcalosis de contracción). • Por último en pacientes con ↑pCO2 crónica (hipoventilantes) puede producirse alcalosis metabólica al normalizar rápidamente su ventilación.

Nivel de manejo del médico general: Diagnóstico Específico Tratamiento Inicial Seguimiento Completo

Aspectos Esenciales • • • • •

Vómitos profusos y diuréticos como causa principal. BE positivo y [HCO3-]st aumentado. Hipoxia tisular, riesgo de tetania y arritmias. Corrección hipovolemia e hipokalemia. Evaluación de pH urinario.

Caso Clínico Tipo Mujer, 32 años, vómitos profusos por gastroenteritis viral, ingesta líquida disminuida, signos de deshidratación. Perfil bioquímico: pCO2:48 mmHg BUN: 31 mg/dl Crea:1,2 mg/dl Na+:141 mEq/l K+:3,2 mEq/l Cl-:90 mEq/l CO2 Total:36 mEq/l pHarterial:7,5 pHorina:5 Na+orina:10 mEq/l.

Diagnóstico Definición

El diagnóstico de este cuadro es de sospecha clínica con confirmación con exámenes La clínica presente en pacientes con alcalosis metabólica deriva de: • La vasoconstricción arteriolar. • Hipocalcemia (cefalea, letargia, delirio, tetania, convulsiones y estupor). • Hipokalemia (aparición de arritmias supraventriculares y ventricula). • Depresión respiratoria. • Aumento de la afinidad de la hemoglobina por oxígeno debido a la alcalemia (hipoxia tisular, hipercapnia, acidosis láctica). De acuerdo a parámetros de laboratorio, la alcalosis metabó-

Trastorno caracterizado por elevación primaria de la [HCO3-] plasmático y un aumento del pH extracelular.

Etiología-Epidemiología-Fisiopatología Sus causas pueden ser múltiples; dentro de las más frecuentes encontramos la pérdida de H+ en tracto digestivo y orina (vómitos y uso de diuréticos). Una causa que no puede pasar inadvertida constituye la hipokalemia. Otras etilologías incluyen hiperaldosteronismo primario y secundario a la pérdida de volumen, pérdidas por sudor, admi-

1 Facultad de Medicina, Universidad de Chile

lica se define por: 1. BE positivo. 2. HCO3- estándar aumentado. 3. Alcalemia. Valores de BE > 2 mEq/Lt, [HCO3-]st > 25 mEq/Lt y pH > 7,45 indican alcalosis metabólica. La presencia de un anion gap>18 mEq/Lt junto con [HCO3-] plasmática < 24 mEq/Lt o la evaluación de una pCO2 no coincidente con la predicha orientan hacia la presencia de un trastorno mixto.

Tratamiento 1. Corrección de hipovolemia con NaCl o KCl en pacientes hipokalémicos. 2. Tratamiento de la etiología de base (antagonistas H2 en caso de vómitos). 3. Inhibidores de anhidrasa carbónica (acetazolamida) o administración de HCl e/v en pacientes edematosos.

Seguimiento Según etiología.

Autor / Editor Nevelline Salgado

Año 2016

2 Facultad de Medicina, Universidad de Chile

Diagnóstico

Anemia en Nefrópata

La anemia observada con ERC se diagnostica en gran medida mediante la exclusión de causas no renales de anemia. En un paciente con enfermedad renal crónica se debe descartar primero causas hemorrágicas, hemolíticas, y de malnutrición que puedan provocar anemias. Por tanto, la evaluación de los pacientes debe incluir índices de glóbulos rojos, recuento de reticulocitos absoluto, hierro sérico, capacidad total de fijación del hierro, porcentaje de saturación de transferrina, ferritina sérica, recuento de glóbulos blancos, recuento diferencial de plaquetas, B12 y concentraciones de folato si la media corpuscular volumen (VCM) esta aumentada, y la prueba de sangre oculta en heces.

Nivel de manejo del médico general: Diagnóstico Sospecha Tratamiento Inicial Seguimiento Derivar

Aspectos Esenciales • • • • •

Anemia normo-normo aparece en etapa IV o V en el contexto de enfermedad renal crónica. Causa: déficit de EPO Aumenta morbilidad cardiovascular Evaluar hierro, B12 y folato; antes de iniciar tratamiento con EPO, Objetivo: hematocrito [30-34]

Es altamente sugerente una anemia normo-normo con índice reticulocitario bajo en un paciente en etapa IV o superior.

Tratamiento

Caso Clínico Tipo

El pilar de tratamiento se basa en la terapia de reemplazo con EPO humana recombinante (epoetina alfa). Más recientemente se ha introducido la darbopoetina alfa con mayor actividad biológica y vida media prolongada. Se debe recordar que la eritropoyetina tiene 3 efectos colaterales: HTA, trombosis y se piensa favorece el crecimiento de células tumorales.

Paciente hombre de 64 años con antecedentes de DM2, con retinopatía y enfermedad renal crónica etapa 4 (muy probablemente a causa de su diabetes) se presenta con anemia resistente al tratamiento con EPO refiere además dolor lumbar. Tiene una PTH 300pg/ml.

El objetivo de hematocrito es aproximadamente 30-34 ya que la mortalidad aumenta si éste es mayor a 36 o menor a 30. Además se debe evaluar siempre la cinética de hierro del paciente porque en los pacientes con ERC suelen ser necesarios los suplementos con hierro para lograr una respuesta adecuada a la EPO, junto con vitamina B12 y folato.

Definición Anemia propia de los enfermos renales crónicos, generalmente de tipo normocítica y normocrómica. La anemia es una característica común en muchos pacientes con enfermedad renal crónica que aún no requieren diálisis, más comunes cuando la velocidad de filtración glomerular disminuye por debajo de 60 ml/min, y por sobre todo, entre los diabéticos.

Seguimiento Etiología-Epidemiología-Fisiopatología

Si la anemia es resistente a la terapia establecida sugiere diálisis inadecuada; hiperparatiroidismo incontrolado; intoxicación con aluminio; hemorragia crónica o hemólisis; y malnutrición, infección crónica o tumor maligno asociado. Preocuparse además de las complicaciones por un tratamiento excesivo con hierro: hemosiderosis, aterosclerosis acelerada.

Aparece en la etapa IV de la enfermedad renal crónica y se vuelve casi universal al avanzar a etapa V. Si no se le procura tratamiento impacta en un peor pronóstico para el paciente, principalmente de orden cardiovascular (aumento del gasto cardíaco y sus complicaciones). En su patogenia intervienen varios mecanismos siendo su causa primaria la producción deficiente de eritropoyetina (EPO) por los riñones enfermos. Además, suelen haber causas concomitantes en el proceso donde podemos encontrar: • • • • • •

Resistencia de los tejidos hematopoyéticos a la hormona (principalmente por el hiperparatiroidismo). Menor sobrevida del eritrocito (por toxinas urémicas). Déficit de hierro (real y funcional). Pérdidas por sangrado. Desnutrición crónica Incluso hemodilución si están muy hipervolémicos.

Autor / Editor Nevelline Salgado

3 Facultad de Medicina, Universidad de Chile

Año 2016

Hipertensión Arterial Esencial

mmHg se realiza diagnóstico. La HTA debe ser clasificada por sus implicancias pronósticas y terapeúticas, entre otras; a futuro.

Nivel de manejo del médico general: Diagnóstico Específico Tratamiento Completo Seguimiento Completo

Aspectos Esenciales • • •

Enfermedad asintomática. Factor de riesgo cardiovascular muy importante. La progresión se mide principalmente por daño en órgano blanco.

Clasificación de la Sociedad Europea de Presión Arterial (PA mmHg)

Tratamiento

Caso Clínico Tipo

a) Tratamiento no farmacológico: • Reducción ingesta de Sal: bajo 6 g/día (una tapa de lápiz bic) • Consumo de Alcohol: La ingesta excesiva de alcohol (más de 21 unidades y 14 unidades de alcohol/semana en varones y mujeres respectivamente) se asocia a una elevación de la PA y a una menor salud cardiovascular y hepática • Restricción consumo de café: máximo 4 tazas al día • Suspender tabaquismo: como medida de prevención de riesgo cardiovascular, más que por disminución de nivel de presión arterial • Terapias de relajación • Ejercicio.

Paciente asintómatico que en controles de salud (EMPA, EFAM) o consulta de morbilidad se sospecha de HTA por cifras elevadas. También pueden tener síntomas inespecíficos: Cefalea, tinnitus, mareos.

Definición Alza y mantención de las cifras de presión arterial sobre valores considerados “normales”, sin una causa subyacente que sea culpable directa de esta condición.

Etiología-Epidemiología-Fisiopatología

b) Tratamiento farmacológico: Indicado desde el diagnóstico en paciente con PA ≥ 160/100 mmHg y aquellas con RCV alto o muy alto y diabéticos con PA ≥ 140/90. Hipertensos con RCV bajo, es posible iniciar cambios en estilo de vida y esperar 3 meses antes de iniciar el tratamiento farmacológico.

La HTA es una enfermedad silenciosa que da síntomas sólo frente a larga progresión. La HTA es el principal factor de riesgo para enfermedad cerebrovascular (ECV) y enfermedad coronaria, siendo la patología cardiovascular la primera causa de muerte en nuestro país. Según la ENS 2010 la prevalencia de HTA es de 26,9%. Los niveles elevados de presión arterial producen cambios estructurales en el sistema arterial que afectan órganos nobles tales como cerebro, corazón y riñón, determinando las principales complicaciones de esta enfermedad que en orden de frecuencia son: ECV, enfermedad coronaria, insuficiencia cardíaca, insuficiencia renal y ateromatosis periférica. Es más frecuente en hombres (28,7% vs 25,3%). La HTA primaria o esencial es aquella en la cual el mecanismo inicial del proceso se desconoce.

La monoterapia es efectiva en alcanzar las metas terapéuticas en un 20-30% de los casos, principalmente en aquellos hipertensos en etapa 1. La terapia combinada tendría sentido en hipertensos con PA > 160/100 mmHg y RCV alto, muy alto, en quienes se recomienda intervenciones más intensas, con metas terapéuticas más exigentes, lográndolas en menos tiempo. Elección del fármaco: 1. Pacientes hipertensos con RCV promedio o bajo y sin comorbilidad: En los menores de 55 años se describe una mejor respuesta terapéutica con fármacos que inhiben la activación del eje renina-angiotensina y betabloqueadores. Los mayores de 55 años se benefician más de calcioantagonistas o diuréticos tiazídicos. Estos pacientes además son más sensibles a la sal, y tienen mayor rigidez arterial. Si luego de tres meses no se logra meta se agrega un

Diagnóstico Se realiza mediante el perfil de PA, realizando al menos dos mediciones de presión arterial en cada brazo, separados al menos por 30 segundos, en días distintos y en un lapso no mayor a 15 días. Si los valores difieren en más de 5 mmHg, se debe tomar lecturas adicionales hasta estabilizar los valores. Si los valores obtenidos promedian mayor o igual a 140/90

4 Facultad de Medicina, Universidad de Chile

fármaco con mecanismos de acción diferentes y complementarios: adicionar IECA a un calcioantagonista o una tiazida en dosis bajas o viceversa en pacientes jóvenes (ver tabla) Si pese a los anterior no se logra meta

se recomienda la combinación de IECA (o ARA-II) más un calcioantagonista y diurético tiazídico. Si con terapia de 3 fármacos no se logra control de HTA se derivará a especialista.

*La combinación de B y D se asocia a mayor incidencia de diabetes en comparación con otra combinación de fármacos.

2. Hipertensos con RCV alto o muy alto, DOB, condiciones clínicas asociadas o comorbilidad, hay recomendaciones

de fármacos específicas por su efectividad en el tratamiento.(ver tablas a continuación).

5 Facultad de Medicina, Universidad de Chile

Seguimiento Las metas terapéuticas propuestas son valores menores a 140/90 mmHg. En pacientes con antecedentes de IAM , ACV, nefropatía o diabéticos, la meta podría ser de 130/80.

Autor / Editor Nevelline Salgado

Año 2016

6 Facultad de Medicina, Universidad de Chile

Hipertensión Arterial Secundaria

glomerulopatías, poliquistosis, entre otras), endocrinológicas (Hiperaldosteronismo, Feocromocitoma, Síndrome de Cushing, Hipo/Hipertiroidismo, hipercalcemia, acromegalia, hiperparatiroidismo), coartación aórtica, SAHOS y diversos fármacos (agonistas adrenérgicos, glucocorticoides, antidepresivos tricíclicos, entre muchos otros).

Nivel de manejo del médico general: Diagnóstico Sospecha Tratamiento Inicial Seguimiento Derivar

Diagnóstico Aspectos Esenciales • • • •

Debe sospecharse HTA secundaria en pacientes con HTA iniciada antes de los 20 o después de los 50 años, PA>180/110mmHg, con presencia de lesiones de órgano blanco (retina, riñón, corazón), soplos abdominales, hipokalemia, signos de enfermedad endocrinológica, HTA de difícil manejo, alzas bruscas de la PA con buen control previo y asimetría del tamaño y/o función renal. La sospecha es más alta (sobre todo para HTA renovascular) en aquellos casos de agravamiento del cuadro hipertensivo en pacientes con buen control previo y en jóvenes que presenten caída de la PAS>50mmHg luego de iniciar tratamiento con iECA. Los exámenes complementarios sólo deben realizarse en presencia de los criterios anteriormente mencionados. Entre las opciones para estudio de la HTA renovascular están el Eco-Doppler, Renograma isotópico con Captopril, AngioTAC/RNM y la arteriografía (Gold standard).

La causa más frecuente de HTA secundaria es la Renovascular. Las causas más frecuentes de HTA renovascular son la Aterosclerosis y la Displasia Fibromuscular. Sospechar en pacientes con HTA refractaria o agravamiento de función renal al tratar con iECA. El único tratamiento definitivo es la revascularización de la arteria renal.

Caso Clínico Tipo Paciente hombre de 25 años, sin antecedentes, consulta por cuadro de HTA que no responde al tratamiento farmacológico. ¿Cuál de las siguientes situaciones sugeriría HTA renovascular en este caso? • Hipertrofia ventricular izquierda en el EKG • Hipokalemia persistente • Hematuria glomerular • HTA de larga data • Insuficiencia renal aguda

Tratamiento El tratamiento debe estar enfocado a la causa del cuadro hipertensivo. Para el tratamiento de la HTA renovascular existen 2 opciones: el tratamiento farmacológico (sólo si la estenosis es <60%, HTA controlada o revascularización imposible) y la revascularización mediante angioplastia intraluminal percutánea (con colocación de stent ya que tiende a reestenosarse en un 35% de los casos) o quirúrgica.

Definición La hipertensión arterial secundaria, no-esencial o inesencial corresponde al conjunto de cuadros de hipertensión arterial que poseen una causa identificable, siendo esta más frecuentemente renal (la cual debe diferenciarse de aquella esencial que cursa con afectación renal secundaria) o endocrina.

Seguimiento Tanto el diagnóstico definitivo, tratamiento y seguimiento debe ser indicado por el especialista.

Etiología-Epidemiología-Fisiopatología La etiología más frecuente es el hiperaldosteronismo primario.La causa nefrológica más frecuente es la de origen vascular renal (renovascular o estenosis de la arteria renal), la que da cuenta de aprox. 1% de todos los cuadros de HTA. Es causada más frecuentemente por 2 cuadros característicos: Aterosclerosis (típicamente en hombres, >50 años, asociada a lesiones ateroscleróticas en coronarias, carótidas y aorta), y Displasia Fibromuscular (más frecuente en mujeres jóvenes, terminando en insuficiencia renal en 1/3 de los casos). Su fisiopatología viene dada por la hipoperfusión del riñón afectado, con secreción de renina y activación del eje Renina-Angiotensina-Aldosterona. Otras causas frecuentes de HTA secundaria son las renales parenquimatosas (infartos renales, pielonefritis crónica,

Autor / Editor Hernán Rubilar

7 Facultad de Medicina, Universidad de Chile

Año 2016

Insuficiencia Renal Crónica

cuales se asocian a diálisis o necesidad de trasplante), estamos en el número 3°, lo cual implica un gran costo asociado de esta patología GES al país y, además, se presenta alta asociación con el desarrollo de enfermedades cardiovasculares.

Nivel de manejo del médico general: Diagnóstico Sospecha Tratamiento Inicial Seguimiento Completo

Etiología: Actualmente en Chile (y en Latinoamérica) la principal causa de esta patología en la población adulta corresponde a la Diabetes Mellitus (DM), seguido por la Hipertensión Arterial (HTA) en una segunda instancia. Vista grosso modo es una patología multifactorial donde además de las 2 enfermedades mencionadas anteriormente se encuentran como causas más comunes, también destacan las glomerulopatías, alteraciones intersticiales, enfermedad poli quística renal, causas congénitas, fármacos, mieloma o progresión de una lesión renal aguda.

Aspectos Esenciales • • • •

Enfermedad importante dentro de la población Chilena con alto costo asociado. Presencia de alta asociación con diabetes mellitus (> 10 años) e HTA, por lo que debe tenerse como sospecha. Requiere un alto manejo cardiovascular asociado. Etapas tempranas (1, 2 y 3) son de manejo en APS y las avanzadas (4 y 5) por nefrólogo.

Fisiopatología: El punto de convergencia de las etiologías anteriores se base en 3 pilares: 1. Pérdida progresiva de nefronas. 2. Adaptación funcional de las nefronas remanentes. 3. Repercusión de éstos trastornos sobre el resto de aparatos y sistemas del cuerpo humano.

Caso Clínico Tipo Paciente de sexo femenino de 72 años, con antecedentes de DM2 (Insulino requirente hace 4 años) e HTA (Diagnosticada hace 10 años), acude a control cardiovascular en APS. Paciente dice que se ha sentido un poco más cansada que lo normal y que en su casa no ha logrado controlar bien el azúcar. Se le piden exámenes de sangre donde destaca creatinina de 1,77 mg/dl y pruebas de orina que muestran relación albuminuria creatininuria de 0.4.

Las distintas enfermedades ayudan de alguna u otra manera a desencadenar el círculo vicioso, donde encontramos que se genera un estrado proinflamatorio con presencia de distintas interleuquinas, y además se asocia a secreción de distintas hormonas con efectos pleiotrópicos en otros sistemas como la angiontensina II; favoreciendo la formación de procesos de ateromatosis de manera sistémica.

Definición

Dentro de las complicaciones de la ERC encontramos difícil manejo ácido – base e hidrosalino, alteraciones osteometabólicas, anemia e infecciones. Estas patologías se evaluarán de manera específica en otros capítulos.

Corresponde a cualquier anormalidad estructural o funcional renal (independiente de su origen) por un periodo de 3 meses o más, con una velocidad de filtración normal o disminuida y que se puede evidenciar en estudios imagenelógicos, histológicos o con alteración persistente en los marcadores de daño renal en sangre u orina.

Clasificación: la ERC la podemos clasificar según una estimación de la VFG, lo cual determinará tanto nuestros objetivos como nuestras metas. Normalmente la estimación se hace en base al clearence de creatinina de manera que siempre debemos pedir creatinina plasmática y urinaria de los pacientes.

Etiología-epidemiología-fisiopatología Epidemiología: Chile a nivel mundial se encuentra en el 18 lugar de incidencia y 8vo lugar en prevalencia de la ERC. Pero si analizamos en niveles terminales de la enfermedad (los

8 Facultad de Medicina, Universidad de Chile

La progresión de la enfermedad es definida de manera arbitraria como una pérdida arbitraria > 5 mL / min de VFG en un año donde encontramos que afectan de manera significativa la hipertensión, episodios de insuficiencia renal aguda y proteinuria (/albuminuria). Otros factores a destacar corres-

ponden edad avanzada, raza negra, diabetes mellitus, sexo masculino, hipoalbuminemia, dislipidemia, sobrepeso, altos niveles de hemoglobina glicosilada, hiperfosfatemia, anemia, tabaquismo entre otros.

9 Facultad de Medicina, Universidad de Chile

Diagnóstico En la ERC existen distintos manera de aproximarse según la etiología de la enfermedad, pero como se dijo en la definición previa, debemos encontrar alteraciones funcional estructurales o funcionales para hablar de ésta enfermedad. De manera que la historia nos guiará, pero lo que deberemos buscar dirigídamente son alteraciones de la VFG, imágenes,

laboratorio (orina y sangre) y, como uno de los últimos recursos, histológicos. • Anamnesis y examen físico: Los síntomas inician en la etapa 3 de la enfermedad, y corresponden a distintas manifestaciones no específicas de la enfermedad y/o sus complicaciones, de manera que es necesario hacer una historia completa y detallada del paciente:

Al examen físico siempre se debe evaluar talla – peso, presión arterial, presencia de arritmias, malformaciones del desarrollo, problemas cardiovasculares y fondo de ojo. • Ecografía: Siempre se debe pedir en paciente con sospecha de ERC; primeramente, para evaluar la existencia de 2 riñones, analizar su morfología y descartar procesos obstructivos. Además, ciertas alteraciones ecográficas nos hacen sospechar ERC tales como tamaño renal pequeño (<9 cm, según la superficie corporal), el adelgazamiento de la cortical renal y una asimetría entre los riñones mayores a 2 cm. No se debe olvidar que existen distintas etiologías que pueden a generar ERC con tamaño de los riñones aumentado, tales como la Diabetes Mellitus, enfermedades por depósito, presencia de una infección concomitante, obstrucción a posterior y masas (tanto tumores como la enfermedad renal poliquística). • Laboratorio: Es importante evaluar los distintos aspectos

de esta enfermedad de manera que es indispensable una evaluación completa del estado del paciente (pruebas de glucosa, hemograma, perfil lipídico, electrolitos, entre otros). Dentro del laboratorio a nivel diagnóstico destacan 2 exámenes: 1. Creatinina: tanto plasmática, como en orina, de manera que permite determinar el clearence de ésta y por ende evaluar la etapa de ERC con la que nos vemos enfrentados. Es indispensable usar la creatinina con una estimación de la VFG y no usarla de manera aislada. Dentro de los estimadores más utilizados encontramos la MDRD y la EPI – CKD. 2. Albuminuria / Proteinuria: ya sea en 24 horas o en base a la relación proteinuria- creatininuria (albuminuria – creatininuria); su importancia recae como factor pronóstico propiamente tal o como marcador de nefropatía diabética como factor etiológico y pronóstico; de manera que cambiará, en parte, la terapéutica a seguir.

10 Facultad de Medicina, Universidad de Chile

Tratamiento y Seguimiento En el caso de esta enfermedad se hace un manejo general en base a la patología, un manejo específico según la etapa en que se encuentre y un manejo etiológico según corresponda. • Manejo General: La enfermedad presenta metas específicas dentro del control de presión arterial, microalbuminuria, proteinuria, hemoglobina glicosilada, colesterol LDL y de cambios de hábitos como dejar de fumar y realización de actividad física (ejercicio aeróbico). Además, se usan medidas como controlar el peso (bajar un 10% o más en caso de IMC > 25), uso de aspirina en 75 – 100 mg si el riesgo cardiovascular a 10 años es > 20%, evitar AINEs y otros nefrotóxicos y vacunación contra la inluenza y neumococo.

• • • •

de las complicaciones urémicas queun paciente puede presentar, que de la creatinina plasmática, de este modo es que muchos nefrólogos prefieren iniciar diálisis precozmente para evitar complicaciones más serias de la uremia. Indicaciones absolutas de de ingreso a diálisis corresponden a: - Hiperkalemia no controlada. - Acidosis metabólica. - Síntomas gastrointestinales y neurológicos. - Hipervolemia.

Intervenciones específicas ERC:

En el caso de presentar hipertensión arterial y/o proteinuria, los fármacos de primera elección debiesen ser IECA / ARA 2 hasta el estadio 3. En el caso de manejo metabólico destaca que en caso de HTA se debe disminuir la ingesta de sodio, en oligoanuria la de potasio y de manera global se sugiere aumentar el consumo de proteínas dado la disminución espontanea del consumo que estos pacientes presenta. • Manejo específico según etapa de ERC: 1. ERC etapas 1 – 3: la mayoría de estos pacientes no progresará a fase terminal de ERC por lo que el énfasis debe ponerse en reducir el riesgo cardiovascular. Estos pacientes pueden ser manejados en APS y en algunos casos se hará derivación a nefrólogo. ERC 1 y 2: Control anual con VFG estimado, examen de orina completo y relación albuminuria – creatininuria, deben tener un control meticuloso de la presión arterial. ERC 3: Control semestral con VFG estimado, examen de orina completo, REAC y además se añaden control de calcemia, fosfatemia y PTH. Debe tener un control meticuloso de presión arterial. 2. Etapas 4 – 5: Derivar al nefrólogo y en caso de ser etapa 5, debe realizarse con urgencia. En este nivel se hace una evaluación de la dieta, corrección de la acidosis, optimización de calcemia, fosfatemia y PTH. Se inmunizará al paciente para la hepatitis B, se preparará acceso vascular; y se informará y discutirá sobre alternativas de reemplazo de función renal. • Inicio de terapia de remplazo renal: La decisión de iniciar terapia de diálisis depende más de la severidad

Cuando derivar al nefrólogo:

11 Facultad de Medicina, Universidad de Chile

Manejo de las personas diabéticas y no diabéticas según etapa de la ERC.

* En ausencia de otras enfermedades sistémicas. ** Factores de Riesgo: Tabaco, hipertensión arterial y dislipidemia *** Controlar K plasmático a la semana de inicio de tratamiento con IECA y ARA II

Autor / Editor Hernán Rubilar

Año 2016

12 Facultad de Medicina, Universidad de Chile

Nefropatía Diabética

tomía de ser necesario (Resistencia a tratamiento). Hay que tener en mente que una mala terapéutica, puede producir Enfermedad Ósea Adinámica (Por exceso de Calcio y Vitamina D).

Nivel de manejo del médico general: Diagnóstico Sospecha Tratamiento Inicial Seguimiento Derivar

Seguimiento Derivar.

Aspectos Esenciales • • • • •

Causada por progresión de ERC. Todo afectado la padecerá a lo largo de su evolución. Es importante buscar dolor óseo, deformidades esqueléticas, esguinces y fracturas. Importante preguntar tratamiento actual. El tratamiento puede requerir cirugía.

Autor / Editor Hernán Rubilar

Definición Alteraciones Osteometabólicas derivadas de la Enfermedad Renal Crónica.

Etiología-Epidemiología-Fisiopatología Se debe a la progresión de la Enfermedad Renal Crónica. La ERC está aumentando su prevalencia, y todo afectado padecerá de Osteodistrofia por esta causa. La enfermedad Renal Crónica produce un deficit de vitamina D activa (1,25 dihidroxicolecalciferol) y un hiperparatiroidismo secundario, el cual produce alteraciones en la homeostasis del Ca y P y alteraciones osteometabólicas.

Diagnóstico •

• •

Anamnesis: Edad, sexo, sintomatología de la ERC, dolores óseos, prurito, miopatía proximal, antecedentes personales (Enfermedad renal crónica, diabetes mellitus, hipertensión arterial, entre otras), y familiares (Antecedentes mórbidos de importancia). Es importante buscar los antecedentes de tratamiento para la ERC (tratamiento dietario, farmacológico, diálisis, entre otros). Ex. Físico: Hay que evaluar signos de ERC, y buscar: deformaciones esqueléticas, ruptura de tendones, fracturas espontáneas, entre otras. Ex. Complementarios: Exámenes generales, medición de Calcio, Fósforo y PTH, radiografía e histología.

Tratamiento Tratamiento debe considerar principalmente: dietas hipofosfémicas, quelantes del fósforo (con calcio o resinas sin calcio. Nunca hidróxido de aluminio), Calcitriol Oral o ev (Cuando PTH sobre 250pg/mL. Se debe vigilar Ca y P), y Paratiroidec-

13 Facultad de Medicina, Universidad de Chile

Año 2016

Pielonefritis Aguda No Complicada

ma de los riñones. La vía hematógena es más rara y suele observarse en la bacteremia por S. Aureus o en la fungemia por Candida. Además existen determinantes que posibilitan una ITU alta, como son: virulencia del patógeno, tamaño del inóculo y estado inmunitario del huésped.

Nivel de manejo del médico general: Diagnóstico Específico Tratamiento Completo Seguimiento Completo

Diagnóstico

Aspectos Esenciales • •

• • •

El diagnóstico es Clínico y de Laboratorio. a) Clínico: Por lo general los síntomas se desarrollan con rapidez, en horas o un día y comprende fiebre, escalofríos, náuseas y vómitos. Además puede acompañarse de síntomas de cistitis. El examen físico se acompaña de taquicardia, dolor muscular generalizado. Es notable el dolor a la presión en una o ambas fosas lumbares o a la palpación abdominal profunda. b) Laboratorio: Cilindros Leucocitarios, Piuria, Bacteriuria, Urocultivo positivo (se aceptan como significativas bacteriurias de 100 UFC/ml de un germen uropatógeno), Leucitosis con desviación izquierda, Proteína C elevada, VHS elevada, Bacteremia (30%).

La mayor parte de ITU alta es por E.coli uropatógena muy virulenta Diagnóstico es Clínico (Fiebre, dolor fosa lumbar y puño-percusión positiva, escalofríos, vómitos) y de Laboratorio (Cilindros leucocitarios, leucocitosis notable, bacteriuria, etc.) Urocultivo siempre. Hemocultivos en los que se hospitalizan (15-20% positivos) Tratamiento ATB por 2 semanas y realización de Urocultivo e imágenes en 1-2 semanas post- tratamiento solo bajo determinadas circunstancias. Resolución clínica esperable en 48-72 horas.

Caso Clínico Tipo

Tratamiento

Paciente de 47 años, sin antecedentes mórbidos, consulta por cuadro de 3 días de evolución, caracterizado por disuria, poliaquiuria, tenesmo vesical, asociado a dolor en hipogastrio y flanco derecho. Al examen físico se constata buen estado general, bien hidratada y puño percusión positiva a derecha. Temperatura axilar de 38,5 °C, presión arterial de 130/90, polipneica y taquicardia.

Su objetivo es la erradicación del germen y del reservorio. 1. Al menos los primeros días los antibiótico se administran vía EV (lo más económico es gentamicina o amikacina en una dosis por día), según estado del paciente, aunque los más leves pueden empezar de inmediato con ATB vía oral. Para esto se pueden utilizar Fluoroquinolonas (habitualmente ciproflofloxacino), Aminoglicósidos (enterococo, bacilos gran negativos), Cefalosporinas (2da y 3era generación), Carboxipenicilamina. 2. El tratamiento se mantiene por 48-72 horas EV, luego se continúa por vía oral (según antibiograma), por 10-14 días. Por lo tanto SIEMPRE debe realizarse Urocultivo.

Definición Inflamación del riñón y de la pelvis renal como resultado de la invasión bacteriana. Puede clasificarse anatómicamente como ITU alta (infección de la porción superior de la vía urinaria).

Los Hemocultivos sólo en pacientes que se hospitalizan. Se espera resolución clínica en 48-72 hrs. No utilizar nitrofurantoína, ácido pipemídico, ácido nalidíxico o fosfomicina.

Etiología-Epidemiología-Fisiopatología

En el caso de paciente embarazada Cefuroxime 500 mg c/12 vía oral o Ceftriaxona 1 gr c/12 o c/24 hrs ev.

Como la vía ascendente es la vía común de infección del aparato urinario, el germen causal que se encuentra con más frecuencia es ‘Escherichia coli uropatógeno tanto en ambiente ambulatorio (80-90%) como hospitalario (50%). Otros gérmenes frecuentes son otras especies de gram negativos, como Proteus y Klebsiella, y también Enterococcus. La incidencia en mujeres de 18 a 49 años que no están hospitalizadas es de 28 casos por 10000 de ellas.

Seguimiento •

Las principales rutas de infección son la ascendente y la hematógena. La mayoría se produce por ruta ascendente. La infección del parénquima renal se produce debido a la multiplicación de los gérmenes en la vejiga y su ascenso por el uréter y la pelvis renal, y la colonización final en el parénqui-

No es de rutina la realización de un urocultivo de control en estos pacientes si hubo urocultivo positivo con antibióticos sensible indicado.

Debe realizarse Urocultivo 1-2 semanas luego del tratamiento ATB. La realización de estudio por imágenes (Ecografía o TAC) en: - No respuesta al 4º día - Clínica atípica (hematuria prolongada, dolor cólico) - ITU en el ambiente pediátrico.

14 Facultad de Medicina, Universidad de Chile

Autor / Editor Matilde Pedrero

15 Facultad de Medicina, Universidad de Chile

Año 2016

Síndrome Nefrítico

ENTRE 15 Y 50 AÑOS: • Glomerulopatía por IgA (Enfermedad de Berger). • Glomerulopatía membranoproliferativa. • Glomérulonefritis lúpica (en la mujer). • Glomerulonefritis aguda post infecciosa.

Nivel de manejo del médico general: Diagnóstico Específico Tratamiento Inicial Seguimiento Derivar

MAYORES DE 50 AÑOS: • Vasculitis • Idiopática • Poliarteritis microscópica • Sd. Goodpasture • Enfermedad de Wegener

Aspectos Esenciales • • • •

Síndrome caracterizado por hematuria, proteinuria, y ↓ VFG (edema, HTA y oliguria). Orientar las probabilidades etiológicas según la edad del paciente. Causa más frecuente postinfecciosa. Buscar enfermedad sistémica (ASO, C3, ANA, anti DNAn, ANCA).

Diagnóstico El diagnóstico clínico se realiza frente a un paciente que presenta: • HEMATURIA de origen glomerular (GR dismórficos, cilindros hemáticos), micro o macroscópica (75% de los casos); • PROTEINURIA en rango no nefrótico (< 3,5 g/24 hr), • EDEMA inicialmente facial luego generalizado e hipertensión venosa yugular. • HTA variable y • ↓ VFG evidenciada por una retención nitrogenada transitoria, con alzas de creatinina hasta 1,5-2,0 mg/dL.

Caso Clínico Tipo Paciente de 25 años con antecedente de faringitis hace dos semanas, consulta por presentar hace 1 día hematuria. Al examen físico destaca edema peripalpebral e HTA. Creatinina sérica 1,4 mg/dL, proteinuria 1,8 g/24 hr. Dg: Sd. Nefrítico (Glomerulonefritis aguda post- infecciosa).

Enfoque Diagnóstico: Una vez confirmada la existencia de un SNA el paso siguiente es averiguar su etiología: 1. Investigar una glomerulonefritis aguda post infecciosa. 2. Buscar enfermedad sistémica (Lab. Inmunológico: ASO, C3, ANA, anti DNAn, ANCA es de gran utilidad) **Se puede ver hipocomplementemia en glomerulonefritis postinfecciosa, lúpica y membranoproliferativa.** 3. Orientar las probabilidades diagnósticas según edad 4. Biopsia renal cuando el diagnóstico clínico no sea obvio o la evolución sea atípica, siempre que no signifique un riesgo mayor.

Definición El Síndrome nefrítico agudo (SNA) es una de las formas de expresión clínica de diversos patrones histológicos de daño glomerular, en respuesta a variadas agresiones renales, que se caracteriza por la aparición en pocos días de: hematuria, proteinuria, y reducción de la velocidad de filtración glomerular (VFG), traduciéndose esta última en edema, oliguria e HTA.

Etiología-Epidemiología-Fisiopatología

Tratamiento

Las causas del SNA (Tabla 1) son agresiones glomerulares por diversos mecanismos inmunológicos. Ocurre un proceso inflamatorio agudo y difuso de todos los glomérulos que determina: a) Hematuria y proteinuria secundarias al aumento de la per meabilidad del capilar glomerular y rupturas de la membrana basal por la glomerulitis. b) Reducción de la superficie filtrante con caída de VFG, que determina oliguria y balance positivo de sodio respons ble de edema e HTA.

Según etiología y sintomático de la sobrecarga hidrosalina y de la insuficiencia renal. En la Glomerulonefritis aguda o proliferativa difusa, el tratamiento es sintomático: reposo, restricción de sodio y el uso juicioso de diuréticos y antihipertensivos. Se prescribe antibióticos sólo si coexiste infección activa. En las glomerulopatías por depósito de IgA no existe un tratamiento específico, en formas severas con insuficiencia renal o sospecha de isquemia o de perforación intestinal: altas dosis de corticoides e inmunosupresión con ciclofosfamida.

MENORES DE 15 AÑOS: • Glomérulonefritis aguda post estreptocócica. • Síndrome hemolítico-urémico. • Púrpura de Schönlein-Henoch.

Seguimiento Su pronóstico es variable, desde la curación como en la glomerulonefritis aguda post estreptocócica a la cronicidad

16 Facultad de Medicina, Universidad de Chile

en otras como la Glomerulonefritis membranoproliferativa (GNMP) o la Nefropatía por IgA. Ocasionalmente las glomerulonefritis rápidamente progresivas pueden debutar como un síndrome nefrítico agudo, con un muy mal pronóstico de no mediar tratamiento. Se deberá derivar los casos al especialista.

Autor / Editor Nicole Fritzche

Año 2016

17 Facultad de Medicina, Universidad de Chile

Síndrome Nefrósico (Nefrótico)

estado de hipercoagulabilidad. La hiperlipidemia se produce por aumento de la síntesis hepática de lipoproteínas (en respuesta a la hipoproteinemia) y por una disminución del clearance de colesterol.

Nivel de manejo del médico general: Diagnóstico Específico Tratamiento Inicial Seguimiento Derivar

MENORES DE 15 AÑOS: • Enfermedad por cambios mínimos (80% entre los 2-10 años). • Glomeruloesclerosis focal y segmentaria (> frecuente en adolescentes).

Aspectos Esenciales • • • • •

Los hechos cardinales son: proteinuria masiva, hipoalbuminemia, edema, hiperlipidemia, lipiduria. Orientar etiología según edad. Causa más frecuente en niños (enfermedad de cambios mínimos), en adultos (GN membranosa idiopática según biopsia y Diabética sin biopsia). Exámenes útiles: C3, ANA, anti DNAn, AgHBs, anti VHC y anti VIH. Proteinuria se trata con IECA. CUIDADO con diuréticos en edema, puede causar IR prerrenal e incluso NTA.

ENTRE 15 Y 40 AÑOS: • Glomerulopatía membranoproliferativa. • Glomeruloesclerosis focal y segmentaría. • Glomerulopatía membranosa (primaria o secundaria). • Nefrosis lipoídica. • Nefropatía diabética (primera causa adulto). • Pre-eclampsia MAYORES DE 40 AÑOS: • Glomerulopatía membranosa (secundaria y primaria). • Glomeruloesclerosis diabética. • Amiloidosis primaría.

Caso Clínico Tipo Diagnóstico

Mujer de 58 años, con antecedente de DM II e HTA en tratamiento hace 10 años. Presenta hace 4 días edema palpebral y edema en ambas piernas con orinas espumosas. Creatinina 0,8 mg/dL, BUN 16mg/dL, Albumina plasmática 2,4 mg/dL, Colesterol total 320mg/dL. Sedimento urinario: cilindros hialinos y gotas de grasa. Proteinuria 8,5 g/24 h.

El diagnóstico requiere de apoyo directo de exámenes de laboratorio dado que los hechos clínicos cardinales para su diagnóstico son: proteinuria masiva, hipoalbuminemia, edema, hiperlipidemia, lipiduria. Los siguientes elementos deben tomarse en cuenta para una aproximación etiológica en un paciente con síndrome nefrótico: 1. Edad de aparición; 2. Determinar si el síndrome nefrótico es “puro” o “impuro” (SN asociado a HTA, IR o hematuria); 3. Investigar la presencia de una enfermedad sistémica; 4. Averiguar antecedentes de uso de drogas productoras de SN.

Definición Es un estado clínico producido por glomerulopatías de múltiples etiologías, con lesiones histológicas diversas, caracterizado por la presencia de edema, proteinuria masiva, hipoalbuminemia, hiperlipidemia y lipiduria. La presencia de HTA, hematuria y/o IR agregados, hacen que el diagnóstico ya no sea un Sd. Nefrótico sino que corresponderá a un Sd. nefrótico impuro, el cual presenta un manejo distinto.

Es importante buscar una enfermedad sistémica, pudiendo ser la glomerulopatía la primera manifestación de ésta (LES). Además de la clínica, son exámenes útiles: C3, ANA, anti DNAn, AgHBs, anti VHC y anti VIH. Siempre hay que ser muy acucioso en la investigación del uso de medicamentos productores de síndrome nefrótico (antiinflamatorios no esteroidales, penicilamina).

Etiología-Epidemiología-Fisiopatología Diversos mecanismos de agresión glomerular según la etiología (tabla 1) producen un aumento de la permeabilidad del capilar glomerular que genera una proteinuria masiva (>3,5 g/24 h) con hipoalbuminemia que conlleva a una reducción de la presión oncótica y por consiguiente a la producción de edema. Este cuadro es sinónimo de la presencia de una glomerulopatía.

El diagnóstico etiológico definitivo se basará en el juicioso uso de los criterios clínicos señalados, más la biopsia renal debidamente procesada, indicada en todo síndrome nefrótico del adulto cuando los datos no revelan una causa clara. En niños solo se realiza biopsia cuando el cuadro no es concordante con enfermedad de cambios mínimos (corticorresistentes, hipocomplementemia).

La hipoproteinemia a nivel hepático induce el aumento de la síntesis de proteína C, proteína S y de fibrinógeno que junto a la pérdida de AT3 por la hiperfiltración producen un

18 Facultad de Medicina, Universidad de Chile

Tratamiento Dependerá de la etiología. El tratamiento inespecífico corresponde a la restricción de la ingesta de sal y/o diuréticos: sólo ante edema severo o derrame pleural, nunca en dosis altas por riesgo de insuficiencia renal aguda o trombosis. Dieta normoproteica.

Seguimiento Derivar al especilista.

Autor / Editor Nicole Fritzche

Año 2016

19 Facultad de Medicina, Universidad de Chile

Síndrome Urémico

• •

Nivel de manejo del médico general: Diagnóstico Específico Tratamiento Inicial Seguimiento Derivar

Cardiovasculares • HTA • ICC • Cardiomiopatías • Pericarditis • Ateroesclerosis acelerada • Arritmias

Aspectos Esenciales • • • •

Hiperfosfatemia e Hipocalcemia Hipermagnesemia

Corresponde a una intoxicación por metabolitos que no se pueden eliminar. Se debe tener en consideración en pacientes con patología nefrológica de base. La clínica puede ser muy variada. El tratamiento puede requerir terapia de remplazo renal.

Gastrointestinales • Anorexia, Náuseas y Vómitos • Aliento Urémico • Estomatitis, Gastritis y Enteritis • Ulcera Péptica • Hemorragia Gastrointestinal Hematológicas e Inmunológicas • Anemia • Diátesis hemorrágica • Inhibición fagocítica • Linfocitopenia y disfunción linfocítica • Aumento de susceptibilidad a infecciones y neoplasias • Neurológicas Decaimiento • Cefaleas • Irritabilidad y alteraciones del sueño • Tremor • Asterixis • Convulsiones • Estupor y Coma • Neuropatía periférica • Piernas inquietas • Debilidad motora

Caso Clínico Tipo Paciente de sexo masculino de 50, con ERC de larga data en mal tratamiento, consulta por cuadro de días de evolución de astenia, adinamia asociado a nauseas, vómitos y disminución de peso. Al examen físico destaca fetor urémico, palidez generalizada y en algunos sectores presencia de equímosis, además de presentar piel seca y con finas escarchas.

Definición La uremia es un síndrome clínico y de laboratorio que refleja una disfunción multiorgánica debido a falta de tratamiento o subtratamiento de una falla renal aguda o crónica.

Endocrinológicas y metabólicas • Intolerancia a los Hidratos de Carbono • Hipertrigliceridemia • Desnutrición Proteica • Infertilidad, Disfunción Sexual, y Amenorrea • Osteodistrofia Renal • Trastorno del Crecimiento • Hiperparatiroidismo Secundario • Hiperuricemia

Etiología-Epidemiología-Fisiopatología Es un estado de intoxicación que afecta los sistemas cardiovascular, gastrointestinal, hematopoyético, inmune, nervioso y endocrino. Es en parte el resultado de la reducción de la función excretora renal con retención de sustancias tóxicas que alteran los mecanismos de regulación celular. Es también la consecuencia del desajuste de las funciones endocrinas y metabólicas reguladas por el riñón. Los síntomas y signos varían de un paciente a otro dependiendo de la velocidad y severidad de la pérdida de función renal.

Dermatológicas • Palidez • Hiperpigmentación • Prurito • Equimosis • Escarcha urémica

Manifestaciones Clínicas

Tratamiento

Fluido y los Electrolítico: • Expansión y contracción de volumen • Hiper e Hiponatremia • Hiper e Hipokalemia • Acidosis Metabólica

Frente a sintomatología urémica (generalmente con VFG <15 ml/min/1,73 m2) está indicada el inicio de la terapia de sustitución renal.

20 Facultad de Medicina, Universidad de Chile

Seguimiento Los casos deben ser derivados para manejo por el Nefrólogo/a.

Autor / Editor Hernán Rubilar

Año 2016

21 Facultad de Medicina, Universidad de Chile

Uropatía Obstructiva

edad del paciente, velocidad de instalación y la presencia de cólico renal pueden aproximar a la etiología. Cuando el sitio de obstrucción es bajo, cursa con retraso para iniciar la micción, disminución de la fuerza y tamaño del chorro y goteo terminal, o con retención aguda de orina. Si el sitio se encuentra sobre la vejiga, se presenta con dolor en el flanco irradiado o no a genitales, dependiendo del nivel de obstrucción. La hipertensión puede estar presente, y, entre los pacientes con obstrucción del tracto urinario inferior, se puede detectar una masa en la parte inferior del abdomen. El análisis de orina puede mostrar hematuria, pero el sedimento es generalmente benigno.

Nivel de manejo del médico general: Diagnóstico Sospecha Tratamiento Inicial Seguimiento Derivar

Aspectos Esenciales • • • •

Obstrucción del flujo urinario. Causa más frecuente en jóvenes es la litiasis. Causa más frecuente en adultos mayores es la hiperplasia prostática. Ante la sospecha realizar ecografía.

El diagnóstico se hace por la imagen, y el ultrasonido es la modalidad de elección, sirve para evaluar presencia de hidronefrosis o alguna alteración anatómica que esté produciendo la obstrucción.

Caso Clínico Tipo



Paciente de 60 años con dificultad para terminar la micción hace varios meses, que además refiere goteo terminal y una disminución del tamaño del chorro de orina.

• •

En el caso de la litiasis el examen de elección es el pieloTAC, que permite visualizar cálculos de al menos 3mm. Si se sospecha patología vesical la cistoscopia permite visualizar las paredes de la vejiga, observar sus características y tomar muestras para biopsia. Un método fácil y económico es el sondeo vesical para descartar una obstrucción baja.

Definición Tratamiento

Es la obstrucción al flujo de la orina en cualquier punto entre los cálices renales y la uretra. La obstrucción del tracto urinario es una causa fácilmente reversible de daño renal que es importante reconocer. Esta obstrucción puede ser causa de insuficiencia renal aguda o crónica, según la forma de presentación de la obstrucción. Si no se corrige, puede predisponer a la infección del tracto urinario (ITU) y sepsis urinaria, y eventualmente culminar en enfermedad renal terminal.

El tratamiento busca restablecer el flujo urinario y debe ser orientado a la causa que genera la obstrucción. Si la obstrucción es infravesical, puede restablecerse el flujo y conseguir el alivio de los síntomas mediante el sondeo vesical, pero debe buscarse y tratarse la causa de la obstrucción. •

Etiología-Epidemiología-Fisiopatología



Las causas de la obstrucción al flujo son múltiples, siendo los lugares más frecuentemente afectados la unión pieloureteral y ureterovesical, el cuello vesical y el meato uretral. La etiología más frecuente en adultos jóvenes es la presencia de cálculos, mientras en adultos mayores destaca la hiperplasia prostática, neoplasias en la vía urinaria y cálculos. La causa más frecuente de obstrucción unilateral es la litiasis, mientras que bilateral la HPB. La obstrucción genera una acumulación retrograda de la orina, produciéndose hidronefrosis unilateral si la causa de obstrucción se encuentra por sobre la vejiga o bilateral si está por debajo de ésta. La uropatía obstructiva puede desencadenar un deterioro de la función renal.



En caso de imposibilidad de instalar una sonda se debe recurrir a la instalación bajo visión directa (cistoscopía) o la cistostomía suprapúbica. Si la obstrucción es alta los estudios de imagen permiten hacer el diagnóstico etiólogico y planificar el tratamiento: litotripsia, extracción quirúrgica del cálculo o extracción tumoral. De existir compromiso de la función renal secundario a la obstrucción debe considerarse como una urgencia y resolverse a la brevedad, aunque sea una solución transitoria como instalación de pigtail o nefrostomía.

Seguimiento Por especialista.

Autor / Editor Hernán Rubilar

Diagnóstico Se debe sospechar con el cuadro clínico, que varía según el lugar donde radica y si la obstrucción es aguda o crónica. La

22 Facultad de Medicina, Universidad de Chile

Año 2016

Crisis Hipertensivas, Emergencias Hipertensivas

Etiología-Epidemiología-Fisiopatología

Nivel de manejo del médico general: Diagnóstico Específico Tratamiento Inicial Seguimiento Completo

Aspectos Esenciales •

• • •

Una urgencia hipertensiva corresponde a una hipertensión severa (PAS 180 mmHg o más, y/o PAD 120 mmHg o más) sin signos de compromiso agudo o de daño de órgano blanco. Una emergencia hipertensiva corresponde a una presión sanguínea elevada con signos de daño agudo de órgano blanco. La emergencia hipertensiva requiere tratamiento inmediato intrahospitalario. Se debe evaluar la rapidez de normalización PA de acuerdo a etiología.

Caso Clínico Tipo Al servicio de urgencia se presenta paciente de 50 años, con antecedente de HTA de larga data sin control, que refiere historia de 12 horas de evolución de cefalea intensa, náusea y visión borrosa; en la última hora se agrega dificultad respiratoria y hemiparesia fascio-braquio-crural izquierda. Al ex. físico destaca déficit neurológico mencionado, estado confusional y crépitos húmedos en ambos campos pulmonares. PA de ingreso: 195/130 mmHg.

No constituye una crisis hipertensiva la HTA con cifras tensionales elevadas, PAD >110 mmHg y <130 mmHg, sin síntomas y sin amenaza de daño a corto plazo de órganos blanco. Frecuentemente estos pacientes consultan al Servicio de Urgencia con síntomas inespecíficos, que coexisten con la HTA pero no son producidos por ella, tales como epistaxis, vértigo paroxístico benigno, cefaleas tensionales o migraña.

Definición Corresponden a situaciones clínicas provocadas por elevación brusca de la presión arterial que requieren manejo de manera rápida, eficiente y vigilada. La urgencia hipertensiva es una presión elevada por sobre PAS 180 mmHg y/o PAD 120 mmHg, sin signos de daño agudo de órgano blanco; requiere manejo rápido (días) pero no inmediato. Una emergencia hipertensiva es aquella en la que la presión arterial es tal magnitud, o aparecen en contextos tales (como por ej., embarazo), que ponen en riesgo la vida del paciente o la integridad de órganos vitales, con signos de compromiso agudo de ellos. No existe un valor de corte de presión para la emergencia hipertensiva, sino que se caracteriza por una presión elevada o alza brusca sobre el basal acompañada de signos de daño de órgano blanco. Requiere manejo inmediato (horas).

La clínica de la emergencia hipertensiva depende del órgano blanco afectado; síntomas comunes son visión borrosa, cefalea intensa, náuseas/vómitos aislados. Dependiendo del grado de afectación puede manifestarse déficit neurológico

23 Facultad de Medicina, Universidad de Chile

como manifestación de infarto cerebral, disnea como síntoma de insuficiencia cardíaca asociada o no a EPA, u otras.

ñino: EPA y disección aórtica requieren normalización de la PA en minutos; en accidentes isquémicos no se normaliza PA antes de 10 días (exceptuando PAM >130 mmHg, PD >120 mmHg, PS >200 mmHg, presencia de IC, presencia o sospecha de disección aórtica y en planificación de trombolisis); Una hemorragia cerebral parenquimatosa se debe tratar si la PAS >170 mmHg, siendo la meta alcanzar presiones sistólicas de 140-160 mmHg y no menos. En hemorragia subaracnoidea la HTA no debe ser tratada, salvo cifras PA >220/130 mmHg, dado el intenso vasoespasmo cerebral asociado.

Tratamiento Emergencia hipertensiva: La meta global de tratamiento consiste en la reducción de la PA a valores bajo 140/90 mmHg. La rapidez para lograr tal objetivo varía respecto a la etiología de la crisis y del órgano blanco afectado, habiendo situaciones en las que un rápido descenso de la PA puede ser da-

Urgencia Hipertensiva:

En pacientes con alza de PA asintomática están contraindicados los hipotensores parenterales y de acción rápida.

Monitorización recurrente de la PA según tratamiento ya indicado, observar y detectar precozmente recidivas en pacientes ya normalizados. Controles cada 24-48 horas (adecuación terapia antihipertensiva oral, evaluación perfusión periférica, certificación PA de pie) en atención primaria en pacientes con alza de PA asintomática o con sintomatología inespecífica.

Seguimiento Una vez que se ha logrado controlar la PA durante 12 – 24 horas para permitir que se restablezca la autorregulación, los medicamentos parenterales debieran retirarse en forma gradual hacia los medicamentos por vía oral. En los próximos días, se pueden efectuar algunos estudios diagnósticos para establecer una posible causa secundaria de HTA.

Autor / Editor Hernán Rubilar

24 Facultad de Medicina, Universidad de Chile

Año 2016

Hiperkalemia Grave

bular renal tipo IV (por enfermedades tubuloinsterticiales o glomeruloesclerosis diabética), hipoaldosteronismo primario o secundario y otros fármacos como: IECA, ARAII, AINES y Heparina.

Nivel de manejo del médico general: Diagnóstico Específico Tratamiento Completo Seguimiento Derivar

Diagnóstico

Hablamos de Hiperkalemia cuando la concentración sérica de potasio es > 5.5 mEq/l. Se considerará grave cuando se asocia a cualquier cambio en el ECG o la kalemia es > 6.5 mEq/l.

Sospecha clínica (aunque puede ser asintomático) y se confirma con exámenes de laboratorio (Electrolitos plasmáticos y ECG principalmente). Las manifestaciones clínicas dependen de la magnitud y velocidad del alza de la kalemia. Estas aparecen generalmente con niveles séricos de K > 6.5 mEq/l, siendo las principales: • Cardiovasculares: bradicardia • Neuromusculares: parestesias, debilidad progresiva de varios grupos musculares y parálisis fláccida ascendente, que puede alcanzar la musculatura respiratoria. • Gastrointestinales: náuseas y vómitos • Exámenes de laboratorio: • Electrolitos plasmáticos: Determinar la existencia de hiperkalemia • ECG: A medida progresa la hiperkalemia se aprecian los siguientes cambios. Ondas T picudas (con intervalo QT normal o ligeramente reducido), prolongación del intervalo PR con depresión de ST, desaparición progresiva de la onda P, bloqueo cardíaco progresivo, arritmias ventriculares, paro cardíaco. • Gases arteriales: Descartar acidosis • Creatinina plasmática y BUN: Descartar insuficiencia renal. • Renina y aldosterona plasmática: Descartar hipoaldosteronismo. • [K+] urinario, Osm plasmática y Osm urinaria: Para calcular la gradiente transtubular de K+ (GTTK= [K+]u x Osmp/ [K+]p x Osmu), que si es < 5 sugiere defecto en la secreción renal de K+ y si es > 5 la secreción renal de K+ es normal.

Etiología-epidemiología-fisiopatología

Tratamiento



1. Evaluar ABC y monitorizar con ECG.

Aspectos Esenciales • • • •

Una hiperkalemia asociada a cambios en el ECG y/o [K+] p > 6,5 mEq/l es una urgencia. Se asocia frecuentemente a arritmias potencialmente fatales y paro Cardio-respiratorio. La alteración en el ECG más constante son las ondas T picudas. La principal causa es la insuficiencia renal y fármacos bloqueadores del eje RAA.

Caso Clínico Tipo Paciente de sexo masculino de 55 años, con ERC etapa IV con mal manejo médico, consulta a APS por cuadro de días de evolución caracterizado por sensación de debilidad en extremidades, parestesias en extremidades inferiores, náuseas y vómitos. Se toman electrolitos plasmáticos que evidencian K+ de 6,2 mEq/L y ECG, donde se evidencian ondas T picudas.

Definición



• •

Pseudohiperkalemia: Muestras de sangre hemolizadas, traumatizadas (por el mal uso de torniquete al extraer muestra), analizadas tardíamente, leucocitosis excesiva y trombocitosis. Desplazamiento intra-extracelular de K+: Destrucción celular (hemólisis masiva, necrosis tumoral, rabdomiolisis, quemaduras y trauma), acidosis, déficit de insulina, aumento del catabolismo celular, hiperosmolaridad (contrastes radiológicos, manitol), β2 Bloqueadores, succinilcolina e intoxicación digitalica. Aumento de aporte de K+: Suplementos de K+ (sal dietética), suplementos nutricionales, penicilina G potásica y hemorragia digestiva. Disminución excreción renal de K+: Insuficiencia renal, diureticos ahorradores de K+ (espironolactona, triamtereno), disminucion del aporte de Na al túbulo distal (ICC, depleción de volumen, AINES, ciclosporina), acidosis tu-

2. Estabilización de la membrana: Es la primera medida, se logra con la administración de calcio endovenoso, esto no afecta a los niveles de potasio plasmático pero modifica el potencial umbral, reestableciendo la diferencia entre potencial de reposo y potencial umbral. El efecto es inmediato y dura 30 minutos. Dosis: Gluconato de calcio 10 ml al 10% en 10 minutos (acción inmediata dura 20 – 30 min). Si no hay corrección ECG repetir a los 5 min. (cuidado en presencia digital). 3. Estimular la entrada de K+ a la célula: • Insulina: Habitualmente asociada a glucosa para evitar hipoglicemia. Promueve la entrada de K+ a la célula disminuyendo las concentraciones de K+ en 0.5 a 1 meq/l de una forma dependiente de la dosis. Dosis: 10 U insuli-

25 Facultad de Medicina, Universidad de Chile





na cristalina + 50ml de glucosa al 50% (acción a los 20-30 min. dura 1-2 hrs). B2 agonista: Se pueden utilizar por vía inhalatoria o endovenosa. Los niveles de potasio comienzan a disminuir en 5 minutos y el efecto es máximo a los 40-80 min. Las dosis necesarias son 4 a 8 veces mayores a las administradas como broncodilatadores. A estas dosis se puede producir taquicardia, temblor, ansiedad y rubor. Dosis: Albuterol o salbutamol e/v o inhalatorio 10 a 20 mg. Inicio de acción a los 5 minutos (efecto máximo 30 a 90 min.) Bicarbonato: Efectivo especialmente en presencia de acidosis.

En ausencia de función renal y con niveles de K+ elevados que ponen en riesgo al paciente, debe adicionarse a la terapia de emergencia la hemodiálisis que es capaz de depurar eficientemente el potasio, logrando una disminución en 1 a 2 meq/l/hora.

Seguimiento Derivar a especialista.

Autor / Editor Hernán Rubilar

Año 2016

26 Facultad de Medicina, Universidad de Chile

Hipokalemia



Nivel de manejo del médico general: Diagnóstico Específico Tratamiento Completo Seguimiento Completo



Aspectos Esenciales • • •



Factor de riesgo importante para arritmias ventriculares y auriculares. Exámenes: niveles de K plasmático y ECG. Tratamiento de hipokalemia grave se realiza por vía IV. Si K+ ≥ 3mEq/L reposición por VO. Si K+ entre 2,5 y 3 mEq/L, sin signos de gravedad ni tratamiento digitálico, reposición VO. Corregir hipomagnesemia asociada.

Pérdidas urinarias: Diuréticos, Hiperaldosteronismo, Diuresis osmótica, Acidosis tubulares tipo 1 y 2, Sd. Bartter – Gitelman, Hipomagnesemia, Sd. de Liddle, Fase poliúrica de NTA. Otras causas: Pseudohipokalemia, Redistribución de K (generalmente transitorias), Disminución del aporte Por incorporación a células metabólicamente activas (LMA con leucocitosis marcada), Parálisis periódica hipokalémica, Insulina, Beta 2 agonistas, Intoxicación por tolueno, Tirotoxicosis, Alcalosis (rara, salvo indigentes), Dieta pobre, Uso de fluidos IV, Geofagia (arcilla).

Diagnóstico Los síntomas de hipokalemia moderada incluyen: debilidad, fatiga, dificultad respiratoria, constipación, íleo paralítico, calambres, parálisis arrefléxica de los músculos esqueléticos, rabdomiólisis, vasodilatación e hipotensión. La hipokalemia severa puede alterar la excitabilidad del tejido cardiaco. • Se debe evaluar la dieta, presencia de diarrea y vómitos, uso de diuréticos y laxantes, presión arterial (HTA se puede asociar a hiperaldosteronismo), signos de hipertiroidismo (asociados a parálisis periódica hipokalémica). • En el laboratorio se debe solicitar ELP, ECG (QT largo, aumento de onda U, descenso del segmento ST, arritmias severas), K urinario (K+ < 30 mEq en 24 hrs. o de 15 mEq/l), estado ácido base (asociación a alcalosis).

Caso Clínico Tipo Mujer de 17 años concurre a SU por cuadro agudo de palpitaciones que se agrega a debilidad y calambres de varios días de evolución. Refiere que hace 6 meses está en tratamiento por obesidad con dieta y preparado magistral cuyos componentes desconoce. Al ingreso ansiosa, disneica, PA: 100/70 mmHg, FC: 110x’ con pulso irregular.

Tratamiento

Definición

Va a depender del grado de hipokalemia. Si el K+ está entre 3mEq/L y 3,5 mEq/L, de favorece la reposición VO, sales de KCL 20 a 80 mEq/día, o SlowK (8 mEq/ comp) y/o con aumento del aporte de K en la dieta, (sal dietética contiene 12 mEq/g, cucharada de té contiene 60 mEq). Pero si el K+ está entre 2,5 y 3 mEq/L sin signos de gravedad ni tratamiento con digitálicos: Reposición VO, 120 – 240 mEq/día.

Se define hipokalemia como valores de K+ sérico < 3,5mEq/L.

Epidemiología La hipokalemia se encuentra en más de 20% de los pacientes hospitalizados y en 10 a 40% de los pacientes que reciben diuréticos tiazídicos.

Si K+ es <2,5 mEq/L o sintomática: Reposición IV, en solución sin dextrosa para impedir translocación al intracelular. Se recomienda que la concentración de K+ en la solución sea de < 50 mEq/L, y en general < 200 mEq/día. Reponer 10 a 20 mEq/hora con monitoreo cardiaco continuo. Si la hipokalemia crónica: el tratamiento debe dirigirse a la patología de base. Uso de diuréticos conservadores de K: espironolactona y amilorida.

Fisiopatología Sus consecuencias son derivadas de alteraciones en el potencial de membrana. A nivel del tejido cardíaco ellas tienden a despolarizarse predisponiendo a arritmias ventriculares y auriculares especialmente en el contexto de cardiopatías preexistentes y en usuarios de digoxina.

Evaluación de la dieta: alimentos según contenido de K: • > 25 mEq/100g: Higos, miel, algas, • > 12,5 mEq/100g: ciruelas secas, nueces, paltas, cereal integral, trigo, legumbres • > 6mEq/100g: espinaca, papas, zanahoria, acelga, brócoli, coliflor, zapallo, plátano, kiwi, naranja, melón.

Las causas más comunes de hipokalemia son: • Pérdidas gastrointestinales (diarrea, laxantes), pérdidas renales (hiperaldosteronismo, hiperglicemia severa, diuréticos eliminadores de potasio), alteraciones en equilibrio ácido – base (alcalosis), y malnutrición, vómitos (5–10 mEq/L)

27 Facultad de Medicina, Universidad de Chile

Seguimiento En hipokalemia grave se debe controlar niveles de K cada 2 a 3 horas e intentar no llegar violentamente a la normalidad. La desaparición de la onda U en el ECG es útil para evaluar respuesta.

Autor / Editor Paulina Lira

Año 2016

28 Facultad de Medicina, Universidad de Chile

Hiponatremia Aguda Grave



Hipervolémica: Insuficiencia Cardiaca, Síndrome Nefrótico, Cirrosis hepática, Insuficiencia renal. • Euvolémica: - Hipotiroidismo, Insuficiencia suprarrenal, SIADH, Diuréticos tiazídicos, Poca ingesta de solutos. 2. Ingesta excesiva de agua • Polidipsia primaria • Soluciones de irrigación sin NaCl • Fórmulas infantiles diluidas • Inmersión

Nivel de manejo del médico general: Diagnóstico Sospecha Tratamiento Inicial Seguimiento Derivar

Aspectos Esenciales • • •

Na+ plasmático <125 mEq/L de instalación rápida y sintomático. Síntomas neurológicos: cefalea, compromiso de conciencia, edema cerebral difuso, herniación, muerte. Corrección rápida con solución salina al 3%. Meta [Na+] p 120-125 mEq/L.

Diagnóstico El cuadro clínico se caracteriza por síntomas principalmente neurológicos de instalación aguda. Inicialmente aparece cefalea, náuseas, vómitos y posteriormente se agrega compromiso de conciencia progresivo, convulsiones y eventualmente muerte en relación a herniación cerebral por edema cerebral difuso. Al examen físico: disminución del turgor cutáneo, mucosas secas, yugulares planas, oliguria, hipotensión y taquicardia de reposo u ortostática. Para evaluar la hiponatremia se debe determinar la osmolalidad y responder ¿es un pseudohiponatremia? ¿es una hiponatremia falsa?.

Caso Clínico Tipo Mujer de 49 años en tratamiento durante 8 semanas con corticoides suspendido bruscamente. Presenta cefalea y compromiso de conciencia progresivo durante las últimas 8 horas. Al ingreso destacan: [Na+] 121 mEq/L, [K+] 6,1 mEq/L. Glicemia 80 mg/dL, Osmolaridad plasmática 246 mOsm/kg. [Na+] urinario 36 mEq/L.

Definición

La seudohiponatremia puede observarse tras la determinación cuantitativa del Na+ sérico en un volumen dado de plasma con contenido de partículas excluyentes de agua (lípidos y proteínas) superior al normal. En este contexto, el porcentaje del volumen plasmático constituido por agua plasmática (donde se encuentra el Na+) es menor, y por ello el valor de concentración de Na+ (expresado en mEq/L) determinado también es menor. No obstante, la concentración plasmática de Na+ es normal y por ello los pacientes son asintomáticos. Es importante prestar atención a la hiperlipidemia o hiperproteinemia.

Concentración de Na+ plasmático ([Na+]p) < 125mEq/L de instalación en < 48 hrs.

Epidemiología Se encuentra hiponatremia entre el 15 y 30% de los pacientes hospitalizados, la mayoría de las veces es leve y asintomática.

La hiponatremia falsa es consecuencia de una osmolalidad sérica elevada (hiperosmolalidad) tal y como puede observarse en caso de hiperglucemia y que es responsable del paso de agua intracelular hacia el espacio extracelular y la consecuente dilución del Na+ en el LEC. Los pacientes con hiponatremia falsa, al contrario que los pacientes con hiponatremia verdadera, no presentan síntomas relacionados con la hipoosmolalidad. En caso de hacerlo, éstos son consecuencia directa de la hiperosmolalidad, y por este motivo se aconseja centrar una especial atención en recuperar los valores normales de osmolalidad. Es importante distinguir estos dos tipos de hiponatremia de la hiponatremia verdadera asociada a hipoosmolalidad, porque tanto las pruebas de evaluación como el tratamiento y el seguimiento terapéutico de los pacientes son diferentes en ambos casos.

Fisiopatología Presenta potencial morbimortalidad en su forma severa y cuando su corrección es demasiado rápida.

Etiología Causas de hiponatremia hipotónica: 1. Disminución de la capacidad de excreción de agua • Hipovolémica: - Pérdida de Na+ renal: Diuréticos, diuresis osmótica, insuficiencia suprarrenal, nefropatía perdedora de sal, bicarbonaturia. - Pérdida de Na+ extrarrenal: Diarrea, vómitos, hemorragia, sudoración, tercer espacio.

29 Facultad de Medicina, Universidad de Chile

Tratamiento Se indica corrección rápida con solución salina al 3% en la hiponatremia severa y sintomática. En este contexto la velocidad de ascenso de la natremia no debe superar un promedio de 8 a 12 mEq/24 horas, ya que sobre esto existe riesgo mayor de desmielinización osmótica. Muchas veces no se conoce el tiempo de instalación, por lo que en ese contexto es mejor manejarlo como hiponatremia crónica. La meta es llevar a un nivel de [Na+]p “seguro” (entre 120 y 125 mEq/L nunca a lo normal) en pocos días.

Seguimiento Completo

Autor / Editor Marcelo Fres

Año 2016

30 Facultad de Medicina, Universidad de Chile

Insuficienca Renal Aguda Obstructiva

coraliforme con hidronefrosis.

Definición

Nivel de manejo del médico general: Diagnóstico Sospecha Tratamiento Inicial Seguimiento Derivar

Aquella insuficiencia renal aguda (IRA) dada por una obstrucción del flujo urinario en la vía urológica (distal al parénquima renal).

Aspectos Esenciales

Etiología

• • • •

Las etiologías se muestran en la tabla. Es más frecuente en pacientes de edad avanzada por enfermedad de próstata.

Azoemia por obstrucción al flujo urinario. Hiperplasia prostática benigna (HPB) principal causa. Ecografía diagnóstica. Aportar volumen por poliuria osmótica post obstructiva.

Fisiopatología

Caso Clínico Tipo Paciente hombre de 68 años llega confuso y con dificultad respiratoria. Relata cuadro de dolor lumbar, tomó analgésicos y a las horas se agregó disnea. No ha orinado en todo el día. El paciente cuenta ser monorreno. Los exámenes muestran elevación de la creatininemia y la ecografía una litiasis

La obstrucción aumenta la presión hidrostática de forma retrógrada intratubular en el espacio de Bowman, ello disminuye el filtrado glomerular, activa el eje renina-angiotensina y éste lleva a una vasoconstricción renal secundaria que reduce el flujo sanguíneo y, más aún, la función renal. Independiente del nivel de la obstrucción, si ésta se mantiene, el daño renal se hace irreversible debido a atrofia tubular.

Diagnóstico

Seguimiento

La clínica es muy variable, con distintos grados de diuresis: anuria, o anuria-poliuria intermitentes, globo vesical. Ella depende de la etiología y por la propia azoemia. El diagnóstico no debiese ser un problema dado el algoritmo de manejo de toda insuficiencia renal que consiste en una ecografía renal (dilatación de la vía urinaria) e instalación de una sonda Foley (poliuria post obstructiva si era debido a patología prostática).

Luego de la descompresión de la vía urinaria se debe evitar la pérdida de volumen producto de la poliuria osmótica post obstructiva con solución salina isotónica (0.9%) de forma inicial y, más tarde, al 0.45% más aporte de K+ (por las pérdidas renales aumentadas).

Autor / Editor Marcelo Fres

Tratamiento Es suficiente con el sondaje vesical en la uropatía obstructiva baja, si es más alta puede ser necesaria la instalación de un catéter ureteral, cistostomía o nefrostomía (requiere evaluación por urólogo).

31 Facultad de Medicina, Universidad de Chile

Año 2016

Insuficienca Renal Aguda Prerrenal

Na orina <20 mEq/L y Osmolaridad urinaria > 500 mOsm/kg H20., FE urea <35% (en caso de uso de diuréticos).

Nivel de manejo del médico general: Diagnóstico Específico Tratamiento Completo Seguimiento Completo

Tratamiento El inicio del tratamiento debe ser precoz para evitar la evolución a necrosis tubular aguda. Como generalmente la causa es prerrenal, se debe tratar el evento desencadenante, si es depleción de volumen se debe expandir volemia cristaloides (suero fisiológico) y tratamiento de la causa subyacente.

Aspectos Esenciales • • • •

Presenta oliguria, hipotensión arterial, taquicardia. La orina se caracteriza por Na+ orina <20 y Osm orina > 500. FE Na menor al 1%. Si el paciente usa diuréticos, la FE Na no sirve, por lo tanto se utiliza la FE urea.

Seguimiento Monitorización de la respuesta a la terapia de restauración de la volemia, mediante la observación de la evolución de los signos de contracción del volumen extracelular. Medición seriada de diuresis, del estado ácido-base y los niveles de

Caso Clínico Tipo

potasio plasmático.

Paciente de 18 años, con síndrome diarreico agudo, de una semana de evolución, con hipotensión, mucosas secas, oliguria, taquicardia. Autor / Editor Marcelo Fres

Definición Deterioro brusco de la función renal, ocurre en situaciones de descenso de perfusión renal. Su velocidad de instalación es de horas a días. La mayoría de casos son reversibles.

Fisiopatología Al disminuir el volumen intravascular se activará el SRAA que generará la liberación de ATII, Aldosterona y ADH, aumentando la reabsorción principalmente de agua y Na. Se produce por disminución severa no compensada del FPR, debido a hipovolemia (contracción del volumen extracelular, hemorragia, 3° espacio), vasodilatación sistémica (shock séptico) o en vasoconstricción de la arteriola aferente glomerular (preeclampsia, sd heptorrenal). Como también, puede producirse por una alteración en la autorregulación de ATII y PG, como en el uso de IECAs o AINEs.

Epidemiología Es la causa de un 20% de IRA.

Diagnóstico En paciente con síntomas sugerentes de depleción volumétrica: hipotensión, taquicardia, mucosas secas y oliguria, y exámenes confirmatorios: BUN/CREA alta (>20), orina muy concentrada con presencia de Cilindros Hialinos, FE Na <1%,

32 Facultad de Medicina, Universidad de Chile

Año 2016

Pielonefritis Aguda Complicada

to y diarrea. Además puede acompañarse de síntomas urinarios bajos. El examen físico se acompaña de taquicardia, dolor muscular generalizado. Es notable el dolor a la presión en una o ambas fosas lumbares o a la palpación abdominal profunda.

Nivel de manejo del médico general: Diagnóstico Sospecha Tratamiento Inicial Seguimiento Completo

En el laboratorio se puede encontrar microhematuria, cilindros leucocitarios, piuria, bacteriuria, urocultivo positivo (se aceptan como significativas bacteriurias de 100.000 UFC/ml de un germen uropatógeno), leucocitosis con desviación izquierda, PCR elevada, VHS elevada, bacteremia (30%).

Aspectos Esenciales • •

PNA complicada: hombres, paciente con alteración funcional o estructural del aparato urinario, mujer embarazada, en pacientes inmunodeprimidos. El tratamiento se hace hospitalizando, uso de ceftriaxona i.v. y continuar v.o. según urocultivo por 14 días Si no hay respuesta a tratamiento considerar eco renal.

Tratamiento

Varón con antecedentes de uropatía obstructiva en tratamiento médico, que presenta fiebre 39°C por 3 días, disuria y compromiso del estado general.

El tratamiento se hace hospitalizado, se debe hidratar adecuadamente. El tratamiento antibiótico de elección es la ceftriaxona 2 g/24 h por vía endovenosa, durante 2-5 días en función de la evolución. Continuar tratamiento oral según urocultivo, y si éste no se obtuvo o es negativo, se recomienda administrar quinolonas de segunda generación, como ciprofloxacino, o de tercera generación, como levofloxacino, cefalosporinas orales de tercera generación (cefpodoxima) o amoxicilina/ácido clavulánico. Se debe completar 14 días.

Definición

Seguimiento

Inflamación del riñón y de la pelvis renal como resultado de la invasión bacteriana. Puede clasificarse anatómicamente como una ITU alta (infección de la porción superior de la vía urinaria). Se considera pielonefritis complicada a todo cuadro infeccioso que ocurre en: todas las ITU en el hombre, ITU con alteración funcional o estructural del aparato urinario, ITU en la mujer embarazada, ITU en pacientes inmunosuprimidos (SIDA, trasplantados).

La ausencia de respuesta al tratamiento a los 3-4 días, si no se dispone de urocultivo o éste es negativo, obliga a la realización de ecografía renal.



Caso Clínico Tipo

Autor / Editor Paulina Lira

Etiología Como la vía ascendente es la vía más frecuente de infección del aparato urinario, el germen causal con más frecuencia es Escherichia coli uropatógeno tanto ambulatorio (80-90%) como hospitalario (50%). La infección del parénquima renal se produce debido a la multiplicación de los gérmenes en la vejiga y su ascenso por el uréter y la pelvis renal, y la colonización final en el parénquima de los riñones. También son importantes Proteus, Klebsiella y Enterococcus. La vía hematógena es más rara y suele observarse en la bacteriemia por S. aureus o en la fungemia por Candida albicans. Además existen determinantes que posibilitan una ITU alta, como son: virulencia del patógeno, tamaño del inóculo y estado inmunitario del huésped.

Diagnóstico La clínica se presenta con fiebre, escalofríos, náuseas, vómi-

33 Facultad de Medicina, Universidad de Chile

Año 2016

Acidosis Metabólica

En cuanto a los mecanismos de compensación son: a) Tampones: En minutos actúan los tampones del extracelular e intracelular que se refleja en caída del bicarbonato plasmático e hiperkalemia respectivamente b) Respiratoria: En horas se produce hiperventilación que genera hipocapnia, disminuyendo la pCO2 en 1.25 mm Hg por cada 1 mEq/l que baja la concentración de bicarbonato. c) Renal: En días se produce aumento de la excreción renal de acido fijo por mayor excreción de amonio (principalmente), acidez titulable (en menor medida) y generación de bicarbonato.

Nivel de manejo del médico general: Diagnóstico Específico Tratamiento Inicial Seguimiento Derivar

Aspectos Esenciales • • • • •

Se caracteriza siempre por acidemia y BE negativo o HCO3- estándar disminuidos. Se clasifican en Acidosis metabólicas con Anión Gap aumentado ([Cl]p normal) y con Anión Gap normal ([Cl]p aumentado). Las principales causas son Diarrea (AG normal), Cetoacidosis diabética (AG alto) y acidosis láctica (AG alto). Las manifestaciones clínicas típicas son respiración de Kussmaul y compromiso de consciencia. El tratamiento se basa en corregir la causa y administrar bicarbonato en acidosis extremas.

Diagnóstico Se base en la Sospecha clínica + confirmación por exámenes de laboratorio En cuanto a las manifestaciones clínicas podemos encontrar: • •

Caso Clínico Tipo Paciente de 50 años (60 kg) en encontrado en vía pública comprometido de conciencia. Al ingreso se objetiva en coma, respiración profunda, 35 x min. Está con VEC levemente disminuido, hay leve aliento a acetona. En los exámenes se encuentra: pH: 7,05 pCO2: 18 mmHg; BE: -24 meq/l. HCO3real: 6,1 meq/l. CO2 total: 7 mmoles/l. Na+ : 140 mEq/l; K+: 7 mEq/l. Cl-: 85 mEq/l.

• • • •

Definición Proceso que genera un exceso de ácidos fijos que lleva a un incremento de protones libres en el plasma (Acidemia o pH < 7,35) y a una disminución de la concentración plasmática de bicarbonato ([HCO3 -]p < 23 mEq/l).

Sistema respiratorio: taquipnea, hiperpnea, respiración de Kussmaul y disnea. Sistema cardiovascular: disminución de la contractibilidad miocárdica, gasto cardiaco y resistencia vascular periférica; hipotensión arterial, disminución del flujo hepático y renal, venoconstriccion esplacnica y arritmias como fibrilación ventricular. Sistema nervioso central: Compromiso de conciencia progresivo hasta el coma. Sistema exocrino: diaforesis e hipersecreción péptica Sistema urinario: reducción de la VFG, activación eje RAA y estímulo de la amonio- génesis Metabolismo: del K+ (hiperkalemia e hipokalemia), del Na+ (natriuresis y contracción del VEC), de las proteínas (hipercatabolismo y proteólisis), del calcio (aumento de la reabsorción osea, hipercalciuria y nefroclacinosis), del fosforo (hiperfosfaturia) y del magnesio (hipermagnesiuria).

Exámenes de laboratorio: a) Para confirmar el diagnóstico: Gases en Sangre Arterial. b) Para el estudio etiológico inicial: • Electrolitos plasmáticos: para calcular Anion Gap y Gap Osmolar • Electrolitos y pH urinarios: para calcular Anion Gap urinario y diferenciar las ATR • Creatinina plasmática y BUN: descartar insuficiencia renal • Glicemia y cetonemia: descartar cetoacidosis diabética o alcohólica • Osmolaridad medida por osmómetro: para calcular Gap Osmolar

Etiología-Epidemiología-Fisiopatología Los mecanismos de producción son: 1. Aumento en el ingreso de ácidos fijos: Cetoacidosis, Acidosis láctica e Ingestión exógena de: aspirina, etilenglicol (solventes y anticongelantes), metanol y cloruro de amonio (NH4Cl). 2. Disminución en la excreción de ácidos fijos: Insuficiencia renal aguda, Insuficiencia renal crónica, Acidosis tubular renal tipo IV (hipoaldosteronismo), Acidosis tubular renal tipo II. 3. Dilución de los tampones extracelulares: infusión de soluciones de grandes volúmenes importantes, sin bicarbonato ni bases (Ej. NaCl isotónico). 4. Pérdida de bicarbonato: Diarrea y Acidosis tubular renal tipo I.

Tratamiento a) Acidosis metabólica con anión gap Normal: • Terapia de la causa de fondo. • Reposición de bicarbonato en caso que no se resuelva

34 Facultad de Medicina, Universidad de Chile

causa de fondo y se asocie pH< 7,20 y/o HCO3 < 10 mEq/l. Para esto calcular Déficit de: HCO3 = 0,6 x peso(Kg) x (HCO3 deseado – HCO3 medido), luego calcular N° de ampollas de HCO3Na al 8,4%= Déficit HCO3/20. Y administrar el 50% de ampollas en infusión 30 min y el otro 50% en 4-6 horas según nuevo control de gases en sangre arterial. b) Acidosis metabólica con anión gap aumentado: • Tratamiento de la causa de fondo. • Terapia con HCO3 en caso de: 1) pH< 7,10 en acidosis láctica o toxica. 2) pH < 6,9 y/o arritmias inducidas por acidemia en casos de Cetoacidosis diabética o alcohólica. Complicaciones del tratamiento con bicarbonato: alcalemia transitoria, hipokalemia, hipernatremia-hiperosmolaridad, sobrecarga de volumen, tetania, alteración de conciencia, convulsiones e inactivación de catecolaminas.

Seguimiento Derivar

Autor / Editor Nevelline Salgado

Año 2016

35 Facultad de Medicina, Universidad de Chile

BIBLIOGRAFÍA •

1. Morales A., “Insuficiencia Renal Crónica”. Nefrología. Bases de la Medicina 2013. Disponible en: www.basesmedicina.cl



2. Guía Clínica “Prevención de Enfermedad Renal Crónica”. Ministerio de Salud, 2010.

36 Facultad de Medicina, Universidad de Chile

consumo de sales ricas en K+ que pueden generar hiperkalemia. Por último la hiperkalemia se puede gatillar por acidosis metabólica, hipoaldosteronismo hiporreninémico (frecuente en DM y enfermedades túbuloinsterticiales) y fármacos (IECA, ARA II o diuréticos ahorradores de Potasio).

Acidosis e Hiperkalemia en insuficiencia renal crónica Nivel de manejo del médico general: Diagnóstico: Sospecha Tratamiento: Inicial Seguimiento: Derivar



Aspectos esenciales    

Son complicaciones frecuentes y potencialmente fatales en pacientes en diálisis. La hiperkalemia se presenta generalmente asociada a sobrecarga aguda de K+ o uso de fármacos que bloquean el eje RAA. La principal medida para manejar la hiperkalemia es la disminución en la ingesta de K+. Se debe controlar la acidosis, ya que aumenta la progresión de la IRC y eleva el riesgo cardiovascular.

Diagnóstico Se debe tener una sospecha clínica y confirmación por exámenes de laboratorio. 

Caso clínico tipo



Mujer de 86 años, DM tipo 2 es llevada a un SU por dificultad respiratoria. Antec: demencia, multi-infarto, AVC, Cardiopatía Isquémica con FE estimada en 20 %. Al Ex. Físico: FR: 26 x min; PA: 180/70; Soplo Holosistólico II/VI en el ápex; Ingurgitación yugular y crépitos en ámbas bases pulmonares. Exámenes de Laboratorio: Hb: 9,6 g/dL; GB: 5.600, Plaq.: 343.000; Creatinina: 5.2 mg/dL; NU: 50 mg/dL; Na: 130 meq/L; K: 6,3 meq/L: Cl: 107 meq/L; Bicarbonato: 16 meq/L; Calcio: 7.4 mg/dL; Fósforo: 6.7 mg/dL; Exámen de Orina: Proteinas +++; Cilindros Granulosos Anchos.

Hiperkalemia: se sospecha en pacientes en diálisis que ingieren una sobrecarga aguda de K+ o usan fármacos que bloquean el eje RAA o presentan los síntomas de hiperkalemia (bradicardia, parestesias y debilidad muscular). Acidosis: se sospecha en pacientes con IRC que presentan las manifestaciones derivadas de: la compensación respiratoria (respiración de Kussmaul), el tamponamiento intracelular (hiperkalemia), el tamponamiento óseo (desmineralización y osteopenia) y el hipercatabolismo crónico muscular.

Exámenes de laboratorio: ELP, hay una kalemia > a 5,5 mEq/l, ECG, puede haber alteraciones inducidas por la hiperkalemia como las Ondas T picudas (Hiperkalemia grave). Gases en sangre arterial: hay un pH < 7,35 con HCO3 < 24 mEq/l o BE < -2 o HCO3 st <24 mEq/l y PCO2 < 40 mmHg. (Acidosis metabólica). Calculo Anion gap (Na+ menos (HCO3- + Cl-): es normal si VFG > 50 ml/min y aumentado si VFG < 30 ml/min, (VN: 12 ± 2).

Definición Disminución del pH arterial bajo 7,35 y aumento de la concentración plasmática de K+ por sobre 5,5 mEq/l en pacientes con disminución crónica (más de tres meses) de la velocidad de filtración glomerular bajo 90 ml/min.

Tratamiento Hiperkalemia: La restricción del potasio en la dieta es la principal medida. Si no es grave se pueden utilizar resinas intercambiadoras de K+, que intercambian K+ por Ca+ o Na+, por vía oral 15 a 30 gr. en 50 a 100 ml asociadas a sorbitol al 20 %. Si es grave se debe realizar hemodiálisis, que logra disminuir la kalemia en 1 a 2 mEq/l/hora. Acidosis: debe tratarse para detener la progresión de la IRC y prevenir la aparición de osteopenia y catabolismo muscular. El bicarbonato de sodio oral es el tratamiento de elección, para mantener una [HCO3-]p > 22mEq/l.

Etiología-epidemiología-fisiopatología Son una de las complicaciones más frecuentes y potencialmente fatales en pacientes con insuficiencia renal crónica terminal en diálisis. 

Homeostasis ácido-base en la IRC: Con una VFG < 50 ml/min, las nefronas remanentes secretan amonio al máximo, pero es insuficiente para regenerar el bicarbonato gastado, produciendo una acidosis metabólica con AG normal. A partir de VFG < 30 ml/min (retención fosfatos) aparece acidosis metabólica con AG aumentado.

Homeostasis del K+ en la IRC: La kalemia se mantiene dentro de los rangos normales, hasta que se presenta oliguria. Esto se logra ya que hay hiperfiltración por los nefrones remanentes y como la Fracción Excretada de K+ es 15% aumenta la excreción neta por el nefrón remanente, lo cual compensa la caida de la VFG de los dañados. También se observa incremento de secreción distal. Finalmente, existe aumento de la excreción colónica en etapas avanzadas. Además disminuye progresivamente la capacidad de manejar la ingesta diaria normal y las sobrecargas agudas de K+ que frente a una ingesta excesiva de frutas o

Seguimiento Se deriva al especialista.

1

mayores que están institucionalizados (15 a 35% en hombres y 25 a 50% en mujeres). Se considera que una alta proporción de personas pertenecientes a esta población presenta bacteriuria en algún momento. Se asocia a pacientes con que utilizan catéter por largos períodos. La prevalencia de bacteriuria asintomática en mujeres sanas aumenta con la edad, de aproximadamente 1 por ciento entre las escolares a> 20% entre las mujeres mayores de 80 años que residen en la comunidad. Se correlaciona con la actividad sexual, la prevalencia es mayor entre las mujeres casadas premenopáusicas que las monjas de la misma edad. Las mujeres embarazadas y no embarazadas tienen una prevalencia similar (2-7%). En las mujeres jóvenes y sanas, la bacteriuria asintomática es transitorio; rara vez dura más de unas pocas semanas. Son varios los factores que favorecen la bacteriuria de los adultos mayores: cambios fisiológicos del tracto urinario; cambios hormonales en la mujer; hipertrofia prostática; disfunción vesical y vesicoureteral relacionadas con enfermedades crónicas comunes en esta población, diabetes, uso de antidepresivos y sedantes; incontinencia vesical y/o intestinal; uso de urosan, etc.

Bacteriuria Asintomática Nivel de manejo del médico general: Diagnóstico: Sospecha Tratamiento: Inicial Seguimiento: Completo

Aspectos esenciales  

  

Bacteriuria asintomática se da sin presencia de síntomas urinarios. La gran mayoría se encuentra en pacientes adultos mayores y aquellos con sonda permanente. El diagnóstico es por Laboratorio, al realizar un Urocultivo. La elección del tratamiento ATB depende de varios factores. Búsqueda y terapia se recomienda antes de procedimientos urológicos, en trasplantados renales y en embarazadas.

Diagnóstico El diagnóstico es de Laboratorio, se basa en el resultado del cultivo o de una muestra recolectada de orina de forma que minimice la contaminación

Caso clínico tipo Paciente de sexo femenino de 67 años que en un control de rutina por su hipertensión arterial (HTA) se detecta la presencia de leucocitos (30-50 l/campo) y bacteriuria en el análisis de orina, siendo el resto normal. Al preguntarle sobre posibles síntomas niega presentar molestias urinarias, dolor lumbar o abdominal, ni fiebre. Hace unos 6 años tuvo una infección de orina que mejoró con tratamiento antibiótico.

 

Mujer asintomática: 2 muestras recuento ≥10 5 UFC /ml de la misma cepa En hombres una muestra con una especie bacteriana aislada en un recuento ≥10 5 UFC/ml.

Tratamiento Recomendado:

Definición

 

Bacteriuria asintomática se define como el aislamiento de un recuento cuantitativo específico de bacterias en una muestra de orina recogidas adecuadamente de un individuo sin síntomas o signos de infección del tracto urinario.

 

En mujeres la bacteriuria asintomática se define por la Sociedad de Enfermedades Infecciosas de América (IDSA) como dos muestras consecutivas de orina recolectada con aislamiento del mismo microorganismo en recuentos cuantitativos ≥105 UFC/ml.

Embarazadas. Pacientes en los que se ha de realizar una resección prostática. Previo a una manipulación urológica mayor (no para sonda vesical). Pacientes con trasplante renal

No recomendado: -Mujeres pre-menopáusicas no embarazadas. -Mujeres diabéticas. -Ancianos que viven en la comunidad. -Ancianos institucionalizados. Personas con daño medular. -Pacientes con sondaje vesical.

Bacteriuria asintomática en los hombres se define según IDSA 2005 como una sola toma limpia de muestra de orina con aislamiento de un solo organismo en recuentos cuantitativos de ≥105 UFC/ml.

El tratamiento preventivo sólo erradica definitivamente la bacteriuria en menos de 20%.

Seguimiento Etiología-epidemiología-fisiopatología

En embarazadas con BA se deben realizar urocultivos de control posterior al tratamiento durante toda la gestación, debido a las frecuentes recurrencias.

Los gérmenes más frecuentemente aislados son los enterobacilos y con menos frecuencia Staphylococcus coagulasa negativo. En los institucionalizados la bacteriuria puede ser polimicrobiana. La prevalencia de bacteriuria es mayor en mujeres que en hombres. Es más remarcable en aquellos adultos 2

Cólico nefrítico, Urolitiasis y Complicaciones (Obstrucción, Sepsis e Insuficiencia Renal)

Las complicaciones se explican por la alteración provocada por la expulsión del propio cálculo (insuficiencia renal de tipo postrenal, por obstrucción bilateral o unilateral en monorreno), sepsis por diseminación bacteriana en plasma, y retención urinaria por obstrucción.

Nivel de manejo del médico general: Diagnóstico: Sospecha Tratamiento: Inicial Seguimiento: Completo

Diagnóstico El diagnóstico se basa en el cuadro clínico característico (dolor cólico en fosa lumbar asociado a hematuria) más una imagen (PieloTAC de elección. Rx: Cálculos cálcicos son radiopacos). Examen orina: visualización cristales hexagonales de cistina.

Aspectos esenciales      

De las complicaciones más importantes que debemos evaluar siempre están:

Principal etiología: litiasis cálcica. El Diagnóstico es Clínico + Imágenes (de elección: PieloTAC). Complicaciones: Sepsis foco urinario, IRA y retención urinaria con globo vesical. Tratamiento agudo: analgesia. Evitar recurrencia: medidas farmacológicas y dietéticas. Complicaciones revierten con la desobstrucción y medidas específicas en cada caso.

1. IRA: Disminución de VFG con elevación de creatininemia y uremia. 2. Sepsis: Infección documentada más fiebre, hipotermia, etc. 3. Retención urinaria: deseo miccional desmesurado, dolor en hipogastrio y ausencia de micción de varias horas de evolución, con matidez palpable en hipogastrio (globo vesical).

Caso clínico tipo

Tratamiento

Paciente de 50 años, DM insulinodependiente, ITU y cólicos nefríticos a repetición. Acude a SU por dolor en fosa renal izquierda de 5 días y actualmente se agregó fiebre, escalofríos y CEG. Al laboratorio se evidencia plaquetopenia, leucocitosis y disminución de la actividad de protrombina. Examen de orina normal. Eco Renal: dilatación moderada de sistema excretor izquierdo.

Manejo Agudo: Se basa en Analgesia con AINES y/o opiaceos, calor local, hidratación a la espera de eliminación en cálculos de < 0.5 cm. Se debe optar por un tratamiento Intervencional en caso de tamaño considerable (se sabe que no se podrá eliminar por si sólo) o que presente condiciones de gravedad (European Association of Urology 2010) tales como que no se logra un alivio suficiente del dolor, cuando existe una obstrucción causada por la propia litiasis acompañada de infección, cuando existe riesgo de pionefrosis o sepsis urinaria, en riñones únicos con obstrucción y en la obstrucción bilateral. Las alternativas intervencionales correspondenden a las siguientes:

Definición Dolor que se origina por la obstrucción aguda a nivel del riñón o de las vías urinarias, produciendo una distensión retrógrada del sistema calicial. Si es provocado por la presencia de concreciones minerales y de matriz orgánica en vía urinaria, se habla de urolitiasis. Existen riesgos de complicación en este proceso debido a la misma expulsión del cálculo, factores del huésped, entre otros.

A. Litotripsia (sobre unión pieloureteral y < 2cms) B. Cirugía (ureteral > 5 cms, segunda opción para cálculos sobre unión pieloureteral y > 2cms vesical o uretral)

Manejo específico de las complicaciones

Etiología-epidemiología-fisiopatología

1. IRA: desobstrucción 2. Sepsis: Antibióticos específicos, desobstrucción y drenaje de la vía urinaria (pigtail) 3. Retención urinaria: Sonda Foley, sino es posible desobstruir por otra via.

Etiología: 80% de los cálculos son causado por urolitiasis cálcica: Oxalato (más frecuente) o fosfato cálcico. Otros: litiasis úrica, cistina, o infecciosa (fosfato amonio magnesiano o estruvita) Epidemiología: Afecta una alta proporción de la población, frecuencia de 5% en mujeres y 10% en hombres, entre los 15 y 45 años de edad. Las complicaciones se dan en una ínfima parte de los pacientes con este trastorno.

Seguimiento Impedir la litiasis recurrente con medidas dietéticas (aumento ingesta de agua, restricción de ingesta de sodio, restricción de proteínas, ingesta de calcio normal) y farmacológicas (Citrato de potasio, tiazidas, alopurinol).

Fisiopatología: Se basa en el aumento de la sobresaturación de la orina con respecto a sales. Además, hay condiciones que aumentan la cristalización (disminución del volumen urinario; pH básico: precipitación cálcica; pH ácido: precipitación ácido úrico); y otras que la disminuyen (citrato). 3

La retención de sodio y agua por los riñones puede ser el proceso principal, como en el edema debido a insuficiencia renal, o puede ser una respuesta compensatoria al volumen arterial efectivo reducido como se observa tanto en el edema de causa cardiaca como de causa hepática.

Edema generalizado grave (Anasarca) Nivel de manejo del médico general: Diagnóstico: Sospecha Tratamiento: Inicial. Seguimiento: Derivar

Diagnóstico El diagnóstico es principalmente clínico. Debido a la naturaleza de las causas de edema generalizado, el paciente típico presenta un gran compromiso del estado general, principalmente se le observa en decúbito forzado, por lo que el edema es más evidente en dorso y flancos del torso que en piernas y muslos. El signo de la fóvea es positivo y muy marcado: la depresión cutánea suele ser mayor a 4 mm y persistir por lo menos 1 minuto. No es infrecuente observar, además de los derrames en serosas, edemas en zonas de laxitud dérmica y celular, como párpados y conjuntivas. Clínicamente podemos diferenciar la etiología según algunas sutilezas clínicas.

Aspectos esenciales    

Se define anasarca como la presencia de edema importante y generalizado. Las causas más comunes son la insuficiencia cardiaca congestiva y el síndrome cirrótico. El diagnóstico es clínico por el examen físico, destacando el signo de la fóvea (Godet) positivo. El tratamiento consiste en hospitalización, monitorización del paciente e iniciar terapia con diuréticos de asa. Además de la evaluación y manejo de la etiología.



Caso clínico tipo



Paciente 68 años, campesino, con antecedente de HTA, DM, 2 IAM, IRC etapa 3 e insuficiencia cardíaca. Hace 3 meses abandona tratamiento por problemas personales. Cursa cuadro caracterizado por CEG progresivo, disnea, DPN y edema vespertino de EEII. Evoluciona con mayor compromiso general, edema EEII continuo y distensión abdominal.





Definición Se define anasarca como la presencia de edema masivo y generalizado, causado por la infiltración de líquido seroso en los tejidos celulares, principalmente subcutáneo. Generalmente compromete cavidades serosas (derrame pleural, ascitis, derrame pericárdico).

En la insuficiencia cardiaca (derecha) podremos encontrar distención yugular, hepatomegalia y signo del galope (derecho). En el daño hepático crónico podremos ver arañas vasculares, circulación colateral, signos de hiperestrogenismo e incluso, en algunos casos, encefalopatía. Además, es posible de ver ascitis de diversa cuantía e hipoalbuminemia. En el síndrome nefrótico es apreciable una proteinuria marcada (>3,5 mg/día) junto a hipoalbuminemia, hiperlipidemias, hipergammaglobulinemia, estados de hipercoagulabilidad e inmunosupresión. En el caso de alteraciones hipotiroideas (mixedema) encontramos la observación de un paciente con un gran estado edematoso, pero que a la palpación se aprecia firme y denso (signo de la fóvea negativo), junto a piel pálido-amarillenta, fría, áspera y seca, pelo ralo, disminución de secreciones sudoríparas y sebáceas, bradipsiquia, constipación, hiporreflexia, entre otros.

Etiología-epidemiología-fisiopatología

Tratamiento

Las causas de los estados edematosos responden a dos procesos básicos: una alteración de la hemodinamia capilar (fuerzas de Starling) y un aumento en la reabsorción de sodio y agua a nivel tubular renal. Las formas más comunes de edema generalizado incluyen:

Hospitalización, monitorización del paciente, iniciar terapia con diuréticos de asa y derivar. El edema pulmonar es la única forma de edema generalizado que es potencialmente mortal y requiere tratamiento inmediato. En todos los demás estados edematosos, la eliminación del exceso de líquido puede proceder más lentamente, ya que no es peligroso para el paciente. El tratamiento farmacológico se inicia preferentemente con un diurético de asa, como la furosemida, y pueden ser necesarias dosis altas, especialmente en pacientes con síndrome nefrótico. En los pacientes con cirrosis, la combinación de espironolactona y un diurético de asa es el régimen diurético inicial predilecto.









Insuficiencia cardíaca congestiva, secundaria al aumento en la retención renal de sodio debido a la caída en el volumen arterial efectivo. Cirrosis hepática, cuyo mecanismo principal responde a la caída de la presión oncótica plasmática por hipoalbuminemia (también observable en estados de gran desnutrición proteíca moderada y grave) Síndrome nefrótico, donde puede observarse tanto reabsorción tubular de sodio aumentada como hipoalbuminemia (explicada en parte por el daño glomerular); Gran quemado o sepsis grave, dónde la permeabilidad capilar está muy aumentada.

Seguimiento Por especialista.

4

neurológicos, respiración acidótica, frote pericárdico); albuminuria e hipoalbuminemia, hiperlipidemia u lipiduria, hipercoagulabilidad (que indican Sd nefrótico). Siempre evaluar la gravedad de la anasarca buscando dirigidamente serositis, y estado de perfusión de órganos (piel, riñón y cerebro). Los principales diagnósticos diferenciales son el mixedema y el linfedema. En el mixedema los mucopolisacáridos no permiten fóvea, es pálido y más amarillento y la piel es fría, seca y escamosa. El linfedema es un edema duro, dado su cronicidad y contenido proteico (que favorece la proliferación fibroblástica).

Edema Generalizado grave (Anasarca) Nivel de manejo del médico general: Diagnóstico: Sospecha. Tratamiento: Inicial. Seguimiento: Derivar.

Aspectos esenciales     

Tratamiento Indica descompensación grave de la enfermedad de base. Se debe realizar diagnóstico etiológico. Asegurar ABC. Uso de diuréticos sólo con una correcta presión de perfusión. Controlar evolución por pesaje del paciente.

El tratamiento debe estar enfocado a la causa que gatilló el edema. Entre las medidas generales que se pueden tomar está el ABC, restricción hídrica para evitar hiponatremia y de Na+ para evitar una progresión del estado edematoso. Se pueden usar diuréticos (furosemida) sólo cuando se asegure clínicamente una buena perfusión de los órganos para evitar IRA por disminución del volumen efectivo. Deben responderse cuatro preguntas fundamentales en el manejo de la anasarca: ¿Con que urgencia debe tratarse? El EPA es la única condición que requiere manejo inmediato; en las demás condiciones puede iniciarse la terapia de forma gradual. En particular debe tenerse cuidado con pacientes en que la etiología es hepática, donde la depleción de volumen y cambios en la volemia pueden inducir un sd. hepato-renal. ¿Cuáles serán las consecuencias de corregir el edema? En las etiologías cardiogénica y hepática el edema es consecuencia del mecanismo compensatorio de retención de sodio para mejorar el volumen circulatorio efectivo; en cambio, en la etiología renal la retención de líquido es inapropiada, estando tanto el VEC como el volumen circulante efectivo ambos expandidos. ¿Con qué velocidad debe corregirse el edema? El uso de diuréticos elimina líquido inicialmente desde el espacio intravascular, por lo la corrección debe acompañarse de monitoreo hemodinámico y de revisión frecuente de adecuada perfusión capilar. En el caso de pacientes que presentan mayormente ascitis, el líquido debe movilizarse primariamente desde los capilares peritoneales, cuya capacidad no supera los 300-500 ml/día. En la mayoría de los pacientes puede removerse 2-3 lts/día sin impacto hemodinámico significativo. ¿Qué régimen diurético debe utilizarse? En pacientes con etiología hepática es de elección la Espironolactona + Diurético de asa; en pacientes con sd. nefrótico son necesarias dosis mayores debido al secuestramiento de diurético por parte de albúmina intratubular. El principio general es buscar la dosis diaria única efectiva para inducir diurésis suficiente para la pérdida de peso deseada; típicamente se usa Furosemida 20-40 mg/día (dosis máxima oral 320 mg). El control del tratamiento se realiza mediante peso diario del paciente, con objetivo de pérdida de 1-1,5 kgs de peso por día.

Caso clínico tipo Hombre de 28 años sin antecedentes se presenta en el servicio de urgencias en estado de anasarca. Al examen físico se aprecia disminución de la masa muscular, PA: 128/88, exámenes muestran hipoalbuminemia y proteinuria nefrótica. Se debe comenzar tratamiento inespecífico (IECA) y derivar para diagnóstico diferencial de posible glomerulopatía primaria.

Definición El edema es el signo clínico definido como la acumulación excesiva de fluido en células, tejidos y cavidades serosas del cuerpo; cuando ocurre de manera excesiva y generalizada se conoce como anasarca.

Etiología-epidemiología-fisiopatología Las principales etiologías son la insuficiencia cardíaca congestiva, la insuficiencia o cirrosis hepática, la insuficiencia renal, síndrome nefrótico y la desnutrición. El mecanismo patológico puede ser por un desbalance entre las fuerzas de Starling (presiones hidrostáticas y oncóticas del capilar e intersticio); disminución del volumen arterial efectivo con consecuente activación del sistema renina angiotensina aldosterona.

Diagnóstico El objetivo del diagnóstico es encontrar la etiología de esta anasarca, para ello se deben conocer las 3 principales etiologías  Cardíaca: Se manifiesta con disnea de esfuerzos, DPN u ortopnea. Aumento presión venosa yugular, galope (S3) y disminución presión diferencial. Puede presentarse al examen físico crépitos húmedos en pulmón, sugerente de falla de corazón derecho.  Hepática: Siendo su causa más frecuente el abuso de alcohol, siendo especialmente sospechoso si la anasarca se presenta en conjunto con signos de DHC (ictericia, eritema palmar, contractura de Dupuytren, telangiectasias, ginecomastia, asterixis u otros de encefalopatía).  Renal: Que se manifiesta frecuentemente con síntomas de uremia (digestivos, trastornos

Seguimiento Una vez estabilizado el paciente, completar el estudio diagnóstico etiológico y comenzar terapia necesaria, con derivación a especialista según necesidad. Controlar respuesta a terapia por la baja de peso (mantener una tasa de 1- 1,5 kg/día).

5

características de la congestión post-isquémica empeoran la hipoxia relativa conduciendo a prolongada injuria celular y muerte celular en los segmentos tubulares predispuestos. Además, se produce obstrucción tubular lo que permite el paso del ultrafiltrado hacia el tejido intersticial al perder el túbulo la integridad de su pared, y, se reduce la reabsorción tubular.

Enfermedad Tubular Aguda Nivel de manejo del médico general: Diagnóstico: Sospecha. Tratamiento: Inicial Seguimiento: Derivar

Etapas: prerenal, iniciación, extensión, mantención y reparación. La recuperación comenzará entre 3-21 días dependiendo del grado de injuria que sufrió el riñón. Es precedida, generalmente por un progresivo aumento del gasto urinario (fase poliúrica). Las funciones tubulares se demoran más tiempo en recuperarse totalmente. En general la mortalidad es alrededor de un 50%, cifra que no ha cambiado en los últimos 40 años. El pronóstico estaría determinado por la enfermedad primaria que llevó a la falla renal aguda.

Aspectos esenciales   

Importante causa de insuficiencia renal. Puede ser causada por agentes nefrotóxicos o isquemia. El tratamiento busca eliminar las complicaciones de la insuficiencia renal.

Diagnóstico Caso clínico tipo Antecedentes de agentes nefrotóxicos, sepsis, falla renal que evolucionó a NTA.

Paciente de 46 años de edad consultó por oliguria y edema progresivo en miembros inferiores y cara, acompañado con compromiso de su estado general 48 hrs después de hacerse un TAC con contraste.

El examen de orina en la NTA presenta perfil de falla renal "renal": Osmolaridad urinaria < 400 mOsm/Kg, Na > 40 mEq/lt, FeNa > 2%, Fe de Urea > 35% y sedimento urinario con células epiteliales y cilindros café.

Definición Tratamiento

Disfunción tubular renal que se traduce generalmente en la alteración de la reabsorción de uno o varios electrolitos, determinantes de las manifestaciones clínicas. Las anomalías en el transporte tubular son frecuente causa de insuficiencia renal presentándose en pacientes de cualquier grupo etario.

El objetivo del tratamiento es prevenir las complicaciones potencialmente mortales de la insuficiencia renal aguda, durante el tiempo que la lesión esté presente. En otras palabras, consiste en la corrección de las alteraciones secundarias al fallo renal (hiperkalemia, sobrecarga hídrica, acidosis metabólica y elevación de los productos nitrogenados en sangre) mediante diálisis u otras técnicas.

Etiología-epidemiología-fisiopatología Etiología: Necrosis tubular aguda (la más frecuente), exposición a medicamentos o productos químicos nefrotóxicos como metales o algunos medicamentos como antibióticos (aminoglicósidos) o medios de contraste radiológiocos, obstrucción tubular, toxinas producidas en situaciones de infección masiva.

En la NTA la prevención es de importancia fundamental ya que no existen tratamientos específicos, siendo el correcto estado del volumen arterial efectivo la mejor protección para la insuficiencia renal. La buena hidratación es la mejor profilaxis contra el daño renal secundario a medios de contraste, cisplatino, AINEs, rabdomiolisis, mieloma. Recordar que la isquemia renal y los nefrotóxicos se potencian especialmente en presencia de sepsis.

La sepsis induce isquemia a través de la combinación de vasodilatación sistémica y de hipoperfusión intrarrenal. Las toxinas generadas en un cuadro de sepsis sensibilizan las células tubulares renales frente a los efectos perjudiciales de la isquemia. El riñón recibe el 20% del gasto cardiaco y la médula menos del 10% del flujo renal. La PO2 en el segmento grueso del asa ascendente es de 10-20 mmHg, por lo tanto, los segmentos medulares son los más susceptibles al daño isquémico.

Seguimiento Derivar.

Fisiopatología: La isquemia y la hipoperfusión provocan alteraciones hemodinámicas que, a través de vasoconstricción y congestión medular, conducen a la marcada reducción del filtrado gromerular, y, daño tubular isquémico. En la NTA se produce una intensa y persistente vasoconstricción renal que reduce el FSR a menos del 50% de lo normal, hay una marcada congestión e hipoperfusión de la medula externa que persiste incluso cuando el flujo cortical mejora durante la reperfusión. Las 6

tipo V con un 17%, tipo III con un 10%, tipo VI con un 2% y finalmente tipo I con un 1%.

Glomerulopatía Lúpica

Diagnóstico

Nivel de manejo del médico general: Diagnóstico: Sospecha Tratamiento: Inicial Seguimiento: Derivar

La Nefritis Lúpica puede presentarse dentro de una variada gama, desde lesiones mínimas aisladas del sedimento urinario, síndrome nefrótico (lo más frecuente), síndrome nefrítico, insuficiencia renal aguda o crónica e hipertensión arterial.

Aspectos esenciales    

Los ANA están presentes en más del 90% de los pacientes con LES sin embargo no son específicos y sus títulos no se correlacionan con el daño renal.

Glomerulopatía secundaria producida por LES. Compromiso renal es igual en ambos sexos. Depósito crónico de complejos inmunes, reacción mensangial y proliferativa. Inmunosupresores en NL severa.

Los AC anti DNA doble cadena son más específicos y se correlacionan con daño renal, se pueden realizar por IF o por ELISA. Los AC anti DNA tipo IgG con gran afinidad por el complemento se asocian a mayor daño renal.

Caso clínico tipo Finalmente la confirmación diagnóstica se hace con biopsia renal.

Mujer de 24 años cursando su primer embarazo de 22 semanas, presenta artralgias en manos, sensación febril, edema en extremidades inferiores y cara; y orina oscura. Al ingreso: PA 145/90, pulso 92. El examen de orina muestra proteinuria en rango nefrótico, microhematuria y cilindros eritrocitarios. ¿Cuál es el examen que confirmaría su sospecha diagnóstica? R: Ac anti DNA.

Tratamiento NL clase I y II: Excelente pronóstico renal. No requieren terapia específica. NL clase III: Corticoides en altas dosis de corta duración si el componente proliferativo afecta pocos glomérulos, no hay lesiones necrotizantes ni formación de crecientes (buen pronóstico). Tratar como proliferativa difusa si la lesión en la biopsia es mayor.

Definición Glomerulopatía secundaria al Lupus Eritematoso Sistémico (LES) caracterizada por formación o depósito de complejos inmunes en los glomérulos, vasos sanguíneos y a lo largo de la membrana basal glomerular (MBG).

NL severa: Esquema de inmunosupresores: ciclofosfamida endovenosa en bolos, al inicio prednisona 1mg/kg/día por 4-8 semanas. Rituximab en caso de NL severa refractaria al tratamiento.

Etiología-epidemiología-fisiopatología Seguimiento

LES es más frecuente en mujeres (8-13: 1), pero el compromiso renal es igual en ambos. Un 60% de los pacientes con LES tendrá compromiso renal en algún momento. El compromiso renal se da por depósito crónico de complejos inmunes circulantes, que produce reacción mesangial y proliferativa. En esta última los complejos están formados por antígenos DNA unidos a Ig G con gran afinidad por el complemento lo que activa la cascada de inflamación. En otros pacientes los depósitos subepiteliales con unión de AC no activan el complemento. La clasificación actual se basa en patrones histológicos en microscopía de luz, electrónica e IF.

Clasificación (2003)

Nefritis

Lúpica

El tratamiento deberá ser inicial, el seguimiento es de resorte del especialista y los casos deben ser derivados.

ISN/RPS

I: Mesangial mínima II: Mesangial proliferativa III: Proliferativa focal (< 50% de los glomérulos) IV: Proliferativa difusa (> 50% de los glomérulos) V: Nefropatía membranosa. VI: Nefritis lúpica esclerosante primaria (Esclerosis > 90%). Dentro de la incidencia dentro de los tipos de nefirtis lúpicas (de los pacientes consultantes) la predominante corresponde a la de clase IV con un 57%, seguido de la 7

Alteraciones en la función de la ADH, central (poca producción) y nefrogénica (resistencia), aumentan la pérdida de agua libre de soluto a nivel de los túbulos colectores, siendo responsables de la poliuria y polidipsia característica de estos pacientes, y este último mecanismo el responsable de las natremias normalesaltas observables en ellos.

Hipernatremia, poliuria Nivel de manejo del médico general: DIagnóstico: Sospecha Tratamiento: Inicial Seguimiento: Derivar

Diagnóstico Aspectos esenciales     

Las causas más comunes de hipernatremia son cuadros infecciosos en pacientes con ingesta disminuída de agua, especialmente pacientes añosos. Descartando iatrogenia o ingesta excesiva de sodio, antecedentes de fiebre, vómitos y diarrea orientan a pérdidas insensibles; poliuria orienta tanto a diabetes mellitus descompensada como a diabetes insípida tanto central como nefrogénica.

Pérdidas insensibles de agua como mecanismo principal. Se debe evaluar presencia cuadros infecciosos: fiebre, vómitos, diarrea. En pacientes poliúricos evaluar antecedentes de diagnóstico DM, evaluar glucosuria. Osmolalidad urinaria orientador diagnóstico. Prueba de DDAVP diagnóstica entre polidipsia primaria y Diabetes Inspida central y nefrogénica.

Alteraciones neurológicas pueden observarse en hipernatremias marcadas o, más frecuentemente, ser causales de pérdida del reflejo de la sed. Osmolalidad urinaria nos permite determinar la indemnidad del eje ADH-renal: Osmur> 500 mOsm/kg, con poca respuesta a la administración de desmopresina (DDAVP) demuestran indemnidad de la función de ADH, lo que sumado a la presencia de glucosuria nos permiten diferenciar entre pérdidas insensibles o diuresis osmótica. Osmur< 300 mOsm/kg con un aumento superior a la mitad dos horas de administrada DDAVP orienta a diabetes insípida central, en cambio poca o ninguna alteración en Osmur tras DDAVP orienta a DI nefrogénica.

Caso clínico tipo Hombre, 84 años, semi-autovalente tras episodio de AVE, es traído por familiares tras episodios de alteración cualitativa de conciencia. NAC activa desde hace una semana. El paciente se muestra confuso témporoespacialmente, presenta signos de deshidratación y oliguria.

El diagnóstico diferencial entre DI central, nefrogénica, y polidipsia primaria se evalúa tras respuesta a DDAVP frente a restricción acuosa (o administración de sodio hipertónico, 5%): aumento de Osmur y disminución de Volur que no varían tras DDAVP demuestran polidipsia primaria. Osmur bajo y Volur elevado que no cambian tras DDAVP demuestran DI nefrogénica, en cambio un aumento de Osmur (hasta 500 mOsm) y disminución de Volur (> 50%) tras DDAVP demuestran DI central.

Definición Concentración plasmática de sodio por sobre 146 mEq/l. Una diuresis mayor a 3 l/día se define como poliuria, y es un signo clínico presente en muchos casos de hipernatremia.

Etiología-epidemiología-fisiopatología El aumento de sodio plasmático responde fundamentalmente a dos eventos: aumento de pérdidas de agua no reemplazadas por la ingesta, o administración de sodio hipertónico. Entre las pérdidas sin reemplazo de agua encontramos las pérdidas insensibles y por sudor (estados febriles, infecciones respiratorias), pérdidas urinarias (diabetes insípida (DI), diabetes mellitus descompensada, administración de manitol), pérdidas gastrointestinales y lesiones hipotalámicas del centro de la sed (raro). 



Tratamiento 





Las pérdidas por sudor, evaporación y diarreas no secretoras poseen una concentración de sodio y potasio menor a la plasmática, por lo consiguiente provocarán una elevación en la natremia. Si bien la hiperglicemia en diabetes mellitus descompensada si bien induce pérdidas de agua por arrastre de ésta a la luz tubular (diuresis osmótica), la hipernatremia producida en consecuencia de lo anterior puede estar “enmascarada” por el movimiento de agua desde el intracelular hacia el plasma producido por la misma hiperglicemia. El tratamiento con insulina en tales pacientes suele hacer evidente tal hipernatremia.

 

Corrección del déficit de agua mediante reposición lenta (no superior a 12 mEq/l/día) con suero glucosado o dextrosa al 5%. En pacientes con diabetes mellitus descompensada, preceder con administración de insulina y observar además para evitar hipokalemia. En pacientes con polidipsia primaria evaluar origen medicamentoso (fenotiazinas) y suspensión. En pacientes con DI central evaluar administración DDAVP o moduladores del eje ADH-riñón. En pacientes con DI nefrogénica es útil el uso de tiazidas y AINEs.

Seguimiento Por especialista.

8

3. VEC disminuido: Suero fisiológico. En caso de tratarse de una reagudización sobre un trastorno crónico deberá tratarse como una hiponatremia aguda grave.

Hiponatremia Crónica Asintomática

Seguimiento

Nivel de manejo del médico general Diagnóstico: Sospecha Tratamiento: Inicial Seguimiento: Derivar

Se debe corregir máximo 12 mEq/Lt al día. Si el paciente está asintomático o con síntomas leves, corregir a 0,5 mEq /Lt /hora.

Aspectos esenciales    

Na+ < 135mEq/L Asintomática por adaptación neural. Algoritmo Dg: Osm plasma > VEC > [Na]urinario. Corregir lentamente con medidas generales

Caso clínico tipo Paciente mujer de 76 años con antecedentes de HTA controlada con tiazidas. En uno de sus controles se pesquisa como hallazgo una hiponatremia con [Na+]=125mEq/L. No presenta sintomatología. Revisar la dosis de diuréticos, suspender y controlar en 2 semanas.

Definición [Na+] < 135mmol/L de instalación gradual en un período > 48 hrs, lo que permite una adaptación osmótica neuronal ocultando posibles síntomas.

Etiología-epidemiología-fisiopatología Las hiponatremias crónicas son principalmente hipotónicas, pudiendo presentar LEC disminuido (pérdidas de Na+ renales o extra renales), aumentado (disminución del volumen circulante efectivo que sobrepasa los mecanismos mediados por aldosterona para asegurar una presión de perfusión), o normal (SIADH, hipotiroidismo, entre otros).

Diagnóstico Los síntomas son principalmente neurológicos, náuseas, vómitos, letargia, confusión y coma, aunque los casos crónicos suelen ser asintomáticos por adaptación neural. El laboratorio es fundamental para un rápido diagnóstico con sólo 4 parámetros: osmolaridad plasmática; osmolaridad urinaria; Na+urinario; K+ urinario, que junto a la clínica orientan a la etiología.

Tratamiento Manejo muy cuidadoso para evitar mielinolisis pontina por corrección rápida. En primer lugar corregir la volemia, luego la osmolaridad según: 1. VEC aumentado: Furosemida. Restricción de Na+ y si es más intensa, restricción de agua. 2. VEC Normal: Restricción de agua. Remplazo hormonal (hipotiroidismo e hipocortisolismo). 9



Hipovolemia Nivel de manejo del médico general: Diagnóstico: Específico Tratamiento: Completo Seguimiento: Completo

Aspectos esenciales 

 

La clínica depende de la magnitud de la hipovolemia, de la velocidad de instalación y de la capacidad del organismo para compensarla. Es importante reconocerla porque afecta órganos noble, produciendo una afectación multisistémica. Tiene diferentes etiologías (sangrado, formación de un tercer espacio como ascitis, ingesta de líquido disminuido, o pérdidas aumentadas).

 



Caso clínico tipo Paciente con historia clara de pérdida de algún tipo de líquido, refiere sed. Al examen físico taquicardia, hipotensión, pérdida de turgencia e hidratación de piel y mucosas.

Territorio venoso: o Inspección de venas yugulares (la forma más práctica de evaluar el territorio venoso). Un paciente que en decúbito dorsal tiene las yugulares planas, refleja una disminución del LEC. Hay que considerar que es difícil de evaluar en personas obesas con cuello corto. También pueden estar ingurgitadas aún en presencia de un cuadro de hipovolemia, si hay falla ventricular derecha). En pacientes críticos se puede evaluar las siguientes: o Presión venosa central. o Presión del capilar pulmonar. Territorio arterial. Signos de hipoperfusión. Se observa en la alteración del estado de conciencia, disminución de diuresis con concentración de la orina. Una buena forma de evaluarla, es determinar cómo esta afecta al riñón. Un aumento de la creatinina plasmática, con aumento del nitrógeno uréico reflejan hipovolemia a nivel renal. Signos de concentración sanguínea. El aumento brusco del hematocrito refleja una disminución del VEC (ej. Hcto que aumenta de 45 a 50%, reflejan una reducción del VEC de un 10%) Es importante puesto que se puede sobreestimar el hematocrito de un paciente (incluso un paciente con anemia podría tener un hematocrito normal, pero hay que recordar que es por hemoconcentración y que el paso de los días refleja el verdadero hematocrito).

Definición Tratamiento

Contracción del líquido extracelular (LEC).

El tratamiento va enfocado a la reposición de volumen, con cristaloides, coloides, hemoderivados dependiente de la etiología.

Etiología-epidemiología-fisiopatología La depleción de volumen resulta de la pérdida de Na+ y agua desde los siguientes sitios anatómicos: pérdidas gastrointestinales (vómitos, diarrea, sangrados), pérdidas renales (efecto de diuréticos, diuresis osmótica, nefropatías e hipoaldostenorismo), pérdidas a travé de la piel (sudoración, quemaduras) y secuestro por formación de un tercer espacio (obstrucción intestinal, pancreatitis aguda).

Seguimiento Completo.

La contracción del LEC, refleja una reducción del contenido de sodio. Para investigarla hay que evaluar:   

Relación entre continente-contenido intravascular (se explora el lado venoso y arterial). Gasto cardíaco (se evalúa por la perfusión sanguínea). Signos de concentración sanguínea (se refleja con un aumento del hematocrito).

Diagnóstico El diagnóstico es clínico. La clínica va a depender del grado de hipovolemia, de la velocidad de instalación de esta y la respuesta del organismo a esta mediante la vasoconstricción. Es importante reconocerla de forma precoz para evitar el daño que se produce en los órganos nobles producto de la hipoperfusión. En el examen físico se debe evaluar:

10



Infección Urinaria Baja (Cistitis) Nivel de manejo del médico general: Diagnóstico: Específico Tratamiento: Completo Seguimiento: Completo

Tratamiento Cistitis aguda no complicada (alternativas de tratamiento):  

Aspectos esenciales    

Urocultivo: Indicado en duda diagnóstica o en sospecha de cistitis complicada. Se considera positivo en ≥105 UFC en paciente con síntomas característicos y piuria.



Clínica típica: Disuria, polaquiuria y tenesmo vesical. Frecuente en mujeres por la cercanía con esfínter anal. Enterobacterias, la más importante E. coli. Tratamiento ATB por tres días.

    

Caso clínico tipo

 

Paciente femenina de 23 años, acude al consultorio refiriendo ardor al orinar y aumento de la frecuencia miccional, pero en baja cantidad. Al examen físico se encuentra dolor frente a la palpación de la región suprapúbica.

Trimetroprim 300mg-400mg c/24 hrs x 3 días. Clotrimoxazol 160mg-800mg c/12 hrs x 3 días. Es preferible utilizar Trimetroprim exclusivamente. Nitrofurantoína 50mg-100mg c/8 hrs x 3 días. No es efectiva contra Proteus spp. Norfloxacino 400mg-800mg c/8 hrs x 3 días. Muy eficaz. Ciprofloxacino 250mg-500mg c/12 hrs x 3 días. Ofloxacino 200mg-400mg c/12 hrs x 3 días. Levofloxacino 250mg-500mg c/24 hrs x 3 días. Cefalexina 250mg-500mg c/8 hrs x 3 días.Útiles en caso de insuficiencia renal. Cefuroxima 250mg c/12 hrs x 3 días. Amoxicilina/Ác. Clavulánico 500mg/125mg c/12 hrs x 3 días.

Infección bacteriana de la vejiga urinaria.

En una mujer embarazada se recomienda realizar urocultivo en el primer control del embarazo (antes de las 12 a 14 semanas). En caso de dar positivo se trata con Amoxicilina, Nitrofurantoína o Cefalosporina por 7 a 10 días. La Cistitis en el hombre se suele tratar con una Quinolona por 7 a 14 días.

Etiología-epidemiología-fisiopatología

Seguimiento

Definición



El 80% de las Infecciones urinarias agudas se deben a la bacteria E. coli. Otros agentes son bacilos gram negativos como Proteus y Klebsiella y, en ocasiones, Enterobacter. Se da en el 1-2% de las niñas en etapa escolar, lo que se ve aumentado hacia la adolescencia por el comienzo de la vida sexual.





Factores que aumentan la prevalencia de esta enfermedad: sexo femenino (40-50% presentará al menos una ITU en su vida) y actividad sexual, embarazo (4 a 10% sufren una ITU). Se puede clasificar en ITU complicada o no, según la presencia de factores de riesgo, obstrucción de la vía urinaria, disfunción vesical neurógena, reflujo vésicoureteral, diabetes u otros tipos de inmunosupresión, lo que se asocia a complicaciones (pielonefritis, absceso perirrenal), y gérmenes multiresistentes.

Diagnóstico El diagnóstico de Cistitis es clínico y con exámenes confirmatorios. Síntomas y signos de ITU Baja: Disuria, polaquiuria, nicturia, tenesmo, dolor suprapúbico y en ocasiones hematuria macroscópica. La presencia de fiebre >38,3°C, náuseas, vómitos y dolor a la palpación de la fosa lumbar, puede indicar una infección renal concomitante. Laboratorio: 

Orina Completa: leucocitos/mm3.

Piuria positiva en ≥ 10

11

El Urocultivo no está indicado en pacientes asintomáticos al término del tratamiento (salvo embarazadas, 1-2 semanas post-término del tratamiento). Si persisten los síntomas o recurren precozmente luego del término del tratamiento: realizar urocultivo y considerar diagnóstico alternativo. En recaídas luego de 2 semanas o más posttérmino de tratamiento, reevaluar desde el comienzo.

Preparación

Instalación de pigtail Nivel de manejo especialista

del

médico

general: Derivar

1. Explicar a paciente procedimiento a realizar, con su atorizaciñon en el consentimiento informado. 2. Realizar estudio de coagulación, examen de orina completa y urocultivo. 3. Vaciamiento de vejiga. 4. Aseptización de zona genital.

al

Introducción 



Procedimiento

El pigtail o catéter doble J es una sonda flexible, de pequeño calibre, caracterizado por presentar curvaturas multiperforadas en ambos extremos, que se instala en el sistema urinario, con un extremo en la pelvis renal y el otro en la vejiga, con el objetivo de restablecer (en caso de obstrucción) o asegurar (cirugías o riesgo de obstrucción) el flujo normal de la orina en las vías urinarias. Puede instalarse vía retrógrada o anterógrada. La vía retrógrada es la más utilizada y se realiza a través de un cistoscopio, y suele asistirse radiológicamente para asegurar la correcta ubicación del catéter y/o guías. La vía anterógrada, en cambio, requiere de una nefrostomía percutánea para acceder con un nefroscopio.



Instalación vía retrógrada. 1. Instrumentalización de vía urinaria con cistoscopio, distención vesical mediante irrigación con suero fisiológico. 2. Instalación de catéter ureteral con guía metálica bajo visión radioscópica hasta alcanzar la pelvis renal o el punto de obstrucción. 3. Se retira el catéter ureteral y se procede a instalar pigtail, deslizándolo por sobre la guía, hasta que su extremo anterior alcance el sistema colector renal. 4. Se retira la guía y se comprueba por radioscopia la ubicación y formación “en jota” del extremo superior del pigtail, y por vía endoscópica del extremo inferior en vejiga. 5. Retiro de instrumental para visualización endoscópica. 6. Sondeo vesical.

Indicaciones   



Urolitiasis obstructiva (tras intentos de litotripsia y/o litotomía). Estenosis del uréter (causa tumoral, caausa inflamatoria, causa postquirúrgica). Fístulas ureterales.

Instalación vía anterógrada 1. La técnica para la inserción anterógrada, después de realizado el trayecto renal percutáneo, sigue esencialmente los mismos pasos descritos anteriormente.

Seguimiento

Contraindicaciones  

Por especialista. Coagulopatías incorregibles. Alergia al contraste (relativa).

Complicaciones       

Hematuria transitoria Hematuria que requiere cirugía (1-3%) Infecciones (1-2,5%) Obstrucción Cálculos por presencia de pigtail Perforación o rotura de uréter Molestias vesicales tipo cistitis

Materiales o insumos necesarios        

Guantes, mascarillas, batas estériles. Apósitos, gasas, sábanas y paños estériles. Suero fisiológico 3L, sistema de irrigación. Jeringas 10, 20, 50cc. Guías y catéteres ureterales. Catéter Doble J. Cistoscopio, lubricante estéril. Contraste yodado y equipo de radioscopía.

12

Insuficienca renal aguda prostrrenal u obstructiva

TABLA 1 INTRALUMINAL INTRAPARIETAL COMPRESIÓN EXTRÍNSECA Fibrosis retroperitoneal (1, neoplasias, metisergida)

Nivel de manejo del médico general: Diagnóstico: Sospecha Tratamiento: Inicial Seguimiento: Derivar

Malformación congénita (riñón en herradura, uréter retrocavo)

Diagnóstico

Aspectos esenciales    

DISFUNCIÓN NEURÓGENA

La clínica es muy variable, con distintos grados de diuresis: anuria, o anuria-poliuria intermitentes, globo vesical. Ella depende de la etiología y por la propia azoemia. El diagnóstico no debiese ser un problema dado el algoritmo de manejo de toda insuficiencia renal que consiste en una ecografía renal (dilatación de la vía urinaria) e instalación de una sonda Foley (poliuria post obstructiva si era debido a patología prostática).

Azoemia por obstrucción al flujo urinario. Hiperplasia prostática benigna (HPB) principal causa. Ecografía diagnóstica. Aportar volumen por poliuria osmótica post obstructiva.

Caso clínico tipo Tratamiento

Paciente hombre de 68 años llega confuso y con dificultad respiratoria. Relata cuadro de dolor lumbar, tomó analgésicos y a las horas se agregó disnea. No ha orinado en todo el día. El paciente cuenta ser monorreno. Los exámenes muestran elevación de la creatininemia y la ecografía una litiasis coraliforme con hidronefrosis.

Es suficiente con el sondaje vesical en la uropatía obstructiva baja, si es más alta puede ser necesaria la instalación de un catéter ureteral, cistostomía o nefrostomía (requiere evaluación por urólogo).

Seguimiento

Definición

Luego de la descompresión de la vía urinaria se debe evitar la pérdida de volumen producto de la poliuria osmótica post obstructiva con solución salina isotónica (0.9%) de forma inicial y, más tarde, al 0.45% más aporte de K+ (por las pérdidas renales aumentadas).

Aquella insuficiencia renal aguda (IRA) dada por una obstrucción del flujo urinario en la vía urológica (distal al parénquima renal). Requiere obstrucción del drenaje urinario de ambos riñones o de un riñón en el caso de monorrenos funcionales.

Epidemiología-etiología-fisiopatología. Representa menos del 5% de los casos de IRA. Las etiologías se muestran en la Tabla 1. Es más frecuente en pacientes de edad avanzada por enfermedad de próstata. La obstrucción aumenta la presión hidrostática de forma retrógrada intratubular en el espacio de Bowman, ello disminuye el filtrado glomerular, activa el eje reninaangiotensina y éste lleva a una vasoconstricción renal secundaria que reduce el flujo sanguíneo y, más aún, la función renal. Independiente del nivel de la obstrucción, si ésta se mantiene, el daño renal se hace irreversible debido a atrofia tubular. Etiología de IRA postrenal según nivel al que se encuentre la obstrucción. TABLA 1 INTRALUMINAL INTRAPARIETAL COMPRESIÓN EXTRÍNSECA Estenosis Litiasis Coágulos Tumores

Tumores Malformaciones congénitas

Hoperplasia prostática

Adenocarcinoma Disfunción de la unidad prostático pielouretral Tumores

Malacoplaquia

DISFUNCIÓN NEURÓGENA Vegija neurógena

Reflujo vesicouretral

13

2. NIC generalmente oligosintomáticas, se hacen manifiestas en etapa de IRC.

Nefritis Tubulointersticial Aguda y Crónica

Laboratorio:    

Nivel de manejo del médico general: Diagnóstico: Sospecha Tratamiento: Inicial Seguimiento: Derivar

Hemograma: Hay eosinofilia en el 50% (excepto en NTI por AINEs). BUN/creatinina: Puede haber insuficiencia renal. GSV: Puede haber acidosis metabólica Orina: proteinuria, leucocituria por piuria, eosinofiluria (excepto NTI por AINEs)

Imágenes: Ecografía, TAC.

Aspectos esenciales     

Biopsia: Confirma el diagnóstico (solicitar en caso de incertidumbre o si no mejora a los días de retirado el fármaco).

Principalmente por fármacos (AINEs y ATB). IRA asociada a reacción alérgica. Biopsia da diagnóstico confirmatorio. Suspender el fármaco sospechoso. Corticoides en casos seleccionados.

Tratamiento 1. NTI Aguda: La suspensión del agente causal, habitualmente es suficiente para la recuperación completa, que en algunos pacientes ésta puede durar varias semanas. Si esto no es suficiente, de segunda línea son corticoides (cuando hay marcado infiltrado intersticial o insuficiencia renal grave), que disminuiría el tiempo de recuperación y disminuiría necesidad de diálisis. 2. NTI Crónica: Tratamiento depende de la causa. Se debe manejar la HTA e insuficiencia renal crónica asociada.

Caso clínico tipo Hombre de 24 años se presenta en el servicio de urgencia por CEG de varios días de evolución. Al examen presenta asterixis, febrícula y exantema en el dorso. En sus exámenes destaca creatinina (1,7) y eosinofilia. Examen de orina: leucocitos ++. Familiar cuenta que se encontraba en tratamiento con antibióticos.

Definición Seguimiento

Heterogéneo grupo de desórdenes que se caracterizan por alteraciones histológicas y funcionales que implican a los túbulos y el intersticio en mayor grado que a los glomérulos y la vasculatura renal. Se clasifican en Agudas (NIA) o Crónicas (NIC).

Tres signos mayores de irreversibilidad: infiltrado intersticial difuso, retraso en respuesta a prednisona, persistencia insuficiencia renal >3 semanas.

Etiología-epidemiología-fisiopatología Existen 5 mecanismos de daño: Depósito de complejos inmunes; autoanticuerpos contra la membrana basal tubular; hipersensibilidad tipo I; mecanismos de inmunidad celular; y por linfoquinas e interleuquinas. 1. Etiología NIA: Drogas (tipo alérgica por AINEs y ATB principalmente 71%); Infecciosas (NO parenquimatosas 15%); Autoinmune (granulomatosas, Sjögren, Sd Tinu 1%) e idiopático (8%). 2. Etiología NIC: incluye a las anteriores más enfermedades genéticas (Alport, poliquística), enfermedades metabólicas, entre otras.

Diagnóstico Clínica: Primero buscar el antecedente de ingesta de fármaco, descartar otras causas de IRA y sedimento urinario. La presencia de un Sd. Tubular es sugerente (Fanconi, acidosis tubular). 1. NIA suele cursar con nicturia y poliuria, fiebre o exantema cutáneo, petequias, poliartralgias, hematuria.

14

Osteodistrofia Renal Crónica

en

Tratamiento

Insuficiencia

Tratamiento debe considerar principalmente: dietas hipofosfémicas, quelantes del fósforo (con calcio o resinas sin calcio. Nunca hidróxido de aluminio), Calcitriol Oral o ev (Cuando PTH sobre 250pg/mL. Se debe vigilar Ca y P), y Paratiroidectomía de ser necesario (Resistencia a tratamiento).

Nivel de manejo del médico general: Diagnóstico Sospecha Tratamiento Inicial Seguimiento Derivar

Hay que tener en mente que una mala terapéutica, puede producir Enfermedad Ósea Adinámica (Por exceso de Calcio y Vitamina D).

Aspectos esenciales     

Seguimiento

Causada por progresión de ERC. Todo afectado la padecerá a lo largo de su evolución. Es importante buscar dolor óseo, deformidades esqueléticas, esguinces y fracturas. Importante preguntar tratamiento actual. El tratamiento puede requerir cirugía.

Derivar.

Caso clínico tipo Paciente masculino de 63 años con antecedente de IRC diagnosticada hace 3 años. Refiere dolores óseos constantes y que varias veces se ha esguinzado, fracturándose en una oportunidad. Al exámen físico se aprecia leve deformidad ósea facial.

Definición Alteraciones Osteometabólicas Enfermedad Renal Crónica.

derivadas

de

la

Etiología-epidemiología-fisiopatología Se debe a la progresión de la Enfermedad Renal Crónica. La ERC está aumentando su prevalencia, y todo afectado padecerá de Osteodistrofia por esta causa. La enfermedad Renal Crónica produce un deficit de vitamina D activa (1,25 dihidroxicolecalciferol) y un hiperparatiroidismo secundario, el cual produce alteraciones en la homeostasis del Ca y P y alteraciones osteometabólicas.

Diagnóstico 





Anamnesis: Edad, sexo, sintomatología de la ERC, dolores óseos, prurito, miopatía proximal, antecedentes personales (Enfermedad renal crónica, diabetes mellitus, hipertensión arterial, entre otras), y familiares (Antecedentes mórbidos de importancia). Es importante buscar los antecedentes de tratamiento para la ERC (tratamiento dietario, farmacológico, diálisis, entre otros). Ex. Físico: Hay que evaluar signos de ERC, y buscar: deformaciones esqueléticas, ruptura de tendones, fracturas espontáneas, entre otras. Ex. Complementarios: Exámenes generales, medición de Calcio, Fósforo y PTH, radiografía e histología.

15

Diagnóstico

Pre-eclampsia

El diagnóstico se basa en la presencia de la diada: 

Nivel de manejo del médico general: Diagnóstico: Sospecha Tratamiento: Inicial Seguimiento: Derivar



Se clasifica en moderada o severa en base a la gravedad de la HTA y proteinuria, y la presencia de síntomas y signos que manifiesten compromiso de otros órganos. Criterios de gravedad

Aspectos esenciales  



Se caracteriza por HTA, proteinuria y edema luego de las 20 semanas de gestación. Alteración inmune que produce isquemia placentaria y liberación de factores vasoconstrictores que provocan lesión endotelial diseminada. La cura definitiva es el parto.

        

Caso clínico tipo Mujer de 20 años, en su semana 34 de gestación, consulta al SU por edema y cefalea. Al ex físico presenta PA 140/90 y proteínas (+) en sedimento urinario.



Definición

La cura definitiva de la preeclampsia es el parto o la interrupción del embarazo. Las decisiones deben ser compartidas con el equipo obstétrico y neonatológico. PE Moderada: Al detectarse hospitalizar, control estricto de signos vitales,realizar estudio completo y evaluación de UFP. Evaluar uso de antihipertensivos: metildopa, hidralazina y labetalol. No se recomienda antagonistas del Calcio por riesgo teratogénico ni diureticos. Interrumpir a las 37-38 semanas.

Etiología-epidemiología-fisiopatología

PE Severa: Hospitalizar, estabilizar y realizar exámenes para evaluar repercusión y diagnóstico precoz de síndrome HELLP, inducción de madurez pulmonar con corticoides si corresponde, uso de sulfato de magnesio E.V. para prevenir la eclampsia, uso de antihipertensivos, Sonda Folley en los cuadros más graves o si evidencia oliguria, evaluación de condición fetal, interrumpir a las 34 semanas o si hay deterioro materno y/o fetal agudo interrumpir de inmediato. Vía interrupción: la vía del parto debe ser preferentemente vaginal, anestesia de conducción, cesárea solo con indicación obstétrica.

La etiologia de la preeclampsia es desconocida. Pero se conocen múltiples factores de riesgo:



PA sistólica ≥ 160 mmHg. PA diastólica ≥ 110 mmHg. Elevación de enzimas hepáticas (GOT > 70 U/l y LDH > 600 U/l), dolor epigástrico o vómitos. Cefalea o alteraciones visuales. Proteinuria ≥ 2 g/24 hr. Hemorragia retiniana, exudado en fondo de ojo o papiledema. Crea > 1,2 mg/dl. Edema pulmonar. Plaquetas < 100.000 o anemia hemolítica microangiopática. Sindrome HELLP: Hemolisis, Elevacion enzimas hepaticas(Liver) ,Plaquetopenia (Low Platelets)

Tratamiento

También conocida como hipertensión proteinúrica gestacional, es una enfermedad inducida por el embarazo. Caracterizada por: HTA, proteinuria y edema, luego de las 20 semanas de gestación, aunque en el 90% de los casos aparece después de la semana 32. Afecta principalmente a primigestas sin antecedentes cardiovasculares o renales. Se clasifica en moderada y severa.



Hipertensión arterial (HTA) (>140 mmHg de sistólica o >90 mmHg de diastólica, tomada en posición sentada) Proteinuria (>0.3 gr. en 24 horas).

Factores Placentarios: Necesita placenta para producirse, relacionado con defecto en la placentación y un fallo en la reorganización de las arterias espirales. Factores Maternos: Nuliparidad, obesidad, antecedentes familiares de preeclampsiaeclampsia, pre-eclampsia en gestación previa, hipertensión crónica, enfermedad renal crónica, DM gestacional, gestación múltiple y la presencia de trombofilias.

Seguimiento Tras resuelto el embarazo, debe confirmarse la desaparición de la hipertensión arterial y proteinuria al término del puerperio (semana 6 post parto). De persistir anormalidades, debe sospecharse una nefropatía subyacente.

Su frecuencia oscila entre 6 a 8% de los embarazos en países desarrollados. En mujeres con factores predisponentes, se produce una alteración inmune que produce isquemia placentaria y liberación de factores vasoconstrictores como el tromboxano, provocando una lesión endotelial diseminada que da lugar a los diferentes síntomas y signos de la enfermedad.

16

Progresión de la Insuficiencia Renal Crónica

Ex. Físico: Se deben evaluar la presencia de los distintos factores de progresión de IRC, y evaluar la evolución de los signos de IRC. Ex. Complementarios: Hemograma, electrolitos plasmáticos, creatininemia y uremia, glicemia (considerando hemoglobina glicosilada), orina completa, entre otros.

Nivel de manejo del médico general: Diagnóstico: Sospecha. Tratamiento: Inicial. Seguimiento: Derivar.

Tratamiento Aspectos esenciales     

Corresponde a un manejo integral: Adquisición de hábitos de vida más saludables, manejo dietético, terapia farmacológica, derivación a especialista, e iniciación de terapia de remplazo cuando sea necesario.

Principal factor etiológico es la proteinuria. Es muy importante conocer sus mecanismos y tratarlos. Importante buscar factores de injuria renal en la anamnesis. Buscar síntomas y signos de progresión de IRC. Su tratamiento involucra cambios de estilos de vida.

Seguimiento Derivar.

Caso clínico tipo A su consulta llega, a control médico, un paciente varón de 48 años de edad con antecedentes de Diabetes Mellitus 2, de 2 años de evolución y aparentemente bien controlada. Al examen físico no se aprecian alteraciones de importancia, pero a entre los exámenes complementarios, destaca un aumento importante de la mircoalbuminuria.

Definición Empeoramiento del daño renal y declinación más rápida de la función renal; de manera que considera tanto el ámbito clínico como el de laboratorio o alteraciones radiológicas - ecográficas.

Etiología-epidemiología-fisiopatología Se debe a múltiples factores, entre los que destaca la proteinuria. La Enfermedad Renal Crónica ha aumentado en el tiempo, por lo cual es de gran importancia conocer y tratar su progresión. Los factores de progresión a través de diversos mecanismos, producen finalmente daño renal y disminución de la función renal.

Diagnóstico Anamnesis: Edad, sexo, factores que promuevan la proteinuria e inflamación glomerular (Por ejemplo: diabetes mellitus), hipertensión arterial, trastornos del metabolismo de los lípidos (Hiperlipidemias, entre otras), Obesidad, Sd. Metabólico, hiperglicemia, acidosis metabólica, hiperuricemia, raza negra y tabaquismo. Además, se deben buscar síntomas de las alteraciones hidroelectrolíticas, Cardiovasculares, Neurológicas, Dermatológicas, Osteometabólicas, Hematológicas, Gastrointestinales y del Sd. Urémico del IRC. 17

Por último, proteinuria de sobrecarga se expresa secundaria a un aumento plasmático de proteínas que supera la tasa de reabsorción tubular; un ejemplo común es la excreción de cadenas livianas por la hipergammaglobulinemia en mieloma múltiple.

Proteinuria Nivel de manejo del médico general: Diagnóstico: Sospecha Tratamiento: Inicial Seguimiento: Completo

Diagnóstico La medición de la excreción diaria de proteína por medio de la recolección de orina en 24 horas es el método de medición tradicional y el gold standard para la cuantificación de la proteinuria. Sin embargo, este método ha probado ser poco práctico y complejo de realizar en la práctica clínica, por lo que dos alternativas son las más utilizadas comúnmente:

Aspectos esenciales    

Presenta múltiples etiologías. La proteinuria glomerular es la causa más prevalente. Cuantificación mediante relación proteína total/creatinina en muestra de orina aíslada. Pesquisar y seguir microalbuminuria en pacientes diabéticos.



Caso clínico tipo

Tiras reactivas: corresponden a una objetivación semicuantitativa sobre una muestra de orina aislada (detecta principalmente albúmina y varía desde 15-30 mg/dl “+” hasta >500 mg/dl “++++”). El screening poblacional con tiras reactivas solamente está indicado en pacientes con riesgo de enfermedad renal crónica. Si es negativo o sólo muestra indicios, está indicado determinar el cuociente albúmina/creatinina en muestra aislada de orina. Si es positivo, debe calcularse el cuociente proteína/creatinina y, de verificarse positivo, está indicada una evaluación diagnóstica renal.

Hombre, 16 años, consulta por edema periorbitario y orina color marrón. No refiere antecedentes mórbidos de importancia salvo cuadro de infección de vía aérea alta hace dos semanas, tres días de duración, con fiebre y CEG, con resolución espontánea. Al examen físico se constata PA 150/110, edema facial, leve dolor garganta. Laboratorio: BUN = 32 mg/dl Crea =2,1 mg/dl Albúm. = 3,7 g/dl Tira reactiva =+ Sedim. orina: múltiples eritrocitos dismórficos, con cilindros hemáticos.



Definición El rango de excreción normal de proteínas por vía urinaria es de 40 a 80 mg/día, con un límite superior de 150 mg/día. Excreción de una cantidad superior de este límite constituye la presencia de proteinuria.

Relación proteína total/creatinina (en orina): son un buen indicador (e.g. si la muestra posee 260 mg/dl de prot. y 40 mg/dl de creat., la proteinuria sería aproximadamente 6,5 g/día/1,73m²). También hay otras variantes como el uso de relación albuminuria / creatininuria.

Los rangos de presentación de proteinuria son: * 150 a 3500 mg/día → proteinuria en rango no nefrótico * > 3500 mg/día → proteinuria en rango nefrótico

Etiología-epidemiología-fisiopatología Tratamiento

De acuerdo al origen del exceso de proteína.

Se debe realizar de acuerdo a patología base. En proteinuria de origen glomerular utilizar medidas antiproteinuricas: bloqueo del eje renina-angiotensinaaldosterona y dieta con restricción proteica moderada.

1. Proteinuria glomerular: nefropatía diabética, glomerulonefritis aguda postinfecciosa, nefropatía IgA, nefritis lúpica, amiloidosis, entre otras enfermedades glomerulares. 2. Proteinuria tubular: nefritis intersticial aguda, enfermedad renal poliquística, etc. 3. Proteinuria de sobrecarga: mieloma múltiple, otras discrasias de células plasmáticas.

Seguimiento Valores crecientes de proteína en orina indican progresión de la patología de base y son indicadores de evolución hacia insuficiencia renal crónica. Evaluar dirigdamente microalbuminuria en todo paciente diabético.

La proteinuria de origen glomerular ocurre debido a un aumento de la permeabilidad de pared glomerular, ya sea por perdida de la selectividad de la barrera de filtración o por inflamación y destrucción de la misma, secundario a depósito de inmunocomplejos y activación del sistema del complemento, anticuerpos anti-membrana-basal, ANCA, microangiopatía diabética o a enfermedades de depósito. La proteinuria de origen tubular ocurre debido a una disrupción del proceso de reabsorción de proteínas a nivel tubular, y generalmente no es un trastorno importante a menos que se asocie con otros defectos en la función tubular (pérdida bicarbonato, fosfato, etc.).

18

Tabla 1. Clasificación de los valores de Orina en 24 horas, Muestra Aislada de Alb/ Proteinuria : Creatininuria.

19

Riñon Poliquístico

La PQRI tiene una herencia autosómica recesiva, la lesión se localiza en el cromosoma 6. Se presenta en 1 de cada 20000 RNV.

Nivel de manejo del médico general: Diagnóstico: Sospecha. Tratamiento: Inicial. Seguimiento: Derivar.

Diagnóstico Ecografía renal: a. EPQRA se ven riñones grandes, asimétricos, con grandes quistes en todo el parénquima, pudiendo también existir quistes en otros órganos, especialmente en el hígado y páncreas. b. PQRI se utiliza como diagnóstico prenatal (2430 semanas) o postnatal; se observan riñones simétricos con superficie lisa, microquistes y nefromegalia bilateral.

Aspectos esenciales 

 

 

La Enfermedad poliquística renal del adulto (EPQRA) presenta como presentación clínica: poliuria, hematuria, HTA, masa palpable en flanco y poliglobulia. EPQRA se asocia con aneurismas cerebrales, quistes hepáticos y diverticulosis colónica. La Poliquistosis renal infantil (PQRI) es una enfermedad autosómica recesiva y se asocia a fibrosis hepática Ambas se diagnóstican por ecografía. Generalmente es un hallazgo. El tratamiento es el control de la HTA, infecciones, hemoglobina y sangrado de los quistes, y cálculos renales.

Estudio genético: a. EPQRA realizar estudio genético en familiares de pacientes con enfermedad diagnósticada. b. PQRI estudio de elección para diagnóstico prenatal, también utilizado en diagnóstico postnatal.

Tratamiento Caso clínico tipo

Es el tratamiento propio de la ERC (destaca control de HTA) y para evitar las complicaciones de los quistes (infecciones y sangrado).

EPQRA: Paciente de sexo masculino, de 60 años, hipertenso que consulta por sintomatología sugerente de enfermedad renal crónica, al examen físico presenta masa palpable en ambos flancos. Al hemograma se constata poliglobulia. Tiene antecedente familiar de poliquistosis renal.

En la PQRI además se debe incluir además tratamiento para la fibrosis hepática. Los antihipertensivos de elección son los IECA. En el caso de ser necesario evaluar transplante hepatorrenal.

PQRI: Paciente pediátrico con enfermedad renal crónica rápidamente progresiva, con hepatoesplenomegalia e hipertensión portal. Sin antecedentes familiares de poliquistosis renal.

Seguimiento Por especialista.

Definición Enfermedad poliquística renal (EPQR), es una enfermedad genética progresiva de los riñones. Existen dos formas de presentación principales: Enfermedad poliquística renal del adulto (EPQRA) y la Poliquistosis renal infantil (PQRI). La EPQRA es un trastorno multisistémico en el que se desarrollan quistes en el tejido renal, que progresivamente ocupan el parénquima normal de éste. Presenta alta asociación con la presencia de aneurismas cerebrales, quistes hepáticos y diverticulosis colónica. La Poliquistosis renal infantil se trata de una combinación variable de quistes múltiples en ambos riñones y fibrosis hepática congénita.

Etiología-epidemiología-fisiopatología La EPQRA tiene una herencia autosómica dominante en genes que codifican la poliquistina. Se presenta en 1 de cada 400 a 1000 RNV.

20



Urolitiasis



Nivel de manejo del médico general: Diagnóstico: Sospecha. Tratamiento: Inicial. Seguimiento: Derivar.

El 80% de los cálculos corresponden a litiasis cálcica (oxalato cálcico, fosfato acido de cálcico) mientras que el 20% a litiasis no cálcica (Ácido úrico, estruvita, cistina, indinavir).

Aspectos esenciales    

Modificaciones en el pH urinario. Un pH urinario bajo favorece la formación de cálculos de ácido úrico. Un pH alcalino favorece la de fosfato cálcico. Disminución o modificación en los inhibidores urinarios de la cristalización o de la agregación cristalina.

Son factores de riesgo para la formación de cálculos de calcio:

El cólico renal es la manifestación más común. El examen de elección es el pieloTAC. En litiasis menores de 4 mm se preferirá el manejo médico. En litiasis mayores de 6 mm se preferirá iniciar con manejo quirúrgico.

        

Caso Clínico Tipo Paciente de 58 años con antecedente de HTA e HPB consulta en SU por cuadro de dolor lumbar de inicio súbito asociado a 1 episodio de vómito. Impresiona inquieto con imposibilidad de mantenerse sentado.

Hipercalciuria: > 200 mg /24 horas (40 a 60 % de los pacientes) Hiperuricosuria: > 800 mg/24 horas (25%) Hiperoxaluria : > 45 mg/24 horas Hipocitruria: < 250 mg/24 horas Volumen urinario: < 1 L pH ácido: < 5,5 Hipomagnesiuria: < 50 mg Cistinuria: > 250 mg/g Otros: Déficit de vitamina A, sedentarismo, malformaciones urinarias.

Diagnóstico Definición

El diagnóstico se sospecha ante la presencia de cólicos nefríticos, aunque exiten "calculos silentes" que se encuentran al realizarse exámenes imagenológicos por otros motivos.

Urolitiasis es la presencia de cálculos (masas sólidas compuestos por microcristales) en cualquier parte de la vía urinaria.

A. Cólico renal: Dolor lumbar de inicio súbito, frecuentemente muy intenso, que logra despertar al paciente. Se acompaña de intranquilidad psicomotora, distensión abdominal, náuseas y vómitos. Esta sintomatología se explica por la sobredistensión de la capsula renal y su inervación parasimpática. El diagnóstico diferencial debe ser con cólico biliar, pancreatitis aguda, apendicitis aguda, salpingitis, diverticulitis, lumbalgia entre otros.

Etiología-epidemiología-fisiopatología Epidemiología: Los cálculos renales son muy comunes, aproximadamente 15% de la población entre 25 y 55 años los presenta. Ocurre 2 a 3 veces más en hombres que mujeres. Más frecuentes entre los 40 - 60 años con un peak en los 50. Se proponen etapas en el proceso de formación y crecimiento del cálculo.

B. Síndrome miccional irritativo: Se produce cuando el cálculo se encuentra en el uréter yuxtavesical o intramural, lo que origina inflamación e irritación, que se traduce en poliaquiuria, tenesmo, disuria. Diagnóstico diferencial con ITU, aunque muchas veces son concomitantes.

1. Sobresaturación de la orina. 2. Germinación cristalina. 3. Aumento de tamaño de las partículas formadas (crecimiento de los cristales o por la agregación de estos entre sí). 4. Retención de una o varias de las partículas formadas en las vías urinarias.

C. Hematuria: De cuantía variable, aunque suele presentarse en el 90% de los casos, muchas veces pasa inadvertida por ser microscópica, aunque la ausencia de ella no descarta la litiasis.

Se han identificado 3 vías de formación de cálculos: 

 

Sobrecrecimiento de las placas intersticiales de apatita (formación idiopática de cálculos de oxalato cálcico, hiperparatiroidismo primario). Depósitos en forma de cristales en los túbulos (casi todas las causas de litiasis). Cristalización libre en solución (cistinuria, hiperoxaluria).

A pesar de las muchas características clínicas, el diagnóstico se confirma con la evidencia de la formación o expulsión del cálculo. Imágenes:

Mecanismos que intervienen en la formación de cálculos: 

Aumento de la concentración urinaria de los componentes del cristal (calcio, oxalato, fosfato) o disminución de la diuresis. 21



Radiografía renal y vesical simple: Solo útil para visualizar cálculos radio opacos, es decir preponderantemente cálcicos.



Ecografía: El aspecto típico es el de una imagen hiperecogénica con sombra sónica posterior.

Tiene mayor sensibilidad (84%), que la Rx de abdomen simple (54%) para detectar cálculos. Único útil en embarazadas. 

Dependiendo de la localización y tamaño del cálculo, su eliminación puede realizarse mediante litotricia extracorpórea mediante ondas de choque (LEOC), litofragmentación endourológica con energía ultrasónica, electrohidráulica o laser a través de ureterorenoscopio, endourología percutánea y cirugía convencional.

PieloTAC: Exámen de elección. Permite evidenciar un cálculo en cualquier parte del sistema urinario, tamaño y la densidad para decidir conducta. Es operador independiente, el tiempo de examen es corto, bajo artefactos en imágenes, costo relativamente bajo, altamente sensible y especifico, ventaja de pesquisar otras patologías.

Seguimiento Se realiza por especialista, con el fin de prevenir recurrencias.

Tratamiento

Los pacientes deben mantener una hidratación que permita una orina con densidad < 1.025, actividad física, disminuir el aporte proteico y de sal. No es necesario disminuir el consumo de calcio. Se debe realizar estudio metabólico en caso de recurrencia.

Depende de la ubicación, tamaño, cantidad de cálculos, forma y evaluar la posibilidad de expulsión.   

≤ 4 mm: Probabilidad de expulsión espontánea de 90% 5 a 6 mm: Probabilidad de expulsión espontánea de 50% > 6mm: Probabilidad de expulsión espontánea menor de 15%

La gran mayoría de los cálculos son pequeños (< 5mm) y lograrán ser expulsados sólo bebiendo abundante agua. En los pacientes con un episodio agudo de litiasis, la medida terapéutica más urgente es la analgesia. Se recomienda iniciar la analgesia con diclofenaco siempre que sea posible y utilizar un medicamento alternativo cuando persista el dolor (metamizol, tramadol). Los antiespasmódicos no tienen indicación en esta patología. A. El tratamiento expulsivo médico: 1. Medicas Generales: Analgesia, calor local, elevada ingesta de líquidos (que permita un aumento de la diuresis, procurando mantener una densidad urinaria < 1.025, que producirá una disminución de saturación urinaria), evitar sobrepeso, alimentación baja en sodio y proteínas (en caso de litiasis oxalocálcica). En caso de litiasis úrica, restringir alimentos ricos en ácido úrico y administrar alopurinol (en pacientes con función renal normal). En caso de cálculos de estruvita la efectividad de tratamiento médico pasa por la exéresis total del cálculo. Recidiva del 20%. 2. Farmacológico específico: Se basa en los efectos beneficiosos de ciertos medicamentos que contribuyen a la relajación del músculo liso ureteral. Se utiliza Tamsulosina (0,4 mg). Se ha planteado el uso asociado de corticoides para disminuir el edema local. Se considera un plazo de 1 mes para lograr la expulsión espontánea del cálculo. B. Tratamiento quirúrgico: En la Guía clínica sobre la urolitiasis (European Association of Urology 2010) se plantea que las indicaciones de extracción activa son: Cuando el diámetro del cálculo es ≥ 7 mm (debido a una tasa baja de expulsión espontánea), cuando no se logra un alivio suficiente del dolor, cuando existe una obstrucción causada por la propia litiasis acompañada de infección, cuando existe riesgo de pionefrosis o sepsis urinaria, en riñones únicos con obstrucción y en la obstrucción bilateral.

22

Vasculitis o glomerulonefritis rápidamente progresiva





Nivel de manejo del médico general: Diagnóstico sospecha Tratamiento inicial Seguimiento derivar

Aspectos esenciales   



Diagnóstico

Las glomerulonefritis (GN) corresponden a la inflamación de los glomérulos. Son procesos de etiología inmunitaria. Si no se trata de forma oportuna, progresa con rapidez a una insuficiencia renal aguda en semanas. Focal vs Difusa: Hace referencia al grado de compromiso glomerular. La lesiones son focales si afecta <80% del glomérulo, y se consideran difusas si afectan >80% de este. Las glomerulonefritis se sospechan en hematuria con eritrocitos dismórficos o cilindros eritrocíticos +/ - proteinuria en rango no nefrótico con insuficiencia renal, HTA y edema. Sin embargo, pueden presentarse como síndrome nefrótico, hematuria macroscópica recurrente, IRA o ERC

La clínica que hace sospechar una glomerulonefritis es la presencia de hematuria con eritrocitos dismórficos o cilindros eritrocíticos +/ - proteinuria en rango no nefrótico con insuficiencia renal, HTA y edema.

Tratamiento Ante la sospecha de una GNRP, se debe administrar 1 gramo de metilprednisolona ev 1 vez al día por 3 días lo antes posible mientras se realiza estudio. A grandes rasgos, el tratamiento de las glomerulonefritis primarias se resumen en la siguiente tabla.

Seguimiento Derivar a nefrólogo

Caso clínico tipo Paciente que inicia cuadro de hematuria, proteinuria y, ocasionalmente,hipertensión arterial y edema. El cuadro clínico es compatible con un síndrome nefrítico (pero la proteinuria podría estar en rango nefrótico). Puede evolucionar con una insuficiencia renal en semanas.

Definición Las glomerulonefritis primarias se caracterizan por la afección única o predominantemente renal (que no es consecuencia de una enfermedad sistémica) Estas pueden cursar como un síndrome nefrótico, síndrome nefrítico, hematuria/proteinuria no nefrótica, hematuria macroscópica recurrente, insuficiencia renal aguda o insuficiencia renal crónica. Dentro de las glomerulonefritis primarias se encuentran:     

Glomerulonefritis con cambios mínimos Glomeruloesclerosis focal Glomerulonefritis membranoproliferativa (GNMP) Glomerulonefritis aguda postinfecciosa (GNA) Glomerulonefritis rápidamente progresiva (GNRP)

Esta última, la glomerulonefritis rápidamente progresiva es la inflamación de los glomérulos de etiología inmunitaria, que se caracteriza por la proliferación de la célula epitelial en el espacio extracapilar con formación de semilunas en el glomérulo. Existen tres tipos 

forma aislada o asociada a hemorragia pulmonar, en este último caso, se denomina enfermedad de Goodpasture) Tipo II (depósito de complejos inmunes, como en lupus eritematoso, glomerulonefritis postinfecciosa, púrpura de Schönlein-Henoch y nefropatía por IgA) Tipo III (es secundaria a vasculitis sistémicas asociadas con ANCA, como la granulomatosis de Wegener, la poliangitis microscópica o el síndrome de Churg-Strauss)

Tipo I (se caracteriza por la presencia de anticuerpos anti-MBG, puede presentarse de

23

24

MÓDULO 1: Medicina Interna

Neurología Compromiso De Conciencia •

Somnolencia: dormido, que despierta fácilmente a estímulo verbal.Somnolencia: dormido, que despierta fácilmente a estímulo verbal. • Sopor: es superficial, si requiere estímulo táctil; medio, si requiere dolor superficial y profundo si requiere estímulo nociceptivo profundo. • Coma: solo hay respuesta refleja. Sin embargo, en la práctica clínica es mejor describir a qué estímulo responde el paciente.

Nivel de manejo del médico general: Diagnóstico Específico Tratamiento Inicial Seguimiento Derivar

Aspectos Esenciales • • • • •

Causas: estructurales (TEC, ECV, CA, etc.) y no estructurales (tóxicas, metabólicas, infecciosas, inflamatorias) Objetivos: Asegurar vía aérea, ventilación y hemodinamia, definir la etiología. Exámenes iniciales: glicemia capilar, hemograma, imágenes, ECG. Tratamiento específico una vez estabilizado el paciente Los trastornos del potasio no producen alteraciones de conciencia

Fisiopatología Conciencia se puede definir como la capacidad de darse cuenta de uno mismo (medio interno) y del medio que lo rodea (medio externo), así como generar respuestas adecuadas a los estímulos de este. Se compone del entendimiento (contenido) y vigilia (nivel de conciencia). El primero depende de la corteza cerebral, y el segundo de la indemnidad del sistema reticular ascendente del tronco cerebral. El compromiso de conciencia es causado por patologías que afectan a distinto nivel el encéfalo, siendo causas estructurales de tipo tumoral, infecciosas, abscesos, traumas, hematomas, infartos y hemorragias. Causas no estructurales, van desde una alteración al equilibrio hidroelectrolítico, endocrinopatías, hasta meningitis y vasculitis. Coma: máximo compromiso de conciencia, pérdida de interacción voluntaria con el entorno, teniendo como única respuesta la refleja a estímulos.

Caso Clínico Tipo Mujer de 72 años, presenta compromiso de conciencia brusco. Familiares relatan que desde hace una semana paciente refería visión doble, presentaba dificultad al hablar y debilidad muscular generalizada. Al examen físico se observan pupilas mióticas, sin reflejos oculocefálicos ni oculovestibulares, y presenta postura de descerebración ante estímulos dolorosos.

Definición Alteración del nivel de conciencia, breve o establecida. Si bien no existe una clasificación universal del niveles de conciencia, se puede hablar de: •

Etiología •

Vigilia-alerta: apertura ocular y comunicación con el medio, respectivamente.

Estructural: lesión de diencéfalo, mesencéfalo, puente, bulbo (por una hernia uncal). - Simétrico: hemorragia subaracnoídea, ECV de tronco,

1 Facultad de Medicina, Universidad de Chile



trauma. - Asimétrico: hematoma subdural, ECV hemisférico, ECV tronco. No estructural - Tóxico: etanol, monóxido de carbono, hipoxia, etc. - Metabólico: hipoglicemia, hiperglicemia, Wernicke, hiponatremia, uremia.

histérico. •

Movimientos oculares espontáneos - Roving, tronco intacto - Bobbing ocular: movimiento rápido hacia abajo y retorno lento: lesión de puente - Desviación conjugada de la mirada: lesiones hemisféricas, estructurales, ojos desviados conjugadamente hacia lado de lesión. Lesiones irritativas o de tronco se desvían hacia contralateral



Posturas - De descerebración: extensión de miembros superiores e inferiores (planchado), lesión de núcleo rojo y núcleos vestibulares. - De decorticación: flexión de codo y extensión EEII, lesiones hemisféricas profundas o bilaterales (núcleo rojo indemne).

Diagnóstico Hay que indagar en la anamnesis enfermedad previa conocida (tratable: DM, DHC; no tratable: cáncer, enf. degenerativas, metástasis cerebrales), o desconocida (síntomas previos, interrogar testigos). Exámen físico: papiledema (HTE, encefalopatía hipertensiva); hemorragias subhialoídeas (HSA); rigidez de nuca (HSA, meningitis); hipopnea, hipotermia, miosis, marcas de punción (intoxicación por opiáceos); hipertermia, piel seca, midriasis (golpe de calor, intox. anticolinérgicos); aliento (OH, DM, uremia, cirrosis); Lengua mordida a lateral, hemorragia gingival (crisis convulsiva, tto fenitoína); rash petequial (meningococcemia, otras sepsis); equimosis extensas (hemorragia cerebral, TEC, Trastorno coagulación).

Exámenes complementarios: ECG, HGT o glicemia, hemograma, electrolitos plasmáticos, creatinina, perfil bioquímico, neuroimágenes (excepto ante en causa metabólica obvia).

Tratamiento

En la exploración neurológica, es importante evaluar: • Respiración - Cheyne-Stokes: lesión cortical difusa, tálamo, etapa inicial de hernia transtentorial, uremia, anoxia, insuficiencia cardíaca. - Hiperventilación neurógena central (Kussmaul): mesencéfalo, protuberancia; cetoacidosis, acidosis láctica, hipoxemia. - Respiración apnéustica: tegmento lateral de protuberancia inferior. -Respiración atáxica: lesión bulbar dorsomedial, sugiere próximo paro respiratorio. •

Pupilas - Pupilas simétricas, pequeñas, reactivas: alteración metabólica o diencefálica. - Pupilas simétricas, grandes, fijas: alteración tectal. - Pupila asimétrica dilatada, fija: alteración III par, herniación uncal, lesión estructural. - Pupilas simétricas, pequeñas fijas (midriáticas arreactivas): lesión mesencefálica. - Pupilas puntiformes, fijas (puntiformes arreactivas): lesión protuberancial.



Reflejos - Corneal ausente: disfunción de tronco en puente. - Oculocefálicos: normal si se mueve en dirección opuesta de cabeza. Respuesta desconjugada o arreflexia es lesión pontomecencefálica. También alterado por barbitúricos. - Óculovestibulares: en respuesta a frío. Fase lenta ausente es lesión de tronco, fase rápida ausente es lesión hemisférica. Si ambos son normales pensar en coma

Va a depender de la etiología. Sin embargo las medidas de soporte vital son importantes como tratamiento inicial.

Seguimiento Derivar a unidad de paciente crítico si procede.

Autor / Editor Paulina Lira

2 Facultad de Medicina, Universidad de Chile

Año 2016

Epilepsia

Caso Clínico Tipo Hombre de 45 años, es traído al servicio de urgencias tras haber sido encontrado en el suelo con desviación de cabeza y movimientos repetitivos de extremidad superior derecha hacia la derecha, con posterior presentación de movimientos descritos como sacudidas con pérdida de conciencia posterior. Se desconoce la duración del episodio. Paciente refiere posterior al cuadro se encontraba confuso y somnoliento. Al examen físico se evidencia cianosis leve y lesiones de mordedura de lengua en la zona lateral.

Nivel de manejo del médico general: Diagnóstico Sospecha Tratamiento Inicial Seguimiento Derivar

Aspectos Esenciales • • • • •

Condición de cronicidad de crisis epilépticas paroxísticas (2 o + de similar presentación). Causas: idiopatica o secundaria. Evaluar patrón de presentación y clasificación en simple o compleja, focal o de rápida generalización. Diagnóstico es clínico asociado a la presencia ideal de un testigo. Exámenes son complementarios. Descartar causas secundarias de urgencia, otras causas de alteración de conciencia o convulsiones y derivar.

Definición Corresponde a la recurrencia de crisis epilépticas paroxísticas sin tener un desencadenante identificable de inmediato, causadas por la descarga sincrónica y exagerada de un grupo de neuronas que puede progresar o no a la descarga sincrónica de ambos hemisferios.

Etiología Causas más frecuentes por edad: <20 años: asfixia perinatal, infección SNC, trastornos metabólicos, traumas, malformaciones arteriales, criptogénica, idiopáticas. 20-45 años: TEC, malformación arterio-venosa, infecciones

SNC (cisticercosis, abscesos cerebrales), abuso de sustancias, abstinencia de alcohol, idiopáticas. >45 años: tumores, enfermedad cerebro-vascular (ECV), cisticercosis, enfermedades neurodegenerativas.

3 Facultad de Medicina, Universidad de Chile

Causas Principales de Epilepsia:



1. Hereditarias: Epilepsias genéticamente determi nadas. 2. Congénita (Hereditarias o Adquiridas): • Displasias o disgenesias cerebrales • Algunos tumores cerebrales • Lesiones intraútero • Malformaciones vasculares • Sindromes neurocutáneos (neurofibromatosis, enfer medad de Sturge-Weber, esclerosis tuberosa) • Anomalías cromosómicas (síndrome de Down-triso mía 21, cromosoma 20 en anillo, síndrome de Angel man-deleciones en cromosoma 15, trisomía o micro deleciones en otros cromosomas) • Trastornos congénitos del metabolismo (aminoaci durias, leucodistrofias) • Miopatías congénitas tipo Fukuyama • Epilepsias mioclónicas progresivas 3. C. Adquiridas: • Traumatismos • Lesiones posquirúrgicas • Lesiones postinfecciosas • Infarto y hemorragias cerebrales • Tumores • Esclerosis del hipocampo (del lóbulo temporal) • Tóxicos (alcohol y otras drogas) • Enfermedades degenerativas (enfermedad de Alzheimer y otras demencias)

Enfermedades metabólicas adquiridas

Epidemiología • • •

17 por 1.000 hab. en Chile, con una incidencia de 114 por 100.000 hab. al año Frecuencia mayor en <20 y >60 años Mortalidad 2 a 3 veces la general.

Fisiopatología Las crisis son producidas por un desequilibrio excitatorio/ inhibitorio del SNC, que deriva en hiperexcitabilidad celular, mediado por aumento de entrada de Ca2+ y Na+ a la célula, activación de NMDA, y disminución del efecto GABA en un grupo focalizado de neuronas (crisis focal). Ésta puede propagarse a través de la excitación de neuronas contiguas por el desbalance de Neurotransmisores a otras áreas del cerebro, pudiendo tener compromiso de un hemisferio y afectar en forma bilateral (crisis de rápida progresión o generalizada). La clínica se producirá dependiendo de la zona inicialmente excitada a nivel focal, evolucionando posteriormente a compromiso de conciencia si llega a afectar ambos hemisferios.

Figura: Esquema del proceso diagnóstico en epilepsia. Neurología Fundamental. Alfredo Yáñez. Editorial Mediterráneo

4 Facultad de Medicina, Universidad de Chile

Tratamiento

La sospecha es eminentemente clínica. El elemento esencial para la sospecha es la presencia de un testigo. Se debe evaluar eventos desencadenantes asociados al episodio (consumo de medicamentos, alcohol, drogas, antecedente de trauma, cefaleas recurrente, etc). Se debe caracterizar el episodio y lograr al menos una clasificación del episodio de crisis, destacando el estereotipo de crisis en relación a cuadros similares anteriores. El diagnóstico definitivo lo hace el especialista.

La decisión de iniciar el tratamiento farmacológico corresponde al especialista una vez confirmado el diagnóstico. La meta es la reducción del número y severidad de crisis, con menor cantidad de efectos colaterales. El fármaco a elección depende del tipo de crisis. Para el inicio del tratamiento, se realiza en forma gradual de acuerdo a la posología correspondiente a cada medicamento, y considerar el balance entre la desaparición de crisis y efectos adversos. De no lograrse con una droga con dosis máxima tolerable, se decidirá cambiar por un segundo medicamento en forma gradual con retiro al momento de lograr los niveles terapéuticos con la segunda droga. Eventualmente de no conseguirse disminución de las crisis, evaluar combinación de medicamentos o asociación de un fármaco de segunda línea. • Crisis focales: Carbamacepina, Fenitoína, Lamotrigina, Levetiracetam. • Crisis Generalizadas Tónico Clónicas: Carbamazepina, Ac. Valprocio, Lamotrigina, Fenitoina, Fenobarbital. • Crisis Mioclónias: Ac. Valproico, Levetiracetam, Topiramato.

Se pueden clasificar las crisis inicialmente en: • Focales simples (o anterior parcial simple): se mantiene nivel de conciencia. Signo o síntoma característico del lóbulo comprometido. Pueden ser motoras (aumento de tono, movimientos repetitivos, automatismo), sensoriales (sonidos, cacosmia, escotomas o visión de halos), psíquicos (deja vú, miedo, despersonalización), con o sin síntomas autonómicos (piloerección, sudoración). Pueden progresar a generalizadas. Puede darse la llamada progresión Jacksoniana (compromiso inicial a nivel de extremidades inferiores con progresión hacia extremidades superiores o viceversa) con posterior parálisis o paresia de las extremidades comprometidas (Parálisis de Todd). •





Focal compleja (o anterior parcial compleja): crisis focal que se acompaña de algún nivel de alteración de conciencia. Puede presentar automatismos (ej. chupeteo), con confusión posterior. No confundir con crisis de ausencia.

En el caso de crisis en embarazadas, derivar inmediamente a alto riesgo y a control con neurología para cambio de medicamentos. Importante: NO UTILIZAR Carbamacepina, Oxcarbacepina, Gabapentina en crisis mioclónicas o de ausencia, ya que pueden desencadenar crisis.

De rápida progresión o generalizada: Se afectan simultáneamente a ambos hemisferios cerebrales desde un inicio. Tiene compromiso de conciencia. Corresponden a las anteriormente llamadas petit mal (crisis de ausencia) o gran mal (tónica-clónica). Puede ser tónico-clónica, tónico, clónica, ausencias o atonías. Suelen tener un periodo de corta duración, menor a 5 minutos (más de 5 minutos corresponde a un status epiléptico), con confusión posterior.

Siempre evaluar estudio neuroimagenológico para descartar lesiones de tipo estructurales, que sean eventualmente susceptibles de resecar en forma quirúrgica. Considerar también en casos refractarios al uso de medicamentos con alto indice de efectos secundarios.

Seguimiento

Mioclónica Juvenil: Se inicia con sacudidas de extremidades, generalmente a nivel de extremidades superiores, con movimientos repetitivos, cíclicos. Pueden asociarse a eventos tónico-clónicos o de tipo ausencia. Generalmente tienen como gatillante la abstinencia de sueño o el consumo de alcohol.

Diagnóstico final, tratamiento y seguimiento corresponden al especialista. Seguimiento en el caso de uso de Carbamacepina y Ac. Valproico con pruebas hepáticas y hemograma.

Para diagnóstico diferencial se pueden ocupar algunos exámenes: Electrocardiograma (síncope) y neuro-imagen (epilepsia de inicio tardío, de inicio focal, epilepsia refractaria, o epilepsia que cambia de patrón y tipo de crisis). Descartar eventos no epilépticos causales de pérdida de conciencia (síncope, accidente isquémico transitorio, hipoglicemia, crisis de pánico) o causales de movimientos anormales (temblor, distonías paroxísticas, vértigo paroxístico, etc.). El electroencefalograma no es requerido para realizar el diagnóstico, sino que es un método complementario al estudio etiológico.

Autor / Editor Paulina Lira

5 Facultad de Medicina, Universidad de Chile

Año 2016

Lumbago Mecánico Y Lumbociática

• • • • • • • •

Nivel de manejo del médico general: Diagnóstico Sospecha Tratamiento Inicial Seguimiento Derivar

Aspectos Esenciales • • • • •

Espondiloartrosis. Raquiestenosis. Síndrome de Leriche (ateroesclerosis aorto-ilíaca). Polineuritis múltiple. Esclerosis múltiple. Tabes. Siringomielia. Lumbociáticas de tipo psicógeno.

Epidemiología

El diagnóstico es clínico. Afecta a personas jóvenes en edad laboral, siendo una de las primeras causas de ausentismo laboral La hernia se produce frecuentemente en forma brusca en jóvenes y lenta en adultos mayores. Lo más importante del tratamiento es el reposo y rehabilitación precoz. Ante un cuadro de lumbociática que presenta signos de síndrome de cauda equina se debe derivar al paciente de forma urgente a un centro neuroquirúrgico

En las lumbociáticas, 96% de los afectados tienen la hernia a nivel de L4-L5 ó L5-S1. El 80% de las personas la tendrán en algún momento, de los cuales la mayoría son hombres.

Fisiopatología Ruptura de la parte posterior del anillo fibroso que rodea al núcleo pulposo, ya sea por sobrecarga o trauma, que permite la herniación de éste, comprimiendo la raíz.

Caso Clínico Tipo Diagnóstico

Paciente masculino de 40 años, panadero, con sobrepeso. Inicia hace tres meses, de manera progresiva dolor lumbar sordo, sin predilección horaria, que cede con analgésicos. En el último mes, el dolor se hace más intenso y se irradia a pierna izquierda por su cara posterior, cediendo al final del día con reposo. Al examen físico, fuerza y sensibilidad conservadas, con signo de Lásegue (+).

Es fundamentalmente clínico, en el caso de la lumbociática se presenta como dolor en la región lumbar irradiado hacia el trayecto del nervio ciático (región glútea, cara posterior del muslo, cara lateral de la pierna y del pie). El dolor característicamente aumenta con la maniobra de Valsalva. Al examen físico, observar marcha en talones y punta de pies, flexión de columna, en la lumbociática destaca la presencia del signo de laségue (dolor al elevar la pierna extendida a menos de 45°, con el paciente en decúbito supino). Signo de Gowers: Aumento del dolor ciático con la dorsiflexión del tobillo. Signo de O’Connell: Dolor en el nervio femoral al producir hiperextensión de la cadera. Se deben evaluar dermatomas de L4, L5 Y S1 en la lumbociática. Pueden también haber alteraciones motoras (parálisis, paresia) o de reflejos patelar, aquiliano y tibial posterior (hiporreflexia o arreflexia).

Definición Lumbociática: Dolor lumbar que se irradia siguiendo el trayecto del nervio ciático (región lumbosacra, región glútea y cara póstero - externa de la extremidad inferior).

Etiología Compresión radicular, que provoca el estiramiento de la raíz al realizar movimiento. Esta compresión es generalmente por hernia del núcleo pulposo, aunque puede también darse por causas traumáticas, infecciosas, inflamatorias, tumorales. Entre ellas: • Lesiones traumáticas tendinosas y musculares. • Fracturas vertebrales, luxofractura. • Espondilolisis. • Espondilolistesis congénita y adquirida. • Artritis reumatoídea, espondilitis. • Bursitis del trocánter mayor. • Tuberculosis, bruselosis. • Tumores intra y extradurales, tumores óseos y metástasis pelvianas. • Neuropatía diabética.

6 Facultad de Medicina, Universidad de Chile

Figura: Exploración de las raíces nerviosas del plexo lumbosacro Manual CTO Medicina y Cirugía. 1° edición. Chile. Neurología y neurocirugía

Tratamiento

Se solicita radiografía de columna lumbosacra de pie antero-posterior y lateral, y radiografía de quinto espacio en proyección lateral a fin de descartar problemas de alineación, espóndilolistesis lítica o ístmica, tumores u otras patologías especialmente visibles en radiografías.

El tratamiento del lumbago mecánico, incluye reposo relativo (no más de 3 días), uso de calor húmedo en área afectada (toallas húmedas calientes). Se puede usar AINEs (por periodos cortos, si no hay contraindicación) y relajantes musculares. Es importante realizar kinesioterapia y ejercicios para fortalecer la musculatura de la espalda después, y manejar aquellas condiciones favorecedoras (sobrepeso, mala posturas)

La RNM es el examen de elección. El TAC se solicita secundario a la RNM, para visualizar mejor el detalle óseo de la columna. Es importante correlacionar la clínica con la imágen, ya que se pueden evidenciar hernias de núcleo pulposo en una RNM y que no son las causantes del cuadro de lumbociática. El estudio neurofisiológico se solicita cuando la clínica no es clara para determinar el nivel y la magnitud del compromiso radicular y es solicitado por el especialista.

El tratamiento de la lumbociática es similar al lumbago mecánico, se puede agregar infiltraciones peridurales. Lo normal es que el 90 % responda a tratamiento médico en 6-12 semanas, sin embargo, si no mejora, si hay déficit progresivo o

7 Facultad de Medicina, Universidad de Chile

dolor invalidante se debe considerar cirugía

Seguimiento Derivar a neurólogo si el tratamiento conservador no es efectivo. En caso de signos sugerentes de síndrome de cauda equina, se debe derivar a una urgencia neuroquirúrgica.

Autor / Editor Daniela Bustos

Año 2017

8 Facultad de Medicina, Universidad de Chile

Migraña

Epidemiología •

Nivel de manejo del médico general: Diagnóstico Específico Tratamiento Completo Seguimiento Completo

• • • •

Aspectos Esenciales • • • • •

Un 12% de la población presenta migraña. Del total de personas migrañosas, alrededor de un 60% corresponde a mujeres. Se inicia en la adolescencia o en la adultez temprana. La migraña sin aura es más frecuente (70%). La teoría más aceptada es la trigémino vascular. Existen antecedentes familiares en 2/3 de los pacientes.

Subtipos Migraña con Aura o clásica: Se da en alrededor de 1 de cada 4 casos de migrañas, que se presenta como una cefalea hemicranea, pulsátil que puede ser acompañada de náuseas, vómitos, fotofobia sonofobia, siendo mucho menos común que se acompañe de disfasia. Dura entre 1 a 72 horas. Precedida de episodios de aura los cuales ocurren 10 a 15 minutos antes corresponden a manifestaciones visuales generalmente como escotomas, visión borrosa, defectos hemianópsicos, estos desaparecen con el inicio de la cefalea.

Diagnóstico clínico Tipos de migraña Banderas rojas Identificar los factores desencadenantes Tratamiento

Caso Clínico Tipo Hombre de 22 años sin antecedentes mórbidos, refiere cuadro de cefela hemicranea derecha de inicio incidioso, intensa llegando a EVA 6, que inicia en la mañana durante clase en la Universidad, asociada a fotofobia. Al interrogatorio dirigido refiere que fue precedida 30 minutos antes de sensación de hormigueo en brazo y antebrazo derecho. Cede al utilizar AINES y con reposo.

Migraña sin aura: Representa el 75% de los casos. Banderas rojas (signos de alarma que orientaran hacia una etiología específica y que requiriera de estudios adicionales): • Cefalea que inicia en personas >55 años. • Cefalea de inicio súbito • Cefalea crónica que cambia de patrón • Asociada a focalidad neurológica. • Signos meníngeos positivos. • Edades extremas. • Asociación con esfuerzo físico. • Antecedente de TEC. • Convulsiones presentadas anteriormente. • Inmunosupresión.

Definición Es una cefalea primaria, un desorden episódico, de origen idiopático, muy común en la práctica clínica. Puede estar asociada a desencadenantes o gatillantes, siendo importante descartar causas secundarias. El primer episodio ocurre en general entre los 10 a 30 años, y un 60-75% de los casos es en mujeres.

Diagnóstico Es un diagnóstico clínico, importante la historia, examen físico y averiguar si hay causas secundarias de esta.

Se puede subdividir en migraña con aura o sin aura, de acuerdo a su modo de presentación clínica.

Siendo un tipo de cefalea primaria, recurrente, de horas de duración y evolución(generalmente de 1 a 72 horas), de carácter pulsátil, intensidad moderada a severa y que impide al individuo continuar con sus actividades cotidianas. Generalmente es hemicránea y en el caso de que sea con aura se acompaña de síntomas vegetativos como náuseas, vómitos e hipersensibilidad a estímulos sensitivos (fotofobia y fonofobia). Puede estar precedida por un conjunto de síntomas de origen cortical (aura), siendo las manifestaciones visuales las más frecuentes (pueden ser auras de tipo sensitivo, motoras, neurovegetativo).

Etiología Se desconoce el gatillante inicial del fenómeno. Hay evidencia del rol que cumplirían las estructuras centrales del tronco (núcleos del rafe medio y locus coeruleus). Habría inicialmente una activación vasomotora neuronal puntual a nivel de tronco encefálico, que posteriormente producirían la activación a nivel de receptores nociceptivos trigeminales enviando una señal al núcleo trigeminal donde sinapta con la vía aferente a hipotálamo, tálamo y luego corteza sensitiva. Lo anteriormente descrito seguiría una progresión de occipital a cefálico. Durante los episodios de migraña se ha demostrado una hipoperfusión cerebral a nivel de la corteza visual que se va extendiendo hacia delante.Neuropatía diabética.

Las crisis suelen desencadenarse posterior a estímulos conocidos por el propio paciente: alimentos condimentados, chocolate, alcohol, exposición al sol, entre otros. Por lo general tienen una presentación estereotipada en el caso de las migrañas con aura, con presentación del aura 150 a 30 mins antes del inicio de la cefalea con posterior descenso

9 Facultad de Medicina, Universidad de Chile

de los síntomas del aura al momento del inicio de la cefalea. El paciente busca el reposo y la oscuridad (a diferencia de la cefalea en cluster que produce inquietud psicomotora). Se debe intentar identificar los factores desencadenantes, para poder intervenir con cambios de hábito.

Tratamiento: •

Tratamiento profiláctico: está indicado en quienes la frecuencia o intensidad de la migraña sea invalidante (más de 2 crisis al mes). Ha demostrado utilidad el uso diario de propanolol (40-120 mg 2-3 veces/día), Atenolol (25-50 mg/día), flunarizina (2,5-5 mg/noche), divalproato de sodio, amitriptilina (25-50 mg/noche) y topiramato. Se recomienda seguir esta terapia por un período de 3 a 6 meses, suspendiendo en forma gradual. Siempre recordar ABCE (Antidepresivos tricíclicos, betabloqueadores, calcioantagonistas, antiepilépticos)



Tratamiento abortivo de la crisis: debe evitarse el uso de ergotamínicos por el riesgo de desencadenar una cefalea por abuso, prefiriendo uso de AINES de acción rápida en caso de migrañas moderadas (aspirina 1000 mg, naproxeno 550 mg, ketorolaco 10 mg) y triptanes en caso de migrañas severas, en forma precoz (naratriptán 2,5 mg, eletriptán 40 mg). Se recomienda el uso de antieméticos (domperidona 10 mg). Fármacos de tercera línea son: hidratación ev, ketorolaco o ketoprofeno 30 mg ev, betametasona 4 mg ev. Es importante también el manejo ambiental, que el paciente repose preferiblemente en una habitación oscura y sin ruidos.

Seguimiento Control al mes de iniciado el tratamiento profiláctico para evaluar respuesta. Derivar a neurólogo si existe resistencia al tratamiento o cambio del perfil del dolor con aparición de banderas rojas.

Autor / Editor Daniela Bustos

Año 2017

10 Facultad de Medicina, Universidad de Chile

Parálisis (Hemiplejias, Tetraplejias, Hemiparesias, Tetraparesias)

Caso Clínico Tipo Mujer de 25 años, estudiante, inicia cuadro clínico caracterizado por debilidad distal simétrica en miembros inferiores y superiores, de instalación progresiva e insidiosa, que en pocos días asciende afectando los músculos de los brazos y cara, impidiéndole deambular. No refiere actualmente disnea ni dificultad para orinar ni a la defecación.

Nivel de manejo del médico general: Diagnóstico Sospecha Tratamiento Inicial Seguimiento No requiere

Definición

Aspectos Esenciales • • • •

Debilidad muscular provocada por defectos del sistema motor. Pueden ser secundarias a lesiones ubicadas a distintos niveles del sistema motor: músculo (miodistrofias), unión neuromuscular (miastenia gravis, síndrome Eaton Lambert), nervio (neuropatías) , vía piramidal (lesiones vasculares, sección medular, etc), corteza motora (lesiones tumorales, ACV, hemorragias intracraneales) y voluntad (conciencia).

La debilidad muscular tiene diversas causas, dependiendo de la estructura dañada: músculo, unión NM, nervio, vía piramidal. Los signos clínicos dependen del nivel de la lesión. Siempre diferenciar entre compromiso de 1era Motoneurona y 2da Motoneurona mediante examen físico. El tratamiento depende del sistema dañado y su etiología.

Figura: Tipos de parálisis: A) hemiplejía B) paraplejía y C) monoplejía Semiología Médica. Goic. 3a edición. Editorial Mediterráneo

La debilidad es un síntoma de consulta muy frecuente y su causa variada. El diagnóstico del mecanismo y la localización es de carácter clínico y que se puede complementar con Imagenología.

prometidos. • Corteza motora: ACV isquémico y hemorrágico, hemorragia Intracraneal, lesiones tumorales, lesiones traumáticas, infecciones, esclerosis múltiple, lesiones desmielinizantes de 1era y 2da motoneurona (ELA). • Tronco encefálico y médula espinal: similares a las presentes a nivel de corteza. • SNP: mononeuropatías, radiculopatías, polineuropatías, mononeuritis múltiple, lesiones desmielinizantes (Sd

Etiología Es muy variada considerando cada uno de los niveles com-

11 Facultad de Medicina, Universidad de Chile

• •

Guillian Barré o CIDP). Unión neuromuscular: miastenia gravis, síndrome Eaton Lambert, botulismo. Miopatías: miopatías congénitas (distrofia de Duchenne, distrofia de Becker) y miopatías no congénitas o adquiridas.

a nivel sensitivo con signos de 2da motoneurona, lesión medular, y dependiendo de la altura puede asociar compromiso de control de esfínteres. Compromiso proximal (grandes articulaciones cadera y cintura escapular), recurrente, asociado a compromiso de pares craneanos (sobre todo III,VI y glosofaríngeo) en enfermedades de la placa motora. verificar compromiso de pares craneanos en relación a la parálisis (síndromes alternos, con compromiso ipsilateral a la lesión, de par craneano con compromiso contralateral a lesión en parálisis inferior). Diferenciar entre armónico (lesiones generalmente posteriores a la corteza, a nivel de tronco encefálico) o disarmónico (compromiso generalmente de corteza cerebral). Los exámenes complementarios se solicitarán dependiendo

Fisiopatología Se pueden subdividir en las de tipo vascular (infartos isquémicos o hemorrágicos), enfermedades del SNP (desmielinización o compromiso axonal o ambas), trastornos de la placa motora (presencia de anticuerpos anti-receptores o bloqueador de receptores de acetilcolina), alteraciones genéticas a nivel molecular de la estructura muscular (miopatías congénitas)

Epidemiología No tienen una predilección etárea y generalmente se pueden ver tanto en servicios de atención primaria como en servicio de urgencia. Se pueden identificar en algunos casos antecedentes familiares (miopatías) o factores de riesgo generales a otras enfermedades (factores de riesgo cardiovascular en ACV). La parálisis cerebral espástica es la más común, llegando a ser el 70-80% del total.

Diagnóstico El diagnóstico de la debilidad se basa en la semiología. Siempre se debe diferenciar entre lesiones de 1era motoneurona y lesiones de 2da motoneurona. Para ello recurrir al examen físico evaluando la presencia de espasticidad (mayor en 1era motoneurona), tono muscular (aumentado en 1era motoneurona), presencia de reflejos osteotendíneos (exaltados en primera motoneurona), reflejos plantar extensor (lesión 1ea motoneurona) o flexor, presencia de fasciculaciones (2da motoneurona), atrofia precoz (en 2da motoneurona). En segundo lugar caracterizar el perfil temporal y evolutivo de la parálisis. En general en los pacientes con PNP el inicio es distal y suele ser simétrico, con distribución en guante y calcetín, con progresión hacia proximal, que puede o no asociar componente sensitivo dependiendo de la causa. En el caso de otras neuropatías es característico su asimetría, compromiso sensitivo asociado, generalmente unilateral más que bilateral. En general considerar lesiones que no son progresivas y de inicio agudo, de carácter vascular. Lesiones de inicio subagudo con progresión a proximal como enfermedades desmielinizantes. Perfil recurrente con debilidad ocasional frente a ejercicio con recuperación posterior enfermedades de la placa motora. Cuadros recurrentes más espaciados con recuperación entre episodios, esclerosis múltiple. En tercer lugar evaluar localización de la parálisis, si es simétrica y distal, pensar en PNP, unilateral, asimétrica una mononeuropatía o radiculopatía, compromiso simétrico asociado

de la causa y de la clínica se puede recurrir a la Imagenología (TAC o RNM según sea el caso, siendo mejor la RNM a nivel de médula ósea o TAC para estudiar precozmente pacientes con sospecha de ACV) o electromiografía. En algunas enfermedades, se tienen exámenes específicos (anticuerpos para miastenia gravis (anticuerpos anti receptores de acetil colina), test de tensilon para miastenia gravis, biopsia muscular

12 Facultad de Medicina, Universidad de Chile

en miopatía).

Tratamiento Dependiendo de la causa se definirá tratamiento una vez confirmado el diagnóstico. La mayor parte de los casos se definirá tratamiento en conjunto con el especialista. • PNP: En el caso de la diabetes, manejo con pregabalina y control glicémico estricto. • Mononeuropatías: Identificar la causa, y liberación quirúrgica del nervio en caso de que sea por compresión mecánica. • Enfermedades de la placa motora: En miastenia gravis, uso de anticolinesterásicos, timectomía y en crisis plasmaféresis si hay compromiso de vía aérea. • Infartos medulares, de troncoencefálico y corteza cerebral: Identificar causa del infarto, manejo de factores de riesgo, eventual TACO dependiendo de la causa, rehabilitación. • Miopatías: dependiendo de la etiología de la miopatía.

Seguimiento Realizado por el especialista

Autor / Editor Daniela Bustos

Año 2017

13 Facultad de Medicina, Universidad de Chile

Síndrome de Hipertensión Endocraneana

Fisiopatología Teoría de Monro-Kellie: Volumen total del contenido intracraneal (parénquima, sangre y LCR) debe permanecer constante, debido a que se encuentran al interior de una cavidad no extensible (cráneo). Un aumento de alguno de estos componentes hará que de manera compensatoria los otros reduzcan su volumen. Cuando este mecanismo de compensación se satura, se produce la HTE, que impedirá la llegada adecuada del flujo sanguíneo cerebral, el cual provee de oxígeno y glucosa al cerebro, generándose de esta forma la sintomatología. Por otro lado, existe la posibilidad que existan herniaciones del parénquima por los agujeros naturales del cráneo (herniación subfalcial, uncal, amigdaliana)

Nivel de manejo del médico general: Diagnóstico Sospecha Tratamiento Inicial Seguimiento No requiere

Aspectos Esenciales • • • •

HTE se define como PIC >15mmHg. Clínica característica incluye cefalea, vómitos y edema papilar. En niños, se aprecia abombamiento de la fontanela y separación de las suturas. Tratar según etiología de la HTE.

Diagnóstico El diagnóstico es clínico. Los síntomas va a depender de la velocidad de instauración de la hipertensión endocraneana. Es importante detectarla pues puede producir síndromes de herniación y llegar al coma y muerte. Para determinar etiología, es útil la neuroimagen (TAC o RNM). Dado el riesgo de herniación, cuando la sospecha diagnóstica requiere la evaluación con punción lumbar, esta debe se realizar con extremo cuidado (teniendo neuroimagen y fondo de ojo previo). El diagnóstico de certeza es mediante la monitorización de la PIC.

Caso Clínico Tipo Paciente de sexo femenino de 32 años de edad refiere cefalea, acompañada de diplopía y vómitos explosivos en 3 ocasiones. La sintomatología comenzó hace una hora, después de golpearse la cabeza sin casco, al caer de su bicicleta. Al examen físico destaca obnubilación y paresia braquiocrural izquierda.

• •

Definición



Conjunto de signos y síntomas desencadenados por aumento de la presión al interior del cráneo, ya sea de inicio agudo o insidioso por sobre los 15 mmHg, de etiología diversa. (PIC normal: 5-15 mmHg). Se requiere mantener una PIC normal para lograr un flujo sanguíneo adecuado. •

Etiología



Las causas más frecuentes son • Traumatismo craneoencefálico (hematoma epidural, hematoma subdural, contusión hemorrágica, Swelling) • Hidrocefalia (normotensiva o hipertensiva) • Procesos Expansivos: Tumores, hematomas, Pseudo tumor cerebral, edema cerebral. • Infecciones: absceso cerebral, empiema subdural, tromboflebitis corticales o de senos venosos, meningitis, ventriculitis, encefalitis virales. • Procesos vasculares: Infarto cerebral, HSA, Encefalopatía hipertensiva, hematoma intraparenquimatoso hemorragia cerebelosas, trombosis venosa. • Anoxia cerebral



Cefalea crónica progresiva, mayor en la noche por la hipercapnia noctura (Provoca vasodilatación cerebral) Vómitos de origen central, matutinos (Que se caracterizan por ser explosivos y sin náuseas) Visión borrosa (Dada inicialmente por la vasodilatación venosa y borramiento del borde papilar, hasta llegar al el edema papilar luego 7 a 10 días aprox). No se necesita detectar la presencia de edema papilar para hacer el diagnóstico, ya que es tardío, y además puede no ser pesquisado por un médico no-oftalmólogo que realiza el fondo de ojo) Hipertensión y bradicardia (Como nemotécnica, es todo lo contrario a la sepsis en fases iniciales) Se va produciendo un compromiso de conciencia, dado por bradipsiquia, somnolencia. Se puede observar parálisis del VI par, que puede producir diplopia

Síndromes de herniación: • Herniación subfalcial (herniación bajo hoz del cerebro o falx): Ictus por oclusión de arteria cerebral anterior. Esto puede ocurrir previo a herniación transtentorial. • Herniación uncal (herniación a través de la tienda del cerebelo): Compresión del III par y empujando pedúnculo cerebral contra tienda del cerebelo, midriasis ipsilateralcomo signo más precoz , hemiplejía contralateral y oclusión de arteria cerebral posterior. • Herniación amigdalina (a través de foramen magno): Produciendo oclusión de bulbo raquídeo con colapso cardiorrespiratorio, vasomotor e incluso muerte súbita.

14 Facultad de Medicina, Universidad de Chile



Desplazamiento hacia abajo del diencéfalo a través de la tienda del cerebelo, resultando en “Pseudotumor Cerebri”, que se caracteriza por no evidenciarse etiología (RM o angioRM) y una punción lumbar que evidencia aumento de presión intracraneana, pero con LCR sin alteraciones.

Tratamiento Estabilización y hospitalización en UCI. Identificar y tratar problema primario que provoca la HTE. Monitorizar PIC (objetivo: PIC<20mm Hg y PPC>70mm Hg). El tratamiento puede incluir: elevar cabecera de cama a 30°, sedación, Osmoterapia con manitol 20% o suero hipertónico, dexametasona en caso de lesiones inflamatorias o en lesiones por TEC, hiperventilación controlada (controversial), tratamiento vasopresor para mantener PAM que asegure una PPC>70mm Hg (manteniendo euvolemia), ventriculostomía, drenaje ventricular externo. Una vez estabilizado, realizar estudio y tratamiento según etiología El pseudotumor cerebri se maneja con acetazolamida, drenaje ventrículo peritoneal para evitar el daño del nervio óptico.

Seguimiento Por especialista.

Autor / Editor Paulina Lira

Año 2016

15 Facultad de Medicina, Universidad de Chile

Cefalea Aguda en Urgencias

de la inflamación neurogénica del seno cavernoso con compromiso de la vía simpática secundariamente. •

Nivel de manejo del médico general: Diagnóstico Específico Tratamiento Inicial Seguimiento Derivar

Cefalea secundaria: Se presenta en menor frecuencia, y responde a etiologías infecciosas, traumáticas, tumorales, vasculares, etc.

Diagnóstico Aspectos Esenciales

Orientado a diferenciar causa primaria de secundaria. El diagnóstico se basa en la historia, un examen físico y neurológico con fondo de ojo (pese a que hipertensión endocraneana no genera edema de papila precozmente) buscando signos de alarma. La anamnesis debe enfocarse en: edad, frecuencia, intensidad y duración de los ataques, factores desencadenantes, localización y características del dolor, uso de analgésicos, etc. En el examen físico, ante la presencia de rigidez de cuello y meningismo se debe considerar la posibilidad de meningitis. Si se observa papiledema, considerar posible masa intracraneal, meningitis o hipertensión intracraneal idiopática. Ante la sospecha de causa secundaria, se debe recurrir a la punción lumbar y el TAC de cerebro.

Diferenciar dolor primario o secundario. • Descartar signos de alarma. • Evaluar utilidad de imágenes o laboratorio (TAC, punción). • Cefalea primaria en general tratamiento con AINES. • Referir a atención primaria o especialista según corresponda.

Caso Clínico Tipo Mujer de 35 años refiere cuadro recurrente de cefalea que cede con AINE. Ahora con dolor hemicráneo pulsátil antecedido por visión de destellos en campo visual derecho. Tuvo 2 vómitos y no pudo seguir trabajando. Consulta al SU sin haber tomado analgésico.



Definición Motivo de consulta frecuente (4,5% de las consultas en urgencia) que generalmente corresponde a un fenómeno benigno (cefaleas primarias), siendo el principal objetivo de la evaluación el descartar alguna cefalea secundaria asociada a patología grave. • Cefalea primaria: Es la cefalea más frecuente (80%), de predominio en el sexo femenino (excepto la cefalea en cluster en hombres jóvenes), de origen intrínseco y con frecuencia asociada a gatillantes (estrés, falta de sueño, alcohol, alimentos). •





Cefalea primaria - Migraña: ≥5 ataques, con o sin aura, invalidante, dura 4 a 72 horas, unilateral, pulsátil, intensidad moderada a grave, se agrava por actividad habitual. Con náuseas o vómitos, fotofobia y fonofobia. Aura visual: amaurosis, escotomas centellantes, teicopsias, hemianopsia, también pueden ser parestesias faciales, braquiales, trastornos del lenguaje etc. Estos fenómenos duran entres 1-20 minutos previo a la cefalea. - Cefalea tensional: ≥10 ataques de 30 min a 7 días de duración. Es bilateral, opresiva, intensidad leve a moderada, no se exacerba con actividades habituales. Puede tener puntos dolorosos. Asociado a estrés. Sin vómitos, fotofobia o fonofobia. - Cefalea en clúster: Hombres jóvenes. 2 o 3 crisis diarias de 15-180 min de duración, hasta 8 crisis diarias, con una duración total de 4-8 semanas, reaparece nuevamente en meses-años. Se acompaña de dolor retro-ocular, penetrante, intenso, con epifora, ojo rojo, miosis, ptosis, rinorrea y tumefacción facial. El paciente está inquieto como si tuviera un cólico renal.

Migraña: Causa frecuente de cefalea, más frecuente en mujeres, en un 70% se presenta sin aura. Episodios de 4 - 72 hrs de duración. Se postula alteración del sistema trigémino vascular con inflamación vascular neurogénica con vasodilatación como vía final común, con progresiva sensibilización y rol regulador de serotonina.

- Cefalea hemicránea paroxística: Hemicránea, intensa y de corta duración, sin náuseas ni vómitos, varias crisis diarias y predominio en mujeres.

Cefalea tensional: Es la más prevalente en su forma infrecuente (<1 día al mes). Tiene origen en la hipersensibilidad de las vías del dolor del SNC generados por input miofascial pericraneano asociado a eventual rol del óxido nítrico.



Cefalea en clúster (<1%): eventualmente causada por activación hipotalámica, con activación secundaria de reflejo trigémino autonómico. Existe también la teoría

Cefalea secundaria: Se sospecha ante signos de alarma y su enfrentamiento es el tratamiento del cuadro de base. Debe ser atendido por especialista - Signos de alarma: Inicio explosivo (vascular); la peor cefalea de toda mi vida (HSA, síndrome de vasocons-

16 Facultad de Medicina, Universidad de Chile

tricción cerebral reversible), meningismo (HSA, infecciosa), Posicional: donde en decúbito es mayor que en bipedestación (aumento de PIC), Asociado a nauseas y/o vómitos (aumento de PIC v/s migraña), Síntomas visuales: diplopía, visión borrosa, disminución agudeza visual (arteritis de células gigantes, glaucoma,aumento de PIC), dolor ocular (glaucoma, cefalea en brotes), Examen neurológico alterado (lesiones estructurales, posibles en migraña), disminución del nivel de conciencia con o sin fiebre (infeccioso, HIC), edad mayor de 50 años, Inmunodepresión (Infecciones SNC, sd de encefalopatía posterior reversible). - Traumático: El paciente puede o no manifestar signos neurológicos. Para fracturas pudiera ser útil la Rx de cráneo (Guía GES), siendo realmente el examen de elección un TAC de cerebro sin contraste que incluya ventana ósea. - Vascular: Sospechar ante inicio abrupto o subagudo que se hace permanente, con compromiso de conciencia, focalización, vómitos, convulsiones. Signos meníngeos sin fiebre orientan a HSA. Requiere TAC de cerebro sin contraste (ante sospecha de HSA puede complementarse con AngioTAC de vasos cerebrales). La punción lumbar en búsqueda de xantocromía y glóbulos rojos es útil, siempre habiendo descartado antes la HTEC. Ante dolor cervical y/o sospecha de disección carotídea es ideal realizar un AngioTAC de vasos cervicales. Infeccioso: Dolor intenso, fiebre, signos meníngeos. Exacerba con movimientos oculares. Confirmación con PL. - Ocupación de espacio: Tumoral, infecciosa (cisticerco, toxoplasma), hemorrágica (hematomas), etc. Cuadro generalmente larvado, con cefalea intermitente que se hace permanente, que puede asociarse a vómitos, focalización y crisis epiléptica. El diagnóstico incial será con TAC de cerebro sin contraste, pero deberá ser complementado con RNM con contraste (especialmente en el dg. diferencial de tumores e infecciones). Sobreuso de fármacos: Cefalea crónica diaria, presente por >15 días al mes, bilateral, opresiva, intensidad leve a moderada, con antecedente de uso por más de 3 meses de ergotamínicos (≥10 d/mes), analgésicos (≥15 d/ mes), opioides (≥10 d/mes), triptanos (≥10 d/mes). - Atribuido a estructuras faciales o craneales: Descartar sinusitis, causa odontológica, disfunción mandibular (ATM), neuralgia trigeminal, arteritis temporal, glaucoma. Correlación anamnesis-examen físico en el estudio de una cefalea de causa secundaria.

• • •







Tratamiento primera línea (oral): AINES (AAS 1000 mg, naproxeno 550 mg, ketorolaco 10 mg, etc.) + antiemético (metoclopramida 10 mg, domperidona 10mg). Segunda línea: Triptanes (naratriptán 2.5mg, eletriptán 40mg, sumatriptán 50-100 mg). Tercera línea: Hidratación parenteral, AINES parenteral (ketorolaco 30 mg, ketoprofeno ev), corticoides (betametasona 4 mg ev), antiemético parenteral (clorpromazina 12,5 mg ev, metoclopramida 10 mg ev). Evaluar hospitalización. Referir a atención primaria. Cefalea Tensional: AINES o paracetamol. En estudio triptanes y relajantes musculares. Siempre considerar las medidas no farmacológicas (masoterapia, calor local, etc.) Cefalea en clúster: Iniciar manejo con oxígeno y triptanes SC (sumatriptán 6mg sc). De haber mala evolución, emplear triptanes intranasales, otreótide SC, lidocaína intranasal o ergotamínicos. Cefalea hemicránea paroxística: Característicamente responden bien a indometacina. Iniciar 25 mg c/8h VO en dosis crecientes hasta 225 mg como dosis diaria total.

Seguimiento En cefalea primaria sin elementos de alarma, tratar crisis y control en APS. Eventualmente algunos pacientes requerirán reposo laboral y manejo con profilaxis a largo plazo. En general, ante cefalea con elementos de alarma en la anamnesis y/o examen físico, se recomienda el estudio en servicio de urgencia, idealmente por especialista, el cual habitualmente contemplará de manera inicial exámenes generales y un TAC de cerebro sin contraste seguido de una punción lumbar según el contexto clínico.

Autor / Editor Paulina Lira

Tratamiento La cefalea primaria requiere tratamiento inicial y la cefalea secundaria puede requerir hospitalización según etiología y debe ser tratada por especialista. Migraña en servicio de urgencia:

17 Facultad de Medicina, Universidad de Chile

Año 2016

Crisis Convulsiva

(parcial o generalizado) de cualquier tipo que se asocia a descarga cortical.

Etiología

Nivel de manejo del médico general: Diagnóstico Sospecha Tratamiento Inicial Seguimiento Derivar

La crisis convulsiva es característica de la epilepsia, que corresponde a una enfermedad neurológica crónica que predispone a estos eventos paroxísticos por descargas eléctricas hipersincrónicas de origen cerebral. Sin embargo, diversas patologías no epilépticas pueden presentar convulsiones, como alteraciones tóxico-metabólicas, trastornos motores, trastornos del sueño, trastornos psiquiátricos, entre otras (malformaciones congénitas, enfermedad cerebral vascular, tumores, infecciones, traumas, hipoglicemia, hiperglicemia no cetócica, hiponatremia, hipocalcemia, hipomagnesemia, falla renal/uremia, hipertiroidismo, alteraciones del metabolismo de las porfirinas, anoxia cerebral (paro cardiorrespiratorio, intoxicación con CO, inmersión o secundario a anestesia), episodios de abstinencia (particularmente a alcohol y a BDZ) y uso de drogas.)

Aspectos Esenciales • • • • •

Movimientos bruscos involuntarios de comienzo súbito, generalmente con alteración de conciencia Descartar patología subyacente Medidas generales Derivar asintomático Status convulsivo es emergencia médica

Caso Clínico Tipo

Atendiendo al origen anatómico y a las manifestaciones clínicas, las crisis se pueden clasificar en: • Parciales o focales: con síntomas motores, sensitivos, autonómicos y/o psíquicos, dependiendo de la función que realice el área cerebral afectada. Se dividen en - Simples (sin alteración del nivel de conciencia) - Complejas - Parciales secundariamente generalizadas • Generalizadas: descarga neuronal de ambos hemisferios. La alteración del nivel de conciencia suele ser el síntoma inicial y la afectación es bilateral desde el comienzo. Se dividen en: • Ausencias • Mioclónicas

Mujer de 18 años que consulta asintomática tras un cuadro súbito de convulsiones generalizadas y pérdida de conciencia con incontinencia urinaria. Duró 30 segundos y luego se mantuvo somnolienta. Al examen presenta contusión en hombro y lesión lateral de lengua.

Definición Evento súbito y transitorio caracterizado por movimientos de carácter tónico, clónico o tónico-clónico, generalizado o focalizado, de inicio súbito y correlacionado a una descarga eléctrica sincrónica cortical. En sentido más amplio, crisis epiléptica corresponde a aquel episodio ictal neurológico

Compendio de Medicina Interna. C. Rozman. V edición. Editorial Elsevier

18 Facultad de Medicina, Universidad de Chile

• Tónico-clónicas generalizadas • Atónicas • Tónicas • Clónicas Las crisis tónico-clónicas generalizadas son las más frecuentes.

según disponibilidad. Recordar siempre que el inicio de antiepilépticos según la evidencia internacional se fundamenta generalmente en: • •

Diagnóstico Anamnesis y examen físico para detallar el cuadro tónico clónico generalizado: fase inicial tónica de aumento del tono muscular, de unos 20 segundos de duración, asociada a veces del “grito epiléptico” inarticulado y con mordedura de lengua lateral. Posteriormente y de manera progresiva, se suceden episodios de relajación que se vuelven más frecuentes (fase clónica) con sacudidas simétricas y bilaterales de las extremidades y de la musculatura maseterina, por alrededor de 30 segundos y asociados a cambios vegetativos (cianosis, palidez, taquicardia, hipertensión, sialorrea y relajación de esfínteres). Finalmente, se sucede al episodio el periodo post ictal de al menos 5 minutos con somnolencia, confusión y relajación muscular con recuperación progresiva del nivel de conciencia.

Paciente con al menos dos crisis epilépticas claras semiológicamente Paciente con una primera crisis epiléptica y: - Alteración en neuroimagen (con causa secundaria) - EEG alterado - Alteración al examen neurológico

Medidas farmacológicas sólo son necesarias cuando la crisis no se autolimita o en Status Convulsivo (definición operacional de 5 minutos en crisis o en 2 ó más crisis sin recuperación entre ellas). Benzodiazepinas son la primera elección: • Lorazepam EV (0,1 mg/Kg pasando a 2 mg/min, dosis máx.: 8 mg), Diazepam EV (10-20 mg pasando a 5 mg/ min, dosis máx: 30 mg): MONITORIZAR FRECUENCIA RESPIRATORIA, porque puede producir depresión respiratoria. •

Siempre caracterizar semiológicamente de manera acuciosa el episodio ictal, sumado a los antecedentes de epilepsia, descartando siempre alguna causa sistémica de convulsiones o secundaria de crisis epiléptica y estableciendo de manera clara la distinción con los principales diagnósticos diferenciales (síncope, TIA, crisis de pánico, disquinesias, aura migrañosa, trastorno del sueño REM, descontrol de impulsos, amnesia global transitoria, vértigo paroxístico, etc) Se debe tomar un ECG para descartar patología cardíaca, determinación de la glicemia o HGT, electrolitos plasmáticos (Na, K, Cl, Ca, P, Mg), gases arteriales.

Repetir dosis de BDZ y agregar Tiamina (100 mg EV con 50 ml de glucosado al 50% de haber hipoglicemia o de no poder descartar ésta).

• Carga de fenitoína (15-20 mg/Kg EV en bolo a no más de 50 mg/min en sol. salina y con monitor cardiaco). De ser necesario, se pueden agregar cargas de 5 mg/Kg EV. Ante crisis de >30 minutos que no ceden con Fenitoína: intubar, trasladar a UCI y administrar Fenobarbital (10-15 mg/ Kg/dosis a 100 mg/min). De persistir en crisis: inducir coma farmacológico con Tiopental, Midazolam y/o Propofol, titulando según efecto, con monitorización de EEG y manteniendo por al menos 12 hrs. con manejo farmacológico que suprimió el status.

Dentro de las neuroimágenes, principalmente TAC cerebral en servicio de urgencias considerando que la imagen de elección de manera electiva es la RNM cerebral con protocolo de epilepsia (con cortes finos en región temporal).

Seguimiento

La punción lumbar siempre debe realizarse ante la sospecha de algún cuadro infeccioso (encefalitis fundamentalmente).

Realizado por especialista.

Cuando el cuadro orienta a una crisis epiléptica y se ha descartado una causa secundaria de urgencia (como enfermedad cerebrovascular, trastorno médico o trauma) y el paciente está asintomático, corresponde referir a neurólogo para estudio ambulatorio con EEG.

Autor / Editor Marcelo Fres

Tratamiento Medidas generales ante crisis epiléptica convulsiva: debe asegurarse cabeza lateralizada, oxigenación, y seguridad ante caídas o movimientos bruscos. Son importantes la monitorización de signos vitales y vías venosas. No deben introducirse objetos a la boca. Tomar ECG y eventualmente EEG

19 Facultad de Medicina, Universidad de Chile

Año 2016

Estado Confusional Agudo

Causas: • Infección extracerebral • Síndrome de abstinencia a alcohol o sedantes • Alteraciones metabólicas (hipoglicemia, falla renal o hepática) • Quemaduras • Hipoxia (secundaria a IAM, arritmia, postquirúrgica, TEP, etc) • Deficiencias vitamínicas (de tiamina, vit B12, folato, etc) • Endocrinas (hiper o hipotiroidismo, trast. de paratiroides, alteración eje adrenal) • Drogas y toxinas, metales pesados (plomo, mercurio) • Infección intracerebral (meningitis, encefalitis) • Vasculares cerebrales (TIA, AVE, encefalopatía hipertensiva, shock) • Trauma cerebral • Patología del SNC (lesiones expansivas, epilepsia en estatus y estados postictales, encefalopatía de Wernicke y otras), etc.

Nivel de manejo del médico general: Diagnóstico Específico Tratamiento Inicial Seguimiento Completo

Aspectos Esenciales • • • •

Alteración principal de atención, alerta y funciones cognitivas. Inicio agudo, fluctuante, de duración menor a 6 meses. Tratamiento es etiológico, si bien los síntomas pueden manejarse con neurolépticos (haloperidol o quetiapina). Benzodiacepinas para el delirium tremens.

Caso Clínico Tipo

Factores predisponentes: > 70 años , enfermedad grave, factores de deterioro del sistema nervioso central (demencia, depresión en ancianos), alteraciones metabólicas, deshidratación, abuso de alcohol, polifarmacia, deprivación sensorial.

Hombre de 75 años, diabético e hipertenso, presenta hace 6 horas compromiso leve de conciencia de tipo cuali-cuantitativo. Al examen mental se muestra desorientado, apático y somnoliento. Familiares refieren que ha presentado períodos de agitación y alucinaciones durante el último tiempo. No muestra focalidades neurológicas. Familiares refieren que en el último no ha estado comiendo en forma regular y antecedentes de consumo de OH crónico, actualmente suspendido.

Factores precipitantes: fiebre, dolor, traumatismos, hipoperfusión, abstinencia a sustancias, anticolinérgicos, psicotrópicos, opioides, restricción física (inmovilidad), procesos patológicos del sistema nervioso (infecciones), AVE. Factores agravantes: UTI, cambios reiterados de habitación, falta de elementos que le permitan orientarse, falta de lentes o audífonos.

Definición Disfunción cerebral aguda y fluctuante caracterizada por alteraciones del nivel de conciencia y funciones cerebrales superiores, especialmente la atención, pudiendo afectar también ciclo sueño-vigilia, coordinación, emoción, movimiento, etc. Se diferencia de las demencias en que es de origen agudo, con compromiso de la atención y nivel de consciencia.

Fisiopatología No se conoce totalmente la fisiopatología del síndrome confusional agudo, siendo lo más aceptado su relación con un desbalance en los sistemas de neurotransmisión colinérgico, serotoninérgico y dopaminérgico. El hecho de que pacientes adultos mayores tengan menos neuronas colinérgicas implica un mayor riesgo en este grupo. El desbalance en la neurotransmisión explica la relación de los fármacos anticolinérgicos en el inicio del síndrome confusional agudo. Durante el estado confusional se comprometen múltiples áreas del encéfalo, planteándose que la disfunción del hemisferio derecho constituye una vía final común en su patogenia, y que los infartos en el territorio de la arteria cerebral media derecha se asocian con mayor frecuencia a delirium de tipo hiperactivo.

Epidemiología Es el trastorno mental más frecuente en pacientes hospitalizados. Están predispuestos niños, ancianos, pacientes con polifarmacia, antecedente de deterioro cerebral (pacientes con antecedentes de TEC o de Hematoma Subdural Crónico) o con abuso de sustancias.

Etiología

Diagnóstico

Típicamente multifactorial, y característicamente las causas neurológicas son las menos frecuentes. En la práctica clínica, delirium es casi siempre una consecuencia directa de un estresor fisiológico general.

La clínica se presenta como un compromiso cualitativo y cuantitativo de conciencia, con alteraciones de la atención, concentración y orientación temporoespacial, alerta (somnolencia o

20 Facultad de Medicina, Universidad de Chile

agitación e hipervigilancia) y percepción (alucinaciones, pseudopercepciones), que se agregan de acuerdo a la gravedad.

• •

Los síntomas esenciales son la inatención y pensamiento desorganizado. Es de inicio brusco (horas a días), fluctuante a lo largo del tiempo, de menos de 6 meses de duración. Pueden presentar también: alteraciones del ánimo, pasando de la ansiedad a la depresión, inversión del ciclo sueño/vigilia, alteraciones de la memoria, con distorsiones y amnesia lacunar, conducta y coordinación inadecuadas y síntomas autonómicos: sudoración, temblor, taquicardia, dilatación pupilar, hipertensión, etc.



• Se clasifica según el estado psicomotor del paciente en: hipoactivos (1%), hiperactivos (35%) y mixto (64%). Para el diagnóstico etiológico deben indagarse las posibles causas en la anamnesis y examen físico, poniendo énfasis en las causas no neurológicas.

Seguimiento

Exámenes complementarios: hemograma, BUN, creatinina, glicemia, orina completa, electrolitos plasmáticos, TAC y EEG. También se puede realizar punción lumbar y determinación de drogas plasmáticas, evaluación de nutrientes (Vit B12, Tiamina) o B1. Instrumentos de evaluación: Examen mental Neurológico. Diagnóstico diferencial: demencias, enfermedades psiquiátricas, estados conversivos, facticios o simulación.

Seguimiento por especialista según la causa etiológica epiléptica corresponde a aquel episodio ictal neurológico (parcial o generalizado) de cualquier tipo que se asocia a descarga cortical.

Tratamiento •



• •

• •

IM. Preferir medicamentos con bajo perfil anticolinérgico Evitar benzodiazepinas solas (evidencia de efecto paradojal en adultos mayores), uso de opiáceos y anticolinérgicos. En casos particulares: administración de Vit B12 intramuscular (déficit de vit B12), aporte de tiamina IV y suero glucosado en los casos que se sospeche encefalopatía de Wernicke, manejo de desequilibrio hidroelectrolítico y de uremia en encefalopatías urémicas y uso de lactulosa en los pacientes con encefalopatías hepáticas. La recuperación es lenta, y no es raro observar una recuperación total pasado un mes de compensada la causa médica original.

Autor / Editor Paulina Lira

Mantener adecuada hidratación y signos vitales. Manejo de la causa médica subyacente, manejo del dolor en caso de estar presente, revisar la medicación regularmente, minimizando uso de fármacos psicoactivos. Identificar fuentes de dolor e incomodidad. Medidas ambientales: dar al paciente una reorientación frecuente, enfatizar contactos interpersonales, dar instrucciones y explicaciones simples, buscar contacto visual frecuente, reducir los déficits sensoriales: traer anteojos y/o audífonos, permitir al paciente participar en la toma de decisiones lo más posible, estimular la movilidad, el autocuidado, e independencia tanto como sea posible, evitar la contención física lo más posible, usar relojes, calendarios y objetos orientadores del hogar, lugares tranquilos, bajos niveles de luminosidad, evitar interrupción del sueño. Evitar cercanía con otros pacientes con delirium. Tratamiento farmacológico: Haloperidol de elección en agitación (VO/IM), usar dosis bajas y aumentar lentamente y observar RAMs extrapiramidales. Dosis: 2.5 a 5 mg VO, que se puede repetir a las 8 horas. Antipsicóticos atípicos (Olanzapina : 2,5 a 20 mg. VO, Risperidona 0,5mg a 4,5 mg, Quetiapina: 25 a 300 mg. VO) En caso de agitación moderada a severa: Haloperidol 2.5 a 5 mg IM, que se puede repetir a los 30-60-90 minutos. También se puede usar Olanzapina: 10 a 20 mg.

21 Facultad de Medicina, Universidad de Chile

Año 2016

Encefalopatía Tóxica Metabólica

Fisiopatología Alteración bioquímica del cerebro que produce disfunción neurológica global. Se ha comprobado disfunción de circuitos GABAérgicos y disminución de la actividad colinérgica.

Nivel de manejo del médico general: Diagnóstico Específico Tratamiento Inicial Seguimiento Derivar

Diagnóstico Hay diversas manifestaciones, dependiendo de la etiología de la encefalopatía. Estas manifestaciones son: trastornos psíquicos (generalmente al inicio de la encefalopatía), convulsiones, trastornos de la conciencia (frecuentes en encefalopatías agudas metabólicas) y trastornos sensoriales (auditivos o visuales), sensitivos (parestesias), motores o reflejos.

Aspectos Esenciales • • •

Existe disfunción neurológica generalizada. Esta condición es muy frecuente en UTI. En examen físico evaluar pupilas, motilidad ocular y respiración.



Caso Clínico Tipo Paciente que, tras beber alcohol por varios días en cantidad excesiva, se muestra confuso y desorientado, con somnolencia fluctuante. Horas después, cae en estado de coma.

• •

Definición

Examen pares craneanos: Pupilas mióticas (excepto en intoxicación con anticolinérgicos, en que están midriáticas) y simétricas, que responden a la luz. Con respecto a la motilidad ocular, al comienzo hay “ojos de muñeca”, lo que desaparece al llegar al coma. Examen respiratorio: En la mayoría de etiologías respiración de Cheyne-Stokes. Se debe realizar hemograma, gases en sangre, electrolitos, glucosa, urea, creatinina, calcio, magnesio, fosfato, bilirrubina, albúmina, enzimas hepáticas, exámenes de coagulación, electrocardiograma, amonio plasmático. Además, cultivos de sangre, orina y LCR si se sospecha de patología infecciosa y evaluación toxicológica si se sospecha intoxicación. Determinar cortisol plasmático, hormonas tiroídeas y depósito de Vit. B12. Si el comienzo de la encefalopatía es rápido, hacer punción lumbar.

Cualquier proceso que altere la función bioquímica del cerebro, afectando la corteza cerebral de forma generalizada, produciendo una disfunción neurológica global. No hay presencia de alteraciones estructurales primarias, aunque algunas encefalopatías (ej. encefalopatía de Wernicke, encefalopatía hipoxico-isquémica) pueden generar daño estructural. Habitualmente son consecuencia de desórdenes sistémicos, con afectación del sistema reticular activador ascendente.



Etiología



Variada. Ejemplos: anormalidades metabólicas, drogas, estados convulsivos, encefalopatía aguda de terapia intensiva, infecciones del SNC, etc.

• •

Epidemiología

Seguimiento

Muy frecuente en UTI. Sin embargo en muchas ocasiones pasa desapercibida en pacientes intubados, sedados, o recibiendo fármacos bloqueantes neuromusculares. Factores de riesgo: • >60 años • drogas con toxicidad potencial conocida sobre el SNC • falla orgánica múltiple • deficiencias nutricionales severas

Derivar a especialista.

Tratamiento Soporte: Controlar balance hidroelectrolítico, nutrición y suplementación de vitaminas. Neurolépticos en caso de agitación (Haloperidol). Benzodiazepinas con precaución, debido a la sedación que causan.

Autor / Editor Paulina Lira

22 Facultad de Medicina, Universidad de Chile

Año 2016

Enfermedad Cerebro Vascular

Epidemiología

Nivel de manejo del médico general: Diagnóstico Sospecha Tratamiento Inicial Seguimiento Derivar

En Chile se estima una incidencia de 130 por 100.000 habitantes/año y una prevalencia de 6 por 1.000 habitantes, aumentando con la edad (hasta 25 x 1.000 sobre los 65 años). Se constituye como la segunda causa de muerte general y representa el 9% de todas las muertes en nuestro país.

Aspectos Esenciales

Fisiopatología

• •

Isquemia cerebral causada por la disminución del flujo sanguíneo. Los síntomas aparecen rápidamente por la privación neuronal de glucosa y oxígeno, única fuente energética de estas células. Si la interrupción del flujo dura bastantes minutos, habrá muerte del tejido encefálico (infarto).

• •

El diagnóstico es clínico. Dependiendo de la presencia o no de una lesión por necrosis tisular en la neuroimagen, dependerá que la denominemos infarto cerebral (IC) o ataque isquémico transitorio (TIA). El tratamiento de primera línea del AVE isquémico es la trombolisis. El tratamiento del AVE hemorrágico es médico.

Diagnóstico El diagnóstico es fundamentalmente clínico, sin embargo la imagenología es crucial para identificar etiología y definir conducta. La clínica esmuy variable, dependiendo de la zona anatómica cerebral afectada. Se pueden incluir: • Hemiparesia (85% de los accidentes isquémicos), alteración de la visión, marcha, expresión o comprensión del lenguaje (afasias), etc. • Cefalea intensa repentina es menos frecuente en el ACV isquémico, más propio de accidentes hemorrágicos. • Debe durar >24 hrs, de lo contrario es un Accidente Isquémico Transitorio (TIA). • La identificación precoz de un ACV es fundamental para iniciar el manejo en el menor tiempo posible y prevenir recurrencias. Para ello, la Asociación Americana del Corazón (AHA) recomienda utilizar la Escala de ACV de Cincinnati que mide 3 parámetros clínicos: • Simetría facial: solicite a paciente que sonría • Prueba de pequeña paresia • Evaluación de lenguaje: pida que repita una oración que usted diga. La presencia de los tres déficits posee una sensibilidad de 100% y especificidad de 90% para ACV. TAC sin contraste: permite diferenciar entre isquémico y hemorrágico. RM: permite conocer con exactitud la extensión y ubicación de un infarto.

Caso Clínico Tipo Paciente de sexo masculino de 67 años de edad, diabético, hipertenso, tabáquico activo, presenta un cuadro de hemiparesia facio-braquio-crural derecha, de comienzo repentino, acompañado de dificultad del lenguaje, con disminución de la fluidez y torpeza para encontrar las palabras que busca.

Definición Las enfermedades cerebrovasculares son trastornos causados por la alteración del flujo sanguíneo cerebral (hemorragia/isquemia), debido a la ruptura de un vaso o a la obstrucción del mismo por un coágulo, causando el daño del parénquima cerebral.

Etiología Enfermedades isquémicas (más frecuentes) o hemorrágicas. Las enfermedades isquémicas se pueden clasificar según etiología en • Aterotrombótico o enfermedad arterial de gran vaso • Lacunar o enfermedad arterial de pequeño vaso • Cardioembólico • Causa inhabitual • Etiología indeterminada.

Tratamiento En accidente isquémico: asegurar ABC de reanimación y tto. de hipoglicemia, si la hay. trombolisis intravenosa (tPA) si lleva menos de 3 - 4,5 horas de iniciado los síntomas, técnicas endovasculares. Si no es candidato para trombolisis, tratamiento antitrombótico (antiagregantes plaquetarios o anticoagulante [si etiología es cardioembólica]), atorvastatina dosis altas (acción pleiotrópica, vasculoprotectora y estabilizadora de la placa aterotrombótica), neuroprotección y

Las enfermedades hemorrágicas se clasifican según su localización en intracerebral o subaracnoidea. La hemorragia intracerebral puede ser intraventricular o parenquimatosa, que a su vez puede ser troncoencefálica, cerebelosa o hemisférica, y esta última lobular, profunda o masiva.

23 Facultad de Medicina, Universidad de Chile

rehabilitación. En ateroesclerosis carotidea, si hay evidencia de estenosis crítica, el segmento enfermo puede ser extraído por cirugía (endarterectomía) o tratado en forma paliativa con endoprótesis o con angioplastía. En hemorragia intracraneal revertir cualquier coagulopatía identificada. Si hay hipertensión endocraneana, usar fármacos osmóticos (manitol, suero hipertónico) e inducir hiperventilación. No se ha demostrado que la evacuación del hematoma supratentorial sea beneficioso. En hemorragia cerebelosa: Hematoma > 3cm se evacúa. Si es entre 1-3 cm se observa, y si es <1cm no requiere extirpación quirúrgica.

Seguimiento Derivar a especialista.

Autor / Editor Paulina Lira

Año 2016

24 Facultad de Medicina, Universidad de Chile

Hipertensión Intracraneana

Etiología Variada. Ejemplos: anormalidades metabólicas, drogas, estados convulsivos, encefalopatía aguda de terapia intensiva, infecciones del SNC, etc.

Nivel de manejo del médico general: Diagnóstico Sospecha Tratamiento Inicial Seguimiento Derivar

Fisiopatología La teoría de Monro Kellie establece que existen tres componentes (parénquima, LCR, Sangre) en una cavidad inextensible. Cada componente contribuye al volumen intracraneano con 1400 cc, 140 cc y 140 cc respectivamente (80%, 10% y 10%), por lo que el aumento de uno de ellos producirá una disminución de los otros dos componentes a modo de compensación. Se produce por un desbalance entre la producción y absorción y/o la obstrucción en circulación de LCR, llevando a un aumento de presión venosa cerebral, presencia de lesión ocupante de espacio, edema cerebral, lo que produce desviación de LCR al saco dural con compresión de venas epidurales, desplazamiento del tejido encefálico con producción de hernias cerebrales, disminución del volumen sanguíneo cerebral. El daño cerebral en este cuadro se debe a dos mecanismos: • Hipoxia global. Esto sucede cuando la presión intracraneal tiende a igualarse a la presión arterial media, lo que ocasiona un descenso de la presión de perfusión cerebral (PPC = PAM - PIC). El valor normal de la PPC es mayor a 70 mm de Hg. • Herniación del parénquima cerebral

Aspectos Esenciales • • • •

Cefalea gravitativa, matinal, vómitos “en escopeta”, edema de papila, bradicardia, bradipsiquia, constipación, parálisis del VI par, somnolencia y visión borrosa. Riesgo de herniación del uncus. Estabilización del paciente y tratamiento de causa. La presión de perfusión cerebral (PPC)= PAM (presión arterial media) - PIC (presión intracraneal).

Caso Clínico Tipo Mujer de 24 años que, en los últimos 2 meses, presenta cuadro de inicio insidioso con episodios matutinos de cefalea, asociada a náuseas y visión borrosa; en el último episodio, presentó además diplopia. En la exploración, sólo cabe destacar papiledema bilateral y obesidad.

Definición

Etiología

Urgencia médica, que consiste en aumento de presión intracraneana por modificación de algún componente de la cavidad craneal (parénquima, sangre o LCR) que puede ser de inicio agudo o crónico. Valor normal de la PIC 3-15 mmHg (70-150 mmH2O).

Las causas pueden ser benignas (como las resumidas en la tabla) o por lesiones ocupantes de espacio (tumores, abscesos, granulomas, hematomas subdurales) del volumen de LCR (hidrocefalia), aumento del volumen cerebral (infarto cerebral, hemorragia intradural, metabólicas como hiponatremia), aumento del volumen cerebral y sanguíneo (TEC, meningitis, eclampsia).

25 Facultad de Medicina, Universidad de Chile

Diagnóstico



El diagnóstico es clínico.

Se puede observar parálisis del VI par, que puede producir diplopia

Síndromes de herniación: • Herniación subfalcial (herniación bajo hoz del cerebro): Ictus por oclusión de arteria cerebral anterior o posterior • Herniación uncal (herniación a través de la tienda del cerebelo): Compresión del III par y empujando pedúnculo cerebral contra tienda del cerebelo, dilatación pupilar ipsilateral, hemiparesia contralateral y oclusión de arteria cerebral posterior. • Herniación amigdalina (a través de foramen magno): Produciendo oclusión de bulbo raquídeo con colapso cardiorrespiratorio. • Desplazamiento hacia abajo del diencéfalo a través de la tienda del cerebelo, resultando en “pseudotumor cerebri”, que se caracteriza por no evidenciarse etiología (RM o angioRM) y una punción lumbar que evidencia aumento de presión intracraneana, pero con LCR sin alteraciones.

Tratamiento Estabilización y hospitalización en UCI. Monitorizar PIC (objetivo: PIC<20mm Hg y PPC>70mm Hg). El tratamiento puede incluir: elevar cabecera de cama 15 a 30°, Osmoterapia con manitol 20% o suero hipertónico, dexametasona en caso de lesiones inflamatorias o en lesiones por TEC, sedación, hiperventilación controlada (controversial), tratamiento vasopresor para mantener PAM que asegure una PPC>70mm Hg (manteniendo euvolemia), ventriculostomía, drenaje ventricular externo. Una vez estabilizado, realizar estudio y tratamiento según etiología

Los síntomas va a depender de la velocidad de instauración de la hipertensión endocraneana. Es importante detectarla pues puede producir síndromes de herniación y llegar al coma y muerte. Para determinar etiología, es útil la neuroimagen (TAC o RNM). Dado el riesgo de herniación, cuando la sospecha diagnóstica requiere la evaluación con punción lumbar, esta debe se realizar con extremo cuidado (teniendo neuroimagen y fondo de ojo previo). • • •

• •

El pseudotumor cerebri se maneja con acetazolamida, drenaje ventrículo peritoneal para evitar el daño del nervio óptico.

Seguimiento Por especialista.

Cefalea crónica progresiva, mayor en la mañana Vómitos de origen central (que se caracterizan por ser explosivos y sin náuseas) Visión borrosa (dada inicialmente por la vasodilatación venosa y borramiento del borde papilar, hasta llegar al el edema papilar luego 7 a 10 días aprox). No se necesita detectar la presencia de edema papilar para hacer el diagnóstico, ya que es tardío, y además puede no ser pesquisado por un médico no-oftalmólogo que realiza el fondo de ojo) Hipertensión y bradicardia (como mnemotécnica, es todo lo contrario a la sepsis en fases iniciales) Se va produciendo un compromiso de conciencia, dado por bradipsiquia, somnolencia.

Autor / Editor Julián Bravo

26 Facultad de Medicina, Universidad de Chile

Revisor de Formato Angel Eloy

Año 2016

Síndromes Meníngeos con Meningitis Purulenta o Bacteriana

Epidemiología La epidemiología varía de acuerdo a la comunidad, hay alrededor de 1.2 millones de casos de meningitis bacteriana al año en el mundo, disminuyendo según mejor desarrollo sanitario de la población. La patología tiene distribución etaria, en adultos el principal agente es neumococo y luego meningococo, en niños neumococo y meningococo, antiguamente H. influenza. Generalmente inicia con colonización nasofaringe, puerta de entrada secundario a cirugía, vía digestiva (listeria monocitogenes), seguido de bacteriemia con evasión de complemento. Puede entrar a SNC por senos venosos, durales o plexos coroídeos. Se caracteriza por fenómenos inflamatorios de espacio subaracnoídeo, aumento de permeabilidad de BHE, aumento de PIC vasogénico, citogénico o intersticial. También hay descenso de flujo sanguíneo cerebral, pérdida de autorregulación con hipoperfusión cortical, anoxia cerebral, acidosis de LCR.

Nivel de manejo del médico general: Diagnóstico Específico Tratamiento Inicial Seguimiento Derivar

Aspectos Esenciales • • • •

Cuadro clásico: cefalea, fiebre, signos meníngeos. Confirmación con punción lumbar, generalmente purulento, hipoglucorraquia, hiperproteinorraquia leve, Tinción Gram positiva para M.O. Tratamiento antibiótico empírico, precoz y asociado. Derivar para tratamiento definitivo.

Caso Clínico Tipo Hombre de 30 años. Recibe tratamiento prolongado con corticoides post trasplante renal. Presenta fiebre de 38ºC, cefalea que se exacerba con movimientos oculares, fotofobia y somnolencia. Al examen con rigidez de nuca, y esbozo de Brudzinski. La punción muestra un LCR turbio.

Diagnóstico La sospecha es clínica, y el cuadro clínico puede ser agudo o subagudo. De sospecharse un cuadro más silente, crónico se sospecha TBC (con LCR linfocitico) o Criptococosis. El cuadro se caracteriza por fiebre, cefalea intensa y raquialgia. Signos meníngeos: rigidez de nuca y columna (predominio cervical, opistótonos antiálgico), Brudzinski (flexión de EEII tras flexión cervical), Kerning (flexión de rodillas tras extensión de EEII). Fotofobia, fonofobia. Hiperalgesia general. Signos de hipertensión intracraneal: vómitos centrales, edema de papila, bradicardia, bradipsiquia, parálisis de VI par. En la meningitis

Definición Enfermedad provocada por inflamación de las leptomeginges. Se caracteriza por un conjunto de signos y síntomas que traducen un proceso irritativo.

Figura: Signo de Brudzinski por irritación meningea: La rigidez severa del cuello produce que las rodillas y cadera del paciente se flexionen cuando se flexiona el cuello Semiología Médica. Goic. 3a edición. Editorial Mediterráneo

27 Facultad de Medicina, Universidad de Chile

meningocócica, se puede asociar meningococcemia: inicio brusco de rush petequial o purpúreo que se generaliza, y signos de shock. Si presenta compromiso parenquimatoso pueden hallarse focalizaciones neurológicas (de vías largas), compromiso de conciencia y alteraciones psíquicas (Meningoencefalitis) En el laboratorio: Hemograma, PCR, Hemocultivo, Punción lumbar con análisis citoquímico, tinción de Gram, látex y cultivo. Estudios específicos se pueden hacer según la sospecha etiológica. Tinta China en Criptococos Neoformans. Imagen en caso de sospecha de hipertensión intracraneana u otras complicaciones. Es mejor RNM que TAC.

Tratamiento Debe ser precoz y empírico al iniciar la sospecha, y ajustarse tras resultado de cultivos. El tratamiento es hospitalizado intermedio o UCI. Debe incluir: • Cefalosporina de tercera generación (ceftriaxona 2 g ev c/12 hrs, cefotaxima 2 g ev c/6 -8 hrs) y vancomicina (2 g c/ 12 hrs). Si se sospecha encefalitis (HSV), se debe agregar aciclovir. • Se recomienda agregar dexametasona 10 mg cada 6 h durante 4 días, iniciando con la primera dosis de antibiótico, en adultos inmunocompetentes con sospecha clara de meningitis neumocócica (disminuye la probabilidad de secuelas neurológicas). • Aislamiento de protección por gotitas. • Tratamiento específico: se ajusta dependiendo del antibiograma. En general para meningocócica se usa penicilina G, o ceftriaxona o cefotaxima en caso de resistencia a penicilina. En caso de Listeria monocytogenes, uso de Ampicilina. Profilaxis a familiares: Meningococo: rifampicina 600mg cada 12 h por 2 días los adultos, y 10 mg/kg cada 12h durante dos días en niños mayores de 1 año, también puede usarse ciprofloxacino 500 mg vo 1 dosis. Neumococo: cefalosporina y vancomicina. H. influenzae: Rifampicina, Adultos 600 mg/día c/24 hr x 4 días, Niños 20 mg x kg c/24 hr x 4 días.

Seguimiento Hospitalizar y derivar a especialista una vez iniciado el tratamiento inicial con sospecha de meningitis.

Autor / Editor Paulina Lira

Año 2016

28 Facultad de Medicina, Universidad de Chile

Status Convulsivo

personas.

Fisiopatología

Nivel de manejo del médico general: Diagnóstico Sospecha Tratamiento Inicial Seguimiento Derivar

Se produce secundario a la falla de los mecanismos que normalmente abortan una crisis convulsiva, causados por la persistencia de una anormalidad, excitación excesiva (activación excesiva de receptores excitatorios) o reclutamiento inefectivo de la inhibición (elementos que antagonizan la función de los receptores de GABA o producen un cambio en su expresión a nivel de membrana). Status con duración entre 30 a 45 minutos pueden causar daño cerebral, mayormente en estructuras límbicas e hipocampo, mediado por la excito-toxicidad mediada por glutamato liberado al espacio sináptico.

Aspectos Esenciales • • •

El status convulsivo es una emergencia neurológica, que debe ser evaluada y tratada en forma expedita y adecuada. La causa más frecuente es mala adherencia al tratamiento para la Epilepsia. El diagnóstico definitivo es con un EEG.

Diagnóstico La clínica permite hacer el diagnóstico, pero el elemento fundamental es el EEG. Considerar que no todas las crisis convulsivas corresponden a crisis tónico-clónicas, por lo que hace el diagnóstico es la alta sospecha de falta de recuperación de conciencia en un paciente con antecedente de crisis convulsivas anteriores. Las crisis pueden ser de tipo tónica, clónica, tónico-clónicas, atónicas, ausencia. Así como también pueden ser focales o generalizadas.

Caso Clínico Tipo Paciente femenino de 15 años con antecedente de Epilepsia llega a urgencias inconsciente, con ausencia total de tono muscular generalizado. Su madre refiere que ha sufrido 2 crisis convulsivas en los últimos 25 minutos, entre las cuales no ha recuperado la conciencia ad integrum.

Para el diagnóstico etiológico se emplea, además de la historia clínica, el examen físico y examen neurológico completo (evidenciar la presencia de focalidad asociada, evaluar posible lugar de inicio de la crisis según la historia clínica, observar la desviación ocular, movimientos repetitivos de extremidades), el análisis bioquímico (alteraciones metabólicas) y TAC o RM cuando el paciente este estabilizado (Puede mostrar alteraciones estructurales). EEG puede demostrar la persistencia de una actividad de fondo con foco localizatorio o de carácter generalizado. Tipos:

Definición Se definia previamente por la ILAE como convulsiones con duración mayor a 30 minituos. Debido a la urgencia clínica que significan, y es necesaria una rápida evaluación, se utiliza la definición operacional de convulsiones continuas mayor o igual a 5 minutos, o más de 2 de convulsiones discretas sin recuperación completa de conciencia. Esto debido al riesgo cardiovascular y que pueden conducir a daño neurológico irreversible o incluso a la muerte, si no se le trata de forma expedita y adecuada. Considerar que éstas pueden ser focales o generalizadas.



Etiología



Las causas más frecuentes son supresión de fármacos antiepilépticos o falta de seguimiento del tratamiento. Otras causas son trastornos metabólicos (desbalances hidroelectrolíticos), toxicidad de los fármacos y consumo de alcohol, infecciones (encefalitis, meningitis), ACV, hipoxia o tumores del SNC, epilepsia resistente y traumatismos craneales.

Status epiléptico convulsivo generalido: Es la forma más dramática con potencial de provocar serios daños y cmplicaciones, aumentando morbilidada y mortalidad. Incluye tanto las causas primarias como secundarias. Hay deterioro de conciencia, aumento del tono bilateral, seguido de clonías rítmicas simétricas. Status epiléptico convulsivo focal: Tiene manifestaciones clínicas variadas, dependiendo de la localización del área epileptogénica. Es la más fácil de reconocer.

Tratamiento Debe aplicarse de forma inmediata, para evitar disfunción cardiorrespiratoria, hipertermia, alteraciones metabólicas, lesiones neuronales irreversibles y la muerte. Lo primero a realizar es atender cualquier complicación cardiorrespiratoria aguda y la hipertermia (ABC de la reanimación), realizar una rápida exploración médica y neurológica, poner vía intravenosa, y enviar muestras de sangre al laboratorio para realizar estudios con hemograma, uremia, glicemia, GSA, ELP, prue-

Epidemiología Mayor frecuencia en niños que en adultos. 12 a 30% de los epilépticos debutan como status epiléptico. 20% de los niños con epilepsia presentarán 1 o más episodios de status epiléptico. Su incidencia total es de 7 a 41 casos cada 100.000

29 Facultad de Medicina, Universidad de Chile

bas coagulación y enzimas hepáticas. Dejar 5 ml de suero y 50 ml de orina para análisis de niveles de anticonvulsivantes y screening toxicológico. Según el contexto clínico, solicitar Rx Tórax, TAC/RM encéfalo y punción lumbar. El tratamiento farmacológico se debe realizar de forma escalonada de la siguiente manera: 1. 0-5 minutos: Lorazepam 0.1 mg/kg ev, velocidad 2 mg/ min, o diazepam 10-20 mg, ev. velocidad 5 mg/min, como droga de segunda línea. 2. 6-10 minutos: Buscar signos neurológicos focales. Si crisis no cesa, repetir dosis de lorazepam o diazepam. Tiamina 100 mg ev. seguido de 50 ml de sol. glucosada al 50% si existe hipoglicemia o no se puede descartar. 3. 10-30 minutos: Si persisten crisis usar Fenitoína 15-20 mg/kg en bolo ev, a no más de 50 mg/min, en s. salina y bajo monitoreo de presión arterial y frecuencia cardiaca. Si al cabo de su administración no cesan las crisis, agregar 5 mg/kg ev. que puede repetirse si es necesario. Otras opciones al uso de Fenitoína son Fenobarbital o Ácido Valproico. 4. 30-90 minutos: Si persisten crisis, intubar y trasladar a UCI. Administrar Fenobarbital 10-15 mg/kg/dosis a 100 mg min. Si no es efectivo, iniciar coma farmacológico con uno de los siguientes: Propofol: 1-2 mg/kg en bolo y luego 2 a 10 mg/kg/hora, titulando según efecto. Midazolam: 0,1-0,2 mg/kg en bolo, y luego 0,05 a 0,5 mg/ kg/hora, titulando según efecto. Tiopental 3-5 mg/kg de carga ev. y luego titular dosis de 0,3 a 9 mg/kg/hora según respuesta. Como alternativa usar pentobarbital a 10 mg/kg ev. a pasar en una hora, seguido de dosis de mantención de 1 mg/kg/hora. Control o monitoreo EEG para obtener patrón estallido-supresión. Mantener al menos 12 horas sin crisis clínicas o eléctricas para luego disminuir dosis progresivamente.

Seguimiento

Derivación inmediata a especialista después de la estabilización

Autor / Editor Daniela Bustos

Año 2017

30 Facultad de Medicina, Universidad de Chile

Status Migrañoso

mente los mecanismos que promueven la persistencia de la sintomatología. Se ha visto que en muchos casos corresponden a cefaleas mixtas con baja respuesta a medicamentos.

Nivel de manejo del médico general: Diagnóstico Específico Tratamiento Inicial Seguimiento Derivación posterior a especialista

Diagnóstico Es clínico, presenta cefalea recurrente de predominio hemicraneal, alternante y carácter pulsátil. Se acompaña de náuseas, vómitos fonofobia y fotofobia. Puede ser precedido de aura. En el status migrañoso estos síntomas duran de 48-72 horas, pueden acompañarse de intensa emesis que obliga a su hospitalización. Corresponden en su generalidad a pacientes que identifican que es una migraña de clínica similar a las que suelen tener, pero de mayor intensidad y duración. El examen físico neurológico debe ser normal, y visualización de fondo de ojo, sin evidencia de hipertensión endocraneana. Para descartar otras causas descompensantes de una migraña previa o que expliquen la persistencia, se debe realizar punción lumbar y neuroimágenes, para descartar otros diagnósticos.

Aspectos Esenciales • • •

Cefalea de 3 días de duración (>72 horas de duración), con máximo de 4 horas sin dolor. Hay muchos factores desencadenantes. Terapia: Triptán + AINE (im) + Antiemético (ev)+ Hidratación (ev)

Caso Clínico Tipo Paciente de sexo femenino de 40 años, con antecedentes de migraña. Consulta por cefalea constante en los últimos 3 días, acompañada de náuseas, vómitos y diarrea, que le impiden realizar sus actividades de la vida diaria. Refiere que hace unos 4 días tuvo una intensa pelea con uno de sus hijos.

Tratamiento Eliminar fármacos desencadenantes (narcóticos, ergotamínicos). Sedación con benzodiacepinas, rehidratar y administrar antieméticos. Primera elección de tratamiento: Triptán (agonistas selectivos de serotonina) por vía subcutánea (sumatriptán 6 mg), intranasal u oral (Sumatriptán 50-100 mg ; Zolmitriptán 2,55mg) + Metoclopramida 10mg IV cada 8 horas Como alternativa, corticoesteroides: Prednisona (1 – 1,5 mg/ Kg/día) o Dexametasona (4 - 20 mg/día) durante 2 a 4 días y con pauta progresivamente descendente + AINES. No farmacológico: Cambio de estilo de vida, terapia físico, terapias cognitivas.

Definición Crisis de cefalea migrañosa invalidante y de alta intensidad que se prolonga por un tiempo igual o superior a las 72 horas. La cefalea puede ser constante o tener 4 horas máximo sin dolor (sin contar el tiempo de sueño).

Etiología-Epidemiología Se observan factores desencadenantes, entre los que podemos encontrar estrés emocional, factores hormonales, ansiedad, depresión, sobreuso de fármacos o fármacos en específico, etc. Afecta al 2% de la población mundial, los factores de riesgo de transformación de episodios de migraña a su forma crónica son el sexo femenino, genética, historia de injuria cerebral o del cuello, bajo nivel educacional, bajo nivel socioeconómico, vida estresante, edad tempranas de la vida, alodinia cutanea, obesidad, ronquidos, trastornos del sueño.

Seguimiento Derivación posterior a especialista.

Autor / Editor Daniela Bustos

Fisiopatología Actualmente se piensa que la causa es una disfunción serotoninérgica en estructuras del tronco cerebral (núcleos del rafe), que produce a través del sistema trigémino-vascular una hiperexctitación, una respuesta inflamatoria y vasodilatadora de vasos meníngeos y de los tegumentos, además de hipersensibilidad central. Con baja respuesta al uso de medicamentos ya sea por la sobremedicación o en algunos casos por la persistencia del evento causal. Se desconoce exacta-

31 Facultad de Medicina, Universidad de Chile

Año 2017

Esclerosis Múltiple

Epidemiología Segundo lugar en frecuencia (después de los traumatismos) como causa de discapacidad neurológica en etapas temprana y media de la adultez. Edad de presentación entre los 20 y 40 años. Relación mujer:hombre = 3:1. Existe una prevalencia creciente en países cercanos al polo norte: a mayor cercanía, mayor prevalencia, en especial en la población caucásica. En Chile se estima una incidencia de 12 - 15 por cada 100.000 habitantes.

Nivel de manejo del médico general: Diagnóstico Sospecha Tratamiento Inicial Seguimiento No requiere

Aspectos Esenciales • • • • •

Trastorno desmielinizante que afecta al SNC sin compromiso de SNP Presentación más frecuente es la Recurrente Remitente (80%). No hay secuencia temporal ni espacial definida del daño. Los síntomas y signos pueden ser muy variados, dependiendo del sector afectado. No hay prueba diagnóstica única que confirme Esclerosis Múltiple. De elección RNM (permite diagnóstico de diseminación temporal y espacial)

Fisiopatología La lesión característica corresponde a áreas o placas de desmielinización localizadas a nivel de SNC por daño secundario del Oligodendrocito, frente a una reacción autoinmune, generalmente ubicadas a nivel periventricular, subpial, tronco encefálico y nervio óptico. Ocurre una infiltración de linfocitos T y macrófagos con activación de la microglía y mediadores de la inflamación (placa activa). Secundario a la desmielinización ocurre un descenso en la velocidad de conducción pudiendo haber fenómenos de bloqueo. El proceso inflamatorio además puede ser de tal magnitud que puede comprometer los axones y somas neuronales causando degeneración axonal walleriana e incluso focos de necrosis y gliosis (placa “inactiva”), lo que explicaría la existencia de Secuelas neurológicas posterior a la regresión de la placa activa.

Caso Clínico Tipo Paciente de 25 años, consulta en servicio de urgencia, por diplopia de inicio en días, que ha ido progresando y pérdida de la visión en el ojo izquierdo. Asociado a ésto refiere paresia de extremidad inferior izquierda, con comienzo posterior a la diplopia, de carácter progresivo. No refiere compromiso de conciencia ni trauma en el último tiempo, ni tampoco consumo de sustancias durante el último mes. Al examen físico se evidencia paresia de extremidad inferior izquierda y leve paresia en ESD, paresia asociada al VI par derecho y Alteración al examen de campimetría con pérdida de visión unilateral izquierda. Al examen neuroimagenológico se evidencian múltiples lesiones captantes en la RNM de Cerebro.

Clasificación (en orden decreciente de prevalencia): • • • •

Forma Remitente Recurrente Forma Progresiva Secundaria Forma Progresiva Primaria Forma Progresiva Recidivante

Diagnóstico

Definición

Tiene un inicio repentino o insidioso. Conocida como “la gran simuladora”, los síntomas son muy variados y dependen del sitio del SNC afectado. Estos incluyen, en orden decreciente en frecuencia: Pérdida sensitiva, neuritis óptica (disminuye agudeza visual y percepción de color), debilidad de las extremidades, parestesias, diplopía, ataxia, vértigo, entre otras. También puede haber demencia, depresión, disfunción vesical, debilidad facial, etc. Lo más característico de esta enfermedad es la fatiga.

Enfermedad de etiología Autoinmune con desmielinización progresiva, recurrente en forma de placas con inflamación y destrucción selectiva de la mielina en el SNC (Enfermedad del Oligodendrocito), quedando el sistema nervioso periférico indemne. Se caracteriza por la tríada inflamación, desmielinización y gliosis (tejido cicatricial). Las lesiones ocurren en cualquier lugar del SNC y en diferentes momentos.

La cantidad de lesiones evidenciables a la RNM no se correlaciona con la gravedad de la clínica. No hay un método de diagnóstico definitivo. Los criterios diagnósticos exigen documentar 2 ó más episodios de síntomas y 2 ó más signos que reflejen alteraciones de los fascículos de la sustancia blanca anatómicamente no vecinos del SNC.

Etiología Es una patología autoinmune en que participan células T sensibilizadas y células plasmáticas. Se postula que se origina por sensibilización del sistema inmune debido a exposición a antígenos virales o bacterianos similares a antígenos de la mielina en edades precoces.

Los síntomas deben durar más de 24 hrs. y surgir como epi-

32 Facultad de Medicina, Universidad de Chile

sodios separados por un mes o más. Una vez descartadas otras patologías capaces de explicar el cuadro desmielinizante. Se puede evidenciar a la punción lumbar la presencia de bandas oligoclonales, que no son patognomónicas apareciendo en infecciones tales como Sífilis o VIH. En la RM se ven lesiones multifocales hipercaptantes de contraste en encéfalo, tronco encefálico y médula espinal. Ningún signo o prueba aislada confirmarán el diagnóstico de Esclerosis Múltiple. Los criterios diagnósticos específicos incluyen imagenología (RM), análisis de LCR y perfil clínico-temporal.

Tratamiento Hay 3 categorías en el tratamiento: • Tratamiento de los ataques agudos: Metilprednisolona vía ev. a dosis de 1000 mg/día durante 5 días, que puede ser seguido de un ciclo de prednisona por vía oral que disminuirá progresivamente en 2 semanas. • Tratamiento inmunomodulador que reduce la actividad de la enfermedad: Están aprobados IFN-β-1a, IFN-β-1b, acetato de glatiramer y natalizumab. • Tratamiento sintomático: Eminentemente paliativo, que incluye, Bloq. De canales de K (debilidad), fisioterapia y ejercicio regular (espasticidad), anticonvulsivos (dolor), jugo de arándano (infecciones urinarias), etc. El manejo debe ser realizado por médico especialista

Seguimiento Derivar

Autor / Editor Paulina Lira

Año 2016

33 Facultad de Medicina, Universidad de Chile

Neuralgia Esencial del Trigémino

en mujeres, siendo la razón M:H = 1,5:1. La hipertensión arterial podría ser un factor de riesgo.

Nivel de manejo del médico general: Diagnóstico Sospecha Tratamiento Inicial Seguimiento Derivar

Fisiopatología La compresión o alteraciones del nervio causan desmielinización de fibras sensitivas que se tornan hiperexcitables y acopladas con fibras nociceptivas por lo cual comienzan a aparecer los síntomas. Posteriormente potenciales ectópicos gatillados por diversos estímulos se transmitirán por fibras sensitivas sensibles a dolor, con posterior irradiación a lo largo del territorio nervioso.

Aspectos Esenciales • • • •

Dolor neuropático, cutáneo paroxístico, breve, gatillado, sin síntomas neurológicos. Frecuente en 2° y 3° rama. Distinguir dolor esencial del secundario. Descartar otras focalidades neurológicas asociadas. Carbamazepina primera línea. Derivar para tratamiento.

Diagnóstico Dolor paroxístico neuropático superficial que ocurre en la distribución del nervio trigémino, de inicio súbito y término abrupto, breve (típicamente segundos), exacerbado con roce de la cara, bostezo, masticación, con la deglución o al hablar). Sin sintomatología neurológica asociada ni lesiones sobre la piel. Examen neurológico normal. Se puede desencadenar el dolor al presionar puntos específicos de la cara. Debe diferenciarse de otras causas de dolor facial o de cefalea. Los diagnósticos diferenciales, incluyen tumores del ángulo ponto-cerebeloso (meningiomas, colesteatomas, tumores epidermoides, etc.), neuralgia del glosofaríngeo, neuralgia trigeminal atípica, cefalea histamínica de Horton, espasmo facial, neuralgia del ganglio esfenopalatino y neuropatía periférica (suele comenzar con parestesias y el dolor es permanente, sumado a déficit sensitivo y/o motor en la región trigeminal y con una duración de semanas o meses)). Se debe evaluar:

Caso Clínico Tipo Mujer de 65 años. En las últimas 2 semanas presenta dolor muy intenso EVA 7 en pómulo izquierdo que caracteriza como electrizante, que inicia cuando apoya la cara en la almohada y de segundos de duración, en al menos 10 accesos por hora. No ha cedido con paracetamol. Al examen clínico no se evidencian otras focalidades clínicas ni lesiones de piel.

Definición Dolor neuropático paroxístico y recurrente, de corta duración que se ubica en la distribución de una o más ramas del nervio trigémino (V craneal), generalmente las ramas inferiores (Maxilar V2 y Mandibular V3).

• •

Etiología



Se pueden subdividir en Lesiones periféricas (posterior a emergencia desde tronco encefálico, mayoría a nivel del ganglio trigeminal (Gasser) ) y Lesiones Centrales (a nivel de los núcleos sensitivos en el tronco Encefálico). La mayoría de los casos son por compresión del trayecto del nervio trigeminal, usualmente a milímetros de la raíz en el puente. La compresión puede ser por loops aberrantes de vasos (90% de los casos), tumores, aneurismas, etc.

• •

Se clasifica en clásica (idiopática y por compresión vascular generalmente sobre el ganglio trigeminal) y secundaria o sintomática por lesiones estructurales y otras que no sean vasculares.

Compromiso de otros pares craneanos (Schwanoma del VIII Par, Esclerosis Múltiple) Lesiones Pupilares (Aneurismas con compresión nerviosa) Bilateralidad en adulto joven (Descartar esclerosis múltiple u otras patologías virales). Cuando se sospecha neuralgia esencial no es necesario el apoyo diagnóstico. Signos de alarma: La neuralgia sintomática (Secundaria) se caracteriza por: - Aparición en menores de 50 años - uni o bilateral - dolor generalmente continuo - puntos gatillos infrecuentes - asociado a otros signos neurológicos.

Tratamiento Tratamiento farmacológico • Fármaco primera línea: Carbamazepina u Oxcarbazepina (50-75% de los pacientes con buena respuesta). La dosis es de 100 a 200 mg dos veces al día, y luego se va aumentado de 200 mg. En general la dosis de manten-

Epidemiología Incidencia de 4-13/100 mil al año. Comienza en >50 años, y especialmente en >70 años. Rara vez es bilateral. Más común

34 Facultad de Medicina, Universidad de Chile

ción llega a 600-800 mg al día. Fármacos de segunda línea: Lamotrigina o Baclofeno. La dosis de Baclofeno va de 40 a 80 mg al día, la dosis de Lamotrigina es de 400 mg al día. Se usa un mes y se retira gradualmente si es eficaz. (evaluar con Hemograma por riesgo de Neutropenia y de lesiones cutáneas por Sd Steven Johnson. ) • Otros: gabapentina, 800-1600 mg/día; amitriptilina, como coadyuvante, difenilhidantoina 200-400 mg/día. Tratamiento quirúrgico (indicado en caso de fracaso a tratamiento médico) • Rizotomía percutánea (destruir las fibras nociceptivas mediante termocoagulación por radiofrecuencia o trauma mecánico a través de un balón hinchable) • Descompresión microquirúrgica (liberación de arteria sobre el ganglio de Gasser). •

Seguimiento Derivar a especialista

Autor / Editor Daniela Bustos

Año 2017

35 Facultad de Medicina, Universidad de Chile

Neuropatía por Atrapamiento

Etiología El STC se produce por neuropraxia del nervio mediano en la muñeca por el túnel del carpo. El aumento de la presión sobre este es la que provoca el dolor. La realización de movimientos repetitivos con flexoextensión de la muñeca, la manipulación de instrumentos vibratorios, posturas de hiperextensión de la muñeca y realizar fuerza sostenida y repetida, se correlacionan con una mayor incidencia de STC.

Nivel de manejo del médico general: Diagnóstico Sospecha Tratamiento Inicial Seguimiento No requiere

Aspectos Esenciales •



Existen otros cuadros que tienen mayor incidencia de STC como: Diabetes, Amiloidosis, Hipotiroidismo, Embarazo, Acromegalia, Artritis Reumatoide, Enfermedad Renal Crónica. La Neuropatía por atrapamiento del nervio cubital (NANC) puede ocurrir en dos niveles: en su paso lateral al olécranon en la zona del codo o a nivel distal en el canal de Guyon, en el borde cubital de la muñeca. Se produce por compresión del codo sobre una superficie plana por un periodo prolongado de tiempo (cirugías prolongadas, coma, etc.) o traumatismo directo en zona del olécranon. En el caso de compresión distal puede ocurrir en forma idiopática, similar al STC.

La neuropatía por atrapamiento más frecuente es el Síndrome del Túnel Carpiano. Clínica: dolor y parestesias a lo largo del trayecto nervioso. En casos más crónicos y severos atrofia muscular. El tratamiento de elección es la descompresión quirúrgica del nervio.

Caso Clínico Tipo Paciente de sexo femenino de 60 años, trabaja de secretaria en banco. Consulta por adormecimiento y sensación de hormigueo de los 3 primeros dedos de la mano izquierda, que se acentúa por la noche. Refiere además que en el último tiempo tiene dificultad para sostener elementos con la mano, por lo cual se le han caido vasos. Al examen físico se objetivan parestesias a la percusión con un dedo en la zona de la muñeca (Tinel +).

La meralgia parestésica corresponde a la compresión del nervio femorocutáneo o cutáneo femoral entre el ligamento inguinal y la espina ilíaca anterosuperior . Ocurre con mayor frecuencia en personas con abdomen globuloso (pacientes con obesidad mórbida,embarazadas, etc.) o como complicación quirúrgica de cirugías en la espina ilíaca antero-superior. El síndrome del túnel del Tarso corresponde al atrapamiento del nervio tibial en el canal retromaleolar medial. La mitad de los casos son idiopáticos y el resto se debe a quistes sinoviales, gangliones, tenosinovitis, fracturas consolidadas en mala posición, etc.

Definición Neuropraxia (bloqueo temporal de la conducción del nervio) de un tronco nervioso único producto de una distorsión mecánica o compresión secundario a banda fibrosa, un túnel fibroso u osteofibroso o tumoración a lo largo del trayecto nervioso. Se subdividen en: • • •

El neuroma interdigital de morton consiste en la compresión del nervio interdigital típicamente en el espacio entre los metatarsianos tercero y cuarto por una razón anatómica (Unión de los nervios plantar externo e Interno donde se alcanza el mayor volumen de tronco nervioso). Existe un desarrollo posterior de un neuroma que marca la localización mencionada anteriormente. Ocurre con mayor frecuencia en mujeres entre la 5ta y 6ta Década.

Neuropatías por Atrapamiento de Miembros Superiores (Más frecuente es el Sd. del Túnel Carpiano (STC) Neuropatía por atrapamiento del nervio cubital (NANC) Neuropatía de Miembros Inferiores (Meralgia Parestésica, Sindrome del Tunel del Tarso, Neuropatía del Safeno y Neuroma interdigital de Morton)

Epidemiología La neuropatía por atrapamiento más común es el STC, que es más frecuente en las mujeres entre 50 y 70 años. Algunas ocupaciones se relacionan con un mayor riesgo de presentar STC (como empaquetadoras, músicos, técnicos dentales, etc.), aunque esto es controversial.

Fisiopatología La Neuropraxia ocurre por aumento de la presión sobre el nervio, provocando un retardo de la conducción nerviosa, lo que genera la sintomatología característica. En grados severos puede alterar la magnitud de la conducción, con posterior atrofia muscular de los músculos inervados.

Diagnóstico Clínica: Los síntomas más comunes son adormecimiento, entumecimiento, parestesias urentes y dolorosas, torpezas y paresias, que aumentan con posiciones particulares de las

36 Facultad de Medicina, Universidad de Chile

extremidades. También, en los casos de larga duración, puede haber cambios tróficos de la piel, hipotonía y atrofia. •

tratamiento de elección en casos severos.

Seguimiento

STC: Los pacientes presentan adormecimiento, disestesias y dolores de los 3 primeros dedos y la mitad del 4° en la zona anterior, siguiendo el territorio sensitivo del nervio mediano, con exacerbación nocturna. En el examen físico se objetiva hipoestesia del territorio del nervio, además del signo de Tinel (parestesia en el territorio del nervio mediano producida al percutir el ligamento del túnel del carpo). Además es positivo el test de Phalen (empeoramiento de los síntomas a la flexión pasiva máxima de la muñeca sostenida por 1 minuto). En casos más severos se puede observar Hipotrofia o Atrofia de la eminencia tenar.

En un porcentaje de la población existe recurrencia de la lesión por lo que se debe mantener seguimiento clínico en atención primaria en caso de reaparición de síntomas.

Autor / Editor Daniela Bustos



NANC: Hay adormecimiento y parestesia del meñique y del borde medial del antebrazo, que aumenta al momento de flexionar el codo. En casos crónicos se evidencia una “garra cubital” y una atrofia hipotenar. • Meralgia Parestésica: dolor y parestesias en cara anterolateral del muslo, que generalmente aparecen cuando el paciente lleva un tiempo caminando, de pie quieto o sentado con flexión forzada de la cadera. • Síndrome del Túnel del Tarso: Cursa con dolor y parestesias en la zona plantar del pie. • Neuroma Interdigital de Morton: dolor urente durante la marcha y la bipedestación, referido al espacio entre el 3 y 4° ortejo e irradiado hacia el pulpejo de los dos ortejos colindantes. Se agrava con el calor y el uso de calzado estrecho. Se reproduce a la compresión lateral de los metatarsianos o se pinza con los dedos el pliegue del espacio afectado. Estudio radiografía de mano para descarte de fracturas o artrosis. Exámenes de sangre en caso de sospechar una neuropatía secundaria a otra patología. Electromiografía es esencial en el estudio, permite evaluar la velocidad de conducción nerviosa bilateral. Idealmente realizada en ambos lados, para definir tipo y magnitud de la afectación (leve a severa) y además, para descartar otras afecciones de nervios periféricos.

Tratamiento Tratamiento sintomático: Dependiendo del tipo de Neuropatía, existen algunas que son transitorias (meralgia parestésica, STC en algunos casos) por lo que se decide tratar el desencadenante (baja de peso, resolución de embarazo, Manejo del hipotiroidismo). Infiltración local con corticoides, reposo, muñequera ortopédica nocturna (STC). AINEs. Ningún procedimiento ha demostrado ser más efectivo que otro. Tratamiento quirúrgico: Descompresión quirúrgica del nervio (en STC, sección del retináculo flexor con liberación del nervio). Demostrado como

37 Facultad de Medicina, Universidad de Chile

Año 2017

Síndromes Miasténicos

plasia tímica y timoma. Más frecuente en mujeres en edades de 20 a 40 años.

Nivel de manejo del médico general: Diagnóstico Sospecha Tratamiento Inicial Seguimiento No requiere

LE: Trastorno autoinmune presináptico provocado por bloqueo de liberación de Ach, por AC anti canal de Ca2+ voltaje dependiente de la placa motora. Más frecuente en hombres. Paraneoplásico (50% asoc. a ca. de pulmón de cél. pequeñas) o asociado a otras enfermedades autoinmunitarias.

Aspectos Esenciales

BO: Trastorno presináptico, hay diferentes tipos entre ellos el alimentario, por heridas, infantil, entérico del adulto y el desconocido. Es producido por toxina botulínica tipo A, B (conservas de vegetales) y E (pescado), las que bloquean liberación de Ach mediada por Ca2+. Más frecuente en lactante, pero puede aparecer a cualquier edad.

• • • • •

Ptosis, diplopia, y debilidad proximal de extremidades. Diferenciar por examen físico, serología y estudio neurofisiológico. Ejercicio empeora miastenia gravis (MG) y mejora Sd. Lambert - Eaton (LE).. LE buscar cáncer (Sd. Paraneoplásico, más frecuente Cancer Pulmonar). MG: timoma. Asegurar soporte y ventilación.

Diagnóstico MG: Se caracteriza por una debilidad asimétrica, fluctuante, empeora con ejercicio y calor, y mejora con reposo y frio. Generalmente extraocular (ptosis, diplopía), de musculatura craneal (disartria, disfagia, voz nasal) y de extremidades proximal. ROT, pupilas, y SNA intactos. Se clasifica en miastenia ocular, bulbar si presenta disfagia y disartria, y generalizada si abarca miembros y paravertebrales.

Caso Clínico Tipo MG: Mujer de 26 años, consulta en Servicio de urgencia por cuadro clínico caracterizado por ptosis palpebral derecha y diplopía a la mirada lateral derecha, fluctuante. Al examen físico se identifica una “voz nasal” y paresia parcial de los elevadores del paladar. No refiere dolor pero debilidad muscular en cintura escapular y de musculatura proximal de EEII. No se identifican otras focalidades neurológicas.

Crisis miasténica es un evento gatillado por fármacos o infecciones, al que se agrega disfagia, e insuficiencia respiratoria. Otras formas: neonatal transitoria por AC de la madre; congénita, por alteraciones hereditarias de transmisión neuromuscular.

LE: Hombre de 68 años, fumador activo y antecedente de tos crónica. Refiere cuadro clínico de 4 meses de evolución caracterizado por baja de peso y debilidad musculatura proximal, en cintura escapular. Al examen físico se evidencian ROT disminuidos globalmente. Al realizarse TACde Tórax, se evidencia masa espiculada localizada en lóbulo pulmonar derecho.

Un test diagnóstico es el ice peak test: 3-5 mins de hielo sobre globo ocular, mejora ptosis y oftalmoparesia. Test de Tensilon (edrofonio): al igual que con el frio, se inhibe la acción de la acetilcolinesterasa, mejora ptosis y oftalmoparesia. AC anti R-Ach hasta en el 90% de MG generalizada 50% oculares. Es diagnóstico pero si es negativo no es excluyente. Estudio neurofisiológico con “test de estimulación repetitiva” diferencia de otros Sd miasténicos. Imágenes de Timo: TAC de tórax.

Definición Enfermedades que afectan la placa motora (PM) caracterizadas por fatigabilidad y debilidad muscular esquelética, generalmente extraocular y de miembros generalmente a nivel proximal: Miastenia gravis (MG), Sd miasténico de Lambert - Eaton (LE) y botulismo (BO). Así mismo, algunos fármacos pueden inducir síndromes miasténicos.

LE: Se caracteriza paresia proximal de extremidades, prinicpalmente de EEII y cintura pélvica, ptosis y diplopía. Mejora con ejercicio. ROT abolidos, reflejos pupilares disminuidos, disautonomías (impotencia, visión borrosa, estreñimiento). Un test diagnósticos es el estudio neurofisiológico característico, presenta mejoría con estimulación repetitiva de alta frecuencia. Mayor sensibilidad detección de AC Anti canal de Ca2+, positivo en 95% de los pacientes. Es importante solicitar TAC de tórax buscando neoplasia. Cursa también con mialgias, fatigabilidad, disfunción autonómica como sequedad bucal, ocular, anhidrosis, estreñimiento e impotencia.

Etiología MG: Trastorno autoinmune mediado por anticuerpos (AC) anti Receptores de Acetilcolina (ACRA) post-sinápticos en PM, que impiden unión de Ach, estimulan endocitosis de R-Ach, y activan complemento contra mb post-sináptica. Todo lo anterior evita una correcta neurotransmisión y activación muscular a nivel de la placa motora. Se asocia a hiper-

BO: Debut 2 días (alimentos) a 2 semanas (heridas) post exposición a toxina. Primero ocurre disfunción gastrointestinal, luego neurológica: oftalmoplejía externa y ptosis, pupilas arreactivas, disartria, disfagia, ROT abolidos, disautonomías.

38 Facultad de Medicina, Universidad de Chile

Posteriormente paresia de miembros ,que no mejora tras ejercicio físico. La progresión de los síntomas es característicamente descendente (céfalo-caudal). En general se presenta afebril y sin compromisos del nivel de consciencia. Test diagnósticos: Estudio electrofisiológico, semejante a LE.

Tratamiento MG: Asegurar ventilación y manejo de crisis en UCI. Anticolinesterásicos son sintomáticos: piridostigmina (mestinon 60mg c/4-6h oral) y neostigmina (metilsulfato 0,5 mg c/3-4h IM; bromuro 15mg c/3-4h oral). Corticoides e inmunosupresores si hay ausencia de respuesta a corticoides (micofenolato, azatiopririna, ciclosporina, tacrolimus). Plasmaféresis e inmunoglobulinas de uso en crisis. Timectomía, tanto ante timoma como en ausencia de él, donde el 85% mejora y 35% consigue remisión, siendo efectiva dentro de los dos primeros años de enfermedad. LE: Terapia incluye inmunosupresores y plasmaféresis. El mejor resultado se obtiene con el tratamiento del tumor subyacente. Para mejorar la transmisión neuromuscular se usa 3,4-diaminopiridina. BO: Soporte vital hasta que desaparezca el efecto de la toxina, antitoxina equina que no es efectiva en niños. Es importante el control de la vía áerea.

Seguimiento No requiere (referir a especialista y buscar enfermedad de base).

Autor / Editor Paulina Lira

Año 2016

39 Facultad de Medicina, Universidad de Chile

Síndrome Miopatico (Distrofias Musculares, Polimiositis)

Caso Clínico Tipo Hombre de 30 años, con calvicie de predominio frontal, con antecedente de cataras y resistencia a insulina. Refiere cuadro clínico caracterizado por paresia facial, déficit extensor de cuello y de las manos y pérdida de fuerza de la musculatura distal de los miembros. Al examen Físico se observa atrofia muscular generalizada y dificultad para la relajación muscular después de una acción en manos (fenómeno miotónico). Dg: Distrofia miotónica de Steinert.

Nivel de manejo del médico general: Diagnóstico Sospecha Tratamiento Inicial Seguimiento No requiere

Aspectos Esenciales • • •

Definición

Paresia progresiva proximal generalmente de inicio temprano. Existen formas hereditarias y adquiridas Formas inflamatorias asociados a cáncer y enfermedades autoinmunes (polimiositis, dermatomiositis)

Alteraciones estructurales y funcionales del músculo estriado, a nivel histoquímico o anatomopatológico, de origen Genético o Adquirido que se caracteriza por disfunción muscular generalizada, sin afectación del SNC o SNP. Disminución de la potencia muscular en la parte más proximal de las extremidades, de instalación subaguda o crónica, que se acompaña de atrofia muscular, conservando los reflejos osteotendíneos, sin alteraciones sensitivas, con definidos cambios electromiográficos e histológicos.

Compendio de Medicina Interna. C.Rozman. V edición. Editorial Elsevier

Etiología

Los tipos principales son: • Distrofinopatías (alteraciones de distrofina en Duchenne, y Becker) • Distrofia facioescapulohumeral • Distrofia miotónica de Steinert: compromiso de cara y distal de extremidades. • Distrofia muscular de las cinturas (pélvica y escapular): “marcha de pato”.

Las miopatías son un grupo de patologías de causas adquiridas (inflamatorias, endocrinológicas, relacionados a enf. sistémicas, drogas, tóxicos) o hereditarias (distrofias, miotonías, canalopatías, congénitas, metabólicas, mitocondriales). Las distrofias musculares son de carácter hereditario, de inicio generalmente durante la niñez, progresivas, se caracterizan por alteración de proteínas estructurales, y se presentan con diversos fenotipos.

40 Facultad de Medicina, Universidad de Chile

Las Miopatías se pueden subdividir en: • Miopatías congénitas (hereditarias, presentes al nacimiento, pueden o no ser progresivas, debutan con hipotonía, aumento de la extensividad articular, cifoescoliosis, pie cavo y luxación de cadera en su gran mayoría). • Miopatías metabólicas (Alteración de la cadena metabólica de HdeC, lípidos o asociado a otras enfermedades sistémicas, cada una con clínica distintiva) • Miopatías mitocondriales (alteración a nivel mitocondrial con acúmulo anormal de éstas a nivel muscular con depósitos característicos intramitocondriales). Como las miopatías en pacientes con Hipertiroidismo o con Sd. de Cushing. • Enfermedades Autoinmunes asociadas a Miopatías: - Polimiositis corresponde a una enfermedad Autoinmune que se caracteriza por fenómenos inflamatorios a nivel muscular relacionado a inmunidad celular T endomisial, de presentación generalmente en el adolescente y adulto. - Dermatomiositis, en niños y adultos, es producida por autoinmunidad humoral B contra capilares, con atrofia e infiltrado inflamatorio perifascicular, CPK elevada, curso monofásico recurrente de paresia proximal, tracto GI alto, miocardiopatía y fibrosis pulmonar. NOTA: Dermatomiositis y polimiositis pueden estar asociados a neoplasias de mama, pulmón, ovario, gástrico y linfoma. Usualmente se superponen a LES, Enfermedad de tejido conectivo, esclerosis sistémica, AR, Sjögren.

alteraciones gastrointestinales y bloqueo AV. Las polimiositis se caracterizan por ser de causa inflamatoria, adquiridas y potencialmente curables. Presenta paresia progresiva, simétrica e insidiosa, con curso más rápido que las distrofias musculares. Al inicio presenta mialgia y dolor a palpación, que puede progresar con atrofia muscular. Pueden tener afectación de musculatura craneal (generalmente disfagia y rara vez extraocular). Generalmente se encuentra asociada a cuadros autoinmunes como LES o infecciones víricas del tipo Coxsackie. La dermatopolimiositis corresponde a la superposición de la PM asociada a cuadros de manifestación cutánea, con lesiones a nivel de nudillos en forma de pápulas bien delimitadas, coloración rosada (Pápulas de Grotton) y eritema violáceo localizado en zonas de fotoexposición y localizado también alrededor de los párpados (eritema heliotropo). Se asocian en 15% a linfoma no Hodgkin y melanoma. Dentro de los exámenes complementarios, se puede encontrar una CPK sangre elevada, electromiografía con aumento de la actividad muscular espontánea y ondas miopáticas, biopsia muscular con infiltrado intramuscular inflamatorio. En el caso de las distrofias musculares se requiere estudio con genetista.

Tratamiento Diagnóstico

Según la etiología, llevado a cabo por el especialista correspondiente. La mayor parte de las terapias de las distrofias musculares son de tipo soporte sin haberse hallado cura. Las miopatías inflamatorias requieren tratamiento con glucocorticoides e inmunosupresores, y son tratadas en general por neurólogos o reumatólogos.

Importante diferenciar desde el principio con la astenia. Las distrofias que son enfermedades genéticas, se caracterizan por debilidad simétrica proximal de miembros superior e inferior de carácter progresiva, ocurre en músculos voluntarios (También pueden llegar a afectar corazón, sistema GI, ojos, cerebro o columna), con conservación de reflejos y sensibilidad. Suelen ser de inicio temprano, generalmente menor a los 20 años. La paresia puede ser intermitente o persistente. En el caso de la distrofinopatía de Duchenne y de Becker, es característica la pseudohipertrofia de los músculos gemelares, debido al reemplazo de las fibras musculares por depósitos de grasa y tejido conjuntivo. Se puede evidenciar la debilidad muscular proximal con la maniobra de Gowers, el paciente se acuesta y para poder levantarse y pararse lo hace “escalando sobre sí mismo”.

Seguimiento Seguimiento se realiza por especialista en Atención Secundaria. En el caso de los pacientes con antecedentes de Polimiositis y Dermatopolimiositis se debe hacer seguimiento para cáncer, sobretodo linfoma no Hodgkin, melanoma, cáncer de ovario y testicular.

En la distrofia fascio-escapulo-humeral es frecuente la afectación muscular facial con incapacidad para reír o cerrar completamente los ojos, escápula alada y atrofia muscular de biceps y triceps braquial.

Autor / Editor Paulina Lira

La distrofia muscular de Steiner, se caracteriza por fenotipo con calvicie de predominio frontal, atrofia muscular facial, presencia de cataratas en forma temprana (cataratas subcapsulares), y fenómeno miotónico con dificultad para la relajación muscular después de acción. Característicamente empeora con las bajas temperaturas y se activa con el ejercicio muscular. Asociado a lo anterior, hay resistencia a la insulina,

41 Facultad de Medicina, Universidad de Chile

Año 2016

Síndromes Convulsivos Sintomáticos

con sintomatología referida a la localización del lóbulo cerebral. De persistir este foco y producirse consecuentemente excitación neuronal contigua, puede ocurrir generalización si llegase a transmitirse al otro hemisferio a través del cuerpo calloso (fenómeno de generalización secundaria).

Nivel de manejo del médico general: Diagnóstico Sospecha Tratamiento Inicial Seguimiento Derivar

Etiología Considerar siempre el diagnóstico de epilepsia ante la recurrencia de los episodios con similares características clínicas. Dependiendo de la edad pueden dividirse en múltiples etiologías:

Aspectos Esenciales • • •

Las crisis convulsivas pueden ser parciales, generalizadas y no clasificadas o criptogénicas. Se debe realizar diagnóstico diferencial con síncope convulsivo, hiperventilación, narcolepsia, isquemia cerebral transitoria, delirios orgánicos y crisis psicógenas. Criterios de TAC craneal urgente incluye: déficit neurológico focal, persistencia de nivel de consciencia alterada, fiebre, cefalea persistente y TEC reciente, entre otras.

• • •

Neonatos / Lactantes y Niños: hipoxia perinatal, infecciones SNC, alteraciones del desarrollo, hemorragia intracraneal, convulsiones febriles, idiopáticas. Adolescentes: traumatismo, idiopáticas, tumores, consumo de tóxicos Adultos jóvenes y adultos mayores: traumatismo, tóxicos, abstinencia de alcohol, tumores, enfermedad cerebro-vascular, enfermedades degenerativas del SNC.

Caso Clínico Tipo

Diagnóstico

Hombre de 20 años, sin antecedentes mórbidos conocidos, luego de sufrir un trauma a nivel de cráneo, presenta episodio aparente de convulsiones referidas por un segundo observador como rigidez de cuerpo seguido por “espasmos” en cuatro extremidades, con desviación de la mirada hacia derecha. No se tienen más datos del incidente.

Esencialmente clínico, con una anamnesis completa y en lo ideal presencia de un observador, que incluya: Ant. de epilepsia, toma de medicación antiepiléptica, probables desencadenantes: alcohol, drogas, falta de sueño, traumatismo craneal, estímulos visuales, fiebre, enfermedad metabólica y consumo de fármacos. Descripción de la crisis: duración, tipo de crisis, distribución corporal, presencia o no de desviación de la mirada, presencia o no de relajación de esfínteres, mordedura de lengua, estado postcrítico (post ictal), presencia o no y descripción detallada del aura. El examen físico puede aportar elementos de sospecha de alguna lesión focal (compromiso de vías largas, pares craneales, cerebeloso, extrapiramidal, etc) o ser normal. Entre los diagnóstico diferenciales están:

Definición Manifestación de una descarga sincrónica neuronal, que es anormal, excesiva, y simultánea, de forma no recurrente, secundario a un evento particular (trauma, consumo de medicamento, intoxicación, infección, etc) o no conocido. Es importante descartar la recurrencia de los episodios en relación a desencandenantes particulares como la falta de sueño, consumo de alcohol o hiperventilación que pueden encubrir cuadros epilépticos no estudiados.

• •

Fisiopatología

Síncope convulsivo: comienzo gradual, mareo, visión borrosa, palidez, rápida recuperación total sin período post ictal. Convulsión: comienzo brusco, mayor duración del episodio de menos de 5 mins, en ocasiones: aura, cianosis, fase post ictal, estupor.

Dentro de los exámenes complementarios: Hemograma, bioquímica, gasometría, ECG, radiografía de tórax y PL según sospecha de la etiología de la crisis. El EEG es un método complementario que demuestra el carácter irritativo de una crisis, si pesquisa la crisis durante el examen. No es requerido para realizar el diagnóstico, 20% de los casos puede ser normal. La TC y la RM son las técnicas de elección para realizar el diagnóstico diferencial, siendo la RM la más sensible para detectar alteraciones estructurales del SNC y la TC una técnica más sencilla y rápida en situación de urgencia.

Son la consecuencia del desequilibrio entre mecanismos inhibitorios y excitatorios del SNC, producto de una noxa tisular o metabólica. Existe una actividad de descarga generada por entrada de Ca+2 y Na+ al interior de la neurona causando una despolarización prolongada de la membrana. Las descargas repetidas originan un aumento del K+ y de Ca+2 extracelular, además de la activación de receptores NMDA, con lo que se evita la hiperpolarización normal. Producto de la excitación prolongada puede ocurrir descarga de las neuronas contiguas, con lo que se genera un episodio focal,

42 Facultad de Medicina, Universidad de Chile

Criterios de TC craneal urgente • Déficit neurológico focal • Persistencia de nivel de consciencia alterada • Fiebre • Cefalea persistente • TEC reciente • Neoplasia conocida • Tratamiento anticoagulante • Sospecha de infección por VIH • Comienzo con crisis parcial.

Tratamiento Una vez el diagnóstico es realizado debe referirse a especialista para corroborar el diagnóstico e iniciar medicación. No existe una pauta específica acerca del tratamiento, pero puede elegirse un fármaco de acuerdo al tipo de crisis. Es importante recordar que la fenitoína y la carbamazepina están contraindicadas en las crisis mioclónicas y crisis de ausencia típicas. Considerar para el manejo de los pacientes, iniciar con un medicamento en forma paulatina hasta lograr control de las crisis. De no lograrse control con dosis máximas de medicamentos, eventual cambio a otro antiepiléptico en forma paulatina con retiro gradual del otro medicamento (no retirar medicamento hasta lograr estabilizar con el medicamento secundario). De persistir aún con mal control de crisis, eventual combinación de medicamentos (Ac. Valproico con Carbamazepina o Fenitoína). Una vez el diagnóstico es realizado debe referirse a especialista para corroborar el diagnóstico e iniciar medicación. Dosis Fenitoína adultos 4-7 mg/kg/día en dosis cada 8 o 12 horas. Dosis Fenobarbital 50 a 100 mg cada 8 o 12 horas. Se debe evitar eventos desencadenantes, como consumo de sustancias o tóxicos, privación de sueño, consumo de alcohol, manejo de infecciones, resección tumoral, evacuación de hemorragia subdural en caso de traumatismo, etc. En casos en que se evidencia una causa estructural (tumor, lesión puntual con esclerosis, absceso) se requerirá resolución quirúrgica.

Seguimiento Por especialista.

Autor / Editor Paulina Lira

Año 2016

43 Facultad de Medicina, Universidad de Chile

Síndromes Sensitivos

sensitiva tienden a ser de presentación en jóvenes (esclerosis múltiple, Síndrome de Guillian-Barré). También puede estar asociado a enfermedas infecciosas como sifilis, VIH, enfermedad de Lyme, entre otras.

Nivel de manejo del médico general: Diagnóstico Inicial Tratamiento Inicial Seguimiento No requiere

Fisiopatología La percepción sensitiva se inicia a nivel de los receptores sensitivos primarios (terminales libres) y receptores sensitivos secundarios (Paccini, Meisnner, Ruffini). Dependiendo del tipo de estímulo sensitivo se pueden subdividir en dos sistemas Sensitivos: • Sistema Antero-lateral (SAL): Va desde el receptor sensitivo, mediante una 1era neurona con su núcleo ubicado en el Ganglio Radicular con axones que ingresan por la parte posterior de la médula espinal para sinaptar en las Placas de Rexed de la Sustancia Gris, con una 2da Neurona. Los axones de esta Segunda Neurona decusan hacia contralateral posterior al ingreso de la médula y se dirigen hacia Anterior y lateral de la médula espinal, desde donde suben por el Tracto espinotalámico lateral y conectan con el Núcleo Ventral Postero Lateral del Tálamo. Desde allí se proyectan a la Corteza Sensitiva. Ésta corresponde a la vía Termalgésica (dolor y temperatura y tacto grueso). • Sistema Columna Dorsal-Lemnisco Medial: desde el receptor sensitivo se dirige con una 1era neurona con su núcleo ubicado en el ganglio sensitivo, con axones que penetran también por el asta posterior de la médula espinal, desde donde sin hacer sinapsis a nivel medular ascienden directamente por las columnas posteriores de la médula espinal (cordones posteriores) hasta los núcleos ubicados a nivel del bulbo raquídeo (Goll y Burdach) donde hacen sinapsis con una 2da Neurona que decusa hacia contralateral y se dirigen al núcleo ventral postero-lateral del tálamo del lado contrario a su ingreso a la médula, para luego proyectar sobre la Corteza Sensitiva. Esta Vía corresponde a la Vía Epicrítica (tacto fino) - Propioceptiva - Vibratoria. Cualquier lesión en alguno de los niveles de estas vías se traducirá en un síndrome Sensitivo con presentación característica.

Aspectos Esenciales • • • •

Síndrome periférico se reconoce por la distribución de su compromiso neurológico. Para distinguir síndrome medular considerar distribución, presencia de nivel sensitivo y modalidad. Síndrome talámico se caracteriza por hipoestesia e hiperpatía, y componente sensitivo único sin alteraciones motoras. Síndrome sensitivo-parietal tiene dos formas: pérdida completa o gnósica-postural.

Caso Clínico Tipo Paciente de sexo femenino, 50 años con antecedentes de DM tipo 2 Insulino-requirente diagnosticada hace 20 años, presenta un cuadro de meses de evolución caracterizado por parestesias, alodinia e hipoestesia en ambas extremidades inferiores de forma simétrica y que ha progresado hasta comprometer sobre los tobillos.

Definición Corresponden a síndromes clínicos que pueden presentar alteración en la percepción táctil, térmica, dolorosa, propioceptiva o vibratoria frente a estímulos externos. Se clasifican según el lugar de la lesión en corticales, talámicos, medulares y periféricos (radiculares y troncos nerviosos).

Etiología Su clasificación es vasculares, tumorales, infecciosas, traumáticas, metabólicas, tóxicas, etc.

Diagnóstico See deben considerar antecedentes, el perfil temporal del cuadro y las características clínicas de este. El diagnóstico etiológico puede requerir apoyo de imágenes (TAC, RNM), laboratorio específico o electrofisiológico. Para diferenciar topográficamente, es necesario considerar las particularidades de cada uno. Dependiendo del nivel entonces tendremos los siguientes síndromes clínicos:

Epidemiología No tiene un rango etáreo único, pero dependiendo del tipo de etiología existe asociación con ciertos de factores de riesgo, sobretodo en lesiones de tipo vasculares (factores de riesgo cardiovascular: dislipidemia, hipertensión arterial, edad, diabetes mellitus, tabaquismo, etc), así como también en las polineuropatías (consumo de Vit B12, diabetes mellitus). Generalmente las lesiones de tipo traumáticas son lesiones que se dan en población más joven. Y por otro lado algunas causas específicas de enfermedades con manifestación



Neuropatías - Polineuropátias (más frecuente causada por DM2) en guante y calcetín, mononeuropático (más frecuente compresión nervio mediano) distribución característica

44 Facultad de Medicina, Universidad de Chile







de un tronco nervioso. - Radiculoneuropatía con distribución a lo largo del número de raíces comprometidas, poliradiculoneuropatía aguda: pensar en Guillian Barré, mononeuritis múltiple con distribución en parcheado generalmente de una extremidad. Síndromes medulares es fundamental las modalidades comprometidas: - Cordones posteriores: produce déficit propioceptivo y vibratorio con alteración del tacto discriminativo (Evidenciable en deficit de Vit B12 y Neurosífilis) cursan con ataxia posterior y con signo de Romberg (+). Pueden cursar con dolor de tipo lascinante en extremidades inferiores y alteracion de esfínteres. - Cordones anterolaterales: produce déficit termoalgésico (Evidenciable en síndrome del asta anterior, asociado a compromiso motor por infarto medular anterior por obstrucción de la Arteria espinal Anterior). Generalmente asociado a paraplejia o tetraplejia con alteración esfinteriana. - Centromedular: compromiso termoalgésico con nivel suspendido (evidenciable en lesiones tumorales periependimarias, o en siringomielia). Existe un compromiso marcado con nivel suspendido con conservación de la sensibilidad táctil hacia inferior y superior. - Hemisección medular (síndrome de Brown Sequard): déficit propioceptivo, vibratorio y táctil ipsilateral y termoalgésico contralateral. Suele ir asociada a compromiso de función motora ipsilateral por compromiso de vía piramidal. Síndrome talámico (Síndrome Dejerine-Roussy): se produce un cuadro de déficit contralateral a la lesión, con hipoestesia e hiperpatía, sin compromiso motor. Puede asociarse a movimientos coreo-atetósicos (pérdida de propiocepción), incapacidad para la convergencia ocular. Sindrome Sensitivo Cortical: se puede alterar la sensibilidad primaria (pérdida completa) o la discriminativa (gnósico-postural), ésta última con fenómenos de inatención y extinción sensitiva, agrafoestesia y astereognosia (incapacidad de poder decir una letra escrita en la mano y la incapacidad de describir un objeto con los ojos cerrados).

patías) Desde el punto de vista de lesiones medulares: dependiendo de la causa estabilización de lesión en caso de traumatismo y evaluar progresión de los días posteriores secundarios al shock medular agudo. Cuidados generales y medidas antitrombóticas en caso de lesiones asociadas con compromiso motor.

Seguimiento Derivar a especialista.

Autor / Editor Daniela Bustos

Tratamiento El manejo clínico dependerá de la etiología del compromiso sensitivo. Desde el punto de vista vascular: manejo similar al ACV, y manejo de los factores de riesgo cardiovascular. Desde el punto de vista metabólico: evaluar niveles de Vit B12 y aporte IM de vitamina por un periodo de 3 meses. Desde el punto de vista de las neuropatías: liberación de nervio (mononeuropatías), manejo de causas compresivas a nivel de raíces nerviosas (HNP en radiculopatías), inicio de medicamento estabilizadores de membrana para el manejo del dolor como la carbamazepina y pregabalina (polineuro-

45 Facultad de Medicina, Universidad de Chile

Año 2017

BIBLIOGRAFÍA •

1.- de Castro P. Paciente con alteración de conciencia en urgencias. An Sist Sanit Navar. 2008;31 Suppl 1:87-97.



Compendio de Medicina Interna. C. Rozman. V edición. Editorial Elsevier



Epilepsia. Neurología fundamental. A. Yañez. 2º ed. Editorial Mediterraneo



Síndrome Lumbociático. Revista Médica de Clínica Las Condes. Vol. 17 N°1 Enero 2006. Dr. Fernando Larraguibel S. Departamento de Ortopedia y Traumtología, Clínica Las Condes



Ortopedia y Traumatología, sección de Patología ortopédica. Dr. Juan Fortune Haverbeck. Universidad Católica de Chile



Sindromes dolorosos regionales. Revista Médica Clínica Las Condes 2012;23:433-44 - DOI: 10.1016/S0716-8640(12)70334-6



Manual Washington de Terapéutica Médica, 34º ed. Lippincott Williams & Wilkins.



Compendio de Medicina Interna. C. Rozman. V Edición. Editorial Elsevier



Síndrome confusional agudo. Neurología fundamental. A. Yañez. 2º ed. Editorial Mediterraneo.



Medicina Intensiva, C. Lovesio. Editorial El Ateneo



Álvaro Moyano V. El accidente cerebrovascular desde la mirada del rehabilitador. Rev Hosp Clín Univ Chile 2010; 21: 348 - 55



Accidente vascular cerebral isquémico. Neurología fundamental. A. Yañez. Editorial Mediterraneo.



Compendio de Medicina Interna. C. Rozman. V edición. Editorial Elsevier



Becker K. Management of increased intracranial pressure. AAN Syllabi 2000; 8AC.006: 1- 20.



Mellado P, Sandoval P. Hipertensión intracraneana. Cuadernos de Neurología. P. Universidad Católica de Chile



-Drislane F. (2016) Convulsive status epilepticus in adults: Classification, clinical features, and diagnosis In: UpToDate, Post TW (Ed), UpToDate, Waltham, MA. (Accessed on February 02, 2017.)



- Manual CTO de Neurología y Neurocirugía 8ª edición. Grupo CTO



-Garza I. (2016) Chronic migraine In: UpToDate, Post TW (Ed), UpToDate, Waltham, MA. (Accessed on February 02, 2017.)



- Manual CTO de Neurología y Neurocirugía 8ª edición. Grupo CTO



-Briemberg H., Amato A. (2015) Approach to the patient with sensory loss. In: UpToDate, Post TW (Ed), UpToDate, Waltham, MA. (Accessed on February 02, 2017.) - Manual CTO de Neurología y Neurocirugía 8ª edición. Grupo CTO



46 Facultad de Medicina, Universidad de Chile

días, se intensifica hacia las tardes. No suele agravarse con esfuerzos, no se acompaña de náuseas ni vómitos, puede presentar fotofobia. Puede presentar dolor a la palpación de musculatura pericraneal. Su aparición suele asociarse a estresores biopsicosociales. Habitualmente son tolerables y no afectan tan severamente las actividades de la vida diaria, pero en ocasiones puede ser tan intensa que los pacientes consultan al SU.

Cefalea tensional Nivel de manejo del médico general: Diagnóstico: específico Tratamiento: completo Seguimiento: completo

Signos de alarma que orientan a causa secundaria (necesario estudio):

Aspectos esenciales  

  

     

Corresponde a la cefalea primaria más frecuente. De carácter opresivo, holocránea, leve-moderada intensidad, de entre minutos a días de duración. No se agrava con esfuerzos ni se acompaña de náuseas o vómitos. Asociada a estrés. Se debe descartar causa secundaria ante signos de alarma. Tratamiento agudo: medidas generales, AINEs, tricíclicos

  

Caso clínico tipo

       

Mujer de 45 años, consulta por cefalea casi diaria, de intensidad leve, bilateral, localizada en región occipital, de años de evolución, sin náuseas, vómitos ni fotofobia. Sin signos de alarma. Examen neurológico es normal.

Definición Cefalea primaria de intensidad leve a moderada con dolor de carácter opresivo, holoencefálico y descrito como sensación de opresión bilateral, que aumenta con el transcurso del día y se alivia con el sueño. Puede persistir por horas o días, suele responder a analgésicos simples o AINES.

En el adulto mayor siempre buscar un sustrato orgánico, no olvidar la posibilidad de una infección del SNC a pesar de no presentar fiebre. Se sugiere realizar imágenes cuando se sospecha un hematoma subdural crónico.

Tratamiento

Epidemiología



Cefalea primaria más frecuente. Genera gran impacto en la calidad de vida. Entre un 30 a 78% de población general la presentará a lo largo de la vida. Tiene su inicio a cualquier edad, afecta principalmente a mujeres en edad media, sin carácter hereditario (mujeres/hombres : 3/1).





Fisiopatología Se postulan mecanismos periféricos y centrales del dolor. 



Cefalea intensa de comienzo súbito. Relato de “la peor cefalea de la vida”. Empeoramiento reciente de una cefalea crónica. Frecuencia y/o intensidad creciente. Localización unilateral estricta. Cefalea con: trastorno de conducta, crisis epilépticas, alteración neurológica focal, papiledema, fiebre, náuseas y vómitos no explicables por migraña ni por enfermedad sistémica. TEC reciente. Signos meníngeos. Precipitada por esfuerzo físico, tos o cambio postural. Características atípicas. Refractaria a tratamiento. Edades extremas. Predominantemente nocturna. Pacientes oncológicos. Imunocomprometidos. Embarazo. Cefalea que se presenta por primera vez en un adulto mayor.



Mecanismos periféricos: Sobreestimulación de nociceptores de estructuras miofasciales epicraneales. Mecanismos centrales: Se inducirían cambios en mecanismos reguladores del dolor, disminuyendo el umbral de percepción del mismo. Se asocia a factores emocionales (estrés).

No farmacológico: Control de estresores. Higiene del sueño. Relajación. Farmacológico: Paracetamol 1g c/6 hrs o Naproxeno 550 mg c/12 hrs vo. No se deben utilizar antimigrañosos ergotamínicos, ya que prolongan y dificultan el manejo de cefaleas tensionales. Profiláctico: Usar en caso de crisis recurrentes que limitan calidad de vida. Mantener medidas no farmacológicas y agregar medidas farmacológicas: Tricíclicos (Amitriptilina 25-50 mg/noche) o Ciclobenzaprina 5-10 mg/noche. En el adulto mayor siempre buscar un sustrato orgánico, no olvidar la posibilidad de una infección del SNC a pesar de no presentar fiebre. Se sugiere realizar imágenes cuando se sospecha un hematoma subdural crónico.

Seguimiento Completo

Diagnóstico El diagnóstico es clínico. El dolor suele ser referido como opresión o abombamiento, sin carácter pulsátil, localización holocránea o bilateral, intensidad levemoderada, no invalidante, de duración entre minutos a 1

una fase de hiperexcitabilidad del córtex, para presentar luego un síndrome confusional y cerebeloso, hasta llegar finalmente a sufrir un coma.

Complicaciones neurológicas de abuso de sustancias

Los efectos psicoactivos asociados con el consumo del marihuana derivan de la interacción del Δ9-THC con receptores cannabinoides CB1 en el sistema nervioso central, principalmente en los ganglios basales, amígdala, capa molecular del cerebelo y ciertas partes del hipocampo. A bajas dosis produce relajacón, reducción de la coordinación, somnolencia, alteración de la percepción y la concentración, alteración del sentido del espacio y el tiempo. Altas dosis, así como el consumo de forma repetida y prolongada, pueden producir alucinaciones, delirios y deterioro de la memoria.

Nivel de manejo del médico general: Diagnóstico: específico. Tratamiento: inicial. Seguimiento: derivar.

Aspectos esenciales  

   

Alcohol (agudo): Amnesia lacunar, intoxicación idiosincrásica, Intoxicación aguda típica. Alcohol (crónico): encefalopatía de Wernicke, neuropatía periférica, degeneración cerebelosa, encefalopatía portocava, etc. Tratamiento Enf. de Wernicke/neuropatía periférica: Vit. B1. Intoxicación por opiáceos: naloxona. Cocaína: Benzodiacepinas. Intoxicación por BZD: Flumazenil.

La cocaína clorhidrato (HCL), la pasta base y el crack, todos alcaloides derivados de la hoja de coca, producen los mismos efectos clínicos, diferenciados sólo en tiempos de aparición y duración. Su efecto principal corresponde estado de euforia (en pasta base de solo minutos o segundos, mientras que en cocaína de cerca de media hora), ocasionada por el aumento en la concentración de dopamina en circuitos del cerebro que regulan el placer y el movimiento. El uso prolongado, aparte de inducir tolerancia puede desencadenar sensaciones paranoides, ilusiones y aumento del tono muscular (se ponen "duros") con movimientos estereotipados (caminar, abrir y cerrar puertas, hablar sin finalidad).

Caso clínico tipo Paciente varón de unos 40 años es traído a urgencias por haber sido hallado en el suelo en la vía pública. El paciente está poco reactivo, presenta una frecuencia respiratoria de 7 por minuto, una presión arterial de 90/60 mmHg y una frecuencia cardíaca de 38 por minuto. Pupilas mióticas.

Diagnóstico Según clínica específica para cada sustancia 

Alcohol: 

Definición El sistema nervioso es sensible a múltiples toxinas exógenas, entre las que se encuentran aquellas administradas por el mismo sujeto en contexto recreativo y/o de trastorno por consumo de sustancias (abuso y dependencia de sustancias), tales como alcohol, opiáceos, marihuana y cocaína. Las manifestaciones neurológicas derivadas de la exposición a dichos tóxicos y drogas pueden ser agudas o crónicas, dependiendo de la dosis, tiempo de exposición y estructura química de la neurotoxina, que define que estructuras del sistema nervioso se verán afectados.





Epidemiología



Según datos del XI Estudio Nacional de Drogas en Población General de Chile, 2014 SENDA; la prevalencia de consumo de alcohol durante el último mes es de 48,9%, de las cuales 2 de cada 5 presentó al menos un episodio de embriaguez durante mismo periodo. En el mismo estudio aparece una prevalencia de 11,3% para marihuana, 1,4% para cocaína y 0,5% para pasta base. Destaca que la mayor concentración del consumo se da en población adolescente, joven y vulnerable social.







El etanol (alcohol) es reconocido por distintas proteínas (receptores, enzimas), que da cuenta de sus múltiples efectos biológicos, siendo el más estudiado el receptor GABA, compartido con las benzodiacepinas, lo que da cuenta de su reactividad cruzada. A medida que van aumentando la alcoholemia, el sujeto presenta primero



2

Intoxicación aguda típica: Sus efectos dependen de la alcoholemia alcanzada y de la tolerancia. Intoxicación idiosincrásica: Se presenta como una agitación extrema tras ingerir dosis mínimas de alcohol, con amnesia lacunar asociada. Supone un síndrome confusional inducido por el tóxico en personas predispuestas (niños, ancianos con demencia, patología previa del SNC) que no suelen beber de forma habitual. Amnesia lacunar: No se recuerda lo sucedido durante la borrachera, sin que se haya perdido el conocimiento en el transcurso de la misma, por afectación de la memoria reciente. Encefalopatía de Wernicke: Aparece en pacientes alcohólicos y malnutridos debido a un déficit de tiamina o vitamina B1. Se caracteriza por la tríada de oftalmoparesia, ataxia y síndrome confusional. Neuropatía periférica: Provoca polineuropatía mixta (sensitivomotora) de predominio distal. Degeneración cerebelosa: Predominio vermiano (hay alteraciones de la postura y de la marcha). Enfermedad de Marchiafava-Bignami: Infrecuente, degeneración del cuerpo calloso y de la comisura blanca anterior que cursa con demencia y con alteraciones de los movimientos. Temblor postural: Sin constituir una patología en sí, forma parte del cuadro





clínico, temblor fino distal, de predominio en manos y lengua. Mielinolisis central pontina, hematoma subdural (por caídas), dilatación de los ventrículos laterales, esclerosis laminar de Morel (degeneración de la capa IV de la corteza cerebral), miopatía alcohólica. Encefalopatía portocava: Síndrome confusional, fetor hepático, "flapping tremor" (asterixis).



Marihuana 

Intoxicación aguda: Estado de relajación y de euforia, con aumento de la sociabilidad y con disminución de abstracción y de concentración. Son frecuentes las alteraciones perceptivas (ralentización del tiempo). En personas predispuestas, puede desencadenar crisis de pánico con despersonalización grave o psicosis tóxicas con ideación paranoide. Los efectos físicos inmediatos son la hiperemia conjuntival con taquicardia y efectos anticolinérgicos (sequedad de boca).

Seguimiento Derivar

Cocaína 



Intoxicación: Síndrome simpaticomimético, con estimulación, euforia y disminución del cansancio. En dosis altas, pueden aparecer alucinaciones (táctiles o visuales) y delirios (paranoides). Provoca midriasis bilateral reactiva, bruxismo y movimientos estereotipados. Aumenta la temperatura corporal, la frecuencia cardíaca y la presión arterial. Riesgo de ictus, síndrome coronario agudo (vasoconstricción coronaria), arritmias, convulsiones y muerte. Consumo crónico: Parkinsonismo e hiperprolactinemia persistente.

Opioides 

Intoxicación aguda (sobredosis): Cursa con depresión respiratoria, disminución del nivel de conciencia y miosis; es posible que se produzca hipotensión, bradicardia, hipotermia y muerte por paro cardiorrespiratorio.

Tratamiento 

 

 

bien al paciente. La hipertermia se controla con medidas físicas. Las crisis de HTA, los fenómenos isquémicos y las arritmias requieren tratamiento específico, debiendo evitarse los B-bloqueantes puros. Intoxicación aguda por opiáceos: Mantener las constantes vitales hasta que pueda administrarse naloxona (i.v. o s.c), que desencadenará un síndrome de abstinencia agudo si el paciente tenía tolerancia. Si tomó propoxifeno, buprenorfina o pentazocina necesitará dosis elevadas de naloxona y precisará ventilación mecánica.

Intoxicación aguda típica alcohólica: tratamiento es sintomático (soporte ventilatorio, manejo de la hipoglucemia); si existe agitación extrema, antipsicóticos o BZD; es necesario vigilar las complicaciones (accidentes, hipoglicemia, hipotermia, aspiración de vómito, síndrome compartimental); en casos extremos ("coma etílico") se precisará soporte ventilatorio Intoxicación idiosincrásica alcohólica: El tratamiento es sintomático. Encefalopatía de Wernicke: Administración inmediata de vitamina B1, inicialmente por vía parenteral y posteriormente por vía oral. Neuropatía periférica: A veces mejora con la abstinencia y con tiamina. Intoxicación aguda por cocaína: Monitorizar la saturación arterial de oxígeno, la PA y el ECG. inicialmente, se manejan con BZD. Las convulsiones se tratan con diazepam y obligan a realizar un TAC craneal. Hay que hidratar muy 3

Cuadros de deterioro orgánico cerebral potencialmente reversibles

Las lesiones que ocupan espacio son posiblemente las más fáciles de diagnosticar. Por eso se recomienda en el estudio de un paciente con un síndrome demencial considerar siempre un estudio de imágenes.

Diagnóstico

Nivel de manejo del médico general: Diagnóstico: sospecha. Tratamiento: inicial. Seguimiento: no requiere.

Son sugerentes de demencia: 

Aspectos esenciales    



Es importante siempre considerarlas en el diagnóstico diferencial de demencias Las lesiones estructurales deben detectarse precozmente pues permiten revertir el cuadro. Dentro de las causas metabólicas hay que considerar el hipotiroidismo y el déficit de vit. B12 Recordar Lúes y HIV.





Caso clínico tipo



Paciente de 78 años, con deterioro cognitivo de 1 mes de evolución, con antecedentes de una caída accidental 2 meses antes. La familia se queja de que ha cambiado su conducta, que está olvidadizo y más apático. No tiene focalidad en el examen neurológico.





Definición



Trastorno cognitivo causado por etiologías cuyo tratamiento determina una mejoría total o parcial del estado mental. Si el deterioro causa trastornos en las actividades de la vida diaria corresponde a una demencia. Es lo que se ha denominado “demencias tratables”. La tasa de reversión total de este cuadro es habitualmente menor que la de reversión parcial. El pronóstico depende de la edad de presentación, de la magnitud del deterioro cognitivo, y el tiempo de instauración del tratamiento.

Dificultad para aprender y retener nueva información. Se vuelve repetitivo, le cuesta recordar una conversación reciente, eventos, citas; pone los objetos en lugares incorrectos. Dificultad para realizar tareas complejas. Le cuesta realizar una labor que requiere varios pasos, tal como cocinar, balancear una chequera, etc. Deterioro en el capacidad de razonar. Es incapaz de responder con un plan para enfrentar problemas en la casa o en el trabajo. Han dejado de importarle las reglas de conducta social. Desorientación espacial. Tiene problemas para manejar, se pierde en la calle, aún en lugares familiares. Deterioro en el lenguaje. Aparece dificultad creciente para encontrar las palabras adecuadas en una conversación. Alteraciones en el comportamiento. Aparece pasividad, o más irritablilidad que lo habitual; puede malinterpretar estímulos visuales o auditivos. Otros: observar si ha cambiado su hábito de vestirse, puede verse desaseado, desordenado; llega tarde a la hora de la consulta, etc. En el algoritmo diagnóstico de un síndrome demencial debe considerarse siempre un estudio mínimo que incluya o Estudio de imágenes (TAC o RNM con y sin contraste) o Función tiroidea o Función hepática y renal o Niveles de cianocobalamina (vit B12) o VDRL o Hemograma, VHS, Electrolitos plasmáticos

Etiología

Tratamiento

Los síndromes demenciales reversibles llegan hasta un 20% de las demencias, dependiendo de los estudios. Las causas se dividen en condiciones:

El tratamiento de la causa determina una mejoría parcial o total del síndrome demencial. La mejoría no es inmediata y demora un par de meses.

 



   

Neuroquirúrgicas: tumores, hematoma subdural agudo o crónico, hidrocéfalo normotensivo Infecciosas/inflamatorias: neurosífilis, neurocisticercosis, encefalitis, VIH, TBC, vasculitis, esclerosis múltiple, LES, enfermedad de Whipple, enfermedad de Lyme Metabólicas: hipo/hipertiroidismo, hipopituitarismo, hipoglicemia, trastornos del sodio, trastornos del calcio, enfermedad de Wilson Carenciales: vit. B12, tiamina, niacina Endocrinas: hipo e hipertiroidismo, trastornos paratiroideos, diabetes mellitus Tóxicas: intoxicación por alcohol, drogas, metales Otras condiciones: depresión (causa más común de deterioro cognitivo tratable), fármacos, encefalitis límbica, etc)

Seguimiento Por neurólogo especialista.

4

Cuadros deficitarios de funciones cerebrales superiores Nivel de manejo del médico general: Diagnóstico: sospecha. Tratamiento: inicial. Seguimiento: derivar.

Aspectos esenciales    

 

Las disfasias mixtas son comunes, y los elementos característicos de las afasias tardan en aparecer. Considerar terapias fonoaudiológicas en afasias. La atención es fundamental en el examen neurológico y la evaluación de otras funciones. La enfermedad de Alzheimer afecta esencialmente la memoria episódica de corto plazo. La entrevista clínica permite determinar las funciones cerebrales superiores. El manejo es por un equipo multidisciplinario.



Caso clínico tipo Paciente de 63 años, hipertenso, tabáquico activo, es traído por su familia por alteración del habla iniciada hace 40 min. Se presenta alerta, atento, y destaca un lenguaje fluido, continuo, con parafasias y neologismos. Es incapaz de obedecer órdenes y su escritura es incomprensible.



Definición Las funciones cerebrales superiores corresponden a aquellas funciones en las que participan procesos cognitivos en la elaboración de respuestas frente a estímulos del medio (externo o interno). Por ejemplo, lenguaje, memoria, cálculo, etc. 

“mesa”) y fónemicas (“perro” por “cerro”). El paciente no tiene conocimiento de su alteración. La lectura, escritura y la comprensión están afectadas. El sitio de la lesión es la región posterosuperior del lóbulo temporal dominante para el lenguaje. o Afasias de conducción: La repetición se ve afectada, mientras que la comprensión y la fluencia se alteran en menor medida. Las lesiones se ubican en el fascículo arcuato (conexión entre área de Broca y Wernicke). o Afasia nominativa: La nominación se afecta en todas las afasias, sin embargo en las afasias nominativas se pierde la habilidad de nombrar objetos. El resto del lenguaje se mantiene relativamente conservado. Ocurre en lesiones pequeñas de la región temporoparietal o lóbulo frontal del hemisferio dominante. Apraxias: Se definen como la incapacidad de realizar actos propositivos complejos en ausencia de deficiencias motoras y teniendo el propósito de realizarlos. Se le puede pedir al paciente que simule que se peina, que encienda un cigarrillo, etc. Generalmente se dan en lesiones en el hemisferio dominante, exceptuando la apraxia de la marcha que se observa en lesiones frontales bilaterales. Se describen 4 apraxias: o apraxia de construcción o apraxia de extremidades o apraxia del vestir o apraxia oral.. Agnosias: Se definen como una incapacidad en el reconocimiento en ausencia de alteraciones de la percepción sensorial o defectos cognitivos generales. Se describen según la función afectada: prosopagnosia (incapacidad de reconocer rostros), alexia (incapacidad de leer palabras), astereognosia (agnosia táctil), etc.

Diagnóstico

Afasias: Trastornos del lenguaje adquirido a causa de una injuria cerebral. Generalmente afectan la modalidad oral y la escrita (unidimensional). Se evalúan básicamente en cuanto a comprensión, repetición, nominación y lectoescritura. Las afasias se pueden dividir en dos grupos: fluentes (se conserva la articulación, entonación y forma gramatical) y no fluentes (alteración de la articulación, línea melódica plana, brevedad de los enunciados, pérdida de estructura gramatical).Se las puede clasificar de manera general en:  Afasias expresivas (anteriores o de Broca): presentan un lenguaje no fluido, con errores en las palabras (agramatismo), las que se produce con dificultad y a menudo con frustración. Tanto la lectura como la escritura están afectadas, sin embargo la comprensión se mantiene intacta. La lesión se ubica generalmente en la región inferolateral del lóbulo frontal dominante para el lenguaje (izquierdo por lo común para los diestros). o Afasias receptivas (posteriores o de Wernicke): Se caracterizan por un lenguaje fluente, vacío en contenido, con parafasias semánticas (“silla” en lugar de

Los métodos de evaluación de las funciones cerebrales superiores son fundamentalmente clínicos. Inicialmente un completo examen mental neurológico, con las pruebas clínicas que incluyen, sirve para determinar el nivel de funcionalidad del paciente, lo que representa su evaluación inicial. Adicionalmente se pueden utilizar escalas validadas de nivel mental como el Mini Mental Scale Examination (MMSE), Frontal Assesment Battery (FAB) o MoCA Test, pudiendo ser utilizadas como herramienta de screening o como punto inicial para determinar la evolución de un paciente. Por último, las evaluaciones neuropsicológicas son las más completas y específicas en determinar las alteraciones de estas funciones.

Tratamiento Multidisciplinario (neurólogo, kinesiólogo, terapeuta ocupacional, fonoaudiólogo). Se debe tratar la etiología específica, ya sea degenerativa, inflamatoria, vascular, neoplásica o irritativa (epiléptica).

Seguimiento Por especialista. 5



Demencia



Nivel de manejo del médico general: Diagnóstico: sospecha. Tratamiento: inicial. Seguimiento: derivar.  

Aspectos esenciales   



El minimental de Folstein permite de forma rápida evaluar funciones cognitivas superiores. La principal causa de demencia es la enfermedad de Alzheimer La segunda causa de demencia son las de etiología vascular (estas se caracterizan por su aparición brusca, focalidad neurológica y curso clínico fluctuante) Para considerar que es una demencia debe tener la atención intacta, y que una de las alteraciones cognitivas sea la memoria





Demencias traumáticas o demencias postraumáticas (por contusión y/o daño axonal) o hematoma subdural crónico Demencias infecciosas o enfermedad de Creutzfeldt Jacob o demencias postencefalitis o postmeningitis o neurosífilis o SIDA o parasitosis cerebrales (cisticercosis, otros) Hidrocefalia normotensiva Enfermedades de la sustancia blanca o esclerosis múltiple o leucodistrofias o desmielinización postanóxica Demencias tóxicas o alcoholismo o cocaína Demencias por fallas metabólicas o carenciales o demencia por carencia de vitamina B12 o encefalopatía hepática, urémica, hipóxica o encefalopatía de Wernicke

Diagnóstico Caso clínico tipo

El diagnóstico es clínico. Una forma rápida para evaluar la memoria, la orientación temporoespacial, el lenguaje, el cálculo y las praxis visuoespaciales e ideomotoras, es a través del Minimental Test de Folstein.

Mujer que 68 años, que inició a los 60 años cuadro caracterizado por errores inexplicables en la actividad cotidiana, descuido en la higiene personal que se fueron acentuando con el paso del tiempo, llegando a depender de forma absoluta de sus familiares.

  

Normal: >27 Deterioro cognitivo leve: 24-27 Demencia: <24

Definición La demencia es la pérdida de al menos 6 meses, de dos o más capacidades cognitivas (una de ellas debe ser la memoria). Esta pérdida debe interferir con las actividades habituales y se debe excluir un compromiso de conciencia (en la demencia la atención no está alterada, en cambio el delirium se caracteriza por la inatención).

Epidemiología   

En adultos mayores entre 65 y 70 años, afecta un 2%, pero en mayores de 80 años, afecta a un 20% La mayoría de las demencias son irreversibles La principal causa de demencia es la Enfermedad de Alzheimer, luego vienen las demencias vasculares

Etiología 



Demencias degenerativas o compromiso motor tardío (enfermedad de Alzheimer, demencia frontotemporal) o compromiso motor precoz (enfermedad de Parkinson, corea de Huntington, parálisis supranuclear progresiva, ataxias espinocerebelosas) Demencias vasculares o demencia por accidente cerebrovascular único, extenso o en un lugar estratégico o demencia por multiinfarto (Binswanger)

Se debe identificar las causas de demencia reversibles. 

6

La enfermedad de Alzheimer es una demencia cortical, de predominio temporoparietal, en ella se pueden encontrar madejas neurofibrilares, pero el dato más característico son las placas de amiloide



compuestos por la proteína b-amiloide (es lo más característico, pero no patognomónico). La enfermedad de Alzheimer, se asocia a factores de riesgo (como la presencia del gen de la proteína precursora amiloide en el cromosoma 21, gen de la presenilina I en el cromosoma 14 y el gen de la presenilina 2 en el cromosoma 1. Los factores protectores son el genotipo apo-E2, AINEs, terapia estrogénica, nivel educativo. Cuando las demencias tienen un inicio brusco, con mayor razón si hay antecedente de enfermedad vascular cerebral o signos de focalidad neurológica, se sospecha de demencia vascular. En esta, se producen múltiples áreas de infarto, y se asocia a HTA y ateroesclerosis. Destacan la demencia multiinfarto por embolías bilaterales recidivantes y la enfermedad de Binswanger (o encefalopatía aterosclerótica subcortical.

Tratamiento En el tratamiento se debe considerar que las demencias frontotemporales, la degeneración corticobasal, la parálisis supranuclear progresiva no tienen tratamiento farmacológico aceptador. En el caso de la enfermedad de Alzheimer son útiles los anticolinesterásicos (donepezilo, rivastigmina) y antiglutamatérgicos (memantina). Tanto si hay o no un tratamiento específico, se deben tratar las alteraciones emocionales y conductuales usando fármacos antidepresivos, tranquilizantes e hipnóticos. En caso de síntomas psicóticos, preferir neurolépticos (pero se debe estar atento a la aparición de síntomas extrapiramidales). Lo ideal frente a cualquiera de estos tratamiento es iniciar con dosis bajas e ir titulando. Es sumamente importante que los pacientes tengan algún tipo de actividad (aunque estén postrados), lo importante es que sean supervisadas para evitar accidentes. Es bueno que compartan con otras personas, pero dando las facilidades para que pueda seguir la conversación (ojalá sea un interlocutor que lo escuche atentamente).

Seguimiento Idealmente todos los pacientes con demencia deben ser evaluados por un especialista en neurología, aunque después siga en control con su médico de cabecera o en atención primaria. SIn embargo dada la realidad en Chile, esto no siempre es posible, por lo que es útil tener en consideración criterios de priorización de derivación, como los siguientes:     

Menores de 65 años Deterioro cognitivo de menos de un año de evolución Focalidad neurológica Trastornos conductuales importantes Solicitud expresa de la familia

7

parenterales. Pero sólo un pequeño porcentaje de los pacientes que utilizan neurolépticos desarrolla distonías agudas. Se ha observado un ritmo circadiano de la reacción distónica aguda; el 80% de ellas ocurre en las horas de la tarde.

Distonía aguda por neurolépticos Nivel de manejo del médico general: Diagnóstico: específico. Tratamiento: completo. Seguimiento: no requiere.

Diagnóstico Es clínico. Las distonías se presentan entre 12 y 48 horas después del consumo del fármaco, aunque puede aparecer incluso pocos minutos después de la ingesta de éste. Entre el 90-95% de los casos aparecen en los primeros 4 días de tratamiento.

Aspectos esenciales   

La distonía aguda por fármacos se observa más frecuentemente por neurolépticos. El compromiso varía según la edad: es generalizada en niños y focal en adultos. El tratamiento de elección son los anticolinérgicos.

Tratamiento Tranquilizar al paciente. Informar del carácter benigno del trastorno.

Caso clínico tipo Hombre de 26 años en reciente inicio de tratamiento con neurolépticos, presenta blefaroespasmo de inicio brusco.

Fármacos: De elección: Anticolinérgicos como el Trihexifenidilo (Tonaril), 2mg vo c/12 a 6-8 hrs. Otra opción es Biperideno, en distonía dolorosa, dosis de 5 mg por vía intravenosa de forma lenta. Si es necesario se puede repetir la dosis cada 30 min sin sobrepasar los 20 mg. En distonía moderada, dosis de 5mg vía intramuscular. En distonía leve, dosis de 2mg vía oral. La respuesta es casi inmediata. Alternativa: Benzodiazepinas como el Diazepam. Dosis de 5-10 mg vía intravenosa.

Definición La distonía consiste en una contracción muscular involuntaria simultánea de músculos agonistas y antagonistas, que puede ser sostenida o repetitiva, y que a menudo causan sacudidas con posturas anormales. Varía desde contracciones menores de un grupo muscular individual hasta el compromiso grave y discapacitante de múltiples grupos musculares. La distonía es la reacción farmacológica hipercinética aguda más frecuente y casi siempre es generalizada en niños y focal en adultos.

Una vez manejada la distonía aguda, se aconsejará la supresión del fármaco que la desencadenó. Si esto no es posible o se ha administrado un neuroléptico de acción prolongada es necesaria la asociación de anticolinérgicos (Trihexifenidilo o Biperideno) por VO a dosis de 4-8 mg/día durante 5 a 7 días, remitiendo al paciente a su médico para modificación del tratamiento. Si el fármaco responsable del cuadro se empleaba como antiemético, se debe sustituir por otros con menores o nulos efectos extrapiramidales como la domperidona.

Etiología La distonía inducida por fármacos se relaciona sobre todo con los fármacos con acción bloqueante dopaminérgica (ejemplo: antieméticos, como la metoclopramida). Se observa más a menudo con los neurolépticos, y dentro de ellos, con los neurolépticos típicos (Clorpromazina, Haloperidol).

Seguimiento No requiere.

Epidemiología Más frecuente en niños, adolescentes y adultos jóvenes, en general menores a 30 años. La proporción según sexo masculino:femenino en este trastorno es de 2:1.

Fisiopatología Se caracteriza por oleadas de contracción en grupos musculares agonistas y antagonistas. Esto se acompaña de pérdida de la inhibición en múltiples niveles del sistema nervioso, así como aumento en la excitabilidad y reorganización cortical. Se desconoce el mecanismo por el que los neurolépticos producen distonías agudas. La hipótesis más aceptada es que se debe a una sobrestimulación colinérgica al desequilibrarse el sistema dopaminérgico-colinérgico de los ganglios basales. A favor de esta hipótesis estaría la respuesta rápida y prácticamente universal a los anticolinérgicos 8

Enfermedad de parkinsonismos

Parkinson

y

Fisiopatología

Nivel de manejo del médico general: Diagnóstico: sospecha. Tratamiento: inicial. Seguimiento: no requiere.

Las neuronas dopaminérgicas nigroestriatales mueren por una combinación de factores, que incluyen vulnerabilidad genética, estrés oxidativo, disfunción de proteosomas, actividad anormal de cinasa y factores ambientales. La neurodegeneración comienza en el bulbo olfatorio y núcleos del rafe medio, y luego avanza a la sustancia nigra (mesencéfalo) y finalmente a la corteza.

Aspectos esenciales

Diagnóstico

Signos cardinales del la enfermedad de Parkinson: temblor de reposo, rigidez y bradicinecia. En el Parkinson no sólo hay alteraciones motoras, sino también neuropsiquiátricas y autonómicas, entre otras. El tratamiento no es curativo.

Se puede establecer el diagnóstico de Enfermedad de Parkinson con cierta confianza si existe el criterio central de bradicinecia y por lo menos 2 de los 3 signos cardinales: temblor de reposo (parte como el “pildoreo”), rigidez (“rueda dentada”) y alteración de los reflejos posturales. Apoyan el diagnóstico el inicio gradual y bilateralidad con predominio de un hemicuerpo de los síntomas, precedidos de sensaciones vagas de debilidad y fatiga, falta de coordinación, dolor y malestar.

Caso clínico tipo Hombre de 72 años presenta temblor de reposo predominantemente al lado derecho. Se aprecia hipomímico, con sialorrea, encorvado y sudoroso. Al examen físico, se haya rigidez “en rueda dentada” bilateral. La marcha es de pasos cortos y presenta dificultad para girar.

Complementan la sintomatología la hipomimia, disminución del parpadeo, postura encorvada y marcha festinante. También hay manifestaciones no motoras, como son síntomas neuropsiquiátricos (depresión, ansiedad y alteraciones cognitivas), trastornos del sueño, anormalidades sensoriales y dolor, anosmia y alteraciones de la función autonómica (hipotensión ortostática, constipación, polaquiuria, sudoración excesiva y sialorrea).

Definición Los parkinsonismos son un grupo de enfermedades neurodegenerativas progresivas caracterizadas por los signos clínicos de la enfermedad de Parkinson, que incluyen como característica esencial la bradicinecia, y al menos dos de los siguientes elementos: temblor de reposo, rigidez muscular, y alteración de los reflejos posturales. Se agregan generalmente, marcha festinante y postura en flexión. Esto se acompaña de síntomas no motores, que incluyen hiposmia, alteraciones autonómicas, sensitivas, del sueño (trastorno conductual del sueño REM), cognitivas y psiquiátricas. La forma más frecuente de parkinsonismo es la Enfermedad de Parkinson.

Existen signos de alarma que nos harán sospechar que estamos en presencia de un parkinsonismo que no corresponde a Enf. de Parkinson, estos son: síntomas bulbares precoces, inestabilidad postural severa precoz, no presentar respuesta a levodopa, desarrollo precoz de demencia o manifestaciones neuropsiquiátricas, predominio inicial de hemicuerpo inferior y compromiso autonómico precoz.

Tratamiento No hay tratamiento curativo, así que el tratamiento es sintomático y se enfoca en mejorar la calidad de vida. El tratamiento no es tóxico y debe iniciarse tan pronto como se diagnostique la enfermedad. Los fármacos que se pueden utilizar son la preparación de levodopa con inhibidor periférico de dopadescarboxilasa (carbidopa o benserazida) que tiene efecto en el temblor, la bradicinesia y la rigidez, agonistas dopaminérgicos (pramiepxol, rotigotina o cabergolina), o inhibidores de MAO-B (rasagilina). En ciertas situaciones se emplea el tratamiento quirúrgico, sin embargo tampoco es curativo. Otras opciones son: inhibidores de COMT (entacapone), anticolinérgicos.

Etiología Factores de riesgo para los parkinsonismos son los antecedentes familiares, sexo masculino, lesión craneal, exposición a pesticidas. Para la enfermedad de Parkinson se ha demostrado como factor de riesgo: el consumo de agua de bebederos y residencia en medios rurales. Las formas familiares comprenden sólo un 5% de los casos.

Epidemiología La enfermedad de Parkinson afecta a 2 por 1.000 de la población general. En Chile se estima una población de entre 30 a 35 mil afectados. Predominio masculino de 3:2 respecto a población femenina. El 75% de los parkinsonismos son por enfermedad de Parkinson; el resto son producto de otros trastornos neurodegenerativos (Parkinson Plus), enfermedades cerebrovasculares, fármacos y drogas (parkinsonismos secundarios).

Seguimiento Por especialista.

9



Examen de Líquido Cefaloraquídeo



Nivel de manejo del médico general: Nivel: Emplea informe

El examen tiene indicación, pero de forma no urgente en los siguientes casos:

Definición del examen



El líquido cefaloraraquídeo corresponde a un líquido claro e incoloro que se produce en el plexo coroideo. Su función principial es mantener separados el sistema nervioso central de la circulación sanguínea, con el fin de prevenir la entrada mediante difusión simple de fluidos, electrolitos u otras sustancias. Sus características citoquímicas y bacteriológicas ayudan en el diagnóstico de patologías infecciosas del sistema nervioso central, permitiendo distinguir etiología viral de la bacteriana y en este último caso determinar el patógeno causante. Adicionalmente sirve de apoyo en el diagnóstico de patologías no infecciosas como en el caso de hemorragia subaracnoidea.

       

El examen de líquido cefaloraquídeo se realiza a través de una punción lumbar. Para su realización se siguen los siguientes pasos:

 

 





endrocraneana

Corresponden principalmente a las contraindicaciones de la punción lumbar.  



Hipertensión idiopática(pseudotumor cerebri) Carcinomatosis meníngea Meningitis tuberculosa Hidrocefalia normotensiva Neurosífilis Vasculitis del SNC Esclerosis múltiple Síndrome Guillain Barré Síndrome paraneoplásico

Contraindicaciones del examen

Forma de realización del examen



Sospecha de infección del sistema nervioso central(fiebre, alteración del estado mental, cefalea, signos meníngeos). Tiene alta sensibilidad y especificidad para dectar meningitis de origen bacteriano y fúngico. Sospecha de hemorragia subaracnoidea con TAC negativo

Sospecha de hipertensión endocraneana. Trombocitopenia o diátesis hemorrágica (incluido el uso de terapia anticoagulante) Sospecha de absceso espinal epidural Infección del sitio de punción

En pacientes con alguno de las siguientes factores de riesgo, se debe realizar un TAC de cerebro previo a la punción lumbar para descartar lesiones con efecto de masa u otras causas de hipertensión endocraneana:

Acomodar al paciente en posición fetal con el cuello, espalda y extremidades en flexión. Limpiar la piel con alcohol y desinfectante(povidona o clorhexidina) a nivel de las apófisis espinosas entre L3-4 o L4-5. Instalar un campo estéril con paño perforado sobre la zona a puncionar. Administrar anestesia local(Lidocaína) e insertar una aguja espinal 20-22 G que contiene un estilete, en el espacio intervertebral. La aguja se inserta de forma lenta con una ligera inclinación hacia cefálico con la idea de separar las fibras del saco dural en vez de cortarlas. Se puede introducir la aguja, retirando periódicamente el estilete para evaluar la salida de LCR. Una vez que aparece el flujo del LCR el paciente debe extender levemente las piernas para permitir las salida libre de LCR para evitar su aspiración, ya que esta maniobra incrementa el riesgo de hemorragia. Se pueden extraen 40 mL sin problema. Por último, se debe introducir el estilete antes de que la aguja espinal sea extraida para reducir el riesgo de cefalea post punción.

 

   

Inmunocomprometidos (pacientes VIH, en tratamieto inmunosupresor o trasplantados) Historia de enfermedad de sistema nervioso central (accidente cerebro vascular, infecciones focales o masas) Convulsiones de inicio reciente (siete dias previos a la consulta) Papiledema Alteración del nivel de conciencia Déficit neurológico focal

Informe que entrega el especialista que realiza el exámen El líquido cefaloraquídeo se analiza en cuanto a sus características macroscópicas, citoquímicas y bacteriológicas. El líquido cefaloraquídeo normal es claro e incoloro. Los procesos infecciosos como los no infecciosos pueden alterar su apariencia. A partir de 200 GB/microL o 400 GR/microL el líquido cefaloraquídeo se ve turbio y con 6000 GR/microL en adelante se ve sangriento.

Indicaciones del examen Se utiliza en en el diagnóstico de infecciones del sistema nerviso central de origen viral, bacteriano y fúngico. Además, se utiliza como apoyo diagnóstico en neoplasias del sistema nervioso central, enfermedades desmielinizantes y el síndrome de Guillain Barré.

Cuando el líquido cefaloraquídeo adquiere un tinte amarillento o rosado se le denomina xantocrómico. La xantocromia ocurre producto de las degradacion de los GR y se observa comparando el frasco conteniendo el LCR con un frasco con agua. La xantocromía indica que la sangre ha estado presente en el LCR por al menos 2 hrs. Su presencia es útil en el diagnóstico de la hemorragia subaracnoidea, ya que se encuentra presenta

El examen de líquido cefaloraquídeo debe ser realizado con urgencia en las siguientes situaciones:

10

en en mas del 90% de los casos desde las 12 hrs desde el sangrado hasta las 2 semanas posteriores a éste.También se presenta cuando hay concentración de proteinas aumentadas(>=150 mg/dL) o cuando hay hiperbilirubinemia sistémica(BIlirrubina serica >10-15 mg/dL).

parámetro se tiende a normalizar de forma más precoz que la concentracián de proteínas y de células en líquido cefaloraquídeo. En las infecciones de origen viral, su concentración es comúnmente normal. Niveles bajos de glucosa en líquido cefaloraquídeo se pueden ver en en enfermedades no infecciosas como en pacientes hipoglicémicos con síntomas neurológicos, pacientes con carcinomatosis leptomeníngea, leucemia, linfoma del sistema nervioso central, hemorragia subaraconideas severas, o neurosarcoidosis.

El líquido cefaloraquídeo normalmente es acelular, sin embargo se considera normal hasta 5 GB y 5 GR cuando la muestra se obtiene por punción lumbar. Sin embargo, a partir de 3 polimorfonucleares se considera anormal. La muestra debe ser procesada rápidamente, ya que el conteo celular puede resultar falsamente bajo si se mide después de 60 minutos de tomada la muestra. Cuando al analizar el líquido cefaloraquídeo se determina que existe un número elevado de células surge el concepto de pleocitosis. La pleocitosis puede ocurrir en patologías infecciosas e inflamatorias no infecciosas. Los siguientes consejos pueden ser útiles para su interpretación: 

La determinación de lactato en el líquido cefaloraquídeo ayuda a diferenciar entre menigitis de origen bacteriano vs viral. Su valor normal es menor a 3,5 mEq/L y su medición resultó ser superior que el recuento celular, la medición de la concentracion de glucosa y de proteinas para diferenciar menigitis de origen bacteriano vs meningitis aséptica. La citología cumple un rol en el diagnóstico de neoplasias del sistema nervioso central. Para estos propósitos se requiere al menos 10-15 mL de LCR para que lo evalúe el patólogo.

Siempre correlacionar los hallazgos con la clínica. Los polimorfonucleares predominan inicialmente en el líquido cefaloraquídeo de hasta 2/3 de los pacientes con meiningitis debida a enterovirus y el cambio a un predominio linfocitico ocurre dentro de las 12-24 hrs posteriores. En cambio, el predominio linfocitario es muy raro en fases tempranas de meningitis bacteriana.



La presencia de eosinófilos es de utilidad limitada, ya que puede ocurrir en la presencia de parásitos, Mycobacterium tuberculosis, Mycoplasma pneumoniae, Rickettsia ricketssii y otras condiciones no infecciosas como linfoma, leucemia, hemorragia subaracnoidea e hidrocefalia obstructiva o no comunicante



Ante la sospecha de una punción lumbar traumática, que pueda haber aumentado el número de GR y GB en el líquido cefaloraquídeo, se debe estimar el número de GB descontanto 1 GB por cada 500-1500 GR medidos en el LCR

El análisis bacteriológico incluye realizar cultivo y tinción de Gram del líquido cefaloraquídeo. La tinción de Gram debe realizarse siempre que exista sospecha de meningitis bacteriana, ya que tiene la ventaja de sugerir el microorganismo causante de forma más temprana que el cultivo. Dentro de este mismo contexto, existen estudios de diagnóstico rápido como los de aglutacion en látex, que detectan los antígenos de los patógenos más comunes cauxantes de meningitis. Sin embargo, no son recomendados para ser realizados de forma rutinaria, ya que su resultado no cambia la decisión de administrar o no antibioterapia y además se han reportado falsos positivos. Otro de los exámenes disponibles, que es útil en la determinacion etiológica de la meningitis tanto de origen viral como bacteriano, corresponde a la PCR. Su rol se encuentra sobretodo en el caso de detección de enterovirus, Herpesvirus(VHS, VZV, CMV, VEB) y en casos en que los microorganismos sospechados son fastidiosos(Mycoplasma spp y Brucella spp, entre otros) o que se ha utilizado tratamiento antibiótico previo.

El analisis químico del líquido cefaloraquídeo consta de la determinacion de la concentración de glucosa y proteinas principalmente.

Por último, otro de los parámetros importantes a evaluar consiste en la medición de la presión de líquido cefaloraquídeo. El nivel normal de la presión de LCR es menor a 150 mm H2O. Su rango normal varía entre 60250 mm H2O medido con un manómetro en un paciente recostado en decúbito lateral con las piernas extendidas. Cuando su valor se encuentra elevado, se puede deber tanto a enfermedades infecciosas como no infecciosas tales como meningitis, sangrado y tumores. Esto se produce principalmente por alteraciones en los mecanismos fisiológicos de producción y reabsorción de líquido cefaloraquídeo.

La concentración normal de proteínas en el líquido cefaloraquídeo fluctúa entre 23-38 mg/dL (menor a 50 mg/dL como valor de corte) en adultos y 20-170 mg/dL en prematuros y neonatos de término. Sus valores pueden estar elevados en diabéticos, hemorragia subaracnoídea, punciones lumbares traumáticas, enfermedades infecciosas y no infecciosas. En los casos de meningitis, los valores pueden permanecer elevados por semanas a meses después de la recuperación, por lo que no es de utilidad en el seguimiento de la enfermedad. Las bandas oligoclonales, que corresponden a inmunoglobulinas oligoclonaes, se encuentran en razon sangre/LCR de 500/1. Su presencia se observa en Esclerosis Multiple, sin embargo, también se encuentran en otras enfermedades como enfermedad de Lyme, enfermedades autoinmunes, tumores cerebrales, y enfermedades linfoproliferativas, por lo que su utilidad es limitada.

Interpretación del informe que entrega exámen Los hallazgos que orientan a meningitis viral son los siguientes:  El recuento celular usualmente es menor a 250/microL y casi siempre menor a 2000/microL. TÍpicamente se encuentra un predominio linfocitario, aunque en la etapa temprana de la infección puede haber un predominio de neutrófilos que dentro de las siguiente 24 hrs cambian a predominio linfocitario.  La concentración de proteínas es usualmente menor a 150 mg/dL. Una concentración de

La concentración normal de glucosa, tiene una relación LCR/plasma mayor a 0,6. Su valor se puede ver alterado en múltiples enfermedades, sin embargo una concentración inferior a 18 mg/dL es altamente predictiva de meningitis bacteriana. Además ,este fenómeno también se puede observar en meningitis por mycobacterias, mycoplasma, treponema y hongos. Este 11

proteínas mayor de 220 mg/dL reduce la probabilidad de una infección viral a un 1% o menos.  La concentración de glucosa es usualmente mayor a un 50% de la concentración del suero Los hallazgos que orientan a meningitis bacterianas son los siguientes;  Recuento celular sobre 1000/microL, usualmente con predominio de neutrófilos  Concentracion de proteínas sobre 250 mg/dL  Concentracion de glucosa bajo 45 mg/dL La tincion de Gram ayuda a orientar en la etiología de la meningitis bacteriana:  Diplococos gram positivos sugieren infección por neumococo  Diplococos gram negativos sugieren infección por meningococo  Cocobacilos gram negativos sugieren infección por Haemophilus influenzae  Cocobacilos gram positivos sugieren infección por Listeria Los hallazgos que orientan a hemorragia subaracnoidea son:  Xantocromía posterior a las 2 hrs de iniciado los síntomas  Aclaramiemiento sanguíneo de la muestra de líquido cefaloraquídeo con una reducción de un 63% en el conteo de GR entra la primera y la última muestra  Presión de LCR elevada

Signos de alarma Los siguientes hallazgos orienta a meningitis bacteriana con un 99% de certeza o más:  GB sobre 2000/microL(Rango <100 a >10000) con un porcentaje de neutrófilos usualmente superior a un 80%  Conteo de neutrófilos >1180/microL  Concentración de proteínas >220 mg/dL(sobre 200 sospechar)  Concentración de glucosa <34mg/dL (bajo 45 sospechar). Razón LCR/suero menor o igual a 0,4 Los siguiente hallazgos orienta a hemorragia subaracnoidea son:  Presión de LCR elevada  Conteo de GR elevado que no disminuye desde la extracción de liquido con el tubo 1 hasta el tubo 4

Riesgos del examen La punción lumbar es un procedimiento relativamente seguro, sin embargo pueden ocurrir complicaciones pese a que se tomen todas las precauciones correspondientes. Dentro de las complicaciones se pueden mencionar:  Cefalea postpunción  Infección  Sangrado  Herniación cerebral  Síntomas neurológicos como dolor radicular o paresia  Inicio tardio de tumores epidermoide del saco tecal  Dolor lumbar

12

severa, a veces descrita como "la peor cefalea de la vida", que se puede acompañar de compromiso de conciencia, náuseas y vómitos, además de signos meníngeos, síntomas y signos neurológicos focales. Ante sospecha clínica de HSA aguda la persona debe ser derivada en forma inmediata a un centro con capacidad de realizar tomografía computarizada de cerebro (TC) dentro de las primeras 24 horas. Si el TC es negativo, dudoso o no se puede realizar en 24 horas, con alta sospecha clínica, se debe realizar PL. Una PL negativa sin sangre ni xantocromía, confirmado por laboratorio, descarta HSA. Si el laboratorio encuentra glóbulos rojos y/o xantocromía debe considerarse el diagnóstico de HSA.

Hemorragia subaracnoídea Nivel de manejo del médico general: Diagnóstico: sospecha. Tratamiento: inicial. Seguimiento: no requiere

Aspectos esenciales    

Urgencia médica que se presenta con cefalea súbita e intensa, vómitos y pérdida de conciencia. Principal etiología: Ruptura de aneurisma intracraneal. Vigilar: Vasoespasmo. Examen de elección: TC las primeras 12 horas del evento.

Caso clínico tipo Paciente de sexo masculino, 49 años de edad, fumador e hipertenso con historia de cefaleas recurrentes de 2 semanas de evolución. El reciente episodio describe una cefalea de inicio brusco, fotofobia y pérdida transitoria de conciencia. Al ingreso el paciente presenta rigidez de nuca y vómitos.

Definición Hemorragia subaracnoídea (HSA) por rotura de aneurisma cerebral, que corresponde al sangrado intracraneano localizado a nivel del espacio subaracnoídeo secundario a la rotura de un aneurisma. Un aneurisma es una debilidad de la pared del vaso arterial que lleva a una dilatación de esta zona con riesgo de sangrado espontáneo y lesiones secundarias de las estructuras nerviosas del encéfalo o la médula espinal con alto riesgo de mortalidad o morbilidad. Frecuentemente, los síntomas no se presentan hasta que ocurre el sangrado, por lo cual, un aneurisma cerebral roto es una condición de emergencia cuando se lo descubre.

Para confirmar o descartar el diagnóstico de aneurisma cerebral roto, realizar una angiografía selectiva intraarterial, idealmente tridimensional (3D) o angioTC multicorte, dentro de las 48 horas siguientes desde que se diagnosticó HSA. En pacientes con HSA demostrada pero la angioTC multicorte es negativa se recomienda realizar angiografía selectiva intraarterial, idealmente 3D. La indicación de una 2° angiografía en las personas con HSA demostrada pero con angiografía selectiva intra-arterial negativa debe ser evaluada por el especialista. En los pacientes con sospecha de HSA de diagnóstico tardío puede ser necesario complementar el estudio diagnóstico mediante RM ponderada en flair y T2 gradiente, a fin de certificar el sangrado.

Epidemiología La tasa de mortalidad por HSA secundaria a rotura de aneurisma cerebral el año 2004, fue de 2,8 por 100.000 habitantes, con 451 defunciones en ambos sexos. El riesgo de muerte por esta condición fue de 1,9 x 100.000 en varones y 3,7 x 100.000 en mujeres, con 149 y 301 fallecidos ese año, respectivamente. El riesgo de muerte en menores de 15 años es muy bajo, 0,1 x 100.000 y aumenta significativamente a partir de los 35 años, con riesgos muy superiores al promedio en los grupos de 5574 años y 75 años y más, tasas de 8,6 por 100.000 y 16,4 x 100.000 respectivamente. El 55.9% de todas las defunciones ocurre en personas de 55 años y más.

Tratamiento Manejo inicial con sospecha de HSA por aneurisma roto:   

Diagnóstico 

Se considerará en fase aguda si los síntomas se iniciaron dentro de los últimos 21 días. Se considerará de diagnóstico tardío posterior a 21 días de iniciado los síntomas. HSA aguda se presenta con cefalea brusca y 13

Medidas de reanimación (ABC) según condición clínica, considerando: Manejo de vía aérea (Intubación con GCS ≤8), Ventilación. Vía venosa permeable, usar soluciones cristaloides isotónicas (solución salina isotónica, con adición de KCL, a razón de 120 ml/kg/h). El objetivo es mantener una PVC entre 5-8 mmHg, y diuresis > 2ml/kg/h. Se debe evitar soluciones hipotónicas y sueros con glucosa. El manejo de la PA dependerá si el aneurisma roto se encuentra excluido o no. En pacientes con aneurismas rotos no excluidos los cambios de PA podrían favorecer el resangrado. En estos casos

  

  

se recomienda mantener la PA dentro de rangos normales, con PAM < 110 mmHg, tratando presión sistólica cuando supera 160 mmHg con captopril o labetalol. En HSA aneurismática con aneurisma excluido, dado el riesgo de vasoespasmo, se recomienda mantener PAS >140 mmHg, PAD> 90 mmHg y PAM > 110 mmHg. Para subir la presión se recomienda el uso de soluciones salinas iso o hipertónicas monitorizando la presión venosa central (PVC) o presión capilar pulmonar (PCP) y el uso de drogas vasoactivas. Mantener normotermia (temperatura ≤37.2º). Mantener normoglicemia, tratando hiperglicemia sobre 140 mg/dL e hipoglicemia bajo 60 mg/dL. Manejo del dolor, manteniendo dolor bajo EVA 4, utilizando Paracetamol (4 g/día), AINEs endovenosos. Opiáceos en caso necesario. Corticoides, sólo cuando los signos meníngeos son severos. No usar aspirina. El paciente con aneurisma roto demostrado debe ser atendido dentro de las próximas 24 horas en un centro con Unidad de paciente crítico (UPC), Disponibilidad de estudio de imágenes (TC o RM) 24 horas, capacidad de neurocirugía endovascular y/o microcirugía.

Tratamiento definitivo será definido por equipo de especialistas, idealmente conformado por neurólogo, neuro-intensivista, neurocirujano y neurorradiólogo, quienes junto a paciente y/o familiares pueden optar por: Abordaje quirúrgico a través del cráneo para clipar el aneurisma (microquirúrgico a través de craneotomía y acceso al aneurisma, el cual se excluye de la circulación con clips); vía endovascular con coils bajo imagen radioscópica, ingresando un catéter a través de una arteria, alcanzando el saco aneurismático por técnica de microcateterización rellenando su lumen con múltiples coils con el objetivo de lograr su exclusión; o abstención terapéutica. Se debe prevenir y pesquisar eventuales complicaciones neurológicas, entre las cuales están: resangrado, vasoespasmo, hematoma intracerebral, hidrocefalia, crisis convulsivas. Las complicaciones extra neurológicas en la etapa aguda del ACV corresponden a trastornos de la deglución, complicaciones respiratorias (neumonía, edema pulmonar neurogénico, neumotórax y embolia pulmonar), arritmias, disfunción diastólica, necrosis subendocárdica, infección del tracto urinario, constipación, úlceras por presión, trombosis venosa profunda, tromboembolismo pulmonar, trastornos de la comunicación.

Seguimiento Por especialista.

14

vez fiebre (menos del 20% de los pacientes). Puede presentarse como una neuritis aguda con dolor urente, punzante o lascinante, intenso y circunscrito a un dermatoma, 1 a 5 días previo a lesiones cutáneas. Mayor frecuencia T3 a L3. Al examen físico las manifestaciones cutáneas, son máculas y pápulas de base eritematosa que evolucionan en lesiones vesiculosas. Al 3º-4º día pueden tornarse purulentas (en caso de sobreinfección), con distribución a lo largo de un dermatoma (hasta 2 o 3 niveles de lesión), sin cruzar la línea media. Lesiones duran 7-10 días. En pacientes con recurrencia las lesiones puede presentar una ubicación inhabitual, lo cual hace difícil el diagnóstico. El herpes oftálmico se producen por afectación de la rama oftálmica del trigémino, que tiene riesgo de dejar secuelas oculares (úlceras corneales), o más infrecuente, del nervio oftálmico. El herpes zóster en niños debe hacer sospechar inmunocompromiso. El cuadro característico no necesita estudio de laboratorio, considerar en cuadro atípico en inmunocomprometidos, o ante sospecha de enfermedad diseminada. Dentro del laboratorio la RT-PCR (más rápido), detección de antígenos por IF o cultivo celular (muestra reciente y efecto citopático a los 7 días).

Hérpes zóster Nivel de manejo del médico general: Diagnóstico: específico. Tratamiento: completo. Seguimiento: completo.

Aspectos esenciales    

Corresponde a la reactivación del VZV. Genera dolor neuropático unilateral en territorio de un dermatoma (usualmente T3 a L3). Eritema y vesículas circunscritas a dermatoma. Tratamiento con valaciclovir 1g vo c/8h x 5-7 días.

Caso clínico tipo Paciente masculino de 73 años, diabético e hipertenso, refiere dolor en dorso derecho, de carácter urente que se gatilla por el roce de la ropa, de reciente aparición. Al examen físico destaca vesículas con borde eritematoso localizados en la zona del dolor ubicados en la zona del dorso hasta el tórax por anterior, sin cruzar la línea media.

Tratamiento El objetivo del tratamiento es disminuir intensidad y duración de sintomatología; evitar sobreinfección, complicaciones oculares y complicaciones secundarias (parálisis nerviosa de nervios craneales, encefalitis herpética, mielitis) y contagio a terceros; prevenir neuralgia postherpética (se define como 30 días con persistencia del dolor, alodinia local, mayor riesgo de incidencia a mayor edad). Para que funcione, se debe tratar dentro de 72 horas de iniciado el cuadro, y mantener piel limpia y seca para evitar sobreinfección, evitar rascado de heridas locales. Dentro de los fármacos están el valaciclovir 1 g vía oral cada 8 horas (3 al día) por 5-7 días. Otras opciones son famciclovir 500 mg vía oral 2 veces al día por 7 días o aciclovir 800 mg vía oral 5 veces al día por 7-10 días. El tratamiento para la neuralgia post-herpética: En caso de dolor severo agregar AINEs, asociados o no a opioides. Derivar siempre en caso de alteración visual o herpes en territorio oftálmico, a oftalmólogo. Los pcientes inmunocomprometidos dada su condición deben ser hospitalización para tratamiento con aciclovir intravenoso.

Definición Neuritis periférica aguda causada por la reactivación de infección por virus varicela-zóster (VZV) latente en los ganglios sensitivo, manifestada como neuralgia y exantema pápulo-vesiculoso unilateral, ambos restringidos en el territorio cutáneo de la raíz nerviosa afectada. La duración del cuadro habitualmnete es de 7 10 días; no obstante, las lesiones cutaneas pueden persistir hasta por cuatro semanas. Los pacientes con herpes zoster pueden transmitir la infección a individuos seronegativos, con la consiguiente varicela.

Etiología Reactivación de virus varicela-zóster. Los factores de riesgo: uso de corticoides en forma prolongada, diabetes mellitus, inmunosupresión, VIH, edad. No se conocen factores causantes de la reactivación.

Epidemiología

Seguimiento

Aparece a cualquier edad con mayor incidencia en mayores de 60 años. Rara recurrencia (1-5%), salvo en inmunocomprometidos.

La vacuna de virus atenuado (no PAI) se recomienda en personas mayores de 50 años, incluyendo aquellos que ya han tenido herpes zóster. No es necesario determinar si el paciente ha tenido varicela o herpes zóster para recibir la vacuna. Está contraindicada en embarazadas, inmunodeficiencias, radio o quimioterapia, trasplantados, uso de corticoides (≥20 mg/día), terapia inmunomoduladora y pacientes VIH con CD4 <200. Volver a consultar: si los síntomas no disminuyen tras una semana de tratamiento o cuando se agravan por aparición de nuevas lesiones, o cuando aparecen síntomas nuevos como fiebre persistente, dolor de cabeza, fatigabilidad muscular, ceguera, etc. En tales situaciones se debe sospechar inmunocompromiso y/o enfermedad diseminada. El 50% de los pacientes mayores de 50 años refieren neuralgia postherpética durante varios meses o años. Control por médico de atención primaria para manejo en APS.

Fisiopatología Los virus libres presentes en las vesículas de varicela durante la primoinfección infectan terminales nerviosas y migran vía axonal hacia el ganglio regional donde establece latencia. Se desconoce el mecanismo de reactivación, pero es más frecuente en inmunocomprometidos. Tras reactivarse el virus puede infectar otras neuronas del ganglio. La reactivación produce inflamación y necrosis hemorrágica del ganglio, con fibrosis subsecuente de fibras nociceptivas aferentes.

Diagnóstico El diagnóstico es clínico. El pródromo se caracteriza por presentar compromiso del estado general, cefalea, rara 15

Etiología

Lumbociáticas y cervicobraquialgias

Siendo la hernia discal la patología neuroquirúrgica más frecuente que provoca el cuadro. Siendo producida por una degeneración del nucleo pulposo y del anillo fibroso que lleva a una compresión radicular, que lleva a la compresión de raíces nerviosas que pasan por el canal. Las más comunes son las que ocurren en los espacios de L4-L5, y mayor en L5-S1. Puede también darse por causas traumáticas, infecciosas, inflamatorias, tumorales. Entre ellas:

Nivel de manejo del médico general: Diagnóstico: sospecha Tratamiento: inicial Seguimiento: derivar

Aspectos esenciales   

Diagnóstico es clínico. La hernia se produce frecuentemente en forma brusca en jóvenes y lenta en adultos mayores. Lo más importante del tratamiento es el reposo y rehabilitación precoz.

Caso clínico tipo Paciente masculino de 40 años, panadero, con sobrepeso. Inicia hace tres meses, de manera progresiva dolor lumbar sordo, sin predilección horaria, que cede con analgésicos. En el último mes, el dolor se hace más intenso y se irradia a pierna izquierda por su cara posterior, cediendo al final del día con reposo. Al examen físico, fuerza y sensibilidad conservadas, con signo de Lásegue (+).

Definición 



Lumbociática: Dolor lumbar que se irradia al miembro inferior, siguiendo el trayecto del nervio ciático (región lumbosacra, región glútea y cara póstero- externa de la extremidad inferior). Cervicobraquialgia: Dolor cervical irradiado a una de las extremidades superiores a través del territorio de una raíz nerviosa cervical baja (C5, C6, C7 y C8).

Epidemiología En las lumbociáticas, 96% de los afectados tienen la hernia a nivel de L4-L5 ó L5-S1. El 80% de las personas la tendrán en algún momento, de los cuales la mayoría son hombres. En la cervicobraquialgia, las raíces más frecuentemente comprometidas son C7 Y C6.

16

Fisiopatología

Signo de O’Connell: Dolor en el nervio femoral al producir hiperextensión de la cadera.

Degeneración del núcleo pulposo y del anillo fibroso del disco intervertebral por lo que el nucleo sobresale del anillo fibroso (Herniación) pudiendo a llegar a salir del espacio intervertebral (extrusión). El disco herniado provoca compresión de las raíces nerviosas que pasan por el canal pudiendo llegar a radiculopatía o mielopatía.

Se deben evaluar dermatomas de L4, L5 Y S1 en la lumbociática, o C5, C6, C7 Y C8 en caso de cervicobraquialgia, en busca de alteración de la sensibilidad. Pueden también haber alteraciones motoras (parálisis, paresia) o de reflejos patelar, aquiliano y tibial posterior (hiporreflexia o arreflexia).

Diagnóstico

Se solicita radiografía de columna lumbosacra de pie antero-posterior y lateral, y radiografía de quinto espacio en proyección lateral a fin de descartar problemas de alineación, espóndilolistesis lítica o ístmica, tumores u otras patologías especialmente visibles en radiografías. La RNM es el examen de elección en los dos casos. El TAC se solicita secundario a la RNM, para visualizar mejor el detalle óseo de la columna.

Es principalmente clínico, en el caso de la lumbociática se presenta en un inicio como dolor en la región lumbar, que luego se puede irradiar hacia el trayecto del nervio ciático (región glútea, cara posterior del muslo, cara lateral de la pierna y del pie) este dolor aumenta con la flexión de columna y la bipidestación, disminuyendo con el reposo. En el caso de la cervicobraquialgia se presenta como dolor cervical irradiado a la extremidad superior, según la raíz cervical afectada. El dolor característicamente aumenta con la maniobra de Valsalva.

El estudio neurofisiológico se solicita cuando la clínica no es clara para determinar el nivel y la magnitud del compromiso radicular.

Al examen físico, observar marcha en talones y punta de pies, flexión de columna, en la lumbociática destaca la presencia del signo de TEPE (dolor al elevar la pierna extendida a menos de 45°, con el paciente en decúbito supino). Signo de Lásegue que se realiza con la rodilla flectada, se provoca flexión de la cadera en 90º y se extiende posteriormente la rodilla provocando dolor irradiado siguiendo el trayecto del ciático, lo cual es positivo. Signo de Gower's: Aumento del dolor ciático con la dorsiflexión del tobillo.

Tratamiento Es principalmente conservador. Se debe realizar rehabilitación precoz. Reposo en cama en la lumbociática por 2-3 días (No se ha demostrado eficacia), o con collarín blando en la cervicobraquialgia, al que se pueden agregar de primera linea paracetanol, si no mejora AINEs además de calor local. Si con lo anterior el dolor no cede se pueden prescribir relajantes musculares, sedantes o infiltraciones con corticoides peridurales y en articulaciones fascetarias (en caso de mucho dolor), asociado a kinesioterapia. Cerca de un 90% de las lumbociáticas por hernia discal se resuelven con tratamiento médico. Si las molestias y el déficit de sensibilidad o de fuerza no desaparecen a las 6 semanas, se considera tratamiento quirúrgico.

Seguimiento Derivar

17

Fisiopatología

Movimientos anormales inducidos por fármacos

En el caso de los parkinsonismos secundarios al consumo de fármacos, se produce un desbalance de neurotransmisores a nivel central, con disminución del sistema dopaminérgico por bloqueo de los receptores DOPA, en relación al sistema colinérgico a nivel de los ganglios basales, lo que condiciona una clínica similar al Parkinson.

Nivel de manejo del médico general: Diagnóstico: sospecha Tratamiento: inicial Seguimiento: no requiere

En el caso de las diskinesias tardías, se sugiere un efecto de hipersensibilidad de receptores de dopamina con un desbalance entre los receptores D1 y D2 ocasionando los síntomas en forma más tardía con consumo más crónico de antidopaminérgicos.

Aspectos esenciales      

Los bloqueadores de la dopamina son los principales agentes etiológicos. La epidemiología varía para cada síndrome y según los distintos factores de riesgo El diagnóstico es esencialmente clínico. Realizar el diagnóstico diferencial (Enf De Parkinson Idiopático, Parkinson Plus). El tratamiento depende del síndrome específico. Es más común el temblor postural

Epidemiología Varía según una serie de factores, como el consumo de fármacos y cronicidad de consumo, sexo, edad, antecedentes personales y familiares de movimientos anormales inducidos por fármacos, lesión cerebral previa, comorbilidades, alcoholismo, pobre respuesta a antipsicóticos, entre otros. Se reconocen factores predisponentes el sexo masculino (H:M= 2:1) y una edad menor a 30 años. Es importante considerar siempre el uso reciente de medicamentos mencionados anteriormente, que pueden tener efecto hasta tres meses posterior a su consumo.

Caso clínico tipo Paciente varón de 25 años, es trasladado al servicio de urgencia por familiares, debido a cuadro clínico de carácter progresivo (de 12 hrs. de evolución) caracterizado por alteración de la conciencia cualicuantitativa, temblor de extremidades en reposo bilateral de inicio reciente, alteración leve de la marcha, fiebre e hipertermia y disfunción autonómica. Al interrogatorio dirigido, la madre refiere que el paciente había consumido neurolépticos con fines recreacionales, dos días antes del inicio de los síntomas. No se evidencian otras focalidades al examen neurológico.

Diagnóstico El diagnóstico es esencialmente clínico, basándose en una buena anamnesis (preguntando por: movimientos anormales, tiempo de evolución, si son episódicos o permanentes, inicio asimétrico o simétrico, predominio en una extremidad o en un hemicuerpo o bilateralidad, además consumo de fármacos recientes y cronicidad del consumo, comorbilidades, alcoholismo, etc.). Algunos cuadros clínicos son:

Definición



Síndromes clínicos caracterizados por movimientos anormales debido al uso o abuso de una amplia variedad de medicamentos, que en la mayoría de los casos provocan el bloqueo en los receptores de Dopamina a nivel cerebral. Se consideran dentro de los diagnósticos diferenciales de enfermedades neurológicas como el Parkinson o diskinesias idiopática.

Etiología Los principales factores etiológicos son los fármacos que modifican el equilibrio dopaminérgico en el SNC. Son de alto riesgo:      



Neurolépticos convencionales y rara vez los atípicos Antidepresivos (Fluoxetina, Litio) Antiepiléptico (Ac. Valproico) Antieméticos (Metoclopramida) Antihipertensivos (Reserpina y alfametildopa) Bloqueadores de Canales de Calcio (Verapamilo, Flunarizina y Cinarizina)



18

Discinesia aguda inducida por bloqueadores dopaminérgicos: Principalmente caracterizada por una distonía aguda, que surge al poco tiempo de empezar el tratamiento o al incrementar la dosis (2-4 días). La distonía es generalmente focal, y las más frecuentes son la bucofacial y tortícolis. Hay que tener en cuenta una forma especial de distonía que se denomina crisis óculogira (desviación tónica de los ojos hacia un lado). Considerar en el caso de las distonías siempre la presencia del "Truco Sensitivo", que corresponden a acciones que se pueden realizar para suprimir la distonía (rascado del mentón, etc). A veces el paciente no lo asocia al uso de medicamentos, pero los antivertiginosos también lo pueden provocar. Acatisia aguda: Ocurre en alrededor de un 2030% de los pacientes que utilizan fármacos bloqueadores de la dopamina, siendo más común en los que lo utilizan de forma crónica. Se caracteriza por necesidad de moverse constantemente para aliviar una sensación de tensión o disconfort poco definible. Discinesia tardía: movimientos repetitivos, especialmente bucolinguales. Para su diagnóstico es necesaria la exposición previa a fármacos neurolépticos de tres o más meses, desde la



aparación de los síntomas. El riesgo es mayor en pacientes de edad y de sexo femenino. Parkinsonismo secundario: Corresponde al espectro clínico de la enfermedad de Parkinson con características clínicas similares, con bradiquinesia, rigidez, alteración de reflejos posturales y temblor de reposo, de inicio generalmente bilateral, simétrico (puede ser asimétrico inicialmente), con menor grado de desequilibrio postural, con el antecedente de consumo de medicamentos antidopaminérgicos en los últimos tres meses.

Tratamiento Es importante la correcta identificación del síndrome, si se trata de una distonía o acatisia aguda, el tratamiento es suspender el fármaco que causó el cuadro y agregar benzodiacepinas (lorazepam o diazepam) agregar anticolinérgicos centrales (trihexifenidilo) por tiempo corto. En caso de una discinesia tardía, se debe modificar el tratamiento farmacológico. Como generalmente se trata de efectos secundarios al uso de antipsicóticos típicos, se recomienda reemplazar por neurolépticos atípicos, y eventualmente usar depletores dopaminérgicos (tetrabenazina). En caso de complicaciones con poca respuesta a la terapia médica, se puede utilizar en forma eventual inyecciones de toxina botulínica para manejo de las distonías recurrente focales, en forma regular.

Seguimiento Derivar

19

Papiloscopía 

Nivel de manejo del médico general: Realizar

Introducción



El fondo de ojo corresponde a un instrumento importante en la evaluación oftalmológica y forma parte del examen físico de cada paciente.







Figura 1: Fondo de ojo normal (ojo izquierdo).

Indicaciones   

Evaluación oftalmológica. Sospecha de hipertensión endocraneana. Evaluación de patologías sistémicas con compromiso ocular (Diabetes, HTA, VIH, algunas parasitosis como toxoplasmosis).

Complicaciones 

Contraindicaciones 

Ninguna, excepto usar midriáticos en pacientes con glaucoma de ángulo cerrado.

Preparación    

Explicar al paciente el procedimiento. Preparar una habitación semioscura Revisar oftalmoscopio directo, que tenga batería. Se pueden utilizar midriáticos tópicos como la tropicamida, hay que tener en cuenta que su efecto dura 6 horas.

Procedimiento 

del vítreo, alteraciones retinales, retinoblastomas o alteraciones coroideas. Al estar alterado el rojo pupilar disminuye la posibilidad de realizar un fondo de ojo adecuado. Acercándose al paciente en 45º se observa el fondo de ojo. Se recomienda examinar el ojo derecho del paciente con el ojo derecho del examinador. En el fondo de ojo se evalúa primero la papila, la cual se encuentra en la región nasal de la retina, se describe la forma redondeada, plana, de color rosado y de bordes netos. En presencia de hipertensión endocraneana se pierden los bordes de la papila, puede haber pequeñas hemorragias en la misma y ausencia del latido venoso. En la papila encontramos una excavación la cual debe tener una relación en el diámetro vertical papila/excavación menor a 0,4, de lo contrario es sugerente de glaucoma. De la papila emergen vasos retinales, 4 ramas de la arteria central de la retina: Rama temporal superior e inferior y nasal superior e inferior. Se evalúa la relación vénula-arteriola que debe ser 4:3. Se observa además la mácula que se encuentra hacia temporal e inferior de la papila que se reconoce por su mayor pigmentación, finalmente se examina el resto de la retina la cual tiene que tener una coloración rosada homogénea, en patologías como altos miopes o enfermedades degenerativas pigmentaria se pierde esta homogeneidad retineana.

Con el paciente sentado con la mirada fija en un punto, se sostiene el oftalmoscopio con la mano hábil. Podría ser útil poner la mano en la frente u hombro del paciente, y acercarse lenta y progresivamente sin perder el reflejo de los medios oculares,hasta llegar a una distancia de 2,5 cm entre el oftalmoscopio y la córnea del paciente, y así lograr observar el rojo pupilar, el cual tiene que estar presente y ser simétrico. Afecciones que alteren el rojo pupilar: leucomas corneales, opacidades de cristalino, opacidades 20

Fotofobia, disminución de agudeza posterior al uso de midriáticos.

visual

Parálisis facial periférica Epidemiología Nivel de manejo del médico general: Diagnóstico: específico. Tratamiento: completo. Seguimiento: completo.

La incidencia anual es de 13 a 34 casos por 100.000. No hay predilección por sexo. riesgo es 3 veces mayor en el embarazo. Un 10 % de los pacientes con diabetes presenta parálisis facial.

Aspectos esenciales

Diagnóstico

  

El diagnóstico es fundamentalmente clínico. Anamnesis, examen físico y otoscopía. La paresia facial se instaura de forma aguda (horas a 3 días), en 2/3 de los casos es total. Hay que evaluar y observar el cierre de ojos, elevación de cejas, mostrar dientes, fruncir labios, tensión de partes blandas del cuello para observar platisma. Puede presentar ardor de ojos, imposibilidad de aspirar o soplar, desviación de la comisura bucal hacia el lado opuesto, otalgia (con frecuencia antes de la aparición de la parálisis), falta de motilidad hemifacial (incapacidad de fruncir la frente, cerrar los ojos o mostrar los dientes), asimetría facial, lagoftalmo (incapacidad de cerrar completamente el ojo), signo de Bell (al cerrar los ojos, del lado paralizado, el ojo se dirige hacia arriba), puede haber pérdida del gusto en los 2/3 anteriores de la lengua ipsilateral, hiperacusia ipsilateral y algiacusia. El paciente debe tener un examen neurológico normal El principal diagnóstico diferencial: parálisis facial central.

Paresia hemicara superior e inferior, generalmente unilateral. Descartar clínicamente causas secundarias. El tratamiento incluye protección ocular, corticoides, aciclovir (discutido) y rehabilitación motora.

Caso clínico tipo Hombre de 12 años presenta dolor mastoideo izquierdo y trastorno del gusto. Al día siguiente nota pérdida de las arrugas de hemifrente izquierda, dificultad para elevar la ceja, cerrar el ojo y gesticular con la boca. Nota desviación de la comisura bucal hacia la derecha. Refiere no tener episodios previos.

Definición Paresia o parálisis de los músculos de una hemicara completa. Esto la diferencia de la parálisis facial de tipo central que involucra preferentemente los músculos de la hemicara inferior.

Etiología 

 

Parálisis facial idiopática o de Bell: Es la forma más frecuente de PFP. Es de etiología desconocida: Se proponen factores metabólicos, autoinmunes post-infecciosos, isquemia vascular e infecciones virales (en algunos casos se ha aislado el genoma del virus herpes simplex 1 y hérpes zóster). Su incidencia aumenta con la edad. Factores de riesgo: embarazo, diabetes mellitus y presencia de antecedentes familiares. Causas secundarias de PFP: Síndrome de Ramsay Hunt (Herpes zoster del pabellón auricular) lesiones de tronco, PFP asociada a síndrome piramidal contralateral (Síndrome alterno), Otitis media, mastoiditis, Sindrome de Guillain-Barré, Neurinoma del acústico, Meningitis piógena, Carcinomatosis meníngea, fracturas de hueso temporal, Enfermedad de Lyme (Borrelia Burgdorferi), HIV, Sífilis.

21

Figura 27148. Parálisis Facial Semiología Médica. Mediterráneo.

Goic.



Edición.

Editorial

Tratamiento 

En la parálisis facial periférica (A) el ojo permanece abierto, con aumento de la apertura palpebral (lagoftalmo). En esta parálisis participa el facial superior, que inerva el orbicular de los párpados (al paralizarse este músculo predomina el elevador del párpado superior, inervado por el III par). En la parálisis facial central (B), la oclusión del ojo es posible porque el facial sólo está comprometido levemente.



Se recomienda realizar estudio con neuroimágenes en:  Ausencia de mejoría después de un mes  Bilateralidad  Déficit de múltiples nervios craneales  Hipoacusia asociada  Déficit de vías largas.  Audiometría ante sospecha de tumor del ángulo.  U otro síntoma atípico

    

Pruebas de laboratorio (hemograma, test de tolerancia a la glucosa, calcemia y serología) en: Fiebre, baja de peso, lesiones cutáneas en CAE, debilidad progresiva y ausencia de mejora al mes. TAC con contraste en casos de signos atípicos o no mejoría en 3 meses. Estudio de LCR ante sospecha de infección o infiltración neoplásica.

Protección ocular para prevenir úlceras corneales: Lágrimas artificiales durante el día y parche oclusivo con ungüento oftálmico durante la noche. Fármacos: Prednisona 25mg c/12hrs por 10 días, a iniciar dentro de las primeras 72 horas del cuadro. El uso de corticoides sólo acelera la recuperación, no debe utilizarse en situaciones de riesgo. Aciclovir: 400mg 5 veces al día por 5 días, no es deletéreo, asociado a corticoides es igual de efectivo que corticoides solos. No ha demostrado por sí solo, ser mejor que los corticoides. Actualmente no existe evidencia suficiente para recomendar el uso de aciclovir en el tratamiento de la parálisis facial idiopática en Atención Primaria. Paracetamol o AINES para manejo del dolor. Vitaminas, o relajantes musculares no tienen indicación. Rehabilitación motora. Cirugía descompresiva: es controversial Y no está recomendada. Toxina botulínica para complicaciones (espasmo hemifacial).

Seguimiento El 75% se recupera sin secuelas en 4-6 semanas (hasta 6 meses), con recurrencia del 10%. Factores de mal pronóstico son: Edad > 55 años, HTA, diabetes mellitus y paresia severa desde el inicio.

22

corticoespinales, retículoespinales y vestíbuloespinales. Se inicia al momento del trauma con recuperación parcial posterior en las próximas 48 horas de clínica, donde se evidencia las lesiones originales que no tendrán recuperación posteriormente. En relación a las lesiones secundarias de tipo vascular, la irrigación terminal ocurre a nivel de la zona anterior de la médula, donde en episodios de shock hemodinámico, aterotrombosis o isquemia secundario a disección aórtica, ocurre infarto de la zona anterior donde descienden las fibras corticoespinales, con compromiso funcional secundario, sin recuperación de clínica posterior.

Paraplejia aguda Nivel de manejo del médico general: Diagnóstico: sospecha Tratamiento: inicial Seguimiento: no requiere

Aspectos esenciales 

   

Sus etiologías son trauma raquimedular, lesiones vasculares, neoplasias, infecciones, patologías autoinmunes, o enfermedades degenerativas. Es importante buscar otras lesiones asociadas ante trauma raquimedular. Puede acompañarse de otros síntomas y signos neurológicos. Importante evaluar la hemodinamia. El tratamiento médico se basa en la administración de Metilprednisolona.

Diagnóstico El diagnóstico de sospecha es clínico, basado en la anamnesis (antecedentes de traumatismos vertebrales recientes, factores de riesgo de ateroesclerosis, presencia de cuadros infecciosos previos, entre otros), y en el examen físico, en el cual se encuentra el paciente.

Parálisis de inicio agudo con compromiso de las 4 extremidades, generalmente simétrico y que puede asociarse a compromiso de Sistema autónomo, cuya lesión generalmente ocurre a nivel cervical.

Paraplejia (fláccida en fase aguda o espástica en fase subaguda-crónica) de inicio agudo, alteración de los reflejos osteotendíneos y profundos de los territorios afectados y característicamente: alteración en el control de esfínteres y presencia de un compromiso sensitivo con un nivel medular claro. Para su evaluación de altura se deben buscar la función motora y sensitiva según la Escala de ASIA, determinándose el dermatoma y miotoma comprometido según raíces medulares. Por otro lado, también es importante evaluar, si hay: compromiso hemodinámico (bradicardia y/o hipotensión), neurovegetativo (por ejemplo: priapismo), entre otros, y pulsos de extremidades. En el caso específico de paraplejias de origen vascular se suele asociar dolor dorso-lumbar en el segmento afectado. Debuta con paraplejia o tetraplejia aguda con disfunción vesical e intestinal y anestesia dolorosa y térmica por debajo de la lesión sin afectación propioceptiva. El diagnóstico generalmente se confirma con RNM, siendo muy útil en el trauma raquimedular y para evidenciar el efecto isquémico o hemorrágico a nivel medular.

Etiología

Tratamiento

Puede ocurrir secundario a traumatismo raquimedular, lesiones vasculares regionales (arteria espinal anterior que irriga la zona de los cordones motores anteriores) o lesiones inflamatorias (desmielinizantes o con compromiso axonal) de la médula. El trauma raquimedular es una patología prevalente. De alto impacto en salud, calidad de vida, y productividad. Puede estar asociada a otras lesiones. El segmento mayoritariamente comprometido es a nivel cervical y en segundo lugar a nivel lumbar. Generalmente ocurre secundario a accidentes de tránsito. El trauma o lesión vascular pueden comprometer los cordones anteriores de la médula espinal, ocasionando, una paraplejia aguda. Puede ser secundario a accidentes vasculares isquémico trombóticos o disección Aórtica. En el caso de las lesiones inflamatorias, el compromiso suele ser de tipo Inmunológico en su mayoría, siendo su progresión mucho más lenta que en los casos anteriores.

Es primordial identificar la etiología del cuadro para poder tomar una decisión. En el caso del trauma raquimedular, consiste en administración de metilprednisolona en altas dosis en las primeras 8 horas del trauma y mantener por 1-2 días (mantener por 24 horas si se inicio dosis en las primeras 3 horas, por 48 horas si se inicio entre las 3 y 8 horas), inmovilización y tracción cervical, y descompresión quirúrgica. (Según sea el caso). En el caso de afectación vascular el manejo es similar al accidente cerebrovascular, con control de presión horaria, evitando los fenómenos de hipotensión, manejo de glicemia (140 mg/dl como valor máximo, evitando hipoglicemias) y manejo de temperatura. Realizar inicio del estudio etiológico para eventual uso de TACO en caso de fuente embólica identificada. En los cuadros inflamatorios se usa metilprednisolona en dosis altas por 5 días, y se inicia el estudio etiológico de la lesión desmielinizante. La rehabilitación motora y funcional se debe iniciar precozmente en todos los casos.

Caso clínico tipo Recibe en el servicio de urgencia a una paciente femenina, de 60 años fumadora, hipertensa e hipercolesterolémica. , Comenzó cuadro hace 1 hora aproximadamente con dolor intenso y signos de isquemia en EEII, que luego se acompaña de parestesias y paresia. Al examen físico dirigido se aprecia hemodinamicamente estable, y parálisis fláccida de EEII, asociada a hiporrefléxia simétrica de los mismos, ausencia de pulsos en ambas extremidaddes, e isquemia moderada de ambos pies.

Definición

Fisiopatología

Seguimiento

En relación al trauma raquimedular, en el momento del trauma ocurre el denominado "shock espinal" con pérdida de todas las funciones neurológicas hacia distal, por interrupción de la estimulación tónica de los haces

Seguimiento realizado por especialista para control de complicaciones neurológicas secundarias (incontinencia urinaria, constipación crónica, escaras, etc). 23

La mononeuritis múltiple consiste en el compromiso de múltiples troncos nerviosos no contiguos, con compromiso parcheado de una extremidad, generalmente secundario a infecciones o enfermedades vasculares reumatológicas (vasculitis).

Polineuropatías, radiculopatías, mononeuropatías

Epidemiología

Nivel de manejo del médico general: Diagnóstico: sospecha. Tratamiento: inicial. Seguimiento: no requiere.

En general no tienen predilección etárea, pero suelen darse mayoritariamente en pacientes jóvenes entre la 2da y la 5ta década. Dependiendo de la causa, algunas tienen factores de riesgo como en el caso de las PNP (diabetes, PNP por enfermedad VIH, infecciones enterales por Campylobacter jejuni o virus del grupo Herpes, o alteraciones degenerativas de la columna en los casos de las radiculopatías).

Aspectos esenciales 



La distribución anatómica es esencial para el diagnóstico de las afecciones del SNP: o Polineuropatías: síntomas distales en calcetín-guante o Mononeuropatía: territorio de un nervio o Radículopatía: territorio dermatómico y miotómico La causa más frecuente de PNP es la Diabetes

Fisiopatología Dependiendo de la etiología se pueden diferenciar lesiones de tipo desmielinizantes o lesiones de tipo axonal. En las primeras se destacan por ser lesiones que muestran enlentecimiento de la velocidad de conducción, mientras que en las segundas se observa pérdida de la amplitud de la conducción. En algunos casos, como la PNP de la diabetes se pueden comprometer ambos componentes, debido a las lesiones de la vasa nervorum en los terminales nerviosos, secundarios a la hiperglicemia. Por otro lado en las mononeuropatías se genera un fenómeno de neuropraxia que dificulta el paso de neurotransmisores a lo largo del axón en los troncos nerviosos. En las mononeuritis múltiples se observan fenómenos de isquemia secundario a la inflamación arterial que irriga los troncos nerviosos.

Caso clínico tipo Paciente de 65 años de sexo femenino, diabética mal controlado desde los 50 años, con HbA1c de 9%, en tratamiento con Insulina NPH. Lleva alrededor de un año con adormecimiento y dolor quemante en los pies, especialmente de noche, que lo obliga a sacar los pies de la cama, en los últimos dos meses se ha agregado adormecimiento de manos. En el examen tiene hipoestesia táctil y dolorosa en calcetín, con test de monofilamento alterado, y arreflexia aquiliana bilateral.

Diagnóstico

Definición

Los síntomas sensitivos y motores distales con distribución en calcetín y guante son característicos de las polineuropatías.

El sistema nervioso periférico (SNP) puede comprometerse en forma simétrica de predominio distal y de forma gradual como en la polineuropatía; en un territorio dermatómico y miotómico en las radículopatías o en el territorio de uno o más nervios (mononeuropatía, mononeuropatía múltiple).

Los defectos sensitivos (hipoestesia-parestesias-dolor) en el territorio de un dermatomo o de un tronco nervioso permiten sospechar una radiculopatía o una mononeuropatía, respectivamente. Las mononeuropatías múltiples característicamente afectan un tronco nervioso y luego otro en distintos tiempos,generalmente de troncos no contiguos pero que pueden ser confluentes y confunden el diagnóstico.

Etiología Las afecciones del SNP son frecuentes. La causa más frecuente de polineuropatía es la diabetes, pero otras causas, tóxicas, inflamatorias paraneoplásicas, fármacos o infecciones deben considerarse.

En el caso de la poliradiculoneuropatía desmielinizante aguda o síndrome de Guillian-Barré, 2/3 de los casos hay antecedentes de infección viral respiratoria, y se evidencia un compromiso distal, simétrico, con progresión hacia proximal durante los próximos días, con signos de motoneurona inferior.

Las mononeuropatías ocurren principalmente secundarios a atrapamientos (síndrome del túnel del carpo) o a otras causas de tipo compresivas como lesiones tumorales, traumatismo, complicación quirúrgica.

La conducción nerviosa y electromiografía ayudan a localizar la lesión así como reconocer el defecto axonal o desmielinizante (diagnóstico y pronóstico).

Las radiculopatías se relacionan frecuentemente con patología degenerativa de la columna vertebral, como en la HNP. En casos particulares como las poliradiculopatías, se deben considerar diagnósticos alternativos como síndrome de Guillian-Barré y CIDP (polirradiculopatía desmielinizante inflamatoria crónica) en el que se comprometen múltiples raíces en forma progresiva de distal hacia proximal por desmielinización progresiva de los troncos nerviosos.

Tratamiento Dependerá de la causa. Además del tratamiento de la causa, las afecciones del SNP se manifiestan con frecuencia con dolor. Por ello es necesario además del 24

tratamiento causal un tratamiento sintomático para el dolor. En general en algunas PNP se pueden utilizar pregabalina, carbamazepina o amitriptilina. La pregabalina a dosis de 50 mg cada 12 horas hasta 150mg cada 12 horas (dosis máxima) es el fármaco más utilizado. En el caso de las mononeuropatías por atrapamiento se recurre a la liberación quirúrgica en los casos más severos (Como en el STC). En el caso de las polirradiculopatías como el síndrome de Guillian Barré se deberá atender en atención secundaria para evaluación y eventual plasmaféresis o dosis de inmunoglobulina IV. En el caso de las mononeuritis múltiples será el control de la patología de base, en la mayor parte de los casos las vasculitis.

Seguimiento Derivar a especialista

25



Síndrome atáxico Nivel de manejo del médico general: Diagnóstico: sospecha Tratamiento: inicial Seguimiento: no requiere

Diagnóstico 

Aspectos esenciales    

Alteración de la postura y coordinación de movimientos voluntarios. Dado por lesiones a nivel de vías propioceptivas, cerebelosas o vestibulares. Caracterizado por alteración de marcha, puede acompañarse de vértigo, nistagmo y disartria. Mediante la clínica se debe diferenciar el sistema involucrado para realizar manejo dirigido a la etiología.



Caso clínico tipo Niña de 4 años con antecedente de varicela hace 2 semanas, presenta marcha inestable, aumento de base de sustentación, imposibilidad de marcha en tándem. Sin signos meníngeos. Punción lumbar con líquido claro, normal.



Definición Trastorno de coordinación, dirección y amplitud del movimiento voluntario, de postura y equilibrio, en ausencia de paresia o de apraxia.

Tratamiento

Se produce por lesiones en las vías implicadas en la coordinación de los movimientos, postura y equilibrio. Los sistemas implicados son:



  

Ataxia sensitiva: trastornos de propiocepción consciente. Afecta simétricamente EEII, y la marcha. Sin vértigo, nistagmus, ni disartria. Hay un aumento de la base de sustentación. Empeora al cerrar ojos: Romberg+, el que no aparece en lesiones cerebelosas. Ocurre por compromiso de los cordones posteriores de la médula ósea (Ej: degeneración combinada subaguda por déficit de Vit B12) Ataxia cerebelosa: Producida por incordinación motora, afecta la bipedestación, marcha y control de extremidades. Persiste con ojos abiertos, no se agrava al cerrarlos. Disartria (habla con palabra escándida), hipotonía, temblor cinético y nistagmus. Signo de Romberg negativo. 1. Cerebeloso vermiano: Ataxia de la marcha; sin ataxia de miembros, nistagmus, hipotonía, disartria ni temblor. Por OH, meduloblastoma en niños. 2.Cerebeloso hemisférico: Ataxia de miembros ipsilaterales a lesión, con hipotonía, nistagmus y temblor. Ataxia vestibular: Trastorno del equilibrio en bipedestación y marcha, sin ataxia de miembros en decúbito. Asociado a vértigo y nistagmus, sin disartria. Puede ser central o periférica y coincide con los síndromes vertiginosos correspondientes. Romberg positivo.

Pruebas complementarias: neuroimágenes en sospecha de ECV, TU, etc. De preferencia RNM.

Fisiopatología



Ataxias heredodegenerativas: Patologías genéticas que causan atrofia de estructuras específicas (Síndrome de Ataxia-Telangectasia, Ataxia espinocerebelosa, ataxia episódica, entre otras)

El tratamiento es específico según la etiología. 

Propiocepción consciente (n. periférico-raíz posterior-cordones posteriores-lemnisco medialtálamo-corteza). Propiocepción inconsciente (haces espinocerebelosos posterior y anteriorpedúnculos cerebelosos-cerebelo). Cerebelo (vérmis-hemisferios cerebelos). Sistema vestibular (canales semicurcularesutrículo-sáculo). Núcleos vestibulares a nivel de Tronco Encefálico.

 



Ataxia sensitiva secundaria a déficit de Vit B12, aporte de Vit B12 en forma IM mensual x 3 meses con control posterior de niveles de Vit B12. Ataxia secundaria a hidrocefalia normotensiva, evaluar eventual válvula derivativa. En caso de lesiones de causa vascular cerebelosa, considerar estudio etiológico para eventual uso de TACO en caso de lesiones de origen embólico y manejo de factores de riesgo cardiovascular en caso de ser aterotrombótico. En caso de lesión tumoral con compresión a nivel medular o cerebelosa, resolución quirúrgica según especialista.

Etiología 



 

Seguimiento

Sensitiva: Neuropatía periférica, Sd. tabético (Sífilis), degeneración combinada subaguda medular (déficit de Vit B12), espondilosis cervical. Cerebelosa: TU, ECV, postinfeccioso (VVZ), tóxico (OH), fármacos (fenitoína). Hidrocefalia normotensiva. Vestibular central: Ictus vertebrobasilar, esclerosis múltiple, TU ángulo pontocerebeloso. Vestibular periférico: Posicional, neuronitis, laberintitis, Ménière.

Derivar

26

éste el nivel más bajo en el que existe función neurológica, motora y/o sensitiva, compromiso esfinteriano y/o compromiso de la función respiratoria (en lesiones por sobre C4, por ser éste el nivel al cual emergen los nervios frénicos). El examen motor y sensitivo permite establecer si la lesión medular es completa o incompleta. Se define como incompleta aquélla en la que existe algún grado de función residual motora o sensitiva más de tres segmentos por debajo del nivel de la lesión, y se define como completa aquella en la que no existe ninguna función neurológica conservada. Solamente un 3 % de los pacientes con lesión medular completa en la primera exploración tendrán alguna mejoría en las primeras 24 horas.

Síndrome cuadrapléjico agudo Nivel de manejo del médico general: Diagnóstico: sospecha. Tratamiento: inicial. Seguimiento: no requiere.

Aspectos esenciales 

  

La cuadriplejia aguda en la matoría de los casos puede asociarse a traumatismos raquimedulares cervicales. El nivel de lesión suele ser por sobre C5, sobre C4 hay compromiso respiratorio. Es clave determinar el nivel de lesión mediante el examen físico neurológico y prueba de imagen. La RNM es el examen de elección para evaluar lesiones cervicales con déficit neurológico.

En una primera etapa post trauma existe una etapa de shock espinal un periodo de parálisis fláccida y arrefléxica que habitualmente se resuelve en el transcurso de 7 a 10 días. Luego de eso, gradualmente se va estableciendo la parálisis espástica e hiperrefléxica.

Tratamiento 

Caso clínico tipo Se recibe a un motociclista de 17 años, que tras choque de alta energía con poste de la vía pública, se presenta consciente Glasgow 14, no presenta lesiones externas relevantes, ni abdomen agudo, mantiene la ventilación espontánea, pero refiere no poder movilizar extremidades y anestesia en todas las extremidades para todas las modalidades. Se realiza imagen que muestra fractura de columna cervical con lesión medular a nivel C3.



Definición Corresponde a la pérdida completa de la movilidad o parálisis, tanto de extremidades superiores como inferiores , por lesiones agudas (frecuentemente traumáticas) de la médula espinal por sobre el nivel C5. Generalmente se asocia a compromiso sensitivo y autonómico concomitante.



Etiología El síndrome cuadrapléjico agudo, suele asociarse en la gran mayor parte de los casos a traumatismos raquimedulares con lesión por sobre el nivel C5. Otras causas menos frecuentes, que ante la falta del antecedente traumático podrían descartarse: lesión desmielinizante aguda de médula (autoinmune o post infecciosa), mielopatía cervicoartrósica agudizada por traumatismo, malformación vascular en segmento cervical, accidente cerebrovascular isquémico o hemorrágico con afectación del tronco encefálico.

Manejo Pre-Hospitalario: El manejo de una tetraplejia de causa traumática debe iniciarse en el propio lugar del accidente e incluye o Inmovilización precoz con collar cervical rígido, control hemodinámico y respiratorio. o Uso de tabla espinal larga, inmovilizadores laterales, para ser trasladado al centro de atención. Manejo Hospitalario: Precisar nivel de lesión, control respiratorio y hemodinámico si se requiere, dependiendo del centro se administra metilprednisolona en altas dosis en las primeras ocho horas tras haberse producido un traumatismo medular, manteniéndola durante 24 horas si ésta se administra en las primeras tres horas (NASCIS II-NASCIS III). Se puede realizar Radiografía en primera instancia , más indicado el TAC o RNM. Manejo de Rehabilitación: Manejo por Kinesiología y Terapia Ocupacional para evitar rigidez posterior en extremidades, evitar aparición de UPP y potenciar la funcionalidad restante. Evitar complicaciones secundarias a la postración (Escaras, Neumonía Aspirativa, etc).

Seguimiento Manejo de complicaciones por Especialista.

De manera mucho más infrecuente SCIWORA (lesión medular sin anomalías radiográficas), más frecuente en niños pero posible en adultos, o polirradiculoneuritis afectación bulbo medular hasta C6, de tipo desmielinizante, generalmente postinfeccioso.

Diagnóstico Se basa en el examen neurológico e imágenes. Es prioritario establecer el nivel de la lesión medular, siendo 27

pacientes jóvenes entre los 20 a 40 años. La incidencia es de 1 a 2 casos cada 100.000 por año, aumenta la probabilidad de tener esta enfermedad en un 20% cada 10 años luego de la primera década de la vida.

Síndrome cuadripléjico fláccido: polirradiculoneuritis aguda o guillain-barré (sgb)

Diagnóstico En la clínica se observa tetraparesia fláccida y arrefléctica, ascendente (comienza en EEII), simétrica, de rápida evolución (horas a días), con escasos síntomas sensitivos (disestesias). Sin afectar esfínteres. El 50% de los pacientes cursan con paresia facial. Progresa hasta afectar todo el cuerpo y en casos graves la musculatura respiratoria. Al inicio puede haber dolor lumbar y de EEII. Disautonomías frecuentes son taquicardia, hipotensión postural, hipertensión, síntomas vasomotores. También se puede encontrar ileo paralítico, disfunción vesical, arritmias, anormalidades en la sudoración.

Nivel de manejo del médico general: Diagnóstico: sospecha Tratamiento: inicial Seguimiento: no requiere

Aspectos esenciales     

Tetraparesia fláccida aguda progresiva de distal a proximal Sin síntomas sensitivos puros Antecedente de infección respiratoria o GI (Campylobacter Jejuni o Virus Familia Herpes) Soporte cardiorespiratorio Tratamiento con plasmaféresis o inmunoglobulinas.

La clínica progresa alcanzando su máximo de desmielinización a las 4 semanas. Se comienza a recuperar las funciones neurológicas 2 a 4 semanas posterior al cese de la progresión del cuadro clínico. Subtipos no clásicos y menos frecuentes: neuropatía axonal motora (AMAN), neuropatía axonal sensitivomotora aguda (AMSAN), síndromes limitados regionales, como el sd Miller-Fisher (ataxia - arreflexia oftalmoparesia, como tip "Guillian Barré Invertido"), Poliradiculopatía desmielinizante inflamatoria aguda (AIDP).

Caso clínico tipo Mujer de 27 años, presenta fiebre y adenopatías. En el transcurso del cuadro comienza con debilidad de MMII de forma simétrica que progresa a MMSS. Es llevada a urgencias donde se constata paresia facial simétrica, dificultad respiratoria y EBV en sangre. Al examen neurológico presenta arreflexia en todas las extremidades y paresia sin carácter de espasticidad, y ausencia de reflejo plantar indiferente.

Dentro de las pruebas complementarios, al realizar una punción lumbar el LCR muestra disociación albuminocitológica (aumento de proteínas sin pleocitosis, al 7° a 10° día). Si hay pleocitosis probablemente es SGB+VIH. Estudios neurofisiológicos con alteraciones de la velocidad de conducción nerviosa característica (abolición de onda F, ralentización de velocidad de conducción, aumento de latencias distales secundarios al efecto desmielinizante de la enfermedad) Algunos de los diagnósticos diferencial son la parálisis diskalémica, mielitis aguda, botulismo, poliomielitis, porfiria, difteria, neuropatías tóxicas (talio, dapsona, nitrofurantoína), neuroborreliosis o enfermedad de Lyme, intoxicación por plomo, etc.

Definición Polirradiculoneuropatía desmielinizante aguda, de origen autoinmune, frecuentemente grave y de evolución rápida (fulminante) en días, manifestada como cuadro de parálisis motora arrefléxica de evolución rápida que comienza de la musculatura distal y progresa hacia proximal, con escasas alteraciones sensitivas.

Etiología

Tratamiento

Patología autoinmune, más frecuente en hombres jóvenes. El 60% de los casos tiene antecedente de infección viral respiratoria o gastrointestinal. Los virus más frecuentemente implicados son herpes (CMV, EBV). También se presenta asociada a Campylobacter jejuni, en pacientes con antecedente de gastroenteritis (similar a EII)

Hospitalizado, en unidad compleja (intermedio o UCI), para soporte cardiorrespiratorio con prevención de infecciones intercurrentes, mantener una buena hidratación es esencial. En caso de ser menos grave, o tener buena evolución se puede manejar en sala común. Esteroides no han demostrado efectividad y existe evidencia sugerente de que empeora pronóstico.

Fisiopatología

El tratamiento de elección es plasmaféresis o inmunoglobulinas intravenosas, que acortan la enfermedad, ambas con la misma efectividad, se recomiendan su utilización de forma precoz. Es importante mantener rehabilitación precoz con manejo Kinesiológico para evitar secuela neurológicas a largo plazo. Considerar siempre en estos pacientes el manejo de patología tromboembólica, secundario a la estasia muscular por la paresia arrefléctica.

La desmielinización es por mecanismo linfocitario y anticuerpos. Se ha descrito la presencia de AC anti-GM1. Se traduce en inflamación, desmielinización y degeneración axonal del SNP, sin compromiso del SNC, de forma segmentaria y multifocal, principalmente a nivel proximal de los axones en raíces nerviosas, alcanzando su máximo a las 4 semanas.

Seguimiento

Epidemiología

Especialista si hay secuela neurológica.

No se evidencian factores de riesgo, ni un rango etáreo en específico, pero se suele presentar mayormente en 28

Síndrome encefalítico

Diagnóstico

Nivel de manejo del médico general: Diagnóstico: sospecha Tratamiento: inicial Seguimiento: no requiere

Además de la enfermedad febril aguda con signos de afección meníngea, característica de la meningitis, el paciente con encefalitis con frecuencia presenta confusión, trastornos conductuales o alteración del nivel de conciencia (cuali-cuantitativo incluso llegando al coma) y signos y síntomas neurológicos focales o difusos (más frecuentes afasia, ataxia, hemiparesia espástica, alteraciones de pares craneales). En casos de encefalitis grave, pueden presentarse incluso convulsiones tónicoclónicas.

Aspectos esenciales   

  

Dentro de los exámenes complementarios, se encuentran  Estudio de LCR: Es indistinguible de la meningitis vírica. Consiste en pleocitosis linfocítica (>5 células/µl), ligera elevación de proteínas y cifra normal de glucosa. En HSV el líquido es hemorrágico con xantocromía. No realizar si hay sospecha de hipertensión endocraneana. Cultivo de LCR es útil en virus Coxsaquie, ECHO, virus parotiditis, pero negativos en casos de HSV-1.  PCR en LCR: Diagnóstico definitivo de HSV. Su sensibilidad y especificidad son iguales o superiores a la de la biopsia de encéfalo.  Pruebas serológicas y detección de antígenos en LCR.  Imagenología: RNM (de elección) para detectar cambios de señal del parénquima, TAC puede mostrar zonas de hipodensidad. HSV: Necrosis hemorrágica de lóbulos temporales.  EEG: signos de focalidad frontotemporal sugiren encefalitis por HVS-1.

La presentación es aguda, de horas de evolución. Afectación del parenquima cerebral. La etiología más frecuente es viral, y entre éstas, la más habitual es el Enterovirus (Encefalitis epidémica) y HSV 1 (Encefalitis esporádica). La encefalitis muchas veces está asociada a una meningitis (meningoencefalitis). Fiebre, Cefalea, Compromiso de conciencia y focalidad neurológica. Tratamiento Aciclovir EV sin demora , de forma empírica.

Caso clínico tipo Paciente masculino de 18 años de edad de sexo femenino, sin antecedentes mórbidos, madre refiere que comienza con, cuadro hace 24 horas caracterizado por fiebre cuantificada en 38.5ºC de aparición brusca, cefalea holocranea y confusión. Señala que se comporta distinto a lo normal. A lo anterior se le agrega hemiparesia izquierda desde hace 1 hora y posterior compromiso de conciencia cuantitativo (la nota más dormida).

Tratamiento Se deben instaurar medidas generales, hospitalizando en sala de paciente complejo o UCI, con monitorización y manejo de vía aérea, ventilación, presión arterial. Se debe vigilar presión intracraneana. Restricción de líquidos y manejo de la fiebre. El tratamiento antiviral aciclovir (10 mg intravenoso/kg de peso cada 8 hrs., por un mínimo de 14 días hasta 21 días) de forma empírica sin demora, pues es efectivo contra herpes virus, reduciendo la mortalidad y secuelas. Suspender al descartarse herpes virus. Se realiza control al finalizar tratamiento para evaluar respuesta a medicamento para eventual extensión de dosis. En los casos en los que se evidencia CMV, tratamiento es con Ganciclovir.

Definición Es la inflamación del parénquima encefálico, secundario a infección generalmente viral, de inicio agudo. Puede asociarse a cuadros con inflamación meníngea (meningoencefalitis). Está caracterizada por deterioro del nivel de consciencia y focalidad neurológica.

Etiología La causa más frecuente es viral. Cientos de virus son capaces de causar encefalitis, pero los que originan casi todos los casos en que se puede determinar la etiología específica son por los virus de la familia Herpes (principalmente HSV-1, también VVZ y EBV, CMV), enterovirus y arbovirus. Se presenta con mayor frecuencia en jóvenes y adultos. Las epidemias de encefalitis son causadas por arbovirus y enterovirus.

Seguimiento Seguimiento de complicaciones y secuelas neurológicas por Especialista

Los virus alcanzan el SNC mediante 2 vías: neuronal o hematógena. Es característica en la encefalitis herpética la afección de la región frontotemporal, cíngulo e ínsula. Se produce un fenómeno inflamatorio con compromiso localizado del parénquima cerebral, con necrosis localizada y hemorragia local, lo que ocasiona las focalidades neurológicas posteriores.

29

Síndromes convulsivos somáticos

La causa más frecuente es la epilepsia, que puede generar crisis parciales (simples, complejas, parciales secundariamente generalizadas) o crisis generalizadas.

Nivel de manejo del médico general: Diagnóstico: sospecha. Tratamiento: inicial. Seguimiento: no requiere

Fisiopatología Corresponde a una descarga sincrónica, simultánea, de forma exagerada de un grupo neuronal, causadas por un desequilibrio entre mecanismos inhibitorios y excitatorios, gatillado por una estimulación externa. Se inicia una descarga generada por la entrada masiva de Ca2+ y Na+ al interior de la neurona, con una despolarización prolongada de membrana, la cual no es posible de ser inhibida por la hiperpolarización de membrana de receptores GABA y canales de K+ (descargas repetidas generan aumento de K+ extracelular y Ca2+ extracelular mediante activación de NMDA). Este proceso puede ser focal (crisis focal, anterior parcial) o generalizarse a otros grupos neuronales estimulando ambos hemisferios (crisis de rápida generalización o anterior generalizada).

Aspectos esenciales     

No toda las crisis convulsivas son epilepsia, y no toda epilepsia se presenta con convulsiones. Hay causas intracraneales y extracraneales/sistémicas. Diagnóstico diferencial con un síncope, o un trastorno disociativo. El TAC de cráneo no es de regla. El manejo es distinto si es primera crisis o si es status epiléptico.

Diagnóstico

Caso clínico tipo

Es clínico. Las crisis tienen que subdivirse según presentación del evento clínico en focales (crisis donde se evidencia un foco claro de descarga neuronal) o de rápida generalización o generalizada (compromiso de ambos hemisferios) y según compromiso de conciencia asociado o no (simple en caso de ausencia de compromiso, o compleja si tiene compromiso de conciencia). Es ideal contar con un observador del evento.

Mujer de 55 años, diabética, hipertensa, es traída por su hermana porque presentó en la tarde un episodio convulsivo generalizado de aproximadamente 3 minutos. Su hermana refiere que hace una semana, se cayó en las escaleras debido a que tropezó, y se pegó en la cabeza.

Definición

Se debe identificar gatillante posible del episodio, si se asoció aura o no anterior al episodio convulsivo, el inicio asociado a una focalidad neurológica o generalizada desde el comienzo, episodios anteriores y características similares al actual, evidencia de consumo de sustancias anterior al episodio, Antecedente de trauma o infecciones concomitantes, estado post-crítico, si recuerda el episodio o no, duración del episodio (si duró minutos, >5 minutos es diagnóstico operacional de status epiléptico), consumo de medicamentos antiepilépticos (antecedente de consumo en los últimos días) Posteriormente categorizar el episodio convulsivo según focal (evidencia de clínicas sugerente de un lóbulo en particular) o generalizada (clónica, tónica, tónico-clónica, mioclónica, atónica, ausencia)

Corresponde a una crisis convulsiva de origen secundario a una causa identificable, en pacientes que pueden o no tener antecedente de Epilepsia. El enfrentamiento clínico en este caso es considerar al paciente como primer episodio convulsivo.

Epidemiología Un 10% de la población presentará al menos una convulsión durante su vida, las crisis convulsivas tienen una incidencia de 50 por cada 100.000 habitantes al año. No presentan una edad en particular de presentación, ya que varía según la etiología.

En el examen físico se debe evaluar el nivel de conciencia, buscar focalidad neurológica al examen físico, fondo de ojo con signos de hipertensión endocraneal, evidencia de relajación de esfínter, evidencia de mordedura de lengua (generalmente es lateral, en episodios facticios suele ser anterior), signos meníngeos, signos de otras afecciones generales que puedan haber causado la crisis.

Etiología Dependiendo de la Etiología, pueden identificarse grupos de riesgo: 





Neonatos, Lactantes y Niños (12 años): Hipoxia Perinatal, Infecciones de SNC (meninigitis, Encefalitis), Trastornos metabólicos (Hipoglicemia), Crisis Febriles, Traumatismo, Idiopático Adolescentes, Adulto Jóvenes (18-35 años): Traumatismo, Drogas y Consumo de Tóxicos, Abstinencia de Alcohol, Tumores Adultos (>35 años): ACV, Tumores, Abstinencia de Alcohol, Trastornos Metabólicos (hipoglicemia, Uremia, Hipocalcemia, Hiponatremia, Intoxicación por CO), Enf Degenerativas del SNC, Idiopática.

Dentro del laboratorio se pide hemograma, PCR, bioquímica sanguínea, glicemia capilar, creatinina, transaminasas, gases arteriales. Se pide TAC de cráneo si: hay signos focales, se trate de estatus epiléptico, TEC reciente, hay signos de hipertensión intracraneal o si se sospecha de infección del SNC y previamente a la punción lumbar. Se puede pedir un EEG posteriormente, en forma complementaria (diagnóstico es clínicoAnamnesis) y un diagnóstico diferencial es síncope.

30

Tratamiento Durante la crisis se debe mantener la calma, poner al paciente en posición lateral de seguridad. Evitar colocar objetos en la boca, ni inmovilizar, es importante contar la duración del episodio. Administración de oxígeno durante el episodio e inicio de tratamiento en caso de prolongación del episodio como manejo de status epiléptico. El tratamiento definitivo es el tratamiento de la causa de base, por lo que es indispensable descartar los diagnósticos diferenciales para tratar la causa. En caso de crisis prolongadas, repetidas, anormalidades focales o con EEG anormal: se inicia un tratamiento farmacológico (Fenitoína, Fenobarbital, Carbamazepina o Ácido Valproico) de acuerdo a la clínica del episodio focal o generalizado. En caso de status epiléptico: se administra oxígeno por mascarilla, idealmente con bolsa de reservorio, permeabilización y mantenimiento de las vías aéreas. Si no se sabe la causa, se inicia infusión de glucosa con tiamina, y tratamiento farmacológico con Lorazepam iv, Diazepam iv, Diazepám rectal, y de segunda línea Fenobarbital.

Seguimiento Seguimiento por especialista.

31

Síndromes vertiginosos centrales Etiología-Epidemiología 

Nivel de manejo del médico general: Diagnóstico: sospecha. Tratamiento: inicial. Seguimiento: no requiere.





Aspectos esenciales 

 



Síntomas principales: desequilibrio, cefalea, síntomas focales, nistagmo espontáneo y de posición. Dg: Prueba de VIII par e imágenes. Definir etiología: Síndromes de línea media de tronco cerebral, Sd. del ángulo pontocerebeloso, Sd. cerebeloso. Frente a un síndrome vertiginoso, lo primero es diferenciar si se trata de una alteración central o periférica (la siguiente tabla reúne las principales diferencias)

 

Síndromes de línea media del tronco encefálico (patología cerebrovascular, esclerosis múltiple, tumores compresivos por vecindad) Síndrome del Ángulo Pontocerebeloso: principalmente tumoral: neurinoma del VIII (80%), meningioma, colesteatoma, etc. Síndrome Cerebeloso: isquemia u hemorragia, Tumores, Afecciones congénitas o inflamatorias (cerebelitis infecciosa o esclerosis múltiple), degenerativas (alcohólica, paraneoplásica, hereditaria), Hidrocefalia. Corresponden al 20% del total de vértigos

Fisiopatología Los estímulos ambientales de movimientos angulares y lineales son sensados inicialmente por los órganos sensoriales a nivel del Oído interno (Canales semicirculares, Utrículo y Sáculo) en forma bilateral. Cada uno mantiene una estimulación tónica (durante la ausencia de movimiento) o estimulación-inhibición (durante los movimientos en forma secuencial) desde ambos órganos sensoriales los cuales posteriormente son transmitidos por el VIII Par Vestibular hasta su ingreso al Tronco encefálico en la Zona del Puente inferior por Anterior, hacia los Núcleos Vestibulares. A nivel Central se procesa dicha información asociada con la información obtenida en forma visual y posteriormente procesada a nivel de Cerebelo.

DIFERENCIAS PRINCIPALES ENTRE LOS VÉRTIGOS PERIFÉRICOS Y CENTRALES Cuadro Vértigo central Vértigo periférico clínico Inicio Raramente agudo (con Agudo la excepción de los ictus y de los brotes de esclerosis múltiple) Síntomas Inestabilidad/mareo Ilusión de rotación Síntomas Poco intensos Muy intensos vegetativo Hipoacusia Rara Frecuente Acúfenos Casi nunca Posibles Signos Déficit neurológicos Hipoacusia asociados (ares craneales, vías largas) Tipo de Variable (rotatorio Casi siempre nistagmo puro, vertical, horizontal en refractario dirección fija dependiente de la (horizonte-rotatorio mirada) en el VPPB) Desviación En el sentido de la fase En el sentido de la fase de índices y rápida del nistagmo lenta del nistagmo tronco (prueba de Barany) Evolución A menudo crónico Paroxístico/recurrente VPPB: vértigo paroxístico posicional benigno. Compendio de Medicina Interna. C. Rozman. V edición

Cualquier lesión considerada a nivel de Tronco Encefálico (ingreso del VIII Par, lesiones a nivel de núcleos vestibulares, lesiones de vías vestíbulo-oculares o lesiones de vías vestíbulo-espinales) o a nivel de cerebelo, producirá una alteración en el procesamiento de la Información que se transmitirá como una deficiencia en el estímulo tónico o incapacidad de estimularse durante el movimiento, traduciéndose así en vértigo.

Diagnóstico Habiendo diferenciado el origen del vértigo, (con las diferencias previamente mencionadas) se debe buscar otros síntomas que orienten al diagnóstico etiológico, como la presencia cefalea, alteración de conciencia, signos de focalidad neurológica (disartria, diplopia, disfagia)

Caso clínico tipo Tratamiento Mujer de 30 años aquejado de mareos, ataxia, nistagmo de rebote, dismetría ocular y aleteo ocular. Además presenta dismetría y disdiadococinesia de miembro superior izquierdo. No refiere otras focalidades neurológicas al examen físico ni a la anamnesis.

Dependiendo de la Etiología se definirá tratamiento por especialista:  Manejo de Factores de riesgo Cardiovascular en aquellos de origen Vascular y dependiendo de la etiología Manejo con AAS y Atorvastatina y eventual anticoagulación según etiología de ACV.  Extirpación de Tumores del Ángulo pontocerebeloso (Schwanoma del VIII Par)  Manejo de soporte en caso de Hidrocefalia en casos de infartos cerebelosos

Definición Lesión que ocurre a nivel de sistema vestibular en tronco encefálico o cerebelo, cuyos síntomas principales son mareos, vértigo (la ilusión de movimiento, generalmente rotacional) y desequilibrio. 32

 

Manejo con Corticoides y terapia secundaria en los casos de Esclerosis Múltiple. Manejo Sintomático puede realizarse con Metoclopramida, Ondasentrón, Antihistamínicos, clorhidrato de Difenidol 1-4 compr./ día cada 12 horas hasta 2 semanas, Dimenhidrato 100mg vo 2-4 veces/día por 2 semanas (inhibición parcial de estímulos a nivel central).

Seguimiento Derivar.

33

Nivel de manejo del médico general: Diagnóstico: específico. Tratamiento: completo. Seguimiento: completo.

vestibular hacia el tronco encefálico para su procesamiento central asociado al sistema vestibulo ocular y vestíbulo espinal proveniente del cerebelo. Lesiones que se producen a este nivel (órganos sensoriales y nervio VIII) producirán una alteración en la transmisión de la estimulación tónica sostenida (individuo en reposo) o estimulación/inhibición (individuo en movimiento) de los estímulos externos, lo que se traducirá en vértigo a nivel central.

Aspectos esenciales

Diagnóstico

Síndromes vertiginosos periféricos



   

Síntomas principales: Mareos, vértigo y desequilibrio, generalmente unilateral, nistagmo unidireccional, horizontal. Definir compromiso central y periférico. Vértigo postural paroxístico benigno: Crisis de vértigo posturales de corta duración y recurrentes. Tratamiento de vértigo: sintomático, de mantención y ejercicios vestibulares. Tratamiento de la causa.

Frente a un síndrome vertiginoso, Reconocer si es central o periférico.

DIFERENCIAS PRINCIPALES ENTRE LOS VÉRTIGOS PERIFÉRICOS Y CENTRALES Cuadro Vértigo central Vértigo periférico clínico Inicio Raramente agudo (con Agudo la excepción de los ictus y de los brotes de esclerosis múltiple) Síntomas Inestabilidad/mareo Ilusión de rotación Síntomas Poco intensos Muy intensos vegetativo Hipoacusia Rara Frecuente Acúfenos Casi nunca Posibles Signos Déficit neurológicos Hipoacusia asociados (ares craneales, vías largas) Tipo de Variable (rotatorio Casi siempre nistagmo puro, vertical, horizontal en refractario dirección fija dependiente de la (horizonte-rotatorio mirada) en el VPPB) Desviación En el sentido de la fase En el sentido de la fase de índices y rápida del nistagmo lenta del nistagmo tronco (prueba de Barany) Evolución A menudo crónico Paroxístico/recurrente VPPB: vértigo paroxístico posicional benigno. Compendio de Medicina Interna. C. Rozman. V edición

Caso clínico tipo Paciente de 44 años con episodios de vértigo breve al levantarse por la mañana y al girar la cabeza. Presenta examen neurológico normal y nistagmus rotacional geotrópico ante maniobras posturales.

Definición Lesiones localizadas a nivel de órganos sensoriales (Utrículo, Sáculo o Canales Semi-circulares) o Nervio Vestibular VIII previo a su ingreso a Tronco Encefálico que se traduce clínicamente en Vértigo (sensación de movimiento rotatorio ilusorio del ambiente).

Etiología 

 

  



es necesario:

Vértigo postural paroxístico benigno: Idiopático, frecuente en tercera edad, post-TEC, alcohol, patología ótica crónica. Neuronitis vestibular: Respuesta inmune cruzada post viral. Hidrops endolinfático: Acumulación de endolinfa y daño de laberinto por aumento de presión al interior del oído interno, de origen idiopático o Síndrome de Menieré, secuela de TEC, otoesclerosis, etc. Neurinoma del Nervio Acústico: Tumor de células de Schwann. Parálisis vestibular súbita: Causa vascular o inflamatoria. Ototoxicidad: aminoglicósidos, paracetamol, digoxina, propanolol, alcohol, anticonvulsivantes, arsénico, etc. Otras causas: infecciones, trastornos degenerativos, metabólicos y efectos de cirugía.



 



Nistagmus posicional: Ocurre al cambiar bruscamente de posición al paciente. Ej: En decúbito supino, giro lateral de cabeza a derecha y luego a izquierda. Nistagmus espontáneo: Se obtiene al mirar en 30º en todas las direcciones. Ley de Alexander: Nistagmus horizontal que aumenta su intensidad al mirar en el sentido de la fase rápida. Realizar pruebas del VIII par: Fx. Coclear y fx. Vestibular (pruebas calóricas).

Buscar etiología: 

Fisiopatología Los estímulos externos de movimientos lineales y angulares son recibidos por los órganos sensoriales del oído interno (utrículo, sáculo y canales semicirculares), los cuales pasan posteriormente por medio del VIII par 34

Vértigo postural paroxístico benigno: crisis de vértigo posturales de corta duración (segundos a minutos) y recurrentes, que se presentan por días a semanas, reaparecen frente a estrés y mejoran durante el día. Examen neurológico: normal o presencia de nistagmus posicional periférico (generalmente canal semicircular posterior). No asocian ni hipoacusia, ni tinnitus. Maniobra Dix-Halpike (nistagmo con periodo de latencia). No se











realizan imágenes, solo si no se está seguro el diagnóstico. Neuronitis vestibular: Inicia con crisis de vértigo espontáneo, náuseas y vómitos de 24 hrs, con lateropulsiones ipsilaterales y nistagmus horizontal periférico hacia lado sano. Luego, crisis posturales por 3-7 días, sin desequilibrio espontáneo y nistagmus puede ceder. Cuadro recurrente en los meses siguientes. No presenta síntomas auditivos. Examen neurológico: audición normal, paresia vestibular del lado de la lesión (prueba calórica) y ocasional nistagmus posicional periférico. Hidrops endolinfático: Inicia con tinnitus unilateral e hipoacusia fluctuante. Luego, crisis de vértigo espontáneo con náuseas y vómitos que cede en horas a minutos. Generalmente síntomas suelen empeorar con maniobra de valsalva o con la tos. Examen neurológico: hipoexcitabilidad vestibular (prueba calórica) e hipoacusia sensorioneural unilaterales. Neurinoma del acústico: Tinitus e hipoacusia progresivos de inicio insidioso (no fluctuantes) y pocos síntomas vestibulares. Tumor puede crecer, produciendo sd del ángulo pontocerebeloso. Examen neurológico: gran paresia vestibular para pocos síntomas (prueba calórica) e hipoacusia sensorioneural unilateral. Parálisis vestibular súbita: Inicia con crisis de vértigo espontáneo, náuseas y vómitos, lateropulsiones ipsilaterales y nistagmus horizontal periférico hacia lado sano, que cede en días. Luego crisis posturales por 4-5 semanas. No es recurrencial y puede tener síntomas auditivos. Examen neurológico: parálisis vestibular o cócleovestibular (prueba calórica) y ocasional nistagmus posicional periférico Ototoxicidad: Tinitus, hipoacusia, mareo, vértigo y desequilibrio y poco nistagmus. Antecedente de fármaco ototóxico. Examen neurológico: paresia vestibular bilateral con o sin daño coclear sensorioneural.

Seguimiento Derivar enfermedad de Menieré y neurinoma del acústico. En VPPB, neuronitis vestibular, parálisis vestibular súbita u ototoxicidad derivar en caso de curso atípico o sospecha de lesión central.

Tratamiento Tratamiento del vértigo:  





Sintomático: hidratación, antieméticos (ej: metoclopramida), ansiolíticos (BDZ). Mantención: antivertiginosos como Cinarizina 40120mg/día cada 8-12horas (Se puede utilizar tambiém difenidol, dimenhidrato, etc). Ejercicios vestibulares: girar varias veces en sucesión varias veces al día (provocando vértigo) en cama. Tratamiento específico: o VPPB: maniobras de reposición vestibular (Epley para canal posterior o Semont). o Neuronitis vestibular: Tratamiento sintomático del vértigo. o Hidrops endolinfático, enfermedad de Menieré: Manejo por especialista. o Neurinoma del acústico: cirugía u observación. o Parálisis vestibular súbita: antiagregantes plaquetarios (aspirina). o Ototoxicidad: Suspender fármaco ototóxico.

35

Diagnóstico

Temblor esencial

El diagnóstico es clínico, se observa un temblor de alta frecuencia en los miembros superiores que se intensifica cuando se intenta mantener una postura (temblor postural) o realizar una acción (temblor cinético). Por lo general es bilateral y simétrico. Puede también afectar la cabeza, y la voz provocando un habla trémula. Temblor de la cabeza puede ser vertical ("sí-sí") u horizontal ("nono") y por lo general se asocia con la mano o temblor de voz. Temblor en las piernas es inusual en TE. Puede también afectar la voz y menos común cara, piernas y tronco.

Nivel de manejo del médico general: Diagnóstico: sospecha. Tratamiento: inicial. Seguimiento: no requiere

Aspectos esenciales    

Temblor de alta frecuencia oscilatoria (hasta 11 Hz). Hasta un 50% de los paciente tiene el antecedente familiar. Se intensifica al intentar mantener una postura o al realizar una acción. Tratamiento Primidona y Propranolol.

Por definición, el temblor debe ser la única manifestación neurológica del TE. Sin embargo, en algunos casos graves, pueden estar presentes trastorno de la marcha y del cerebelo. Algunos pacientes con TE desarrollan mayor temblor fisiológico debido a la ansiedad u otros mecanismos adrenérgicos, agravando así el temblor subyacente. El TE se suele aliviar con pequeñas cantidades de alcohol, pero, en contraste con el temblor fisiológico, no suele agravarse por la cafeína.

Caso clínico tipo

Se diferencia de la enfermedad de Parkinson por la ausencia de bradicinesia, rigidez, temblor de reposo, micrografía, entre otras y por su evolución prolongada sin empeoramiento clínico importante.

Paciente de 75 años que consulta por temblor cefálico horizontal y de ambas extremidades superiores, de 30 años de evolución que se ha intensificado el último año, según lo resaltado por sus familiares. Temblor es de acción y actitud y desaparece en reposo.

Tratamiento Definición

En casos leves no se requiere tratamiento, pero si interfiere con la vida diaria del paciente, el tratamiento farmacológico estándar es con Primidona y Propranolol. El propranolol y primidona tienen eficacia similar cuando se utilizan como terapia inicial para tratar temblor de las extremidades en en TE, cualquiera de los fármacos puede ser utilizado en función de los problemas médicos concomitantes y los efectos secundarios potenciales. El propranolol puede utilizarse en dosis de 60 a 320 mg/día para reducir el TE, la supresión máxima del temblor es más probable que ocurra con una dosis entre 160 y 320 mg / día. Las contraindicaciones relativas para propranolol incluyen bloqueo cardíaco y enfermedad broncoespástica.

Temblor es un movimiento rítmico y oscilante, como resultado de contracción alternante o sincrónica de grupos musculares opuestos. El temblor es el más común de todos los trastornos del movimiento, y el temblor esencial (TE) es la causa neurológica más común de temblor postural o acción. El TE es un temblor de alta frecuencia (hasta 11 Hz) que afecta sobre todo a los miembros superiores. Puede ser temblor postural o de reposo.

Etiología-Fisiopatología Se desconoce, aunque 50% de los casos tienen antecedente familiar, con patrón autosómico dominante, es importante el factor genético, pero el ambiente también juega un rol. Los familiares de primer grado de pacientes con TE tienen un mayor riesgo de desarrollar el trastorno, sobre todo cuando la persona afectada desarrolla TE a una edad temprana.

El tratamiento con primidona debe comenzar a una dosis de 12,5 y 25mg antes de dormir y ajustarse cuidadosamente durante varias semanas según la tolerancia y de acuerdo con la respuesta terapéutica, hasta 750 mg/día. El uso de primidona puede ser limitado por efectos secundarios, a menudo agudos, tales como sedación, somnolencia, fatiga, náuseas, vómitos, ataxia, malestar general, mareos, inestabilidad, vértigo, y confusión.

Se sugiere que la neuropatología estaria localizada en locus ceruleus y en cerebelo (este último más discutido).

Cuando no hay respuesta al tratamiento médico y perjudica la calidad de vida del paciente, un especialista puede plantear cirugía (talamotomía unilateral o talámico estimulación cerebral profunda del núcleo ventral intermedio del tálamo) es un tratamiento eficaz para el temblor de la extremidad contralateral.

Epidemiología Es el trastorno del movimiento más frecuente, con una prevalencia estimada en todo el mundo de hasta el 5% de la población. La incidencia de TE aumenta con la edad, aunque a menudo afecta a las personas jóvenes, especialmente cuando es familiar. Antecedente familiar se presenta en el 30 a 70% de los pacientes.

Seguimiento Por especialista.

36

sueño es más común en hombres. En general aumentan con la edad.

Trastornos del sueño

Fisiopatología

Nivel de manejo del médico general: Diagnóstico sospecha Tratamiento inicial Seguimiento derivar

Los organismos poseen un ritmo circadiano, mantenido por la secreción cíclica de melatonina influenciada por la luz y la oscilación circadiana, que permite mantener al SNC en funcionamiento para mantener el orden de los diferentes sistemas. Este control homeostático permite la generación del sueño y de la vigilia con duración adecuada según los requerimientos del organismo. Alteraciones ya sea por sustancias externas, alteraciones orgánicas internas o psiquiátricas modifican la secreción de melatonina o evitan una correcta homeostasis del ciclo circadiano lo que produce alteración en la calidad, cantidad, inicio y consolidación del sueño.

Aspectos esenciales Dificultad para conciliar o mantener el sueño.     

Somnolencia diurna. Evaluar consumo de fármacos y drogas. Tratar comorbilidades. Promover higiene del sueño. Derivar al especialista.

Diagnóstico Motivos de consulta: Incapacidad de dormir adecuadamente; somnolencia y fatiga crónica. Se puede evidenciar cambios en la conducta (apatía, trastornos del ánimo) e impedimento cognitivo con afectación del rendimiento laboral.

Caso clínico tipo Paciente obeso de 45 años que refiere quedarse dormido en varios lugares durante el día. Además refiere fatiga y que a pesar de dormir lo necesario despierta cansado.

Los síntomas principales son el insomnio (de conciliación, mantenimiento o terminal), movimientos anormales, alteraciones conductuales durante el sueño o despertares nocturnos, somnolencia diurna excesiva. SAHOS: en pacientes generalmente con sobrepeso, no siempre asociado a roncopatía, cefalea.

Definición Procesos que alteran la iniciación, consolidación, calidad y cantidad del sueño, deteriorando la calidad de vida. Se estima la cantidad de horas de sueño según la edad de los pacientes siendo en adulto lo normal entre 4,5 a 8,5 horas, en adolescentes 9 a 9,5 horas y en escolares y preescolares 12 a 14 horas. 



  



Se debe identificar en que momento se presenta la alteración del sueño: inicio, consolidación, duración, calidad. Es importante preguntar por ingesta de medicamentos, hábitos de higiene de sueño, alteraciones psiquiátricas, alcohol, síndrome apnea hipopnea obstructiva del sueño (SAHOS), Patrones anormales de sueño, evidencia de alteraciones orgánicas (movimientos anormales del dormir, síntomas respiratorios, ronquidos), preguntar por estilo de vida (turnos), viajes recientes.

Disomnias: Trastornos primarios como dificultad de inicio y mantenimiento (Insomnios), como somnolencia diurna excesiva (hipersomnias). Parasomnias: eventos que perturban el sueño nocturno, y se dividen de acuerdo al momento de aparición. (Sonambulismo, terrores nocturnos, pesadillas). Trastornos del ciclo sueño-vigilia: Jet-lag. Trastornos del sueño asociado a Movimientos. Trastornos del sueño asociados a enfermedades médicas o psiquiátricas: Como por ejemplo SAOS, Narcolepsia, hipersomnia idiopática, Sd. de Kleine-Levine. Trastorno del sueño asociado a medicamentos







Etiología



Variada, puede asociarse a comorbilidades, trastornos psiquiátricos, enfermedades respiratorias asociadas (SAHOS), fármacos, uso de sustancias ilícitas, estrés psicológico, ambiente, eventos laborales (Turnos) etc.





Epidemiología 

Aprox. 30% de la población de EEUU afirma haber tenido algún trastorno del sueño, aumentando la prevalencia con la edad. El insomnio es el trastorno más común, principalmente en mujeres. La apnea obstructiva del



37

Patrones anormales de sueño: Síndrome de sueño retardado (Adolescentes: insomnio vespertino, somnolencia matinal), Síndrome del sueño adelantado (Adulto Mayor: Somnolencia vespertina, Insomnio Matinal). Alteraciones del Ciclo Sueño-Vigilia: Jet-lag, Síndrome por trabajo de Sistema de Turnos, Síndrome de Ciclo sueño vigilia diferente de 24 horas. Parasomnias: Fenómenos anormales durante el sueño: Trastornos del Despertar: Despertar confusional, Sonambulismo, Terrores Nocturnos. Trastornos de la Transición Sueñovigilia: Movimientos Rítmicos, Hablar nocturno, Calambres nocturnos en EEII. Trastornos Asociados al Sueño REM: Pesadillas, Parálisis del Sueño, Erecciones Dolorosas. Otras: Bruxismo, Enuresis, Distonía Paroxística Asociados a Patología Respiratoria: SAHOS, con Apneas que pueden ser de origen Central o Obstructiva o Mixtas. Narcolepsia (hipersomnia): Ataque de sueño o somnolencia excesiva (no se puede reprimir, en cualquier situación, sueño se inicia con REM). Se

caracteriza por la triada: cataplexia (caída brusca o pérdida del equilibrio por hipotonía), parálisis del sueño (escuchan todo pero no pueden moverse) y alucinaciones hipnagógicas (alucinaciones del despertar). Dentro del estudio, está el laboratorio del sueño. La polisomnografía permite el diagnóstico de SAHOS (lo realizan en algunos hospitales), narcolepsia, movimientos periódicos de las piernas, epilepsia nocturna, terrores nocturnos, insomnio. El test de latencias múltiples de sueño (TLMS): para somnolencia diurna excesiva, narcolepsia.

Tratamiento Siempre recomendar hábitos de higiene del sueño:           

Horarios regulares de acostarse y despertar. No dormir durante el día (máx. siesta 10-15 mins.). Levantarse inmediatamente al despertar. No consumir líquidos al menos 1 hora previo al dormir. No usar estimulantes (café, cigarros, bebidas cola) ni beber alcohol antes de dormir. No auto-medicarse con fármacos para dormir. No realizar actividades en dormitorio (trabajo, comida, TV). Aumentar la actividad durante el día: ejercicio 2030 minutos/día. Dormitorio ventilado, oscuro, silencioso. Relajarse un tiempo antes de dormir. Evaluar comorbilidades, consumo de fármacos o sustancias que puedan causar los trastornos y evaluar la disminución de dosis durante 2 semanas en dosis bajas o suspensión.

Seguimiento Derivar al especialista.

38

  

Traumatismo encéfalo-craneano grave (TEC grave)

TEC leve: 14-15 TEC moderado: 9-13 moderado TEC grave: 3-8 grave

Etiología-Epidemiología Nivel de manejo del médico general: Diagnóstico: sospecha. Tratamiento: inicial. Seguimiento: no requiere.

Es de tipo traumática, siendo la causa más frecuente los accidentes de tránsito. Es la 1° causa de pérdida del conocimiento en la población general. Afecta más a hombres que a mujeres, teniendo una relación de 2:3. El TEC tiene una incidencia de 200 por cada 100.000 habitantes, por cada 300 TEC leve hay 10 graves.

Aspectos esenciales     

Glasgow 3-8 grave o descenso de 3 puntos en forma progresiva. El examen de elección para lesiones intracraneales es el TAC. Derivar a especialista. Manejo en UCI, intubar. Importante es mantener normotensión, normotermia y buena oxigenación y ventilación.

Fisiopatología Existe una transmisión de energía mecánica durante el impacto del cráneo contra objetos contundentes. Esta transmisión de energía se transmite al contenido intracraneal, con efecto distinto para el mecanismo de energía. Se considera daño primario, a aquel producido directamente por el golpe (fractura craneales, contusión, laceración cerebral y daño axonal difuso) y daño secundario, al que es consecuencia de eventos que acompañan al trauma (por ejemplo, alteraciones provocadas por hipoxia e hipotensión, y lesiones de origen vascular como los hematomas subdurales, hematomas epidurales y la hemorragia subaracnoídea)

Caso clínico tipo Paciente de 22 años que presenta importante compromiso de conciencia tras accidente de tránsito cuando iba manejando una moto. Durante el accidente se identificó el fallecimiento de uno de los pasajeros. Ingresa a urgencia con Glasgow 7. Se realiza TAC cerebral, que muestra hematoma subdural de localización frontoparietal derecha.

Se produce un aumento de neurotransmisores excitatorios como acetilcolina, glutamato, aspartato y producción de radicales libres que llevan a daño secundario. El glutamato provoca activación de receptores AMPA que alteran la permeabilidad de la membrana, aumentando el agua intracelular que estimula producción de proteinasas, lipasas y endonucleasas, llevando a apoptosis celular.

Definición Intercambio brusco de energía mecánica que genera deterioro físico y/o funcional del contenido craneal (compromiso de conciencia, amnesia, sd. vertiginoso, etc.), provocado por fuerzas de aceleración y desaceleración. Definido como Escala de Glagow < 8 puntos (TEC Grave).

Diagnóstico El antecedente del traumatismo y nivel de conciencia del paciente evaluado con la Escala de coma de Glasgow (1415 leve, 9-13 moderado, < 8 grave). Es recomendable examinar orientación, memoria, concentración. Considerar los factores de riesgo que pueden hacernos suponer la presencia de un TEC que aún con clínica leve puede evolucionar a un TEC grave en las próximas horas. Es importante buscar signos de focalidad neurológica, tamaño pupilar, reflejos troncoencefálicos (corneal, oculocefálicos, oculovestibulares, nauseoso, etc.) y realizar pruebas radiológicas para diagnosticar lesiones intracraneales asociadas. TAC cerebral con ventana ósea es examen radiológico de elección. La RNM se utiliza en pacientes en los cuales persisten con secuelas postraumáticas.

Tratamiento Manejo prehospitalario: lo principal es prevenir hipotensión e hipoxia. Además, se debe derivar a un centro neuroquirúrgico y avisar que va el paciente, dado que la mortalidad es muy elevada en pacientes con TEC grave.

39

Tabla 14-1. Mortalidad prehospitalaria en TEC. Manejo agresivo de la vía aérea

Seguimiento

Intubados 22,8% 41% 8,6%

Por especialista.

TEC General Glasgow 3 Glasgow 4

No intubados 49,6% 69% 22%

Neurología Fundamental. Alfredo Yañez. 







Asegurar Vía Aérea: intubación endotraqueal y oxigenación al 100% (paciente Glagow <8, Aquellos >8 con traslado a centro lejano, presencia de insuficiencia respiratoria, compromiso circulatorio). No se recomienda el uso de Cánulas Orofaríngeas (causa vómitos por reflejo estimulado). Instalar collar de inmovilización cervical. Ventilación: Descartar lesiones vitales graves en tórax (Neumotórax a Tensión, Hemotórax Masivo, Tórax Volante), Aportar Oxígeno para asegurar SatO2 95% y vigilar Capnografía. Circulación: Restitución de Volemia con dos vías periféricas en extremidades superiores con solución Isotónica (Solución Fisiológica 0,9%) y asegurar PAM mayor a 80. Exposición y Evaluación Neurológica: Clasificar según Glasgow y evaluación pupilar. Investigar consumo de OH, sustancias tóxicas, fármacos o presencia de alteraciones metabólicas.

Manejo de urgencia ABCDE y reevaluación de Escala de Glasgow y Evaluación pupilar.

TAC, hospitalización en UCI, medidas generales de neuroprotección. El resto va a depender de la evaluación del neurocirujano  



  

Posición 30° en paciente hemodinámicamente estable. Mantener estabilidad hemodinámica (solución salina isotónica) y eventual uso de drogas vasoactivas. Optimizar oxigenación cerebral: Mantener SatO2 95%, Hb mayor a 10gr/dl y evitar hipercapnia mayor a 35. Mantener analgesia y eventualmente sedación con BZD, propofol o tiopental. Control de glicemia, natremia y temperatura (mantener normotermia). Sedación en caso de actividad convulsiva

Se debe consignar Glasgow, tamaño pupilar, movimiento de extremidades. Reevaluar e indicar TAC en caso necesario, por ejemplo en agitación psicomotora, reducción del Glasgow, cefalea intensa, vómitos persistentes, signos focales, etc. En caso de herniación transtentorial o deterioro neurológico progresivo, se aconseja uso de manitol, solución hipertónica de NaCl e hiperventilación. En caso refractario considerar propofol o craniectomía descompresiva. Evaluación por Neurocirugía para definir tratamiento según casos especiales.

40

Tabla 14-3. Mortalidad de acuerdo con el puntaje GCS Glasgow Mortalidad 3 65% 4 45% 5 33% 6 24% 7-13 10-15%

Traumatismo encéfalo-craneano leve (TEC leve) Nivel de manejo del médico general: Diagnóstico: específico. Tratamiento: completo. Seguimiento: completo.

Neurología Fundamental. Alfredo Yañez.

Aspectos esenciales     

Las lesiones del encéfalo pueden ser primaria (herida cuero cabelludo, fractura de cráneo y contusión cerebral) o secundarias (hematoma extradural, hematoma subdural agudo o crónico y contusiones hemorrágicas). Las secundarias son potencialmente evitables o tratables con alta prevalencia en los TEC graves. Según la indemnidad meníngea se clasifican en:

Escala Glasgow mide respuesta verbal, motora y ocular. Glasgow 14-15: TEC leve. El examen de elección para lesiones intracraneales es el TAC craneal. Radiografía simple si no es posible realizar TAC. Pacientes asintomáticos, sin factores de riesgo y que no presentan pérdida de conciencia (Glasgow 15) pueden ser dados de alta con indicaciones

 

Caso clínico típico

Diagnóstico

Paciente de 11 años presenta herida contusa de localización frontal tras golpearse con el canto de una puerta. Refiere cefalea intensa. Se ingresa a urgencia con Glasgow 15.

El nivel de consciencia es el principal factor pronóstico en el TEC. Para su evaluación se utilizada la escala de coma de Glasgow, que evalúa 3 parámetros clínicos: apertura ocular, respuesta verbal y respuesta motora. Según el grado de compromiso en cada categoría se asigna un puntaje total (Tabla 1). Dicho puntaje comprende desde los 3 puntos (máximo compromiso) a 15 puntos (mínimo compromiso).

Definición El trauma encefalocraneano es definido como un intercambio brusco de energía mecánica que genera deterioro físico y/o funcional del contenido craneal. Se consigna como alteración del contenido encefálico:    

TEC abierto: lesión con solución de continuidad de las envolturas meníngeas. TEC cerrado: lesión sin comunicación del encéfalo con el exterior.

Es importante buscar signos de focalidad neurológica, tamaño pupilar (normal 2-5 mm), reflejos troncoencefálicos (corneal, oculocefálicos, oculovestibulares, nauseoso, etc.) y respuesta motora. En general, el estudio imagenológico de elección corresponde al TAC craneal. Cuando no puede realizarse, deben tomarse radiografías simples de cráneo si Glasgow <15 o Glasgow 15 con pérdida de consciencia, amnesia postraumática, herida contusa o hematoma subgaleal importante.

Compromiso de conciencia. Cefalea holocránea persistente y progresiva (con o sin vómitos). Amnesia postraumática. Síndrome vertiginoso o mareos persistentes.

Se distingue así TEC de contusión cerebral, que corresponde al impacto mecánico de la bóveda craneana sin alteración del contenido craneano. La definición de TEC leve es aquél cuyo Glasgow es de 14-15.

También debe realizarse estudio de imágenes si el mecanismo involucrado es de alta energía, como caídas de altura o accidentes automovilísticos, pese a una clínica poco alarmante, o si la clínica es poco clara. El estudio de imágenes debe incluir radiografía de columna cervical de tres proyecciones.

Epidemiología El TEC es una importante causa de morbimortalidad en la población general, sobretodo en varones jóvenes, en estrecha relación con los accidentes de tránsito. Corresponden a la primera causa de pérdida de consciencia en la población general y a la primera causa de muerte en la población de 20-40 años en Chile. En la población infantil, el TEC constituye un 3% de las consultas en urgencia y es la causa de muerte de alrededor de un tercio de los pacientes menores de 18 años que fallecen por un traumatismo. Se estima que tiene una incidencia de 200 por cada 100.000 habitantes, máximo riesgo en personas entre 15 a 30 años.

Tratamiento Manejo prehospitalario: clasificar según Glasgow.   

41

Considerar alcohol, sustancias tóxicas, alteraciones metabólicas y fármacos, si es posible. Realizar control hemodinámico precoz. Instalar collar de inmovilización cervical en paciente que presenten factores de riesgo de lesión de columna cervical: Glasgow <15, dolor o rigidez cervical, daño neurológico focal, parestesias en extremidades, otra sospecha de daño de columna cervical.

Manejo de urgencia: Los pacientes asintomáticos, sin factores de riesgo y que no presentan/presentaron alteración del nivel consciencia alguna pueden ser dados de alta sin estudio imagenológico, pero bajo observación domiciliaria por persona responsable. Debe educarse e indicarse el regreso al servicio de urgencia ante signos de posible lesión intracraneal. En aquellos pacientes asintomáticos pero que sí presentan factores de riesgo (coagulopatías, abuso de drogas, antecedentes neuroquirúrgicos, epilepsia, edad avanzada, factores sociales, etc.) se recomienda la realización de TAC para decidir el manejo. Pacientes sintomáticos (amnesia postraumática, convulsiones, vómitos, cefalea progresiva, etc.) o que presentan/presentaron alteración del nivel de consciencia alguna deben estudiarse con TAC craneal y hospitalizarse para observación durante algunas horas. Se puede dar alta si:   

 

El paciente presenta Glasgow 15. No persiste efecto residual de depresores del SNC (alcohol, drogas). No hay factores de riesgo (shock, pérdida de LCR, lesión no accidental en niños, accidente con mecanismo de alta energía, antecedentes de epilepsia, edad mayor de 65 años, terapia anticoagulante, alcoholismo, abuso de drogas, escasa red social de apoyo). No hay signos neurológicos no tratados. Se debe educar y entregar indicaciones por escrito especificando los signos/síntomas ante los cuales debe consultar precozmente y cuidado en domicilio. (compromiso de conciencia progresivo, cefalea progresiva, vómitos, déficit neurológico)

Seguimiento Derivar a especialista: Se derivan a especialista si Glasgow <14, deterioro de nivel de conciencia postingreso, signos focales progresivos, heridas penetrantes, fístula de LCR, imágenes de complicaciones intracraneales, etc.

42

que es debido a alteraciones metabólicas secundarias al trauma, hipoxia, isquemia, edema, apoptosis y disrupciones en la homeostasis, esto ocurre de minutos a horas posteriores.

Traumatismo raquimedular Nivel de manejo del médico general: Diagnóstico: sospecha. Tratamiento: inicial. Seguimiento: derivar.

Diagnóstico Es fundamentalmente clínico y requiere además de estudio imagenológico (Rx y TAC). Se debe sospechar traumatismo raquimedular en toda víctima de trauma de cierta importancia (antecedente de muerto en accidente, choque de alta velocidad, trauma mayor visible, pérdida total del vehículo, Eyección desde el vehículo). Si hay traumatismo con compromiso de conciencia, o un trauma menor con síntomas referidos espinales (cervicalgia, dolor, paresia, parestesias)

Aspectos esenciales      

Debe considerarse en todo paciente politraumatizado. Afecta a hombres jóvenes. Pueden tener lesiones en otros sistemas. El diagnóstico es fundamentalmente clínico. Imágenes permiten complementar sobre el nivel de la lesión. Requiere manejo inmediato.

Se deben buscar signos de lesión medular (respiración abdominal, priapismo, nivel sensitivo claro, alteración en control de esfínteres). El estudio clínico permite diferenciar diferentes tipos de lesión:

Caso clínico tipo



Paciente hombre de 25 años que sufre accidente automovilístico. Llega al SU luego de un buen manejo en el lugar del accidente. En el examen neurológico se constata paresia de EESS e inferiores con mayor afectación de las extremidades superiores, además de pérdida de la sensibilidad variable.





Definición Trauma de la columna vertebral que como consecuencia genera lesión, compresión o sección de estructuras nerviosas, ya sea médula espinal, raíces o envolturas medulares.



Epidemiología Habitualmente afecta a personas jóvenes en etapa productiva. La mayoría de los casos son hombres. La causa más frecuente es debida a accidentes de tránsito (48%), también se observa secundaria a caídas de altura (16%), violencia por heridas de bala (12%), por deportes (10%) y otras (14%). Su incidencia es de 40 por 1 millón de personas al año. Los hombres representan del 77-80% de los casos, y el alcohol presenta un rol fundamental en al menos un 25%.



Completa: cuando no hay función motora o sensitiva más de tres niveles bajo el nivel de lesión. 3% de pacientes puede tener alguna recuperación en las primeras 24 horas, pasado ese plazo no desarrollan recuperación. Incompleta: preservan función motora o sensitiva más de tres niveles bajo la lesión: sensación táctil o postural, función sacra, reflejo del control de esfínteres. Considerar siempre durante el inicio del cuadro la posibilidad de Shock Espinal. Síndrome medular central: alteración sensitiva termalgésica bilateral en el nivel afectado, con preservación de la propiocepción, paresia y compromiso de los reflejos en el segmento motor y disfunción vesical. En casos de hiperextensión severa cervical, se produce cuadriplejia, de predominio distal de miembros superiores, con nivel sensitivo y disfunción vesical Síndrome de Brown-Sequard o de hemisección medular: Contralateral a la lesión hay pérdida sensitiva disociada, conserva tacto fino. Ipsilateral sin propiocepción y hay parálisis motora. Mejor pronóstico. Síndrome medular anterior: paraplejia y pérdida sensitiva disociada (preserva propiocepción, pierde termoalgesia). Es de mal pronóstico. Es importante considerar también dentro del inicio del cuadro posterior a un trauma raquimedular, el shock neurogénico que debuta con bradicardia e hipotensión secundario a la alteración del SNA.

Fisiopatología

Estudio radiológico

Los segmentos móviles cervical y lumbar, son los más afectados. Las fracturas cervicales ocurren por angulación aguda del cuello en sacudidas de flexoextensión (accidentes automovilísticos) o en flexión marcada (caídas de altura). El trauma daña la médula por compresión directa (por hueso, ligamento, disco) o por interrumpir su irrigación o por tracción o sección propiamente por tracción de las estructuras secundarias. En la mayoría de las lesiones medulares, pequeños vasos intramedulares son lesionados, lo cual produce hemorragia disminuyendo el flujo tisular, hay isquemia y luego necrosis de la sustancia gris central y posteriormente de la sustancia blanca. Debe tenerse en cuenta además el daño que ocurre posterior al trauma el

La radiografía: de columna cervical, torácica y lumbar así como de pelvis y huesos largos de regla y de acuerdo al grado de traumatismo. La tomografía computada: complementa el estudio de la lesión y debe solicitarse en los pacientes cuya radiografía no permite descartar lesión ósea. A diferencia de la RNM, tiene mejor visualización de los cuerpos óseos. El gold estándar es la RNM de médula, cumple un rol indispensable en la visualización de lesiones ligamentosas, discales y medulares. Es un examen imprescindible en TRM, aunque más caro.

43

Tratamiento Manejo inicial: Debe iniciarse en el sitio del accidente. CAB del trauma. 





  

Inmovilización cervical inmediata y asegurar vía aérea permeable, evitar el movimiento y lateralización de columna cervical. Ventilación y aporte de oxígeno, evaluación de lesiones a nivel de tórax que comprometan mecánica ventilatoria. Control de hemorragia y aporte de volúmen con dos vías periféricas de grueso calibre y control de la presión arterial media con meta PAM >80. Evaluación en el sitio de alteraciones neurológicas. Traslado rápido y cuidadoso a un centro médico con la infraestructura adecuada. Siempre considerar la posibilidad del paciente en shock neurogénico con hipotensión y bradicardia, por lo que en algunos casos se requerirá incluso manejo con drogas vasoactivas.

El tratamiento farmacológico: dependerá de las normas de cada centro. Metilprednisolona (NASCIS2-NASCIS3): durante las primeras 8 horas. Si fue entre 0 y 3 horas mantener aporte por 24 horas, si fue entre 3 y 8 horas, mantener por 48 horas. Tratamiento quirúrgico 

Cirugía descompresiva: tiene como objetivo descomprimir el canal medular, alinear el raquis si no ha sido posible por medios ortopédicos y estabilizar la columna vertebral lo que va a permitir una rehabilitación precoz. Se sugiere en lesiones de carácter incompletas que se evidencien al momento del accidente. Contraindicada en lesión completa de más de 24 horas de evolución; medicamente inestable o por posible síndrome medular central.

Seguimiento Derivar con especialista para manejo de complicaciones asociadas. Tratamiento multidisciplinario para maximizar calidad de vida.

44

Tumores de la region selar 

Nivel de manejo del médico general: Diagnóstico: sospecha. Tratamiento: inicial. Seguimiento: derivar

Aspectos esenciales     

El adenoma es el tumor más frecuente de la región selar. Los adenomas pueden ser funcionantes (más frecuentes) o no funcionantes. Entre los funcionantes el más común es el prolactinoma. Tumor selar con quistes y calcio orienta a craneofaringeoma. El tratamiento es esencialmente quirúrgico.

 

funcionantes (etiología correspondiente a gonadotropinomas) El craneofaringeoma representa aproximadamente el 4% de los tumores intracraneales, (incidencia global de 0,5-2,5 por millón de habitantes al año); es igual de frecuente en ambos sexos; la edad más frecuente de aparición es la infancia (mayor distribución de 5 a 14 años) representando el 5 a 10% de los tumores intracraneales y adolescencia temprana, aunque también pueden aparecer en adultos de 50 a 75 años (distribución bimodal); su localización más frecuente es la región supraselar; surge habitualmente del tallo hipofisario y se extiende hacia el hipotálamo. El carcinoma es muy infrecuente y afecta a adultos principalmente. Tumor de células granulares se presenta habitualmente en la edad adulta (quinta y sexta década), con mayor incidencia en mujeres; son muy infrecuentes.

Fisiopatología Caso clínico tipo

Depende de la funcionalidad del tumor en el caso de los Adenomas, se puede presentar clínica de acuerdo al aumento de secreción de la hormona correspondiente. En el caso de los Prolactinomas la hipersecreción de Prolactina explica los síntomas y signos clínicos. En el caso de los tumores no funcionantes, los productores de gonadotropinas, secretan una cantidad muy baja de hormonas no detectable por laboratorio ni funcional, por lo que son tumores biológicamente ineficientes. Su clínica se explica por el aumento de secreción a nivel de Hipófisis secundario a la inhibición del tono inhibitorio de la Dopamina dado por el Hipotálamo sobre la Hipófisis en las células productoras de Prolactina. Por otro lado la compresión de la Hipófisis puede traducirse en una deficiencia hormonal lo que también se evidenciará en la clínica dependiendo del grupo afectado (TSH, ACTH, FSH, LH).

Mujer de 24 años consulta por amenorrea de 3 meses a lo que se suma en el último mes galactorrea. Al examen físico se evidencia leve hemianopsia heterónima bitemporal. Paciente se encontraba con uso de ACO y trae dos test de Embarazo Negativo, además de BHCG intedectable.

Definición Corresponden a tumores cerebrales primarios. Dentro de esta clasificación se engloban diferentes tipos de tumores como los adenomas, el craneofaringeoma, carcinoma hipofisiario y el tumor de células granulares. Se presentan en general como detección incidental, por síntomas neurológicos o por anormalidades hormonales.

Diagnóstico

Etiología 



 

El diagnóstico es clínico, de laboratorio y neuroimagenológico (TAC o RM). Las manifestaciones clínicas son disfunción o hiperfunción endocrina, alteraciones visuales (hemianopsia heterónima bitemporal o alteración de la agudeza visual progresiva) y cefalea (efecto mecánico compresivo o invasivo sobre la silla turca, la hipófisis, el quiasma óptico y/o el diencéfalo). En relación a las manifestaciones endocrinas se reconocen cuatro síndromes relevantes:  Síndrome galactorrea-amenorrea-disminución de líbido, por hiperproducción de prolactina.  Gigantismo en niños y la Acromegalia en adultos.  Enfermedad de Cushing.  Panhipopituitarismo (evidenciable también en casos de Apoplejía Hipofisiaria).  Cuando hay compromiso de la neurohipófísis puede producir alteraciones en la secreción de ADH causando Diabetes Insípida. Importante tener en cuenta que los adenomas frecuentemente dan síntomas endocrinos y que los craneofaringiomas pueden provocar obstrucción al flujo de LCR y causar hidrocefalia.

Adenoma: Es un tumor benigno de la adenohipófisis. Puede ser funcionante (situación dada en el 70%, siendo el más frecuente el prolactinoma) o no funcionante. Microadenoma cuando es menor a 1 cm y macroadenoma cuando es mayor a 1cm. Craneofaringioma: Es un tumor benigno, parcialmente quístico, que se origina en el epitelio de la bolsa de Rathke. Carcinoma: tumor maligno con capacidad de diseminación metastásica intra o extracraneal. Tumor de células granulares: Tumor epitelial benigno originado a partir de pituicitos granulares de la neurohipófisis.

Epidemiología 

Los adenomas son la tercera neoplasia intracraneal más frecuente (con una prevalencia de aproximadamente el 10% en la población general) y los más frecuentes de la región selar. Aparecen típicamente en la edad adulta, y son más frecuentes en mujeres. El 70% son funcionantes (Prolactinomas) y 30% no 45



Cuando son tumores grandes pueden provocar panhipopituarismo, apoplejía hipofisiaria (cefalea, oftalmoplejia, y disminución de la conciencia)

Las pruebas de laboratorio incluyen estudio hormonal para comprobar si existe hipersecreción hormonal por parte del tumor y para evaluar la función del resto del tejido hipofisario. En el caso de los Prolactinomas su secreción suele ser sobre 200 ng/ml, en el caso de lo tumores no funcionantes suele ser menor de 100 ng/ml. Suelen determinarse de forma rutinaria los niveles de prolactina, TSH, T4 libre, FSH, LH, IGF-1, testosterona (en hombres), estradiol (en mujeres) y cortisol basal. En cuanto a la neuroimagen: es de elección la RM hipofisaria, por sobre el TAC con contraste.

Tratamiento El tratamiento de elección es quirúrgico y consiste en la extirpación del tumor, generalmente vía transesfenoidal. En el caso de los microprolactinomas, el tratamiento de elección es el tratamiento médico, ya que algunos de éstos mediante el tratamiento con agonistas dopaminérgicos disminuirá el tamaño del tumor. En cambio, en los macroprolactinomas se puede iniciar tratamiento médico y seguimiento. Se decidirá de acuerdo a la respuesta clínica. El tratamiento médico consiste en la administración de Bromocriptina (agonista dopaminérgico) o Cabergolina en el caso del prolactinoma y el octreótido o análogos en los secretores de GH. Esto con el fin de disminuir el volumen del tumor previo a ser extirpado. En los casos en los que se debuta con insuficiencia endocrina hipofisiaria, es importante considerar la reposición hormonal correspondiente previo a la cirugía. La radioterapia suele ser un tratamiento coadyuvante luego de la citoreducción quirúrgica en los tumores más agresivos; se emplea en pacientes con resección de difícil acceso.

Seguimiento Por especialista.

46

MÓDULO 1: Medicina Interna

Reumatología Artritis Reumatoídea Etiología-Epidemiología-Fisiopatología

Nivel de manejo del médico general: Diagnóstico Sospecha Tratamiento Inicial Seguimiento Derivar

Prevalencia en el mundo es del 0,5-1%, siendo más frecuente en mujeres con relación 3:1. Se inicia entre los 40 y 60 años. En Chile es la enfermedad del tejido conectivo más frecuente. Histológicamente se caracteriza por la aparición de un infiltrado inflamatorio crónico en el tejido sinovial, constituidos por linfocitos T, linfocitos B, plasmocitos, macrófagos y células dendríticas. La intensa actividad inmunológica transforma el tejido sinovial en un órgano linfoide secundario y la proliferación de las células sinoviales residentes, en respuesta a un exceso de diversas citokinas producidas localmente, lleva a la formación del pannus. Este último es un tejido sinovial hiperplásico que produce erosión del cartílago articular y degradación del hueso subyacente.

Aspectos Esenciales • • • •

Enfermedad inflamatoria crónica con afección poliarticular erosiva. Más frecuente en mujeres entre los 40 y 60 años. FR y Anti CCP de gran utilidad en el diagnóstico. Tratamiento farmacológico, con especial importancia de fármacos modificadores de enfermedad.

La interacción de genes de predisposición a la enfermedad y el ambiente provocan una ruptura de la tolerancia inmunitaria a autoantígenos articulares que gatillan una reacción inflamatoria crónica, la cual a su vez, inducen hiperplasia sinovial. Ambos procesos determinan un aumento de la osteoclastogénesis, responsable en gran parte del daño articular.

Caso Clínico Tipo Mujer de 50 años que consulta por dolor y rigidez matinal de manos de más 2 horas. Al examen físico se pesquisa dolor e inflamación en las articulaciones MCF e IFP de distribución simétrica. Serología evidencia FR y CCP (+).

Diagnóstico

Definición

Clínica: Comienzo gradual e insidioso en la mayoría de los casos. El compromiso articular (artritis) se caracterizan por dolor y edema, a lo cual se agrega aumento de temperatura y limitación funcional, siendo el compromiso articular la mayoría de las veces simétrico. El compromiso de manos refleja el daño general del organismo, siendo lo más característico la afección en las IFP y MCF (articulación más comprometida), sin compromiso de IFD (a diferencia de la artropatía psoriática y el compromiso no inflamatorio de la artrosis). La afección crónica en estas articulaciones puede llevar a deformaciones características tales como dedos en cuello de cisne, dedos en boutonniere, desviación en ráfaga cubital y pulgar en Z.

Enfermedad inflamatoria crónica multisistémica de causa desconocida, que se caracteriza por sinovitis inflamatoria persistente, en la mayoría de los casos con compromiso articular periférico simétrico. Es característica la presencia de erosiones óseas, destrucción del cartílago, deformidad articular, si bien su evolución puede ser muy variable. Factores tanto ambientales como genéticos participan en la instauración de este cuadro, se ha relacionado en particular con HLA-DR4.

1 Facultad de Medicina, Universidad de Chile

Además del compromiso de manos existe compromiso en muñecas, codos, hombros, temporo-mandibular, cricoaritenoídeo, caderas, rodillas, pies y columna cervical (subluxación atlantoaxoídea). El compromiso de pies involucra a cerca el 90% de los pacientes en el transcurso de la enfermedad, siendo de igual o mayor frecuencia al compromiso de manos.

primeros años), compromiso extra-articular, aumento de VHS y PCR persistente, FR a títulos altos, mala respuesta a metrotrexato. •

El compromiso extra-articular se manifiesta de diversas formas y es marcador de mal pronóstico. Lo más frecuente es la presencia de nódulos reumatoídeos (20-50%) de localización subcutánea y en superficies de extensión. Pueden observarse manifestaciones hematológicas como anemia (2530%), generalmente secundaria a inflamación crónica, pero también asociada al uso de DMARDs y AINEs. De manera muy infrecuente puede haber leucopenia en el contexto del Sd. Felty (leucopenia y esplenomegalia). Además puede existir compromiso pulmonar (derrame pleural, enfermedad intersticial), compromiso cardíaco (pericarditis), compromiso ocular (Sd. Sjögren 2°, epiescleritis, escleritis), compromiso neurológico (mononeuritis múltiple, mielopatía cervical) y afección vasculítica. Compromiso renal es solo secundario a fármacos.

Tratamiento no farmacológico: educación, terapia física y ocupacional, ejercicios intercrisis, reposo en crisis, férulas para prevenir deformidades. Fundamental resulta el cese del tabaquismo, el cual no sólo está involucrado en la evolución y pronóstico de la enfermedad, sino en la patogenia de la misma.

• Tratamiento farmacológico: 1. AINES: manejo sintomático. De uso limitado por efecto adversos. 2. Corticoides: no modifican curso de enfermedad, pero resultan útiles en manejo sintomático. Se evita uso prolongado y se intenta usar el mínimo tiempo posible o la mínima dosis posible asociados a agentes modificadores de enfermedad (DMARD). 3. DMARDs: Retardan progresión, por lo que son el pilar del manejo farmacológico de la AR. Lo agentes más tradicionales de este grupo son el metrotrexato, leflunomida y sulfasalazina, siendo el primero, el más utilizado. El uso de hidroxicloroquina asociado como inmunomodulador es frecuente y muy útil.

Laboratorio: Factor reumatoideo (FR) (sensibilidad de 70% y especificidad menor al 80%), anti CCP similar (sensibilidad sólo un poco mayor a FR, pero muy específico (96%)). Suele haber aumento de parámetros inflamatorios como VHS y PCR.



Imágenes: importante como herramienta diagnóstica. Los hallazgos más característicos a la radiografía son: disminución del espacio articular, osteopenia yuxtaarticular, erosiones óseas subcondrales, aumento de volumen de partes blandas, desviaciones óseas y subluxaciones en algunos casos. La radiografía cervical resulta útil para diagnóstico de subluxación atlanto-axoídea, la cual puede ser asintomática. Los criterios de clasificación de 1987 fueron modificados el 2010, lo cual permitió mejorar la sensibilidad y con ello lograr un diagnóstico e inicio de tratamiento precoz. Estos incluyen 4 pilares: compromiso articular, serología, reactantes de fase aguda y tiempo de evolución, siendo compatible con AR si cumple 6 o más criterios. Todo paciente debe tener como mínimo radiografía de manos y pies AP, columna cervical AP, lateral y en flexión máxima y de tórax.

Terapia biológica: reservado para pacientes refractarios a uso de los medicamentos previamente mencionados o con enfermedad agresiva. Dentro de estos se incluye anti TNF (Infliximab, Etanercept, Adalinumab, Golimumab), anti CD 20 (Rituximab) y anti IL-6 (Tocilizumab).

Seguimiento Se deriva a todo paciente con sospecha de AR a Reumatología. Todo paciente con AR debe realizarse EMPA anual (al presentar una enfermedad inflamatoria crónica se asocia a mayor riesgo cardiovascular por ateroesclerosis acelerada, debiendo controlarse factores asociados como hipertensión arterial, diabetes y dislipidemia).

GES: Todo médico que sospeche debe derivar de inmediato a reumatología para diagnóstico, debe ser evaluado por especialista en un plazo de 90 días.

Autor / Editor Paulina Lira

Tratamiento Todo paciente debe iniciar tratamiento en período <1 mes desde confirmación diagnóstica. Factores de mal pronóstico: inicio poliarticular (>20 articulaciones), inicio en >75 años, mala capacidad funcional inicial, presencia de erosiones (radiografía de pie o manos en 2

2 Facultad de Medicina, Universidad de Chile

Año 2016

Diagnóstico

Fibromialgia

Clínica: Los hallazgos más comunes son: polialgia (que empeora con estrés y frío), rigidez matinal generalizada, fatiga (síntoma constante y que no mejora con el sueño ni con reposo), sueño fragmentado, no reparador,trastornos del ánimo, en especial depresión y alteraciones cognitivas como fallas en la memoria. En ocasiones, el dolor se acompaña de parestesias en las extremidades, sensación de calambres, quemazón y hormigueos, debiendo hacerse el diagnóstico diferencial con neuropatías.

Nivel de manejo del médico general: Diagnóstico Sospecha Tratamiento Inicial Seguimiento Derivar

Aspectos Esenciales • • • •

Dolor musculo-esquelético generalizado, fatiga y trastornos del sueño. Afecta predominantemente a mujeres en edad fértil. Se asocia fuertemente con depresión. Debe realizarse una evaluación de laboratorio básica para descarte de causas orgánicas (Hemograma, VHS, calcemia, CK y TSH).

Al examen físico, destaca la presencia de puntos gatillo en localizaciones específicas del sistema musculoesquelético. Los criterios diagnósticos clásicos exigen la presencia de al menos 11 de los 18 puntos en contexto de dolor generalizado de al menos 3 meses de evolución. El año 2010, se establecieron los criterios de Wolfe para el diagnóstico de fibromialgia, los cuales eliminan como instrumento la presencia de puntos gatillo y agregan lo demás síntomas descritos, no considerados en los primeros criterios. Debido a que los síntomas de fibromialgia pueden ser secundarios a otras patologías, debe realizarse un estudio básico en todo paciente en que se plantea el diagnóstico, el cual debe incluir como mínimo hemograma con VHS (para descarte de patología inflamatoria sistémica), hormonas tiroídeas (pues el hipotiroidismo puede asemejar sus síntomas a la fibromialgia), creatinkinasa (para el diagnóstico diferencial de miopatías inflamatorias) y calcemia + fosfemia (para evaluación inicial de hiperparatiroidismo, el cual puede cursar con dolor generalizado. Si la sospecha es alta, deberá solicitarse Parathormona).

Caso Clínico Tipo Mujer de 40 años que refiere presentar dolor en hombros, manos y cuello de 4 meses de evolución. Además refiere cansancio generalizado durante el día y sueño no reparador. Se ha exacerbado durante el último mes en relación a problemas en su núcleo familiar.

Definición Enfermedad caracterizada por dolor musculoesquelético crónico difuso, rigidez, parestesias, fatiga y trastornos del sueño. Si bien muchas veces el dolor de esta patología puede confundirse con reumatismos articulares, éste se localiza en músculos, ligamentos y tendones, por lo que la fibromialgia está clasificada dentro de los reumatismos de partes blandas.

Tratamiento Comprende medidas no farmacológicas como ejercicio físico aeróbico y graduado que ha demostrado ser parcialmente eficaz en el manejo del dolor. La terapia farmacológica comprende de AINEs, inhibidores de recaptación de serotonina y noradrenalina (duloxetina), pregabalina y relajantes musculares rincipalmente, siendo necesario el apoyo de unidades de salud mental para el manejo integral del paciente. Se utilizan BZD o antidepresivos tricíclicos (amitriptilina) para los trastornos del sueño asociados.

Epidemiología Constituye la causa más frecuente de dolor musculoesquelético difuso no inflamatorio. Afecta entre el 0.5-5% de la población general, en especial a mujeres desde los 45 años, lo que representa entre el 80-90% de los casos. Se reconocen factores genéticos asociados, avalados por el mayor riesgo de padecer la enfermedad en familiares de primer grado. Se han postulado por otro lado, varios desencadenantes tales como infecciones y traumas emocionales, pero su patogenia aún no está aclarada. La fibromialgia se genera dentro del contexto del síndrome de sensibilización central, en el cual los pacientes son más sensibles al dolor y donde se altera el proceso de inhibición del mismo.

Seguimiento Derivar a especialista.

Autor / Editor Paulina Lira

3 Facultad de Medicina, Universidad de Chile

Año 2016

situaciones que incluyen caída “sobre el hombro”) o no traumático. En este último caso, deben evaluarse causas extrínsecas como origen del dolor (cervicobraquialgia, isquemia miocárdica o incluso cuadros infradiafragmáticos).

Síndrome De Hombro Doloroso Nivel de manejo del médico general: Diagnóstico Sospecha Tratamiento Inicial Seguimiento Completo

Cuando la omalgia no es referida, el siguiente paso es determinar si la afección es articular o extraarticular. En el último, los síntomas se asocian a ciertas posiciones o actividades reproducibles, no presentan limitación a la movilización pasiva de la articulación y generalmente el dolor es localizado, mientras que en los de origen articular, el dolor es más difuso, se reproduce en todas direcciones, genera alteraciones tanto a la movilidad pasiva como activa y puede estar relacionado a signos inflamatorios locales.

Aspectos Esenciales • • • •

Distinguir origen traumático/no traumático, intrínseco/ extrínseco y articular/extraarticular. La causa más frecuente corresponde a lesiones del manguito rotador. El diagnóstico es clínico. Objetivo del tratamiento es evitar el hombro congelado.

El dolor de origen extrarticular producido por lesión en manguito de rotador (la causa más frecuente en dicho contexto), suele ocurrir en personas mayores de 40 años, se presenta en región superolateral del hombro, característicamente se incrementa en la noche y se exacerba con movimientos de flexión y abducción. Para examinar el manguito rotador deben realizarse maniobras orientadas a evaluar los movimientos del hombro tanto pasiva como activamente. Dentro de la evaluación del hombro, destaca el test de Jobe para el supraespinoso, que se realiza con el brazo a 90° de abducción, 30°de flexión y en rotación interna con el pulgar apuntando al piso, y se evalúa el dolor y la debilidad contra resistencia. Para evaluar los rotadores externos (infraespinoso y redondo menor), se pide al paciente realizar el movimiento espontáneo (manos detrás de la cabeza) y contra resistencia, misma situación para evaluar al rotador interno (subescapular), pidiendo al paciente simular que se abrocha el sostén o que intente tocar sus escápulas. Debe tenerse la precaución de examinar el hombro fijando la escápula, debido al importante rol de la articulación escapulotorácia en la movilidad del hombro.

Caso Clínico Tipo Paciente masculino de 68 años, consulta por omaglia derecha, sin antecedentes traumáticos. Al examen, destaca dolor a la abducción entre 70 y 100°, lo cual no ocurre ante movilidad pasiva. La ecografía muestra tendinopatía del supraespinoso.

Definición El dolor de hombro (omalgia) es, luego del dolor lumbar y cervical, el síntoma musculoesquelético más frecuente.

Epidemiología

El signo del arco doloroso consiste en solicitar al paciente la abducción completa del miembro. Si presenta dolor entre los 70 y 100°, el cual cede ante la superación de dicho ángulo (momento en el cual el deltoides asume la función), se comprueba el compromiso del manguito rotador. Si por otro lado, el dolor se genera sólo al superar los 100-120°, debe sospecharse alteración acromioclavicular. La radiografía tiene utilidad limitada considerando que la principal causa de omalgia es secundaria a compromiso de partes blandas. Sí es útil cuando se sospecha compromiso articular por examen físico, para la evaluación de la presencia de calcificación tendínea o para obtener información indirecta en el caso de la rotura del supraespinoso, en el cual se verá ascenso de la cabeza humeral. Debe solicitarse en AP con rotación interna, externa y outlet. El ultrasonido es un método de bajo costo, no invasivo, rápido y seguro en la evaluación de lesiones del manguito rotador y bursitis, siendo de elección para evaluar omaglia. El método diagnóstico más completo en la evaluación del hombro es la resonancia magnética, la cual debe ser reservada sólo en casos de duda diagnóstica.

Es responsable del 15% de las consultas por dolor musculoesquelético. Si bien son muchas las causas de dolor de hombro (traumáticas, degenerativas, inflamatorias, vasculares, tumorales y dolor referido), la más común tiene su origen en los tejidos blandos periarticulares como tendones y las bursas, con frecuencia asociada a la enfermedad del manguito rotador. Cuando el compromiso es articular, las causas subyacentes son las artritis infecciosas, por cristales y autoinmunes, entre otras. La razón de la alta frecuencia de esta patología se sustenta en que la articulación del hombro es compleja e inestable debido a su amplio rango de movilidad - característica relacionada al inicio de la bipedestación en el hombre - por lo que requiere de la función integrada de numerosos músculos y ligamentos.

Diagnóstico La clínica es la base del diagnóstico. Lo primero a determinar es si el origen del dolor es traumático (relacionado a fractura o a disyunción acromioclavicular, esta última sugerente ante

4 Facultad de Medicina, Universidad de Chile

Depósitos de calcio en tendón del supraespinoso (flecha blanca) y bursa subdeltoídea (flecha negra).

Tratamiento

Seguimiento

El objetivo fundamental del tratamiento de la omalgia es la prevención de la capsulitis adhesiva (hombro congelado), por lo que debe evitarse la inmovilización prolongada e incluirse precozmente la rehabilitación kinésica para preservación de rango articular y fortalecimiento muscular. Dependiendo del caso, en relación a severidad del cuadro o presencia de bursitis, puede indicarse infiltración con corticoides.

En sospecha de una tendinitis del manguito de rotador, el manejo puede realizarse por médico general. En caso de sospecha de rotura de manguito, en especial en pacientes jóvenes o de una patología sistémica con repercusión en la articulación del hombro, debe derivarse de forma oportuna a traumatología.

Autor / Editor Paulina Lira

Infiltración bursa subacromial.

5 Facultad de Medicina, Universidad de Chile

Año 2016

Lumbago Mecánico



Nivel de manejo del médico general: Diagnóstico Sospecha Tratamiento Completo Seguimiento Completo

interapofisarias posteriores Por sobrecarga funcional o postural: dismetrías pélvicas, hipotonía muscular abdominal, hipertonía muscular posterior, sobrecargas musculares y discales, embarazo, sedentarismo, hiperlordosis, deportivas.

Diagnóstico Aspectos Esenciales • • • • •

El diagnóstico es generalmente clínico en casos de lumbago mecánico puro y lumbociática. Rara vez se indica la realización de exámenes en la lumbalgia aguda o subaguda, pero si se deben realizar ante presencia de factores de riesgo. La RM y la TC son los exámenes de elección ante sospecha de alteraciones estructurales de columna. La electromiografía y pruebas de conducción nerviosa permiten evaluar la función de nervios periféricos.

Más del 90% de los lumbagos son mecánicos Afecta a personas jóvenes en edad laboral, siendo una de las primeras causas de ausentismo laboral El diagnóstico es clínico Es un cuadro de origen agudo autolimitado, cuyo tratamiento se basa en reposo, analgésicos, antiinflamatorios, relajantes musculares. Siempre se deben buscar banderas rojas, ya que la presencia de estas orientan a lumbago de origen inflamatorio (infeccioso, inmunológico, tumoral o traumático), en cuyo caso se necesita realizar estudio.

Siempre hay que buscar banderas rojas, que nos orientaran a causas no mecánicas del dolor (tumorales, infecciosas, inflamatorias) que son: •

Caso Clínico Tipo •

Paciente de sexo masculino de 37 años, cargador de la vega, sin antecedentes mórbidos que consulta por cuadro de 2 días de evolución de dolor lumbar bajo, intenso, bilateral, sin irradiación, que inició posterior a levantar mal una carga. El dolor se intensifica durante el día, llegando a inhabilitarlo en su trabajo. Al examen físico, mantiene movilidad de la columna, levemente limitado por dolor. Se palpa contractura muscular paravertebral, sin déficit neurológico.

• • • • • • • •

Definición Dolor lumbar por la utilización excesiva de una estructura anatómica normal, por lesión o malformación de una estructura anatómica. Agudo: < 4 semanas Subagudo: 4-12 semanas Crónico: > 12 semanas



Epidemiología Episodios agudos se resuelven el 90% a las 8 semanas, pero hay un porcentaje de pacientes que hacen crisis repetidas o padecen de lumbago crónico.

Primer episodio post 50 años o previo a 20 años (sin embargo este factor por sí sólo no es una bandera roja, pero cobra mucha relevancia si está asociado a otras banderas rojas) Dolor nocturno que despierta al paciente o le impide dormir (hay que diferenciar si el paciente se despertó por otra causa y sintió el dolor) Dolor de reposo (ya que los lumbagos mecánicos deberían aliviarse con este) Baja de peso Fiebre Déficit neurológico Rigidez posterior al reposo Antecedente de patología tumoral Trauma. Lumbago mecánico (70%): dolor mecánico lumbar puro, limitado a la parte baja de la espalda y bilateral, aparece en forma aguda, después de alguna actividad física. La exploración física es anodina. Es frecuente el dolor a la palpación en la parte baja de la espalda. Lumbociática (4%): dolor lumbar radicular, que se irradia siguiendo el dermatomo de la raíz afectada (más frecuente L5-S1). El dolor aumenta con Valsalva y al sentarse, y disminuye en decúbito. Puede acompañarse de alteraciones sensitivas, paresia y asimetría de reflejos. Hay signos radiculares: Tepe, Tepe contralateral, Gowers, Lasègue. Su origen más frecuente es la enfermedad discal. Es muy importante descartar Sd. cauda equina (hipoestesia en silla de montar, dolor perineal, alteración de esfínteres, sd ciático bilateral) ya que es una urgencia.

Etiología •

Alteraciones de las estructuras vertebrales: espondilolisis, espondilolistesis, escoliosis, patología discal, artrosis

6 Facultad de Medicina, Universidad de Chile

Hay que tener en cuenta que: • Si los síntomas persisten por 6 a 8 semanas, se debe tomar radiografías buscando otras causas, y si el estudio es negativo, derivar a nivel secundario. • En lumbociáticas con deterioro progresivo del estado neurológico está indicada la RNM. • Un lumbago puede ser diagnosticado como mecánico, pero si aparecen banderas rojas en la evolución, también se debe realizar estudio.

El tratamiento de la lumbociática es similar al lumbago mecánico, se puede agregar infiltraciones peridurales. Lo normal es que el 90 % responda a tratamiento médico en 6-12 semanas, sin embargo, si no mejora, si hay déficit progresivo o dolor invalidante se debe considerar cirugía

Seguimiento Completo

Tratamiento El tratamiento del lumbago mecánico, incluye reposo relativo (no más de 3 días), uso de calor húmedo en área afectada (toallas húmedas calientes). Se puede usar AINEs (por periodos cortos, si no hay contraindicación); los opioides no han demostrado ser significativamente superiores a los AINEs en la lumbalgia aguda. Relajantes musculares como la ciclobenzaprina pueden ser útiles, pero es común la sedación como efecto secundario. Es importante realizar kinesioterapia y ejercicios para fortalecer la musculatura de la espalda después, y manejar aquellas condiciones favorecedoras (sobrepeso, mala posturas).

Autor / Editor Nicole Fritzche

7 Facultad de Medicina, Universidad de Chile

Año 2016

relacionada con la formación de autoanticuerpos que, ya sea daño directo o mediante la formación de inmunocomplejos, generarán el daño multisistémico de la enfermedad. Dentro de los autoanticuerpos, vale destacar los antinucleares (ANA) como los más sensibles para LES. Un ANA (+) es sólo la punta del iceberg, pues se debe determinar si esos anticuerpos se dirigen contra estructuras proteicas no histonas (ENA) o contra el DNA (anticuerpos antiDNA), los cuales son altamente específicos para LES. Además es posible que factores relacionados con las hormonas sexuales, genéticos y ambientales posean relevancia en la patogenia.

Lupus Eritematoso Sistémico Nivel de manejo del médico general: Diagnóstico Sospecha Tratamiento Inicial Seguimiento Derivar

Aspectos Esenciales • • •

Es más frecuente en mujeres en edad reproductiva. La artritis es poliarticular no erosiva y no deformante en la mayoría de los casos. Clínica junto a ANA (+) es altamente sensible para diagnóstico.

Diagnóstico Criterios clínicos e inmunológicos: Se requiere de 4 de 11 criterios para clasificar a un paciente como LES (ACR 1997). •

Caso Clínico Tipo



Mujer de 22 años consulta por cuadro febril de una semana de evolución, artralgias, astenia y adinamia. Al examen físico ausencia de murmullo pulmonar en base pulmonar derecha. Hemograma con Hb: 10,6, Hcto: 31%, VCM: 92.5, GB 3700, linfocitos 40% y plaquetas 130.000. ANA (+), DNA (+) e hipocomplementemia.



Definición



Enfermedad inflamatoria crónica de patogenia desconocida y etiología autoinmune. Se caracteriza por la formación de autoanticuerpos, inmunocomplejos y un heterogéneo compromiso multiorgánico, en el cual los tejidos y células experimentan lesión mediada por autoanticuerpos fijadores de tejido y complejos inmunitarios.



Epidemiología La incidencia en EEUU es de 1,8-7,6 x 100.000 habitantes y la prevalencia de 40-50 x 100.000 habitantes. Es más frecuente en mujeres, presentando una relación de 9-12:1 respecto a los varones. La edad promedio de aparición en mujeres blancas es de 37 años. Es más frecuente en raza negra y el antecedente de familiar de primer grado con LES implica un aumento en 3,3-9,1 veces el riesgo de presentar la enfermedad.





Etiología



La etiología es desconocida, pero se presume la participación de componentes genéticos, ambientales y hormonales. Estos últimos generarían una disfunción en las células T y una producción anormal de citokinas que conllevarían a una activación de linfocitos B. Dicha activación está directamente

Eritema malar: Representa la clásica erupción eritemoescamosa en región malar y nasal. Se presenta en aproximadamente el 30-40% de los pacientes. Eritema discoide: Lesiones solevantadas, descamativas, de borde irregular y que pueden desencadenar una cicatriz atrófica. Generalmente están ubicadas en mejillas y detrás de las orejas. Fotosensibilidad: Luego de la exposición al sol se genera la aparición de nuevas lesiones, la exacerbación de lesiones antiguas o el empeoramiento del proceso sistémico. Este fenómeno en mayor o menor grado ocurre en cerca de la mitad de los pacientes. Úlceras orales: Se pueden localizar en mucosa del paladar ginguival, labial e incluso lengua. Pueden ser dolorosas o asintomáticas. Ocurre en aproximadamente el 25% de los pacientes. Artritis no erosiva que comprometa 2 o + articulaciones periféricas: Manifestación muy frecuente, se encuentra desde el 53-95% según las distintas series. La gran mayoría son poliartritis no erosiva no deformante. Las articulaciones más afectadas son la IFP, rodillas, muñecas y MCF. Entre el 10-50% de los casos puede encontrarse deformaciones similares a AR pero no erosivas, fenómeno denominado Artropatía de Jaccoud. Serositis: Pleuritis (historia de dolor pleurítico o frote auscultado, o evidencia de derrame pleural) o pericarditis documentada por ECG o frote o evidencia de derrame pericárdico. Enf. renal: Proteinuria persistente mayor a 0,5 g/día o mayor de +++ si no hizo cuantificación o cilindros celulares (puede abarcar compromiso glomerular (lo más frecuente), túbulointersticial y vascular). Las anormalidades de orina van de hematuria asintomática a proteinuria en rango nefrótico. Se caracteriza por positividad de anticuerpo anti DNA y disminución del complemento. Enf. Neurológica: Convulsiones o psicosis en ausencia de medicamentos o condiciones que pudieran provocarla (las manifestaciones neurológicas no son extrañas en el LES, si bien tanto la psicosis como las convulsiones no son manifestaciones frecuentes, distintos grados de disfunción cognitiva pueden ocurrir en hasta el 66% de

8 Facultad de Medicina, Universidad de Chile







los pacientes con LES). Alt. hematológica: Anemia hemolítica con reticulocitosis o leucopenia menor 4.000 x mm³ o linfopenia menor de 1.500 x mm³ o trombocitopenia menor de 100.000 x mm³, pudiendo esta última ser la primera y única manifestación de la enfermedad (uno de los hallazgos más frecuentes, pero que no forma parte de los criterios diagnósticos, es la anemia normocítica normocrómica de enfermedad crónica). Alt. Inmunológica: Anti DNA (+) (presente en 40-70% de los LES, y generalmente se asocia a enfermedad renal con hipocomplementemia), o anti Sm (+) (sólo en 1530% de los pacientes con LES, pero es el anticuerpo dentro de los ENA, más específico) o prueba serológica para sífilis falso (+) por 6 meses como mínimo o anticuerpos antifosfolípidos (anticoagulante lúpico, anticardiolipina o anti β2-glicoproteína I). Anticuerpos antinucleares (ANA): Presentes en el 9599% de los pacientes con LES, pero de baja especificidad debido a su positividad en otras patologías reumatológicas (EMTC 100%, esclerodermia 60-90%, Sjögren 40-70%, artritis reumatoide 50-60% y en otras enfermedades no reumatológicas (hepatitis crónica, tiroiditis de Hashimoto y miastenia gravis). Existen nuevos criterios de LES publicados el año 2012 (criterios SLICC), los cuales aún no son de uso masivo, pero que cumplen con aumentar la sensibilidad mediante la consideración de manifestaciones antes no incluidas como la alopecia, la hipocomplementemia, alteraciones neurológicas como la mononeuritis múltiple y la mielitis. Requiere también 4 o más criterios para ser clasificado como LES, con al menos 1 criterio clínico y 1 inmunológico. Incluye además, la presencia de una biopsia renal compatible con nefropatía lúpica asociada a ANA o antiDNA (+) como criterio en sí mismo para el diagnóstico, sin necesidad de otras manifestaciones.

go de referencia en caso de prueba de ELISA). 3. Anti-Sm: Presencia de anticuerpos contra el antígeno Sm nuclear. 4. Anticuerpos antifosfolípidos positivos. 5. Complemento bajo. 6. Resultado positivo de Coombs directo.

Tratamiento Medidas generales como disminución de factores de riesgo cardiovascular y fotoprotección. • Corticoides son piedra angular en el tratamiento, colaborando con la resolución de las manifestaciones inflamatorias. Las altas dosis de prednisona o los pulsos de metilprednisolona, están reservadas para situaciones con compromiso de órganos vitales o riesgo de muerte. • Antimaláricos (hidroxicloroquina): Inmunomoduladores efectivos en síntomas leves y moderados. Resultan muy útiles en manifestaciones cutáneas y articulares, con efecto protector en nefropatía lúpica y de uso seguro y necesario en embarazo. • Inmunosupresores: Ciclofosfamida, azatioprina, micofenolato, utilizados ante compromiso orgánico severo.

Seguimiento Derivar a especialista

Autor / Editor Diego Vidal

Criterios clínicos: • Lupus cutáneo agudo, incluyendo lupus rash malar, lupus rash fotosensible, etc. • Lupus cutáneo crónico, incluyendo lupus discoide y otras. • Úlceras orales. • Alopecia. • Sinovitis involucran 2 o más articulaciones o sensibilidad en dos o más articulaciones y por lo menos con 30 minutos de rigidez matinal. • Serositis. • Compromiso renal: proteinuria (≥500 mg proteínas/24 horas) o hematuria. • Compromiso neurológico (por ejemplo, convulsiones, psicosis, neuritis, neuropatía, etc.). • Anemia hemolítica. • Leucopenia o linfopenia. • Trombocitopenia • Criterios inmunológicos 1. ANA nivel por encima del rango de referencia de laboratorio. 2. Anti-dsDNA de anticuerpos nivel por encima del rango de referencia de laboratorio (o mayor de 2 veces el ran-

9 Facultad de Medicina, Universidad de Chile

Año 2016



Artritis por cristales: la gota presenta episodios de inflamación severos, en que la máxima intensidad se alcanza en poco tiempo. Clásicamente se origina en la primera MTF (podagra). Se asocia a hiperuricemia y es típica en hombres. En el caso de la enfermedad por cristales de pirofosfato cálcico (pseudogota), la afección en su forma clásica, compromete rodilla. A diferencia de la gota, es más frecuente en mujeres añosas. Vale destacar que muchas veces, el cuadro clínico entre artritis séptica y artritis por cristales puede ser indistinguible. Una monoartritis de rodilla asociada a fiebre, leucocitosis y elevación marcada de parámetros inflamatorios, no permite hacer la diferencia; será el contexto clínico la clave para la sospecha diagnóstica: Dicha monoartritis en una paciente de 80 años que ocurre durante una hospitalización, tendrá como primera opción causal la pseudogota; mientras que la misma situación, pero en un paciente con bacteremia por S. aureus, tendrá como opción principal la artritis séptica. • Artritis traumática: monoartritis originada por traumatismos en articulación. • PEP: Tanto la artritis reactiva, artritis psoriásica, espondilitis anquilosante y artritis relacionada a EII pueden presentarse como monoartritis. Característicamente compromete extremidades inferiores. • Infección por hongos o mycobacterias: La característica fundamental de estos agentes es que pueden generar monoartritis crónica y poco dolorosa. Son infrecuentes y de difícil diagnóstico. El examen fundamental es el análisis del líquido sinovial, por lo que el dogma es: “Toda monoartritis debe puncionarse” (rodilla hinchada, rodilla pinchada). Valores de recuentos leucocitarios mayores a 50.000 células x mm3 y PMN > 95% son altamente sugerentes de artritis séptica. La presencia de cristales es un orientador al diagnóstico de artritis por cristales, pero siempre se debe considerar que la presencia de cristales no descarta necesariamente la artritis séptica. Además de la visualización de cristales, el recuento leucocitario debe estar entre 2.000 y 50.000 leucocitos x mm3. Si bien valores mayores son sospechosos de artritis séptica, no excluyen la primera posibilidad. La artritis por trauma suele presentar recuento leucocitario cercano a lo normal. El hemograma, VHS y PCR pueden ser orientadores de patologías inflamatorias o infecciosas.

Monoartritis Nivel de manejo del médico general: Diagnóstico Específico Tratamiento Inicial Seguimiento Derivar

Aspectos Esenciales • • • •

Inflamación de 1 articulación. Las 3 principales causas: séptica, cristales y trauma. El diagnóstico se realiza con examen de líquido sinovial (toda monoartritis se debe puncionar). En caso de sospecha de artritis séptica iniciar medidas terapéuticas de forma inmediata. Es una urgencia.

Caso Clínico Tipo Mujer de 70 años consulta por dolor de rodilla derecha desde hace 1 día. Al examen físico aumento de volumen y limitación de la movilidad articular en todos los rangos. Sin antecedentes de traumas previo. La punción articular evidenció 60.000 leucocitos, con cristales de pirofosfato cálcico.

Definición Compromiso articular inflamatorio limitado a 1 articulación. Se caracteriza por presentar dolor, aumento de volumen, calor y en ocasiones eritema. Cualquier trastorno articular puede iniciar como monoartritis, siendo de importancia descartar de forma temprana el diagnóstico de artritis séptica.

Etiología Las 3 causas más frecuentes a considerar son la artritis séptica, la artritis traumática y la artritis por cristales. Otras causas son la AR inicial, LES inicial (Aunque ambas evolucionan mayoritariamente a poliartritis), PEP, artritis por hongos o mycobacterias, Enfermedad de Lyme, sarcoidosis, tumores malignos (osteosarcoma, condrosarcoma), tumores benignos (hemangioma).

Tratamiento Según etiología (fundamental la punción articular). En el caso de la artritis séptica, el tratamiento debe ser inmediato con drenaje quirúrgico de la articulación y antibióticos endovenosos. Esto debido a la rápida progresión del compromiso articular. En el caso de la artritis por cristales, el tratamiento es con AINEs, colchicina o corticoides.

Diagnóstico •

Artritis séptica: inflamación articular con intenso dolor y aumento de volumen. Característicamente se acompaña de síntomas generales como fiebre y CEG. Al examen físico destaca restricción a la flexión, particularmente en rodiila. La principal causa en adultos jóvenes es la artritis gonocócica y luego S. aureus, en ocasiones, en contexto de infecciones protésicas.

Seguimiento Derivar a especialista.

10 Facultad de Medicina, Universidad de Chile

Autor / Editor Paulina Lira

Año 2016

Oligoartritis



Nivel de manejo del médico general: Diagnóstico Específico Tratamiento Inicial Seguimiento Derivar



Aspectos Esenciales • •



En la evaluación descartar artritis séptica y artritis por cristales mediante estudio de líquido sinovial. Evaluar posible patología sistémica acorde a sospecha.



Caso Clínico Tipo Paciente de 26 años, sexo masculino, hace 3 semanas presentó un cuadro diarreico agudo. Consulta por oligoartritis y ojo rojo doloroso.

Definición •

Compromiso articular inflamatorio de más de 1 articulación, pero menos de 5. Se caracteriza por aumento de volumen, calor y en ocasiones eritema.

cas distales. No es raro observar el compromiso axial y sacroilíaco asociado. En la fiebre reumática existe una oligoartritis de comienzo agudo, de extremidades inferiores y que característicamente es de tipo migratorio. Puede haber compromiso de piel, fiebre y nódulos subcutáneos. Enfermedad de Behcet: Se caracteriza por la presencia de úlceras orales, úlceras genitales, uveítis (muchas veces con hipopion), test de patergia (+), eritema nodoso, con artritis generalmente de extremidades inferiores. Sarcoidosis: Enfermedad granulomatosa caracterizada por eritema nodoso, adenopatías perihiliares, compromiso pulmonar intersticial y oligo o poliartritis,con afección principalmente de rodillas, tobillos y manos. Artritis por cristales: La gota, en casos crónicos, puede presentar afección de más de una articulación. La presencia del antecedente de ataques de gota o de tofos ayuda en el diagnóstico. La pseudogota puede debutar con baja frecuencia como cuadros oligoarticulares, pues su manifestación típica es monoarticular. Lo mismo ocurre con la artritis séptica, lo cual apoya el hecho de que no sólo las monoartritis deben ser siempre puncionadas, sino también las oligoartritis e incluso los poliartritis, siempre y cuando existan articulaciones grandes puncionables (rodillas, muñecas, tobillos, etc) y exista duda diagnóstica. Se debe realizar artrocentesis, si es posible, para descartar artritis séptica y por cristales. En caso de sospechar una PEP, debe estudiarse esqueleto axial si existe clínica sugerente de su compromiso.

Etiología

Tratamiento

Dentro de las causas clásicas está al grupo de las pelviespondilopatías (artritis reactiva, artropatíapsoriática, espondilitis anquilosante y artritis enteropáticas), las artritis por cristales (gota y pseudogota), la artritis séptica, la fiebre reumática, la enfermedad de Behcet, la sarcoidosis, la enfermedad de Lyme, entre otras. Vale decir, que en este grupo deben considerarse todas las causas de poliartritis como el LES y la AR, pues en ocasiones se inician como oligoartritis que evolucionarán con el tiempo a cuadros de compromiso poliarticular.

Derivación a especialista. El tratamiento de las artritis séptica y por cristales fue descrito en el capítulo respectivo.

Seguimiento Derivar.

Diagnóstico •



Autor / Editor Paulina Lira

Pelviespondilopatías, la artritis reactiva presenta el antecedente de infección, ya sea gastrointestinal o genitourinaria. Posterior a ello, se inicia la inflamación articular que generalmente es de extremidades inferiores, se acompaña de entesitis, dactilitis y puede manifestar tanto poli, oligo como monoartritis. Además, se observa compromiso extraarticular como conjuntivitis, uveítis y úlceras orales. Artropatía psoriática existe el antecedente de psoriasis en el 75% de los casos. Presenta un compromiso articular asimétrico, que puede ser tanto oligoartritis como poliartritis. Destaca la afección de articulaciones interfalángi-

11 Facultad de Medicina, Universidad de Chile

Año 2016

dosis ≥ 7.5 mg/día de prednisona en período > a 3 meses y menopausia precoz. Existen 2 tipos de osteoporosis: • Primaria: que puede ser senil (característica de hombres y mujeres de edad avanzada) o postmenopáusica (relacionada a hipoestrogenismo). • Secundaria a patologías concomitantes (hipertiroidismo, hiperparatiroidismo, hipercortisolismo, déficit GH, hipogonadismo, insuficiencia renal crónica), fármacos (corticoides, fenitoína, inhibidores de aromatasa, heparina), inmovilización.

Osteoporosis Nivel de manejo del médico general: Diagnóstico Específico Tratamiento Inicial Seguimiento Completo

Aspectos Esenciales • • • •

Enfermedad caracterizada por disminución de masa ósea con riesgo aumentado de fractura. Diagnóstico DSMO, utilizando T-score en pacientes postmenopáusicas. Estudiar causas secundarias en pacientes sin factores de riesgo. Tratamiento con calcio y Vitamina D.

Diagnóstico La historia clínica y examen físico no son suficientes para el diagnóstico de certeza. Es una enfermedad silenciosa y su diagnóstico sólo se hace evidente cuando el hueso está lo suficientemente débil como para generar fractura por trauma. Las fracturas vertebrales presentan dolor lumbar crónico y suelen presentarse después del esfuerzo, aunque la mayoría (2/3 de los casos) son asintomáticas y se sospechan por disminución de la estatura (alta probabilidad en pacientes que han disminuido más de 6 cm de estatura).

Caso Clínico Tipo Paciente de 70 años, fumadora y obesa, presenta fractura de cadera ante trauma menor. La radiografía evidencia osteopenia. La DSMO determina un T-score de - 3.0. Se indica tratamiento con calcio, vitamina D y bifosfonatos.

El examen definitivo para el diagnóstico es la densitometría ósea (DSMO), en donde valores de T-Score < -2,5 indican osteoporosis (si se asocia a fractura, se denomina osteoporosis severa), T-score entre -1 y -2,5 indica osteopenia y T-score > 1 es normal (T-score: valor de la DSMO comparado con valor promedio de sujetos jóvenes de 20-30 años). La DSMO con la evaluación del T-score está validada sólo para mujeres postmenopáusicas, por lo que en otros contextos, debe utilizarse el Z-score.

Definición Enfermedad ósea caracterizada por disminución de la masa y pérdida de la arquitectura del esqueleto generando deterioro en la resistencia y fortaleza del hueso, predisponiendo a mayor riesgo de fracturas. Las fracturas más frecuentes ocurren en vértebras y en extremidad distal del radio (fractura de Colles). El sistema óseo presenta alteraciones de la microarquitectura, de la conectividad ósea trabecular y adelgazamiento del hueso cortical.

El screening con DSMO está indicado en: mujeres > 65 años o menores de esa edad, pero con factores de riesgo, con fractura ante traumas de bajo impacto, osteopenia radiológica, tratamiento corticoidal crónico, presencia de factores predisponentes de osteoporosis secundaria y para monitoreo terapia en paciente con osteoporosis conocida.

Etiología-Epidemiología-Fisiopatología La remodelación ósea es constante. Los osteoclastos se encargan de la resorción ósea y los osteoblastos de la formación. En edades avanzadas, la formación ósea es ligeramente inferior a la resorción, lo que determina una baja gradual en la masa ósea. Esta se acumula en las 2 primeras décadas, alcanzando su peak entre los 20-40 años, estabilizándose por 2 décadas y luego declinando a una tasa de 0,1-0,5% por año. Las mujeres aceleran esta pérdida durante la menopausia debido a la disminución de los estrógenos. Tras el cese de la menstruación, durante 6-7 años pueden llegar a perder hasta 4% al año. Después de los 50 años de edad, el riesgo de fractura osteoporótica es de 40% en mujeres y 15% en hombres.

Tratamiento Modificación de estilo de vida: Suspensión de tabaco, restricción de consumo de alcohol, realización de actividad física (atenúa la pérdida ósea). Terapia con calcio y vitamina D: El requerimiento de Calcio en adultos >50 años es de 1.000-1.500 mg/día (1 taza de leche contiene aproximadamente 300 mg de calcio). El requerimiento de Vitamina D es 800-1.000 UI/día. Generalmente las formulaciones farmacológicas contienen 500 mg de calcio y 400 UI de vitamina D, por lo que la mayoría de las veces, el paciente requiere 2 comprimidos al día.

Factores de riesgo: Nulíparas, obesidad o bajo peso, consumo de OH, tabaco, uso de corticoides, específicamente el de

Bifosfonatos: Son fármacos que se depositan ávidamente en huesos en zonas de resorción, inhibiendo a osteoclastos, con

12 Facultad de Medicina, Universidad de Chile

una comprobada efectividad en la disminución del riesgo de fracturas por osteoporosis. El más utilizado es el alendronato a dosis de 70 mg/semanal vía oral. Los bifosfonatos deben ser administrados en ayuna, con abundante agua y el paciente debe permancer de pie durante al menos 30 minutos. Esta indicación se debe a que, de forma muy frecuente generan esofagitis, intolerancia digestiva o úlceras. En segunda instancia podría usarse Teriparatide, TRH, Ranelato de estroncio y calcitonina. Terapia de reemplazo hormonal: entrega beneficios a nivel óseo, pero no es la principal indicación de tratamiento.

Seguimiento Si DSMO inicial normal, control en 3 años. Si existe osteoporosis, puede controlarse el tratamiento con DSMO al año.

Autor / Editor Catalina Luvecce

Año 2016

13 Facultad de Medicina, Universidad de Chile

Diagnóstico

Síndrome De Túnel Carpiano

Clínica: Los síntomas más comunes son dolor y parestesias generalmente nocturnas, referidos a la zona de inervación del nervio mediano (cara palmar del pulgar, dedo medio, índice y parte del anular), pudiendo también generarse dolor en zonas más proximales, con irradiación que puede abarcar antebrazo, brazo e incluso hombro. Además de los cambios sensitivos al examen (hipoestesia y disminución de la capacidad de discriminación entre dos puntos) podemos encontrar alteraciones motoras como debilidad (especialmente a la abducción) y atrofia de los músculos de la región tenar. El signo de Tinel consiste en que la percusión del nervio mediano en el túnel del carpo reproduce el dolor y las parestesias en los sitios mencionados previamente, presentándose sólo en el 60% de los casos. La maniobra de Phalen consiste en realizar una flexión de la muñeca durante 1 minuto, lo cual reproduce la sintomatología y se encuentra presente en el 75% de los casos. El signo de Flick consiste en que el paciente sacude la mano afectada cuando se le pregunta qué hace al sentir los síntomas.

Nivel de manejo del médico general: Diagnóstico Específico Tratamiento Inicial Seguimiento Derivar

Aspectos Esenciales • • • •

Es la neuropatía por compresión más frecuente. Se asocia a tenosinovitis de los flexores del carpo, secundaria a movimientos repetidos. Se presenta con dolor y parestesias en zona de inervación del nervio mediano. El tratamiento inicialmente es médico, indicándose cirugía ante compromiso severo, refractario o motor.

Caso Clínico Tipo

Electromiografía: Necesario para confirmar la impresión clínica y para descartar una radiculopatía cervical como origen del dolor. Presenta una disminución de la velocidad de conducción sensitiva y motora del nervio mediano a través de la muñeca. Debe realizarse en pacientes sintomáticos al momento de la evaluación. Está alterada sólo en el 70% de los casos, por lo que un resultado normal no descarta el síndrome. La ecografía puede solicitarse en casos de duda dagnóstica, la cual puede evidenciar aumento del diámetro del nervio mediano.

Paciente de sexo femenino, 45 años, secretaria, con antecedentes de hipotiroidismo, consulta por cuadro de dolor nocturno y parestesias en cara palmar de dedo índice, medio y pulgar. Al examen físico Signos de Phalen y Tinel (+).

Definición Es el síndrome de compresión nerviosa más conocido. Corresponde a la compresión del nervio mediano en la muñeca en su paso entre los tendones del músculo flexor de los dedos en el interior del túnel del carpo.

Tratamiento En los casos leves, el evitar la actividad que genera las molestias, la fisioterapia, calor local y el uso de férulas que eviten la flexión total de la muñeca, constituyen medidas suficientes para la resolución de los síntomas. En ocasiones puede ser necesaria la inyección local de glucocorticoides o la administración de medicamentos neuromoduladores como tricíclicos, gabapentina, pregabalina, etc. En los casos severos, refractarios a tratamiento o con compromiso motor, es necesaria la intervención quirúrgica para liberar el nervio.

Etiología-Epidemiología-Fisiopatología Es la mononeuropatía periférica por atrapamiento más común. Presenta una relación 7:1 de mujeres con respecto a hombres y generalmente afecta a adultos entre 40 y 60 años. La principal causa es la tenosinovitis de los tendones de los músculos flexores de la mano, que genera una disminución del espacio en el interior del túnel del carpo (compartimiento inextensible) con la consecuente compresión del nervio mediano.

Seguimiento

La tenosinovitis generalmente se genera en asociación a movimientos repetidos y forzados de las manos, sin embargo, también puede presentarse en el contexto de patologías sistémicas tales como endocrinopatías (diabetes mellitus, acromegalia, hipotiroidismo), embarazo, neoplasias, patologías reumatológicas (artritis reumatoide), enfermedad renal crónica,amiloidosis, entre otras.

Derivar a especialista.

Autor / Editor Victor Valdes

14 Facultad de Medicina, Universidad de Chile

Año 2016

puncionarse. Dentro de los cuadros que afectan las bursas profundas, es importante destacar la bursitis trocantérea, isquioglútea y anserina. La bursitis trocantérea genera una patología que se caracteriza por dolor en trocánter mayor y por presentar una asociación frecuente con discrepancia de la longitud de los miembros inferiores, espondilosis lumbar y coxartrosis. Por otra parte la bursitis isquioglútea (ubicada entre la tuberosidad isquática y el glúteo mayor) es una de las causas de dolor y tumefacción en la región glútea, y se ha asociado a ocupaciones que causan fricción repetida sobre las tuberosidades isquiáticas, entre las que destaca el tejido manual artesanal. La bursitis anserina es una causa frecuente de dolor en la cara interna de la rodilla, siendo más frecuente su aparición en mujeres con sobrepeso y/o osteoartritis de rodilla. En las bursitis profundas, el origen séptico o por cristales no es relevante.

Tendinopatías y Bursitis Nivel de manejo del médico general: Diagnóstico Específico Tratamiento Inicial Seguimiento Completo

Aspectos Esenciales • • •

Se caracterizan por dolor local periarticular. Las bursitis superficiales deben puncionarse para descarte de infección o cristales. Buena respuesta a tratamiento médico.

Tendinopatías: Las tenosinovitis estenosantes consisten en la inflamación de la vaina tendínea, cuyo engrosamiento genera obstrucción del movimiento del tendón que transcurre a través de ella. Si la vaina afectada corresponde a un tendón flexor de los dedos, se produce el cuadro clínico denominado dedo en resorte o en gatillo, mientras que de ser afectada la vaina de los tendones extensor corto y abductor largo del pulgar, se genera el cuadro denominado tenosinovitis de Quervain. El síndrome epicondilar lateral (codo de tenista) y el medial (codo del golfista), son tratados en el capítulo respectivo. El tendón de Aquiles puede ser asiento de patología de origen no inflamatorio (por abuso de tendón) o de patología inflamatoria sistémica (entesitis en contexto de pelviespondilopatías, donde la zona afectada es el sitio de inserción del tendón). Se asocia a factores intrínsecos como alteraciones de la alineación y movilidad del antepie o del retropié, y a factores extrínsecos como ejercicios exagerados.

Caso Clínico Tipo Paciente hombre de 46 años de edad, acude a consulta por dolor y aumento de volumen en cara anterior de rótula. Dirigidamente refiere el día anterior haber estado de rodillas durante horas en una festividad religiosa. Al examen físico existe aumento de volumen prepatelar.

Definición Los tendones son estructuras de colágeno altamente organizadas y de gran fuerza tensora, cuya principal función es transmitir la fuerza muscular al hueso. Las tendinopatías (o tendinosis) son entidades en las cuales se generan alteraciones en la estructura de los tendones y se manifiestan con síntomas que dependen del tendón afectado. No es correcta la utilización del término tendinitis, pues las biopsias de tendones afectados muestran degeneración de fibras y no infiltrado inflamatorio.

Diagnóstico

Por otra parte, las bursas son sacos sinoviales cuya función es promover el deslizamiento de los tejidos adyacentes y actuar como amortiguadores. Existen bursas superficiales y profundas. Su inflamación se denomina bursitis.

Bursitis: • Bursitis olecraneana y bursitis prepatelar: Difiere según etiología: Traumática: Varía de una tumefacción fría e indolora de larga data a una distensión tensa con sensibilidad local en la zona afectada. Séptica: dolor agudo, tumefacción y enrojecimiento de la bursa. Dolor a la extensión-supinación del brazo o extensión de rodilla según el caso. Gota: similar a séptica. • Bursitis trocantérea: Dolor en la región lateral de la cadera asociado con la marcha, que se incrementa en decúbito lateral. Dolor a compresión del trocánter mayor. • Bursitis isquioglútea: Dolor localizado en el isquion en la posición sentada o acostada, más hipersensibilidad isquiática. • Bursitis anserina: Dolor en la cara interna de la rodilla e hipersensibilidad 3-5 cm distal a la interlinea articular. • Tendinopatía:

Etiología-Epidemiología-Fisiopatología Bursitis: Las bursas superficiales son la olecraneana y la prepatelar, que son homólogas anatómicamente y presentan una patología similar. La forma más común de bursitis superficial se origina en base a microtraumas repetidos en un contexto laboral o recreativo. La bursitis inicialmente descrita fue la rodilla de religiosa o mucama, por distensión de la bursaprepatelar, pero en la actualidad la bursitis olecraneana es más frecuente y se debe a la costumbre de apoyarse sobre los codos. Además de las causas traumáticas, pueden tener origen infeccioso o secundario a cristales (generalmente de urato monosódico), por ello las bursitis superficiales deben

15 Facultad de Medicina, Universidad de Chile







Tenosinovitis de Quervain: Dolor radial de la muñeca. Maniobra de Filkenstein: se coloca pulgar dentro del puño y se cubitaliza la muñeca. Es positiva cuando se reproduce el dolor. Dedo en gatillo o en resorte: Dolor palmar al flectar los dedos, en especial el pulgar, índice y medio con nódulo firme y doloroso próximo a la articulación metacarpofalángica, el cual interfiere en el deslizamiento del tendón, quedando el dedo bloqueado en flexión con resalte al forzar la extensión. Tendinitis Aquiliana: Presentación más común es con dolor tendíneo durante o después de la actividad física, pudiendo incluso comprometer la actividad cotidiana.

Tratamiento Bursitis: • Bursitis olecraneana y bursitis prepatelar: Traumática: Evitar el microtrauma repetido y esperar la resolución espontánea en un curso de 1-3 meses. Séptica: Tratamiento antibiótico y aspiración de bursa. Dependiendo del caso, bursectomía. Por cristales: Tratamiento análogo a artritis gotosa. • Bursitis trocantérea: AINEs, fisioterapia e infiltración con glucocorticoides. Cirugía en casos refractarios. • Bursitis isquioglútea: Cojín con orificio central, AINEs y medidas físicas. Infiltración con glucocorticoides en casos refractarios. • Bursitis anserina: Medidas físicas locales, AINEs, corrección del sobrepeso e infiltración con glucocorticoides. Tendinopatía: • Tenosinovitis de Quervain: Reposo relativo y AINEs. En casos refractarios infiltración con corticoides y si persiste, cirugía. • Dedo en gatillo o en resorte: Reposo relativo y AINEs. Infiltración con corticoides. Si recurre: Escisión quirúrgica de la vaina tendínea. • Tendinitis Aquiliana: Modificación de factores predisponentes, reposo deportivo, elevadores del talón, AINEs.

Seguimiento Completo en APS. Derivar en caso de sospecha de patología sistémica o refractariedad.

Autor / Editor Marcelo Fres

Año 2016

16 Facultad de Medicina, Universidad de Chile

Fisiopatología:

Vasculitis Sistémicas

Mecanismos de daño: • Elevación de IgE 1. Aumento de eosinófilos.

Nivel de manejo del médico general: Diagnóstico Sospecha Tratamiento Inicial Seguimiento Derivar

• • 1. 2.

Aspectos Esenciales • • •

Grupo heterogéneo de síndromes por inflamación de pared vascular. Siempre descartar causa secundaria (más frecuente). Biopsia certifica la vasculitis, de no poder realizarse es necesario ir a realizar imagen con angiografía.

Daño por complejos inmunes: Depósito de complejos inmunes subendoteliales con: Activación de complemento Atracción de PMN

• Daño directo por anticuerpos 1. ANCA (anticuerpos anticitoplasma de neutrófilos) - Anca-C (antiproteinasa 3) - Anca-P (anti mieloperoxidasa) 2. Anticuerpos anti-cel. endotelial

Caso Clínico Tipo Mujer de 73 años que consulta por cefalea intensa de predominio frontotemporal bilateral y episodios de amaurosis fugax, de 3 días de evolución. Trae un hemograma del día previo que es normal, pero tiene VHS de 78 mm 1ª hora. Al examen físico sólo destaca sensibilidad a la palpación de ambas regiones temporales y disminución de la amplitud del pulso en las arterias temporales.

• 1. 2. 3.

Por células: Activación de células T Formación de granulomas Secreción de citoquinas proinflamatorias.



Inducción del proceso por antígenos exógenos (infecciones, drogas, otros)

Clasificación:

Epidemiología

Primarias: Se basa en diámetro de los vasos predominantemente afectados, aunque puede existir traslape en el tamaño de las arterias afectadas, tenemos: • Vasculitis de vaso grande: 1. Arteritis de Takayasu 2. Arteritis de Células Gigantes (Arteritis de la temporal) • Vasculitis de vaso mediano 1. Poliarteritis nodosa 2. Enf. Kawasaki • Vasculitis de vaso pequeño 1. ANCA (+): - Poliangeítis Granulomatosa con Eosinofilia o EGPA (Schurg Strauss) - Granulomatosis con Poliangeítis o GPA (Wegener) - Poliangeitis Microscópica o MPA 2. ANCA (-) o Asociadas a inmunocomplejos: - Vasculitis leucocitoclasticas - Vasculitis mediada por IgA (Schönlein-Henoch) - Crioglobulinemia - Enfermedad mediada por Ac-Anti MB. Basal Glomerular (Good Pasture) • Vasculitis de vaso de tamaño variable

Existen pocos datos epidemiológicos, además los distintos tipos están asociados a diferentes poblaciones, por ejemplo, Kawasaki principalmente en niños, Takayasu, en mayores de 30 años, asi como la arteritis de la temporal se sospecha desde los 50 años, aumentando su incidencia en la 8° y 9° década de vida.

Secundarias: Asociadas a diferentes factores, como: • Drogas • Neoplasias • Infecciones • Mesenquimopatías

Definición Grupo heterogéneo de síndromes clínicos definidas por inflamación de las paredes vasculares. Las consecuentes rupturas de pared que conducen a sangrado y el compromiso del lumen vascular lleva a isquemia distal y necrosis en distintos territorios.

Etiología En general, los vasos afectados varían en tamaño, tipo y ubicación, asociado al distinto tipo de vasculitis resultan en variados síndromes. Las vasculitis pueden ocurrir como una enfermedad primaria o secundaria a una enfermedad subyacente. La etiología exacta es largamente desconocida. A menudo son enfermedades de cuidado e incluso fatales, por lo que requieren pesquisa y terapia precoz.

17 Facultad de Medicina, Universidad de Chile

Clasificación de Vasculitis según territorio vascular afectado

Diagnóstico: Clínica: Es importante destacar que dependiendo del tipo de vaso afectado y su localización, se van a desarrollar diferentes cuadros clínicos.

Manifestaciones clínicas de vasculitis según territorio vascular afectado

18 Facultad de Medicina, Universidad de Chile

A grandes rasgos, los síntomas asociados a este gran grupo de enfermedades corresponden a: • Enfermedad inflamatoria crónica: CEG, pérdida de peso, mialgias, artralgias, artritis, fiebre... • • •



• Mantenimiento de dicha remisión 1. Disminución gradual de la dosis de glucocorticoides, según tolerancia. 2. Glucocorticoides e inmunosupresores, se pueden continuar o suspender, según la condición, acorde a los protocolos de cada patología específica Piel: 3. Objetivos: Ulceras, púrpura, nodulos, rash... - Mantener controlada la actividad de la patología - Prevenir la recurrencia de la enfermedad Neuropatía: - Minimizar los riesgos de toxicidad Mononeuritis multiple • Monitoreo SNC: 1. De la actividad de la enfermedad Cefalea, cambios visuales, AVE, convulsiones, compromi2. De la toxicidad a fármacos durante el tratamiento activo so de conciencia... 3. De la recurrencia Renal: HTA, proteinuria, GNF necrotizante...

Seguimiento:



Tracto respiratorio: Derivar a especialista. Alveolitis, hemorragia alveolar, Infiltrados nódulos, asma, sinusitis...



Gastrointestinal: Diarrea, náuseas, vomitos, dolor abdominal, hemorraAutor / Editor Año gia... Valeska Lavanderos 2016 Claudicación muscular: En extremidades, en mandibula...



Al laboratorio se objetiva: • Alteraciones inespecíficas (anemia, VHS elevada, leucocitosis) • ANA, ANCA (P ó C) • Hipocomplementemia • Electromiografía • Deberá solicitarse siempre que haya alteraciones neurológicas periféricas y servirá para certificar el hallazgo de una mononeuritis múltiple y ayudará a obtener un buen sitio para biopsia. • Biopsia, y de estar contraindicada se debe realizar angiografía. • SIEMPRE descartar una causa secundaria inicialmente.

Tratamiento: Está basado en tres pilares: • Inducción de la remisión de la enfermedad, que involucra: 1. Dosis medianas-altas de glucocorticoides 2. Uso de inmunosupresores 3. La presentación inicial de las vasculitis a menudo es rápida y la demora en el reconocimiento de la enfermedad e inicio del tratamiento puede llevar a una importante morbilidad, y en algunos casos, mortalidad. Es por esto, que el tratamiento inicial suele ser más intensivo, que las etapas posteriores.

19 Facultad de Medicina, Universidad de Chile

titis B, parvovirus B19, estas también pueden ocurrir en contexto de endocarditis infecciosa o fiebre reumática. Si se subdivide por edad: lactantes (S. aureus, enterobacterias y streptococo grupo B); menor de 5 años (S. aureus, streptococo grupo A y H. influenzae si no ha sido vacunado); 16-40 años (gonococo si es sexualmente activo, S. aureus en caso contrario).

Artritis Séptica Nivel de manejo del médico general: Diagnóstico Sospecha Tratamiento Inicial Seguimiento Derivar

La vía de infección puede ser hematógena (más frecuente), directa (heridas, inyecciones intraarticulares o infección post quirúrgica) o por contigüidad (extensión de una osteomielitis, bursitis). Los factores predisponentes para adquirir esta infección son infecciones sistémicas, diabetes, inmunosupresión, uso de corticoides y drogadicción.

Aspectos Esenciales • • • • •

Es la inflamación de una o más articulaciones debido a infección. Su etiología varía de acuerdo a la forma de presentación clínica, siendo las monoartritis agudas bacterianas las más frecuentes. Clínicamente se caracterizan por dolor articular inflamatorio asociado a fiebre y síntomas generales. Es una emergencia médica por el riesgo potencial de desarrollar un shock séptico y la muerte. El tratamiento inicial es con antibióticos con cobertura para S. aureus.

Diagnóstico Clínica + punción articular (citoquímico, citológico, gram y cultivo) + identificación del agente (confirmatorio). Clínicamente se manifiesta por un cuadro de comienzo agudo o subagudo caracterizado por una artralgia, habitualmente monoarticular, que aumenta con los movimientos, asociada a aumento de volumen local, enrojecimiento de la piel, aumento de temperatura loca, impotencia funcional de la articulación comprometida y manifestaciones generales como síndrome febril, calofríos y compromiso del estado general. En el caso de lactantes, puede manifestarse como sepsis. En artritis gonocócica se describe una triada característica: artritis, dermatitis (papulopustulas diseminadas) y tenosinovitis.

Caso Clínico Tipo Un varón de 27 años acude a urgencias por dolor y signos inflamatorios en la rodilla derecha. No presenta fiebre y resto de la exploración era normal. Se practicó una artrocentesis. El análisis del líquído articular mostró: 52.000 células, 80% PMN, glucosa 27 mg/dL. En la tinción de Gram se observaron cocos gramnegativos.

Exámenes que se deben realizar ante la sospecha clínica de una artritis infecciosa: • Punción articular: el líquido en infecciones bacterianas agudas tiene las siguientes características: color blanco, un volumen >4 ml, leucocitosis entre 30.000-300.000/ mm³ con >90% de PMN. En caso de infecciones por Mycobacterias y hongos leucocitosis entre 10.000-30.000/ mm³ con 50 a 70% de PMN y 50 a 30% de mononucleares. • Radiografía articular: los signos radiológicos son tardíos (aparecen después de los 10 a 15 días). Entre estos destacan el aumento de partes blandas periarticulares, disminución del espacio articular (sólo se observa de forma tardía en contexto de artritis séptica erosiva, inicialmente puede haber aumento del espacio intraarticular), desmineralización ósea subcondral y epifisiaria, borramiento y posterior irregularidad del contorno articular, destrucción de las superficies articulares. • Identificación del agente etiológico: mediante análisis de frotis teñidos del líquido sinovial, hemocultivos y cultivos del líquido articular, y detección de ácidos nucleicos y proteínas microbianas por PCR y técnicas inmunológicas. Pueden realizarse hemocultivos y urocultivos y, en adultos sexualmente activos, cultivo en medio Thayer-Martin (frotis uretral, cervical, rectal o faríngeo buscando gonococo).

Definición Inflamación de una o más articulaciones debido a infección.

Etiología-Epidemiología-Fisiopatología Son más frecuentes en los niños y adolescentes. El 90% son monoartritis agudas. En hombre adulto joven se debe sospechar en primera instancia artritis gonocócica. La ubicación más frecuente son las rodillas (niños y adolescentes), caderas (lactantes y niños menores) y hombro. La etiología varía de acuerdo a la forma de presentación clínica. En caso de una mono u oligoartritis aguda la causa más frecuente es bacteriana, por Staphylococco aureus, neumococo, bacilos gram negativos, gonococo y otras. En caso de una mono u oligoartritis con evolución subaguda o crónica la causa más frecuente son Mycobacterias y hongos, pero con evolución en brotes inflamatorios las causas más frecuentes son Treponema pallidum (sífilis) y Borrelia burgdorferi (enfermedad de Lyme). Por último, en caso de poliartritis las causas más frecuentes son bacterianas donde destaca la Neisseria meningititis y gonorrhoeae y virales donde destaca la hepa-

20 Facultad de Medicina, Universidad de Chile

Tratamiento Debe ser de urgencia, puesto que existe el riesgo de que se produzca un shock séptico y eventualmente la muerte, y consiste en: • Tratamiento antibiótico inicialmente empírico de amplio espectro y con cobertura para Staphylococus aureus meticilino resistente (vancomicina + betalactámico) y se espera confirmación de agente etiológico para partir con tratamiento específico. Si se confirma artritis gonocócica se debe tratar con ceftriaxona 1-2 g/día ev, también se puede usar ciprofloxacino. • Drenaje del exudado purulento por artrotomía, punciones articulares repetidas o artroscopías, dejando artroclisis. • Inmovilización de la articulación, reposo y rehabilitación.

Seguimiento Derivar a especialista.

Autor / Editor Paulina Lira

Año 2016

21 Facultad de Medicina, Universidad de Chile

(cerveza), enfermedades crónicas como la obesidad, HTA, DM, dislipidemia, aterosclerosis, fármacos como diuréticos (hidroclorotiazida), levodopa, AAS en dosis altas, etc.

Gota Aguda Nivel de manejo del médico general: Diagnóstico Sospecha Tratamiento Completo Seguimiento Derivar

Diagnóstico a) Gota aguda: Dolor agudo e intenso nocturno o matinal asociado a calor, hipersensibilidad e inflamación. Habitualmente compromete una sola articulación, la más frecuente es la primera articulación metatarsofalángica (podagra), no obstante puede presentarse de forma poliarticular en los episodios subsiguientes. Los episodios pueden ser precipitados por enfermedades graves, alcohol, trauma o ayuno prolongado. Al principio pueden ser crisis autolimitadas que ceden luego de 3 - 10 días, luego se hacen más frecuentes e intensas. Si el tratamiento no es oportuno puede llegar a ser una artropatía crónica invalidante.

Aspectos Esenciales • • • • • •

Depósito de cristales urato monosódico. Mayor en hombres pasado edad media. Crisis precipitadas por fármacos, excesos dieta y alcohol. Podagra y tofos. Cristales con birrefringencia (-) a la luz. Tratamiento agudo: nunca dar alopurinol en crisis.

Caso Clínico Tipo

b) Período intercrítico: Puede ser asintomático y a largo plazo dar gota poliarticular crónica generalmente asimétrica con compromiso de bursas subdeltoideas y olecraneanas, tendón de Aquiles y zonas periarticulares. Se debe descartar artritis reumatoide con radiografías y factor reumatoideo.

Hombre de 50 años con antecedentes de hipertensión arterial y dislipidemia. Día posterior a un asado presenta dolor agudo en primer ortejo, asociado a inflamación. Refiere haber presentado episodio similar una vez antes, sin haber consultado. En laboratorio presenta leucocitosis con aumento VHS, se punciona ortejo que muestra líquido con cristales de birrefringencia (-).

c) Gota tofácea crónica: En casos de diagnóstico tardío o tratamiento inadecuado, se acumulan tofos en olecranon, hélix-antihélix de oreja, superficie cubital del antebrazo y tendón calcaneo. En esta etapa aparece el compromiso renal que puede asociarse a litiasis. Exámenes: solicitar uricemia, BUN, creatininemia, perfil lipídico, orina completa. Hemograma con leucocitosis y aumento de VHS. Si se punciona articulación (diagnóstico diferencial con artritis séptica) se puede evidenciar la presencia de cristales con birrefringencia (-) a la luz polarizada.

Definición Enfermedad metabólica de prevalencia familiar caracterizada por crisis de artritis por cristales de urato monosódico, que se pueden acumular en leucocitos de líquido sinovial, articulaciones, cartílagos, tendones, tejido celular subcutáneo (tofos). En algunos casos crónicos puede existir compromiso de función renal por acumulación de cristales en túbulos colectores y uréteres.

Tratamiento Medidas generales: disminuir consumo de carnes rojas, mariscos y alcohol, bajar de peso si es necesario. Evitar otros factores desencadenantes. a) Crisis aguda: • AINES dosis máxima vía oral: naproxeno 500 mg c/12 hrs, piroxicam 20 mg/día, diclofenaco 50 mg c/8 hrs, celecoxib 200 mg o rofecoxib 25 mg por 1-2 veces/día. Tratamiento al menos por 10 días sin crisis. • En contraindicación absoluta a AINES: colchicina 1-2 mg vía oral de carga inicial, luego 0,5 mg c/1 hr (dosis máxima 5 mg/día), RAM: gastrointestinales (lo más frecuente, destaca diarrea), desequilibrio hidroelectrolítico, mielosupresión. • En contraindicación de AINES (hemorragia digestiva o terapia anticoagulante) o colchicina (insuficiencia hepática, renal): corticoides orales (ej. prednisolona 30 mg/ día por 5 días), intramusculares o intraarticulares (betametasona 4 mg/dl o metilprednisolona 20-40 mg) han

Etiología-Epidemiología-Fisiopatología Es más común en hombres y a mayor edad. El ácido úrico es el producto final del catabolismo de las purinas. Su concentración sérica normal es entre 3,5 y 6 mg/dl. Los límites superiores fluctúan entre 6,9 a 7,5 en hombres y 5,7 a 6,6 en mujeres. La hiperuricemia es prerrequisito de la gota, en la cual existe precipitación de estos cristales en las articulaciones y otros tejidos. Esto produce inflamación que se manifiesta en las crisis agudas. Factores de riesgo: defectos enzimáticos del metabolismo de ácido úrico (síntesis y excreción), aumento de síntesis de ácido úrico en enfermedades mielo y linfoproliferativas como mieloma múltiple, tumores, anemia hemolítica o perniciosa, psoriasis, etc, defecto de excreción renal de ácido úrico, excesos en dieta (carnes, mariscos, fructosa, etc) y alcohol

22 Facultad de Medicina, Universidad de Chile



demostrado similares resultados (por especialista). Nunca usar alopurinol en crisis aguda porque las exacerba. Tampoco se debe suspender si el paciente ya lo estaba recibiendo previo a la crisis.

b) Tratamiento intercrisis: Sólo cuando son repetidas, hiperuricemia mayor a 9 mg. • Alopurinol 100-300 mg/día. Disminuye el ácido úrico sintetizado bloqueando la xantino oxidasa, se debe corregir por función renal (riesgo de necrolisis, agranulocitosis, hepatitis, vasculitis). • Probenecid 0,5 g/día. En uricosuria disminuida (menor a 600 mg en 24 hrs), aumenta la excreción renal. RAM: nefrolitiasis (alcalinizar orina y aumentar ingesta de líquidos). No se aconseja uso en APS. • IECA, ARA II y fibratos: tienen acción uricosúrica, por lo que son de elección en manejo de comorbilidades (hipertensión arterial y dislipidemia).

Seguimiento Derivar a especialista.

Autor / Editor Cristian Aguirre

Año 2016

23 Facultad de Medicina, Universidad de Chile

Diagnóstico

Poliartritis

En la historia clínica deben considerarse los siguientes aspectos: • Forma de inicio y evolución: Las poliartritis crónica pueden tener una evolución intermitente (artritis por cristales, enfermedad de Lyme, laxitud ligamentosa, etc) o progresiva. Además el patrón de compromiso puede ser migratorio (artritis virales, infecciones por Neisseria, enfermedad de Lyme) o aditivas. • Síntomas y signos articulares: La presencia de rigidez articular prolongada matinal orienta a artritis reumatoide, elevación de la temperatura local, eritema y derrame orienta a patología infecciosa o inflamatoria. • Síntomas sistémicos: La presencia de fiebre, baja de peso, sudoración nocturna orientan a enfermedades de tejido conectivo (vasculitis, LES, AR), infeciosas o neoplásicas. • Síntomas y signos sistémicos de enfermedades del tejido conectivo: Fotosensibilidad, eritema facial, alopecia, úlceras orales, compromiso cutaneo, renal, del SNC o polineuropatías, pleuropericarditis.

Nivel de manejo del médico general: Diagnóstico Sospecha Tratamiento Inicial Seguimiento Completo

Aspectos Esenciales • • •

Poliartritis < 6 semanas: Sospechar artritis virales. Poliartritis ≥ 6 semanas: Sospechar AR, LES o Sjögren, entre otras. Si bien la serología reumatológica es útil para el diagnóstico, debe tenerse en cuenta que algunas artritis virales pueden mostrar ANA y FR (+), de manera transitoria.

Caso Clínico Tipo Mujer de 42 años refiere poliartritis de manos (MCF, IFP), muñecas y rodillas, de 2 semanas de evolución. Tiene a su hijo de 2 años enfermo (le dijeron que se trataba de una infección viral). Ha presentado exantema eritematoso de tronco. Tiene FR y ANA (+), además de IgM para parvovirus (+). Se da tratamiento sintomático y en 2 semanas, la paciente está sin artritis y su serología es negativa.

Según etiología: • Artritis virales: Considerada la causa más frecuente de poliartritis aguda, puede ser generada por virus como VHB, Parvovirus B19, VIH, CMV, VEB y rubeola. Puede ser simétrica o asimétrica. En muchos casos, puede acompañarse de fiebre y exantema. En el caso del Parvovirus B19, es fundamental el dato anamnéstico del contacto con niños enfermos. Generalmente presentan remisión espontánea y debe tenerse en cuenta que el fenómeno inmunológico provocado por algunos virus, puede generar positividad de las pruebas reumatológicas (especialmente ANA y FR), la cual tiende a ser transitoria y puede transformarse en un problema diagnóstico si no se sigue la evolución del paciente luego de las 6 semanas.

Definición Compromiso articular inflamatorio de 5 o más articulaciones. La clasificación más útil para esta entidad es aquella que considera el patrón temporal: Poliartritis de < 6 semanas de evolución (agudas) y poliartritis de ≥ 6 semanas (crónicas).

Etiología Las causas más importantes de poliartritis aguda corresponden a las artritis virales, mientras que en las crónicas, destacan las clásicas patologías reumatológicas autoinmune, esto es: artritis reumatoídea, lupus eritematoso sistémico y síndrome de Sjögren. Con menor frecuencia pueden manifestarse así, algunas formas de artropatía psoriática, poli y dermatomiositis, esclerodermia y artritis por cristales (en especial en enfermedad de larga data). La enfermedad de Still es otra causa a considerar como parte del diagnóstico diferencial. Se debe considerar además que un hasta un 19% de las artritis bacterianas pueden presentarse como poliarticulares, especialmente en el caso de pacientes inmunocomprometidos, con diabetes mellitus o enfermedad renal crónica. Es importante recalcar que toda poliartritis crónica que se evalúa antes de la sexta semana de evolución será considerada en ese momento como aguda, por lo que el seguimiento después de las 6 semanas es fundamental para catalogarla.



Artritis reumatoídea: Poliartritis erosiva, deformante y simétrica con compromiso característico de manos, específicamente MCF e IFP. Puede ser aditiva y se acompaña de síntomas extraarticulares.



Lupus eritematoso sistémico: Poliartritis no erosiva y no deformante que puede ser muysimiliar a la AR en su distribución. Presenta manifestaciones extraarticulares con mucha mayor frecuencia que AR, destacando el compromiso renal y cutáneo.



Artropatía psoriática: En la mayoría de los casos, asociada a psoriasis cutánea, pero hasta un 15% de los pacientes no la presenta al momento de la evaluación. En la gran mayoría de los casos es asimétrica y en ocasiones con compromiso axial característico de las PEP. A diferencia de AR, en la artropatíapsoriática las IFD pueden estar comprometidas. En ocasiones puede ser similar a AR en distribución, diferenciándose por marcadores inmunológicos propios de esta última (FR, AntiCCP).

24 Facultad de Medicina, Universidad de Chile



Artritis por cristales: tanto la gota como la pseudogota pueden al largo plazo presentar clínica de poliartritis.



Enfermedad de Still: Cursa con poliartritis, fiebre, odinofagia, a veces adenopatías y un característico exantema asalmonado evanescente. La serología reumatológica es negativa y destaca al laboratorio, niveles de ferritina muy elevados (generalmente sobre 1.000 ng/mL)

Dentro de los exámenes, se solicita hemograma, VHS y PCR evidencian un aumento de parámetros inflamatorios, pero no ayudará a identificar la causa. El FR y Anti CCP son útiles en el diagnóstico de AR, por lo que en un cuadro de poliartritis simétrica con disposición característica debiese realizarse estos exámenes. ANA y complemento bajo orientan a LES. Según sospecha, debe solicitarse serología viral.

Tratamiento Sintomático con AINES o corticoides hasta determinar la causa, con lo cual puede darse tratamiento específico.

Seguimiento Derivar a especialista.

Autor / Editor Paulina Lira

Año 2016

25 Facultad de Medicina, Universidad de Chile

Diagnóstico

Síndrome De Sjögren

Clínica: Los síntomas clásicos son xeroftalmia, xerostomía (los cuales en conjunto se denominan síntomas sicca) y artralgias/itis. Puede encontrarse hipertrofia parotídea hasta en el 80% de los casos. La xerostomía es el síntoma más frecuente.

Nivel de manejo del médico general: Diagnóstico Sospecha Tratamiento Inicial Seguimiento Derivar

Criterios clínicos (4 de 6, excluyendo otras causas): 1. Síntomas oculares (1 o +): molestias diarias y persistentes de sequedad ocular por 3 meses o sensación de arenilla en los ojos o necesidad de utilizar lágrimas artificiales > 3 veces por día. 2. Síntomas orales (1 o +): Sensación diaria de sequedad bucal por 3 meses o tumefacción de alguna glándula salival de forma recurrente o persistente o necesidad de tomar líquidos durante las comidas. El paciente se puede quejar de dificultad para comer alimentos secos, dificultad para dar un discurso, aumentos de caries. 3. Signos oculares (1 o +): Test Schirmer < 5 mm o prueba rosa de Bengala > 4. 4. Afección de glándulas salivales (1 o + ): Alteración en gammagrafía salival o sialografía parotídea o flujo salival no estimulado en <15 minutos. 5. Datos histopatológicos: Biopsia concordante. 6. Anticuerpos: ANA (40-70%), Anti-Ro (60-80%), Anti-LA (50%), Factor reumatoide (70-90%).

Aspectos Esenciales • • • •

Afecta a glándulas exocrinas, en especial salivales y lagrimales. Compromiso extraglandular más frecuente: articular y tiroídeo. Alta asociación a otras enfermedades reumatológicas. Síntomas característicos: xeroftalmia, xerostomía, artralgias/itis.

Caso Clínico Tipo Paciente sexo femenino de 51 años, refiere sensación de sequedad bucal, necesidad de beber líquidos constantemente para poder tragar alimentos, sensación de arenilla en los ojos y poliartralgias.

NOTA: Se pueden comprometer glándulas de todo el organismo generándose atrofia mucosa gástrica, mayor predisposición a infecciones respiratorias, dispareunia, pancreatitis subclínica. Compromiso extraglandular: Son propias del Sd. Sjogren primaria y rara vez ocurren en el secundario. La afección más frecuente es la tiroídea(tiroiditis de Hashimoto). Es también frecuente encontrar artritis, la cual no es erosiva, a excepción del Sd. Sjögren secundario a una AR, donde esta última es la que comanda la clínica articular. A nivel hepático, entre el 40-75% de los pacientes con cirrosis biliar primaria presentan Sd. Sjögren asociado. A nivel renal, el compromiso más frecuente es la nefritis intersticial, expresado como una acidosis tubular distal, en ocasiones con hipokalemia severa. A nivel pulmonar, puede encontrarse un infiltrado intersiticial linfocítico que tiende a ser subclínico. El fenómeno de Raynaud aparece en un 30% de los pacientes con Sd. Sjogren primario. Por su implicancia pronóstica, es importante conocer que los pacientes con Sd. de Sjögren primario presentan 44 veces más riesgo de desarrollar un linfoma (no Hodgkin de células B). Esta última condición debe ser descartada en caso de tumefacción parotídea prolongada o la aparición de múltiples adenopatías mediastínicas.

Definición Enfermedad autoinmune sistémica caracterizada por disfunción progresiva de glándulas exocrinas (exocrinopatía autoinmune) debida a infiltración de linfocitos CD4 asociada a manifestaciones sistémicas extraglandulares. Muy amenudo ocasiona sequedad bucal y ocular sintomática. Puede además presentar alteraciones extraglandulares. Puede ser primario o secundario al presentarse asociado a otras patologías autoinmunes.

Etiología-Epidemiología-Fisiopatología Se presenta entre el 0,1-0,6% de la población, siendo un 90% de los pacientes de sexo femenino, generalmente entre 45 y 55 años. Existen 2 formas clásicas reconocidas para el Sd.Sjögren: el Sjögren primario, que corresponde a la manifestación clásica del síndrome con afección glandular (fundamentalmente lagrimales y salivales) y extraglandular característica, y el Sjögren secundario, en el cual las características propias del síndrome se encuentran asociada a otras enfermedades reumatológicas, especialmente artritis reumatoide y en segundo lugar lupus eritematoso sistémico (hasta el 25% de los pacientes con estas patologías presentan un Sjögren secundario).

Laboratorio: es frecuente la elevación de la VHS y anemia de trastornos crónicos. El FR suele estar elevado en el 80% de los casos. Los anticuerpos más específicos son los antiRO y antiLA que se encuentras positivos en el 60% y 50% de los casos respectivamente. Se presentan con mayor frecuencia en el Sd. Sjogren primario y su positividad se asocia a enfermedad de inicio más precoz, con mayor compromiso extra-

26 Facultad de Medicina, Universidad de Chile

glandular. Es relevante hacer el diagnóstico diferencial con el síndrome SICCA del paciente VIH. Recordar que la proporción hombre:mujer es de 1:9 en el caso del Sd. Sjögren por lo que sospecharlo en hombres requiere necesariamente descartar VIH.

Tratamiento Sintomático con hidratación de mucosas orales y manejo de la xeroftalmia (gotas artificiales, lubricantes oftálmicos, gotas de ciclosporina). La pilocarpina puede ayudar a las manifestaciones de sequedad. Los glucocorticoides no son eficaces para los síntomas de sequedad, pero pueden ser útiles en el manejo de las manifestaciones extraglandulares. El uso de hidroxicloroquina puede ser util en el manejo de las manifestaiones articulares.

Seguimiento Derivar a especialista.

Autor / Editor Victor Valdes

Año 2016

27 Facultad de Medicina, Universidad de Chile

travascular, que puede ser tanto venosa como arterial (a diferencia de las trombofilias hereditarias, en las cuales las trombosis son casi exclusivamente venosas). Se pueden ver comprometidos vasos sanguíneos de cualquier tamaño, desde capilares a grandes arterias y sin fenómenos inflamatorios asociados (sin vasculitis). La afección de vasos pequeños suele manifestarse como microangiopatía trombótica. Si dicho fenómeno se extrapola a los vasos sanguíneos placentarios, se entiende por qué esta patología se asocia a abortos a repetición, parto prematuro y preeclampsia.

Síndrome Antifosfolípido (SAAF) Nivel de manejo del médico general: Diagnóstico Sospecha Tratamiento Inicial Seguimiento Derivar

Aspectos Esenciales • • • •

Una de las trombofilias adquiridas más frecuente. Afecta principalmente a mujeres, muchas veces asociado a LES. Se caracteriza por abortos a repetición o episodios de trombosis vasculares tanto arteriales como venosas. El tratamiento es con AAS y TACO dependiendo del caso.

Diagnóstico Para establecer el diagnóstico se requiere un criterio clínico + un criterio de laboratorio. Criterios clínicos: • Trombosis vascular sin vasculitis (1 o + ya sea arterial, venosa o de pequeños vasos). • Morbilidad durante el embarazo (1 o + muertes fetales en fetos > 10 semanas o 1 o + partos prematuros > 34 semanas debido a eclampsia, preeclampsia o insuficiencia placentaria o 3 o + pérdidas fetales en < 10 semanas). Debe sospecharse en pacientes jóvenes o sin factores de riesgo cardiovasculares que desarrollan accidentes vasculares o infartos miocárdicos.

Caso Clínico Tipo Paciente de sexo femenino 36 años, con historia de abortos recurrentes. Consulta en Urgencias por hemiplejia facio-braquio-crural izquierda. Anticuerpos anticardiolipinas (+) IgM e IgG a títulos altos.

Definición

Criterios de laboratorio: • Anticoagulante lúpico (+) en plasma en 2 o más ocasiones separado por 12 semanas. • Anticardiolipina IgM o IgG por ELISA en títulos medios o altos (> percentil 99) en 2 ocasiones separados por 12 semanas. • Anti β2 glicoproteína I IgG o IgM por ELISA en títulos medios o altos (> percentil 99) en 2 ocasiones separado por 12 semanas.

Enfermedad caracterizada por la presencia de eventos clínicos trombóticos y por la persistente presencia de anticuerpos específicos (Anticardiolipina, Anti β2-glicoproteína I). Es una de las trombofilias adquiridas más frecuentes. Puede ocurrir de manera primaria o secundaria a otras patologías autoinmunes. Se asocia a abortos espontáneos recurrentes en mujeres de edad fértil.

En relación a los anticuerpos anticardiolipinas, aquellos IgG tienen mayor peso diagnóstico que los IgM, los cuales pueden elevarse en infecciones y neoplasias. Su presencia explica el VDRL falsamente (+) en muchos de estos pacientes. El requisito de la persistencia de éstos por 12 semanas es fundamental para dar valor diagnóstico a dicha prueba, al igual que para la β2 glicoproteína I, autoanticuerpo más sensible y específico. El anticoagulante lúpico no es un autoanticuerpo, sino una prueba funcional que genera in vitro anticoagulación, pero que in vivo tiene el efecto contrario, si bien es poco sensible, su presencia es de alta especificidad para SAAF. Otras manifestaciones clínicas del SAAF son: lívedo reticularis, anemia hemolítica y trombocitopenia (en rango leve).

Etiología-Epidemiología-Fisiopatología En personas sanas, la presencia de anticuerpos anticardiolipinas a títulos bajos, ocurre en cerca del 10%, generalmente de forma transitoria. Títulos moderados y altos o la presencia de anticoagulante lúpico, se observa en menos del 1 %. De los pacientes con LES, el 10-40% presentan anticuerpos antifosfolípidos, mientras que en AR, menos del 20%. Esta situación implica que su sola presencia no da el sello a la enfermedad, sino que su conjunción con aspectos clínicos. El SAAF afecta principalmente a mujeres de edad fértil. Puede dividirse en primario y secundario, con similares característica clínicas, pero con la diferencia en que el SAAF secundario se asocia a otra patología autoinmune, generalmente LES. En ocasiones el SAAF es la primera manifestación del LES, por lo tanto, para definir el diagnóstico de SAAF primario se requiere un seguimiento del paciente en el tiempo.

Tratamiento Profilaxis primaria con aspirina y profilaxis secundaria con TACO, con objetivo de mantener un INR de 2,5 a 3,5. En embarazadas se ha mejorado el pronóstico con el uso de acido

El aspecto clínico central del síndrome es la trombosis in-

28 Facultad de Medicina, Universidad de Chile

acetilsalicílico 80 mg y heparina (sin tratamiento, el 80% de los embarazos no llega a término). Las inmunoglobulinas IV también mejoran el pronóstico en el embarazo, mientras que el uso de corticoides en estos casos no son eficaces. En casos severos (SAAF catastrófico), se utilizan corticoides y plasmaféresis.

Seguimiento Derivar a especialista.

Autor / Editor Paulina Lira

Año 2016

29 Facultad de Medicina, Universidad de Chile

impotencia funcional y fiebre. Sin embargo la expresión clínica en ocasiones en bastante discreta, presentando una evolución afebril, lo que contribuye a un diagnóstico tardío. Suele haber un evento médico o infeccioso previo que actúa como desencadenante y el germen causal más aislado es el S aureus. En este sentido, cualquier paciente que presenta una bacteremia por dicho germen y que evoluciona con lumbalgia, debe ser estudiado por la posibilidad de una espondilodiscitis. Para dicho diagnóstico es fundamental realizar hemocultivos y estudio por imágenes. En el caso de presentar el antecedente de contacto con TBC o inmunosupresión, sospechar una TBC vertebral o mal de Pott. Sin embargo, debe recalcarse que el compromiso de esta patología suele ser predominantemente cervical o dorsal más que lumbar.

Lumbago Infeccioso y Tumoral Nivel de manejo del médico general: Diagnóstico Sospecha Tratamiento Inicial Seguimiento Derivar

Aspectos Esenciales • • •

Sospecha en paciente añoso con dolor nocturno y refractario a tratamiento. Sospechar metástasis ósea en pacientes con antecedentes de cáncer prostático, tiroídeo, mamario , pulmonar. Cintigrama óseo y resonancia magnética son herramientas claves para el diagnóstico.

Imagenología: Las alteraciones radiográficas son siempre tardías, por lo que las imágenes por resonancia magnética pasan a ser una herramienta fundamental para la pesquisa precoz de la espondilodiscitis, además de la determinación de su extensión y posibles complicaciones relacionadas a la ocupación del espacio raquídeo. La medicina nuclear, a través del cintigrama óseo puede ser una aproximación inicial muy útil para la sospecha diagnóstica, con la ventaja que permite realizar un estudio posterior con resonancia localizado en la zona hipercaptante. En relación al lumbago tumoral, la característica de la imagen en relación a la presencia de una lesión lítica vs una blástica, puede determinar una aproximación diagnóstica valiosa.

Caso Clínico Tipo Paciente masculino de 72 años, consulta por dolor en zona lumbosacra de 3 meses de evolución, de carácter sordo, EVA 5/10 que es predominantemente nocturno. Dirigidamente refiere síntomas urinarios obstructivos y no ha tenido evaluación urológica.

Definición Dolor lumbar de origen infeccioso o tumoral. Característicamente presenta dolor persistente, que no alivia con el reposo, rigidez intensa, y asociado a signos de compromiso de estado general.

Tratamiento

El lumbago infeccioso es predominantemente bacteriano (cocos gram (+), bacilos gram (-) y en mucha menor medida TBC) y el lumbago tumoral es predominantemente secundario a metástasis prostáticas, tiroídeas, mamarias y pulmonares. Los tumores primarios son menos frecuentes. No olvidar mieloma múltiple.

Las infecciones en la columna lumbar son graves. El tratamiento antibiótico debe ser iniciado precozmente, previa toma de cultivos, con un esquema antibiótico intravenoso de amplio espectro que cubra gram (+) y (-). El esquema antibiótico será ajustado según los resultados bacteriológicos y mantenido por un mínimo de 3 semanas intravenoso y similar periodo vía oral. La cirugía se considera en los casos en que la infección compromete el canal raquídeo y produce compresión de estructuras neurales vecinas, generando déficit motor y sensitivo de extremidades o de la función vesical por síndrome de cola de caballo.

Diagnóstico

Seguimiento

Clínica: La sospecha de compromiso infeccioso o tumoral de la zona lumbosacra se hará presente frente a la presencia de una o más de las clásicas banderas rojas del lumbago: primer episodio en pacientes > 50 años, refractariedad a tratamiento apropiado, lumbago sin alivio con reposo, de predominio nocturno y con síntomas asociados como fiebre, anemia, anorexia y baja de peso. La espondilodiscitis infecciosa se debe a la llegada de gérmenes al espacio discal, lo que genera dolor axial, rigidez,

Derivación a especialista.

Etiología

Autor / Editor Paulina Lira

30 Facultad de Medicina, Universidad de Chile

Año 2016

BIBLIOGRAFÍA



Carlos Rodríguez Pago. Síndrome de túnel carpiano. Guías clínicas - Reumatología, tema 4. Fisterra.



Tabares Neyra, Horacio, Díaz Quesada, Juan Miguel, Tabares Sáez, Horacio, & Tabares Sáez, Laura. (2016). Tratamiento quirúrgico del síndrome de túnel del carpo en adultos mayores. Revista Cubana de Ortopedia y Traumatología, 30(1), 40-52.



Dr. Agustín Pecho Vega. Síndromes dolorosos del codo, antebrazo y mano. Cirugía ortopédica y traumatología. 2000.

Derivation and validation of the Systemic Lupus International Collaborating Clinics classification criteria for systemic lupus erythematosus. Arthritis Rheum. 2012 Aug;64(8):2677-86.



El Ultrasonido en el Estudio del Síndrome de Codo Doloroso. Marcelo Audisio, Marta Aliste. Rev. chil. reumatol. 2011; 27(1):31-37



Otros trastornos musculoesqueléticos - Capítulo 176. Harrison. Manual de medicina. 18º edición. Editorial McGraw Hill.



Maria Cinta Cid Xutglà, Roser Solans. Avances en vasculitis sistémicas. 1° edición, 2012. Marge Médica Books.



Dra. R. Sequeira González; Dr. R. Navarro Navarro; Dr. J.A. Ruíz Caballero; Dr. J. F. Jiménez Díaz; Dra. E. Brito Ojeda. Hombro doloroso. 23° Jornadas Canarias de Traumatología y Cirugía Ortopédica, 99-104,2009



Caso clínico MIR 1997-1998-F



Dra. Roxana Pérez Loyola, MSc. Dra. Zoila M. Pérez Rodríguez. Ventajas de la electroforesis, la magnetoterapia y el ejercicio en las lesiones calcificadas de hombro. Revista Cubana de Medicina Física y Rehabilitación 2015;7(2):149-159

Gota, seudogota y enfermedades relacionadas - Capítulo 175. Harrison. Manual de medicina. 18º edición. Editorial McGraw Hill.



Dra. María Eugenia Martínez. Diagnóstico Diferencial Monoartritis y Poliartritis. Apuntes de Reumatología. 2009. P. Universidad Católica de Chile.



Dra. Verónica Wolff C. Síndromed dolorosos regionales, columna, hombro y rodilla. REV. MED. CLIN. CONDES - 2012; 23(4) 433-444



Moutsopoulos HM, Tzioufas AG. Síndrome de Sjögren, cap. 324. Harrison. Principios de medicina interna, 18° edición. McGraw Hill.



Wolff V. Sindromes dolorosos regionales. Revista Médica Clínica Las Condes 2012;23:433-44 - DOI: 10.1016/S0716-8640(12)70334-6



2. Síndrome de Sjögren. Carlos Arberto Cañas Dávila, MD. Internista Reumatólogo. Fundación Clínica Valle del Lili.



Dolor de espalda y cuello - Capítulo 54. Harrison. Manual de medicina. 18º edición. Editorial McGraw Hill.





Rev. Chilena Dermatol. 2015; 31 (3) : 230 - 237. Puntos de encuentro entre la dermatología y la reumatología. Jara I.

Moutsopoulos HM, Vlachoyiannopoulos PG: Síndrome por anticuerpos antifosfolípidos, cap. 320. Harrison. Principios de medicina interna, 18° edición. McGraw Hill.



2. Loreta Rodríguez Pérez, Dra. Dunia Castillo González, Dra. Yusselys Cabrera Payne, Lic. Maribel Tejeda González. Síndrome antifosfolípido en mujeres con pérdidas recurrentes de embarazo: diagnóstico de laboratorio. Revista Cubana de Hematología, Inmunología y Medicina Transfusional.



Massardo L. Lumbago. Apuntes de reumatología. P. Universidad Católica de Chile.



Lupus eritematoso sistémico, artritis reumatoide y otras enfermedades del tejido conjuntivo - Capítulo 169. Harrison. Manual de medicina. 18º edición. Editorial McGraw Hill.



Manriquez J, Cardoso I and Wulf A. Fibromialgia: Epidemiología, fisiopatología, diagnóstico y estrategias de tratamiento. Rev Chil Med Fam 2004; 5 (1): 14-22







Derivation and validation of the Systemic Lupus International Collaborating Clinics classification criteria for systemic lupus erythematosus. Arthritis Rheum. 2012 Aug;64(8):2677-86.



Petri M. Review of classification criteria for systemic lupus erythematosus. Rheum Dis Clin North Am. 2005 May;31(2):245-54.



Lupus eritematoso sistémico, artritis reumatoide y otras enfermedades del tejido conjuntivo - Capítulo 169. Harrison. Manual de medicina. 18º edición. Editorial McGraw Hill.



Martínez M. Diagnóstico Diferencial Monoartritis y Poliartritis. Apuntes de Reumatología. P. Universidad Católica de Chile.



Lauro Gilberto Nunes R, Raquel Moresco V, Marley G, Cristina da Silva B, Matesanz Pérez, P. Síndrome de Behçet. Av. Odontoestomatol 2004; 21-4: 183-187.



Guía de Buena Práctica Clínica en OSTEOPOROSIS, 2da edición. Organización Médica Colegial de España.

31 Facultad de Medicina, Universidad de Chile

Artritis psoriásica

pero solamente en aquella con predominantemente axial (50% aprox.).

Nivel de manejo del médico general: Diagnóstico: sospecha. Tratamiento: inicial. Seguimiento: derivar.

Diagnóstico

compromiso

El diagnóstico es clínico, y se hace en un paciente con compromiso articular y con antecedente de psoriasis. Se apoya con un FR negativo, y anti-CCP negativo (que lo diferencia de la AR).

Aspectos esenciales

Existen 5 formas clínicas de manifestación articular: 

 

 



Forma parte del conjunto de las espondiloartropatías seronegativas, por lo tanto clásicamente se asocia a Factor reumatoídeo (FR) negativo, sacroilitis, entesopatía, manifestaciones ocular, tiene agregación familiar, y tienen elevada prevalencia del antígeno HLA-B27 Artropatía psoriática (APS)= Atropatía + Psoriasis. Habitualmente ocurre primero el compromiso cutáneo, y luego de 5 a 15 años ocurre el compromiso articular, aunque pueden ser al revés o iniciarse de forma simultánea La imagen radiológica “lápiz-copa” de la IFD es muy característica La presencia de alteraciones ungueales (pitting, onicolisis) apoya fuertemente el diagnóstico, pero puede no estar presentes No hay un marcador de laboratorio específico, pero el FR negativo y Anti CCP negativo apoyan el diagnóstico.











Oligoartritis asimétrica: suele afectar articulaciones interfalángicas distales (IFD) y proximales de manos y pies, rodillas, tobillos, muñecas. Poliartritis simétrica (40%): similar a artritis reumatoide, pero con FR negativo, ausencia de nódulos reumatoides. Artritis mutilante (3-5%): forma agresiva y destructiva, con compromiso articular severo y disolución ósea. Compromiso predominante en IFD (15%): frecuentemente asociada con alteraciones ungueales. Compromiso axial (20-40%): Espondilitis, sacroilitis o ambas, puede presentarse en ausencia de compromiso periférico.

La forma clínica de poliartritis simétrica es muy similar a la artritis reumatoide, y para hacer la diferencia es muy útil la presencia compromiso ungueal (punteado o pitting ungueal y onicolisis) que está presente en el 80% de los pacientes con artropatía psoriática. También puede haber compromiso de las articulaciones IFD, y es importante recordar que este compromiso puede ser también por artrosis, y la forma de diferenciarlas, es que la artropatía psoriática tiene resorción de las falanges distales (acroosteólisis) “lápiz-copa”, incluso llegando al telescopaje. Se pueden producir entesitis (siendo frecuente la tendinitis aquileana y la fascitis plantar) y se pueden observar sindesmofitos y sacroilitis unilateral o asimétrica (a diferencia de la espondilitis anquilosante que es simétrica)

Caso clínico tipo Paciente sexo masculino, 50 años, antecedentes de placas eritematodescamativas en superficies extensoras de ambos codos. Consulta por inflamación del dedo índice de mano izquierda y lumbago tipo inflamatorio.

Definición Artropatía inflamatoria asociada a la psoriasis cutánea. Pertenece al grupo de las espondiloartropatías seronegativas. Generalmente, el cuadro articular es varios años posterior a la enfermedad cutánea. Sin embargo, esto no ocurre siempre.

Epidemiología La prevalencia de psoriasis cutánea varía entre 1-2%, y entre el 6-8% de estos presenta artropatía inflamatoria. Ocurre con la misma frecuencia en ambos géneros, pero se ha encontrado que el compromiso axial es más frecuente en el sexo masculino y el compromiso periférico, en el femenino. Se inicia entre los 35-50 años.

Etiología Es desconocida, involucra tanto factores ambientales como genéticos. Se ha encontrado fuerte evidencia de la asociación entre la región HLA-C del complejo mayor de histocompatibilidad y la psoriasis cutánea. La presencia de HLA Cw6 es la región de susceptibilidad para psoriasis cutánea y de igual forma se observa asociación, aunque menor, con artropatía psoriática. Con respecto a HLA B27 se ha encontrado asociación con artropatía psoriática 1

Tratamiento Similar al de la AR, usando AINES para el manejo sintomático de la enfermedad y usando fármacos modificadores de la enfermedad o que eviten la progresión. El fármaco modificador de elección es el metotrexato (ya que es muy eficaz en controlar las manifestaciones articulares y cutáneas). Pero puede ser deletéreo en pacientes con VIH y pareciera ser poco efectivo si hay afectación axial o compromiso mutilante. El uso de prednisona puede empeorar el cuadro cutáneo.

Seguimiento Derivar a especialista.

2

Artritis reactivas

Fisiopatología El proceso resulta de la interacción entre los agentes infecciosos mencionados anteriormente y factores de susceptibilidad individual, debido a que sólo 1-12% de los pacientes que sufren infección desarrollan posteriormente artritis. El factor de susceptibilidad más aceptado para el desarrollo de artritis reactiva es la presencia de HLA B27 que se observa en 60-80% de los casos.

Nivel de manejo del médico general: Diagnóstico: específico. Tratamiento: inicial. Seguimiento: derivar.

Aspectos esenciales  



Diagnóstico

Cuadros de inflamación articular con antecedente de infección urogénica o enterogénica. Es frecuente la aparición de entesitis, comúnmente en fascia plantar o en tendón aquiliano. Lo más frecuente es la manifestación oligoarticular.

Clínica: El inicio de los síntomas articulares es entre 1-4 semanas posterior a la infección inicial. La artritis puede ser mono, oligo o poliarticular, pero la formas más común es la oligoartritis. Compromete principalmente los miembros inferiores y con distribución asimétrica, pero también puede afectar a miembros superiores y generar sacroileítis.

Caso clínico tipo

Dentro de las manifestaciones extra-esqueléticas encontramos la afección ocular (conjuntivitis, uveítis), muco-cutánea (úlceras orales, balanitis circinada y queratoderma blenorrágica), gastrointestinales (diarrea) y genito-urinarias (uretritis, cervicitis).

Hombre de 31 años que presenta hace 1 semana dolor en el talón izquierdo sin causa aparente y dolor glúteo izquierdo. Refiere el antecedente de haber tenido episodios diarreicos y de dolor abdominal hace aproximadamente 2 semanas.

Los episodios de artritis pueden durar desde pocas semanas hasta 8 meses. Cerca del 50% presenta sólo un episodio, el resto cursa con brotes recurrentes, y hasta el 20% puede desarrollar artritis crónica.

Definición El término define a una artritis no purulenta que se desarrolla posterior a una infección extra-articular, con cultivo (-) en el líquido sinovial. Por consenso, el concepto queda restringido a la infección por bacterias de foco urogénico (Chlamydia) y enterogénico (Shigella, Salmonella, Yersinia, Campylobacter). La concomitancia de artritis, conjuntivitis y uretritis descrita por Hans Reiter constituye apenas uno de los múltiples complejos sintomáticos, por lo tanto, el epónimo de Síndrome de Reiter está relegado sólo a una cita histórica. El término se ha utilizado principalmente para referirse a las espondiloartritis posteriores a infecciones genitourinarias o entericas que se presentan predominantemente en pacientes HLA-B27 positivos. El pronóstico es variable; persistencia o recurrencias en un 30 a 60% de los afectados, un 15 - 25% sufre invalidez permanente.

Laboratorio: No existe ninguna prueba específica para el diagnóstico. Puede existir elevación de parámetros inflamatorios, principalmente en el período agudo, por lo que su ausencia no excluye el diagnóstico. En casos crónicos puede observarse anemia de enfermedad crónica. Se observa negatividad de FR y Anti CCP, y líquido sinovial inflamatorio con cultivos (-). Puede indagarse el origen de la artritis reactiva buscando la presencia de uretritis, aunque no es infrecuente la infección asintomática. Siempre debe descartarse la infección por gonococo, principal diagnóstico diferencial en estos casos.

Tratamiento Para manejo sintomático se utilizan AINEs. En casos crónicos se utilizan DMARDs (Sulfasalazina) y ante refractariedad, Anti TNFalfa. En casos de confirmación de uretritis clínica o pesquisa de chlamydia en secreción uretral, así como de cuadro diarreico relacionado a enterobacterias, debe iniciarse tratamiento antibiótico.

Epidemiología La artritis reactiva urogénica es más frecuente en el sexo masculino y constituye la causa más común de artritis en varones de 15-40 años. En países desarrollados representan la gran mayoría de las artritis reactivas. La artritis enterogénica se presenta con la misma frecuencia en hombres y mujeres, siendo más frecuentes en países sub-desarrollados.

Seguimiento Derivar a especialista

Además de la artritis destaca la entesitis, principalmente de tendón aquiliano y fascia plantar, que puede observarse en cerca de la mitad de los casos, y la dactilitis. Ambas características se comparten con la artropatíapsoriática.

3

y menos frecuentemente IFP. El compromiso puede generar un aumento de volumen en las articulaciones por hipertrofia ósea, los cuales se denominan nódulos de Heberden (IFD) y Bouchard (IFP). A diferencia de AR, el aumento de volumen es duro y sin signos inflamatorios. En rodillas genera limitación en flexión, con presencia de crujido articular al examen físico. En caderas se manifiesta inicialmente como trastorno de marcha con limitación en la rotación interna y luego en la flexión y abducción. El compromiso vertebral lleva a dolor cervical y lumbar, no irradiado y de localización inespecífica. Imágenes: Los hallazgos característicos son disminución del espacio articular, esclerosis subcondral, osteofitos y geodas. No hay correlación entre hallazgos imagenológicos y clínicos, pudiendo ser totalmente discordantes. Es fundamental la sospecha de artrosis secundaria (a artritis, trastornos metabólicos, endocrinológicos, estructurales o traumáticas), lo cual debe considerarse ante aparición de artrosis en menores de 40 años, más aún en varones, localización atípica (hombro, muñeca, tobillo), signos de compromiso sistémico o alteraciones de laboratorio sospechosas (elevación de VHS, hipercalcemia, etc).

Artrosis Nivel de manejo del médico general: Diagnóstico: específico. Tratamiento: inicial. Seguimiento: completo.

Aspectos esenciales      

Enfermedad reumatológica más común. Mujeres mayores de 60 años. Dolor que aumenta con actividad y disminuye en reposo. Rigidez de menor duración a la de AR. Tratamiento no farmacológico es fundamental. Descartar causas secundarias.

Caso clínico tipo Mujer de 65 años con cuadro de 3 años de evolución de dolor al caminar en rodillas y rigidez matinal que dura menos de 30 minutos. Al examen físico limitación y crujido. Se solicita Rx rodillas que muestra; disminución del espacio articular, esclerosis subcondral y presencia de osteofitos.

Tratamiento Ambulatorio en APS. GES incluye tratamiento médico de artrosis de rodillas y caderas para personas >55 años, remplazo articular con endoprótesis total de cadera en personas > 65 años con artrosis de cadera y limitación funcional severa y entrega de órtesis para adultos > 65 años.

Definición



Enfermedad degenerativa articular de evolución crónica caracterizada por daño al cartílago y los tejidos periarticulares. La característica patológica del cuadro es la pérdida de cartílago hialino articular, asociado a esclerosis subcondral, osteofitos, estiramiento de la cápsula y debilidad de los músculos estabilizadores relacionados. Puede afectar cualquier articulación, pero predomina en aquellas que sostienen peso o se movilizan con frecuencia (columna vertebral, cadera, rodilla, manos).



Epidemiología Cuadro reumático más frecuente en > 65 años y una de las principales causas de discapacidad en este grupo etario. Afecta más frecuente en mujeres. Durante muchos años se aceptó que la artrosis era un problema de desgaste por uso, actualmente se considera que no es una consecuencia obligada. Involucra un proceso patológico que tiene origen en una falla metabólica del cartílago que lo lleva a la desintegración gradual generando los síntomas característicos. Dentro de los factores de riesgo se incluye la edad (factor de riesgo principal), sexo, raza, obesidad (mujeres obesas 4 veces más riesgo), ocupacionales, genéticas, tabaquismo.



No farmacológicos: baja de peso, ejercicio aeróbico, siendo especialmente beneficiosos aquellos en agua, modalidades térmicas y educación paciente. Farmacológico: o Analgesia: Paracetamol 1g/8 horas, en caso de no ser suficiente se puede utilizar AINEs por períodos cortos como Ibuprofeno (No se ha descrito ningún AINEs más efectivo que otro, incluyendo a los COX- 2 selectivos que representan beneficios disminuyendo los efectos adversos). o Corticoides intraarticulares solamente útiles en brotes agudos, y no utilizar más de 2 cada año. o Los opioides pueden utilizarse en contexto de terapia escalonada sólo si la respuesta es insuficiente con las medidas anteriores. o El uso de Glucosamina y viscosuplementación puede considerarse en el manejo sintomático. Quirúrgico: a evaluar en paciente con dolor intratable y/o alteración grave de la función articular. Se realiza reemplazo articular con endoprótesis, evaluar por traumatología.

Seguimiento Diagnóstico Control del dolor y progresión de enfermedad. Derivar si: dolor intenso incapacitante, sospecha de artrosis secundaria, compromiso neurológico secundario a compromiso de columna.

Clínica: El dolor es el síntoma cardinal, pudiendo ser de inicio insidioso e intensidad leve, intermitente y autolimitado. Las zonas más afectadas son las manos, rodillas, caderas y columna vertebral. En las manos las zonas más afectadas son las IFD, trapecio-metacarpiana 4

cervical tiene su origen en una causa mecánica. Alrededor de un 1% puede desarrollar manifestaciones neurológicas

Cervicalgia

Etiología

Nivel de manejo del médico general: Diagnóstico: específico. Tratamiento: completo. Seguimiento: completo.

La principales causas de cervicalgia, y cervicobraquialgia se resumen en las siguientes tablas.

Diagnóstico

Aspectos esenciales     

El diagnóstico es fundamentalmente clínico. La causa más frecuente es de origen mecánico. Se debe preguntar el antecedente de trauma y buscar focalidad neurológica Se debe indagar signos de alarma que podrían implicar una causa sistémica Tratamiento con analgésicos y relajantes musculares.

El diagnóstico es clínico. Cuando existe el antecedentes de trauma previo, la evaluación inicial incluye radiografía cervical en 3 proyecciones (AP, lateral y oblicua). En caso

Caso clínico tipo Paciente de sexo femenino, 43 años, con dolor localizado en cara posterior de región cervical, sin irradiación, sin déficit neurológico, que empeora con el movimiento y disminuye en reposo. Consulta por persistencia del dolor durante 1 semana, con limitación en la movilidad cervical.

Definición La cervicalgia corresponde al dolor cervical, que en la mayoría de los casos tiene un origen de tipo muscular. En caso de asociarse a alteraciones neurológicas con clínica de irradiación a los miembros superiores, se denomina cervicobraquialgia.

Epidemiología Por lo menos el 80% de la población ha experimentado cervicalgia alguna vez en su vida. Dentro de los factores de riesgo más trascendentes encontramos situaciones de estrés, depresión, obesidad y factores mecánicos ocupacionales. En cerca del 90% de los casos, el dolor

de presentar dolor cervical asociado a síntomas neurológicos, la resonancia magnética será el examen más útil. Si ese antecedente no está presente, lo más importante es determinar si se está frente a un trastorno

5

localizado o ante una manifestación local de una enfermedad sistémica, a lo que ayudan la identificación de signos de alarma (fiebre, pérdida de peso, dolor intenso en reposo, rigidez matinal prolongada, dolor óseo localizado, antecedentes de cáncer, antecedentes de infección grave reciente, espasticidad de miembros inferiores, pérdida de función vesical e intestinal). Si el dolor es eléctrico y se irradia a los miembros superiores corresponde a una cervicobraquialgia. En estos casos el dolor radicular es la expresión del efecto compresivo de una protrusión discal, de osteofitos o la combinación de ambos sobre las raíces cervicales. En estos casos el dolor usualmente se acompaña de hiporreflexia, disestesias y disminución de la fuerza en el territorio de la raíz comprometida.

Tratamiento Las cervicalgias de origen mecánico responden en su mayoría al tratamiento médico. El tratamiento farmacológico habitual consiste en manejo del dolor con analgésicos o anti-inflamatorios y relajantes musculares. La indicación fisioterapia es fundamental en los cuadros de evolución crónica. El uso de collar cervical blando es especialmente útil para el alivio del dolor nocturno. En caso de presentar signos de alarma se requiere de una derivación oportuna.

Seguimiento Completo

6

o o

Columna dolorosa 

Nivel de manejo del médico general: Diagnóstico: específico. Tratamiento: completo. Seguimiento: completo.

Lesiones de partes blandas. Psicógenas De origen en el plexo braquial: o Traumáticas o Adenopatías cervicales: infecciosas (tuberculosa).

tumorales,

Causas dolor lumbar: 

Aspectos esenciales 



Siempre evaluar banderas rojas: o Primer episodio post 50 años o previo a 20 años. o Dolor en decúbito, dolor óseo localizado, dolor visceral, rigidez prolongada. o Dolor nocturno mayor a diurno. o Parestesias o irradiación bilateral. o Síntomas constitucionales: baja de peso, fiebre, anorexia. o Antecedentes de cáncer. o Factores de riesgo de infección (VIH, drogas ev). o Trauma mayor (menor en ancianos); osteoporosis. o Déficit motor o sensitivo. o Babinski, clonus, hiperreflexia. o Disfunción vesical, incontinencia anal. o Hipoestesia perineal-perianal (en silla de montar).

 

Diagnóstico Anamnesis 



Caso clínico tipo Hombre de 23 años, con antecedentes de dolor lumbar insidioso de 3 meses de evolución, acompañado de rigidez lumbar matinal. Además presenta talalgia al caminar desde hace 15 días.

 

Definición Conjunto de síndromes cuyo síntoma en común es dolor en alguna parte de la columna, de múltiples etiologías y se pueden dividir en sectores para su estudio.

Epidemiología El dolor de columna, principalmente el lumbar, es la segunda causa más frecuente de consulta médica, de ellos el 30-80% será autolimitado. Es la primera causa de ausentismo laboral. El 90% de los casos tiene origen en causas biomecánicas.

Etiología Causas cervicalgia - cervicobraquialgia: Para más información, ver apunte específico. 

Con columna sana: sobrecarga (vicio postural, sobrepeso y tensión nerviosa). Con columna enferma: espondilolistesis y espondilolisis, artrosis, espondilolistesis degenerativa, AR, artritis anquilopoyética, escoliosis, tumores (primarios malignos o benignos, metástasis, de partes blandas, pseudotumores), osteopatías metabólicas (gota, osteoporosis). Causa extra raquídea: ginecológica, urológica, hepatobiliar, pancreática, muscular (miositis). Secundario a enfermedad general: AR, gota, grupe, amigdalitis aguda, etc.



De origen cervical o Traumáticas o Inflamatoria: infecciosas (TBC, tífica, etc.), no infecciosa (AR, enfermedad reumática generalizada, etc.) o Neoplásicas: metástasis, mieloma, hemangioma, osteoma osteoide, etc. 7

Antecedentes: Trauma, enfermedades infecciosas (TBC, tifus, infecciones estafilocócias), neoplasias (CA de mama, riñón, próstata), tipo de trabajo, estado psíquico. Historia actual: Semiología del dolor, limitación funcional, rigidez, signos de compromiso radicular, síntomas generales (fiebre, baja de peso, anorexia, sudoración nocturna, síntomas depresivos). Banderas rojas: excluyen causa biomecánica, siempre deben preguntarse dirigidamente. Examen físico, Inspección (columna estática y en movimiento, alteraciones de ejes (xifosis, lordosis, escoliosis), palpación. Examen neurológico: Muy importante, buscar parestesias o hipoestesia que siguen dermatoma: radiculopatía; si no siguen raíz: lesión de plexo o nervio, simulación. Buscar signos de irritación radicular: TEPE (+) indica compresión ciática, O'Connel (+) indica compresión nervio crural.Reflejos: patelar L3-4; Aquiliano L5-S1. Fuerza muscular: dorsiflexión contra resistencia (pie L4 y 1º ortejo L5); inversión del pie; flexión de cadera contra resistencia L2; caminar en puntillas S1 y talones L4 - cuclilla (fuerza distal y proximal). Incontinencia urinaria y fecal. Sensibilidad cara interna de muslo y perineal. Síndrome cola de caballo: hipoestesia en silla de montar, dolor perineal, alteración de esfínteres, sd. ciático bilateral. Derivar en forma urgente. Derivar para estudio ante sospecha o pedir TAC Cintigrama que objetive. Espondilitis infecciosaVIH, drogas ev, inmunosuprimidos. Dolor de reposo y movimientos + fiebre. Derivar para hospitalización: Fractura vertebral, anciano; usuarios corticoides. Con o sin antecedentes de trauma. Dolor de inicio reciente, localizado en línea media, percusión de apófisis dolorosa; no cede con reposo y aumenta con movimientos. Sin síntomas neurológicos. Se deben pedir exámenes complementarios ante la sospecha de una infección, enfermedad sistémica,





daño articular, compresión nerviosa, evaluar estado de tendones, bursas y en presencia de banderas rojas. Si no tiene banderas rojas no pedir exámenes hasta 4ª semana de evolución. Imágenes, la radiografía simple AP, L, oblicua según cuadro clínico; en flexoextensión evalúa hipermovilidad de espondilolistesis. TAC para evaluar fracturas, raquiestenosis, lesión ósea, Paget. RNM para evaluar partes blandas, enfermedad facetaria, HNP. Cintigrama esquelético en sospecha de infección, neo, fractura. Laboratorio: Según la sospecha se puede incluir mielografía, angiografía u otros generales (HG, VHS, etc.).

Tratamiento Si el dolor lumbar inespecífico: conservador. Reposo por 2 días si es muy intenso o tiene incapacidad funcional, mantener actividad física, el reposo en cama puede ser contraproducente. Paracetamol 1g c/8-6hrs; AINEs: si en 7-10 días no hay respuesta, cambiar de familia; relajantes musculares; calor o frío loca. Lumbociática: AINEs, neuromoduladores, KNT, derivar a las 6 semanas si no hay mejoría de síntomas. Crónico: evaluar banderas amarillas (comportamiento, trabajo, emociones, familia, etc.). KNT, fisiatra, reumatólogo, traumatólogo, etc. Considerar antidepresivos o tramadol.

Seguimiento Si es inespecífico por APS. Otras causas evaluar individualmente. En presencia de banderas rojas se debe derivar

8

Condrocalcinosis

Diagnóstico

Nivel de manejo del médico general: Diagnóstico: específico. Tratamiento: inicial. Seguimiento: derivar.

Clínica

Artritis aguda: Es el cuadro más característico. Afecta principalmente a rodillas (25% de los casos), muñecas y con menor frecuencia tobillos y hombros, con un inicio súbito o gradual y con intensos síntomas inflamatorios: la articulación comprometida presenta edema, eritema, y dolor intenso. Esta presentación imita a la gota (pseudogota) y a la artritis séptica. Hasta el 50% de los casos puede presentarse fiebre. Si bien el inicio monoarticular es lo más característico, también puede presentar afección poliarticular.

Aspectos esenciales     

Condrocalcinosis: presencia de cristales de Pirofosfato de Calcio (PC) (birrefringencia débil). Pseudogota: fenómeno inflamatorio secundario a los cristales de PC. Diagnóstico mediante observación de cristales en líquido sinovial. Tratamiento similar a gota. Todo paciente menor de 50 años con la enfermedad, debe ser estudiado en búsqueda de causas secundarias.

Artropatía similar a OA: En mujeres mayores de 65 años, con clínica similar a OA, pero distribución articular diferente. Además, puede generar moderada inflamación. Se manifiesta con dolor crónico y rigidez. Pseudoartritis reumatoide: similar a AR, pero con afección asimétrica, ausencia de osteoporosis yuxtaarticular y FR (-). En algunos casos familiares puede ser idéntica a AR, con compromiso de MCF. Pseudoespondilitis anquilosante: depósito en discos, caderas, inserciones músculo-tendinosas, sínfisis del pubis y sacroilíacas. Asintomático (condrocalcinosis). Exámenes: líquido sinovial inflamatorio entre 2.000 y 20.000 leucocitos x mm³ con predominio de PMN >50%. La confirmación es con la observación de cristales de PC en líquido sinovial. En la radiografía de la articulación afectada se observa una calcificación característica.

Caso clínico tipo Mujer de 70 años con dolor súbito de rodilla derecha asociado a aumento de volumen. Se punciona articulación afectada que evidencia 10.000 leucocitos x mm³ con 65% PMN y cristales de pirofosfato cálcico.

Definición Entidad que se caracteriza por el hallazgo de cristales de pirofosfato de calcio dihidratado (PC) en el líquido sinovial, cartílagos, fibrocartílagos y meniscos. Con menos frecuencia se observan depósitos en bursas y tendones. Debe diferenciarse dos conceptos: 



Tratamiento El manejo de la crisis es con AINES, corticoides intraarticulares o colchicina. En caso de recurrencia, usar colchicina profiláctica. Las dosis son equivalentes a las usadas en gota. La colchicina posee una eficacia variable. Todo paciente menor de 50 años que presente la enfermedad debe ser estudiado por causas secundarias.

Condrocalcinosis: Se refiere a la presencia de cristales de PC sin clínica, generalmente como hallazgo radiológico. Pseudogota: Se refiere a la artropatía inflamatoria secundaria a la presencia de cristales de PC (enfermedad por depósito de PC). Suelen comprometerse con más frecuencia la articulación de la rodilla y otras de gran tamaño.

Seguimiento Derivar a especialista.

Epidemiología Enfermedad rara en menores de 50 años, su prevalencia puede alcanzar hasta el 10% en sujetos entre los 60-75 años y llegar hasta el 30% en personas mayores de 80 años. Es más frecuente en mujeres. En condrocalcinosis, aumenta la concentración de sales de calcio en la articulación en forma de PC, haciéndose insoluble y precipitando. Estos cristales forman una capa cálcica dentro del cartílago y no produce síntomas. Lo que produce la clínica es la reacción inflamatoria a los microcristales libres. Existe la forma esporádica, la más frecuente y característica en pacientes de mayor edad y en casos más raros las formas familiares que debutan a edades tempranas.

Causas secundarias o asociadas de condrocalcinosis son: hiperparatiroidismo, hemocromatosis, hipotiroidismo, hipomagnesemia, hipofosfemia.

9

el origen del músculo extensor del carpo en el epicóndilo lateral. El codo del golfista se asocia al uso excesivo y reiterado de la extremidad superior en relación a la flexión de muñeca y pronación contra resistencia.

Epicondilalgia, epitroclealgias Nivel de manejo del médico general: Diagnóstico: sospecha. Tratamiento: inicial. Seguimiento: completo.

Diagnóstico

Aspectos esenciales  



En la epicondilalgia, el paciente refiere dolor en cara lateral del codo, inmediatamente distal al epicóndilo, el cual se reproduce frente a la extensión forzada contra resistencia de la muñeca. La clínica es categórica por lo que no se requiere estudios complementarios. En la epitroclealgia, el paciente consulta por dolor en la cara medial del codo, intensificado a la palpación del epicóndilo medial. El dolor se reproduce ante la flexión contra resistencia de la muñeca.

Epicondilalgia es la causa más común de dolor en la zona del codo. Es conocida como “codo del tenista” y característicamente presenta dolor en la zona lateral del codo que se exacerba ante la extensión de la muñeca. La epitroclealgia es conocida como “codo del golfista” y característicamente presenta dolor en la zona medial del codo que se exacerba ante la flexión de la muñeca.

Tratamiento Caso clínico tipo Reposo de zona afectada, evitando la actividad que generó el cuadro. AINEs sistémico para manejo sintomático. Pueden utilizarse férulas, en especial ante epicondilalgia, cuya función no es inmovilizar, sino distribuir la fuerza ejecutada en la extensión de la muñeca en un área mayor que la del tendón de los músculos extensores. Si el dolor es agudo y refractario a tratamiento, pueden utilizarse infiltraciones con corticoides. La indicación de cirugía está dada sólo ante casos que no responden a las medidas descritas.

Paciente sexo masculino de 43 años. Consulta por dolor en la cara lateral del codo que aumenta con la extensión y supinación de la muñeca.

Definición La epicondilalgia corresponde al dolor en la zona de inserción de los músculos extensores del carpo en el epicóndilo (epicóndilo lateral). La epitroclealgia corresponde al dolor en la zona de inserción de los músculos flexores del carpo en la epitróclea (epicóndilo medial).

Seguimiento Controlar hasta remisión rehabilitación con fisioterapia.

Epidemiología La epicondilitis se presenta con mayor frecuencia en el género masculino, entre 30 a 50 años de edad. Característicamente compromete el brazo dominante. Su incidencia es de un 1% - 3% en la población general, la que aumenta en deportistas. La epitrocleítis posee una prevalencia hasta 7 veces menor que la epicondilitis, presentándose en un 75% en el brazo dominante.

Etiología La epicondilalgia (síndrome epicondilar lateral o codo del tenista) es la causa más común de dolor en la región del codo, pudiendo presentarse a cualquier edad, mientras que la epitroclealgia (síndrome epicondilar medial o codo del golfista) es más rara y se presenta a mayor edad. El codo del tenista es una entesopatía mecánica que se asocia al uso excesivo y reiterado de la extremidad superior, en especial en aquellas personas que por su labor, están obligadas a extender el codo y la muñeca de forma repetida contra una resistencia. El sitio afectado es 10

de

síntomas.

Luego

Telangectasias): infiltración cutánea distal a codos y rodillas, respeta tronco. Inicio insidioso y pronóstico bueno ya que tiene menor porcentaje de afectación visceral. El fenómeno de Raynaud es temprano y precede por años al compromiso de piel (Importante diferenciar de Raynaud primario que presenta las siguientes características: sin evidencia de enfermedad sistémica, inicio en jóvenes e historia de larga data (< 30% en > 40 años), afección a varios miembros de la familia, ataques episódicos simétricos, ausencia de lesiones isquémicas, capilaroscopía normal, ANA (-) y VHS normal).

Esclerosis sistémica progresiva Nivel de manejo del médico general: Diagnóstico: sospecha. Tratamiento: inicial. Seguimiento: derivar.

Aspectos esenciales  





El compromiso sistémico es de menor gravedad que en la variedad difusa, destacando síntomas articulares (generalmente artralgias), compromiso digestivo con afección de la musculatura lisa del esófago y con ello disfagia esofágica, asociación con cirrosis biliar primaria e hipertensión pulmonar sin daño en parénquima, siendo esta última un marcador pronóstico fundamental. En los exámenes de laboratorio ANA (+) patrón anticentrómero en el 70% de los casos.

Mesenquimopatía con afección sistémica, pero principalmente cutánea. Existen varias manifestaciones de la enfermedad. Las 2 presentaciones clásicas en la esclerodermia sistémica: difusa y limitada. Ambas con características distintivas. Compromiso pulmonar: Hipertensión pulmonar en variedad limitada y enfermedad pulmonar intersticial en la difusa. Actualmente sin tratamiento curativo.

Esclerodermia sistémica difusa: infiltración cutánea proximal y distal sin respetar tronco, con inicio rápido, progresivo y fenómeno de Raynaud. El compromiso visceral es de mayor gravedad y presenta peor pronóstico que la variedad limitada. Destacan la enfermedad pulmonar intersticial que se presenta con clínica respiratoria restrictiva y se asocia a Anti Scl-70, y el compromiso renal (Crisis renal esclerodérmica) que se caracteriza por alteración de la función renal e HTA, muchas veces desencadenado por uso de corticoides a altas dosis. Fue la principal causa de morbimortalidad en estos pacientes, situación que ha mejorado notablemente con el uso de IECAs como medida preventiva.

Caso clínico tipo Mujer de 40 años, con fenómeno de Raynaud, piel de los dedos esclerótica, poliartralgias y síntomas de pirosis y regurgitación. Al examen físico piel engrosada en tercio distal de brazos.

Definición

En los exámenes de laboratorio destaca ANA (+) con ENA Anti Scl-70 (+) en 22-30% de los casos. Existen otras variedades de la enfermedad como la esclerosis sistémica sin esclerodermia, la enfermedad mixta del tejido conectivo o síndromes esclerodertiformes. Todos estos diagnósticos corresponden a especialidad.

Enfermedad del tejido conectivo caracterizada por cambios degenerativos e inflamatorios que conducen a la fibrosis. El compromiso cutáneo (esclerodermia) le otorga el sello a la enfermedad, pero el compromiso sistémico (especialmente a nivel cardíaco, pulmonar, renal y digestivo) es el que marca el pronóstico de la misma y determina su tratamiento. La mayoría de las manifestaciones de esta entidad tienen un sustrato vascular (telangiectasias, fenómeno de Raynaud, nefroesclerosis, etc).

Tratamiento Tratamiento difícil, sin fármacos que detengan la progresión de la enfermedad.  

Epidemiología



Afecta predominantemente al sexo femenino 94%, y su peak es entre los 45 y 55 años. Se caracteriza por generar anormalidades en 3 tipos de células: fibroblastos (asociada a la fibrosis cutánea y visceral), células endoteliales (se asocia a obliteración de vasos y Raynaud) y linfocitos (generación de anticuerpos). La etiología es desconocida.



 

Piel: lubricación, cuidado de heridas. Raynaud: Bloqueadores de canales de calcio, aspirina, IECAs. RGE: Inhibidores de bomba de protones, prokinéticos. Enfermedad pulmonar intersticial: Inmunosupresores (Ciclofosfamida, Azatioprina, Ciclosporina). Hipertensión pulmonar: Sildenafil, bosentán, Prostaciclina. Renal: IECAs.

Diagnóstico Seguimiento

La esclerodermia sistémica se caracteriza por compromiso de piel y compromiso sistémico. Dentro de ésta encontramos 2 variedades, la variedad difusa y la limitada.

Derivar a especialista.

Esclerodermia Sistémica limitada (CREST: Calcinosis, Raynaud, Compromiso Esofágico, Esclerodactilia,

11

Osteodistrofia renal crónica

en

Ex. Físico: Hay que evaluar signos de ERC, y buscar: deformaciones esqueléticas, ruptura de tendones, fracturas espontáneas, entre otras.

insuficiencia

Ex. Complementarios: Exámenes generales, medición de Calcio, Fósforo y PTH, radiografía e histología. Nivel de manejo del médico general: Diagnóstico: Sospecha. Tratamiento: Inicial. Seguimiento: Derivar.

Tratamiento

Aspectos esenciales     

El objetivo del tratamiento es la normalización de los niveles de PTH junto a concentraciones plasmáticas normales de calcio y fósforo. Esto se puede esquematizar en 3 fases: repleción de los depósitos de vitamina D, control de la ingesta de fósforo con fijadores, y administración de vitamina D activa. Por tanto, el tratamiento debe considerar principalmente: dietas hipofosfémicas, quelantes del fósforo (con calcio o resinas sin calcio, nunca hidróxido de aluminio), calcitriol oral o ev (cuando PTH sobre 250pg/mL. Se debe vigilar Ca y P), y paratiroidectomía de ser necesario (resistencia a tratamiento).

Causada por progresión de ERC. Todo afectado la padecerá a lo largo de su evolución. Es importante buscar dolor óseo, deformidades esqueléticas, esguinces y fracturas. Importante preguntar tratamiento actual. El tratamiento puede requerir cirugía.

Caso clínico tipo

Hay que tener en mente que una mala terapéutica, puede producir Enfermedad Ósea Adinámica (por exceso de Calcio y Vitamina D).

A su consulta asiste un paciente masculino de 63 años de edad con antecedentes de IRC por DM2 diagnosticadas hace 3 años. En la entrevista, el paciente refiere dolores óseos constantes, y refiere que varias veces se ha esguinzado y una vez se fracturó. Al ex. Físico se aprecia leve deformidad ósea facial.

Seguimiento Derivar.

Definición La osteodistrofia renal se refiere a diversos trastornos minerales óseos presentes en la enfermedad renal crónica. Su prevalencia aumenta a mayor descenso de la función glomerular.

Etiología-epidemiología-fisiopatología Se debe a la progresión de la Enfermedad Renal Crónica. La ERC está aumentando su prevalencia, y todo afectado padecerá de Osteodistrofia por esta causa. La enfermedad Renal Crónica produce un déficit de vitamina D activa (1,25 dihidroxicolecalciferol) y un hiperparatiroidismo secundario, el cual produce alteraciones en la homeostasis del Ca y P y alteraciones osteometabólicas.

Diagnóstico Anamnesis: Edad, sexo, sintomatología de la ERC, dolores óseos, prurito, miopatía proximal, antecedentes personales (Enfermedad renal crónica, diabetes mellitus, hipertensión arterial, entre otras), y familiares (Antecedentes mórbidos de importancia). Es importante buscar los antecedentes de tratamiento para la ERC (tratamiento dietario, farmacológico, diálisis, entre otros).

12

Etiología

Pelviespondilopatías seronegativas

La etiología es desconocida. Actualmente se sospecha el rol de diversos agentes infecciosos virales o bacterianos como desencadenantes del cuadro. En el caso particular de la artritis reactiva (síndrome de Reiter) se ha observado que se puede desarrollar posterior a infecciones del tracto urinario o digestivo por Campilobacter, Clamidia, Salmonella y Yersinia. Hasta ahora se reconoce que la interacción entre HLA B27 y elementos exógenos podría jugar un papel fundamental en la reacción inflamatoria, siendo especialmente importante en la espondilitis anquilosante. En población blanca el 1-2% de la población HLA B27 (+) presenta EA y el 90% de los pacientes con EA presentan HLA B27 (+). En el caso de la artropatía asociada a EII solo se ha observado relación en aquellas con compromiso axial.

Nivel de manejo del médico general: Diagnóstico: sospecha. Tratamiento: inicial. Seguimiento: derivar.

Aspectos esenciales    

Artritis periférica con predominancia en extremidades inferiores. Compromiso axial que se caracteriza por dolor lumbar inflamatorio. Presenta asociación con HLA B-27. Rx de columna es una herramienta importante en el diagnóstico.

Diagnóstico Este grupo de enfermedades comparten ciertas características en común: artritis periférica con predominio en extremidades inferiores, sacroileítis, dolor lumbar inflamatorio (inicio en menores de 40 años, insidioso, persistencia por más de 3 meses, rigidez matinal, mejoría con el ejercicio), factor reumatoídeo negativo, asociación con HLA B27, entesitis, agregación familiar y manifestaciones extra-articulares donde destaca la uveítis. La espondilitis anquilosante en su etapa temprana presenta dolor glúteo secundario a compromiso sacroilíaco bilateral y en etapas tardías, una importante disminución de la movilidad de la columna, secundario al proceso inflamatorio-reparativo con generación de sindesmofitos que puede llevar finalmente a la anquilosis total de la misma. La afección de articulaciones periféricas es típico en caderas y rodillas, y dentro de las manifestaciones extra-articulares destaca la uveítis anterior. Las pelviespondilopatías secundarias a enfermedad inflamatoria intestinal, pueden presentar compromiso inflamatorio periférico, en especial de extremidades inferiores y compromiso axial, siendo este último asociado a HLA B27. No hay exámenes específicos para el diagnóstico de pelviespondilopatías, el diagnóstico es clínico. Puede ayudar la ausencia de FR y Anti CCP. Puede existir aumento en parámetros inflamatorios, pero no es excluyente su ausencia. También puede existir anemia de enfermedad crónica en algunos casos. El HLA B27 no es bajo ninguna circunstancia diagnóstico de pelviespondilopatía, pero puede representar una herramienta importante. La radiografía de articulaciones sacroilíacas representa una gran herramienta para la evaluación del compromiso axial, sin embargo, los hallazgos radiográficos pueden resultar tardíos, situación en la cual la TAC y la RNM constituyen herramientas fundamentales, en especial esta última, para el diagnóstico precoz.

Caso clínico tipo Hombre de 23 años con dolor lumbar insidioso de 3 meses, que mejora con el ejercicio y se acompaña de rigidez matinal lumbar. Al examen físico dolor en la inserción del tendón aquiliano.

Definición Corresponden a un grupo de enfermedades que comparten un conjunto de signos y síntomas característicos, secundarios a compromiso inflamatorio del esqueleto axial, así como de afección extraesquelética. Incluye 2 subgrupos, el primero de pelviespondilopatías diferenciadas (espondilitis anquilosante, artritis reactiva, artropatíapsoriática y artritis asociada a enfermedad inflamatoria intestinal) y el segundo de formas indiferenciadas (pelviespondilopatía indiferenciada y cuadros de oligoartritis, entesitis y uveítis aislados).

Epidemiología Constituyen el grupo de enfermedades reumáticas crónicas más frecuente en el adulto joven de sexo masculino. La incidencia está directamente relacionada con la frecuencia de HLA B27 en la población, siendo predominante en las población de raza blanca. La prevalencia de HLA B27 en la población chilena es de un 4%. Aproximadamente un 2% de los individuos positivos para HLA B27 desarrollaran la enfermedad, lo que aumenta hasta un 20% en caso de agregarse antecedentes de la patología en familiares de primer grado. La Espondilitis anquilosante característicamente afecta más a hombres que a mujeres en relación 3:1, siendo más frecuente en menores de 40 años. Dentro de las enfermedades inflamatorias intestinales, la artritis periférica se observa en cerca del 15% de los pacientes con enfermedad de Crohn y 10% en colitis ulcerosa, mientras que el compromiso axial ocurre entre el 15-20% en enfermedad Crohn y 10-15% en colitis ulcerosa.

Tratamiento Se requieren de medidas generales de ejercicio físico para evitar la restricción de la movilidad articular. El uso de fármacos incluye los AINEs como medida sintomática, DMARDs y terapias biológicas dependiendo del caso.

Seguimiento Derivar a especialista 13

Polimiositis, dermatomiositis Nivel de manejo del médico general: Diagnóstico: Sospecha. Tratamiento: Inicial. Seguimiento: Derivar.



Aspectos esenciales     

PM: Debilidad muscular proximal. DM: se agregan manifestaciones cutáneas como exantema heliotropo y pápulas de Grotton Se confirma con EMG y biopsia muscular. Potencialmente mortal, por lo que debe iniciarse tratamiento precoz (corticoides a altas dosis). Debe hacerse screening de neoplasia oculta (muy relevante).





Caso clínico tipo 

Paciente de 45 años, que presenta una historia de un mes de debilidad muscular de cintura escapular y pélvica. En la piel se observan zonas eritematosas en articulaciones MCF y coloración violácea en región palpebral. El laboratorio muestra CK de 5.000 U/L.

Tratamiento Rehabilitación muscular temprana, fotoprotección.

Definición

El pilar farmacológico son los corticoides en altas dosis (prednisona 1 mg/Kg/día) durante 3 a 4 semanas con reducción posterior muy gradual, los cuales deben iniciarse lo antes posible, sin esperar el resultado de la biopsia, especialmente si existen signos o síntomas de mal pronóstico como disfagia faríngea, debido a la asociación a neumonía aspirativa. En caso de refractariedad se pueden usar inmunosupresores; aproximadamente un 75% requerirá tratamiento inmnosupresor: habitualmente azatioprina 3 mg/kg/día, o metotrexato 7,5 mg/semana con aumento gradual hasta 25 mg/semana.

Enfermedades inflamatorias autoinmunes incluidas dentro del grupo de las miopatías inflamatorias idiopáticas, que habitualmente cursan con debilidad muscular progresiva y simétrica, y muchas veces compromiso sistémico.

Etiología-epidemiología-fisiopatología De etiología desconocida. Son enfermedades de baja incidencia, en EEUU se ha descrito 5 x 1.000.000 habitantes. La polimiositis (PM) y dermatomiositis (DM) son más frecuentes en mujeres (2:1). En la PM la edad promedio de inicio es entre los 50-60 años, mientras que la DM presenta 2 peak de incidencia: entre los 5-15 años y entre los 45-65. Hasta en el 20% de los pacientes, en especial los mayores de 45-50 años con dermatomiositis, puede existir asociación con tumores malignos (generalmente cáncer de ovario, pulmón, páncreas y tracto digestivo).

Seguimiento Derivar a especialista.

Diagnóstico 

heliotropo (lesión eritematosa violácea en párpados, muy característica, que también puede encontrarse en región malar y sobre el puente de la nariz) y en ocasiones, telangectasias periungueales y fenómeno de Raynaud. Laboratorio: Elevación sérica de CK, LDH o transaminasas. Además entre el 60-80% presenta ANA (+). Dentro de los ENA, destaca la presencia de Anti Jo-1 en el 20-30% de la PM y el 10% de las PM, lo cual se asocia muchas veces a enfermedad pulmonar difusa. Es así como la conjunción de miositis, compromiso pulmonar intersticial, presencia de Anti Jo-1 y las llamadas “manos de mecánico” (Lesiones hiperqueratósicas con zonas agrietadas), conforman la entidad denominada Síndrome antisintetasa. Electromiografía: Evidencia signos característicos de miopatía inflamatoria que ayuda al diagnóstico. Histología: Biopsia muscular y cutánea compatible. En el caso de la PM, el blanco es la miofibrilla y en la DM, son los vasos sanguíneos. En todo paciente con una miopatía inflamatoria debe hacerse screening de neoplasia oculta.

Clínica: La manifestación más característica es la debilidad muscular proximal, con especial compromiso en la flexión anterior del cuello, lo cual es referido por los pacientes como dificultad para peinarse, subir escaleras o levantarse de una silla. Es común además la disfagia por debilidad de la musculatura faringea; en casos avanzados puede verse afectada la musculatura respiratoria. Los músculos extraoculares resultan ilesos. La evolución de la sintomatología en el 55% de los casos ocurre en 3-6 meses. Las manifestaciones cutáneas (que generalmente anteceden a las musculares) permiten diferenciar la DM de la PM. Las lesiones características son las pápulas de Gottron (pápulas rosadas sobre articulaciones de manos, principalmente MCF), eritema 14

Related Documents


More Documents from "Veronica Llumiquinga"